Download as pdf or txt
Download as pdf or txt
You are on page 1of 658

NEUROSURGERY

SELF-ASSESSMENT
This page intentionally left blank

www.ketabpezeshki.com 66485438-66485457
NEUROSURGERY
SELF-ASSESSMENT
Questions and Answers

Rahul S. Shah, BSc(Hons), MBChB(Hons), MRCS(Eng)


Specialty Registrar in Neurosurgery and
Wellcome Trust Clinical Research Fellow
University of Oxford
Oxford, UK

Thomas A.D. Cadoux-Hudson, DPhil, FRCS, MB BS


Honorary Consultant Neurosurgeon
Department of Neurosurgery
Oxford University Hospitals NHS Trust
Oxford, UK

Jamie J. Van Gompel, MD


Associate Professor of Neurosurgery and Otolaryngology
Mayo Clinic College of Medicine
Rochester, MN, USA

Erlick A.C. Pereira, MA, BM BCh, DM, FRCS(Neuro.Surg),


SFHEA
Senior Lecturer in Neurosurgery and Consultant Neurosurgeon
Atkinson Morley Neurosciences Centre, St George’s Hospital
St George’s, University of London
London, UK

Foreword by

Edward C. Benzel, MD
Chairman, Department of Neurosurgery
Center for Spine Health, Cleveland Clinic
Cleveland, OH, USA

For additional online content visit ExpertConsult.com

Edinburgh London New York Oxford Philadelphia St Louis Sydney Toronto 2017

www.ketabpezeshki.com 66485438-66485457
© 2017, Elsevier Inc. All rights reserved.

The right of Drs. Rahul S. Shah, Thomas A.D. Cadoux-Hudson, Jamie J. Van Gompel, Erlick A.C. Pereira to
be identified as author of this work has been asserted by them in accordance with the Copyright, Designs and
Patents Act 1988.

No part of this publication may be reproduced or transmitted in any form or by any means, electronic or
mechanical, including photocopying, recording, or any information storage and retrieval system, without
permission in writing from the publisher. Details on how to seek permission, further information about the
Publisher’s permissions policies and our arrangements with organizations such as the Copyright Clearance
Center and the Copyright Licensing Agency, can be found at our website: www.elsevier.com/permissions.

This book and the individual contributions contained in it are protected under copyright by the Publisher
(other than as may be noted herein).

Notices
Knowledge and best practice in this field are constantly changing. As new research and experience
broaden our understanding, changes in research methods, professional practices, or medical treatment may
become necessary.

Practitioners and researchers must always rely on their own experience and knowledge in evaluating
and using any information, methods, compounds, or experiments described herein. In using such
information or methods they should be mindful of their own safety and the safety of others, including
parties for whom they have a professional responsibility.

With respect to any drug or pharmaceutical products identified, readers are advised to check the
most current information provided (i) on procedures featured or (ii) by the manufacturer of each product to
be administered, to verify the recommended dose or formula, the method and duration of administration,
and contraindications. It is the responsibility of practitioners, relying on their own experience and
knowledge of their patients, to make diagnoses, to determine dosages and the best treatment for each
individual patient, and to take all appropriate safety precautions.

To the fullest extent of the law, neither the Publisher nor the authors, contributors, or editors,
assume any liability for any injury and/or damage to persons or property as a matter of products
liability, negligence or otherwise, or from any use or operation of any methods, products, instructions,
or ideas contained in the material herein.

ISBN: 978-0-323-37480-4

Printed in China

Last digit is the print number: 9 8 7 6 5 4 3 2 1

Content Strategist: Lotta Kryhl


Content Development Specialist: Humayra Rahman Khan
Project Manager: Srividhya Vidhyashankar
Design: Miles Hitchen
Illustration Manager: Lesley Frazier
Marketing Manager: Rachael Pignotti

www.ketabpezeshki.com 66485438-66485457
CONTENTS

FOREWORD, VII 16 NEUROPSYCHOLOGY AND NEUROLOGICAL


REHABILITATION, 236
PREFACE, IX
17 STATISTICS, 241
HOW TO PASS NEUROSURGICAL
EXAMINATIONS, XI 18 PROFESSIONALISM AND MEDICAL
ETHICS, 242

19 SURGICAL TECHNOLOGY AND


PART I PRACTICE, 256

BASIC SCIENCE
1 NEUROANATOMY, 1 PART III
2 EMBRYOLOGY, 22 CRANIAL NEUROSURGERY
3 NEUROPHYSIOLOGY, 34 20 GENERAL NEUROSURGERY AND CSF
DISORDERS, 257
4 NEUROPATHOLOGY I: BASICS, 55
21 CRANIAL TRAUMA, 268
5 NEUROPATHOLOGY II: GROSS
PATHOLOGY, 66 22A CRANIAL VASCULAR NEUROSURGERY I:
ANEURYSMS AND AVMS, 290
6 NEUROPATHOLOGY III: HISTOLOGY, 86
22B CRANIAL VASCULAR NEUROSURGERY II:
7 PHARMACOLOGY, 121 CEREBRAL REVASCULARIZATION
AND STROKE, 306

23 CRANIAL ONCOLOGY, 322


PART II
24 SKULL BASE AND PITUITARY
CARE OF THE SURGERY, 340
NEUROSURGICAL PATIENT 25 CRANIAL INFECTION, 354

8 NEUROLOGY AND STROKE, 127

9 NEURO-OPHTHALMOLOGY, 161 PART IV


10 NEURO-OTOLOGY, 176 SPINAL NEUROSURGERY
11 NEUROINTENSIVE AND PERIOPERATIVE
CARE, 185 26 SPINE: GENERAL PRINCIPLES, 364

12 INFECTION, 201 27 SCOLIOSIS AND SPINAL


DEFORMITY, 377
13 SEIZURES, 203
28 SPINAL TRAUMA AND ACUTE
14 NEURORADIOLOGY, 216 PATHOLOGY, 386

15 RADIOTHERAPY AND STEREOTACTIC 29 DEGENERATIVE SPINE, 403


RADIOSURGERY, 226
v

www.ketabpezeshki.com 66485438-66485457
vi CONTENTS

30 SPINAL INFECTION, 418 PART VII


31 SPINAL ONCOLOGY, 424 PEDIATRIC
32 SPINAL VASCULAR NEUROSURGERY, 440
NEUROSURGERY
38 PEDIATRIC NEUROSURGERY: GENERAL
PRINCIPLES AND NORMAL
PART V DEVELOPMENT, 513
FUNCTIONAL 39 CRANIOSYNOSTOSIS, 521
NEUROSURGERY
40 CONGENITAL CRANIAL AND SPINAL
33 PAIN SURGERY, 448 DISORDERS, 527

34 ADULT AND PEDIATRIC EPILEPSY 41 PEDIATRIC NEUROSURGERY: GENERAL AND

SURGERY, 459 HYDROCEPHALUS, 546

35 ADULT MOVEMENT DISORDERS, 471 42 PEDIATRIC NEURO-ONCOLOGY, 557

36 SURGERY FOR PSYCHIATRIC 43 PEDIATRIC HEAD AND SPINAL TRAUMA, 569


DISORDERS, 485 44 PEDIATRIC VASCULAR NEUROSURGERY, 584
45 PEDIATRIC MOVEMENT DISORDERS AND

PART VI SPASTICITY, 589

PERIPHERAL NERVE 46 NEUROSURGERY AND PREGNANCY, 593


SURGERY
INDEX, 600
37 PERIPHERAL NERVE, 489

www.ketabpezeshki.com 66485438-66485457
FOREWORD

Neurosurgery Self-Assessment: Questions and the multiple choice question strategy employed
Answers by Shah, Cadoux-Hudson, Van Gompel by the authors particularly relevant to modern
and Pereira is a true masterpiece. All neurosur- day foundational neurosurgery information
geons need ‘refreshers’; some for certification, acquisition and retention.
some for maintenance of certification, and others I commend the authors for their tried and true,
for the mere need to ‘keep up’. With over 1000 but uncommonly used, approach to education. It
questions and 700 images available both in print takes the agony out of reading a chapter. It min-
and interactively online, this volume provides imizes the laborious efforts required to gather
an extensive coverage of neurosurgery from top information via searches and other strategies. It
to bottom, and all points in between. Multiple brings the art and craft of neurosurgery to life
choice questions are used to test foundation of in an enjoyable and relatively painless format.
knowledge and, most importantly, educate. Finally, it provides a near complete coverage of
As adults, we learn most efficiently and effec- the field – at least as complete as is humanly pos-
tively when our minds are exercised and stressed. sible in the space afforded.
When multiple modalities are employed (such as So, whether you have an impending examina-
questions, answers and explanations), learning tion, or you simply desire to ‘spiff up’ on your
becomes more efficient, with a greater long term neurosurgical foundations, this book is for you.
retention of the newly acquired information. Use it as one might use a bedside novel. Use it
This becomes particularly relevant to those who to prepare. Use it to simply stay at the top of
are to soon be ‘tested’ in the form of certification your field. This book can truly fulfill all of these
or maintenance of certification examinations. needs – and much, much more.
Reading, thinking, answering, and then the con-
templation of answers and their rationales makes Ed Benzel

vii

www.ketabpezeshki.com 66485438-66485457
This page intentionally left blank

Neurosurgery Books Full


www.ketabpezeshki.com 66485438-66485457
PREFACE

Neurosurgical training is delivered worldwide those about sit their examinations who require a
with the goal of producing a surgeon who is safe mix of questions (in terms of both topic and diffi-
for independent practice. Today, neurosurgical culty), this is provided by the interactive question
residents and their trainers are trying to achieve bank accessed via the online Inkling platform and
this goal in the face of reduced working hours, smartphone app. This book consists of single best
increasing demand on services, individual sur- answer (SBA) and extended matching item (EMI)
geon outcome publication, and increasing litiga- questions constructed according to the guidelines
tion, to name but a few challenges. In this from the US National Medicine Licensing Board
environment, the value of targeted learning mate- and the UK Joint Committee on Intercollegiate
rials and advanced surgical simulation is clear. Examinations, to enable the user to become
The content of this question book aims to reflect familiar with the respective formats before the
the evolving expectations placed on residents in exam. While SBA- and EMI-style questions are
an age of evidence-based practice, subspecializa- not yet universal in postgraduate neurosurgical
tion, and multidisciplinary teams: one must also examinations across the world, we hope all trainees
be familiar with allied specialties advancing just find them valuable and cost-effective for self-
as fast as our own. study.
As a counterpoint to currently available self- Finally, I would like to thank Elsevier—their
assessment books, we have organized questions support has ensured that this book could also
by the highly specific topic areas outlined in most serve as a comprehensive and representative cat-
modern neurosurgical textbooks and training cur- alogue of commonly examined clinical images
ricula. Furthermore, most questions are accompa- and investigation results in a single resource for
nied by in-depth answers and, where appropriate, neurosurgical residents. I hope you enjoy using it!
suggestions for further reading. We hope this will
enable junior trainees to use it as a learning aid and Rahul S. Shah
for focused revision prior to rotating onto partic- Oxford
ular neurosurgical firms. For senior trainees or July 2016

ix
Neurosurgery Books Full
www.ketabpezeshki.com 66485438-66485457
This page intentionally left blank

Neurosurgery Books Full


www.ketabpezeshki.com 66485438-66485457
HOW TO PASS NEUROSURGICAL
EXAMINATIONS
LEARNING BY MULTIPLE For the vast majority of multiple choice ques-
tions (MCQs) in this book, we provide a detailed
CHOICE QUESTIONS explanation of the correct answer with references
to current evidence-based data where appropri-
The World Federation of Neurosurgical Societies
ate. Like the real examinations, questions test
estimates that there are 30,000 neurosurgeons
the reader's knowledge of basic and clinical neu-
worldwide. In the United States, there are approx-
rosciences and neurosurgery, and are arranged by
imately 3500 board certified neurosurgeons and
topic to be useful to doctors in neurology, neuro-
800 neurosurgical residents. In the United radiology, and neuropathology, and medical stu-
Kingdom, there are close to 300 consultants and
dents. Illustrations include anatomical pictures,
200 trainees, with a total of approximately 8000
graphs, tables, radiology images, and histology
qualified neurosurgeons and trainees in Europe.
slides in questions and answers where required.
Due to international collaboration through
We suggest the following approach to using
research and education, neurosurgical training
this book and learning by MCQs:
curricula have become increasingly standardized
• Firstly, start early! Learning throughout
across most countries. Both UK and US-style
one's training will lead to reinforcement
examinations are well established in other coun-
and consolidation of deep knowledge not
tries (e.g. India and Brazil, respectively), and
easily forgotten. Use books like this at the
recently developed training programs in Africa beginning, middle, and end of training,
have based their examinations on the UK format.
and relate them to your clinical practice.
Additionally, the need for already qualified neuro-
• Secondly, let this book be a guide to con-
surgeons to demonstrate continuing professional solidate the information learnt. Annotate
development for revalidation purposes has also
material from other resources like compre-
increased the demand for courses and objective
hensive textbooks. Use the “red,” “amber,”
self-assessment tools in neurosurgery.
and “green” gradings to distinguish bet-
Although the duration of postgraduate neuro-
ween lower-yield and more difficult ques-
surgical training varies by country, completion
tions and high-yield easy questions. Make
of training usually requires the candidate to pass
connections between different subspe-
both written and oral examinations set by the
cialties and general principles, and focus
relevant national training board or committee.
on material most likely to be tested.
For the written examinations, questions are gener-
Remember that this is neither a compre-
ally multiple choice and cover the basic and clinical hensive review book nor a panacea for inad-
sciences; short answer and essay questions are used
equate preparation in the last few months
in some regions. Topics include neuroanatomy,
before the exam.
neurophysiology, neuropharmacology, critical
• Thirdly, prime your memory by returning
care, fundamental clinical skills, neuroradiology,
to challenging and annotated questions in
neuropathology, neurology, neurosurgery, and
the final days before the exam. This book
other disciplines deemed suitable and important
can serve as a useful way of retaining key
(e.g. statistics, medical law, medical ethics). Ques-
associations and refreshing important facts
tions relating to clinical neurosurgery also cover
fresh in your memory for the exam. Finally,
the main subspecialties, including trauma,
contribute to the book to enable active
neuro-oncology, skull base and pituitary surgery,
learning. Email us if you find errors or see
vascular neurosurgery, spinal surgery, pediatric
ways in which the book can be updated.
neurosurgery, peripheral nerve surgery, and
functional/epilepsy/pain surgery.

xi
Neurosurgery Books Full
www.ketabpezeshki.com 66485438-66485457
xii HOW TO PASS NEUROSURGICAL EXAMINATIONS

HOW TO TACKLE SINGLE BEST in most cities in front of desktop computers with
headphones, pencil, and paper available, and the
ANSWER (SBA) AND EXTENDED software is controlled by a mouse. Residents
MATCHING ITEM (EMI) QUESTIONS taking the US examination use certified laptops
provided by the residency program. Both have
Test performance is influenced not just by your high-quality, distinct images, and sometimes
knowledge but also by your test-taking skills. include audio and video material.
You can improve your performance by honing Given the artificial environment of computer-
your test-taking skills and strategies well in based testing, it is important to become familiar
advance of the exam so that you can concentrate with it before the actual exam. Most examination
on the information and your knowledge during boards offer a downloadable or interactive mock
the test itself. The following strategies may be examination with a few sample questions to famil-
useful. iarize yourself with the environment. Skipping
Try to deal with each question in turn, identi- the tutorial on the exam day sometimes adds extra
fying it as easy, workable or impossible from your time to answer the actual questions in the test
own perspective; our green, amber, and red clas- itself. Learn how to mark questions, go back to
sification provides an approximate examiner's them and if there are any rules preventing going
guide to difficulty for someone having completed back to previous blocks. Become familiar with
their neurosurgical training. Aim to answer all the how to view images and spot the icons for playing
easy questions, resolve the workable ones in rea- audio and video clips. Be vigilant that some multi-
sonable time, and make quick educated guesses at part questions prevent changing the answer to the
any apparently impossible ones. There are differ- first part of the question once the second part has
ent techniques for question reading that include been revealed.
reading the stem, thinking of the answer, and
turning to the choices or skimming the answer
choices and the last part of the question before US, UK, AND EUROPEAN
returning to the stem. Try different techniques
to see what work best for you and yields the high- NEUROSURGICAL EXAMINATION
est marks. Our online testing area should help STRUCTURE
with that.
Set a good pace for answering the questions. MCQ tests generally form the first part of most
Divide the total time for the exam by the number neurosurgical examinations, with the subsequent
of questions and be strict with yourself. If you are parts being a combination of oral and clinical
taking too long then mark the question, pick your examinations. The 2015 ABNS Primary Exami-
best answer, and come back to it later if you have nation consisted of 350 questions (in 6 h 45 min),
time at the end. Avoid burnout by practicing while the UK FRCS Written Examination is in
timed tests to develop endurance. Use extra time two parts, the first consisting of 135 SBA questions
to check marked questions. Never give up—take a (in 2 h) and the second part of 110 EMI questions
short one-minute break and come back to the test (in 2.5 h). The European Association of Neuro-
if too disheartened. surgical Societies Part 1 examination consists of
Answer all test questions—even if it means approximately 200 MCQs to be answered in 3 h.
guessing! Whereas in the past many neurosurgi- Questions in all three examinations cover neuro-
cal examinations were negatively marked, that anatomy, neurobiology, neuropathology, neuro-
process has largely been superseded by only pos- logy, neuroradiology, clinical neurosurgery
itively marked exams, so there is no harm in an (including subspecialties), fundamental clinical
educated or instinctive guess, or even just a blind skills, and other disciplines deemed suitable and
punt. If you have to guess, go on a hunch and pick important.
an answer you are vaguely familiar with rather The marking of such MCQ examinations is
than something you have never heard of. now quite standardized and relies upon principles
of statistics and psychology. Many examination
boards use the modified Angoff method, whereby
COMPUTER-BASED TESTING experts are briefed then allowed to take part or all
of the test with the performance levels in mind.
The UK FRCS (Neurological Surgery) examina- They are then asked to provide estimates for each
tion has been using computer-based testing for question of the proportion of minimally accept-
several years, the American Board of Neurologi- able candidates that they would expect to get
cal Surgery moved to a web-based format for the the question correct. The final determination of
Primary Examination in 2015, and the EANS the cut score is then made by averaging the esti-
Part 1 remains a pencil-and-paper test. The UK mates. Controversial questions—those that
exam takes place in dedicated test centers found polarized the candidates' answers between two
Neurosurgery Books Full
www.ketabpezeshki.com 66485438-66485457
HOW TO PASS NEUROSURGICAL EXAMINATIONS xiii

answers or those that candidates scoring highly ability to satisfy patients and colleagues that those
overall got wrong whereas those scoring poorly passing have attained a minimum standard
overall got right—are scrutinized and potentially of basic and applied science knowledge and clin-
removed from the overall scoring at examiners' ical decision-making to practice independently.
standard setting meetings. It is good practice Oral examinations are crucial in this process as
for a trainee representative who has sat the exam- they assess communication skills, clinical skills,
ination to participate in the whole process. and decision-making and professionalism in a
Whereas the written examination explores an high-pressure environment. In contrast, MCQs
applicant's knowledge in various relevant disci- focus on assessing knowledge and analytical and
plines, the oral examination explores knowledge decision-making skills. More clinically integra-
and judgment in clinical neurosurgical practice tive questions test higher orders of Bloom's tax-
after an applicant has been an independent prac- onomy and are more effective than simple
titioner. The oral examination is accomplished factual questions in assessing and developing
in a series of face-to-face examinations. The the clinical problem-solving skills of trainee
applicant is presented with a series of clinical surgeons.
vignettes using real patients, clinical descriptions, Patients fundamentally wish for their treating
radiographs, computerized images, anatomical surgeon to be as independent as possible in order
models, and/or diagrams. The examiners grade to maximize their chances for an excellent
the applicant on specific tasks including diagnos- outcome. Therefore, when setting minimum
tic skills, surgical decision-making, and manage- standards for independent practice, an expert
ment of complications. peer group of examiners is accountable to
patients, other neurosurgeons and healthcare
professionals, and the general public. Postgrad-
STANDARDS FOR INDEPENDENT uate medical examinations have therefore gen-
NEUROSURGICAL PRACTICE erally evolved to become as standardized and
fair as possible, while maintaining rigor, expand-
The credibility of professional examinations ing, and adapting as trends change in clinical
taken at the end of surgical training rests on their practice.

Neurosurgery Books Full


www.ketabpezeshki.com 66485438-66485457
This page intentionally left blank

Neurosurgery Books Full


www.ketabpezeshki.com 66485438-66485457
PART I
BASIC SCIENCE
CHAPTER 1

NEUROANATOMY
SINGLE BEST ANSWER (SBA) QUESTIONS
1. From inferior to superior (i.e. ascending), 5. Lesion of which structure increases extensor
what is the 4th branch of the external carotid tone?
artery in the neck? a. Dentate nucleus
a. Maxillary artery b. Pedunculopontine nucleus
b. Occipital artery c. Red nucleus
c. Facial artery d. Ventral tegmentum
d. Lingual artery e. Superior olive
e. Posterior auricular artery
6. Which one of the following drain into the
2. The pathway best describing how sympa- cavernous sinus?
thetic fibers of the autonomic nervous system a. Superior ophthalmic vein
exit the spinal cord is: b. Superior petrosal sinus
a. Via the dorsal roots and white rami c. Inferior petrosal sinus
communicans d. Basal vein of Rosenthal
b. Via the ventral roots and white rami e. Vein of Labbe
communicans
c. Via the dorsal roots and gray rami 7. Persistent trigeminal artery is commonly:
communicans a. Found in 3-5% of people
d. Via the ventral roots and gray rami b. Found to connect to the proximal basilar
communicans artery
e. Via the ventral roots and spinal nerves c. Found to branch off from the ICA just
proximal to the meningohypophyseal
3. The left vertebral artery usually arises from trunk
the: d. Found to have a vascular abnormality in
a. Arch of the aorta approximately 50% of cases
b. Brachiocephalic trunk e. Found in conjunction with internal
c. Left common carotid carotid artery aplasia
d. Left subclavian artery
e. Costocervical trunk 8. The afferent loop of the Hering-Breuer
inflation and deflation reflexes is mediated
4. Hemiballismus results from lesioning which by:
basal ganglia target? a. CN XIII
a. Globus pallidus interna b. CN IX
b. Subthalamic nucleus c. CN X
c. Substantia nigra pars reticularis d. CN XI
d. Striatum e. C2
e. Pedunculopontine nucleus

1
Neurosurgery Books Full
www.ketabpezeshki.com 66485438-66485457
2 PART I BASIC SCIENCE

9. Which one of the following nerves is outside e. Preganglionic fibers synapse in either the
the annulus of Zinn? sympathetic chain or prevertebral ganglia
a. Abducens
b. Nasociliary 14. Nervi erigentes are responsible for:
c. Trochlear a. Inhibition of the external anal sphincter
d. Oculomotor (superior division) b. Inhibition of the internal vesicle sphincter
e. Oculomotor (inferior division) c. Inhibition of the internal anal sphincter
d. Inhibition of the external vesicle sphincter
10. The C2 vertebra has how many secondary e. Inhibition of the rectal muscles
ossification centers?
a. 2 15. Parasympathetic sensory afferents terminate
b. 3 in which one of the following?
c. 4 a. Nucleus ambiguus
d. 5 b. Solitary nucleus
e. 6 c. Edinger-Westphal nucleus
d. Red nucleus
11. A line drawn between the highest point of the e. Superior colliculus
iliac crests across the back usually denotes:
a. L1/2 interspace 16. Which one of the labels in the diagram below
b. L2/3 interspace of the internal auditory canal identifies the
c. L3/4 interspace facial nerve?
d. L4/5 interspace
e. L5/S1 interspace

12. Which one of the following is labeled X in


the image below? A
C
B
D

E
F
X

17. Blood supply to the posterior pituitary gland


arises from branches of which internal
carotid artery segment?

a. Ophthalmic division of the trigeminal nerve


b. Meckel’s cave
c. Oculomotor nerve
d. Maxillary division of trigeminal nerve
e. Abducens nerve

13. Which one of the following statements about C7


C6
the sympathetic nervous system is FALSE? C4
C5
a. Innervation of thoracic viscera arises from
T1-T4 spinal segments
C2
b. Splanchnic nerves are unmyelinated C3
c. Preganglionic fibers enter the sympa-
thetic chain via white rami communicans C1
d. Sensory afferent fibers are important for
visceral pain sensation

Neurosurgery Books Full


www.ketabpezeshki.com 66485438-66485457
1 NEUROANATOMY 3

a. C1 (Cervical) 27. Internal auditory canal:


b. C2 (Petrous)
c. C3 (Lacerum)
d. C4 (Cavernous)
e. C5 (Clinoid)
f. C6 (ophthalmic/supraclinoid)
g. C7 (communicating)
C D E F G
QUESTIONS 18–25 H

Additional questions 18–25 available on


ExpertConsult.com I
B A J

EXTENDED MATCHING ITEM (EMI)


QUESTIONS
26. Cavernous sinus imaging: For each of the following descriptions, select the
most appropriate answers from the image above.
Each answer may be used once, more than once
or not at all.
1. AICA
2. Basal turn of cochlea
3. Cochlear nerve

28. Cavernous sinus anatomy:

G B A

A F
B H
C
F
C
G
D D H

E I

E J
K
F L

For each of the following descriptions, select the


most appropriate answers from the image above.
Each answer may be used once, more than once
For each of the following descriptions, select the
or not at all.
most appropriate answers from the diagram
1. Right optic nerve above. Each answer may be used once, more than
2. Oculomotor nerve once or not at all.
3. Abducens nerve 1. ACA
2. Maxillary division of CN V (V2)
3. Oculomotor nerve (III)

Neurosurgery Books Full


www.ketabpezeshki.com 66485438-66485457
4 PART I BASIC SCIENCE

29. Internal auditory canal: 31. Basal Ganglia:


A
B
C R
D
F E F
VI A
Pons
G G
Q AICA B
1
2 H
C
I
VIII D

H P IX
E J
K
X
I
O 1 Lateral medullary lamina

XI L 2 Medial medullary lamina

J N For each of the following descriptions, select the


K most appropriate answers from the image above.
Each answer may be used once, more than once
L or not at all.
M
1. Caudate nucleus
For each of the following descriptions, select the 2. Claustrum
most appropriate answers from the image above. 3. Globus pallidus interna
Each answer may be used once, more than once 4. Internal capsule
or not at all. 5. Putamen
1. Facial nerve
2. Superior vestibular nerve 32. Projection and association tracts:
3. Greater superficial petrosal nerve a. Central tegmental tract
4. Posterior semicircular canal b. Lamina terminalis
c. Median forebrain bundle
30. Internal auditory canal: d. Stria medullaris
e. Stria terminalis
f. Postcommissural Fornix
g. Nucleus of the diagonal band of Broca
(vertical limb)
h. Retinohypothalamic tract
i. Supraopticohypophyseal tract
C
j. Tuberoinfundibular
D A (tuberohypophyseal) tract
k. Trapezoid body
B l. Thalamic fasciculus (Forel’s field H1)
m. Nucleus of the Diagonal band of Broca
(horizontal limb)
A n. Mammillothalamic tract
o. Tapetum
A E For each of the following descriptions, select the
B G H most appropriate tracts from the list above. Each
F
answer may be used once, more than once or not
at all.
For each of the following descriptions, select the 1. Conducts fibers to the posterior pituitary
most appropriate answers from the images above. gland
Each answer may be used once, more than once 2. Arcuate nucleus to hypophyseal portal sys-
or not at all. tem of infundibulum
1. Anterior inferior cerebellar artery 3. Septal nuclei to hippocampus
2. Vestibulocochlear nerve 4. Connects sepal area, hypothalamus, basal
3. Facial nerve olfactory areas, hippocampus/subiculum
to midbrain, pons and medulla
5. Hippocampus to cingulate gyrus

Neurosurgery Books Full


www.ketabpezeshki.com 66485438-66485457
1 NEUROANATOMY 5

33. Vascular territories: 35. Offending Artery:


a. Middle cerebral artery a. A1 portion of anterior cerebral artery
b. Basilar artery b. Anterior choroid artery
c. Perforators from internal carotid artery c. Anterior communicating artery
d. Ophthalmic artery d. Anterior inferior cerebellar artery
e. P2 portion of posterior cerebral artery e. Basilar arteries
f. Vertebral artery f. Facial artery
g. Superior cerebellar artery g. Internal carotid artery
h. Posterior inferior cerebellar artery h. M3 portion of middle cerebral artery
i. Anterior inferior cerebellar artery i. Ophthalmic artery
j. Posterior communicating artery j. Posterior cerebral artery
k. A2 portion of anterior cerebral artery k. Posterior communicating artery
l. P3 portion of posterior cerebral artery l. Posterior inferior cerebellar artery
m. Recurrent artery of Heubner m. Superior cerebellar artery
n. Vertebral artery
For each of the following descriptions, select the
most appropriate answers from the list above. For each of the following descriptions, select the
Each answer may be used once, more than once most appropriate answers from the list above.
or not at all. Each answer may be used once, more than once
1. Posterior limb of the internal capsule or not at all.
2. Medial and lateral geniculate nuclei 1. Glossopharyngeal neuralgia
3. Anterior limb of internal capsule and head 2. Trigeminal neuralgia
of caudate 3. Hemifacial spasm
4. Posterior pituitary gland 4. Horner’s syndrome
5. Splenium of corpus callosum 5. CN III palsy

34. Cerebral veins: 36. Autonomic nervous system:


a. Erdinger-Westphal nucleus
B C b. Superior salivatory nucleus
A
c. Inferior salivatory nucleus
d. Dorsal nucleus
D
e. Ciliary ganglion
f. Pterygopalatine ganglion
E g. Otic ganglion
h. Submandibular ganglion
F i. CNII
j. CNV
R G k. Chorda tympani
H
l. Vidian nerve
Q m. Superior cervical ganglion
I n. Greater petrosal nerve
P
o. Lesser superficial petrosal nerve
J
O p. Auriculotemporal nerve
N
L K
M
For each of the following descriptions, select the
most appropriate answers from the list above.
For each of the following descriptions, select the Each answer may be used once, more than once
most appropriate answers from the image above. or not at all.
Each answer may be used once, more than once 1. Mediates bronchoconstriction
or not at all. 2. Receives preganglionic parasympathetic
1. Inferior anastamotic vein of Labbe fibers via CNIII
2. Superficial middle cerebral vein of Silvius 3. Postganglionic parasympathetic fibers to
3. Superior anastamotic vein of Trolard parotid gland
4. Basal vein of Rosenthal 4. Preganglionic parasympathetic fibers to the
5. Vein of Galen submandibular ganglion
5. Origin of preganglionic parasympathetic
fibers transmitted in GSPN IX

Neurosurgery Books Full


www.ketabpezeshki.com 66485438-66485457
6 PART I BASIC SCIENCE

37. Projection and association tracts: 39. Thalamus:


a. Ansa lenticularis
b. Fasciculus retroflexus C
A
c. Lenticular fasciculus (Forel’s field H2) D
C
d. Postcommissural fornix
C
e. Precommissural fornix B
F J
f. Thalamic fasciculus (Forel’s field H1)
E M
g. Nucleus of the diagonal band of Broca H
G
h. Mammillothalamic tract I
i. Tapetum
j. Uncinate fasciculus
K L
k. Commissure of Probst
l. Central tegmental tract
m. Lamina terminalis For each of the following descriptions, select the
n. Median forebrain bundle most appropriate part of the thalamus from the
o. Stria medullaris image above. Each answer may be used once,
more than once or not at all.
For each of the following descriptions, select the 1. Receives major input from inferior colliculi
most appropriate option from the list above. Each 2. Major projection to the primary visual cortex
answer may be used once, more than once or not 3. Receives major projections from mammillary
at all. body
1. Globus pallidus interna to thalamus 4. Auditory relay nucleus
through internal capsule 5. Contains the area of face representation
2. Globus pallidus interna to thalamus around
internal capsule 40. Projection and association tracts:
3. Septal nuclei to amygdala a. Inferior collicular commissure
4. Temporal lobe to occipital lobe b. Cingulate fasciculus
5. Connection between nuclei of lateral c. Arcuate fasciculus
lemniscus d. Corpus callosum
e. Posterior commissure
38. Thalamus: f. Hypothalamospinal tract
g. Brachium conjunctivum
A H h. Brachium pontis
I
i. Restiform and juxtarestiform bodies
B j. Dorsal longitudinal fasciculus
C k. Medial longitudinal fasciculus
J
l. Uncinate fasciculus
D m. Lamina terminalis
E n. Commissure of Probst
o. Stria medullaris
F
K
For each of the following descriptions, select the
G L most appropriate X from the list above. Each
answer may be used once, more than once or
not at all.
For each of the following descriptions, select the 1. Periventricular hypothalamus and mam-
most appropriate part of the thalamus from the millary bodies to midbrain central gray
image above. Each answer may be used once, matter
more than once or not at all. 2. Covered with indusium griseum
1. Pulvinar 3. Contains crossing fibers of pretectal
2. Ventral anterior nucleus nucleus for light reflex
3. Ventral posterolateral nucleus 4. Connects Wernicke and Broca’s areas
4. Lateral geniculate nucleus 5. Interruption can result in Horner’s
5. Medial geniculate nucleus syndrome

Neurosurgery Books Full


www.ketabpezeshki.com 66485438-66485457
1 NEUROANATOMY 7

41. For each of the following descriptions, select 43. Sulci and gyri:
the most appropriate answers from the image
A B
below. Each answer may be used once, more C1
C
than once or not at all. D
E
B1 F
A1
G
Z
G Y
H
I
X
H W J
A
V
U
B I T K

S L
R
C
J Q
P M
D O N
K
Frontal lobe Parietal lobe Temporal lobe Occipital lobe
E
L
F
For each of the following descriptions, select the
most appropriate answers from the image above.
Each answer may be used once, more than once
1. Cisterna magna or not at all.
2. Interpeduncular cistern 1. Angular gyrus
3. Chiasmatic cistern 2. Supramarginal gyrus
3. Pars opercularis of inferior frontal grus
42. Cranial Nerve Nuclei: 4. Middle frontal gyrus
5. Parieto-occipital sulcus

44. Sulci and gyri:


A
D1 A
B C1 B
C L
B1 C
A1 D
D M
Z E
E
N F
F
Y
B
G
G X G Septum
H O H
Superior
W R S
I V Thalamus I
J P Lateral U
K
T J
Dorsal
cochlear S K
R L
Efferent cranial Inferior Q M
nerve nuclei Medial P N
O
Vestibular nuclei Frontal lobe Limbic lobe Temporal lobe Parietal lobe Occipital lobe
Afferent cranial
nerve nuclei

For each of the following descriptions, select the


For each of the following descriptions, select the most appropriate answers from the image above.
most appropriate answers from the image above. Each answer may be used once, more than once
Each answer may be used once, more than once
or not at all.
or not at all.
1. Marginal sulcus
c
1. Abducens nerve nucleus 2. Calcarine sulcus h
2. Principal sensory nucleus of trigeminal nerve 3. Cuneus f
3. Solitary tract nucleus 4. Collateral sulcus n
4. Facial nerve motor nucleus 5. Lamina terminalis
5. Nucleus ambiguus t

Neurosurgery Books Full


www.ketabpezeshki.com 66485438-66485457
8 PART I BASIC SCIENCE

45. Sulci and gyri: 47. Cranial Nerve Nuclei:


C
H
A D
A
I E
B
J SC
F
C IC
K
G
D G
S L H
I
E M
J
K
F N
L
O M
G N
O
IO
P
Q
For each of the following descriptions, select the R
most appropriate answers from the image above. S
Each answer may be used once, more than once T
or not at all.
1. Central sulcus h B
U

2. Paracentral sulcus B
3. Calcarine sulcus N
4. Marginal sulcus J
5. Precuneus
k
46. Fourth ventricular floor: For each of the following descriptions, select the
most appropriate answers from the image above.
A
Each answer may be used once, more than once
J or not at all.
K 1. Red nucleus
B
2. Erdinger-Westphal nucleus
L 3. Oculomotor nucleus
C 4. Trochlear nucleus
D
M 5. Abducens nucleus
N 6. Facial nucleus
O
7. Nucleus ambiguus of vagus nerve
E
F
48. Medulla at sensory decussation:
P
G Q A
H R L
B
I S M
C
N
D
O
For each of the following descriptions, select the P
most appropriate answers from the image above. E Q
Each answer may be used once, more than once F
R
or not at all. G S
1. Facial colliculus L H
T
2. Striae medullaris d I
U
3. Sulcus limitans p V
4. Median sulcus c J
W
5. Vagal trigone g
X
K
Y

Neurosurgery Books Full


www.ketabpezeshki.com 66485438-66485457
1 NEUROANATOMY 9

For each of the following descriptions, select the 1. Posterior cochlear nucleus
most appropriate answers from the image above. 2. Vestibulocochlear nerve
Each answer may be used once, more than once 3. Spinal trigeminal nucleus
or not at all. 4. Medial longitudinal fasciculus
1. Nucleus gracilis 5. Nucleus ambiguus
2. Nucleus cuneatus
3. Spinothalamic tract 51. Caudal pons:
4. Posterior spinocerebellar fibers
A
B P
49. Medulla and vagal nuclei: C Q
R
K
A O
L
S
B M
D
T
U
C N
E V
O F
D
P G
W
Q H
E R I
F S J X
K
T
Y
U
G L Z
V A1
M
W N B1
H
X
I For each of the following descriptions, select the
Y
J most appropriate answers from the image above.
Z
Each answer may be used once, more than once
or not at all.
For each of the following descriptions, select the 1. Facial nucleus
most appropriate answers from the image above. 2. Facial nerve
Each answer may be used once, more than once 3. Superior olivary nucleus
or not at all. 4. Abducens nucleus
1. Solitary nucleus and tract 5. Abducens nerve
2. Dorsal motor vagal nucleus
3. Reticular formation 52. Mid-pons:
4. Principal olivary nucleus (inferior olivary A N
nucleus) B
O
M
5. Medial lemniscus C P
D

50. Rostral medulla: Q


E
F
A
N R
B O G
C P S
Q
H
R T
E
S I
D
J U
F
G K V
T W
L
H U
I V
W
J For each of the following descriptions, select the
X
K
most appropriate answers from the image above.
Y Each answer may be used once, more than once
L Z or not at all.
M 1. Locus ceruleus
2. Corticospinal fibers
For each of the following descriptions, select the 3. Principal trigeminal sensory nucleus
most appropriate answers from the image above. 4. Fourth ventricle
Each answer may be used once, more than once 5. Brachium pontis
or not at all.
Neurosurgery Books Full
www.ketabpezeshki.com 66485438-66485457
10 PART I BASIC SCIENCE

53. Rostral pons: For each of the following descriptions, select the
I most appropriate answers from the image above.
J Each answer may be used once, more than once
A or not at all.
K
B L 1. Medial lemniscus
M 2. Medial longitudinal fasciculus
C
N
D 3. Trochlear nerve
O
P 4. Central tegmental tract
E
5. Tectobulbospinal tract
Q

F
QUESTIONS 54–58
G R
Additional questions 54–58 available on
S ExpertConsult.com
H

SBA ANSWERS
1. c—Facial artery 2. b—Via the ventral roots and white rami
communicans
The external carotid artery has several branches in
the neck (SALFOPSI in ascending order): superior 3. d—Left subclavian artery
thyroid, ascending pharyngeal, lingual, facial (aka
external maxillary), occipital, posterior auricular, Each vertebral artery arises from its ipsilateral
superficial temporal, maxillary (aka internal maxil- subclavian artery. The aortic arch gives off three
lary). It can be distinguished on angiogram (figure) branches in order: brachiocephalic trunk (or
from the ICA, which has no branches in the neck. innominate artery), left common carotid and left
During EC/IC bypass procedures for Moya Moya subclavian arteries (A). The second commonest
disease, anastomosis of the superficial temporal branching pattern (termed a “bovine arch”) is
artery to the middle cerebral artery (or less com- where the left common carotid arises from the
monly occipital artery to the posterior cerebral brachiocephalic trunk (B).
artery/posterior inferior cerebellar artery) may be
performed.
Right carotid Right carotid
artery Left carotid Right artery Left carotid
Right artery
artery vertebral
vertebral
Left artery Left
artery
vertebral vertebral
artery artery
Right Right
subclavian Left Left
subclavian
artery subclavian subclavian
artery
artery artery
Innominate Innominate
artery artery

A B
Image redrawn from Layton KF, Kallmes DF, Cloft HJ, Lindell EP, Cox VS. Bovine aortic arch variant in humans:
Clarification of a common misnomer. AJNR Am J Neuroradiol 2006;27:1541-1542. In: Low M, Som PM, Naidich
TP. Problem solving in neuroradiology. Elsevier.

Neurosurgery Books Full


www.ketabpezeshki.com 66485438-66485457
1 NEUROANATOMY 11

4. b—Subthalamic nucleus After the Pcomm, persistent primitive trigeminal


artery is the next commonest remnant of the fetal
Hemiballismus is a condition characterized by circulation. It is seen in 0.1-0.6% of cerebral angio-
unilateral, involuntary, violent flinging of the grams. It connects the cavernous ICA (just proxi-
limbs. Lesion is based in the contralateral subtha- mal to meningohypophyseal trunk) to the basilar
lamic nucleus or its connections and due to vascu- artery between superior cerebellar and anterior
lar cause (PCA territory) but can occur in MS. inferior cerebellar arteries. Its persistence is usually
Often settles spontaneously and drug treatment associated with a hypoplastic basilar and vertebral
is ineffective. arteries proximal to the anastomosis, as well as a
hypoplastic PcommA. Its frequency is explained
5. c—Red nucleus as the order of regression during embryogenesis
is otic/acoustic artery first, then hypoglossal fol-
Factors normally inhibiting extensor action in the lowed by trigeminal. Vascular abnormalities
arms and legs are: (AVM, aneurysm) is seen in 25%. Characterized
(A) Cortical inhibition of lateral vestibular by the tau sign (flow void) on sagittal MRI.
nucleus (vestibulospinal tract) and pontine
reticular formation PCOMM
(B) Red nucleus projections to spinal cord
(rubrospinal tract; possibly arms only)
(C) Medullary reticular formation

Disconnection lesion involving red nucleus Trigeminal


Otic
results in loss of normal inhibition of extension
(rubrospinal and medullary reticular formation)
and loss of cortical inhibition of extensor action Proatlantal
Hypoglossal
of LVN and pontine RF, producing hyperreflexia
and increased extensor tone (decerebrate rigid-
ity). Disconnection lesions above the red nucleus
result in extension in legs, but flexion in arms
(decorticate rigidity). This is explained as in
humans the rubrospinal tract terminates in the ICA
cervical spine, meaning intact rubrospinal
input could counteract vestibulospinal (extensor)
input in the arms but it remains unopposed in Vertebral artery
the legs.

6. a—Superior ophthalmic vein Image from Law M, Som P, Naidich T. Problem Solving
in Neuroradiology, Elsevier, Saunders, 2011.
The cavernous sinus receives the superior and
inferior ophthalmic veins, sphenoparietal sinus
and the superficial middle cerebral vein (coursing 8. c—CN X
from superiorly to inferiorly in the Sylvian fis-
sure). It drains via superior petrosal sinus (to The Hering-Breuer inflation and deflation reflexes
the junction of the transverse and sigmoid are thought to play a role in controlling the depth
sinuses), inferior petrosal sinus (to the internal of breathing, although may be less important in
jugular vein). Right and left cavernous sinuses humans at rest. Their overall effect is to prevent
are also connected across the midline anterior overinflation and extreme deflation of the lungs.
and posteriorly to the pituitary gland via the ante- The inflation reflex is mediated by pulmonary
rior and posterior intercavernous sinuses, result- stretch receptor afferents signaling via CNX dur-
ing in the circular sinus. Each cavernous sinus is ing lung inflation to inhibit medullary inspiratory
also connected to the pterygoid venous plexus center and the pontine apneustic center, as well
via small branches in the foramen Vesalii, fora- as inhibiting cardiac vagal motor neurons resulting
men ovale and foramen lacerum. in sinus tachycardia. The deflation reflex also acts
via CNX and directly activates medullary inspira-
7. c—Found to branch off from the ICA just tory centers, stopping expiration and initiating
proximal to the meningohypophyseal trunk inspiration.

Neurosurgery Books Full


www.ketabpezeshki.com 66485438-66485457
12 PART I BASIC SCIENCE

9. c—Trochlear nerve Cartilaginous stage—chondrification centers


appear in the centrum and vertebral arches,
The Annulus of Zinn (or annular tendon) is a causing cartilaginous fusion, and spinous
fibrous ring which surrounds the optic nerve, and transverse processes develop from
and which is continuous with the dura of the mid- extensions of the chondrification centers in
dle cranial fossa. It is divided into upper (superior the vertebral arches. Chondrification
tendon of Lockwood) and lower (inferior tendon spreads until a cartilaginous vertebral col-
of Zinn) parts which together give rise to the umn is formed.
four recti muscles (superior, inferior, medial, Bony stage—By the end of the embryonic period
lateral) and superior oblique. The remaining two each vertebrae usually has three primary ossi-
extraocular muscles, inferior oblique and levator fication centers (centrum and each half verte-
palpabrae superioris arise from the maxillary and bral arch), and the cartilaginous connection
sphenoid bones respectively. The Annulus of between the arch and centrum allows growth
Zinn contains the optic nerve, ophthalmic artery, as the spinal cord enlarges after birth. After
superior division of CNIII, nasociliary division puberty, five secondary ossification centers
of CNV1, CNVI, and the inferior division of appear—tip of spinous process, tip of both
CNIII. transverse processes and annular epiphyses
of the vertebral body.
Recurrent Superior Superior rectus Levator palpebrae
meningeal orbital fissure superioris
artery Primary
Common Ossification Secondary
tendinous ring Vertebra Centers Ossification Centers
IV
Superior C1 2 posterior 1 anterior
Lacrimal nerve oblique
(atlas)
Rim of C2 (axis) 1 centrum 1 tip of dens
Frontal nerve optic canal
and 2 1 ring apophysis
Superior Dural vertebral arch 1 spinous process
ophthalmic vein III sheath
2 base of and 2 transverse
Nasociliary nerve VI Optic dens (odontoid process
nerve
III peg)
Lateral rectus
Medial
rectus
Inferior
orbital fissure
Ophthalmic 11. d—L4/5 Interspace
artery

Zygomatic nerve Inferior Intercristal line (Tuffier’s line)—space between


rectus
L4 and L5 spinous process, or through L4 spi-
Infraorbital nerve
Inferior
ophthalmic
nous process. In infants this is at the L5/S1 level.
and artery vein

Image from Mancall EL. Gray's Clinical Neuroanatomy: 12. b—Meckel’s cave (containing Gasserian
The Anatomic Basis for Clinical Neuroscience, Elsevier, ganglion). Axial view in T2 MRI is shown
Saunders, 2011. below

Image from Som PM, Curtin HD. Head and Neck


10. d—5 Imaging, vol. 2, 5th ed., Elsevier, Mosby, 2011.

Development of the vertebral column occurs in


three stages:
Mesenchymal stage—where somites gives rise to
sclerotomes (condensation of mesenchymal
cells around notochord and neural tube,
divided into a loosely packed upper half
and a densely pack lower half) and myo-
tomes. The centrum (primordial vertebral
body) forms from the lower half of a cranial
sclerotome and the upper half of the imme-
diately caudal sclerotome, such that the
intervertebral disc forms at the level oppo-
site the myotome and the vertebral body is
Image from Naidich TP. Imaging of the Brain, Saunders,
at the level between two myotomes. Elsevier, 2013.

Neurosurgery Books Full


www.ketabpezeshki.com 66485438-66485457
1 NEUROANATOMY 13

13. b—Splanchnic nerves are unmyelinated Visceromotor to rectal muscles, inhibitor to


internal anal sphincter
Pre-ganglionic sympathetic fibers (myelinated) Motor to bladder wall, inhibitor to internal
arise in the lateral horn of gray matter T1-L2 vesicle sphincter
and exit the cord via the anterior (ventral) root Vasodilator fibers to cavernous sinuses of
then as white rami communicans to reach the penis/clitoris
sympathetic ganglion. Here they may synapse
onto unmyelinated post-ganglionic fibers or pass 15. b—Solitary nucleus
through unchanged as splanchnic nerves (which
later synapse in prevertebral ganglia and inner- Parasympathetic sensory afferents:
vate the viscera). Post-ganglionic fibers exit the Afferent fibers from GI, respiratory, cardiac
sympathetic chain at the same or different level and mouth/pharynx travelling in CN VII/
(after ascending or descending), via a gray ramus XI/X terminate in the solitary nucleus of
communicans which relays fibers to an existing the medulla
spinal nerve. Sympathetic chain runs from the Sacral afferents terminate in the S2-S4
skull base to coccyx on both sides of the vertebral gray matter
column. Important in maintaining visceral reflexes
Spinal segments responsible for sympathetic
innervation (e.g. vasoconstrictor to skin, pilomo- 16. b—Facial nerve
tor to hair, sudomotor to sweat glands)
T1-T2: head and neck via ICA/vertebral The internal auditory canal runs lateral and poste-
arteries riorly from the porus on the medial surface of the
T2-T5: upper limb temporal bone to fundus (entry to middle ear).
T1-T4: thoracic viscera via cardiac/pulmo- The lateral portion of the internal canal is divided
nary/esophageal plexus into superior and inferior compartments by the fal-
T4-L2: abdominal viscera via splanchnic ciform or transverse crest. The superior compart-
nerves to coeliac/hypogastric plexuses (ex- ment is further divided into anterior and
cept adrenal medulla which receives a pre- posterior portions by the vertical crest (Bill’s bar).
ganglionic fiber which has also traversed Thus the IAC contains four main neural compo-
the coeliac plexus) nents in quadrants: the facial nerve (superior and
T10-L2: pelvic viscera via splanchnic nerves to anterior), the superior vestibular nerve (superior
pelvic plexus and posterior), the cochlear nerve (anterior and
T11-L2: lower limb inferior), and the inferior vestibular nerve (poste-
Sympathetic sensory afferents terminate in the rior and inferior). The inferior compartment does
intermediate zone of gray matter in the cord and not have a bony division for the cochlear and infe-
are important in the appreciation of visceral pain. rior vestibular nerves, but the cochlear nerve leaves
the IAC through a multiperforate osseous plate to
14. b—Inhibition of the internal vesicle enter the cochlear modiolus.
sphincter
Image adapted from Quiñones-Hinojosa A. Schmidek
and Sweet's Operative Neurosurgical Techniques, 6th
Sacral parasympathetic outflow: ed., Saunders, Elsevier, 2012.
Anterior (ventral) primary rami of S2, S3 and
(occasionally) S4 give off fibers termed pelvic 17. d—C4 (cavernous)
splanchnic nerves or nervi erigentes which join
the pelvic sympathetic plexus for distribution to Image from Naidich TP. Imaging of the Brain, Saunders,
pelvic viscera: Elsevier, 2013.2. PICA

Bouthillier Classsification of ICA Segments

C1 (Cervical) Extends from the origin of the internal carotid artery to its entry into the skull base
C2 (Petrous) Portion of the artery within the carotid canal of the petrous temporal bone. Initially,
ascends vertically within the canal (vertical portion) and then turns anteriorly, medially,
and superiorly within the canal (genu) and continues horizontally (horizontal portion)
toward the petrous apex, where it exits the temporal bone
Vidian artery
Caroticotympanic artery may (variably)
Continued on following page

Neurosurgery Books Full


www.ketabpezeshki.com 66485438-66485457
14 PART I BASIC SCIENCE

Bouthillier Classsification of ICA Segments (Continued)


C3 (Lacerum) Begins where the internal carotid artery exits from the carotid canal and extends up to the
level of the petroclinoid ligament. Passes over (not through) covered foramen lacerum
C4 (Cavernous) Begins at the superior aspect of the petroclinoid ligament and includes the portion of
the internal carotid artery that courses through the cavernous sinus until the proximal
dural ring
Meningohypophyseal artery, arises from the posterior genu of C4 and gives rise to three
major branches: Inferior hypophyseal artery—posterior pituitary gland
Tentorial artery (of Bernasconi and Cassinari)
Dorsal clival (meningeal) artery
The inferolateral trunk commonly arises from the horizontal portion of C4 and courses
laterally to supply CN III, IV, and VI in addition to the trigeminal ganglion and the dura
covering the cavernous sinus
C5 (Clinoid) Short segment between the proximal and distal dural reflections (rings) related to the
anterior clinoid process
C6 (ophthalmic/ Begins at the distal dural ring/reflection (continuous with the falx) around the anterior
supraclinoid) clinoid process at which point it is considered to be intradural (in the subarachnoid
space) and extends to the origin of the posterior communicating artery
Ophthalmic artery typically arises from the medial aspect of the C6 segment and courses
with the optic nerve through the optic canal into the orbit. The ophthalmic artery gives
rise to multiple ocular, orbital, and extraorbital branches. The ocular branches include
the central retinal artery and ciliary arteries
Superior hypophyseal artery arises from the medial aspect of C6, anastomoses with its
contralateral counterpart, and forms a vascular plexus about the pituitary stalk. This
plexus supplies the anterior pituitary gland, tuber cinereum, optic nerve, and optic
chiasm
C7 (communicating) Begins just proximal to the origin of the posterior communicating artery and terminates
where the internal carotid artery bifurcates into the anterior and middle cerebral arteries
Posterior communicating artery—courses posteriorly through the suprasellar cistern to
anastomose with the PCA. Large PCoA size suggests that it directly supplies the PCA
territory as a persistent fetal PCA The origin of the PCoA often exhibits a focal
enlargement, designated the infundibulum. CN III courses through the suprasellar
cistern close to the PCoA, so it is often affected by aneurysms of the PCoA. The anterior
thalamoperforating arteries arise from the PCoAs and course superiorly to supply
portions of the medial hypothalamus, thalamus, and lateral aspect of the third ventricle
Anterior choroid artery arises from the posterior aspect of C7 just above the PCoA. Its
long course is divided into three segments. The anterior choroid artery first courses
through the suprasellar cistern just medial to the uncus of the temporal lobe (cisternal
segment). It then turns laterally and passes through the choroid fissure to enter the
temporal horn of the lateral ventricle. Within the ventricle, the anterior choroid artery
supplies the choroid plexus and courses posterosuperiorly with the choroid plexus up to
and around the pulvinar of the thalamus. In its course, the anterior choroid artery
supplies the medial temporal lobe, the optic tract and lateral geniculate body, the dorsal
globus pallidus, the inferior half of the posterior limb of the internal capsule, the lateral
aspect of the cerebral peduncle, the tail of the caudate nucleus, and the choroid plexus
N.b. Multiple other systems for classifying carotid artery segments exist (e.g. Gibo/Rhoton, Fischer, Ziyal)

oculomotor nerve; b, trochlear nerve; c, abducens


ANSWERS 18–25 nerve; d, ophthalmic nerve; e, maxillary nerve; f,
Additional answers 18–25 available on
internal cerebral artery; cavernous segment;
ExpertConsult.com g, right optic nerve; h, pituitary gland.
Image from Naidich TP. Imaging of the Brain, Saunders,
Elsevier, 2013.

27. 1—e, 2—c, 3—f


EMI ANSWERS
Axial T2W MR images at the level of the facial
26. 1—g, 2—a, 3—c colliculi. a, abducens nucleus; b, superior cerebel-
lar peduncle (brachium conjunctivum); c, basal
Coronal scan through the cavernos sinus. MR turn of the cochlea, scala vestibule/scala tympani;
contrast-enhanced FIESTA sequence. a, d, osseous spiral lamina; e, anterior inferior

Neurosurgery Books Full


www.ketabpezeshki.com 66485438-66485457
1 NEUROANATOMY 15

cerebellar artery (AICA); f, cochlear nerve; g, Image adapted from Winn HR. Youman’s Neurological
second turn of cochlea; h, vestibule; i, lateral and Surgery, 4-Volume Set, 6th ed., Elsevier, Saunders,
posterior semicircular canals; j, inferior vestibular 2011.
nerve.
30. 1—c, 2—b, 3—a
Image from Naidich TP. Imaging of the Brain, Saunders,
Elsevier, 2013. a, Facial nerve; b, vestibulocochlear nerve; c, ante-
rior inferior cerebellar artery (AICA); d, abducens
28. 1—b, 2—m, 3—g nerve; e, superior vestibular nerve; f, cochlear nerve;
g, inferior vestibular nerve; h, cerebellum.
a, MCA; b, ACA; c, circular sinus; d, Dura pro-
pria; e, Periosteal dura; f, ICA; g, Oculomotor Image with permission from Naidich TP. Imaging of the
nerve (III); h, inner membranous layer; i, Medial Brain, Saunders, Elsevier, 2013.
temporal lobe dura; j, IV; k, VI; l, V1; m, Maxil-
lary division of CN V (V2). 31. 1—a, 2—i, 3—c, 4—f, 5—b

Image adapted from Yousem DM, Grossman RI. Neurora- a, Caudate nucleus; b, Putamen; c, Globus palli-
diology: The Requisites, 3rd ed., Mosby, Elsevier, 2010. dus (External segment); d, Globus pallidus (Inter-
nal segment); e, Substantia innominate; f, Internal
29. 1—q, 2—o, 3—b, 4—l capsule; g, External capsule; h, Extreme capsule;
i, Claustrum; j, Amygdala; k, Hippocampus; l,
a, Labyrinthine segment; b, Greater superficial Thalamus.
petrosal nerve; c, Cochlea; d, Geniculate gan-
glion; e, Stapes; f, Malleus; g, Incus; h, Tympanic Image adapted with permission from Crossman A. Neu-
segment of facial nerve; i, Vertical (mastoid) roanatomy: An Illustrated Colour Text, 5th ed., Churchill
Livingstone, Elsevier, 2015.
segment of facial nerve; j, Stylomastoid foramen;
k, Horizontal (lateral) semicircular canal; l, Pos-
terior semicircular canal; m, Superior semicircu- 32. 1—i, Supraopticohypophyseal tract; 2—j,
lar canal; n, Inferior vestibular nerve; o, Superior Tuberoinfundibular (tuberohypophyseal)
vestibular nerve; p, Internal auditory canal; tract; 3—g, Nucleus of the diagonal band
q, Facial nerve; r, Meatal foramen. of Broca (vertical limb); 4—c, Median fore-
brain bundle; 5—f, Postcommissural Fornix

Projection and Association Tracts

A. Central tegmental tract Connects rostral solitary nucleus (gustatory) to medial thalamic VPM
and red nucleus to inferior olive
B. Lamina terminalis Closed rostral end of the neural tube
C. Median forebrain bundle Connects septal area, hypothalamus, basal olfactory areas,
hippocampus/subiculum to midbrain, pons and medulla
D. Stria medullaris Connects the septal area, hypothalamus, olfactory area and anterior
thalamus to the habenulum
E. Stria terminalis Amygdala to hypothalamus
F. Postcommissural Fornix Hippocampus to cingulate gyrus
G. Nucleus of the diagonal band of Septal nuclei to hippocampus
Broca (vertical limb)
H. Retinohypothalamic tract Retinal ganglion cells to suprachiasmatic nuclei and other
hypothalamic nuclei (circadian rhythm)
I. Supraopticohypophyseal tract Supraoptic/paraventricular nuclei to neurohypophysis (posterior
pituitary)
J. Tuberoinfundibular Neuroendocrine neurons from arcuate nucleus to hypophyseal portal
(tuberohypophyseal) tract system (release dopamine and growth hormone releasing hormone
into portal blood which cause anterior pituitary to release prolactin
and growth hormone respectively)
K. Trapezoid body Ventral cochlear nuclei to contralateral superior olive
Continued on following page

Neurosurgery Books Full


www.ketabpezeshki.com 66485438-66485457
16 PART I BASIC SCIENCE

Projection and Association Tracts (Continued)


L. Thalamic fasciculus (Forel’s field Combination of ansa lenticularis, lenticular fasciculus and
H1) cerebellothalamic tract to VA/VL thalamus
M. Nucleus of the Diagonal band of Connects septal nuclei to amygdala
Broca (horizontal limb)
N. Mammillothalamic tract Mammillary body to anterior thalamic nuclei
O. Tapetum Corpus callosum fibers connecting temporal to occipital lobes

33. 1—j, 2—e, 3—n, 4—c, 5—m Parasympathetic motor efferents—This system is
divided into cranial and sacral components, and
34. 1—i, 2—q, 3—b, 4—g, 5—f parasympathetic efferents only synapse with post-
ganglionic cells close to or within target viscera
a, Thalamostriate and choroid veins; b, Superior (allowing local discrete responses).
anastomotic vein; c, Superior sagittal sinus; d, Cranial outflow:
Inferior sagittal sinus; e, Internal cerebral vein; • Edinger Westphal nucleus (midbrain)—
f, Great cerebral vein (Vein of Galen); g, Basal CN III (preganglionic fibers)—ciliary gan-
vein; h, Straight sinus; i, Inferior anastomotic glion—postganglionic fibers to ciliary mus-
vein; j, Transverse vein; k, Occipital sinus; l, Sig- cle, sphincter pupillae
moid sinus; m, Inferior petrosal sinus; n, Superior • Superior salivatory nucleus (pons)—CN
petrosal sinus; o, Cavernous sinus; p, Deep mid- VII branches A) nervus intermedius via
dle cerebral vein; q, Superficial middle cerebral greater petrosal nerve to pterygopalatine
vein; r, Anterior cerebral vein. ganglion and lacrimal gland and B) chorda
tympani and lingual nerve to submandibu-
Image adapted with permission from Mancall EL. lar ganglion for secretomotor to subman-
Gray's Clinical Neuroanatomy: The Anatomic Basis
for Clinical Neuroscience, Elsevier, Saunders, 2011. dibular/sublingual glands
• Inferior salivatory nucleus (medulla)—CN
35. 1—h, Posterior inferior cerebellar artery; IX via lesser petrosal nerve to otic ganglion
2—g, Superior cerebellar artery; then postganglionic in auriculotemporal
3—d, Anterior inferior cerebellar artery; nerve to parotid gland
4—g, Internal carotid artery; 5—k, Posterior • Dorsal nucleus (medulla)—CN X to the
communicating artery plexuses on the walls of respiratory, cardiac
and abdominal viscera
36. 1—d, 2—e, 3—p, 4—n, 5—d
37. 1—c, 2—a, 3—g, 4—i, 5—k

Projection and Association Tracts

A. Ansa lenticularis Globus pallidus interna around IC to thalamus


B. Fasciculus retroflexus Habenulum to midbrain and interpeduncular nuclei
C. Lenticular fasciculus Globus pallidus interna through IC to thalamus
(Forel’s field H2)
D. Postcommissural Fornix Hippocampus to cingulate gyrus
E. Precommissural Fornix Hippocampus to septal nuclei, hypothalamus, mammillary bodies, anterior
thalamus
F. Thalamic fasciculus Combination of ansa lenticularis, lenticular fasciculus and cerebellothalamic
(Forel’s field H1) tract to VA/VL thalamus
G. Diagonal band of Broca Connects septal nuclei to amygdala
H. Mammillothalamic tract Mammillary body to anterior thalamic nuclei
I. Tapetum Corpus callosum fibers connecting temporal to occipital lobes
Continued

Neurosurgery Books Full


www.ketabpezeshki.com 66485438-66485457
1 NEUROANATOMY 17

J. Uncinate fasciculus Anterior temporal lobe to orbitofrontal gyrus


K. Commissure of Probst Dorsal nucleus of lateral lemniscus to inferior colliculus
L. Central tegmental tract Connects rostral solitary nucleus (gustatory) to medial thalamic VPM and red
nucleus to inferior olive
M. Lamina terminalis Closed rostral end of the neural tube
N. Median forebrain bundle Connects septal area, hypothalamus, basal olfactory areas, hippocampus/
subiculum to midbrain, pons and medulla
O. Stria medullaris Connects the septal area, hypothalamus, olfactory area and anterior
thalamus to the habenulum

38. 1—j, 2—b, 3—g, 4—l, 5—k 39. 1—m, Pulvinar; 2—k, Lateral geniculate; 3—
a, Anterior; 4—l, Medial geniculate; 5—i,
a, Anterior nucleus; b, Ventral anterior nucleus; c, Ventral posteromedial
Lateral dorsal nucleus; d, Ventral lateral nucleus
(oral part); e, Ventral lateral nucleus (caudal part); a, Anterior (A); b, Ventral anterior (VA); c, Dor-
f, Lateral posterior nucleus; g, Ventral posterolat- somedial (DM); d, Lateral dorsal (LD); e, Ventral
eral and ventral posteromedial nuclei; h, lateral (VL); f, Lateral Posterior (LP); g, Ventral
Dorsomedial nucleus (Magnocellular); i, Dor- posterolateral (VP); h, Centromedian (CM); i,
somedial nucleus (Parvicellular); j, Pulvinar; k, Ventral posteromedial (VPM); j, Parafascicular
Medial geniculate nucleus; l, Lateral geniculate (PF); k, Lateral geniculate (LGB); l, Medial
nucleus. geniculate (MGB); m, Pulvinar (P).

Image adapted from Haines DE. Fundamental Neurosci- Image adapted from Haines DE. Fundamental Neurosci-
ence for Basic and Clinical Applications, 4th ed., Saun- ence for Basic and Clinical Applications, 4th ed., Saun-
ders, Elsevier, 2013. ders, Elsevier, 2013.

Thalamic Nuclei: Inputs, Outputs and Function


Type Nucleus Specific Inputs Output Function
Relay Anterior Mammillothalamic tract, Cingulate gyrus Limbic (emotion and
hippocampus memory)
Lateral dorsal Hippocampus (fornix) Cingulate gyrus Limbic (memory)
(LD)
Ventral Globus pallidus, SN, Premotor (area 6) and Planning of movement
anterior, (VA) cerebellum PFC (cortico-subcortico-cortical
loop)
Ventral lateral Cerebellum, GP, SN Primary motor (area 4) Initiation of movement
(VL) and premotor (area 6) (cortico-cerebello-cortical
loop)
Ventral Cerebellum Primary motor cortex Coordination of movement
intermediate (area 4)
(VIM)
Ventral Medial lemniscus (body), Somatosensory cortex Somatosensory (body)
posterolateral spinothalamic tract (body) (area 1-3)
(VPL)
Ventral Medial lemniscus (face), Somatosensory cortex Somatosensory (head and
posteromedial spinothalamic tract (face) (area 1-3) taste)
(VPM) Central tegmental tract Insula
(taste)
Medial Brachium of the inferior Auditory cortex (area Auditory relay
geniculate colliculus 41,42)
(MGN)
Continued on following page

Neurosurgery Books Full


www.ketabpezeshki.com 66485438-66485457
18 PART I BASIC SCIENCE

Thalamic Nuclei: Inputs, Outputs and Function (Continued)


Type Nucleus Specific Inputs Output Function
Lateral Optic tract Visual cortex (Area 17) Visual relay
geniculate
(LGN)
Association Dorsomedial Prefrontal cortex, Prefrontal cortex Limbic (Emotional
(DM) olfactory and limbic response to pain and
structures memory)
Lateral Unknown Parietal association Unknown
posterior (LP) cortex
Pulvinar Parietal, occipital, and Parietal-occipital- Visual association
temporal lobes temporal association
cortex
Regulatory Reticular Thalamus and cortex All thalamic nuclei Attention
Centromedian Brainstem Putamen and motor Attention and arousal
(CM) cortex
Parafascicular Brainstem Caudate nucleus and Attention and arousal
PFC

40. 1—j, 2—d, 3—e, 4—c, 5—f

Projection and Association Tracts

A. Inferior collicular commissure Connects the inferior colliculi


B. Cingulate fasciculus Cingulate gyrus to entorhinal cortex
(cingulum bundle)
C. Arcuate fasciculus Wernicke’s area to Broca’s (temporal, parietal and frontal)
D. Corpus callosum Connects both hemispheres. Covered with indusium griseum
(supracallosal gyrus) which is a thick layer of unmyelinated fibers
arranged as medial and lateral longitudinal striae of Lanscisi
E. Posterior commissure Crossing fibers from pretectal nucleus for the consensual light reflex
F. Hypothalamospinal tract Hypothalamus to ciliospinal center in intermediolateral column of T1-T2
spinal cord
G. Brachium conjunctivum AKA superior cerebellar peduncle
H. Brachium pontis AKA middle cerebellar peduncle
I. Restiform and juxtarestiform AKA inferior cerebellar peduncle
bodies
J. Dorsal longitudinal fasciculus Periventricular hypothalamus/mammillary bodies to midbrain central
gray matter
K. Medial longitudinal fasciculus Optokinetic and vestibularocular reflexes and saccadic eye movements
Descending fibers: superomedial vestibular nuclei, tectospinal tract,
vestibulospinal tract
Ascending fibers: vestibular nucleus to nuclei for III, IV, VI
L. Uncinate fasciculus Anterior temporal lobe to orbitofrontal gyrus
M. Lamina terminalis Closed rostral end of neural tube
N. Commissure of Probst Dorsal nucleus of lateral lemniscus to inferior colliculus
O. Stria medullaris Septal area, hypothalamus, olfactory area and anterior thalamus to
habenulum

Neurosurgery Books Full


www.ketabpezeshki.com 66485438-66485457
1 NEUROANATOMY 19

41. 1—f, 2—g, 3—j commissure; p, Parahippocampal gyrus; q, Uncus;


r, Optic chiasm; s, Temporal pole; t, Lamina ter-
a, Massa intermedia; b, Third ventricle; c, Supra- minalis; u, Anterior commissure; v, Subcallosal
tectal cistern; d, Fourth ventricle; e, Pontomedul- area; w, Fornix; x, Frontal pole; y, Callosal sulcus;
lary cistern; f, Cisterna magna; g, Interpeduncular z, Cingulate gyrus; a1, Cingulate sulcus; b1, Supe-
cistern; h, Optic chiasma; i, Cistern of lamina ter- rior frontal gyrus; c1, Paracentral sulcus; d1, Ante-
minalis; j, Chiasmatic cistern; k, Basilar Artery; l, rior paracentral gyrus.
Prepontine cistern.
Image adapted with permission from Haines DE. Funda-
Image from Standring S (Ed.), Gray’s Anatomy, 40th mental Neuroscience for Basic and Clinical Applica-
ed., Churchill Livingstone, Elsevier, 2008. tions, 4th ed., Saunders, Elsevier, 2013.

42. 1—e, 2—m, 3—p, 4—f, 5—j 45. 1—h, 2—b, 3—n, 4—j, 5—k

a, Edinger-Westphal nucleus; b, Oculomotor a, Anterior paracentral gyrus; b, Paracentral sul-


nucleus; c, Trochlear nucleus; d, Trigeminal cus; c, Superior frontal gyrus; d, Cingulate sulcus;
motor nucleus; e, Abducens nucleus; f, Facial e, Cingulate gyrus; f, Rostrum of corpus callosum;
motor nucleus; g, Salivatory nuclei (superior); g, Body of corpus callosum; h, Central sulcus; i,
h, Salivatory nuclei (inferior); i, Dorsal vagal Posterior paracentral gyrus; j, Marginal sulcus;
motor nucleus; j, Nucleus ambiguous; k, Hypo- k, Precuneus; l, Parieto-occipital sulcus; m,
glossal nucleus; l, Trigeminal mesencephalic Cuneus; n, Calcarine sulcus; o, Tentorium
nucleus; m, Trigeminal main sensory nucleus; cerebelli.
n, Trigeminal spinal nucleus; o, Dorsal cochlear
nucleus; p, Nucleus of tractus solitaries. Image adapted with permission from Haines DE. Funda-
mental Neuroscience for Basic and Clinical Applica-
tions, 4th ed., Saunders, Elsevier, 2013.
Image adapted from Mancall EL. Gray's Clinical Neuro-
anatomy: The Anatomic Basis for Clinical Neurosci-
ence, Elsevier, Saunders, 2011. 46. 1—l, 2—d, 3—p, 4—c, 5—g

43. 1—h, 2—d, 3—u, 4—y, 5—g a, Anterior medullary velum; b, Middle cerebellar
peduncle; c, Median sulcus of rhomboid fossa; d,
a, Central sulcus; b, Post central gyrus; c, Post- Striae medullares; e, Foramen of Luschka; f,
central sulcus; d, Supramarginal gyrus; e, Supe- Hypoglossal trigone; g, Vagal trigone; h, Tela
rior parietal lobule; f, Intraparietal sulcus; g, choroidea (cut edge); i, Gracile tubercle; j, Supe-
Parieto-occipital sulcus; h, Angular gyrus; i, rior cerebellar peduncle; k, Medial eminence of
Superior occipital gyrus; j, Inferior occipital fourth ventricle; l, Facial colliculus; m, Superior
gyrus; k, Preoccipital notch; l, Middle temporal fovea; n, Vestibular area; o, Lateral recess; p, Sul-
sulcus; m, Inferior temporal sulcus; n, Middle cus limitans; q, Restiform body; r, Inferior fovea;
temporal gyrus; o, Superior temporal sulcus; p, s, Cuneate tubercle.
Superior temporal gyrus; q, Lateral sulcus; r,
Image adapted with permission from Haines DE. Funda-
Temporal pole; s, Orbital surface; t, Inferior fron- mental Neuroscience for Basic and Clinical Applica-
tal gyrus (Pars triangularis); u, Inferior frontal tions, 4th ed., Saunders, Elsevier, 2013.
gyrus (Pars opercularis); v, Inferior frontal gyrus
(Pars orbitalis); w, Frontal pole; x, Inferior frontal 47. 1—a, 2—c, 3—d, 4—f, 5—i
sulcus; y, Middle frontal gyrus; z, Superior frontal
gyrus; a1, Superior frontal sulcus; b1, Precentral a, Red nucleus; b, Accessory nucleus; c, Edinger-
sulcus; c1, Precentral gyrus. Westphal preganglionic nucleus; d, Oculomotor
nucleus; e, Mesencephalic nucleus; f, Trochlear
Image adapted with permission from Haines DE. Funda-
mental Neuroscience for Basic and Clinical Applica- nucleus; g, Principal sensory nucleus; h, Trigem-
tions, 4th ed., Saunders, Elsevier, 2013. inal motor nucleus; i, Abducens nucleus; j, Inter-
nal genu of facial nerve; k, Superior salivatory
44. 1—c, 2—h, 3—f, 4—n, 5—t nucleus; l, Spinal trigeminal nucleus, pars oralis;
m, Facial motor nucleus; n, Inferior salivatory
a, Central sulcus; b, Posterior paracentral gyrus; c, nucleus; o, Solitary nucleus (and tract); p, Dorsal
Marginal sulcus; d, Precuneus; e, Parieto-occipital motor vagal nucleus; q, Nucleus ambiguous; r,
sulcus; f, Cuneus; g, Hippocampal commissure; h, Hypoglossal nucleus; s, Solitary nucleus (and
Calcarine sulcus; i, Occipital pole; j, Lingual gyrus; tract); t, Spinal trigeminal nucleus, pars caudalis;
k, Isthmus of cingulate gyrus; l, Pineal; m, Occipi- u, Substantia gelatinosa (spinal lamina II).
totemporal gyri; n, Collateral sulcus; o, Posterior

Neurosurgery Books Full


www.ketabpezeshki.com 66485438-66485457
20 PART I BASIC SCIENCE

Image adapted with permission from Haines DE. Funda- and fourth ventricle; n, Prepositus nucleus; o,
mental Neuroscience for Basic and Clinical Applica- Medial longitudinal fasciculus; p, Tectobulbosp-
tions, 4th ed., Saunders, Elsevier, 2013. inal system; q, Inferior salivatory nucleus; r, Sol-
itary tract and nucleus; s, Reticular formation; t,
48. 1—b, 2—q, 3—f, 4—p Rubrospinal tract; u, Anterolateral system; v,
Nucleus ambiguous; w, Posterior accessory oli-
a, Gracile fasciculus; b, Gracile nucleus; c, Cuneate vary nucleus; x, Medial accessory olivary nucleus;
fasciculus; d, Cuneate nucleus; e, Spinal trigemi- y, Medial lemniscus.
nal: Nucleus (pars caudalis); f, Spinothalamic tract;
g, Internal arcuate fibers; h, Reticular formation; i, Image adapted with permission from Haines DE. Funda-
Lateral reticular nucleus; j, Hypoglossal nerve; k, mental Neuroscience for Basic and Clinical Applica-
tions, 4th ed., Saunders, Elsevier, 2013.
Pyramid (corticospinal fibers); l, Central canal;
m, Central gray; n, Solitary nucleus and tract; o,
Dorsal motor vagal nucleus; p, Posterior spinocer- 51. 1—g, 2—h, 3—j, 4—p, 5—z
ebellar fibers; q, Accessory cuneate nucleus; r,
Hypoglossal nucleus; s, Nucleus ambiguous; t, a, Medial longitudinal fasciculus; b, Tectobul-
Anterolateral system; u, Medial longitudinal fas- bospinal system; c, Internal genu of facial nerve;
ciculus; v, Tectobulbospinal system; w, Principal d, Superior salivatory nucleus; e, Spinal trigemi-
olivary nucleus; x, Medial accessory olivary nal: Tract; f, Spinal trigeminal: Nucleus (pars ora-
nucleus; y, Medial lemniscus. lis); g, Facial nucleus; h, Facial nerve; i,
Rubrospinal tract; j, Superior olivary nucleus; k,
Image adapted with permission from Haines DE. Funda- Central tegmental tract; l, Transverse pontine
mental Neuroscience for Basic and Clinical Applica- fibers (pontocerebellar); m, Corticospinal fibers;
tions, 4th ed., Saunders, Elsevier, 2013. n, Nucleus raphe magnus; o, Facial colliculus;
p, Abducens nucleus; q, Inferior cerebellar
49. 1—p, 2—n, 3—c, 4—x, 5—z peduncle: Juxtarestiform body; r, Inferior cere-
bellar peduncle: Restiform body; s, Vestibular
a, Medial vestibular nucleus; b, Inferior vestibular nuclei: Superior; t, Vestibular nuclei: Medial; u,
nucleus; c, Reticular formation; d, Hypoglossal Vestibular nuclei: Lateral; v, Solitary tract and
nerve; e, Spinal trigeminal: Nucleus (pars interpo- nucleus; w, Anterolateral system; x, Anterior tri-
laris); f, Tract; g, Anterior spinocerebellar tract; h, geminothalamic tract; y, Medial lemniscus; z,
Lateral reticular nucleus; i, Hypoglossal nerve; j, Abducens nerve; a1, Pontine nuclei; b1,
Pyramid (corticospinal fibers); k, Choroid plexus Trapezoid body.
and fourth ventricle; l, Hypoglossal nucleus; m,
Sulcus limitans; n, Dorsal motor vagal nucleus; Image adapted with permission from Haines DE, Funda-
o, Accessory cuneate nucleus; p, Solitary tract mental Neuroscience for Basic and Clinical Applica-
tions, 4th ed., Saunders, Elsevier, 2013.
and nucleus; q, Restiform body; r, Medial longitu-
dinal fasciculus; s, Tectobulbospinal system; t,
Nucleus ambiguous; u, Rubrospinal tract; v, Ante- 52. 1—p, 2—l, 3—e, 4—o, 5—h
rolateral system; w, Posterior accessory olivary
nucleus; x, Principal olivary nucleus; y, Medial a, Tectobulbospinal system; b, Brachium con-
accessory olivary nucleus; z, Medial lemniscus. junctivum; c, Mesencephalic tract; d, Mesence-
phalic nucleus; e, Trigeminal nuclei: Principal
Image adapted with permission from Haines DE. Funda- sensory; f, Trigeminal nuclei: Motor; g, Reticulo-
mental Neuroscience for Basic and Clinical Applica- tegmental nucleus; h, Brachium pontis; i,
tions, 4th ed., Saunders, Elsevier, 2013. Rubrospinal tract; j, Transverse pontine fibers
(pontocerebellar); k, Pontine nuclei; l, Corticosp-
50. 1—e, 2—h, 3—f, 4—o, 5—v inal fibers; m, Nucleus raphe pontis; n, Medial
longitudinal fasciculus; o, Fourth ventricle; p,
a, Medial vestibular nucleus; b, Inferior vestibular Locus ceruleus; q, Anterior spinocerebellar
nucleus; c, Lateral recess of fourth ventricle; d, fibers; r, Lateral lemniscus; s, Anterolateral sys-
Cochlear nuclei: Anterior; e, Cochlear nuclei: tem; t, Central tegmental tract; u, Anterior trige-
Posterior; f, Spinal trigeminal: Nucleus (pars ora- minothalamic fibers; v, Medial lemniscus; w,
lis); g, Spinal trigeminal: Tract; h, Vestibuloco- Pontine nuclei.
chlear nerve; i, Glossopharyngeal nerve; j,
Anterior spinocerebellar tract; j, Principal olivary Image adapted with permission from Haines DE. Funda-
nucleus; k, Anterior trigeminothalamic fibers; l, mental Neuroscience for Basic and Clinical Applica-
tions, 4th ed., Saunders, Elsevier, 2013.
Pyramid (corticospinal fibers); m, Choroid plexus

Neurosurgery Books Full


www.ketabpezeshki.com 66485438-66485457
1 NEUROANATOMY 21

53. 1—p, 2—d, 3—k, 4—n, 5—e Image adapted with permission from Haines DE. Funda-
mental Neuroscience for Basic and Clinical Applica-
a, Periaqueductal gray; b, Mesencephalic tract tions, 4th ed., Saunders, Elsevier, 2013.
and nucleus; c, Locus ceruleus; d, Medial longitu-
dinal fasciculus; e, Tectobulbospinal system; f,
Anterior trigeminothalamic fibers; g, Rubrosp-
inal tract; h, Corticospinal fibers; i, Frenulum; ANSWERS 54–58
j, Fourth ventricle-cerebral aqueduct transition;
Additional answers 54–58 available on
k, Trochlear nerve; l, Posterior raphe nucleus; ExpertConsult.com
m, Brachium conjunctivum; n, Central tegmental
tract; o, Anterolateral system; p, Medial lemnis-
cus; q, Middle cerebellar peduncle; r, Pontine
nuclei; s, Central superior nucleus (of the raphe).

Neurosurgery Books Full


www.ketabpezeshki.com 66485438-66485457
CHAPTER 2

EMBRYOLOGY
SINGLE BEST ANSWER (SBA) QUESTIONS
1. Which one of the following correctly e. SHH/morphogen secretion on D14
describes the order of embryological stages causes the neural plate to form median
of CNS development? hinge points and start invaginating along
a. Blastogenesis, gastrulation, dorsal induc- its central axis to form a neural groove
tion, ventral induction, neural proliferation, (with neural folds on either side)
neuronal migration, and axonal myelination
b. Dorsal induction, ventral induction, gas- 4. Which one of the following statements about
trulation, neural proliferation, neuronal secondary neurulation and retrogressive dif-
migration, and axonal myelination ferentiation is most accurate?
c. Gastrulation ventral induction, dorsal a. Important for the formation of the conus
induction, neural proliferation, neuronal medullaris but not the filum terminale
migration, and axonal myelination b. Involves canalization of a caudal men-
d. Neural proliferation, gastrulation dorsal senchymal cell mass
induction, ventral induction, neuronal c. Is completed by days 24-26 of embryonic
migration, and axonal myelination development
e. Ventral induction, gastrulation, dorsal d. Responsible for the formation of thoracic,
induction, neural proliferation, axonal lumbar, sacral, and coccygeal neural tube
myelination, and neuronal migration e. Retrogressive differentiation is a mitotic
process
2. Which one of the following statements
regarding gastrulation is most accurate? 5. Which one of the following statements about
a. It is the process by which the bilaminar ventral induction is most accurate?
disc is converted into a trilaminar disc a. It includes development of the primary
b. It can result in lipomyelomeningocele if brain fissure
disturbed b. It includes development of the secondary
c. It is not dependent on bone morphoge- brain vesicles and brain flexures
netic protein expression c. It includes formation of the neural plate
d. It starts with closure of the cranial d. It includes formation of the notochord
neuropore e. It includes primary neurulation
e. It occurs from embryonic days 10-12
6. The disencephalon does not give rise to
3. Which one of the following statements about which one of the following?
primary neurulation is most accurate? a. 3rd ventricle
a. Anterior neuropore closure approxi- b. Mamillary bodies
mately occurs on D19 c. Optic vesicle
b. Disjunction results in formation of the spi- d. Posterior pituitary
nal canal below the posterior neuropore e. Superior colliculus
c. Fusion of the neural folds starts at the
anterior neuropore and proceeds caudally 7. Mesencephalon does NOT give rise to which
in a zip-like fashion until it reaches the one of the following?
posterior neuropore a. Cerebral aqueduct
d. Notochord induces the overlying ecto- b. Edinger-Westphal nucleus
derm to differentiate into a flat area of c. Pineal body
specialized neuroectoderm called the d. Red nucleus
neural plate e. Substantia nigra

22
Neurosurgery Books Full
www.ketabpezeshki.com 66485438-66485457
2 EMBRYOLOGY 23

8. Which one of the following statements about 13. Which one of the following is the first to
the rhombencephalon is most accurate? form in the developing brain?
a. It contains the cerebral aqueduct at a. Anterior commissure
its center b. Genu of corpus callosum
b. It gives rise to diencephalon and myelen- c. Hippocampal commissure
cephalon secondary brain vesicles d. Posterior commissure
c. It gives rise to the inferior colliculi and pons e. Splenium of corpus callosum
d. It is separated from the mesencephalon by
the isthmus rhombencephalii 14. Which one of the following statements about
e. Pontine flexure indents the rhomben- the developing spinal cord are most accurate?
cephalon ventrally a. Alar columns form the intermediolateral
horn
9. Which one of the following statements about b. Alar columns form the ventral horns
cerebellar development is most accurate? c. Dorsally the floor plate marks where the
a. Brainstem input to the cerebellum is via paired basal columns meet
parallel and climbing fibers d. Laterally, the alar and basal plates abut at a
b. Commences at week 15 groove called the sulcus limitans
c. Golgi cells come to reside in the e. Ventrally the roof plate marks where the
molecular layer paired alar columns meet
d. Granule cells develop axons called Mossy
fibers
e. Granule cells migrate inward past Pur- QUESTIONS 15–25
kinje cells with the help of Bergmann glia
Additional questions 15–25 available on
ExpertConsult.com
10. Which one of the following is important in
dorsoventral patterning of the neural tube?
a. BF-1
b. BMP-4 and BMP-7
c. EMX1 and EMX2 EXTENDED MATCHING ITEM (EMI)
d. FGF-8
e. HOX QUESTIONS
f. SHH
26. Embryological terms:
11. Which one of the following best describes a. Ectoderm
cells forming the mantle layer in the develop- b. Endoderm
ing neural tube? c. Induction
a. Ependymal cells d. Mesenchyme
b. Glioblasts e. Mesoderm
c. Neuroblasts f. Neural crest
d. Postmitotic young neurons g. Notochord
e. Radial cells h. Paraxial mesoderm
i. Primitive streak
12. Which one of the following statements about j. Sclerotome
cerebral cortex formation is most accurate? k. Somite
a. Cortical layers are laid down from most
superficial to deep For each of the following descriptions, select the
b. Germinal matrix zone is superficial to the most appropriate answers from the list above.
ventricular zone Each answer may be used once, more than once
c. Intermediate zone contains axons of cor- or not at all.
tical pyramidal neurons 1. Population of cells arising from the lateral
d. Migration of cortical pyramidal neurons lips of the neural plate that detach during
occurs tangentially formation of the neural tube and migrate
e. The neocortex usually has four layers in to form a variety of cell types/structures.
the adult 2. The first morphological sign of
gastrulation.

Neurosurgery Books Full


www.ketabpezeshki.com 66485438-66485457
24 PART I BASIC SCIENCE

3. The process in which one embryonic For each of the following descriptions, select the
region interacts with a second embryonic most appropriate answers from the list above.
region, thereby influencing the behavior Each answer may be used once, more than once
or differentiation of the second region. or not at all.
1. Origin of neural crest cells
27. Central nervous system formation: 2. Failure of closure results in spina bifida
a. Diencephalon 3. Structure signaling to midline ectoderm to
b. Mescencephalon form neural tube
c. Metencephalon 4. Formed by neural crest cells
d. Myelencephalon
e. Prosencephalon 30. Pharyngeal arch derivatives:
f. Rhombencephalon a. 1st pharyngeal arch
g. Telencephalon b. 2nd pharyngeal arch
c. 3rd pharyngeal arch
For each of the following descriptions, select the d. 4th pharyngeal arch
most appropriate answers from the list above. e. 5th pharyngeal arch
Each answer may be used once, more than once f. 6th pharyngeal arch
or not at all. g. Ductus thyroglossus
1. Contains cerebral aqueduct h. Foramen caecum
2. Gives rise to the cerebellar hemispheres i. Sinus cervicalis
j. Tuberculum impar
28. Embryology: k. Tuberculum laterale
a. Days 2-3
b. Days 4-5 For each of the following descriptions, select the
c. Day 6 most appropriate answers from the list above.
d. Days 8-12 Each answer may be used once, more than once
e. Days 14-17 or not at all.
f. Day 18 1. Common carotid and internal carotid
g. Day 20 artery and glossopharyngeal nerve
h. Days 24-26 2. Recurrent laryngeal branch of CN X
i. Days 26-28 3. Parts of CN V2 and V3
j. Day 31 4. Facial nerve
k. Day 35
l. Day 42 31. Disorders of CNS development:
a. Adrenoleukodystrophy
For each of the following descriptions, select the b. Caudal regression syndrome
most appropriate answers from the list above. c. Dandy-Walker spectrum
Each answer may be used once, more than once d. Heterotopia
or not at all. e. Intradural lipoma
1. Formation of the neural plate f. Lipoma of filum terminale
2. Closure of the posterior neuropore g. Pelizaeus-Merzbacher disease
3. Five secondary brain vesicles h. Schizencephaly
i. Segmental spinal dysgenesis
29. Neurulation: j. Split cord malformation
a. Alar plate k. Sturge-Weber syndrome
b. Basal plate l. Terminal myelocystocele
c. Caudal neuropore
d. Cranial neuropore For each of the following descriptions, select the
e. Dorsal root ganglion most appropriate answers from the list above.
f. Neural fold Each answer may be used once, more than once
g. Neural groove or not at all.
h. Notochord 1. Disorder of neural proliferation
i. Primary neurulation 2. Disorder of notochordal integration during
j. Primitive node gastrulation
k. Primitive streak 3. Disorder of ventral induction
l. Secondary neurulation

Neurosurgery Books Full


www.ketabpezeshki.com 66485438-66485457
2 EMBRYOLOGY 25

SBA ANSWERS
1. a—Blastogenesis, gastrulation, dorsal induc- developing structures, including the cortex, hip-
tion, ventral induction, neural proliferation, pocampus and the cerebellum set the stage for
neuronal migration, and axonal myelination. differential periods of vulnerability to insults in
a regionally specific manner. Timings for individ-
Below is a simplified timeline of neural develop- ual events vary between sources for events beyond
ment. It is worth noting that different brain ventral induction, but the general sequence is as
regions have a unique course of ontogeny. Late follows:

Blastogenesis D1-13 Development from a fertilized embryo into a bilaminar blastocyst


implanted in the uterus with amniotic and yolk sacs

Gastrulation D14-17 It is the process by which the bilaminar disc is converted into a
trilaminar disc, including integration of bilateral notochordal
anlagen into a single notochordal process and segmental notochordal
formation

Dorsal induction D17-28 (3rd-4th Formation and closure of neural tube (primary neurulation).
week) Development of three primary brain vesicles, and two flexures
(D21 mesencephalic and D28 cervical)

Secondary D28-D48 Formation of the neural tube caudal to the posterior neuropore
neurulation (below S2/3) from mesenchyme by cavitation

Ventral induction 5th-10th week Existing three primary brain vesicles (prosencephalon,
mesencephalon, and rhombencephalon) differentiate into five vesicles
(telencephalon, diencephalon, mesencephalon, metencephalon, and
myelencephalon) and subsequently forebrain, midbrain, and hindbrain
structures. Pontine flexure forms on D32

Neural 6-12 weeks Neuroblasts (primitive neurons) proliferate in the subependymal


proliferation zone of the neural tube adjacent to the central canal of the spinal cord or
the ventricles of the brain. Glio

Migration 8 weeks- Neuroblasts become neurons which then use radial glial cell fibers
(histogenesis) as scaffolds to reach their eventual destination in cortex or
subcortical nuclei. Radial glial cells also have a progenitor
function in the late stages of neurogenesis—their asymmetric division
produces a new radial glial cell and a postmitotic neuron

Axonal/dendritic 16 weeks- Neuronal arborization and branching in an attempt to establish


outgrowth appropriate connections

Apoptosis 18 weeks- Approximately 50% of all neurons are eliminated before birth to
allow dramatic morphological rearrangements to increase efficiency of
synaptic transmission (a second wave of
overproduction and elimination occurs later in life during
periadolescence)

Synaptogenesis 20 weeks- Formation of synapses as part of CNS maturation, second wave


occurs in periadolescence

Myelination 6 months- Axons become insulated with myelin sheaths allowing rapid
adulthood transmission of action potentials between nodes of
Ranvier. Completion of myelination marks maturity of the
nervous system

Synaptic 12 months Competitive elimination of synapses during functional development


elimination/ postnatally of each brain region, e.g., 40% reduction in fontal cortex synaptic
pruning onwards density between 7 and 15 years of age

Neurosurgery Books Full


www.ketabpezeshki.com 66485438-66485457
26 PART I BASIC SCIENCE

2. a—It is the process by which the bilaminar of specialized neuroectoderm (neural


disc is converted into a trilaminar disc. plate). Relative to Hensen's node, the neu-
ral plate expands cranially and narrows/
Gastrulation occurs between D14 and D17, and is elongates the parts on either side of the pri-
the process by which the bilaminar disc (consisting mary streak—these areas will form the
of epiblast facing the amniotic cavity and the hypo- brain and spinal cord, respectively. This
blast facing the yolk sac) becomes a trilaminar disc process is regulated by multiple genes,
with formation of an intervening third layer, the including brachyury and Wnt.
mesoblast (future mesoderm). On day 14 or 15 a • SHH/morphogen secretion on D18 causes
strip of thickened epiblast/ectoderm (primitive the neural plate to form median hinge
streak) appears caudally in the midline of the dorsal points and start invaginating along its cen-
surface of the embryo to define the craniocaudal tral axis to form a neural groove (with neu-
axis. The cranial end of the primitive streak forms ral folds on either side).
the primitive (Hendersen's) node, and shows a cen- • These folds progressively increase in size
tral depression called the primitive pit. Ectodermal and flex to approach each other, until they
cells start migrating towards the primitive streak, eventually fuse in the midline to form the
pass inward at the primitive pit to the interface of neural tube (regulated by PAX3 genes).
ectoderm and endoderm, and then migrate laterally Fusion occurs in a zip-like fashion, proba-
to form the mesoderm. The two paired notochordal bly at multiple sites but first at the level of
anlagen (primordia) then fuse in the midline to form the 4th somite (future craniocervical
a single notochordal process (“notochordal integra- junction).
tion”; D16). The primitive node defines the cranio- • The cranial end of the neural tube (anterior
caudal axis, the right and left sides and the dorsal and neuropore) closes first at the site of the lam-
ventral surfaces of the embryo. Prospective noto- ina terminalis on D24-26, followed by the
chordal cells in the wrong craniocaudal position posterior neuropore on D26-28 to com-
undergo apoptosis maintaining segmental noto- plete primary neurulation. Note that the
chordal formation. Multiple signaling molecules, posterior neural pore is not located at the
such as bone morphogenetic protein (BMP), fibro- caudal tip of the neural tube. The caudal
blast growth factor (FGF), and Wnt are essential for part of the spinal cord and the lowest
gastrulation to occur. BMP is very important in sacrum portion is formed from the solid
establishing the rostrocaudal polarity. In addition, core of neuroepithelium (tail bud) during
multiple factors and genes are implicated in pattern- secondary neurulation.
ing the primitive body axis (e.g., brachyury, sonic • Ectodermal cells progressively disconnect-
hedgehog (SHH), and HNF-beta genes). Defects ing from the lateral walls of the neural
in gastrulation (integration or segmental formation) folds during formation of the neural tube
affect development and differentiation of all three differentiate into the neural crest cells (form
primary cell layers and cause abnormalities from branchial arch derivatives, dorsal roots/
the occiput downwards, e.g., split cord malforma- dorsal root ganglia, autonomic ganglia and
tion (diastematomyelia and diplomyelia), neuren- adrenergic cells).
teric, dermoid, and epidermoid cysts, anterior and • Disjunction: Immediately after neural tube
posterior spina bifida, intestinal malformation, closure it becomes separated from the over-
duplication and fistula formation, and anterior lying superficial ectoderm (forms the skin)
meningocele. by dorsally migrating mesenchyme (forms
meninges, neural arches of the vertebrae
3. d—Notochord induces the overlying ecto- and paraspinal muscles).
derm to differentiate into a flat area of special-
ized neuroectoderm called the neural plate 4. b—Involves canalization of a caudal men-
senchymal cell mass
Dorsal induction (3rd-4th weeks; D17-D28)
includes primary neurulation, secondary neurula- The location of the caudal end of the neural plate
tion and formation of the “true” notochord. Pri- (posterior neuropore) is approximately at the S3
mary neurulation involves separation of level. The remaining caudal sacral and coccygeal
neuroectoderm in the neural plate from cutane- portions of the neural tube, including the conus
ous ectoderm to form the neural tube (brain medullaris and filum terminale are formed by sec-
and spinal cord) as far caudal as S2/3. The steps ondary neurulation and retrogressive differentia-
are summarized below: tion (days 28-48). During secondary neurulation,
• Neural induction and formation of the neu- a secondary neural tube is formed caudad to the
ral plate: the notochord induces the overly- posterior neuropore. A caudal cell mass of undif-
ing ectoderm to differentiate into a flat area ferentiated, totipotential cells initially appears as a

Neurosurgery Books Full


www.ketabpezeshki.com 66485438-66485457
2 EMBRYOLOGY 27

result of fusion of neural ectoderm with the lower appear dorsally and rostrally, which form the
portion of the notochord. Multiple small vacuoles cerebral hemispheres as the central cavities form
then appear in the caudal cell mass and progres- the lateral ventricles. The posterior part of the
sively coalesce to form a central canal (canaliza- prosencephalon becomes the diencephalon,
tion), which will merge with the canal formed which later develops into the thalami, hypothala-
during primary neurulation. Retrogressive differ- mus, epithalamus, optic cups, and neurohypoph-
entiation is an apoptotic process in which a com- ysis. The central cavity in the region of
bination of regression, degeneration and further diencephalon forms the third ventricle. Simulta-
differentiation of the caudal cell mass into the neously, two lateral outpouchings (optic vesicles)
tip of the conus medullaris, ventriculus termina- grow from the telencephalon on each side. These
lis, and filum terminale. optic vesicles form the retina and optic nerve.
Cells of the diencephalon and telencephalon
5. b—It includes development of the secondary originate from the germinal matrix lining of the
brain vesicles and brain flexures. third and lateral ventricles, respectively. The tel-
encephalon grows rapidly and covers the devel-
By the end of dorsal induction/primary neurulation oping diencephalon, midbrain and hindbrain,
the neural tube is closed and three primary brain because the outer regions grow more rapidly than
vesicles (prosencephalon, mesencephalon, and the floor. This growth of the cerebral hemi-
rhombencephalon) are present. During ventral spheres within the developing cranial cavity gives
induction (5th-10th weeks of gestation) the pri- the characteristic “C” shape to the developing lat-
mary brain vesicles differentiate into five secondary eral ventricles. The mesenchymal tissue trapped
brain vesicles by day 35 (telencephalon, dience- in the midline between the developing hemi-
phalon, mesencephalon, metencephalon, and mye- spheres develops into the cerebral falx.
lencephalon) which then form forebrain, midbrain,
and hindbrain structures. Between the 4th and 8th 7. c—Pineal body
weeks, the brain tube folds sharply at three loca-
tions. The first of these folds to develop is the The mesencephalon undergoes the least amount
cephalic flexure (between diencephalon and mes- of change during the expansion from three pri-
encephalon), followed by the cervical flexure mary to five secondary brain vesicles, and forms
between myelencephalon and spinal cord—both the midbrain. The central cavity decreases in size
flexures are ventral and produce an inverted U to form the aqueduct of Sylvius. The neuroblasts
shape. The last flexure is dorsally located between from the dorsal alar plates migrate and appear as
metencephalon and myelencephalon (pontine flex- two swellings that form the superior and inferior
ure) and changes the shape to an M. By the 8th colliculi (tectal plate). Some cells of the alar plate
week, deepening of the pontine flexure has folded also migrate ventrally to form the red nucleus and
the metencephalon (including the developing cer- substantia nigra. The basal plate of the mesen-
ebellum) back onto the myelencephalon. Any insult cephalon forms the midbrain tegmentum (which
during this phase affects the development of brain include the somatic and general visceral efferent
vesicles and the formation of the facial skeleton. columns, and crus cerebri).
Ocular and nasal anomalies are frequently associ-
ated with forebrain malformation because the optic 8. d—It is separated from the mesencephalon
placode and forebrain develop at the same time, by the isthmus rhombencephalii
with subsequent formation of the olfactory vesicle
1 week later. The commonly seen forebrain ventral With rapid growth of the embryonic brain, the neu-
induction malformations are (1) holoprosence- ral tube bends on itself in a zigzag fashion. Two flex-
phaly, (2) atelencephaly, (3) olfactory aplasia, (4) ures developed initially are the cephalic and the
agenesis of the corpus callosum, and (5) agenesis cervical flexures, and these are concave ventrally
of the septum pellucidum (septo-optic dysplasia, so the neural tube forms a wide upside-down
cavum vergae and pellucidum). Hindbrain anoma- U-shaped configuration. The mescencephalon
lies include vermian dysgenesis (e.g., Dandy- and rhombencephalon are separated by a constric-
Walker spectrum). tion (isthmus rhombencephalii). Around 6 weeks of
gestation, the pontine flexure develops dorsally
6. c—Optic vesicle between the two rhombencephalic vesicles—
metencephalon (future pons and cerebellum) and
The prosencephalon is the most rostral of the myelencephalon (future medulla). This flexure is
three brain vesicles and gives rise to a caudal dien- concave dorsally, thereby converting the shape of
cephalon and a rostral telencephalon. A pair of the developing neural tube into a broad “M” shape.
diverticula, known as the telencephalic vesicles, Hindbrain structures form as follows:

Neurosurgery Books Full


www.ketabpezeshki.com 66485438-66485457
28 PART I BASIC SCIENCE

• Pons—develops from a thickening in the choroid plexus and residual PMA (i.e., residual
floor and lateral walls of the metencephalon. rhombencephalic roof plate) form the definitive
• Medulla oblongata—develops from the tela choroidea of the 4th ventricle. Folding, trans-
thickened floor and lateral walls of the mye- verse fissure formation and foliation result in
lencephalon which is continuous inferiorly anterior lobe (cerebellar vermis and hemisphere
with the spinal cord. above primary fissure), posterior lobe (vermis
• Cerebellum—alar plates of the and rhombic and hemispheres below primary fissure) and a
lips of the metencephalon form the flocculonodular lobe.
cerebellum. Development of the cerebellar cortex and deep
nuclei (dentate, globose, emboliform, and fasti-
9. e—Granule cells migrate inward past Purkinje gial) occurs as follows:
cells with the help of Bergmann glia • Week 8—Metencephalon consists of typical
ventricular, mantle and marginal layers and
Development of the pontine flexure result in: rhombic lips have started to form the cere-
• The cranial and the caudal ends of the 4th bellum. The ventricular layer produces four
ventricle approximate together dorsally. types of neurons forming the mantle layer
• The rhombencephalic roof plate is folded which will subsequently migrate to the cor-
inward towards the cavity of the 4th tex: Purkinje cells, Golgi cells, basket cells,
ventricle. and stellate cells, as well as their associated
• The alar columns are splayed laterally glia (astrocytes including Bergmann glia,
because of the bending of the pons and and oligodendrocytes).
eventually lie dorsolateral to the basal • Week 12—Two additional layers form: an
columns. external germinal/granular layer derived
Therefore, the roof plate of the developing 4th from the rhombic lips, from which granular
ventricle remains thin, is wide at its fold/waist and cells migrate inwards to form a new internal
tapers superiorly and inferiorly (diamond shaped). germinal layer between the ventricular and
Mesenchyme inserts itself into the roof fold and marginal layers (cells of the mantle layer
forms the plica choroidalis (choroid plexus precur- have now dispersed into the marginal layer
sor) which divides the roof of the 4th ventricle into a where they will form a distinct cortical pat-
superior anterior membranous area (AMA) and tern). External germinal layer also produces
inferior posterior membranous area (PMA). The primitive nuclear neurons which also
alar laminae along the lateral margins of the migrate inwards to form the deep cerebellar
AMA become thickened to form two rhombic lips, nuclei. Migration of granule cells takes place
which enlarge to approach each other and fuse in along Bergman (radial) glia. Purkinje cells
the midline dorsally (covering the rostral half of migrate toward the cortex, it reels out an
the 4th ventricle and overlapping the pons and the axon that maintains synaptic contact with
medulla). As the rhombic lips grow to form the cer- neurons in the developing deep cerebellar
ebellar hemispheres and midline vermis, the AMA nuclei. These axons will constitute the only
regresses by incorporation into the developing efferents of the mature cerebellar cortex.
choroid plexus. Growth and backward extension • Week 15—From superficial to deep the cere-
of the cerebellum pushes the choroid plexus inferi- bellum consists of: external granular layer
orly, whereas the PMA greatly diminishes in the (persists until approximately 15 months post-
relative size compared with the overgrowing cere- natally), Purkinje cell layer, molecular layer
bellum. Subsequently there is development of a (stellate, basket cells), and granular layer
marked caudal protrusion of the 4th ventricle, caus- (Golgi cells; granule cells and their parallel
ing the PMA to expand as the finger of a glove. This fibers), white matter (Mossy fibers from brain-
Blake's pouch consists of ventricular ependyma sur- stem nulcei, climbing fibers from inferior
rounded by condensation of the mesenchymal tis- olivary nucleus) and deep cerebellar nuclei.
sues and is initially a closed cavity that does not
communicate with the surrounding subarachnoid 10. f—Sonic hedgehog
space of the cisterna magna. The network between
the vermis and the Blake's pouch progressively Some of the molecular signals patterning brain and
becomes condensed, whereas the other portions spinal cord development include homeobox-
about the evagination become rarified resulting in containing genes (e.g., HOX, PAX, OTX, EMX).
permeabilization of the Blake's pouch to form the A homeobox is a 180 bp DNA sequence found
foramen of Magendie. The foramina of Luschka within genes involved in anatomical development
also probably appear late into the 4th month of ges- (morphogenesis) and are important in establishing
tation. From superior to inferior, the residual AMA, body axes and cellular differentiation:

Neurosurgery Books Full


www.ketabpezeshki.com 66485438-66485457
2 EMBRYOLOGY 29

Homeobox-containing genes coding tran- (folding of the cerebral hemispheres will


scription factors: alter its position to subcortical).
• PAX3 and PAX7 are expressed by the entire After production of neurons is waning in the ven-
neural plate. tricular layer, this layer begins to produce a new
• Homeotic (HOX) genes control the body cell type, the glioblast which differentiate into glia
plan of an embryo along the craniocaudal of the CNS—astrocytes and oligodendrocytes.
axis, e.g., the rhombencephalon is divided Glia provide metabolic and structural support
into eight segments called rhombomeres, to the neurons of the central nervous system.
which are regulated by an overlapping The last cells produced by the ventricular layer
HOX gene expression. are the ependymal cells; these line the brain ven-
• Other homeobox genes are important in tricles and the central canal of the spinal cord and
establishment of forebrain and midbrain produce CSF.
boundaries and are expressed even before
the formation of neural fold (e.g., Lim1 12. c—Intermediate zone contains axons of cor-
and OTX2). Later, once the neural folds tical pyramidal neurons
and pharyngeal arches appear, additional
homeobox genes, including OTX1, The cerebral cortex is made up of several cell
EMX1, and EMX2, are expressed in an layers (or laminae) that vary in number from three
overlapping pattern to further specify the in the phylogenetically oldest parts to six in the
identity of these brain regions. dominant neocortex. Compared to the rest of
Other factors: the CNS, cerebral cortex has an “inside-out”
• Sonic hedgehog (SHH) is a protein arrangement of gray and white matter.
secreted by the notochord and floor plate • Proliferating cells of the ventricular layer
which downregulates the expression of undergo a series of regulated divisions to
PAX3 and PAX7 in the midline and ventral produce waves of neurons that migrate
half of the neural tube (dorsoventral peripherally (on radial cell processes span-
patterning). ning the full thickness of the cortex) and
• Wnt signaling pathway is active in the mid- establish the neuronal layers of the cortex.
line and dorsal half of the neural tube (dor- The first wave of neurons form a cortical
soventral patterning) and axon guidance. layer is termed the preplate.
• Bone morphogenetic protein (BMP-4 and • Axons extend from preplate cells back
BMP-7) are growth factors important in towards the ventricular zone producing an
dorsolateral patterning. They are secreted intermediate zone (white matter).
by the adjacent non-neural ectoderm, main- • As neurogenesis proceeds, new neurons
tain and upregulate PAX3 and PAX7 expres- are increasingly formed in an accessory ger-
sion in the dorsal half of the neural tube minative zone lying deep to the ventricular
which stimulates alar plate formation. zone, called the subventricular (germinal
• Fibroblast growth factor-8 is secreted by matrix) zone.
the anterior neural ridge (an organizing • Multiple cortical layers are laid down in a
center in the neural plate) which induces sequence from deep to superficial, that is, the
expression of the brain factor-1 (BF-1) tran- neurons of each wave migrate through the
scription factor that regulates the develop- preceding layers to establish a more superfi-
ment of the telencephalon. cial layer. This is thought to be mediated by
reelin (glycoprotein) secreted by transient
11. d—Postmitotic young neurons Cajal-Retzius cells which migrate to the
marginal layer (lamina I) tangentially after
Except in the telencephalon, neurogenesis estab- being born in a dorsal midline telencephalic
lishes the following architecture of the neural structure. As such, after normal cortical histo-
tube (from central to peripheral): genesis has been achieved in principle, only
1. Central canal. lamina II-VI persist in the adult.
2. Ventricular layer—neuroepithelial (radial) • As the production of neurons tapers off, the
cells which give rise to all other layers. ventricular layer gives rise to various kinds
3. Mantle layer—contains cell bodies of post- of glia and then to the ependyma.
mitotic young neurons which have
migrated laterally from the ventricular layer More numerous but smaller than the pyramidal
and will form eventual gray matter. neurons are the inhibitory interneurons—the gran-
4. Marginal layer—outermost layer contains ule cells, which originate in the ganglionic emi-
the axons of neurons in the mantle layer, nences of the ventral telencephalon and migrate
and will form eventual white matter dorsally into the cortex via a tangential route.

Neurosurgery Books Full


www.ketabpezeshki.com 66485438-66485457
30 PART I BASIC SCIENCE

13. a—Anterior commissure intermediolateral cell columns contain the visceral


motoneurons that constitute the central auto-
The commissures that connect the right and left nomic motoneurons of the sympathetic division,
cerebral hemispheres form from a thickening at whereas the intermediolateral cell columns in
the cranial end of the telencephalon, which rep- the sacral region contain the visceral motoneurons
resents the zone of final neuropore closure. This that constitute the central autonomic motoneu-
area can be divided into a dorsal commissural rons of the parasympathetic division.
plate and a ventral lamina terminalis:
• 7th week—anterior commissure forms in
the commissural plate and interconnects
the olfactory bulbs and olfactory centers ANSWERS 15–25
of the two hemispheres.
• 9th week—hippocampal (forniceal) com- Additional answers 15–25 available on
ExpertConsult.com
missure forms between the right and left
hippocampi.
• 9th week (late)—corpus callosum linking
together the right and left neocortices along
their entire length. The most anterior part
(the genu) of the corpus callosum appears
EMI ANSWERS
first, and its posterior extension (the sple-
nium) forms later in fetal life. 26. 1—f, Neural crest; 2—i, Primitive streak;
3—c, Induction (see table of definitions
14. d—Laterally, the alar and basal plates abut at below)
a groove called the sulcus limitans.

Cell bodies in subependymal zone: in the spinal


cord cells remain near the subependymal zone Term Definition
to form the central gray matter of the spinal cord
Ectoderm Outermost of the three primary
(mantle layer) and extend axonal processes toward germ layers, which forms the
the periphery of the spinal cord. neural tube, skin and pigmented
Axons (white matter) surrounds gray: the sur- cells
rounding spinal cord white matter is comprised of
Endoderm Inner most of the three primary
local and ascending white matter tracts generated germ layers, which forms the
in the spinal cord gray matter and descending respiratory tract, gastrointestinal
tracts from supranuclear sources. tract and appendages, urinary tract
Starting at the end of the 4th week, the neurons amongst others
in the mantle layer of the spinal cord become Induction The process in which one
organized into four plates that run the length of embryonic region interacts with a
the cord: a pair of dorsal (alar) columns and a pair second embryonic region, thereby
of ventral (basal) columns. Laterally, the two influencing the behavior or
plates abut at a groove called the sulcus limitans, differentiation of the second
region. Most, and perhaps all,
dorsally the roof plate and ventrally the floor plate tissues require inductive
(both non-neurogenic). The cells of the ventral interactions for normal
columns become the somatic motoneurons of development
the spinal cord and innervate somatic motor struc-
Mesenchyme Loosely associated embryonic cells
tures such as the voluntary (striated) muscles of derived from mesoderm that
the body wall and extremities. The cells of the differentiates into connective or
dorsal columns develop into association neurons hemopoietic tissue. This is in
receiving synapses from afferent (incoming) fibers distinction to epithelial cells,
which are tightly connected at
from the sensory neurons of the dorsal root gang- specific cell junctions, forming
lia, and either synapsing with ipsilateral/contralat- sheets or tubes
eral motoneurons to form a reflex arc or it may
ascend to the brain. The outgoing (efferent) motor Mesoderm One of the three germ layers,
it gives rise to the muscles
neuron fibers exit via the ventral roots. In most and skeleton of the body as well
regions of the cord—at all 12 thoracic levels, at as connective tissue, the
lumbar levels L1 and L2, and at sacral levels S2- reproductive and excretory
S4—the neurons in more dorsal regions of the organs and most of the
ventral columns segregate to form intermediolat- cardiovascular tissue
eral cell columns. The thoracic and lumbar Continued

Neurosurgery Books Full


www.ketabpezeshki.com 66485438-66485457
2 EMBRYOLOGY 31

Term Definition Term Definition


Neural crest Population of cells arising from Primitive The first morphological sign of
cells the lateral lips of the neural streak gastrulation. Made up of the
plate that detach during formation primitive pit, the primitive node,
of the neural tube and migrate to and the primitive groove
form a variety of cell types/
structures Sclerotome Portion of the somite that gives
rise to the vertebral column and
Notochord Rod of mesoderm that lies beneath ribs
the neural tube along the
central midline of the embryo. Somite Segmented mesodermal
Derived from the notochordal structures that first appear at
process. Involved in induction of about day 20 by segmentation
the neural tube and vertebral of the paraxial mesoderm.
bodies, induction of muscle in the Gives rise to the axial skeleton
somite, and establishment of of the trunk, all skeletal
dorsal-ventral polarity of the muscle of the trunk and limbs,
neural tube and the dermis of the skin of
the trunk
Paraxial Thick bands of embryonic
mesoderm mesoderm immediately adjacent
to the neural tube and notochord.
In the trunk, paraxial mesoderm
gives rise to somites

Continued

27. 1—b, Mesencephalon; 2—c, Metencephalon

Secondary
Primary Vesicles Vesicles
(Week 4) (Week 5) Adult Derivatives Ventricles CN
Prosencephalon Telencephalon Cortex—neocortex and hippocampus Lateral I
Striatum (caudate/putamen) ventricles
Globus pallidus
Amygdala
Olfactory bulbs

Diencephalon Alar plate—Thalamus, epithalamus and 3rd ventricle II


subthalamic nuclei
Basal plate—Midbrain tegmentum, pituitary
and hypothalamus
Roof plate—pineal body, choroid plexus

Mesencephalon Mesencephalon Roof/alar plate—Midbrain tectum (colliculi) Aqueduct III


Floor/basal plate—Midbrain tegmentum
Rhombencephalon Metencephalon Alar plate—Cerebellum 4th ventricle IV-VIII
Basal plate—Pons

Myelencephalon Medulla oblongata 4th ventricle IX-XII

Neurosurgery Books Full


www.ketabpezeshki.com 66485438-66485457
32 PART I BASIC SCIENCE

28. 1—f, Day 18; 2—h, Day 26-28; 3—k, Day 35 29. 1—f, neural fold; 2—c, caudal neuropore;
Approximate timetable for CNS development is 3—g, neural groove; 4—e, dorsal root
shown below (exact numbers vary depending on ganglion
source). 30. 1—c, 3rd pharyngeal arch; 2—f, 6th pharyn-
geal arch; 3—a, 1st pharyngeal arch; 4—b,
2nd pharyngeal arch
Stage Days Week
Morula 2-3 1st
Pharyngeal arches consist of a mesenchymal
Bilaminar disc 4-5 1st core (mesoderm and neural crest cells) that is
covered on the outside with ectoderm and lined
Implantation 6 1st
on the inside with endoderm. Each arch contains
Amniotic and yolk sacs formed 8-12 2nd (1) a central cartilaginous skeletal element
(derived from neural crest cells); (2) striated
Trilaminar disc with primitive streak 14-17 3rd muscle rudiments (derived from head meso-
Neural plate and groove 18 3rd derm) innervated by an arch-specific cranial
nerve; and (3) an aortic arch artery. Like so many
Appearance of three primary brain 20 3rd other structures in the body, the pharyngeal
vesicles
arches form in craniocaudal succession: the 1st
Anterior neuropore closure 24-26 4th arch forms on D22; the 2nd and 3rd arches form
sequentially on D24; and the 4th and 6th arches
Posterior neuropore closure 26-28 4th form sequentially on D29. The pharyngeal
(formation of neural tube)
arches of human embryos initially resemble
Anterior and posterior roots 31 4th-5th the gill arches of fish, except that they never
become perforated to form gill slits.
Five cerebral vesicles and 35 5th
developing flexures

Primordium of cerebellum 42 6th

Corpus callosum 10th

Cerebellar cortex and Purkinje cells 12th

Dentate nucleus 15th

Primary cerebral fissure 20th

Arch Nerve Artery Muscle Bone


1st (Mandibular) V2 and V3 Maxillary, Mastication Maxilla, mandible, zygoma, temporal
ECA Anterior belly of bone, incus, malleus, Meckel's
diagastric cartilage, sphenomandibular ligament
Mylohyoid
Tensor tympani
Tensor veli palatini

2nd (Hyoid) VII Stapedial, Muscles of facial Stapes, temporal styloid process,
hyoid expression lesser horn and upper body of hyoid,
Buccinator stylohyoid ligament, Reichert's
Platysma cartilage
Stapedius
Stylohyoid
Posterior belly of
diagastric
Auricular

3rd IX CC/ICA Stylopharyngeus Greater horn and lesser body of hyoid,


thymus, inferior parathyroids

Continued

Neurosurgery Books Full


www.ketabpezeshki.com 66485438-66485457
2 EMBRYOLOGY 33

Arch Nerve Artery Muscle Bone


4th X, superior Right: Cricothyroid Thyroid cartilage, superior
laryngeal subclavian Intrinsic muscle of parathyroids, epiglottic cartilage
Left: aortic soft palate (except
arch tensor veli
palatine)

6th X, recurrent Right: Intrinsic muscle of Cricoid cartilage, arytenoid cartilage,


laryngeal pulmonary larynx (except corniculate cartilage, cuneiform
Left: cricothyroid) cartilage
pulmonary
and ductus
arteriosus

31. 1—k, Sturge-Weber syndrome; 2—j, Split cord malformation; 3—c, Dandy-Walker spectrum

Gastrulation (notochordal integration) Gastrulation (segmental notochordal formation)

Dorsal enteric fistula Segmentation defects


Split cord malformation Indeterminate/block vertebrae
Neurenteric cyst Caudal regression syndrome
Dermal sinus* Segmental spinal dysgenesis

Primary neurulation Secondary neurulation

Myelomeningocele Lipoma of filum terminale (fatty filum)/conus


Myelocele Tight filum terminale
Lipomyelomeningocele Persistent terminal ventricle
Lipomyelocele Terminal myelocystocele
Intradural lipoma
Dermal sinus*

Ventral induction Neural proliferation

Holoprosencephaly Microencephaly
Septo-optic dysplasia Macroencephaly
Pituitary anomalies Neurocutaneous syndromes
Inferior cerebellar vermis hypoplasia Aqueduct stenosis
Dandy-Walker spectrum
Mega cisterna magna
Rhombencephalosynapsis
Joubert syndrome

Neuronal migration Myelination

Lissencephaly Adrenoleukodystrophy
Heterotopia Metachromatic leukodystrophy
Pachygyria Pelizaeus-Merzbacher disease
Schizencephaly
Polymicrogyria
Agenesis of corpus callosum

*Hemimyelocele and hemimyelomeningocele are considered disorders of both gastrulation and primary neurulation and are
extremely rare.

Neurosurgery Books Full


www.ketabpezeshki.com 66485438-66485457
CHAPTER 3

NEUROPHYSIOLOGY
SINGLE BEST ANSWER (SBA) QUESTIONS
1. Which one of the following is NOT a e. Basolateral membrane Na influx via
component of the blood-brain barrier? Na+/H+ exchange and Na+/HCO3 
a. Capillary endothelial cells cotransport channels.
b. Astrocytic foot processes
c. Basement membrane 5. Which one of the following statements
d. Tight junctions regarding axonal transport is LEAST
e. Microglia accurate?
a. Large membranous organelles are trans-
2. Which one of the following regions has an ported by fast kinesin dependent antero-
intact blood-brain barrier? grade transport and dynein dependent
a. Subforniceal organ retrograde transport
b. Area postrema b. Cytosolic proteins are transported by fast
c. Median eminence transport
d. Posterior pituitary c. Occurs by retrograde transport
e. Pineal gland d. Anterograde transport is dependent upon
f. Subcommissural organ microtubules and the ATPase kinesin
g. Organum vasculosum of lamina e. Rabies virus spreads by retrograde axonal
terminalis transport

3. Which one of the following statements 6. Which one of the following statements
regarding the area postrema is LEAST regarding the concentration of ions in extra-
accurate? cellular and intracellular compartments is
a. It is located in the dorsomedial medulla in LEAST accurate?
the caudal part of the fourth ventricle a. Extracellular sodium ion concentration is
b. Its blood supply is mostly from the ante- approximately 140 mM (140 mEq/l)
rior inferior cerebellar artery b. Intracellular potassium ion concentration
c. It is a circumventricular organ is approximately 160 mmol/l (160 mEq/L)
d. It plays a role as a chemoreceptor c. Extracellular chloride ion concentration is
trigger zone approximately 110 mM (110 mEq/l)
e. It expresses 5-HT3 receptors d. Intracellular calcium ion concentration is
approximately 2 mM (4 mEq/l)
4. Which one of the following statements e. Extracellular bicarbonate ion concentra-
regarding the production of CSF by choroid tion is approximately 22-26 mmol/l
plexus cells is LEAST accurate?
a. Requires ultrafiltration of plasma to form 7. Which one of the following statements con-
extracellular fluid at basolateral cerning the resting membrane potential is
membrane most accurate?
b. Formation is primarily generated by net a. Maintenance of the resting membrane
secretion of Na+, Cl, and HCO3  into potential is an energy dependent process
ventricles requiring Na/K-ATPase
c. Water is actively pumped into the ventri- b. A membrane is depolarized when there is
cles via Aquaporin 1 channels in the apical an increase in separation of the charge
membrane across it from baseline
d. Active transport of Na+ into the ventricles c. Neurons become depolarized when the
via Na+/K+ ATPase occurs at the basolat- charge inside the cell becomes more neg-
eral membrane ative compared to its resting state

34
Neurosurgery Books Full
www.ketabpezeshki.com 66485438-66485457
3 NEUROPHYSIOLOGY 35

d. Hyperpolarization of a cell membrane c. Length constant is greater in unmyelin-


occurs when the outside of the cell ated and large diameter axons
becomes more negatively charged com- d. The time constant is a function of the
pared to its resting state membrane’s resistance and capacitance
e. Resting potential difference across a e. The time constant characterizes how rap-
membrane is not dependent on the sepa- idly current flow changes the membrane
ration of charged ions across it potential

8. Which one of the following statements 12. Which one of the following statements
regarding ion channels is LEAST accurate? regarding the generation of the action poten-
a. Nicotinic AChR is a ligand-gated ion tial is LEAST accurate?
channel a. It is an all-or-nothing, regenerative wave
b. NMDA receptor is a ligand-gated cation of depolarization
channel b. It can propagate bidirectionally
c. Voltage-gated sodium channels open in c. Repolarization is due to inactivation of
response to hyperpolarization of the cell sodium channels combined with increased
membrane conductance in potassium channels
d. Cyclic AMP is generated by activation of d. Hyperpolarization occurs due to increases
beta-adrenoceptors in potassium conductance lasting beyond
e. GABA-B receptor is a ligand-gated ion the point of return to resting membrane
channel potential
e. Repolarization is required for inactivated
9. Which one of the following statements sodium channels to return to the closed state
regarding the membrane potentials is most
accurate? 13. Which one of the following sites acts as the
a. The Nernst equation can be used to calcu- trigger zone that integrates incoming signals
late the resting membrane potential of from other cells and initiates the action
a cell potential?
b. The Goldman equation can be used to a. Soma
calculate the intracellular concentration b. Dendritic shaft
of sodium c. Dendritic spines
c. The equilibrium potential for potassium d. Axon hillock and initial segment
is approximately +70 mV e. Axon trunk
d. Equilibrium potential of an ion maintains
a unique ion gradient for it exists across a 14. Which one of the following statements
cell membrane regarding phenomena relevant to action
e. At electrochemical equilibrium, the potential conduction is LEAST accurate?
chemical and electrical driving forces act- a. Accommodation is dependent on postsyn-
ing on an ion are equal and opposite, and aptic receptor phagocytosis
no further net diffusion occurs b. Saltatory conduction occurs to high resis-
tance to transmembrane current leak in
10. Which one of the following best describes ions myelinated segments of nerve
responsible for membrane hyperpolarization? c. Absolute refractory period is due to inac-
a. Chloride and sodium tivation of voltage-gated sodium channels
b. Chloride and potassium d. Relative refractory period occurs when
c. Potassium and sodium populations of inactivated voltage-gated
d. Sodium and calcium sodium channels return to the closed state
e. Sodium only e. Unidirectional propagation is function of
the refractory periods associated with
11. Which one of the following statements action potentials
regarding the passive membrane properties
of neurons is LEAST accurate? 15. Which one of the following synapse types is
a. The length constant is the distance where characterized by gap junctions?
the initial voltage response to current flow a. Axodendritic synapses
decays to 1/e (or 37%) of its value b. Axoaxonic synapses
b. Smaller length constant means passive c. Axosomatic synapses
flow of an action potential will stop at a d. Dendrodendritic synapses
shorter distance along an axon e. Electrical synapses

Neurosurgery Books Full


www.ketabpezeshki.com 66485438-66485457
36 PART I BASIC SCIENCE

16. Which one of the following statements d. Dopamine-Dopamine beta-hydroxylase-


regarding neurotransmission at chemical Norepinephrine
synapses is LEAST accurate? e. Tryptophan is converted to serotonin
a. The action potential stimulates the by tryptophan 5-hydroxylase and a
postsynaptic terminal to release decarboxylase
neurotransmitter
b. Release of the transmitter into the synaptic 20. Which one of the following statements
cleft by exocytosis is triggered by an influx regarding dopaminergic neurotransmission
of Ca2+ through voltage-gated channels is LEAST accurate?
c. Postsynaptic current produces an excit- a. Dopamine is loaded into synaptic vesicles
atory or inhibitory postsynaptic potential via a vesicular monoamine transporter
d. Neurotransmitters may undergo degrada- (VMAT)
tion in the synaptic cleft or be transported b. The neostriatum is the major site of dopa-
back into the presynaptic terminal minergic transmission in the brain
e. Vesicular membrane is retrieved from the c. Dopamine is derived from norepineph-
plasma membrane after exocytosis rine
d. Cocaine inhibits the Na-dependent dopa-
17. Which one of the following statements mine transporter
regarding cholinergic neurotransmission is e. monoamine oxidase (MAO) and catechol
LEAST likely? O-methyltransferase (COMT)
a. synthesized in nerve terminals from the
precursors acetyl coenzyme A 21. Which one of the following statements
b. acetylcholinesterase (AChE) hydrolysis regarding GABAergic neurotransmission is
Ach into acetate and choline LEAST accurate?
c. Nicotinic AChR are a nonselective cation a. Glutamic acid decarboxylase (GAD) con-
channel complex consisting of five subunits version of glutamate to GABA
arranged around a central membrane- b. Vitamin B6 is important in the function of
spanning pore glutamic acid decarboxylase
d. α-bungarotoxin binds to muscarinic c. The mechanism of GABA removal from
AChRs the synaptic cleft is similar to that for
e. mAChRs are metabotropic G-protein glutamate
coupled receptors d. GABAA and GABAC receptors are iono-
tropic receptors and are Ca2+ conductors
18. Which one of the following statements e. GABAB receptors are metabotropic and
regarding glutamatergic neurotransmission increase K conductance
is LEAST accurate?
a. At depolarized membrane potentials, an 22. Which one of the following areas does the
Mg2+ blocks the pore of the NMDA superior temporal gyrus (Heschl’s gyrus)
receptor primarily receive inputs from?
b. most prevalent precursor for glutamate a. Centromedian thalamic nucleus
synthesis is glutamine b. Medial geniculate thalamic nucleus
c. glutamine is taken up into presynaptic ter- c. Dorsomedial thalamic nucleus
minals and metabolized to glutamate by d. Anterior thalamic nucleus
the mitochondrial enzyme glutaminase e. Centromedian-parafascicular nucleus
d. Activation of metabotropic GluRs leads to
inhibition of postsynaptic Ca2+ and Na+ 23. Which one of the following cell types
channels involved in vision is able to generate an action
e. AMPA receptors are a type of metabo- potential?
tropic GluR a. Ganglion cells
b. Bipolar cells
19. Which one of the following enzymatic con- c. Horizontal cells
version pathways is LEAST accurate? d. Rods and cones
a. Tyrosine-tyrosine hydroxylase-DOPA e. Amacrine cells
(dihydroxyphenylalanine)
b. DOPA-catechol O-methyltransferase- 24. Which one of the following statements
dopamine regarding cones and rods is LEAST accurate?
c. Histidine-histidine decarboxylase- a. In the dark, rods have a high resting mem-
Histamine brane potential of about 70 mV

Neurosurgery Books Full


www.ketabpezeshki.com 66485438-66485457
3 NEUROPHYSIOLOGY 37

b. In the dark, both rods and cones tonically lingual nerve to the chorda tympani and
release glutamate onto synapsing finally into CN VII (facial)
bipolar cells d. Posterior one-third of the tongue detects
c. Photon absorption by rhodopsin results in bitter and sour tastes and signal through
reduced cyclic GMP and hyperpolariza- glossopharyngeal and vagus nerves
tion of the rod cell e. All taste fibers synapse in the nucleus
d. Photon absorption by cone opsin results ambiguus
in reduced cyclic GMP and hyperpolari-
zation of the rod cell 28. Which one of the following statements con-
e. Reduced glutamate secretion can cause cerning neurotransmission at the neuromus-
both hyperpolarization or depolarization cular junction is most accurate?
in bipolar cells a. It is dependent upon the release of norepi-
nephrine from the nerve ending
25. Which one of the following events during b. End plate potential amplitude can be
visual processing is LEAST accurate? much larger than that of excitatory or
a. The on-center bipolar depolarizes in inhibitory postsynaptic potentials
response to reduced tonic glutamate c. It is an all-or-none response
release d. It is not directly related to the concentra-
b. The on-center ganglion cell will produce tion of transmitter released from the pre-
a burst of action potentials if a spot of light synaptic terminals
is shone on the receptive field center e. It is dependent on the opening of ligand-
c. The ganglion cell that receives its input gated calcium channels
from an off-center bipolar cell will reduce
its firing rate in response a spot of light is 29. Which one of the following statements
shone on the receptive field center regarding peripheral nerve injury is most
d. The receptive fields of on-center and off- accurate?
center ganglion cells do not overlap a. Neuropraxia involves disruption of the
e. Glutamate released from a cone cell has myelin sheath only with some evidence
differential effect in different cells with of Wallerian degeneration
which it synapses b. Recovery after neuropraxia is likely to be
incomplete
26. Which one of the following statements about c. Neurotmesis is ideally managed with
olfaction is LEAST accurate? expectant management
a. Bowman glands secrete a fluid that bathes d. Axonotmesis shows Wallerian degenera-
the cilia of the receptors and acts as a sol- tion distal to injury
vent for odorant molecules e. A dense motor and sensory deficit fol-
b. Mucus-coated olfactory epithelium lines lowing a penetrating injury is due to
the anterodorsal parts of the nasal cavities neuropraxia
c. Binding of odor molecules generates
action potentials in a G-protein coupled 30. Which one of the following ensures sufficient
mechanism contraction of the striated portion of intrafu-
d. Fibers of CN I synapse with the mitral sal fibers to enable monitor changes in mus-
cells of the olfactory bulb cle length?
e. Olfactory tract and lateral olfactory stria a. Unmyelinated C fibers
project to the primary olfactory cortex b. 1A fibers
and amygdala c. Gamma motor neurons
d. Alpha motor neurons
27. Which one of the following statements e. General visceral efferent fibers
regarding taste sensation is LEAST accurate?
a. Receptors for molecules associated with
sweet and bitter tastes utilize second QUESTIONS 31–45
messengers
Additional questions 31–45 available on
b. Sour and salty-tasting molecules act ExpertConsult.com
directly upon the ion channels
c. Taste buds on the anterior two thirds of
the tongue send signals through the

Neurosurgery Books Full


www.ketabpezeshki.com 66485438-66485457
38 PART I BASIC SCIENCE

EXTENDED MATCHING ITEM (EMI) 4. Phenylethanolamine-N-methyl transferase


is required for production of this
QUESTIONS transmitter
5. Required in addition to glutamate for
46. Neurotoxins and other agents: co-activation of NMDA receptor, but acts
a. 3,4-Methylenedioxy-methamphetamine as an inhibitory neurotransmitter via Cl
b. Botulinum toxin channels
c. Bungarotoxin
d. Curare 48. Sensory receptors:
e. Chlorotoxin a. Free nerve endings
f. Conotoxin b. Golgi tendon organs
g. Ethanol c. Meissner’s corpuscles
h. Phencyclidine (PCP) d. Merkel’s tactile discs
i. Tetraethylammonium (TEA) e. Nuclear bag fibers
j. Tetrodotoxin (TTX) f. Nuclear chain fibers
g. Pacinian corpuscles
For each of the following descriptions, select the h. Pain nociceptors
most appropriate answers from the list above. i. Peritrichial nerve endings
Each answer may be used once, more than once j. Ruffini’s organs
or not at all.
1. Uncompetitive NMDA receptor antagonist For each of the following descriptions, select the
producing dissociative state most appropriate answers from the list above.
2. Works by presynaptic competitive reuptake Each answer may be used once, more than once
inhibition followed by inhibition of VMAT or not at all.
and TAAR1 receptor resulting in synaptic 1. Signal the onset of muscle stretch via A-
accumulation of monoamines. alpha myelinated fibers
3. Neurotoxic peptides isolated from venom 2. Two-point discriminative fine touch
of marin cone snail. 3. Deep pressure and vibration sense
4. Blocker of voltage-gated K+ channels and
competitive inhibitor of nicotinic AChRs 49. CNS cells:
5. Blocker of voltage-gated Na+ channels a. Astrocytes
found in Pufferfish b. Basket cells
c. Betz cells
47. Neurotransmitters: d. Ependymal cells
a. Acetylcholine e. Golgi type 2 cells
b. Dopamine f. Granule cells
c. Epinephrine g. Martinotti cells
d. GABA h. Microglia
e. Glutamate i. Oligodendroglia
f. Glycine j. Purkinje cells
g. Histamine k. Schwann cells
h. L-DOPA l. Stellate cells
i. Met-enkephalin
j. Serotonin For each of the following descriptions, select the
k. Substance P most appropriate answers from the list above.
l. Taurine Each answer may be used once, more than once
m. Tyramine or not at all.
n. VIP 1. These cells are derived from neural crest
origin and myelinate neurons of the PNS
For each of the following descriptions, select the 2. These cells arise from monocytes (hemato-
most appropriate answers from the list above. poietic precursor) and thus are the resident
Each answer may be used once, more than once macrophages of the CNS. Their function is
or not at all. to protect the CNS. When the brain is
1. The immediate precursor of norepinephrine damaged or infected, they become acti-
2. The immediate precursor of dopamine vated and multiply quickly to perform
3. Rate limiting step in the production of this functions such as phagocytosis and present-
neurotransmitter is tryptophan hydroxylase ing antigen
activity

Neurosurgery Books Full


www.ketabpezeshki.com 66485438-66485457
3 NEUROPHYSIOLOGY 39

3. These cells myelinate neurons within the 51. Cerebellar cortex:


CNS (one cell myelinates multiple neurons).
A
4. These are the most abundant and largest of B
the glial subtypes. Their most notable role G
is the metabolism and recycling of certain
neurotransmitters (glutamate, serotonin, C

and gamma-aminobutyric acid [GABA]). D

They also buffer the extracellular potassium


concentration, respond to injury (gliosis), H
and make up the blood-brain barrier I

5. These ciliated cells line the cavities of the J

CNS (ventricular system) in the choroid E * K


plexus, where they are involved in the pro- F L
*
duction of cerebrospinal fluid (CSF) and are
part of the blood-CSF barrier
K
M
50. Cerebellum:

A For each of the following descriptions, select the


HII J most appropriate answers from the image above.
I HIII
Each answer may be used once, more than once
II HIV K
B III
HV
or not at all.
IV HVI 1. Purkinje cell axon
C V 2. Basket cell
HVII
VI L 3. Climbing fiber
VII M 4. Golgi cell
D HVII
E VIII 5. Parallel fiber
F
G
IX
HVIII 52. Hypothalamic-pituitary axis:
H HIX
a. ACTH
N
I HX
b. ADH (vasopressin)
X
c. Cortisol
O
P
d. Epinephrine
e. FSH/LH
f. GH
For each of the following descriptions, select the g. TSH
most appropriate answers from the image above. h. Oestradiol
Each answer may be used once, more than once i. Oxytocin
or not at all. j. Prolactin
1. Posterior lobe k. Testosterone
2. Flocculus
3. Primary fissure For each of the following descriptions, select the
4. Dentate nucleus most appropriate answers from the list above.
5. Anterior inferior cerebellar artery Each answer may be used once, more than once
or not at all.
1. Uterine contractions in labor and milk ejec-
tion reflex
2. Renal water conservation
3. Excess may cause galactorrhea

Neurosurgery Books Full


www.ketabpezeshki.com 66485438-66485457
40 PART I BASIC SCIENCE

53. Peripheral nerve: For each of the following descriptions, select the
most appropriate answers from the image above.
The internal structure of a peripheral nerve Each answer may be used once, more than once
or not at all.
A C 1. Endoneurium
2. Perineurium
3. Mesoneurium
D 4. External epineurium
E

B G

H
I
J

SBA ANSWERS
1. e—Microglia 3. a—It is located in the ventral medulla at a
position that is caudal to the fourth ventricle
2. f—Subcommissural organ
The area postrema is found in the dorsomedial
The brain regions lacking a blood-brain barrier medulla oblongata and can be observed as two con-
are the circumventricular organs with neuroen- vex prominences bulging into the most caudal part
docrine function. They may be sensory organs: of the fourth ventricle. It is a V-shaped structure
subforniceal organ, area postrema, and organum diverging from an apex at the obex, and receives
vasculosum of lamina terminalis which can sense blood supply from pyramidal branches of the pos-
levels of various plasma molecules and signal to terior inferior cerebellar arteries which run along
the autonomic system. Alternatively, they may its lateral edge. It is thought to be a chemoreceptor
be secretory organs: median eminence of the trigger zone for vomiting and inhibition of 5-HT3
hypothalamus, pineal gland, posterior pituitary, receptors here (as well as peripherally on vagal
and subcommissural organ, which deliver hor- afferents) is effective in reducing the nausea associ-
mones/glycoproteins into the bloodstream in ated with cancer chemotherapy.
response to neural signals. Overall they form
part of feedback loops involved in body water 4. c—Water is actively pumped into the ventri-
regulation, feeding, thirst, cardiovascular func- cles via Aquaporin 1 channels in the apical
tion, immune response and reproductive behav- membrane
ior. The dura and choroid plexus also lack a
blood-brain barrier. Generally, lipophilic/ CSF forms in two sequential stages. First, ultrafil-
hydrophobic substances can cross the BBB (e.g., tration of plasma occurs across the fenestrated
O2, CO2, ethanol, caffeine, nicotine), whereas capillary wall into the ECF beneath the basolat-
lipophobic/hydrophilic/large molecules sub- eral membrane of the choroid epithelial cell. Sec-
stances cannot. ond, choroid epithelial cells secrete fluid into the

Neurosurgery Books Full


www.ketabpezeshki.com 66485438-66485457
3 NEUROPHYSIOLOGY 41

ventricle. Fluid secretion into the ventricles is allows replication in the cell body and spread to
mediated by an array of ion transporters unevenly adjacent neurons.
positioned at the blood-facing (basolateral) or
CSF-facing (apical) membranes. Many ionic spe- 6. d—Intracellular calcium ion concentration is
cies are involved in CSF production (e.g., K+, approximately 2 mM (4 mEq/l)
Mg2+, and Ca2+). However, fluid formation is pri-
marily generated by net secretion of Na+, Cl,
and HCO3  into ventricles as water molecules Equilibrium
Plasma Intracellular Potential
follow them passively down a chemical gradient
via Aquaporin1 channels in the apical membrane. Na+ 140 mM 10 mmol/l +68 mV
Na+ transport into CSF occurs due to active (140 mEq/l) (10 mEq/l)
transport via Na+/K+ ATPase exchange pump at K+ 4.5 mM 160 mmol/l 93 mV
the apical membrane, and is replaced by basolat- (4.5 mEq/l) (160 mEq/l)
eral membrane Na influx via Na+/H+ exchange
and Na+/HCO3  cotransport channels. Trans- Cl 110 mM 9 mmol/l 86 mV
port of Cl into CSF occurs via passive diffusion (110 mEq/l) (9 mEq/l)
via apical Cl selective channels (and possibly Ca2+ 2 mM (4 mEq/l) 0.0001 mmol/l +129 mV
Na+/K+/Cl cotransport), and is replaced at the (mEq)
basolateral membrane in exchange for HCO3  .
HCO3 22-26 mmol/l 10 mmol/l
Intracellular HCO3  is accumulated by (i) hydra- (mEq)
tion of CO2 catalyzed by carbonic anhydrase and
(ii) influx via basolateral membrane Na/HCO3 
cotransport, then can enter the CSF at the apical
membrane either by anion channel or Na/HCO3 7. a—Maintenance of the resting membrane
cotransport. CSF has lower concentrations of K+ potential is an energy dependent process
and amino acids than plasma does, and it contains requiring Na/K-ATPase
almost no protein.
The voltage, or potential difference, across the
5. b—Cytosolic proteins are transported by fast cell membrane (resting membrane potential) is
transport a result of the separation of positively and nega-
tively charged ions across it, the balance of which
Nerve cells have an elaborate transport system that is actively maintained by ATP-dependent mem-
moves organelles and macromolecules between the brane pumps. At rest, the inside of a cell holds
cell body and the axon and its terminals. Axonal more negative charge than the extracellular fluid
transport from the cell body toward the terminals outside it. Membrane depolarization is said to
is called anterograde; transport from the terminals occur when the separation of charge across the
toward the cell body is called retrograde. Antero- membrane is reduced from the resting/baseline
grade axonal transport is classified into fast and value (i.e., the inside of cell becomes more posi-
slow components. Fast transport, at speeds of up tively charged), whereas hyperpolarization is said
to 400 mm/day, is based on the action of an to occur if the separation of charge is increased
ATPase protein called kinesin which moves (i.e., the inside of the cell becomes more nega-
macromolecule-containing vesicles and mitochon- tively charged than at rest). There is a tendency
dria along microtubules. Slow transport carries for ions to passively leak in or out of the cell
important structural and metabolic components against their respective electrochemical gradi-
from the cell body to axon terminals (e.g., cytoskel- ents, hence the requirement for continuously
etal protein components such as actin, myosin, active ATP-dependent membrane pumps to pre-
tubulin, and cytosolic enzymes required for neuro- vent an overall change in the resting membrane
transmitter synthesis in the presynaptic terminal) potential. The propensity for ion flux across the
but the mechanism is less clear. Retrograde axonal membrane passively down artificially membrane
transport along axonal microtubules is driven by pump produced and maintained electrochemical
the protein dynein and allows the neuron/cell body gradients is exploited and forms the basis for
to respond to molecules taken up near the axon ter- action potentials during which ion channels open
minal by either pinocytosis or receptor-mediated up to allow passive ion flux on a magnitude and
endocytosis (e.g., growth factors). In addition, this time scale at which ATP-dependent membrane
form of transport functions in the continual recy- pumps cannot prevent, allowing depolarization/
cling of components of the axon terminal (e.g., hyperpolarization to act as a high fidelity way of
mitochondria). Retrograde transport of rabies virus information transfer.

Neurosurgery Books Full


www.ketabpezeshki.com 66485438-66485457
42 PART I BASIC SCIENCE

8. e—GABA-B receptor is a ligand-gated ion stop further diffusion of the cation. Equally, if an
channel anion diffuses down its concentration gradient, it
carries a negative charge, which will retard and then
Ion channels are transmembrane proteins that stop further diffusion of the anion. The equilibrium
permit the selective passage of ions with specific potential is the diffusion potential that exactly bal-
characteristics (size and charge) down their elec- ances or opposes the tendency for diffusion down
trochemical gradient by passive diffusion when the concentration difference. At electrochemical
open. Ion channels are controlled by gates, and, equilibrium, the chemical and electrical driving
depending on the position of the gates, the chan- forces acting on an ion are equal and opposite,
nels may be open or closed. The higher the prob- and no further net diffusion occurs. The Nernst
ability that the channel is open, the higher is its equation is used to calculate the equilibrium poten-
conductance or permeability. The gates on ion tial for an ion at a given concentration difference
channels are controlled by three types of sensors: across a membrane, assuming that the membrane
• Voltage-gated channels have gates that is permeable to that ion. By definition, the equilib-
are controlled by changes in membrane rium potential is calculated for one ion at a time. For
potential. a given ion X with charge z at 37 °C, the equilibrium
• Second messenger-gated channels have gates potential (Ex)¼ (60/z) log10([intracellular con-
that are controlled by changes in levels of centration of X in mmol/l]/[extracellular concen-
intracellular signaling molecules such as tration of X in mmol/l]). For example, E(Na) ¼
cyclic AMP (e.g., beta-adrenoceptors, (60/+1) log10(10/140) ¼ +68.8 mV. Whereas
alpha2-adrenoceptors, M2 muscarinic for E(k) ¼ (60/+1) log10 (140/10) ¼  87 mV.
AChR) or inositol 1,4,5-triphosphate (IP3; The Goldmann equation can be used to calculate
e.g., alpha1-adrenoceptors, M1/M3 musca- the exact resting membrane potential based on all
rinic AChR). In general, Gs/Gi G-protein the permeable ions across it, but in practice since
coupled receptor activation causes adenylyl in neurons 80% of conductance is due to K+ (resid-
cyclase to convert ATP to cAMP, which then ual is 15% due to Na+ and 5% due to Cl), the rest-
activates protein kinase A to phosphorylate ing membrane voltage (Vm) of approximately
downstream proteins. In contrast, Gq G- 70 mV is much closer to that of the equilibrium
protein coupled receptors cause activation potential for K+.
of phospholipase C which hydrolyzes mem-
brane phospholipid (phosphatidylinositol 10. b—Chloride and potassium
4,5-bisphosphate; PIP2) to diacyl glycerol
(DAG) and inositol 1,4,5-trisphosphate (IP3). Assuming normal intracellular and extracellular
• Ligand-gated channels have gates that are concentrations of ions, both potassium and chlo-
controlled by hormones and neurotrans- ride ions have a negative equilibrium potential
mitters. The sensors for these gates are hence will result in hyperpolarization of the cell
located on the extra-cellular side of the if allowed to flow down their electrochemical
ion channel (e.g., nicotinic AChR allows gradients. Chloride influx into the cell down
Na+ and K+ passage on binding its electrochemical gradient results in a gain of
acetylcholine). negative charge, whereas efflux of potassium
reflects a loss of positive charge in the intracellu-
9. e—At electrochemical equilibrium, the lar compartment to achieve this. Physiological
chemical and electrical driving forces acting electrochemical gradients for both sodium and
on an ion are equal and opposite, and no fur- calcium favor influx into the cell, and would
ther net diffusion occurs cause depolarization due to net gain of positive
charge.
The concept of equilibrium potential is simply an
extension of the concept of diffusion potential. If 11. c—Length constant is greater in unmyelin-
there is a concentration difference for an ion across ated and large diameter axons
a membrane and the membrane is permeable to that
ion, a potential difference (the diffusion potential) is The passive flow of electrical current plays a cen-
created. Eventually, net diffusion of the ion slows tral role in action potential propagation, synaptic
and then stops because of that potential difference. transmission, and all other forms of electrical sig-
In other words, if a cation diffuses down its concen- naling in nerve cells. For the case of a cylindrical
tration gradient, it carries a positive charge across axon, subthreshold current injected into one part
the membrane, which will retard and eventually of the axon spreads passively along the axon until

Neurosurgery Books Full


www.ketabpezeshki.com 66485438-66485457
3 NEUROPHYSIOLOGY 43

the current is dissipated (decays) by leakage out plasma membrane such that: τ ¼ RmCm. The values
across the axon membrane. The decrement in of Rm and Cm depend, in part, on the size of the neu-
the current flow with distance is described by a ron, with larger cells having lower resistances and
simple exponential function: Vx ¼ V0ex/λ where larger capacitances. In general, small nerve cells
Vx is the voltage response at any distance x along tend to have long time constants and large cells brief
the axon, V0 is the voltage change at the point
time constants. Regarding achieving threshold
where current is injected into the axon, e is the base
for action potential generation, long time constants
of natural logarithms ( 2.7), and λ is the length
constant of the axon. As evident in this relation- favor temporal summation of EPSPs, whereas short
ship, the length constant is the distance where time constant allows coincidence detection
the initial voltage response (V0) decays to 1/e (or through spatial summation of EPSPs/IPSPs.
37%) of its value. The length constant is thus a
way to characterize how far passive current flow FURTHER READING
spreads before it leaks out of the axon, with leakier Chapter 3 Voltage dependent membrane permeability. In:
axons having shorter length constants. The length Purves D, et al. (Eds.), Neuroscience, 3rd ed. MA: Sinauer.
constant depends upon the physical properties of p. 60-61.
the axon, in particular the relative resistances of
the plasma membrane (Rm), the intracellular axo- 12. b—It can propagate bidirectionally
plasm (Ri), and the extracellular medium (R0).
The relationship between these parameters is: The action potential, as classically defined, is an
λ ¼ √(Rm/[R0+Ri]). Hence, to improve the passive all-or-nothing, regenerative, directionally propa-
flow of current along an axon (i.e., slow the rate of gated, depolarizing nerve impulse. At rest, the
decay), the resistance of the plasma membrane membrane has high K+ conductance and Vm is
should be as high as possible (e.g., myelination) near the Nernst equilibrium potential for K+
and the resistances of the axoplasm and extracellu- (EK). Spread of an action potential from an adja-
lar medium should be low. Another important cent area of the membrane brings the membrane
consequence of the passive properties of neurons potential Em, to a threshold potential (approxi-
is that currents flowing across a membrane do mately 40 to 55 mV) causing a large increase
not immediately change the membrane potential. in Na+ conductance of the membrane and Na+
These delays in changing the membrane potential influx such that Vm approaches the Nernst poten-
are due to the fact that the plasma membrane tial for Na+ (ENa) and the membrane depolarizes.
behaves as a capacitor, storing the initial charge Depolarization causes voltage-gated sodium
that flows at the beginning and end of the current channels to change from an open to an inactivated
pulse. For the case of a cell whose membrane state, preventing further rises in membrane
potential is spatially uniform, the change in the potential, and at the same time there is an increase
membrane potential at any time, Vt, after begin- in conductance of delayed-rectifier K channels
ning the current pulse can also be described by causing K efflux and movement of Vm towards
an exponential relationship: Vt ¼ V1(1 et/τ) the equilibrium potential for potassium (repolar-
where V1 is the steady-state value of the mem- ization). This increased K+ conductance usually
brane potential change, t is the time after the cur- lasts slightly longer than the time required to
rent pulse begins, and τ is the membrane time bring the membrane potential back to its normal
constant. The time constant is thus defined as resting level, hence there is an overshoot (hyper-
the time when the voltage response (Vt) rises to polarization) which subsequently decays. An
1  (1/e) (or 63%) of V1. After the current pulse absolute refractory period for action potential fir-
ends, the membrane potential change also declines ing is seen when sodium channels are in their
exponentially according to the relationship inactivated state, but as repolarization progresses
Vt ¼ V1et/τ During this decay, the membrane more Na channels move from an inactivated to a
close state, and thus could be reopened in the
potential returns to 1/e of V1 at a time equal to t.
presence of a supratheshold stimulus (relative
The time constant characterizes how rapidly refractory period). The figure below shows the
current flow changes the membrane potential. action potential (yellow), and underlying changes
The membrane time constant also depends on in membrane conductance to sodium (purple) and
the physical properties of the nerve cell, specifically potassium (red) due to opening/inactivation of
on the resistance (Rm) and capacitance (Cm) of the channels.

Neurosurgery Books Full


www.ketabpezeshki.com 66485438-66485457
44 PART I BASIC SCIENCE

current can flow passively such that the time-


50 ENa consuming process of action potential generation
occurs only at specific points along the axon,

Open channels per µm2 of membrane


80 called nodes of Ranvier, where there is a gap in
Em the myelin wrapping (rather than adjacent mem-
60 brane in a depolarization wave). As it happens, an
Membrane potential (mV)

0 action potential generated at one node of Ranvier


Voltage-gated
40 elicits current that flows passively within the axo-
Thresh- Na+ channels
old
plasm of the myelinated segment until the next
Voltage-gated
20 node is reached and another action potential is
K+ channels
–50 generated, and the cycle is repeated along the
0 length of the axon. Because current flows across
EK the neuronal membrane only at the nodes, action
potentials “leap” from node to node and this is
–100
termed salutatory conduction. Myelination
0 1 2 3 4 greatly speeds up action potential conduction
Time (ms) (velocities up to 150 m/s) compared to unmyelin-
ated axons (0.5-10 m/s). (In: Purves D, et al.
Open Closed Open (Eds.), Neuroscience, 3rd ed. MA: Sinauer.)
K+ leak
channels
15. e—Electrical synapses
Image with permission from Pollard TD et al. (Eds.), Cell
Biology, 2nd ed. Elsevier, 2008.
Electrical synapses only represent a small minor-
ity of synapses (e.g., some neuroendocrine cells in
hypothalamus) and are characterized by very
13. d—Axon hillock and initial segment
closely apposed pre and post-synaptic membranes
connected by a gap junction. These junctions
14. a—Accommodation is dependent on post-
contain aligned paired channels so that each
synaptic receptor phagocytosis
paired channel forms a pore (larger than those
Unidirectional propagation is due to the inactive observed in ligand-gated channels) and allows
state of the sodium channel, and this wave of for the bidirectional transmission. Chemical syn-
inactivation immediately following the action apse types include:
potential prevents it from reversing direction. Axosecretory—axon terminal secretes directly
Accommodation occurs when subthreshold stim- into bloodstream (e.g., hypothalamus)
ulus will stimulate channels to open, but at a rate Axodendritic—axon terminal ends on den-
that is too slow for there to be a sufficient number dritic spines or shaft (type I excitatory
of open channels at any one time to fire an AP but synapse)
sufficient for channel inactivation. Absolute Axoaxonic—axon terminal secretes onto
refractory period is the time period immediately another axon
after/during the action potential upstroke when Axoextracellular—axon with no connection
most of the neuron’s sodium channels are inacti- secretes into extracellular fluid
vated and cannot be opened to elicit a second Axosomatic—axon terminal ends on cell soma
action potential. The relative refractory period (type II inhibitory synapse, e.g., basket cell
refers to the period during repolarization when onto Purkinje cell)
inactivated Na channels return to a closed state Axosynaptic—axon terminal ends on presyn-
and a second action potential can be generated aptic terminal of another axon
but is more difficult than normal (becomes pro-
gressively less difficult to elicit an action potential 16. a—The action potential stimulates the post-
during the relative refractory period until it synaptic terminal to release neurotransmitter
returns to normal). Myelination of axons involves
wrapping the axon in myelin, which consists of Neurotransmission at a chemical synapse
multiple layers of closely opposed glial cell mem- requires a neurotransmitter to be synthesized
branes (i.e., oligodendrocytes in CNS, Schwann and stored in the presynaptic vesicles. The arrival
cells in PNS). Myelination electrically insulates of an action potential at the presynaptic terminal
the axonal membrane, reducing the ability of cur- results in depolarization dependent opening of
rent to leak out of the axon and thus increasing voltage-gated Ca2+ channels and calcium influx.
the distance along the axon that a given local Then, there is Ca2+ through these channels,

Neurosurgery Books Full


www.ketabpezeshki.com 66485438-66485457
3 NEUROPHYSIOLOGY 45

causing the vesicles to fuse with the presynaptic subunits also bind other ligands, such as nicotine
membrane in a mechanism mediated by synapto- and α-bungarotoxin. At the neuromuscular junc-
tagmin 1 and SNAP-25 (SNARE) calcium sensi- tion, the two α subunits are combined with up to
tive proteins. The transmitter is then released four other types of subunit—β, γ, δ, ε—in the
into the presynaptic cleft (by exocytosis) and ratio 2α:β:ε:δ. Neuronal nAChRs typically differ
binds to receptor molecules in the postsynaptic from those of muscle in that they lack sensitivity
membrane. This leads to the opening or closing to α-bungarotoxin, and comprise only two recep-
of postsynaptic channels. The resultant current tor subunit types (α and β), which are present in a
results in an EPSP or IPSP, which causes a ratio of 3α:2β. In all cases, however, five individ-
change in excitability of the postsynaptic cell. ual subunits assemble to form a functional,
The vesicular membrane is then retrieved from cation-selective nACh receptor. Each subunit of
the plasma membrane by endocytosis. If summa- the nAChR molecule contains four transmem-
tion of EPSPs or IPSPs exceeds threshold poten- brane domains that make up the ion channel por-
tial at the axon hillock, an axon potential is tion of the receptor, and a long extracellular
generated. To prevent repetitive stimulation, region that makes up the ACh-binding domain.
neurotransmitters are either degraded in the pre- A second type of ACh receptors is activated by
synaptic cleft or taken up by endocytosis in muscarine and thus they are referred to as mus-
presynaptic cell. carinic ACh receptors (mAChRs). mAChRs are
metabotropic and mediate most of the effects of
17. d—α-bungarotoxin binds to muscarinic ACh in brain via G-protein signaling. Several
AChRs subtypes of mAChR are known. Muscarinic
ACh receptors are highly expressed in the stria-
In addition to the action of ACh as the neuro- tum and various other forebrain regions, where
transmitter at skeletal neuromuscular junctions they exert an inhibitory influence on dopamine-
as well as the neuromuscular synapse between mediated motor effects. These receptors are also
the vagus nerve and cardiac muscle fibers, ACh found in the ganglia of the peripheral nervous
serves as a transmitter at synapses in the ganglia system and autonomic effector organs—such as
of the visceral motor system, and at a variety of heart, smooth muscle, and exocrine glands—
sites within the central nervous system. Acetyl- and are responsible for the inhibition of heart
choline is synthesized in nerve terminals from rate by the vagus nerve. Nevertheless, mACh
the precursors acetyl coenzyme A (acetyl CoA, blockers that are therapeutically useful include
which is synthesized from glucose) and choline, atropine (used to dilate the pupil), scopolamine
in a reaction catalyzed by choline acetyltransfer- (effective in preventing motion sickness), and
ase (CAT). Choline is present in plasma at a high ipratropium (useful in the treatment of asthma).
concentration (about 10 mM) and is taken up into (In: Purves D, et al. (Eds.) Neuroscience 3rd
cholinergic neurons by a high-affinity Na+/cho- ed. MA: Sinauer.)
line transporter. After synthesis in the cytoplasm
of the neuron, a vesicular ACh transporter loads 18. e—AMPA receptors are a type of
approximately 10,000 molecules of ACh into each metabotropic GluR
cholinergic vesicle. The postsynaptic actions of
ACh at many cholinergic synapses terminated Nearly all excitatory neurons in the central ner-
by acetylcholinesterase (AChE) hydrolysis Ach vous system are glutamatergic, and it is estimated
into acetate and choline. The choline produced that over half of all brain synapses release this
by ACh hydrolysis is transported back into nerve agent and cause excitotocity in ischemic brain.
terminals and used to resynthesize ACh. Many of Glutamate is a nonessential amino acid that does
the postsynaptic actions of ACh are mediated by not cross the blood-brain barrier and therefore
the nicotinic ACh receptor nAChR which is a must be synthesized in neurons from local pre-
nonselective cation channels that generate excit- cursors. The most prevalent precursor for gluta-
atory postsynaptic responses a large protein com- mate synthesis is glutamine, which is released by
plex consisting of five subunits arranged around a glial cells. Once released, glutamine is taken up
central membrane-spanning pore. In the case of into presynaptic terminals and metabolized to
skeletal muscle AChRs, the receptor pentamer glutamate by the mitochondrial enzyme gluta-
contains two α subunits, each of which binds minase. Glutamate can also be synthesized by
one molecule of ACh. Because both ACh-binding transamination of 2-oxoglutarate, an intermedi-
sites must be occupied for the channel to open, ate of the tricarboxylic acid cycle. Hence, some
only relatively high concentrations of this neuro- of the glucose metabolized by neurons can also
transmitter lead to channel activation. These be used for glutamate synthesis. The glutamate

Neurosurgery Books Full


www.ketabpezeshki.com 66485438-66485457
46 PART I BASIC SCIENCE

synthesized in the presynaptic cytoplasm is pack- excitatory ionotropic glutamate receptors,


aged into synaptic vesicles by transporters, mGluRs cause slower postsynaptic responses that
termed VGLUT. Once released, glutamate is can either increase or decrease the excitability of
removed from the synaptic cleft by the excitatory postsynaptic cells. (In: Purves D, et al. (Eds.)
amino acid transporters (EAATs). Glutamate Neuroscience 3rd ed. MA: Sinauer.)
taken up by glial cells is converted into glutamine
by the enzyme glutamine synthetase; glutamine is 19. b—DOPA—catechol O—methyltransferase—
then transported out of the glial cells and into dopamine
nerve terminals. In this way, synaptic terminals
cooperate with glial cells to maintain an adequate There are five well-established biogenic amine
supply of the neurotransmitter. This overall neurotransmitters: the three catecholamines—
sequence of events is referred to as the dopamine, norepinephrine (noradrenaline), and
glutamate-glutamine cycle. Receptors of these epinephrine (adrenaline)—and histamine and
are ionotropic receptors called, respectively, serotonin. Their synthesis is as follows:
NMDA receptors, AMPA receptors, and kainate • Tyrosine-tyrosine hydroxylase-DOPA
receptors. These glutamate receptors are named (dihydroxyphenylalanine)
after the agonists that activate them: NMDA • DOPA-DOPA decarboxylase-dopamine
(N-methyl-D-aspartate), AMPA (α-amino-3- • Dopamine-Dopamine beta-hydroxylase-
hydroxyl-5-methyl-4-isoxazole-propionate), and Norepinephrine
kainic acid. All of the ionotropic glutamate recep- • Norepinephrine-Phenylethanolamine N-
tors are nonselective cation channels similar to methyltransferase-Epinephrine
the nAChR, allowing the passage of Na+ and • Histidine-histidine decarboxylase-Histamine
K+, and in some cases small amounts of Ca2+. • Tryptophan-tryptophan 5-hydroxylase-
NMDA receptor ion channels allow the entry 5-hydroxytryptophan-Aromatic L-amino
of Ca2+ in addition to monovalent cations such acid decarboxylase-Serotonin (5-Hydroxy
as Na+ and K+. As a result, EPSPs produced by tryptamine)
NMDA receptors can increase the concentration
of Ca2+ within the postsynaptic neuron; the Ca2+ 20. c—Dopamine is derived from
concentration change can then act as a second norepinephrine
messenger to activate intracellular signaling cas-
cades. Another key property is that they bind Dopamine is present in several brain regions,
extracellular Mg2+. At hyperpolarized membrane although the major dopamine-containing area
potentials, this ion blocks the pore of the NMDA of the brain is the corpus striatum, which receives
receptor channel. Depolarization, however, major input from the substantia nigra and plays an
pushes Mg2+ out of the pore, allowing other cat- essential role in the coordination of body move-
ions to flow. This property provides the basis for a ments. Dopamine is also believed to be involved
voltage-dependence to current flow through the in motivation, reward, and reinforcement, and
receptor and means that NMDA receptors pass many drugs of abuse work by affecting dopami-
cations (most notably Ca2+) only during depolar- nergic synapses in the CNS. In addition to these
ization of the postsynaptic cell, due to either acti- roles in the CNS, dopamine also plays a poorly
vation of a large number of excitatory inputs and/ understood role in some sympathetic ganglia.
or by repetitive firing of action potentials in the Dopamine is produced by the action of DOPA
presynaptic cell. These properties are widely decarboxylase on DOPA. Following its synthesis
thought to be the basis for some forms of infor- in the cytoplasm of presynaptic terminals, dopa-
mation storage at synapses, such as memory. mine is loaded into synaptic vesicles via a vesicular
Another unusual property of NMRA receptors monoamine transporter (VMAT). Dopamine
is that opening the channel of this receptor action in the synaptic cleft is terminated by reup-
requires the presence of a coagonist, the amino take of dopamine into nerve terminals or
acid glycine. In addition to these ionotropic glu- surrounding glial cells by a Na+-dependent dopa-
tamate receptors, there are three types of metabo- mine transporter, termed DAT. Cocaine appar-
tropic glutamate receptor (mGluRs). These ently produces its psychotropic effects by
receptors, which modulate postsynaptic ion chan- binding to and inhibiting DAT, yielding a net
nels indirectly, differ in their coupling to intracel- increase in dopamine release from specific brain
lular signal transduction pathways and in their areas. Amphetamine, another addictive drug, also
sensitivity to pharmacological agents. Activation inhibits DAT as well as the transporter for
of many of these receptors leads to inhibition of norepinephrine (see below). The two major
postsynaptic Ca2+ and Na+ channels. Unlike the enzymes involved in the catabolism of dopamine

Neurosurgery Books Full


www.ketabpezeshki.com 66485438-66485457
3 NEUROPHYSIOLOGY 47

are monoamine oxidase (MAO) and catechol this degradation, GABA transaminase and succi-
O-methyltransferase (COMT). Both neurons nic semialdehyde dehydrogenase, are mitochon-
and glia contain mitochondrial MAO and cyto- drial enzymes. Inhibitory synapses employing
plasmic COMT. Inhibitors of these enzymes, GABA as their transmitter can exhibit three
such as phenelzine and tranylcypromine, are used types of postsynaptic receptors, called GABAA,
clinically as antidepressants. Once released, GABAB, and GABAC. GABAA and GABAC
dopamine acts exclusively by activating G-pro- receptors are ionotropic receptors, while
tein-coupled receptors. Most dopamine receptor GABAB receptors are metabotropic. The iono-
subtypes act by either activating or inhibiting tropic GABA receptors are usually inhibitory
adenylyl cyclase. Activation of these receptors because their associated channels are permeable
generally contribute to complex behaviors; for to Cl; the flow of the negatively charged chlo-
example, administration of dopamine receptor ride ions inhibits postsynaptic cells since the
agonists elicits hyperactivity and repetitive, ste- reversal potential for Cl is more negative than
reotyped behavior in laboratory animals. Activa- the threshold for neuronal firing. Like other
tion of another type of dopamine receptor in ionotropic receptors, GABA receptors are penta-
the medulla inhibits vomiting. Thus, antagonists mers assembled from a combination of five types
of these receptors are used as emetics to induce of subunits (αβγδρ). Benzodiazepines, such as
vomiting after poisoning or a drug overdose. diazepam and chlordiazepoxide, are tranquilizing
Dopamine receptor antagonists can also (anxiety reducing) drugs that enhance GABAer-
elicit catalepsy, a state in which it is difficult to ini- gic transmission by binding to the α and δ sub-
tiate voluntary motor movement, suggesting units of GABAA receptors. Metabotropic GABA
a basis for this aspect of some psychoses. (In: receptors (GABAB) are also widely distributed in
Purves D, et al. (Eds.) Neuroscience 3rd ed. brain. Like the ionotropic GABAA receptors,
MA: Sinauer.) GABAB receptors are inhibitory. Rather than
activating Cl selective channels, however,
21. d—GABAA and GABAC receptors are iono- GABAB-mediated inhibition is due to the activa-
tropic receptors and are Ca2+ conductors tion of K+ channels. A second mechanism for
GABAB-mediated inhibition is by blocking Ca2+
Most inhibitory synapses in the brain and spinal channels, which tends to hyperpolarize pos-
cord use either γ-aminobutyric acid (GABA) or tsynaptic cells. Unlike most metabotropic recep-
glycine as neurotransmitters. It is now known tors, GABAB receptors appear to assemble as
that as many as a third of the synapses in the heterodimers of GABAB R1 and R2 subunits.
brain use GABA as their inhibitory neurotrans- (In: Purves D, et al. (Eds.) Neuroscience 3rd
mitter. GABA is most commonly found in local ed. MA: Sinauer.)
circuit interneurons, although cerebellar Pur-
kinje cells provide an example of a GABAergic 22. b—Medial geniculate thalamic nucleus
projection neuron. The predominant precursor
for GABA synthesis is glucose, which is metabo- 23. a—Ganglion cells
lized to glutamate by the tricarboxylic acid cycle
enzymes (pyruvate and glutamine can also act as The output of the retina is determined by gan-
precursors). The enzyme glutamic acid decar- glion cells which can generate action potentials
boxylase (GAD), which is found almost exclu- and give rise to optic nerve. The other cell types
sively in GABAergic neurons, catalyzes the display graded depolarizing/hyperpolarizing
conversion of glutamate to GABA. GAD responses and amacrine cells show calcium spikes.
requires a cofactor, pyridoxal phosphate, for In general, 99% of all ganglion cells are con-
activity. Because pyridoxal phosphate is derived cerned with details of image formation and
from vitamin B6, a B6 deficiency can lead to receive input from rods and cones via synaptic
diminished GABA synthesis. Once GABA is syn- relays through the layers of the retina, are
thesized, it is transported into synaptic vesicles involved in circadian rhythms and the pupillary
via a vesicular inhibitory amino acid transporter light reflex. The second type, melanopsin-
(VIATT). The mechanism of GABA removal is containing ganglion cells, comprise less than
similar to that for glutamate: Both neurons and 1% of all ganglion cells, are intrinsically sensitive
glia contain high-affinity transporters for GABA, to light and will generate action potentials (even
termed GATs. Most GABA is eventually con- without rods/cones, particularly blue light); are
verted to succinate, which is metabolized further not concerned with image formation, and have
in the tricarboxylic acid cycle that mediates cel- connections to the suprachiasmatic and pretectal
lular ATP synthesis. The enzymes required for nuclei maintaining circadian rhythm. This type of

Neurosurgery Books Full


www.ketabpezeshki.com 66485438-66485457
48 PART I BASIC SCIENCE

ganglion cell explains why those blind due to rod/ minimize light scattering on the way to the
cone disease (e.g., retinitis pigmentosa) may still receptors. In addition, within the fovea, the ratio
have an intact pupillary reflex and maintain circa- of photoreceptors to ganglion cells falls dramat-
dian rhythm. ically. Most foveal receptors synapse on only one
bipolar cell, which synapses on only one gan-
24. a—In the dark, rods have a high resting glion cell. Because each ganglion cell is devoted
membrane potential of about 70 mV to a very small portion of the visual field, central
vision has high resolution. In other words, the
Rod cells are named for the shape of their outer receptive field of a foveal ganglion cell (i.e.,
segment, which is a membrane-bound cylinder the region of stimulus space that can activate
containing hundreds of tightly stacked membra- it) is small. At the periphery, the ratio of recep-
nous discs. In the dark, cGMP levels in the rod tors to ganglion cells is high; thus, each ganglion
outer segment are high facilitating a inward Na cell has a large receptive field. The large recep-
and Ca current results in a relatively high resting tive field reduces the spatial resolution of the
membrane potential for rod cells, about peripheral portion of the retina but increases
40 mV, and at the rod spherule there is tonic its sensitivity because more photoreceptors col-
release of glutamate. With light, rhodopsin lect light for a ganglion cell. Lastly, the magni-
absorbs photons and undergoes a conformational tude of phototransduction amplification varies
change causing reduced levels of cGMP, causing with the prevailing levels of illumination (light
closure of sodium channels, a wave of hyperpo- adaptation). At low levels of illumination, photo-
larization and a transient reduction in this tonic receptors are the most sensitive to light. As levels
release of glutamate. Cone outer segments also of illumination increase, sensitivity decreases
consist of a membranous stack of constantly (due to reduction in calcium currents in the
decreasing diameter (from cilium to tip), giving rod outer segment), preventing the receptors
the cell its characteristic shape. Cone opsin from saturating and thereby greatly extending
absorbs photons and undergoes a conformational the range of light intensities over which they
change, resulting in a hyperpolarization of the operate.
cell membrane. This hyperpolarization propa-
gates passively to the cone’s synaptic ending, 25. d—The receptive fields of on-center and off-
the cone pedicle, in the outer plexiform layer. center ganglion cells do not overlap
Like rods, cones release the neurotransmitter
glutamate tonically in the dark and respond to Most of the information in visual scenes consists
light with a decrease in glutamate release. There of spatial variations in light intensity. Each gan-
are three types of cones, each tuned to a different glion cell responds to stimulation of a small cir-
light wavelength. L-cones (red cones) are sensi- cular patch of the retina, which defines the cell’s
tive to long wavelengths, M-cones (green cones) receptive field. Turning on a spot of light in the
to medium wavelengths, and S-cones (blue receptive field center of an on-center ganglion
cones) to short wavelengths. Because any pure cell produces a burst of action potentials. The
color represents a particular wavelength of light, same stimulus applied to the receptive field cen-
each color will be represented by a unique com- ter of an off-center ganglion cell reduces the rate
bination of responses in the L-, M-, and S-cones. of discharge, and when the spot of light is turned
At the posterior pole of the eye is a yellowish off, the cell responds with a burst of action
spot, the macula lutea, the center of which is a potentials. Complementary patterns of activity
depression called the fovea centralis Cones, are also found for on-center versus off-center
which are responsible for color vision, are the cell type when a dark spot is placed in the recep-
only type of photoreceptor present in the fovea. tive field center. Thus, on-center cells increase
In contrast, rods, which are most sensitive at low their discharge rate to luminance increments in
levels of illumination, are the predominant pho- the receptive field center, whereas off-center
toreceptors in the periphery of the retina. The cells increase their discharge rate to luminance
visual world is a composite formed from a suc- decrements in the receptive field center. On-
cession of foveal images carrying form and color and off-center ganglion cells are present in
information supplemented with input from the roughly equal numbers. Their receptive fields
peripheral retina carrying motion information. have overlapping distributions, so that every
Several adaptations of the fovea allow it to medi- point on the retinal surface (i.e., every part of
ate the highest visual acuity in the retina. Neu- visual space) is analyzed by several on-center
rons of the inner layer of retina are actually and several off-center ganglion cells. In practice,
displaced laterally to the side of the fovea to silencing on-center ganglion cells in primates

Neurosurgery Books Full


www.ketabpezeshki.com 66485438-66485457
3 NEUROPHYSIOLOGY 49

caused a deficit in their ability to detect stimuli illumination that falls on the receptive field center
that were brighter than the background; how- and the level of illumination that falls on the sur-
ever, they could still see objects that were darker round—that is, to luminance contrast. The center
than the background. These observations imply of a ganglion cell receptive field is surrounded by
that information about increases or decreases a concentric region (surround) that, when stimu-
in luminance is carried separately to the brain lated, antagonizes the response to stimulation of
by the axons of these two different types of ret- the receptive field center (center antagonism).
inal ganglion cells. Having separate luminance In practice this means that firing of an on-center
“channels” means that changes in light intensity, ganglion cell is (i) increased above baseline when
whether increases or decreases, are always con- a spot of light shines on receptive field center, (ii)
veyed to the brain by an increased number of at baseline when the spot of light is on the center/
action potentials. Because ganglion cells rapidly surround border or outside of the receptive field
adapt to changes in luminance, their “resting” completely, and (iii) reduced below baseline when
discharge rate in constant illumination is rela- shined on the surround alone. Off-center gan-
tively low. Although an increase in discharge rate glion cells demonstrate surround antagonism.
above resting level serves as a reliable signal, a Much of the antagonism is thought to arise via
decrease in firing rate from an initially low rate lateral connections established by horizontal cells
of discharge might not. Thus, having luminance and photoreceptor terminals (lateral inhibition).
changes signaled by two classes of adaptable cells Thus, the information supplied by the retina to
provides unambiguous information about both central visual stations for further processing does
luminance increments and decrements. On- not give equal weight to all regions of the visual
and off-center ganglion cells have dendrites that scene; rather, it emphasizes the regions where
arborize in separate strata of the inner plexiform there are differences in luminance. In addition
layer, forming synapses selectively with the ter- to making ganglion cells especially sensitive to
minals of on- and off-center bipolar cells that light-dark borders in the visual scene, center-
respond to luminance increases and decreases, surround mechanisms make a significant contri-
respectively. As mentioned previously, the prin- bution to the process of light adaptation as back-
cipal difference between ganglion cells and bipo- ground/ambient level of illumination is less
lar cells lies in the nature of their electrical important than scaled differences in light inten-
response. Like most other cells in the retina, sity. (In Chapter 10 Visual System In: Purves
bipolar cells have graded potentials rather than D, et al. (Eds.) Neuroscience 3rd ed. MA:
action potentials. Graded depolarization of bipo- Sinauer.)
lar cells leads to an increase in transmitter release
(glutamate) at their synapses and consequent 26. b—Mucus-coated olfactory epithelium lines
depolarization of the on-center ganglion cells the anterodorsal parts of the nasal cavities
that they contact via AMPA, kainite, and NMDA
receptors. The selective response of on- and off- Smell is detected by olfactory receptor cells,
center bipolar cells to light increments and decre- which are situated in mucus-coated olfactory
ments is explained by the fact that they express epithelium that lines the posterodorsal parts of
different types of glutamate receptors. Off-center the nasal cavities. Olfactory glands (Bowman
bipolar cells have ionotropic receptors (AMPA glands) secrete a fluid that bathes the cilia of
and kainate) that cause the cells to depolarize in the receptors and acts as a solvent for odorant
response to glutamate released from photorecep- molecules. Olfactory receptor cells (first-order
tor terminals. In contrast, on-center bipolar cells neurons) are stimulated by the binding of odor
express a G-protein-coupled metabotropic gluta- molecules to their cilia—G protein activation
mate receptor (mGluR6). When bound to gluta- and activation of adenylyl cyclase, a rise in intra-
mate, these receptors activate an intracellular cellular cAMP with causes opening of a cyclic-
cascade that closes cGMP-gated Na+ channels, nucleotide gated ion channel allowing influx of
reducing inward current and hyperpolarizing Na+ and Ca2+ causing neuronal depolarization.
the cell. Decrements in light intensity naturally The axons of the olfactory receptor cells form
have the opposite effect on these two classes of CN I (olfactory nerve); these project through
bipolar cells, hyperpolarizing on-center cells and the cribriform plate at the base of the cranium to
depolarizing off-center ones. Retinal ganglion synapse with the mitral cells of the olfactory bulb
cells are relatively poor at signaling differences in olfactory glomeruli. The map of glomerular
in the level of diffuse illumination. Instead, they activation patterns within the olfactory bulb are
are sensitive to differences between the level of thought to represent the quality of the odor being

Neurosurgery Books Full


www.ketabpezeshki.com 66485438-66485457
50 PART I BASIC SCIENCE

detected. The mitral cells of the olfactory bulb are An action potential in presynaptic neuron
excitatory, second-order neurons. The output causes calcium influx and release of acetylcho-
axons of the mitral cells form the olfactory tract line (ACh) from presynaptic vesicles stored in
and lateral olfactory stria, both of which project terminal bouton. Diffusion of ACh occurs
to the primary olfactory cortex (prefrontal cortex) across the synaptic cleft and it binds to postsyn-
and the amygdala. aptic nicotinic ACh receptors which are ligand-
gated ion channels selective for Na+ and K+
27. e—All taste fibers synapse in the nucleus ions, with subsequent current flow producing
ambiguus membrane depolarization (end-plate potential,
EPP). The EPP is a graded potential (rather
Taste is detected by taste receptor cells, which are than an all-or-none response) with an ampli-
located on specialized papillae of the taste buds tude directly related to the quantity of
and are stimulated by taste chemicals. The cellu- neurotransmitter (ACh) released from the pre-
lar mechanism for transduction of taste stimuli synaptic terminals. The amplitude of the EPP
depends upon the stimulus. Receptors for mole- can be much greater that of the excitatory and
cules associated with sweet and bitter tastes utilize inhibitory postsynaptic potentials in CNS syn-
second messengers, while those associated with apses. At the neuromuscular junction, ACh is
sour and salty-tasting molecules act directly upon enzymatically degraded by acetylcholinesterase
the ion channels. Taste buds on the anterior into acetate and choline. Choline is then taken
two thirds of the tongue have fungiform papillae up by the presynaptic terminal.
and primarily detect sweet and salty tastes. They
send signals centrally through the lingual nerve to 29. d—Axonotmesis shows Wallerian degenera-
the chorda tympani and finally into CN VII tion distal to injury
(facial). Taste buds on the posterior one third of
the tongue have circumvallate papillae and foliate At times, it is difficult to tell what form of injury
papillae, which detect bitter and sour tastes. Most a patient has sustained. Certainly, if a patient
of them send signals centrally through CN IX has a dense motor and sensory deficit following
(glossopharyngeal); however, some located in the a penetrating injury, it probably represents
back of the throat and epiglottis send signals a neurotmesis and the patient will benefit from
centrally through CN X (vagus). CN VII, IX, an exploration and nerve repair. On the other
and X synapse with the tractus solitarius (solitary hand, if a patient sustained blunt trauma to the
nucleus). Second-order neurons leave the solitary upper extremity and now has a partial sensory
nucleus and project ipsilaterally to the ventral and motor deficit, it is difficult to know what
posterior medial nucleus of the thalamus. Neurons form of nerve injury they have sustained. Explo-
from the thalamus project to the taste cortex ration of this wound may not be indicated imme-
located in the primary somatosensory cortex. diately following the injury and the wait-and-see
Taste discrimination and perception occur as a approach may be more appropriate. Surgical
result of the comparison of the activation pattern repair may involve end-to-end neurorrhaphy
of different groups of taste fibers. (either epineural repair or fascicular repair with
cable nerve grafts), nerve graft reconstruction
28. b—End plate potential amplitude is larger of peripheral nerve (using donor nerves), neural
than that of excitatory or inhibitory postsyn- conduit (e.g., if significant peripheral nerve gap)
aptic potentials and less frequently, end-to-side neurorrhaphy.

Classification of Peripheral Nerve Injury


Classification Grade Pathology Return of Function

Neuropraxia I Disruption of myelin sheath only. No Nearly complete in days to weeks


Wallerian degeneration

Axonotmesis II Disruption of axon only. Wallerian Nearly complete at a rate of 1 mm/day as


degeneration distal to injury endoneurial conduit remain intact
(regenerating axons do not need to traverse a
coaptation site)

Continued

Neurosurgery Books Full


www.ketabpezeshki.com 66485438-66485457
3 NEUROPHYSIOLOGY 51

Classification Grade Pathology Return of Function


III Disruption of axon and endoneurial Variable
tubes

IV Disruption of axon, endoneurial Variable


tubes, and perineurium

Neurotmesis V Disruption of axon, endoneurial Incomplete after microsurgical repair due to


tubes, perineurium and epineurium mismatching of fascicles, scarring at the
(complete transection) nerve coaptation site, and loss of axons
during regeneration

Adapted from Cederna PS, Chung KC. Nerve Repair and Nerve Grafting, In Guyuron B (Ed.), Plastic Surgery: Indications and
Practice, Elsevier, 2009.

30. c—Gamma motor neurons and unable to monitor changes in muscle length.
However, where descending inhibition on
Neuromuscular spindles are stretch receptor gamma motor neurons is impaired (e.g., UMN
organs within skeletal muscles which are respon- lesion), this can result in exquisitely sensitive
sible for the regulation of muscle tone via the stretch receptors and hyperreflexia.
spinal stretch reflex. They lie parallel to the mus-
cle fibers, embedded in endomysium or perimy-
sium. Each spindle contains 2-10 modified
skeletal muscle fibers called intrafusal fibers,
which are much smaller than skeletal extrafusal
fibers. The intrafusal fibers have a central non- Gamma
striated area in which their nuclei tend to be con- motor nerve
centrated. The two types of intrafusal fibers are
nuclear bag fiber and nuclear chain fiber. Asso- Extrafusal fibers
ciated with the intrafusal fibers are branched
non-myelinated endings of large myelinated sen- lntrafusal fibers
sory fibers which wrap around the central non- Capsule
striated area, forming annulospiral endings. Nuclear chain
Additionally, flower-spray endings of smaller fiber
myelinated sensory nerves are located on the Nuclear bag
striated portions of the intrafusal fibers. These fiber
sensory receptors are stimulated by stretching Annulospiral
of the intrafusal fibers, which occurs when the sensory nerve
(extrafusal) muscle mass is stretched. This stim-
ulus evokes a simple two-neuron spinal cord Flower spray
reflex, causing contraction of the extrafusal mus- sensory nerve
cle mass. This removes the stretch stimulus from
the spindle and equilibrium is restored (e.g.,
Alpha
knee jerk reflex). The sensitivity of the neuro- motor nerve
muscular spindle to stretch is modulated via
small gamma motor neurons controlled by the
extra-pyramidal motor system. These gamma
Image from Young B, et al, Wheater's
motor neurons innervate the striated portions Functional Histology, 6th ed. Churchill Livingstone,
of the intrafusal fibers; contraction of the intra- Elsevier, 2014.
fusal fibers increases the stretch on the fibers and
thus the sensitivity of the receptors to stretching
of the extrafusal muscle mass. During a normal ANSWERS 31–45
movement, both alpha and gamma motor neu-
rons are co-activated. If only the alpha motor Additional answers 31–45 available on
ExpertConsult.com
neurons were activated the muscle would con-
tract and the central non-contractile portion of
intrafusal muscle fibers would become slack

Neurosurgery Books Full


www.ketabpezeshki.com 66485438-66485457
52 PART I BASIC SCIENCE

EMI ANSWERS antagonises binding of ACh to the postsynaptic


nicotinic receptor.
46. 1—h, PCP; 2—a, 3,4-methylenedioxy-meth-
amphetamine; 3—f, Conotoxin; 4—i, Tetra- 47. 1—b, Dopamine. The conversion of dopamine
ethylammonium (TEA); 5—j, to norepinephrine comes about by the action of
Tetrodotoxin (TTX) the enzyme dopamine #-hydroxylase
2—h, L-DOPA. The biosynthesis of catechol-
amines includes the following steps: tyrosine
Conotoxins include: α-conotoxin inhibits nico-
is converted into L-dihydroxyphenylalanine
tinic acetylcholine receptors at nerves and mus-
(L-DOPA) by tyrosine hydroxylase.
cles, δ-conotoxin inhibits the inactivation of
L-DOPA is then decarboxylated by a decar-
voltage-dependent sodium channels, κ-conotoxin
boxylase to form dopamine (and CO2)
inhibits potassium channels, μ-conotoxin inhibits
3—j, Serotonin. Tryptophan is converted to
voltage-dependent sodium channels in muscles,
5-hydroxytryptophan by tryptophan
ω-conotoxin inhibits N-type voltage-dependent
hydroxylase and by 5-hydroxytryptophan
calcium channels. Chlorotoxin is found in the
decarboxylase into serotonin
venom of deathstalker scorpion and acts as a
4—c, Epinephrine. Norepinephrine is
Cl channel blocker. Botulinum toxin irrevers-
converted into epinephrine by
ibly blocks release of acetylcholine (ACh) from
phenylethanolamine-N-methyl transferase
presynaptic nerve terminal. The α-toxins, includ-
5—f, Glycine
ing α-bungarotoxin, can produce postsynaptic
effects similar to that observed with curare, 48. 1—e, Nuclear bag fibers; 2—c, Meissner’s
by binding specifically to the subunits of the corpuscles; 3—g, Pacinian corpuscles
nicotinic ACh receptor. Curare competitively

Types of Sensory Receptors


Type Modality Fibers Tracts

Nociceptors

Pain/Temperature Pain (mechanical, A-delta (fast) myelinated Anterolateral system


thermal, chemical) C-fiber (slow) unmyelinated

Mechanoreceptors

Free nerve endings Touch, pressure A-delta, C-fibers Anterolateral system


Merkel’s tactile discs Discriminative touch, A-beta myelinated Dorsal column-medial
superficial pressure lemniscus
Meissner’s Two-point
corpuscles discriminative fine
touch
Pacinian corpuscles Deep pressure and
vibration
Ruffini’s corpuscles Skin pressure/stretch
Peritrichial nerve Hair movement
endings

Muscle and tendon mechanoreceptors


Nuclear bag fibers Onset of muscle A-alpha myelinated fibers, muscle Dorsal column-medial
stretch spindle afferents, secondary lemniscus and
afferents spinocerebellar
Nuclear chain fibers Progress of muscle
stretch

Golgi tendon organ Tendon stretch A-alpha myelinated fibers

Neurosurgery Books Full


www.ketabpezeshki.com 66485438-66485457
3 NEUROPHYSIOLOGY 53

49. 1—k, Schwann cells; 2—h, Microglia; 3—i, 51. 1—m, 2—d, 3—k, 4—a, 5—g
Oligodendrocytes; 4—a, Astrocytes; 5—d,
Ependymal cells A, Dendritic tree of one Golgi cell; b, Stellate
cells; c, Molecular layer; d, Basket cell; e, Den-
Neurons are classified as shown below: dritic tree of one Purkinje cell; f, Axon of basket
1. Multipolar (several dendrites and one axon) cell; g, Parallel fibers; h, Granule cell; i, Synaptic
neuron with either a long axon (Golgi type glomerulus; j, Ramification of Golgi cell axon in
I, e.g., Betz cell, Martinotti cell) or short/no granular layer; k, Climbing fibers; l, Mossy fibers;
axon (Golgi type II, e.g., interneuron) m, Axons of Purkinje cells.
2. Bipolar (single dendrite and axon, cell body
close to sensory receptor) neuron, e.g., ret- Afferent projections to the cerebellum arise princi-
ina, auditory, vestibular, olfactory system pally from the spinal cord (spinocerebellar fibers),
3. Unipolar (single process which is structur- inferior olivary nucleus (olivocerebellar fibers), ves-
ally and functionally an axon, cell body far tibular nuclei (vestibulocerebellar fibers) and pons
from sensory receptor) neuron, e.g., (pontocerebellar fibers). Afferent axons mostly ter-
peripheral sensory afferents minate in the cerebellar cortex, where they are
excitatory to cortical neurons. Fibers enter the cer-
50. 1—l, 2—o, 3—k, 4—d, 5—h ebellum through one of the cerebellar peduncles
and proceed to the cortex as either mossy fibers or
a, Superior cerebellar arteries; b, Superior cere- climbing fibers, depending upon their origin. All
bellar surface; c, Inferior cerebellar surface; d, afferents originating elsewhere than the inferior
Dentate nucleus; e, Emboliform nucleus; f, Cer- olivary nucleus end as mossy fibers. Mossy fibers
ebellar nuclei: Globose; g, Cerebellar nuclei: Fas- branch to supply several folia and end in the gran-
tigial; h, Anterior inferior cerebellar arteries; i, ular layer, in synaptic contact with granule cells.
Posterior inferior cerebellar artery; j, Anterior The axons of granule cells pass towards the surface
lobe; k, Primary fissure; l, Posterior lobe; m, Hor- of the cortex and enter the molecular layer. Here
izontal fissure; n, Posterolateral fissure; o, Floccu- they bifurcate to produce two parallel fibers that
lonodular lobe: Flocculus; p, Flocculonodular are oriented along the long axis of the folium.
lobe: Nodulus. The Purkinje cell layer consists of a unicellular layer
of the somata of Purkinje neurons. The profuse
An unfolded view of the cerebellum showing dendritic arborizations of these cells extend
medial (vermal including nodulus, gray), interme- towards the surface of the cortex, into the molecular
diate (paravermal, green), and lateral (hemispheric layer. The arborizations are flattened and oriented
including flocculus, blue) zones on the right and at right angles to the long axis of the folium. They
their functionally associated deep cerebellar are, therefore, traversed by numerous parallel
nuclei (fastigial, globose/emboliform, and dentate fibers, from which they receive excitatory synaptic
respectively) on the left in the corresponding input. Inhibitory modulation of intracortical cir-
color. The general areas of cortex and nuclei cuitry is provided by numerous other neurons
served by the cerebellar arteries are also indicated known as Golgi, basket and stellate cells. The axons
on the left. Roman numerals indicate lobules of of Purkinje cells are the only axons to leave the cer-
the vermis; numerals preceded by H indicate ebellar cortex. Most of these fibers do not leave the
the corresponding lobules of the hemisphere. cerebellum entirely but end in the deep cerebellar
Note that the IX represents the uvula and HIX nuclei. The other type of afferent fiber entering
the cerebellar tonsil. In general, cerebellar struc- the cerebellar cortex, the climbing fiber, originates
ture is classified by gross anatomical features from the inferior olivary nucleus of the medulla.
(anterior, posterior and flocculonodular lobes These fibers provide relatively discrete excitatory
with primary, horizontal and posterolateral fis- input to Purkinje cells. At the same time, axon col-
sures), by organization of cortical projections laterals of climbing fibers excite the neurons of the
(vermal, paravermal, hemispheric) and by func- deep cerebellar nuclei. Purkinje cells utilize GABA
tional modules (vestibulocerebellum, spinocere- as their neurotransmitter, which means that the
bellum, and pontocerebellum). output of the whole of the cerebellar cortex is medi-
ated through the inhibition of cells in the cerebellar
Image adapted from Haines DE. Fundamental Neurosci-
ence for Basic and Clinical Applications, 4th ed. Saun- nuclei.
ders, Elsevier, 2013.
Image adapted from Mancall EL, Gray's Clinical Neuro-
anatomy: The Anatomic Basis for Clinical Neurosci-
ence, Elsevier, Saunders, 2011

Neurosurgery Books Full


www.ketabpezeshki.com 66485438-66485457
54 PART I BASIC SCIENCE

52. 1—i, Oxytocin; 2—b, ADH (Vasopressin); 3—j, Prolactin

Clinical Features of Endocrine Dysfunction


Hormone Effect Deficit Excess

TSH Production of T3/T4 in Hypothyroidism Hyperthyroidism


thyroid

ACTH Adrenal cortex Secondary adrenal Cushing’s disease (most common)


production of cortisol insufficiency
and mineralocorticoids

Oxytocin Uterine contractions in


labor, milk ejection
reflex

Prolactin Lactation Lactation failure, male Galactorrhea, amenorrhea,


reduced fertility, anovulatory infertility,
ovarian dysfunction gynaecomastia, loss of libido, sexual
dysfunction

ADH (vasopressin) Renal water Diabetes insipidus SIADH


conservation

GH Stimulate production GH deficiency Acromegaly


of insulin-like growth
factors

FSH Ovulation in women Infertility


Sperm production in
men

LH Sex hormone Hypogonadotrophic


production by gonads (secondary)
hypogonadism

Cortisol Metabolism and stress Addison’s disease Cushings syndrome


response (primary adrenal
insufficiency)

53. 1—i, 2—f, 3—a, 4—d polyfascicular. The fascicular pattern of a periph-
eral nerve has important implications when
a, Mesoneurium; b, Grouped fascicle; c, Longitu- trying to repair an injured or divided peripheral
dinal vessels on the extrinsic epineurium; d, nerve. The size, quantity, and ultrastructure of
External epineurium; e, Internal epineurium; f, nerve fascicles vary greatly along the length of a
Perineurium; g, Myelin; h, Fascicle; i, Endoneur- given nerve. Within the perineurium, individual
ium; j, Axon. nerve axons are surrounded by a layer of connec-
Peripheral nerves are composed of connective, tive tissue known as the endoneurium. Each nerve
vascular, and neural tissue. The connective tissue has a vascular network along its entire length
components include the epineurium, perineu- including arteries, veins, and capillaries. These vas-
rium, and endoneurium. The epineurium consists cular components of the peripheral nerve are ana-
of loose collagenous connective tissue that either tomically separated from the neural components of
encloses groups of nerve fascicles (external epi- the nerve by a blood-brain barrier. Finally, the
neurium) or cushions fascicles from external pres- peripheral nerve contains myelinated and unmy-
sure and trauma to prevent injury (internal elinated motor, sensory, and sympathetic nerve
epineurium). The amount of external and inter- fibers. The size and number of axons varies greatly
nal epineurium varies greatly among individuals, within all nerves. The cell bodies for these nerves
peripheral nerves, and even within a single nerve. fibers are contained within the spinal cord.
The perineurium surrounds individual nerve
fascicles and defines the fascicular pattern of a Image adapted from Berger RA, Weiss APC. Hand Sur-
given nerve: monofascicular, oligofascicular, or gery, Volume 1. Lippincott, Williams and Wilkins, 2003.

Neurosurgery Books Full


www.ketabpezeshki.com 66485438-66485457
CHAPTER 4

NEUROPATHOLOGY I: BASICS
SINGLE BEST ANSWER (SBA) QUESTIONS
1. A 43-year-old South Asian man is brought 4. Which one of the following pathologies is
into the emergency department with general- most likely to exhibit the finding shown?
ized seizures and fever >38 (101F). CT head
does not show any abnormality. LP is per-
formed with an opening pressure of 22 cm
H2O and CSF analysis shows: WCC 748
(Polymorphs 113, Lymphocytes 635), RBC
28, normal protein and normal glucose.
Which one of the following is the most likely
cause?
a. Enterovirus
b. Listeria monocytogenes
c. Mycobacterium tuberculosis
d. Streptococcus pneumoniae
e. Wegener's granulomatosis

2. A 45-year-old woman presents with sudden


onset headache and photophobia. CT head a. Ataxia telangiectasia
is unremarkable and she undergoes a lumbar b. Neurofibromatosis-1
puncture. CSF analysis shows WCC 3, RBC c. Neurofibromatosis-2
15000 and subarachnoid hemorrhage cannot d. Sturge-Weber syndrome
be excluded due to the presence of oxyhemo- e. Tuberous sclerosis
globin. CSF xanthochromia is detected by
which one of the following assays? 5. Which one of the following is most accurate
a. Fluorescence in situ hybridization regarding tumors with 0-6-methylguanine-
b. Immunoprecipitation DNA methyltransferase methylation?
c. Light microscopy a. More susceptible to alkylating agents
d. Spectroscopy b. More susceptible to antimetabolites
e. Western blotting c. More susceptible to antitumor antibiotics
d. More susceptible to topoisomerase
3. Which one of the following is the most inhibitors
appropriate marker for tumor proliferation? e. More susceptible to ribunucleotide reduc-
a. GFAP tase inhibitors
b. Ki-67
c. LDH 6. Which one of the following genetic muta-
d. P53 tions are associated with improved brain
e. S100 tumor prognosis?
a. Loss of 1p/19q
b. Loss of 1p/22q
c. Loss of 1p/10q
d. Loss of 1p/10q
e. Loss of 1p/10q

55
Neurosurgery Books Full
www.ketabpezeshki.com 66485438-66485457
56 PART I BASIC SCIENCE

7. Which one of the following types of cerebral 11. Which one of the following best describes
edema is seen in malignant hypertension? the finding below?
a. Cytotoxic
b. Hydrostatic
c. Interstitial
d. Osmotic
e. Vasogenic

8. Cerebral ischemia is usually seen when global


cerebral blood flow is below:
a. 60 ml per 100 g tissue per min
b. 50 ml per 100 g tissue per min
c. 40 ml per 100 g tissue per min
d. 30 ml per 100 g tissue per min
e. 20 ml per 100 g tissue per min

9. Which one of the following descriptions sug-


gest WHO grade II astrocytoma? a. Cowden syndrome
a. Microcystic change b. Gorlin syndrome
b. Nuclear atypia and hyperchromasia c. MEN1
c. >10 mitoses per high power field d. Tuberous sclerosis
d. Numerous mitoses and anaplasia e. Von Hippel Lindau
e. Microvascular proliferation or necrosis
12. Which one of the following genetic mutations
10. Which one of the following best describes is most likely seen with the finding below?
the finding below?

a. 9q34/16p13
b. 3p25
c. 17p13
a. Ash-leaf (macule) d. 9q22
b. Café-au-lait spot e. 5q21
c. Plexiform neurofibroma
d. Port wine stain
e. Shagreen patch

Neurosurgery Books Full


www.ketabpezeshki.com 66485438-66485457
4 NEUROPATHOLOGY I: BASICS 57

13. Which one of the following best describes 15. Which one of the following best describes
the finding shown? the finding shown?

a. Angiofibroma
b. Collagenoma
c. Neurofibroma
d. Neuroma a. Cowden syndrome
e. Periungual fibroma b. McCune-Albright syndrome
c. Neurofibromatosis type 1
14. Which one of the following findings are most d. Neurofibromatosis type 2
likely associated with the clinical feature below? e. Rhabdoid tumor syndrome

16. Which one of the following best describes


the finding shown?

a. Brainstem arteriovenous malformation


b. GI polyps
c. Optic glioma
d. Retinal hamartoma
e. Sensorineural deafness

a. Acanthosis nigricans
b. Legius syndrome
c. Muenke syndrome
d. Neurofibromatosis type 2
e. Pfeiffer syndrome

Neurosurgery Books Full


www.ketabpezeshki.com 66485438-66485457
58 PART I BASIC SCIENCE

17. Which one of the following best describes 19. Which one of the following is most likely in
the finding shown? the image shown?

a. Cowden syndrome a. Crouzon syndrome


b. Legius syndrome b. Familial adenomatous polyposis syndrome
c. Neurofibromatosis type 1 c. Gardener's syndrome
d. Rhabdoid tumor syndrome d. Gorlin syndrome
e. Tuberous sclerosis complex e. Osler-Weber-Rendu syndrome

18. Which one of the following is most likely in 20. Which one of the following CNS manifesta-
the image shown? tions are associated with the condition sug-
gested below?

a. Aqueduct stenosis
b. DNET
c. Leptomeningeal angiomatosis
d. Skull base meningioma
e. Sphenoid wing dysplasia

QUESTIONS 21–23
a. Basal cell naevus syndrome (Gorlin) Additional questions 21–23 available on
b. Hereditary Hemorrhagic Telangiectasia ExpertConsult.com
c. Neurofibromatosis type 2
d. Sturge-Weber syndrome
e. Tuberous sclerosis

Neurosurgery Books Full


www.ketabpezeshki.com 66485438-66485457
4 NEUROPATHOLOGY I: BASICS 59

EXTENDED MATCHING ITEM (EMI) e. CA15-3/CA27.29


f. CA19-9
QUESTIONS g. Calcitonin
h. CEA
24. Molecular assays of NS tumors: i. Chromogranin-A
a. B-cell and T-cell receptor gene rearran- j. Cytokeratin fragment 21-1
gement k. HE-4
b. BRAF-KIAA1549 gene fusion/ l. LDH
duplication m. PSA
c. Chromosome 1p/19q loss
d. EGFR gene amplification/10q loss For each of the following descriptions, select the
e. EWSR1 gene rearrangement most appropriate answers from the list above.
f. MGMT promotor methylation status Each answer may be used once, more than once
g. Monosomy chromosome 6 or not at all.
h. MYC gene amplification 1. Neuroendocrine tumors
i. SMARCB1 gene loss/INI1 protein absent 2. Multiple myeloma
j. Wnt signaling pathway upregulation 3. Liver and germ cell cancers
For each of the following descriptions, select the 27. Cytopathology of neurones and glia:
most appropriate answers from the list above. a. Central chromatolysis
Each answer may be used once, more than once b. Cowdry type A intranuclear inclusions
or not at all. c. Flexner Wintersteiner rosette
d. Hirano Bodies
1. Diagnosis of atypical teratoid/rhabdoid e. Homer-Wright rosette
tumor f. Inclusion bodies
2. Response to alkylating agents in high grade
g. Lewy bodies
glioma h. Negri bodies
3. Found in most pilocytic astrocytomas i. Neurofibrillary tangles
25. Tumor syndromes: j. Perivascular pseudorosette
a. APC k. Pick Bodies
l. Pick Cells
b. MEN1
m. Schiller-Duval bodies
c. NF-1
d. NF-2 n. Verocay bodies
e. PTCH
For each of the following descriptions, select the
f. PTEN
g. SMARCB1 most appropriate answers from the list above.
Each answer may be used once, more than once
h. TP53
or not at all.
i. TSC1/TSC2
j. VHL 1. Schwannoma
2. Ependymoma
For each of the following descriptions, select the 3. Retinoblastoma
most appropriate answers from the list above. 4. Rabies virus
Each answer may be used once, more than once 5. Alzheimer's disease
or not at all.
1. Gliomas and medulloblastoma QUESTIONS 28–29
2. AT/RT
3. Hemangioblastoma Additional questions 28–29 available on
ExpertConsult.com
26. Tumor markers:
a. Alpha-fetoprotein (AFP)
b. Beta-2-microglobulin
c. Beta-human chorionic gonadotropin
d. CA-125

Neurosurgery Books Full


www.ketabpezeshki.com 66485438-66485457
60 PART I BASIC SCIENCE

SBA ANSWERS
1. a—Enterovirus

Normal CSF Bacterial meningitis Viral Chronic


Pressure 10-20 Normal/increased Normal/increased Normal/increased
Appearance Clear Turbid/purulent Clear/cloudy Clear/cloudy
WCC (/mm3) <5 >1000 500-1000 <500
Predominant cell type Lymphocyte Polymorphonuclear leukocyte Lymphocytes Lymphocytes
Protein (mg/dl) <40 >100-200 Normal or <100 Normal or <100
Glucose 2/3 serum level Reduced Normal Normal or reduced
Lactic acid (mmol/l) <3.5 >3.5 <3.5 <3.5

2. d—Spectroscopy such as temozolomide (TMZ). The less func-


tional MGMT is present in a rapidly proliferating
3. b—Ki-67 tumor such as GBM, the more effective alkylating
agents are likely to be in killing cells off by induc-
MIB-1 antibody is directed against the cell cycle- ing irreparable cytotoxic DNA damage. There-
associated antigen Ki-67 expressed in the nucleus of fore, MGMT methylation status can be used to
cells that have entered the cell cycle (i.e. exited the stratify tumors into likely TMZ-responders and
G0 (resting) phase). It aids assessment of mitotic fig- non-responders. It has been shown that GBM
ures in order to estimate the proliferative potential patients with a good performance score and
and thus aggressiveness of a tumor. Generally grade MGMT methylation benefit from post-operative
II gliomas have MIB-1 indices of 2-5% and combined TMZ and radiotherapy treatment
glioblastomas of >10%. However, tumors with (Further reading Hegi ME, Diserens AC, Gorlia
the highest proliferation index are PNETs (20- T. MGMT gene silencing and benefit from temozolo-
60%) and high-grade lymphomas (40-90%). mide in glioblastoma, N Engl J Med. 2005 Mar
10;352(10):997-1003), resulting in a significant
4. b—Neurofibromatosis-1 survival benefit.
Individuals with light irises tend to have orange or 6. a—Loss of heterozygosity on Chr 1p and 19q
brown round Lisch nodules. In an individual with
a dark iris, on slit-lamp examination reveals light- This is the signature genetic defect in classical oli-
colored nodules appear like splattered putty or godendrogliomas (WHO grade II and III). They
white paint. often occur together and can be the result of a trans-
location of 1p and 19q [t(1,19)(q10;p10)]. The pres-
Image with permission from Liu GT, et al. Neuro-
Ophthalmology: Diagnosis and Management, 2nd ed.,
ence of these mutations typically indicates an
Saunders, Elsevier, 2010. improved prognosis particularly for patients with
WHO grade III anaplastic oligodendrogliomas
5. a—More susceptible to alkylating agents (irrespective of treatment). It was also thought that
this genetic signature (particularly LOH 1p) was
Promoter methylation is a mechanism of gene- associated with a good response to early PCV che-
silencing that occurs spontaneously in many motherapy resulting in overall improved survival.
tumor types. MGMT is one of many DNA repair 1p/19q loss can also occur in mixed oligoastrocyto-
enzymes and in a tumor with MGMT gene mas. If it is the main genetic abnormality in these
silencing by promoter hypermethylation little tumors, it usually also indicates a somewhat
functional MGMT enzyme is produced. MGMT improved prognosis. There are presumed oligo-
is particularly effective in repairing DNA damage dendroglioma tumor-suppressor genes on 1p and
induced by alkylating chemotherapeutic agents 19q, but their specific identity remains unclear.

Neurosurgery Books Full


www.ketabpezeshki.com 66485438-66485457
4 NEUROPATHOLOGY I: BASICS 61

7. b—Hydrostatic

Cytotoxic Vasogenic Hydrostatic Interstitial Osmotic


Pathophysiology Metabolic failure of Increased vascular Increased Transependymal Reduced
NaKATPase pump— permeability cerebral flow of CSF plasma
reversal of osmotic capillary osmolality
gradient—Net pressure
intracellular
accumulation of Na
and H2O
Location ICF ECF ECF ECF ECF > ICF
Causes Infarction/ischemia/ Tumor, abscess, Malignant Hydrocephalus Dialysis,
anoxia, trauma, encephalitis, hypertension, SIADH, water
trauma, hypothermia, late infarction intoxication
infection/meningitis
Treatment Reversal of cause corticosteroids, Reversal of Treatment of Reversal of
reversal of primary cause hydrocephalus cause
insult

8. e—20 ml per 100 g tissue per min precisely about 70-80 ml per 100 g per min in
gray matter and 20-45 ml per 100 g per min
Normal global cerebral blood flow is 55-60 ml in white matter. Physiological and EEG
per 100 g of brain tissue per min (i.e. 700 ml/min changes associated with different CBF is out-
or 15% of resting cardiac output); more lined below:

CBF
(ml/100 g/min) EEG Physiological
35-70 Normal Decreased protein synthesis
25-35 Loss of fast beta frequencies Anaerobic metabolism; neurotransmitter release
(glutamate)
18-25 Slowing of background to 5-7 Hz Lactic acidosis; declining ATP
(theta)
12-18 Slowing of background to 1-4 Hz Ischemia: Na-K-ATPase pump failure, cytotoxic edema
(delta)
<10-12 Suppression of all frequencies Infarction: Calcium accumulation, anoxic
depolarization, cell death

9. b—Nuclearatypia and hyperchromasia or progress to higher grade lesions (survival gen-


erally >5 years). Grade III is generally reserved
WHO grade is an independent prognostic factor for tumors with histological anaplasia and brisk
and currently particularly used in determining mitotic activity; recurrence and progression is
need for adjuvant therapies (usually indicated in the rule (survival 2-3 years). Grade IV lesions
WHO III/IV lesions). Grade I lesions (pilocytic are highly aggressive tumors prone to necrosis
astrocytomas, meningiomas) are tumors with a and relentless progression to within a year if left
low proliferation rate which often can be cured untreated (note that treatment of some WHO IV
by surgery alone. Grade II lesions are often infil- lesions such as medulloblastoma and germinoma
trative, and in the glioma categories, tend to recur can be quite successful).

Neurosurgery Books Full


www.ketabpezeshki.com 66485438-66485457
62 PART I BASIC SCIENCE

10. a—Ash-leaf spot although having café-au-lait spots alone does


not allow for definitive diagnosis.
Tuberous sclerosis. Several lance-ovate (ash-leaf)
and thumbprint white macules are noted on this Image with permission from Paller AS, Mancini AJ. Hur-
infant's back. witz Clinical Pediatric Dermatology, 4th ed., Saunders,
Elsevier, 2011.
Image with permission from Paller AS, Mancini AJ. Hur-
witz Clinical Pediatric Dermatology, 4th ed., Saunders, 16. b—Legius syndrome
Elsevier, 2011.
Legius syndrome (Neurofibromatosis type 1-like
11. d—Tuberous sclerosis syndrome) is an autosomal dominant RASopathy
often mistaken for NF-1. Patients show multiple
Facial angiofibromas (“adenoma sebaceum”) are café-au-lait spots, axillary freckling, lipomas,
typically 1-4 mm, skin-colored to red, dome- macrocephaly, learning disabilities among others.
shaped papules with a smooth surface. It lacks Lisch nodules, bone abnormalities, neu-
Image with permission from Paller AS, Mancini AJ, Hur- rofibromas, optic pathway gliomas and malignant
witz Clinical Pediatric Dermatology, 4th ed., Saunders, peripheral nerve sheath tumors. Axillary freckling
Elsevier, 2011. (Crowe's sign) is present in 20-50% of individuals
with NF1 and commonly appears between 3 and
12. a—9q34 (TSC1)/16p13 (TSC2) 5 years of age.
The shagreen patch is characteristically found at Image with permission from Paller AS, Mancini AJ. Hur-
the lumbosacral area and has a peau d'orange witz Clinical Pediatric Dermatology, 4th ed., Saunders,
Elsevier, 2011.
texture.
Image with permission from Paller AS, Mancini AJ, 17. c—Neurofibromatosis type 1
Hurwitz Clinical Pediatric Dermatology, 4th ed., Saunders,
Elsevier, 2011. Neurofibromatosis type 1. Dermal and subcuta-
neous neurofibromas are rarely found before ado-
13. e—Periungual fibroma lescence. These tumors, which originate from
Schwann cells, increase in number progressively
Tuberous sclerosis. Periungual and subungual thereafter.
fibromas on the fourth finger of this adolescent
boy. Image with permission from Paller AS, Mancini AJ.
Hurwitz Clinical Pediatric Dermatology, 4th ed., Saunders,
Image with permission from Paller AS, Mancini AJ. Hur- Elsevier, 2011.
witz Clinical Pediatric Dermatology, 4th ed., Saunders,
Elsevier, 2011. 18. c—Neurofibromatosis type 2
14. e—Sensorineural deafness Plexiform neurofibromas are commonly present
at birth and can resemble giant café-au-lait spots,
Waardenburg syndrome is a group of four auto- although borders are often more irregular. With
somal dominant disorders characterized by a advancing age, plexiform neurofibromas may
white forelock (hair depigmentation), hetero- enlarge and become more elevated with a firm
chromia irides, cutaneous depigmentation and or “bag of worms” consistency.
congenital sensorineural deafness. Individuals
with the commonest type I have characteristic Image with permission from Paller AS, Mancini AJ,
facial features—broad nasal root, lateral displace- Hurwitz Clinical Pediatric Dermatology, 4th ed., Saunders,
Elsevier, 2011.
ment of the medial canthi and lacrimal punctua of
the lower lids (dystopia canthorum).
19. d—Gorlin syndrome
Image with permission from Paller AS, Mancini AJ. Hur-
witz Clinical Pediatric Dermatology, 4th ed., Saunders, Gorlin (basal cell nevus) syndrome. Shallow ery-
Elsevier, 2011. thematous depressions on the plantar surface of
an adult female with BCNS (acral pits).
15. c—Neurofibromatosis type 1
Image with permission from Paller AS, Mancini AJ. Hur-
The presence of six or more café-au-lait spots witz Clinical Pediatric Dermatology, 4th ed., Saunders,
Elsevier, 2011.
>0.5 cm in diameter in children and 1.5 cm in
adolescents suggests the possibility of NF1,

Neurosurgery Books Full


www.ketabpezeshki.com 66485438-66485457
4 NEUROPATHOLOGY I: BASICS 63

20. c—Leptomeningeal angiomatosis at detecting the classic cortical calcifications,


which are also seen. These calcifications follow
Port-wine stain (PWS) (nevus flammeus). This the convolutions of the cerebral cortex and are
lesion involves both the V1 and V2 trigeminal characterized by double-contoured parallel
dermatomes in this infant with Sturge-Weber streaks of calcification (“tram lines”). Ocular
syndrome (SWS). Nevus flammeus, or PWS, is involvement occurs in around 60% of patients
a congenital capillary malformation that may with SWS. Glaucoma is the most frequent ocular
occur as an isolated lesion or in association with finding, and it may present at any time between
a variety of syndromes (e.g. SWS, Klippel- birth and the fourth decade. It may be unilateral
Trenauny syndrome, Von Hippel Lindau syn- or bilateral, with the latter being more common
drome, Wyburn-Mason syndrome, amongst in patients with bilateral facial PWS. Vascular
others). It is often, but not always, unilateral malformations of the eye in patients with SWS
and the most common site of involvement is may involve the conjunctiva, episclera, choroid,
the face, although they may occur on any cutane- and retina. Other eye findings include nevus of
ous surface. SWS (encephalofacial or encephalo- Ota, buphthalmos, and blindness. Dermatologic,
trigeminal angiomatosis) is a neuroectodermal neurologic, and ophthalmologic follow-up is
syndrome characterized by a PWS in the distri- indicated and the primary care provider must
bution of the first (ophthalmic) branch of the provide anticipatory guidance and support.
trigeminal nerve (V1) in association with lepto- Although the primary management for seizures
meningeal angiomatosis (presenting usually with is with pharmacologic agents, surgical therapy
seizures) and glaucoma. Central nervous system may become necessary. Visually guided lobec-
disease in SWS: seizures are the most common tomy with excision of the angiomatous cortex is
CNS feature, and often have their onset during considered the primary surgical approach in
the first year of life. The seizures of SWS may patient with focal lesions. Hemispherectomy is
be difficult to control, and both early onset and often advised for patients with intractable seizures
increased seizure intensity are associated with and unihemispheric involvement. This radical
future developmental and cognitive delay. Head- therapy is often successful, with decreased seizure
aches (including migraines), stroke-like episodes, activity and, in some patients, cognitive and
focal neurologic impairments, cognitive deficits behavioral improvement.
and emotional and behavioral problems, includ-
ing depression, violent behavior, and self- Image with permission from Paller AS, Mancini AJ.
inflicted injury, are also more common in SWS. Hurwitz Clinical Pediatric Dermatology, 4th ed., Saunders,
Leptomeningeal angiomatosis is a classic compo- Elsevier, 2011.
nent of the syndrome, and lesions are frequently
ipsilateral to the cutaneous vascular stain. Cere-
bral atrophy is a frequent radiologic finding, as
ANSWERS 21–23
is enlargement of the choroid plexus and venous
abnormalities. Magnetic resonance imaging is the Additional answers 21–23 available on
modality of choice for identifying these changes, ExpertConsult.com
although computed tomography scans are better

Neurosurgery Books Full


www.ketabpezeshki.com 66485438-66485457
64 PART I BASIC SCIENCE

EMI ANSWERS

24. 1—i, 2—f, 3—b

Principal
Molecular Target Assay Clinicopathological Setting
BRAF-KIAA1549 gene fusion/duplication PCR/FISH Found in most pilocytic astrocytomas
Chromosomes 1p/19q—allelic loss PCR/FISH Aligns with chemoresponsive/good outcome
oligodendrogliomas
EGFR gene—amplification/chromosome FISH Found in most small cell astrocytic tumors
10q—loss
MGMT promoter methylation status PCR Aligns with response to alkylating agents
(e.g. temozolomide) by high-grade gliomas
Wnt signaling pathway—upregulated IHC for Aligns with low-risk childhood medulloblastoma
b-catenin
Chromosome 6—monosomy FISH Aligns with low-risk childhood medulloblastoma
MYC gene—amplification FISH Aligns with high-risk childhood medulloblastoma
SMARCB1 gene—loss/INI1 protein—absent FISH/IHC Diagnosis of atypical teratoid/rhabdoid tumor
EWSR1 gene—rearrangement FISH Diagnosis of skull/meningeal pPNET/Ewing
sarcoma
B cell/T cell receptor genes—rearrangement PCR Demonstration of clonal populations of lymphoid
cells

25. 1—a: APC (Turcot syndrome), 2—g: SMARCB1 (Rhabdoid tumor predisposition syndrome),
3—j: VHL (Von Hippel Lindau)

Gene
Syndrome Locus Gene Type(s) of CNS Neoplasm
Neurofibromatosis type 1 17q11 NF1 Neurofibroma, malignant nerve sheath tumor, optic
nerve glioma, meningioma
Neurofibromatosis type 2 22q12 NF2 Schwannoma, meningioma, ependymoma
Tuberous sclerosis 9q34/16p13 TSC1/TSC2 Subependymal giant cell astrocytoma
Von Hippel Lindau 3p25 VHL Hemangioblastoma
Li-Fraumeni syndrome 17p13 TP53 Glioma
Gorlin syndrome 9q22 PTCH Medulloblastoma
Turcot syndrome 5q21 APC Astrocytoma, glioblastoma, medulloblastoma
Cowden disease 10q23 PTEN Dysplastic gangliocytoma of cerebellum
Multiple endocrine 11q13 MEN1 Pituitary adenoma
neoplasia type 1
Rhabdoid tumor 22q11 SMARCB1 Atypical teratoid/rhabdoid tumor
predisposition syndrome

Neurosurgery Books Full


www.ketabpezeshki.com 66485438-66485457
4 NEUROPATHOLOGY I: BASICS 65

26. 1—i: Chromogranin A, 2—b: Beta-2-microglobulin, 3—a: Alpha-fetoprotein

Tumor marker Tumor


Alpha-fetoprotein Liver, germ cell
Beta-2-microglobulin Multiple myeloma, CLL, some lymphomas
Beta-human chorionic gonadotropin Choriocarcinoma, testicular cancer
Cytokeratin fragment 21-1 Lung cancer
CA15-3/CA27.29 Breast cancer
CA19-9 Pancreatic, gallbladder, bile duct, gastric cancer
CA-125 Ovarian cancer
Calcitonin Medullary thyroid cancer
CEA Colorectal cancer, breast cancer
PSA Prostate cancer
HE-4 Ovarian cancer
Chromogranin-A Neuroendocrine tumors
LDH Germ cell tumors

27. 1—n: Verocay, 2—j: Pseudorosette, 3—c: Flexner Wintersteiner rosette, 4—h: Negri bodies, 5—i:
Neurofibrillary tangles

ANSWERS 28–29
Additional answers 28–29 available on
ExpertConsult.com

Neurosurgery Books Full


www.ketabpezeshki.com 66485438-66485457
CHAPTER 5

NEUROPATHOLOGY II: GROSS


PATHOLOGY
SINGLE BEST ANSWER (SBA) QUESTIONS
1. Which one of the following is most likely 3. Which one of the following is most likely
based on the image shown below? based on the image shown below?

a. Hydrocephalus
a. Congestive edema b. Subfalcine herniation
b. Diffuse cytotoxic edema c. Tonsillar herniation
c. Focal cytotoxic edema d. Transtentorial herniation
d. Interstitial edema e. Upwards herniation
e. Vasogenic edema
4. Which one of the following is most likely
2. Which one of the following is most likely based on the image shown below?
based on the image shown below?

a. Brain stem compression


a. Arachnoid granulations b. Demyelination
b. Calcification c. Infarction
c. Dural metastasis d. Primary brainstem hemorrhagic stroke
d. Meningitis e. Subarachnoid hemorrhage
e. Venous thrombosis

66
Neurosurgery Books Full
www.ketabpezeshki.com 66485438-66485457
5 NEUROPATHOLOGY II: GROSS PATHOLOGY 67

5. Which one of the following is most likely 7. Which one of the following is most likely
based on the image shown below? based on the image shown below?

a. Alobar holoprosencephaly
a. Chiari malformation III
b. Arhinencephaly
b. Craniospinal rachischisis
c. Lobar holoprosencephaly
c. Dandy-Walker malformation
d. Semilobar holoprosencephaly
d. Semilobar holoprosencephaly
e. Syntelencephaly
e. Syntelencephaly
6. Which one of the following is most likely
8. Which one of the following is most likely
based on the image shown below?
based on the image shown below?

a. Focal cortical dysplasia


b. Lissencephaly type 1
c. Lissencephaly type 2
d. Pachygyria
e. Pick's disease
a. Chiari I malformation
b. Chiari II malformation
c. Dandy-Walker Malformation
d. Joubert syndrome
e. Rhombencephalosynapsis

Neurosurgery Books Full


www.ketabpezeshki.com 66485438-66485457
68 PART I BASIC SCIENCE

9. Which one of the following is most likely 11. Which one of the following is most likely
based on the image shown below? based on the image shown below?

a. Cortical dysplasia a. Germinal matrix hemorrhage


b. Periventricular nodular heterotropias b. Kernicterus
c. Polymicrogyria c. Periventricular leukomalacia
d. Ventriculitis d. Wilson's disease
e. X-linked lissencephaly e. X-linked adrenoleukodystrophy

10. Which one of the following is most likely 12. Which one of the following is most likely
based on the image shown below? based on the image shown below?

a. Acute diffuse hypoxia a. Huntington's disease


b. Canavan disease b. Intraventricular hemorrhage
c. Carbon monoxide poisoning c. Multiple sclerosis
d. Cerebral amyloid angiopathy d. Multiple system atrophy
e. Pachygyria e. Periventricular leukomalacia

Neurosurgery Books Full


www.ketabpezeshki.com 66485438-66485457
5 NEUROPATHOLOGY II: GROSS PATHOLOGY 69

13. Which one of the following is most likely 15. Which one of the following is most likely
based on the image shown below? based on the image shown below?

a. Left ACA infarct a. Cerebral Toxoplasmosis


b. Left pericallosal infarct b. HSV encephalitis
c. Left SCA infarct c. Mycotic aneurysm
d. Right PCA infarct d. Rosenthal fibers
e. Right pericallosal infarct e. Tuberculous meningitis

14. Which one of the following is most likely 16. Which one of the following is most likely
based on the image shown below? based on the image shown below?

a. Arteriovenous malformation
a. Brain contusion b. Capillary telangiectasia
b. CNS lymphoma c. Caverous hemagioma
c. Malignant infarction d. Developmental venous anomaly
d. Non-accidental injury e. Dural arteriovenous fistula
e. PRES

Neurosurgery Books Full


www.ketabpezeshki.com 66485438-66485457
70 PART I BASIC SCIENCE

17. Which one of the following is most likely 19. Which one of the following is most likely
based on the image shown below? based on the image shown below?

a. Germinal matrix hemorrhage


b. Hydrancephaly
c. Lissencephaly
d. Multicystic encephalopathy
a. Aneurysmal subarachnoid hemorrhage
e. Porencephalic cyst
b. Contrecoup contusion
c. Hemorrhagic stroke
20. Which one of the following is most likely
d. Ischemic stroke
based on the image shown below?
e. Vasculitis

18. Which one of the following is most likely


based on the image shown below?

a. Agyria
b. Cobblestone cortex
c. Pachygyria
d. Porencephaly
a. Kernicterus e. Schizencephaly
b. Multicystic encephalopathy
c. Pontosubicular necrosis
d. Status marmoratus
e. Ulegyria

Neurosurgery Books Full


www.ketabpezeshki.com 66485438-66485457
5 NEUROPATHOLOGY II: GROSS PATHOLOGY 71

21. Which one of the following is most likely 23. Which one of the following is most likely
based on the image shown below? based on the image shown below?

a. Astrocytoma
b. Caseous necrosis a. Idiopathic intracranial hypertension
c. Cerebral abscess b. NF-1
d. Cerebral metastasis c. Retinal detachment
e. Tumefactive demyelination d. Retinoblastoma
e. Terson's syndrome
22. Which one of the following is most likely in a
patient where the findings shown affect mul- 24. Which one of the following is most likely
tiple (3 or more) lobes of the brain? based on the image shown below?

a. Cerebral infarct a. Diffuse astrocytoma


b. Gliomatosis cerebri b. Germinal matrix hemorrhage
c. Kernicterus c. Perventricular heterotopia
d. Periventricular leukomalacia d. Tuberous sclerosis
e. Primary CNS lymphoma e. Von Hippel-Lindau

Neurosurgery Books Full


www.ketabpezeshki.com 66485438-66485457
72 PART I BASIC SCIENCE

25. Which one of the following is most likely 27. Which one of the following is most likely
based on the image shown below? based on the image shown below?

a. Fourth ventricular subependymoma a. Ependymoma


b. Duret hemorrhage b. Glioblastoma multiforme
c. Infarct of cerebellar vermis c. Meningioma
d. Myxopapillary ependymoma d. Oligodendroglioma
e. Tanycytic ependymoma e. Supratentorial PNET

26. Which one of the following is most likely 28. Which one of the following is most likely
based on the image shown below? based on the image shown below?

a. Atypical teratoid/rhabdoid tumor a. Arteriovenous malformation


b. Cerebral abscess b. Choroid plexus papilloma
c. Choroid plexus papilloma c. Glioma
d. Intraventricular meningioma d. Intracranial aneurysm
e. Mesial temporal sclerosis e. Meningioma

Neurosurgery Books Full


www.ketabpezeshki.com 66485438-66485457
5 NEUROPATHOLOGY II: GROSS PATHOLOGY 73

29. Which one of the following is most likely 31. Which one of the following is most likely
based on the image shown below? based on the image shown below?

a. Abscess a. Cerebral abscess


b. Aneurysm b. Cystic Meningioma
c. Arachnoid cyst c. Ex-vacuo dilatation
d. Glioma d. Germinoma
e. Schwannoma e. Hemangioblastoma

30. Which one of the following is most likely 32. Which one of the following is most likely
based on the image shown below? based on the image shown below?

a. Colloid cyst
b. Optic glioma
c. Pineal cyst
a. Dermoid cyst d. Pituitary adenoma
b. Diffuse astrocytoma e. Sheehan's syndrome
c. Hemorrhagic stroke
d. Intraventricular meningioma
e. Primary CNS lymphoma

Neurosurgery Books Full


www.ketabpezeshki.com 66485438-66485457
74 PART I BASIC SCIENCE

33. Which one of the following is most likely a. Cerebral vasculitis


based on the image shown below? b. Multifocal glioma
c. Multiple abscesses
d. Multiple metastasis
e. Neurofibromatosis

36. Which one of the following is most likely


based on the image shown below?

a. Arachnoid cyst
b. Dermoid cyst
c. Epidermoid cyst
d. Pineal cyst
e. Rathke's cleft cyst

34. Which one of the following is most likely


based on the image shown below?
a. Epidermoid
b. Hemangioblastoma
c. Lhermitte-Duclos disease
d. Melanoma
e. Teratoma

37. Which one of the following is most likely


based on the image shown below?

a. Colloid cyst
b. Craniopharyngioma
c. Epidermoid cyst
d. Pituitary adenoma
e. Teratoma
a. Arachnoid villus
35. Which one of the following is most likely b. Choroid plexus
based on the image shown below? c. Ecchordosis physaliphora
d. PICA aneurysm
e. Schwannoma

Neurosurgery Books Full


www.ketabpezeshki.com 66485438-66485457
5 NEUROPATHOLOGY II: GROSS PATHOLOGY 75

38. Which one of the following is most likely 40. Which one of the following is most likely
based on the image shown below? based on the image shown below?

a. Mycobacterium Tuberculosis
a. Cerebral malaria b. Pseudomonas
b. Cysticercosis c. Spirochetes (Lyme disease)
c. Herpes encephalitis d. Staphylococcus aureus
d. Purulent meningitis e. Streptococcus pneumoniae
e. Subdural empyema

39. Which one of the following is most likely 41. Which one of the following is most likely
based on the image shown below? based on the image shown below?

a. Aspergillosis
b. Candidiasis
a. Cavum septum pellucidae
c. Cryptococcosis
b. Periventricular leukomalacia
d. Cystercicosis
c. Subependymal giant cell astrocytoma
e. Toxoplasmosis
d. Subependymal heterotopia
e. Ventriculitis
42. Which one of the following is most likely
based on the image shown below?

Neurosurgery Books Full


www.ketabpezeshki.com 66485438-66485457
76 PART I BASIC SCIENCE

a. Aspergillosis a. Amoebic meningoencephalitis


b. Cerebral abscess b. Diffuse subarachnoid hemorrhage
c. Cerebral malaria c. Meningeal carcinomatosis
d. Hyatid disease d. Post-radiotherapy change
e. Listeria encephalitis e. Tuberculous meningitis

43. Which one of the following is most likely 45. Which one of the following is most likely
based on the image shown below? based on the image shown below?

a. CMV ventriculitis
b. HIV encephalitis
c. HSV encephalitis
d. Hypoxic-ischemic encephalopathy
a. Cavernoma
e. TORCH infection
b. Cerebral metastasis
c. Developmental venous anomalies
46. Which one of the following is most likely
d. Neurocysticercosis
based on the image shown below?
e. Neuronal migration disorder

44. Which one of the following is most likely


based on the image shown below?

a. Cortical tuber
b. Diffuse astrocytoma
c. Pleomorphic xanthoastrocytoma
d. Progressive multifocal leukoencephalo-
pathy
e. Rabies encephalitis

Neurosurgery Books Full


www.ketabpezeshki.com 66485438-66485457
5 NEUROPATHOLOGY II: GROSS PATHOLOGY 77

47. Which one of the following is most likely 49. Which one of the following is most likely
based on the image shown below? based on the image shown below?

a. Amyotrophic lateral sclerosis


a. Alzheimer's disease b. Cord infarction
b. Huntington's disease c. Guillain-Barre syndrome
c. Lhermitte-Duclos disease d. Radiation myelopathy
d. Parkinson's disease e. Transverse myelitis
e. Wilson's disease
50. Which one of the following is most likely
48. Which one of the following is most likely based on the image shown below?
based on the image shown below?

a. Double cortex syndrome


a. Alzheimer's disease b. Hippocampal atrophy
b. Focal cortical dysplasia c. HSV encephalitis
c. Lissencephaly type 2 d. Huntington's disease
d. Parkinson's disease e. Toxic leukoencephalopathy
e. Pick disease

Neurosurgery Books Full


www.ketabpezeshki.com 66485438-66485457
78 PART I BASIC SCIENCE

51. Which one of the following is most likely 52. Which one of the following is most likely
based on the image shown below? based on the image shown below?

a. Alzheimer's dementia
a. Amyotrophic lateral sclerosis
b. Cortical tubers
b. Cord infarction
c. Periventricular leukomalacia
c. Friedreich's ataxia
d. Subependymal heterotopia
d. Multiple sclerosis
e. Vascular dementia
e. Subacute combined degeneration

QUESTIONS 53–68

Additional questions 53–68 available on


ExpertConsult.com

SBA ANSWERS
1. e—Vasogenic edema secondary to GBM. Image with permission from Yachnis AT, Rivera-
Widened gyri, narrowing of sulci, compres- Zengotita ML. Neuropathology, High-Yield Pathology
Series, Saunders, Elsevier, 2014.
sion of ventricles may be focal or diffuse.
Vasogenic edema often associated with focal 4. a—Brain stem compression.
lesions, tumors, abscess.
Duret (secondary) hemorrhage. Hemorrhages of
Image with permission from Yachnis AT, Rivera-
Zengotita ML. Neuropathology, High-Yield Pathology the basis pontis may result from brain stem com-
Series, Saunders, Elsevier, 2014. pression secondary to downward mass effect and
herniation from above.
2. a—Arachnoid granulations. These whitish
granular structures are located at the superior Image with permission from Yachnis AT, Rivera-
Zengotita ML. Neuropathology, High-Yield Pathology
medial aspect of the cerebral hemispheres Series, Saunders, Elsevier, 2014.
near the sagital sinus. They function in resorp-
tion of CS. 5. c—Lobar holoprosencephaly.
Image with permission from Yachnis AT, Rivera-
Zengotita ML. Neuropathology, High-Yield Pathology Two distinct cerebral hemispheres have formed,
Series, Saunders, Elsevier, 2014. but there is fusion of inferior-medial structures
including the thalamus and mammillary bodies.
3. b—Subfalcine herniation There is no septum pellucidum.
Holoprosencephaly represents a spectrum of
In this image, a lesion, not visible in this image (at midline patterning defects that involve the fore-
least the lesion is not), causing significant mass brain and midline facial structures; brain mal-
effect in the right frontal lobe has caused right formation results from failure of prosencephalon
cingulate gyrus herniation under the falx. to develop into two telencephalic vesicles. Rare:

Neurosurgery Books Full


www.ketabpezeshki.com 66485438-66485457
5 NEUROPATHOLOGY II: GROSS PATHOLOGY 79

1 per 10,000 live births (but 1 in 250 spontaneous herophili; unlike Dandy-Walker) with
abortions); equal gender distribution. Genetic downward herniation of cerebellar vermis/
abnormalities (25-50%): trisomy 13 and 18, brainstem into foramen magnum and
deletion/duplication 13q, SHH (sonic hedgehog), upward herniation of midbrain (tectal beak-
ZIC2 (zinc finger protein of the cerebellum ing—prominent inferior colliculus) with
2), SIX3, TGIF. Non-genetic: maternal diabetes, aqueduct compression causing hydroceph-
retinoic acid, drug/alcohol abuse, hypercholester- alus (clival hypoplasia). Associated with
olemia. Genetic counseling: risk of recurrence lumbosacral myelomeningocele.
after affected sibling estimated 6%. Presentation 3. Type III (rare): cerebellar vermis, cerebel-
variable: arhinencephaly least severe (anosmia, lar hemisphere and brain stem ¼/ part
single central incisor), cleft lip/palate, hypotelor- of fourth ventricle protrude through fora-
ism, flat single nostril nose/cebocephaly, micro- men magnum; associated with occipital
cephaly, hydrocephalus, most severe cyclopsia encephalocele.
with proboscis-like structure emanating from 4. Type IV (rare): cerebellar hypoplasia.
forehead. Prognosis depends on type and associ-
ated anomalies—high incidence of fetal demise Image with permission from Yachnis AT, Rivera-
Zengotita ML. Neuropathology, High-Yield Pathology
in severe cases, cognitive delay, epilepsy, mental Series, Saunders, Elsevier, 2014.
retardation, endocrine abnormalities; less severe
cases have normal brain development with mild 7. c—Dandy-Walker malformation is charac-
facial anomalies. Classification: terized by loss of cerebellar vermis with
1. Alobar—complete failure in forebrain sep- dilatation of fourth ventricle. Incidence 1 in
aration resulting in single holospheric 25,000-30,000. Genetics: possible loci on
cerebrum. chromosomes 3 (ZIC1), 6 (ZIC4), 9, partial
2. Semilobar—frontal and parietal lobes trisomy 13q, 18, autosomal dominant 2q36;
appear fused but posterior interhemi- possible association with first trimester infec-
spheric fissure present. tions and warfarin. Presentation: delayed
3. Lobar—only rostral most areas of cerebral motor development, increasing head circum-
hemispheres show fusion. ference, raised ICP, abnormal breathing pat-
4. Syntelencephaly (middle hemisphere vari- terns, associated congenital heart defects,
ant)—hemispheres separated rostrally and visual problems (nystagmus, cataracts, retinal
caudally except near posterior frontal lobe/ dysgenesis, coloboma). Prognosis depends
parietal lobe. on severity of brain and systemic manifesta-
5. Arrhinencephaly—absent olfactory bulbs, tions. Imaging—posterior fossa cyst. Gross
olfactory tracts and gyri recti. pathology: partial or complete absence of cer-
ebellar vermis, posterior fossa cyst continuous
Image with permission from Yachnis AT, Rivera-
Zengotita ML. Neuropathology, High-Yield Pathology with fourth ventricle and congenital hydro-
Series, Saunders, Elsevier, 2014. cephalus. Other cerebellar vermian malforma-
tions include Joubert syndrome (autosomal
6. b—Chiari II malformation—small cerebellum recessive; vermis agenesis, molar tooth sign
with marked tonsillar and vermian herniation. as deep interpeduncular fossa with thick-
ened/elongated superior cerebellar peduncles)
Chiari malformations are structural defects of the and rhombencephalosynapsis (fused cerebel-
cerebellum and brain stem associated with reduced lar hemispheres, no vermis, associated septo-
volume posterior fossa. Incidence 1 per 1000 live optic pituitary dysplasia, poly/syndactyly).
births; commoner in females; Chiari type 1 com-
monest. Presentation: neck pain, balance/incoordi- Image with permission from Yachnis AT, Rivera-
nation, weakness, numbness, swallowing, hearing, Zengotita ML. Neuropathology, High-Yield Pathology
Series, Saunders, Elsevier, 2014.
vomiting, insomnia, depression, high pressure
headache; asymptomatic (incidental); syringomye- 8. d—Pachygyria—enlarged gyri adjacent to
lia. Type I is usually asymptomatic—surgery only sylvian fissure.
to reduce symptoms/halt CNS injury, shunting
for hydrocephalus in Chiari II. Classification: Neuronal migration defects result in abnormal
1. Type I (commonest): extension of tonsils cortical development due to abnormal migration
through FM without brainstem involve- of young neurons from periventricular sites of
ment; synringomyelia. production to the cortex, but also likely to involve
2. Type II (“Arnold-Chiari” malformation): dysfunctional stem cell generation, neuronal
small posterior fossa (low lying torcular differentiation, synaptogenesis and functional

Neurosurgery Books Full


www.ketabpezeshki.com 66485438-66485457
80 PART I BASIC SCIENCE

organization. Rare; neurons and glial cells pro- unilateral or bilateral and symmetric or
duced in subventricular zones migrate to cortex asymmetric; acquired cases CMV infection,
in inside out fashion—neurons forming deep cor- hypoxic injury, in utero vascular occlusion (in
tical layers migrate first then more superficial association with schizencephaly); mutation
ones; most migration defects have genetic basis. of SRPX2 (bilateral sylvian polymicrogyria),
Presentation: seizures, poor muscle tone/func- PAX6, TBR2, GPR56.
tion, developmental delay, mental retardation, 7. Focal cortical dysplasia-microdysgenesis—
failure to grow/thrive, feeding difficulty, micro- focally thickened cortex with disordered
cephaly. Associated dysmorphic facial features cryoarchitecture (large abnormally ori-
or syndactyly depending on cause. Gross classifi- ented neurons, hypertrophic astrocytes);
cation though can occur in combination: for example, cortical tubers (TSC); intrac-
1. Abnormal proliferation (megancephaly) or table epilepsy.
apoptosis (microcephaly).
2. Neurons do not migrate away from subven- Image with permission from Yachnis AT, Rivera-
Zengotita ML. Neuropathology, High-Yield Pathology
tricular zone—subependymal/periventri- Series, Saunders, Elsevier, 2014.
cular heterotopias.
3. Neurons only migrate half way to cortex— 9. b—Periventricular nodular heterotropias.
subcortical band heterotopias.
4. Neurons reach cortex but abnormal Image with permission from Yachnis AT, Rivera-
cortical lamination—lissencephaly type Zengotita ML. Neuropathology, High-Yield Pathology
1/pachygyria. Series, Saunders, Elsevier, 2014.
5. Neurons overshoot cortex and end up in
10. a—Acute diffuse hypoxia/anoxia. A ribbon
subarachnoid space—marginal zone hetero-
effect is produced under conditions of acute
topia (leptomeningeal glioneuronal hetero-
hypoxia where the white matter appears dif-
topia) and lissencephaly type 2
(cobblestone cortex). fusely dusky while cortical ribbon appears
pale. Periventricular leukomalacia (PVL)
6. Late stage migration defects with abnormal
encompasses focal necrotic lesions and dif-
cortical organization or neuronal morphol-
fuse white matter gliosis resulting from selec-
ogy—polymicrogyria and focal cortical
tive ischemic injury of periventricular white
dysplasia/microdysgenesis.
matter during the fetal/perinatal period.
Histological classification:
Commonest ischemic brain injury in pre-
1. Periventricular heterotopia—unorganized
mature infants (4-25%); greatest risk
nodules of neurons under the ependyma of
<32/40; hypotension, sepsis, congenital car-
lateral ventricles; mutation in FNLA gene
diac disease, diaphragmatic hernia, acute
at Xq28 producing filamin A1 actin binding
chorioamnionitis. Pathophysiology: periven-
protein (fatal in males, heterogeneous in
tricular white matter is watershed perfusion
females).
zone, increased metabolic demand of mye-
2. Lissencephaly type 1 (agyria/pachygyria)—
smooth hemispheric surface lacking sulci/ linating white matter, poorly developed
autoregulatory mechanisms. Presentation:
gyri and only four cortical layers; mutation
cerebral palsy (fixed or nonprogressive motor
in LIS1 gene (17p13), complete loss fatal,
disorder resulting from lesions acquired dur-
partial ¼ seizures + retardation.
ing fetal/perinatal period; spastic diplegia),
3. Lissencephaly type 2 (cobblestone cor-
quadriplegia in severe PVL, poor suck reflex,
tex)—neuroglial tissue interrupts pia as it
developmental delay, coordination problems,
enters subarachnoid space resulting in fine
vision and hearing impairment. Prognosis
stippling; marked disorganization of neu-
depends on severity of brain injury; out-
rons, glia and blood vessels.
come/cerebral palsy difficult to predict in neo-
4. X-linked lissencephaly (double cortex syn-
drome): subcortical band heterotopia natal period. Emphasis on prevention: good
prenatal care, prompt treatment of maternal
within centrum ovale; mutation of
infection/other conditions. Gross pathology:
double-cortin gene (DCX; X22.3-q23).
5. Pachygyria—broad gyri and thick cortex ribbon effect in acute diffuse hypoxia, cavities
in periventricular deep white matter, periven-
with abnormal cryoarchitecture; metabolic
tricular lesions may become hemorrhagic.
CNS disorders.
6. Polymicrogyria—hemispheric surfaces have Image with permission from Yachnis AT, Rivera-
multiple festoon-like convolutions with Zengotita ML. Neuropathology, High-Yield Pathology
four cortical layers only; diffuse or focal, Series, Saunders, Elsevier, 2014.

Neurosurgery Books Full


www.ketabpezeshki.com 66485438-66485457
5 NEUROPATHOLOGY II: GROSS PATHOLOGY 81

11. a—Germinal matrix hemorrhage (16-18 13. d—Right PCA infarct. Remote infarct in
weeks of gestation). region of right posterior cerebral artery
appears as a depressed, cavitated area.
Germinal matrix zone is a fetal periventricular
structure that forms between the developing deep Image with permission from Yachnis AT, Rivera-
Zengotita ML. Neuropathology, High-Yield Pathology
cerebral nuclei and ependymal lining; 13-36
Series, Saunders, Elsevier, 2014.
weeks gestation; composed of immature neuroe-
pithelial cells and thin walled blood vessels with 14. 14. c—Malignant infarction. A massive
little supportive stroma. Germinal matrix hemor- right cerebral infarct (recent) resulted in
rhage refers to bleeding into the subependymal hyperemia, swelling, and right cingulate gyrus
germinal matrix zone with or without subsequent herniation.
intraventricular extension.
Classification: Image with permission from Yachnis AT, Rivera-
Grade I—subependymal hemorrhage Zengotita ML. Neuropathology, High-Yield Pathology
Grade II— IVH without HCP Series, Saunders, Elsevier, 2014.
Grade III— IVH with HCP
Grade IV— IPH 15. c—Mycotic aneurysm. Vasoinvasive fungi
are revealed by Gomori methenamine silver
(GMS) stain.
FURTHER READING
Papile LA, et al. Incidence and evolution of subependymal and Image with permission from Yachnis AT, Rivera-
intraventricular hemorrhage: a study of infants with birth Zengotita ML. Neuropathology, High-Yield Pathology
weights less than 1500 g. J Pediatr 1978;92(4):529-34. Series, Saunders, Elsevier, 2014.

Risk factors: prematurity (neonates with 16. d—DVA—congenital venous malformation


birth weight 500-750 g at highest risk 45%; consisting of dilated but fully functional veins
affects 20% of neonates weighing <1500 g; rare of the superficial or subcortical cerebral vascu-
after 35 weeks; occurs in the first 5 postnatal days: lature. Most common vascular malformation
50% D1, 25% D2, 15% D3, 10% D4 onward), (2% people); accounts for 60% of all CNS vas-
acute chorioamnionitis. Causes: hypertension, cular malformations; 30% associated with
venous congestion, hypoxia, traumatic rupture. another vascular malformation (typically
Presentation: silent/asymptomatic (25-50%), AVM). Dilated-appearing superficial veins
saltatory/gradual (respiratory distress, abnormal may arise in the region of the Sylvian fissure
eye movements, hypotonia) or catastrophic as shown in the image. Benign lesion not
(acute IVH with bulging fontanelle, split sutures, requiring intervention.
reduced GCS, focal neurology, hypotension).
Image with permission from Yachnis AT, Rivera-
Sequelae: cerebral palsy, seizures, mental retarda- Zengotita ML. Neuropathology, High-Yield Pathology
tion, coma/death, posthemorrhagic hydrocepha- Series, Saunders, Elsevier, 2014.
lus, myelination delay. Mortality: 5% in those
with Grade I + II GMH (7% develop hydrocepha- 17. b—Contrecoup contusions. The orbital
lus), 20% in Grade III + IV (75% develop frontal gyri and inferior lateral surfaces of
hydrocephalus). Management is supportive with the temporal lobes are typical sites of contre-
intraventricular taps and shunting for those with coup contusional injury.
posthemorrhagic hydrocephalus.
Image with permission from Yachnis AT, Rivera-
Image with permission from Yachnis AT, Rivera- Zengotita ML. Neuropathology, High-Yield Pathology
Zengotita ML. Neuropathology, High-Yield Pathology Series, Saunders, Elsevier, 2014.
Series, Saunders, Elsevier, 2014.
18. a—Kernicterus. Yellow discoloration of the
12. e—Periventricular leukomalacia (hemor- subthalamic nuclei with lighter yellow stain-
rhagic). Areas of periventricular necrosis ing of the thalamus and basal ganglia. Neuro-
may undergo extensive hemorrhage. nal necrosis and yellow staining of deep
cerebral and brain stem nuclei associated
Image with permission from Yachnis AT, Rivera-
with infantile hyperbilirubinemia.
Zengotita ML. Neuropathology, High-Yield Pathology
Series, Saunders, Elsevier, 2014.
Image with permission from Yachnis AT, Rivera-
Zengotita ML. Neuropathology, High-Yield Pathology
Series, Saunders, Elsevier, 2014.

Neurosurgery Books Full


www.ketabpezeshki.com 66485438-66485457
82 PART I BASIC SCIENCE

19. e—Porencephaly (“hole in brain”; porence- 22. b—Gliomatosis cerebri. Gross brain section
phalic cysts) refers to a spectrum of cystic showing subtle effacement of gray and white
lesions resulting from loss of neural tissue matter structures. Compare the affected right
(encephalomalacia) between the subpial cor- basal ganglia and surrounding structures with
tical surface and ependymal lining of ventri- the unaffected left side. Tumor cells were
cles. Prevalence <1 per 200,000; thought to found in contiguous frontal, temporal, and
result from large vessel occlusion/spasm dur- parietal lobes.
ing gestation (emboli, lupus, maternal
cocaine), but familial version due to mutation Image with permission from Yachnis AT, Rivera-
Zengotita ML. Neuropathology, High-Yield Pathology
in COL4A1 gene. Presentation: delayed Series, Saunders, Elsevier, 2014.
growth and development, spastic paresis,
hypotonia, poor or absent speech, epilepsy, 23. b—NF-1 associated optic nerve glioma. Pilo-
hydrocephalus, mental retardation. Progno- cytic astrocytoma causing fusiform enlarge-
sis depends on the size and location of the ment of the optic nerve (left) in a patient
cyst, and the presence of other abnormalities. with neurofibromatosis type I.
Gross pathology: Basket brain—bilateral
severe porencephaly with persistence of Image with permission from Yachnis AT, Rivera-
mesial structures. Porencephalic cysts are Zengotita ML. Neuropathology, High-Yield Pathology
lined with white matter, in contrast to schi- Series, Saunders, Elsevier, 2014.
zencephaly, where the cyst is lined with het-
24. d—Tuberous sclerosis. This gross image is
erotopic gray matter. They are intra-axial, in
from a patient with tuberous sclerosis that
contrast to arachnoid cysts, which are extra-
shows a sharply circumscribed subependymal
axial.
giant cell astrocytoma (SEGA) arising from
Image with permission from Yachnis AT, Rivera- the lateral wall of the left lateral ventricle.
Zengotita ML. Neuropathology, High-Yield Pathology A cortical “tuber” is present in the lower left
Series, Saunders, Elsevier, 2014. side of the image.
20. e—Schizencephaly is a rare cortical malfor- Image with permission from Yachnis AT, Rivera-
mation in which gray-matter lined clefts arise Zengotita ML. Neuropathology, High-Yield Pathology
near the sylvian fissure, often with adjacent Series, Saunders, Elsevier, 2014.
polymicrogyria in the lining dysplastic gray
25. a—Fourth ventricular subependymoma.
matter. Ependyma and pia mater meet in
Grossly, subependymomas are lobular neo-
the cleft at the pial-ependymal seem. Presen-
plasms that are well demarcated from adjacent
tation is with seizures, motor and develop-
CNS tissue as in this fourth ventricular
mental delay. Gross/imaging: may be
example situated between cerebellum and
unilateral or bilateral; open lip/type II (com-
medulla. Focal hemorrhage is present.
monest type in bilateral) cleft walls separated
and filled with CSF; closed-lip/type I (com- Image with permission from Yachnis AT, Rivera-
monest in unilateral cases) cleft walls are in Zengotita ML. Neuropathology, High-Yield Pathology
apposition; frequently associated with Series, Saunders, Elsevier, 2014.
septo-optic dysplasia, gray matter heteroto-
pia, absent septum. 26. a—Atypical teratoid/rhabdoid tumor. Grossly,
tumors are soft gray, tan, and demarcated from
Image with permission from Yachnis AT, Rivera- the surrounding brain (right side of image).
Zengotita ML. Neuropathology, High-Yield Pathology
Series, Saunders, Elsevier, 2014. Image with permission from Yachnis AT, Rivera-
Zengotita ML. Neuropathology, High-Yield Pathology
21. a—Astrocytoma. Gross specimen showing an Series, Saunders, Elsevier, 2014.
ill-defined lesion with loss of gray-white
matter demarcation toward the left of 27. c—Meningioma. Meningiomas are typically
the image. firm, solid, well-circumscribed neoplasms
that are attached to the dura (upper right).
Image with permission from Yachnis AT, Rivera-
Zengotita ML. Neuropathology, High-Yield Pathology Image with permission from Yachnis AT, Rivera-
Series, Saunders, Elsevier, 2014. Zengotita ML. Neuropathology, High-Yield Pathology
Series, Saunders, Elsevier, 2014.

Neurosurgery Books Full


www.ketabpezeshki.com 66485438-66485457
5 NEUROPATHOLOGY II: GROSS PATHOLOGY 83

28. e—This intraventricular meningioma arose 35. d—Multiple metastasis. Grossly, metastatic
from the choroid plexus on the left side. It tumors are found at the junctions between
had histologic features of a fibrous cortical gray and white matter.
meningioma.
Image with permission from Yachnis AT, Rivera-
Image with permission from Yachnis AT, Rivera- Zengotita ML. Neuropathology, High-Yield Pathology
Zengotita ML. Neuropathology, High-Yield Pathology Series, Saunders, Elsevier, 2014.
Series, Saunders, Elsevier, 2014.
36. d—Melanoma. Hemorrhagic metastases may
29. e—Schwannoma. This inferior view of the be associated with significant mass effect.
brain shows a large solid tumor (left) com- Metastatic tumors that tend to undergo hem-
pressing the brain stem at the orrhage include renal cell carcinoma, mela-
cerebellopontine angle. noma, and choriocarcinoma.
Image with permission from Yachnis AT, Rivera- Image with permission from Yachnis AT, Rivera-
Zengotita ML. Neuropathology, High-Yield Pathology Zengotita ML. Neuropathology, High-Yield Pathology
Series, Saunders, Elsevier, 2014. Series, Saunders, Elsevier, 2014.

30. e—Primary CNS lymphoma. Grossly, 37. c—Ecchordosis physaliphora. Gross image
tumors are located deeply within the cerebral of skull base showing the optic chiasm (left
hemispheres in periventricular locations and center), basilar artery (left), and a focal gelat-
may contain extensive necrosis. inous mass adjacent to the basilar artery.
Such incidental notochordal rests (remnants)
Image with permission from Yachnis AT, Rivera- can be seen in 1-2% of autopsies usually
Zengotita ML. Neuropathology, High-Yield Pathology
Series, Saunders, Elsevier, 2014. located in the retroclival prepontine region,
but can be found anywhere from the skull
31. e—Hemangioblastoma. This image shows a base to the sacrum. Ecchordosis physaliphora
classic gross appearance of a large cerebellar arise from remaining notochord cells along
cyst with a hyperemic mural tumor nodule. the axis of the spine after embryogenesis.
Unfortunately, ecchordosis physaliphora and
Image with permission from Yachnis AT, Rivera- chordoma are histologically indistinguishable,
Zengotita ML. Neuropathology, High-Yield Pathology other than by examining the margins, the later
Series, Saunders, Elsevier, 2014. demonstrating infiltrative growth.
32. d—Large soft tan-brown pituitary adenoma. Image with permission from Yachnis AT, Rivera-
Zengotita ML. Neuropathology, High-Yield Pathology
Image with permission from Yachnis AT, Rivera- Series, Saunders, Elsevier, 2014.
Zengotita ML. Neuropathology, High-Yield Pathology
Series, Saunders, Elsevier, 2014.
38. d—Purulent meningitis. A purulent exudate
covers the frontal region of the brain in this
33. e—Rathke's cleft cyst. Grossly, Rathke's cleft
superior view. Hyperemia of the superficial
cysts have a thin cyst wall and may be adher-
blood vessels is also typical.
ent to the adjacent infundibular stalk or infe-
rior hypothalamus. Image with permission from Yachnis AT, Rivera-
Zengotita ML. Neuropathology, High-Yield Pathology
Image with permission from Yachnis AT, Rivera- Series, Saunders, Elsevier, 2014.
Zengotita ML. Neuropathology, High-Yield Pathology
Series, Saunders, Elsevier, 2014.
39. e—Ventriculitis. The acute purulent lepto-
meningeal exudate can extend into the ven-
34. a—Colloid cyst of third ventricle. This col-
tricular system to cause ependymitis
loid cyst fills the third ventricle and obstructs
(ventriculitis) that can lead to obstructive
both foramina of Monro causing significant
hydrocephalus.
hydrocephalus.
Image with permission from Yachnis AT, Rivera-
Image with permission from Yachnis AT, Rivera- Zengotita ML. Neuropathology, High-Yield Pathology
Zengotita ML. Neuropathology, High-Yield Pathology Series, Saunders, Elsevier, 2014.
Series, Saunders, Elsevier, 2014.

Neurosurgery Books Full


www.ketabpezeshki.com 66485438-66485457
84 PART I BASIC SCIENCE

40. a—Mycobacterium Tuberculosis. Thick, (80% HIV; also hematological malignancy,


grayish exudates are typically located at the post-transplant, other malignancy). Gross
base of the brain. Basilar meningitis is com- image showing confluence of multiple areas
monly TB or cryptococcal, and less com- of demyelination to forma unifocal lesion
monly syphilis, spirochetes and autoimmune on the right. The adjacent cortical ribbon
conditions. Risk factors include AIDS and is spared.
other causes of immunocompromise.
Image with permission from Yachnis AT, Rivera-
Image with permission from Yachnis AT, Rivera- Zengotita ML. Neuropathology, High-Yield Pathology
Zengotita ML. Neuropathology, High-Yield Pathology Series, Saunders, Elsevier, 2014.
Series, Saunders, Elsevier, 2014.
47. a—Alzheimer's disease. Most cases show dif-
41. c—Cerebral cryptococcosis. The presence of fuse cerebral atrophy with widening of the
multifocal gelatinous cysts within the bilat- sulci and narrowing of the gyri. There is also
eral basal ganglia is a classic pattern of symmetrical ventriculomegaly with blunting
CNS involvement by Cryptococcus in the of the lateral ventricular angles (ex vacuo
immunocompromised host. hydrocephalus).
Image with permission from Yachnis AT, Rivera- Image with permission from Yachnis AT, Rivera-
Zengotita ML. Neuropathology, High-Yield Pathology Zengotita ML. Neuropathology, High-Yield Pathology
Series, Saunders, Elsevier, 2014. Series, Saunders, Elsevier, 2014.

42. a—Aspergillosis. Multiple hemorrhagic in- 48. e—Pick disease. Characterized by sulcal wid-
farcts resulting from vasoinvasive Aspergillus ening and gyral atrophy in frontal and tem-
arose in a child with severe combined immu- poral lobes, but sparing of parietal and
nodeficiency syndrome. Typical lesions are occipital lobes.
circumscribed, hemorrhagic, and softened.
Image with permission from Yachnis AT, Rivera-
Image with permission from Yachnis AT, Rivera- Zengotita ML. Neuropathology, High-Yield Pathology
Zengotita ML. Neuropathology, High-Yield Pathology Series, Saunders, Elsevier, 2014.
Series, Saunders, Elsevier, 2014.
49. a—Amyotrophic lateral sclerosis. Lateral cor-
43. d—Neurocysticercosis. Grossly two thin- ticospinal tract—degeneration manifest by
walled cysts (arrows) with scolex present in pallor on myelin-stained sections is typical.
the larger cyst.
Motor neuron disease (or amyotrophic lateral
Image with permission from Yachnis AT, Rivera- sclerosis) is a progressive neuromuscular disease
Zengotita ML.Neuropathology, High-Yield Pathology
characterized by degeneration of upper and lower
Series, Saunders, Elsevier, 2014.
motor neurons resulting in progressive skeletal
44. a—Amoebic meningoencephalitis. Gross muscle wasting and weakness leaking to respira-
pathology shows hemorrhagic necrosis of tory failure and death. Genetics: most common
basal frontal lobes, destruction of olfactory familial form is AD ALS associated with mutation
bulbs/tracts, cerebral edema, diffuse hemor- of copper/zinc superoxide dismutase (SOD1)
rhagic meningeal exudate. gene on chromosome 21. Classification: primary
lateral sclerosis affects predominantly UMNs,
Image with permission from Yachnis AT, Rivera- while progressive muscular atrophy affects
Zengotita ML. Neuropathology, High-Yield Pathology LMNs. Incidence 2 per 100,000 per year; preva-
Series, Saunders, Elsevier, 2014. lence 5 per 100,000; mean age of onset for sporadic
60 years; familial cases make up 10%. Presenta-
45. a—Cytomegalovirus ventriculitis and tion: asymmetric extremity weakness, dysphagia
encephalitis. Grossly, the ventricular surfaces and dysarthria; signs of UMN (weakness, hyperre-
are discolored and necrotic. flexia, spasticity) and LMN degeneration (weak-
Image with permission from Yachnis AT, Rivera-
ness, atrophy, hyporeflexia, fasciculations).
Zengotita ML. Neuropathology, High-Yield Pathology Median survival 3-5 years after symptom onset;
Series, Saunders, Elsevier, 2014. glutamate inhibitor may slow disease. MRI usually
normal but T2 hypointensity of motor cortex.
46. d—Progressive multifocal leukoencephalo- Histology: loss of anterior horn cells, lateral and
pathy—CNS demyelinating disease caused anterior corticospinal tract degeneration, Bunina
by the ubiquitous JC virus (John Cunning- bodies, Skein-like inclusion, Lewy-like inclu-
ham virus; papovavirus/polyomavirus), usu- sions, loss of Betz cells in M1/cranial nerve
ally affecting immunocompromised patients nuclei. Immunohistochemistry: Bunina bodies

Neurosurgery Books Full


www.ketabpezeshki.com 66485438-66485457
5 NEUROPATHOLOGY II: GROSS PATHOLOGY 85

cystatin-C positive and ubiquitin negative; Dif- Longer repeat size associated with earlier age
ferential: spinal muscular atrophy, hereditary of onset, increased severity/progression rate
spastic paraplegia, myasthenia gravis. and increased neurologic impairment (shows
genetic “anticipation”). FA is the commonest
Image with permission from Yachnis AT, Rivera- hereditary ataxia (50% cases); children and
Zengotita ML. Neuropathology, High-Yield Pathology
Series, Saunders, Elsevier, 2014.
young adults; mean 15 years, 85% before age
20; incidence 2 per 100,000 per year; preva-
50. d—Huntington's disease. Huntington's dis- lence 1 in 30,000; rare in Black/Asian popula-
ease is an autosomal dominant neurodegener- tions. Presentation: progressive lower limb
ative disease characterized by choreiform and gait ataxia, sensory loss, areflexia, dysar-
movements, psychiatric symptoms, dementia thria; progression of the disease leads to loss
and genetic expansion of the trinucleitide of vibration/JPS, areflexia of all extremities,
(CAG) repeat in the HD gene (chromosome foot deformity, scoliosis, diabetes mellitus,
4p). IAgre range 2-85 (mean 40); incidence hypertrophic cardiomyopathy/myocardial
2-4 per 1,000,000 per year, prevalence 5-8 fibrosis. Prognosis—loss of ambulation within
per 100,000; no gender predilection; inter- 15 years onset, life expectancy 30-40 years
action of mutated Huntington protein with after onset; coenzyme Q and vitamin E may
other proteins results in neuronal death. slow disease progression. Gross appearance:
Presentation: insidious onset of chorea, psy- brain usually appears normal, but possibly
chiatric symptoms (irritability, depression, atrophy of cerebellar vermis and dentate
anxiety) and cognitive impairment; successive nucleus; diffuse atrophy of spinal cord, partic-
generations have expanded repeats leaded to ularly dorsal and lateral columns. Differential:
earlier onset and more severe phenotype other hereditary ataxias, hereditary spastic
(anticipation). Eventually fatal disease. paraplegia, ataxia telangiectasia, acquired
Imaging: striatal and cortical atrophy. Gross ataxias.
appearance: marked atrophy of the caudate
Image with permission from Yachnis AT, Rivera-
nucleus and putamen is associated with Zengotita ML. Neuropathology, High-Yield Pathology
enlarged lateral ventricles (“ex vacuo hydro- Series, Saunders, Elsevier, 2014.
cephalus”). Histology shows striatal atrophy
(caudate/putamen) due to degeneration of 52. e—Vascular dementia. Foci of complete white
medium spiny neurons, and reactive gliosis. matter cavitation (upper left) may be evident.
There is also marked thinning of the corpus
Image with permission from Yachnis AT, Rivera- callosum and symmetrical widening of the lat-
Zengotita ML. Neuropathology, High-Yield Pathology
Series, Saunders, Elsevier, 2014. eral ventricles because of widespread white
matter disease.
51. c—Friedreich's ataxia. Sections of spinal cord
Image with permission from Yachnis AT, Rivera-
typically show degeneration (pallor) of the Zengotita ML. Neuropathology, High-Yield Pathology
posterior columns and lateral corticospinal Series, Saunders, Elsevier, 2014.
tracts as shown in this lumbar level. Frie-
dreich's ataxia is an autosomal recessive spino-
cerebellar degeneration due to mutation of
FRDA (frataxin) gene on chromosome 9q— ANSWERS 53–68
trinucleide repeat (GAA) expansion results
Additional answers 53–68 available on
in reduced frataxin protein (involved in oxida- ExpertConsult.com
tive phosphorylation and iron homeostasis).

Neurosurgery Books Full


www.ketabpezeshki.com 66485438-66485457
CHAPTER 6

NEUROPATHOLOGY III:
HISTOLOGY
Histological sections shown in this chapter are hematoxylin and eosin stained unless otherwise stated.

SINGLE BEST ANSWER (SBA) QUESTIONS


1. Which one of the following is the main fea-
a. Caudate nucleus
ture demonstrated in this histological section?
b. Cortex
c. Optic nerve
d. Posterior pituitary
e. Thalamus

3. Which one of the following is most likely


demonstrated in this histological section?

a. Herring bodies
b. Hirano bodies
c. Neuritic plaques
d. Rosenthal fibers
e. Verocay bodies

2. Which one of the following areas in the brain


a. Creutzfeldt-Jakob disease
is most likely demonstrated in this histologi-
b. Gemistocytic astrocytoma
cal section?
c. Lymphoma
d. Oligodendroglioma
e. Pineoblastoma

86
Neurosurgery Books Full
www.ketabpezeshki.com 66485438-66485457
6 NEUROPATHOLOGY III: HISTOLOGY 87

4. Which one of the following is the main 6. Which one of the following is the most likely
feature demonstrated in this histological diagnosis demonstrated in this histological
section? section?

a. Anaplastic astrocytoma a. Atypical meningioma


b. Pituitary adenoma b. DNET
c. Psammomatous meningioma c. Glioblastoma multiforme
d. Schwannoma d. Medulloblastoma
e. Teratoma e. Primary CNS lymphoma

5. Which one of the following is the most likely 7. Which one of the following features is dem-
diagnosis demonstrated in this histological onstrated in the center of this histological
section? section?

a. Diffuse astrocytoma
b. Germinoma
c. Glioblastoma multiforme
d. Schwannoma
e. Secretory meningioma a. Balloon cells
b. Fried egg cells
c. Glomeruloid microvascular proliferation
d. Mitotic figure
e. Pseudopalisading necrosis

Neurosurgery Books Full


www.ketabpezeshki.com 66485438-66485457
88 PART I BASIC SCIENCE

8. Which one of the following is the most likely 10. Which one of the following is the most likely
diagnosis demonstrated in this histological diagnosis demonstrated in this histological
section? section?

a. DNET a. Choroid plexus papilloma


b. Gemistocytic astrocytoma b. Gliomatosis cerebri
c. Glioblastoma multiforme c. Normal white matter
d. Medulloblastoma d. Pineocytoma
e. Oligodendroglioma e. Teratoma

9. Which one of the following is the most likely 11. Which one of the following is most likely
diagnosis demonstrated in this histological based on the histology below (low magnifica-
section? tion showed microcystic areas)?

a. Anaplastic astrocytoma
a. Anaplastic oligodendroglioma
b. Anaplastic oligodendroglioma
b. Ependymoma
c. Glioblastoma multiforme
c. Fibrous meningioma
d. Renal metastasis
d. Medulloblastoma
e. Secretory meningioma
e. Pilocytic astrocytoma

Neurosurgery Books Full


www.ketabpezeshki.com 66485438-66485457
6 NEUROPATHOLOGY III: HISTOLOGY 89

12. Which one of the following is the most likely 14. This child presented with seizures and a
diagnosis demonstrated in this histological known mutation on chromosome 16p13. A
section? supratentorial lesion was resected with the
histological appearance shown below. Which
one of the following diagnoses is most likely?

a. Alexander's disease
b. Oligodendroglioma
c. Pleomorphic xanthoastrocytoma a. Gorlin syndrome
d. Psammomatous meningioma b. NF-2
e. Reactive gliosis c. Tuberous sclerosis complex
d. Turcot syndrome
13. This histological section from an intraven- e. Von Hippel-Lindau syndrome
tricular tumor is most likely to be which
one of the following 15. Which one of the following is the most likely
diagnosis demonstrated in this histological
section?

a. Chordoma
b. Choroid plexus papilloma
c. Pineoblastoma a. Anaplastic oligodendroglioma
d. Pituitary adenoma b. Ependymoma
e. Subependymal giant cell astrocytoma c. Glioblastoma multiforme
d. Medulloblastoma
e. Retinoblastoma

Neurosurgery Books Full


www.ketabpezeshki.com 66485438-66485457
90 PART I BASIC SCIENCE

16. Which one of the following diagnoses is most 18. Which one of the following is shown in this
likely from this histological section? histological section?

a. Central neurocytoma a. Ependymal rosette


b. Chordoid glioma b. Flexner-Wintersteiner rosette
c. Ependymoma c. Homer Wright rosette
d. Pleomorphic xanthoastrocytoma d. Neurocytic rosette
e. Subependymal giant cell astrocytoma e. Perivascular pseudorosette

17. Which one of the following is the most likely 19. Which one of the following is the most likely
diagnosis demonstrated in this histological diagnosis demonstrated in this histological
section? section?

a. Central neurocytoma a. Central neurocytoma


b. Ependymoma b. Craniopharyngioma
c. Pilocytic astrocytoma c. Germinoma
d. Pituitary adenoma d. Papillary ependymoma
e. Retinoblastoma e. Subependymal giant cell astrocytoma

Neurosurgery Books Full


www.ketabpezeshki.com 66485438-66485457
6 NEUROPATHOLOGY III: HISTOLOGY 91

20. Which one of the following is the most likely 22. Histology of this fourth ventricular tumor
diagnosis demonstrated in this histological extending out of foramen of Luscka laterally
section? is most likely to represent which one of the
following?

a. Atypical meningioma
b. Choroid plexus papilloma a. Anaplastic ependymoma
c. Clear cell ependymoma b. Atypical teratoid/rhabdoid tumor
d. Glioblastoma multiforme c. Pilocytic astrocytoma
e. Schwannoma d. Pineoblastoma
e. Pontine glioma
21. Which one of the following is the most likely
diagnosis demonstrated in this histological 23. A 65-year-old male with sciatica undergoes
section from a spinal cord lesion? excision of an L3/4 spinal lesion. Which
one of the following is most likely based on
histological appearances below?

a. Chordoma
b. Oligodendroglioma
c. Pituitary adenoma a. Chordoma
d. Retinoblastoma b. Gemistocystic astrocytoma
e. Tanycytic ependymoma c. Myopapillary ependymoma
d. Pineocytoma
e. Schwannoma

Neurosurgery Books Full


www.ketabpezeshki.com 66485438-66485457
92 PART I BASIC SCIENCE

24. Histology of this lateral ventricular tumor in 26. Histology of this lateral ventricular tumor in a
a 65-year-old patient is most likely to repre- child with Li-Fraumeni syndrome is most
sent which one of the following? likely to represent which one of the following?

a. Choroid plexus carcinoma a. Choroid plexus carcinoma


b. Choroid plexus papilloma b. Hemangioblastoma
c. Ependymoma c. Hemangiopericytoma
d. Meningioma d. Intraventricular meningioma
e. Subependymoma e. Subependymal giant cell astrocytoma

25. Which one of the following is the most likely 27. A 3 year old child with known chromosome
diagnosis demonstrated in this histological 17p loss presents with a posterior fossa tumour
section from ventricular lesion? with the histological appearance shown.
Which one of the following is most likely?

a. Chordoid glioma
b. Chordoma a. Cortical tuber
c. Choroid plexus papilloma b. Embryonal carcinoma
d. Hypothalamic hamartoma c. Gemistocytic astrocytoma
e. Yolk sac tumor d. Medulloblastoma
e. Meningioma

Neurosurgery Books Full


www.ketabpezeshki.com 66485438-66485457
6 NEUROPATHOLOGY III: HISTOLOGY 93

28. In the context of positive INI-1 staining (not 30. This histological section from a suprasellar
shown), this histological section from a pos- lesion is INI-1 negative but positive for
terior fossa tumor is most likely to represent vimentin and epithelial membrane antigen.
which one of the following? Which one of the following are most likely:

a. Atypical teratoid/rhabdoid tumor a. Atypical teratoid/rhabdoid tumor


b. Hemangioblastoma b. Chordoma
c. Large cell/anaplastic medulloblastoma c. Craniopharyngioma
d. Nodular/desmoplastic medulloblastoma d. Pituitary adenoma
e. Pilocytic astrocytoma e. Pure germinoma

29. This INI-1 positive (not shown) tumor aris- 31. Which one of the following is the most likely
ing in the right frontal lobe of a 1 year old is diagnosis demonstrated in this histological
most likely to represent which one of the section?
following?

a. Chordoma
a. Chordoma b. Ganglioglioma
b. Germinoma c. Meningioma
c. Glioblastoma multiforme d. Oligodendroglioma
d. Supratentorial primitive neuroepithelial e. Schwannoma
tumor (PNET)
e. Teratoma

Neurosurgery Books Full


www.ketabpezeshki.com 66485438-66485457
94 PART I BASIC SCIENCE

32. Which one of the following is the most likely 34. Which one of the following is the most likely
diagnosis demonstrated in this histological diagnosis demonstrated in this histological
section? section?

a. Central neurocytoma a. Antoni B Schwannoma


b. Dysembryoplastic neuroepithelial tumor b. Hemangioblastoma
c. Fibrillary astrocytoma c. Hemangiopericytoma
d. Meningothelial meningioma d. Neurofibroma
e. Pineoblastoma e. Pilocytic astrocytoma

33. Which one of the following is the most likely 35. Which one of the following is the most likely
diagnosis demonstrated in this histological diagnosis demonstrated in this histological
section? section?

a. Central neurocytoma a. Choroid plexus papilloma


b. Chordoma b. Glioblastoma multiforme
c. Glioblastoma multiforme c. Meningothelial meningioma
d. Hemangioma d. Pineoblastoma
e. Oligodendroglioma e. Pituitary adenoma

Neurosurgery Books Full


www.ketabpezeshki.com 66485438-66485457
6 NEUROPATHOLOGY III: HISTOLOGY 95

36. Which one of the following is the most likely 38. Which one of the following is the most likely
diagnosis demonstrated in this histological diagnosis demonstrated in this histological
section? section?

a. Germinoma
a. Antoni A Schwannoma b. Glioblastoma multiforme
b. Fibrillary astrocytoma c. Retinoblastoma
c. Fibrous fibroblastic meningioma d. Schwannoma
d. Malignant peripheral nerve sheath tumor e. Secretory meningioma
e. Medulloblastoma
39. Which one of the following is the most likely
37. Which one of the following is the most likely diagnosis demonstrated in this histological
diagnosis demonstrated in this histological section?
section?

a. Angiomatous meningioma
a. Angiocentric glioma b. Choroid plexus papilloma
b. Central neurocytoma c. Glioblastoma multiforme
c. Chordoma d. Hemangiopericytoma
d. Medulloblastoma e. Psammomatous meningioma
e. Psammomatous meningioma

Neurosurgery Books Full


www.ketabpezeshki.com 66485438-66485457
96 PART I BASIC SCIENCE

40. Which one of the following is the most likely 42. Which one of the following is the most likely
diagnosis demonstrated in this histological diagnosis demonstrated in this histological
section? section?

a. Atypical meningioma a. Chordoma


b. Atypical teratoid/rhabdoid tumor b. Clear cell meningioma
c. Diffuse astrocytoma c. Gemistocytic astrocytoma
d. Medulloblastoma d. Pilocytic astrocytoma
e. Schwannoma e. Pituitary adenoma

41. Which one of the following is the most likely 43. Which one of the following is the most likely
diagnosis demonstrated in this histological diagnosis demonstrated in this histological
section? section?

a. Anaplastic astrocytoma a. Anaplastic meningioma


b. Chordoid meningioma b. Central neurocytoma
c. Choroid plexus papilloma c. Gemistocytic astrocytoma
d. Oligodendroglioma d. Hemangiopericytoma
e. Pituitary adenoma e. Medulloblastoma

Neurosurgery Books Full


www.ketabpezeshki.com 66485438-66485457
6 NEUROPATHOLOGY III: HISTOLOGY 97

44. Which one of the following is the most likely 46. Which one of the following is the most likely
diagnosis demonstrated in this histological diagnosis demonstrated in this histological
section? section?

a. Choroid plexus carcinoma a. Angiocentric glioma


b. Fibrillary astrocytoma b. Neurofibroma
c. Oligodendroglioma c. Primary CNS lymphoma
d. Parkinson's disease d. Psammomatous meningioma
e. Rhabdoid meningioma e. Pseudopsammomatous bodies

45. Which one of the following is the most likely 47. Which one of the following is the most likely
diagnosis demonstrated in this histological diagnosis demonstrated in this histological
section? section?

a. Central neurocytoma a. Astrocytoma


b. Choroid plexus papilloma b. Germinoma
c. Medulloblastoma c. Neurofibroma
d. Oligodendroglioma d. Schwannoma
e. Papillary meningioma e. Teratoma

Neurosurgery Books Full


www.ketabpezeshki.com 66485438-66485457
98 PART I BASIC SCIENCE

48. Which one of the following is the most likely 50. Which one of the following is the most likely
diagnosis demonstrated in this histological diagnosis demonstrated in this histological
section? section?

a. Antoni A schwannoma a. Bunina bodies


b. Epidermoid b. Hirano bodies
c. Malignant peripheral nerve sheath tumor c. Lewy bodies
d. Meningioma d. Pick bodies
e. Neurenteric cyst e. Verocay bodies

49. This dumbbell shaped mass in the spinal cord 51. Which one of the following is the most likely
most likely represents which one of the diagnosis demonstrated in this histological
following? section?

a. Antony B schwannoma a. Angiocentric glioma


b. Malignant peripheral nerve sheath tumor b. Atypical teratoid/rhabdoid tumor
c. Myxopapillary ependymoma c. Choriocarcinoma
d. Neurofibroma d. Malignant peripheral nerve sheath tumors
e. Tanycytic ependymoma e. Secretory meningioma

Neurosurgery Books Full


www.ketabpezeshki.com 66485438-66485457
6 NEUROPATHOLOGY III: HISTOLOGY 99

52. Deep cerebral lesion with the histological 54. A pineal region mass with the histological
appearance shown is most likely to be which appearance shown is most likely to be which
one of the following? one of the following?

a. Angiofibromatous meningioma a. Choriocarcinoma


b. Germinoma b. Ependymoma
c. Hemangiopericytoma c. Germinoma
d. Primary CNS lymphoma d. Pineal parenchymal tumor of intermedi-
e. Toxoplasmosis ate differentiation
e. Teratoma
53. Which one of the following is the most likely
diagnosis demonstrated in this histological 55. A child with a RB1 mutation develops a
section? supratentorial lesion with the histological
appearance shown. Which one of the follow-
ing is most likely?

a. Choroid plexus carcinoma


b. DNET
c. Gemistocytic astrocytoma a. Fibrillary astrocytoma
d. Oligodendroglioma b. Germinoma
e. Pineocytoma c. Pilocytic astrocytoma
d. Pineoblastoma
e. Secretory meningioma

Neurosurgery Books Full


www.ketabpezeshki.com 66485438-66485457
100 PART I BASIC SCIENCE

56. Which one of the following is the most likely 58. Which one of the following is the most likely
diagnosis demonstrated in this histological diagnosis demonstrated in this histological
section? section?

a. Anaplastic astrocytoma a. Craniopharyngioma


b. Epidermoid b. Creutzfeldt-Jakob disease
c. Germinoma c. Pick's disease
d. Hemangioblastoma d. Teratoma
e. Oligodendroglioma e. Yolk sac tumor

57. Which one of the following is the most likely 59. Which one of the following is the most likely
diagnosis demonstrated in this histological diagnosis demonstrated in this histological
section? section?

a. AT/RT a. DNET
b. Chordoma b. Embryonal carcinoma
c. Metastatic melanoma c. Fibrillary astrocytoma
d. Pituitary adenoma d. PNET
e. Teratoma e. Rhabdoid meningioma

Neurosurgery Books Full


www.ketabpezeshki.com 66485438-66485457
6 NEUROPATHOLOGY III: HISTOLOGY 101

60. Which one of the following is the most likely 62. Which one of the following is the most likely
diagnosis demonstrated in this histological diagnosis demonstrated in this histological
section? section?

a. Choriocarcinoma a. Craniopharyngioma
b. Clear cell meningioma b. Pineocytoma
c. Pick bodies c. Pituitary adenoma
d. Progressive nuclear palsy d. Psammomatous meningioma
e. Renal cell carcinoma metastasis e. Teratoma

61. Which one of the following is the most likely 63. Which one of the following is the most likely
diagnosis demonstrated in this histological diagnosis demonstrated in this histological
section? section?

a. Central neurocytoma
b. Hemangioblastoma
c. Neurofibroma a. Adamantinomatous craniopharyngioma
d. Schwannoma b. Chordoma
e. Secretory meningioma c. Dermoid cyst
d. Pineal parenchymal tumor of intermedi-
ate differentiation
e. Rhabdoid meningioma

QUESTIONS 64–81

Additional questions 64–81 available on


ExpertConsult.com

Neurosurgery Books Full


www.ketabpezeshki.com 66485438-66485457
102 PART I BASIC SCIENCE

SBA ANSWERS
1. d—Rosenthal fibers (RFs) and consists of neoplastic astrocytes in dense fibril-
lary background, cells often have inconspicuous
These are intracytoplasmic aggregates of glial cytoplasm: “naked” nuclei which can be atypical,
fibrillary acidic protein (GFAP) and chaperone pro- with elongation and irregular nuclear contours
teins. They are bright eosinophilic in H&E-stained and often has extensive microcyst formation.
sections and cork-screw-like or beaded. Eosino- Gemistocytic astrocytoma tumor composed of
philic granular bodies are related to Rosenthal plump, angular cells with eosinophilic, glassy cyto-
fibers and they often occur together. RFs occur plasm and short, haphazardly arranged processes
most commonly in pilocytic astrocytoma, but may (gemistocytes; at least 20% are neoplastic) with
also be seen in Grade I ganglioglioma or Grade II eccentrically located, round, hyperchromatic
pleomorphic xanthoastrocytoma, and Alexander's nuclei; perivascular lymphocytic infiltrates are
disease. They can also be seen in reactive gliosis common; higher rate of malignant transformation.
(piloid gliosis), particularly around chronic lesions Immunohistochemistry: GFAP in 100% (not spe-
in the hypothalamus, spinal cord or cerebellum cific), TP53 in >50%, S-100, Ki-67 mitotic index
(e.g., craniopharyngioma, AVM, syrinx, or <4%. Histologic differential: reactive gliosis,
granulomatous inflammation). oligodendroglioma.
Image with permission from Perry A, Brat DJ. Practical Image with permission from Yachnis AT, Rivera-
Surgical Neuropathology: A Diagnostic Approach. Zengotita ML. Neuropathology, High-Yield Pathology
Churchill Livingstone, Elsevier, 2010. Series, Saunders, Elsevier, 2014.

2. d—Posterior pituitary 4. a—Anaplastic astrocytoma (WHO III)

Posterior pituitary is formed by axonal projec- Intermediate-grade infiltrating glioma derived from
tions of neurons from the hypothalamus together malignant astrocyte-like cells; may arise from lower-
with primary glial cells and pituicytes, and Her- grade diffuse astrocytoma, and may progress to glio-
ring bodies (eosinophilic axonal dilatations that blastoma. Peak incidence in fifth decade, slight male
store neurosecretory peptides) can be seen preponderance; 10% of astrocytic tumors. Genetics:
throughout the posterior gland. 60% TP53 mutations, 40-60% loss of chromosome
10q, 10-20% PTEN losses, 30-50% p16 losses. Pro-
Image with permission from Perry A, Brat DJ. Practical
Surgical Neuropathology: A Diagnostic Approach.
gression from diffuse to anaplastic astrocytoma
Churchill Livingstone, Elsevier, 2010. accompanied by increased seizures and neurologic
deficits. Management is biopsy or resection, fol-
3. b—Gemistocytic astrocytoma lowed by chemotherapy and radiotherapy. Mean
survival 3 years from diagnosis; mean transformation
Diffuse astrocytomas are infiltrating glial neoplasm time to GBM is 2 years; better prognosis if young
with astrocytic features. WHO grade II. Peak inci- and complete resection; worse prognosis with larger
dence in fourth decade, accounting for 5% of all pri- size. MRI: T1 hypointense, T2 hyperintense, but
mary intracranial tumors (10-15% of astrocytic may have foci of gadolinium enhancement. Gross
tumors). Genetics: 60% have p53 mutation (90% appearances ill-defined without associated tissue
in gemistocytic) and subset have IDH-1 mutations. destruction. Histology similar to diffuse astrocytoma
Commonly present with seizures and progress to (fibrillary or gemistocytic) but with increased cellu-
higher grade lesions. Management is surgical resec- larity and cellular atypia; most important distin-
tion and radiotherapy. Favorable prognosis with guishing feature is presence of mitotic figures.
younger age, complete resection, and seizure at pre- May represent undersampled glioblastoma multi-
sentation. Mean survival 6-8 years. Worse progno- forme in some cases. Staining positive for GFAP,
sis if older, large size and focal neurological deficit. S100, p53 (60%), Increased Ki-67 (MIB1) labeling
MRI appearances T1 hypointense, T2/FLAIR index 5-10%; mutated IDH-1 in subset of cases. His-
hyperintense. Gross appearances of ill-defined tologic differential: diffuse astrocytoma, glioblas-
lesion with blurring of gray-white matter junction, toma multiforme.
colored gray/yellow-white without tissue destruc-
Image with permission from Yachnis AT, Rivera-
tion, possibly cystic. Histologically may be fibrillary Zengotita ML. Neuropathology, High-Yield Pathology
or gemistocytic. Fibrillary astrocytoma commonest Series, Saunders, Elsevier, 2014.

Neurosurgery Books Full


www.ketabpezeshki.com 66485438-66485457
6 NEUROPATHOLOGY III: HISTOLOGY 103

5. c—Glioblastoma multiforme Commonest primary brain tumor in adults


(15% of all intracranial); accounts for 70% of
This image shows pseudopalisading necrosis. astrocytic tumors. Mean age at presentation
Nuclear palisades may be considered “primary” 60 years old, slight male preponderance, rare in
when they reflect a natural tendency of the nuclei children. Most arise rapidly de novo (95%; pri-
to develop this distinctive pattern of growth or mary GBM) but some arise from progressive
“secondary” when the alignment forms as a transformation of lower-grade gliomas in youn-
response to external influences such as necrosis. ger patients (5%; secondary GBM). Genetics:
The latter have been termed “pseudopalisades”: Primary GBMs show loss of heterozygosity chro-
garland-like array of nuclei surrounding a region mosome 10 (70%), epidermal growth factor
of necrosis. Palisades are most often seen in (but receptor (EGFR) amplification (40%), p16 dele-
are not pathognomonic) of schwannomas whereas tion (30%), p53 mutation (30%), PTEN muta-
pseudopalisades are pathognomonic of GBM. tion (25%), mutated isocitrate dehydrogenase-1
(IDH-1) (<10%); secondary GBMs show
Image with permission from Yachnis AT, Rivera- mutated isocitrate dehydrogenase-1 (IDH-1)
Zengotita ML. Neuropathology, High-Yield Pathology (>80%), loss of heterozygosity chromosome 10
Series, Saunders, Elsevier, 2014.
(63%), p53 mutation (60%), EGFR amplification
(8%), PTEN mutation (5%). Most occur in sub-
cortical white matter, and may cross corpus callo-
sum into contralateral hemisphere (butterfly
glioma). Symptoms present abruptly and progress
rapidly, with symptoms of raised ICP, and sei-
zures in 30%. Poor prognosis even with maximal
therapy, including surgical resection (complete
Nuclear palisade
resection usually impossible given extensive infil-
Anuclear zone tration of tumor) and chemoradiotherapy (Stupp
Verocay body protocol). Prognosis is better in younger patients,
those with secondary GBM and tumors with
methylated MGMT promotor (respond better
to alkylating chemotherapy agents like temozolo-
A mide). MRI shows ring/heterogenous enhance-
ment on T1 + GAD, central hypointense lesion
with zone of surrounding edema on T2. Gross
appearance of poorly defined lesion, gray periph-
erally and yellow/tan necrosis centrally with occa-
sional hemorrhage/cyst. Histology: nuclear
atypia, pleomorphism (Figure 1), mitoses, micro-
vascular proliferation (multilayered plump endo-
thelial cells with increased mitotic activity, often
forming “glomeruloid tufts”), thrombosis and
necrosis (pseudopalisading necrosis is pathogno-
monic but not always present); “multiforme”
indicates, there can be a marked variability
between tumors which may be composed of small
monotonous cells (small cell glioblastoma), giant
Necrosis
cells, gemistocytes, granular cells, and they may
Nuclear pseudopalisade
rarely have metaplastic epithelial or mesenchymal
B elements. Immunohistochemistry: GFAP-
Redrawn with permission from Zülch KJ. Histology of positive with a high KI-67 (MIB-1) labeling
brain tumors. In: Brain Tumors: Their Biology and
Pathology. Berlin, Germany: Springer-Verlag, 1986, p.
index; p53 immunostaining is positive in tumors
118-34. with TP53 mutation; mutated isocitrate
dehydrogenase-1 (IDH-1)—positive in second-
ary GBM. Histological differential: anaplastic oli-
6. c—Glioblastoma Multiforme (“glomeruloid godendroglioma, metastatic carcinoma,
tuft” or microvascular proliferation) lymphoma, radiation necrosis.
Image with permission from Yachnis AT, Rivera-
This is the most malignant of gliomas, and has Zengotita ML. Neuropathology, High-Yield Pathology
astrocytic differentiation; WHO grade IV. Series, Saunders, Elsevier, 2014.

Neurosurgery Books Full


www.ketabpezeshki.com 66485438-66485457
104 PART I BASIC SCIENCE

8. e—Oligodendroglioma

Oligodendrogiomas are diffuse infiltrating glio-


mas with cells resembling oligodendroglia—
WHO grade II. Account for 3% of primary
brain tumors and 10% of gliomas; peak in
fourth and fifth decades; usually arise in cortex;
slight male predominance. Genetics: Isocitrate
dehydrogenase-1 (IDH-1) mutations in 75%;
80% have loss of 1p and 19q associated with
better prognosis. Presentation: seizures, head-
ache, focal deficit. Imaging: T1 hypointense
without contrast enhancement, T2 hyperin-
tense with ill-defined edges; calcification on
CT. Gross appearance: soft pink-gray, blurring
Image with permission from Yachnis AT, Rivera-
Zengotita ML. Neuropathology, High-Yield Pathology of gray-white junction; gritty if calcifications.
Series, Saunders, Elsevier, 2014. Histology: monomorphic tumor cells with ten-
dency to form small clusters, perinuclear haloes
(fried egg appearance) are an artefact of forma-
7. d—Mitotic figure (center—late anaphase) lin fixation; delicate chicken wire capillary
vasculature; perineuronal satellitosis. Immuno-
Mitosis is a continuous process that is tradition-
histochemistry: GFAP variably reactive, S100
ally divided into five phases, prophase, prometa-
positive, p53 negative, IHD-1 reactive in
phase, metaphase, anaphase and telophase, each
75%, Ki67 labeling index variable. Differential:
stage being readily recognizable with the light
DNET, diffuse astrocytoma, clear cell ependy-
microscope (see figure below).
moma. It should be noted that DNET has cells
Image with permission from Yachnis AT, Rivera- similar in histology to oligodendroglioma how-
Zengotita ML. Neuropathology, High-Yield Pathology ever are arranged in cords and have more
Series, Saunders, Elsevier, 2014. superstructure.
Image with permission from Yachnis AT, Rivera-
Zengotita ML. Neuropathology, High-Yield Pathology
Series, Saunders, Elsevier, 2014.

G2 phase Prophase Prometaphase Metaphase Astral


microtubules

Anaphase Kinetochore Telophase Cytokinesis G1 phase


microtubules
Interpolar
microtubules

Image with permission from Young B, et al. Wheater's Functional Histology, 6th ed. Churchill Livingstone: Elsevier,
2014.

Neurosurgery Books Full


www.ketabpezeshki.com 66485438-66485457
6 NEUROPATHOLOGY III: HISTOLOGY 105

9. b—Anaplastic oligodendroglioma Image with permission from Yachnis AT, Rivera-


Zengotita ML. Neuropathology, High-Yield Pathology
Anaplastic oligodendroglioma is an oligoden- Series, Saunders, Elsevier, 2014.
droglial tumor with malignant features— 11. e—Pilocytic astrocytoma
WHO grade III. Accounts for 1% of primary
brain tumors and 20-30% of oligodendroglial Pilocytic astrocytomas are WHO grade I circum-
tumors; peak age 45-50; commonly frontal then scribed, well-differentiated tumors usually pre-
temporal cerebrum. Genetics: IDH-1 mutation senting as a cystic cerebellar mass but also arise
present in majority; 1p/19q loss in 80% and in hypothalamus, optic pathway and brainstem
offers better prognosis. Presentation: seizures; (midbrain tectum or exophytic). Commonest
average progression from oligodendroglioma childhood brain tumor; overall accounts for 2%
(WHO II) to anaplastic variant is 7 years. of brain tumors and 6% of gliomas. Average
Treatment involves surgical resection, radio- age of diagnosis is 13 years old. Genetics: BRAF
therapy and chemotherapy; increased survival gene duplications on chromosome 7q34 lead to
in younger patients; median survival 7 years in KIAA1549:BRAF fusions in 80% of cerebellar,
those with 1p/19q loss, while 3 years in those 60% of optic pathway/hypothalamic and 15%
without. Imaging: as oligodendroglioma but of cerebral pilocytic astrocytomas; tumors arising
may enhance with contrast. Gross appearance: in NF1 have distinct alterations, with loss or
Well-defined soft pink-gray tumors, may have mutation in NF1 gene. Presentation can be
focal necrosis. Histology: as oligodendroglioma hydrocephalus (fourth ventricle or aqueduct
but increased cellularity and pleomorphism, compression), visual loss (optic pathway) or endo-
increased mitotic activity, microvascular prolif- crine dysfunction (hypothalamic). Prognosis is
eration, necrosis, gliofibrillary oligodendrocytes good with approximately 80% survival at 20 years;
and minigemistocytes, focal microcalcifications. better prognosis in context of NF1 and worse if
Immunohistochemistry: GFAP highly variable, supratentorial location. Imaging appearance in
negative p53, mutated IDH-1 positive. Differ- cerebellum is a cystic mass with enhancing mural
ential: oligodendroglioma (WHO II), anaplastic nodule; diffusely enhancing or cystic in hypothal-
astrocytoma, small cell glioblastoma. amus; fusiform in optic pathway. Gross appear-
Image with permission from Yachnis AT, Rivera-
ance is well-demarcated lesion with large cyst
Zengotita ML. Neuropathology, High-Yield Pathology and pink-tan, soft mural tumor nodule; may be
Series, Saunders, Elsevier, 2014. calcified. Histological appearance is biphasic
(dense microfibrillar areas vs loose, microcystic
10. b—Gliomatosis cerebri areas), bipolar piloid cells (elongated hair-like
cytoplasmic processes), round to oval nuclei, oli-
Gliomatosis cerebri is an extensively infiltrating godendrogliomatous appearance in some areas,
glioma involving at least three lobes of the brain; multinucleate cells with clustering of nuclei
typically resembles diffuse astrocytoma but (pennies on a plate), eosinophilic granular bodies,
occasionally oligodendroglioma—WHO grade Rosenthal fibers, microvascular hyperplasia,
III. Peak in fifth decade; bilateral in 75% cases; mitoses are rare. Immunohistochemistry: GFAP
can arise de novo or secondary to diffuse infiltra- positive, S-100 variable, PAS (identifies eosino-
tion by locally infiltrative glioma. Genetics: philic granular bodies and Rosenthal fibers).
unclear, some have p53 mutation. Present with Differential: Pilomyxoid variant, diffuse astrocy-
seizures, gait disturbance or dementia. Treat- toma, oligodendroglioma, reactive piloid gliosis
ment is chemotherapy or whole brain radiother- (craniopharyngioma, hemangioblastoma, syrin-
apy—poor prognosis; favorable factors are gomyelia, pineal cysts).
young age, good performance status, lower
WHO grade and histological subtype. Imaging: Image with permission from Yachnis AT, Rivera-
diffuse T2 hyperintense lesion, foci of contrast Zengotita ML. Neuropathology, High-Yield Pathology
Series, Saunders, Elsevier, 2014.
enhancement (multifocal glioma). Gross: blur-
ring/effacement of gray-white junction, gyral wid- 12. c—Pleomorphic xanthoastrocytoma
ening. Histology: diffusely infiltrating tumor with
irregular pleomorphic nuclei consistent with astro- Pleomorphic xanthoastrocytoma is a circum-
cytoma, can have other glial morphologies within scribed astrocytic neoplasm with reticulin depo-
same tumor, typically no necrosis/microvascular sition and significant pleomorphism (variability
proliferation. Immunohistochemistry: variable of cell/nuclear size, shape and staining) usually
GFAP and S100. Differential: diffuse astrocytoma, found in superficial cortex and involving the
anaplastic astrocytoma, oligodendroglioma, pro- meninges. Most are WHO grade II (anaplastic
gressive multifocal leukoencephalopathy. PXA is WHO III). Accounts for <1% all CNS

Neurosurgery Books Full


www.ketabpezeshki.com 66485438-66485457
106 PART I BASIC SCIENCE

tumors; two thirds occur in those under 18 years. Prognosis depends on severity of other TSC
Genetics: BRAF gene (7q34) point mutation manifestations. Imaging appearance is solitary,
(V600E) in two thirds of cases; 50% of lesions circumscribed intensely enhancing intraventricu-
will have loss of chromosome 9; multiple other lar mass. Gross appearance is solid well-
genetic alterations can be found but are not spe- circumscribed mass, calcification, spontaneous
cific. Usually present with prolonged history of hemorrhage. Histological appearance: astrocyte-
seizures. Surgical excision often curative, adju- like tumor cells can appear polygonal with glassy
vant therapy indicated if incomplete resection, eosinophilic cytoplasm, spindled, or epithelioid;
recurrent or anaplastic features. Malignant cells typically arranged in fascicles or nests sepa-
transformation occurs in 15-20%. Favorable rated by fibrillary areas; “ganglioid” cells with
prognosis with 80% 5-year and 70% 10-year neuronal-like features also seen; nuclei with finely
survival. Imaging appearance is cystic mass with granular chromatin and distinct nucleoli; perivas-
enhancing mural nodule (or solid mass) and cular pseudorosette-like arrangement of tumor
meningeal involvement. Gross appearance is cells is frequent; nuclear pleomorphism and multi-
firm, well-demarcated solid or cystic mass, often nucleated cells often present; mitotic figures can
with calcification. Histological appearance: sig- be present but have no impact on prognosis.
nificant cellular and nuclear pleomorphism, Immunohistochemistry: Strong GFAP in subset,
spindled cells with astrocytic features, stori- S100 reactive, individual cells or processes may
form/fascicular growth pattern, multinucleated be positive for neuronal markers. Differential:
cells, lipidized tumor cells (in 25%), eosinophilic central neurocytoma, subependymoma, ependy-
granular bodies and perivascular lymphocytes; moma, choroid plexus tumor.
usually very low mitotic rate (no microvascular
proliferation or necrosis)—increased mitotic Image with permission from Yachnis AT, Rivera-
Zengotita ML. Neuropathology, High-Yield Pathology
rate (>5 mitoses per high power fields) and Series, Saunders, Elsevier, 2014.
necrosis suggestive of anaplastic PXA. Immuno-
histochemistry: GFAP and S100 positive, neuro- 14. C—Tuberous sclerosis complex
nal markers (synaptophysin/neurofilament)
positive in 25%, Ki67 mitotic index <3%, PAS Cerebral cortical hamartomas (tubers): circum-
staining shows eosinophilic granular bodies, scribed areas of disorganized CNS tissue; dystro-
reticulin deposition surrounds small groups of phic neurons and “balloon cells”; highly
tumor cells. Differential: ganglioglioma, glio- associated with seizures. The other CNS lesions
blastoma variants. seen in TSC are subependymal giant cell astrocy-
toma (SEGA) and subependymal hamartomatous
Image with permission from Yachnis AT, Rivera- nodules (candle gutterings).
Zengotita ML. Neuropathology, High-Yield Pathology
Series, Saunders, Elsevier, 2014. Image with permission from Yachnis AT, Rivera-
Zengotita ML. Neuropathology, High-Yield Pathology
13. e—Subependymal giant cell astrocytoma Series, Saunders, Elsevier, 2014.

Subependymal giant cell astrocytomas (SEGA) 15. b—Ependymoma


are benign intraventricular neoplasms with astro-
cytic and neuronal features associated with tuber- Ependymomas are a slowly growing tumor arising
ous sclerosis complex—WHO grade I. Rare in the central region of the spinal cord in adults or
(<1% of all brain tumors); seen in 5-15% of intraventricular in children—WHO grade II. Rep-
TSC patients, usually in first two decades of life. resent 5-7% of all CNS tumors. Most common
Genetics: TSC is autosomal dominant (but 50% intra-axial spinal cord tumor in adults and third
cases arise de novo), 50-90% cases due to TSC2 most common posterior fossa tumor in children.
gene mutation (chromosome 16p13; codes for Infratentorial ependymomas have bimodal age dis-
tuberin) and 10% due to TSC1 mutation (chro- tribution with first peak in childhood (2-16 years)
mosome 9q34; codes for hamartin). Presentation and second peak at 30-40 years; supratentorial
may be due to hydrocephalus as SEGAs arise in ependymomas have no age predilection. Genetics:
lateral ventricle near foramen of Monro (or 30% incidence of aberration involving chromo-
sometimes third ventricle); seizures are due to some 22, associated with neurofibromatosis type
cortical hamartomas (tuber) not SEGA; other fea- 2 involving tumor suppressor NF2 gene located
tures of TSC include facial angiofibromas, ungula at 22q12 (distinct from the 22q mutation found in
fibroma, cardiac rhabdomyoma, renal angioli- many incidental ependymomas of the spinal cord).
poma, lymphangiomatosis, retinal hamartomas Presentation depends on location: infratentorial
and hypomelanotic macules. Management is sur- (hydrocephalus/visual disturbances, ataxia), supra-
gical excision (often curative) for primary tumor tentorial (seizures, focal deficits), spinal cord
or recurrence—malignant transformation is rare. (motor and sensory deficit). Prognosis is good with

Neurosurgery Books Full


www.ketabpezeshki.com 66485438-66485457
6 NEUROPATHOLOGY III: HISTOLOGY 107

overall 60-70% 5-year survival with surgery; best resembles a tubule lumen and contains no
prognosis in adults with spinal cord ependymoma; fiber-rich neuropil or central cytoplasmic projec-
recurrence is common in children hence goal is tions. These tubule-like structures, as well as
gross total resection. Imaging shows well- more elongated versions known as ependymal
demarcated masses with variable enhancement on canals, may represent an attempt by the tumor
MRI. Gross appearance is soft pink-tan well- cells to recapitulate the formation of ventricles
circumscribed tumor with occasional hemor- with ependymal linings. This rosette provides
rhage/necrosis; fourth ventricular tumors may strong evidence of ependymal differentiation at
extend out of foramen of Luscka laterally into sub- the light microscopic level. Unfortunately, true
arachnoid space/cistern. Histological appearances: ependymal rosettes and canals are found in only
uniform appearance of cells (salt and pepper chro- a minority of the most well-differentiated epen-
matin), distinctive perivascular pseudorosettes dymomas and most commonly in infratentorial
(radially arranged tapering cell processes extending examples.
to intratumoral blood vessels) are commoner than Image with permission from Yachnis AT, Rivera-
true ependymal rosettes (tumor attempting to form Zengotita ML. Neuropathology, High-Yield Pathology
little ependymal canal-like channels with central Series, Saunders, Elsevier, 2014.
lumen). Variants by location and histology include
papillary ependymomas, clear cell ependymomas,
tanycytic ependymomas, myxopapillary ependy- Rosette Type Associated Tumors
momas (WHO grade I) and anaplastic ependymo- Homer Wright Neuroblastoma,
mas (WHO grade III). Immunohistochemistry: rosette medulloblastoma, primitive
GFAP-positive perivascular cell processes, dot-like neuroectodermal tumor,
EMA reactivity within inner lining of ependymal pineoblastoma
rosettes, CD99 (nonspecific), Ki67 mitotic index Flexner- Retinoblastoma,
low <5%. Electron microscopy: luminal cilia and Wintersteiner pineoblastoma,
microvilli in ependymal rosettes, zipperlike junc- rosette medulloepithelioma
tional complexes. Differential diagnosis by loca- True ependymal Ependymoma
tion—fourth ventricle tumor in child rosette
(medulloblastoma, pilocytic astrocytoma, choroid Perivascular Ependymoma,
plexus tumor), adult intra-axial spinal cord tumor pseudorosette medulloblastoma, primitive
(diffuse astrocytoma), supratentorial (central neu- neuroectodermal tumor,
rocytoma, choroid plexus tumor, astroblastoma). central neurocytoma,
glioblastoma, monomorphous
A fourth type of rosette is the perivascular pilomyxoid astrocytomas
pseudorosette. In this pattern, a spoke-wheel
arrangement of cells with tapered cellular pro- Pineocytomatous Pineocytoma
rosette
cesses radiates around a wall of a centrally placed
vessel. Perivascular pseudorosettes are encoun- Neurocytic Central neurocytoma
tered in most ependymomas regardless of grade rosette
or variant. As such, they are significantly more Table with permission from Wippold II FJ, Perry A,
sensitive for the diagnosis of ependymomas than Neuropathology for the neuroradiologist: rosettes and
true ependymal rosettes. Unfortunately, perivas- pseudorosettes. AJNR March 2006;27:488-92.
cular pseudorosettes are also less specific in that
they are also encountered in medulloblastomas,
PNETs, central neurocytomas, and less often in 17. e—Retinoblastoma
glioblastomas, and a rare pediatric tumor, mono-
Flexner-Wintersteiner Rosette—retinoblas-
morphous pilomyxoid astrocytomas.
toma—tumor cells surrounding central lumen
Image with permission from Yachnis AT, Rivera-
containing cytoplasmic extensions. The tumor
Zengotita ML. Neuropathology, High-Yield Pathology cells that form the Flexner-Wintersteiner rosette
Series, Saunders, Elsevier, 2014. circumscribe a central lumen that contains small
cytoplasmic extensions of the encircling cells;
16. c—Ependymoma—section showing true however, unlike the center of the Homer Wright
ependymal rosettes (bottom left) and perivas- rosette, the central lumen does not contain the
cular pseudorosettes fiber-rich neuropil. Like the Homer Wright
rosette, the Flexner-Wintersteiner rosette signifies
In contrast to the Homer Wright and the a specific form of tumor differentiation. This con-
Flexner-Wintersteiner rosettes, the empty- tention is supported by electron microscopy where
appearing lumen of the true ependymal rosette the tumor cells forming the Flexner-Wintersteiner

Neurosurgery Books Full


www.ketabpezeshki.com 66485438-66485457
108 PART I BASIC SCIENCE

rosette have ultrastructural features of primitive generally thought to reflect differentiation of the
photoreceptor cells. In addition, special staining tumor, in this case neuronal. The cells of the pine-
properties of the rosette lumen resemble those ocytomatous and neurocytic rosettes are also
seen in rods and cones. Although this type of considered to be much more differentiated than
rosette is particularly characteristic of retinoblas- the cells forming Homer Wright rosettes in that
tomas, it may also be seen in pineoblastomas and the nuclei are slightly larger, more rounded,
medulloepitheliomas, where it is similarly thought much less mitotically active, and paler or less
to represent retinal differentiation. hyperchromatic.
Image with permission from Gault J, Vander JF, Ophthal- Image with permission from Perry A, Brat DJ. Practical
mology Secrets in Color, 3rd ed. Mosby, Elsevier, 2007. Surgical Neuropathology: A Diagnostic Approach,
Churchill Livingstone, Elsevier, 2010.
FURTHER READING
Wippold II FJ, Perry A, Neuropathology for the neuroradiolo- FURTHER READING
gist: rosettes and pseudorosettes. AJNR March 2006;27:488-92. Wippold II FJ, Perry A, Neuropathology for the neuroradiolo-
gist: rosettes and pseudorosettes. AJNR March 2006;27:488-92.
18. c—Homer Wright rosette
19. d—Papillary ependymoma
James Homer Wright (1869-1928), recognized a
group of adrenal and sympathetic nervous system Papillary ependymomas is a rare variant with uni-
tumors, which became known as neuroblastomas. form epithelial surfaces along the border with
The typical Homer Wright rosette with its central CSF with papillary (nipple-like) or pseudopapil-
lumen or hub filled with fiber-like processes can lary architecture. Differential: choroid plexus
also be found in medulloblastomas and histologi- tumor, metastasis.
cally similar tumors occurring outside of the cere-
Image with permission from Yachnis AT, Rivera-
bellum, designated PNETs. Although the cellular Zengotita ML. Neuropathology, High-Yield Pathology
mechanisms responsible for the formation of Series, Saunders, Elsevier, 2014.
rosettes within medulloblastomas and the signifi-
cance of these rosettes are not fully understood, 20. c—Clear cell ependymomas
most investigators believe that their presence indi-
cates neuronal differentiation. The delicate fibril- This is a mimic of oligodendroglioma but has
lary material found within the central lumen of the well-demarcated border with adjacent brain; usu-
Homer Wright rosette is composed of neuropil, ally located in cerebral hemisphere of young
which contains primitive neuronal processes or adults; may behave more aggressively. Electron
neurites. Although the identification of Homer microscopy shows deep nuclear invaginations in
Wright rosettes in a posterior fossa tumor is nearly the clear cell variant.
pathognomonic of the diagnosis of medulloblas-
toma, the rosettes are encountered in only a third Image with permission from Yachnis AT, Rivera-
of these tumors. Moreover, Homer Wright Zengotita ML. Neuropathology, High-Yield Pathology
Series, Saunders, Elsevier, 2014.
rosettes may be found in other tumors such as
supratentorial PNETs and pineoblastomas.
21. e—Tanycytic ependymomas
Pineocytomas and central neurocytomas repre-
These usually arise in the spinal cord as a discrete
sent well-differentiated neuronal neoplasms with mass that is well demarcated from the adjacent
small rounded nuclei, analogous to those nor-
neural tissue; forms fascicles of highly fibrillated
mally encountered in the internal granular layer
bipolar spindle cells; mimic of diffuse astrocy-
of the cerebellum or the dentate fascia of the hip-
toma; ependymal rosettes usually absent with
pocampus. Although they likely originate from
only vague pseudorosettes.
slightly different precursors, the histologic fea-
tures of these two tumors are virtually identical, Image with permission from Yachnis AT, Rivera-
including their tendency to form neuropil-rich Zengotita ML. Neuropathology, High-Yield Pathology
rosettes, referred to as pineocytomatous rosettes Series, Saunders, Elsevier, 2014.
in pineocytomas and neurocytic rosettes in cen-
tral neurocytoma. Both are quite similar to the 22. a—Anaplastic ependymoma
Homer Wright rosette, but they are generally
larger and more irregular in contour. As in the Anaplastic ependymoma is an malignant ependy-
other types of rosettes discussed, the presence of mal tumor seen most commonly in posterior fossa
pineocytomatous or neurocytic rosettes is in children—WHO grade III. Treatment

Neurosurgery Books Full


www.ketabpezeshki.com 66485438-66485457
6 NEUROPATHOLOGY III: HISTOLOGY 109

included surgical resection (extent is key predic- within background of eosinophilic fibrillary pro-
tor of outcome), chemotherapy and radiotherapy. cesses, microcystic change (if in lateral ventricle),
Worse prognosis if <3 years old, incomplete vague perivascular pseudorosette-like pattern.
tumor resection and CSF dissemination at pre- Immunohistochemistry shows GFAP-positive
sentation. Ependymomatous features (e.g., peri- cell processes, negative for synaptophysin, low
vascular pseudorosettes) present, but also mitotic index. Differential: ependymoma, central
malignant features such as increases cellularity, neurocytoma, astrocytoma.
mitoses, microvascular proliferation and pseudo-
palisading necrosis. Differential: ependymoma, Image with permission from Yachnis AT, Rivera-
Zengotita ML. Neuropathology, High-Yield Pathology
poorly differentiated embryonal tumors, astrocy- Series, Saunders, Elsevier, 2014.
toma, anaplastic oligodendroglioma.
Image with permission from Yachnis AT, Rivera- 25. c—Choroid plexus papilloma
Zengotita ML. Neuropathology, High-Yield Pathology
Series, Saunders, Elsevier, 2014. Choroid plexus papilloma (CPP) is an intraventric-
ular papillary neoplasm—WHO grade I. Repre-
23. c—Myxopapillary ependymoma sents 0.3-0.6% of brain tumors and 2-4% of
pediatric brain tumors (10-20% of brain tumors
Myxopapillary ependymoma is a slow growing in <1 year olds). Most common in children
ependymal tumor commonly arising in the conus <15 years; fourth ventricular CPP shows no age
medullaris, cauda equina and filum terminale— predilection while lateral ventricular CPPs arise
WHO grade I. Represents 10% of all ependymo- in younger patients (<20 years). Location is lateral
mas, and 50% of all spinal cord ependymomas; ventricle (50%) > fourth (40%) > third ventricle
twice as common in men. Presents as chronic (5%). Genetics: Association with SV-40 T antigen;
back pain, sciatica, sensorimotor deficit and May arise in Aicardi syndrome (sporadic mutation
sphincter disturbance. Management is surgical linked to X chromosome; partial or total agenesis
resection (high cure rate), greater than 10-year of corpus callosum, chorioretinal lacunae, infantile
survival; recurrence may be seen in tumors with spasms); no mutations in p53 gene unlike choroid
nerve root invasion. Imaging shows enhancing, plexus carcinoma. Presentation is with increasing
well-demarcated ovoid masses attached to filum head circumference or hydrocephalus. Manage-
terminale. Gross appearance is soft, lobulated, ment is surgical resection and prognosis is good.
white-tan tumor easily separable from surround- Imaging appearances of well-circumscribed intra-
ing structures. Histological appearance cuboidal/ ventricular mass, enhancing with gadolinium
spindled tumor cells arranged radially around and T2 hyperintense. Gross appearance of well-
papillary vascular cores, myxoid matrix with defined cauliflower-like mass that may adhere to
microcystic structures. Immunohistochemistry: ventricular wall. Histological appearance: resem-
GFAP positive, S100 and vimentin reactivity, bles normal choroid plexus but has increased
absent cytokeratin activity. Differential: filum cellularity, nuclear crowding, solid areas and strat-
terminale region tumors (schwannoma, meningi- ification; fibrovascular tissue core surrounded by
oma, paraganglioma), sacral tumors (chordoma, single layer of columnar-cuboidal epithelium with
chondrosarcoma), metastatic adenocarcinoma. basally oriented nuclei; atypical CPP (WHO grade
II) has increased mitoses, pleomorphism and foci
Image with permission from Yachnis AT, Rivera- of necrosis. Immunohistochemistry: Immunoreac-
Zengotita ML. Neuropathology, High-Yield Pathology
Series, Saunders, Elsevier, 2014. tive to transthyretin, cytokeratins (CK), and synap-
tophysin; 75% are CK7-positive and CK20-
24. e—Subependymoma negative; Ki-67 (MIB1) labeling range from 0%
to 6%; S-100 staining in 55% to 90% of reported
Subependymoma is a benign glial neoplasm usu- cases; Focal glial fibrillary acidic protein (GFAP)
ally attached to wall of lateral ventricles near the reactivity. Differential: normal/hypertrophy of
foramen of Monro or floor of fourth ventricle— choroid plexus, choroid plexus carcinoma; ependy-
WHO grade I. Mean age of presentation approx- moma; metastatic carcinoma.
imately 60, mostly incidental on imaging or Image with permission from Yachnis AT, Rivera-
autopsy, though can present with hydrocephalus. Zengotita ML. Neuropathology, High-Yield Pathology
Treatment is total or subtotal resection, with low Series, Saunders, Elsevier, 2014.
recurrence. Imaging shows small, nodular, dis-
crete mass (<2 cm), occasional foci of calcifica- 26. a—Choroid plexus papilloma
tion/hemorrhage/cystic change. Grossly distinct
from adjacent brain tissue. Histology shows nod- Choroid plexus carcinoma is a malignant cho-
ular growth pattern, clusters of small bland nuclei roid plexus neoplasm (WHO grade III; 80%

Neurosurgery Books Full


www.ketabpezeshki.com 66485438-66485457
110 PART I BASIC SCIENCE

arise in children). Genetics: Reported associa- • Rubinstein-Taybi syndrome (with menin-


tion with p53 germline mutation or Li- giomas and oligodendrogliomas): mutations
Fraumeni syndrome; nearly all demonstrate in CREB-binding protein gene on chromo-
p53 immunoreactivity; no INI-1 mutations some 16p13 with cognitive impairment,
despite histologic resemblance to atypical tera- growth retardation, microcephaly, facies
toid/rhabdoid tumors. Present with hydroceph- Presentation is with raised ICP or cerebellar
alus and focal neurology. Management is signs—75% arise from vermis and fill the
surgical resection, chemotherapy and irradia- fourth ventricle; 30% show CSF spread with
tion. Poor prognosis due to CSF dissemination. spinal drop metastases at presentation. Manage-
Imaging: large ventricular poorly defined lesion, ment is maximal surgical resection followed by
edema of adjacent brain. Gross appearance: chemotherapy and craniospinal irradiation—
gray-tan tumor with areas of hemorrhage and overall 5-year survival 65%. Good prognostic
necrosis, brain invasion may be evident. Histo- factors: 3-22 years old at presentation, gross
logical criteria (at least four of the following): total excision, no CSF spread, WNT signaling
frequent mitoses (>5 of 10 high power fields), activation (nuclear b-catenin immunoreactiv-
increased cellularity, nuclear pleomorphism, ity), TrkC neurotrophin receptor overexpres-
blurring of papillary pattern, necrosis. Immuno- sion, balanced 17q, monosomy chromosome
histochemistry: Cytokeratin reactivity, less reac- 6, nodular/desmoplastic and extensively nodular
tive than CPP to S100 and transthyretin, GFAP variants. Poor prognostic factors: <3 years old,
20%, EMA negative. Differential: AT/RT, met- subtotal resection, CSF dissemination at pre-
astatic carcinoma. sentation, C-MYK/N-MYC amplification, iso-
chromosome 17q (unbalanced), 17p loss, large
Image with permission from Yachnis AT, Rivera- cell/anaplastic variant (aggressive and extra-
Zengotita ML, Neuropathology, High-Yield Pathology
Series, Saunders, Elsevier, 2014. neural metastasis has been reported). Imaging:
solid heterogenous enhancing mass adjacent
27. d—Medulloblastoma or extending into fourth ventricle; nodular/
desmoplastic variant location in cerebellar
Medulloblastomas are malignant embryonal hemisphere; extensively nodular variant has
tumors (PNET) arising in the posterior fossa of enhancing nodules which cluster (look grape-
children—WHO grade IV. Commonest like). Gross appearances: solid masses of friable
PNET—0.5/100,000; peak incidence 5-10 years gray-white tissue involving cerebellar folia and
(adults cluster in 30s); accounts for 20% of brain leptomeninges. Histological appearance: small
tumors in children, and is the commonest malig- round blue cell tumor composed of sheets of
nant brain tumor in children. Genetics: undifferentiated cells with indistinct cytoplasm,
• Classic medulloblastoma: most common hyperchromatic angulated nuclei (molding);
genetic alteration: 17p loss with isochromo- frequent mitoses; Homer Wright rosettes in
some 17q formation about 40% of cases; apoptosis or necrosis may
• Large cell/anaplastic medulloblastoma be present but more prominent in large cell/a-
associated with C-MYC oncogene naplastic variants; overall architecture may be
amplifications swirling, fascicular, diffuse; scanty stroma con-
• Desmoplastic medulloblastoma: PTCH taining small blood vessels (occasional micro-
gene mutations (sonic hedgehog pathway) vascular proliferation). Immunohistochemistry:
• Gorlin syndrome (nevoid basal cell carci- Most are synaptophysin immunoreactive;
noma): autosomal dominant germline muta- nodular foci of neuronal differentiation (synap-
tions of patched gene at 9q22.3 with tophysin, neurofilament); GFAP-positive ele-
odontogenic keratocysts, pitting of palms ments appear to be focally entrapped; positive
and soles, skeletal anomalies, lamellar calcium for INI-1 (hSNF5/SMARCB1) using BAF47
deposition in falx cerebri and diaphragma antibody to exclude AT/RT; S-100 may be
sellae, calcifying ovarian fibromas, and multi- seen in melanocytic variant; Desmin and myo-
focal, early-onset basal cell carcinomas globin positivity in myogenic cells; Ki-67 index
• Li-Fraumeni syndrome/p53 mutation is variable, but generally >20%. Differential:
syndromes AT/RT, ependymoma, lymphoma, small cell
• Turcot syndrome (Type 2): medulloblas- carcinoma, metastasis, high-grade glioma.
toma arises in setting of autosomal domi-
nant adenomatous polyposis of colon Image with permission from Yachnis AT, Rivera-
resulting from APC mutations on chromo- Zengotita ML. Neuropathology, High-Yield Pathology
some 5q21 APC Series, Saunders, Elsevier, 2014.

Neurosurgery Books Full


www.ketabpezeshki.com 66485438-66485457
6 NEUROPATHOLOGY III: HISTOLOGY 111

28. c—Large cell/anaplastic medulloblastoma hyperchromatic nuclei with granular chromatin,


frequent mitoses, necrosis. Immunohistochemis-
Large cell/anaplastic medulloblastomas are typi- try: variably positive for neuronal antigens (NF,
cally grouped together and probably represent a synaptophysin, NSE, beta-III tubulin), high
spectrum; they share frequent mitoses, apoptotic Ki67 labeling index, positive for INI-1. Differen-
cells, pleomorphism, cell “wrapping” (seen in sec- tial: AT/RT, germ cell tumors, GBM.
tion). Large cell medulloblastoma: cells are large
with rounded vesicular nuclei, prominent nucle- Image with permission from Yachnis AT, Rivera-
Zengotita ML. Neuropathology, High-Yield Pathology
oli, and conspicuous eosinophilic cytoplasm;
Series, Saunders, Elsevier, 2014.
discohesive with numerous mitotic and apop-
totic cells. Anaplastic medulloblastoma: marked
30. a—Atypical teratoid/rhabdoid tumor
variation in nuclear size and shape with some
bizarre forms: multinucleated, giant cells, cell
Atypical teratoid/rhabdoid tumors (AT/RT) are
wrapping/molding.
high-grade embryonal tumors expressing epithe-
Nodular/desmoplastic medulloblastoma: reti- lial, neuronal and mesenchymal lineage anti-
culin rich, proliferatively active cellular tumor
gens—WHO grade IV. Represent 1-2% of all
with nodular reticulin-free “pale islands” Neuro-
pediatric brain tumors (10% of infant brain
nal differentiation in nodules (synaptophysin,
tumors); >90% cases in under 5 year olds (mean
neurofilament immunoreactive) Reduced mitotic
age 20 months); 1.5-2:1 male preponderance;
activity in nodules.
supratentorial location (cerebrum, suprasellar)
Medulloblastoma with extensive nodularity
slightly more common than infratentorial (cerebel-
(previously called “cerebellar neuroblastoma”):
lar, CPA with brainstem involvement). Genetics:
more advanced neurocytic maturation with more
monosomy/deletions or mutations of INI1
lobular (nodular), pale areas with neuropil-like
(hSNF5/SMARCB1) gene on chromosome
stroma. 22q11.2 (INI1 protein functions in chromatin
• Predilection for children <3 years of age
remodeling); rhabdoid predisposition syndrome:
with a more favorable prognosis than other
germline loss or inactivation of INI1 gene. Presen-
subtypes tation usually nonspecific irritable, vomiting,
• Greatly decreased proliferation index, lin-
lethargic, headache. Management influenced by
ear pattern of tumor cell nuclei
CSF dissemination (present in 1/3 at presentation)
• Rare maturation to benign ganglioneurocy-
and age (risks of radiotherapy if <3 years); surgery
tic or gangliogliomatous elements
can confirm diagnosis but is not curative; chemo-
• “Neuroblastic” foci: may be found in any
therapy is mainstay. Poor survival 50% at 6 months;
given tumor case
survival >2 years is exceptional. Imaging: T1
hypointense, variable enhancement, diffusion
Image with permission from Yachnis AT, Rivera-
Zengotita ML, Neuropathology, High-Yield Pathology restricting, large, partly cystic/hemorrhagic. Gross:
Series, Saunders, Elsevier, 2014. large, soft demarcated mass. Histology: Heteroge-
neous lesion; rhabdoid cells (large, distinct cell bor-
29. d—Supratentorial PNET ders with eccentric nuclei, macronucleoli, and
prominent pink cytoplasmic inclusions); occasional
Supratentorial/CNS-PNET area heterogenous vacuolization; some areas have overlapping histol-
group of embryonal tumors composed of ogy with other embryonal tumors (such as medullo-
primitive-appearing neuroepithelial cells expres- blastoma or CNS-PNET); spindled mesenchymal
sing neuronal or glial antigens and most com- or even myxoid-like components may be promi-
monly arise in cerebral hemispheres. Represent nent; overall architecture can be nested, spindled,
1-3% of pediatric CNS neuroepithelial tumors; sheet-like, or even glandular. Immunohistochemis-
age range 4 weeks to 20 years; in older patients try: INI1 protein negative (BAF47 antibody),
may be mistaken as PNET-like areas within immunoreactive for vimentin and EMA, variably
GBM. Present with raised ICP, localizing signs positive for SMA, neurofilament, GFAP, synapto-
or seizures. Management is chemotherapy (stem physin, and cytokeratins (AE1/3; CAM 5.20; Ki-
cell rescue) and radiotherapy as usually large 67 [MIB-1] labeling index is typically high
and non-resectable. Overall 5-year survival [>50%]). Differential: medulloblastoma, supraten-
20-30%. Imaging: large enhancing lesions with torial PNET, choroid plexus tumors, germ cell
mass effect, restricted diffusion, calcification. tumors, high-grade glioma.
Gross—large soft, tan-gray tumors with areas
of necrosis or calcification. Histological appear- Image with permission from Yachnis AT, Rivera-
Zengotita ML, Neuropathology, High-Yield Pathology
ance: highly cellular with scant cytoplasm, Series, Saunders, Elsevier, 2014.

Neurosurgery Books Full


www.ketabpezeshki.com 66485438-66485457
112 PART I BASIC SCIENCE

31. b—Ganglioglioma cell tumors cortical dysplasia may be found adjacent to the
neoplasm; eosinophilic granular bodies, bipolar
These are well-differentiated CNS tumors with astrocytes; no mitotic figures, necrosis, or endo-
mature neurons (ganglion cells) as a defining thelial hyperplasia. Immunohistochemistry:
feature: floating neurons are positive for synaptophysin/
1. Ganglioglioma—ganglion cell tumor with a NF/neuron specific enolase; GFAP in astrocytes;
low grade glial element (WHO grade I) S100 in oligodendrocyte-like cells; mucin is
2. Gangliocytoma—ganglion cell tumor with- Alcian blue positive; Ki67 index <1-2%. Differ-
out a glial element (WHO grade I) ential: oligodendroglioma, astrocytoma with
3. Anaplastic ganglioglioma—ganglion cells microcystic change, ganglioglioma/cytoma, pap-
and anaplastic glial elements (WHO illary glioneuronal tumor, rosette-forming glio-
grade III) neuronal tumor of the fourth ventricle.
Represent 4% of primary brain tumors (incidence
0.2/100,000 per year); commoner in children and Image with permission from Yachnis AT, Rivera-
Zengotita ML, Neuropathology, High-Yield Pathology
temporal lobe. Presentation with seizures or loca- Series, Saunders, Elsevier, 2014.
tion specific symptoms. Management is surgery
(>90% 10-year survival), while combination with 33. a—Central neurocytoma
radiotherapy for inoperable or anaplastic (WHO
III) cases. Imaging: variably enhancing solid or This is an intraventricular tumor composed of
cystic tumor, possibly calcified. Gross appearance: uniform round cells with neuronal differentia-
demarcated tumor mass with or without cystic tion—WHO grade II. Rare 0.25-0.5% of
change or calcification. Histology: disorganized intracranial tumors; peak age 20-40 (rare
collection of mature ganglion cells with variable <10 years or >70 years). Presentation is with
glial background; clusters of mature neurons, with raised ICP/hydrocephalus; tumor typically
large nuclei, prominent nucleoli in disordered/ attached to septum pellucidum near foramen
haphazard arrangements; ganglion cells may of Monro. Good prognosis with complete sur-
exhibit binucleation or have bizarre-appearing gical excision—incomplete excision associated
nuclei; Rosenthal fibers, eosinophilic granular with recurrence/progression; radiotherapy
bodies; perivascular lymphocytes, microcalcifica- used in some cases. Imaging: enhancing intra-
tions also can be present; exclude anaplastic fea- ventricular mass attached to septum pelluci-
tures. Immunohistochemistry: synaptophysin, dum (calcified/cystic). Gross appearance: well
chromogranin, neurofilament reactive neoplastic demarcated, gray friable tumor—calcified/
ganglion cells but NeuN negative; GFAP-positive cystic/hemorrhagic. Histology: monomorphic
in astrocytic elements; Ki-67/MIB1 usually low, bland appearing cells with oval nuclei and
<2%. Differential: pilocytic astrocytoma, diffuse salt and pepper chromatin, background
astrocytoma, DNET, developmental lesion, cor- fibrillary neuropil is eosinophilic, mimic of
tical dysplasia, tuberous sclerosis. oligodendroglioma, rare Homer Wright
rosettes and ganglion cells, perivascular
Image with permission from Yachnis AT, Rivera-
Zengotita ML. Neuropathology, High-Yield Pathology pseudorosettes, perinuclear haloes, rarely ana-
Series, Saunders, Elsevier, 2014. plastic features. Immunohistochemistry:
Synaptophysin/NeuN positive, GFAP
32. b—Dysembryoplastic neuroepithelial tumor for astrocytes, Ki67 low <2%, unlike oligoden-
(DNET) drogliomas do not show loss of 1p/19q. Differ-
ential: oligodendroglioma, ependymoma,
This is a benign neuronal-glial neoplasm arising pineocytoma, DNET.
from gray matter—WHO grade I. Accounts for
Image with permission from Yachnis AT, Rivera-
<1% primary brain tumors; affects children/ Zengotita ML. Neuropathology, High-Yield Pathology
young adults (mean age 9 years); first seizure usu- Series, Saunders, Elsevier, 2014.
ally <20 years of age; supratentorial and cortical
origin (temporal > frontal). Present with seizures. 34. c—Hemangiopericytomas
Excellent prognosis with surgery—recurrence is
rare. Imaging: T1 hypointense, T2 hyperintense These are rare dural-based mesenchymal
non-enhancing lesion located within expanded tumors—WHO grade II or III. Account for
cortical ribbon. Gross appearance: multinodular 0.4% of primary CNS tumors; common between
or cystic; gelatinous/mucoid mass with discrete second and sixth decades (mean age 45). Presenta-
margins. Histological: specific glioneuronal ele- tion is due to local compression/invasion, com-
ment consisting of oligodendrocyte-like cells monly headache and seizures or myelopathy
arranged along bundled axons separated by a (10% arise as spinal lesions). Primary treatment
myxoid matrix that contains floating neurons; is surgical resection but 90% recurrence rate and

Neurosurgery Books Full


www.ketabpezeshki.com 66485438-66485457
6 NEUROPATHOLOGY III: HISTOLOGY 113

metastasis in up to 20%; radiotherapy/radiosur- chromatin with central clearing, nuclear


gery in some cases. Imaging: dural-based pseudoinclusions. Immunohistochemistry: Posi-
contrast-enhancing masses. Gross appearance: tive for vimentin and EMA, S100 variable, 60%
solid, gray to red/brown tumors with tendency progesterone receptor positive (mostly females).
to bleed excessively during surgery. Histological Differential: Metastatic carcinoma, schwannoma,
appearance: monomorphous cellular spindle cell solitary fibrous tumor, sarcoma.
tumor with staghorn-like blood vessels, focally
storiform architecture, no intranuclear inclusions, Image with permission from Yachnis AT, Rivera-
Zengotita ML, Neuropathology, High-Yield Pathology
no whorls/psammoma bodies, necrosis uncom- Series, Saunders, Elsevier, 2014.
mon; WHO III have >5 mitoses per 10 HPF with
moderate to high cellular atypia and cellularity. 36. c—Fibrous (fibroblastic) meningioma: spin-
Immunohistochemistry: tumor cells negative for dled cells forming interlacing, parallel fasci-
CD34 but blood vessels positive for CD34, Ki67 cles with surrounding collagenous stroma;
index mean 5-10% (up to 40%), reticulin rich, psammoma bodies and whorls uncommon.
vimentin positive but negative for S100 and Differential: Schwannoma (versus fibroblas-
EMA. Differential: meningioma, solitary fibrous tic meningioma).
tumor (positive for CD34, Bcl-2, CD99) menin-
geal sarcoma, metastasis, meningeal lymphoid Image with permission from Yachnis AT, Rivera-
tumors. Zengotita ML, Neuropathology, High-Yield Pathology
Series, Saunders, Elsevier, 2014.
Image with permission from Yachnis AT, Rivera-
Zengotita ML, Neuropathology, High-Yield Pathology 37. e—Psammomatous meningiomas are typi-
Series, Saunders, Elsevier, 2014. cally found in thoracic spinal region of
middle-aged women. Psammomatous menin-
35. c—Meningiomas gioma: variant having abundant psammoma
bodies; most common in the thoracic spinal
These are tumors arising from meningothelial cord of middle-aged women. Differential:
(arachnoidal) cells attached to the inner surface Reactive process with calcification (psammo-
of the dura mater—most are WHO grade I. Rep- matous meningioma).
resent 25-30% of primary intracranial tumors;
1.4% at autopsy; most common in fifth to seventh Image with permission from Yachnis AT, Rivera-
decades, commoner in females 3:1 (10:1 for spinal Zengotita ML. Neuropathology, High-Yield Pathology
meningiomas); multiple in 10%. Genetics: many Series, Saunders, Elsevier, 2014.
arise secondary to radiation exposure, deletion of
chromosome 22, multiple meningiomas seen in 38. e—Secretory meningioma: “classic” appear-
NF2 (22q11-13.1 mutation; Merlin peptide pro- ing tumor cells with numerous interspersed
duction), DAL-1 loss (18p11.3). Presentation: intracellular lumina containing eosinophilic
headache and seizures commonest, but local com- secretions (pseudopsammoma bodies); may
pression related also. Location: intracranial, have abundant mast cells; may have peritu-
intraspinal or orbital; intracranial locations are moral edema. Pseudopsammoma bodies of
convexity (half of these parasaggital), olfactory secretory variant are carcinembryonic anti-
groove, sellar/parasellar (cavernous sinus), gen (CEA) immunoreactive (and periodic
petrous ridge, tentorium, posterior fossa; rarely acid-Schiff [PAS] positive). Differential:
in intraventricular/extradural location. Manage- Metastatic adenocarcinoma (versus secretory
ment is gross total resection; recurrence if not meningioma).
completely excised; radiotherapy/radiosurgery Image with permission from Yachnis AT, Rivera-
for recurrence or small surgically inaccessible Zengotita ML. Neuropathology, High-Yield Pathology
lesions (cavernous sinus). Imaging: circumscribed, Series, Saunders, Elsevier, 2014.
isodense/isointense contrast-enhancing lesion
with dural tail sign; peritumoral edema due to vas- 39. a—Angiomatous meningioma: predomi-
cular compromise; calcification or bone forma- nance of small hyalinized blood vessels; back-
tion/hyperostosis on CT. Gross appearance: ground tumor cells may demonstrate marked
well-demarcated, firm, rubbery, yellow-tan, degenerative nuclear atypia. Differential:
round/lobulated mass compressing adjacent Vascular malformation, hemangioblastoma
brain/spinal cord, invasion of skull/hyperostosis. (versus angiomatous meningioma).
Histology (commonest meningothelial variant):
Image with permission from Yachnis AT, Rivera-
lobules of uniform cells in a syncytium, whorls
Zengotita ML. Neuropathology, High-Yield Pathology
and psammoma bodies, bland oval nuclei, fine Series, Saunders, Elsevier, 2014.

Neurosurgery Books Full


www.ketabpezeshki.com 66485438-66485457
114 PART I BASIC SCIENCE

40. a—Atypical meningiomas at all ages; slight female predominance. Clear cell
meningioma: tendency to arise in the cerebello-
These are those dural-based meningothelial pontine angle or cauda equina; also found along
tumors which are WHO grade II (includes atypi- cerebral convexities; may have headache and cra-
cal, chordoid and clear cell variants). Account for nial nerve palsies. Histology: Clear cell meningi-
5-20% of meningiomas; clear cell and chordoid oma: sheets of polygonal tumor cells with
usually affect younger individuals (third decade). abundant clear cytoplasm, bland round to oval
Genetics (in addition to classic mutations) losses nuclei, separated by bands of collagen and inter-
on 1p, 6q, 10q, 14q, 18q; gains on 1p, 9q, 12q, spersed vessels with perivascular hyalinization;
15q, 20q. Surgical resection is primary treatment; rare mitoses; nuclear pleomorphism and necrosis
30-40% recurrence; radiotherapy/radiosurgery in uncommon. Differential: Metastatic clear cell
some cases; 80% 10-year survival; 25% may pro- carcinoma, microcystic meningioma (versus clear
gress to malignant (anaplastic) meningioma cell meningioma).
WHO grade III. Gross appearance: larger than
benign variants, necrosis and adherence to adja- Image with permission from Yachnis AT, Rivera-
cent brain tissue. Histological appearance: high Zengotita ML. Neuropathology, High-Yield Pathology
Series, Saunders, Elsevier, 2014.
mitotic index (>4 per 10 HPF) or 3 of
hypercellularity/sheet-like growth pattern/small 43. a—Anaplastic meningiomas
cell formation/macronuclei/necrosis/brain inva-
sion/chordoid features/clear cell features. Immu- Anaplastic meningiomas (WHO grade III; malig-
nohistochemistry: Vimentin positive but less nant, papillary or rhabdoid) have markedly
consistent EMA. Differential: meningioma increased mitotic activity or frank anaplasia.
(WHO I or III), schwannoma, solitary fibrous Account for 1-3% of meningiomas; 50% papil-
tumor, chordoma, metastatic clear cell carcinoma. lary meningiomas occur in children. Genetics:
gains on 17q, losses on 9p (CDKN2A, p14ARF,
Image with permission from Yachnis AT, Rivera-
Zengotita ML. Neuropathology, High-Yield Pathology CDKN2B), losses on 1p, 6q, 14q, 18q. Can occur
Series, Saunders, Elsevier, 2014. de novo, in tumor recurrence or post-radiation;
commonly located at falx or convexity. Treat-
41. b—Chordoid meningioma ment is with surgical resection followed by
radiotherapy; high recurrence rate (50% in pap-
This affects younger individuals (mean age 35); illary meningioma and 90% in rhabdoid meningi-
slight female predominance; rare association with oma); extracranial metastasis (20% in papillary
Castleman disease, iron-refractory anemia, bone meningioma, 10% in rhabdoid). Similar imaging
marrow plasmacytosis with dysgammaglobuline- and gross appearance to meningioma, but inva-
mia. Chordoid meningioma: usually in supraten- sion of adjacent brain/skull and hemorrhage/
torial location; rarely found intraventricular, near necrosis. Histopathology: high mitotic rate
the foramen jugulare, and intraorbital; most com- (>20 per 10 HPF) or frank anaplasia with areas
mon presenting symptoms are headache, mental resembling carcinoma/sarcoma/melanoma.
or visual disturbances, and seizures. Chordoid
meningioma: chordoma-like tumor with cords, Image with permission from Yachnis AT, Rivera-
Zengotita ML. Neuropathology, High-Yield Pathology
nests, or trabeculae of epithelioid and spindled Series, Saunders, Elsevier, 2014.
tumor cells with eosinophilic, often vacuolated
cytoplasm; myxoid stroma; foci of typical menin- 44. e—Rhabdoid meningioma: >50% of tumor
gotheliomatous features; foci of chronic inflamma- composed of loosely cohesive sheets of large
tion may be prominent. Chordoid meningiomas cells with eosinophilic inclusion-like cyto-
are typically PAS positive. Differential: Chordoma, plasm sometimes appearing as globular or
other meningioma (versus chordoid meningioma). whorled filamentous inclusions; vesicular
Image with permission from Yachnis AT, Rivera- nuclei with prominent nucleoli; other histo-
Zengotita ML. Neuropathology, High-Yield Pathology logic features of malignancy—including
Series, Saunders, Elsevier, 2014. increased mitoses, high proliferative indices,
and necrosis—should be present; foci of con-
42. b—Clear cell meningioma ventional meningioma morphology may be
present; concurrent papillary features may
Rare variant also affecting children and young be present.
adults (mean age 29 years old) although reported

Neurosurgery Books Full


www.ketabpezeshki.com 66485438-66485457
6 NEUROPATHOLOGY III: HISTOLOGY 115

• Focal rhabdoid-like histology in otherwise Differential: malignant peripheral nerve sheath


typical meningioma not sufficient for diag- tumor, schwannoma, perineurinoma.
nosis of rhabdoid variant
• Metastatic carcinoma/melanoma, rhabdo- Image with permission from Yachnis AT, Rivera-
Zengotita ML. Neuropathology, High-Yield Pathology
myosarcoma, gemistocytic astrocytoma Series, Saunders, Elsevier, 2014.
(versus rhabdoid meningioma)
47. c—Neurofibroma
Image with permission from Yachnis AT, Rivera-
Zengotita ML, Neuropathology, High-Yield Pathology Image with permission from Yachnis AT, Rivera-
Series, Saunders, Elsevier, 2014. Zengotita ML. Neuropathology, High-Yield Pathology
Series, Saunders, Elsevier, 2014.
45. e—Papillary meningioma: discohesive menin-
gothelial tumor cells around fibrovascular pap- 48. a—Schwannomas
illary or pseudopapillary cores; perivascular
pseudorosettes may be apparent; increased These are encapsulated nerve sheath tumor com-
mitoses, necrosis, and pleomorphism; brain posed of differentiated Schwann cells—WHO
invasion common. Ependymoma, metastatic grade I. Represent 8-10% of intracranial tumors
carcinoma, astroblastoma, choroid plexus and 30% of spinal tumors; peak incidence in
tumor (versus papillary meningioma). fourth to sixth decade; most are sporadic and sol-
itary; 4% are associated with NF2
Image with permission from Yachnis AT, Rivera- or schwannomatosis; no gender predilection.
Zengotita ML. Neuropathology, High-Yield Pathology
Series, Saunders, Elsevier, 2014. Genetics: bilateral vestibular schwannomas
involving CN VIII are pathognomonic for NF2
46. b—Neurofibromas (22q frameshift mutation of gene coding for
tumor suppressor merlin found in 60% of
These are peripheral nerve sheath tumors com- schwannomas); Schwannomatosis syndrome
posed of differentiated Schwann cells, characterized by multiple painful schwannomas
perneurial-like cells (epithelioid myofibroblasts in a segmental distribution in absence of
in perineurium), fibroblasts and nerve fibers— other NF2 features; psammomatous/melanotic
WHO grade I. Relatively common sporadic schwannomas associated with Carney complex
tumor that affects all ages and both genders (mutation of PRKAR1A gene on chromosome
equally; multiple and plexiform neurofibromas 17q; autosomal dominant disorder characterized
associated with NF1 (mutation on 17q). Presen- by lentiginous facial pigmentation, cardiac myx-
tation: localized painless cutaneous nodule/mass oma, and endocrine disorders). Presentation:
(commonest; sporadic); diffuse cutaneous nod- Account for 85% of cerebellopontine angle
ules (extraneural types); plexiform neurofibromas tumors (most arise from vestibular branch of
in NF1 (multinodular tangles “bag of worms,” VIII [acoustic neuroma], occasionally V, VII);
enlargement/deformity of plexus or nerve trunk, intraspinal schwannomas arise from sensory
multiple neurofibromas, >5 café-au-lait spots, roots. Management is surgical resection for
axillary/inguinal freckling, optic glioma, Lisch large tumors, radiosurgery possible for small
nodules, sphenoid dysplasia). Treatment is (<2.5 cm); malignant transformation is rare.
surgical resection—sporadic neurofibromas are Imaging: intracranial circumscribed/cystic
benign, 3-5% of NF1 plexiform neurofibromas enhancing mass with extension into the internal
undergo malignant transformation. Gross auditory canal; spinal schwannomas are generally
appearance: firm to soft tan pink with variable intradural extramedullary (but occasionally extend
myxoid stroma; intraneural tumors are well- into extradural space creating dumbbell shape).
circumscribed fusiform shape vs diffusely infil- Gross appearance: well-circumscribed, occasion-
trating extraneural tumor. Histology: delicate ally cystic mass, heterogenous cut surface will
spindle cells in matrix of collagen fibers, mucus, yellow or hemorrhagic foci. Histology: encapsu-
axons; Schwann cells have wavy nuclei; expansion lated, differentiated Schwann cells form two
of multiple nerve fascicles in plexiform neurofi- architectural patterns Antoni A (closely apposed
broma; vascular unlike schwannomas; nuclear spindled tumor cells with palisading, elongated
atypia and increased cellularity but mitoses rare; nuclei; Verocay bodies are alternating parallel
uncommon—pseudo-Meissnerian bodies, mela- rows of nuclear palisades with areas devoid
nin pigmentation, dense aggregations of of nuclei occurring within Antoni A) and
hyperchromatic nuclei. Immunohistochemistry: Antoni B (less cellular areas of loosely arranged
S100 and vimentin positive, EMA for perineu- tumor cells with indistinct processes and
rium, CD34 positive cells (non-Schwann). microcystic change); nuclear inclusions, nuclear

Neurosurgery Books Full


www.ketabpezeshki.com 66485438-66485457
116 PART I BASIC SCIENCE

pleomorphism; hyalinized vessels; larger schwan- arranged in herringbone pattern, hyperchromatic


nomas may undergo degenerative change and nuclei, moderate to marked pleomorphism, mito-
necrosis. Variants: cellular Schwannoma (hyper- ses and necrosis. Variants: Epithelioid MPNST,
cellular, Antoni A without Verocay bodies, MPNST with mesenchymal differentiation
low mitoses, paravertebral and cranial nerve (including Triton tumor), MPNST with glandu-
locations), plexiform Schwanomma (affect skin/ lar differentiation, melanotic MPSNT. Immuno-
subcutaneous tissues of extremities in schwanno- histochemistry: scattered S100 reactivity in
matosis) and psammomatous/melanotic schwan- 50-70%, p53 reactive, Leu-7 focally positive,
noma (50% associated with Carney complex; Ki67 5-65%. Differential: fibrosarcoma, synovial
10% undergo malignant transformation). Immu- sarcoma, leiomyosarcoma, melanoma.
nohistochemistry: Reactive for S100, Leu-7,
calretinin; basement membrane type IV collagen Image with permission from Yachnis AT, Rivera-
Zengotita ML. Neuropathology, High-Yield Pathology
and laminin. Differential by location: CPA— Series, Saunders, Elsevier, 2014.
meningioma/epidermoid/ependymoma, spinal—
meningioma/myxopapillary ependymoma, 52. d—Primary CNS lymphoma is a malignancy
skin—neurofibroma, large nerve root—MPNST. arising independently within the CNS with-
out evidence of systemic lymphoma. Com-
Image with permission from Yachnis AT, Rivera-
Zengotita ML. Neuropathology, High-Yield Pathology prise 1-5% of primary brain tumors; peak
Series, Saunders, Elsevier, 2014. incidence is in immunocompetent individ-
uals age 55-77 years; increased risk in inher-
49. a—Antony B schwannoma ited or acquired immunodeficiency (AIDS,
transplant patients, Wiskott-Aldrich syn-
Image with permission from Yachnis AT, Rivera- drome) where it is associated with EBV;
Zengotita ML, Neuropathology, High-Yield Pathology
Series, Saunders, Elsevier, 2014. HAART therapy has reduced incidence in
AIDS patients; 50% of post-transplantation
50. e—Verocay bodies PCNSL occur in first year. Present as focal
or multifocal neurologic deficit, neuropsy-
Image with permission from Yachnis AT, Rivera- chiatric symptoms, seizures, raised ICP. Sur-
Zengotita ML. Neuropathology, High-Yield Pathology vival without therapy is 6 months; treatment
Series, Saunders, Elsevier, 2014. is systemic and intrathecal methotrexate che-
motherapy with or without whole brain
51. d—Malignant peripheral nerve sheath tumor
radiotherapy; also rituximab (anti-CD20
(MPNST)
monoclonal antibody) and autologous stem
These are high-grade sarcomas arising from cell transplantation. Five-year survival in
immunocompetent individuals is 75% (under
peripheral nerve or extraneural soft tissue with
60 years) and 20% (over 60)—in general
nerve sheath differentiation. Represent 5% of
malignant soft tissue tumors; over half arise from approaches 48 months. HIV-infected
patients undergoing HAART and radiation
neurofibromas in NF1 (usually plexiform); spo-
reported median survival 36 months. Imag-
radic cases peak fifth decade, NF1 associated ear-
ing: highly variable appearance, usually deep
lier (second and third decades). Genetics:
and periventricular location; iso-
MPNSTs associated with NF-1 have both NF1
hyperintense T2, less peritumoral edema than
alleles inactivated; alterations in p53, p16, and
metastasis/abscess, in context of AIDS diffi-
p27; RB and p53 pathways are altered. Presenta-
cult to distinguish from opportunistic infec-
tion: progressively enlarging painful mass on an
tions, steroids may cause disappearance
extremity; medium to large nerves, sciatic nerve
(glucocorticoids are lymphocytic) but usually
commonest; radicular pain with spinal cord
lesions; cranial nerves rare (spontaneously from returns. Gross appearance: single or multifo-
cal (commoner in immunocompromised)
schwannoma/neurofibroma). Treatment is surgi-
deep lesions, necrosis is present, lymphomato-
cal resection with irradiation, but poor prognosis:
sis cerebri. Histology: 92-98% are aggressive
60% mortality (80% for paraspinal lesions), local
non-Hodgkin B-cell lymphoma (95% are
recurrence 40-65%, metastasis 30-80% (usually
large cell), express pan-B-cell lineage antigens
lungs), overall 16-39% 5-year survival. Imaging:
(CD20, CD79a), atypical lymphoid cells,
Inhomogenous contrast enhancement with irreg-
angiocentric growth pattern, mitotic activity,
ular contours and invasion. Gross appearance:
necrosis, background reactive astrocytes; low
fusiform/globoid, thick pseudocapsule, light tan
grade B-cell lymphoma and T cell lymphoma
fleshy tumor with necrosis and hemorrhage. His-
tology: cellular/hypercellular bipolar spindle cells less common. Immunohistochemistry: CD

Neurosurgery Books Full


www.ketabpezeshki.com 66485438-66485457
6 NEUROPATHOLOGY III: HISTOLOGY 117

20, CD79a (CD19 flow cytometry); majority pineal parenchymal tumors, germ cell
express BCL-6 and BCL-2 (but not indica- tumors, metastasis, normal pineal gland.
tive of t(14;18) translocation); 90-100%
express MUM-1; high Ki 67 index 80%. Image with permission from Yachnis AT, Rivera-
Zengotita ML. Neuropathology, High-Yield Pathology
Differential: glioma, opportunistic infection Series, Saunders, Elsevier, 2014.
in immunocompromised individuals, metas-
tasis of systemic lymphoma (meningeal > 55. d—Pineoblastoma is an aggressive embryo-
cerebral). nal tumor of pineal region—WHO grade
IV. Accounts for 40% of all pineal parenchy-
Image with permission from Yachnis AT, Rivera-
Zengotita ML. Neuropathology, High-Yield Pathology mal tumors; present in first two decades of
Series, Saunders, Elsevier, 2014. life. Genetics: 3% of patients with bilateral
retinoblastoma have pineoblastoma (“trilat-
53. e—Pineocytoma is a primary pineal paren- eral retinoblastoma”; RB1 mutation; pineo-
chymal tumor composed of small, uniform blastoma must be concurrent/sequential
pinealocyte-like cells—WHO grade I. rather than pineal retinoblastoma metasta-
Accounts for 14-60% of pineal parenchymal sis). Treatment is resection and radiotherapy;
tumors; mean age 38. Presentation: raised infiltration into adjacent brain and CSF dis-
ICP/hydrocephalus from aqueduct com- semination is common; median postsurgical
pression, Parinaud's syndrome (upgaze survival 24-30 months; prognosis for trilat-
palsy, light-near dissociation, convergence/ eral retinoblastoma is <1-year survival from
retraction nystagmus, lid retraction/sun set- diagnosis. Imaging: ill-defined lobular mass
ting), brainstem/cerebellar dysfunction, >3 cm, heterogenous contrast enhancement.
hypothalamic dysfunction. Treatment is Histology: sheets of densely packed small
surgical resection; 5-year survival 85% blue cells, no pineocytomatous rosettes but
(debulking), does not metastasize. Imaging: occasional Homer Wright or Flexner-
contrast enhancing, T2 hyperintense. Wintersteiner rosettes; necrosis/calcification
Gross: gray-tan tumor, well demarcated. common; high nuclear/cytoplasmic ratio;
Histology: pineocytomatous rosettes, sheets increased mitotic index. Immunohistochem-
of well-differentiated pinealocyte-like cells, istry: positive for neuronal markers and
low mitotic rate, no necrosis, occasional retinal-S-antigen. Differential: pineal paren-
multinucleate giant cells, ganglion cells. chymal tumors, medulloblastoma metastasis,
Immunohistochemistry: positive neural AT/RT, germ cell tumors, metastasis.
markers (synaptophysin, NF, NSE), positive
retinal S-antigen and rhodopsin, Ki67 Image with permission from Yachnis AT, Rivera-
Zengotita ML. Neuropathology, High-Yield Pathology
< 3%. Differential: normal pineal gland tis- Series, Saunders, Elsevier, 2014.
sue (lobular and contains calcifications),
pineal cyst, pineal tumor of intermediate 56. c—Germinoma is a germ cell tumor (resem-
differentiation or pineoblastoma, germ cell bling testicular seminoma and ovarian dys-
tumors, metastasis. germinoma) arising in the pineal or
suprasellar region. Account for 3% of pediat-
Image with permission from Yachnis AT, Rivera-
Zengotita ML. Neuropathology, High-Yield Pathology ric intracranial tumors; 65% of CNS germ
Series, Saunders, Elsevier, 2014. cell tumors are germinomatous (i.e., germi-
nomas; lack cellular differentiation), 35%
54. d—Pineal parenchymal tumors of intermedi- are non-germinomatous (display tissue type
ate differentiation define those intermediate- differentiation; teratoma, yolk sac, embryo-
grade malignancies between pineocytoma nal carcinoma, choriocarcinoma); male pre-
and pineoblastoma. Account of 20% of pineal dominance; present teens/early 20s.
parenchymal tumors. Similar presentation Presentation: 80% are midline pineal (hydro-
and imaging features to pineocytoma. Treat- cephalus, Parinaud's) or suprasellar lesions
ment is resection with 40-75% 5-year sur- (visual field defect, panhypopituitarism, dia-
vival rate. Gross: solid gray-tan tumor. betes insipidus). Given the lack of differenti-
Histology: diffuse or lobular architecture, ation in germinomas serum and CSF markers
moderate/high cellularity, mitotic index may often be negative. Primary treatment is
<15%, occasional giant cells and Homer radiotherapy—10-year survival 85% or
Wright rosettes. Immunohistochemistry: more; poorer prognosis with mixed germi-
positive for neuronal markers, variable for noma (germinoma with syncytiotrophoblas-
NF, chromogranin, retinal S-antigen and tic giant cells, mixed germinoma/teratoma).
S100. Ki67 index 3-10%. Differential: other Imaging: hyperdense, enhancing lesions.

Neurosurgery Books Full


www.ketabpezeshki.com 66485438-66485457
118 PART I BASIC SCIENCE

Gross: solid, soft tan-white tumor. Histol- Image with permission from Yachnis AT, Rivera-
ogy: biphasic with large neoplastic cells Zengotita ML. Neuropathology, High-Yield Pathology
Series, Saunders, Elsevier, 2014.
with abundant cytoplasm intermixed with
fibrovascular septae harboring lymphocytic
infiltrate, occasional granulomas/syncytio- 58. e—Yolk sac (endodermal sinus) tumor.
trophoblastic cells/foci of other germ cell
Yolk sac tumors may have gelatinous appearance
tumor. Immunohistochemistry: positive
Yolk sac (endodermal sinus) tumors: highly vari-
CD117 (c-kit), OCT3/4, SALL4, placental
able histology with cuboidal/elongated epithelial
alkaline phosphatase (PLAP); if syncytiotro-
cells surrounding fibrovascular cores (Schiller-
phoblasts present betaHCG or human
Duval bodies) or having eosinophilic hyaline
placental lactogen may be present. Differen-
globules within cytoplasm. Immunoreactive for
tial: primary pineal/suprasellar tumor, non-
AFP, SALL4, glipican-3.
germinomatous germ cell tumors.
Image with permission from Yachnis AT, Rivera-
Image with permission from Yachnis AT, Rivera- Zengotita ML. Neuropathology, High-Yield Pathology
Zengotita ML. Neuropathology, High-Yield Pathology Series, Saunders, Elsevier, 2014.
Series, Saunders, Elsevier, 2014.

57. e—Teratoma. Numerous germ cell ele- 59. b—Embryonal carcinoma. This highly
ments are present including cartilage malignant neoplasm consists of large cells
(upper left), squamous epithelium with skin with prominent nucleoli and abundant clear
adnexal structures (upper right), bone to eosinophilic cytoplasm forming solid
(lower right), and neuroepithelial elements sheets, nests, or lining glandlike spaces; fre-
(center). quent mitoses and necrosis. Embryonal car-
cinoma: positive for cytokeratins, CD30,
Non-germinomatous germ cell tumors OCT4, SALL4, PLAP.
(NGGCTs) include teratomas, embryonal carci-
Image with permission from Yachnis AT, Rivera-
nomas, yolk sac tumor (endodermal sinus tumor) Zengotita ML. Neuropathology, High-Yield Pathology
and choriocarcinomas. Prognosis depends on his- Series, Saunders, Elsevier, 2014.
tological subtype of GTC: good prognosis (pure
germinoma, mature teratoma), intermediate prog- 60. a—Choriocarcinoma. These highly hemor-
nosis (germinoma with syncytiotrophoblasts, rhagic tumors contain neoplastic cytotropho-
immature teratoma, teratoma with malignant blastic (left) and syncytiotrophoblastic giant
transformation, mixed germinoma/teratoma), and cells (right center). Choriocarcinoma: com-
poor prognosis (choriocarcinoma, yolk sac tumor, posed of both neoplastic cytotrophoblastic
embryonal carcinoma, mixture of these). Surgical and syncytiotrophoblastic giant cells with
resection of mature teratoma can be curative. extensive hemorrhagic necrosis. Choriocar-
Teratomas are comprised of cells originating cinoma: syncytiotrophoblastic cells positive
from usually all three of endoderm, mesoderm for β-HCG, HPL, PLAP (variable).
and ectoderm. Intra-axial teratomas are rare
but are the commonest cause of fetal brain Image with permission from Yachnis AT, Rivera-
tumors (25-50%); located in cerebral hemi- Zengotita ML. Neuropathology, High-Yield Pathology
spheres presenting antenatally or in the neonatal Series, Saunders, Elsevier, 2014.
period with increased head circumference.
Extra-axial (suprasellar or pineal) teratomas are 61. b—Hemangioblastoma is a highly vascular-
commoner and present in childhood/early adult- ized tumor composed of stromal cells and cap-
hood. Mature teratoma contain differentiated illaries. Accounts for 1-2% of intracranial
skin, brain, cartilage, fat, respiratory/enteric epi- tumors; peaks in third to fifth decade (affects
thelium and may be cystic (n.b. dermoids are children as part of VHL); mostly sporadic;
ectodermal in origin only hence will not contain 25% associated with von Hippel-Lindau
fat). Immature teratomas have foci of incom- disease (autosomal dominant inherited
pletely differentiated tissue elements including defect in VHL gene on 3p25-p26; multiple
immature neuroepithelium, embryonal mesen- hemangioblastomas, renal cell carcinoma,
chymal tissue, abortive retinal epithelium; malig- phaeochromocytoma, pancreatic/liver cysts,
nant transformation of any component can endolymphatic sac tumor). Presentation: spo-
occur. Immunohistochemistry: cytokeratin posi- radic hemangioblastomas commonest in cere-
tive in epithelial elements. bellum, while those in VHL in cerebellum,

Neurosurgery Books Full


www.ketabpezeshki.com 66485438-66485457
6 NEUROPATHOLOGY III: HISTOLOGY 119

brain stem, spinal cord (associated with syr- 8. Gonadotrophins (FSH/LH): amenorrhoea
inx), cerebrum, leptomeninges, retina and in females, impotence in males
peripheral nerves; 10% associated with sec- Medical treatment of prolactinomas with dopa-
ondary polycythemia (stromal EPO produc- mine receptor agonist (bromocriptine or caber-
tion). Surgical resection is treatment of goline) and GH-secreting tumors with
choice for these benign lesions; can bleed somatostatin analogs (octreotide). Transphenoi-
extensively hence may require preop emboli- dal resection for symptomatic non-functioning
zation; multiple lesions and recurrence seen in lesions or resistant to medical therapy. Radio-
VHL. Imaging: commonly cystic lesion with therapy for residual tumor, recurrence or inva-
contrast-enhancing mural nodule in cerebel- sive. Management of pituitary apoplexy
lum. Gross: circumscribed, non-encapsulated includes glucocorticoids, close monitoring and
usually cystic with vascularized red mural surgical decompression if vision deteriorating.
nodule or yellow from lipid cells. Histology: Imaging: circumscribed, variably enhancing
lipid rich stromal cells with vacuolated cyto- compared to adjacent pituitary, macroadenomas
plasm ovoid nuclei surrounded by network expand the sella. Gross: circumscribed, non-
of capillaries; intratumoral sclerosis/hemor- encapsulated tan-brown tumor (microadenoma
rhage; mitoses/necrosis rare; Rosenthal fibers <10 mm, macroadenoma >10 mm). Histology:
and reactive gliosis in cyst wall. Immunohisto- loss of normal lobar architecture of anterior
chemistry: Stromal cells positive for NSE, pituitary; sheets and cords of monomorphic
inhibin A, aquaporin 1, S100, CD56, vimen- cells; basophilic granules in ACTH secreting
tin, GFAP; unlike renal cell carcinoma also types; eosinophilic cytoplasmic granules
negative for CD10, EMA, cytokeratin; capil- (cytokeratins; CAM5.2) in GH-secreting types.
laries positive for CD31, CD34, and reticulin. Immunohistochemistry: reticulin staining
Differential: metastatic RCC, pilocytic astro- demonstrates loss of architecture; chromogra-
cytoma, endocrine neoplasm. nin, cytokeratin, EMA positive; hormone
stains. Differential: pituicytoma, craniopharyn-
Image with permission from Yachnis AT, Rivera- gioma, granular cell tumor, sellar meningioma/
Zengotita ML. Neuropathology, High-Yield Pathology
Series, Saunders, Elsevier, 2014. schwanomma, Rathke cleft cyst, germ cell
tumor, metastasis, hypothalamic hamartoma/
62. c—Pituitary adenomas are tumors of the glioma, carotid cavernous fistula/aneurysm.
anterior pituitary gland. Represent 10-15%
Image with permission from Yachnis AT, Rivera-
of all intracranial tumors; incidence 0.2-2.8 Zengotita ML. Neuropathology, High-Yield Pathology
per 100,000 per year; 20-30% non-function- Series, Saunders, Elsevier, 2014.
ing, 25% prolactin-secreting, 20% growth
hormone, 10% ACTH, 10-15% secrete 63. a—Adamantinomatous Craniopharyngioma
FSH/LH, 1-3% secrete TSH; occasional
bihormonal adenomas, usually prolactin Craniopharyngiomas are epithelial tumors
and GH. Genetics: most sporadic; 3% associ- derived from Rathke's pouch. Accounts for 3%
ated with multiple endocrine neoplasia type 1 of intracranial tumors; Incidence is 0.5-2.5 per
(11q13 mutation; pituitary, pancreatic and million/year; no gender predilection; adamanti-
parathyroid tumors). Presentation: nomatous subtype (children; 5-10% of pediatric
1. General: headache, facial pain, fatigue, intracranial tumors) and papillary subtype
weight loss (exclusively adults). Presents due to local com-
2. Mass effect: bitemporal hemianopia, cranial pression in suprasellar region: hypothalamus
nerve palsy, stalk effect (prolactin rise with- (endocrine dysregulation/diabetes insipidus),
out prolactinoma) third ventricle (hydrocephalus), optic chiasm
3. Pituitary apoplexy: hemorrhagic necrosis of (bitemporal hemianopia). General symptoms:
adenoma—sudden headache, visual loss, cognitive impairment, N&V, somnolence. Man-
Addisonian crisis agement is surgical resection, with >90% 10-
4. Prolactin excess: amenorrhoea, irregular year recurrence free survival depending on
periods, galatorrhoea, infertility in women; extent of resection (limited by adherence to adja-
hypogonadism, loss of libido, impotence cent structures); rare reports of malignant trans-
in men formation to SCC post-radiotherapy. May need
5. Growth hormone excess: gigantism in chil- hormone replacement if hypopituitarism. Imag-
dren, acromegaly in adults ing: suprasellar cystic mass (T1 hyperintense)
6. ACTH excess: Cushing disease but occasionally solid enhancing tumor (T1
7. TSH excess: thyrotoxicosis hypointense). Gross: solid and cystic mass with

Neurosurgery Books Full


www.ketabpezeshki.com 66485438-66485457
120 PART I BASIC SCIENCE

calcification, motor-oil like material in adaman-


tinomatous type. ANSWERS 64–81
Additional answers 64–81 available on
Histology: ExpertConsult.com
1. Adamantinomatous: nests and trabeculae of
epithelium in lose fibrous stroma, palisad-
ing at periphery of tumor, stellate reticulum
(loosely arranged epithelium), wet keratin
(eosinophilic), cysts, calcification, xantho- GLOSSARY
granulomatous reaction (cholesterol clefts,
foreign body giant cells, inflammation), P: chromosome short (petit) arm
gliosis/Rosenthal fibers at lesion edges Q: chromosome long arm
2. Papillary: circumscribed papillary tumor Myxoid: mucous like
with fibrovascular cores lined by well- Papillary: with small nipple-like projections
differentiated monotonous squamous epi- Spindle: cell long and slender
thelium (i.e., no stellate reticulum, wet ker- Palisade: row of elongated nuclei parallel to
atin, calcification, xanthogranulomatous each other
change). Rosette: ring or garland-like arrangement of
Immunohistochemistry: cytokeratin and focal nuclei with or without a central channel
HCG, CK7, CK20; adamantinomatous subtype Piloid: thin hair-like cells arranged in
associated with beta-catenin mutations resulting parallel array
in nuclear reactivity. Differential: xanthogranulo- Fibrillary cells: have projecting fibres
matous inflammation, Rathke cleft cyst, epider- Clear cells: have abundant clear cytoplasm
moid cyst, metastasis, pilocytic astrocytoma. Small cells: primitive-appearing cells with little
Image with permission from Yachnis AT, Rivera- cytoplasm
Zengotita ML. Neuropathology, High-Yield Pathology Storiform: cartwheel pattern (i.e., radiating out
Series, Saunders, Elsevier, 2014. from center)

Neurosurgery Books Full


www.ketabpezeshki.com 66485438-66485457
CHAPTER 7

PHARMACOLOGY
SINGLE BEST ANSWER (SBA) QUESTIONS
1. Which one of the following best describes 6. Which one of the following best describes
the main mechanism of propofol sedation? the mechanism of dexamethasone action in
a. Cyclo-oxygenase inhibition reducing cerebral edema?
b. Depolarizing neuromuscular blockade a. Reduces cytotoxic edema through nitric
c. Endocannabinoid activation oxide inhibition
d. Non-depolarizing neuromuscular blockade b. Reduces cytotoxic edema through VEGF
e. Potentiates GABA-A receptor activity inhibition
c. Reduces vasogenic edema through VEGF
2. Which one of the following combinations of inhibition
clotting factors are affected by warfarin? d. Reduces vasogenic edema through upre-
a. II, IX, X, Protein C gulation of aquaporins
b. II, VII, IX, X e. Reduces vasogenic edema through nitric
c. II, VII, IX, X, Protein C, Protein S oxide signaling
d. II, VII, X, Protein C
e. II, VII, X, XII 7. Propofol-related infusion syndrome is
usually characterized by which one of the
3. Which one of the following blood tests following?
would you perform to monitor the effect of a. Acute refractory bradycardia with meta-
low molecular weight heparin? bolic alkalosis
a. Factor VII b. Acute refractory bradycardia with meta-
b. Factor VIII bolic acidosis
c. Factor Xa activity c. Acute refractory bradycardia with respira-
d. Prothrombin time tory alkalosis
e. Von Willebran Factor d. Acute refractory bradycardia with respira-
tory acidosis
4. A 75-year-old patient presents with GCS
e. Acute refractory bradycardia with normal
E3V4M5 and due to ICH. INR is 5.0 on war-
acid-base balance
farin for atrial fibrillation. Assuming you have
access to all of the following therapies, which
8. Red man syndrome is seen with which one of
one of the following is the most appropriate
the following medications?
next treatment?
a. Levodopa
a. Fresh frozen plasma
b. Procyclidine
b. Protamine
c. Prothrombin complex concentrate c. Propofol
d. Rifampicin
d. Recombinant factor VIIa
e. Vancomycin
e. Vitamin K

5. A patient with a right extradural hematoma 9. Which one of the following is the initial
fixes and dilates his right pupil on the way to treatment for a dystonic reaction to
theater. The anesthetist administers 100 ml levodopa?
of 20% mannitol and his pupil normalizes a. Adenosine
after 2 min. Which one of the following best b. Bromocriptine
explains the immediate effect of mannitol? c. Cyclizine
a. Autoregulatory vasoconstriction d. Procyclidine
b. Diuretic effect e. Topiramate
c. Increased cerebral blood volume
d. Osmotic effect reducing interstitial 10. Which one of the following would you mon-
brain fluid itor during infusion of an intravenous loading
e. Local effect on pupillary constrictors dose of phenytoin?
121
Neurosurgery Books Full
www.ketabpezeshki.com 66485438-66485457
122 PART I BASIC SCIENCE

a. Capillary blood glucose For each of the following descriptions, select the
b. Cardiac monitoring most appropriate answers from the list above.
c. Nystagmus Each answer may be used once, more than once
d. Peak flow rate or not at all.
e. Urine output 1. Microtubule inhibitor
2. VEGF inhibitor
QUESTIONS 11–20 3. SEGA treatment
4. Chemotherapy wafer for GBM
Additional questions 11–20 available on
ExpertConsult.com 23. Bleeding:
a. Abciximab/epitifibatide/tirofiban
b. Aspirin
c. Clopidogrel/prasugrel
d. Dalteparin
EXTENDED MATCHING ITEM (EMI) e. Desmopressin
QUESTIONS f. Dipyridamole
g. Enoxaparin
21. Anticonvulsants: h. Fondaparinux
a. Acetazolamide i. Protamine
b. Carbamazepine j. Rivaroxiban
c. Ethosuxamide k. Tranexamic acid
d. Gabapentin l. Warfarin
e. Lacosamide For each of the following descriptions, select the
f. Lamotrigine most appropriate answers from the list above.
g. Levetiracetam Each answer may be used once, more than once
h. Perampanel or not at all.
i. Phenytoin 1. ADP receptor antagonist
j. Pregabalin 2. Glycoprotein IIa/IIIb antagonist
k. Sodium Valproate 3. Used preoperatively in factor VIII
l. Topiramate dysfunction
m. Vigabatrin 4. Irreversible cyclo-oxygenase inhibitor
n. Zonisamide 5. Adenosine reuptake inhibitor

For each of the following descriptions, select the 24. Anticoagulants:


most appropriate answers from the list above. a. Antithrombin protein
Each answer may be used once, more than once b. Batroxobin
or not at all. c. Citrate
1. Risk of thrombocytopenia and increased d. Dabigatran
warfarin effect e. EDTA
2. Affect T-type calcium current and used in f. Fondaparinux
absence seizures g. Hirudin
3. Associated with weight loss h. Low molecular weight heparin
i. Rivaroxiban
22. Chemotherapy: j. Unfractionated heparin
a. Bevacizumab k. Warfarin
b. Carmustine (BNCU)
c. Cisplatin For each of the following descriptions, select the
d. Erlotinib most appropriate answers from the list above.
e. Everolimus Each answer may be used once, more than once
f. Lomustine (CCNI) or not at all.
g. Methotrexate 1. Synthetic pentasaccharide factor Xa inhibitor
h. Procarbazine 2. Direct inhibitor of factor Xa
i. Tamoxifen 3. Direct thrombin inhibitor
j. Temozolomide
k. Vincristine

Neurosurgery Books Full


www.ketabpezeshki.com 66485438-66485457
7 PHARMACOLOGY 123

25. Anesthetic drugs: 26. Reversal of clotting abnormalities:


a. Atracurium a. Desmopressin
b. Clonidine b. Factor IX
c. Desmopressin c. Factor VIII
d. Dobutamine d. Fresh frozen plasma
e. Fludrocortisone e. Platelets
f. Glycopyronium f. Protamine
g. Ketamine g. Prothrombin complex concentrate
h. Labetalol h. Recombinant factor VIIa
i. Mannitol i. Tranexamic acid
j. Midazolam j. Tranexamic acid
k. Noradrenaline k. Vitamin K
l. Propofol
m. Remifentanil For each of the following descriptions, select the
n. Thiopentone most appropriate answers from the list above.
o. Vasopressin Each answer may be used once, more than once
or not at all.
For each of the following descriptions, select the 1. Preoperatively in patients with mild
most appropriate answers from the list above. hemophilia A
Each answer may be used once, more than once 2. Major hemorrhage in warfarinized patient
or not at all.
1. Associated with Ondine's curse
2. Associated with a low serum potassium
despite normal total body potassium when
chronic infusion used
3. Associated with raised intracranial pressure

SBA ANSWERS
1. e—GABA(A) receptor activation. While pro- blood viscosity (reduction in volume, rigidity,
pofol may have multiple effects, the main and cohesiveness of RBC). Altered blood rheology
mechanism of action is thought to result from reduces cerebral vascular resistance, increases
activation of GABA(A) receptors, causing cerebral blood flow and CPP. Autoregulatory
increased transmembrane chloride conduc- vasoconstriction then reduces CBV (to restore
tance and hyperpolarization of the neuron normal CPP) and reduces ICP. These immediate
preventing generation of an action potential. rheological effects may also explain why ICP
reduction with mannitol occurs in situations
2. c—II, VII, IX, X, Protein C, Protein S. Pro- where the BBB is not intact. The osmotic effect
tein C has a short half-life (8 h) compared of mannitol in causing brain shrinkage by drawing
with other vitamin K-dependent factors and water out requires an intact BBB (across which an
therefore is rapidly depleted with warfarin osmotic gradient can be set up) and can take up to
initiation, resulting in a transient 30 min to develop. Adverse effects of mannitol
hypercoagulable state. include hypotension, renal failure (especially if
serum osmolality >320) and rebound rise in ICP
3. c—Factor Xa activity. (penetration of osmotically active solutes into
edematous brain reversing osmotic gradient).
4. c—Prothrombin complex concentrate
6. c—Reduces vasogenic edema through
5. a—Autoregulatory vasoconstriction VEGF inhibition
Tumor-related disruption in the blood-brain bar-
Mannitol has hemodynamic, osmotic, and diuretic rier resulting in vasogenic edema is caused by
effects. Following a bolus of hyperosmolar manni- local factors increasing the permeability of vessels
tol, body water is drawn (including from RBC) in (VEGF, glutamate, leukotrienes) and absence of
to plasma causing expansion and reduction in normal tight endothelial junctions in tumor

Neurosurgery Books Full


www.ketabpezeshki.com 66485438-66485457
124 PART I BASIC SCIENCE

vessels as they grow in response to VEGF and presence of one or more of the following: meta-
bFGF. In large part, VEGF is responsible for bolic acidosis, rhabdomyolysis, hyperlipidemia,
the loss of integrity of the blood-brain barrier and enlarged or fatty liver. There is an association
in brain tumors. Gliomas, meningiomas, and between PRIS and propofol infusions at doses
metastatic tumors all have upregulation of higher than 4 mg/kg/h for greater than 48 h
VEGF. VEGF is secreted by tumor cells as well duration. It is proposed that the syndrome may
as host stromal cells and binds to its receptors be caused by either a direct mitochondrial respi-
VEGFR1 and VEGFR2, which are located pri- ratory chain inhibition or impaired mitochondrial
marily on the surface of endothelial cells. VEGF fatty acid metabolism mediated by propofol.
stimulates the formation of gaps in the endothe- ECG shows new right bundle branch block with
lium, a process that leads to fluid leakage into the convex-curved (“coved type”) ST elevation in the
brain parenchyma, thereby resulting in vasogenic right precordial leads (V1 to V3). Risk factors
edema. Most patients with brain tumors and peri- include young age, severe critical illness of central
tumoral edema can be adequately managed with nervous system or respiratory origin, exogenous
glucocorticoids. Reduction of intracranial pres- catecholamine or glucocorticoid administration,
sure and improvement in neurologic symptoms inadequate carbohydrate intake and subclinical
usually begins within hours. A decrease in capil- mitochondrial disease. Hemodialysis or hemoper-
lary permeability (i.e. improvement in blood- fusion with cardiorespiratory support has been
brain barrier function) can be identified within the most successful treatment.
6 h and changes of diffusion-weighted MRI indi-
FURTHER READING
cating decreased edema are identifiable within
Kam PC1, Cardone D, Propofol infusion syndrome. Anaes-
48-72 h. However, adequate reduction in ele- thesia 2007 Jul; 62(7):690-701
vated ICP resulting from peritumoral edema
may take several days with glucocorticoid therapy 8. e—Vancomycin
alone. Dexamethasone is the standard agent,
because its relative lack of mineralocorticoid Red man syndrome is characterized by a complex of
activity reduces the potential for fluid retention. symptoms including: pruritis, urticaria, erythema,
In addition, dexamethasone may be associated angioedema, tachycardia, hypotension, occasional
with a lower risk of infection and cognitive muscle aches, and a maculopapular rash that usually
impairment compared to other glucocorticoids. appears on the face, neck, and upper torso. The eti-
The mechanism of action of glucocorticoids for ology is thought to be due to a non-immune related
control of vasogenic edema is not fully understood. release of histamine.
Dexamethasone has recently been shown to upre-
gulate Ang-1, a strong BBB-stabilizing factor, 9. d—Procyclidine
whereas it downregulates VEGF, a strong permea-
bilizing factor, in astrocytes and pericytes. Gluco- 10. b—Cardiac monitoring. During intravenous
corticoids may also increase the clearance of loading of phenytoin for control of seizures
peritumoral edema by facilitating the transport of cardiac monitoring is essential due to the risk
fluid into the ventricular system, from which it is of bradycardia and heart-block. Elderly
cleared by cerebrospinal fluid (CSF) bulk flow. patients requiring multiple intravenous doses
of phenytoin should also be monitored for
FURTHER READING
purple glove syndrome.
Kim H, Lee JM, Park JS, Jo SA, et al, Dexamethasone
coordinately regulates angiopoietin-1 and VEGF: a mecha-
nism of glucocorticoid-induced stabilization of blood-brain
barrier Biochemical and Biophysical Research Communica-
ANSWERS 11–20
tions, 2008, 372(1):243-248.
Additional answers 11–20 available on
7. b—Acute refractory bradycardia with meta- ExpertConsult.com
bolic acidosis

Propofol infusion syndrome (PRIS): acute refrac-


tory bradycardia leading to asystole, in the

Neurosurgery Books Full


www.ketabpezeshki.com 66485438-66485457
7 PHARMACOLOGY 125

EMI ANSWERS
21. 1—k, Valproate; 2—c, Ethosuxamide; 3—l, Topiramate

Features of Common Antiepileptic Drugs


Drug Mechanism Clearance/Half-Life Side Effects

Carbamazepine Voltage gated Na channel Hepatic CYP450 CYP450, hyponatremia,


12-17 h neurotoxicity, leukopenia, aplastic
anemia, Stevens Johnson syndrome,
hepatitis
Sodium Voltage dependent Na Hepatic Weight gain, insulin resistance,
Valproate channels, increases GABA, 9-12 h thrombocytopenia, hepatotoxicity,
T-type Ca current teratogenicity, pancreatitis,
hyperammonemia
Gabapentin Voltage dependent Ca Renal Weight gain, sedation
channel, GABA(B) reduces 4-6 h
glutamate
Pregabalin Voltage gated Ca, glutamate, Renal Euphoria, myoclonus, weight gain
norepinephrine, substance P 6h
Lacosamide Voltage gates Na channels, 13 h Dizziness, ataxia, blurred vision
modulated CRMP-2
Ethosuxamide T-type Ca current in thalamus Hepatic Insomnia, pancytopenia,
30-60 h hyperactivity
Levetiracetam Binds SV2A and inhibits Renal Aggression, depression
presynaptic Ca channels 6-12 h
Topiramate GABA(A) receptors, NMDAR Renal Cognitive impairment, weight loss,
antagonist, weak Ca inhibitor 20 h mood disturbance, somnolence,
metabolic acidosis, renal stones
Phenytoin Voltage dependent Na Hepatic amine oxidase Gingival hypertrophy, osteomalacia,
channels, synaptic 7-40 h teratogenicity, hirsutism, rash/SJS,
transmission, Ca-calmodulin lymphadenopathy
phosphorylation
Phenobarbital GABA(A) receptor—increase Hepatic Lethargy, cognitive impairment,
chloride current 24-100 h teratogenic
Perampanel AMPAR antagonist Hepatic Dizziness, aggression, weight gain
100 h
Zonisamide Voltage dependent Na and T- Hepatic/renal Renal stones, anorexia, rash/SJS,
type Ca channels, CA inhibitor 60 h agranulocytosis
Lamotrigine Na channel blocker Liver glucoronidation, Rash/SJS, angioedema, multiorgan
renal excretion, failure/DIC, somnolence, drug
10-60 h interaction, myoclonus
Vigabatrin GABA transaminase inhibitor Renal Concentric visual field loss, weight
6-8 h gain, depression

Neurosurgery Books Full


www.ketabpezeshki.com 66485438-66485457
126 PART I BASIC SCIENCE

22. 1—k, Vincristine; 2—a, Bevacizumab; 3—e, Everolimus; 4—b, Carmustine

Features of Common Antiepileptic Drugs


Chemotherapy
Agent Mechanism Comments

Bevacizumab VEGF inhibitor GBM; hypertension, delayed wound healing, bowel perforation, ICH,
(Avastin) thrombosis
Erlotinib Tyrosine kinase Metastatic non-small cell lung cancer
inhibitor (EGFR)
Carmustine Alkylating agent GBM, astrocytoma, medulloblastoma; wafer may cause seizures,
(BNCU) cerebral infarction; intravenous may cause N&V, fatigue, respiratory
complications/pulmonary fibrosis, bone marrow suppression
Cisplatin Platinum alkylating Glioma, medulloblastoma, others; minor pancytopenia, ototoxicity,
agent peripheral neuropathy, nephrotoxicity, N + V (carboplatin can cause
alopecia)
Everolimus mTOR inhibitor GBM, SEGA
Tamoxifen Estrogen receptor Breast cancer
antagonist
Methotrexate Dihydrofolate Lymphoma; myelosuppression, mucositis, N + V, nephrotoxic, hepatic
reductase inhibitor fibrosis, pulmonary, neurotoxicity if intrathecal
Lomustine Alkylating agent PCV regimen for oligodendroglima/mixed oligoastrocytoma;
(CCNI)
Procarbazine Alkylating agent PCV regimen for oligodendroglima/mixed oligoastrocytoma;
malignant hypertension when taken with tyramine containing food.
Temozolomide Alkylating agent GBM; constipation, nausea/vomiting, fatigue, headache
Vincristine Microtubule PCV regimen for oligodendroglima/mixed oligoastrocytoma;
inhibitor pancytopenia, neuropathy, N + V, mouth ulcers, fatigue

23. 1—c, Clopidogrel; 2—a, Abciximab; 3—e, 25. 1—m, Remifentanil; 2—n, Thiopentone;
Desmopressin; 4—b, Aspirin; 5—f, 3—g, Ketamine
Dipyridamole
26. 1—a, Desmopressin; 2—g, Prothrombin
24. 1—f, Fondaparinux; 2—i, Rivaroxiban; 3—d, complex concentrate
Dabigatran

Neurosurgery Books Full


www.ketabpezeshki.com 66485438-66485457
PART II
CARE OF THE
NEUROSURGICAL PATIENT
CHAPTER 8

NEUROLOGY AND STROKE


SINGLE BEST ANSWER (SBA) QUESTIONS
1. An 84-year-old man has been brought into 2. A 70-year-old gentleman attends outpatient
hospital because of self-neglect. He lives clinic with his wife. She reports that her hus-
alone in a ground floor flat and has daily band’s behavior has changed and that he has
carer who have found him to be increasingly become increasingly forgetful over the past
suspicious, accusing them of stealing and year. He has gained 10 kg of weight over
moving his property and becoming physically the past 6 months. His wife reports that he
aggressive. In the past month he had been has an uncontrollable appetite occasionally
refusing to let them in. He also seemed to eating to the point of vomiting. She also
be experiencing auditory hallucinations and states that he has a lack of interest when
had lost weight. Examination was normal the grandchildren visit. Over the last 4 weeks
except for BMI 19 and MMSE 18/30. Bloods, she has noticed that her husband has become
CXR, urine, cultures normal. CT is shown. more unsteady on his feet having had a num-
Which one of the following is most likely? ber of falls. On examining him in clinic he has
impaired word comprehension, reduced
safety awareness on mobilizing and a positive
palmomental reflex. There is no tremor,
rigidity or shuffling gait. MMSE is 22/30.
CT head is shown. Which one of the follow-
ing is most likely?

a. Alcoholic hallucinosis
b. Alzheimer’s disease
c. Delirium
d. Paranoid schizophrenia
e. Pick’s disease

127
Neurosurgery Books Full
www.ketabpezeshki.com 66485438-66485457
128 PART II CARE OF THE NEUROSURGICAL PATIENT

a. Depression
b. Hypomania
c. Pick’s disease
d. Lewy body dementia
e. CJD

3. A 64-year-old man presents with a 6 month


history of abnormal behaviors which have
been noticed by his wife. He has described
seeing vivid visual hallucinations of clowns
in his living room which sometimes talk to
him and appear very real. He believes that
he is the head of a circus and is about to go
on a world tour although this is not true.
At times he is lucid and is fully independent
but at other times he is disorientated in time
and place and is unable to perform simple
tasks such as preparing food and going to
the shops. His wife thinks that his mood is
also lower since the onset of symptoms. He
presented in A +E today because of having a
second fall in 2 weeks. There is no history a. Alzheimer’s disease
of infective symptoms. He went to see his b. Creutzfeldt-Jakob disease
GP two days ago who thought that he may c. Carbon monoxide poisoning
have a UTI and prescribed trimethoprim. d. Huntington’s disease
He has a history of stroke 10 years ago and e. Pick’s disease
hypertension and takes warfarin, amlodipine,
and enalapril. Physical examination is unre- 5. A 81-year-old male experiences progressive
markable except for slightly increased tone cognitive decline over the past 10 years.
on the left side compared to the right. Which His wife reports that every 6 months or so
one of the following is most likely? she will notice another significant decrease
a. Alzheimer’s disease in his functioning. It is now at the point where
b. Semantic dementia he is aggressive and has little short-term mem-
c. Hypothyroidism ory. Past medical history includes hyperten-
d. Lewy body dementia sion and percutaneous coronary intervention
e. Schizophrenia after a myocardial infarction. Examination
findings include poor attention and memory,
4. A 55-year-old man presents with cognitive mild left hemiparesis (face, arm, and leg), and
decline over a 6-month period. He continues brisk reflexes throughout with extensor plan-
to progress and develops myoclonus and a left tar reflex bilaterally and a shuffling
hemiparesis. On examination, he is alert and gait. Which one of the following is most
orientated to time and place but appears easily appropriate?
startled every time you start a sentence. There a. Referral for subthalamic nucleus deep
is bilateral finger-nose and heel-shin dysme- brain stimulation
tria, mild postural tremor and mild speech b. Treat cardiovascular risk factors
slurring. Blood tests are normal including thy- c. Carotid endarterectomy
roid and liver function. Lumbar puncture: d. Commence carbidopa/levodopa
WCC < 1, RBC 16, Protein 0.5 g/l, Glucose e. Commence memantine
3.4 mmol/l, gram stain negative, and no
organisms cultured. An EEG demonstrated
brief periodic spikes. A MRI head (FLAIR
sequence) is shown. Which one of the follow-
ing is most likely?

Neurosurgery Books Full


www.ketabpezeshki.com 66485438-66485457
8 NEUROLOGY AND STROKE 129

6. A 55-year-old man presents with a 2-month


history of weakness in his right arm. He has
also noticed that his voice has become softer.
He is finding it hard to use door handles and
open jars. On two occasions his wife has
noticed him stumbling whilst walking. On
examination he has fasciculations over his
right deltoid muscle and wasting of the inter-
ossei muscles of the right hand. Power is 4/5 in
right shoulder abduction with absent reflexes
in the right arm but present elsewhere.
Coordination and sensation are normal with
a negative Romberg's test. Which one of the
following is the most likely diagnosis?
a. Cervical myelopathy
b. Diabetic neuropathy
c. Amyotrophic lateral sclerosis
d. Multiple sclerosis a. Sulfadiazine + pyrimethamine
e. Hereditary sensory motor neuropathy b. Dexamethasone
c. Methotrexate
7. An 18-month-old girl presents with leg weak- d. Amphotericin B
ness. Tremors, primarily of the hands, had e. Image guided aspiration and intravenous
been noted since 4 months of age. She was antibiotics.
crawling by 9 months of age and cruising
about the furniture by 12 months. Her lan- 9. A 31-year-old man was diagnosed with HIV
guage development was normal. Her 4-year- 5 years ago and had been taking highly active
old sister was developing normally. Cranial antiretroviral therapy until 8 months ago when
nerve examination was normal, and specifi- he decided to stop. He had been doing well on
cally, fasciculations of the tongue were not highly active antiretroviral therapy, but stopped
noted. She was able to sit, crawl, and pull to a taking his medications 8 months ago because he
stand. She could walk holding onto furniture thought that he would be better off. Two
but could not walk independently. Deep ten- months ago, he was successfully treated for
don reflexes were absent throughout, and there Pneumocystis carinii pneumonia. He now pre-
were no Babinski signs. Sensory examination sents with confusion and speech deficit. His
was normal. Which one of the following is CD4 count is 155/ul. MRI appearances are
the next appropriate test? shown below. CSF PCR is positive for JC virus.
a. Serum ceruloplasmin Which one of the following is most likely?
b. Electromyography
c. Nerve conduction studies
d. Survival motor neuron gene testing
e. MRI head

8. A 41-year-old man presents with confusion


and headaches for the last few weeks. He
was diagnosed with HIV 15 years ago and
has been stable on highly active antiretroviral
treatment. Other past medical history
includes an episode of Pneumocystis jirovecii
pneumonia 1 year ago. His latest CD4 count
is 29 cells/μl. An MRI (T1 C+) is shown. The
enhancing lesions on MRI show increased
uptake on Thallium-201 Chloride SPECT
scan. Which one of the following is likely to
be required?

Neurosurgery Books Full


www.ketabpezeshki.com 66485438-66485457
130 PART II CARE OF THE NEUROSURGICAL PATIENT

a. Adrenoleukodystrophy no space occupying lesion; no hydrocephalus.


b. Multiple sclerosis MRI brain with gadolinium: diffuse pachyme-
c. Subacute sclerosing panencephalitis ningeal enhancement without leptomeningeal
d. Progressive multifocal leukoencephalopa- enhancement; subtle downward displacement
thy of brain on sagittal views. Which one of the
e. AIDS dementia complex following would be appropriate next in acute
management?
10. A 43-year-old man has been having nightly,
unilateral, throbbing headaches with the pain
focused at the back of his left eye. They have
been occurring daily for the past week. The
patient recalls having had a similar headache
5 years ago that lasted for several weeks. The
patient has noticed that the headache is asso-
ciated with lacrimation and nasal congestion.
Which one of the following would be appro-
priate next in acute management?
a. Dihydroergotamine
b. Glyceryl trinitrate
c. Indometacin
d. Inhaled 100% oxygen a. Epidural blood patch
e. Propanolol b. Flat bed rest
c. Laminectomy dural repair and sealant
11. A 18-year-old female presents with a severe d. Lumbar puncture
right-sided throbbing headache associated e. MRI whole spine with STIR
with nausea, vomiting, and photophobia which
failed to respond to ibuprofen. There are no 13. A 26-year-old female presents with difficulty
other neurological features in the history. walking and complains of problems with her
She has been having similar headaches 3-4 vision in her right eye. She had an episode of
times per month for the past year. Her mother diarrhea a week ago, but has no other relevant
had a similar problem. Her examination is past medical history apart from problems with
normal. Which one of the following would her left eye 3 months earlier which had
be appropriate next in acute management? resolved. On examination there is a right rela-
a. Amitriptyline tive afferent papillary defect. Visual acuity and
b. Propanolol color vision are 6/6 (20/20) with 17/17 Ishihara
c. Sumatriptan plates on the left, and 6/60 (20/200) with 0/17
d. Topiramate Ishihara plates on the right. She reports no dip-
e. Verapamil lopia with a full range of eye movements, no
facial weakness and normal facial sensation.
12. A 45-year-old man is referred urgently to Fundoscopy was unremarkable. Examination
hospital with a severe headache. The pain revealed 2/5 power on the left arm and leg in
had started gradually three days before and all movements; and 4/5 in all movements in
was now severe. The patient reported the right arm and leg, brisk reflexes bilaterally with
headache was exacerbated by an upright pos- extensor plantar responses. There is patchy
ture with relief obtained by lying flat. Since loss of sensation to cotton wool on right
the headache started the patient had been lateral wrist and anterior aspect left lateral shin.
unable to stand for more than a few minutes Anal tone and saddle sensation are intact. MRI
at a time but was reasonably comfortable brain is normal and MRI spine (Sagittal T2
when lying down. The patient denied any + T1 with gad) shown below. CSF shows
focal neurological symptoms and was consti- WCC 12/mm3, RBC <1/mm3, Glucose
tutionally well. Clinical examination did not 4.5 mmol/dl, Protein 0.9 g/l, and negative for
demonstrate any focal neurological signs or oligoclonal bands. Which one of the following
features of meningism. CT brain: no evi- tests is likely to be positive?
dence of intra-axial or extra-axial bleeding;

Neurosurgery Books Full


www.ketabpezeshki.com 66485438-66485457
8 NEUROLOGY AND STROKE 131

had a similar episode 2 years ago which she


recovered from completely. On examination
she has weakness in wrist extension and finger
abduction in the left hand and visual acuity in
the left eye was measured at 6/24 with an asso-
ciated reduction in color saturation. Blood
tests were unremarkable. Her MRI scan is
shown (Axial T1 with contrast and FLAIR).
Which one of the following options should
be used in acute management?

a. Anti-acetylcholine receptor antibody


b. Anti-aquaporin 4 antibody
c. Anti-muscle specific kinase antibody
d. Anti-voltage gated calcium channel anti-
body
e. Anti-voltage gated potassium channel
antibody

14. A 10-year-old girl presents with subacute


mental status change and left arm weakness.
She had a viral illness 1 week ago. On exam-
ination she appears drowsy. She has a left
sided hemiparesis with bilateral nystagmus.
Fundoscopy reveals papilledema. There are
no skin rashes. MRI head FLAIR sequence
is shown. MRI spine showed a longitudinally
extensive transverse myelitis. Which one of
the following is most likely?

a. Commence high dose oral prednisone


and wean over a month
b. IV methylprednisolone
a. Multiple sclerosis c. Natalizumab infusion
b. Acute disseminating encephalomyelitis d. Interferon beta
c. Neurosarcoidosis e. Biopsy
d. Neuromyelitis optica
e. Systemic lupus erythematosis 16. A 43-year-old female presents with a second
f. Lyme disease episode of loss of sensation in her left anterior
thigh and right foot. This is her second episode
15. A 35-year-old female presents with three days within the past 4 months. She had recently
of increasing weakness in the right arm and reported an episode of left anterior shin numb-
reduced visual acuity in the left eye. She has ness 1 year ago, when an MRI with gadolinium
Neurosurgery Books Full
www.ketabpezeshki.com 66485438-66485457
132 PART II CARE OF THE NEUROSURGICAL PATIENT

demonstrated “spots in her spinal cord” and a. Cerebral abscess


she was diagnosed with transverse myelitis. b. High grade tumor
Her past medical history also includes ulcera- c. Metastasis
tive colitis, diagnosed aged 27 years old and pri- d. Primary CNS lymphoma
mary sclerosing cholangitis. Routine bloods e. Demyelination
are normal except for mild derangement of
liver function tests. Which one of the following 18. A 31-year-old female suffered multiple cuts
is most appropriate? and burns to both arms. On examination
there is marked wasting of brachioradialis
and the small muscles in both hands, with
reduced biceps and brachioradialis reflex.
She is weak in both arms, distally more than
proximally. Her lower limb and cranial nerve
examination is unremarkable. On testing
upper limb sensation, vibration and proprio-
ception are intact but there appears to be
reduced pain and temperature sensation over
the C3/C4/C5 dermatomes. Which one of
the following is most likely?
a. Chiari malformation
b. Chronic inflammatory demyelinating
polyneuropathy
c. Guillain-Barré syndrome
d. Miller Fisher syndrome
a. Interferon beta e. Multiple sclerosis
b. Glatiramer acetate
19. A 64-year-old man presents with sudden
c. Fingolimod
d. Natalizumab onset severe headache while watching televi-
e. Mitoxanthrone sion, followed by confusion and a tonic-
clonic seizure. Past medical history included
a 20 pack year smoking history, hyperten-
17. A 43-year-old female presents with a 2 week
history of mild left arm weakness and head- sion, hypercholesterolemia and myocardial
ache. MRI was done at presentation (shown). infarction two years ago requiring stenting.
On examination, GCS M5V4E3 but was pro-
She was discharged on dexamethasone 2 mg
twice daily due to her focal neurology with a tecting his own airway. Pupils were equal and
plan for awake craniotomy and resection. An reactive. The patient was spontaneously
moving all his limbs and had downgoing
image guidance scan is repeated one week later
but there is no longer any ring-enhancement. plantar reflexes. Cardiovascular, respiratory
Which one of the following is most likely? and abdominal examination was unremark-
able. Initial observations were blood pressure
220/115 mmHg, heart rate 89 beat/min, O2
sats (15 l O2) 100%, Respiratory rate 19/
min, temperature 37.1°C. CT brain is nor-
mal and lumbar puncture shows WCC 3/
mm3, RBC 3, protein 0.6 g/l, glucose
5.4 mmol/l, and no xanthochromia. MRI is
shown (FLAIR). Which one of the following
is most likely?

Neurosurgery Books Full


www.ketabpezeshki.com 66485438-66485457
8 NEUROLOGY AND STROKE 133

have weakness about the hip and shoulder


girdle. She has purplish-red discoloration of
the skin around her eyes, erythematous dis-
coloration over the finger joints and purplish
nodules over the elbows and knees. Which
one of the following is the most likely
diagnosis?
a. Becker muscular dystrophy
b. Dermatomyositis
c. Inclusion body myositis
d. Myotonic dystrophy
e. Polymyositis

23. A 67-year-old male is investigated for chest


pain and painful swallowing progressing over
the last few months with no response to pro-
ton pump inhibitors. There is no history of
weight loss or anorexia or smoking. On
examination you note a left-sided partial pto-
sis, and he reports diplopia on testing extro-
a. Acute disseminated encephalomyelitis cular muscle movements. Sustained upward
b. Herpes simplex virus encephalitis gaze exacerbates his ptosis. There is no limb
c. Multiple sclerosis muscle weakness or sensory disturbance.
d. Posterior circulation stroke CXR is shown. Which one of the following
e. Posterior reversible encephalopathy syn- tests is likely to be helpful?
drome

20. A 12-year-old boy with Lyme disease and


bilateral facial weakness is being treated with
a cephalosporin. The child’s facial strength
improves, but he notices twitching of the left
corner of his mouth whenever he blinks his
eye. This involuntary movement disorder is
probably an indication of which one of the
following?
a. Horner’s syndrome
b. Marcus Gunn phenomenon
c. Mononeuritis multiplex
d. Parinaud syndrome
e. Recurrent meningitis

21. A 25-year-old woman has progressive gait


disorder. The initial physical examination
reveals hepatosplenomegaly and left sided
ataxia and abnormal finger-nose test. Urinal- a. Anti-acteylcholine receptor antibodies
ysis reveals proteinuria and microscopic b. Anti-GM1 antibody
hematuria. Which one of the following find- c. Anti-GQ1b antibody
ings is most likely? d. Anti-muscle specific kinase antibody
a. Neurofibromas e. Anti-voltage gated calcium channel
b. Ash leaf spots antibody
c. Retinal telangiectasia
d. Kayser-Fleisher rings 24. A 65-year-old presents with a 3 month his-
e. Facial angiofibromas tory of progressive weakness. She had ini-
tially noticed difficulty opening jars, but
now also has difficulty walking up stairs.
22. A 62-year-old female has discomfort in her She denied any pain or sensory symptoms.
limbs and trouble getting off the toilet. She Past medical history included osteoporosis,
is unable to climb stairs and has noticed a rash type 2 diabetes mellitus and hypertension.
on her face. On examination, she is found to
Neurosurgery Books Full
www.ketabpezeshki.com 66485438-66485457
134 PART II CARE OF THE NEUROSURGICAL PATIENT

On neurological examination there were no she could not move at all for 2 h. She reports
fasciculations, tone was normal and sensation no loss of consciousness and was aware
was intact. Power was reduced in finger flex- throughout the episode. There is no other sig-
ion (3/5), wrist flexion (4/5), knee extension nificant past medical history or epilepsy. Rou-
(3/5), and hip flexion (4/5) bilaterally. Upper tine systemic and neurological examination is
limb reflexes were present but diminished, normal. A 12 lead ECG demonstrated a jerky
but the knee jerk was absent and there were baseline with flat T waves. What one of the fol-
flexor plantar responses bilaterally. There lowing is most likely?
was no tenderness over any muscle groups. a. Andersen-Tawil syndrome
Cranial nerve examination was unremark- b. Cataplexy
able. Blood results were normal except for c. Hyperkalemia periodic paralysis
CRP 10 mg/l, ESR 41 mm/h, CK 290 u/l. d. Hypokalemic period paralysis
Which one of the following is most likely? e. Night terror
a. Diabetic amyotrophy
b. Inclusion body myositis 28. A 10-year-old presents to your neurology
c. Polymyalgia rheumatica clinic reporting 9 months of subtle and gradual
d. Polymyositis onset, progressive lower limb weakness. For
e. Chronic inflammatory demyelinating the past 18 months, he has noticed a difficulty
polyneuropathy in keeping up with his peers in PE lessons,
which he initially put down to “not being very
25. A 77-year-old male presents with a 2-day his- sporty.” However, he feels weak whenever he
tory of right temporal throbbing headache. walks and has particular difficulty getting up
He has had migraines previously but never from a chair. His appearance is shown below.
this severe and usually occipital. There was Formal examination of power is 4/5 bilaterally
no other past medical history of note. On in shoulder abduction, adduction and normal
examination, his right scalp is tender and a 5/5 distally. 4/5 is also noted in hip flexion
prominent right temporal artery is noted. and extension, 4+/5 in knee flexion and exten-
He is apyrexic with no skin rashes. His blood sion, 5/5 in ankle plantar and dorsiflexion. The
tests are as follows: Hb 13.1 g/dl, Plt 450  10- weaknesses demonstrated are not fatiguable
9/l, WCC 11.5, ESR 85, Na 142, K 4.0., Urea and are persistent. Reflexes are present in all
10, Cr 118 umol/l, CRP 23 mg/l. Which one areas, plantars are downgoing. He has no other
of the following would you do next? past medical history. What is the likely
a. CT angiogram diagnosis?
b. Biopsy
c. Start prednisolone
d. Start azothiaprine
e. Carotid duplex ultrasound

26. A 65-year-old male has been diagnosed with


small cell lung cancer and is currently undergo-
ing chemotherapy. Over the last few months he
has noticed his vision deteriorating and com-
plains of diplopia. He also feels weaker in his
upper limbs although his symptoms do fluctu-
ate. On examination he has mild ptosis of
the eyelids bilaterally and a complex ophthal-
moparesis affecting both eyes. He also has
reduced power proximally in the upper limbs.
Which one of the following may be associated
with this clinical picture?
a. Anti-Ro antibody
b. Anti-voltage gated potassium channel
antibody
c. Anti-voltage gated calcium channel
antibody
d. Anti-Hu antibody
e. Anti-GQ1b antibody
27. A 17-year-old girl presents with a second epi-
sode on waking earlier in the morning where
Neurosurgery Books Full
www.ketabpezeshki.com 66485438-66485457
8 NEUROLOGY AND STROKE 135

a. Becker muscular dystrophy recognize people she had never met. On


b. Duchenne muscular dystrophy examination pupils were reactive, there
c. Emery-Dreifuss syndrome was an ophthalmoparesis, nystagmus on
d. Facial-scapulo humeral syndrome attempted horizontal gaze and ataxic gait.
e. Limb-girdle dystrophy Motor and sensory systems were normal.

53. Poisoning:
a. Aluminum
QUESTIONS 29–51 b. Arsenic
c. Carbon monoxide
Additional questions 29–51 available on d. Cyanide
ExpertConsult.com
e. Ergot
f. Lead
g. Manganese
h. Mercury
i. Organophosphates
EXTENDED MATCHING ITEM (EMI) j. Thallium
QUESTIONS
For each of the following descriptions, select the
52. Alcohol-related neurological disorders: most appropriate answers from the list above.
a. Alcoholic cerebellar degeneration Each answer may be used once, more than once,
b. Alcoholic hallucinosis or not at all.
c. Alcoholic neuropathy 1. A 34-year-old male working in a felt-
d. Alcohol withdrawal seizures processing plant develops tremors, memory
e. Beriberi disturbances and personality change over
f. Delerium tremens the course of months. On examination, he
g. Marchiafava-Bignami disease has prominent gait ataxia, limb and facial
h. Tobacco-alcohol amblyopia tremors, and decreased pain and tempera-
i. Wernicke’s encephalopathy ture sense in his feet.
j. Wernicke-Korsakoff syndrome 2. A 23-year-old volunteers abroad painting
houses during a 3-month exchange.
For each of the following descriptions, select the Towards the end of the trip he develops
most appropriate answers from the list above. weakness in both wrists. On examination,
Each answer may be used once, more than once, there is bilateral wrist drop without any sen-
or not at all. sory deficit. An EMG reveals evidence of a
1. A 55-year-old male known alcoholic is peripheral motor neuropathy.
found confused on a street by a policeman 3. A 45-year-old worker in an insecticide
not orientated in time or place. He is able factory complains of severe stomach pain.
to follow your commands in lifting his She also has had problems with her
upper and lower limbs during his neurolog- memory, excessive drowsiness, and a senso-
ical exams. All reflexes were present. He rimotor neuropathy with absent tendon
fails to follow your finger with his eyes on reflexes.
cranial nerve examination and you note 4. A 22-year-old farm worker has seizure.
horizontal nystagmus. His gait is grossly Neurological examination reveals fascicula-
ataxic. tions and occasional myoclonus. He is
2. A 45-year-old heavy drinker (30 units per ataxic and has absent deep tendon reflexes.
day) presents 72 h after his last drink with A sensory neuropathy is evident in his legs.
agitation, and pointing around the room Ulcers are evident on his fingers and toes.
as if having hallucinations. He has a coarse He says his diet was poor and mostly made
tremor, sinus tachycardia at 120 bpm and food from his rye crop.
sweating. 5. A 38-year-old miner develops a shuffling
3. A 60-year-old female is brought in by police gait, tremor, and drooling. His speech is
as she was wandering and confused. She was difficult to understand and becomes quieter
disorientated in place and time, did not as he talks. On examination, cogwheel
remember her birthday, was unable to recall rigidity is evident in his arms and legs.
three objects after 5 min and identified the His tremor is most evident when his limbs
hospital cleaner as her father and seemed to are at rest.

Neurosurgery Books Full


www.ketabpezeshki.com 66485438-66485457
136 PART II CARE OF THE NEUROSURGICAL PATIENT

54. Genetic syndromes with learning disability: 1. A 48-year-old male develops weakness of
a. Angelman’s syndrome his left wrist extensors and digits over one
b. Cri du Chat syndrome week, followed by involvement of his right
c. Down’s syndrome hand and foot over the subsequent 6 weeks.
d. Fragile X syndrome On examination, he has wasting and 2/5
e. Neurofibromatosis power in the left wrist and digit extensors.
f. Prader-Willi syndrome There is evidence of clawing of the right
g. Rett syndrome ring and little fingers along with wasting
h. Tuberous sclerosis of the small muscles of the right hand
i. Velocardiofacial syndrome (except the thenar eminence and the first
j. Williams syndrome two lumbricals). He has a right foot drop
along with wasting of the anterior tibial
For each of the following descriptions, select the and perineal muscles on that side. Fascicu-
most appropriate answers from the list above. lations are seen in all of the areas of weak-
Each answer may be used once, more than once, ness. Sensory examination and reflexes are
or not at all. normal, no clonus and a flexor plantar
1. A 15-year-old boy has moderate mental response.
retardation, attention deficit disorder, a 2. A 56-year-old female has a 6 month history
long face, enlarged ears, and macroorchid- of worsening numbness and paresthesias
ism. Development has been steady but distally in the hands and feet, as well as
always at a delayed pace. proximal muscle weakness. Bulbar muscles
2. An 11-year-old girl presents with obesity, are normal. An EMG shows multifocal con-
excessive and indiscriminate gorging, small duction block, slowing of nerve conduction,
hands, feet, hypogonadism and mental and minimal loss of amplitude of muscle
retardation. action potentials. CSF examination shows
3. A 7-year-old boy is noted to have character- an elevation in protein, but no increase in
istic elfin facies, short stature and cardiovas- the number of cells.
cular defects. 3. A 40-year-old male has a prophylactic dose
4. A 13-year-old boy has a history of thymus of phenytoin for 7 days after conservative
abnormalities, ear deformities, cleft palate, management of head injury. He presents
cardiac defects and short stature. He has with confusion, psychosis, abdominal pain,
annual blood tests to assess serum calcium. and vomiting. On examination, he is tachy-
5. A 5-year-old girl with progressive deterio- cardic, hypertensive, and febrile and appears
ration in cognitive function and loss of lan- delirious. His arms are weak and areflexic
guage displays stereotypic hand but sensation is relatively preserved.
movements. 4. A 25-year-old woman with a prior history of
visual loss in the left eye and a spastic gait
55. Neuropathy: develops impaired pain and temperature per-
a. Acute intermittent porphyria ception in her feet. She was diagnosed with
b. Charcot-Marie Tooth multiple sclerosis (MS) shortly after her
c. Chronic axonal neuropathy visual loss. Her left fundus reveals optic atro-
d. Chronic inflammatory demyelinating phy, and her facial movements are asymmet-
polyneuropathy ric. Chest X-ray reveals large hilar lymph
e. Diabetic neuropathy nodes. Mammogram reveals no apparent car-
f. Hereditary sensory and autonomic neu- cinoma. Serum ACE is positive.
ropathy 5. A 55-year-old female presents with 3 weeks
g. HIV neuropathy of bilateral tingling sensation in her medial
h. Mononeuritis multiplex one and half digits at night. She has noted a
i. Multifocal motor neuropathy with con- clawing of her 4th and 5th digits and she is
duction block particularly concerned by the cosmetic ele-
j. Paraneoplastic neuropathy ments. She also complains of a left sided
k. Paraproteinemic neuropathy foot drop present over the past 8 months.
l. Sarcoidosis She has also had multiple admissions
m. Vasculitis for surgery to her feet at childhood but
she is unaware of further details. On exam-
For each of the following descriptions, select the ination, she clinically has a left common
most appropriate answers from the list above. peroneal palsy with bilateral thin calves,
Each answer may be used once, more than once, and loss of sensation in bilateral ulnar nerve
or not at all. territories.

Neurosurgery Books Full


www.ketabpezeshki.com 66485438-66485457
8 NEUROLOGY AND STROKE 137

6. A 29-year-old female with Type 1 diabetes e. Episodic ataxia type 2


mellitus presents with weakness in several f. Fragile X tremor ataxia syndrome
muscles in different limbs. The pattern is g. Friedreich’s ataxia
lower motor neuron and does not fit with h. Machado-Joseph disease (SCA3)
any particular peripheral, plexus, or root i. Mitochondrial disorders
localization. j. Spinocerebellar ataxias (SCA1)

56. Paraneoplastic disorders: For each of the following descriptions, select the
a. Dorsal root ganglionopathy most appropriate answers from the list above.
b. Guillain-Barré syndrome Each answer may be used once, more than once,
c. Hypercalcemia or not at all.
d. Lambert-Eaton myasthenic syndrome 1. A 3-year-old boy presents with gait ataxia,
e. Limbic encephalitis choreoathetoid movements in his right
f. Motor neuron disease hand and recurrent ear infections or epi-
g. Myasthenia gravis sodes of unexplained fever. On examination
h. Opsoclonus-myoclonus he had dilated venules on his ear, mild dys-
i. Paraneoplastic cerebellar degeneration arthria and a wide based ataxic gait. Deep
j. Paraproteinemic neuropathy tendon reflexes were absent.
k. Stiff man syndrome 2. A 24-year-old presents with progressive
unsteadiness on walking over the past
For each of the following descriptions, select the 6 months. Over the past 3 months, he
most appropriate answers from the list above. has noticed a lack of articulation with
Each answer may be used once, more than once, his speech. On examination, his cardio-
or not at all. vascular, respiratory and abdominal sys-
1. A 67-year-old female has a 2 month history tems are normal. His finger-nose test
of progressive gait disturbance. On exami- is impaired bilaterally and he is unable
nation, she has dysmetria of the limbs, a to tandem walk. He denies any neck stiff-
wide-based, unsteady gait; and hypermetric ness or headache. He has a full range of
saccades. A hard, firm breast lump is eye movements. He has absent reflexes in
discovered. his lower limbs and upgoing plantars
2. A 70-year-old male with a history of lung bilaterally.
cancer develops nausea and vomiting and 3. A 36-year-old male presents with a 5-year
then becomes lethargic. On examination, history of increasing, progressive “clumsi-
he is lethargic but arousable, disoriented, ness.” He cannot write legibly or even
and inattentive. He is weak proximally hold a key still using either hand to open
and has diminished reflexes. a door. On examination, his cranial nerves
3. A 57-year-old female with a history of were unremarkable except for mild multi-
smoking has a 3-month history of hip and directional nystagmus at primary gaze.
shoulder weakness. She also complains of Fundoscopy was normal. Limb examina-
xerostomia. There are no sensory symp- tion revealed significant impairment of
toms, and she is cognitively intact. On finger-nose and heel-shin testing. His
examination, she is orthostatic. There is gait, tone, power, sensation and reflexes
proximal muscle weakness, but she has were normal with downgoing plantars.
increasing muscle strength with repetitive A brief mini-mental state examination
activity of her muscles. Eye movements scored 30/30. Serum ANA was negative.
are normal. 4. At age 5, a child is noted to have the loss of
4. A 65-year-old female develops pain and ankle jerks. At age 10, limb ataxia develops,
paresthesias in her feet. On examination, followed by a peripheral neuropathy. Dur-
she has stocking distribution sensory loss, ing adolescence, retinitis pigmentosa
and mild distal weakness with areflexia. develops. Peripheral blood smear shows
Serum protein electrophoresis reveals a acanthocytosis.
monoclonal gammopathy, and bone mar-
row biopsy reveals plasma cell dyscrasia. 58. Nutritional deficiency:
a. Folate
57. Hereditary ataxia: b. Niacin
a. Abetalipoproteinemia c. Pyridoxine (vitamin B6) deficiency
b. Ataxia telangiectasia d. Riboflavin
c. Ataxia with isolated vitamin E deficiency e. Thiamine (vitamin B1) deficiency
d. Episodic ataxia type 1 f. Vitamin A deficiency

Neurosurgery Books Full


www.ketabpezeshki.com 66485438-66485457
138 PART II CARE OF THE NEUROSURGICAL PATIENT

g. Vitamin B12 deficiency For each of the following descriptions, select the
h. Vitamin C deficiency most appropriate answers from the list above.
i. Vitamin E deficiency Each answer may be used once, more than once,
j. Vitamin D deficiency or not at all.
k. Vitamin K 1. Two brothers, 4 and 6 years of age, but not
their 9-year-old sister, exhibit limb ataxia,
For each of the following descriptions, select the nystagmus, and learning disability, and have
most appropriate answers from the list above. abnormally low serum cortisol levels.
Each answer may be used once, more than once, 2. A 3-month-old boy exhibits nystagmus and
or not at all. limb tremors unassociated with seizures.
1. A 36-year-old man with tuberculosis is Over the next few years, he develops optic
started on therapy with isoniazid, rifampin, atrophy, choreoathetotic limb movements,
and ethambutol. After 2 months his liver en- seizures, and gait ataxia. He dies during sta-
zymes are slightly deranged and he reports tus epilepticus and at autopsy is found to
pins-and-needles sensations in his feet. have widespread myelin breakdown with
Neurological examination reveals preserved myelin preservation in islands about the
power, but hypoactive deep tendon reflexes blood vessels. The pathologist diagnoses a
in the legs and impaired position sense. sudanophilic leukodystrophy to describe
2. A 47-year-old known alcoholic female is the pattern of staining observed on slides
found wandering and brought to the emer- prepared to look for myelin breakdown
gency room. She is disoriented to time, products.
place, and person, but has no external evi- 3. A 17-month-old boy had developed nor-
dence of head trauma. Examination reveals mally until approximately 13 months of
ataxic gait, paresis of conjugate gaze, age, when he began having progressive gait
and horizontal nystagmus. She does not problems. On examination, the patient is
have any ethanol in her bloodstream on spastic, yet nerve conduction studies
testing. (NCS) reveal slowed motor and sensory
3. A 59-year-old man developed progressive conduction velocities. Cerebrospinal fluid
cramping of his legs, gait unsteadiness and (CSF) protein is elevated. MRI reveals
paresthesia affecting his hands and feet over white matter abnormalities. Leukocyte test-
1 year. He has also had some episodes of ing reveals deficient arylsulfatase A activity.
urinary incontinence. On examination, he 4. A 6-month-old child has a rapid regression
has a spastic paraparesis with severe distur- of psychomotor function and loss of sight.
bance of position and vibration sense in his There is increased urinary excretion of N-
legs, absent knee and ankle reflexes. Blood acetyl-L-aspartic acid.
results show a megaloblastic anemia.
4. A 5-year-old boy develops progressive gait 60. Sphingolipidosis:
ataxia and limb weakness over the course a. Batten’s disease
of 3 months. Examination reveals diffusely b. Cerebrotendinous xanthomatosis
absent deep tendon reflexes, proximal mus- c. Fabry disease
cle weakness, ophthalmoparesis, and poor d. Gaucher’s disease
pain perception in the feet. Blood tests e. GM1 gangliosidosis
reveal elevated creatine kinase levels and f. Krabbe disease
evidence of liver disease without features g. Metachromatic leukodystrophy
of liver failure. h. Neimann-Pick disease
i. Sandhoff’s disease
59. Leukodystrophies: j. Tay-Sachs disease
a. Acute disseminated encephalomyelitis k. Wolman disease
b. Alexander disease
c. CADASIL For each of the following descriptions, select the
d. Canavan disease most appropriate answers from the list above.
e. Cerebrotendinous xanthomatosis Each answer may be used once, more than once,
f. Krabbe Disease or not at all.
g. Metachromatic leukodystrophy 1. A 12-month-old develops progressive
h. Pelizaeus-Merzbacher disease blindness and delayed cognitive milestones.
i. Refsum disease Fundal examination shows a cherry
j. X-linked Adrenoleukodystrophy red spot. There is a deficiency of
k. Zellweger syndrome hexaminosidase A.

Neurosurgery Books Full


www.ketabpezeshki.com 66485438-66485457
8 NEUROLOGY AND STROKE 139

2. An 8-month-old boy develops spasticity, 3. A 7-year-old child presents with severe skele-
head retraction, and difficulty swallowing. tal dysplasia, short stature. Test shows abnor-
There is abnormal accumulation of gluco- mal accumulation of keratan sulfate secondary
cerebroside and the child will deteriorate to deficiency of galactose-6-sulfate sulfatase.
and die within 3 years.
63. Mitochondrial disorders:
61. Antibodies: a. Alpers-Huttenlocher syndrome
a. Anti-muscle specific kinase b. Ataxia neuropathy syndromes
b. Anti-NMDA receptor antibody c. Chronic progressive external ophthalmo-
c. Anti-voltage gated calcium channel plegia
antibody d. Kearns-Sayre syndrome
d. Anti-Aquaporin 4 antibody e. Leber’s hereditary optic neuropathy
e. Anti-Hu (ANNA-1) f. Leigh syndrome
f. Anti-Yo (PCA-1) g. MELAS
g. Anti-Ri (ANNA-2) h. MEMSA
h. Anti-Tr i. MERRF
i. Anti-GAD j. NARP
j. Anti-voltage gated potassium channel
antibody For each of the following descriptions, select the
k. Anti-TA Ma2 most appropriate answers from the list above.
l. Anti-GQ1b Each answer may be used once, more than once,
m. Anti-myelin associated glycoprotein/sul- or not at all.
fated glucoronul paragloboside 1. A 19-year-old male presents with diplopia,
n. Anti-GD1b ataxia, and heart block.
2. A 24-year-old male presents with reduced
For each of the following descriptions, select the vision in both eyes over several weeks,
most appropriate answers from the list above. and has developed “spasms” of his left hand
Each answer may be used once, more than once, and complains of palpitations. ECG is sug-
or not at all. gestive of ventricular pre-excitation.
1. Non-paraneoplastic limbic encephalitis 3. A 34-year-old female with a history of
2. Neuromyelitis optica (Devic’s disease) migraines present’s with stroke-like epi-
3. Guillain-Barré syndrome sodes affecting her left arm and leg, lactic
4. Myasthenia gravis acidosis, and cognitive impairment.
5. Lambert-Eaton myasthenic syndrome
64. Dementia:
62. Mucopolysaccharidoses: a. Alzheimer’s disease
a. Hunter syndrome b. B12 deficiency
b. Hurler syndrome c. Cortical Lewy body disease
c. Sanfilippo syndrome A d. Corticobasal degeneration
d. Sanfilippo syndrome B e. Creutzfeldt-Jacob disease
e. Sanfilippo syndrome C f. Depression
f. Sanfilippo syndrome D g. Frontotemporal dementia
g. Maroteaux-Lamy syndrome h. HIV dementia complex
h. Morquio syndrome A i. Huntington’s disease
i. Morquio syndrome B j. Hypothyroidism
j. Natowicz syndrome k. Neurosyphilis
k. Sly syndrome l. Normal pressure hydrocephalus
m. Transient global amnesia
For each of the following descriptions, select the n. Vascular dementia
most appropriate answers from the list above.
Each answer may be used once, more than once, For each of the following descriptions, select the
or not at all. most appropriate answers from the list above.
1. A 4-year-old is found to have dwarfism, Each answer may be used once, more than once,
mental retardation, and clouding of his cor- or not at all.
neas. Tests show an α-L-iduronidase 1. A 73-year-old man steps out of the shower on
deficiency. a Saturday evening and is unable to remember
2. A 6-year-old child is diagnosed with X- that he and his wife have tickets to a play. He
linked recessive mild learning difficulty asks her repeatedly, “Where are we going”?
due to iduronate sulfatase deficiency. He appears bewildered, but is alert, knows

Neurosurgery Books Full


www.ketabpezeshki.com 66485438-66485457
140 PART II CARE OF THE NEUROSURGICAL PATIENT

his own name, speaks fluently, and has no normal. Routine laboratory work up was
motor deficits. He has no history of memory normal. Echocardiography was normal.
disturbance and after 8 h returns to normal. 2. A 51-year-old female presents to the ER
2. A 50-year-old woman began having double complaining of a sudden syncopal episode
vision and blurry vision 3 months ago and while climbing up the stairs. Her PE
has since had diminishing interaction with showed that she is in respiratory distress,
her family, a paucity of thought and expres- BMI of 40, hemodynamically stable, cardiac
sion, and unsteadiness of gait. Her whole examination showed normal. Routine
body appears to jump in the presence of a laboratory work up was normal. Echocardi-
loud noise. MRI is normal but CSF is pos- ography was normal apart from mild MR
itive for 14-3-3 protein. and mild TR.
3. A 17-year-old girl develops mild dementia, 3. A 72-year-old with a 10-year history of
tremor, and rigidity. Her father died in his Parkinson’s disease presents with recurrent
fourth decade of life of a progressive syncopal episodes on standing up from a
dementing illness associated with jerking sitting position.
(choreiform) limb movements. On expo-
sure to L-dopa, she becomes acutely agi- 66. Headache:
tated and has jerking limb movements. a. Analgesic overuse headache
4. A 62-year-old man has had 2 years of pro- b. Atypical facial pain
gressive memory loss and inappropriate c. Cluster headache
behavior. He has been delusional. More d. Hypnic headache
recently, he has developed tremors, myoc- e. Migraine with aura
lonus, dysarthria, and unsteadiness of gait. f. Migraine without aura
The CSF shows a lymphocytic pleocytosis, g. Opthalmoplegic migraine
protein of 150, and positive VDRL. h. Paroxysmal hemicranias
5. A 44-year-old woman presents with inat- i. Postherpetic neuralgia
tentiveness, poor concentration, weight j. Retinal migraine
gain and lethargy. She has paranoid delu- k. Sinusitis
sions. There is mild proximal weakness l. Short-lasting unilateral neuralgiform
and ataxia. On general examination, she headache with conjunctival injection
has edema, coarse and pale skin, macroglos- and tearing (SUNCT)
sia and delayed relaxation of the ankle m. Tension headache
reflexes. n. Trigeminal neuralgia
o. Vertebrobasilar migraine
65. Syncope:
a. Autonomic failure For each of the following descriptions, select the
b. Cardiac arrhythmia most appropriate answers from the list above.
c. Carotid sinus syncope Each answer may be used once, more than once,
d. Cerebrovascular steal or not at all.
e. Dehydration 1. A 22-year-old complains of regular, right-
f. Drug-induced orthostatic syncope sided throbbing headaches. Changes in her
g. Epileptic seizure vision that precede the headache by 20 min
h. Hypoglycemia include scintillating lights just to the left of
i. Neurally mediated (vasovagal) syncope her center of vision progressing to a blind
j. Non-epileptic attack disorder spot which then clears before the headache
k. Situational syncope starts. It rarely lasts more than 1 h, but is usu-
l. Structural cardiac disease ally accompanied by nausea and vomiting.
2. A 35-year-old man has severe throbbing
For each of the following descriptions, select the pain waking him from sleep at night and
most appropriate answers from the list above. persisting into the day. This pain is usually
Each answer may be used once, more than once, centered about his left eye and appears on a
or not at all. nearly daily basis for several weeks or
1. A 36-year-old female complianed of a syn- months each year. He becomes combative
copal episode. She described dizziness and and agitated during the onset, but never
fatigue on her way to her bedroom, shortly vomits or develops focal weakness.
after she passed out. She regained con- 3. An 81-year-old man with chronic lympho-
sciousness after a few minutes, feeling tired cytic leukemia develops pain and burning
and soaked in her sweat. Her PE was over the right side of his face. Within a
normal. Her resting ECG was within few days, a vesiculopapular rash in the

Neurosurgery Books Full


www.ketabpezeshki.com 66485438-66485457
8 NEUROLOGY AND STROKE 141

distribution of the first division of the tri- The headaches resolve quickly when he lies
geminal nerve appears. The vesicles down and are accompanied by mild nausea.
become encrusted, and the burning associ- His examination is normal.
ated with the rash abates. Within 1 month
the rash has largely resolved, but the man 68. Disorders of language:
is left with a dull ache over the area of the a. Akinetic mutism
rash that is periodically punctuated by b. Anomic aphasia
shooting pains. c. Broca’s aphasia
d. Cerebellar mutism
67. Headache: e. Conduction aphasia
a. Analgesic rebound headache f. Global aphasia
b. Aseptic meningitis g. Mixed transcortical aphasia
c. Carotid artery dissection h. Transcortical motor aphasia
d. Glioblastoma multiforme i. Transcortical sensory aphasia (fluent)
e. Idiopathic intracranial hypertension j. Wernicke’s aphasia
f. Paroxysmal hemicranias
g. Post-traumatic headache For each of the following descriptions, select the
h. Raeder syndrome most appropriate answers from the list above.
i. Spontaneous intracranial hypotension Each answer may be used once, more than once,
j. Thunderclap headache or not at all.
1. A 45-year-old woman with chronic atrial
For each of the following descriptions, select the fibrillation discontinues warfarin treatment
most appropriate answers from the list above. and abruptly develops problems with lan-
Each answer may be used once, more than once, guage comprehension. She is able to pro-
or not at all. duce some intelligible phrases and
1. An obese 37-year-old woman has had a produces sound quite fluently; however,
daily headache, worse in the morning, for she is unable to follow simple instructions
1 year. She has episodes of transient visual or to repeat simple phrases. On attempting
obscurations affecting each eye and also to write, she becomes very frustrated and
hears a pulsatile tinnitus. Examination is agitated. Emergency MRI reveals a lesion
notable for bilateral papilledema. MRI is of the left temporal lobe that extends into
normal. the superior temporal gyrus.
2. A 42-year-old man presents with a sudden 2. A 62-year-old man has had a left hemi-
and severe headache associated with nausea sphere stroke. He has impaired naming
during coitus. The headache reaches maxi- and repetition. His speech is nonfluent.
mal intensity within 5 s. He has no prior Comprehension is preserved.
history of headache. Examination is unre- 3. A 28-year-old woman is hit in the left neck
markable. CT head is normal and CSF while playing lacrosse. Approximately 2 h
shows a traumatic tap and cannot exclude later she begins having language difficulties.
subarachnoid hemorrhage. A good quality Her speech is fluent and nonsensical.
CT angiogram is negative for aneurysm, She cannot understand commands, but
dissection or AVM. repeats well.
3. A 29-year-old man relates that he has had
recent headaches only when standing up.

SBA ANSWER
1. b—Alzheimer’s disease and most cases are sporadic; 5% of cases are inher-
ited as an autosomal dominant trait mutations in
Alzheimer's dementia can present as self-neglect the amyloid precursor protein (chromosome 21),
and weight loss, especially when the patient is liv- presenilin 1 (chromosome 14), and presenilin 2
ing alone. Paranoid ideation is also quite common (chromosome 1) genes are thought to cause the
and may be used by the patient as an explanation inherited form. Risk of Alzheimer’s disease is
for symptoms of memory loss (e.g. misplacing increased in those with the apolipoprotein E allele
items), as is physical aggression whereas auditory E4 (present in 20% of population) is 15 times
and visual hallucinations are less common. Alzhei- higher than those with two E3 alleles. Mild AD
mer’s disease is the commonest cause of dementia, is characterized by minor behavioral changes, loss

Neurosurgery Books Full


www.ketabpezeshki.com 66485438-66485457
142 PART II CARE OF THE NEUROSURGICAL PATIENT

of memory of recent events (e.g. conversations, localized to frontal and temporal lobes only.
events), misplace items, struggle to find the right Microscopic findings include Pick bodies,
word in conversation, confused or lose track of gliosis, neurofibrillary tangles, and senile
day/date, difficulty planning and making decisions, plaques.
visuospatial impairment, and lose interest in peo- 2. Progressive non-fluent aphasia (chronic
ple or activities. Moderate AD will need reminders progressive aphasia). Patients have non-
about self-care, increasingly forgetful, not recog- fluent speech, they make short utterances
nize people, place themselves/others at risk (e.g. that are agrammatic but comprehension is
miss medication, leave gas stove on), easily relatively preserved.
upset/angry/aggressive, night-day reversal, agita- 3. Semantic dementia: Here the patient has a
tion, socially inappropriate, delusions/hallucina- progressive fluent aphasia but speech lacks
tions. Severe AD is characterized by increasing content and conveys little meaning. Unlike
dependence on others for nursing care, bed/ in Alzheimer's memory is better for recent
wheelchair bound, weakness, unable to recognize rather than remote events.
familiar objects/people, incontinence, difficulty
eating/swallowing and gradual loss of speech. Image with permission from Hinds SR II, Stocker DJ,
Bradley YC. Role of positron emission tomography/
Death is usually 8-10 years after symptom onset. computed tomography in dementia, Radiol Clin North
Pathological changes include widespread cerebral Am. 2013;51(5):927-34.
atrophy, particularly involving the cortex and hip-
pocampus. In AD, FDG-PET can show hypome- 3. d—Lewy body dementia
tabolism in the temporoparietal regions and/or the
posterior cingulum. On microscopy there are cor- Lewy body dementia is an increasingly recog-
tical plaques due to deposition of type A-Beta- nized cause of dementia, accounting for up to
amyloid protein and intraneuronal neurofibrillary 20% of cases. The characteristic pathological fea-
tangles caused by abnormal aggregation of the tau ture is alpha-synuclein cytoplasmic inclusions
protein (excessive phosphorylation). There is also (Lewy bodies) in the substantia nigra, paralimbic,
reduced acetylcholine due to damage to ascending and neocortical areas. The relationship between
forebrain projection, hence acetylcholinesterase Parkinson's disease and Lewy body dementia is
inhibitors (donepezil, galantamine, and rivastig- complicated, particularly as dementia is often
mine) as options for managing mild to moderate seen in Parkinson's disease. Also, up to 40% of
Alzheimer's disease. Memantine (a NMDA recep- patients with Alzheimer's have Lewy bodies.
tor antagonist) is reserved for patients with moder- Neuroleptics should be avoided in Lewy body
ate—severe Alzheimer's disease. dementia as patients are extremely sensitive and
may develop irreversible Parkinsonism. Features
Image with permission from Loevner L. Brain Imaging: include progressive cognitive impairment, Par-
Case Review Series, 2nd ed. Mosby: Elsevier, 2009.
kinsonism, and visual hallucinations (other fea-
tures such as delusions and non-visual
2. c—Pick’s disease
hallucinations may also be seen). Two out of
Frontotemporal lobar degeneration (FTLD) is three are needed for diagnosis. The visual hallu-
the third most common type of cortical dementia cinations are often very vivid. He also has a few
after Alzheimer's and Lewy body dementia. supportive features of Lewy body dementia hallu-
Common features of frontotemporal lobar cinations in other modalities, delusions, depres-
dementias include: Onset <65 years, insidious sion and repeated falls. Diagnosis is usually
onset, relatively preserved memory and visuospa- clinical, but SPECT is increasingly used. It is cur-
tial skills, personality change and social conduct rently commercially known as a DaT scan. Dopa-
problems. CT shows cortical loss in the frontal minergic iodine-123-I FP-CIT is used as the
and temporal lobes, and FDG-PET/CT shows radioisotope. The sensitivity of SPECT in diag-
hypometabolism. There are three recognized nosing Lewy body dementia is around 90% with
types of FTLD: a specificity of 100%. Currently, evidence best
1. Frontotemporal dementia (Pick's disease). supports cholinesterase inhibitors in the treating
Most common type and is characterized of Lewy body dementia. It must be remembered
by personality change and impaired social that these patients have high sensitivity to neuro-
conduct. Other common features include leptics so Olanzapine should not be used here.
hyperorality, disinhibition, increased appe-
4. b—Creutzfeldt-Jakob disease
tite, and perseveration behaviors. Focal
gyral atrophy (“knife-blade” atrophy) is
Creutzfeldt-Jakob disease is a rapidly progressive
characteristic of Pick's disease and is
spongiform encephalopathy due to accumulation

Neurosurgery Books Full


www.ketabpezeshki.com 66485438-66485457
8 NEUROLOGY AND STROKE 143

of prion proteins resistant to proteases. Sporadic point towards a diagnosis of motor neuron disease
CJD accounts for 85% of cases whereas 10-15% include absence of sensory signs/symptoms, both
of cases are familial. The mean age of onset is UMN and LMN symptoms, no cerebellar signs,
65 years, except for new variant CJD which no ocular signs and abdominal reflexes are usually
affects younger patients (mean age 25 years). Fea- preserved and sphincter dysfunction is a late fea-
tures include dementia, myoclonic jerks (often ture. MRI is usually performed to exclude the dif-
stimulus-sensitive), startle response, and less ferential diagnosis of cervical cord compression or
commonly extrapyramidal signs. New variant cranial lesion, and shows T2 hyperintensity (better
CJD usually has psychological symptoms such seen on FLAIR) along the length of the corticosp-
as anxiety, withdrawal and dysphonia. MRI shows inal tract. The diagnosis of motor neuron disease is
high signal on the cortical sulci surfaces (ribbon- clinical, but EMG and nerve conduction studies
ing) and increased signal in putamen and caudate will show normal sensory conduction with abnor-
head. EEG shows periodic spikes with sharp mal spontaneous (fasciculation) and evoked muscle
waves in sporadic CJD. CSF profile is usually potentials. Riluzole is the only drug that has been
normal but positive for 14-3-3 protein. proven to demonstrate a disease modifying effect
in motor neurone disease, increasing survival from
Image with permission from Naidich T, Castillo M, Cha diagnosis from 12 to 15 months and should be
S, Smirniotopoulos J. Imaging of the Brain, Saunders: started; therapies to reduce oxidative stress such
Elsevier, 2013.
as addition of vitamin E and N-acetylcysteine
5. b—Treat cardiovascular risk factors (NAC) are not recommended. Non-invasive ven-
tilation is the only other therapy that seems to pro-
Vascular dementia is one of the most common long life expectancy but only if the patient can
causes of dementia after Alzheimer's disease, caus- tolerate greater than 4 h of NIV per day and does
ing around 15% of cases. The history and findings not have severe bulbar dysfunction. NIV is recom-
are most suggestive of a vascular dementia caused mended when the patient has developed signs of
by multiple strokes, hence management of stroke respiratory distress, type 2 respiratory failure,
risk factors is the primary option as there is no FVC < 50% or the patient has reported orthop-
licensed treatment for it. Targeted treatment is nea/nocturnal hypoventilation. However, patients
made more difficult as there are multiple subtypes with severe bulbar palsy or cognitive impairment
of vascular dementia depending on profile of are excluded. Tracheostomy and long term inva-
ischemia: multi-infarct (cortical) dementia, small sive mechanical ventilation have been used in
vessel (subcortical white matter) dementia, hypo- selected cases with respiratory deterioration
perfusion dementia (watershed infarcts), hemor- despite being largely neurologically intact. Prog-
rhagic dementia, CADASIL, and mixed vascular- nosis poor: 50% of patients die within 3 years
Alzheimer’s disease type.
7. d—Survival motor neuron gene testing
6. c—Amyotrophic lateral sclerosis
Spinal muscular atrophy is a congenital lower
Motor neuron diseases (MND) result in progres- motor neuron disorder manifesting as progressive,
sive degeneration of upper motor neurons (Betz symmetric proximal muscular weakness occurring
cells) and/or lower motor neurons (anterior horn in 1 in 6000 to 1 in 10,000 births (second most
cells). Genetic studies have implicated Cu/Zn common autosomal recessive disease in humans
superoxide dismutase-1 (SOD1) gene in sporadic after cystic fibrosis). It is the leading inherited
cases. They rarely presents before 40 years of cause of infant death. Spinal muscular atrophy is
age. Various patterns/subtypes are recognized classified clinically by the age at symptom onset
including: amyotrophic lateral sclerosis (Lou Geh- and disease severity into type I (Werdnig-
rig disease), progressive bulbar palsy (bulbar onset Hoffman disease, acute), type II (intermediate
ALS), primary lateral sclerosis, spinal muscular form, usually 7-18 months old and can sit unsup-
atrophy, X-linked spinobulbar muscular atrophy ported but can’t walk independently), type III
(Kennedy disease) and hereditary spastic parapar- (Kugelberg-Welander disease, mildest form, pre-
esis. ALS is the most common form and may sents >18 months and able to achieve independent
present with limb symptoms (tripping, foot drop, walking), and type IV SMA (adult-onset). Spinal
wasting of the small hand muscles, wrist drop) in muscular atrophy is inherited in autosomal reces-
75% or bulbar symptoms (slurred speech, hoarse- sive fashion or is sporadic. Mutations or deletions
ness, decreased volume of speech, aspirating/chok- in the telomeric SMN (survival of motor neuron)
ing on meals) in 25% of cases. Progression of gene occur in most patients. The loss of functional
disease results in muscle atrophy, fasciculations, SMN protein results in premature neuronal cell
spasticity, muscle cramps, voice changes, dyspha- death. The SMN protein has a role in cardiac
gia, dysarthria, and drooling. Other clues which development. If the history and physical

Neurosurgery Books Full


www.ketabpezeshki.com 66485438-66485457
144 PART II CARE OF THE NEUROSURGICAL PATIENT

examination suggest spinal muscular atrophy, a dexamethasone due to raised ICP or focal deficits
positive DNA test for deletion of the survival which (i) further reduces the chance of diagnostic
motor neuron gene eliminates the need for elec- LP and (ii) may cause the lesion to “disappear” and
trophysiological testing and muscle biopsy. How- prevent accurate brain biopsy. In those AIDS
ever, the SMN gene is deleted only in 96% of patients in whom diagnostic uncertainty between
patients, serum creatine kinase activity may be 1 cerebral toxoplasma and PCNSL remains after
to 2 times normal. Electromyography reveals large non-diagnostic CSF, brain biopsy should be
motor units; nerve conduction velocities and sen- considered in the context of negative serological
sory conduction times are normal, ruling out screening for toxoplasma and thallium-enhanced
motor neuropathies. Muscle biopsy reveals group SPECT scan results (negative in toxoplasma, pos-
atrophy of type 1 and type 2 muscle fibers as itive in PCNSL). Treatment for toxoplasma
opposed to the normal checkerboard pattern. In includes sulfadiazine and pyrimethamine, whereas
the most severe cases (Type I), children never gain it is methotrexate chemotherapy for PCNSL (or
the ability to sit unsupported and severe respira- radiotherapy as second-line therapy).
tory problems mean children rarely survive
beyond two years of age. Type II SMA may Image with permission from Tang YZ, Booth TC, Bhogal
P, et al. Imaging of primary central nervous system
shorten life expectancy, but improvements in care lymphoma, Clin Radiol 2011;66(8):768-77.
standards mean the majority of people can live
long, fulfilling, and productive lives. Survival into 9. d—Progressive multifocal leukoencephalo-
adulthood is now expected. Life expectancy is usu- pathy
ally unaffected in Types III and IV.
Adrenoleukodystrophy, MS, SSPE, are all demye-
8. c—Methotrexate linating diseases, but PML is the only one linked
to JC virus in the context of immunocompromised
Common focal cerebral lesions in HIV patients are patients (e.g. AIDS, post-transplantation). General-
toxoplasmosis (50%), primary CNS lymphoma ized CNS disorders in patients with HIV include
(30%), and less commonly cerebral tuberculosis. viral encephalitis, Cryptococcus meningitis, PML
Typically, PCNSL in immunocompetent individ- and AIDS dementia complex. Progressivemultifocal
uals (whether HIV positive or not) will appear as a leukoencephalopathy (PML) results in widespread
single homogenously enhancing lesion, or spread demyelination due to infection of oligodendrocytes
across the corpus callosum (butterfly pattern) by JC virus (a polyoma DNA virus; papovavirus)
and has a dramatic response to dexamethasone resulting in subacute onset of behavioral changes,
treatment hence is easier to differentiate from speech, motor, and visual impairment. On CT this
infection. In immunocompromised patients, how- may appear as single or multiple lesions, no mass
ever, imaging appearances of PCNSL are more effect, no enhancement but MRI clearly shows wide-
variable—with smaller lesions and faster growth spread high T2 and FLAIR signal. Encephalitis may
outstripping blood supply leading to necrosis be due to CMV or HIV itself rather than HSV.
(ring-enhancing lesions) and making them chal- Cryptococcus is the most common fungal infection
lenging to differentiate from other ring-enhancing of CNS and presents with meningism, seizures, and
lesions seen in immunocompromised individuals focal neurological deficit with raised CSF pressure
such as toxoplasma (multiple lesions) and tubercu- on LP and positive India Ink staining. AIDS demen-
losis (usually only single abscess). Several limita- tia complex is caused by HIV itself (i.e. HIV enceph-
tions of diagnostic testing also complicate alopathy/encephalitis) and correlates with high viral
matters and are worthy of note. Firstly, even when loads and the duration of the infection. With the use
primary CNS lymphoma presents with its classical, of HAART, a milder form of cognitive dysfunction,
homogeneously enhancing imaging appearance a minor cognitive motor disorder (MCMD) has
cytological diagnosis is still required before treat- become common. MCMD accounts for approxi-
ment with methotrexate can start. In this scenario, mately 30% of patients with HIV infection, while
if there is little intracranial mass effect lumbar HIV-associated dementia accounts for less than
puncture can be performed and CSF cytology, 10%. Imaging findings include widespread cortical
flow cytometry, PCR for immunoglobin clonal atrophy, ventricular enlargement and white matter
gene rearrangements (to establish monoclonality) damage.
and EBV PCR (80% positive in AIDS-related
PCNSL). Despite this, CSF is often non- Image with permission from Tan CS, Koralnik IJ. Pro-
diagnostic and serial samples may be required or gressive multifocal leukoencephalopathy and other dis-
alternatively brain biopsy which is the gold stan- orders caused by JC virus: clinical features and
dard. Additionally, many patients receive pathogenesis. Lancet Neurol 2010;9:430.

Neurosurgery Books Full


www.ketabpezeshki.com 66485438-66485457
8 NEUROLOGY AND STROKE 145

10. d—Inhaled 100% oxygen puncture headache (PLPH), a common compli-


cation of a lumbar puncture (30%) and is
This patient describes features of cluster head- thought to be caused by excess leakage of cere-
ache, which is most likely to be terminated with brovascular fluid causing a relatively low intra-
inhalation of pure oxygen within minutes. Cluster cranial pressure. Risk factors include: factors
headaches usually occur at night when the patient which may contribute to headache; increased
is asleep, and so practical access to the oxygen needle size; direction of bevel; not replacing
tank is possible. Propranolol is a β-adrenergic- the stylet; increased number of LP attempts;
blocking agent that is useful in the prophylaxis factors that do not contribute to headache;
of some vascular headaches, but it is of no value increased volume of CSF removed; bed rest fol-
in aborting a cluster headache. Dihydroergota- lowing procedure; increased fluid intake post
mine suppositories may abort some vascular procedure; opening pressure of CSF; and posi-
headaches, but they do not have as obvious an tion of patient. Management is conservative,
effect in cluster as in classic or common migraine including administering analgesia and sufficient
syndromes. fluids and caffeine. If this fails to resolve the
headache after 72 h an epidural blood patch.
11. c—Sumatriptan
Image with permission from Inamasu J, Nakatsukasa
This patient has common migraine (migraine M. Blood patch for spontaneous intracranial hypoten-
without aura). Of the agents listed, only suma- sion caused by cerebrospinal fluid leak at C1-2, Clin
Neurol Neurosurg 2007;109(8):716-19.
triptan is generally considered of use to abort a
headache. The triptans are a group of medica- 13. b—Anti-aquaporin 4 antibody
tions that act as agonists at serotonergic recep-
tors (specifically, the 5HT-1 receptors), and Neuromyelitis optica (NMO; Devic’s disease) is
they have been found to be very effective at stop- monophasic or relapsing-remitting demyelinating
ping migraine headaches. Additional agents that CNS disorder Although previously thought to be a
might be of benefit in abortive therapy include variant of multiple sclerosis, it is now recognized to
ibuprofen, aspirin, acetaminophen, be a distinct disease, particularly prevalent in Asian
isometheptene, or ergotamine. Several medica- populations. Features of optic neuritis include uni-
tions are effective as prophylactic agents in the lateral decrease in visual acuity over, poor discrim-
treatment of migraine. These include amitripty- ination of colors, “red desaturation,” pain worse
line hydrochloride, propranolol, verapamil, and on eye movement, relative afferent pupillary
valproate. Verapamil and amitriptyline may be defect and a central scotoma. Diagnosis of
used as prophylactic (preventative) therapy. NMO requires bilateral optic neuritis, transverse
Most experts recommend initiating prophylactic myelitis and 2 of the following:
therapy only when headaches occur at least one 1. Spinal cord lesion involving three or more
to two times per month. Metoclopramide hydro- spinal levels (longitudinally extensive trans-
chloride, sumatriptan, and ergotamine tartrate verse myelitis)
are appropriately used to treat an acute attack 2. Initially normal MRI brain
of migraine and should not be prescribed on a 3. Aquaporin 4 positive serum antibody (pos-
daily basis. Daily use of these medications can itive in 80%)
establish a rebound syndrome that results in a Adults are especially likely to develop a
chronic daily headache. pattern more typical of relapsing-remitting MS
after an initial episode of neuromyelitis optica.
12. a—Epidural blood patch Management of acute episodes is with predniso-
lone or plasma exchange, and recovery from acute
The clinical presentation is consistent with
optic neuritis usually takes 4-6 weeks. Long
spontaneous intracranial hypotension, in partic-
term treatment is with immunosuppression
ular given the strong relationship of pain to (e.g. azothiaprine, rituximab). Disease modifying
upright posture. The patient has known con-
drugs used in MS are not used in the treatment
nective tissue disease and so is at increased risk
of NMO.
for this diagnosis. The MRI brain with gadolin-
ium images are characteristic for spontaneous Image with permission from Sheerin F, Collison K,
intracranial hypotension and so confirm the Quaghebeur G. Magnetic resonance imaging of acute
diagnosis. Lumbar puncture is often difficult intramedullary myelopathy: radiological differential
in spontaneous intracerebral hypotension and diagnosis for the on-call radiologist, Clin Radiol
would not change initial management unless 2009;64(1):84-94.
CSF infection needed excluding. Post lumbar

Neurosurgery Books Full


www.ketabpezeshki.com 66485438-66485457
146 PART II CARE OF THE NEUROSURGICAL PATIENT

14. b—Acute disseminating encephalomyelitis (Marcus Gunn pupil), and internuclear opthal-
moplegia. Atypical presentations include trigem-
Acute disseminated encephalomyelitis (ADEM, inal neuralgia, seizures and acute psychiatric
postinfectious encephalomyelitis) is a demyelin- disturbance. Diagnosis of MS requires demon-
ating disease of the brain, brainstem, and spinal stration of lesions disseminated in time and space
cord that is indistinguishable from MS on MRI. (McDonald criteria), hence after a single episode
It is, however, monophasic, meaning that it it is termed “clinically indeterminate syndrome”
occurs acutely on a single occasion and not in a (unless there is past medical history and old and
recurrent fashion like MS. It usually develops newer lesions on MRI). If there are >3 white-
within days or weeks of a viral illness (e.g. scarlet matter lesions on MRI the 5-year risk of develop-
fever, measles, chickenpox) or immunization. It is ing multiple sclerosis is c. 50%. MRI features of
characterized by an acute onset of multifocal neu- demyelinating plaques include T1 hypointense
rological symptoms with rapid deterioration, (black holes), T2 and FLAIR hyperintense, and
which can be fatal if untreated. Non-specific signs if new/active inflammation they enhance on T1
such as headache, fever, nausea, and vomiting + GAD sequences. T2/FLAIR imaging may also
may also accompany the onset of illness. Motor show linear regions of perivenous demyelination
and sensory deficits are frequent and there may perpendicular to the corpus callosum known as
also be brainstem involvement including occulo- Dawson’s fingers. Other investigations include
motor defects. The diagnosis is suggested by the lumbar puncture which may show a raised protein
MRI or CT picture of rapidly evolving white and in 80% positive for oligoclonal bands (in CSF
matter damage associated with a high ESR and but not in serum) signifying increased intrathecal
a CSF under increased pressure with elevated synthesis of IgG. Visual evoked potentials may be
red cell and white cell counts and elevated protein delayed, but well preserved waveform. Treatment
content. The CSF glucose content is usually in multiple sclerosis is focused at reducing dura-
normal. Management involves intravenous glu- tion of relapses (acute) and reducing the fre-
cocorticoids and the consideration of IV immu- quency of relapses (disease modifying drugs) as
noglobulins where this fails. there is no cure. For acute relapses high dose ste-
roids (e.g. oral or IV methylprednisolone) may be
Image with permission from Yachnis AT, Rivera- given for 3-5 days to shorten the length of an
Zengotita ML. Neuropathology, High-Yield Pathology
Series, Saunders, Elsevier, 2014 acute relapse, although they do not alter the
degree of recovery (i.e. whether a patient returns
15. b—IV methylprednisolone to baseline function).

Multiple sclerosis is a chronic, predominantly Image with permission from Sicotte NL. Magnetic reso-
nance imaging in multiple sclerosis: the role of conven-
autoimmune demyelinating disease of the central tional imaging, Neurol Clin 2011;29(2):343-56.
nervous system (CNS) characterized by subacute
neurologic deficit (relapses last at least 24 h) cor- 16. b—Glatiramer acetate
relating with CNS lesions separated in time and
space, excluding other possible disease. Peak pre- Clinical and imaging features (focal lesion that
sentation at 20-40 years. Subtypes include: does not exceed two vertebral segments in length
Relapsing-remitting MS (80%): relapses followed and does not affect more than half the cross-
by complete or near-complete recovery, most of sectional area of the cord) suggest relapsing-
which later transition to secondary progressive remitting MS with plaques in the cervical spinal
MS during which there progression of disability cord. In general, current evidence suggests start-
with few or no relapses. Primary progressive ing disease-modifying therapy at the point of diag-
MS (20%) shows progression of disability from nosis of relapsing-remitting forms of MS since
the onset, rarely with relapses. Presentation is damage continues to occur (based on MRI studies)
with optic neuritis, neurological symptoms even between relapses. As such, disease modifying
related to transverse myelitis (e.g. bladder dys- therapy is used in those with “active” relapsing MS
function, myelopathy) or intracranial plaques of defined either as 2 or more relapses in the last
demyelination. Lhermitte’s sign is an electrical 2 years or one recent relapse and/or signs of new
sensation radiating down the spine when the neck lesions on MRI. Interferon beta 1a or 1b can be
is passively flexed and is believed to signify spinal used in relapsing remitting MS, secondary pro-
cord demyelination. Uhthoff’s phenomenon gressive MS if there are still significant relapses
describes the worsening of MS symptoms with and clinically isolated syndrome, but is contraindi-
higher body temperature (e.g. hot weather, exer- cated with deranged liver function. Glatiramer
cise, fever). Eye signs include nystagmus, RAPD acetate may act as a myelin decoy for the immune

Neurosurgery Books Full


www.ketabpezeshki.com 66485438-66485457
8 NEUROLOGY AND STROKE 147

system and is not contraindicated in liver dysfunc- neurologic deficits. Equally, ring-enhancing
tion. Fingolimod is the only oral drug and is a lesions in the setting of MS should not be consid-
sphingosine 1 phosphate receptor modulator ered to be TDL—neoplasia and abscess should be
affecting lymphocyte migration that has been excluded first.
proven to reduce number of relapses and slow
the rate of number of new MRI lesions. However, Image with permission from Adam A, et al., editors.
Grainger & Allison's Diagnostic Radiology, 6th ed.
it was also associated with increased incidence of Churchill Livingstone: Elsevier, 2014.
varicella zoster, tumor formation, and progressive
multifocal leucoencephalopathy (PML) hence 18. a—Chiari malformation
reserved for patients who fail 1st line therapies.
Similarly, while natalizumab is effective in modi- The presence of loss of pain and temperature sen-
fying multiple sclerosis progression, it is also asso- sation in a “cape-like” distribution is highly sug-
ciated with PML and not considered a 1st line gestive of syringomyelia, which is commonly
treatment. Mitoxanthrone is a chemotherapy associated with Chiari I malformation. It may
agent that inhibits DNA synthesis and repair, be slowly progressive, cause wasting and weak-
associated with significant cardiotoxicity, reserved ness of the arms, spinothalamic tract deficit (pain
as last resort. Other problems may also need symp- and temperature), loss of reflexes and upgoing
tomatic treatment: plantars, and Horner’s syndrome.
• Fatigue—exclude common causes (e.g. ane-
mia, hypothyroid or depression), amantadine, 19. e—Posterior reversible encephalopathy
mindfulness training and CBT. syndrome
• Spasticity—physiotherapy, baclofen, and
gabapentin are first-line. Other options Posterior reversible leucoencephalopathy syn-
include diazepam, dantrolene, and tizanidine. drome may present with thunderclap headache,
Botox. usually followed rapidly by confusion, seizures
• Bladder dysfunction—ultrasound first to and visual symptoms. The most common causes
assess bladder emptying. If significant residual of PRES are hypertensive encephalopathy and
volume ! intermittent self-catheterization, eclampsia. Hypertension is commonly observed.
whereas if no significant residual volume anti- CT brain and lumbar puncture results are usu-
cholinergics may improve urinary frequency. ally normal or near normal. The elevation of
• Oscillopsia may respond to gabapentin. CSF protein with hypertensive encephalopathy
is variable because intracranial hemorrhage
Images with permission from Saraf-Lavi, Efrat, Spine may occur with the hypertensive crisis, but most
Imaging: Case Review Series, 3rd ed. Saunders, Else-
vier, 2014. patients will have moderate increases in CSF
protein.
17. e—Demyelination Diagnosis is made by evidence of vasogenic
brain edema on MRI brain. PRES is often associ-
Tumefactive demyelination is inflammatory ated with reversible cerebrovascular vasoconstric-
demyelinating disease which presents as a soli- tion syndrome with vasospasm on cerebral
tary large (>2 cm) focus of demyelination angiography. Management in this case will
within a cerebral hemisphere with associated require blood pressure control in the critical care
edema that may simulate neoplasm or abscess. setting. Other causes of thunderclap headache
Presentation is acute (3 weeks) with headache, include aneurysmal subarachnoid hemorrhage,
seizures, and focal neurologic deficits. Often it cerebral venous sinus thrombosis, internal carotid
may be a monophasic episode of disease without artery dissection, pituitary apoplexy reversible
recurrence, but some may evolve into relapsing- cerebral vasoconstriction syndrome, and benign
remitting MS. Imaging features in 50% have coital headache.
contrast enhancement in the form of an incom-
plete ring, without enhancement at junctions Image with permission from Loevner L. Brain Imaging:
Case Review Series, 2nd ed. Mosby: Elsevier, 2009.
with gray matter (or basal ganglia depending on
orientation), and there is usually minimal mass
20. b—Marcus-Gunn phenomenon
effect. Advanced MR imaging techniques may
be useful, and the rCBV values are significantly Aberrant regeneration of a cranial nerve is not all
lower than for high-grade glial neoplasms. Man- that uncommon, but it is more often seen after
agement is with high-dose corticosteroid therapy. injury to the third nerve than to the seventh.
Radiation or surgical excision of lesions misdiag- For unknown reasons, the regenerating motor
nosed as tumor will cause additional irreversible fibers miss their original targets and innervate

Neurosurgery Books Full


www.ketabpezeshki.com 66485438-66485457
148 PART II CARE OF THE NEUROSURGICAL PATIENT

new destinations. With cranial ALS, facial twitch- receptors. A functional acetylcholine deficiency
ing occurs, but it is not preceded by unilateral develops at the synapse because receptors are
weakness, and it is seen as the weakness evolves, blocked or inefficient. Myasthenia is more com-
not as it remits. Sarcoidosis may produce facial mon in women (2:1). Approximately 1/3 of patient
weakness with aberrant regeneration, but this have a thymoma (commonest tumor of anterior
patient’s history does not suggest this idiopathic mediastinum) which can cause death by airway
granulomatous disease. There is nothing to sug- compression or cardiac tamponade. The chest
gest that his Lyme disease is recurring, although X-ray shows a partially delineated mediastinal
recurrent meningitis may develop with inade- mass (anterior mediastinum) with regular borders,
quate treatment. bulging the left upper mediastinal contour sugges-
tive of a thymoma. Other associations include
21. c—Retinal Telangiectasias thymic hyperplasia and autoimmune disorders
(pernicious anemia, autoimmune thyroid disor-
The association of erythrocytosis with cerebellar ders, rheumatoid, SLE). Presentation is with ocular
signs, microscopic hematuria, and hepatospleno- weakness (90%) including ptosis, opthalmoparesis
megaly suggests von Hippel-Lindau syndrome. generally worse with sustained upward gaze. More
This hereditary disorder is characterized by severe disease includes limb weakness, difficulty
polycystic liver disease, polycystic kidney disease, with swallowing, and respiratory difficulties. The
retinal angiomas (telangiectasia), and cerebellar key feature is muscle fatigability—muscles become
tumors. This is an autosomal dominant inherited progressively weaker during periods of activity
disorder with variable penetrance. Men are more and slowly improve after periods of rest. Patients
commonly affected than women. Although neo- usually report fatigue that increases as the day
plastic cysts may develop in the cerebellum in progresses. Investigations include single fiber
persons with von Hippel-Lindau syndrome, electromyography (high sensitivity 92-100%),
these usually do not become sufficiently large to CT thorax to exclude thymoma, CK is normal,
cause an obstructive hydrocephalus. Other abnor- around 85-90% of patients have antibodies to
malities that occur with this syndrome include acetylcholine receptors. In the remaining patients,
adenomas in many organs. Hemangiomas may about 40% are positive for anti-muscle-specific
be evident in the bones, adrenals, and ovaries. tyrosine kinase antibodies. Tensilon test (intrave-
Hemangioblastomas may develop in the spinal nous edrophonium) reduces muscle weakness
cord or brainstem, as well as in the cerebellum. temporarily—not commonly used anymore due
to the risk of cardiac arrhythmia. Management
22. b—Dermatomyositis includes initiating long-acting anticholinesterases
(e.g. pyridostigmine), immunosuppression and
Dermatomyositis is an inflammatory disorder thymectomy. Myasthenic crises (severe enough to
causing a symmetrical, proximal muscle weakness require intubation) may be triggered by other med-
and skin lesions. Polymyositis is a variant where ications, infection or other physiological stressors
skin lesions are not prominent. It may be idio- and necessitate plasma exchange or intravenous
pathic, associated with connective tissue disorders immunoglobulin.
or a paraneoplastic syndrome in about 20% of
cases overall. Lung, ovarian, gastrointestinal Image with permission from Mattler FA. Essentials of
tract, breasts, and other malignancies can cause Radiology, 3rd ed. Saunders, Elsevier, 2014.
it hence a thorough search for a primary is indi-
cated. Skin manifestations include a lilac (helio- 24. b—Inclusion body myositis
trope) rash around the eyes, photosensitive skin,
macular rash over back and shoulder, Gottron’s Inclusion body myositis is the most common pri-
papules, nail fold capillary dilatation and mary myopathy in the elderly manifesting as a
flat-topped purplish nodules over the elbows slowly progressive weakness, usually affecting fin-
and knees. Other features are proximal muscle ger and wrist flexion initially (but does affect
weakness with tenderness, Raynaud’s syndrome, both proximal and distal muscles). Lower limb
respiratory muscle weakness, interstitial lung dis- weakness may also occur with quadriceps. In
ease and dysphagia/dysphonia. upper and lower limbs flexors affected more than
extensors. Reflexes are usually diminished as in
23. a—Anti-acteylcholine receptor antibodies other myopathies. Creatine kinase levels are usu-
ally normal or only mildly raised, in contrast to
Myasthenia gravis is an autoimmune damage polymyositis (where creatine kinase levels are
that occurs at the neuromuscular junction, speci- usually markedly elevated). Associated with cyto-
fically at postsynaptic membrane acetylcholine plasmic inclusions on muscle biopsy. Muscles are

Neurosurgery Books Full


www.ketabpezeshki.com 66485438-66485457
8 NEUROLOGY AND STROKE 149

often tender in polymyositis, the distal muscles unremarkable in between attacks. Diagnosis
are usually not affected until the disease is of hypokalemic periodic paralysis is often made
advanced and CK is significantly raised. Diabetic clinically by episodes of paralysis, typically
amyotrophy is characterized by painful wasting of occur at night and may be triggered by carbo-
the proximal lower limb muscles. hydrate meals, in association with low serum
potassium but genetic testing can help if known
25. c—Start prednisolone mutations are present. Management is with
lifelong potassium supplementation.
Temporal (giant cell) arteritis. is large vessel vas-
culitis which overlaps with polymyalgia rheuma- 28. b—Duchenne muscular dystrophy
tica (PMR) typically in a patient >60 years
old, usually rapid onset with evidence of head- Duchenne muscular dystrophy (DMD) and
ache and jaw claudication in many, and visual dis- Becker muscular dystrophy (BMD) are X-linked
turbances secondary to anterior ischemic optic recessive disorders caused by mutations in the
neuropathy in the presence of a tender, palpable dystrophin gene on Xp21 (DMD occurs in 1 in
superficial temporal artery. In contrast, PMR 3000-6000 live births; BMD is much less com-
usually presents with myalgia, morning stiffness mon). Dystrophin is part of a large membrane
in proximal limb muscles (not weakness), leth- associated protein in muscle which connects the
argy, depression, low-grade fever, anorexia, night muscle membrane to actin, part of the muscle cyto-
sweats with a raised ESR but normal CK. Inves- skeleton. In DMD there is a frameshift mutation
tigations for temporal arteritis include raised resulting in one or both of the binding sites are lost
inflammatory markers: ESR >50 mm/hr and leading to a severe form while in BMD there is a
CRP may also be elevated, normal CK. Treat- non-frameshift insertion in the dystrophin gene
ment with high dose prednisolone is started on resulting in both binding sites being preserved
clinical suspicion (due to the risk to vision) while leading to a milder form. DMD usually presents
awaiting temporal artery biopsy, and also because with skeletal muscle weakness before the age of
histology shows changes which characteristically 5 years, calf pseudohypertrophy, Gower's sign
“skips” certain sections of affected artery whilst (child uses arms to stand up from a squatted posi-
damaging others hence a negative temporal artery tion) and intellectual impairment (in 30%) which
biopsy does not rule out temporal arteritis. progresses if untreated such that boys become
Patients with visual symptoms should be seen wheelchair-bound by their early teens. Histori-
the same-day by an ophthalmologist as visual cally, death occurs by age 25 years, primarily from
damage is often irreversible. If there is no respiratory dysfunction and less often from heart
response to prednisolone the diagnosis should failure. A multidisciplinary treatment approach
be reconsidered. including steroids, scoliosis surgery, ventilatory
support, and cardiac therapy has improved sur-
26. c—Anti-voltage gated calcium channel vival. BMD is associated with a more variable pre-
antibody sentation of skeletal muscle weakness from the age
of 10 onwards and absence of intellectual impair-
Lambert-Eaton myesthenic syndrome (LEMS) ment, and carries a better prognosis, with most
is a paraneoplastic myesthenic syndrome associ- patients surviving to the age of 40-50 years. Car-
ated with small-cell lung cancer where antibodies diac involvement is seen in both disorders, and
to voltage-gated calcium channels (VGCCs) the severity is not correlated with the severity of
have been reported in 75-100%. It presents skeletal muscle involvement. CK is elevated to
similarly to myasthenia gravis, with proximal 5-10 times normal in both. Facioscapulohumeral
muscle weakness, hyporeflexia and autonomic muscular dystrophy is autosomal dominant and
features but does not show fatiguability of muscle the third most common after the DMD and myo-
strength—strength actually improves with tonic dystrophy. Muscle weakness tends to follow a
greater effort. slowly progressive but variable course. The patient
initially presents with facial and/or shoulder girdle
27. c—Hyperkalemia periodic paralysis muscle weakness, which progresses to involve the
pelvic musculature. The limb-girdle muscular dys-
Periodic paralysis can be classified into hypoka- trophies are a group of disorders with a limb-
lemic and hyperkalemic periodic paralysis and shoulder and pelvic girdle distribution of weakness,
Andersen-Tawil (long-QT) syndrome, due to but with otherwise heterogeneous inheritance and
mutations in skeletal muscle ion channels. genetic cause. The onset of muscle weakness is var-
Onset is most commonly in childhood and ado- iable but usually occurs before age 30 with com-
lescents, with attacks of paralysis lasting hours plaints of difficulty with walking or running
and neurological examination is normally secondary to pelvic girdle involvement. As the

Neurosurgery Books Full


www.ketabpezeshki.com 66485438-66485457
150 PART II CARE OF THE NEUROSURGICAL PATIENT

disease progresses, involvement of the shoulder EMI ANSWER


muscles and then more distal muscles occurs, with
sparing of facial involvement. Emery-Dreifuss 52. 1—i, Wernicke encephalopathy. Thiamine
muscular dystrophy is a rare X-linked disorder in deficiency can result in this classic triad of nys-
which skeletal muscle symptoms are often mild tagmus, opthalmoplegia and ataxia requiring
but with cardiac involvement that is both common urgent replacement. 2—f, Delerium tremens.
and serious. It is characterized by a triad of early 3—j, Wernicke-Korsakoff syndrome. Repre-
contractures of the elbow, Achilles tendon, and sents the addition of anterograde amnesia, ret-
posterior cervical muscles; slowly progressing rograde amnesia and confabulation to
muscle weakness and atrophy, primarily in humer- Wernicke’s encephalopathy.
operoneal muscles; and cardiac involvement.
53. 1—h, Mercury; 2—f, Lead; 3—b, Arsenic;
Image with permission from Daroff RB, et al., editors.
Bradley’s Neurology in Clinical Practice, 7th ed. 4—e, Ergotism; 5—g, Manganese.
Elsevier, 2016.
Clinical features of course depend on the precise
metal involved but acute poisoning typically
ANSWERS 29–51 include encephalopathy, gastrointestinal upset
and myalgia in addition to peripheral neuropathy.
Additional answers 29–51 available on Significant sensory neuropathy causing neuro-
ExpertConsult.com pathic pain is commonly prominent.

Environmental Poisoning: Clinical Features


Toxin Effects Treatment

Lead Ataxia and tremor in children exposed to relatively low levels. NaEDTA, dimercaprol or
Chronic exposure routinely impairs psychomotor N-acetyl-penicillamine
development and may lead to substantial retardation in very
young children. Acute toxicity in children can cause brain
edema and may be lethal even with efforts to relieve the
intracranial pressure. In adults, it produces a painless bilateral
neuropathy often targeting the radial nerve and resulting in a
wrist drop abdominal pain, constipation, anemia, basophilic
stippling of erythrocyte precursors, and a linear discoloration
along the gingival margin (lead lines)

Arsenic Diarrhea, polyneuropathy, gastrointestinal pain, vomiting, Dimercaprol


shock, coma, renal failure Penicillamine

Manganese Long-term exposure to this metal may produce Parkinsonism Levodopa


but axial rigidity and dystonia may also develop. Chronic EDTA
poisoning results in polyneuropathy

Mercury Depends on route (vapor, ingestion, skin). Vapor toxicity may Dimercaprol
show fatigue, weakness, abdominal cramp, headache, and
fever. Chronic toxicity usually associated with tremors,
gingivitis and erethism (behavior change). Other features are
peripheral neuropathy, ataxia, visual disturbance

Aluminum Encephalopathy and osteodystrophy in renal dialysis patients Desferrioxamine

Carbon monoxide Confusion and headache at carboxyhemoglobin levels of 20% Pure oxygen, hyperbaric
to coma, posturing, and seizures at levels of 50% to 60%. oxygen
Characteristic of CO poisoning is delayed neurological
deterioration occurring 1-3 weeks after the initial event.
Typically, this takes the form of an extrapyramidal disorder
with Parkinsonian gait and bradykinesia

Ergot Ergot is a potent vasoconstricting agent derived from the rye Discontinue ergots,
fungus, Claviceps purpurea. Ergotism is convulsive (diarrhea, Anti-platelets
paresthesias, headache) or gangrenous (dry gangrene due to
vasoconstriction affecting fingers and toes)

Continued

Neurosurgery Books Full


www.ketabpezeshki.com 66485438-66485457
8 NEUROLOGY AND STROKE 151

Toxin Effects Treatment


Organophosphates Severe abdominal cramps, blurred vision, twitching, and loss Atropine (muscarinic
of consciousness bronchospasm and diaphragmatic paralysis features)
Oximes (nicotinic
features)
May require intubation
and ventilation.

Cyanide Onset of symptoms is usually rapid and can be within seconds Resuscitation and
to minutes for inhalation and within an hour for oral exposure. decontamination.
The “classical” presentation is rapid onset of coma, seizures, Dicobalt edetate,
shock and profound lactic acidosis hydroxocobalamin

Thallium Hair loss, stupor, gastrointestinal distress, seizures, and Gastrointestinal


headaches, as well as a painful, symmetric, primarily sensory decontamination
neuropathy Prussian blue or
activated charcoal

54. 1—d, Fragile X syndrome; 2—f, Prader-Willi syndrome; 3—j, Williams syndrome; 4—i, Velocar-
diofacial syndrome; 5—g, Rett syndrome

Genetic Syndromes Associated with Learning Disabilities


Genetic syndrome Cause Features

Tuberous sclerosis Usually sporadic but can be Skin changes (adenoma sebacium, shagreen patches,
autosomal dominant (Ch 9,11,16) ash leaf spots), seizures, cortical tubers, subependymal
giant cell astrocytoma, retinal hamartoma, renal
angiomyolipoma, cardiac rhabdomyoma

Down’s syndrome Trisomy 21 due to non- Characteristics include facial appearance, simian crease,
dysjunction, translocation, clinodactyly, congenital heart defects, GI defects,
mosaicism atlantoaxial instability, dementia, thyroid disorders,
diabetes, sleep apnea, hearing loss, and visual problems

Rett syndrome X-linked defect of methyl CpG- Most boys die before birth. Girls developmental
binding protein 2 retardation, mutism, and movement disorder from early
childhood, generally during the second year of life. There
is loss of previously acquired language skills and
effective eye contact, as well as purposeful hand
movement. Stereotypic hand movements develop (hand
wringing, tapping, patting, and at times hand-mouth
movements). Seizures may also occur

Cri du Chat Chromosome 5 deletion High pitched cry, microcephaly, hypotonia, facial
syndrome features, including widely set eyes (hypertelorism), low-
set ears, a small jaw, and a rounded face. Some children
with cri-du-chat syndrome are born with a heart defect

Neurofibromatosis Usually autosomal dominant. The disorder is characterized by numerous benign


May be sporadic (Ch 17 + 22) Neurofibromas, but also ependymoma, meningioma,
schwannoma and childhood chronic myelogenous
leukemia

Prader-Willi 70% sporadic (deletion on Ch 15 Inheriting the deletion through the mother gives rise to
syndrome of paternal origin) Angelman syndrome, which is characterized by short
Angelman stature, severe mental retardation, spasticity, seizures,
syndrome and a characteristic stance. Inheriting the deletion from
the father produces the more common Pader-Willi
syndrome, which is characterized by obesity, excessive
and indiscriminate gorging, small hands, feet,
hypogonadism and mental retardation. In rare cases,
uniparental disomy involving chromosome 15 produces

Continued on following page

Neurosurgery Books Full


www.ketabpezeshki.com 66485438-66485457
152 PART II CARE OF THE NEUROSURGICAL PATIENT

Genetic Syndromes Associated with Learning Disabilities (Continued)


Genetic syndrome Cause Features
PWS when both copies are inherited from the mother and
AS when both copies are inherited from the father

Velocardiofacial Usually sporadic, may be Thymus abnormalities, ear deformities, cleft palate,
syndrome (22q11 autosomal dominant cardiac defects and short stature. Hypocalcemia in
deletion, DiGeorge (microdeletion Ch 22) adolescence
syndrome)

Fragile X X-linked (FMR-1 gene has Boys with syndrome have long faces, prominent jaws,
syndrome expansion of trinucleotide large ears, and are likely to be mentally retarded. Affected
repeats at fragile site on X men have large ears, a high-arched palate, hypotelorism,
chromosome). Shows and large testes. Autism also occurs among affected men
anticipation.

Williams Deletion in Chromosome 7 Characteristic elfin facies, short stature, cardiovascular


syndrome defects, dental problems and GI disorders (among
others). Hypercalcemia

55. 1—i, Multifocal motor neuropathy with con- It is characterized by recurrent attacks of
duction block (MMNCB). Younger to abdominal pain, psychosis, and neuropathy
middle-aged men who develop focal arm (motor and autonomic). Autonomic neurop-
weakness in the distribution of a named nerve athy results in gastroparesis, constipation/
relatively rapidly (e.g. a week). Over several pseudoobstruction (hence abdominal pain),
months additional named motor nerves and autonomic instability. Attacks may be
become involved asymmetrically such that provoked by drugs, such as barbiturates, anti-
it may resemble MND (MMNCB shows convulsants, sulfonamide antibiotics, and
conduction block due to segmental demye- estrogens. Treatment is best accomplished
lination. MND does not. MMNCB is a with use of intravenous hematin when sup-
demyelinating condition, MND is axonal). portive measures are not adequate or the case
Nerve conduction studies help to decide is severe. 4—l, Sarcoidosis. Neurosarcoid
whether a motor neuropathy is axonal or affects both central and peripheral nervous
demyelinating. Anti-GM1 antibodies may system, with optic atrophy, facial nerve palsies
also be present. MMNCB, Guillain-Barré, and peripheral neuropathy. 5—f, Hereditary
and CIDP are all examples of demyelinating sensory and motor neuropathy (HSMN,
neuropathies, all of which therefore respond Charcot-Marie tooth disease). HSMN type
to intravenous immunoglobulin (IVIG). I autosomal dominant due to defect in
2—d, Chronic inflammatory demyelinating PMP-22 gene (which codes for myelin) result-
polyneuropathy (CIDP). Similar to ing in a predominantly demyelinating neu-
Guillain-Barré syndrome (acute inflamma- ropathy. Features often start at puberty,
tory demyelinating polyneuropathy, AIDP), motor symptoms predominate with distal
but it takes a slowly progressive or remitting muscle wasting, pes cavus, clawed toesfoot
course rather than one of acute onset. It is a drop, and leg weakness. HSMN type II is pri-
polyradiculoneuropathy affecting the proxi- marily axonal neuropathy. 6—h, Mononeuri-
mal portions of the nerves where they exit tis multiplex. In this disorder, individual
the spinal cord (i.e. nerve root). Because nerves are transiently disabled over the course
nerve roots are affected, patients experience of minutes to days, and the recovery of func-
proximal and distal weakness and sensory loss tion may require weeks to months. Diabetes is
from the onset. Raised CSF protein may be the commonest cause.
present due to the inflammatory response
affecting nerve roots within the thecal sac. 56. 1—i, Paraneoplastic cerebellar degeneration.
3—a, Acute intermittent porphyria. A disor- Characterized by subacute, progressive
der of heme biosynthesis in the liver resulting ataxia, dysarthria, and nystagmus. Myoclo-
from increased production and excretion of nus, opsoclonus (irregular jerking of the eyes
porphobilinogen and θ-aminolevulinic acid. in all directions), diplopia, vertigo, and

Neurosurgery Books Full


www.ketabpezeshki.com 66485438-66485457
8 NEUROLOGY AND STROKE 153

hearing loss may also occur. The most com- of voltage gated calcium channels by autoan-
mon associated tumor types are small cell tibodies. A characteristic feature is the
carcinoma of the lung, ovarian/breast carci- increase in strength briefly after repeated
noma, and lymphoma. Anti-Purkinje cell muscle activation. Most cases are associated
antibodies (anti-Yo antibodies) may be pre- with small cell lung cancer, or in the context
sent in 50%. Paraneoplastic cerebellar of other autoimmune diseases. 4—j, Parapro-
degeneration may precede the symptoms of teinemic neuropathy. Polyneuropathy may
the underlying tumor itself. 2—c, Hypercal- occur in up to 15% of patients with multiple
cemia. It may be a result of parathyroid- myeloma presents as a chronic distal sym-
related peptide secreted by the tumor itself metrical sensory or sensorimotor neuropa-
(usually lung cancer) or of bone destruction thy. CSF protein may be elevated if there is
by metastatic disease. The elevated serum a chronic inflammatory demyelinating
calcium decreases membrane excitability, polyneuropathy-like picture. Up to 20% of
leading to the clinical syndrome of fatigabil- patients referred for evaluation of polyneuro-
ity, lethargy, generalized weakness, and are- pathy may a monoclonal gammopathy of unde-
flexia progressing to coma and even termined significance, but a hematologic
convulsions. Symptoms usually do not occur malignancy may later declare itself.
until levels reach 14 mg/dL (3 mmol/l) or
higher. 3—d, Lambert-Eaton myasthenic 57. 1—b, Ataxia telangiectasia; 2—g, Friedreich
syndrome (LEMS) shows subacute proximal ataxia; 3—j, Spinocerebellar ataxia; 4—a,
muscle weakness and spares the bulbar mus- Abetalipoproteinemia
culature, and is due to presynaptic blockade

Hereditary Ataxias: Clinical Features


Hereditary ataxia Features

Spinocerebellar ataxia Autosomal dominant ataxias many of which are trinucleotide repeat disorders,
(SCA1-SCA10) demonstrate anticipation and tend to present in the third and fourth decades.
Frequently associated with pyramidal signs such as hyperreflexia and spasticity.
Sensory neuropathy is common, and patients may display dystonia, chorea, or
cognitive decline. Machado-Joseph disease (SCA3) is one of the commonest and
ataxia, hyperreflexia, nystagmus/ophthalmoplegia, and dysarthria develop early in
the disease and may also show a levodopa-responsive rest tremor

Freiderich’s ataxia The most common genetic ataxia (1 in 50,000) and occurs secondary to a GAA repeat
located on chromosome 9 (autosomal recessive) but unusually does not demonstrate
anticipation. Loss of frataxin affects mitochondrial iron homeostasis, making the cell
susceptible to oxidative stress. The typical age of onset is 10-15 years old with gait
ataxia and scoliosis followed by progressive loss of neuromuscular function, with the
patient wheelchair-bound 10-20 years after symptom onset. Neurological features are
absent ankle jerks/extensor plantars, cerebellar ataxia, optic atrophy, spinocerebellar
tract degeneration. Other features hypertrophic obstructive cardiomyopathy (90%,
most common cause of death) diabetes mellitus (10-20%) and high-arched palate

Ataxia telangiectasia Ataxia-telangiectasia is the second most common autosomal recessive ataxia, with a
frequency of 1 in 100,000 persons. Due to a deficit in the DNA repair pathway, and
patients are predisposed to the development of neoplasms. Onset is in early childhood,
with postural instability and ataxia first becoming apparent as the child begins to walk.
Later, hypotonia, bradykinesia, areflexia, and proprioceptive deficits develop. Chorea
may occasionally be seen. The eponymous telangiectasias develop later in childhood.
Patients are often wheelchair bound by their second decade, and death usually occurs
in the fourth to fifth decade as a result of either pulmonary infection or malignancy.
EMG shows a sensory neuropathy and MRI shows cerebellar atrophy

Fragile X-associated Fragile X-associated tremor ataxia syndrome (FXTAS) is characterized by the
tremor ataxia syndrome presence of a permutation for fragile X syndrome (CGG triplet repeat in insufficient
number to cause the full fragile X syndrome). It is accompanied by intention tremor,
gait ataxia, rigidity, bradykinesia, polyneuropathy, and autonomic manifestations

Continued on following page

Neurosurgery Books Full


www.ketabpezeshki.com 66485438-66485457
154 PART II CARE OF THE NEUROSURGICAL PATIENT

Hereditary Ataxias: Clinical Features (Continued)


Hereditary ataxia Features
Abetalipoproteinamia Usually becomes symptomatic during early childhood. The peripheral blood smear
will exhibit acanthocytes, and the plasma lipid profile will reveal a very low
cholesterol and triglyceride content. These are an unusual hematologic findings in
patients with ataxia and are often diagnostic. The initial complaints are similar to the
spinocerebellar signs of Friedreich disease. Ataxia, areflexia, distal muscle atrophy,
intestinal symptoms, loss of vibratory sense; retinitis pigmentosa; steatorrhea (fat
malabsorption) resulting in vitamin A, E, K deficiency; acanthocytes on smear.
Position sense is lost and extensor plantar responses develop as the disease
progresses. Deficits accumulate over the course of years. Vitamin E supplementation
may slow the disease’s progression

Episodic ataxias Eight forms of episodic ataxia have been thus far been described (EA1 to EA7 and
DYT9), all caused by ion channel mutations. EA1 and EA2 are of particular interest
because they arise from defects in the same chromosome (chromosome 19) but have
different phenotypes. EA1 is caused by mutations in a potassium channel gene and
EA2 by mutations in a voltage-dependent calcium channel. EA1 is characterized by
kinesigenic attacks of myokymia and ataxia that last seconds to minutes. EA2 is
characterized by non-kinesigenic episodes of ataxia and vertigo that can last days.
Although the symptoms of EA1 abate over time, EA2 is a lifetime affliction

Ataxia with isolated Mimic of Freiderich’s ataxia but treatment with vitamin E can slow or reverse the
vitamin E deficiency disease

FURTHER READING isolated pyridoxine deficiency and subsequent


Haq IU, Foote, KD, Okun, MS. Clinical overview of move- peripheral neuropathy. 2—e, Thiamine
ment disorders. In: Winn RH, editor. Youmans Neurological deficiency. Symptoms suggest Wernicke’s
Surgery, 6th ed. Saunders: Elsevier, 2011. encephalopathy. 3—g, Vitamin B12 deficiency.
Symptoms suggest subacute combined
58. 1—c, Pyridoxine deficiency. Patients on antitu- degeneration of the cord. 4—i, Vitamin E
berculous therapy with isoniazid may develop deficiency.

Vitamin Deficiency States: Clinical Features


Nutrient Findings in deficiency Causes (other than dietary factors)

Thiamine (B1) Wet beriberi, dry beriberi, Wernicke’s Alcoholism, chronic diuretic use,
encephalopathy, Wernicke-Korsakoff psychosis hyperemesis, thiaminases in food

Riboflavin Magenta tongue, angular stomatitis, seborrhea, Alcoholism


chellosis

Niacin Pellagra (rare) Alcoholism, deficiency of Vit B6 or


riboflavin or tryptophan

B6 (Pyridoxine) Seborrhea, glossitis, convulsions, neuropathy, Alcoholism, isoniazid


depression, confusion, microcytic anemia

Folate Megaloblastic anemia, atrophic glossitis, Alcoholism, sulfasalazine,


depression, increased homocysteine pyrimethamine, triamterene

B12 Megaloblastic anemia, subacute combined Pernicious anemia, terminal ileal


degeneration of the spinal cord, increased disease, metformin, acid-reducing
homocysteine and methylmalonic acid drugs

Vitamin A Xeropthalmia, night blindness, Bitot’s spots, Fat malabsorption, infection,


follicular hyperkeratosis, immune dysfunction, measles, alcoholism, protein-energy
impaired embryonic development malnutrition (e.g. marasmus,
kwashiorkor)

Continued

Neurosurgery Books Full


www.ketabpezeshki.com 66485438-66485457
8 NEUROLOGY AND STROKE 155

Nutrient Findings in deficiency Causes (other than dietary factors)


Vitamin C Scurvy (inflamed/bleeding gums, petechiae, Smoking, alcoholism
ecchymosis, fatigue, joint effusion, poor wound
healing)

Vitamin D Rickets Aging, lack of sunlight, fat


malabsorption, deeply pigmented
skin

Vitamin E Peripheral neuropathy, spinocerebellar ataxia, Fat malabsorption, genetic


skeletal muscle atrophy, retinopathy abnormalities of transport

Vitamin K Coagulopathy (factors I, II, VII, IX) Fat malabsorption, liver disease,
antibiotic use

59. 1—j, X-linked adrenoleukodystrophy. Pro- myoclonic seizures may develop. There is
duces rapidly evolving brain damage in male accumulation of N-acetylaspartic acid in the
infants or boys, with survival from onset of blood and urine, but elevated levels in the
symptoms usually limited to 3 years. Long- brain establish the diagnosis.
chain fatty acids accumulate in adrenal corti-
cal and other cells, resulting in adrenal insuf- 60. 1—j, Tay-Sachs disease. This is a ganglioside
ficiency and CNS disease. 2—h, Pelizaeus- storage disease that occurs more commonly
Merzbacher disease. Leukodystrophy with in Ashkenazi Jews than in the general popu-
significant Sudan-staining typically become lation. The early-onset form will produce
symptomatic during the first months of life, macrocephaly and a cherry red spot in the
but survival may extend into the third decade fundus (retinal ganglion cells become dis-
of life. Most affected persons are male. 3—g, tended with glycolipid making retinal pale
Metachromatic leukodystrophy. Sphingoli- compared to fovea lacking ganglion cells).
pidosis due to arylsulfatase-A deficiency Children exhibit mental retardation, sei-
resulting in accumulation of galactosyl sulfa- zures, blindness and die prematurely. 2—d,
tides. The affected person usually has retar- Gaucher disease.
dation, ataxia, spasticity, and sensory
disturbances usually symptomatic during All the sphingolipids are nothing more than lipids
infancy. 4—d, Canavan disease may produce that contain a sphingosine moiety. Storage diseases
developmental regression at about 6 months of sphingolipids are mostly autosomal recessive and
of age, with extensor posturing, rigidity and can lead to CNS degeneration and early death.

Sphingolipidoses: Clinical Features


Sphingolipidosis Enzyme deficiency Abnormal accumulation Features

Niemann-Pick Sphingomyelinase Sphingomyelin and Autosomal recessive; death by age


disease cholesterol 3 year

Tay-Sachs Hexaminosidase A GM2 ganglioside Autosomal recessive; death by age


disease 3 year; cherry-red spot on fundus.

Krabbe disease Galactosylceramide Galactocerebroside Autosomal recessive; optic atrophy,


β-Galactosidase spasticity, early death

Gaucher disease β-Glucocerebrosidase Glucocerebroside Autosomal recessive; “crinkled


accumulation in liver, paper” appearance of cells,
brain, spleen, and bone hepatosplenomegaly
marrow

Fabry disease α-Galactosidase A Ceramide trihexosidase X-linked recessive

Metachromatic Arylsulfatase-A Galactosyl sulfatide Autosomal recessive; retardation,


leukodystrophy ataxia, spasticity, and sensory
disturbances; early death

Neurosurgery Books Full


www.ketabpezeshki.com 66485438-66485457
156 PART II CARE OF THE NEUROSURGICAL PATIENT

61. 1—j, Anti-voltage gated potassium channel Ab; 2—d, Anti-Aquaporin 4; 3—l, Anti-GQ1b; 4—a,
Anti-muscle specific kinase; 5—c, Anti-voltage gated calcium channel Ab

Autoantibody-Associated Neurological Disorders


Antibody Associated disorders

Anti-acetylcholine receptor antibody (AChR) Myasthenia gravis


and anti-muscle specific kinase (MuSK)

Anti-NMDA receptor antibody NMDAR-antibody encephalopathy

Anti-voltage gated calcium channel antibody Lambert Eaton myasthenic syndrome, Paraneoplastic
cerebellar degeneration

Anti-Aquaporin 4 antibody Neuromyelitis optica (Devic’s disease)

Anti-Hu (ANNA-1) Subacute sensory neuropathy, limbic encephalitis, brain stem


encephalitis, paraneoplastic encephalomyelitis

Anti-Yo (PCA-1) Paraneoplastic cerebellar degeneration

Anti-Ri (ANNA-2) Myoclonus/opsoclonus

Anti-Tr Paraneoplastic cerebellar degeneration

Anti-GAD Stiff person syndrome/cerebellar ataxia

Anti-voltage gates potassium channel antibody Neuromyotonia, limbic encephalitis (non-paraneoplastic)

Anti-Ta/Ma2 Limbic encephalitis (paraneoplastic)

Anti-GQ1b Guillain-Barré syndrome/Miller-Fisher variant

Anti-myelin associated glycoprotein/sulfated IgM Paraproteinemic neuropathy


glucoronul paragloboside

Anti-GD1b Paraproteinemic neuropathy, CANOMAD (chronic ataxic


neuropathy, ophthalmoplegia, M protein, cold agglutinins,
anti-disialosyl antibodies).

62. 1—b, Hurler syndrome; 2—a, Hunter syn- tendons, corneas, skin, and connective tissue.
drome; 3—h, Morquio syndrome A Buildup of glycosaminoglycans collect in the
cells, blood and connective tissues cause progres-
Mucopolysaccharidoses are a group of metabolic sive cellular damage which affects appearance, tis-
disorders caused by the absence or malfunction- sue function and mental development. All are
ing of lysosomal enzymes needed to break down autosomal recessive except for Hunter syndrome.
glycosaminoglycans forming bone, cartilage, Treatment is with enzyme replacement.

Mucopolysaccharidoses: Clinical Features


MPS Enzyme deficiency Accumulated product Clinical features

Hurler syndrome α-L-Iduronidase Heparan sulfate, Dwarfism, facial


dermatan sulfate dysmorphism, Clouding of
cornea, mental retardation,
hepatosplenomegaly, death
by age 10 from respiratory/
cardiac complications.

Continued

Neurosurgery Books Full


www.ketabpezeshki.com 66485438-66485457
8 NEUROLOGY AND STROKE 157

MPS Enzyme deficiency Accumulated product Clinical features


Hunter syndrome iduronate sulfatase Mild form of Hurler
syndrome; no corneal
clouding, mild mental
retardation. X-linked
recessive
Sanfilippo Heparan sulfamidase Heparan sulfate Initially symptom-free, then
syndrome A progressive intellectual
decline/speech delay,
hyperactivity and sleep
disturbance followed by
motor disease. Death in third
decade.
Sanfilippo N-Acetylglucosaminidase
syndrome B
Sanfilippo Heparan-α-glucosamidine
syndrome C N-acetyltransferase
Sanfilippo N-Acetylglucosamine-
syndrome D 6-sulfatase

Morquio Galactose-6-sulfate sulfatase Keratan sulfate, Severe skeletal dysplasia,


syndrome A chondroitin 6-sulfate short stature, cord
compression, cardiac
abnormalities, corneal
clouding, death at an early
age.

Morquio β-Galactosidase Keratan sulfate


syndrome B

Maroteaux-Lamy N-Acetylgalactosamine-4- Dermatan sulfate Normal intellectual


syndrome sulfatase development, but shares
features of Hurler syndrome.

Sly syndrome B-glucoronidase Heparan sulfate, Hydrops fetalis but those that
dermatan sulfate, survive to birth have similar
chondroitin 6-sulfate to Hurler syndrome

Natowicz Hyaluronidase Hyaluronic acid Short stature, periarticular


syndrome soft tissue masses

63. 1—d, Kearns-Sayre syndrome; 3—e, Leber’s myopathies, encephalomyopathies, or respiratory


hereditary optic neuropathy; 3—g, MELAS chain disorders, are a heterogeneous group of dis-
eases resulting from abnormalities in mitochon-
Mitochondrial DNA is inherited maternally, and drial DNA and respiratory chain function,
some of these disorders are thus transmitted hence manifest in tissue with a high respiratory
from mother to children of both sexes. The mito- workload such as brain, retina and skeletal mus-
chondrial disorders, also termed mitochondrial cle, especially extraocular and cardiac muscle.

Mitochondrial Disorders: Clinical Features


Mitochondrial disorder Features

Ataxia neuropathy syndromes Sensory ataxia, neuropathy, dysarthria, ophthalmoplegia


(SANDO). May also have epilepsy and myopathy

Chronic progressive external ophthalmoplegia External ophthalmoplegia, bilateral ptosis. May also show
mild proximal myopathy

Kearns-Sayre syndrome Early onset (<20 years) progressive external ophthalmoplegia,


pigmentary retinopathy, and one of: cerebellar ataxia, heart
block, CSF protein >1 g/l. Other associations: bilateral
deafness, myopathy, dysphagia, diabetes mellitus,
hypoparathyroidism, and dementia

Continued on following page

Neurosurgery Books Full


www.ketabpezeshki.com 66485438-66485457
158 PART II CARE OF THE NEUROSURGICAL PATIENT

Mitochondrial Disorders: Clinical Features (Continued)


Mitochondrial disorder Features
Leigh syndrome Infantile onset of subacute relapsing encephalopathy with
cerebellar/brain stem signs. Also associated with basal ganglia
lucencies, and maternal history positive

Leber’s hereditary optic neuropathy Subacute bilateral painless visual failure in young men (M:F
4:1, median age 24). Associated with dystonia and cardiac pre-
excitation syndromes

MEMSA (myoclonic epilepsy, myopathy, Seizures, cerebellar ataxia, and myopathy. Associated with
sensory ataxia) dementia, peripheral neuropathy and spasticity

NARP (neurogenic weakness with ataxia and Late-childhood or adult onset peripheral neuropathy, ataxia
retinitis pigmentosa) and pigmentary retinopathy. Associated basal ganglia
lucencies and sensorimotor neuropathy

MELAS (mitochondrial encephalomyopathy, Commonest mitochondrial disorder. Presents with stroke-like


lactic acidosis, and stroke-like episodes) episodes in those <40 years old, seizures and/or dementia, and
lactic acidosis. Associated with diabetes mellitus,
cardiomyopathy, bilateral deafness, pigmentary retinopathy
and cerebellar ataxia

MERRF (myoclonic epilepsy with ragged-red Myoclonus, seizures, cerebellar ataxia and myopathy (ragged-
fibers) red fibers on biopsy). Associated with dementia, optic atrophy,
bilateral deafness, peripheral neuropathy, spasticity and
multiple lipomata

Alpers-Huttenlocher syndrome Hypotonia, seizures, liver failure with/without renal


tubulopathy

FURTHER READING supported by the finding of periodic sharp


Chinnery PF. Mitochondrial disorders overview. In: Pagon waves at a 1-2 Hz frequency on EEG and
RA, Adam MP, Ardinger HH, et al., editors. GeneReviews the finding of elevated protein 14-3-3 in
Seattle, WA: University of Washington, Seattle; 1993-2015. CSF. 3—i, Huntington’s disease. 4—k, Neu-
http://www.ncbi.nlm.nih.gov/books/NBK1224. rosyphilis. General paresis is a chronic, often
insidious meningoencephalitis that may be
64. 1—l, Transient Global Amnesia. Episode of delayed up to 20 years after the original spi-
complete and reversible anterograde and ret- rochetal infection. Clinically, it manifests as
rograde memory loss lasting up to 24 h. dementia, delusions, dysarthria, tremor,
Patients have a persistent loss of memory myoclonus, seizures, spasticity, and Argyll
for the duration of the attack, which is char- Robertson pupils. Other presentations of
acterized by bewilderment and repeating neurosyphilis include meningitis, meningo-
questions, but retained personal identity vascular syphilis causing infarcts, optic atro-
and ability to perform complex cognitive phy, and tabes dorsalis (characterized by
and motor tasks. TGA usually affects ataxia, urinary incontinence, and lightning
middle-aged or older men and often occurs pains caused by degeneration of the posterior
in the setting of an emotional or other spinal roots). 5—j, Hypothyroidism in adults
stressor, such as physical or sexual exertion. may present with headache, dementia, psy-
Although it shares features of transient ische- chosis, and decreased consciousness. Neuro-
mic attack, it is not associated with an muscular findings are also common, and they
increased risk of stroke. 2—e, Creutzfeldt- include a proximal myopathic weakness and a
Jakob disease. Patients may have ataxia, delay in the relaxation phase of reflexes.
myoclonus, clumsiness, or dysarthria, as well
as diplopia, distorted vision, blurred vision, 65. 1—i, Neurally mediated (vasovagal) syncope;
field defects, changes in color perception, 2—b, Cardiac arrhythmia; 3—a, Autonomic
and visual agnosia. Ultimately, cortical blind- failure
ness may occur. The diagnosis may be

Neurosurgery Books Full


www.ketabpezeshki.com 66485438-66485457
8 NEUROLOGY AND STROKE 159

Classification of Syncope
Classification Pathophysiology Discriminating clinical features

Cardiac Syncope

Arrythmia Tachyarrhythmia Palpitations, HR, ECG, cardiac history, family history,


Bradyarrhythmia ECHO

Structural cardiac Valve disease Abnormal cardiac exam, ECG, cardiac history, family
disease Outflow obstruction history, ECHO

Pacemaker malfunction ECG, pacemaker check

Non-Cardiac Syncope

Neurocardiogenic Exaggerated vagal response Prolonged standing, heat, situational, normal exam,
Cardioinhibitory to stimulus normal ECG, tilt table test
Vasodepressor

Orthostatic hypotension Autonomic failure (diabetes, Change in body position, lying/standing BP, ECG, drop
Parkinson’s) in BP without change in HR. Medication history
Drug induced

Seizure - Preceding aura, rhythmic jerking, unresponsive,


tongue biting, urinary incontinence, post-ictal period,
known epilepsy, Babinski positive, pupillary response

CVA/TIA Hemorrhage/infarct Other neurological symptoms, fall due to weakness/


loss of balance rather than syncope

Vertigo Steal phenomenon Room spinning, nystagmus


BPPV

Others Hypoglycemia
Dehydration

66. 1—e, Migraine with aura. Classic migraine is described as originating in the eye and spread-
usually familial, involves a unilateral, throb- ing over the temporal area as the headache
bing head pain, and diminishes in frequency evolves. In contrast to migraine, men are more
with age. The blind spot, or scotoma, that often affected than women, and extreme irrita-
may develop as part of the aura is usually hom- bility may accompany the headache. The pain
onymous hemianopia. It typically enlarges and usually abates in less than 1 h. Affected persons
may intrude on the central vision. The margin routinely have autonomic phenomena associ-
of the blind spot is often scintillating or daz- ated with the headache that include unilateral
zling. Homonymous hemianoptic defects of nasal congestion, tearing from one eye, con-
the sort that develop during the aura of a classic junctival injection, and pupillary constriction.
migraine indicate an irritative lesion that is The autonomic phenomena are on the same
affecting one part of the occipital cortex in side of the face as the pain. 3—i, Postherpetic
one hemisphere of the brain. Other focal neu- neuralgia. HZV/VZV may reactivate in
rological phenomena may precede classic immunocompromised patients or the severely
migraine; the most common are tingling of ill elderly and result in this neuropathic pain
the face or hand, mild confusion, transient syndrome once the acute infection has settled.
hemiparesis, and ataxia. Fatigue, irritability,
and easy distractibility often develop before a 67. 1—e, Idiopathic intracranial hypertension
migraine. Affected persons usually also have (primary pseudotumor cerebri). Symptoms
hypersensitivity to light and noise during an include headaches, transient visual obscura-
attack. 2—c, Cluster headache. Cluster head- tions, progressive visual loss, pulsatile tinni-
ache refers to the tendency of these headaches tus, diplopia, and shoulder and arm pain in
to cluster in time. They may be distinctly sea- obese/overweight females. Papilledema or
sonal, but the triggering event is unknown. optic atrophy and occasionally sixth nerve
The pain of cluster headache is usually palsies may be present. Diagnoses requires

Neurosurgery Books Full


www.ketabpezeshki.com 66485438-66485457
160 PART II CARE OF THE NEUROSURGICAL PATIENT

exclusion of structural (e.g. mass lesion or puncture, head trauma, neurosurgery, or


venous sinus obstruction) or other secondary even pneumonectomy (thoracoarachnoid fis-
causes, and opening pressure >25 cm H2O tula), pituitary tumors, dural tear in the spinal
on lumbar puncture. Treatment options root sleeves, traumatic nerve root avulsion or
include serial lumbar puncture, diuretics, dehydration.
ventriculoperitoneal shunting, and optic
nerve sheath fenestration. 2—j, Thunderclap 68. 1—j, Wernicke’s aphasia; 2—c, Broca’s apha-
headache. 3—i, Spontaneous intracranial sia, I—Transcortical sensory aphasia
hypotension. Causes of intracranial hypoten-
sion include persistent CSF leak after lumbar

Classification of Aphasias
Fluency Comprehension Repetition Diagnosis Lesion/Features
Fluent Good comprehension Good repetition Anomic aphasia Isolated word finding
deficit; least localized.
Poor repetition Conduction aphasia Arcuate fasciculus
damage.
Poor comprehension Good repetition Transcortical sensory aphasia White matter
underlying
Wernicke’s area
Poor repetition Wernicke’s (receptive) aphasia Posterior lesion.
Paraphrasias and
neologisms.
Non-fluent Good comprehension Good repetition Transcortical motor aphasia Frontal white matter
lesion.
Poor repetition Broca’s aphasia Inferior frontal lobe.
Telegraphic speech
Poor comprehension Good repetition Mixed transcortical aphasia Watershed Infarct
affecting speech
areas
Poor repetition Global aphasia Large territory infarct.

Neurosurgery Books Full


www.ketabpezeshki.com 66485438-66485457
CHAPTER 9

NEURO-OPHTHALMOLOGY
SINGLE BEST ANSWER (SBA) QUESTIONS
1. The horizontal gaze center is formed by a. Non-organic disorder
which one of the following: b. Optic nerve drusen
a. Pontine paramedian reticular formation c. Optic neuritis
b. Reticular medial longitudinal fasciculus d. Idiopathic intracranial hypertension
c. Preganglionic Erdinger-Westphal nucleus e. Leber's hereditary optic neuropathy
d. Brodman Area 6
e. Superior colliculus 3. A 50-year-old man complains of 2 days' dou-
ble vision. Has abducting nystagmus in the
2. A 25-year-old woman presents with a several right eye and vertical gaze is preserved bilat-
week history of diplopia, with an 8 month his- erally. Which one of the following is most
tory of generalized headache which has been likely?
particularly bad over the last 2 months. On
examination she is obese, has a normal pupillary
light reflex and no RAPD. Appearance of
fundi are shown. Which one of the following
is most likely cause of her complaint?

a. Left internuclear ophthalmoplegia


b. Orbital apex syndrome
c. Left incomplete oculomotor palsy
d. Superior orbital fissure syndrome
e. Left one-and-a-half syndrome

161
Neurosurgery Books Full
www.ketabpezeshki.com 66485438-66485457
162 PART II CARE OF THE NEUROSURGICAL PATIENT

4. A 53-year-old woman reports drooping of her eyelids for the last 6 months. She experiences dip-
lopia when driving for any extended period of time. Which one of the following is most likely?

a. Cavernous sinus thrombosis 6. A 22-year-old female complains of reduced


b. Myasthenia gravis vision and pain in the left eye starting 3 days
c. Oculomotor nerve palsy ago. On examination vision is normal in the
d. Pituitary apoplexy right eye but 6/12 (20/40) in the left eye, Left
e. Thyroid ophthalmopathy relative afferent pupillary defect, pain when
looks to right or left, no redness or photophobia.
5. Forty minutes after bilateral instillation of a. Arteritic anterior ischemic optic neuropathy
10% cocaine eyedrops, the left pupil dilates, b. Central retinal artery occlusion
but the right does not. c. Iritis
d. Optic neuritis
e. Orbital cellulitis

a. Horner's syndrome
b. Myasthenia gravis
c. Oculomotor palsy
d. Trochlear palsy
e. Thyroid ophthalmopathy

Neurosurgery Books Full


www.ketabpezeshki.com 66485438-66485457
9 NEURO-OPHTHALMOLOGY 163

7. A 34-year-old man present with intermittent 8. On examination, this 15-year-old boy was
diplopia, worse when looking to the left. Exa- found to have upgaze paresis, ocular tilt
mination findings are shown below. Which reaction (right superior rectus skew devia-
one of the following is most likely? tion and head tilt), papilledema, and aniso-
coria. The pupils were moderate in size
and poorly reactive to light, but reactive to
near stimuli.

a. Left abducens nerve palsy


b. Left conjugate gaze paresis
c. Left internuclear ophthalmoplegia
d. Left hypotropia
e. Right trochlear palsy
B

a. Adie's tonic pupil


b. Argyll Robertson pupil
c. Horner syndrome
d. Parinaud syndrome
e. Wallenberg syndrome

Neurosurgery Books Full


www.ketabpezeshki.com 66485438-66485457
164 PART II CARE OF THE NEUROSURGICAL PATIENT

9. Which urine test would aid diagnosis in this a. Carotid dissection


child? b. Congenital
c. Neuroblastoma
d. Hypothalamic tumor
e. Cervical cord ependymoma
12. A 54-year-old male presented after being told
he had unequal pupils by a colleague. Exam-
ination findings are shown to light, accom-
modation and 30 min after administration
of 0.1% pilocarpine to both eyes. Which
one of the following is most likely?

a. Urinary Bence-Jones protein A


b. Urinary catecholamines
c. Urinary ketones
d. Urinary casts
e. Urinary protein/creatinine ratio

10. An 82-year-old man had sudden, profound


vision loss in his right eye. This was preceded
by 2 days of brief episodes of transient vision
loss in the affected eye lasting seconds. He
also complained of “tender cords” on his B
scalp, jaw claudication, and weight loss.
The erythrocyte sedimentation rate (ESR)
was normal and fundoscopy showed a dif-
fusely swollen right optic disc.
a. Anterior ischemic optic neuropathy
b. Iritis
c. Central retinal artery occlusion
d. Central retinal vein occlusion
e. Optic neuritis
C
11. Which one of the following is the most likely
cause of this patient's Horner's syndrome?

a. Left Argyll Robertson pupil


b. Left Horner's syndrome
c. Right Adie's tonic pupil
d. Right oculomotor palsy
e. Right relative afferent pupillary defect

Neurosurgery Books Full


www.ketabpezeshki.com 66485438-66485457
9 NEURO-OPHTHALMOLOGY 165

13. A 69-year-old presented with recurrent falls 14. A 5-year-old child presents with diplopia and
and unsteadiness. Visual acuity is normal. numbness over right forehead. Examination
Examination findings are shown to at rest, findings shown in primary gaze, down gaze,
to light and to accommodation. Which one left gaze and right gaze. Which one of the
of the following is demonstrated? following are most likely?

C D

a. Adie's tonic pupil a. Gradenigo's syndrome


b. Argyll Robertson pupils b. One-and-a-half syndrome
c. Parinaud syndrome c. Orbital apex syndrome
d. Marcus-Gunn pupil d. Parinaud's syndrome
e. Marcus-Gunn phenomenon e. Third nerve palsy

Neurosurgery Books Full


www.ketabpezeshki.com 66485438-66485457
166 PART II CARE OF THE NEUROSURGICAL PATIENT

15. A 17-year-old presents after a head injury in right gaze, primary gaze, left gaze, right
with diplopia, particularly worse when walk- head tilt, left head tilt and at rest. Which
ing down stairs and two images are oblique to one of the following is most likely?
each other. Examination findings are shown

A B

C D

E F

a. Left abducens palsy d. Right exotropia


b. Left fourth nerve palsy e. Right fourth nerve palsy
c. Left hypotropia

Neurosurgery Books Full


www.ketabpezeshki.com 66485438-66485457
9 NEURO-OPHTHALMOLOGY 167

16. Which one of the following is most likely in the images below?

A B

C D

E F

a. Cavernous sinus syndrome d. Left trochlear nerve palsy


b. Internuclear ophthalmoplegia e. Right Horner's syndrome
c. Left oculomotor nerve palsy

Neurosurgery Books Full


www.ketabpezeshki.com 66485438-66485457
168 PART II CARE OF THE NEUROSURGICAL PATIENT

17. A 45-year-old man presents with bilateral 18. Which one of the following is the most likely
adduction deficit during attempted gaze (with cause?
nystagmus of contralateral abducting eye).
Adduction is intact during converging (and
accommodating) during viewing of a near
target. Which one of the following is most
likely?

a. Bilateral internuclear ophthalmoplegia


b. Left internuclear ophthalmoplegia
c. Light-near dissociation
B d. Left one-and-a-half syndrome
e. Tabes dorsalis

QUESTIONS 19–23

Additional questions 19–23 available on


ExpertConsult.com

C
EXTENDED MATCHING ITEM (EMI)
QUESTIONS
a. Bilateral internuclear ophthalmoplegia
b. Superior orbital fissure syndrome 24. Ophthalmoplegia:
c. Light-near dissociation a. Abducens palsy
d. Oculomotor neuropathy b. Bilateral internuclear ophthalmoplegia/
e. Myasthenia gravis WEBINO
c. Cavernous sinus thrombosis
d. Oculomotor palsy
e. Kearns-Sayre syndrome
f. Miller Fisher syndrome
g. Myasthenia gravis
h. One-and-a-half syndrome
i. Orbital apex syndrome
j. Parinaud's syndrome
k. Steele-Richardson-Olszewski syndrome
l. Superior orbital fissure syndrome
m. Thyroid ophthalmopathy
n. Trochlear palsy
o. Unilateral internuclear ophthalmoplegia
p. Wallenberg syndrome

Neurosurgery Books Full


www.ketabpezeshki.com 66485438-66485457
9 NEURO-OPHTHALMOLOGY 169

For each of the following descriptions, select the For each of the following descriptions, select the
most appropriate answers from the list above. most appropriate answers from the list above.
Each answer may be used once, more than once Each answer may be used once, more than once
or not at all. or not at all.
1. An 18-year-old presents with progressive 1. A 54-year-old with chronic cough and
bilateral symmetrical ptosis and ophthal- weight loss presents with a left ptosis and
moplegia, pigmentary retinopathy and car- miosis. Hydroxyamphetamine testing
diac arrhythmia. dilates the miotic pupil.
2. A 32-year-old presents with ophthalmople- 2. A 22-year-old with recurrent sinusitis pre-
gia, ataxia and areflexia. There is a recent sents with pyrexia, headache and complex
history of diarrheal illness. bilateral ophthalmoplegia and loss of cor-
3. A 72-year-old admitted with aspiration neal reflex on the left.
pneumonia is noted to have impaired 3. A 3-year-old child presents with a right
downgaze, nuchal rigidity, slurred speech Horner's syndrome. There is no iris hetero-
and loss of Bell's phenomenon. chromia, no evidence or history of trauma.
Urinary catecholamines are increased.
25. Pupillary abnormalities:
a. Aberrant regeneration of oculomotor nerve 27. Third nerve palsy:
b. Adie's tonic pupil a. Atherosclerosis
c. Argyll Robertson pupil b. Basilar tip aneurysm
d. Holmes-Adie's syndrome c. Carotid cavernous fistula
e. Horner's syndrome d. Cavernous sinus syndrome
f. Marcus-Gunn pupil e. Chordoma
g. Pretectal pupil (Parinaud's syndrome) f. Chronic progressive external ophthal-
h. Physiologic anisocoria moplegia
i. Pupil involving third nerve palsy g. Clival meningioma
h. Giant cell arteritis
For each of the following descriptions, select the i. Intraorbital lesion
most appropriate answers from the list above. j. Peripheral neuropathy
Each answer may be used once, more than once k. Posterior communicating artery aneurysm
or not at all. l. Trauma
1. A 43-year-old presenting with progressive m. Uncal herniation
headache and vomiting. Examination reveals
light-near dissociation associated with upgaze For each of the following descriptions, select the
paresis. most appropriate answers from the list above.
2. A 22-year-old female with right optic neu- Each answer may be used once, more than once
ritis. Swinging flashlight test appears to or not at all.
cause dilatation of the right pupil when light 1. A 21-year-old motorcyclist involved in a
is swung from the left eye to the right eye. road traffic collision where he was thrown
20 ft. He has an obvious right sided scalp
26. Horner's syndrome: laceration and GCS was E2V2M4 at scene.
a. Brachial plexus injury He was intubated and in the ambulance his
b. Carotid dissection right pupil dilates to size 6 and is unreactive
c. Carotid thrombosis to light.
d. Cavernous sinus lesion 2. A 47-year-old type 1 diabetic develops
e. Cluster headache diplopia. On examination he has a left
f. Head and neck surgery ptosis and weakness of superior rectus.
g. Hypothalamic injury There is no anisocoria or abnormal pupil-
h. Intraoral trauma lary reflex.
i. Lateral medullary stroke
j. Midbrain injury
k. Neuroblastoma QUESTIONS 28–30
l. Pancoast tumor
m. Pontine injury Additional questions 28–30 available on
ExpertConsult.com
n. Small vessel ischemia
o. Spinal cord lesion

Neurosurgery Books Full


www.ketabpezeshki.com 66485438-66485457
170 PART II CARE OF THE NEUROSURGICAL PATIENT

SBA ANSWERS
1. a—Pontine paramedian reticular formation MLF cause ipsilateral eye adduction failure.
The contralateral eye can abduct, but with nys-
Gaze is a complex eye movement that is mainly tagmus. In lateral gaze palsy, lesions in the PPRF
controlled by the combination of CNs III, IV, cause ipsilateral eye abduction and contralateral
and VI. Gaze can be simply classified as lateral, eye adduction failure. Lateral gaze to the contra-
vertical, or conjugate. The frontal eye field (Area lateral side is preserved in these cases. Lesions in
8) and supplementary eye field (Area 6) are both the MLF and the PPRF result in ‘one-
regions of the cerebral cortex that control gaze, and-a-half’ syndrome with ipsilateral eye
and they innervate the contralateral paramedian abduction and adduction failure and contralateral
pontine reticular formation (PPRF). Fibers from eye adduction failure. The contralateral eye can
the PPRF reach the ipsilateral CN VI nucleus and abduct, but with nystagmus. The control center
contralateral CN III nucleus through the contra- for vertical gaze is located at the rostral interstitial
lateral MLF to control lateral gaze. Lateral gaze nucleus of the MLF, near the CN III nucleus
palsies are classified as internuclear ophthalmo- in the midbrain. In addition, it has been suggested
plegia (INO), lateral gaze palsy, or one-and-a- that a center for convergence is located near
half syndrome, and the corresponding lesions CN III nucleus. A lesion in the midbrain can
are located in the MLF, PPRF, and both MLF cause vertical gaze palsy and convergence palsy,
and PPRF, respectively. In INO, lesions in the and this is known as Parinaud syndrome.

MLF MLF MLF

III III III III III III

VI VI VI VI VI VI

PPRF PPRF PPRF PPRF PPRF PPRF

Image with permission from Sakai K, Yokota H, Akazawa K, Yamada K. Brainstem white matter tracts and the control
of eye movements, Semin Ultrasound CT MR. Oct;35(5):517-26, 2014.

2. d—Idiopathic intracranial hypertension affected only very late in its course. IIH typically
occurs in obese women between puberty and men-
Elevated intracranial pressure can be transmitted to opause. Men and non-obese women frequently
the optic nerve head and usually results in bilateral harbor identifiable causes of elevated intracranial
optic disc swelling. The optic disc may be markedly hypertension (e.g. Chiari malformation, occult
elevated, with hemorrhages of the nerve head and dural arteriovenous malformation, dural venous
surrounding retina. Usually, there is no visual loss sinus occlusion, sleep apnea) and should not be con-
acutely, except for enlargement of the physiologic sidered to have IIH until an exhaustive clinical
blind spot from elevation and compression of the investigation confirms that there is no identifiable
peripapillary retina. Chronic papilledema can cause cause. Fundoscopic features of papilledema are
loss of the peripheral visual field, with central vision highlighted below.

Neurosurgery Books Full


www.ketabpezeshki.com 66485438-66485457
9 NEURO-OPHTHALMOLOGY 171

the superior cervical ganglion by means of the


Papilledema
(Common) Acute Chronic carotid plexus sympathetic nerves to the pupillary
dilator. Postganglionic interruption (at or distal
Disc Disc Champagne cork to the superior cervical ganglion) is commonly
elevation hyperemia appearance
benign or idiopathic, but preganglionic or central
Venous Cotton wool Pseudodrusen (gliosis oculosympathetic pareses are associated with
distension/ spots and extruded malignancy in about half the cases.
tortuosity axoplasm)
Pharmacologic evaluation in Horner's
Blurred disc Peripapillary Disc atrophy syndrome
margin hemorrhages (pale) (1) Apraclonidine 0.5% is a direct alpha recep-
Absent Venous tor agonist. It does not cause pupil dilata-
venous collaterals tion in the presence of an intact
pulsations Peripapillary sympathetic innervation, but will dilate
subretinal
neovascularization any pupil with sympathetic denervation.
In Horner's it will thus reverse the aniso-
coria and elevate the ptotic lid.
Image with permission from Kaiser PK, Friedman NJ. (2) Cocaine 10% dilates the normal (non-
Case Reviews in Ophthalmology, Elsevier, Saunders, Horner's) pupil by blocking the reuptake
2012. of norepinephrine into sympathetic nerve
3. a—Left internuclear ophthalmoplegia endings producing a prolonged mydriasis
over 45 min. Due to the lack of norepi-
Image with permission from Kaiser PK, Friedman NJ. nephrine in sympathetic denervated
Case Reviews in Ophthalmology, Elsevier, Saunders,
2012.
pupil (Horner's) blocking reuptake does
not produce a mydriasis. Persistence/wors-
4. b—Myasthenia gravis ening of anisocoria establishes the diagnosis.
(3) Hydroxyamphetamine actively releases
Ocular myasthenia gravis can mimic an isolated or norepinephrine from adrenergic nerve
combined neuropathy of CN III (except pupil endings to dilate a normal pupil. In Hor-
involvement), IV, or VI, but often presents with ner's syndrome due to first- or second-
diplopia or ptosis that is not easily categorized. order neuron damage, intact third-order
Ocular involvement is seen early and may be the neurons will respond to hydroxyampheta-
only system affected in some patients, with an mine as those in a normal pupil and dilate.
inability to forcefully close their eyes against resis- However, with Horner's due to a third-
tance. Patients frequently report worsening symp- order (postganglionic) lesion, the damaged
toms as the day progresses and relief after a nap or nerve endings cannot release norepineph-
rest. The diagnosis is sometimes difficult to make rine and the pupil will not dilate.
because the results of edrophonium chloride Image with permission from Kaiser PK, Friedman NJ. Case
(Tensilon) testing, serum acetylcholine receptor Reviews in Ophthalmology, Elsevier, Saunders, 2012.
antibody assay, repetitive electromyographic
nerve stimulation, or trials of pyridostigmine bro- 6. d—Optic neuritis
mide (Mestinon) may be equivocal. The ice test is
also a useful screen. Single-fiber electromyogra- UK (6 m) 6/6 6/9 6/12 6/18 6/24 6/36 6/60
phy has the greatest sensitivity and specificity US (20 ft) 20/20 20/30 20/40 20/60 20/80 20/120 20/200
but is technically difficult. CT of the thorax is per-
formed to exclude a thymoma.
7. a—Left abducens nerve palsy
Image with permission from Kaiser PK, Friedman NJ.
Case Reviews in Ophthalmology, Elsevier, Saunders, Trauma is a common cause of abducens palsy in
2012. all age groups. Acute, painful abducens paresis in
a patient 45 years or older who has hypertension
5. a—Horner's syndrome
or diabetes suggests an ischemic cranial mono-
Right Horner's syndrome with right ptosis, miosis neuropathy. Recovery over a period of 2-6 months
and facial anhidrosis due to right lateral medullary is the rule. In young patients, abducens palsies may
infarction. Relative miosis and a slight ptosis (1-2 occur as a postviral syndrome, but the clinician
mm) may result from interruption of the oculosym- must remain alert for the possibility of tumor.
pathetic pathway anywhere along the three-neuron Elevation of intracranial pressure can affect the
chain: from the hypothalamus to the ciliospinal function of one or both VI nerves.
center of Budge-Waller (C8–T1), across the lung Image with permission from Kaiser PK, Friedman NJ. Case
apex to the superior cervical ganglion, or from Reviews in Ophthalmology, Elsevier, Saunders, 2012.

Neurosurgery Books Full


www.ketabpezeshki.com 66485438-66485457
172 PART II CARE OF THE NEUROSURGICAL PATIENT

8. d—Parinaud syndrome 12. c—Right Adie's tonic pupil

Dorsal midbrain syndrome (Parinaud's syndrome) The right pupil is mid-sized and larger than the
consists of upgaze palsy, convergence spasm or left, poorly reactive to light, but reactive to near
paresis, bilateral mid-dilated pupils with light- stimulus (light-near dissociation). It constricted
near dissociation (poor reaction to light and good with 0.1% pilocarpine while the left pupil did
reaction to near), and convergence-retraction not indicating denervation hypersensitivity on
nystagmus. The nystagmus is unique to this syn- the right. Adie's (or Holmes-Adie's) syndrome which
drome and consists of rapid convergence with is a symptom complex consisting of tonic pupil(s)
retraction of the globes on attempted upgaze. and absent deep tendon reflexes and is the com-
Common causes include pinealomas, aneurysms monest cause. Viral or bacterial infection causing
of the vein of Galen, and noncommunicating inflammation to the ciliary ganglion damages para-
hydrocephalus with distention of the third ventri- sympathetic postganglionic input to the eye, and to
cle and the anterior aqueduct of Sylvius. This syn- dorsal root ganglia in the spine. The use of stronger
drome may also occur with stroke as one of the 1% pilocarpine would be effective in constricting
“top of the basilar” syndromes. normal pupils, as well as third nerve-related mydri-
asis, tonic pupils, and other pre- and postganglionic
Image with permission from Liu GT. Neuro- parasympathetic disorders because in these cases
Ophthalmology: Diagnosis and Management, 2nd ed.,
Elsevier, Saunders, 2010. the receptors at the iris constrictor muscle are either
normal or hypersensitive. Pupils dilated with anti-
9. b—Urinary catecholamines cholinergic agents such as atropine, tropicamide, or
cyclopentolate or sympathomimetic agents such as
Horner's syndrome in children may be caused by phenylephrine or neosynephrine, are generally
neuroblastomas, and 50% are adrenal in origin. large (> 7-8 mm) and unreactive to light or near
stimulation.
Image with permission from Liu GT. Neuro-
Ophthalmology: Diagnosis and Management, 2nd ed., Image with permission from Liu GT. Neuro-
Elsevier, Saunders, 2010. Ophthalmology: Diagnosis and Management, 2nd ed.,
Elsevier, Saunders, 2010.
10. a—Anterior ischemic optic neuropathy
13. b—Argyll Robertson pupils
This is the commonest cause of acute visual loss
in older patients, and thought to be due to ischemia Here the pupils are bilaterally small and show light-
in posterior circulation of the gobe—predomi- near dissociation (“accommodate but do not
nantly vessels supplying the optic nerve at its exit react”). Apart from pupil size, the other main way
from the eye. It is classified as non-arteritic or arteri- of differentiating between Argyll Robertson pupils
tic, the former being much more common. In all and Adie's pupil is their speed of constriction to
cases of AION, it is vital to determine whether there near vision—immediate in the former and slow/
is any evidence of an arteritic cause (e.g., giant cell prolonged in the latter. In tabes dorsalis (demyelin-
arteritis). Untreated giant cell arteritis can cause ation secondary to syphilis) Argyll Robertson pupils
rapid, sequential, or simultaneous blindness in both are associated with absent deep tendon reflexes, loss
eyes. If giant cell arteritis is suspected because of an of vibratory sense and proprioception in the lower
elevated ESR or CRP, or both, or because extremities, and Charcot joints. Lesion locations
of symptoms such as headache, scalp and temple of the various types of light-near dissociation are
tenderness, myalgias, arthralgias, low-grade fever, outlined below.
anemia, malaise, weight loss, anorexia, or jaw clau-
dication, oral or intravenous steroid treatment Cause of Light-Near
should be instituted immediately. In acute cases, Dissociation Lesion
the patient may benefit from high-dose intravenous
Adie's tonic pupil Ciliary ganglion
steroids. Biopsy of the temporal artery should follow
within days of steroid therapy. In arteritic AION, Optic neuropathy/ Anterior visual pathway
the visual loss is usually profound, and the optic Severe retinopathy
nerve is often diffusely swollen and pale. Argyll Robertson pupil Lesion of midbrain
tectum
11. b—Congenital Parinaud syndrome Lesion of midbrain
tectum
Iris heterochromia in idiopathic congenital
Horner's syndrome. Aberrant regeneration Aberrant innervation of
of CNIII papillary fibers
Image with permission from Liu GT. Neuro- Peripheral neuropathy Short posterior ciliary
Ophthalmology: Diagnosis and Management, 2nd ed., nerves
Elsevier, Saunders, 2010.
Neurosurgery Books Full
www.ketabpezeshki.com 66485438-66485457
9 NEURO-OPHTHALMOLOGY 173

Image with permission from Liu GT. Neuro- muscle; the clinician looks at how well the right
Ophthalmology: Diagnosis and Management, 2nd ed., eye can move down in left gaze and how well
Elsevier, Saunders, 2010. the left eye moves down in right gaze. The vital
role of the superior oblique muscle in ocular cyclo-
14. c—Orbital apex syndrome
torsion explains why patients with trochlear palsy
often describe diplopia with one image tilted.
Cause Description One can occasionally elicit which fourth nerve is
affected by asking the patient to look at a horizon-
Cavernous sinus CN III, IV, VI palsy
syndrome Pain in distribution of tal straight object (e.g. a pen). If the patient sees
V1-2 (and V3 is posterior sinus two images, they can be asked to describe how they
involved) intersect. A patient with a unilateral fourth nerve
Horner's syndrome palsy will see a horizontal line and a tilted line
(sympathetic fibers on ICA, VI,
V1)
below it, intersecting on the side of the abnormal
Lid malposition (ptosis or lid eye (the arrow points to the side of the affected
edema) fourth nerve).
Mixed CNIII palsy and
Horner's gives a small or mid- Image with permission from Liu GT. Neuro-
dilated poorly reactive pupil. Ophthalmology: Diagnosis and Management, 2nd ed.,
Elsevier, Saunders, 2010.
Superior orbital As for cavernous sinus
fissure syndrome syndrome but never has
V3 involvement; also no optic
16. c—Left oculomotor nerve palsy
nerve involvement
The left eye has complete ptosis, defective
Orbital apex Optic nerve (RAPD/visual elevation, absent adduction, a down and out posi-
syndrome loss), III, IV, VI, V1
(hypoesthesia in ophthalmic tion in primary gaze with a large unreactive pupil,
division); proptosis common intact abduction, and deficient downgaze. The
oculomotor nerve innervates all the extraocular
muscles (including the levator palpebrae), except
for the lateral rectus (VI) and superior oblique
muscles (IV). It also carries parasympathetic
Image with permission from Liu GT. Neuro- efferent fibers to the pupillary sphincter and
Ophthalmology: Diagnosis and Management, 2nd ed., ciliary muscle through the ciliary ganglion.
Elsevier, Saunders, 2010. Complete third cranial nerve palsy produces an
eye that is turned down and out (because of
15. b—Left fourth nerve palsy
remaining function of the superior oblique and
lateral rectus muscles) and a dilated pupil with
The main causes are head trauma, microvascular
infarction, congenital, tumors and demyelination. ptosis. Partial or incomplete paresis may present
It is the only cranial nerve which exits the brain- a more confusing picture. The pupillary fibers
stem dorsally (has the longest intracranial course), run superficially in the nerve and are preferen-
tially affected by compression, such as from a pos-
it decussates to supply the contralateral superior
oblique muscle and is enveloped in the anterior terior communicating artery aneurysm or uncal
medullary velum where it is vulnerable to head herniation. Ischemic cranial mononeuropathy of
the vasa nervosum of the oculomotor nerve
trauma. Assessment of a trochlear nerve palsy is
challenging, but a three step test is best used: generally shows relative pupillary sparing with
(1) Which eye is hypertropic in primary gaze? the pupil being less affected than motility. Aber-
rant regeneration after disruption of the axons
(2) Is the hypertropia worse in left or right gaze?
(3) Is the hypertropia worse in left or right of the oculomotor nerve may produce clinical
head tilt? signs pupillary constriction or lid elevation with
The left hypertropia, which is worse in right gaze attempted adduction or paradoxical motility.
and left head tilt, is consistent with weakness of Aberrant regeneration never occurs with an
the left superior oblique muscle. The trochlear ischemic (diabetic) mononeuropathy and always
nerve innervates the superior oblique muscle and implies that the nerve has been injured in such
because of its redirection at the trochlea and atta- a way that the myelin sheath and perineurium
chment to the globe it intorts, depresses, and have been breached (i.e., aneurysm, tumor, or
abducts the eye. When the globe is adducted, how- trauma).
ever, the angle of the muscle's insertion minimizes Image with permission from Liu GT. Neuro-
all actions except pure depression. This fact sim- Ophthalmology: Diagnosis and Management, 2nd ed.,
plifies clinical evaluation of the superior oblique Elsevier, Saunders, 2010.

Neurosurgery Books Full


www.ketabpezeshki.com 66485438-66485457
174 PART II CARE OF THE NEUROSURGICAL PATIENT

17. a—Bilateral internuclear ophthalmoplegia should constrict equally irrespective of which


(INO) eye is being illuminated (intact direct and consen-
sual pupillary reflexes). Marcus-Gunn pupil is a
In INO, convergence may or may not be intact. relative afferent pupillary defect (RAPD) indicat-
Severe bilateral INOs may also result in a exotro- ing a decreased pupillary response to light in the
pia in primary gaze termed wall-eyed bilateral affected eye, such that during the test the pupil
INO (WEBINO). in the affected eye experiences a relative dilatation
during direct illumination.
Image with permission from Liu GT. Neuro-
Ophthalmology: Diagnosis and Management, 2nd ed.,
Elsevier, Saunders, 2010.
26. 1—l, Pancoast tumor; 2—d, Cavernous sinus
lesion; 3—k, Neuroblastoma
18. d—Left one-and-a-half syndrome

On attempted right gaze, the patient has a left


internuclear ophthalmoplegia (defective adduc- 1st order neuron
tion of the left eye and abducting nystagmus of
the right eye). On attempted left gaze there is a
conjugate gaze paresis so that neither eye can 3rd order neuron
move past midline. This is due to a lesion in the to pupil dilator
and lid
pons affecting the left PPRF and MLF.
Image with permission from Liu GT. Neuro-
Ophthalmology: Diagnosis and Management, 2nd ed.,
3rd order neuron
Elsevier, Saunders, 2010.
to facial
sweat glands
ANSWERS 19–23
2nd order neuron
Additional answers 19–23 available on
ExpertConsult.com Common
carotid artery
Subclavian artery

Apex of lung
EMI ANSWERS
Images with permission from Liu GT. Neuro-
24. 1—e, Kearns-Sayre syndrome is a form of Ophthalmology: Diagnosis and Management, 2nd ed.,
Elsevier, Saunders, 2010.
chronic progressive external ophthalmople-
gia (CPEO; mitochondrial myopathies) with
The sympathetic innervation of the eye con-
onset before the age of 20 years. In addition
sists of three neurons connected in series: first-
to ophthalmoplegia, pigmentary retinopathy
order neurons, second-order neurons, and
and complete heart block they may also have
third-order neurons. The first-order neurons
cerebellar ataxia, dementia, deafness, short
(central neurons) extend from the posterior
stature and endocrine disturbance.
hypothalamus to the C8 to T2 level of the spinal
2—f, Miller Fisher syndrome (variant of Guillain- cord. The second-order neurons (preganglionic
Barré syndrome), neurons) leave the spinal cord and travel over
the lung apex, around the subclavian artery,
3—k, Steele-Richardson-Olszewski syndrome and along the carotid artery to the superior cer-
(Progressive supranuclear palsy) is an neurode- vical ganglion. The third-order neurons (post-
generative disorder and included as one of the ganglionic neurons) diverge and take two
Parkinson plus syndromes. paths: Those to the pupil and lid muscles travel
along the internal carotid artery through the cav-
25. 1—g, Parinaud syndrome; 2—f, Marcus- ernous sinus to reach the orbit; those to the facial
Gunn pupil sweat glands travel with the external carotid
artery to the face. Lesions in any of these neu-
In the swinging flashlight test, a light is alternately rons cause Horner syndrome and distinct associ-
shone into the left and right eyes. Both pupils ated physical signs.

Neurosurgery Books Full


www.ketabpezeshki.com 66485438-66485457
9 NEURO-OPHTHALMOLOGY 175

27. 1—m, Uncal herniation; 2—j, Peripheral


Horner's Syndrome: Clinical Features neuropathy
Associated
Neuron Cause of Horner's Symptoms
Oculomotor (III) Palsy: Causes
First- CVA Contralateral
order MS hemianesthesia Cause Comments
Vertebral artery Contralateral
dissection hemiparesis Infarction Microvascular infarction usually
VI palsy involves axial portion—pupil
Wallenberg spared.
syndrome Diabetes, hypertension,
hyperlipidemia, atherosclerosis.
Second- Schwannoma Hoarse voice
order Lung/breast tumor Cough Nucleus/ Weber syndrome
Trauma/surgery Scapular pain fascicular Nothnagel syndrome
Epidural anesthetic lesion Benedikt syndrome

Third- Carotid artery Pain Subarachnoid Posterior communicating artery


order dissection Reduced taste space aneurysm
Neck trauma Dysphagia Uncal herniation
Neck tumors/ Palatal
inflammation hemianesthesia Cavernous Cavernous sinus syndrome
Cluster headache Headache sinus involving III, IV, VI, V1 and V2.
Raeder's Involvement of III, Orbit Superior division—levator
paratrigeminal IV, V, VI palpebrae superioris, superior
neuralgia rectus
Cavernous sinus Inferior division—pupillary
lesions constrictor, other recti and
inferior oblique

ANSWERS 28–30

Additional answers 28–30 available on


ExpertConsult.com

Neurosurgery Books Full


www.ketabpezeshki.com 66485438-66485457
CHAPTER 10

NEURO-OTOLOGY
SINGLE BEST ANSWER (SBA) QUESTIONS
1. Which of the right ear pure tone audiograms shown below most resemble the pattern expected with
Meniere's disease?
2,896 5,792 11,584 2,896 5,792 11,584
128 256 512 1,024 2,048 4,096 8,192 128 256 512 1,024 2,048 4,096 8,192
–20 –20

0 0

20 20

40 40

60 60

80 80

100 100

a b

2,896 5,792 11,584 2,896 5,792 11,584


128 256 512 1,024 2,048 4,096 8,192 128 256 512 1,024 2,048 4,096 8,192
–20 –20

0 0

20 20

40 40

60 60

80 80

100 100

c d
2,896 5,792 11,584 2,896 5,792 11,584
128 256 512 1,024 2,048 4,096 8,192 128 256 512 1,024 2,048 4,096 8,192
–20 –20

0 0

20 20

40 40

60 60
Air conduction (AC)
80 80
Bone conduction (BC)
100 100
e f

2. Vestibular schwannoma (acoustic neuroma) 3. Vestibular-ocular reflex elicited during a


most commonly affects which one of the fol- right head turn is best described by which
lowing nerves? one of the following?
a. Cochlear nerve a. Relative motion between membranous
b. Facial nerve labyrinth and endolymph causes right
c. Inferior vestibular nerve horizontal canal cupula to deflect towards
d. Superior vestibular nerve the utricle with reflex movement of eyes to
e. Trigeminal nerve left with saccades to right

176
Neurosurgery Books Full
www.ketabpezeshki.com 66485438-66485457
10 NEURO-OTOLOGY 177

b. Relative motion between membranous d. Resection of acoustic neuroma in a deaf


labyrinth and endolymph causes left hor- patient
izontal canal cupula to deflect towards the e. Vestibular neurectomy for intractable
utricle with reflex movement of eyes to tinnitus
left with saccades to right
c. Relative motion between membranous 7. Damage to hair cells in the basal turn of the
labyrinth and endolymph causes right cochlea is likely to result in which one of the
horizontal canal cupula to deflect towards following?
the utricle with reflex movement of eyes to a. High frequency hearing loss
left with saccades to left b. High-intensity hearing loss
d. Relative motion between membranous c. Low-frequency hearing loss
labyrinth and perilymph causes right hor- d. Low-intensity hearing loss
izontal canal cupula to deflect towards the e. Mid-frequency hearing loss
utricle with reflex movement of eyes to
left with saccades to right 8. Which one of the following sites in the inner
e. Relative motion between membranous ear does gentamicin exert its ototoxic effect?
labyrinth and perilymph causes right hor- a. Apical turn of cochlea
izontal canal cupula to deflect away from b. Cochlear nerve
the utricle with reflex movement of eyes c. Hair cells
to right with saccades to right d. Macula densa
e. Striavascularis
4. In the brainstem auditory evoked response,
which one of the following structures gives 9. A 34-year-old female presents with a House-
rise to wave V? Brackmann grade IV facial palsy. Which one
a. Cochlear nerve of the following best describes the clinical
b. Inferior colliculus findings?
c. Lateral lemniscus a. Complete facial paralysis
d. Superior olivary complex b. Obvious asymmetry (not disfiguring);
e. Ventral cochlear nucleus noticeable synkinesis, contracture, or
hemifacial spasm; complete eye closure
5. A 25-year-old male had occasional difficulty with effort.
in understanding speech over the telephone c. Obvious weakness or disfiguring asymme-
with his left ear. Brainstem auditory evoked try; normal symmetry and tone at rest;
response is shown for both ears. Which one incomplete eye closure.
of the following is most likely? d. Only barely perceptible motion with
asymmetry at rest
I
IV–V e. Slight weakness noticeable on close
IV–V
I
II III III inspection; slight synkinesis

10. Which one of the following best describes


RIGHT LEFT
the target region for an auditory brainstem
implant?
0 2 4 6 8 10 0 2 4 6 8 10
a. Cochlear nucleus
Time in msec
b. Inferior colliculus
c. Inferior olivary nucleus
d. Superior olivary nucleus
a. Aminoglycoside toxicity e. Vestibular nucleus
b. Gentamicin ototoxicity
c. Left acoustic neuroma 11. A 35-year-old NF-2 patient has sensorineu-
d. Right cochlear ischemia ral hearing loss and paresthesia of the poste-
e. Right glomus jugulare tumor rior aspect of his right ear canal. MRI shows a
large cerebellopontine angle tumor. Com-
6. In which one of the following situations is it pression of which one of the following best
NOT appropriate to use intraoperative explains the altered sensation?
brainstem auditory evoked response a. Facial nerve
monitoring? b. Glossopharyngeal nerve
a. Basilar artery aneurysm surgery c. Inferior vestibular nerve
b. Microvascular decompression of CN V d. Superior vestibular nerve
c. Posterior fossa surgery e. Vagus nerve

Neurosurgery Books Full


www.ketabpezeshki.com 66485438-66485457
178 PART II CARE OF THE NEUROSURGICAL PATIENT

12. Which of the labels below refers to the c. Electrically evoked auditory potentials
modiolus? d. Masking
e. Otoacoustic emission
f. Play audiogram
g. Pure-tone audiometry
h. Speech recognition threshold
i. Tympanometry

For each of the following descriptions, select the


most appropriate answers from the list above.
Each answer may be used once, more than once
a or not at all.
e
b 1. A 9-year-old girl who is struggling to hear
c f
d
in classes at school.
g
2. A 4-year-old boy whose parents are con-
h cerned he can't hear them when not in
the same room.
3. A neonate with a family history of sensori-
neural deafness.
13. Which one of the following is most accurate 4. A 1-year-old who is being considered for
regarding caloric testing in the right ear? cochlear implantation.
a. Cold water irrigation causes endolymph in
lateral portion to become dense and fall 15. Hearing Loss:
pulling the right horizontal canal cupula a. Acoustic neuroma
away from the utricle, reducing the firing b. Acute otitis media
rate and causes a nystagmus with fast c. Genetic
phase to the left. d. Glue ear (otitis medial with effusion)
b. Cold water irrigation causes endolymph in e. Meniere's disease
lateral portion to become dense and fall f. Meningitis
pulling the right horizontal canal cupula g. Noise-induced hearing loss
away from the utricle, reducing the firing h. Ossicular dislocation
rate and causes a nystagmus with fast phase i. Otosclerosis
to the right. j. Ototoxicity
c. Cold water irrigation causes perilymph in k. Perilymph fistula
lateral portion to become dense and fall l. Presbycusis
pushing the right horizontal canal cupula m. Tympanosclerosis
towards the utricle, reducing the firing
rate and causes a nystagmus with fast For each of the following descriptions, select the
phase away from the stimulus. most appropriate answers from the list above.
d. Warm water irrigation causes endolymph Each answer may be used once, more than once
in lateral portion to become less dense and or not at all.
fall pulling the horizontal canal cupula 1. A 24-year-old man is involved in a road
away from the utricle, reducing the firing traffic accident and sustains a significant
rate and causes a nystagmus with fast head injury requiring admission to inten-
phase away from the stimulus. sive care. After discharge to the ward he
e. Warm water irrigation causes perilymph reports hearing loss in his left ear. Pure tone
in lateral portion to become dense and fall audiogram shows a 50 dB conductive hear-
pulling the horizontal canal cupula away ing loss across all frequencies.
from the utricle, reducing the firing rate 2. A 57-year-old with progressive bilateral
and causes a nystagmus with fast phase hearing loss over several years. Audiogram
away from the stimulus. shows sensorineural hearing loss particu-
larly at the 4 kHz frequency.
3. A 40-year-old woman sustained a head
EXTENDED MATCHING ITEM (EMI) injury following a fall from a pushbike.
QUESTIONS Since then she has experienced intermit-
tent hyperacusis and fullness in her left
14. Audiometry: ear, with dizziness. These are especially
a. Acoustic reflex particularly worse after heavy lifting or
b. Auditory brainstem evokes response coughing.

Neurosurgery Books Full


www.ketabpezeshki.com 66485438-66485457
10 NEURO-OTOLOGY 179

16. Genetic hearing loss: 18. Anatomy:


a. Alport syndrome
c b a
b. Apert syndrome
c. Crouzon syndrome q p o

d. Hersh syndrome n

e. Jervell Lange-Nielsen syndrome


m
f. Neurofibromatosis type II
g. Pendred syndrome d

h. Pierre-Robin sequence l
k
i. Refsum syndrome
j. Seckel syndrome j

k. Treacher-Collins syndrome e f
i
l. Usher syndrome g
h
m. Waardenburg syndrome

For each of the following descriptions, select the For each of the following descriptions, select the
most appropriate answers from the list above. most appropriate answers from the image above.
Each answer may be used once, more than once Each answer may be used once, more than once or
or not at all. not at all.
1. A child with iris heterochromia, hyperte- 1. Scala vestibuli
lorism and white forelock born with 2. Scala media
hearing loss. 3. Organ of Corti
2. A child with hematuria and high frequency 4. Tectorial membrane
hearing loss 5. Oval window
3. A child born with profound deafness and
prolonged QT interval. 19. Cochlear anatomy:
17. Dizziness:
a i
a. Benign paroxysmal positional vertigo
b
b. Central vestibular dysfunction
c. Head injury Cochlear
duct
d. Labyrinthine fistula Internal
spiral
c
e. Meniere's disease Inner
tunnel
tunnel

f. Migraine
g. Psychiatric disease d Tympanic Nerve
h. Vertebro-basilar ischemia cavity
i. Vestibular neuronitis
e f g h
j. Vestibular schwannoma

For each of the following descriptions, select the For each of the following descriptions, select the
most appropriate answers from the list above. most appropriate answers from the image above.
Each answer may be used once, more than once Each answer may be used once, more than once
or not at all. or not at all.
1. A 54-year-old who has had two previous 1. Inner hair cell
episodes of acute vertigo lasting a few 2. Outer hair cells
hours only, each preceded by hearing 3. Tectorial membrane
loss, tinnitus and aural fullness in the 4. Osseous spiral lamina
right ear.
2. A 72-year-old woman who has a 3 month 20. Audiogram symbols:
history of vertigo when turning her head a. O
in bed, each lasting less than 1 min. Pure b. X
tone audiogram shows symmetrical bilat- c. ]
eral high frequency hearing loss. d. [
3. A 65-year-old man presenting with e. >
progressive unsteadiness, intermittent ver- f. <
tigo and right sided tinnitus. Pure tone g. □
audiogram shows asymmetrical right senso- h. ↘
rineural hearing loss. i. ↙
j. Δ

Neurosurgery Books Full


www.ketabpezeshki.com 66485438-66485457
180 PART II CARE OF THE NEUROSURGICAL PATIENT

For each of the following descriptions, select the


most appropriate answers from the list above. QUESTIONS 21–22
Each answer may be used once, more than once Additional questions 21–22 available on
or not at all. ExpertConsult.com
1. Right ear, air conduction, masked
2. Left ear, air conduction, masked
3. Right ear, bone conduction, unmasked

SBA ANSWERS
1. c—Sensorineural hearing loss predomi- 4. b—Inferior colliculus
nantly of the lower frequencies may be seen
dB
in Meniere's disease II III IV V
I 80
VI
a. Normal right audiogram. b. Sensorineural
hearing loss: the commonest cause is presbyacu-
sis, with usually high-frequency loss. c. Sensori- 60
µ Volts

neural hearing loss predominantly of the lower


frequencies may be seen in Meniere's disease. 40
d. Mixed conductive and sensorineural hearing
loss: seen in patients with a combination of pres- 20
byacusis and middle ear pathology, or in a peri-
lymph fistula. e. Conductive hearing loss: the
difference between AC and BC demonstrates Time (msec)
the conductive loss. f. Noise-induced hearing Image with permission from Winn HR. Youman's
loss: commonly affects the frequencies around Neurological Surgery, 4-Volume Set, 6th ed. Elsevier,
Saunders, 2011.
4 kHz initially.
Image with permission from Dhillon R, East CA. Ear,
Nose and Throat and Head and Neck Surgery, 4th ed. The main indication for BAER is when an acoustic
Elsevier, Churchill Livingstone, 2012. neuroma is suspected due to asymmetrical senso-
rineural hearing loss. BAER testing is more cost
2. d—Superior vestibular nerve effective than MRI, but MRI provides additional
information. The most reliable indicator for
Superior vestibular nerve supplies the superior acoustic neuromas from the BAER is the increased
semicircular canal, lateral semicircular canal and interaural latency in wave V. Sensitivity of BAER
the utricle. Inferior vestibular nerve supplies the for acoustic neuroma is approximately 90%.
posterior semicircular canal and saccule

3. a—Relative motion between membranous


labyrinth and endolymph causes right hori-
zontal canal cupula to deflect towards utricle Wave Source
(and left away) with reflex movement of eyes I Cochlear nerve (first-order neuron)
to left with saccades to right.
II Cochlear nucleus

III Superior olivary complex (first-order


neuron)

IV Lateral lemniscus

V Inferior colliculus (first-order neuron)


VI/VII Not routinely studied—medial geniculate,
auditory radiation

Neurosurgery Books Full


www.ketabpezeshki.com 66485438-66485457
10 NEURO-OTOLOGY 181

5. c—Left acoustic neuroma


House-Brackmann Classification of VII
Preoperative deafness on the operative side elim- Palsy
inates the possibility of recording intraoperative
Grade Description
BAEPs. Cases in which BAEPs are commonly
monitored include microvascular decompression I Normal facial function
of cranial nerves (especially V and VII), resection
II Slight weakness noticeable on close
of acoustic neuroma, posterior fossa exploration inspection; slight synkinesis
for vascular or neoplastic lesions, clipping of bas-
ilar artery aneurysm, and section of nerve VIII for III Obvious asymmetry (not disfiguring);
intractable tinnitus. Changes in BAER can be noticeable synkinesis, contracture, or
caused by section/retraction of CNVIII, cerebel- hemifacial spasm; complete eye closure
with effort.
lum, or brain stem; hypotension and hypocarbia;
drilling around the internal auditory canal; irriga- IV Obvious weakness or disfiguring
tion of nerve VIII; severe cerebellar edema; and asymmetry; normal symmetry and tone at
positioning of the head for retromastoid craniot- rest; incomplete eye closure.
omy. Patients with transient or persistent V Only barely perceptible motion with
increases in latency or decreases in amplitude asymmetry at rest
can be expected to have unchanged or only slight
worsening of hearing postoperatively. Patients VI Complete facial paralysis
with complete but reversible loss of BAEP will
also have unchanged or mild worsening of hear-
ing postoperatively. Patients with complete irre- 9. c—Obvious weakness or disfiguring asym-
versible loss of BAEP will most likely have metry; normal symmetry and tone at rest;
complete or near complete loss of hearing in incomplete eye closure.
the ipsilateral ear postoperatively.
10. a—Cochlear nucleus
Image with permission from Winn HR. Youman's Neu-
rological Surgery, 4-Volume Set, 6th ed. Elsevier, Saun-
ders, 2011. 11. a—Facial nerve

6. d—Resection of acoustic neuroma in a deaf Altered sensation in the posterior aspect of the
patient external auditory canal (Hitzelberger's sign) is
secondary to compression of sensory fibers in
7. a—High frequency hearing loss the nervusintermedius branch of VII.

8. c—Hair cells. Gentamicin ototoxicity results 12. a—Modiolus, the conical central axis of the
in high tone sensorineural deafness through cochlea of the ear.
its effects on outer hair cells in basal turn of Image with permission from Lowe JS, Anderson PG.
the cochlea. Stevens & Lowe's Human Histology, 4th ed. Elsevier,
Mosby, 2015.

13. a—Cold water irrigation causes endolymph


in lateral portion to become dense and fall
pulling the right horizontal canal cupula
away from the utricle, reducing the firing rate
and causes a nystagmus with fast phase to
the left.

Neurosurgery Books Full


www.ketabpezeshki.com 66485438-66485457
182 PART II CARE OF THE NEUROSURGICAL PATIENT

EMI ANSWERS
14. 1 ¼ g, Pure-tone audiometry, 2 ¼ f, Play audiometry, 3 ¼ e, Otoacoustic emission testing, 4 ¼ b,
Auditory brainstem response

Audiometric Tests

Tympanometry Relative change in impedance with a change in ear canal air pressure at the plane of the
tympanic membrane. The tympanogram provides indirect evidence of the mechanical
integrity of middle ear structures

Acoustic reflex Measurement of the reflex stapedius muscle contraction bilaterally on presentation
of an acoustic stimulus. In patients with a retrocochlear site of the lesion (cranial
nerve VIII and low brainstem), the acoustic reflex may be elevated or absent

Pure-tone audiometry For adults and older children (>5 years). Using headphones, each ear is tested
individually for air conduction and, if necessary, bone conduction thresholds. The
results are usually plotted as a graph. A result of 0 dB (decibels) is the average
normal threshold for hearing in young adults

Masking When a patient has a substantial difference in hearing sensitivity between the two
ears, it is necessary to rule out the potential participation of the better hearing ear
when testing the poorer hearing ear. Masking is defined as the amount by which the
threshold of audibility of a sound is raised by the presence of another (masking)
sound

Speech recognition Measurement of the hearing sensitivity/threshold (in dB) for speech
threshold

Speech recognition score Measurement of the speech recognition (discrimination) ability when presented
above the SRT. Those with conductive hearing loss typically score high, whereas
those with sensorineural hearing loss show decreased discrimination. When the
conductive mechanism is normal but lesions of the auditory system affect the
cochlear or retrocochlear structures, the ability to understand the consonant
elements of speech is affected. When the cochlear structures are normal but cranial
nerve VIII or low-brainstem structures are affected speech recognition can be
severely affected. One of the early diagnostic signs of lesions of cranial nerve VIII or
the low brainstem is severely reduced speech recognition scores in the presence of
mild or moderate pure-tone hearing loss

Brainstem auditory evoked Gives objective assessment of hearing thresholds (see earlier discussion)
response

Otoacoustic emission Universally used for neonatal screening of hearing. Presentation of an acoustic
stimulus to the ear and monitoring of energy in the ear canal allows
measurements of emissions from the outer hair cells of the cochlea. In the
case of a retrocochlear lesion, when there has been no retrograde degeneration
of the outer hair cells, normal OAE can be evoked in the presence of significant
sensorineural hearing loss. OAE represents the first available auditory function
test with which it may be possible to differentiate neural from cochlear sites of a
lesion when the potential exists for each site to be implicated in sensorineural
loss. Any middle ear lesion typically precludes measurement of OAE

Electrically evoked auditory Allow assessment of neural integrity, evaluation of cochlear implant function, and
potentials estimation of the psychophysical measures needed to program the cochlear
implant speech processor, as well as an indication of performance after cochlear
implantation

15. 1 ¼ h, Ossicular dislocation, 2 ¼ g, Noise- 17. 1 ¼ e, Meniere's disease, 2 ¼ a, BPPV, 3 ¼ j,


induced hearing loss, 3 ¼ k, Perilymph fistula Vestibular schwannoma

16. 1 ¼ m, Waardenburg syndrome, 2 ¼ a, Alport


syndrome, 3 ¼ e, Jervell Lange-Nielsen
syndrome

Neurosurgery Books Full


www.ketabpezeshki.com 66485438-66485457
10 NEURO-OTOLOGY 183

18. 1 ¼ p, 2 ¼ n, 3 ¼ l, 4 ¼ m, 5 ¼ q variation imparts motion to the basilar mem-


brane, causing a wave to travel along it. The
Sound waves contact the tympanic membrane, basilar membrane is stiffest at its base and
and the acoustic energy is transformed and becomes progressively more flexible toward its
transmitted to the inner ear through the ossic- tip. Therefore any given frequency of sound
ular chain in the middle ear. Displacement of (pure tone) will cause a wave in the basilar
the oval window by the stapes footplate results membrane that has maximum displacement that
in instantaneous transmission of the pressure is precisely timed and spaced along the mem-
through perilymph in the scalavestibuli, Reiss- brane. For high tones, this point is close to
ner's membrane, endolymph, basilar mem- the base of the cochlea, and for lower frequen-
brane, and perilymph in the scala tympani and cies, it is more distal. The response of hair cells
in outward displacement of the round window to the tone is strongest at the point of greatest
membrane. Hair cell transduction occurs as displacement. Therefore the position from base
the basilar membrane containing the organ of to apex along the spiral of the basilar membrane
Corti moves relative to the tectorial membrane. and organ of Corti is directly related to the fre-
The plunger-like motion of the stapes in the quency of the tone that will elicit a response
oval window compresses the perilymph. In the (tonotopy).
fluid medium of the cochlea, this pressure

Malleus Incus Stapes

Scala vestibule Reissner


Oval window (perilymph) membrane

Scala media
(endolymph)

Tectorial
membrane
External
auditory
canal
Organ of corti
Basilar membrane

Scala tympani
(perilymph)

Tympanic Middle ear Spiral ganglion


membrane
Round window Cochlear division of CN VIII

Image with permission from Winn HR. Youman's Neurological Surgery, 4-Volume Set, 6th ed. Elsevier, Saunders,
2011.

19. 1 ¼ g, 2 ¼ b, 3 ¼ a, 4 ¼ h

Inner hair cells are extremely sensitive transducers membrane. This causes ion channels at the tips
that convert the mechanical force applied to the of the stereocilia to open, allowing potassium flow
hair bundle into an electrical signal. Endolymph, down the electrical gradient to depolarize the cell.
like extracellular fluid, has a high concentration When a hair cell depolarizes, voltage-gated cal-
of potassium. In contrast, perilymph, like cerebro- cium channels at the base of the cell open, and
spinal fluid, has a high concentration of sodium. the resulting influx of calcium causes synaptic ves-
The potential difference between the endolymph icles to fuse to the cell membrane and to release a
and the perilymph is maintained by selective secre- neurotransmitter into the synaptic cleft between
tion and absorption of ions by the striavascularis. the hair cell and the cochlear nerve fibers. The
As the basilar membrane moves up in response transmitter causes depolarization of the afferent
to fluid movement in the scala tympani, the taller fiber, and an action potential is transmitted along
stereocilia are displaced against the tectorial the cochlear nerve fiber.

Neurosurgery Books Full


www.ketabpezeshki.com 66485438-66485457
184 PART II CARE OF THE NEUROSURGICAL PATIENT

Tectorial membrane
Spiral limbus
Outer hair cells

Cochlear
duct
Internal
Spiral spiral
tunnel
ligament Inner
tunnel

Basilar
membrane Tympanic Nerve
cavity
Inner
hair cell
Phalangeal cells Pillar cells Osseous
spiral lamina
Image with permission from Lowe JS, Anderson PG. Stevens & Lowe's Human Histology, 4th ed. Elsevier, Mosby,
2015.

20. 1 ¼ j, Δ, 2 ¼ g, □, 3 ¼ f, <

Right ear Left ear

AC unmasked O X

AC masked Δ □

BC unmasked < >

BC masked [ ]
No response (imposed over other symbol) ↙ ↘

ANSWERS 21–22
Additional answers 21–22 available on
ExpertConsult.com

Neurosurgery Books Full


www.ketabpezeshki.com 66485438-66485457
CHAPTER 11

NEUROINTENSIVE AND
PERIOPERATIVE CARE
SINGLE BEST ANSWER (SBA) QUESTIONS
1. Which one of the following statements through the choroid plexus in the
regarding cerebral blood flow is LEAST ventricles
accurate? b. Third arterial wave is the percussion wave
a. Cerebral blood flow to white matter is c. First wave is the tidal wave which reflects
approximately 25 ml/100 g/min brain compliance
b. Total cerebral blood flow is approxi- d. Second wave is the dicrotic wave that
mately 750 ml/min in adults reflects aortic valve closure
c. Regional cerebral blood flow tends to e. Intracranial hypertension increase in the
track cerebral metabolic rate of oxygen peak of the tidal and dicrotic waves
consumption rather than cerebral meta-
bolic rate of glucose consumption 4. Regarding cerebral autoregulation in adults,
d. Cerebral blood flow to gray matter is which one of the following statements is
approximately 80 ml/100 g/min LEAST accurate?
e. Brain tissue accounts for 20% of basal a. Increasing hypoxia results in increasing
oxygen consumption and 25% of basal cerebral blood flow
glucose consumption b. Cerebral blood flow is relatively constant
over a range of cerebral perfusion pres-
2. Which one of the following statements sures from 50 to 150 mmHg
regarding intracranial compliance is LEAST c. Cerebral blood flow is directly propor-
accurate? tional to cerebral perfusion pressure
a. Increase in the volume of one intracranial (CPP) when CPP is greater than
compartment will lead to a rise in ICP 150 mmHg or less than 50 mmHg
unless it is matched by an equal reduction d. A pCO2 of 4.0 kPa (30 mmHg) is associ-
in the volume of another compartment ated with an average cerebral blood flow
b. Cerebral compliance is equal to intracra- of approximately 50 ml/100 g/min
nial volume displaced divided by the resul- e. Cerebral blood flow ¼ cerebral perfusion
tant change in intracranial pressure pressure/cerebral vascular resistance
c. CSF and CBV compartments normally
represent a volume of approximately 5. Which one of the following statements
1400 ml regarding control of cerebral vascular tone
d. Additional intracranial volume is initially is LEAST accurate?
accommodated with little or no change a. CO2 causes vasoconstriction at low ten-
in ICP sions in the blood, and vasodilatation at
e. Once craniospinal buffering capacity is higher tensions
exhausted further small increases in intra- b. Alpha2 and beta-1 adrenergic stimulation
cranial volume lead to substantial rises cause vasodilatation
in ICP c. Prostaglandins PGE2 and PGI2 are
vasodilators
3. Which one of the following statements d. Increase in perivascular K+ causes
regarding the intracranial pressure pulse vasodilatation
waveform is most accurate? e. Thromboxane A2 is a potent
a. Percussion wave, which reflects the ejec- vasoconstrictor
tion of blood from the heart transmitted

185
Neurosurgery Books Full
www.ketabpezeshki.com 66485438-66485457
186 PART II CARE OF THE NEUROSURGICAL PATIENT

6. Maintenance of which one of the following 10. Which one of the following statements
requires the highest proportion of energy regarding the role of hypothermia in the
expenditure in the brain? management of traumatic brain injury is
a. Transmembrane electrical and ionic LEAST accurate?
gradients a. Eurotherm trial showed a significant
b. Membrane structure and integrity increase in odds of unfavorable outcome
c. Synthesis and release of neurotransmitters but not death at 6 months in the mild
d. Neurogenesis hypothermia group
e. Axonal transport b. Two trials of hypothermia therapy in chil-
dren with TBI have shown no improve-
7. Immediately below which one of the follow- ment in neurologic or other outcomes
ing regional cerebral blood flow values does one pediatric trial showed a nonsignificant
the onset of infarction occur if sustained for increase in mortality
more than 2-3 h? c. Eurotherm trial RCT included patients
a. Less than 50 ml/100 g/min with TBI last 10 days and hypothermia
b. Less than 23 ml/100 g/min was induced if the ICP climbed above
c. Less than 17 ml/100 g/min 20 mmHg for 5 min refractory to tier 1
d. Less than 10 ml/100 g/min management
e. Less than 5 ml/100 g/min d. Statistically significant increase in the odds
of an unfavorable outcome in the group
8. Which one of the following statements allocated to therapeutic hypothermia
regarding neuroprotection during anesthesia e. Statistically significant increase in the odds
is LEAST accurate? of death at 6 months (HR 1.45 (1.01-2.10))
a. Burst suppression must be achieved hence discontinued due to futility
before any neuroprotective effects are
seen with barbiturates 11. A patient in the emergency department has
b. Hyperglycemia exacerbates ischemic been intubated and ventilated. CT head has
injury shown a right EDH with significant mass effect.
c. Mild hypothermia for low-grade aneu- His right pupil is larger than the left and the
rysm clipping and for head injury may anesthetist is concerned about hemodynamic
not be of benefit instability. What ASA grade is this patient?
d. Hyperthermia should be treated a. 1
e. Volatile anesthetics reduce the vulnerabil- b. 2
ity of the brain to ischemic injury c. 3
d. 4
9. Which one of the following statements e. 5
regarding successful strategies for cerebral f. 6
protection during cerebrovascular surgery is
LEAST accurate? 12. A 27-year-old man undergoes general anes-
a. For a given total vessel occlusion time, thesia for a hernia repair. As the anesthesia
brief-repetitive occlusions rather than a begins, his jaw muscles tense and he becomes
longer-single occlusion where possible generally rigid. He becomes febrile, tachy-
should be the goal cardic, and tachypneic. Which one of the fol-
b. Collateral blood flow can be increased by lowing treatments is most appropriate?
inducing hypertension (e.g. target MAP a. Atropine
150 mmHg) b. Procyclidine
c. Preoperative perfusion imaging to help c. Succinylcholine
identify patients who have low cerebro- d. Dantrolene
vascular reserve and may be at higher risk e. Thiopental
for iatrogenic ischemia
d. Intraoperatively, vessel or graft patency 13. Which one of the following is LEAST likely to
can be confirmed by be associated with massive blood transfusion?
e. IHAST2 trial showed improvement in a. Iron overload
outcome for clipped ruptured aneurysms b. Hyperkalemia
(WFNS1 and 2) given mild hypothermia c. Hypocalcemia
compared to normothermia d. Hypothermia
e. Coagulopathy

Neurosurgery Books Full


www.ketabpezeshki.com 66485438-66485457
11 NEUROINTENSIVE AND PERIOPERATIVE CARE 187

14. Which one of the following statements 18. Which one of the following statements regard-
regarding intraoperative blood loss manage- ing shock is LEAST accurate?
ment techniques applied in patients refusing a. Cardiac tamponade is a cause of
blood product transfusion is LEAST obstructive shock
accurate? b. Sepsis can cause a distributive shock
a. Meticulous attention to hemostasis and c. Spinal shock can cause bradycardia and
technical blood losses during surgery are hypotension
not usually important d. Hypovolemic shock is managed with res-
b. Phlebotomy should be rationalized toration of the circulating volume
c. Jehovah’s witnesses generally accept pro- e. Neurogenic shock is due to peripheral
thrombin complex concentrate vasoconstriction
d. Intraoperative cell saver use should be
considered if appropriate 19. Which one of the following is the most
e. DDAVP (vasopressin) can be used as a appropriate approximate blood volume for
procoagulant a term neonate?
a. 90-105 ml/kg
15. Which one of the following statements b. 80-90 ml/kg
regarding the oxygen-dissociation curve is c. 70-80 ml/kg
LEAST accurate? d. 70 ml/kg
a. It is sigmoidal due to cooperative binding e. 65 ml/kg
of oxygen to hemoglobin
b. The Bohr effect is a shift of the dissocia- 20. A 44-year-old male sustains a major trauma
tion curve to the left and is found with vomitus in his airway at
c. Reducing pH shifts the oxygen- scene with a GCS on E2V2M4. Primary sur-
dissociation curve to the left vey suggests isolated head injury and is admit-
d. The fetal oxygen-dissociation curve is ted to intensive care for medical management
shifted to the left reflecting the increased of intracranial pressure. ICP is 19 mmHg
oxygen affinity of fetal hemoglobin caused therefore he is kept sedated and ventilated.
by the presence of the gamma subunit of On day 2 he starts to desaturate and CXR is
hemoglobin performed. PaO2/FiO2 ratio is 113 mmHg
e. Increased temperature shifts the oxygen- (15 kPa). TTE is normal and there is no evi-
dissociation curve to the left dence of peripheral edema. Which one of the
following is the most likely diagnosis?
16. Which one of the following statements
regarding mechanical ventilation is LEAST
accurate?
a. PEEP and CPAP aim to keep alveoli open
during inspiration
b. Delivery of machine breaths may be trig-
gered by time or start of a patients spon-
taneous breath
c. SIMV allows patients to breath spontane-
ously between machine breaths
d. Patients with sufficient spontaneous
respiratory drive can be managed with
pressure support ventilation alone
e. Tidal volume is usually calculated as
6-8 ml/kg of ideal body weight a. Lower respiratory tract infection
b. ARDS
17. A 70 kg man has lost 1.7 l of blood from a stab c. ALI
wound. Which one of the following is the d. Congestive cardiac failure
LEAST likely to be showing? e. Hemopneumothorax
a. Respiratory rate 20-30
b. Narrow pulse pressure
QUESTIONS 21–38
c. Urine output 5-15 ml/h
d. Confusion Additional questions 21–38 available on
e. Pulse rate 120-140 bpm ExpertConsult.com

Neurosurgery Books Full


www.ketabpezeshki.com 66485438-66485457
188 PART II CARE OF THE NEUROSURGICAL PATIENT

EXTENDED MATCHING ITEM (EMI) QUESTIONS


39. Anesthetic agents in NICU: i. Platelet function assay
a. Etomidate j. Thrombin clotting time
b. Halothane
c. Isoflurane For each of the following descriptions, select the
d. Ketamine most appropriate answers from the list above.
e. Midazolam Each answer may be used once, more than once,
f. Opiates or not at all.
g. Propofol 1. Used to assess fibrinogen deficiency
h. Sevoflurane 2. Has largely been replaced by platelet func-
i. Succinylcholine tion assays
j. Vancuronium 3. Helpful to determine if prolonged PT or
aPTT is due to patient clotting factor defi-
For each of the following descriptions, select the ciency or due to presence of clotting
most appropriate answers from the list above. inhibitors
Each answer may be used once, more than once, 4. Test for lupus anticoagulant
or not at all.
1. Avoided in head injury patients due to rise 42. Acid-base balance:
in CMRO2, CBF and ICP a. Acute metabolic acidosis
2. Causes a decrease in CMRO2, CBF b. Acute metabolic alkalosis
and ICP c. Acute respiratory acidosis
3. Anticonvulsant effect but no effect on d. Acute respiratory alkalosis
CMRO2, CBF and ICP e. Compensated metabolic acidosis
f. Compensated respiratory acidosis
40. Bleeding diatheses: g. Partially compensated metabolic acidosis
a. Antiplatelets h. Partially compensated metabolic alkalosis
b. Disseminated intravascular coagulation i. Partially compensated respiratory acidosis
c. Factor V deficiency j. Partially compensated respiratory
d. Glanzmann’s thrombasthenia alkalosis
e. Hemophilia
f. Liver failure For each of the following descriptions, select the
g. Thrombocytopenia most appropriate answers from the list above.
h. Uremia Each answer may be used once, more than once,
i. Von Willebrand’s disease or not at all.
j. Warfarin or vitamin K deficiency 1. pH 7.21, pO2 108 mmHg (14.3 kPa), pCO2
15 mmHg (1.99 kPa), HCO3 15 mmol/l,
For each of the following descriptions, select the Base Excess 10 mmol/l
most appropriate answers from the list above. 2. pH 7.55, pO2 113 mmHg (15.1 kPa), pCO2
Each answer may be used once, more than once, 25 mmHg (3.3 kPa), HCO3 22 mmol/l,
or not at all. Base Excess +7 mmol/l
1. Normal PT, raised APTT, normal bleed- 3. pH 7.18, pO2 67 mmHg (8.9 kPa), pCO2
ing time, normal platelet count 74 mmHg (9.8 kPa), HCO3 11 mmol/l,
2. Prolonged PT, prolonged APTT, pro- Base Excess 12 mmol/l
longed bleeding time, reduced platelets 4. pH 7.35, pO2 76 mmHg (10.1 kPa), pCO2
3. Normal PT, Normal APTT, prolonged 55 mmHg (7.33 kPa), HCO3 29 mmol/l,
bleeding time, reduced platelets Base Excess 1 mmol/l
41. Coagulation assays: 43. Electrolyte disturbance:
a. APTT a. Hypernatremia
b. Bleeding time b. Hyponatremia
c. Dilute Russell’s viper venom time c. Hyperkalemia
(aRVVT) d. Hypokalemia
d. Factor V Leiden e. Hypocalcemia
e. Factor VII assay f. Hypermagnesemia
f. Factor VIII assay g. Hypomagnesemia
g. INR h. Hypophosphatemia
h. Mixing (50:50) test i. Hyperphosphatemia

Neurosurgery Books Full


www.ketabpezeshki.com 66485438-66485457
11 NEUROINTENSIVE AND PERIOPERATIVE CARE 189

For each of the following descriptions, select the g. Furosemide


most appropriate answers from the list above. h. Lidocaine
Each answer may be used once, more than once, i. Pacemaker
or not at all. j. Vagal maneuver plus adenosine
1. Tall T waves progressing to widened QRS k. Verapamil
complexes
2. U wave For each of the following descriptions, select the
3. Prolonged QT interval most appropriate answers from the list above.
Each answer may be used once, more than once,
44. Arrhythmia: or not at all.
a. Amiodarone 1. First line for stable ventricular tachycardia
b. Atropine 2. First line for narrow complex tachycardia
c. Bisoprolol 3. First line for persistent bradyarrhythmia
d. Digoxin causing symptoms
e. Electrical cardioversion 4. Third degree heart block
f. Flecainide

SBA ANSWERS
1. c—Regional cerebral blood flow tends to 2. c—CSF and CBV compartments normally
track cerebral metabolic rate of oxygen con- represent a volume of approximately 1400 ml
sumption rather than cerebral metabolic rate
of glucose consumption The volume of intracranial contents is approxi-
mately 1700 ml and can be divided into three
Despite its relatively small size (about 2% of total physiologic compartments: Brain parenchyma 
body mass; adult brain weighs approx 1.4 kg), the 1400 ml (80%, of which 10% is solid material
high metabolic activity of the brain (20% of basal and  70% is tissue water), Cerebral blood volume
oxygen consumption and 25% of basal glucose (CBV)  150 ml (10%), CSF  150 ml (10%).
consumption) requires reliable and responsive Intracranial pressure (ICP) is the pressure within
cerebral blood flow (CBF). The brain receives the intracranial space relative to atmospheric pres-
15% of cardiac output (750 ml/min in adults) at sure. “Normal” ICP is generally less than 10-
rest, which equates to an average CBF of about 15 mmHg and varies with age being lower in
50 ml/100 g/min. Mean CBF values for white infants and children. However, it is rarely constant
and gray matter vary between 20-30 and 75- and is normally subject to substantial individual
80 ml/100 g/min, respectively. Regional CBF variations and physiologic fluctuations, for exam-
therefore parallels metabolic activity and varies ple, with change in position, straining, and cough-
between 10 and 300 ml/100 g/min. Transmission ing. The generalized Monro-Kellie doctrine states
of electrical impulses by the brain is achieved by that an increase in the volume of one intracranial
energy-dependent neuronal membrane ionic gra- compartment will lead to a rise in ICP unless it
dients. Increases in local neuronal activity, there- is matched by an equal reduction in the volume
fore, are accompanied by increases in the regional of another compartment. Because brain paren-
cerebral metabolic rate. CBF changes parallel chyma is predominantly represented by incom-
these metabolic changes (i.e. flow-metabolism pressible fluid, the vascular and CSF
coupling). However, increases in regional CBF compartments play the key role in buffering addi-
during functional activation tend to track the tional intracranial volume by increasing venous
cerebral metabolic rate of glucose utilization outflow or reducing CBF and by displacing or
but may be far in excess of that required for the reducing the amount of intracranial CSF. In
cerebral metabolic rate of oxygen consumption infants, an open fontanelle provides an additional
(CMRO2). The regulatory changes involved in mechanism of volume compensation. Because
flow-metabolism coupling have a short latency the size of the CBV and CSF compartments is rel-
(about 1 s) and may be mediated by regional met- atively small, many pathologic processes easily lead
abolic or neurogenic pathways. In health, flow to increases in ICP by exceeding this compensa-
and metabolism are closely matched, with tory capacity. The dynamic relationship between
remarkably little variation in the oxygen extrac- changes in intracranial volume and pressure can
tion fraction (OEF) across the brain despite wide be graphically presented as a “pressure-volume”
regional variations in CBF and CMRO2. curve. It is evident from the exponential shape of

Neurosurgery Books Full


www.ketabpezeshki.com 66485438-66485457
190 PART II CARE OF THE NEUROSURGICAL PATIENT

the curve that additional intracranial volume is ini- 35 W2 elevation Abnormal pulse waveform
tially accommodated with little or no change in 30
ICP (flat part of the curve), but when craniospinal 25
buffering capacity is exhausted (point of decom-

ICP mmHg
20
pensation), further small increases in intracranial 15 W1
volume lead to substantial rises in ICP. Intracranial W2
10 W3
compliance can serve as measure of craniospinal
5
compensatory reserve (position on the pressure- Normal pulse waveform
0
volume curve) and is described by the following
equation, in which ΔV is change in volume and Image with permission from Vincent JL, et al. Textbook
ΔP is change in pressure: C ¼ ΔV/ΔP. Cerebral of Critical Care, 6th ed., Elsevier, Saunders, 2011.
compliance can be measured directly with invasive
devices. A substantial increase in ICP may lead to a
reduction in cerebral blood flow (CBF), and this 4. d—A pCO2 of 4.0 kPa (30 mmHg) is associ-
finding triggered interest in estimation of cerebral ated with an average cerebral blood flow of
perfusion pressure (CPP¼ mean arterial pressure approximately 50 ml/100 g/min (see graph)
[MAP] ICP).
Cerebral blood flow (CBF) is regulated at the
level of the cerebral arteriole. It depends on the
Irreversible increase in ICP
pressure gradient across the vessel wall (which
C ~ ΔV Loss of regulatory mechanisms in turn is the result of CPP) and Pa CO2 value
ΔP
Small dV causes large dP (which depends on ventilation). Cerebral autore-
gulation is dominant to ICP homeostasis and
keeps CBF constant in the face of changes in
Intracranial pressure

CPP or mean arterial pressure (MAP). It does


Poor compensatory reserve this through alterations in cerebral vasomotor
Compliance starts to fail
tone (i.e. cerebrovascular resistance [CVR])
Good compensatory reserve such that CBF ¼ CPP/CVR ¼ [MAP  ICP]/
Small dP for each dV
CVR. Chronic hypertension or sympathetic acti-
vation shifts the autoregulatory curve to the
Zone I Zone II Zone III
right, whereas sympathetic withdrawal shifts it
to the left. Cerebral autoregulation is normally
functional for CPP values of 50-150 mmHg
Volume (brain, blood, CSF, mass) and is impaired by intracranial and extracranial
(e.g. chronic systemic hypertension) pathologic
Image with permission from Swaiman K, et al. (Eds.), conditions and anesthetics. Tissue perfusion will
Swaiman's Pediatric Neurology: Principles and Prac- decrease proportionally when CPP is below the
tice, 5th ed., Elsevier, 2012. lower limit of autoregulation (e.g. <50 mmHg
if normal autoregulation is intact). Above the
upper autoregulation limit, the high CPP causes
3. e—Intracranial hypertension increase in the forced dilation of cerebral arterioles with resul-
peak of the tidal and dicrotic waves tant increases in CBV and ICP, disruption of
the blood-brain barrier, reversal of hydrostatic
Typically, the normal ICP waveform consists of gradients, and cerebral edema or hemorrhage
three arterial components superimposed on the (or both). CBF is proportional to arterial carbon
respiratory rhythm. The first arterial wave is dioxide tension (PaCO2), subject to a lower limit,
the percussion wave, which reflects the ejection below which vasoconstriction results in tissue
of blood from the heart transmitted through hypoxia and reflex vasodilation, and an upper
the choroid plexus in the ventricles. The second limit of maximal vasodilation. As such, target
wave is the tidal wave, which reflects brain com- pCO2 in head injury is 4-4.5 kPa (30-35 mmHg),
pliance; and finally, the third wave is the dicrotic and hyperventilation is avoided as brining CO2
wave that reflects aortic valve closure. Under down further to vasoconstrict can reduce CBV
physiologic conditions, the percussion wave is because dangerously low regional CBF and resul-
the tallest, with the tidal and dicrotic waves hav- tant cerebral ischemia can develop. CBF is
ing progressively smaller amplitudes. When unchanged until arterial oxygen tension (PaO2)
intracranial hypertension is present, cerebral falls below about 7 kPa (53 mmHg) but rises
compliance is diminished. This is reflected by sharply below that, such that raised ICP may
an increase in the peak of the tidal and dicrotic occur in hypoxic individuals. However, the actual
waves exceeding that of the percussion wave threshold may vary based on arterial oxygen

Neurosurgery Books Full


www.ketabpezeshki.com 66485438-66485457
11 NEUROINTENSIVE AND PERIOPERATIVE CARE 191

content (CaO2) related primarily to hemoglobin 5. b—Alpha2 and beta-1 adrenergic stimulation
oxygen carriage (and thus oxygen saturation) cause vasodilatation
rather than PaO2. Because of the shape of the
hemoglobin-oxygen dissociation curve, CaO2 is Multiple factors can regulate regional cerebral
relatively constant over this range of PaO2. Below blood flow. Vasodilatation may be due to beta-1
about 7 kPa, CBF exhibits an inverse linear rela- adrenoceptor stimulation, high pCO2, nitric
tionship with CaO2. Hypoxemia-induced vasodi- oxide, prostaglandins (increase in ECF/CSF dur-
lation shows little adaptation with time but may be ing hypotension), perivascular K + (rises due to
substantially modulated by PaCO2 levels. Other hypoxia and seizures), and local adenosine (hypo-
global factors affecting CBF include hematocrit; tension and hypoxia). Vasoconstriction may be
sympathetic tone, with β1-adrenergic stimulation related to alpha-2 adrenergic stimulation, free
causing vasodilation and α2-adrenergic stimula- calcium ions, thromboxane A2, and endothilin
tion causing vasoconstriction predominantly in (via action of vascular smooth muscle endothilin
the larger cerebral vessels; and elevated central A receptors)
venous pressure, which may elevate ICP and
reduce CPP. Temperature changes CBF by about 6. a—Transmembrane electrical and ionic
5% per 1° C and also decreases both CMRO2 and gradients
CBF, whereas autoregulation, flow-metabolism
coupling, and carbon dioxide reactivity remain The human brain accounts for only approxi-
intact. Ischemia results at levels of CBF below mately 2% of total body weight but receives about
20 ml/100 g/min unless CPP is restored (by 15% of resting cardiac output (750 ml/min) and
increasing MAP or decreasing ICP) or cerebral consumes about 20% (150 μmol/100 g/min) of
metabolic demand is reduced (through deepened the oxygen and 25% (30 μmol/100 g/min) of
anesthesia or hypothermia). Increased ICP result- the glucose required by the body at rest. This
ing in reduced CPP is met by cerebral arteriolar high energy expenditure results mainly from
relaxation; in parallel, MAP is increased via the maintenance of transmembrane electrical and
systemic autonomic response. As a result, a vicious ionic gradients ( 60%), but also from mainte-
cycle can be established, particularly in the pres- nance of membrane structure and integrity and
ence of impaired intracranial homeostasis, as the synthesis and release of neurotransmitters
cerebral vessel relaxation increases cerebral blood ( 40%). Although the energy requirements of
volume (CBV), thus further raising ICP. In addi- the brain are substantial, it has a very small
tion, an acute reduction in CPP or MAP tends to reserve of metabolic substrates. Therefore, nor-
acutely increase ICP (the so-called vasodilatory mal functioning of the central nervous system is
cascade). Reductions in PaCO2 induce vasocon- highly dependent on adequate and continuous
striction, reducing CBF, CBV, and thus ICP Con- provision of energy substrates and removal of
versely, hypercapnia increases ICP and should be the waste products of metabolism. Glucose is
prevented in the perioperative period. This makes the brains main substrate for generating ATP,
hyperventilation a useful tool for the acute control with its oxidation into pyruvate generating
of intracerebral hyperemia and elevated ICP. 2ATP molecules, but subsequent conversion of
pyruvate into acetyl-CoA and oxidation in the
citric acid (Krebs) cycle resulting in a net yield
of approximately 30 ATP molecules (compared
100 PaCO2 to 2ATP molecules for anaerobic respiration).
MAP The brain has a high metabolic requirement for
Cerebral blood flow

oxygen (40-70 ml O2/min) that must be met by


(ml/100g/min)

delivery within blood, which depends on the


50 oxygen content of the blood (typically, 20 ml
PaO2 per 100 ml blood) and blood flow (typically,
ICP 50 ml per 100 g brain per minute). Therefore,
under normal circumstances, delivery (150 ml/
0 50 100 150 500 min) is much greater than demand (40-70 ml/
Cerebral perfusion pressure (mmHg) min), and around 40% of the oxygen delivered
Image with permission from Cottrell JE, Young WL. in blood is extracted. This oxygen extraction frac-
Cottrell and Young's Neuroanesthesia, 5th ed., Elsevier, tion (OEF) can be increased for short periods
Mosby, 2010. when either delivery is reduced or demand is
increased. In states of prolonged starvation and
FURTHER READING in the developing brain, ketone bodies (acetoace-
Nortje J. Cerebral blood flow and its control. In Gupta K. tate and β-hydroxybutyrate) can become impor-
(Ed.), Essentials of neuroanaesthesia and neurointensive care, tant metabolic substrates within the brain. In
Elsevier, Saunders, 2008.
addition, some amino and organic acids can be
Neurosurgery Books Full
www.ketabpezeshki.com 66485438-66485457
192 PART II CARE OF THE NEUROSURGICAL PATIENT

taken up and metabolized within the brain. Over- can consume lactate as a substrate, particularly
all, these are minor energy substrates except dur- during periods of hypoglycemia or elevated blood
ing periods of metabolic stress, such as during lactate.
acute hypoglycemia and ischemia. The brain

7. c—Less than 17 ml/100 g/min

Cerebral Blood Flow and Ischaemia


CBF Cell State Time to Infarction Consequences

50 (20-80) Normal – Normal

<23 Oligemia >6 h EEG slowing

10-17 Penumbra Several hours Flatline EEG, absent evoked potentials

<10 Death Several minutes Membrane pump failure

Under normal circumstances, CBF is main- 8. a—Burst suppression must be achieved


tained at a relatively constant rate of 50 ml/ before any neuroprotective effects are seen
100 g/min. With a reduction in cerebral perfu- with barbiturates
sion pressure, CBF declines gradually. Physio-
logic indices of ischemia are not apparent until An anesthetized brain is less vulnerable to ische-
CBF is reduced to about 20-25 ml/100 g/min; mic injury. There do not appear to be any differ-
at that time, electroencephalographic (EEG) ences among anesthetic agents with respect to
slowing is apparent. Such slowing indicates that their neuroprotective efficacy.
the brain has a substantial blood flow reserve. • Barbiturates, propofol, and ketamine have
Below a CBF of 17 ml/100 g/min, the electroen- been shown to have neuroprotective effi-
cephalogram is suppressed and evoked potentials cacy. With regard to barbiturates, doses less
are absent. It is not until the CBF is less than 10 than those that produce burst suppression of
that ATP energy failure occurs resulting in neu- the electroencephalogram achieve the same
ronal depolarization, excitotoxicity, cytotoxic degree of protection as higher doses do
edema and cell death. Within the ischemic terri- • Volatile anesthetics reduce the vulnerability
tory, the region supplied by end arteries of the brain to ischemic injury
undergoes rapid death and is referred to as the • Maintenance of CPP within the normal
core. Surrounding the core is a variable area of range for a patient who is at risk for cerebral
the brain called the penumbra. The penumbra ischemic injury is essential. Modest increases
is rendered sufficiently ischemic to be electrically in blood pressure (5-10%) may be of benefit
silent but has not yet undergone ischemic depo- to those who have suffered from an ischemic
larization. The penumbra is viable for several insult. Hypotension is deleterious
hours and can be salvaged by restoration of flow. • Arterial pCO2 should be maintained in the
If reperfusion is not established, the penumbra is normal range unless hyperventilation is used
gradually recruited into the core. Depending on for short-term brain relaxation. Prophylac-
the severity of the injury, blood-brain barrier tic hyperventilation should be avoided
breakdown occurs about 2-3 days after injury. • Hyperglycemia exacerbates ischemic injury
This permits the entry of plasma proteins into and should be treated with insulin. A reason-
the brain substance, which further increases able threshold for treatment is 150 mg/dl. If
cerebral edema significantly and is called vaso- insulin treatment is initiated, blood glucose
genic edema. The development of post-ischemic should be closely monitored and hypoglyce-
edema can be significant enough to result in mia prevented
substantial increases in ICP and neurological • The routine induction of mild hypothermia
deterioration. In the region surrounding the for low-grade aneurysm clipping and for
infarction, autoregulation and CO2 reactivity head injury may not be of benefit. In
are re-established in most situations in about high-grade SAH patients (WFNS 4-5),
4-6 weeks. the utility of mild hypothermia for purposes
of brain protection remains to be defined
FURTHER READING • Hyperthermia should be avoided
Heiss WD, Graf R, The ischemic penumbra, Curr Opin Neu- • Seizures can worsen cerebral injury and
rol. 1994;7(1):11-19. should be treated with anticonvulsants
Neurosurgery Books Full
www.ketabpezeshki.com 66485438-66485457
11 NEUROINTENSIVE AND PERIOPERATIVE CARE 193

9. e—IHAST2 trial showed improvement in 10. a—Eurotherm trial showed a significant


outcome for clipped ruptured aneurysms increase in odds of unfavorable outcome
(WFNS1 and 2) given mild hypothermia but not death at 6 months in the mild
compared to normothermia hypothermia group

Several strategies may be used in an attempt to pro- Induced hypothermia has been a proposed treat-
vide cytoprotection during cerebrovascular proce- ment for TBI based upon its potential to reduce
dures with use of temporary arterial occlusion for ICP as well as to provide neuroprotection and
dissection of aneurysms and permanent clipping. prevent secondary brain injury. Induced hypo-
Limiting the duration of ischemia is probably the thermia has been shown to be effective in
most intuitive and direct method of reducing ische- improving neurologic outcome after ventricular
mic injury. The duration of focal ischemia that can fibrillation cardiac arrest. National Acute Brain
be tolerated safely without clinically evident Injury Study: Hypothermia II, plus two trials
sequelae varies between individuals and vascular of hypothermia therapy in children with TBI
territories. The current consensus for temporary have shown no improvement in neurologic or
vessel occlusion is brief repetitive clipping periods, other outcomes; one pediatric trial showed a
which provides increased safety and less risk for nonsignificant increase in mortality. Eurotherm
postoperative neurological deficit than a single epi- RCT looked at hypothermia in patients with
sode of occlusion does, but variation in other TBI (last 10 days) admitted to a critical care
intraoperative parameters means no single occlu- environment with invasive ICP monitoring
sion time is accepted as “safe.” Collateral blood and an approach to participate was triggered if
flow can be increased by inducing hypertension the ICP climbed above 20 mmHg for 5 min
(e.g. target MAP 150 mmHg). Preoperative mea- refractory to tier 1 management (usually
surement of flow rates through cerebral vessels includes head elevation, encouragement of
can be achieved with perfusion imaging to help venous drainage, intubation, sedation and venti-
the surgeon identify patients who have low cere- lation to appropriate targets). Cooling protocol
brovascular reserve and may be at higher risk for was a 20-30 ml/kg bolus of cold saline followed
iatrogenic ischemia. Intraoperatively, vessel or by maintenance of hypothermia as they saw fit
graft patency can be confirmed by a number of but directed to a duration of at least 48 h, with
modalities, including direct microvascular Doppler additional time as needed to control ICP. Tem-
or transcranial Doppler (TCD) ultrasound, ultra- perature was optional between 32 and 35 °C in
sonic flow probe, intraoperative angiography, the intervention arm, titrated to ICP. All
EEG, electrocorticography, multimodality evoked patients were followed up for 6 months and
potential (MEP) and somatosensory evoked poten- the Extended Glasgow outcome scale (EGOS)
tial (SEP) monitoring, brain tissue oxygenation, was used as the primary outcome measure. From
and fluorescent angiography (e.g. fluorescein 387 patients, the data committee found a statis-
sodium, indocyanine green). Decreasing the meta- tically significant increase in the odds of an unfa-
bolic activity of tissue at risk can be achieved by vorable outcome and death at 6 months in the
mild hypothermia (33-34.5 °C) and by the use of group allocated to therapeutic hypothermia,
certain anesthetic agents to induce EEG burst sup- hence it was discontinued due to futility. Criti-
pression (e.g. pentobarbital, propofol, etomidate). cisms of this trial include very early use (i.e. tier
The role of mild hypothermia in neurovascular 2) of hypothermia with high potential variability
surgery is less clear after Intraoperative Hypother- of subsequent management, inclusion criteria
mia After Aneurysm Surgery Trial (IHAST2), an any head injury in previous 10 days, only ICP
international double-blind trial in which 1001 used as a guide (not CPP).
WFNS1-3 aneurysmal SAH patients undergoing
aneurysm clipping were subjected to mild hypo- FURTHER READING
thermia (33 °C at clip placement) versus normo- Hypothermia therapy after traumatic brain injury in children.
thermia (36.5 °C) with no difference in outcome. N Engl J Med 2008; 358:2447-2456.
Hypothermia with circulatory arrest is most often Beca J, et al. Hypothermia for Traumatic Brain Injury in
used during aneurysm surgery for giant and com- Children-A Phase II Randomized Controlled Trial. Crit Care
plex posterior circulation aneurysms including: Med. 2015;43(7):1458-1466.
those not amenable to endovascular treatment; Clifton GL, et al. Very early hypothermia induction in
those with significant intra-aneurysmal thrombus, patients with severe brain injury (the National Acute Brain
broad necks, or a projection endangering dissec- Injury Study: Hypothermia II): a randomised trial. Lancet
tion and preservation of perforators; those adher- Neurol. 2011;10(2):131-139.
ing to vital structures; and fusiform aneurysms Hypothermia for intracranial hypertension after traumatic
with a distal vessel not suitable for arterial bypass. brain injury. N Engl J Med 2015; 373:2403-2412.

Neurosurgery Books Full


www.ketabpezeshki.com 66485438-66485457
194 PART II CARE OF THE NEUROSURGICAL PATIENT

11. e—ASA 5 • Coagulation factor deficiency (relative)—


leading to a coagulopathy if concomitant
American Society of Anesthesiologists Physical cryoprecipitate/FFP not also transfused.
Status classification is as follows: 1—a normal • Ineffective tissue oxygenation due to
healthy patient (non-smoker, minimal/no alcohol); reduced volume of 2,3 bisphosphoglycerate,
2—a patient with mild systemic disease (smoker, which does not store well
pregnancy, obesity); 3—a patient with severe • Hypothermia unless blood adequately warmed
systemic disease (e.g. uncontrolled diabetes/ • Hypocalcemia: Due to chelation by the cit-
hypertension, alcoholism, dialysis, >3 months rate in the additive solution and may worsen
since MI/TIA/CVA); 4—a patient with severe coagulopathy
systemic disease which is a threat to life (e.g. • Hyperkalemia: Due to progressive potas-
<3 months since MI/TIA/CVA, ongoing cardiac sium leakage from the stored red cells
ischemia, sepsis); 5—moribund patient not
expected to survive without the operation (e.g. rup-
tured AAA, massive trauma, intracranial bleed Types of Transfusion Reaction
with mass effect, bowel ischemia with organ fail-
Acute (hours) Delayed (days to weeks)
ure); 6—declared brain dead patient undergoing
organ harvesting. Acute hemolytic Delayed hemolytic
intravascular (ABO) intravascular
12. d—Dantrolene Acute hemolytic Delayed hemolytic
extravascular (Rh) extravascular
Febrile non-hemolytic Viral infection
Malignant hyperthermia is characterized by acute reaction Transfusion graft-
severe fever, tachypnea, tachycardia, and rigidity, Allergy/anaphylaxis versus-host disease
and high mortality rate if left untreated. It is typ- Septic contamination
ically precipitated by volatile anesthetics, espe- Transfusion related
cially halothane, or muscle relaxants such as acute lung injury
succinylcholine. Patients may become severely
acidotic and develop rhabdomyolysis. Pathology
shows diffuse segmental muscle necrosis. It 14. c—Jehovah’s witnesses generally accept pro-
appears to be a metabolic myopathy in which thrombin complex concentrate
there is abnormal release of calcium from the sar-
coplasmic reticulum and ineffectual uptake after- In general, while a hemoglobin (Hgb) between 7
ward. Genetic defects in the ryanodine receptor, and 8 g/dl appears to have no immediate adverse
involved in calcium flux in the sarcoplasmic retic- effect on mortality, there is a clear risk of death in
ulum, are responsible for about 10% of cases, the 30-day postoperative period when hemoglo-
although as yet unidentified abnormalities of this bin fell much below 7 g/dl. Where transfusion
or related proteins probably play a role in most is not an option (e.g. Jehovah’s witnesses, multi-
cases. It is inherited in an autosomal dominant ple alloantibodies) the key emphasis is on opti-
fashion. Certain other myopathies, including mizing hematopoiesis, minimizing bleeding and
Duchenne muscular dystrophy and central core blood loss (blood conservation), and harnessing
myopathy, are associated with this condition as and optimizing physiological tolerance of anemia
well. Treatment consists of discontinuation of (through application of all available therapeutic
anesthesia, administration of dantrolene, which resources). Meticulous attention to hemostasis
prevents release of calcium from the sarcoplasmic and technical blood losses during surgery (e.g.
reticulum, and supportive measures. use of hemostatic surgical devices, fibrin glue
and tissue adhesives; controlled hypotension; ele-
13. a—Iron overload vating the surgical field above the rest of the
body), minimizing phlebotomy are important.
A massive transfusion is defined as a transfusion In addition, clarity must be sought (and docu-
equaling the patients’ blood volume within 12- mented) from the individual about each available
24 h. The specific additional problems related blood product and extracorporeal procedure that
to this scenario include: is acceptable to them in the following groups:
• Volume overload resulting in non- • Allogenic human blood and blood products.
cardiogenic pulmonary edema Whole blood, RBC, plasma, platelets, white
• Thrombocytopenia: following storage blood cells, blood from specific donors
there is a reduction of functioning platelets, • Human blood fractions and medications
so that there is a dilutional thrombocytope- that contain human blood fractions: cryo-
nia following a large transfusion precipitate, cryosupernatant, albumin,

Neurosurgery Books Full


www.ketabpezeshki.com 66485438-66485457
11 NEUROINTENSIVE AND PERIOPERATIVE CARE 195

plasma protein fraction, human immuno- 16. a—PEEP and CPAP aim to keep alveoli
globulin (e.g. Rh immune globulin, IVIG), open during inspiration
plasma derived clotting factor concentrates
(e.g. fibrinogen, VIII, IX), tissue adhesive/ Ventilation parameters:
fibrin glue • PEEP (positive end expiratory pressure) or
• IV fluids and medications not derived from CPAC (continuous positive airway pres-
human blood: hydroxyethyl starch (hetas- sure) at the end of expiration to prevent
tarch, pentastarch), balanced salt solutions, lung atelectasis and improve oxygenation
recombinant clotting factor concentrates The benefits are redistribution of (1) lung
(rVIII, rIX, recombinant VIIa), recombi- water, (the redistribution of extravascular
nant erythropoietin, antifibrinolytic chemi- water leads to improved oxygenation, lung
cals (e.g. tranexamic acid, aminocaproic compliance, and ventilation-perfusion
acid), procoagulant chemicals (e.g. matching) (2) increasing FRC (shunting is
DDAVP, vitamin K) decreased and thus oxygenation improved)
• Extracorporeal techniques for blood con- (3) decreasing work of breathing. Patients
servation or treatment: intraoperative who benefit are suitable are cardiovascularly
hemodilution, intraoperative blood salvage stable, do not have raised ICP, and
(cell saver), autologous banked blood (self- lungs that can be expanded by PEEP, and
donation), cardiopulmonary bypass, chest bilateral lung lesions. PEEP can cause
drainage autotransfusion, plasmapheresis, hypotension due to excessive positive pres-
hemodialysis sure. At lower level of PEEP (3-10 cmH2O)
prevents the alveolar collapse, at higher
15. b—The Bohr effect is a shift of the dissocia- levels reopen or recruit collapsed alveolar
tion curve to the left unit (alveolar recruitment)
• FiO2: 1.0% or 100% oxygen during unstable
The sigmoidal shape of the oxygen dissociation hemodynamic, CPR or initially put on ven-
curve reflects the progressive nature with which tilator 0.4-0.5 to prevent hypoxemia (PaO2
each oxygen molecule binds to hemoglobin such 100-50 mmHg) 0.3-0.4 to keep PaO2
that the binding of one oxygen molecule facili- 60-80 mmHg or SpO2 90-92% for patient
tates the binding of the next. The Bohr effect is with high risk for oxygen toxicity e.g. preterm
a shift of the dissociation curve to the right, sig- • Tidal volume: Normally 6-8 cc/kg of ideal
nifying a reduction of the oxygen affinity of the body weight e.g. patient who weighs 70 kg,
hemoglobin molecule and thus a greater ten- the tidal volume 7 x 70 ¼ 490 cc and look
dency to off-load oxygen into the tissues (i.e. in for PIP which is not too high (not more than
acute or chronically underperfused tissue). This 50 cmH2O)
change is caused by increased body temperature, • Inspiratory:Expiratory ratio is 1:2 in normal
acidosis, chronic hypoxia (increased 2,3-BPG) patients, 1:3 in COPD or asthmatic patient
and hypercapnia. The fetal oxygen-dissociation who need longer expired time or reverse I:E
curve is shifted to the left, reflecting the increased ratio for severe ARDS patients
oxygen affinity of fetal hemoglobin compared to • Respiratory rate 8-25/min to keep normo-
maternal hemoglobin molecule and allowing oxy- carbia or permissive hypercarbia. Inspired
gen transfer (gamma subunit instead of the alpha time 1-2 s depends to respiratory rate. If
that cannot form covalent bonds with 2,3-BPG). higher respiratory rate, lower inspired time
to keep constant I:E ratio
• Peak flow rate inversely correlate with
100
↓ H+ (↑ PH) inspired time and also affects I:E VAC or
↓ CO2 SIMV, the inspiratory flow rate is usually
80 ↓ Temperature set at 40-90 l/min
↓ 2,3-DPG
↑ H+ (↓ PH)
60
↑ CO2
SO2

↑ Temperature
40 ↑ 2,3-DPG

20
P50
0
0 20 40 60 80 100
PaO2
Image with permission from Hayward MP, Medical
Secrets, 5th ed., Elsevier, Mosby, 2012.

Neurosurgery Books Full


www.ketabpezeshki.com 66485438-66485457
196 PART II CARE OF THE NEUROSURGICAL PATIENT

Mechanical Ventilation: Types


Pressure
Controlled Synchronized
Ventilation Intermittent
Continuous Mechanical (PCV) or Mechanical
Ventilation (CMV) or “Volume “Pressure Pressure Support Ventilation
Assist” Assist” Ventilation (PSV) (SIMV)

Breath trigger Time (e.g. 10/min) or patient Time or patient Patient Time or patient
spontaneous breath sensor spontaneous spontaneous spontaneous
(patient draws flow from breath breath breath
circuit or creates negative
pressure)

Breath delivery Fixed flow rate Fixed pressure Fixed pressure Fixed flow rate
or pressure

Breath Delivered preset Tidal volume Completed Inspiratory flow Delivered


termination preset decreased to preset preset tidal
(cycling) inspiratory time percentage of peak volume
flow

Comments Flow target and frequency that Pressure target Tidal volume, Patient can
at least equals the preset rate and frequency inspiratory time breathe
that at least and frequency are spontaneously
equals the preset determined by the with or without
rates patient PSV between
machine
breaths

Advantages Not comfortable but delivery Comfortable, Comfortable, Not comfortable


of minute ventilation minute inadequate minute but delivery of
guaranteed ventilation not ventilation if minute
guaranteed insufficient ventilation
respiratory drive guaranteed

17. a—Respiratory rate 20-30

Classification of Haemorrhagic Shock


Class I Class II Class III Class IV

Blood loss <750 ml (<15%) 750-1500 (15-30%) 1500-2000 (30-40%) >2000 ml (>40%)

Heart rate <100 100-120 120-140 >140

Blood pressure Normal Normal Reduced Reduced

Pulse pressure Normal or Reduced Reduced Reduced


widened

RR 14-20 20-30 30-40 >35

Urine output Normal (>30 ml/h) Oliguria (20-30 ml/h) Oliguria (5-15 ml/h) Anuria
(70 kg male)

CNS Normal Agitated Confused Lethargic

Neurosurgery Books Full


www.ketabpezeshki.com 66485438-66485457
11 NEUROINTENSIVE AND PERIOPERATIVE CARE 197

18. e—Neurogenic shock is due to peripheral a spinal cord injury, and if between T1-L2 spinal
vascoconstriction cord segment (i.e. sympathetic outflow) will cause
neurogenic shock due to loss of vasomotor tone,
Spinal shock refers to complete flaccid paralysis
and if above T5 may also cause bradycardia.
(voluntary and reflex function) below the level of

Features of Different Types of Circulatory Shock


Type of
Shock Mechanism Causes Features Treatment

Hypovolemic Heart pumping Dehydration, Hypotension, tachycardia, weak Volume


well but not hemorrhage, thread pulse, cool/pale/moist skin, replacement
enough burns reduced urine output
circulating
volume

Cardiogenic Heart failing to Arrhythmia, MI, Hypotension, tachycardia, weak Inotropes


pump out blood cardiomyopathy, thread pulse, cool/pale/moist skin, Pacing
pericarditis, PE reduced urine output, pulmonary PCI
edema, tachypnea

Obstructive Heart is PE, cardiac Hypotension, tachycardia, venous Relieve


obstructed from tamponade, congestion obstruction
pumping blood tension
pneumothorax

Distributive Heart pumps Sepsis, Hypotension, tachycardia, Supportive


well, but there is anaphylaxis tachypnea, flushed skin, fever, ABC
peripheral reduced urine output Antibiotics
vasodilatation Epinephrine

Neurogenic Bradycardia (if cord injury T5 or Supportive


above), Hypotension (massive ABC
vasodilatation), warm skin below Atropine for
level of injury (loss of bradycardia
vasoconstriction), poikilothermia. Vasopressors
Associated spinal shock (loss of for
voluntary motor and reflex activity hypotension
below level of injury)

19. b—80-90 ml/kg aspiration (30%), trauma (>20%), massive blood


transfusion, smoke inhalation, pancreatitis,
In general, approximate blood volumes are: pre- cardio-pulmonary bypass, burns. Management
mature neonate 90-105 ml/kg, term neonate including treat the underlying cause (e.g. antibi-
80-90 ml/kg, child 70-80 ml/kg, male adolescent otics if signs of sepsis), aiming for a slightly neg-
70 ml/kg and adolescent female 65 ml/kg. ative fluid balance, lung/alveolar recruitment
maneuvers (e.g. prone ventilation, PEEP),
20. b—ARDS mechanical ventilation strategy using low tidal
volumes, as conventional tidal volumes may cause
Defined as an acute condition characterized by lung injury (only treatment found to improve
bilateral pulmonary infiltrates and severe hypox- survival rates).
emia (PaO2/FiO2 ratio <200 or 26.6 kPa irre-
Image with permission from Khan A, Kantrow S, Taylor
spective of PEEP) in the absence of evidence DE. Acute Respiratory Distress Syndrome, Hospital
for cardiogenic pulmonary edema (clinically or Medicine Clinics, 2015, 4(4); 500-512.
pulmonary capillary wedge pressure
<18 mmHg). In is subdivided into two stages.
Early stages consist of an exudative phase of
injury with associated edema. The later stage is ANSWERS 21–38
one of repair and consists of fibroproliferative
Additional answers 21–38 available on
changes. Subsequent scarring may result in poor ExpertConsult.com
lung function. Causes include infection (>40%),

Neurosurgery Books Full


www.ketabpezeshki.com 66485438-66485457
198 PART II CARE OF THE NEUROSURGICAL PATIENT

EMI ANSWERS
39. 1—d, Ketamine, 2—g, Propofol; 3—e, Midazolam

Physiological Effects of Different Anaesthetic Agents


Anticonvulsant
Drug CMRO2 CBF ICP Effect Comment

Intravenous

Etomidate Decrease No effect No effect No

Propofol Decrease Decrease Decrease Yes May reduce CPP

Midazolam No effect No effect No effect Yes

Ketamine Increase Increase Increase No Neuropsychiatric side effect

Thiopental Decrease Decrease Decrease Yes

Inhalational

Nitrous oxide Increase Increase Increase No Muscle rigidity

Desflurane Decrease Increase at Increase No Below MAC 1.0, reduced


Isoflurane MAC 1.0 at MAC CMRO2 compensates for
Sevoflurane 1.0 vasodilatation

Other

Succinylcholine Increase Increase Increase No Secondary to increased


(non- (non- muscle
significant) significant)

Vercuronium No effect No effect No Effect No

Optiates Minimal Minimal Variable No


decrease decrease

40. 1—e, Hemophilia, 2—b, Disseminated intravascular coagulation, 3—g, Thrombocytopenia

Effects of Different Bleeding Disorders on Blood Tests


Condition PT APTT Bleeding Time Platelet Count

Warfarin or Vit K Prolonged Normal or mildly Normal Normal


deficiency Prolonged

DIC Prolonged Prolonged Prolonged Reduced

Uremia Normal Normal Prolonged Normal

Aspirin/Clopidogrel Normal Normal Prolonged Normal

Thrombocytopenia Normal Normal Prolonged Reduced

Liver failure Prolonged Normal or Prolonged Normal or Normal or


Prolonged Reduced

Von Willebrand’s disease Normal Normal or Prolonged Prolonged Normal

Factor V deficiency Prolonged Prolonged Normal Normal

Hemophilia Normal Prolonged Normal Normal

Factor XII deficiency Normal Prolonged Normal Normal

Glanzmann’s Normal Normal Prolonged Normal


thrombasthenia

Neurosurgery Books Full


www.ketabpezeshki.com 66485438-66485457
11 NEUROINTENSIVE AND PERIOPERATIVE CARE 199

41. 1—j, Thrombin clotting time, 2—b, Bleeding time, 3—h, 50:50 mixing study, 4—c, Dilute Russell’s
viper venom time

Common Blood Coagulation Screen Tests


Assay Description

Thrombin clotting Thrombin added to anticoagulated (citrate/ Prolonged with heparin, fibrin
time (TT) oxalate tube) blood and time to clot formation degradation products and fibrinogen
measured deficiency

Bleeding time Not a laboratory test—involves making a cut in Prolonged in platelet dysfunction, but
skin and measuring time for bleeding to stop. Has also other circumstances where
been replaced by platelet function assays bleeding tendency not reflected in
laboratory tests (e.g. uremia)

Platelet function Blood clotting assessed on collagen/epinephrine Abnormal Col/Epi but normal Col/ADP
assay (Col/Epi) and collagen/ADP membranes. Normal suggests aspirin induced platelet
Col/Epi closure time (<180 s) excludes significant dysfunction
platelet function defect. If Col/Epi time prolonged, Abnormal Col/Epi and abnormal Col/
Col/ADP (Normal <120 s) is automatically ADP suggests anemia,
performed thrombocytopenia, von Willebrand
disease

INR/PT Tests extrinsic + common pathway—factors I, II, V, Prolonged PT due to vitamin K


VII and X. Performed by adding tissue factor (III) to deficiency, warfarin, liver disease or
anticoagulated sample and measuring time to DIC
clot formation. Each batch of tissue factor has
differences, hence this is standardized in INR

Activated partial Tests intrinsic + common pathway I, II, V, VIII, IX, Prolonged APTT suggests heparin,
thromboplastin XI, XII—will be normal despite factor VII or XIII antiphospholipid antibody, factor
time (aPTT) deficiency. Anticoagulated blood (oxalate/citrate deficiency or anti-factor antibodies
sample) mixed with phospholipid, activator and
calcium and time to clot formation measured.
“Partial” refers to absence of tissue factor (III)
from mixture

Mixing (50:50) Plasma of patient with abnormal PT/APTT mixed Correction of PT/aPTT suggests factor
study with normal plasma in 50:50 ratio deficiency in patient serum
Persistence of abnormal PT/aPTT
suggests presence of inhibitor in
patient serum (warfarin, heparin,
lupus anticoagulant, coagulation
factor antibody)

Factor V Leiden Requires genetic test. Mutated factor V cannot be


inactivated by Protein C

Dilute Russell’s Test for lupus anticoagulant (antiphospholipid Prolonged aRVVT as antiphospholipid
viper venom time antibodies) as venom requires phospholipid for antibodies impair venom activation
(aRVVT) activation

42. 1—a, Acute metabolic acidosis, 2—d, Acute respiratory alkalosis, 3—i, Partially compensated respi-
ratory acidosis (Type 2 respiratory failure), 4—f, Compensated respiratory acidosis

Acid-Base Balance
Acid-Base Disturbance pH PaCO2 HCO3 Common Causes

Acute respiratory acidosis <7.35 High Normal Acute Asthma, suffocation

Partly/fully compensated <7.35 or High High COPD, MND


respiratory acidosis Normal

Acute respiratory alkalosis >7.45 Low Normal Pain, type 1 respiratory failure

Continued on following page

Neurosurgery Books Full


www.ketabpezeshki.com 66485438-66485457
200 PART II CARE OF THE NEUROSURGICAL PATIENT

Acid-Base Balance (Continued)


Acid-Base Disturbance pH PaCO2 HCO3 Common Causes
Partly/fully compensated >7.45 or Low Low CNS disturbance, pregnancy
respiratory alkalosis Normal

Acute metabolic acidosis <7.35 Normal Low DKA, lactic acidosis; (anion gap present)

Partly/fully compensated <7.35 or Low Low Diarrhea, renal tubular acidosis; (no
metabolic acidosis Normal anion gap)

Acute metabolic alkalosis >7.45 Normal High Vomiting (hypochloremic), primary


hyperaldosteronism

Partly/fully compensated >7.45 or High High Diuresis (hypochloremic), Cushing’s


metabolic alkalosis Normal syndrome

Mixed or complex acid-base disorders may also changes in ether HCO3 (in respiratory acidosis
occur, and can be diagnosed due to a mismatch in or alkalosis) or pCO2 (in metabolic acidosis or
expected compared to actual compensatory alkalosis).

43. 1—c, Hyperkalemia, 2—d, Hypokalemia, 3—e, Hypocalcemia

Electrolyte Disturbances: Clinical Features


Electrolyte
Disturbance Features ECG Changes

Hypernatremia Thirst, restlessness, brisk reflexes, seizures –

Hyponatremia Muscle cramp, weakness, raised ICP (confusion, seizures) Non-ischemic ST elevation

Hyperkalemia Confusion, hyperreflexia, weakness, paresthesia, Tall T waves progressing to


ascending flaccid paralysis widened QRS complexes

Hypokalemia Leg cramps, fatigues, weakness, muscle breakdown, U wave


paralytic ileus, paralysis, apnea

Hypercalcemia Bone pain, flank pain, constipation, hyporeflexic Shortened QT interval

Hypocalcemia Paresthesia, tetany, facial nerve twitching (Chvostek’s Prolonged QT interval


sign), carpopedal spasm (Trousseau’s sign), confusion,
seizures

Hypermagnesemia CNS depression, lethargy, confusion, weakness Prolonged PR interval,


Widened QRS complexes

Hypomagnesemia Tremor, tetany, myoclonus, seizures T wave flattening, ST


depression, U waves

Hypophosphatemia Asymptomatic or weakness, bone pain, rhabdomyolysis, –


altered mental status

Hyperphosphatemia Asymptomatic or signs of hypocalcemia (tetany, –


Chvostek’s)

44. 1—a, Amiodarone, 2—j, Vagal maneuver FURTHER READING


plus adenosine, 3—b, Atropine, 4—i, 2015 American Heart Association Guidelines for Cardiopul-
Pacemaker monary Resuscitation and Emergency Cardiovascular Care.

Neurosurgery Books Full


www.ketabpezeshki.com 66485438-66485457
This page intentionally left blank

Neurosurgery Books Full


www.ketabpezeshki.com 66485438-66485457
CHAPTER 12

INFECTION
SINGLE BEST ANSWER (SBA) QUESTIONS
1. A 45-year-old man presents to emergency 5. Central line infections are most commonly
department with a history of fever and rigors caused by which one of the following?
for 2 weeks. On examination he is pyrexial a. Candida
tachycardic and noted to have splinter hemor- b. Enterococci
rhages on right middle and index fingers. c. Gram negative bacilli
Where on the echocardiogram are vegeta- d. Staphylococci
tions most likely to occur? e. Streptococci
a. Aortic and mitral valve
b. Aortic valve 6. Which one of the following statements
c. Mitral valve regarding ventilator associated pneumonia
d. Pulmonary valve (VAP) in adults is most accurate?
e. Tricuspid valve a. Defined as pneumonia that occurs more
than 1 week after endotracheal intubation
2. A 61-year-old man presented with a right fron- b. Risk factors include age over 70, aspira-
tal cerebral abscess which was drained under tion and depressed conscious level
image-guidance, and investigations revealed c. Methicillin-resistant Staphylococcus aureus
infective endocarditis (IE). Blood cultures (MRSA) is responsible in >50%
and pus samples grew Streptococcus bovis. Which d. Diagnosis is primarily made on chest X-
one of the following is the next appropriate ray showing bilateral infiltrates
investigation? e. Antibiotics should not be started until
a. Bronchoscopy microbiological confirmation
b. Chest CT
c. Colonoscopy 7. Which one of the following is not a mecha-
d. Repeat blood cultures nism of antibiotic resistance?
e. Transesophageal echo a. Decreased permeability of cell to anti-
biotic
3. A 72-year-old woman presents 3 months after b. Efflux pump (active transport)
having a metallic mitral valve replacement. c. Inactivating enzyme
She was fatigued, pyrexia 38.6 °C (101.4 °F), d. Sequestration in cytoplasmic vesicles
BP 155/80, and pulse rate 124. On ausculta- e. Target site modification
tion she had a mechanical first heart sound
and a late diastolic murmur best heard at the 8. Which one of the following antimicrobials is
apex. Blood cultures were taken from three not first line therapy for active Mycobacterium
sites. What is the most likely organism? tuberculosis infection?
a. Candida a. Ethambutol
b. Coagulase negative Staphylococcus b. Isoniazid
c. Gram negative bacilli c. Pyrazinamide
d. No growth d. Rifampicin
e. Staphylococcus aureus e. Streptomycin

4. Which one of the following antibiotics are 9. Which one of the following statements about
associated with high risk of Clostridum difficile human immunodeficiency virus (HIV) infec-
pseudomembranous colitis? tion is most accurate?
a. Clindamycin a. Pneumocystis carnii pneumonia is likely if
b. Gentamicin the CD4 count is 300
c. Metronidazole b. Toxoplasma encephalitis is likely when
d. Tetracycline the CD4 count is 200
e. Vancomycin
e1
Neurosurgery Books Full
www.ketabpezeshki.com 66485438-66485457
e2 PART II CARE OF THE NEUROSURGICAL PATIENT

c. In an HIV positive individual AIDS is 2. Commonly cause dental caries, subacute


diagnosed when CD4 count is below 200 endocarditis, and brain abscesses.
d. HIV seroconversion usually occurs 2-6 3. Can cause pharyngitis, skin infection, scarlet
weeks after infection without a drop in fever, toxic shock syndrome, rheumatic
CD4 count fever, and acute post-streptococcal glomer-
e. Highly active antiretroviral therapy is ulonephritis.
normally initiated when the CD4 count
is below 400 12. Common Gram negative organisms:
a. Campylobacter
10. An 82-year-old is brought into the emer- b. Escherichia coli
gency department with pyrexia and reduced c. Haemophilus
level of consciousness. She has known d. HACEK group
chronic obstructive airways disease and was e. Klebsiella pneumoniae
recently started on antibiotics by her doctors f. Neisseria meningitidis
for a lower respiratory tract infection. Her g. Pseudomonas
temperature is 39 °C (102.2 °F), heart rate h. Salmonella
124, and respiratory rate 32. Blood pressure i. Shigella
on arrival was 95/67. Which one of the fol-
lowing best classifies her condition? For each of the following descriptions, select the
a. Multiple organ dysfunction syndrome most appropriate answers from the list above.
(MODS) Each answer may be used once, more than once
b. Sepsis or not at all.
c. Septic shock 1. Can produce hemolytic uremic syndrome
d. Severe sepsis 2. Associated with Guillain-Barre syndrome
e. Systemic inflammatory response syn- 3. Associated with Waterhouse-Friderichsen
drome syndrome

13. Antibiotic classes:


a. Aminoglycosides
b. Antimetabolite
EXTENDED MATCHING ITEM c. Carbapenems
(EMI) QUESTIONS d. Cephalosporin
e. Fluoroquinolones
11. Common Gram Positive organisms: f. Glycopeptides
a. Actinomyces israelii g. Lincosamides
b. Clostridium difficile h. Lipopeptides
c. Enterococcus faecalis i. Macrolide
d. Streptococcus pneumoniae j. Monobactam
e. Streptococcus pyogenes k. Oxazolidinones
f. Streptococcus bovis l. Penicillins
g. Staphylococcus aureus m. Rifampin
h. Staphylococcus epidermidis n. Tetracyclines
i. Streptococcus agalactiae
j. Viridans streptococci For each of the following descriptions, select the
most appropriate answers from the list above.
For each of the following descriptions, select the Each answer may be used once, more than once
most appropriate answers from the list above. or not at all.
Each answer may be used once, more than once 1. Class of beta lactam antibiotic where newer
or not at all. generations have reduced gram positive cov-
1. No Lancefield antigen, appears as lancet- erage and increased gram negative coverage
shaped diplococci and contains a polysaccha- 2. Antiribosomal antibiotic with gram positive
ride capsule. These organisms are α- cover including MRSA and vancomycin-
hemolytic and a common cause of pneumo- resistant Enterococcus (VRE), but no activity
nia and meningitis in adults and of otitis against most gram negatives
media and conjunctivitis in children. 3. DNA gyrase inhibitors

Neurosurgery Books Full


www.ketabpezeshki.com 66485438-66485457
12 INFECTION e3

SBA ANSWERS
1. c—Mitral valve and extremes of age; however these are not mod-
ifiable. The risk of infection increases after day 3.
Valve involvement in order of decreasing fre- Higher rates of infection in adults are seen with
quency is mitral > aortic > aortic and mitral > tri- femoral vein insertion sites, then jugular and
cuspid > pulmonary (rare). Vegetations occur on the least with subclavian. The skin pathogens
low pressure surface of the valve, that is, atrial involved are Staphylococci (coagulase-negative
aspect of mitral, ventricular aspect of aortic. About and S. aureus; 50%), Gram negative bacilli (30%),
70% of cases of IE on native valves are caused by enterococci (10%), and Candida species (10%).
viridans group Streptococci (usually after dental Paired blood cultures can be obtained from a
infection), Streptococcus bovis and Enterococcus. peripheral vein and via the catheter [bearing in
Another 25% are caused by Staphylococcus infec- mind that false-positives (i.e. contaminants) are
tions and these are more acute and severe. more common from line cultures]. Culture of a
catheter tip is only useful if the catheter is thought
2. c—Colonoscopy to be infected. If line related sepsis is suspected, the
catheter should be removed, cultures taken
S. bovis bacteremia is associated with colorectal (peripheral, tip of central venous catheter) and anti-
cancer in a significant number of cases. microbial therapy should be initiated. Management
is guided by the identification of the causative
3. e—Staphylococcus aureus organism: S. aureus bacteremia (SAB) should be
treated for at least 14 days, coagulase negative
Prosthetic valves are involved in 10-20% of cases
Staphylococcus for 5-7 days, Gram negatives for
of IE and eventually 5% of all prosthetic valves
10-14 days, and Candida for 14 days. Echocardiog-
will become infected. Early prosthetic valve IE
raphy is absolutely indicated in SAB. Most centers
(<60 days postop) is associated with coagulase
have rates of MRSA high enough to empirically
negative staph, Candida, and gram negative
include cover for this pathogen until culture results
bacilli. Late prosthetic valve infection (>60 days
are available. Prevention is via the use of strict asep-
postop) is associated with S. aureus, V. streptococci,
tic technique by an appropriately experienced per-
and enterococci. Metallic valves more likely to
son, hand washing/strict aseptic technique, full
become infected <3 months while bioprosthetic
barrier precautions, chlorhexidine for skin disinfec-
valves at >1 year. tion, antiseptic impregnated sponge dressings. Vig-
4. a—Clindamycin ilant catheter care is essential, in particular
antiseptic wipes on ports before access. All catheter
insertion sites should be assessed daily for infection.
Risk of C. Difficile Infection Varies by Removal of unnecessary catheters is vital, but often
Antibiotic: overlooked. If inserted under emergency condi-
tions they should be removed/replaced within
Frequently Occasionally Rarely 48 h. Guide-wire exchange techniques should not
Associated Associated Associated be used, as higher rates of bacteremia result.
Fluoroquinolones Macrolide Metronidazole
(e.g. ciprofloxacin) (e.g. FURTHER READING
erythromycin) Majumdar SS, Padiglione AA. Nosocomial infections in the
Clindamycin Trimethoprim Vancomycin intensive care unit. Anaesth Intensive Care Med 2012;13
(5):204-8.
Penicillins Sulfonamide Aminoglycosides
(broad-spectrum)
6. a—Defined as pneumonia that occurs more
Cephalosporins Tetracyclines than 1 week after endotracheal intubation
(broad-spectrum)
Carbapenems Chloramphenicol Hospital-acquired pneumonia (i.e. pneumonia
that begins 48 h or more after admission) is the
leading cause of hospital acquired infection lead-
5. d—Staphylococci ing to mortality. VAP occurs 48 h or more after
endotracheal intubation. Mechanical ventilation
Intravascular catheter-related infections are a increases the risk of pneumonia due to micro-
major cause of morbidity and mortality in the aspiration of oropharyngeal microorganisms via
intensive care unit. Risk factors: host predispos- leakage around the endotracheal tube cuff or
ing factors include immunosuppression, burns, directly through the tube. Risk factors for VAP
malnutrition, use of total parenteral nutrition, include: age over 70 years, chronic lung disease,

Neurosurgery Books Full


www.ketabpezeshki.com 66485438-66485457
e4 PART II CARE OF THE NEUROSURGICAL PATIENT

depressed consciousness, and aspiration. The key 8. e—Streptomycin


modifiable factors increasing risk for VAP are pre-
vious antibiotic exposure, use of paralytic agents, M. tuberculosis is transmitted by respiratory secre-
re-intubation or prolonged intubation, frequent tions (e.g. coughing). It does not stain well with
ventilator circuit changes, presence of a nasogas- the Gram stain (cell wall contains mycolic acids
tric tube, or intra-cranial pressure monitor. rather than peptidoglycan), but is visualized with
Local epidemiology varies significantly, but data the acid-fast Ziehl-Neelsen stain and grows on
from a large US study showed the major patho- Lowenstein-Jensen agar. Several characteristics
gens to be MRSA (14.8%), Pseudomonas aeruginosa of the tubercle bacillus make it difficult to control
(14.3%), and other Staphylococcus species (8.8%). quickly. One problem is its intracellular location,
A combination of clinical, microbiological, and where drugs do not penetrate well. In addition,
radiological criteria is required for diagnosis. the bacillus is often found in large cavities with
The principles of treating VAP include early anti- avascular centers, into which drugs do not pene-
microbial therapy after appropriate specimens are trate well either. Finally, the tubercle bacillus has
taken (guided by the local microbiology), then de- a very slow generation time. First line therapy
escalation according to culture and susceptibility includes: rifampin, isoniazid (also used for pro-
results. In many units this will result in empiric phylaxis), pyrazinamide and ethambutol (RIPE
combination therapy to cover multi-resistant therapy). They are used in combination because
organisms. A total duration of 5-7 days of effective there is a high incidence of resistance to these
therapy is adequate for most pathogens, though drugs. Side effects are outlined here. Isoniazid:
most physicians would treat longer for Pseudomo- peripheral neuropathy (relative pyridoxine defi-
nas or true staphylococcal pneumonia. Prevention ciency), hepatotoxicity, lupus-like syndrome,
strategies include reducing need for ventilation, P450 inhibitor. Rifampin induces hepatic P-450
reducing colonization, and reducing aspiration. enzymes, including those that metabolize opioids,
as turning urine, sweat, tears an orange color.
FURTHER READING Ethambutol has a side effect of optic neuropathy
Majumdar SS, Padiglione AA. Nosocomial infections in the in- (decreased visual acuity and color blindness).
tensive care unit. Anaesth Intensive Care Med 2012;13(5):204-8.
9. c—In an HIV positive individual, AIDS is
Kollef MH. Clinical characteristics and treatment patterns
among patients with ventilator-associated pneumonia. Chest
diagnosed when CD4 count is below 200
2006;129:1210e8.
HIV is an encapsulated retrovirus. Two viral
7. d—Sequestration in cytoplasmic vesicles envelope glycoproteins, gp120 and gp41, allow
the virus to infect CD4+ T cells (as well as some
macrophages, dendritic cells, and microglial cells)
Antibiotic Resistance Mechanisms that express an appropriate coreceptor (the che-
mokine receptors CXCR4 or CCR5). Upon entry
Resistance into the cell, the virus efficiently copies its RNA
Mechanism Examples
genome into double-stranded DNA using the
Decreased Downregulation of porins or viral enzyme reverse transcriptase. The viral
permeability of replacement with more DNA copy is integrated into the host cell
cell to antibiotic selective channels
genome, aided by the viral enzyme integrase.
Efflux pump Overexpression of efflux This pro-virus form may remain latent in the cell
(active transport) pumps until its expression is signaled. Expression of
Target site Altered penicillin-binding functional proteins by the virus involves a virally
modification proteins encoded protease that cleaves polyproteins into
Altered DNA-gyrase smaller functional proteins. When the provirus
Inactivating Hydrolysis: beta-lactamases is expressed to form new virions, the host cells will
enzyme (e.g. penicillinase, often lyse. With acute HIV infection, the individual
cephalosporinase, ESBL, may remain asymptomatic or develop an acute
NDM-1, carbapenemases)
Modification by transfer of a illness that resembles influenza or infectious
chemical group: mononucleosis; symptoms usually develop within
aminoglycoside-modifying 2-6 weeks after infection. During this stage,
enzymes antibodies to HIV are generally undetectable.
Seroconversion usually occurs during clinical
latency, an asymptomatic period that would last
FURTHER READING approximately 7-10 years in an untreated patient.
Blair JMA, et al. Molecular mechanisms of antibiotic Low-level (but persistent) replication of HIV
resistance. Nat Rev Microbiol 2015;13:42-51. causes a gradual decrease in CD4+ T cells, and

Neurosurgery Books Full


www.ketabpezeshki.com 66485438-66485457
12 INFECTION e5

minor opportunistic infections may occur. During 10. b—Sepsis


the crisis phase, escalation of viral replication leads
to a more rapid T-cell decline. This is clinically Term Definition
apparent as weight loss, fever, fatigue, and lymph-
Systemic Two or more of:
adenopathy. Acquired immunodeficiency syn- inflammatory Temperature >38 °C
drome (AIDS) is the diagnosis for a person who response syndrome (100.4 °F) or <36 °C
is HIV-positive and has a T-cell count below (96.8 °F),
200 μL 1 or presents with one of the AIDS- Heart rate >90,
defining opportunistic infections/malignancies RR > 20 or
PaCO2 < 32 mmHg
(e.g. Kaposi's sarcoma, non-Hodgkin's lym- white blood cells > 12 or
phoma, cervical cancer). Progression or response <4 or >10% bands
to antiretrovirals (reverse transcriptase inhibitors,
Sepsis systemic inflammatory
protease inhibitors) is monitored CD4+ T-cell count response syndrome
indicates the damage that has occurred to the + source of infection
immune system (normally 500-1500 μL 1), while
Severe sepsis Sepsis + organ
viral load serves as a marker for disease progression dysfunction (lactic
and drug therapy effectiveness by measuring the acidosis, SBP < 90)
amount of actively dividing HIV virus.
Septic shock Sepsis with refractory
hypotension (SBP < 90 or
MAP < 70 despite fluid
HIV-Related Infections challenge with 30°ml/kg
crystalloid)
CD4 Count Opportunistic Infection
MODS Dysfunction of two or
<200 Pneumocystis carnii/jirovecii more organ systems
pneumonia (PCP)
<100 Toxoplasma encephalitis,
Cryptococcal meningitis
<50 Mycobacterium avium complex,
EMI ANSWERS
cytomegalovirus retinitis
11. 1—d, Streptococcus pneumoniae; 2—j, Viridans
streptococci; 3—e, Streptococcus pyogenes

Common Gram Positive Organisms


Streptococcus pneumoniae Streptococcus pneumoniae has no Lancefield antigen, appears as lancet-
shaped cocci in pairs (diplococci), and contains a polysaccharide capsule.
These organisms are α-hemolytic. S. pneumoniae is a common cause of
pneumonia and meningitis in adults and of otitis media and conjunctivitis in
children. The pneumococcal vaccine should be given to
immunocompromised patients, older adults, those with HIV, and those
without spleens because asplenic patients cannot protect themselves against
encapsulated bugs
Viridans streptococci Heterogeneous group of streptococci not defined by a single Lancefield
(including milleri group) antigen, but share common features location in the gastrointestinal tract
(especially saliva) and are α-hemolytic. They cause the following diseases:
1. Dental caries
2. Subacute (slow-onset) bacterial endocarditis following a dental procedure,
especially on previously damaged or prosthetic heart valves (acute
endocarditis is usually staphylococcal)
3. Brain or abdominal abscesses
Streptococcus pyogenes Lancefield group A strep and are β-hemolytic. Multiple virulence factors
produce disease through direct invasion, toxin release, or delayed antibody
response. It can cause pharyngitis, skin infection, scarlet fever, toxic shock
syndrome, rheumatic fever, and acute post-streptococcal glomerulonephritis
Streptococcus agalactiae Lancefield group B Streptococcus (group B strep, or GBS) is β-hemolytic. It
lives in the vagina of 25% of women and is thus associated with infections of
the newborn as they traverse the birth canal; neonatal meningitis, pneumonia,
and sepsis are most common. Pregnant women are tested for GBS
colonization at 35-37 weeks' gestation and receive prophylactic penicillin
during labor if positive. GBS can also cause sepsis in pregnant women, leading
to sepsis of the fetus and stillbirth. Patients with coexistent medical
complications and older adults are also susceptible to infection

Continued on following page


Neurosurgery Books Full
www.ketabpezeshki.com 66485438-66485457
e6 PART II CARE OF THE NEUROSURGICAL PATIENT

Common Gram Positive Organisms (Continued)


Streptococcus bovis Group D Streptococcus bacteremia is highly associated with colon cancer
Enterococcus faecalis Enterococcus faecalis and Enterococcus faecium are normal inhabitants of
bowel (grow well in bile) and commonly cause infections in hospitalized
patients: urinary tract infections, biliary tract infections, subacute bacterial
endocarditis, wound infections, and bacteremia/sepsis from intravenous (IV)
catheters. May show resistance to VRE
Staphylococcus aureus Present in nasal cavity 30% of individuals, with higher rates among the ill and
hospital personnel. Spread occurs through contact and is reduced by
handwashing. S. aureus is the most virulent staph species in humans and can
survive extreme conditions for long periods of time. A variety of surface
adhesion proteins contribute to its virulence (coagulase, protein A, lipase,
etc.). Colonization followed by local invasion of skin usually leads to a furuncle/
skin abscess, then invasion of other organs through lymphatics or blood and/
or production of potential toxins, which are then disseminated throughout the
body. Most Staphylococcus species are resistant to penicillin because of the
production of penicillinase, but may be susceptible to other beta-lactam
antibiotics. MRSA is resistant to multiple beta-lactam antibiotics and may
represent over 50% of Staphylococcus aureus encountered. In the case of life-
threatening MRSA infection, vancomycin is first-line treatment
Staphylococcus epidermidis Staphylococcus epidermidis inhabits the skin of normal individuals and hence a
frequent contaminant of blood cultures. Its ability to form a biofilm
(polysaccharide scaffold) is what allows it to bind to synthetic materials,
commonly infecting Foley catheters and IV lines of hospitalized patients, as well
as indwelling prostheses (mechanical heart valves, stents, prosthetic joints)
Clostridium difficile Clostridium difficile causes pseudomembranous enterocolitis, usually within a
month after broad-spectrum antibiotic use (e.g. clindamycin, ampicillin),
which knocks out normal GI flora, allowing C. difficile to superinfect the colon.
Patients experience abdominal cramps, diarrhea, and fever due to toxin
release. Diagnosis is by toxin detection in the stool. Treatment is with
metronidazole (oral or IV) or vancomycin (oral only, IV is ineffective in C.
difficile colitis)
Actinomyces israelii A gram-positive, beaded, filamentous anaerobe which is part of the normal
oral flora and causes actinomycosis, an eroding abscess following trauma to
the oral or GI mucosa that can create draining sinus tracts. The pus from an
abscess, when viewed under the microscope, has yellow “sulfur” granules
that are actually microcolonies of A. israelii. Treatment is with penicillin G and
drainage of the abscess

FURTHER READING
Chapter 5 Microbiology. In: O’Connell TX, Pedigo RA, Blair TE. Crush Step 1: Ultimate USMLE Step 1 review, 1e.

12. b—E. coli; a—Campylobacter; f—Neisseria meningitidis

Common Gram Negative Organisms


Neisseria N. meningitidis (“meningococcus”) has a polysaccharide capsule. Meningococcus lives in
the nasopharynx and causes disease when it invades the bloodstream, causing meningitis
or sepsis (meningococcemia). The bacteria release endotoxin (lipopolysaccharides),
causing vascular hemorrhage (seen as a petechial rash) and an acute inflammatory
response (seen as spiking fever, chills, arthralgia, and muscle pain). Adrenal hemorrhage
can also occur, known as fulminant meningococcemia or Waterhouse-Friderichsen
syndrome, leading to adrenal insufficiency. Infants 6 months to 2 years, groups living in
close quarters (army recruits, dorm dwellers) are at risk
HACEK group The acronym HACEK refers to a grouping of gram-negative bacilli with an enhanced ability
to cause infective endocarditis (IE). Haemophilus species (Haemophilus parainfluenzae,
Haemophilus aphrophilus, Haemophilus paraphrophilus), Actinobacillus
actinomycetemcomitans, Cardiobacterium hominis, Eikenella corrodens, and Kingella
species. HACEK organisms are responsible for approximately 3% of cases of native valve IE.
They are also the most common cause of gram-negative endocarditis among persons who
do not abuse intravenous drugs

Continued

Neurosurgery Books Full


www.ketabpezeshki.com 66485438-66485457
12 INFECTION e7

Klebsiella Causes nosocomial sepsis (second most common after E. coli), Foley catheter-associated
pneumoniae UTIs, and pneumonia in those susceptible to aspiration of stomach contents (alcoholic
patients, intubated patients, and debilitated hospitalized patients). Pneumonia can form
cavitations and sputum is classically currant jelly colored
Escherichia Normally present in the GI tract. If it gains virulence factors through DNA exchange with
other enterics, it can cause diarrhea in those who are not immunized (infants in the
developing world, and adults in the developed world who travel to developing nations).
Depending on the virulence factors, three types of diarrhea can result (enterotoxigenic E.
coli (ETEC), enterohemorrhagic E. coli (EHEC), enteroinvasive E. coli (EIEC)). E. coli is the
most common cause of UTIs, especially affecting women and patients with Foley catheters.
E. coli meningitis is common in neonates. It is the most common cause of gram-negative
sepsis in hospitalized patients, and it commonly causes nosocomial (hospital-acquired)
pneumonia
Salmonella There are three clinically important species of Salmonella: S. typhi, S. cholerae-suis, and
S. enteritidis. The first is only found in humans; the latter two live in animal GI tracts and
infect humans through food or water contaminated with animal feces. S. typhi invades
intestinal epithelium, but then goes on to invade lymph and seeds multiple organs. The
resulting typhoid fever includes fever, diarrhea, headache, abdominal pain, and
inflammation of involved organs (e.g. the spleen). Light-skinned people may show a
transient pink-spotted (salmon-colored) rash on the belly
Shigella Transmission is fecal-oral (usually by contaminated water or hand-to-hand contact by
someone who has not washed properly) and occurs most often in preschools and nursing
homes. Bacteria latch onto the intestinal epithelium, invade cells, and release Shiga toxin,
which inhibits protein synthesis and causes cell death. The result is an inflammatory
response with blood-and-pus-speckled diarrhea and abdominal pain
Haemophilus Nonencapsulated (“untypeable”) H. flu mostly causes otitis media in children and upper
respiratory infections in adults with previous lung disease (e.g. chronic obstructive
pulmonary disease). Lack of the capsule limits invasiveness. Since routine vaccination with
purified type b capsule protein began, however, the incidence of Hib meningitis, acute
epiglottitis, and septic arthritis has been significantly reduced
Pseudomonas Tends to infect sick and immunocompromised patients and is very resistant to antibiotics.
When visible, it looks blue-green and smells like sweet grapes. It may cause problems in
burns patients, endocarditis in IV drug abusers, pneumonia (immunocompromised,
ventilator-dependent, cystic fibrosis), sepsis, otitis externa/media (especially diabetic), UTI/
pyelonephritis, osteomyelitis (diabetic, IV drug abusers, children)
Campylobacter Campylobacter jejuni is one of the three leading causes of gastroenteritis worldwide (the
other two are Rotavirus and ETEC), affecting mostly children. Its reservoir is animals, and it
is acquired by the fecal-oral route or unpasteurized milk. A prodrome of fever and headache
gives way in half a day to bloody diarrhea and abdominal cramps. Associated with Guillain-
Barre syndrome

FURTHER READING
Chapter 5 Microbiology. In: O’Connell TX, Pedigo RA, Blair TE. Crush Step 1: Ultimate USMLE Step 1 review, 1e.

13. d—Cephalosporins; k—Linezolid; e—Fluoroquinolones

Antimicrobial Classification
Antibiotic Example Mechanism Coverage and Resistance
Beta-lactam antibiotics
Penicillins Amoxicillin Cell wall Penicillin V/G: gram positives
Flucloxacillin synthesis Amoxicillin: as penicillin plus gram negative rods
Co-amoxiclav inhibitors Flucloxacillin: as penicillin plus MSSA
Piperacillin- Piperacillin: gram positives plus Pseudomonas,
tazobactam gram negative rods, and anaerobes
Beta-lactamases confer resistance (overcome by
combining penicillins with beta-lactamase
inhibitors, e.g. clavulanic acid, tazobactam)

Continued on following page

Neurosurgery Books Full


www.ketabpezeshki.com 66485438-66485457
e8 PART II CARE OF THE NEUROSURGICAL PATIENT

Antimicrobial Classification (Continued)


Antibiotic Example Mechanism Coverage and Resistance
Carbapenems Meropenem Cell wall Broad spectrum gram positive, negative, and
synthesis anaerobic (NOT MRSA)
inhibitors
Cephalosporin Cefuroxime (2nd Cell wall With newer generation, gram positive cover
gen) synthesis decreases and gram negative cover increases
Ceftriaxone (3rd) inhibitors
Ceftazidime (3rd)
Monobactam Aztreonam Cell wall Gram negative aerobes only
synthesis
inhibitors
Antiribosomal antibiotics
Aminoglycosides Gentamicin Protein Gram-negative aerobic bacteria
synthesis
inhibitors
Macrolide Erythromycin Protein Atypicals (Legionella, Mycoplasma, Chlamydia)
Clarithromycin synthesis and some gram positive cocci
inhibitors
Tetracyclines Tetracycline Protein Intracellular pathogens and specific gram
synthesis negatives
inhibitors
Lincosamides Clindamycin Protein Anaerobes, some aerobic gram positives
synthesis (including MRSA but not enterococci)
inhibitors
Amphenicol Chloramphenicol Protein Broad-spectrum (gram positive, negative, and
synthesis anaerobes) but limited used as high toxicity
inhibitors
Oxazolidinones Linezolid Protein Gram positive including MRSA and VRE, no
synthesis activity against most gram negatives
inhibitors
Others
Fluoroquinolones Ciprofloxacin DNA gyrase Narrow gram negative coverage (1st gen) plus
(2nd) inhibitor atypicals (2nd gen) plus Streptococcus (3rd gen)
Levofloxacin (3rd) plus anaerobes (4th gen)
Moxifloxacin (4th)
Nitroimidazoles/ Metronidazole Enzyme Metronidazole—gram negatives and enteric
furans Nitrofurantoin damage and anaerobes
DNA damage
Glycopeptide Vancomycin Cell wall Gram positive including MRSA; orally can be used
synthesis to treat gut C. difficile colitis
inhibitor
Lipopeptide Daptomycin Cell membrane Gram positive organisms including methicillin and
dysfunction vancomycin resistant
Antimetabolites Trimethoprim Folate Gram positive and gram negatives; parasites
Sulfamethoxazole synthesis including Toxoplasma, Pneumocystis jiroveci
inhibitor

FURTHER READING
Chapter 5 Microbiology. In: O’Connell TX, Pedigo RA, Blair TE. Crush Step 1: Ultimate USMLE Step 1 review, 1e.

Neurosurgery Books Full


www.ketabpezeshki.com 66485438-66485457
CHAPTER 13

SEIZURES
SINGLE BEST ANSWER (SBA) QUESTIONS
1. Approximately what proportion of patients spike and wave pattern. Which one of the fol-
with epilepsy have medically refractory lowing is most likely?
seizures? a. Breath holding attack
a. 0-20% b. Cardiac syncope
b. 20-40% c. Childhood absence epilepsy
c. 40-60% d. Partial complex seizures
d. 60-80% e. Rolandic epilepsy
e. 80-100%
5. A 12-year-old boy with diagnoses of
2. Which one of the following is a predictor of attention-deficit disorder and hyperactivity
spontaneous epilepsy remission? was noted to have “staring episodes” begin-
a. Abnormalities on neurological examina- ning over 1 year before this evaluation. About
tion or developmental delay 6 months earlier, the staring episodes became
b. Identification of epileptogenic substrate accompanied by some eye blinking and
c. Inadequate seizure control for greater mouth twitching. He has had one generalized
than 4 years tonic-clonic seizure in the last year. Which
d. Persistent epileptiform abnormalities one of the following is most likely diagnosis?
on EEG a. Childhood absence epilepsy
e. Younger age at onset b. Day dreaming
c. Juvenile absence epilepsy
3. The approximate rate of sudden unexpected d. Neurocardiogenic syncope
death in epilepsy (SUDEP) in patients with e. Reflex anoxic seizure
medically intractable seizures is which one
of the following? 6. A 19-year-old man presents with a history of
a. 1 in 200 per year seizures. The first was at age 12 when he got
b. 1 in 400 per year up very early in the morning to play computer
c. 1 in 600 per year games and was found by his mother a few
d. 1 in 800 per year hours later sitting at the computer blinking
e. 1 in 1000 per year and unresponsive. Two years later he experi-
ences a generalized tonic-clonic seizure. Cur-
4. A 7-year-old right-handed girl presented rently he has a seizure every 1-3 weeks and can
with onset of seizures at 3-years old. Typi- involve a minor jerk of the arms, staring with
cally, her eyes rolled up and she often had blinking, or generalized tonic-clonic seizures.
either a left or right body twitch. These He believed that sleep deprivation often pre-
events lasted 10-12 s and occurred often in cipitated seizures. Which one of the following
activities such as eating or talking. Immedi- is most likely diagnosis?
ately after these seizures, she spontaneously a. Benign familial convulsions
returned to her baseline. They typically b. Benign myoclonic epilepsy in infancy
occurred 6-10 times a day. She had been on c. Epilepsy with myoclonic absences
numerous medications in the past but d. Juvenile absence epilepsy
continues to have frequent breakthrough e. Juvenile myoclonic epilepsy
seizures on a daily basis. EEG shows a 3Hx

203
Neurosurgery Books Full
www.ketabpezeshki.com 66485438-66485457
204 PART II CARE OF THE NEUROSURGICAL PATIENT

7. Which one of the following abnormalities is demonstrated in the ictal EEG below?

Sens TC HF Pat Ref 2002-08-09 Elapsed Epoch


50 µV 0.1 s 70 Hz GK5 OFF 16:33:14 00:01:11 8/13

1 Fp1-Fp2
2 F7-F3
3 F3-Fz
4 Fz-F4
5 F4-F8
6 A1-T7
7 T7-C3
8 C3-Cz
9 Cz-C4
10 C4-T8
11 T8-A2
12 P7-P3
13 P3-Pz
14 Pz-P4
15 P4-P8
16 Fp1-A1
17 Fp2-A2
18 O1-*O2
19 *O1-*A1
20 *O2-*A2
Clin stop hyperventilation
21 *EKG-*A -
M 1s 250uV

a. 3-Hz spike and wave discharge d. Rhythmic temporal theta burst of


b. Hypsarrhythmia bdrowsiness
c. Lambda wave e. Sleep spindles

8. Which one of the following seizure types is classically associated with the pathology shown below?

A B

a. Absence seizure d. Gelastic seizure


b. Complex partial seizure e. Rolandic seizure
c. Focal motor seizure

Neurosurgery Books Full


www.ketabpezeshki.com 66485438-66485457
13 SEIZURES 205

9. Background interictal awake EEG in a 10-month-old female infant presenting with clusters of arm
abduction and head drop. Which one of the following is most likely?

Sens TC HF Pat Ref 2004-09-12 Elapsed Epoch


15 µ V 0.1 s 70 Hz LB OFF 10:08:59 00:00:40 5/28

1 Fp1-F3
2 F3-C3
3 C3-P3
4 P3-O1
5 Fp2-F4
6 F4-C4
7 C4-P4
8 P4-O2
9 Fp1-F7
10 F7-T7
11 T7-P7
12 P7-O1
13 Fp2-F8
14 F8-T8
15 T8-P8
16 P8-O2
17 Fz-Cz
18 Cz-Pz
M 1s 75uV

a. Benign Rolandic epilepsy d. Lennox-Gastaut syndrome


b. Gelastic epilepsy e. West syndrome
c. Landau-Kleffner syndrome

10. A 12-year-old, right-handed, developmentally delayed boy presented with three seizure types: gen-
eralized tonic-clonic, tonic, and atonic. The patient generally had at LEAST three brief seizures (1-
10 s) per day of the tonic and atonic types but rarely had generalized seizures. He presented to the
emergency department after one generalized tonic-clonic seizure followed by a series of atonic drop
attacks concurrent with a streptococcal infection. The patient remained lethargic, and status epi-
lepticus was a concern. Which of the following is the most likely diagnosis?
a. Aicardi syndrome d. Otahara syndrome
b. Angelman syndrome e. West syndrome
c. Lennox-Gastaut syndrome

11. When the patient, a young boy, was about age 6½ his mother first noticed an episode in which he
failed to respond to her on the telephone. Within 3 months, he had become “deaf” with an auditory
aphasia and poor articulation. An EEG during sleep showed continuous epileptiform discharges
from the posterior left temporal region. His language markedly improved with valproic acid and
prednisone. Which of the following is the most likely diagnosis?
a. Absence seizures d. Parietal lobe epilepsy
b. Continuous spike and wave during slow- e. Rassmussen's syndrome
dwave sleep syndrome (CSWS) f. Temporal lobe epilepsy
c. Landau-Kleffner syndrome

Neurosurgery Books Full


www.ketabpezeshki.com 66485438-66485457
206 PART II CARE OF THE NEUROSURGICAL PATIENT

12. Which one of the following is the most common cause of neonatal seizures?
a. Fetal inflammatory response d. Hypoxia-ischemia
b. Hypocalcemia e. Pyridoxine-dependent seizures
c. Hypoglycemia

13. A 24-year-old male presents with episodes characterized by facial (mouth and tongue) clonic move-
ments (which may be unilateral), laryngeal symptoms, articulation difficulty, swallowing or chewing
movements and hyper-salivation. Sensory (e.g. epigastric) and experiential (e.g. fear) aura and auto-
nomic (urogenital, gastrointestinal, cardiovascular or respiratory) features are common. Gustatory
hallucinations are particularly common. Which one of the following areas may the seizure focus arise?
a. Cingulate gyrus e. Motor cortex
b. Dorsolateral frontal f. Orbitofrontal
c. Fronto-parietal operculum g. Supplementary sensorimotor area
d. Frontopolar

14. Which one of the following is the likely origin of this secondarily generalized seizure recorded in
the EEG below?

Sens TC HF Pat Ref 2007-01-02 Elapsed Epoch


7 µV 0.1 s 70 Hz LB OFF 15:33:50 00:01:50 12/19

1 Fp1-F3
2 F3-C3
3 C3-P3
4 P3-O1
5 Fp2-F4
6 F4-C4
7 C4-P4
8 P4-O2
9 Fp1-F7
10 F7-T7
11 T7-P7
12 P7-O1
13 Fp2-F8
14 F8-T8
15 T8-P8
16 P8-O2
17 Fz-Cz
18 Cz-Pz
19 EKG-Aav
M EEG frontocentral fast 50uV
1s

a. Frontal poles d. Occipital


b. Mesial temporal lobe e. Parietal
c. Motor cortex

Neurosurgery Books Full


www.ketabpezeshki.com 66485438-66485457
13 SEIZURES 207

15. A 10-year-old right-handed boy presented with episodes of numbness around the mouth, drooling,
right-sided facial twitching, and inability to speak on waking up from sleep. There was retained aware-
ness and memory. Episodes were frequently followed by headaches. He experienced several episodes
per week lasting up to 1-2 min. Sleep EEG shown below. Which one of the following is most likely?

Sens TC HF Pat Ref 2007-10-13 Elapsed Epoch


10 µV 0.1 s 70 Hz Trace OFF 23:23:49 00:00:50 6/37

1 FP1-F7
2 F7-T7
3 T7-P7
4 P7-O1
5 Fp2-F8
6 F8-T8
7 T8-P8
8 P8-O2
9 FP1-F3
10 F3-C3
11 C3-P3
12 P3-O1
13 Fp2-F4
14 F4-C4
15 C4-P4
16 P4-O2
17 Fz-Cz
18 Cz-Pz
19 EKG-Aav
M 1s 50uV

a. Benign Rolandic epilepsy loss of awareness that might be associated with


b. Childhood absence epilepsy fumbling automatisms and lip smacking.
c. Juvenile myoclonic epilepsy Which one of the following is most likely?
d. Pyridoxine-dependent epilepsy a. Mesial temporal sclerosis
e. West syndrome b. Occipital cortical dysgenesis
c. Parietal meningioma
16. A 54-year-old has been experiencing three types d. Subependymal giant cell astrocytoma
of odd episodes for the last few years. The first e. Third ventricular colloid cyst
type involves a wave of non-painful sensation
emanating from the left elbow up and down 18. A 37-year-old man was diagnosed at age 25
her left arm and then progressing with “electric- when he had his first generalized tonic-clonic
ity” pain marching into her neck and head. This seizure while bowling. He was then noted to
could evolve into a more severe second type of have complex partial seizures during which
seizure in which the left side became very pain- he would repeatedly respond by saying “what”
ful and she felt afraid, had difficulty breathing, to all questions, lip smack, and throw up his
and then had clonic activity of the left face right arm. He was unaware of any auras and
and arm with loss of awareness (about 2 per was amnestic for his seizures. Most of his
month). The third type was a “hyperawareness” seizures were from sleep with a frequency of
in which she knew something was going to hap- about 2 per week. He had been involved in sev-
pen and objects looked larger. Which one of the eral car accidents secondary to seizures. Which
following is most likely? one of the following is most likely?
a. Absence seizures a. Absence seizure
b. Frontal seizures b. Complex partial seizure
c. Mesial temporal lobe seizures c. Focal motor seizure
d. Occipital seizures d. Generalized tonic-clonic seizure
e. Parietal seizures e. Simple partial seizure

17. A 24-year-old female presented with a history


of severe migraines as a child associated with QUESTIONS 19–20
nausea and vomiting. She continued to have sei-
Additional questions 19–20 available on
zures beginning with flashing lights and colors ExpertConsult.com
in the left visual field followed by amaurosis and
Neurosurgery Books Full
www.ketabpezeshki.com 66485438-66485457
208 PART II CARE OF THE NEUROSURGICAL PATIENT

EXTENDED MATCHING ITEM (EMI) For each of the following descriptions, select the
most appropriate answers from the list above.
QUESTIONS Each answer may be used once, more than once
or not at all.
21. EEG terminology: 1. Anterior temporal lobe
a. Alpha 2. Posterior temporal lobe
b. Beta 3. Superior frontal lobe
c. Delta
d. Fast 23. Mimics of epilepsy:
e. Gamma a. Cardiogenic syncope
f. Lambda b. Cough syncope
g. Sharp c. Gastroesophageal reflux
h. Slow d. Narcolepsy
i. Spike e. Neurally mediated syncope (vasovagal)
j. Theta f. Night terrors
g. Non-epileptiform attack disorder
(pseudoseizure)
For each of the following descriptions, select the h. Paroxysmal dyskinesias
most appropriate answers from the list above. i. Paroxysmal vertigo
Each answer may be used once, more than once j. Rages
or not at all. k. Reflex anoxic seizure
1. Frequency 8-13 Hz l. Shuddering attack
2. Frequency under 4 Hz m. Tic
3. Frequency 14-40 Hz
For each of the following descriptions, select
22. EEG electrode position: the most appropriate answers from the list
a. A1 and A2 above. Each answer may be used once, more
b. C3 and C4 than once or not at all.
c. F3 and F4 1. A 2-year-old child accidentally hits his
d. F7 and F8 head on the kitchen table when running.
e. Fp1 and Fp2 He crys, turns pale and collapses to the floor.
f. O1 and O2 He starts to respond after 30 s but is groggy.
g. P3 and P4 2. A 6-month-old baby is seen to exhibit par-
h. P7 and P8 (or T5 and T6) oxysmal dystonic posturing after feeding.
i. Sp1 and Sp2 3. A 14-year-old boy starts to feel dizzy then
j. T7 and T8 (or T3 and T4) loses consciousness during a car journey
on a hot day. He slumps in his seat and
his limbs are seen to jerk.

SBA ANSWERS
1. b—20-40% onset, (v) inadequate control of seizures for lon-
ger than 4 years, (vi) presence of multiple seizure
FURTHER READING types and frequent generalized tonic-clonic
Devinsky O. Patients with refractory seizures. N Engl J Med seizures. Seizure duration of over 10 years also
1999;340(20):1565-70. decreases likelihood of achieving seizure control
in those who undergo epilepsy surgery.
2. e—Younger age at onset
3. a—1 in 200 per year
In general, predictors for low probability of
epilepsy remission are (i) symptomatic localized Overall rates of SUDEP in adults is 1 in 1000 and
epilepsy secondary to remote CNS injury, lower in children at 0.2-0.4 in 1000 per year.
(ii) abnormalities on neurological examination However, in adults and children with medically
or developmental delay, (iii) persistent epilepti- intractable epilepsy rates are higher at 1 in 100-
form abnormalities on EEG, (iv) older age at 200 per year. Cause of death in SUDEP is unclear

Neurosurgery Books Full


www.ketabpezeshki.com 66485438-66485457
13 SEIZURES 209

but thought to be due to seizure related cardiac altered alertness (sometimes “complex absence”
arrhythmias and/or respiratory compromise. with blinking or head nodding). Seizures are
The most reliable risk factor is severity and fre- typically triggered by hyperventilation or sleep
quency of seizures (particularly generalized deprivation. EEG: Interictal background activity
tonic-clonic seizures). Other risk factors include is usual normal, spike and wave slightly faster
nocturnal seizures, young adult, poor adherence (3.5-4 Hz).
to treatment, earlier age of seizure onset, longer
duration of epilepsy, symptomatic epilepsy and 6. e—Juvenile myoclonic epilepsy
male gender.
Juvenile myoclonic epilepsy comprises 5-10%. It is
FURTHER READING an idiopathic generalized epilepsy with myoclonic
Epilepsy Foundation (www.epilepsy.com/learn/impact/ jerks; may also have typical absence seizures, gen-
mortality/sudep). eralized tonic-clonic seizures, or all three seizure
4. c—Childhood absence epilepsy types. The majority have myoclonic and general-
ized tonic-clonic seizures shortly after awakening
Childhood absence epilepsy is characterized by from sleep. Seizure types have age-specific onset:
brief (4-20 s) unresponsive, staring episodes during absence (5-14 years), myoclonic (9-18 years), and
high-amplitude bisynchronous (generalized) generalized tonic-clonic seizures (9-26 years).
approximately 3-Hz spike and slow wave dis- Sleep deprivation is identified as a precipitating
charges on ictal EEG (with a normal interictal factor in the large majority (>90%) of individuals.
EEG). Accounts for 8-15% of all childhood epilep- The EEG showing paroxysmal spike, polyspike,
sies; onset 4-10 years; history of febrile seizures in and wave complexes that may be regular at 3-
up to 30%, family history of epilepsy in 15%; 5 Hz but often are irregular with 2- to 10-Hz com-
may be induced by hyperventilation. The frequent ponents. A photoconvulsive effect on the EEG has
seizures impair attention and learning and may been described in at least one third of cases.
lead to an inaccurate diagnosis of attention-deficit Hyperventilation less reliably than in childhood
disorder or daydreaming. The majority have absence epilepsy or juvenile absence epilepsy.
accompanying non-stereotyped motor activity
(e.g. blinking, myoclonus of eyelid/mouth, oral 7. a—3-Hz spike and wave discharges
automatisms, picking/rubbing movements of Image with permission from Werz MA, Pita Garcia IL.
hands/feet) and are termed complex absence Epilepsy Syndromes, Elsevier, Saunders, 2011.
seizures. Absence seizures usually remit in adoles-
cence, but a significant number of children go on 8. d—Gelastic seizures
to have generalized tonic-clonic seizures as adults.
Hypothalamic (or tuber cinereum) hamartomas
5. c—Juvenile absence epilepsy are associated with gelastic seizures, visual prob-
lems, central precocious puberty (increased
Juvenile absence epilepsy is a type of idiopathic GnRH) and behavioral change. Gelastic seizures
generalized epilepsy. Onset is from age 8 to manifest as typically short (<30 s) bursts of un-
age 16 years, with a peak occurrence at 10-12 controllable laughter with preservation of
years of age. The frequency of absence seizures consciousness.
in juvenile absence epilepsy is lower than that
in childhood absence epilepsy. A higher fre- Image with permission from Loevner L. Brain Imaging:
quency of generalized tonic-clonic seizures is Case Review Series, 2nd ed., Elsevier, Mosby, 2009.
seen compared with childhood absence epilepsy,
9. e—West syndrome
and there is an increased probability of epilepsy
continuing into the adult years. They also noted
West syndrome is the triad of infantile spasms, hyp-
that 11% of patients with the disorder report a
sarrhythmia on EEG and developmental delay or
family history of epilepsy. Absence seizures are
regression. Onset is typically around 6 months of
predominant. The impairment of consciousness
age (almost all begin within the first year of life);
in juvenile absence epilepsy is moderate and
incidence is 1 in 3225 live births. Spasms may
not generally as severe as in childhood absence
involve brief contractions of predominantly flexor
epilepsy. The level of retained consciousness
or extensor muscle groups ranging from large
may vary significantly from seizure to seizure
“jack-knife” type motions to subtle head bobbing.
in the same patient. Unlike childhood absence
These movements are at times difficult to differ-
epilepsy they may occur once a day or in a cluster entiate from less serious non-epileptic events
in the hour after awakening. The classic clinical
such as gastroesophageal reflux, colic and benign
feature is “simple absence” with staring and

Neurosurgery Books Full


www.ketabpezeshki.com 66485438-66485457
210 PART II CARE OF THE NEUROSURGICAL PATIENT

myoclonus of infancy. Precipitating factors mostly of the head or whole body. Clonic (clusters of
include when falling asleep/waking up, being han- myoclonus) seizures involving the arms, face, or
dled, loud noise, feeding, infection, fever, excite- legs may occur in isolation or in clusters. The
ment, hunger and excessive environmental EEG shows a bisynchronous spike and wave or
temperatures. Cognitive disorders may include polyspikes and wave pattern. Sixty to seventy per-
mental retardation, speech delay, autistic features cent of patients exhibit atypical absence seizures.
and visuomotor dyspraxia. EEG of hypsarrhythmia Hyperventilation does not provoke this seizure
consists typically of a diffuse, very high voltage, dis- type. At least one episode of status epilepticus
organized, chaotic and asynchronous pattern of occurs in more than half of patients over their life-
multifocal spike and wave discharges. This pattern time. Typical presentations include mental status
is seen while the child is both awake and in non- changes or persistent tonic seizures. Medications
rapid-eye-movement (NREM) sleep. In REM are often chosen for the most debilitating seizure
sleep, there is marked reduction or even disappear- type. Broad-spectrum anticonvulsants such as val-
ance of the HA pattern. EEG with simultaneous proic acid have been cited as good choices because
pyridoxine injection is often performed to rule of their efficacy in many seizure types. Surgical
out pyridoxine-dependent seizures (very rare, <1 interventions including vagal nerve stimulator
in 100,000; treatment is with high doses of vitamin placement and corpus callosotomy have been used
B6). Video-EEG analysis is considered the gold in some patients as palliative procedures with vary-
standard to diagnose the spasms, and to assess for ing degrees of success. Lennox-Gestaut is one of
focal features of the spasm semiology or EEG trac- the few epilepsies particullarily responsive to deep
ings. Metabolic screens and LP may help identify brain stimulation of the centromedian nucleus
cause. MRI should be done to look for surgical (Velasco et al.).
lesions. Adrenocorticotropic hormone (ACTH) is
effective in the short-term treatment of infantile 11. c—Landau-Kleffner syndrome
spasms and in the resolution of HA (takes about
2 weeks with an “all or nothing” response). Vigaba- Landau-Kleffner syndrome, or acquired epilep-
trin indicated for pediatric patients aged from tic aphasia, is characterized by a progressive ver-
1 month to 2 years with infantile spasms (but risk bal agnosia in a child with previous normal
of irreversible vision loss; need ophthalmology language development, paroxysmal EEG abnor-
assessment). If EEG and MRI demonstrate a focal malities, and epileptic seizures. Associated symp-
causative lesion surgical resection can be per- toms include psychomotor and behavioral
formed, while functional hemispherectomies may disturbances consisting of motor hyperactivity,
be used in more diffuse abnormalities (e.g. impulsivity, and aggressive behavior. There is a
Sturge-Weber). slight male predominance, and it typically pre-
sents at between 3 and 8 years of age. Onset is
Image with permission from Werz MA, Pita Garcia IL. usually acute and presents initially as difficulty
Epilepsy Syndromes, Elsevier, Saunders, 2011. comprehending language, followed by verbal
agnosia. Subsequently, expressive language is
10. c—Lennox-Gastaut syndrome also affected and may lead to mutism. The
EEG background activity during wakefulness is
Lennox-Gastaut syndrome is characterized by
usually normal; severe and variable abnormalities
multiple generalized seizure types that are refrac-
during sleep. Neuroimaging without any struc-
tory to treatment, cognitive dysfunction, and an
tural brain lesion is required for diagnosis.
interictal slow spike and wave pattern with a slow
PET scans have demonstrated regions of hypo-
background on EEG. May be idiopathic or symp-
metabolism predominantly in the temporal
tomatic (e.g. hypoxic-ischemic, traumatic brain lobes. Seizures are usually well controlled with
injury). Onset is generally between 2 and 6 years
traditional antiepileptic drugs but the neuropsy-
of age. A wide range of behavioral problems will
chological manifestations have a more variable
develop in about half of patients. Approximately
response. Valproate, clobazam, and ethosuxi-
25% of patients will have a history of infantile
mide, either monotherapy or in polytherapy,
spasms. The most common manifestation is a drop
are successful in controlling seizures and are
attack. Drop attacks may range from a simple head
occasionally successful in reversing language
drop to a fall if proximal legs are involved. Because
regression. Carbamazepine, phenobarbital, and
the drop attack may be due to a tonic, atonic, or
phenytoin can lead to worsening of seizures
myoclonic seizure, video-EEG monitoring may
and conversion to electrical status epilepticus
be necessary. Tonic seizures may involve the arms, during slow sleep because they may increase
legs, or whole body. Tonic seizures often occur in
cortical synchronization. Corticosteroids can pro-
sleep. Atonic seizures appear as loss of postural tone
vide dramatic improvement in language, cognition,

Neurosurgery Books Full


www.ketabpezeshki.com 66485438-66485457
13 SEIZURES 211

and behavior in some children. In CSWS, seizures automatism. There are frequently bilateral and
are the first symptom in 70-80% of children, fol- involve both legs and arms with features of run-
lowed by neuropsychological regression that may ning, pelvic thrusts, and bizarre behavior. Upper
be global or selective regression of cognitive func- extremity automatisms tend to be irregular,
tions, excluding the acquired aphasia characteristic involving proximal muscles. Later during the sei-
of Landau-Kleffner syndrome. zures laughing and crying may be observed.
Finally, hypermotor activity characterized by
12. d—Hypoxia-ischemia complex movements of the proximal segments
of limbs and trunks may occur. In contrast, tem-
Hypoxia-ischemia (i.e. asphyxia) is traditionally poral lobe seizures have early and prominent
considered the most common cause associated oroalimentary automatism and repetitive upper
with neonatal seizures. Intrauterine factors before extremity automatism involving mainly the distal
labor can result in fetal asphyxia without later segments. Consciousness is more frequently
documentation of acidosis at birth. Both antepar- affected or lost in temporal lobe epilepsy.
tum and intrapartum maternal and placental MRI Abnormalities include encephalomalacia,
illnesses associated with thrombophilia, pre- neoplasm, vascular malformations, cortical
eclampsia, or specific uteroplacental abnormali- dysplasias, and migrational disorders.
ties such as abruptio placentae or cord
compression may contribute to fetal asphyxial
stress leading to metabolic acidosis. Antepartum
maternal trauma and chorioamnionitis are addi-
tional conditions that also contribute to the intra- Focus Typical Description
uterine asphyxia secondary to uteroplacental Supplementary Bilateral tonic or dystonic
insufficiency. Intravascular placental thromboses sensorimotor area postures that are usually
and infarction of the placenta or umbilical cord asymmetric. Despite
vocalization or speech arrest,
documented after birth are markers for possible consciousness is preserved.
fetal asphyxia. Meconium passage into the amni- The typically described fencing
otic fluid also promotes an inflammatory posture is rarely seen
response within the placental membranes, poten- Orbitofrontal Staring, alteration of
tially causing vasoconstriction and resultant consciousness, olfactory
asphyxia. Postnatal include persistent pulmonary hallucinations and oral and
hypertension of the newborn, cyanotic congenital upper extremity automatisms
and autonomic signs. Difficult to
heart disease, sepsis, meningitis, encephalitis, and distinguish from TLE
primary intracranial hemorrhage are leading
diagnoses. Fetal inflammatory response may Frontopolar Near-onset head version with
seizures associated loss of
increase in the risk of unexplained early-onset sei- consciousness
zures after intrapartum maternal fever. Other
causes include hypoglycemia, hypocalcemia and Dorsolateral Can start with an unspecific aura
frontal of cephalic sensation or fear.
pyridoxine-dependent epilepsy (rare). This is followed by head and eye
version and contralateral tonic
13. c—Fronto-parietal operculum and clonic activity. If Broca's
area is involved, speech arrest
Frontal lobe epilepsy is the second most common or postictal aphasia may be
localization-related epilepsy, accounting for observed
20-30% of surgical series. These types of seizures Fronto-parietal Rolandic seizures are
are usually short, occur more frequently during operculum characterized by facial (mouth
and tongue) clonic movements
sleep, and tend to cluster. Lateralizing signs (which may be unilateral),
include version and unilateral clonic, tonic, or laryngeal symptoms,
dystonic activity that correlate with contralateral articulation difficulty,
onset. Unless there is secondary generalization, swallowing or chewing
responsiveness persists throughout the seizure. movements and hyper-
salivation. Sensory (e.g.
Complex partial seizures of frontal lobe origin epigastric) and experiential (e.g.
are characterized by partial or complete loss of fear) aura and autonomic
consciousness. Most patients report an initial aura (urogenital, gastrointestinal,
of vague general body sensation or unspecified cardiovascular or respiratory)
features are common.
cephalic aura. The patient initially may develop Gustatory hallucinations are
staring and behavioral arrest. Motor manifesta- particularly common
tions consist of prominent semi-purposeful
Continued on following page

Neurosurgery Books Full


www.ketabpezeshki.com 66485438-66485457
212 PART II CARE OF THE NEUROSURGICAL PATIENT

seizures during daytime, repeated generalized


Focus Typical Description tonic-clonic seizures, prolonged seizures, and sta-
Cingulate gyrus Usually involve loss of tus epilepticus as well as seizure onset before the
awareness, oral and upper age of 4 years. Carbamazepine may control sei-
extremity automatism,
behavioral alterations, and
zures in up to 65% of patients.
autonomic manifestations that
include tachycardia and Image with permission from Werz MA, Pita Garcia IL.
tachypnea. In addition, absence- Epilepsy Syndromes, Elsevier, Saunders, 2011.
like events are observed with
mesial structure foci 16. e—Parietal seizures
Motor cortex Produce tonic or clonic activity
seizures of the contralateral face, arm, or Parietal lobe seizures are infrequent in both medical
leg depending on the area of and surgical cases, representing from 4.4% to 6% of
origin. Jacksonian march (distal all epilepsies. The etiology of parietal lobe seizures
limb to ipsilateral face) is not has a strong association with space-occupying
uncommon especially with
involvement of the motor
lesions, which may include tumors, congenital
extremity. A postictal Todd's anomalies, postinflammatory brain scarring, and
paralysis is occasionally seen vascular lesions. As a group, parietal lobe seizures
are usually suspected by somatosensory symptom-
atology at onset. All sensory modalities may be
experienced, and often more than one type is per-
14. a—Frontal pole ceived in each seizure. The two most common man-
ifestations include paresthetic seizures and painful
This ictal EEG begins with attenuation and then seizures. Elementary paresthesia is the most com-
low-amplitude fast frequencies frontocentrally mon somatosensory perception in seizures. These
(Fp1-F3, Fp1-F7 and Fp2-F4, Fp2-F8) before most commonly are reported as tingling and/or
evolving into more generalized irregular poly- numbness. Other descriptions may be pins and nee-
spike and wave. dles, prickling, or a crawling under the skin. Sensa-
tion usually starts in a segment of the limb, usually
Image with permission from Werz MA, Pita Garcia IL. distal, and then spreads to involve the whole extrem-
Epilepsy Syndromes, Elsevier, Saunders, 2011. ity in a march-like fashion. Painful seizures are usu-
ally experienced in conjunction with other sensory
15. a—Benign Rolandic epilepsy perceptions. They are described as severe, stabbing,
throbbing, or cramp-like. The hands are most com-
Benign focal epilepsy with centrotemporal spikes monly involved, especially distal, followed by the
(BECTS; Benign Rolandic epilepsy) is the most head, face, and legs.
frequent focal epilepsy syndrome in childhood. Less frequent somatosensory seizures of parie-
Age at onset ranges from 3 to 13 years of age, tal lobe origin include thermal perception, sexual
and peak incidence is usually around age 8 years. seizures, ideomotor apraxia, and disturbances of
Seizures present usually at night, often shortly body image. Tonic posturing, clonic activity,
after falling asleep or before waking up. The pre- contralateral version, and hypermotor activity
sent as somatosensory aura with perioral pares- may characterize subsequent spread of the ictal
thesias, a sensation of choking, and jaw or focus to the frontal lobe. If the focus spreads to
tongue stiffness. Hemifacial or hemibody motor the temporal lobes it produces automatism and
seizures, frequently with unilateral clonic, but alteration of consciousness. Posterior seizure
also tonic or tonic-clonic, activity are seen in up spread results in visual auras.
to 34% of patients. Motor features involve the
face, lips, tongue, pharynx, and larynx, and this 17. b—Occipital cortical dysgenesis
may be associated with speech arrest. Generalized
tonic-clonic seizures without focal onset have Occipital lobe epilepsies are uncommon (5%) and
been described in 54%. The characteristic inter- identified by the presence of visual phenomena in
ictal EEG finding is a distinct high-amplitude, the early seizure symptoms. In patients with sei-
diphasic spike with prominent aftergoing slow zures arising from the medial occipital lobe, visual
wave typically in C3/C4 or T3/T4 electrodes. field deficit can be found in 20-40%. The vision
Spikes are more frequently observed during loss often is unnoticed by the patient, may be sub-
sleep, and sleep activation of spikes is a salient fea- tle enough to be missed on confrontational test-
ture of BECTS. The condition remits spontane- ing as part of the neurologic examination, and
ously in almost all patients around the age of may require formal visual field testing to be
16 years hence indications for treatment include appreciated. The most common auras are simple

Neurosurgery Books Full


www.ketabpezeshki.com 66485438-66485457
13 SEIZURES 213

visual hallucinations (white or colored lights that 1. The most common aura is a rising epigas-
can be constant, flashing, stationary or moving). tric sensation.
Ictal blurring or amaurosis (blindness) is reported 2. Fear is common as well and must be distin-
in 25-40% of case series and can be described as guished from panic attacks.
either a whiteout or blackout. When propagation 3. Experiential auras may include dreamy
is infrasylvian and lateral to the temporoparietal states, déjà vu, and jamais vu.
visual association areas, complex visual hallucina- 4. Sensory auras include smells, which have
tions and illusions (achromatopsia, micro/macro- reliable localization but are actually quite
psia, metamorphopsia, micro/macroproxiopia, or rare. Other sensory manifestations that
palinopsia) may occur. In contrast, propagation to are not localizing include cephalic sensa-
the mesial temporal lobe may result in reports of tion, numbness, tingling, and hearing music
rising epigastric sensation, smells, and nausea and or phrases. Spread of the seizures to the sec-
complex partial seizures. Suprasylvian parietal ondary sensory area that sits in the posterior
spread may lead to reports of somatosensory phe- operculum may produce unusual distribu-
nomena such as paresthesias and vertigo. Spread tions of sensory symptoms that may be ipsi-
from the occipital lobes may evolve to complex lateral or bilateral.
partial seizures typical of temporal origin with 5. Auras may manifest with autonomic symp-
prominent oral and gestural automatisms, to lat- toms such as changes in heart rate, piloerec-
eral frontal lobe with clonic motor activity, or to tion, and urinary urgency.
mesial frontal lobe with tonic seizure manifesta- 6. Simple auditory auras such as buzzing,
tions typical of supplementary motor seizures. roaring, and muffling usually arise from
Apart from focal lesions occipital seizures also the temporal neocortex on the superior
occur as part of neurodegenerative disorders such temporal gyrus near Heschl's gyrus.
as the progressive myoclonic epilepsies (LaFora 7. Complex visual phenomena usually occur
body disease, ceroid lipofuscinosis), mitochon- from the posterior temporal lobe near the
drial disorders (MERRF and MELAS) and poste- occipital lobe.
rior reversible leukoencephalopathy syndrome. Auras often gradually evolve to complex partial
seizures. These seizures can be extremely bland
18. b—Complex partial seizures with a simple stare and behavioral arrest or can
classically have oroalimentary automatisms such
Most adult focal epilepsies arise from the tempo- as lip smacking, chewing, or swallowing along
ral lobes (60%). Auras (simple partial seizures) with gestural automatisms such as picking or fum-
and complex partial seizures are the hallmarks bling movements. Seizure spread to the frontal
of temporal lobe seizures. Unfortunately, patients lobes may produce proximal automatisms such
evolving to complex partial seizures may become as bicycling or thrashing. Secondarily, generalized
amnestic for the aura. Auras of right temporal ori- tonic-clonic seizures may also occur, although
gin are more commonly remembered than those typically infrequently.
from the left. Most seizures originate from the
mesial temporal structures, including the amyg-
dala and hippocampal formation. The amygdala ANSWERS 19–20
has associations with emotion and autonomic
and olfactory systems. The hippocampus has Additional answers 19–20 available on
roles in memory and experiential phenomena. ExpertConsult.com
Common auras are described below:

EMI ANSWERS
21. 1—a, Alpha, 2—c, Delta, 3—b, Beta

Alpha Frequency 8-13 Hz


Delta Frequency <4 Hz
Beta Frequency 14-40 Hz
Theta Frequency 4-8 Hz
Gamma Frequencies >40 Hz

Continued on following page

Neurosurgery Books Full


www.ketabpezeshki.com 66485438-66485457
214 PART II CARE OF THE NEUROSURGICAL PATIENT

Lambda Diphasic sharp transient occurring over the occipital regions of the head of waking subjects during
visual exploration. The main component is positive relative to other areas. Time-locked to saccadic
eye movement. Amplitude varies but is generally below 50 mV
Fast Activity of frequency faster than alpha (i.e. beta and gamma activity)
Slow Activity of frequency slower than alpha (i.e. theta and delta activity)
Sharp A transient, clearly distinguished from background activity, with pointed peak at a conventional
paper speed or time scale and duration of 70-200 ms
Spike A transient, clearly distinguished from background activity, with pointed peak at a conventional
paper speed or time scale and a duration from 20 to 70 ms

22. 1—d, F7 and F8, 2—j, T7 and T8 (or T3 and T4), 3—c, F3 and F4

NZ

Fp1 Fp2
FPZ FP2
FP1

F9 AF9 AF8 F10


AF3 AFZ AF4
F7 F8 F7 F8
F3 F4 F5 F6
FZ F3 F4
F1 FZ F2
FT9 FT7 FT8 FT10
FC5 FC6
FC3 FC FC FC2 FC4
1 Z

A1 T3 C3 CZ C4 T4 A2 C2 C4 C6 T10 A2
A1 T9 T7 C5 C3 C1 CZ T8

CP3 CP1 CPZ CP2 CP4 CP6


TP7 CP5 TP8
PZ TP9 P1 PZ P2 TP10
P3 P4 P6 P3 P4 P6
P7 P8
T5 T6
P9 PO3 POZ PO4 P10
PO7 PO8
O1 OZ O2
O1 O2
A B IZ

Images with permission from a, Libenson MH. Practical Approach to Electroencephalography, Elsevier, Saunders,
2010 and b, American Clinical Neurophysiology Society Guideline 5: guidelines for standard electrode position
nomenclature. J Clin Neurophysiol 2006;2:107-110.

The electrode nomenclature for the original Electrode Common Name


10-20 electrode system is shown. This original
naming system is still in use in many EEG labo- P7 and P8 (T5 and T6) Posterior temporal
ratories. Newer modified version of 10-20 P3 and P4 Parietal
replaces T3 and T4 with T7 and T8, to fit with Pz Parietal midline
the 10-10 system (right).
O1 and O2 Occipital
Sp1 and Sp2 Sphenoidal
Electrode Common Name
A1 and A2 Auricular
Fp1 and Fp2 Frontopolar or frontal polar
Table modified from Libenson MH, Practical Approach to
F7 and F8 Anterior temporal Electroencephalography, Elsevier, Saunders, 2010.
F3 and F4 Superior frontal
Fz Frontal midline
T7 an T8 (T3 and T4) Midtemporal
C3 and C4 Central
Cz Vertex or central midline

Continued

Neurosurgery Books Full


www.ketabpezeshki.com 66485438-66485457
13 SEIZURES 215

23. 1—k, Reflex anoxic seizure, 2—c, Gastroesophageal reflux, 3—e, Neurally mediated syncope

Reflex anoxic seizure (pallid Common in young children, especially under 2 years. Unexpected stimulus
Breath holding spell) (pain, shock, fright) causes excessive vagal activity—heart and respiration
stops transiently and child becomes pale
Cyanotic breath holding spell Reflex expiration in response to anger/frustration causes child to become
cyanotic
Cardiogenic syncope Syncope resulting from structural or functional cardiac abnormality—no
convulsive movements associated
Cough syncope Prolonged cough spasms (e.g. asthmatic, infection) can reduce venous
return and lead to syncope with incontinence
Gastroesophageal reflux Can lead to paroxysmal dystonic posturing associated with meals due to
discomfort
Narcolepsy Sudden loss of muscular tone secondary to catapexy, usually an emotional
trigger, no postictal state or loss of consciousness, EEG shows recurrent
attacks of REM sleep
Night terrors Brief nocturnal episodes of terror without typical convulsive movements,
common in ages 4-6
Paroxysmal dyskinesias Precipitated by sudden movement or startle, no associated change in
consciousness
Paroxysmal vertigo Common in toddlers, triggered by fright/crying—seen to stagger, fall and
possibly vomit
Non-epileptiform attack disorder No EEG changes except movement artefact during episode
(pseudoseizure)
Rage attacks/Tantrum Common in children aged 6-12 years, outburst is explosive and out of
proportion to trigger (tantrums are goal directed)
Shuddering attack Shiver-like movement of the trunk with associated stiffening, neck flexion and
arm adduction, each episode lasts seconds and there is no change in
consciousness
Vasovagal syncope Loss of consciousness triggered by postural change, heat or emotion; there is
presyncopal dizziness, clouded/tunnel vision before a slow collapse
Tic Involuntary, non-rhythmic, repetitive movements not associated with
impaired consciousness

Neurosurgery Books Full


www.ketabpezeshki.com 66485438-66485457
CHAPTER 14

NEURORADIOLOGY
SINGLE BEST ANSWER (SBA) QUESTIONS
1. A patient with severe renal failure but not on 4. A ring-enhancing lesion on contrast CT has
dialysis required CT angiogram. In addition the below appearance on DWI (C- DWI,
to keeping the patient well hydrated, what D- ADC map) sequence. Which one of the
may be given to the patient to minimize following is of most concern?
contrast nephropathy?
a. Hydrocortisone
b. Magnesium sulfate
c. N-Acetylcysteine
d. Ramipril
e. Sodium bicarbonate

2. Early subacute subdural hematoma (4-7 days


old) is most likely to display which one of the
following characteristics on MRI?
a. T1 hypointense, T2 hypointense
b. T1 hypointense, T2 isointense
c. T1 hypointense, T2 hyperintense A B
d. T1 isointense, T2 hypointense
e. T1 isointense, T2 hyperintense
f. T1 hyperintense, T2 hypointense
g. T1 hyperintense, T2 isointense a. Cerebral abscess
h. T1 hyperintense, T2 hyperintense b. Encephalitis
c. Glioblastoma multiforme
3. True diffusion restriction is best assessed by d. Metastasis
looking at which one of the following MR e. Radiation necrosis
sequences?
a. Diffusion tensor imaging 5. Which one of the following combinations of
b. Diffusion weighted image and apparent findings on perfusion weighted MR suggests
diffusion coefficient map early infarction
c. Fractional anisotropy map a. Bright on DWI, low rCBF, long rMTT,
d. T2 gradient echo low CBV
e. T1 with gadolinium b. Bright on DWI, high rCBF, long rMTT,
low CBV
c. Bright on DWI, low rCBF, short rMTT,
low CBV
d. Bright on DWI, low rCBF, long rMTT,
high CBV
e. Dark on DWI, low rCBF, long rMTT,
low CBV

216
Neurosurgery Books Full
www.ketabpezeshki.com 66485438-66485457
14 NEURORADIOLOGY 217

6. Which one of the following tracts is most 8. Which one of the following tracts is most
likely depicted in the figure below? likely depicted in the figure below?

a. Cingulum
b. Corona radiata
c. Corpus callosum
d. U fibers
e. Vertical occipital fasciculus
a. Corpus callosum
7. Which one of the following tracts is most b. Corona radiata
likely depicted in the figures below? c. Corticospinal tract
d. Inferior frontooccipital fasciculus
e. Superior frontooccipital fasciculus

9. Which one of the following tracts is most


likely depicted in the figure below?

a. Anterior commissure a. Corona radiata


b. Extreme capsule b. Corticospinal tract
c. Fornix c. Medial longitudinal fasciculus
d. Posterior commissure d. Reticulospinal tract
e. Posterior limb of internal capsule e. Spinothalamic tract

Neurosurgery Books Full


www.ketabpezeshki.com 66485438-66485457
218 PART II CARE OF THE NEUROSURGICAL PATIENT

10. Which one of the following tracts is most 12. Which one of the following tracts is most
likely depicted in the figure below? likely depicted in the figure below?

a. Arcuate fasciculus
b. Cingulum bundle
c. Fornix
d. Inferior longitudinal fasciculus
a. Arcuate fasciculus e. Superior longitudinal fasciculus
b. Cingulum bundle
c. Corona radiata 13. Which one of the following tracts is most
d. Corticospinal tract likely depicted in the figure below?
e. SFOF

11. Which one of the following tracts is most


likely depicted in the figure below?

a. Inferior fronto-occipital fasciculus


b. Inferior longitudinal fasciculus a. Cingulum
c. Fornix b. Posterior commissure
d. Superior longitudinal fasciculus (II and III) c. Superior fronto-orbital fasciculus
e. U-fibers d. Superior longitudinal fasciculus
e. U association fiber

Neurosurgery Books Full


www.ketabpezeshki.com 66485438-66485457
14 NEURORADIOLOGY 219

14. Which one of the following tracts is most 16. Which one of the following tracts is most
likely depicted in the figure below? likely depicted in the figure below?

a. Anterior commissure
b. Anterior region of the corona radiata
c. Fornix
d. Optic tract
e. Uncinate fasciculus

17. In MR Spectroscopy, Hunter's angle is best


described as which one of the following?

NAA
15
a. Arcuate fasciculus
b. External capsule
c. Fornix
d. Inferior fronto orbital fasciculus
10
e. Inferior longitudinal fasciculus
Signal intensity

Cr

15. Which one of the following tracts is most


likely depicted in the figure below? Cho
5

4 3 2 1
Chemical shift / ppm

a. The rough angle formed with the x axis


when a line is drawn between choline
and creatine peaks
b. The rough angle formed with the x axis
when a line is drawn between NAA and
creatine peaks
c. The rough angle formed with the x axis
when a line is drawn between choline, cre-
atine and NAA peaks
d. The rough angle formed with the x axis
a. Commissure of Probst when a line is drawn between lactate and
b. Corona radiata lipid peaks
c. Inferior fronto orbital fasciculus e. The rough angle formed with the x axis
d. Inferior longitudinal fasciculs when a line is drawn between choline
e. Internal capsule and lactate peaks

Neurosurgery Books Full


www.ketabpezeshki.com 66485438-66485457
220 PART II CARE OF THE NEUROSURGICAL PATIENT

g. Gradient echo
QUESTIONS 18–26 h. MR Spectroscopy
Additional questions 18–26 available on
i. Phase-contrast
ExpertConsult.com j. STIR fat suppression
k. Susceptibility weighted imaging
l. T1
m. T1 with gadolinium
n. T2
EXTENDED MATCHING ITEM (EMI) o. Time of flight
QUESTIONS For each of the following descriptions, select the
most appropriate answers from the list above.
27. Tractography:
Each answer may be used once, more than once
a. Anterior commissure
or not at all.
b. Anterior region of the corona radiata
1. CSF flow studies
c. Arcuate fasciculus
2. Cavernous malformations
d. Corticobulbar tract
3. Resection of an eloquent temporal glio-
e. Corticospinal tract
ma in an intact patient presenting with
f. Cingulum
headache
g. Cerebral peduncle
4. Consideration of microvascular decom-
h. Fornix
pression for trigeminal neuralgia
i. Inferior fronto-occipital fasciculus
j. Inferior longitudinal fasciculus 29. Imaging Modalities:
k. Optic tract a. B-mode Ultrasound
l. Superior fronto-occipital fasciculus b. CT intracranial angiogram
m. Superior longitudinal fasciculus c. CT myelogram
n. Uncinate fasciculus d. CT venogram cerebral
o. Vertical occipital fasciculus e. DAT scan
f. Doppler ultrasound
For each of the following descriptions, select the g. FDG-PET CT
most appropriate answers from the list above. h. Indium-11 Diethylenepentaacetic acid
Each answer may be used once, more than once study
or not at all. i. MIBG scan
1. Pathway important for recognition of lan- j. MR perfusion
guage and appropriate response. k. Somatostatin-PET CT
2. Pathway important for object recognition l. SPECT
and discrimination, semantic processing m. Xenon-133 CT
and visuospatial processing.
3. Pathway with role in linking object repre- For each of the following descriptions, select the
sentations to their lexical labels and face most appropriate answers from the list above.
recognition. Each answer may be used once, more than once
4. Pathway with role in peripheral vision, visual or not at all.
perception of motion, and visual spatial 1. Postoperatively to distinguish between
processing. tumor recurrence and radiation necrosis
5. Pathway with role in initiation of motor activ- 2. Cisternography
ity and higher-order control of body-centered 3. Intraoperative localization of spinal
action, spatial attention and gestural compo- cord tumor
nents of language and orofacial working
memory.
30. MR Spectroscopy:
28. MRI Sequences: a. Alanine
a. BOLD Functional MRI b. Choline
b. Diffusion tensor imaging c. Citrate peak
c. Diffusion weighted imaging (Echo planar) d. Creatine/phosphocreatine
d. Fast spin echo e. Gamma-aminobutyric acid
e. FIESTA f. Glucose
f. FLAIR CSF suppression g. Glutamate (Glu)/Glutamine peak

Neurosurgery Books Full


www.ketabpezeshki.com 66485438-66485457
14 NEURORADIOLOGY 221

h. Lactate h. T2 isointense signal, DWI isointense or


i. Myo-inositol (ml) high signal, ADC low signal
j. N-acetyl aspartate i. T2 isointense signal, DWI high, ADC
low signal
For each of the following descriptions, select the j. T2 isointense signal, DWI high signal in
most appropriate answers from the list above. border and low within lesion, ADC high
Each answer may be used once, more than once signal
or not at all. k. T2 high signal, DWI low signal, ADC
1. Reference metabolite in MR spectroscopy high signal
2. Marker for cellular turnover l. T2 high signal, DWI isointense or low
3. Marker for ischaemia and necrosis signal, ADC low signal
m. T2 high signal, DWI high signal in bor-
31. Diffusion weighted imaging: der and low within lesion, ADC low
a. T2 low signal, DWI low signal, ADC low signal
signal n. T2 high signal, DWI high, ADC isoin-
b. T2 low signal, DWI isointense or low tense signal
signal, ADC low signal o. T2 high signal, DWI high signal, ADC
c. T2 low signal, DWI isointense or high low signal
signal, ADC low signal
d. T2 low signal, DWI high signal in border For each of the following descriptions, select the
and low within lesion, ADC low signal most appropriate answers from the list above.
e. T2 low signal, DWI high signal, ADC Each answer may be used once, more than once
high signal or not at all.
f. T2 isointense signal, DWI low signal, 1. Epidermoid
ADC low signal 2. Arachnoid cyst
g. T2 isointense signal, DWI isointense or 3. Ring-enhancing cerebral abscess
low signal, ADC low signal 4. Ring-enhancing glioblastoma multiforme

SBA ANSWERS
1. c—N-acetylcysteine The table below describes the appearance of sub-
dural blood clots on MRI as they age:
2. f—T1 hyperintense, T2 hypointense

Clot Age T1WI T2WI Explanation Mnemonic*


Hyperacute Isointense Hyperintense Large amount of diamagnetic oxyhemoglobin IBBY
(4-6 h) which does not affect T1/T2. Appearance due
to high water content.
Acute (7-72 h) Isointense Hypointense Deoxyhemoglobin is paramagnetic and IDDY
causes T2 effect but not T1
Early Hyperintense Hypointense Intracellular methemoglobin is paramagnetic BIDDY
Subacute
(4-7d)
Late subacute Hyperintense Hyperintense Hemolysis results in extracellular BABY
(1-4w) methemoglobin increasing T2 hyperintensity
Early chronic Hyperintense Hyperintense Pool of dilute free methemoglobin surrounded
(weeks to by thin rimmed T2 hypointense wall of ferritin
months) and hemosiderin deposits
Late chronic Hypointense Hypointense Ferritin and hemosiderin diffuse DUDU
(months to throughout clot
years)

*I ¼ isointense, B ¼ Bright (hyperintense), D ¼ Dark (hypointense).

Neurosurgery Books Full


www.ketabpezeshki.com 66485438-66485457
222 PART II CARE OF THE NEUROSURGICAL PATIENT

3. b—A Diffusion weighted image and appar- bimanual finger movements. Commissural
ent diffusion coefficient maps fibers passing through the posterior corpus cal-
losum play an important role in visual and
Diffusion restriction is assessed with a diffusion visuospatial integration. Callosal fibers are also
weighted imaging protocol which includes a T2 important for higher-order cognition, includ-
weighted sequence (low b ¼ 0), diffusion weighted ing normal social, attentional, and emotional
sequence (high b ¼ 1000) and apparent diffusion function.
coefficient map. The b-value parameter identifies
the measurement's sensitivity to diffusion and Image with permission from Naidich T, Castillo M,
determines the strength and duration of the diffu- Cha S, Smirniotopoulos J. Imaging of the Brain, Else-
sion gradients. A b value of 0 produces a T2 vier, Saunders, 2013.
weighted image for anatomical reference. In the
7. c—Fornix
range of clinically relevant b values (i.e. up to
1000) the greater the b value the stronger the dif-
The fornix is part of the dorsal limbic system and
fusion weighting and the higher the contrast in the Papez circuit. It participates in high-level
pathogenic regions. A minimum of two b-values
mental processes relevant to episodic memory
must be acquired for an apparent diffusion coeffi-
and emotion. It also provides the main choliner-
cient map, which is a measure of the strength of
gic input to the hippocampus.
diffusion in tissue after eliminating any overlying
T2 shine through/contrast effects. T2 shine- Image with permission from Naidich T, Castillo M, Cha
through refers to high signal on DWI images that S, Smirniotopoulos J. Imaging of the Brain, Elsevier,
is not due to restricted diffusion, but rather to high Saunders, 2013.
T2 signal which “shines through” to the DWI
image. T2 shine through occurs because of long 8. b—Corona Radiata
T2 decay time in some normal tissue. This is most
often seen with subacute infarctions due to vaso- The fibers of the corona radiata interconnect the
genic edema but can be seen in other pathologic cerebral cortex with the thalamus and brainstem
abnormalities e.g. epidermoid cyst. To confirm true in both directions. From anterior to posterior,
restricted diffusion one should always compare the they include (1) the thalamic connections to
DWI image to the ADC. In cases of true restricted the frontal lobes and the frontopontine motor
diffusion, the region of increased DWI signal will fibers that pass through the anterior limb of the
demonstrate low signal on ADC. ADC is a value internal capsule; (2) the thalamic connections to
that measures the effect of diffusion independent the anterior parietal lobe and the corticonuclear
of the influence of T2 shine-through. ADC maps motor projections that pass through the genu;
thus portray restricted diffusion, such as in ischemic and (3) the thalamic connections to the central
injury, as hypointense lesions relative to normal parietal and occipitotemporal lobes and corticosp-
brain. In contrast, in cases of T2 shine-through, inal, corticopontine, and corticotegmental motor
the ADC will be normal or high signal. fibers that pass through the posterior limb of the
internal capsule. The thalamic radiations to and
4. a—Cerebral abscess from the cortex are grouped into four thalamic
peduncles.
Images with permission from Adam A, et al. (Eds.).
Grainger & Allison's Diagnostic Radiology, 6 th ed., Image with permission from Naidich T, Castillo M, Cha
Elsevier, Churchill Livingstone, 2014. S, Smirniotopoulos J. Imaging of the Brain, Elsevier,
Saunders, 2013.
5. a—Bright on DWI, low rCBF, long rMTT,
low CBV 9. b—Corticospinal tract

6. c. The corpus callosum facilitates interhemi- The corticospinal tract is the predominant
spheric interactions for communicating and pathway for the relay of impulses for voluntary
integrating perceptual, cognitive, learned, skilled movements of the upper extremities,
and volitional information. It is important for trunk, and lower extremities. It connects pyra-
the performance of visual and tactile tasks that midal Giant cells of Betz in layer V of primary
require transfer of sensory information motor cortex to alpha motor neurons, decussat-
between the cerebral hemispheres. Commis- ing in the medulla.
sural fibers crossing through the anterior por-
tion and body of the corpus callosum are Image with permission from Naidich T, Castillo M, Cha
essential to perform temporally independent S, Smirniotopoulos J. Imaging of the Brain, Elsevier,
Saunders, 2013.

Neurosurgery Books Full


www.ketabpezeshki.com 66485438-66485457
14 NEURORADIOLOGY 223

10. b—Cingulum This connection provides for the ability to recog-


nize language and respond to it appropriately. Indi-
The cingulum bundle is the major component of viduals with more symmetric patterns of
the dorsal limbic pathway. It is involved in a wide connection perform better overall on word tasks
range of motivational and emotional aspects of of semantic association. It has been considered by
behavior and participates in spatial working some to be the fourth portion of SLF.
memory. It interconnects the hippocampus and
parahippocampal gyrus (critical for memory) with Image with permission from Naidich T, Castillo M, Cha
the (1) prefrontal areas (important for manipulat- S, Smirniotopoulos J. Imaging of the Brain, Elsevier,
ing information, monitoring behavior and work- Saunders, 2013.
ing memory) and (2) rostral cingulate gyrus
(involved in motivation and drive). It can be 13. c—Superior fronto-orbital fasciculus
lesioned or stimulated to treat pain, obsessive
compulsive disorder or depression. The limbic The SFOF is significant for peripheral vision,
system is also important for high-level mental visual perception of motion, and visual spatial pro-
processes relevant to memory and emotion. It is cessing. The SFOF connects the superior parietal
part of the Papez circuit that links the hippocam- gyrus (parastriate areas important for peripheral
pus, parahippocampal gyrus, mammillary bodies, vision and visual appreciation of motion) with
thalamus, and cingulate gyrus. Other structures the dorsolateral prefrontal cortex of the middle
subsequently integrated into the limbic system and inferior frontal gyri (necessary for attention)
include the amygdala, septal region, and olfactory
Image with permission from Naidich T, Castillo M, Cha
bulb. These structures have been implicated in S, Smirniotopoulos J. Imaging of the Brain, Elsevier,
dementia, epilepsy, and schizophrenia. Saunders, 2013.

Image with permission from Naidich T, Castillo M, Cha 14. e—Inferior longitudinal fasciculus
S, Smirniotopoulos J. Imaging of the Brain, Elsevier,
Saunders, 2013.
The inferior longitudinal fasciculus has a role in
the ventral visual stream for object recognition,
11. d—Superior longitudinal fasciculus (II discrimination, and memory. It appears to medi-
and III)
ate the fast transfer of visual signals to anterior
temporal regions and neuromodulatory back-
The SLF is significant for initiation of motor
projections from the amygdala to early visual
activity and higher-order control of body-
centered action. It connects the superior parietal areas. It likely plays a role in linking object repre-
sentations to their lexical labels. Face recognition
lobule (important for limb and trunk location in
probably depends on the ILF, because disruption
body-centered space) with premotor areas
of the tract has been implicated in associative
(engaged in higher aspects of motor behavior).
visual agnosia, prosopagnosia, visual amnesia,
The SLF is also significant for spatial attention,
and visual hypoemotionality.
because it connects the inferior parietal lobule
(concerned with visual spatial information) with Image with permission from Naidich T, Castillo M, Cha
the posterior prefrontal cortex (important for S, Smirniotopoulos J. Imaging of the Brain, Elsevier,
perception and awareness). Furthermore, the Saunders, 2013.
SLF is relevant to gestural components of lan-
guage and orofacial working memory, because 15. c—Inferior fronto orbital fasciculus
it connects the supramarginal gyrus (concerned
with higher order somatosensory information) This fascicle may be a major component of the
with the ventral premotor area (containing mirror ventral subcortical “what” pathway important
neurons for action imitation). for object recognition and discrimination. The
IFOF most likely also has a significant role in
Image with permission from Naidich T, Castillo M, Cha semantic processing, because it interconnects
S, Smirniotopoulos J. Imaging of the Brain, Elsevier, the occipital associative extrastriate cortex with
Saunders, 2013.
the temporobasal region, two areas important
to semantic processing. The IFOF also functions
12. a—Arcuate fasciculus in visuospatial processing and enables the interac-
tion between emotion and cognition.
The classical (direct) arcuate fasciculus intercon-
nects Wernicke's receptive, auditory word proces- Image with permission from Naidich T, Castillo M, Cha
sing area in the superior temporal lobe with Broca's S, Smirniotopoulos J. Imaging of the Brain, Elsevier,
speech production area in the inferior frontal lobe. Saunders, 2013.

Neurosurgery Books Full


www.ketabpezeshki.com 66485438-66485457
224 PART II CARE OF THE NEUROSURGICAL PATIENT

16. e—The uncinate fasciculus is a ventral limbic EMI ANSWERS


pathway that is critical for processing novel
information, for positive/negative valuations 27. 1—c, 2—i, 3—j, 4—l, 5—m
of the emotional aspects of data, and for
self-regulation. The fibers of the uncinate 28. 1—i, Phase-contrast, 2—k, Susceptibility
fasciculus link the rostral superior temporal weighted imaging, 3—a, BOLD Functional
gyrus (important for sound recognition), MRI, 3—e, FIESTA.
the rostral inferior temporal gyrus (impor-
tant for object recognition), and the medial 29. 1—g, FDG-PET CT, 2—h, Indium-11
temporal area (important for recognition Diethylenepentaacetic acid study, 3—a, B-
memory) with the orbital, medial, and pre- mode ultrasound
frontal cortices (involved in emotion, inhibi-
tion, and self-regulation). The uncinate 30. 1—d, Creatine, 2—b, Choline, 3—h,
fasciculus may also be critical in visual Lactate
learning.
In order to interpret MRS, one needs to under-
Image with permission from Naidich T, Castillo M, Cha stand the function of the different molecules
S, Smirniotopoulos J. Imaging of the Brain, Elsevier,
Saunders, 2013.
being measured. NAA is synthesized in the
mitochondria of neurons, and its function is
17. c—The rough angle formed with the x axis unknown. Clinically, NAA serves as a marker
when a line is drawn between choline, creatine for the presence of neurons, including neuronal
and NAA peaks. Hunter's angle is the formed axons in white matter. Creatine (Cr) is used
by a line approximately joining the ascending clinically as a marker for energy metabolism.
peaks of the metabolites in MR spectroscopy Low levels of creatine suggest that the area of
and is roughly 45°. Myoinositol, Choline, interest is highly metabolically active. Creatine
Creatine and N-Acetyl aspartate peaks are is also often assumed to be stable and is used
ascending in normal spectrum, any alteration for calculating metabolite ratios (e.g., Cho:Cr
in the ascending nature of the peaks means and NAA:Cr). Choline (Cho) is found in the
spectrum is abnormal. cell membrane. It serves as a marker for the cel-
lular turnover of a lesion. Choline is elevated
Image with permission from Mangrum WI. Duke both in the setting of increased cellular produc-
Review of MRI Principles: Case Review Series, Elsevier, tion, such as in a tumor, and in the setting of cel-
Mosby, 2012. lular breakdown, such as in leukodystrophy and
multiple sclerosis. Lactate is a marker for
anaerobic metabolism. Normally, lactate levels
ANSWERS 18–26 in the brain are so low that they cannot be mea-
sured by spectroscopy. Increased anaerobic
Additional answers 18–26 available on metabolism, such as with ischemia or tumor
ExpertConsult.com necrosis, results in lactate peaks. Myoinositol
is a sugar. It is absent from neurons but present

Chemical
Molecule Shift Function Classic Association (" Increased; # Decreased)
Lipids 0.8-1.5 ppm Fat ": Diploic space and subcutaneous fat
Lactate 1.33 ppm Anaerobic activity ": Ischemia, infarction, seizures, metabolic disorders,
necrotic tumors
NAA 2.02 ppm Neuronal/axonal # Leukodystrophy, malignant neoplasm, multiple
marker sclerosis, infarction
" Elevated in Canavan disease
Creatine 3.02 ppm Marker of metabolic Assumed to be unchanged and used to calculate ratios
activity (Cho:Cr and NAA:Cr)
# Tumors.
Choline 3.22 ppm Cellular turnover " Increased in tumors, inflammation, infection, multiple
sclerosis …
Myoinositol 3.56 ppm Glial marker " Gliosis, astrocytosis, Alzheimer's disease

Table with permission from Mangrum WI. Duke Review of MRI Principles: Case Review Series, 2012, Elsevier, Mosby.

Neurosurgery Books Full


www.ketabpezeshki.com 66485438-66485457
14 NEURORADIOLOGY 225

in glial cells. It is used as a marker for glial pro- arachnoid cysts may also be seen as a lesion in
liferation or an increase in glial size. Lipids are the posterior fossa demonstrating similar signal
markers for fat, as is seen in the subcutaneous intensity to CSF but do not show restricted diffu-
tissues or in the diploic space of the calvarium. sion (low DWI, high ADC). Cerebral abscess
Description of Common Spectroscopy Mole- must be considered when a ring enhancing
cules: Their Chemical Shifts, Main Functions, necrotic lesion (usually surrounded by vasogenic
and Classic Associations edema) demonstrates restricted diffusion (bright
DWI, dark ADC). An expansive ring enhancing
31. 1—n, 2—k, 3—o, 4—j cystic/necrotic lesion, surrounded by vasogenic
edema/infiltrative lesion, demonstrating restricted
Epidermoids are usually an expansive lesion in diffusion and high perfusion in its borders but
the left aspect of the posterior fossa, and despite unrestricted diffusion within the lesion is more
being solid they demonstrate similar signal inten- consistent with glioblastoma or metastasis. Other
sity to CSF on T2WI, but demonstrate diffusion highly cellular brain tumors demonstrating
restriction on diffusion-weighted imaging (high restricted diffusion on DWI include lymphoma,
signal DWI, isointense ADC). In contrast, medulloblastoma and anaplastic astrocytoma.

Neurosurgery Books Full


www.ketabpezeshki.com 66485438-66485457
CHAPTER 15

RADIOTHERAPY AND
STEREOTACTIC RADIOSURGERY
SINGLE BEST ANSWER (SBA) QUESTIONS
1. Which one of the following statements about d. Target volume is late responding but only
stereotactic radiosurgery is most accurate? in abnormal tissue and with marked radio-
a. A high dose of radiation is delivered in a biologic effect
single sitting often to a volume limited e. Target volume shows small, early effect
target, typically one lesion is treated how- on abnormal tissue but bigger, late effect
ever more than one can be on normal tissue
b. It is used to deliver prophylactic craniosp-
inal irradiation 5. In which one of the following forms of radi-
c. Radiation is delivered in multiple sessions ation therapy is the Bragg peak utilised to
to a single target focus treatment and minimise collateral dam-
d. Requires a cobalt source for generating age to non-targeted structures?
ionizing radiation a. Brachytherapy
e. Requires a copper source for generating b. CyberKnife radiosurgery
non-ionizing radiation c. Gamma Knife surgery
d. Linac-based radiosurgery
2. Which one of the following utilizes a cobalt- e. Proton beam therapy
60 source for photon production?
a. 3D conformal radiotherapy 6. Which one of the following is not an impor-
b. Carbon-ion therapy tant biological factor explaining the efficacy
c. Gamma Knife surgery of fractionation?
d. Linac-based radiosurgery a. Radiosensitivity
e. Proton beam therapy b. Reassortment (redistribution)
c. Reduction
3. In radiotherapy planning, which one of the d. Reoxygenation
following terms best describes the volume e. Repair
that should be treated to account for the f. Repopulation
tumor, microscopic spread, and setup errors
(systematic and random)? 7. Stereotactic radiotherapy for brain metastasis
a. Clinical target volume is LEAST likely to be appropriate in which
b. Gross tumor volume one of the following situations?
c. Planning organ at risk volume a. CNS and systemic progression of disease,
d. Planning target volume with few systemic treatment options and
e. Systematic target volume poor performance status
b. Local relapse after surgical resection of a
4. Which one of the following descriptions does single brain metastasis
not describe the radiobiological response of c. Oligometastases (1-3) metastases espe-
common intracranial targets for radiosurgery? cially if primary tumor is known to be
a. Target volume contains no abnormal radiotherapy resistant
tissue and normal tissue shows early d. Postsurgical resection of a single BM,
radiobiologic effect especially if 3 cm or smaller and in the
b. Target volume contains no normal tissue posterior fossa
and abnormal tissue shows early radiobio- e. Salvage therapy for recurrent oligometas-
logic effect tases (1-3) after WBRT
c. Target volume is late responding as con-
tains normal and abnormal tissue
226
Neurosurgery Books Full
www.ketabpezeshki.com 66485438-66485457
15 RADIOTHERAPY AND STEREOTACTIC RADIOSURGERY 227

8. Which one of the following statements c. Radiotherapy is associated with reduced


regarding radiotherapy for low-grade glioma facial nerve and trigeminal nerve toxicity
is most accurate? compared to microsurgery.
a. Early postoperative radiotherapy increa- d. Stereotactic radiosurgery is more effec-
ses survival tive than fractionated stereotactic radio-
b. Early postoperative radiotherapy is asso- therapy.
ciated with better seizure control e. Stereotactic radiosurgery is preferred for
c. Early postoperative radiotherapy is asso- lesions bigger than 3 cm.
ciated with reduced side-effects
d. Early postoperative radiotherapy results 13. What is the risk of secondary neoplasm after
in increased malignant transformation of SRS at 15 years?
residual tumor a. 0.004%
e. Early postoperative radiotherapy shows no b. 0.04%
advantage in time to tumor progression c. 0.4%
d. 4.0%
9. What is the target for stereotactic radiosur- e. 4.4%
gery treatment for trigeminal neuralgia? f. 44%
a. Cisternal portion of the trigeminal nerve
adjacent to the brainstem 14. Which one of the following is most accurate
b. Sensory trigeminal nucleus in pons about stereotactic radiosurgery for pituitary
c. Superficial cerebellar artery adenomas?
d. Trigeminal ganglion in Meckel's cave a. At doses above 5 Gy, the rate of impaired
e. Trigeminal nerve in foramen ovale thyrotropic function at 5 years was 50%.
b. Contraindicated for tumors within 5 mm
10. Which one of the following scenarios of the optic chiasm.
describes the most appropriate action taken? c. For secretory tumors there is no significant
a. 1 cm Spetzler-Martin grade 3 AVM in the difference in response by tumor subtype.
left thalamus treated with embolization d. Improved rates of tumor control are seen
followed by stereotactic radiosurgery when radiation therapy is combined with
b. 1.5 cm Spetzler-Martin grade 2 AVM in left dopamine agonists.
parietal cortex treated with embolization e. Radiation-induced hypopituitarism occurs
c. 1.5 cm Spetzler-Martin grade 3 AVM in in 10% of patients undergoing SRS.
right thalamus treated with stereotactic
radiosurgery 15. Which one of the following is most
d. 4 cm Spetzler-Martin grade 3 AVM in accurate about radiation treatment for
right frontal lobe treated with stereotactic craniopharyngiomas?
radiosurgery a. Bleomycin can be used for intracystic
e. 5 cm Spetzler-Martin grade 3 AVM in left irradiation.
temporal lobe treated with surgical excision b. Intracystic interferon alpha is utilized to
sensitize the cystic component to SRS.
11. Which one of the following has not been c. Fractionated conformal radiotherapy is
shown to increase the risk of developing commonly used to treat residual tumour.
edema after stereotactic radiosurgery for the d. SRS with intracystic irradiation is effec-
treatment of meningiomas is most accurate? tive for mixed-type tumors.
a. Brain-tumor interface >1 cm2 e. There is no significant role for radiother-
b. Clinical treatment volume of >5 cm3 apy in management.
c. Location in cavernous sinus
d. Presence of edema on pretreatment scan
e. Radiation dose greater than 16 Gy QUESTIONS 16–17
Additional questions 16–17 available on
12. Which one of the following is true of SRS for ExpertConsult.com
acoustic neuroma?
a. Microsurgery is associated with better
preservation of serviceable hearing than
radiotherapy.
b. Microsurgery is preferred for lesions
smaller than 3 cm.

Neurosurgery Books Full


www.ketabpezeshki.com 66485438-66485457
228 PART II CARE OF THE NEUROSURGICAL PATIENT

EXTENDED MATCHING ITEM (EMI) For each of the following descriptions, select the
most appropriate answers from the list above.
QUESTIONS Each answer may be used once, more than once
or not at all.
18. Tissue tolerance to radiosurgery: 1. Treatment targeted at anterior internal
a. Brain lesion 0-2 cm capsule bilaterally
b. Brain lesion 2-3 cm 2. Latency period of 1-3 years before maximal
c. Brain lesion 3-4 cm treatment effect seen
d. Brainstem 3. Target is cisternal segment of the
e. Cochlea trigeminal nerve
f. Cranial nerves III, IV, VI
g. Facial nerve 20. Radiotherapy and radiosurgery:
h. Optic chiasm/nerve a. Carbon-ion therapy
i. Pituitary b. Conformal radiotherapy
j. Trigeminal nerve c. Fast-neutron therapy
d. Gamma Knife surgery
For each of the following descriptions, select the e. Fractionated stereotactic radiotherapy
most appropriate answers from the list above. f. Intensity-modulated
Each answer may be used once, more than once radiotherapy (IMRT)
or not at all. g. Linac radiosurgery
1. Has a single fraction maximal tolerated h. Proton beam therapy
dose of 3.7 Gy i. TomoTherapy
2. Has a single fraction maximal tolerated j. Volumetric modulated arc radiotherapy
dose of 8 Gy k. Whole brain radiotherapy (WBRT)
19. Indications in stereotactic radiosurgery: For each of the following descriptions, select the
a. Arteriovenous malformation most appropriate answers from the list above.
b. Brain metastasis Each answer may be used once, more than once
c. Chordoma or not at all.
d. Cluster headache 1. Non-conformal photon based therapy with
e. Epilepsy opposed lateral fixed beams
f. Glioblastoma multiforme 2. Shapes the radiotherapy beams to allow dif-
g. Glomus jugulare tumors ferent doses of radiotherapy to be given to
h. Low-grade glioma different parts of the treatment area and
i. Meningioma more effectively spare organs at risk
j. Obsessive compulsive disorder 3. Use multileaf collimator in a helmet
k. Pituitary adenoma attached to a stereotactic head frame allow-
l. Vestibular schwannoma ing multiple small beams to deliver high
dose to small target small deep lesions

SBA ANSWERS
1. a—A high dose of radiation is delivered in a is minimized while targets less than 4 cm large
single sitting only to a single target receive a high dose. Effective radiosurgical treat-
Radiosurgery usually implies a single outpatient ment of targets larger than 4 cm with would
treatment with high dose delivered to a small tar- require an unacceptable increase in dose to adja-
get, with multiple beams creating a high dose gra- cent normal brain tissue. It can be performed
dient. As no single beam contributes significantly using various devices including linear accelera-
to the cumulative dose the amount of radiation tors, Gamma Knife (GK), and particle beam
delivered to normal tissues in the beams' paths devices.

Neurosurgery Books Full


www.ketabpezeshki.com 66485438-66485457
15 RADIOTHERAPY AND STEREOTACTIC RADIOSURGERY 229

2. c—Gamma Knife surgery

Proton Beam
LINAC (e.g., CyberKnife) Gamma Knife Therapy
Source Linear accelerator shoots electrons at tungsten Cobalt-60 decay Cyclotron
target
Rays X-rays Gamma rays Proton
Head Stereotactic frame or frameless (fiducials) Stereotactic frame Immobilization
immobilization
Machine moves around patient during Equipment Equipment
treatment stationary stationary
Multileaf In machine In patient helmet
collimator
Use Whole body Intracranial only Whole body

3. d—Planning target volume (PTV)

Gross tumor volume Volume of macroscopic tumor that is visualized on imaging studies
(GTV)
Clinical target Volume that should be treated to a high dose, typically incorporating both the
volume (CTV) GTV and volumes that are assumed to be at risk due to microscopic spread of
the disease
Planning target Volume that should be treated in order to ensure that the CTV is always treated,
volume (PTV) including considerations of systematic and random daily setup errors and
intertreatment and intratreatment motion
Organ at risk (OR) Organ whose damage is especially dangerous and where small amount of
radiation damage would produce a severe clinical manifestation, e.g., spinal
cord. Their radiation sensitivity influences treatment planning or prescribed
radiation dose
Planning organ at risk Margin added around the OR to account for uncertainties in planning and
volume (PRV) delivery
Systematic target Margin added to the CTV to account for systematic errors arising from treatment
volume (STV) planning

4. a—Target volume contains no abnormal tissue and normal tissue shows early radiobiologic effect

Target volume late responding as contains normal and abnormal AVM


tissue
Target volume late responding but only in abnormal tissue and with Benign tumor, e.g., meningioma,
marked radiobiologic effect pituitary adenoma, vestibular
schwannoma
Target volume shows small, early effect on abnormal tissue but Low-grade glioma
bigger, late effect on normal tissue (radiosurgery only indicated in
some cases)
Target volume contains no normal tissue and abnormal tissue shows Metastasis
early radiobiologic effect

FURTHER READING
Shrieve DC, et al. Radiosurgery. In: Leibel & Philips Textbook of Radiation Oncology, 3rd ed.

Neurosurgery Books Full


www.ketabpezeshki.com 66485438-66485457
230 PART II CARE OF THE NEUROSURGICAL PATIENT

5. e—Proton beam therapy


Relevance to Fractionation
Term Efficacy
As a photon beam passes through material and is
absorbed, the overall intensity of the beam is Normal tissue: at 4 weeks post-
radiotherapy in early responding
reduced. In contrast, particles such as protons and tissue
ions travel a finite distance, which is termed the
range. They deposit a disproportionate amount of Malignant tissue: come tumors
show accelerated repopulation
energy in the last few millimeters of their path. This at 4-5 weeks—dangerous
large transfer of energy is known as the Bragg peak. phenomenon where tumor
The physical depth penetrated by the particles shows marked increase in
depends on tissue density and the beam's energy. growth fraction and doubling
time

120 Increasing the length of the


Bragg’s peak
course to over 4 weeks will thus
100 reduce normal tissue reactions,
but increase the risk of
Relative dose

80 accelerated repopulation in the


tumor requiring additional
60 fractions
40 Reassortment/ When radiotherapy is given
redistribution to a population of cells, they
20
may be in different parts of
0 the cell cycle. Cells in S-phase
are typically radioresistant,
0 5 10 15 20
whereas those in late G2 and
Depth in water (cm) M phase are relatively
10 MV x-rays
radiosensitive. A small dose
18 MeV electrons
of radiation delivered over a
150 MeV protons short time period will kill a lot
of the sensitive cells and less
of the resistant cells. Over
Image with permission from Winn HR. Youman's time, the surviving cells will
Neurological Surgery, 4-Volume Set, 6th ed. Elsevier, redistribute the proportion
Saunders, 2011. in each cell cycle phase.
Surviving cells that now moved
into a more radiosensitive
6. c—Reduction phase can now be killed
with a second fraction of
radiotherapy
The five Rs of radiobiology are:
Reoxygenation Radiotherapy works through the
production of free radicals (by
interaction of photons with water
Relevance to Fractionation
or oxygen) which cause DNA-
Term Efficacy
damage to normal and tumor
Repair Ionizing radiation can cause cells. Normal cells are better able
lethal, sublethal and potentially to repair this damage, hence
lethal damage to cells. If survive better than tumor cells.
radiotherapy is fractionated, Tumors can become acutely or
sublethal damage can be chronically hypoxic due to
repaired before the next dose— changes in blood supply—
and this repair occurs more reducing the number of free
effectively in normal tissue radicals produced during a
compared to malignant cells fraction of radiotherapy and
(e.g., TP53 mutation). If multiple increasing their radioresistance/
fractions are to be given on the reducing radiosensitivity.
same day, the repair half-life Fractionating radiotherapy
of the normal tissue must be reduces the chances that the cell
considered, e.g., 4 h repair will be acutely hypoxic during
half-life in spinal cord therefore treatments, and gives multiple
separate fractions by 8 h opportunities for normoxic tumor
cells near capillaries to be killed
Repopulation The increase in cell division that
is seen in surviving normal and Radiosensitivity Refers to the intrinsic
malignant tissue post- radiosensitivity/radioresistance
radiotherapy—determines of certain normal and tumor cell
length and timing of course types:

Continued Continued

Neurosurgery Books Full


www.ketabpezeshki.com 66485438-66485457
15 RADIOTHERAPY AND STEREOTACTIC RADIOSURGERY 231

• Large (>4 cm) brain metastasis not amena-


Relevance to Fractionation
Term Efficacy
ble to SRS
• Postsurgical resection of a dominant hemi-
Radiosensitive: hematological sphere brain metastasis with multiple (4-10)
cells, epithelial stem cells,
gametes; hematological and remaining BMs
germ cell malignancy • Salvage therapy for recurrent BM after SRS
or WBRT failure
Radioresistant: myocytes,
neurons; melanoma, sarcoma
FURTHER READING
Lin X, DeAngelis LM. Treatment of brain metastases. J Clin
Oncol 2015. pii:JCO.2015.60.9503.

Linskey ME, Andrews DW, Asher AL, et al. The role of ste-
reotactic radiosurgery in the management of patients with
7. a—CNS and systemic progression of disease, newly diagnosed brain metastases: a systematic review and
with few systemic treatment options and poor evidence-based clinical practice guideline. J Neurooncol
performance status 2010;96(1):45-68.

Both surgery and SRS have a proven survival ben-


efit in the management of a single brain metasta- 8. b—Early postoperative radiotherapy is asso-
sis. Typically, surgery is preferred in patients with ciated with better seizure control
good performance status, large lesions (>3 cm),
In a single prospective RCT (n ¼ 311) People with
or symptomatic tumors with substantial vaso-
LGG who undergo early postoperative radiother-
genic edema. In patients who are good candidates
apy showed an increase in time to progression
for either surgery or SRS, there are no random-
compared with people who were observed and
ized data currently available to indicate which is
had radiotherapy at the time of progression (mean
the preferred treatment modality. In general,
5.3 vs. 3.4 years). There was no significant differ-
the current debate regarding patients with multi-
ence in overall survival between people who had
ple metastases surrounds whether to use WBRT,
early versus delayed radiotherapy; however, this
SRS, or both. Survival, recurrence, focal neuro-
finding may be due to the effectiveness of rescue
logical deficit and neurocognitive outcome are
radiotherapy in the delayed arm (required in
key considerations. Proponents of SRS suggest
65% of this group). People who underwent early
that highly targeted therapy spares normal brain
radiation had better seizure control at 1 year than
tissue and preserves neurocognitive function,
people who underwent delayed radiation. Early
while WBRT supporters argue that SRS will
radiation therapy was associated with skin reac-
not treat the invisible micrometastatic foci which
tions, otitis media, mild headache, nausea, and
will grow to cause neurological deterioration later
vomiting. There were no cases of radiation-
on. Current practices may be summarized as:
induced malignant transformation of LGG. How-
Consider SRS when:
ever, it remains unclear whether there are differ-
• Oligometastases (1-3) or multiple (4-10)
ences in memory, executive function, cognitive
metastases especially if primary tumor is
function, or quality of life between the two groups
known to be radiotherapy resistant
since these measures were not evaluated.
• Postsurgical resection of a single BM, espe-
cially if 3 cm or bigger and in the
posterior fossa FURTHER READING
• Local relapse after surgical resection of a Sarmiento JM, Venteicher AS, Patil CG. Early versus delayed
single brain metastasis postoperative radiotherapy for treatment of low-grade gliomas.
• Salvage therapy for recurrent oligometas- Cochrane Database Syst Rev 2015;(6). Art. No.: CD009229.
tases (1-3) after WBRT
Consider WBRT in brain metastasis when: 9. a—Cisternal segment of trigeminal nerve
• CNS and systemic progression of disease,
with few systemic treatment options and For patients refractory to medications SRS is the
poor performance status least invasive procedure, though microvascular
• Multiple (4-10) brain metastasis especially if decompression remains superior in candidates fit
primary tumor known to be radiotherapy for surgery. Typical doses are 70-90 Gy in a single
sensitive (NB current data support SRS fraction directed at the dorsal root entry zone of
use in up to 3 metastasis but there is a grow- cranial nerve V near the pons. Initial SRS direc-
ing trend to use it in up to 10) ted at the gasserian ganglion produced inferior
results. Pain relief is experienced with a latency

Neurosurgery Books Full


www.ketabpezeshki.com 66485438-66485457
232 PART II CARE OF THE NEUROSURGICAL PATIENT

of approximately 1 month. Paresthesia is the most 11. c—Location in cavernous sinus


common side effect. Some pain relief (partial or
complete) is seen in 60-70% of patients treated. Radiosurgery plays an important role in the treat-
ment of small lesions (<3 to 4 cm) that are surgi-
cally inaccessible, such as those in cavernous sinus
FURTHER READING or posterior parasagittal locations, or those that
McHaffie DR, et al. Stereotactic irradiation. In: Gunderson have been subtotally resected but consistently
Clinical Radiation Oncology, 3rd ed. have been shown to result in high recurrence rates.
Two of the largest series that have examined
10. c—1.5 cm Spetzler-Martin grade 3 AVM in results of SRS are from the Mayo Clinic and the
right thalamus treated with stereotactic University of Pittsburgh, both of which have
radiosurgery shown local control in more than 90% for benign
meningiomas at 5 years; doses 12-16 Gy, due to
Arteriovenous malformations (AVMs) harbor a increased risk of edema above this dose.
risk of hemorrhage of about 2-4% per year, mor- The most common toxicities include cranial
tality of 10-15% and morbidity of 50% and given nerve deficits for basal tumors and peritumoral
the cumulative lifetime risk treatment is often edema for non-basal tumors. The risk of optic
considered in an asymptomatic patient. Treat- neuropathy is very low, with maximum dose con-
ment options include observation, embolization, straints to the optic nerves and chiasm of between
surgery, or stereotactic radiosurgery. Surgery is 8 and 10 Gy. Risk of peritumoral edema is
the treatment of choice, when feasible, as it increased by: high dose, a treatment volume of
immediately removes the risk of hemorrhage more than 5 cm3, a brain-tumor interface of
(compared to the persisting risk of hemorrhage >1 cm, the presence of pretreatment edema,
during the latency period between SRS and even- and parasagittal location. These factors should
tual AVM obliteration). As such, SRS is an be considered when deciding whether or not to
approved treatment option for intracranial AVMs include none of the dural tail or only a portion
that are not treatable via microsurgery. For low- of it within the clinical target volume. Small
grade deep AVMs smaller than 3 cm, SRS alone meningiomas can be controlled with radiosurgery
can be performed when microsurgery is not pos- in the majority of patients, with initial results
sible. Smaller AVMs allow large doses of radia- comparable to those of complete resection.
tion to be applied safely and thus have a higher
obliteration rate. The high dose of radiation pre-
FURTHER READING
sumably unleashes a cytokine cascade that
McHaffie DR, et al. Stereotactic irradiation. In: Gunderson
induces fibrointimal reaction, thrombosis, and
Clinical Radiation Oncology, 3rd ed.
eventual obliteration of the AVM nidus over
1-3 years (latency period). Given the concomitant
12. c—Radiotherapy is associated with reduced
increased risks of adverse radiation effects, high
facial nerve and trigeminal nerve toxicity
doses of radiation cannot be safely used in large
compared to microsurgery
AVMs, thereby resulting in a worse obliteration
rate. Thus, a multimodal approach consisting of Vestibular schwannomas represent 6-8% of pri-
a combination of embolization and SRS has been mary intracranial tumors, arising at the point
widely used. Embolization may reduce the size of where nerve sheaths are replaced by fibroblasts
larger AVMs, making them more amenable to (Obersteiner-Redlich zone; usually in IAC).
radiosurgery. In addition, intranidal aneurysms Though benign, the lesions can cause severe
and arteriovenous fistulas associated with AVMs local symptoms. Typical growth rates are less
not only have a high risk of hemorrhage but are than 2 mm per year. Common symptoms
also less sensitive to radiosurgery and can be trea- include unilateral sensorineural hearing loss
ted using embolization followed by radiosurgery. (>90%), tinnitus, unsteady gait, facial numbness
or weakness, mastoid pain, and headaches. Late
FURTHER READING presentations can include brain stem compres-
Xu F, Zhong J, Ray A, Manjila S, Bambakidis NC. Stereotactic sion. The typical appearance on T1-weighted
radiosurgery with and without embolization for intracranial MRI with contrast shows a homogenously
arteriovenous malformations: a systematic review and meta- enhancing mass within the cerebellopontine
analysis. Neurosurg Focus 2014;37(3):E16. angle with widening of the internal auditory
canal. Treatment options include observation,
microsurgical resection, SRS, or fractionated
radiotherapy; recommendations are influenced
by age, comorbidities, tumor size, presenting

Neurosurgery Books Full


www.ketabpezeshki.com 66485438-66485457
15 RADIOTHERAPY AND STEREOTACTIC RADIOSURGERY 233

symptoms, hearing loss/presence of serviceable control using SRS is generally in excess of 90%
hearing, proximity to the brainstem or cochlea, for non-secretory tumors. Radiation-induced
and patient preference. The goals of treatment hypopituitarism occurs in more than 50% of
include maximizing tumor control while pre- patients and is the most common late toxicity. A
serving hearing and facial nerve and trigeminal mean pituitary dose of 15 Gy was found to pose
nerve function. All interventions (surgery, little risk of subsequent thyrotropic, gonado-
SRS, FSRT) appear to result in a tumor control tropic, or adrenocorticotropic function; but at
probability of more than 90%, and utility is sum- 5 years half of patients had low gonadotropic
marized below: and thyrotropic function at doses above 17 Gy
• Microsurgery is preferred for lesions larger and low adrenocorticotrophic function at doses
than 2.5 cm. The risk of cranial nerve injury above 20 Gy. Dose to the pituitary stalk and hypo-
with microsurgical resection is highly thalamus may also contribute to hypopituitarism
dependent on tumor size, operative following SRS. For secretory tumors, SRS
approach (retrosigmoid, middle cranial appears to result in a shorter time to hormone
fossa, or translabyrinthine), and the sur- normalization than fractionated radiotherapy.
geon's skill and experience. Though tumor control remains high, hormonal
• SRS is an option for lesions smaller than remission is seen in 25-75% of patients and
2.5 cm. The risk of sensorineural hearing depends on the tumor subtype (i.e., prolactinoma,
loss is related to the dose of radiation deliv- Cushing's disease, Nelson's syndrome, or acro-
ered to cranial nerve VIII, the cochlea, and megaly). Cytostatic medical management of
the ventral cochlear nucleus. Compared to secreting pituitary adenomas is often employed,
microsurgery for similar tumors, SRS but should be discontinued pre-radiotherapy if
shows better serviceable hearing preserva- symptoms allow because patients receiving
tion (50-89%) and reduced facial nerve octreotide or dopamine agonists show markedly
and trigeminal nerve toxicity (<5%). inferior control rates in several series.
• Fractionated stereotactic radiotherapy
(FSRT) has comparable efficacy with FURTHER READING
SRS; some series have reported poorer McHaffie DR, et al. Stereotactic irradiation. In: Gunderson.
hearing preservation and increased trigem- Clinical Radiation Oncology, 3rd ed.
inal nerve injury with SRS.
15. d—SRS with intracystic irradiation is effec-
tive for mixed-type tumors.
FURTHER READING
McHaffie DR, et al. Stereotactic irradiation. In: Gunderson.
Maximal safe resection is the mainstay of treatment
Clinical Radiation Oncology, 3rd ed. for craniopharyngiomas and leaving residual
tumour is often necessary to preserve vision or
13. b—0.04% hypothalamic function. Radiotherapy (RT) has
emerged as a valuable adjuvant treatment modality
FURTHER READING
for recurrent or residual craniopharyngiomas. Sev-
Patel TR, Chiang VL. Secondary neoplasms after stereotactic
eral radiotherapeutic modalities, including confor-
radiosurgery. World Neurosurg 2014;81(3-4):594-9.
mal radiotherapy, single-fraction stereotactic
radiosurgery, fractionated stereotactic radiother-
14. b—Contraindicated for tumors within 5 mm
apy, and proton beam therapy offer reasonable rates
of the optic chiasm
of tumor control. With advances in neuroimaging
and RT modalities, dose delivery is more accurate
Pituitary adenomas represent 10-15% of intracra-
and focused, resulting in decreased long-term com-
nial neoplasms. Irradiation is generally reserved
plication rates over time (hypopituitarism, visual
for patients who have incompletely resected
deterioration, cranial nerve deficit, radiation
tumors or recurrent disease. Based on the size
effects). The cystic component of a craniopharyn-
and location, either SRS or conformal EBRT
gioma commonly presents a problem for radiation
may be considered. SRS is typically contraindi-
therapy and radiosurgery. Tumor growth and cyst
cated for tumors within 3-5 mm of the optic chi-
enlargement can be independent: the solid compo-
asm, respecting a maximum dose constant of
10 Gy or less to the optic nerves and chiasm. Frac- nent of the tumor can usually be controlled by radi-
ation while the cystic component may require
tionated radiotherapy, used successfully for more
treatment with one of the following options:
than 50 years to treat pituitary adenomas, should
• Stereotactic aspiration (e.g., acute presenta-
be recommended for tumors abutting optic nerve
tion or poor surgical candidate)
or chiasm and for diffuse or large tumors. Local

Neurosurgery Books Full


www.ketabpezeshki.com 66485438-66485457
234 PART II CARE OF THE NEUROSURGICAL PATIENT

• Placement of an Ommaya reservoir allow- EMI ANSWERS


ing intermittent aspiration of a cyst that
cannot be completely resected 18. 1—e, Cochlea; 2—h, Optic chiasm and nerves
• Sclerosis of the cyst wall by chemothera-
peutic drugs for treatment-resistant cysts With increasing international experience with
(e.g., bleomycin, interferon alpha) SRS, guidelines for reducing the risk of normal-
• Internal irradiation (i.e., brachytherapy) tissue toxicity have emerged. In general, normal
with implanted radioisotopes for treatment- tissues at risk depend on the location of the target
resistant cysts (Phosphorus-32) volume. These include the brain parenchyma
Although the beneficial effect of radiation in the (edema, necrosis), brainstem (edema, necrosis,
treatment of craniopharyngiomas has been well neuropathy), cranial nerves (neuropathy), and
recognized, several issues remain sources of sig- hypothalamic-pituitary axis (hypopituitarism).
nificant controversy: The interaction of dose and volume irradiated has
1. On the basis of the complications associated not been clearly defined for most structures at risk.
with aggressive resection and the proven
efficacy of radiation for craniopharyngio-
mas, several authors have recommended Structure Single-Fraction Dose Constraint
subtotal resection and RT as an acceptable
Brain lesion 24 Gy (less than 20% risk of
alternative to gross-total resection. Further <2 cm serious complication)
support for this approach comes with
increasing recognition that while neurolog- Brain lesion 18 Gy (less than 20% risk of
2-3 cm serious complication)
ical deficits and endocrine dysfunction due
to radical resection can be managed, the Brain lesion 15 Gy (less than 20% risk of
associated psychosocial consequences for 3-4 cm serious complication)
pediatric patients growing into adults sig- Brainstem 16 Gy—less than 5% cranial
nificantly affect quality of life. nerve deficit if less than 1/3
2. The role of RT immediately after resection brainstem gets 16 Gy
without first monitoring for tumor pro- Pituitary 15 Gy—no risk of
gression (up-front vs. salvage treatment) is hypopituitarism below this level
debated with early radiotherapy showing Optic chiasm/ 8 Gy—no risk of visual loss
some evidence of lower rates of morbidity nerve below this level
and improved tumor control in children
Cranial nerves 30 Gy
but not adults. III, IV, VI
3. SRS as a primary treatment has shown
higher tumor control rates in single-type Trigeminal nerve <12.5-13 Gy
tumors (solid or cystic) compared to mixed Facial nerve <12.5-15 Gy
solid-cystic tumors. Solid-type tumors and Cochlea <3.7 Gy
the solid portions of mixed tumors may
be less responsive to brachytherapy than Spinal cord 50 Gy
cystic tumors, hence a combination of
radioisotope instillation and SRS has been
19. 1—j, Obsessive compulsive disorder; 2—a,
suggested as primary treatment for mixed
AVM; 3—d, Cluster headache (as well as tri-
solid-cystic tumors.
geminal neuralgia)
FURTHER READING
Lee CC, Yang HC, Chen CJ, Hung YC, et al. Gamma Knife
20. 1—k, Whole brain radiotherapy; 2—f,
surgery for craniopharyngioma: report on a 20-year experi-
Intensity-modulated radiotherapy; 3—d,
ence. J Neurosurg 2014;121(Suppl.):167-78.
Gamma Knife surgery

Radiotherapy can be given externally (external


beam radiotherapy) or internally (brachytherapy
ANSWERS 16–17 and radionuclide therapy). External beam radio-
therapy can be further divided into photon based
Additional answers 16–17 available on (i.e., X-ray) and particle based therapies. The
ExpertConsult.com table below describes some terms that usually
describe subtle differences in planning or delivery
of radiotherapy.

Neurosurgery Books Full


www.ketabpezeshki.com 66485438-66485457
15 RADIOTHERAPY AND STEREOTACTIC RADIOSURGERY 235

Type Notes
Radiotherapy
Conformal radiotherapy (3DCRT) This uses a device inside the radiotherapy machine to shape the
radiotherapy beams to the target in three dimensions (height, width,
and depth). The desired cross sectional shape of the beam can be
formed using blocks or a multileaf collimator. Beams can be fixed or
intensity-modulated
Whole brain radiotherapy (WBRT) Non-conformal, two dimensional radiotherapy (opposed lateral fixed
beams)
Intensity-modulated radiotherapy Shapes the radiotherapy beams to allow different doses of
(IMRT) radiotherapy to be given to different parts of the treatment area. This
means lower doses of radiotherapy can be given to normal tissue,
hence often used close to organs at risk
TomoTherapy® Hybrid between CT scanner and IMRT—the radiation source for both
radiotherapy and CT imaging can move completely around the patient
in a helical arc. CT scans performed immediately before treatment.
Highly conformal and precise, conformal avoidance of normal tissue
but slower than VMAT. No comparison studies available currently
Volumetric modulated arc radiotherapy Type of IMRT using rotational (arc) delivery. The angle of the beam, the
(VMAT, e.g., RapidArc®) dose rate and the leaf speed are all independently controlled, making
this a very accurate form of treatment. Arc therapy treatments also take
much less time to deliver than other radiotherapy techniques
Image guided radiotherapy (IGRT) Refers to any mode of radiotherapy where imaging of the tumor is
performed during treatment to ensure treatment precision. Could be
between several fractions, immediately prior to each dose (e.g.,
tomotherapy), or in real-time (e.g., CyberKnife radiosurgery)
Photon (X-ray) radiosurgery
Gamma Knife® Use multileaf collimator in a helmet attached to a stereotactic head
frame allowing multiple small beams to deliver high dose to small
target small deep lesions
Frame-based Linac Uses stereotactic head frame to and multileaf collimator in the linac
radiosurgery machine (with a moving arm)
Frameless Linac (CyberKnife® Robotic Uses a moving couch and a small linear accelerator on a robotic arm to
Radiosurgery) deliver multiple beams of radiation from different angles. It works best
on small tumors with well-defined edges. Due to real-time image-
guidance it can also adjust the delivery, for example to match the
patient's breathing motion
Fractionated stereotactic radiotherapy Combines the similar dose conformality, precise dose delivery, and
steep dose falloff outside the target volume of stereotactic radiosurgery
with the radiobiologic advantages of dose fractionation. Fractionation
safely treats larger tumor volumes intimate to critical structures such as
the optic apparatus
Particle (Hadron) radiosurgery
Proton beam therapy Protons deliver a dose of radiation in a much more confined way to the
tumor tissue than photons (X-rays, gamma rays). After they enter the
body, protons release most of their energy within the tumor region and,
unlike photons, deliver only a minimal dose beyond the tumor
boundaries. Therefore, especially for smaller tumor sizes, the dose of
radiation may conform much tighter to the tumor and there may be less
damage to healthy tissue. In particular its use in young children has
been increasing, especially where conventional radiotherapy may be
contraindicated due to its effects on CNS development
Fast-neutron therapy Theoretical advantage over photons in low-oxygen (hypoxic)
conditions, but interest in the use of neutron therapy has waned
because it has shown no advantages in terms of outcome over
irradiation with other types of particles
Carbon-ion therapy Combines the dose-distribution advantages of protons with an
increase in biologic effectiveness toward the end of the particle range
Boron-neutron capture therapy Clinical studies undergoing in recurrent malignant gliomas

Neurosurgery Books Full


www.ketabpezeshki.com 66485438-66485457
CHAPTER 16

NEUROPSYCHOLOGY AND
NEUROLOGICAL REHABILITATION
SINGLE BEST ANSWER (SBA) QUESTIONS
1. A 63-year-old male presents as a World Feder- 5. Which one of the following domains of cogni-
ation of Neurosurgical Societies (WFNS) tive impairment in normal pressure hydro-
grade III subarachnoid hemorrhage and cephalus is LEAST likely to improve with
undergoes coiling of a basilar tip aneurysm. shunt insertion?
After a prolonged Intensive Care Unit (ICU) a. Delayed verbal recall
stay, he is ready to be discharged from the ward. b. Frontal lobe executive function
He is able to walk with assistance and needs c. Psychomotor speed
help with toileting and showering. What are d. Visual memory
his modified Rankin and Glasgow Outcome e. Visuoconstructional abilities
Scale scores respectively?
a. mRS 2 and GOS 2 6. Which one of the following statements regard-
b. mRS 2 and GOS 3 ing cognitive decline following cranial irradia-
c. mRS 3 and GOS 2 tion for brain metastasis is most accurate?
d. mRS 3 and GOS 3 a. There is no difference in cognitive decline
e. mRS 4 and GOS 3 at 12 months post Whole Brain Radio-
f. mRS 4 and GOS 4 therapy (WBRT) compared to controls
b. There is a greater cognitive decline at
2. Which one of the following Karnofsky 12 months in patients with Stereotactic
performance scores is commonly used as a Radiosurgery (SRS) + WBRT compared
cutoff for functional independence in to SRS alone
neuro-oncology? c. There is a greater cognitive decline at
a. 40 12 months in SRS alone compared to
b. 50 WBRT alone
c. 60 d. There is no difference in cognitive decline
d. 70 at 12 months in patients with SRS
e. 80 + WBRT compared to SRS alone
e. There is no increase in cognitive decline
3. Cerebellar mutism occurs most commonly at 12 months in those receiving 36 Gy of
after resection of which one of the following irradiation compared to 25 Gy
posterior fossa tumors in children?
a. Ependymoma 7. Which one of the following statements
b. Hemangioblastoma regarding cognitive outcome in aneurysmal
c. Medulloblastoma subarachnoid hemorrhage in those patients
d. Meningioma treated with clipping versus those treated
e. Pilocytic astrocytoma with coiling is most accurate?
a. Cognitive outcomes are poorer in the
4. Cerebellar cognitive affective syndrome does coiling group overall
not generally involve which one of the b. Cognitive outcomes are poorer in the
following? clipping group overall
a. Dysprosodia c. Coiling may offer a superior cognitive
b. Impaired executive function outcome in the short term and clipping the
c. Mutism superior cognitive outcome in the long term
d. Personality change d. Cognitive outcomes are poorer in the
e. Visuospatial impairment clipping group for anterior circulation
aneurysms
236
Neurosurgery Books Full
www.ketabpezeshki.com 66485438-66485457
16 NEUROPSYCHOLOGY AND NEUROLOGICAL REHABILITATION 237

e. Coiling may offer an inferior cognitive c. Management in local hospitals by general


outcome in the short term and clipping medical teams to maximize family support
the inferior cognitive outcome in the in the acute stage
long term d. Outpatient management
e. Task-oriented therapy
8. Which one of the following statements
regarding return to work in aneurysmal sub- 11. Neurorehabiliation in the context of spinal
arachnoid hemorrhage is most accurate? cord injury has all of the following goals
a. Return to previous occupation is approx- EXCEPT:
imately 30% in aneurysmal subarachnoid a. Ensure that required adaptations and
hemorrhage equipment are identified and provided
b. Return to previous occupation is approx- b. Management of excretion from bowels
imately 40% in aneurysmal subarachnoid and bladder
hemorrhage c. Minimize the risk of preventable compli-
c. Return to previous occupation is approx- cations (e.g. pressure sores)
imately 50% in aneurysmal subarachnoid d. Recognition and management of auto-
hemorrhage nomic dysreflexia
d. Return to previous occupation is approx- e. Routine recruitment into neural stem cell
imately 60% in aneurysmal subarachnoid transplantation trials for spinal cord injury
hemorrhage
e. Return to previous occupation is approx-
imately 70% in aneurysmal subarachnoid EXTENDED MATCHING ITEM (EMI)
hemorrhage QUESTIONS
9. Which one of the following is currently seen 12. Neuropsychological tests:
as the best marker of severity of traumatic a. Affect and personality
brain injury in survivors? b. Attention
a. Glasgow coma score c. Effort (embedded and free-standing)
b. Length of coma d. Executive functions
c. Mechanism of injury e. Language
d. Initial CT scan f. Memory
e. Post-traumatic amnesia g. Motor processing
h. Visuospatial and visuomotor processing
10. Neurorehabiliation in the context of
stroke is characterized by which one of the For each of the following descriptions, select the
following? most appropriate answers from the list above.
a. Baclofen for dyskinesias resulting from Each answer may be used once, more than once
basal ganglia strokes or not at all.
b. Botulinum toxin injections to facilitate 1. Wisconsin card sorting test
physiotherapy to the paretic limb (s) 2. Beck depression inventory
3. Finger oscillation test

SBA ANSWERS
1. e—mRS 4 and GOS 3

mRS Description
0 No symptoms at all
1 No significant disability despite symptoms; able to carry out all usual duties and activities
2 Slight disability; unable to carry out all previous activities, but able to look after own affairs without
consequence
3 Moderate disability; requiring some help but able to walk without assistance
4 Moderately severe disability; unable to walk without assistance and unable to attend to own bodily needs
without assistance
5 Severe disability; bedridden, incontinent, and requiring constant nursing care and attention
6 Death

Neurosurgery Books Full


www.ketabpezeshki.com 66485438-66485457
238 PART II CARE OF THE NEUROSURGICAL PATIENT

more recent studies have demonstrated that per-


GOS Term Definition
sistent impairment of motor speech is common
1 Dead No life and complete recovery of speech and language
2 Vegetative state Unaware of self and is infrequent. The underlying neuroanatomical
environment locus may be the dentatothalamocortical outflow
3 Severe Unable to live
tracts from the cerebellar nuclei through the
disability independently brainstem.
4 Moderate Able to live independently
disability FURTHER READING
Charalambides C, Dinopoulos A, Sgouros S. Neuropsycho-
5 Mild disability Able to return to work/
logical sequelae and quality of life following treatment of
school
posterior fossa ependymomas in children. Childs Nerv Syst
2009;25(10):1313-20.

2. d—70 4. c—Mutism

The cerebellum is divided into three parts, based


KPS Description on the arrangement of the afferent fiber projec-
tion: the vestibulocerebellum (equilibrium and
100 Normal
eye movements), the spinocerebellum (posture,
90 Capable of normal activity, few symptoms muscle tone, and execution of limb movements),
80 Normal activity with some difficulty, some and the pontocerebellum (coordination of skilled
symptoms movements initiated at a cerebral cortical level).
70 Cares for self, cannot work, no normal activity
More recently there has been growing apprecia-
tion that cerebellar damage can produce cognitive
60 Requires some help, can do most of personal deficits. In patients with right cerebellar injury
care
linguistic processing was impaired, while left cer-
50 Requires help often and frequent medical ebellar injury produced visual-spatial defects were
support noted. Neurpsychological studies have identified
40 Disabled, requires special care and help a cerebellar cognitive affective syndrome, pre-
dominantly in adults:
30 Severely disabled, may need hospital but no
risk of death
• Impairments of executive function
• Visual-spatial disorganization and impaired
20 Very ill, requires urgent treatment visual- spatial memory
10 Moribund, rapidly fatal disease process • Personality change with blunting of affect
or disinhibited and inappropriate behavior
0 Dead
• Difficulties with language production
including dysprosodia, agrammatism, and
mild anomia
3. c—Medulloblastoma Mutism is considered to be part of the initial pre-
sentation of many children with cerebellar cogni-
Cerebellar mutism is a distinct clinical syndrome tive affective syndrome, but is less common in
described following surgery for posterior fossa adults. The major distinction between the two
tumors both in adults and children but most com- is the chronicity of the symptoms with cerebellar
monly in children. Its incidence varies from 2% mutism being more transient.
to 40% in different series, particularly with ver-
mian location of the lesion—hence its occurrence FURTHER READING
particularly after medulloblastoma resection. It Steinlin M, Imfeld S, Zulauf P, Boltshauser E, L€ ovblad KO,
consists of diminished speech output, hypotonia, Ridolfi Lüthy A, Perrig W, Kaufmann F. Neuropsychological
ataxia, and emotional lability. Typically a patient long-term sequelae after posterior fossa tumour resection during
childhood. Brain 2003;126(Pt 9):1998-2008.
who is initially fine in the first few days postoper-
atively develops mutism without any correspond- 5. b—Frontal lobe executive function
ing focal neurological signs. The deficit usually
recovers with time over a period of a few weeks FURTHER READING
to 6 months with an immediate return of full Devito EE, Pickard JD, Salmond CH, Iddon JL, Loveday C,
words and sentences. Resolution of the muteness Sahakian BJ. The neuropsychology of normal pressure hydro-
is often followed by a period of dysarthria, and cephalus (NPH). Br J Neurosurg 2005;19(3):217-24.

Neurosurgery Books Full


www.ketabpezeshki.com 66485438-66485457
16 NEUROPSYCHOLOGY AND NEUROLOGICAL REHABILITATION 239

6. b—There is a greater cognitive decline at function in coiled patients compared to clipped


12 months in patients with SRS + WBRT patients at 4-6 months. Further studies/longer
compared to SRS alone term follow up is needed, but each may have dif-
ferent effects on cognitive outcome at different
The efficacy of WBRT for treatment of cerebral time points since treatment.
metastasis is well documented but the establish-
FURTHER READING
ment of SRS for high precision delivery of radia-
Al-Khindi T, Macdonald RL, Schweizer TA. Cognitive and
tion has questioned its necessity. Given that functional outcome after aneurysmal subarachnoid hemorrhage.
survival is comparable between the two modali- Stroke 2010;41(8):e519-36.
ties, the controversy centers on cognitive and
neurological preservation. Supporters of SRS 8. d—Return to previous occupation is approxi-
point to evidence suggesting that focal radiation mately 60% in aneurysmal subarachnoid
is highly effective in preventing tumor progres- hemorrhage
sion in the irradiated volume and that irradiation
of normal or near-normal brain tissue increases FURTHER READING
the risk of cognitive decline in a brain that is Al-Khindi T, Macdonald RL, Schweizer TA. Cognitive and
already burdened with disease. Equally, propo- functional outcome after aneurysmal subarachnoid hemor-
nents of WBRT argue that focal radiation does rhage. Stroke 2010;41(8):e519-36.
not address potential micrometastatic foci that 9. e—Post-traumatic amnesia
are invisible to conventional imaging which, in
the absence of radiation treatment, can develop Post traumatic amnesia represents the length of
into larger lesions that compromises the patient's time from injury until return of orientation and
neurological and cognitive function. Ultimately, continuous memory for events. Its duration has
the debate revolves around the trade-off between been associated with presence or extent of skull
preserving the function of cerebrum that is not fracture, intracranial hemorrhage, raised intracra-
grossly infiltrated with tumor and the harmful nial pressure, residual neurological deficits, extent
effect of tumor growth from micrometastatic foci. of neuropathology, as well as with longer-term
In summary: functional outcomes and return to employment.
• Cognitive impairment at 12 months is 12% Recent studies investigating individuals surviving
in control and 41% in prophylactic cranial to discharge from hospital provide support for post
irradiation (30 Gy) traumatic amnesia as a stronger predictor of longer-
• Cognitive impairment at 12 months is 62% term functional outcome, return to employment,
for 25 Gy and 85% for 36 Gy prophylactic and cognitive impairment than Glasgow Coma
irradiation Score (GCS) or length of coma. It also accounts
• Cognitive impairment at 4 months is 24% for more variance in outcome than socio-
in SRS alone vs 52% in SRS + WBRT demographic factors.
FURTHER READING
FURTHER READING Ponsford J, Spitz G, McKenzie D. Using post-traumatic
McDuff SG, Taich ZJ, Lawson JD, et al. Neurocognitive amnesia to predict outcome following traumatic brain injury.
assessment following whole brain radiation therapy and radio- J Neurotrauma 2015.
surgery for patients with cerebral metastases. J Neurol Neuro-
surg Psychiatry 2013;84(12):1384-91. 10. e—Task oriented therapy

7. e—Coiling may offer an inferior cognitive The main principles underlying stroke rehabilita-
outcome in the short term and clipping the tion are as follows:
• The patient should be under the care of a
inferior cognitive outcome in the long term
specialist stroke rehabilitation unit whilst
The majority of studies suggest that clipped ver- in hospital, and a specialist stroke rehabili-
sus coiled patients do not differ in the main tation service when back in the community.
domains of cognitive and functional outcome. • Therapy should be task oriented (i.e. prac-
Some (non-randomized) studies suggest a poorer ticing an activity is the best way to improve
outcome with clipping at 1 year, with greater at that activity).
imaging evidence of focal encephalomalacia • The patient should be set both short- and
and infarction compared to coiled patients. long-term goals, and those goals should
Equally, some studies have shown poorer cognitive be relatively challenging and set at the level
of activities or social participation.

Neurosurgery Books Full


www.ketabpezeshki.com 66485438-66485457
240 PART II CARE OF THE NEUROSURGICAL PATIENT

11. e—Routine recruitment into neural stem cell used with caution. Primary advantages of the flexi-
transplantation trials for SCI ble approach to neuropsychological evaluation
include: (1) a potentially shorter administration
Rehabilitation has several general goals in SCI: time; (2) economical favorability; and (3) adapt-
• It should aim to minimize the risk of all pre- ability to differing patient situations and needs. Dis-
ventable complications, including through advantages of the flexible approach include: (1) the
patient education. need for greater clinical experience; (2) a lack of
• Teach the patient how to manage their standardized administration rules for some tests;
impairments and it needs to ensure that (3) a potential lack of comprehensiveness; and (4)
all required adaptations and equipment limitations in establishing systematic databases.
are identified and provided. A non-exhaustive list of commonly used tests is
• It may need to teach others how to provide shown below:
additional support to the patient if neces-
sary. In general patients with lesions below
the cervical level of the spinal cord can live Attention Trail Making Test, Stroop
fully independently, whereas patients with Test, Digit span (Wechsler
cervical spinal-cord lesions will need assis- Adult Intelligence Scale III
tance to a greater or lesser extent. and Wechsler Memory Scale
• In patients with spinal cord injury particular III)
attention needs to be paid to the manage- Memory Wechsler Memory Scale III,
ment of excretion from bowels and bladder, Rey-Osterrieth Complex
sexual function, and skin care. Figure Test, California Verbal
Learning Test-II, Rey
• Medical recognition and management of Auditory-Verbal Learning
autonomic dysreflexia (e.g. treat urinary Test
retention, blood pressure control with imme-
Executive functions Wisconsin Card Sorting Test,
diate nifedipine). Trail Making Test, Stroop
Test, Category Test
Language Boston Diagnostic Aphasia
EMI ANSWERS Examination, Reiten-Indiana
Aphasia Screening Test,
Word fluency
12. 1—d: Executive function, 2—a: Affect and
personality, 3—g: Motor processing Visuospatial and Facial Recognition Test, Rey-
visuomotor Osterrieth Complex
A single neuropsychological test may assess multi- processing Figure Test
ple domains of neuropsychological performance, Motor processing Finger oscillation test,
hence they may be performed as part of a fixed Grooved Pegboard test
(e.g. Halstead-Reitan) or flexible battery. Advan- Affect and personality Beck Depression and
tages of the fixed battery approach to neuropsycho- Anxiety Inventories
logical assessment include: (1) it provides a Effort (imbedded and Word memory test, tests of
comprehensive assessment of multiple cognitive free-standing) memory malingering, dot
domains; and (2) it uses a standardized format that counting test
allows the test data to be incorporated into data-
bases for clinical and scientific analysis. Disadvan-
tages of the fixed battery approach include (1)
time and labor intensiveness; and (2) a lack of flex- FURTHER READING
ibility in different clinical situations; specifically, Podell K, Gifford K, Bougakov D, Goldberg E. Neuropsycho-
multiple, nonequivalent data sets exist and specific logical assessment in traumatic brain injury. Psychiatr Clin
normative data with TBI patients should be North Am 2010;33(4):855-76.

Neurosurgery Books Full


www.ketabpezeshki.com 66485438-66485457
CHAPTER 17

STATISTICS
SINGLE BEST ANSWER (SBA) QUESTIONS
1. A new blood test for glioblastoma multiforme hemorrhage patients. What is the number
(GBM) has recently been discovered. Which of patients that need to be treated with the
one of the following best indicates the pro- new therapy to prevent one infarction?
portion of patients without GBM in whom a. 5
the test will be negative? b. 10
a. Negative predictive value c. 15
b. Positive predictive value d. 20
c. Power e. 25
d. Sensitivity
e. Specificity 6. The power of a study is best described as
which one of the following:
2. The proportion of patients experiencing a. The probability of a significant finding
postoperative CSF leak is to be compared b. The probability of correctly rejecting the
between two differently treated groups. null hypothesis
Which one of the following statistical tests c. The probability of needing further data at
is most appropriate to evaluate this? the end of the study period
a. Mann-Whitney U test d. The probability of obtaining an answer
b. Multiple linear regression e. The probably of accepting the null
c. Spearman's correlation coefficient hypothesis
d. Wilcoxon's test
e. χ 2 (Chi-squared) test 7. You wish to assess rare adverse effects
thought to occur in less than 0.1% of patients
3. A new test for cauda equina syndrome (CES) taking a new antiepileptic. What type of
has 80% sensitivity and 90% specificity. If, study is most appropriate for assessing these
on average 1 in 10 patients with CES symp- effects?
toms referred to neurosurgery actually has a. Phase I
CES, what are the odds that a patient with b. Phase II
CES symptoms testing positive on the new c. Phase III
test actually has CES? d. Phase IV
a. 4 in 10 e. Phase V
b. 4 in 15
c. 4 in 20 8. Which one of the following is a parametric
d. 4 in 25 statistical test?
e. 4 in 5 a. Multiple linear regression
b. Spearman's correlation coefficient
4. Standard deviation of a set of values is best c. Student's t-test
described as: d. Wilcoxon's test
a. The measure of spread of the values e. χ 2 (Chi-squared) test
around the mean
b. The simplest nonparametric statistical test 9. In a clinical trial of a new treatment for pitu-
c. The square of the variance of the values itary adenoma which one of the following are
d. The square root of the standard error of most likely to result in a type II error?
the mean a. False hypothesis
e. The standard error of the mean b. Large effect size
c. Normally distributed data
5. A new therapy reduces the rate of in- d. Small sample size
farction from 25% to 5% in subarachnoid e. Use of multiple statistical tests

e9
Neurosurgery Books Full
www.ketabpezeshki.com 66485438-66485457
e10 PART II CARE OF THE NEUROSURGICAL PATIENT

10. Recall bias is most likely to be an issue in significance of gender on survival in a cohort
which one of the following? of patients with subarachnoid hemorrhage?
a. Case-control studies a. Cox regression model
b. Cross-over trials b. Kaplan-Meier survival curve
c. Meta-analysis c. Linear regression
d. Prospective cohort studies d. Logistic regression
e. Randomized controlled trial e. Spearman's rank correlation

11. In a given population, which one of the fol- 17. A new brain scan to diagnose radiation
lowing best reflects a change in the balance necrosis (RN) is performed on 200 patients
of etiological factors of a particular disease? with suspected RN. Of 20 people eventually
a. Five year mortality rate diagnosed with RN, 5 tested positive on the
b. Incidence new scan. In addition, 45 people without
c. Period prevalence RN tested positive with the brain scan. What
d. Point prevalence is the sensitivity and specificity of the test?
e. Standardized mortality ratio a. Sensitivity 25, specificity 75
b. Sensitivity 25, specificity 90
12. A new therapy for cerebral venous sinus throm- c. Sensitivity 75, specificity 25
bosis finds that it reduces the mortality rate d. Sensitivity 90, specificity 25
from 4.4% to 3%. Which one of the following e. Sensitivity 90, specificity 75
best describe the absolute risk reduction?
a. 0.0014%
b. 0.014%
c. 0.14%
EXTENDED MATCHING ITEM (EMI)
d. 1.4% QUESTIONS
e. 14%
18. Statistical tests:
13. Which one of the following best explains a. Analysis of variance (ANOVA)
higher death rates in head injury patients b. Bonferroni correction
treated with thiopentone infusion? c. Cox proportional hazards regression
a. Attrition bias model
b. Confounding bias d. Fisher's exact test
c. Detection bias e. Friedman's test
d. Publication (reporting) bias f. Kruskal-Wallis test
e. Researcher bias g. Linear regression
f. Selection bias h. Log-rank test
i. Logistic regression
14. A trial of a new antibiotic for meningitis finds j. Mann-Whitney U test
that it reduces the mortality rate from 35% to k. Multiple linear regression
20%. What is the relative risk? l. Pearson correlation coefficient
a. 0.0015 m. Spearman's rank correlation coefficient
b. 0.015 n. Student's t-test
c. 0.15 o. Wilcoxon's signed-rank test
d. 1.5 p. χ 2 (Chi-squared) test
e. 15
For each of the following descriptions, select the
15. Which one of the following terms best most appropriate answers from the list above.
describes data collected about systolic blood Each answer may be used once, more than once
pressure of patients presenting with inciden- or not at all.
tal intracranial aneurysms? 1. Test to compare survivalbetween two unpaired
a. Bayesian variable groups using Kaplan-Meier estimates.
b. Continuous interval scale variable 2. Test to compare paired blood pressure
c. Continuous ratio scale variable measurements before and after starting
d. Nominal variable nimodipine.
e. Ordinal variable 3. Test to compare occurrence of postoperative
CSF leak after foramen magnum decompres-
16. Which one of the following types of test is sion when two different methods for closure
best used to estimate the effect and statistical were used in 10 patients.

Neurosurgery Books Full


www.ketabpezeshki.com 66485438-66485457
17 STATISTICS e11

19. Statistical terms: For each of the following descriptions, select the
a. Alpha most appropriate answers from the list above.
b. Beta Each answer may be used once, more than once
c. Confidence interval or not at all.
d. Degrees of freedom 1. Thoroughly examines a number of valid
e. Hazard ratio studies on a topic and mathematically com-
f. Intention to treat bine the results using accepted statistical
g. Meta-analysis methodology to report the results as if it were
h. NNT one large study.
i. Nonparametric 2. Focus on a clinical topic and answer a spe-
j. Odds ratio cific question. An extensive literature search
k. Parametric is conducted to identify studies with sound
l. Power methodology. The studies are reviewed,
m. Relative risk reduction assessed for quality, and the results sum-
n. Two tailed test marized according to the predetermined
o. Type I error criteria of the review question.
p. Type II error 3. Describes the relationship between diseases
and other factors at one point in time in a
For each of the following descriptions, select the defined population.
most appropriate answers from the list above.
Each answer may be used once, more than once 21. Levels of evidence:
or not at all. a. 1a
1. The proportion by which an intervention b. 1b
reduces the risk of an event c. 1c
2. Enables the null hypothesis to be rejected d. 2a
whether the new treatment is better or worse e. 2b
the current treatment (i.e. in either direction) f. 2c
3. Where patients are included in the analysis g. 3a
according to the group into which they were h. 3b
randomized, even if they did not actually i. 4
receive the treatment. j. 5

20. Study Design: For each of the following descriptions, select the
a. Case series most appropriate answers from the list above.
b. Case-control study Each answer may be used once, more than once
c. Cohort study or not at all.
d. Cross-sectional design 1. Individual randomized controlled trial
e. Cross-over design 2. Systematic review of case-control studies
f. Experimental study
g. Meta-analysis
h. Observational study
i. Prospective study
j. Randomized controlled trial
k. Retrospective study
l. Systematic review

SBA ANSWERS
1. e—Specificity Positive predictive value—probability of truly
having the disease if you test positive.
Sensitivity—proportion of patients with the dis- Power—The probability of correctly rejecting
ease who test positive. High sensitivity ¼ low false the null hypothesis.
negative.
Specificity—proportion of patients without 2. e—χ 2 (Chi-squared) test—this is a test of
the disease who test negative. High specifici- association between two categorical variables
ty ¼ low false positive. (e.g. gender, treatment group, etc).
Negative predictive value—probability of truly
3. e—4 in 5
not having the disease if you are test negative.

Neurosurgery Books Full


www.ketabpezeshki.com 66485438-66485457
e12 PART II CARE OF THE NEUROSURGICAL PATIENT

Odds of CES in patient referred with CES 7. d—Phase IV


symptoms and positive test ¼ pre-test odds in
symptomatic patients x likelihood ratio of
a positive test (sensitivity/(1  specificity)) ¼ Phase I Tests new drug or treatment in small
number of individuals (10-20) for the
1/10  0.8/(1  0.9) ¼ 8/10 ¼ 4 in 5. first time to evaluate safety, determine
Likelihood ratio for a positive result tell us safe dosage range and identify side
how much the odds of the condition increase effects
when the test result is positive. Phase II Drug or treatment given to a
larger people (100-200) to confirm
4. a—The measure of spread of the values efficacy and further evaluate
around the mean side effects
Phase III Done in larger groups of people (>1000)
Standard deviation (SD) of a population is the to confirm effectiveness and monitor
square root of the variance. For normally distrib- side effects
uted data, 68% of the values are within 1SD of the Phase IV Post-marketing safety surveillance of
mean, 95% within 2SD, and 99% within 3SD. new drug or treatment; may show
Standard error of the mean (SEM) is the stan- adverse effects in previously untested
dard deviation (of the population) divided by the groups or rare events
square root of the number of values in any sample Phase V New indication of drug or treatment and
from that population—it is a measure of how close repeat phases II and III
the sample mean is likely to be to the population
mean.

5. a—5
8. d—Student's t-test
Intervention Control
Group Group Total
Comparing Two Groups Only
Events 5 25 30 Data
Distribution Unpaired Data Paired Data
No events 95 75 170
Normal Student's Paired
Total 100 100 200 (parametric) t-test t-test
Event rate 0.05 0.25 - Non-normal Mann-Whitney Wilcoxon's
(nonparametric) U test signed-rank
Absolute Risk Reduction (ARR) ¼ risk in control group—risk
test
in intervention group ¼ 0.2 (20% reduction in
infarction rate). Comparing More Than Two
Relative risk reduction (RRR) ¼ (25  5)/25 ¼ 80%. Groups
Relative Risk (RR) ¼ relative difference in event rate between
intervention and control groups¼ (5/100)/(25/100) ¼ 0.2. Normal ANOVA Repeated-
Odds Ratio (OR) ¼ odds of event in intervention group/ (parametric) measures
odds of event in control group ¼ (5/95)/(25/75) ¼ 0.157. ANOVA
Number needed to treat is calculated as
1/ARR ¼ 1/(0.2) ¼ 5. Non-normal Kruskal-Wallis Friedman's
Number needed to harm (NNH) indicated how many (nonparametric) test test
patients on average need to be exposed to a risk factor
over a specific period to cause harm to one patient who
would not otherwise have been harmed.

6. b—The probability of correctly rejecting the Nonparametric


null hypothesis Parametric Tests Tests
Student's t-test Wilcoxon's signed-
The power of a study is the likelihood of it cor- rank test
rectly rejecting the null hypothesis when it is false
Analysis of variance χ 2 (Chi-squared)
(or 1  beta), and is normally expressed as a per- (ANOVA) test
centage. Power analysis can be used to calculate
the minimum sample size required so that one Analysis of covariance Mann-Whitney
(ANCOVA) U test
can be reasonably likely to detect an effect of a
given size, and vice versa. In most cases adequate Pearson's correlation Spearman's rank
power is accepted as 0.8, with beta 0.2 (Type II correlation
Kruskal-Wallis test
error—false negative rate) and alpha 0.05 (Type Fisher's exact test
1 error—false positive rate) although these may Friedman's test
vary depending on situation.

Neurosurgery Books Full


www.ketabpezeshki.com 66485438-66485457
17 STATISTICS e13

9. d—Small sample size


Confounding Present when the association
bias between an exposure and an
A type II error is the mistaken acceptance of the outcome is distorted by an
null hypothesis when it is incorrect (false nega- extraneous third variable
tive). This is most likely to occur when the power Researcher Investigators personal beliefs
of a study to detect differences is small, e.g. small bias influence the choice of methodology
sample size, small effect size. A type I error (false or question
positive) is if multiple statistical tests are used Attrition bias Systematic differences between
without any correction for multiple comparisons. groups in withdrawals from a study
leading to results based on
10. a—Case-control studies incomplete outcome data
Detection Systematic differences between
Recall bias is most likely to occur when partici- bias groups in how outcomes are
pants are asked about their experience, risk fac- determined/measured, especially if
tors or behavior after they have been diagnosed unblended subjects or assessors
with the disease under study. Carry-over In cross-over trials, carry-over of
effect treatment effect from one period to the
11. b—Incidence next means that the observed
difference between the treatments
depends upon the order in which they
were received
Incidence The number of new occurrences
of a disease in a given population
over a period of time. As only
new cases are included, this best 14. c—0.15
reflects changing likelihood of
developing the disease in
response to changing etiological 15. c—Continuous ratio scale variable
factors.
Point prevalence The prevalence of a condition in a
population at one point in time. Bayesian variable Combines new data to
Period The prevalence of a condition in a previous data before giving
prevalence population over a period of time result (compared to our
(e.g. 5 years). current practice of first
analyzing new data in
Standardized Ratio quantifying the increase isolation).
mortality ratio (SMR > 1) or decrease (SMR < 1)
in mortality of a study cohort with Continuous Has ordinal ranking and equal
respect to the general interval scale intervals but no true zero point
population. variable (e.g. temperature in degrees
Celsius)
Continuous ratio Has ordinal ranking, equal
12. d—1.4% scale variable intervals and absolute zero,
e.g. height, age, weight
13. f—Selection bias Nominal variable Numbers represent a
categorical label only (e.g.
gender, race, eye color)
Ordinal variable Numbers represent a
Selection Methods for selection result in categorical label with implicit
bias systematic differences in baseline rank ordering (e.g. severity
characteristics of groups being scales, age groups)
compared
Publication Research with positive results is
bias more likely to be published than
research showing no effect, hence 16. a—Cox regression model
interventions may seem more
effective than they actually are
Regression analysis is a technique for finding the
Reporting Selective within-study reporting of relationship between variables, one of which is
bias those analyses with statistically dependent on the other.
significant differences between
intervention groups more frequently
than nonsignificant differences

Continued

Neurosurgery Books Full


www.ketabpezeshki.com 66485438-66485457
e14 PART II CARE OF THE NEUROSURGICAL PATIENT

17. a—Sensitivity 25, specificity 75


Cox regression Used to investigate the
model relationship between an event (i.e.
death) and possible explanatory
RN + RN 2 Total
variables and provides estimates
of the effect that different factors Test positive 5 (True 45 (False 50
have on the time until the end positive) positive)
event (hazard ratio).
Test negative 15 (False 135 (True 150
Logistic Logistic regression is a variation negative) negative)
regression of linear regression that is used
when there are only two possible Total 20 180
outcomes (i.e. categorical
dependent/outcome variable). Sensitivity ¼ test positive/disease positive ¼ 5/20 ¼ 25%
Specificity ¼ test negative/disease negative ¼ 135/
Multiple linear used when dependent/outcome 180 ¼ 75%
regression variable is continuous NPV ¼ true negative/test negative ¼ 135/150 ¼ 90%
PPV ¼ true positive/test positive ¼ 5/50 ¼ 10%
Log-rank test A nonparametric test used for the
comparison of survival estimates
using Kaplan-Meyer or life-table
estimates to determine if there is
a statistically significant
EMI ANSWERS
difference.
18. 1—h, Log-rank test, 2—n, Paired t-test,
Kaplan-Meier Recalculates the survival rate
survival curve every time an end event (i.e. 3—d, Fisher's exact test
death) occurs in the data set
rather than at fixed intervals and The selection of the most appropriate test gener-
clearly demonstrates this on a ally depends on hypothesis, type of data (contin-
survival plot. uous or categorical variables), number of groups
(two vs. more than two), whether data is normally
distributed and whether data is independent
(unpaired) or dependent (paired; same individual
or possibly matched individuals). The tables
below should be used as guides only.

Outcome Variable

Quantitative Quantitative
Nominal Quantitative Continuous Continuous
Nominal (2) (>2) Ordinal Discrete (Non-Gaussian) (Gaussian)
Input Nominal (2 Fisher's exact χ2 Mann-Whitney Mann-Whitney Mann-Whitney Student's
variable categories) test or χ 2 or Log-rank test t-test
Nominal (>2 χ2 χ2 Kruskal-Wallis Kruskal-Wallis Kruskal-Wallis ANOVA
categories)
Ordinal Mann-Whitney - Spearman's Spearman's Spearman's Spearman's
(ordered/ correlation correlation correlation correlation
ranked
categories)

Quantitative Logistic - - Spearman's Spearman's Spearman's


discrete regression correlation correlation correlation or
Linear
regression
Quantitative Logistic - - - Spearman's Spearman's
continuous regression correlation correlation or
(non- Linear
Gaussian) regression
Quantitative Logistic - - - Linear Pearson
continuous regression regression correlation or
(Gaussian) Linear
regression

Table with permission from Swinscow TDV. Chapter 13: Study design and choosing a statistical test. In: Swinscow TDV.
Statistics at square one, 9th ed., BMJ, 1997.

Neurosurgery Books Full


www.ketabpezeshki.com 66485438-66485457
17 STATISTICS e15

Type of Data
Rank/Score/ Binomial (Two
Measurement Non-Gaussian Categories;
Aim of Study (Gaussian) Measurement Non-Gaussian) Survival Time
Describe one group Mean and SD Median, Proportion Kaplan-Meier
interquartile survival curve
range
Compare one group to One-sample t-test Wilcoxon's test χ 2 or Binomial test -
hypothetical value
Compare two groups Student's t-test Mann-Whitney Fisher's exact test Log-rank test
(unpaired) test (or χ 2 if sample large
enough)
Compare two groups Paired t-test Wilcoxon's test McNemar's test Conditional
(paired/matched) proportional
hazards
regression
Compare three or more One-way ANOVA Kruskal-Wallis χ 2 test Cox proportional
groups (unmatched) test hazards
regression
Compare three or more Repeated-measures Friedman's test Cochrane Q Conditional
groups (matched) ANOVA proportional
hazards
regression
Quantify association Pearson's Spearman's rank Contingency -
between two variables correlation correlation coefficients
Predict value from Simple linear Nonparametric Simple logistic Cox proportional
another measured regression or regression regression hazards
variable nonlinear regression regression
Predict value from Multiple linear or Multiple logistic Cox proportional
several measured or multiple nonlinear regression hazards
binomial variables regression regression

Table with permission from Motulsky H. Intuitive biostatistics: a nonmathematical guide to statistical thinking. OUP, 1995,
p. 298.

19. 1—m, Relative risk reduction, 2—n, Two tailed test, 3—f, Intention to treat analysis

Parametric Refers to statistical tests which assume the data needs to follow a certain distribution
(commonly a normal distribution)
Nonparametric Refers to statistical tests which do not depend on the distribution of the data
test
Absolute risk The difference between the even rate in the intervention group and that in the control group
reduction (reciprocal of NNT)
Relative risk The proportion by which an intervention reduces the risk of an event
reduction
Odds ratio The odds of an event happening in one group divided by the odds of it happening in another
group.
Hazard ratio The ratio of the hazard (chance of something harmful happening) of an event in one group
of observations divided by the hazard of an event in a different group. A HR of 1 implies no
difference in risk between the two groups, an HR of 2 implies double the risk. The HR should
be stated with its confidence intervals.
Likelihood ratio The likelihood that a test result would be expected in patients with a certain condition
divided by the likelihood that the same result would be in those without the condition
Confidence A range of values within which we are fairly confident the true population value lies (usually
interval 95% certain)
Meta-analysis A method of combining results from a number of independent studies to give one overall
estimate of effect.

Continued on following page


Neurosurgery Books Full
www.ketabpezeshki.com 66485438-66485457
e16 PART II CARE OF THE NEUROSURGICAL PATIENT

Intention to treat Where patients are included in the analysis according to the group into which they were
randomized, even if they did not actually receive the treatment.
NNT Number of patients that need to be treated for one to benefit the treatment.
Type I error A hypothesis that is correct is rejected—same as the p-value.
Type II error A hypothesis that is incorrect is accepted
Alpha Equivalent to the P-value
Beta The probability of accepting a hypothesis that is false
Power 1-beta; the probability that a study will detect a statistically significant difference
Kappa Measure of the level of agreement between two categorical variables. It is often used to
determine how accurately a test can be repeated, e.g. by different people.
Degrees of The number of independent pieces of information available to use in the calculation
freedom
One tailed test Enables the null hypothesis to be rejected if it is worse than the current treatment but not if it
is better (or vice versa).
Two tailed test Enables the null hypothesis to be rejected whether the new treatment is better or worse the
current treatment (i.e. in either direction)

Table with permission from Harris M, Taylor G. Medical Statistics Made Easy, 3rd ed., Scion, 2014.

20. 1—g, Meta-analysis, 2—l, Systematic review, 3—d, Cross-sectional design

Case series Consist of collections of reports on the treatment of individual patients or a report on a
single patient without controls
Case-control study Retrospective study in which patients who already have a specific disease (cases) are
compared with people who do not have the condition (controls) to see if past exposure of
any factor has occurred more or less frequently in cases than controls
Cohort study Identifies a group of patients who are already taking a particular treatment or have an
exposure, follow them forward over time, and then compare their outcomes with a
similar group that has not been affected by the treatment or exposure being studied
Cross-sectional Describe the relationship between diseases and other factors at one point in time in a
design defined population
Cross-over trials Allocate each participant to a sequence of interventions. A simple randomized cross-
over design is an “AB/BA” design in which participants are randomized initially to
intervention A or intervention B, and then “cross over” to intervention B or intervention
A, respectively
Observational study Investigator does not intervene in the care of the patient but records what happens, e.g.
cohort study, case-control study
Experimental study Investigator intervenes in the care of the patient in a pre-planned way and records the
outcome, e.g. RCT, laboratory studies
Prospective study A prospective study watches for outcomes, such as the development of a disease, during
the study period and relates this to other factors such as suspected risk or protection
factor(s). The study usually involves taking a cohort of subjects and watching them over
a long period
Retrospective study A retrospective study looks backwards and examines exposures to suspected risk or
protection factors in relation to an outcome that is established at the start of the study
Randomized Carefully planned experiments that introduce a treatment or exposure to study its effect
controlled trial on real patients. They include methodologies that reduce the potential for bias
(randomization and blinding) and that allow for comparison between intervention and
control groups. A randomized controlled trial is a planned experiment and can provide
sound evidence of cause and effect
Meta-analysis Thoroughly examine a number of valid studies on a topic and mathematically combine
the results using accepted statistical methodology to report the results as if it were one
large study
Systematic review Focus on a clinical topic and answer a specific question. An extensive literature search is
conducted to identify studies with sound methodology. The studies are reviewed,
assessed for quality, and the results summarized according to the predetermined criteria
of the review question

Neurosurgery Books Full


www.ketabpezeshki.com 66485438-66485457
17 STATISTICS e17

21. 1—b, Level 1b, 2—g, Level 3a

Level Description
1a Systematic review of RCTs
1b Individual RCT (with narrow CI)
1c All or none evidence since introduction of therapy e.g. all died before Rx now some survive on it, or some
died before Rx became available now none die on it
2a Systematic review of cohort studies
2b Individual cohort study (including low-quality RCT)
2c Outcomes research
3a Systematic review of case-control studies
3b Individual case-control study
4 Case series or poor-quality cohort/case-control studies
5 Expert opinion without critical appraisal, or based on physiology/bench research/first principles

Table with permission from Oxford Centre for evidence based medicine—levels of evidence (March 2009).

Neurosurgery Books Full


www.ketabpezeshki.com 66485438-66485457
CHAPTER 18

PROFESSIONALISM AND MEDICAL


ETHICS
SINGLE BEST ANSWER (SBA) QUESTIONS
1. Which one of the following is NOT a required cancer presents with multiple new cerebral
component for assessing an individual's capac- lesions, including several radiation necrosis
ity to make a specific treatment decision? lesions from previous radiosurgery. She is
a. Understand the information mildly drowsy but understands her situation
b. Retain the information for long enough to and is competent to make decisions regarding
be able to make the decision her care. During the consultation palliative
c. Use or weigh up the information to make care is discussed, but her husband demands
the decision that she be treated with any available life
d. Communicate their decision prolonging treatment. She says she doesn't
e. Make a rational decision want to go through it any more, to which he
responds by threatening to leave her if she is
2. An 8-year-old boy requires a posterior fossa just going to give up. Which one of the follow-
craniotomy for a space occupying lesion. His ing principles is potentially at risk in this
parents consent to the treatment plan. Which situation?
one of the following would you try to establish a. Capacity
with respect to the child himself? b. Voluntariness
a. Informed consent c. Disclosure of relevant information
b. Informed assent d. Authorization
c. Parens patriae e. Justice
d. Gillick competence
e. Coercion 5. Which one of the following is/are NOT part
of the four principles of biomedical ethics?
3. A 61-year-old male is admitted with a right a. Autonomy
frontal space occupying lesion, consistent with b. Utilitarianism
a glioblastoma multiforme. His imaging shows c. Beneficence
a significant amount of edema and midline d. Non-maleficence
shift and he is started on high-dose dexameth- e. Justice
asone. He is confused with a mild left hemipar-
esis. His family arrive a few hours later, and 6. A mother does not want her son, a 12-year-old
you explain that he will need an operation bright, good athlete without neurological def-
and that the mass is probably cancerous. As icits, to know that his cerebellar astrocytoma
you take them back to his bedside, they ask has only been partially removed. She thinks
you not to tell their father about the likely that knowing this fact would place her son in
diagnosis. Which one of the following princi- emotional jeopardy, because a second proce-
ples would be most compromised by operating dure could diminish his sporting abilities.
on him at this point in time? Over time, the follow-up MRI showed a slow
a. Autonomy but clear progress of the tumor requiring fur-
b. Non-maleficence ther surgery. Which one of the following eth-
c. Justice ical principles are most relevant?
d. Beneficence a. Autonomy and beneficence
e. Futility b. Beneficence and justice
c. Autonomy and justice
4. A 31-year-old female who has been treated for d. Justice and non-maleficence
recurrent cerebral metastases from breast e. Autonomy and non-maleficence

242
Neurosurgery Books Full
www.ketabpezeshki.com 66485438-66485457
18 PROFESSIONALISM AND MEDICAL ETHICS 243

7. A 45-year-old female presents with WFNS UK-SPECIFIC QUESTIONS


grade I subarachnoid hemorrhage on evening.
Vascular imaging reveals a 1.2 cm left supracli- 10. Which one of the following is NOT required
noid internal carotid artery aneurysm. The for a valid advanced decision?
operating neurosurgeon specializes in func- a. Mental capacity present when made
tional neurosurgery and elects to perform a b. It applies to the situation where it is being
clipping the following morning as there is no considered
aneurysm surgeon available for a further c. You must be aged 25 or over
36 h. During the dissection around the aneu- d. Must be signed by you and a witness if
rysm, an intraoperative rupture occurs and you wish to refuse life-sustaining treat-
the surgeon struggles to obtain proximal con- ments
trol leading to intraoperative hypotension e. Must have been made without harassment
from blood loss and prolonged cerebral ische- by anyone else
mia from temporary clipping. Postoperatively
the patient wakes up on the neurointensive 11. Frasier guidelines are best described as clar-
care unit with complete hemiplegia and global ifying circumstances surrounding which
aphasia. Which one of the following factors is one of the following?
LEAST relevant to this surgical complication? a. Competence of a child to consent to medi-
a. Task factors cal treatment without parental involvement
b. Individual factors b. Competence of a child to consent to men-
c. Team factors tal health disorder treatment without
d. Patient factors parental involvement
e. Organizational factors c. Competence of a child to consent to con-
f. Situational factors traceptive advice and treatment without
parental involvement
8. A 34-year-old woman has a long history of epi- d. Competence of a child to withhold con-
lepsy since the age of 24. She only has seizures sent to medical treatment without paren-
when she sleeps, and her last seizure was 3 weeks tal involvement
ago. She is known to be compliant with every e. Competence of a child to withhold con-
medication she's been given. She retains the sent to mental health disorder treatment
driving license which she applied for last year. without parental involvement
Which one of the following is most accurate?
a. She must stop driving because her seizures 12. You are a junior doctor in the Emergency
are not well controlled Department (ED). A 5-year-old boy who
b. She may be able to continue driving as her has been in ED four times previously this
seizures only occur during sleep year with several episodes of trauma that
c. She must stop driving immediately as she did not seem related. Today, the child is
has not been seizure free for 1 year brought with a complaint of “slipping into
d. The doctor must inform the DVLA and a hot bathtub” with a small burn wound on
stop her driving his lower leg. Which one of the following
e. She must give up driving indefinitely would you do next?
a. Admit the child to remove him from pos-
9. A 7-year-old boy presents with a reduced GCS sibly dangerous environment
and CT head scan shows significant intraven- b. Phone the patient’s family doctor
tricular hemorrhage and hydrocephalus. You c. Report your concerns to the local social
discuss the imaging and the plan for an emer- services
gency external ventricular drain with the d. Accept the parent’s explanation
father but he is not willing to proceed if there e. Ask the parent whether there has been
is any chance his son will be left a “vegetable.” any abuse
What is the next step in management?
a. Call social services 13. A 14-year-old boy presents with precocious
b. Obtain a court order to proceed with surgery puberty and headaches. Cranial imaging
c. Do not perform surgery as lacking parental reveals a pineal region mass with hydroceph-
consent alus. An endoscopic third ventriculostomy is
d. Proceed to surgery in the best interests of planned and discussed with the family, but
the child the boy refuses to have the operation. He is
e. Keep the child sedated and ventilated on aware that without surgery, death is likely.
NICU until consent is gained

Neurosurgery Books Full


www.ketabpezeshki.com 66485438-66485457
244 PART II CARE OF THE NEUROSURGICAL PATIENT

What is the next appropriate step in 17. A 44-year-old victim of a car accident with a
management? severe closed head injury was admitted to the
a. Gain parental consent and proceed to ICU. The patient rapidly deteriorated and
surgery was declared brain dead within 24 h. He is
b. Apply to High Court for wardship not on the organ donor register. Which one
c. Respect the boy’s decision and do not of the following statements is most accurate?
operate a. It is not possible for others to consent on
d. Proceed to treat in best interests under behalf of a deceased patient
Mental Capacity Act 2005 b. Organ donation is not valid if the poten-
e. Call child protection services tial donor is not on the register
c. It is not possible for children to consent to
14. A 7-year-old boy is a Jehovah’s Witness and organ donation even if they are Gillick
was involved in a RTA (road traffic accident). competent
He is in hemorrhagic shock and requires emer- d. Organ donation is not an option in this
gency blood transfusion but his mother refuses situation
to give parental consent. Which one of the fol- e. Family members cannot legally veto
lowing is the most appropriate next step? organ donation if the patient is on the
a. Call child protection services organ donor register
b. Give blood anyway as this is an emergency
situation 18. Disclosure of confidential information with-
c. Do not give blood transfusion due to lack out patient consent may occur under certain
of parental consent circumstances. Which one of the following
d. Contact the courts by telephone is unlikely to meet the criteria for such
e. Get advice from the Hospital Liaison disclosure?
Committee for Jehovah’s Witnesses a. Discussion with a competent patient's
family member
15. A 16-year-old girl is a Jehovah’s Witness. She b. Terrorist act
refuses a lifesaving blood transfusion. She is c. Notifiable infectious disease
aware of and understands the consequences. d. Criminal act
What is the next step in management? e. Significant risk of harm to others
a. Gain parental consent to give blood
b. Give blood anyway as it is an emergency
situation
c. Give blood anyway as she is not US-SPECIFIC QUESTIONS
competent
d. Do not give blood transfusion but involve 19. Which one of the following remains valid
courts when the principal dies?
e. Call child protection a. Durable power of attorney
b. Ordinary power of attorney
16. Hilary is a 30-year-old schizophrenic patient. c. Specific powers of attorney
She has an abscess in her chest. But she is d. General powers of attorney
refusing treatment for it. She is also refusing e. Last will and testament
to take her medication for schizophrenia for
the last 2 weeks. She is thought to have full 20. A patient under your care recently diagnosed
capacity, and understands the effects and con- with glioblastoma multiforme and under-
sequences of not being treated for her chest gone gross total resection has threatened to
abscess or taking any antipsychotic medica- seriously harm their partner, who he believes
tion. Which one of the following is true? is having an affair. He is being discharged
a. She can be detained for treatment of both today, but as you leave the room to seek
her abscess and her schizophrenia advice from another colleague regarding his
b. She can be detained for treatment of her comments you turn to see him place a large
abscess only serrated kitchen knife into his bag. Which
c. She can be detained for treatment of her one of the following may become relevant?
mental health disorder only a. Tarasoff decision
d. She cannot be detained as she is b. Doctrine of double effect
competent c. Waiver
e. She cannot have capacity as she has a d. Virtuism
mental health disorder e. Categorical imperative

Neurosurgery Books Full


www.ketabpezeshki.com 66485438-66485457
18 PROFESSIONALISM AND MEDICAL ETHICS 245

21. In the USA, the Emergency Medical Treat- c. Invalid consent


ment and Active Labor Act makes all of the d. Implied consent
provisions except for which one of the e. Valid advance directive
following? f. Invalid advance directive
a. Transfer of a stable patient for non- g. Gillick competence
emergency care h. Assent
b. Right to request a medical screening i. Dissent
examination to exclude an emergency j. Competence
medical condition k. Capacity
c. Transfer of an unstable patient when the
treating physician feels the benefits out- For each of the following descriptions, select the
weigh the risks most appropriate answers from the list above.
d. Treatment of an emergency medical con- Each answer may be used once, more than once
dition until it is resolved or stabilized or not at all.
e. Duty to report inappropriate transfers 1. Doctor tells patient they need a blood test and
patient holds out arm and rolls up sleeve.
22. In those aged under 18 years, which one of the 2. On the insistence of a close relative, an 80-
following circumstances would parental con- year-old man with advanced dementia signs
sent for treatment still usually be required? a form stating he does not want medical treat-
a. Emancipated minor ment if he becomes unwell.
b. Decision regarding birth control 3. A patient with learning difficulties who is
c. Decision regarding substance abuse unable to read or write has the nature and pur-
treatment pose of a lumbar puncture explained to him in
d. Decision regarding a life or limb- terms which he can understand and agrees to
threatening conditions be treated.
e. Decision regarding tethered cord release
25. Medically assisted dying:
a. Suicide
b. Physician-assisted suicide
EXTENDED MATCHING ITEM (EMI) c. Murder
QUESTIONS d. Attempted murder
e. Voluntary euthanasia
23. Ethical concepts: f. Non-voluntary euthanasia
a. Autonomy g. Involuntary euthanasia
b. A positive right h. Doctrine of double effect
c. A negative right
d. Beneficence For each of the following descriptions, select the
e. A categorical imperative most appropriate answers from the list above.
f. Justice Each answer may be used once, more than once
g. Non-maleficence or not at all.
h. Virtue ethics 1. Deliberately ending the life of a person who is
i. Futility incapable of expressing any wishes about
j. Utilitarianism whether they want to live or die, motivated by
a consideration of that person's best interests.
For each of the following descriptions, select the 2. Deliberately ending the life of an elderly per-
most appropriate answers from the list above. son who is gravely ill to make the hospital bed
Each answer may be used once, more than once available who can get more use of it.
or not at all. 3. A competent, able bodied person has a pro-
1. The principle of doing good or improving the gressive illness which will render her incapable
welfare of patients. at a later stage. She seeks medical assistance for
2. The idea that we should create laws that max- medication with which she may end her
imize benefit. own life.
3. Respecting decisions made by those capable of 4. A competent patient who is in pain and distress
making decisions. but unable to take his own life asks his doctor
to administer a lethal injection.
24. Consent: 5. A doctor increases the dosage of painkillers/
a. Written consent sedation, at appropriate levels to control
b. Valid consent symptoms in the knowledge that this may

Neurosurgery Books Full


www.ketabpezeshki.com 66485438-66485457
246 PART II CARE OF THE NEUROSURGICAL PATIENT

have a life shortening side effect. The doctor For each of the following descriptions, select the
is adamant that she only intends the pain- most appropriate answers from the list above.
killing effect of the medication and not side Each answer may be used once, more than once
effect. or not at all.
1. To treat equals equally and unequals
26. Ethical theory: unequally according to morally relevant
a. Altruism inequality.
b. Aristotelian justice 2. The right not to be killed and to possess
c. Communitarianism property.
d. Consequentialism 3. Take from each according to ability and give
e. Deontology to each according to need.
f. Hippocratic oath 4. A rational person who makes a decision behind
g. Libertarianism a veil of ignorance will look after the least
h. Principalism well off.
i. Utilitarianism 5. Act to maximize welfare for the greatest num-
j. Virtuism ber at the least cost.

SBA ANSWERS
1. e—Make a rational decision responding, and (4) soliciting an expression of
the child's willingness to accept the proposed care.
A lack of capacity can be permanent or tempo- In other jurisdictions (e.g., England, Australia,
rary and should be assessed in relation to a spe- Canada), this presumption may be rebutted
cific decision to be made. A person with the through proof that the minor is “mature” (e.g.,
capacity to consent to therapy should be able “Gillick competent” in the UK) although it is still
to understand the relevant information (treat- good practice to also seek parental consent/agree-
ments purpose, nature, likely effects and risks, ment. Although there is no lower age limit defined
chances of success and alternatives to the pro- for which a child can be deemed Gillick competent,
posed treatment), retain the information, it is unlikely to apply for children under 13 years
weigh up the information to make a decision old. In cases of incompetent minors, informed con-
and be able to communicate it in some way. In sent is usually required from a person with parental
the UK, the Mental Capacity Act 2005 a might responsibility. If the person with parental respon-
be used to give treatment for physical health sibility refuses to consent for a specific treatment
problems to someone aged over 16 years who and is deemed negligent, medical treatments can
lacks capacity in their best interests (e.g., because be given in the best interests of the child in emer-
of a mental illness, dementia, learning difficul- gencies or the treating team can make an applica-
ties, unwell). tion to the High Court which can exercise its
power as parens patriae (legal protector of citizens
2. b—Informed assent unable to protect themselves) by making the child
a ward of the Court, such that it takes on the
Historically, children have been thought to lack responsibility for consenting for the child.
capacity and are hence unable to provide consent
on their own. Typically, decisions were made by FURTHER READING
their surrogate, usually a parent or guardian, and American Academy of Pediatrics, Committee on Bioethics.
often without the input of the child. More Informed consent, parental permission, and assent in pediatric
recently, developmentally capable minors can practice. J Pediatr 1995;102:169–76.
be allowed to consent on their own, and those
without the developmental capacity still partici- Gillick v. West Norfolk and Wisbech Area Health Authority
pate in the process of decision-making through [1986] AC 112.
assent. The American Academy of Pediatrics
issued a policy statement in 1995 on assent that 3. a—Autonomy
should be followed where possible. The process
of assent involves (1) helping the child achieve a This case highlights challenges to the consent
developmentally appropriate awareness of the process often seen in neurosurgical patients.
nature of his/her condition, (2) telling the child Firstly, his capacity to make autonomous deci-
what they can expect from tests and treatments, sions is compromised because of the effects of
(3) assessing the child's understanding of the situ- his tumor. Therefore, the immediacy of the clin-
ation and the factors influencing how they are ical situation will dictate whether we can afford to

Neurosurgery Books Full


www.ketabpezeshki.com 66485438-66485457
18 PROFESSIONALISM AND MEDICAL ETHICS 247

wait and see if he regains capacity after a period of • Autonomy—freedom of the patient to choose
dexamethasone treatment or if a surrogate deci- and be an advocate for their own health.
sion maker (e.g., partner, family member) is con- • Beneficence—what is considered to be of
sulted about what they think the patient would the patient's best interest.
request to have done if they had the capacity. Sec- • Non-maleficence—the harm that may
ond, the family has asked that information be come to a patient because of a specific deci-
withheld from the patient. In a patient with com- sion/treatment (“first do no harm”).
petence or capacity, the withholding of informa- • Justice—the legal aspects that impact upon
tion does not allow the patient to make an the ethical scenarios.
informed decision hence any subsequent consent
cannot be valid. For patients, full disclosure of 6. a—Autonomy and beneficence
relevant information (including risks and bene-
fits) is a right but not a duty—they may not wish When parents request information to be kept
to have this information disclosed to them. In this from their children, it may be legally permissible,
situation they are effectively waiving their right to but at the same time compromises the right of the
consent (as valid consent must be informed)— child to autonomy. Therefore, a careful assess-
hence it must be well documented and reasons ment of the following aspects is obligatory:
explored. In the case above, while it may be rea- (1) The ability of the minor to fully understand
sonable to withhold certain information while he the situation and to anticipate and evaluate
remains confused, if he regains capacity before future consequences.
any planned operation then an attempt to get (2) Whether the parental surrogate decision-
informed consent must be made. making is in the best interests of the child
or is it obstructing beneficence.
FURTHER READING After assessment of these aspects, it is the duty of
McDonald P. Informed consent (chapter). In: Ammar A, the physician to form a personal opinion (with
Bernstein M (Eds.), Neurosurgical Ethics in Practice: help from ombudsmen or other authoritative
Value-Based Medicine. Springer, 2014, p. 54.
persons or bodies, as needed), based on the con-
cept of beneficence, and to try to act accordingly
4. b—Voluntariness to work with the parents to take the right
approach.
An action such as consenting to a treatment is con-
sidered voluntary if it is undertaken freely, without
undue influence or coercion from others. How- FURTHER READING
ever, medical decisions are almost always influ- Tan TC, Ammar A. Privacy and confidentiality. In: Ammar A,
enced by the opinion of doctors, family, friends, Bernstein M (Eds.), Neurosurgical Ethics in Practice: Value-
and past experience or knowledge. Identifying Based Medicine. Springer, 2014, p. 63.
the difference between persuasion which is allow-
able and under certain circumstance perhaps even
obligatory (e.g., if a particular option is clearly in
the best interests of a particular patient), and the Task factors Clear protocol, information,
coercion demonstrated in the husband's threat to omission of necessary steps
leave her if she doesn't want to keep going is key Individual Mental readiness, technical
to maximizing patient autonomy. factors performance, fatigue
Team factors Effective communication,
FURTHER READING leadership, confidence, ability
McDonald P. Informed consent. In: Ammar A, Bernstein M to manage unexpected events
(Eds.), Neurosurgical Ethics in Practice: Value-Based Medi- Patient factors Obesity, anatomical
cine. Springer, 2014, p. 57. variations, severity of disease,
comorbidities
5. b—Utilitarianism Organizational Appropriate staff, equipment,
factors scheduling/timing of procedure,
substitution of team members
There are four principles widely used as a frame-
work for analyzing ethical problems. The four Situational Distractions, interruptions, poor
factors equipment design
principles must be applied in the appropriate con-
text and should have equal importance (allowing See Ammar A, Bernstein M. Neurosurgical Ethics in
conflicts to arise). They are: Practice: Value-Based Medicine. Springer.

Neurosurgery Books Full


www.ketabpezeshki.com 66485438-66485457
248 PART II CARE OF THE NEUROSURGICAL PATIENT

7. f—Situational factors with epilepsy for at least 10 years and during that
8. b—She may be able to continue driving time she is only known to have seizures during
because she only has sleep seizures her sleep. She is also very compliant with her
doctor's treatments. Given she has reapplied for
When patients have their first seizure, they the license recently, the licensing agency may
should inform the licensing agency and must stop be satisfied she does not pose any danger to the
driving. In this case, patient has been diagnosed public.

UK REGULATIONS KEY POINTS • Withdrawal: stop driving during period of


(WWW.EPILEPSY.ORG.UK) withdrawal and for 6 months after
Group 1 Licence (Cars)
US REGULATIONS (VARIABLE DEPENDING ON STATE;
• Awake seizures affecting consciousness: ADAPTED FROM WWW.EPILEPSY.COM)
allowed when 12 months seizure free after
Ordinary Driver's Licence
most recent seizure
• Awake seizures not impairing consciousness: Allowed if free of seizures that affect consciousness
allowed if in the last 12 months you have only for a certain period of time, often at least 1 year.
had seizures which do not affect your con- Recently, shorter intervals of seizure freedom
sciousness OR if you have only ever had sei- are being required, for example 3-6 months
zures not affecting your consciousness Physician usually fills out a form with the date of last
• Sleep seizure(s): allowed if seizure free or only seizure, seizure type, and other relevant information.
having sleep seizures for 12 months (and never Some states ask for the doctor's recommendation
had seizures while awake) OR awake seizures in about the person's ability to drive while others leave
the past but only sleep seizures for last 3 years this to the Department of Motor Vehicles (DVM)
• Isolated seizure (after being seizure free for
5 years): allowed after 6 months seizure free- Review and Decision Process
dom if neurologist thinks another seizure is In most states, the medical information and license
unlikely and deemed not a risk to the public application is reviewed by the state's DMV. In com-
• Provoked seizure (e.g., eclamptic, neurosur- plex cases, or when the decision is not clear, informa-
gery, TIA/stroke): assessed on individual basis tion is forwarded to a consulting doctor or the state's
Group 2 Licence (HGV, Minibus, etc.) medical advisory board
• If you have had two or more seizures to regain Commercial Driver's Licenses
group 2 entitlements you need to have Allowed in people with a history of epilepsy/seizures
regained your group 1 licence AND been sei- who have been seizure-free off medication for
zure free for 10 years AND not taken epilepsy 10 years
medication in the last 10 years
• If you have had 1 isolated seizure the same rules Personal Liability
usually apply, but you may be able to regain it if A person with epilepsy may be civilly or criminally lia-
you have been seizure free for 5 years AND ble for a motor vehicle accident caused by seizures.
off epilepsy medication for 5 years AND Liability may occur when a person drives against
assessed as fit to drive by a neurologist medical advice, without a valid license, without noti-
• If you have had a provoked seizure you will be fying the state department of motor vehicles of the
assessed individually medical condition, or with the knowledge
Driving During Antiepileptic Withdrawal/Change that he or she is prohibited from driving
• Change: advised by doctor if need to stop driv-
ing and for how long, no need to inform DVLA
or return license

See Nashef L, Capovilla G, Camfield C, Camfield P, Nabbout R. Transition: driving and exercise. Epilepsia 2014;55
(Suppl. 3):41–5.

Neurosurgery Books Full


www.ketabpezeshki.com 66485438-66485457
18 PROFESSIONALISM AND MEDICAL ETHICS 249

9. d—Proceed to surgery as in best interests of [1985], was presided over by Lord Scarman and
the child Lord Frasier and regarded legal action taken
against the advice given to doctors in a health cir-
In an emergency where you consider that it is the cular that they could prescribe contraception to
child's best interests to proceed, you may treat the minors at their discretion. Victoria Gillick felt
child, provided it is limited to that treatment prescribing contraception in under 16s was illegal
which is reasonably required in that emergency. because the doctor would commit an offence of
Therefore, in this case, the surgery should be per- encouraging sex with a minor, and that it would
formed. In the UK this is governed by the Family be treatment without parental consent. The case
Reform Act 1969. had two main outcomes relevant to health
professionals:
10. c—You must be 25 or over to make an 1. The concept of “Gillick competence”:
advanced decision which declared the parental right to deter-
mine whether or not their minor child below
An advance decision to refuse treatment (Living the age of 16 will have medical treatment ter-
Will; advance directive) is legally binding as minates if and when the child achieves suffi-
long as it complies with the Mental Capacity cient understanding and intelligence to
Act 2005, applies to the situation and is valid; it understand fully what is proposed (Lord
aims to take the place of best interest decisions Scarman).
made for you by other people. Advance decisions 2. Frasier Guidelines which outline the
are valid if: criteria which must be met for doctors to
• you are aged 18 or over and had the capacity to lawfully provide contraceptive advice and
make, understand and communicate your treatment to under 16s without parental
decision when you made it consent.
• you specify clearly which treatments you wish
to refuse 12. c—Report your concerns to the local author-
• you explain the circumstances in which you ity social services
wish to refuse them
• it is signed by you and by a witness if you want The British Medical Association guidance for
to refuse life-sustaining treatment doctors who have concerns about a child state
• you have made the advance decision of your “where a doctor has a reasonable belief that a
own accord, without any harassment by child is at serious risk of immediate harm, he or
anyone else she should act immediately to protect the inter-
• you haven't said or done anything that would ests of the child, and this will involve contacting
contradict the advance decision since you one of the three statutory bodies with responsibil-
made it (for example, saying that you have ities in this area: the police, the local authority
changed your mind) social services or the NSPCC, and making a full
report of their concerns.”
11. c—Competence of a child to consent to con-
traceptive treatment without parental 13. a—Gain parental consent and proceed to
involvement surgery

The House of Lords case Gillick versus West


Norfolk and Wisbech Area Health Authority

Neurosurgery Books Full


www.ketabpezeshki.com 66485438-66485457
250 PART II CARE OF THE NEUROSURGICAL PATIENT

UK Regulations Regarding Consent for Treatment


Others Able
Able to Consent Able to Refuse Can be Treated to Consent
Capacity to Treatment? Treatment? in Best Interests? for Them?
Adult
Competent Yes Yes (can be rational, No No
irrational or
groundless)
Incompetent No No Yes—Mental Yes—Court
Capacity Act 2005
16- and 17-year-olds
Competent Yes—Capacity assumed No—decision to Yes—but need Yes—persons with
under Family Law Reform withhold consent can be court's authority parental
Act 1969 overturned by person responsibility,
with parental High Court,
responsibility specific issues
or court order
Incompetent No No Yes—but may Yes—persons with
need court's parental
authority if responsibility,
disagreement High Court,
about MCA specific issues
2005 order
Under 16-year-olds
Competent Yes—only if deemed No—decision to Yes—but need Yes—persons with
Gillick competent, but withhold consent can be court's authority parental
if possible also with overturned by person responsibility,
consent of person with parental High Court,
with parental responsibility specific issues
responsibility or court order
Incompetent No No Yes—but need Yes—persons with
court's authority parental
responsibility,
High Court,
specific issues
order
See Shaw M. Competence and consent to treatment in children and adolescents. Adv Psychiatr Treat 2001;7:150–9.

14. b—Give blood anyway because this is an a child. Two commonly used arguments when
emergency situation parents refuse treatment are parental rights to
raise children as they see fit and religious
Section 8 of the Family Reform Act 1969 states freedom. Courts throughout the Western world
that in an emergency where you consider that it recognize parental rights, but these rights are
is in the child's best interests to proceed, you not absolute. Parental rights to raise children
may treat the child, provided it is limited to are qualified by a duty to ensure their health,
that treatment which is reasonably required in safety, and wellbeing. Parents cannot make deci-
that emergency. Therefore, the child should be sions that may permanently harm or otherwise
given the transfusion. Traditionally, where young impair their healthy development. If treatment
children are concerned, the power to give or refusal results in a child suffering, parents may
withhold consent to medical treatment on their be criminally liable. However, before any harm
behalf lies with those with parental responsibility. comes to the child the courts are usually asked
Legally, except in an emergency, parental consent to exercise their power under the doctrine of
is necessary to perform any medical procedure on parens patriae which allows state interference to

Neurosurgery Books Full


www.ketabpezeshki.com 66485438-66485457
18 PROFESSIONALISM AND MEDICAL ETHICS 251

protect a child's welfare. This principle applies that the court will override the refusal in the
whether or not the child is in imminent danger, child's best interests. In Scotland, although the
as parents are always required to make decisions Age of Legal Capacity (Scotland) Act does not
in the child's best interests. specifically refer to treatment refusal, the infer-
ence is that a child deemed competent could
FURTHER READING refuse, as well as consent to, treatment. In North
Wolley S. Children of Jehovah's Witnesses and adolescent America, the situation for mature minors is state/
Jehovah's Witnesses: what are their rights? Arch Dis Child province dependent.
2005;90:715–19.

Child Parental
Consent to Consent to
Transfusion? Transfusion? Plan
Competent (16- or 17-year- Yes No Transfuse
old) or Gillick competent No No In emergency in England/Wales, aim to
(Under 16) telephone the court for declaration that
treatment is lawful if time permits. The young
person must not be allowed to die for want of
blood
In less urgent cases, aim to persuade child/
family, involve Hospital Liaison Committee of
the Jehovah's Witnesses
No Yes Transfuse with parental right to consent
Incompetent N/A No Transfuse if in best interest
N/A Yes Transfuse with parental right to consent
See Wolley S. Children of Jehovah's Witnesses and adolescent Jehovah's Witnesses: what are their rights? Arch Dis
Child 2005;90:715–19.

15. d—Do not give blood transfusion but involve 16. c—She can be detained for treatment of her
courts mental health disorder only

The rights of adolescents to refuse medical treat- The Mental Capacity Act 2005 can only be used to
ment vary throughout the world and this judicial give treatment to somebody aged over 16 without
inconsistency creates confusion among health- their consent if that person is assessed as lacking
care workers. In England and Wales, mature capacity to make a specific decision at that partic-
minors (Gillick competent or over 16) may con- ular time, and if treatment would be in their “best
sent to, but not refuse, treatment, with the courts interests.” Mostly, the doctor who will be respon-
using the “best interests” test to override the sible for giving the medical treatment will be
opinions of adolescents. In 1969, the Family responsible for making the “best interests” deci-
Law Reform Act set the age of consent for medical sion and in order to reach this decision, he or
treatment at 16 but did not specifically deal with she must work through a process and statutory
parental-child conflict. The implication, how- checklist which is contained in the Mental Capac-
ever, is that a child's consent to a procedure over- ity Act 2005. If hospital staff want to detain some-
rides parental opinion. The logical inference one for treatment under the Mental Capacity Act,
from Gillick is that competent children are com- they need to use the “Deprivation of liberty safe-
petent to both accept and refuse treatment; yet guards” procedure. This should not be used to
subsequent decisions suggest that a child's refusal detain someone for treatment for a mental health
may be overridden by a proxy's consent to that problem—this is the role of the Mental Health Act
treatment and that the child's refusal, while 1983 (2007). This law allows people living in
important, may not be conclusive. Where treat- England and Wales who have a “mental disorder”
ment refusal was religion based, there was con- to be admitted, detained and treated for their men-
cern about the child's freedom of choice in the tal disorder in hospital without their consent (even
context of a religious upbringing in addition to if they have capacity/are competent)—either for
concerns about whether the child fully grasped their own health or safety, or for the protection
the implications of treatment refusal. Thus, while of other people. This can be done under various
a child's refusal should be considered, it is likely sections of the Mental Health Act 1983 (2007):

Neurosurgery Books Full


www.ketabpezeshki.com 66485438-66485457
252 PART II CARE OF THE NEUROSURGICAL PATIENT

doctor to disclose information depending on


MHA
Section Use the situation.
Section 2 Admission for assessment and 19. e—Last will and testament
initiation of treatment (up to
28 days)
A power of attorney is a document in which one
Section 3 Admission for treatment (up to
6 months) competent person (the principal) appoints
another person (the attorney-in-fact) to act for
Section 4 Allows emergency admission when
not enough time to organize S2/S3 him or her. It becomes invalid when the principal
(up to 72 h) dies and cannot be used to bequeath property
Section 5(2) Allows a doctor to stop a voluntary upon the death of the principal (this is the func-
inpatient from leaving hospital (for tion of the last will and testament).
72 h)
Section 5(4) Allows a nurse to stop a voluntary
inpatient from leaving hospital (for Durable power Remains valid even if the
6 h) of attorney principal later becomes mentally
Section 135 Allows police to gain entry into incompetent
someone's premises to allow an Ordinary power Only valid as long as the principal
assessment under the MHA, or to of attorney is capable of acting for
return someone who is absent themselves and is not valid if
without leave from hospital they become incompetent
Section 136 Allows police to take someone with a Specific powers Gives the attorney-in-fact
mental disorder from a public place of attorney authority to act for a particular
to a place of safety if they think he/ purpose (e.g., to buy or sell a
she needs immediate care particular piece of property)
or control
General powers Gives the attorney-in-fact the
of attorney authority to do anything the
principal could do him or
17. e—Family members cannot legally veto herself
organ donation if the patient is on the organ Last will and When a person dies, the executor
donor register testament appointed in the person's last
will and testament takes control
Organ donation is possible posthumously when of the deceased person's
no prior consent has been given, if a person in a property and distributes it
according to the instructions in
“qualifying relationship” (ranked by the Human the will. If there is no will (or if the
Tissue Act 2004) consents to it. It is possible to will is invalid), each jurisdiction
carry an organ donor card, which will make it has intestacy legislation that
much easier to confirm consent to organ dona- distributes the deceased
person's property to his or her
tion if it is found on the person, but it is not nec- relatives according to a set
essary. According to the Human Tissue Act 2004, of rules
it is legal to preserve bodies after death, e.g., by
ventilating the patient, to continue perfusion to
the organs until consent for donation is estab-
lished. Competent minors can also consent to 20. a—Tarasoff decision
posthumous organ donation, and those with
parental responsibility should be informed of Physicians cannot disseminate confidential infor-
such a decision by the child. Although relatives mation about their patient without consent. This
can no longer legally veto consent to organ dona- principle applies to speaking with families,
tion, in reality, hospitals will respect the wishes of friends, the court, or other doctors (only commu-
relatives and organ donation is unlikely to pro- nication for the purpose of patient care is accept-
ceed if there is disagreement. able). Exceptions to confidentiality are generally
focused on preventing harm and include the
18. a—Discussion with a competent patient's following:
family member 1. Tarasoff decision: physician–patient confiden-
tiality must legally be breeched if the patient
The patient needs to give permission for their has threatened to harm another person. The
information to be disclosed to their family, healthcare provider should try to detain
although in some cultures this may not be the the patient, contact the police, and warn the
norm. All of the other options may require a potential victim.

Neurosurgery Books Full


www.ketabpezeshki.com 66485438-66485457
18 PROFESSIONALISM AND MEDICAL ETHICS 253

2. Child abuse/elder abuse. • A physician certifies the medical benefits


3. Dangerous driving: patients must be reported expected from the transfer outweigh the
to the Department of Motor Vehicles if they risks OR.
experience a seizure or otherwise present a • A patient makes a transfer request in writing
danger (e.g., visual loss). after being informed of the hospital's obliga-
4. Reportable diseases: many diseases must be tions under EMTALA and the risks of transfer.
reported to local authorities (and to the In addition, the transfer of unstable patients must
patient's partner in the case of STDs). be “appropriate” under the law, such that (1) the
5. Waiver: the patient may waive confidentiality transferring hospital must provide ongoing care
so discussions can be held with family mem- within its capability until transfer to minimize
bers or disclosures made to the insurance transfer risks, (2) provide copies of medical
company. records, (3) must confirm that the receiving facil-
ity has space and qualified personnel to treat the
21. a—Transfer of a stable patient for non- condition and has agreed to accept the transfer,
emergency care and (4) the transfer must be made with qualified
personnel and appropriate medical equipment.
Hospitals have three main obligations under
EMTALA: 22. e—Decision regarding tethered cord release
• Any individual who comes and requests must
receive a medical screening examination to Traditionally, US minors (under 18) have no
determine whether an emergency medical legal rights and remain under parental jurisdic-
condition exists. Examination and treatment tion until they reach the age of majority. Over
cannot be delayed to inquire about methods the past century, however, legislation has altered
of payment or insurance coverage. this, allowing minors to obtain treatment for spe-
• If an emergency medical condition exists, cific conditions without parental consent and, in
treatment must be provided until the emer- some states, make medical treatment decisions.
gency medical condition is resolved or stabi- In general, the need for parental consent should
lized. If the hospital does not have the nearly always be respected but in life- or limb-
capability to treat the emergency medical con- threatening emergencies, treatment should not
dition, an “appropriate” transfer of the patient be delayed despite parental objection. In urgent
to another hospital must be done in accor- situations, legal options can be pursued to make
dance with the EMTALA provisions. the child a ward of the court (e.g., parents cannot
• Hospitals with specialized capabilities are obli- refuse life-saving therapy for a minor with can-
gated to accept transfers from hospitals who cer). Established exceptions to the need for
lack the capability to treat unstable emergency parental consent in minors are:
medical conditions. 1. Emancipated minors: married, serving in the
• A hospital must report instances when it may military, self-supporting, or parents to
have inappropriately received an individual children.
who has been transferred in an unstable emer- 2. Reproductive health: sexually transmitted dis-
gency medical condition from another hospital. eases (STDs), birth control, prenatal care.
EMTALA governs how patients are transferred 3. Substance abuse treatment.
from one hospital to another. Under the law, a
patient is considered stable for transfer if the
treating physician determines that no material
deterioration will occur during the transfer
between facilities. EMTALA does not apply to
the transfer of stable patients; however, if the
patient is unstable, then the hospital may not
transfer the patient unless:

Neurosurgery Books Full


www.ketabpezeshki.com 66485438-66485457
254 PART II CARE OF THE NEUROSURGICAL PATIENT

EMI ANSWERS
23. 1—d, Beneficence; 2—j, Utilitarianism; 3—a, Autonomy

Autonomy Principle that all patients have the right to make their own informed medical decisions for
their own reasons. Competent patients have a nearly limitless ability to exercise autonomy
even if it means their own death or if it conflicts with their physician's personal ethical
principles
A positive right A right to be subjected to an action of another person or group (i.e., oblige action)
A negative right A right not to be subjected to an action of another person or group (i.e., oblige inaction)
Beneficence Physicians have a duty to do what is best for their patients
Categorical A rule that is true in all circumstances, which Kant phrased in two ways:
imperative 1. Always act in such a way that you would be willing for it to become general law for everyone
else to do the same as you in the same situation
2. People should always be treated as valuable in themselves, and not just be used in order to
achieve something else
Justice All people should be treated similarly regardless of age, race, or ability to pay. Medical
resources should be allocated fairly
Non-maleficence Physicians should weigh the relative risks and benefits of an intervention, acknowledging that
most treatments have inherent risk and that it may be better to do nothing at all
Virtue ethics Suggests that the right act is the action a virtuous person would do in the same
circumstances. The problem with this is that virtues may vary by culture and historical
context
Futility Futile care can be defined as the initiation or prolongation of “ineffective, pointless, or
hopeless” treatments

24. 1—d, Implied consent; 2—f, Invalid advance directive—AD must be made by a person with capac-
ity, be specific and not as a result of coercion; 3—b, Valid consent

25. 1—f, Non-voluntary euthanasia; 2—c, Murder; 3—b, Physician-assisted suicide; 4—e, Voluntary
euthanasia; 5—h, Doctrine of double effect

Suicide The act of trying to intentionally end one's own life


Physician-assisted Physician provides an individual with the information, guidance, and means to take his or
suicide her own life with the intention that they will be used for this purpose
Murder Where a person of sound mind unlawfully kills another human being, with the intent to kill
or cause grievous bodily harm
Attempted murder In contrast to the offence of murder, attempted murder requires the existence of an
intention to kill, not merely to cause grievous bodily harm
Voluntary The intentional killing by act or omission of a dependent human being for his or her alleged
euthanasia benefit at their request (usually to relieve pain or suffering)
Non-voluntary The intentional killing by act or omission of a dependent human being for his or her alleged
euthanasia benefit without them requesting or consenting to it
Involuntary The intentional killing by act or omission of a dependent human being for his or her alleged
euthanasia benefit but against their wishes
Doctrine of double View that it is permissible to cause a harm as a side effect (or “double effect”) of bringing
effect about a good result even though it would not be permissible to cause such a harm as a
direct means to bringing about the same good end

Neurosurgery Books Full


www.ketabpezeshki.com 66485438-66485457
18 PROFESSIONALISM AND MEDICAL ETHICS 255

26. 1—b, Aristotelian justice; 2—g, Libertarianism; 3—c, Communitarianism; 4—e, Deontology; 5—i,
Utilitarianism

Aristotelian justice When individuals receive benefits according to their merits, or virtue: those most virtuous
should receive more of whatever goods society is in a position to distribute
Hippocratic oath Articulates a commitment by the physician to do their best for the patient, cause no harm,
and not cause injustice to the patient
Utilitarianism Says that the best action is that which produces the greatest amount of happiness for the
individual, community, or entities concerned, or the best outcome for the largest number of
people. Utilitarians are concerned not about intentions or means, but the consequences of
adopting the choice
Libertarianism The rights of individuals to liberty, to acquire, keep, and exchange their holdings, and
considers the protection of individual rights the primary role for the state; moral view that
agents initially fully own themselves and have certain moral powers to acquire property
rights in external things
Communitarianism This is community-based moral theory in which values are determined by the community
and what is best for the community trumps what is best for an individual
Deontology Duty-based ethics which focus on the intrinsic rightness or wrongness of actions
themselves rather than their consequences. In this way, one should do the right thing even if
it produces more harm (or less good) than doing the wrong thing
Consequentialism Action-based ethics which say that at any point in time, the morally right action to take is the
one that will produce the best overall consequences. Different theories suggest what good
thing should be maximized: utilitarianism (human welfare) and hedonism (human
pleasure)
Principalism System of ethical analysis derived from four principles of common morality: autonomy
(free will or agency), beneficence (to do good), non-maleficence (not to harm), and justice
(fair distribution of benefits and burdens). Each is a prima facie principle, is equal to all the
others, and may override others in different situations, but all remain important in
considering execution of decision
Altruism States that one should choose the action that benefits other's well-being apart from oneself
Virtuism States that the right action is that which would be done by a virtuous person, i.e., the moral
character of the person performing the action is the most important thing in deciding if an
action is right

Neurosurgery Books Full


www.ketabpezeshki.com 66485438-66485457
CHAPTER 19

SURGICAL TECHNOLOGY
AND PRACTICE

SINGLE BEST ANSWER (SBA) QUESTIONS


1. You use a 2-0 coated vicryl suture to close the e. Middle cerebral artery (MCA) vasospasm
galea during closure of a craniotomy. Which is suggested by a Vmean of >180 cm/s
one of the following statements is correct?
a. It is a synthetic, braided absorbable 4. Which one of the following statements regard-
suture ing intraoperative ultrasound is most accurate?
b. It is a synthetic, braided non-absorbable a. It is highly useful in the brain but not spi-
suture nal cord due to anatomical constraints
c. It is a synthetic, monofilament absorbable b. It can aid in adjusting for intraoperative
suture brain shift and in detection of intraopera-
d. It is a natural, braided absorbable suture tive hemorrhage or hydrocephalus
e. It is a synthetic, monofilament non- c. Its utility in identification of critical tumor
absorbable suture margins during surgery and unintentional
residual has not been shown
2. During the resection of an intracranial tumor d. Contrast-enhanced intraoperative ultra-
your boss asks the scrub nurse to prepare the sound cannot be performed in those with
ultrasonic aspirator. Which one of the follow- a known allergy to iodinated contrast
ing statements are correct regarding this piece e. Intraoperative Doppler angiography can
of equipment? be used to improve accuracy of functional
a. Composed of an ultrasound generator, a mapping during eloquent tumor surgery
sucker, and an irrigator
b. Ultrasonic dissector/aspirator that is sele- 5. Which one of the following statements
ctive for tissue with high water and low regarding high frequency focused ultrasound
collagen content (HIFU) is LEAST accurate?
c. Fragmentation of solid tissues, relative a. It is a minimally invasive technique
sparing of vessels b. It utilized a piezoelectric ultrasound
d. It could induce false-negative results dur- transducer
ing motor brain mapping with direct elec- c. A single treatment volume can be >5 cm3
trical stimulation d. Tissue response is assessed by MRI at
e. Aspiration, ultrasonic dissection and irri- 6 weeks
gation properties can be controlled e. Intraprocedural ultrasound provides bet-
individually ter temperature mapping than MRI

3. Which one of the following statements 6. Which one of the following statements
regarding transcranial Doppler ultrasound regarding the WHO surgical safety checklist
is LEAST accurate? is most accurate?
a. Doppler effect is the apparent change in a. If there is a risk of >250 mL of blood loss it
frequency of a wave as the source moves must be formally discussed during the sign in
relative to the observer b. It is designed to reduce the number of sur-
b. Transcranial Doppler takes advantage of gical never events by increasing
reflection of sound waves off blood c. Surgical site should be marked during the
vessel walls time out
c. May be useful in distinguishing vasospasm d. VTE prophylaxis should be decided
from hyperemia before leaving the operating room
d. A Lindegaard ratio (LR) of >3 suggests e. Essential imaging must be displayed dur-
vasospasm ing the time out
e18
Neurosurgery Books Full
www.ketabpezeshki.com 66485438-66485457
19 SURGICAL TECHNOLOGY AND PRACTICE e19

7. The Leksell stereotactic frame is most accu- 11. Which of the labels in the picture of an
rately described as which one of the following operating microscope shown below is the
types of stereotactic system? beam splitter?
a. Polar coordinate
b. Arc-radius (arc-quadrant)
D
c. Burr-hole mounted
d. Arc-phantom target E
e. Cartesian coordinate F

8. Which one of the following statements


regarding frameless stereotaxy is most
G
accurate?
a. Does not require manual mapping of
coordinate space of preoperative image
onto surgical field
b. Point-pair registration is generally less A B C
reliable than point-pair registration
c. Accurate optical tracking requires the
patient’s head to be immobilized in a 12. Which one of the following statements
clamp regarding fluorescein sodium is LEAST
d. Surface-contour registration usually re- accurate?
quires fiducial markers a. It emits light in the 600-700 nm range
e. Electromagnetic frameless stereotaxy is b. It is preferentially taken up by neoplastic
not suitable in pediatric neurosurgery cells
c. It relies on breakdown of the blood-brain
9. Which one of the following statements barrier
regarding airflow systems in operating suits d. It can guide where laser scanning confocal
is most accurate? microscope should be focused
a. Laminar air flow has been consistently e. It can assess patency of EC-IC bypass and
shown to reduce surgical infections at risk vessels during aneurysm surgery
b. Positive pressure ventilation is usually
used in isolation rooms 13. Which one of the following statements
c. Negative pressure ventilation is usually regarding indocyanine green (ICG) is
used in operating suites LEAST accurate?
d. Bacterial contamination of surgical a. It emits light in the 700-850 nm range (far
wounds is most commonly from red/infrared)
shedding b. It has better cellular penetration than
e. Bacterial content of air in laminar flow 5-ALA
systems is lower than in conventional ven- c. It has higher levels of non-specific bind-
tilation systems ing in tumor surgery
d. It can be used for intraoperative cerebral
10. Which one of the following statements about angiography
operating microscope optics is most e. Tumor visualization is most specific
accurate? between 1 and 2 h post injection
a. The higher the magnification, the smaller
the width of field 14. Which one of the following statements
b. The higher the magnification, the smaller regarding 5-ALA is most accurate?
the depth of field a. It emits light in the 400-500 nm range
c. The higher the magnification, the smaller b. It is normally produced from protopory-
the working distance phyrin IX in the heme synthesis pathway
d. The higher the magnification, the smaller c. 5-ALA based fluorescence does not occur
the interpupillary distance in red blood cells
e. The higher the magnification, the higher d. Neoplastic cells preferentially take up 5-ALA
the focal length of the objective lens e. Photosensitivity is a recognized side effect

Neurosurgery Books Full


www.ketabpezeshki.com 66485438-66485457
e20 PART II CARE OF THE NEUROSURGICAL PATIENT

15. You are setting up the stereotactic frame in a. Assume that anyone born before 1st
the operating room. Your consultant asks January 1997 and was eating beef before
you to adjust the anterior-posterior measure- then could potentially have latent CJD
ment using the Vernier scale for submillime- b. Adult neurosurgery departments have had
ter accuracy. What reading is currently to keep a separate set of instruments only
shown in the example below? to be used on those at high risk of CJD
since 2006
Rule scale c. There is no effective way of decontami-
0 5 10 mm nating surgical instruments of CJD
d. Screening questionnaires should be con-
ducted in all adult patients to assess their
risk of having CJD
e. Single use instruments should be used
where possible in those patients with high
risk of CJD
0 5 10
19. In the UK, which one of the following groups
Vernier scale
would you not consider high risk of poten-
a. 3.0 mm tially carrying CJD?
b. 2.1 mm a. Individuals receiving blood components
c. 2.0 mm from a person who later developed variant
d. 3.1 mm CJD
e. 1.1 mm b. Individual who received human-derived
pituitary hormones in 1972
c. Individuals who underwent intradural
16. Which one of the following statements brain/spinal cord surgery or received
regarding image-guided neurosurgery in human-derived dural graft before August
pediatric patients is most accurate? 1992
a. Optical frameless stereotactic navigation d. Individuals who have had surgery using
is ideal for children with unfused skull instruments that had been used on some-
sutures one who was at high risk of CJD
b. Electromagnetic frameless stereotactic e. Individuals who have been identified as
navigation is more appropriate for neu- having received blood or blood compo-
roendoscopy in children nents from 300 or more donors since
c. Reference array for electromagnetic nav- January 2000
igation must be fixed to bone
d. Surface contour registration is not re- 20. Which one of the following LEAST accu-
quired in electromagnetic navigation rately describes how surgeons interact with
e. Electromagnetic navigation requires a clear currently available surgical robots?
visual line of sight between the tracker and a. Master-slave
the stylet b. Supervisory controlled
c. Telesurgical
17. Which one of the following statements reg- d. Shared-control
arding cranioplasty materials is LEAST e. Augmented reality
accurate?
a. Autologous bone flaps in children may 21. Which one of the following statements regard-
undergo resorption ing neuroendoscopy is LEAST accurate?
b. MMA use can cause an exothermic burn a. Image guidance systems are not currently
reaction compatible with neuroendoscopes
c. PEEK implants require 3D planning CT b. Generally do not provide stereoscopic
d. Titanium mesh is associated with a higher vision
infection rate c. Single anatomic corridor can result in
e. Ceramic plates are prone to shattering instrument clashing
d. Usually requires an assistant to hold scope
18. In the UK, which of the statements about if the primary surgeon wishes to bimanu-
neurosurgical management of patients at risk ally manipulate a lesion
of latent Creutzfeldt-Jacob Disease (CJD) is e. There is an operator learning curve
LEAST accurate?

Neurosurgery Books Full


www.ketabpezeshki.com 66485438-66485457
19 SURGICAL TECHNOLOGY AND PRACTICE e21

22. Which one of the following statements EXTENDED MATCHING ITEM (EMI)
regarding electrocautery and electrosurgery
are incorrect?
QUESTIONS
a. Flow of electrons produces current
b. Potential difference must exist for elec- 26. Hemostatic agents in neurosurgery:
trons to flow against resistance in a circuit a. Arista (Davol, CR Bard Inc, Rhode
c. Impedance is the resistance to the flow of Island, USA)
current b. Avitene (Davol, CR Bard Inc, Rhode
d. Heat generated is governed by Joule’s Island, USA)
first law c. Floseal (Baxter International Inc, West-
e. In the isolated circuit used in diathermy lake Village, CA, USA)
machines, loosening of the dispersive elec- d. Hydrogen peroxide
trode will result in thermoelectric burn e. Spongostan (Johnson & Johnson)
f. Surgicel (Ethicon, Johnson & Johnson)
23. Which one of the following statements g. Surgiflo (Ethicon, Johnson & Johnson)
regarding bipolar diathermy is most accurate? h. TachoSil (Takeda Nycomed AS, Baxter
a. Cutting setting uses a pure continuous International)
sine wave of low voltage
b. Coagulation setting uses a pure continu- For each of the following descriptions, select the
ous sine wave of high voltage most appropriate label shown above. Each answer
c. Dessication requires avoidance of tissue may be used once, more than once or not at all.
contact with the diathermy probe 1. Oxidized cellulose
d. Blended waveforms should be avoided 2. Microfibrillar collagen
e. Coagulation requires a waveform with a 3. Human derived thrombin and bovine
high duty cycle derived gelatin matrix

24. Which one of the following statements 27. Dural substitutes:


regarding complications of diathermy is most a. DuraGen (Integra Life Sciences, Plains-
accurate? boro, NJ, USA)
a. Surgical smoke does not contain any toxic b. DuraGuard (Synovis Surgical Innova-
substances tions, St Paul, MN, USA)
b. Electromagnetic interference of cardiac c. DuraMatrix (Stryker, Cambridge, MA,
pacemakers is higher with short, low USA)
power bursts of diathermy d. Durepair (TEI Biosciences, Medtronic
c. Capacitive coupling may cause thermo- Neurosurgery, MN, USA)
electric burns even in an isolated diathermy e. Durasis (Cook Biotech Inc, Bloomington,
circuit IN, USA)
d. Burns at tissue pedicles are due to the low f. Fascia lata
current density passing through the g. TachoSil (Takeda Nycomed AS, Baxter
narrow point International)
e. Pools of alcohol-based skin prep are not a h. TissuDura (Baxter International Inc,
fire risk Westlake Village, CA, USA)
i. Neuro-Patch (B. Braun, Melsungen,
25. Which one of the following dural sealants is Germany)
made from human fibrinogen and thrombin j. Pericranium
only?
a. DuraSeal (Confluent Surgical Inc, Wal- For each of the following descriptions, select
tham, MA, USA) the most appropriate label shown above. Each
b. Bioglue (Cryolife, Kennesaw, GA, USA) answer may be used once, more than once or
c. Tissucol (Baxter International Inc, West- not at all.
lake Village, CA, USA) 1. Human thrombin and fibrinogen with
d. Tisseel (Baxter International Inc, West- equine collagen
lake Village, CA, USA) 2. Bovine pericardium cross linked with
e. Evicel (Johnson & Johnson Wound Man- gluteraldehyde
agement, Ethicon Inc, Somerville, NJ,
USA)

Neurosurgery Books Full


www.ketabpezeshki.com 66485438-66485457
e22 PART II CARE OF THE NEUROSURGICAL PATIENT

SBA ANSWERS
1. a—It is a synthetic, braided absorbable suture a sucker, and an irrigator that, together, form an
ultrasonic dissector/aspirator that is selective for
Suture choice will depend on tensile strength tissue with high water and low collagen content.
required, local availability and surgeon prefer- Its usefulness and widespread use are attributable
ence. Size varies from 5 (largest; not the same to several advantages, comprising fragmentation
as 5-0) to 11-0 (smallest). Needle shape (e.g. of solid tissues, relative sparing of vessels, reduc-
straight, half curved, ½ circle) and point geometry tion of surgical time (by combining aspiration,
(e.g. cutting needle, taper, reverse cutting) also ultrasonic dissection, and irrigation in a single
vary. Natural sutures include silk and catgut, operation), improved safety (by adjusting set-
while the remainder are synthetic and have differ- tings) and supposed minimal trauma to surround-
ent structure (braided vs. monofilament) and ing neural brain structures. On the other hand,
absorption (absorbable vs. non-absorbable) prop- the use of ultrasonic dissection also has some dis-
erties. Absorbable suture breaks down over time advantages, primarily related to the inadequate
in the body. The amount of time it takes a suture and unknown control of its energy, which may
to break down in the body depends on a few fac- lead to sudden and sometimes unexpected events
tors such as suture type, size and the location it is (vascular injuries and/or neural tissue damages)
placed. Suture absorption rates can increase in and its possible interaction with intraoperative
patients with fever, infection or protein defi- brain mapping. In motor testing performed
ciency. Also, the strength to a suture will decrease through direct electrical stimulation a positive
significantly prior to its complete breakdown response is given by the active movement (or by
time. Therefore a suture with higher tissue ten- its recording), whereas in language testing a pos-
sion will have a greater chance of failure prior itive response is actually a negative event (e.g.
to the amount of time listed above. Braided suture inability to name objects). If CUSA use negatively
have a number of strands woven together like a interferes with electrical conduction in adjacent
string. It is thought that non-braided sutures tissue it could lead to the misdetection of a motor
cause less reactivity in the body and are not as site, reducing the sensitivity of the technique (if
prone to becoming infected because they lack CUSA and DES are used simultaneously) by
the grooves and rough surface for things to causing false-negative results and increasing the
adhere. However non-braided sutures can have risk of motor deficit. Equally, when the CUSA
a greater tendency to loosen at the surgical knot is used during intraoperative language testing,
with the lack of grip. through its negative interference with neuro-
logical function it could show true-positive
eloquent sites.

Braided/non- Silk, Ethibond


absorbable FURTHER READING
Carrabba G, Mandonnet E, Fava E, Capelle L, Gaini SM, Duf-
Braided/absorbable Coated Vicryl (4.5 weeks), fau H, Bello L. Transient inhibition of motor function induced
(breakdown time) Vicryl Rapide (2 weeks) by the Cavitron ultrasonic surgical aspirator during brain
mapping. Neurosurgery 2008;63(1):E178-9 [discussion E179].
Monfilament/non- Ethilon, Prolene
absorbable
3. b—Transcranial Doppler takes advantage of
Monofilament/ Undyed Monocryl reflection of sound waves off blood vessel
absorbable (3 weeks), Dyed Moncryl walls
(breakdown time) (4 weeks), PDS (9 weeks),
Catgut
Doppler effect (shift) is the apparent change in
frequency of a wave as the source moves towards
or away from the observer. According to this
2. d—It could induce false-negative results dur- principle, ultrasound waves emitted from the
ing motor brain mapping with direct electri- Doppler probe are transmitted through the skull
cal stimulation and reflected by moving red blood cells within the
intracerebral vessels. The difference in the fre-
CUSA is commonly used during surgical removal quency between the emitted and reflected waves
of gliomas, meningiomas, schwannomas, cra- (Doppler shift frequency) is directly proportional
niopharyngiomas, and spinal cord neoplasms. to their speed. Physiological variables affecting
Briefly, it is composed of an ultrasound generator, TCD measured mean blood velocity are age,

Neurosurgery Books Full


www.ketabpezeshki.com 66485438-66485457
19 SURGICAL TECHNOLOGY AND PRACTICE e23

gender, hematocrit, viscosity, carbon dioxide, MRI. Furthermore, newer technologies such as
temperature, blood pressure, and mental or contrast-enhanced US (i.e. microbubble con-
motor activity. Four main acoustic windows have trast), or the integration of US with navigation
been described: transtemporal, transorbital, sub- systems, or with functional navigation systems
mandibular and the suboccipital windows. as a method of assessing and adjusting for brain
Although each window has unique advantages shift may add additional benefits. In particular,
for different arteries and indications, a complete navigable 3D ultrasound (3D iUS) is a novel tool
TCD examination should include measurements that combines navigation technology with a ded-
from all four windows and the course of blood icated cranial insonation probe capable of gener-
flow at various depths within each major branch ating 2D/3D images that can be superimposed on
of the circle of Willis should be assessed. Specific preoperative MR imaging and repeatedly updated
arteries of the circle of Willis are identified using to provide a more accurate anatomic orientation,
the following criteria: relative direction of the as well as be used independently without preop-
probe within a specific acoustic window, direction erative imaging to improve gross total resection
of blood flow relative to the probe, depth of ultra- levels to those seen with 5-ALA and intraopera-
sound (insonation), and response to carotid com- tive MRI. intraoperative doppler-angiography
pression or vibration (when difficult to enabled the delineation of vascular anatomy in
differentiate anterior from posterior circulation). real-time. The specific limitations include con-
The LR, defined as the ratio between the time cerns regarding image quality and difficulties
mean average (Vmean) velocity of the MCA to with orientation of the 2D ultrasound plane,
ICA, is the most established of such ratios and learning curve, artifacts during resection may
helps differentiate hyperemia from vasospasm. compromise image accuracy, and in particular,
Hyperemia would result in flow elevations in hemorrhage and tissue handling have been shown
both the MCA and ICA and result in an to decrease the specificity and positive predictive
LR < 3, whereas vasospasm would preferentially value of iUS. Furthermore, there is no functional
elevate the MCA flow velocity over the ICA with information provided and electrophysiological
LR > 6. LR between 3 and 6 is a sign of mild VSP functional monitoring must still be considered
and >6 is an indication of severe VSP. TCD flow when dealing with lesions involving eloquent
velocity criteria appear to be most reliable for structures.
detecting angiographic MCA and basilar artery
vasospasm. Findings in MCA vasospasm include 5. a—It is a minimally invasive technique
MCA Vmean 180 cm/s, a sudden rise in MCA
Vmean by >65 cm/s or 20% increase within 24 h High-intensity focused ultrasound (HIFU) abla-
during days 3-7 post-subarachnoid hemorrhage, tion is a non-invasive technique that uses a beam
LR  6, and abrupt increase in Pulsatility index generated by a piezoelectric ultrasound transducer
>1.5 in two or more arteries suggesting increases propagates through tissue as a high-frequency
in ICP and/or vasospasm. TCD is most useful in (0.5-4 MHz) pressure wave. The beam is focused
monitoring the temporal course of angiographic on to the targeted tissue, and each treatment
vasospasm following SAH to help guide the timing volume is approximately 0.8  0.2  0.2 cm3. The
of diagnostic and therapeutic angiographic energy from the beam raises the temperature of
interventions. the focused area to 60-95 °C within a few seconds
without causing damage to the adjacent tissues,
FURTHER READING thereby leading to very localized protein denatur-
Purkayastha S, et al. Transcranial Doppler ultrasound: tech- ation and coagulative necrosis. The available
nique and application. Seminars Neurol 2012;32(4):411-20. HIFU devices are generally integrated with either
MRI or ultrasound imaging in order to plan treat-
4. b—It can aid in adjusting for intraoperative ment and monitor response in real time. MRI has
brain shift and in detection of intraoperative the advantage of excellent anatomical resolution,
hemorrhage or hydrocephalus high sensitivity for lesion detection but tempera-
ture mapping is challenging; ultrasonography
Intraoperative US offers real-time information offers real-time visualization of the targeted vol-
about the location, size, vascular relationships ume (compensates for movement) and guidance
and adjacent structures of brain and spinal cord of energy deposition within the treated area
lesions. This allows adjustment for brain shift, through a hyper-echogenic cross visible during
detection of untoward intraoperative events such pulse application. In addition, ultrasonography
as hemorrhage or hydrocephalus, critical tumor provides a rapid real-time assessment of the vol-
margins and residual tumor. It is easily available, ume of coagulative necrosis/temperature during
convenient, fast and easy to use and may provide treatment by visualization of a hyper-echogenic
a more practical alternative to intraoperative spot on the screen. HIFU ablation provides a

Neurosurgery Books Full


www.ketabpezeshki.com 66485438-66485457
e24 PART II CARE OF THE NEUROSURGICAL PATIENT

completely non-invasive therapy, thus avoiding 7. b—Arc-radius (arc-quadrant)


potential complications associated with general
anesthesia and surgery. It has been used for the Stereotactic frame-based guidance systems have
treatment of liver, kidney, prostate, brain, bone generally been based on four systems: polar coor-
and breast cancer. dinate, arc-radius, focal point, and phantom tar-
get. The polar coordinate system is based on
the necessity that guidance of the probe be
6. e—Essential imaging must be displayed described with regard to the angle relative to a
during the time out skull entry point. Usually, a minimum of two
angles (in planes orthogonal to each other) is
To assist operating teams to reduce the number required to characterize a unique trajectory; in
of patient safety events in the surgical environ- addition to these angles, the distance of the probe
ment, a core set of standards have been identified to the target lesion needs to be calculated (e.g.
by the WHO that can be applied universally Spiegel-Wycis, Horsley-Clarke). The arc-radius
within any healthcare setting to address issues system (e.g. Leksell frame) is based on the con-
including correct site surgery, hemorrhage risk, cept that a probe equivalent to the radius of a
antibiotic prophylaxis, airway management and semicircular arc will reach the center of the arc
the risk of allergies. WHO surgical safety check- when introduced perpendicular to any point
list is designed to reduce the number of errors and along the arc. Adjustment of the arc (vertically
complications resulting from surgical procedures and anterior-posterior) permits targeting of dif-
by improving team communication and by verify- ferent points; the base of the arc is attached to a
ing and checking essential care interventions. It rigid head frame on the patient’s head which
consists of sign in, time out and sign out stage defines the corners of a cube visible on CT. After
assessments below: the geometric center of the cube is defined, the
relative location of the target can be calculated
and the semicircular arc manipulated accordingly
to place that lesion at the center of the arc. Arc-
WHO Surgical Safety Checklist phantom systems (e.g. CRW frame) allow
mounting of the arc onto a phantom base frame
Sign Out to check the accuracy of the trajectory immedi-
(Before ately before transferring it to the patient. A burr
Sign In (Before Time Out Leaving
Induction of (Before Start Operating hole-mounted system provides a limited range of
Anesthesia) of Surgery) Room) possible intracranial target points with a fixed
entry point (usually over non-eloquent brain):
Patient Team Name of
confirms introduction procedure
provided two angular degrees of freedom (hori-
identity, site, Surgeon/ recorded zontal plane, vertical plane) and a depth adjust-
procedure, anesthetist/ Instrument/ ment. Cartesian geometry is the underpinning
consent OPD confirm swab/sharp for all frame types, and describes the location of
Surgical site identity, site, count correct? a point in space with three coordinates (x, y, z).
marked procedure, Specimens
Anesthetic consent labeled
equipment and Anticipated Equipment FURTHER READING
medication critical events problems
Olivi A, et al. Frame and frameless stereotactic brain biopsy.
Allergies Anesthetic identified
Difficult airway/ concerns Key concerns In: Winn HR (Ed.), Youman’s Neurological Surgery,
aspiration risk Instrument/ for recovery 4-Volume Set, 6th ed., Elsevier, Saunders, 2011.
Risk of >500 mL equipment and
blood loss? concerns management 8. c—Accurate optical tracking requires the
Surgical site patient’s head to be immobilized in a clamp
infection
- antibiotic
prophylaxis For both frame-based and frameless systems the
- hair removal relationship between the coordinate space for
- patient the preoperative images and that for the surgical
warming field must be established. In frame-based systems,
- glycemic
control
by using the same frame for both preoperative
VTE image acquisition and performing the surgery,
prophylaxis the relationship between the two coordinate sys-
Essential tems is known and no further transformation is
imaging required. For frameless systems, point-pair regis-
displayed
tration or surface contour registration can be used

Neurosurgery Books Full


www.ketabpezeshki.com 66485438-66485457
19 SURGICAL TECHNOLOGY AND PRACTICE e25

to establish the relationship between the pre- When a recirculating system is used, the air
operative images and that for the surgical field. return duct should have a high efficiency partic-
Point-pair matching requires a set of at least three ulate air (HEPA) filter built into the system. In
non-colinear points to be defined in the coordi- an OR where inhalational anesthetics are used,
nates of the images. These ‘fiducial points’ can there should be separate systems for ventilation,
consist of either natural anatomic landmarks vacuum (patient and surgical suction), and waste
(i.e. nasion, lateral canthus, or tragus), skin- anesthetic gas disposal. Studies have demon-
applied markers, or bone-implanted markers. strated that most of the causes of wound contam-
These same fiducial points are defined again ination in the operating suite are the result of the
within the coordinate system of the surgical field patient’s skin flora and bacteria shed on airborne
in the operating room. Software is then used to particles from the personnel. Room ventilation
establish the relationship between the coordi- affects the distribution of these airborne particles
nates of the fiducial points in the image space in four ways: total ventilation (dilution), air distri-
and their counterpart in the surgical space. bution (directional airflow), room pressurization
In surface contour registration, mapping a (infiltration barrier), and filtration (contaminant
radiographic surface can be achieved by touching removal). As the air flows of the room increases,
a number of multiple random points (termed the greater the dilutional effect on airborne par-
cloud of points) or by scanning a surface with ticles. However, resultant turbulent flow also
laser registration. These surface-based algo- increases microbial distribution throughout the
rithms allow the use of imaging obtained before room. Low-velocity unidirectional (laminar) flow
the intent to operate, but are less accurate com- minimizes the spread of microbes in the room.
pared to the combination of natural landmarks Directional flow can be from the outside into
and skin-applied fiducial markers. Once the the OR (negative pressure), or from the OR to
spatial relationship is established with either the outside (positive pressure). Negative pressure
point-pair registration or surface contour regis- ventilation is used for highly infective rooms in
tration, it is used throughout the remainder of the hospital (e.g. isolation rooms for tuberculosis
the operation to map anatomic targets of the patients) and positive pressure ventilation is used
patient to that of the preoperative images. This for protective environments (e.g. ORs and rooms
thus requires that the patient's head be immobi- with immunocompromised patients). This is
lized with a head clamp and the operating table described as an entire body of air within a desig-
not be moved during surgery. To overcome the nated space (theatre suite) moving with uniform
error that may be induced with patient or operat- velocity in a single direction along parallel
ing table movement, tracking units attached to flow lines. True laminar flow is only achieved
the patient's head or a surgical head clamp can when approximately 100% HEPA filter coverage
be used. Electromagnetic (rather than optical) occurs. Laminar flow ventilation comprises a
tracking is the latest development in frameless continuous flow of air with bacteria less than 10
stereotaxy and is used in pediatric neurosurgery colony-forming units/m3. Despite this, infection
as it avoids the need for head clamping (not pos- rates for joint replacement surgery have actually
sible in those without fused skull sutures). been show to increase in laminar flow versus con-
ventional theatre ventilation (reasons remain
9. e—Bacterial content of air in laminar flow unclear).
systems is lower than in conventional ventila-
tion systems 10. a—The higher the magnification, the smaller
the width of field
The goals of the ventilation system are comfort of
patient and staff, removal of pollutants/aerosols, Resolving power of an optical system is its ability
temperature and humidity control, and control to make clear and distinguishable two separate
air movement to minimize the transfer of air- entities. The resolving power of the unaided
borne contaminants from less clean to clean areas. human eye is only 0.2 mm. Most people who view
Airborne contaminants may enter an operating two points closer than 0.2 mm will see only 1
room via the following routes: (a) through the point. Moving closer to an object increases
supply air; (b) shed by operating staff; (c) through resolution up to a point, but objects closer than
surgical activities; and (d) transferred from adja- 10-12 cm go out of focus. Optical aids (e.g.
cent spaces. A recirculating system is one that loupes, OMs, surgical headlamps, fiberoptic
recirculates some or all of the inside air back to handpiece lights) can improve resolution by many
the OR suites or some other part of hospital, orders of magnitude. For example, a common
whereas in a non-recirculating system, all air OM can raise the resolving limit from 0.2 to
brought to the room is conditioned, outside air. 0.006 mm (6 μm), a dramatic improvement.

Neurosurgery Books Full


www.ketabpezeshki.com 66485438-66485457
e26 PART II CARE OF THE NEUROSURGICAL PATIENT

Stereopsis, or 3D perception, is critical to achiev- • Eyepieces—magnify the interim image


ing precision surgery, and is an advantage over generated in the binocular tubes. Eyepiece
2D endoscopes. Several factors are important selection not only determines the magnifi-
for increasing resolution without compromising cation, but also the size of the field of view.
ergonomics, eyestrain, head/neck fatigue: • Binoculars—the precise adjustment of the
interpupillary distance (by adjusting the dis-
tance between binocular tubes holding the
Microscope Parameters eyepieces) is the basic pre-requisite for the
stereoscopic view of the operation area.
Focal Distance between eye/lens and Longer the focal length of binoculars, the
length surface being manipulated. As focal greater the magnification and narrower
length decreases, the eyes must the field of view. Many microscopes now
converge, creating eyestrain and
also reduces the depth of field. The include a beam splitter and a second set of
working distance is the distance teaching binoculars (non-stereoscopic as
from the microscope objective lens they split light from a single objective).
to the point of focus of the optical • Magnification changer (binocular objec-
system. This value is fixed and is
dependent on the chosen focal
tives)—one cylinder, into which two
length of the objective lens Galilean telescope systems with various
magnification factors are built. The combi-
Depth of the far-near range within which nation of the magnification changer with
field objects remains in focus without
having to adjust working distance.
varying objective lenses and eyepiece yields
As magnification increases, depth of an increasing magnification line when the
field decreases hence focusing on control is adjusted.
multiple objects at different depths • Objective lens—its focal length determines
with high magnification will require the working distance between the lens and
more changes in microscope
position. Also, the smaller the field of the surgical field.
view, the shallower the depth of field • Lighting unit as optimal illumination is
necessary with high magnifications. Surgi-
Width of the width and height of the area cal microscope uses coaxial fiber-optic illu-
field visible through a lens system, which
decreases with increasing
mination producing an adjustable, bright,
magnification uniformly illuminated, shadow-free, circu-
lar spot of light that is parallel to the optical
Illumination By increasing light levels, one can viewing axis.
increase apparent resolution (the
ability to distinguish two objects close Image with permission from Cordero I. Understanding
to each other as separate and distinct). and caring for an operative microscope. Commun Eye
Light intensity is determined by the Health 2014;27(85):17.
inverse square law, which states that
the amount of light received from a 12. a—It emits light in the 600-700 nm range
source is inversely proportional to the
square of the distance
Fluorescein sodium has an excitation maximum of
494 nm and an emission maximum of 521 nm. It
readily crosses capillaries, provides fluorescent
11. g—A ¼ Light fiber; B ¼ objective lens; contrast in the extracellular matrix, and has a urine
C ¼ magnification changer; D ¼ teaching clearance of 24-36 h after intravenous injection.
binoculars; E ¼ eyepiece; F ¼ main binocu- The amount of fluorescein delivered to a tumor
lars; G ¼ beam splitter site is increased by the breakdown of the normal
blood-brain barrier. Fluorescein has been used
The surgical microscope is a complicated system in cerebral angiography to detect arteriovenous
of lenses that allows stereoscopic vision at a malformations, assess superficial temporal artery-
magnification of approximately 4-40  with an MCA anastomoses patency, and to aid in treating
excellent illumination of the working area. The cerebral aneurysms. Fluorescein is not a tumor-
separation of the reflected light into two beams specific agent, but it is excellent for visualizing
within a microscope is what produces the stereo- regions of compromised neurovasculature. The
scopic effect that allows the clinician to see depth presumption in its use for neuro-oncology is
of field. The light beams fall parallel onto the ret- that these areas correspond to the enhancing
inas of the observer so that no eye convergence is regions, which also correspond to bulk tumor.
necessary and the demand on the lateral rectus Rates of 100% GTR have been achieved for
muscles is minimal. The optical unit consists of: non-eloquent tumors using microscope was

Neurosurgery Books Full


www.ketabpezeshki.com 66485438-66485457
19 SURGICAL TECHNOLOGY AND PRACTICE e27

developed equipped with dichroic mirrors specific overcome these limitations. In contrast, macro-
for fluorescein-guided resection of malignant glio- scopic detection is more subjective to the surgeon.
mas. Fluorescein-guided laser scanning confocal Intravenously administered ICG is immediately
microscopy in humans had been able to provide localized to a tumor site, and the fluorescent signal
assessment of tumor grade, tumor histology, and remains in the tumor up to 1 h after injection, with
tumor margins for a variety of tumor subtypes. constant imaging, demonstrating limited photo-
The results from these two studies demonstrate bleaching and clearance. The delivery of the dye
the feasibility of this technology as a diagnostic to the site of the tumor relies on binding to serum
and therapeutic tool, as it can help identify many proteins and the damaged vasculature primarily
of the pathognomonic cytoarchitectural features located at the site of the tumor. However, with
of various brain tumors and aid in the intraopera- time the dye will diffuse into surrounding tissue.
tive diagnosis and resection of various central ner- These properties allow real-time, in vivo assess-
vous system tumors. Fluorescein in this case can ment that differentiates glioma tissue and normal
both guide the operator grossly to areas to be brain. In contrast to visible wavelength fluoro-
viewed with LSCE and provide the contrast to phores, ICG provides visualization of deeper tissue
allow cellular visualization. Fluorescein is FDA structures due to its infrared excitation and emis-
approved and is widely used in the field of ophthal- sion spectra. However, depth of imaging still
mology as well as in GI studies. Most fluorescein remains limited to a few hundred microns deep
clinical studies in neurosurgery report no serious to the imaging surface. ICG is considered to have
adverse effects with use of the fluorophore. fewer risks than other FDA approved intravenous
fluorophores.
FURTHER READING
Zehri AH, et al. Neurosurgical confocal endomicroscopy: a
review of contrast agents, confocal systems and future imaging FURTHER READING
modalities. Surg Neurol Int 2014;5:60. Zehri AH, et al. Neurosurgical confocal endomicroscopy: a
review of contrast agents, confocal systems and future imaging
13. e—Tumor visualization becomes most spe- modalities. Surg Neurol Int 2014;5:60.
cific between 1 and 2 h post injection
14. c—5-ALA based fluorescence does not occur
ICG is a near-infrared fluorescent agent with in red blood cells
maximal excitation at 778 nm and emission
Produced in the mitochondria, 5-ALA is a natural
spectra range of 700-850 nm in serum. It is fairly
precursor for the production of protoporphyrin
water-soluble, which allows it to be given intrave-
nously and cleared through renal and bile excre- IX (PpIX) in the heme synthesis pathway found
in all cells. PpIX is a fluorescent molecule that
tion. ICG concentration within a tumor site is
binds membrane lipids and has an excitation
enhanced by breakdown of the normal blood-
brain barrier. ICG has a greater tissue penetrance range of 375-440 nm and emission range of
640-710 nm in vivo. Overloading this pathway
than visible-wavelength fluorophores, such as 5-
with exogenous 5-ALA causes the collection of
ALA and fluorescein. ICG is an anionic, amphi-
PpIX to fluorescently detectable levels in cells.
philic, tricarbocyanine probe, which allows it to
As the production of PpIX occurs in situ in mito-
have a high affinity for proteins, such as albumin,
chondria, fluorescence is limited to cells.
and allows visualization of solid tumors, but may
This decreases fluorescent signal in blood or
also cause higher levels of nonspecific binding.
edematous regions of the operative field. Further-
Similar to fluorescein, ICG provides nonspecific
more, neoplastic cells demonstrate preferential
contrast in areas of permeable neurovasculature.
uptake of exogenous 5-ALA and increased collec-
ICG has been given intravenously for blood vessel
angiography, identifying extrahepatic bile ducts tion of PpIX, making the fluorescent signal more
robust in these abnormal tissues and making
and detecting liver metastases. It can also be given
intraoperative visualization feasible, especially in
subcutaneously for sentinel lymph node mapping
intracranial tumors of higher grades. Photo-
for breast, anal, and GI cancer as well as assessing
bleaching occurs with PpIX fluorescence level
lymphatic drainage for lymphedema. ICG was able
dropping to 36% after 25 min under violet light
to contrast the fluorescent glioma tumor tissue
or 87 min under white light. It can be limited
within 1 mm of the histological tumor margins
by allowing excitation and white light to pene-
in an animal model. However, this technique is
trate as small an area as possible in the surgical
not effective in distinguishing between malignant
field. Phototoxicity is limited with 5-ALA, since
cells and other areas of the brain that may inciden-
tally uptake the injected dye. Cellular visualization fluorescent microscopes do not produce suffi-
cient energy for significant ROS production.
using ICG contrast with an infrared LSCE may
Currently, the FDA has approved 5-ALA and

Neurosurgery Books Full


www.ketabpezeshki.com 66485438-66485457
e28 PART II CARE OF THE NEUROSURGICAL PATIENT

its derivatives for research diagnostic applications FURTHER READING


in endoscopic, photodynamic detection of blad- Warren BF, Davies JD. Pierre Vernier's invention: a neglected
der cancer and residual glioma, as well as the tool of our trade. Histopathology 1991;18(4):361-2.
treatment of basal cell carcinoma and actinic
keratosis. 5-ALA macrofluorescence predicts 16. e—Electromagnetic frameless stereotactic
malignant glioma tissue with 90% accuracy, navigation is more appropriate for neuroen-
showing residual tumor better than post-op doscopy in children
MRI, and the level of residual solid tissue fluores-
cence was shown to correlate with patient sur- Both frame-based and frameless optical neurona-
vival. In an RCT, its use improved gross total vigation have been limited in their use in young
resection rates and overall progression-free pediatric patients, largely due to the hazards of
survival time. In low-grade glioma, macroscopi- fixing young patients’ heads in pinned cranial fix-
cally detectable fluorescence does not occur but ation. The recent development of electromag-
hand held intraoperative confocal microscope netic neuronavigation techniques has led to the
allowed visualization of 5-ALA-induced fluores- greater use of stereotactic navigation in patients
cence of low-grade glioma tumor and tumor in whom cranial fixation would not be tolerated.
margins in vivo that corresponded with standard Endoscopic surgery in pediatric patients is well
histopathology. Some controversy still exists as to described for a variety of intracranial pathologies.
the biology of 5-ALA metabolism in low-grade The use of stereotactic navigation as an adjunct to
gliomas and the need for complete resection by neuroendoscopy is similarly a recognized and
MRI. Though 5-ALA administration is considered growing technique. The improved accuracy and
safe, minor systemic side effects have been reported, safety afforded the surgeon by stereotaxy has
including nausea, vomiting, and hypotension as allowed the development of neuroendoscopic
well as increased sensitivity to sunlight up to 48 h techniques for biopsy or resection of intraventric-
after administration. As with other fluorophores, ular and periventricular lesions, the fenestration
these side effects are limited with lower dosing. of cysts, cerebrospinal fluid diversion procedures
and the placement of catheters, even in the pres-
ence of abnormal or difficult anatomy. However,
FURTHER READING due to the restrictions imposed by the risks of
Zehri AH, et al. Neurosurgical confocal endomicroscopy: a
rigid pinned cranial fixation, these techniques
review of contrast agents, confocal systems and future imaging
modalities. Surg Neurol Int 2014;5:60. were not always suitable for very young patients.
The equipment required for the procedures com-
15. b—2.1 mm prised the StealthStation AxiEM Neuronaviga-
tion System (Medtronic, Louisville, CO, USA),
The top scale is a simple rule scale in cm and mm, which utilizes a coil to generate an electromag-
the other is a Vernier scale with ten divisions. netic field around the patient’s head. Navigation
Whilst one of these scales is fixed, the other is able stylets within this field are accurately localized
to move with the stage. The Vernier scale’s ten divi- with reference to a small array, which is secured
sions are equal to nine small divisions on the rule to the patient’s head (either adhesive-based scalp
scale. Therefore each Vernier division is equal to or fixed to the cranial bone). Previous optical
0.9 mm. The difference between a rule scale divi- technology necessitates the immobilization of
sion and a Vernier division is thus: 1  9/10 ¼ the patient’s head with rigid cranial fixation in
0.1 mm. Hence, the instrument can be read to an pins and a visible reference frame with an unim-
accuracy of approximately 0.1 mm. To take a peded line of sight and utilized probes that attach
reading, note the division on the rule scale immedi- to the endoscope, and these can hinder the free
ately prior to the 0 of the Vernier scale (this will manipulation of the instrument by the surgeon.
provide your first decimal place), then look along We found that the electromagnetic stereotactic
the Vernier scale until you find a mark on it system afforded the desired degree of naviga-
which is in line with a mark on the rule scale-this tional accuracy with a number of advantages over
provides your second decimal place. As such, the optical systems. The navigation stylets were less
reading is 2.1 mm in the example. Vernier scales cumbersome than the equivalent optical probes,
are seen on stereotactic frame axes and inaccuracies and the placement of the navigation stylets within
in set up can significantly alter targeting in the channel of the endoscope did not affect the
3D space. system’s accuracy.

Figure adapted from Warren BF, Davies JD. Pierre 17. d—Titanium mesh is associated with a
Vernier's invention: a neglected tool of our trade. higher infection rate
Histopathology 1991;18(4):361-2.

Neurosurgery Books Full


www.ketabpezeshki.com 66485438-66485457
19 SURGICAL TECHNOLOGY AND PRACTICE e29

Analysis of the Advantages and Disadvantages of Common Cranioplasty Materials


Material Advantages Disadvantages
Autologous bone Accepted by host, low rate of fracture Bone resorption, infection

MMA Strong, heat resistant, inert, low cost, Infection, fracture, exothermic burn
ease of use reaction, inflammation, lack of incorporation

Hydroxyapatite Noninflammatory, decent chemical bonding Low tensile strength, brittle, infection,
to bone, excellent cosmesis and contouring fragmentation, lack of osteointegration
ability

Titanium mesh Noninflammatory, noncorrosive, strong, Expensive, image artifact on imaging


malleability, low infection rate, good cosmesis

Alumina Hard, chemically stable, tissue compatible, Expensive, prone to shatter


ceramics low infection rate

PEEK implant Radiolucent, chemically inert, strong, elastic, Cost, need for additional 3D planning and
does not create artifacts on imaging, imaging, difficult to bond to other materials,
comfortable, does not conduct temperature infection

Table with permission from Shah A, et al. Materials used in cranioplasty: a history and analysis. Neurosurg Focus
2014;36(4):E19.

18. b—Adult neurosurgery departments have assumption that effective methods for
had to keep a separate set of instruments only removing CJD infectivity from instruments
to be used on those at high risk of CJD was likely to be available within 5 years.
since 2006 However, this has not occurred and the
patients born in 1997 are now coming to
Two main issues dominate the management of the adult neurosurgical centers which will
CJD in neurosurgical patients in the UK: now have to keep two separate pools of
1. Implementation of CJD screening ques- instruments or risk them being potentially
tions and quarantining of neurosurgical being infected with the vCJD agent. As such,
instruments. These questions were some are now arguing that the costs are dis-
designed to reduce the risk of spread of spo- proportionate to the risks (which have been
radic CJD, from patients who suffer from revised down) and guidance should change.
the disease, have received a human dural
graft or those that have had human derived FURTHER READING
hormone treatments, as well as individuals NICE guidance IPG196 (2006).
who have had massive blood transfusions
from multiple donors. 19. e—Individuals who have been identified
2. Since it has been causally proven that eating as having received blood or blood compo-
beef with bovine spongiform encephalopa- nents from 300 or more donors since
thy (BSE) can cause vCJD in humans, the January 2000
assumption has been that anyone who has
had beef in the 1980s can be harboring latent CJD is a progressive, fatal neurological disease that
vCJD. Since BSE has been eliminated in belongs to a wider group of neurodegenerative dis-
the food chain by 1997, patients born after orders known as transmissible spongiform
1997 are thought to have no risk of develop- encephalopathies (TSEs) or prion diseases. Prions
ing vCJD. NICE have therefore recom- are infectious particles composed of abnormally
mended a new set of instruments be used folded forms of the prion protein that are thought
on these patients, lest they contract the dis- to cause TSEs, including CJD. They resist com-
ease from neurosurgical instruments used on plete inactivation by conventional hospital decon-
older patients who may have been harboring tamination techniques. Individuals undergoing
vCJD. The requirement to have (i) a whole surgery may therefore be infected by prion-
set of instruments to be kept completely contaminated instruments previously used on
separate and used on a specific cohort of patients with CJD. Sporadic CJD (85-90% of
patients presents huge financial and cases) is of unknown etiology with a worldwide
logistical challenges (e.g. tracking systems, annual incidence of about 1 in 1 million people.
single use instruments in high risk patients). Inherited CJD (10-15% of cases) is associated with
This was temporary advice based on the coding mutations, insertions or deletions in the

Neurosurgery Books Full


www.ketabpezeshki.com 66485438-66485457
e30 PART II CARE OF THE NEUROSURGICAL PATIENT

prion protein gene. Iatrogenic CJD (less than 1% 20. e—Augmented reality
of cases) arises from accidental exposure to human
prions through surgical or medical procedures. A Surgical robots can be broadly classified into
novel form of human prion disease, variant CJD, three categories on the basis of how surgeons
was first recognized in the UK in 1996 and is interact with them: supervisory-controlled robot
believed to result from consumption of food systems in which the surgeon plans the operation,
derived from cattle with BSE. There have been and the robot then carries it out autonomously
seven cases of iatrogenic transmission of (pre- under the supervision of the surgeon; telesurgical
sumed sporadic) CJD via contaminated neurosur- (master-slave) systems in which the surgeon
gical instruments or intracerebral electrodes. Five (master) remotely controls the robots actions
cases resulted from neurosurgical instruments: (slave); and handheld shared-controlled systems
four in the UK and one in France. All of the in which the surgeon and robot share control of
UK cases occurred over 30 years ago. the instrument. Augmented reality systems refer
Increased risk of CJD to fusion of virtual 3D brain models and the actual
1. Recipients of hormone derived from operating field, may enhance the operating room
human pituitary glands, e.g. growth hor- workflow and improve safety by eliminating the
mone, gonadotrophin, are “at increased need for surgeons to repeatedly interrupt opera-
risk” of transmission of sporadic CJD. In tions and look away from the surgical field at
the UK the use of human-derived gonado- image guidance. Such systems are already com-
trophin was discontinued in 1973, and use mercially available for use with surgical micro-
of cadaver-derived human growth hor- scopes, but have not yet been widely applied to
mone was banned in 1985. However, use neuroendoscopy.
of human-derived products may have con-
tinued in other countries after these dates. FURTHER READING
2. Individuals who underwent intradural brain Marcus HJ, Seneci CA, Payne CJ, et al. Robotics in keyhole
or intradural spinal surgery before August transcranial endoscope-assisted microscopic surgery: a critical
review of existing systems and proposed specifications for new
1992 who received (or might have received)
robotic platforms. Neurosurgery 2014;10(Suppl. 1):84-95
a graft of human-derived dura mater are “at [discussion 95-6].
increased risk” of transmission of sporadic
CJD (unless evidence can be provided that
21. a—Image guidance systems are not currently
human-derived dura mater was not used).
compatible with neuroendoscopes
3. Individuals who have had surgery using
instruments that had been used on someone In minimally invasive neurosurgical procedures,
who went on to develop CJD, or was “at the relative paucity of anatomic landmarks often
increased risk” of CJD; necessitates more frequent use of image guidance,
4. Individuals who have been identified as which may have a considerable impact on surgical
having received blood or blood compo- performance. Second, while endoscopes do
nents from 300 or more donors since provide an extended viewing angle compared with
January 1990. operating microscopes, some deliver lower-quality
5. Individuals who have given blood to some- imaging, and most lack stereoscopy, limiting the
one who went on to develop vCJD. appreciation of complex spatial relationships within
6. Individuals who have received blood from the brain. Third, the use of endoscopes makes
someone who has also given blood to a bimanual manipulation difficult or impossible.
patient who went on to develop vCJD, Within the brain, which is incompatible with gas
7. Individuals who have been treated with cer- insufflation, debris quickly clouds the endoscopic
tain implicated UK sourced plasma prod- field unless a sucker is concurrently used; a single
ucts between 1990 and 2001 surgeon can therefore not easily view and manipu-
Increased risk of vCJD late tissue simultaneously. Although an additional
Individuals who have received labile blood surgeon may assist, the use of a single anatomic
components (whole blood, red cells, white cells corridor makes it difficult to do so without the
or platelets) from a donor who later went on to operating surgeons obstructing each other or their
develop vCJD. instruments clashing. Instrument holders have
Individuals who have received an organ or tis- recently been developed but can again lead to
sue from a donor infected with CJD or “at crowding of instruments, and inevitably interrupt
increased risk” of CJD. the operative workflow, because they must be
repeatedly repositioned. Moreover, even when
FURTHER READING specially designed tube-shaft instruments are
NICE guidance IPG196 (2006).
used, manipulation through uniportal keyhole

Neurosurgery Books Full


www.ketabpezeshki.com 66485438-66485457
19 SURGICAL TECHNOLOGY AND PRACTICE e31

neurosurgical approaches is almost coaxial, to the electrode. The current density is greatest at the
major detriment of surgical dexterity. Advances in active electrode and heating occurs here. In bipo-
augmented reality, intraoperative imaging, stereo- lar mode the surgical tool is a pair of forceps
endoscopy (3D HD), and bimanual manipulation/ where one blade represents the active electrode
jointed wrist instruments will be required to and the other the return electrode. The current
overcome these issues. flows at high current densities from one tip to
the other via the bite of tissue between, rather
FURTHER READING than through the body; this eliminates the need
Marcus HJ, Seneci CA, Payne CJ, et al. Robotics in keyhole for a dispersive pad. Modern devices are capable
transcranial endoscope-assisted microscopic surgery: a critical of sealing vessels up to 7 mm in diameter by a
review of existing systems and proposed specifications for new combination of mechanical pressure and dia-
robotic platforms. Neurosurgery 2014;10(Suppl. 1):84-95 [dis-
thermy. Microprocessors use tissue response gen-
cussion 95-6].
erators to adjust current and voltage based on the
22. e—In the isolated circuit used in diathermy sensed tissue impedance, so vessels can be reliably
machines, loosening of the dispersive elec- sealed to withstand three times normal systolic
trode will result in thermoelectric burn pressure. The surgeon has the ability to alter
the power setting, the waveform, the diathermy
The source voltage is created by the diathermy tool and how it is applied:
generator, which converts mains 50 Hz current
into a high-frequency current of 0.2-3 MHz. At
these radiofrequencies there is minimal risk of Diathermy Settings
muscle or nerve stimulation. Instead, electrical
energy is converted to heat as the electrons over- Cutting Using a pure continuous sine wave
come the impedance of the body. The heat gene- of low voltage rapidly produces high
rated is governed by Joule’s first law: Q (heat in temperatures that vaporize tissue
joules) ¼ I2 (current density)  R (resistance)  t fluid causing cells to explode
forming a gap in the tissues. The
(time). Current density is the current per unit electrode need not be in contact with
cross-sectional area. If the current is concentrated tissue, as an arc is formed producing
at a single point such as the active electrode (e.g. a clean cut. With the coagulation
tip of monopolar), the effect will be greater than if setting on high power, cutting also
occurs, but this produces a larger
spread out over a large area (the dispersive elec- zone of greater thermal damage
trode plate). Tissue temperature at the active
electrode tip is 1000°C, but 1 cm away from the Coagulation Using an intermittent sine wave with
tip it only reaches 38°C. For a given voltage the a short ‘on’ time and a longer ‘off’
current is the same, but if the area through which time (low duty cycle), and a higher
voltage—the low rate of heat rise
the current flow falls, resistance increases and and produces a coagulation effect
more heat is produced. Historically, diathermy
used mains voltage referenced to earth and the Fulguration Coagulation voltage is high enough
dispersive electrode was earthed. If this pad to arc causing a zone of thermal
damage around the vaporized tissue
became loose, was incorrectly applied, or the cur-
rent found an alternative path to earth (e.g. via a Dessication Requires the probe to be in contact
limb in contact with the table), burns would result with the tissue but using heat
but modern diathermy generators use an isolated insufficient for cutting. This dries out
the tissue and a coagulum is formed
circuit (loosening of dispersive electrode would
break the circuit). Blend It is also possible to use a blended
waveform. A separate waveform
FURTHER READING with higher duty cycles giving more
Boyd DE, Palmer JM. Surgical diathermy. Anaesthesia Inten- cut and less coagulation
sive Care Med 2010;11(11):461-3.

23. a—Cutting setting uses a pure continuous


sine wave of low voltage FURTHER READING
Boyd DE, Palmer JM. Surgical diathermy. Anaesthesia Inten-
There are two types of diathermy circuits: mono- sive Care Med 2010;11(11):461-3.
polar and bipolar. In monopolar mode, current
flows from the diathermy device through the sur- 24. c—Capacitive coupling may cause thermo-
gical active electrode, through the body, and back electric burns even in an isolated diathermy
to the diathermy device via the dispersive circuit

Neurosurgery Books Full


www.ketabpezeshki.com 66485438-66485457
e32 PART II CARE OF THE NEUROSURGICAL PATIENT

Fires can occur when pools of flammable, alcohol- bovine albumin [Cryolife, Kennesaw, GA,
based skin preparations are ignited. Thermoelec- USA]), Tissucol (human fibrinogen, thrombin,
tric burns occur where the current density rises albumin, and animal aprotinin), Tisseel (human
inadvertently, such as where tissue in a pedicle fibrinogen, thrombin, and aprotinin [Baxter Inter-
burns, at the dispersive electrode if it is applied national Inc., Westlake Village, CA, USA]), and
incorrectly or not of sufficient size, so the site Evicel (human fibrinogen and thrombin [Johnson
should be well perfused, be distant to any metal and Johnson Wound Management, Ethicon Inc.,
implants that may become heated, and be free Somerville, NJ, USA]). These products are used
from hair. Burns have also been reported from alone to reinforce primarily repaired dura or as
insulation failure at the active electrode or acci- adjuncts to dural substitutes.
dental contact between the active electrode and
another conductor. Dispersive electrode contact
monitoring pads have a dual foil design that allows
measurement of skin impedance to ensure ade- EMI ANSWERS
quate skin contact. An audible alarm alerts staff
to electrode failure. The isolated circuit reduces 26. 1—f, surgicel; 2—b, avitene; 3—c, floseal
the risk of burns from contact with earthed objects,
as the current must return to the generator to com- Adequate hemostasis is a prerequisite in neuro-
plete the circuit; however, burns can still occur due surgery, to prevent dramatic postoperative bleed-
to capacitive coupling. Diathermy can interfere ings and their consequences. Different sorts of
with both electroencephalography and electrocar- local hemostatic agents have been developed,
diography monitoring electrodes. Of greater con- with a variable efficacy. Some of them have been
sequence is the interaction between EMI and used for years, none being perfect. The residual
cardiac pacemakers or implantable defibrillators. presence of these agents may behave as foreign
Bipolar is safer than monopolar, but can still cause bodies and induce inflammation, infection, and
EMI. The effects are unpredictable and include even delayed bone growth. Safety is another con-
inappropriate pacing, damage to the device, inap- cern since most of modern agents contain more
propriate defibrillation, and myocardial heat dam- or less human and animal components.
age. Modern pacemakers have a titanium shell and
interference monitor to protect them from EMI. 27. 1—g, tachoSil; 2—b, DuraGuard
The use of a magnet to reset pacemakers to asyn-
chronous continuous pacing is not predictable. Dural substitutes are designed to be either placed
Current advice suggests limiting the use of dia- as an onlay over dural defects or sutured into
thermy to short low power bursts and avoiding place. These could be autologous tissues, such
monopolar where possible. Placement of the dis- as pericranium or fascia lata, or artificial dural
persive electrode away from the device increases substitutes. The latter are generally derived from
safety. Where appropriate the device should be bovine tendon [DuraGen, DuraGen Plus (Inte-
checked and reprogrammed to monitoring mode gra LifeSciences, Plainsboro, NJ, USA), Dura-
prior to surgery. Diathermy smoke consists of Matrix (Stryker, Cambridge, MA, USA),
95% steam and 5% cellular debris, containing a TissuDura (Baxter Healthcare S.A., Opfikon,
variety of toxic mutagenic chemicals including Switzerland)], fetal bovine skin [Durepair (Med-
hydrogen cyanide and benzene. Viruses and viable tronic Inc., Minneapolis, MN, USA), bovine
cancer cells can be transmitted in surgical smoke pericardium/dura [Dura-Guard (Synovis Surgical
and diathermy machines now contain a surgical Innovations, St. Paul, MN, USA)] or porcine col-
smoke evacuator with a 0.1 m filter, attached to lagen [Durasis (Cook Biotech Inc., Bloomington,
the diathermy pencil <2 cm from the site of smoke IN, USA)] sources, polyurethane [Neuro-Patch
production. (B. Braun, Melsungen AG, Melsungen,
Germany)], polyglactin (Ethisorb Dura Patch
FURTHER READING [Johnson & Johnson, New Brunswick, NJ,
Boyd DE, Palmer JM. Surgical diathermy. Anaesthesia Inten- USA)], polytetrafluoroethylene-based sheets
sive Care Med 2010;11(11):461-3. [Preclude, Gore-Tex (W.L. Gore and Associates
Inc., Flagstaff, AZ, USA)], human acellular der-
25. e—Evicel mis [AlloDerm (LifeCell Corporation, Branch-
burg, NJ, USA)] grafts, and TachoSil [human
Dural sealants, including DuraSeal (polyethylene fibrinogen and thrombin with equine collagen
glycol hydrogel [Confluent Surgical Inc., Wal- (Takeda Pharmaceutical Company, Osaka,
tham, MA, USA]), Bioglue (glutaraldehyde, Japan)].

Neurosurgery Books Full


www.ketabpezeshki.com 66485438-66485457
19 SURGICAL TECHNOLOGY AND PRACTICE e33

Hemostat Notes

Surgicel (Johnson & Oxidized cellulose


Johnson)

Floseal (Baxter) Human-derived thrombin with bovine-derived gelatin matrix granules which are
mixed at the time of use

Surgiflo (Johnson & Porcine gelatin


Johnson)

Spongostan (Johnson & Porcine gelatin


Johnson)

Hydrogen peroxide Vasoconstrictive, disrupts blood-barrier causing an aggregation of the platelets


and neutrophils leading to thrombus formation. The free diffusion of H2O2 through
the vessel walls and its conversion to water and 02 leads to intra-luminal bubbles,
micro-embolisms and vessels obstruction

TachoSil (Takeda) Fibrin glues contain thrombin and fibrinogen in separate chambers; when
Tisseel (Baxter combined, the fibrinogen is activated by thrombin and converted into fibrin
International) monomer which interacts with patient factor VIII and calcium to form a fibrin
Evicel (Ethicon, Johnson & polymer that allows for platelets activation and hemostasis
Johnson)

Avitene (Davol) Microfibrillar collagen

Arista (Medafor) Potato starch

Table with permission from Lapierre F, D'Houtaud S, Wager M. Hemostatic agents in neurosurgery. In: Francesco Signorelli
(Ed.), Explicative Cases of Controversial Issues in Neurosurgery. InTech; 2012. Available from: http://www.intechopen.
com/books/explicative-cases-of-controversial-issues-inneurosurgery/topical-hemostatic-agents-and-neurosurgery.

Neurosurgery Books Full


www.ketabpezeshki.com 66485438-66485457
PART III
CRANIAL NEUROSURGERY
CHAPTER 20

GENERAL NEUROSURGERY AND


CSF DISORDERS
SINGLE BEST ANSWER (SBA) QUESTIONS
1. Which one of the following statements regar- e. In a patient with atrial fibrillation risk
ding intracerebral hemorrhage is LEAST of ICH on warfarin is calculated using
accurate? HASBLED score
a. Rate of spontaneous intracerebral hemor-
rhage in a 70-year-old is approximately 3. Which one of the following statements
0.15% per year regarding hydrocephalus is most accurate?
b. Rate of intratumoral hemorrhage in gli- a. Communicating hydrocephalus is usually
oma is 2-4% caused by the presence of intracranial mass
c. Enoxaparin increases the risk of hemor- lesions
rhage in melanoma and renal cell carci- b. Noncommunicating hydrocephalus is
noma brain metastasis usually treated with spinal CSF diversion
d. Risk of recurrent intracerebral hemor- c. Excess CSF production is likely to
rhage after an initial bleed is 3-5% produce a communicating, nonobstruc-
per year tive hydrocephalus
e. Long-term aspirin therapy reduced the d. Endoscopic third ventriculostomy is
risk of ICH in patients with unruptured appropriate in cases of foramen of Monro
intracranial aneurysms obstruction
e. Ventriculoperitoneal shunts are contrain-
2. Which one of the following statements dicated where there is obstruction of
regarding balancing the risk of thromboem- CSF entering the cortical subarachnoid
bolism with risk of intracranial bleeding is space
most accurate?
a. In patient with metallic mitral valve 4. A 75-year-old male having problems with his
replacement risk of thrombosis off antic- gait urinary incontinence for the past
oagulation is 8% per year 6 months, and recently, his short-term
b. In a patient with brain metastasis from memory. When he started to walk he had a
nonsmall cell lung cancer risk of ICH “magnetic” gait but normal armswing.
on LMWH is 35% Mini-mental state examination was 27/30.
c. In a patient with glioma risk of ICH on His CT head is shown below. Which one
warfarin is threefold higher of the following is the next appropriate step
d. In a patient with acute ICH and a proxi- in management?
mal DVT, the risk of fatal PE off anticoa-
gulation is lower than the risk of recurrent
ICH on anticoagulation

257
Neurosurgery Books Full
www.ketabpezeshki.com 66485438-66485457
258 PART III CRANIAL NEUROSURGERY

A c. He is likely a shunt nonresponsive NPH


patient
d. ICP monitoring as an inpatient is
appropriate
e. Adjusting the shunt setting down further
should be tried

7. Which one of the following approaches is


LEAST likely to be appropriate for manage-
ment of entrapped 4th ventricle?
a. 4th ventricular-peritoneal shunt
b. Endoscopic Aqueductal stent and lateral
ventriculoperitoneal shunt
c. Endoscopic third ventriculostomy and
aqueductoplasty
d. Foramen magnum decompression with
expansion duraplasty
a. Reassure that this is part of normal aging e. 4th ventricular-cisternal shunt
and discharge
b. Amantadine trial 8. A 3-year-old female presents with headache
c. ICP monitoring and transient obscurations of her vision.
d. Lumbar puncture tap test Which one of the following would you start
e. Ventriculoperitoneal shunt her on?

5. Which one of the following statements


regarding management of idiopathic normal
pressure hydrocephalus is most accurate?
a. CSF outflow resistance is normally higher
than 13 mmHg/mL/min A
b. CSF outflow resistance greater than
18 mmHg/mL/min correlated with shunt
responsiveness in NPH
c. Lundberg A waves usually visible during
sleep in NPH patients
d. Presence of Lundberg B waves for greater
than 80% of ICP monitoring suggests
poor outcome with VP shunt
e. External lumbar drainage of CSF carries a
lower risk of meningitis compared to lum-
bar infusion study

6. A 75-year-old man diagnosed with normal


pressure hydrocephalus underwent VP shunt a. Acetazolamide
insertion 3 months ago. A Codman Hakim b. Bromocriptine
Progammable valve was set to 160 mmHg c. Corticosteroids
on discharge. CT head today shows no sig- d. Furosemide
nificant change in ventricular size (persistent e. Topiramate
ventriculomegaly). He reports no significant
improvement in his symptoms. You under- 9. Which one of the following statements
take a shunt reservoir tap which records a regarding surgical management of pseudotu-
pressure of 5 cm H2O in the supine and mor cerebri is most accurate?
10 cm H2O in the sitting position. Which a. Optic nerve sheath fenestration provides
one of the following statements is the most better visual outcomes than VP shunt in IIH
accurate conclusion? b. Choroid plexotomy is routinely used in IIH
a. There is evidence of shunt obstruction in patients with recurrent shunt blockage
b. There is evidence of shunt overdrainage

Neurosurgery Books Full


www.ketabpezeshki.com 66485438-66485457
20 GENERAL NEUROSURGERY AND CSF DISORDERS 259

c. Venous sinus stenting is first line treat- a. Anterior commissure


ment in IIH patients with MRI proven b. Fornix
venous sinus stenosis c. Chiasm
d. Endoscopic third ventriculostomy is d. Infundibulum
appropriate substitute when optic nerve e. Tuber cinereum
sheath fenestration is not possible f. Fenestration point for ETV
e. Lumboperitoneal shunt rate failure in IIH g. Mammillary bodies
is 10% h. Intermammillary space

10. Which one of the following statements regar-


ding hydrocephalus is LEAST accurate? 22. Operative approaches:
a. Negative pressure hydrocephalus may be a. Bifrontal
associated with a CSF leak b. Extended middle fossa
b. Low pressure hydrocephalus may be due c. Far lateral
to alteration in transmantle pressure d. Interhemispheric
c. Benign external hydrocephalus usually e. Midline suboccipital
resolves by 2-years of age f. Orbito-zygomatic
d. Subdural hygromas are difficult to distin- g. Petrosal (retrolabryrinthine)
guish from chronic subdural hematomas h. Pterional
radiologically i. Retrosigmoid
e. Normal pressure hydrocephalus is diag- j. Subfrontal
nosed using ICP monitoring k. Transphenoidal

For each of the following descriptions, select the


QUESTIONS 11–20 most appropriate answers from the list above.
Each answer may be used once, more than once
Additional questions 11–20 available on or not at all.
ExpertConsult.com
1. Approach commonly used for a foramen
magnum meningioma
2. Approach commonly used for a large olfac-
tory groove meningioma
EXTENDED MATCHING ITEM (EMI) 3. Approach commonly used for clipping of a
pericallosal artery aneurysm
QUESTIONS 4. Approach commonly used for clipping a
AICA aneurysm
21. In the endoscopic view during third ventricu- 5. Approach commonly used for clipping a
lostomy shown below (top of picture is ante- PICA aneurysm
rior, bottom is posterior) which one of the
following is the ideal point of fenestration? 23. CSF diversion complications:
A B
a. CSF pseudocyst
A
b. Distal catheter migration
c. Infection
d. Intraparenchymal hemorrhage
e. Mechanical obstruction
f. Overdrainage
g. Pseudomeningocele
L
h. Shunt allergy
C
i. Shunt disconnection
D
R j. Slit ventricle syndrome
E k. Subdural hematoma
F For each of the following descriptions, select the
most appropriate answers from the list above.
P
Each answer may be used once, more than once
H or not at all.

Neurosurgery Books Full


www.ketabpezeshki.com 66485438-66485457
260 PART III CRANIAL NEUROSURGERY

1. A 44-year-old female presents after recent 2. A 57-year-old man presents with symptoms
shunt insertion with shunt failure and of shunt obstruction and nonspecific
CSF with persistent eosinophilia despite abdominal pain. His wounds are clean and
negative cultures dry. XR shunt series appears normal

SBA ANSWERS
1. c—Enoxaparin increases the risk of hemorrhage in melanoma and renal cell carcinoma brain
metastasis

Risk of ICH with and without Antithrombotic Drugs


Risk of ICH Risk of ICH with Anticoagulation Risk of ICH Antiplatelet

General 0.016-0.033 per year 0.3-1% per year (INR 2-3) Excess risk of 0.2-1.2
population (0.15% per year in per 1000-patient-years
70-year-olds) with aspirin

Unruptured <7 mm anterior Paucity of data. Worse outcome of Lower risk of aneurysm
intracranial circulation: 0.1% per year aneurysmal SAH if on warfarin. IV rupture in cohort of
aneurysm 7-12 mm anterior circn: thrombolysis appears safe in acute stroke ISUIA on aspirin
2.5% per year patients with unruptured intracranial
>13 mm or posterior aneurysm
circulation: 3-20% per
year

Glioma 2-4% 2-4% (INR well controlled)

Brain 15% (NSCLC) to 35% No change from baseline (enoxaparin or


metastasis (melanoma/RCC) well controlled warfarin)

Acute ICH ICH expansion in first 3-5% After 24 h Unclear


24 h: 15-38% of patients
ICH expansion
24 h-2 weeks: 1-2%

Prior ICH 2-3% per year 3-5% per year Possibly with lobar ICH

Acute Unclear 0.9% (warfarin) 0.2% (aspirin)


infarction 1% (clopidogrel or
dual)

FURTHER READING Wijdicks EFM, et al. The use of antithrombotic therapy in


Rordorf G, et al. Spontaneous intracerebral hemorrhage: patients with an acute or prior intracerebral haemorrhage.
pathogenesis, clinical features, and diagnosis. Uptodate Topic Uptodate Topic 1323 Version 19.0.
1133 Version 12.0. Donato J, et al. Intracranial haemorrhage in patients with
Freeman DW, et al. Risk of intracerebral bleeding in patients brain metastases treated with therapeutic enoxaparin: a
treated with anticoagulants. Uptodate Topic 1328 Version 16.0. matched cohort study. Blood 2015; 126(4).

Gaasch WH, et al. Complications of prosthetic heart valves. 2. a—In patient with metallic mitral valve
Uptodate Topic 8121 Version 12.0. replacement risk of thrombosis off anticoagu-
Cucchiara BL, et al. Antiplatelet therapy for secondary pre- lation is 8% per year
vention of stroke. Uptodate Topic 1086 Version 35.0.
The use of IVC filters in patients with brain
Wen PY. Anticoagulant and antiplatelet therapy in patients tumors has been associated with substantial
with brain tumors. Uptodate Topic 5201 Version 17.0. complication rates (over 50% experience
Alejandro A. Anticoagulant and antiplatelet therapy in patients recurrent VTE, IVC or filter thrombosis, or
with an unruptured intracranial aneurysm. Uptodate Topic post-thrombotic syndrome), and the risk of
1320 Version 8.0. hemorrhage secondary to anticoagulation is

Neurosurgery Books Full


www.ketabpezeshki.com 66485438-66485457
20 GENERAL NEUROSURGERY AND CSF DISORDERS 261

not as high as originally feared. The relatively comparisons are not available in brain tumor
low incidence of intratumoral hemorrhage is a patients, indirect comparisons from case series
particularly important issue for patients who suggest that carefully controlled oral anticoagu-
need anticoagulation for reasons other than lation with warfarin is reasonably safe and asso-
VTE (e.g. atrial fibrillation) in whom IVC filter ciated with fewer serious complications than
is not appropriate. Although randomized routine use of IVC filters.

Balancing the Risk of Venous Thromboembolism with the Risk of ICH


Risk of
Thromboembolic Thromboembolism
Event off Risk on
Diagnosis Anticoagulation Antithrombotic Risk of ICH on Antithrombotic

Glioma 20% risk VTE Recurrent VTE in all 2%—If INR well controlled, not different
cancer patients at from glioma patients not on
6 months: anticoagulation (2-4%)
Warfarin 17%
LMWH 9%

Brain metastasis 20% risk VTE Recurrent VTE in all 20% significant bleed rate at 1 year—no
cancer patients at difference between those treated with
6 months: enoxaparin and controls, but difference
Warfarin 17% between nonsmall cell lung cancer
LMWH 9% (15-19%) and renal cell/melanoma (35%)

Mechanical MVR 8% per year 4% per year after 0.3-1% per year
1 month (INR
2.5-3.5)

Bioprosthetic MVR 2.4% per year after 0.3-1% per year


3 months

Prosthetic AVR 1.9% per year after N/A 0.3-1% per year
3 months

Proximal DVT or Risk of fatal PE 25% Beyond 24 h: 3-5% risk of ICH recurrence
nonfatal PE in
patient with
acute ICH

Atrial fibrillation Calculate using 1-2% per year 0.3-1% per year
CHAD2vasc score Calculate all cause risk of major
hemorrhage using HASBLED score

FURTHER READING Wen PY. Anticoagulant and antiplatelet therapy in patients


McKenzie DB, et al. management of patients with mechanical with brain tumors. Uptodate Topic 5201 Version 17.0.
valves and intracerebral haemorrhage. Br J Cardiol
2008;15:145-8. Alejandro A. Anticoagulant and antiplatelet therapy in patients
with an unruptured intracranial aneurysm. Uptodate Topic
Donato J, et al. Intracranial haemorrhage in patients with 1320 Version 8.0.
brain metastases treated with therapeutic enoxaparin: a
matched cohort study. Blood 2015;126(4). Wijdicks EFM, et al. The use of antithrombotic therapy in
patients with an acute or prior intracerebral haemorrhage.
Rordorf G, et al. Spontaneous intracerebral hemorrhage: Uptodate Topic 1323 Version 19.0.
pathogenesis, clinical features, and diagnosis. Uptodate Topic
1133 Version 12.0. 3. c—Excess CSF production is likely to
Freeman DW, et al. Risk of intracerebral bleeding in patients produce a communicating, nonobstructive
treated with anticoagulants. Uptodate Topic 1328 Version 16.0. hydrocephalus
Gaasch WH, et al. Complications of prosthetic heart valves. Walter Dandy’s classification of hydrocephalus
Uptodate Topic 8121 Version 12.0. is still commonly used and makes the distinction
Cucchiara BL, et al. Antiplatelet therapy for secondary pre- between communicating (no obstruction in CSF
vention of stroke. Uptodate Topic 1086 Version 35.0. pathway from ventricles to subarachnoid space)

Neurosurgery Books Full


www.ketabpezeshki.com 66485438-66485457
262 PART III CRANIAL NEUROSURGERY

and obstructive (CSF cannot flow from ventricu- and true “communicating hydrocephalus” with-
lar system to subarachnoid space; “noncommuni- out a point of obstruction would only be pro-
cating”) hydrocephalus as these are the clinically duced by overproduction of CSF (e.g. choroid
most common and also reflect differing manage- plexus papilloma). A more nuanced system that
ment options. However, almost all hydrocephalus takes advantage of tremendous advances in imag-
involves an obstruction to CSF flow and it is just ing is now possible (although developmental
the point of obstruction which varies (e.g. within forms of hydrocephalus often have multiple
ventricles, arachnoid villi, venous sinus outflow), points of obstruction):

Hydrocephalus Classification Study Group: Point of Obstruction Model


Point of Obstruction Differential Treatments

Foramen of Monro Tumor, congenital absence, Shunt (unilateral or bilateral)


ventriculitis, functional Endoscopic septostomy

Aqueduct of Sylvius Tumor, congenital stenosis, secondary Shunt


Endoscopic third
ventriculostomy

4th ventricle foramina Infection, tumor, severe Chiari Shunt


Endoscopic third
ventriculostomy
Surgical opening

Between spinal and cortical Subarachnoid hemorrhage Shunt


subarachnoid space Infection Endoscopic third
ventriculostomy
LP shunt

Arachnoid villi Subarachnoid hemorrhage VP or LP shunt


Infection

Venous hypertension Pseudotumor cerebri (PTC) Bariatric surgery for obesity-


Congenital hydrocephalus related PTC
Sinus thrombosis VP or LP shunt
Anticoagulation
Venous sinus stent (debated)

FURTHER READING characterized by a broad base and slow, small steps


Rekate HL. A consensus on the classification of hydro- with reduced height clearance as though the feet
cephalus: its utility in the assessment of abnormalities are “stuck to the floor” but also include unsteadi-
of cerebrospinal fluid dynamics. Childs Nerv Syst. ness, recurrent falls, shuffling, and reduced walk-
2011;27(10):1535-41. ing speed (confused with parkinsonism). Urinary
incontinence maybe neurological or a conse-
4. d—Lumbar puncture tap test quence of gait disturbance or the cognitive impair-
ment. NPH is estimated to account for less than
Normal pressure hydrocephalus is a clinical syn- 5% of all cases of dementia hence commoner
drome characterized by gait apraxia (90% of causes such as Alzheimer’s must be excluded, as
patients), dementia, and incontinence with ventri- well as mimics such as Binswanger’s disease (also
culomegaly and normal CSF pressure on lumbar produces a frontal dysexecutive syndrome). Imag-
puncture. If the gait improves after a single ing should show ventricular enlargement not
large-volume lumbar puncture (30-50 mL), serial entirely attributable to cerebral atrophy or con-
large-volume lumbar punctures can be performed genital enlargement (Evans’ index >0.3 or compa-
daily for 3 days the diagnosis is probable, and, rable measure), bicaudate ratio >0.25, and another
more importantly, the patient will likely respond supportive feature: enlargement of the temporal
to treatment by shunting. In those where lumbar horns of the lateral ventricles not entirely attribut-
puncture fails or is contraindicated, a ventricular able to hippocampal atrophy; callosal angle of 40
reservoir can be inserted and testing undertaken degrees or greater; evidence of altered brain water
a few months later. Gait in patients with NPH content, including periventricular signal changes
is described as being “magnetic” in nature, not attributable to microvascular ischemic changes

Neurosurgery Books Full


www.ketabpezeshki.com 66485438-66485457
20 GENERAL NEUROSURGERY AND CSF DISORDERS 263

or demyelination; presence of aqueductal or fourth Dutch NPH study, outflow resistance greater than
ventricular flow void on MRI. Current guidelines 18 mmHg/mL/min had a specificity of 87% and a
deal primarily with idiopathic as opposed to sec- sensitivity of 46%. Although isolated measure-
ondary NPH, which can occur years after trauma, ments of CSF pressure in patients with communi-
subarachnoid hemorrhage, intracranial surgery, or cating hydrocephalus and NPH may be in the
meningitis. Continuous ICP monitoring has dem- normal range, overnight ICP monitoring may
onstrated the presence of waves of increased ICP, reveal dynamic phenomena such as increased
particularly during rapid eye movement (REM) Lundberg “B waves.” B waves are slow waves of
sleep. It has been suggested that these abnormal ICP lasting 20 s to 2 min. The presence of B waves
CSF pressure spikes, called B waves, slowly for more than 80% of the period of ICP monitor-
increase ventricular size by exerting intermittent ing is thought to indicate that it is much more
high pressure on the brain parenchyma that results likely than not that shunting would be helpful.
in ischemic damage. Abnormalities of the aging
brain parenchyma may make it more susceptible FURTHER READING
to these forces. Despite the uncertainty regarding Marmarou A, et al. The value of supplemental prognostic
its evolution, NPH is a syndrome that is treatable tests for the preoperative assessment of idiopathic normal-
pressure hydrocephalus. Neurosurgery 2005;57(3 Suppl.
by CSF diversion (i.e. shunt insertion).
S2):17-284.
Image with permission from Budson AE. Memory Loss,
Alzheimer's Disease, and Dementia, 2nd ed., Elsevier, 6. e—Adjusting the shunt setting down further
2016. should be tried
FURTHER READING It is the patient in whom the association between
Relkin N, Marmarou A, Klinge P, et al. Diagnosing idiopathic clinical findings and ventriculomegaly is uncer-
normal pressure hydrocephalus. Neurosurgery 2005;57(3 tain (e.g. NPH) and fails to improve after shunt
Suppl.):S4-S16, discussion ii-v.
surgery (or only minimally improves) who repre-
sents a clinical challenge. As a result, the failure to
5. b—CSF outflow resistance greater than
improve might be attributed to an incorrect diag-
18 mmHg/mL/min correlated with shunt
nosis, or a shunt nonresponder (e.g. if valve at
responsiveness in NPH
lowest setting and shunt patency confirmed). If
imaging reveals a reduction in ventricular size, a
There is no single test for idiopathic NPH, but
patient should be considered a nonresponder if
supplementary tests can increase the prognostic
no clinical improvement occurred. For patients
accuracy to greater than 90%. A lumbar puncture
in this scenario who remain with significant ven-
“tap test” has been shown to produce a specificity
triculomegaly, strategies for improving drainage
of 100% with a sensitivity of 26%, provided that it
is performed at a high volume (i.e. withdrawal of should be considered (e.g. removal of antisiphon
device, or a programmable valve with a lower
40-50 mL of CSF). Symptomatic improvement
pressure limit) as they may have a low-pressure
after removal of CSF has a high positive predictive
value (73-100%) of a probably favorable outcome hydrocephalus state. Downward adjustments in
valve opening pressure are unlikely to benefit
with shunt placement. It has to be remembered
the patient and instead increase the risk of sub-
that improvement after a shunt is often delayed
dural hematoma. Even if shunt flow is documen-
in many patients, so a simple tap test would not
ted, one should pursue other interventions as one
be expected to reveal all patients who might benefit
cannot exclude functional underdrainage. For
from a shunt. However, the low sensitivity of the
example, if there is an ASD, remove it. If the
“tap test” precludes using this method as a diag-
patient has a fixed-pressure valve or a flow-
nostic tool for exclusion. Prolonged external lum-
restricting valve, change it to an adjustable
bar drainage in excess of 300 mL is associated with
differential pressure valve (no ASD). It is our
high sensitivity (50-80%), specificity (80%), and
positive predictive value (80-100%), but requires observation that ventriculoatrial shunts provide
more drainage than ventriculoperitoneal shunts
inpatient stay and carries a risk for the complica-
do, and therefore we offer a shunt revision to a
tions of nerve root irritation, hemorrhage, and
CSF infection. Measurement of CSF outflow ventriculoatrial shunt as well. It is only the case
in which the patient has a ventriculoatrial shunt
resistance (reflecting the capacity of CSF absorp-
with a differential pressure valve set to 30 mm
tion pathways) via a daycase lumbar infusion or
H2O or less that an operative intervention is
ventricular reservoir infusion test with a
not recommended.
pressure-volume study is also established. In the

Neurosurgery Books Full


www.ketabpezeshki.com 66485438-66485457
264 PART III CRANIAL NEUROSURGERY

7. d—Foramen magnum decompression with population, rising to 3.5/100,000 in women aged


expansion duraplasty 15-44 years and 19.3/100,000 in women aged
20-44 years who weigh 20% or more than their
Entrapped fourth ventricle has been used to ideal body weight. When no secondary cause is
describe the situation in which the fourth ventri- identified (e.g. venous obstruction, endocrine
cle no longer communicates with either the third disorders, medications), the syndrome is primary
ventricle and/or the basal cisterns. It is thought and termed Idiopathic Intracranial Hypertension
that secondary aqueduct stenosis from adhesions, (IIH). Symptoms include headache, transient
obstruction of the foramina of Luschka or visual obscurations, pulsatile tinnitus, visual loss,
Magendie, or infective debris pooling in the basal diplopia. Signs include 6th nerve palsy and papil-
cisterns may be responsible for this condition. ledema. In addition, for the diagnosis to be made
Patients may have the typical symptoms and signs brain imaging must show no structural causes of
of hydrocephalus or more atypical symptoms raised ICP, and lumbar puncture CSF opening
such as lower cranial nerve dysfunction. Patients pressure >25 cm H2O in relaxed adults and
with prolonged infection or multiple shunt >28 cm H2O in a (sedated) child with normal
operations are particularly at risk for this syn- CSF analysis. In the absence of papilledema or
drome. In cases of hydrocephalus caused by 6th nerve palsy, possible diagnosis of PTCS is
membranous occlusion or short segment stenosis suggested by MRI showing at least three of:
of the aqueduct of Sylvius, endoscopic aqueduc- empty sella, flattening of the posterior aspect of
toplasty (EA) with and without stenting has been the globe, distension of perioptic subarachnoid
reported. The burr hole for EA is placed more space (¼/- tortuous optic nerve), transverse
anteriorly than the one for standard ETV. Stent- venous sinus stenosis. The main goal of treat-
ing of the aqueduct may be performed for ment is preservation of vision. As such, patients
patients at high risk for aqueductal restenosis or presenting with deteriorating vision require
patients with a trapped fourth ventricle. The stent more aggressive initial management. General
is usually a ventricular catheter with additional management strategied include weight loss
holes. Shunted patients with a trapped fourth (including bariatric surgery in some cases), salt
ventricle often have slit-like lateral ventricles, restriction, acetazolamide, topiramate (headache
making them poor candidates for the standard and causes weight loss), ventriculoperitoneal or
EA so a suboccipital approach for retrograde lumboperitoneal CSF shunting, optic nerve
aqueductoplasty and stenting can be performed. sheath fenestration and in some cases venous
EA restores the physiologic CSF pathways and sinus stenting.
eliminates the risk for basilar artery injury. There
are no arachnoidal adhesions around the aque- Image with permission from Scholes MA. ENT Secrets,
4th ed., Elsevier, 2016.
duct to interfere with CSF flow. The risk for
injuring the hypothalamus is avoided, especially
during cases when the floor of the third ventricle FURTHER READING
Friedman DI. The pseudotumor cerebri syndrome. Neurol
is thickened and a considerable amount of force is
Clin 2014;32:363-96.
required to perforate the floor. Strictures at the
aqueduct are usually not as tough to penetrate; 9. a—Optic nerve sheath fenestration provides
thus, less force is required for fenestration. A better visual outcomes than VP shunt in IIH
major risk of EA is injuring the periaqueductal
gray matter and the floor of the fourth ventricle. Surgery is usually indicated for visual loss or
Other complications reported, especially in long worsening of vision that is attributable to papille-
stenoses, include midbrain injury causing tran- dema, rather than chronic headache. The two
sient or permanent dysconjugate eye movements, surgical treatments are optic nerve sheath fenes-
Parinaud-syndrome, and cranial nerve palsies. In tration (ONSF) and shunting. There have been
cases with long stenoses, ETV may be a more no prospective, randomized trials of surgical
appropriate procedure. treatments. ONSF as the preferred treatment of
visual loss from IIH, perhaps because visual out-
8. a—Acetazolamide comes were better documented with this proce-
dure. ONSF tends to be more effective in acute
The pseudotumor cerebri syndrome (PTCS) is papilledema than chronic papilledema and is
a perplexing syndrome of increased intracranial not indicated once the papilledema has resolved.
pressure without a space-occupying lesion. Studies show that bilateral improvement in vision
annual incidence of pseudotumor cerebri syn- often occurs after a unilateral procedure. The
drome (PTCS) as 0.9/100,000 in the general complications of ONSF include failure, ischemic

Neurosurgery Books Full


www.ketabpezeshki.com 66485438-66485457
20 GENERAL NEUROSURGERY AND CSF DISORDERS 265

optic neuropathy, transient diplopia, and tran- may be an option for the long-term management
sient blindness. Because the ventricles are not of IIH in morbidly obese patients but is not help-
enlarged in PTCS, lumboperitoneal shunting ful for acute management. Transverse sinus ste-
was previously preferred over ventriculoperito- nosis in association with IIH prompted
neal (VP) shunting but usually require multiple endovascular stenting as a treatment for the dis-
revision and failure rate is approximately 50% order, though there is debate as to whether it is
in PTCS. The most common reasons for revision cause or a sign of raised ICP. Despite some pos-
are shunt obstruction, intracranial hypotension/ itive results, the need for long term anticoagula-
subdurals, and lumbar radiculopathy. Visual dete- tion has resulted in subdural/epidural hematoma
rioration may be the only sign of shunt failure and (and also complicates other surgical interventions
may occur even if the shunt is functioning. Other which may need performing), anaphylaxis, and
complications include infection, abdominal pain, hearing loss.
CSF leak, hindbrain herniation, and migration of
the peritoneal catheter. Cisterna magna shunting FURTHER READING
has been described, but image-guided placement Friedman DI. The pseudotumor cerebri syndrome. Neurol
of VP shunts is now favored. Bariatric surgery Clin 2014;32:363-96.

10. e—Normal pressure hydrocephalus is diagnosed using ICP monitoring

Other CSF Disorders

Negative or low pressure Rare conditions where patient exhibit features of raised ICP and show
hydrocephalus ventriculomegaly on imaging despite very low or negative ICP. Clinical
suspicion for low- or negative-pressure hydrocephalus should be high when
patients with enlarged ventricles have repeated “shunt failures” that do not
improve with shunt revisions. Presence of a CSF leak from the cortical
subarachnoid space and/or loss of patency between the ventricles and the
cortical subarachnoid space (e.g. arachnoiditis, mechanical) may affect
transmantle pressure (ventricular pressure—cortical subarachnoid space
pressure) and brain turgor, leading to ventriculomegaly and neurological signs

Benign enlargement of The course is self-limited, manifesting in the first 6 months of life and resolving
subarachnoid space spontaneously by 2 years of age and thought to be due to transient mismatch in
maturation/ability of arachnoid villi to absorb increasing volumes of CSF being
produced

Subdural hygroma Subdural fluid collection resembling CSF, on similar spectrum as subdural
effusion and chronic subdural hematoma

Normal pressure Constellation of dementia, incontinence, and gait apraxia with evidence of
hydrocephalus ventriculomegaly on imaging (without underlying mass lesion) with normal
CSF pressure on lumbar puncture

FURTHER READING
Filippidis AS, et al. Negative-pressure and low-pressure ANSWERS 11–20
hydrocephalus: the role of cerebrospinal fluid leaks resulting
from surgical approaches to the cranial base Report of 3 cases. Additional answers 11–20 available on
ExpertConsult.com
JNS 2012;116(5):1031-7.
Cardoso ER. External hydrocephalus in adults. Report of
three cases. JNS 1996;85:1143-7.

Neurosurgery Books Full


www.ketabpezeshki.com 66485438-66485457
266 PART III CRANIAL NEUROSURGERY

EMI ANSWERS Image with permission from Jandial, R, McCormick PC,


Black PM. Core Techniques in Operative Neurosurgery,
Elsevier, Saunders, 2011.
21. f—6. 1—Anterior commissure; 2—Fornix;
3—Chiasm; 4—Infundibulum; 5—Tuber 22. 1—c, Far lateral; 2—a, Bifrontal; 3—d, Inter-
cinereum; 6—Fenestration point for hemispheric, 3—i, Retrosigmoid; 4—c, Far
ETV; 7—Mammillary bodies; 8— lateral
Intermammillary space

General Approaches in Neurosurgery


Approach Positioning Notes Indications

Medial subfrontal Supine, neck Frontal bone flap extends to Unilateral tumor or
flexed, head midline. Can also take orbital rim vascular lesion in anterior
extended (cranio-orbital approach) cranial fossa

Pterional (lateral Supine, ipsilateral extends anteriorly in a curvilinear Anterior and middle cranial
frontotemporal) shoulder elevated manner toward the supraorbital rim fossa lesions
30°, head rotated just above the superior temporal
30° line (key hole), then inferiorly
parallel to the supraorbital rim to the
frontozygomatic process, then
below this and posteriorly (crossing
the sphenoid wing) to the squamous
portion of the sphenoid bone, and
then back up toward the superior
temporal line posteriorly

Orbito-zygomatic Supine, ipsilateral As pterional approach followed by Anterior and middle cranial
shoulder elevated en bloc removal of supraorbital rim, fossa lesions
30°, head rotated frontozygomatic process posterior
30° half of the body of the zygoma, and
the arch of the zygoma in a second
osteotomy

Extended middle Supine with head Removal of the petrous apex Middle/posterior cranial
fossa (Anterior turned between the foramen ovale fossa:
petrosectomy) contralateral to anteriorly, the arcuate eminence of Petrous apex, superior
lesion side the cochlea posteriorly, and the clival, anterior CPA lesions,
greater superficial petrosal nerve posterior cavernous sinus,
laterally basilar artery and anterior
brain stem

Bifrontal Supine, neck Zygomatic arch to zygomatic arch Bilateral anterior cranial
flexed, head incision. Extended bifrontal fossa:
extended approach involves removal of Midline tumors
orbital bar (may cause Large olfactory groove/
trigeminocardiac reflex) planum sphenoidale/
tuberculum sella
meningiomas, large
craniopharyngiomas

Interhemispheric Supine, head Bipartite box flap crossing midline Midline lesions, e.g. distal
slightly flexed or (ipsilateral half is completely lateral ACA aneurysm, falcine
semilateral, head to SSS) meningioma, corpus
turned 90 deg. to callosum lesion, lateral and
table with 45 deg. third ventricle lesions
upward tilt of
vertex

Transnasal Supine, body Anterior wall of sphenoid sinus Sellar lesions with or
Transphenoidal flexed, nose in removed followed by sellar floor, without suprasellar
sniffing position can be advanced to include extension or for clival
tuberculum sellae and planum lesions
sphenoidale

Continued

Neurosurgery Books Full


www.ketabpezeshki.com 66485438-66485457
20 GENERAL NEUROSURGERY AND CSF DISORDERS 267

Approach Positioning Notes Indications


Petrosal Supine, bolster C-shaped temporal-occipital Upper 2/3 clivus and
(retrolabryrinthine- under ipsilateral craniotomy followed by anterior brain stem/basilar
middle shoulder, and 60° mastoidectomy. Gives a combined artery
fossa) head turn supra and infratentorial exposure.
contralaterally Tentorium and superior petrosal
sinus sectioned. In certain cases
sigmoid sinus may be divided

Midline Prone with head Craniotomy below transverse sinus Cerebellar hemispheres,
suboccipital flexed, chin tuck or to lip of foramen magnum. C1 midline dorsal medulla/
sitting position laminectomy may be required for pons, pineal region
(obese, large 4th ventricular lesions
breasted)

Retrosigmoid Supine, bolster Suboccipital craniotomy which also CPA tumors, cranial nerves
suboccipital under ipsilateral exposes junction of transverse and V-XI, anterolateral pons,
shoulder, and 90° sigmoid sinus. Bone over sigmoid AICA aneurysm, middle 1/3
head turn to floor, sinus can be drilled off using clivus
vertex tilt as diamond burr. neurovascular
required structures of the temporal bone are
avoided at the expense of cerebellar
retraction

Far lateral ¾ prone, park A suboccipital craniotomy is Lateral and anterolateral


suboccipital bench (lateral performed from midline to the foramen magnum, lower 1/
decubitus) sigmoid sinus laterally. The occipital 3 clivus, PICA aneurysm
position, head condyle can be drilled down to the
tucked forward and hypoglossal canal. A C1 cervical
rotated 120° from laminectomy is also performed
vertical and vertex
tilted

23. 1—h, Shunt allergy; 2—a, CSF peudocyst giant cells. There are documented cases of
immune responses to unpolymerized silicone in
True shunt allergies are rare. CSF often demon- the literature. There are several management
strates persistent eosinophilia (3-36%), with neg- strategies. One is to consider an endoscopic third
ative cultures. Recurrent shunt failure is a ventriculostomy and to remove the offending
common presentation. Pathologic examination shunt. A second is to use a shunt system devoid
of the ventricular catheter often demonstrates of silicone, such as a polyurethane shunt system
mechanical obstruction by inflammatory debris or hyperextruded silicone components.
consisting of eosinophils and multinucleated

Neurosurgery Books Full


www.ketabpezeshki.com 66485438-66485457
CHAPTER 21

CRANIAL TRAUMA
SINGLE BEST ANSWER (SBA) QUESTIONS
1. Which one of the following approximate b. It is associated with increased glutamate
ratios for the proportion of traumatic brain release, and a hyperglycotic and hyperme-
injuries that are mild, moderate or severe is tabolic state in brain tissue
most accurate? c. Clinically significant brain injury is pre-
a. 22 : 1.5 : 1 sent in 10-15% of cases
b. 20 : 3.5 : 1 d. The majority of patients are symptom free
c. 20 : 2.5 : 2 by 7-10 days post-concussion
d. 18 : 3.5 : 3 e. Approximately 0.5-1% result in death or
e. 18 : 4.5 : 2 require neurosurgical intervention due
to underlying significant brain injury
2. Which one of the following statements
regarding the pathophysiology of traumatic 5. A football player sustains a head injury during
brain injury is LEAST accurate? a game. On examination he is GCS 15/15.
a. An acute extradural hematoma is a type of Which one of the following statements about
primary brain injury complications following concussion is most
b. Diffuse axonal injury is a type of second- accurate?
ary brain injury a. Second impact syndrome is a common cause
c. Cerebral contusions are a type of primary of rapid fatal brain swelling after concussion
brain injury b. The commonest symptoms of post-
d. Glutamate excitotoxicity is a type of concussion syndrome are ongoing cogni-
secondary brain injury tive impairment
e. Hypoxia is a cause of secondary brain c. Chronic traumatic encephalopathy is char-
injury acterized clinically by neurodegeneration
d. The risk of late seizures (post-traumatic
3. Which one of the following statements regar- epilepsy) in those with mild traumatic
ding prognosis in TBI is LEAST accurate? brain injury is thought to be threefold
a. Mortality rate in severe TBI (GCS 3-8) is higher in the next 5 years.
approximately 40% e. Dysfunction of cerebral autoregulation after
b. Hypoxia and hypoglycemia are the extra- an initial concussion is thought to underlie
cranial insults which most strongly affect the risk for second impact syndrome
prognosis after TBI
c. Prognostication based on CT head 6. Which one of the following statements regard-
appearance may be done using Marshall ing return to play of an athlete who has sus-
or Rotterdam classifications tained a concussion is LEAST accurate?
d. Mortality rate in those with mild TBI a. An athlete cannot return to play that day
(GCS 13-15) is <1% overall even once concussion symptoms have
e. Mortality rate at 14 days in those with cleared
GCS 13 and bilaterally reactive pupils b. Risk of a second concussion is particularly
after a significant, isolated head injury increased in the 10 days following the first
may be up to 30% concussion
c. Return to play guidelines are based on risk
4. Which one of the following statements of second impact syndrome
regarding mild traumatic brain injury (GCS d. Have full resolution of their symptoms
13-15) patients presenting to emergency (off medication) and approval by an
departments are LEAST accurate? LHCP to return to play
a. Loss of consciousness is not a requirement e. Neuroimaging should be obtained based
for diagnosis on the presence of risk factors for clini-
cally significant brain injuries
268
Neurosurgery Books Full
www.ketabpezeshki.com 66485438-66485457
21 CRANIAL TRAUMA 269

7. In the UK, which one of the following head a. GCS 3-8 with abnormal CT scan
injury scenarios necessitates a CT scan within b. GCS 3-8 with normal CT scan age
1 h of being identified (NICE head injury >40 years and SBP <90 mmHg
guideline CG176)? c. Postoperative period after removal of
a. More than 30 min of retrograde amnesia acute subdural hematoma
of events immediately before the head d. GCS of 3-8 and diffuse injury type III
injury (Marshall CT classification)
b. Loss of consciousness and dangerous e. Diffuse injury type II (Marshall CT
mechanism of head injury classification)
c. Isolated post-traumatic seizure
d. Amnesia since the injury and history of 12. A 17-year-old boy is struck on the right side
easily bruising (currently on aspirin) of the head during a sports match. He is
e. Loss of consciousness since the head dazed initially and is taken off the field
injury and age >65 years despite saying he feels fine. He becomes
more sleepy over the subsequent 15 min
8. Which one of the following statements and an ambulance is called. In the emergency
regarding CT head scanning in those with department his GCS is E2V3M5 with a
mild head injury is most accurate? sluggish right pupil and weakness on the
a. New Orleans criteria apply to head left side of his body. CT head is shown.
injured patients with GCS 13-15 and loss Which one of the following is LEAST likely
of consciousness with no focal deficit on to be an indication for surgery in this type of
neurological examination pathology?
b. Canadian CT head rule high risk group
includes those with GCS 13-15 and are
aged 70 or older
c. Canadian CT head rule medium risk cri-
teria include dangerous mechanism of
head injury
d. Canadian CT head rule medium risk cri-
teria include two or more episodes of
vomiting
e. New Orleans criteria apply to GCS 15
head injured patients without loss of con-
sciousness and with a normal examination

9. Which one of the following statements


regarding moderate traumatic brain injury
is LEAST accurate?
a. Include head injuries with GCS 9-13
b. 30% chance of having a brain lesion
(intra- or extra-axial)
c. Mortality is around 30%
d. Account for 5-7% of head injury atten-
dances in the emergency department
e. Includes patients who “talk and die”

10. Which one of the following is a biomarker


for traumatic brain injury? a. Lesion volume greater than 30 cm3 with
a. GFAP anisocoria and 20 mm maximal thickness
b. TP53 b. Lesion volume greater than 30 cm3
c. ATRX c. GCS 8 or less with evidence of anisocoria
d. VEGF d. Lesion volume 15 cm3 with anisocoria but
e. IDH-1 no midline shift
e. Lesion volume 35 cm3 with atrophic brain
11. Which one of the following is the LEAST without midline shift
appropriate indication for placement of
ICP monitor in TBI?

Neurosurgery Books Full


www.ketabpezeshki.com 66485438-66485457
270 PART III CRANIAL NEUROSURGERY

13. A 27-year-old presents to ED after an assault.


His GCS is 15/15 but he has evidence of
facial fractures involving the frontal sinus
and evidence of some CSF rhinorrhea. He
is admitted for observation and initial conser-
vative management of CSF leak. On D3 post
injury, he developed three episodes of vomit-
ing and became drowsy. On examination he
was obtunded and lethargic, but arousable.
His BP was 140/90 mmHg, heart rate 59/
min, and respiratory rate 20/min and main-
tained a saturation of 92% on room air.
Examination revealed a dilated right pupil
whereas the rest of neurological and systemic
examination was normal. CT head was
repeated (shown). Which one of the follow-
ing is most appropriate acute management?

a. High flow oxygen


b. Burr hole decompression
c. Cranialization of the frontal sinus
a. Neurological dysfunction or deteriora-
d. Decompressive craniectomy
tion referable to the lesion
e. Minicraniotomy
b. Distortion, dislocation, or obliteration of
the fourth ventricle
14. A 57-year-old patient presents with head
c. Compression or loss of visualization of the
injury after falling backwards and sudden
temporal horns
onset of agitation. Two hours later her blood
d. Presence of obstructive hydrocephalus.
pressure is 220/110 mmHg, her heart rate is
e. Underlying depressed skull fracture
39 beats per minute, and her consciousness is
fluctuating. She is intubated and ventilated.
CT is shown. Regarding this type of lesion
and its location, which one of the following
is LEAST likely to be an indication for sur-
gical evacuation?

Neurosurgery Books Full


www.ketabpezeshki.com 66485438-66485457
21 CRANIAL TRAUMA 271

15. A 19-year-old female is involved in a high for a minor head injury 6 weeks ago. On
speed motor vehicle accident and sustained examination there is no focal deficit. CT
facial injuries from hitting the dashboard. head is shown. In the UK, which one of the
She was GCS 4/15 at the scene and was intu- following is appropriate next management?
bated for transfer. Her pupils are reactive to
light. CT head was performed as part of the
trauma protocol but did not show any mass
lesions or fractures. As this was an isolated
head injury a decision was made to wean
sedation and assess neurology, but she
remained unresponsive. CT is shown below.
These are most likely indicative of which one
of the following?

a. Twist drill craniotomy


b. Single burr hole with or without
subdural drain
c. Human prothrombin complex concentrate
d. Dexamethasone 2 mg twice daily
e. Minicraniotomy and excision of
membranes
a. Subarachnoid hemorrhage 18. A 29-year-old is assaulted and is GCS
b. Diffuse axonal injury E3V4M5 at the scene. On examination there
c. Cortical contusion is significant bruising to the right side of his
d. Global hypoxic brain damage head with a 5-cm laceration and does not
e. Cerebral venous sinus thrombosis appear to have any focal neurological deficit.
He is awaiting a CT scan when you are told
16. Which one of the following statements he is less responsive. On examination, he is
regarding chronic subdural hematomas is GCS E2V3M4, his right pupil is dilated
LEAST accurate? and on application of painful stimulus
a. They form due to separation of the dural appears to only to be moving his left side.
border cell layer Which one of the following is most likely
b. They may arise secondary to acute sub- to be responsible for these findings?
dural hematoma with fails to be resorbed a. Transalar (transsphenoidal) herniation
c. They may arise secondary to repeated b. Unilateral uncal herniation
hemorrhage into a subdural hygroma c. Ascending transtentorial herniation
d. They are prone to forming membranes d. Tonsillar herniation through the foramen
with a tendency to bleed magnum
e. They are more common in females than e. Subfalcine herniation
males at all ages

17. A 72-year-old male develops a worsening


headache over the course of 3 weeks. There
is no relevant past medical history, except

Neurosurgery Books Full


www.ketabpezeshki.com 66485438-66485457
272 PART III CRANIAL NEUROSURGERY

19. A 45-year-old is involved in a high speed road 21. Which one of the following statements
traffic accident and was GCS E2V3M4 at the regarding the lesion shown is LEAST
scene. He was intubated and ventilated and accurate?
transferred to a trauma center. En route,
his left pupil became sluggish and mannitol
was administered. CT head scan is shown.
Given the scenario and type of injury shown,
which one of the following is the LEAST
appropriate indication for surgery according
to current guidelines?

a. Pattern represents about 20% of cases of


moderate TBI cerebral contusions
b. Due to acceleration/deceleration injury
c. Associated with ascending transalar
herniation
d. Decompressive craniectomy would require
cutting the anterior falx
e. Cerebral edema usually peaks between
day 5 and 10
a. Thickness greater than 10 mm
b. Midline shift greater than 5 mm 22. Which one of the following AP diameters of
a decompressive hemicraniectomy flap is the
c. GCS score less than 9, with or without
minimum size thought to prevent local com-
ICP >20 mmHg
d. Documented drop in GCS by 2 or more plications relating to brain herniation?
a. 10 cm
points since injury
b. 12 cm
e. Asymmetric or fixed and dilated pupils
c. 14 cm
20. A 23-year-old female sustains a head injury d. 16 cm
e. 18 cm
after accidentally falling from a motorcycle
at low speed. She is drowsy at scene and say-
ing inappropriate words, but obeys com- 23. Which one of the following statements
regarding decompressive craniectomies is
mands and remained in this state for several
LEAST accurate?
hours in the emergency department. CT
head showed a small amount of convexity a. DECRA trial has shown the utility of
early decompressive craniectomy in
traumatic subarachnoid blood. Which one
neuroprotection
of the following best reflects her risk of devel-
b. DECRA trial utilized decompressive
oping post-traumatic epilepsy?
craniectomy at tier 2 management of ICP
a. <1%
c. Primary decompressive craniectomy is
b. 5%
usually performed during evacuation of
c. 10%
d. 15% an acute subdural hematoma due to con-
cerns regarding brain swelling
e. 20%

Neurosurgery Books Full


www.ketabpezeshki.com 66485438-66485457
21 CRANIAL TRAUMA 273

d. Secondary decompressive craniectomy is For each of the following descriptions, select the
usually undertaken as a last-tier therapy most appropriate answers from the list above.
when a patient has intractable intracranial Each answer may be used once, more than once,
hypertension or not at all.
e. Level I evidence of the effectiveness 1. Assesses changes in the chromophores oxy-
of decompressive craniectomy for refrac- hemoglobin (HbO2), deoxyhemoglobin
tory intracranial hypertension is still (Hb), and cytochrome oxidase
lacking 2. Released from phospholipids
3. Independent marker of increased anaerobic
respiration
QUESTIONS 24–33 4. Gold standard for ICP measurement
5. Intact cerebral autoregulation results in a
Additional questions 24–33 available on
ExpertConsult.com negative value

36. Complications of decompressive cra-


niectomy and cranioplasty:
a. Aseptic bone flap reabsorption
EXTENDED MATCHING ITEM (EMI) b. Bone flap depression/cosmetic defect
c. Bone flap/prosthesis infection
QUESTIONS d. Cerebral abscess
e. CSF leak
34. Management of ICP: f. Extradural/subdural empyema
a. Anticonvulsant g. Fall on unprotected cranium
b. Decompressive craniectomy h. Hemorrhagic progression of contusion
c. External ventricular drain and/or hema- i. Hydrocephalus
toma evacuation j. Meningitis/ventriculitis
d. ICP monitor k. New contralateral/remote hematoma
e. Head up 15° l. New ipsilateral hematoma
f. Hyperosmolar therapy m. Paradoxical herniation
g. Midazolam n. Subdural hygroma
h. Normocapnia o. Superficial complications
i. Normothermia p. Syndrome of the trephined
j. Propofol
k. Thiopentone For each of the following descriptions, select the
most appropriate answers from the list above.
For each of the following descriptions, select the Each answer may be used once, more than once,
most appropriate answers from the list above. or not at all.
Each answer may be used once, more than once, 1. Commonest hemorrhagic complication
or not at all. after decompressive craniectomy
1. Medical therapy only used in highest tier of 2. Commonest overall reported complication
ICP management after decompressive craniectomy
2. Second tier management of raised ICP 3. Commonest complication seen in adult and
3. Surgical procedure considered above tier 3 pediatric autologous cranioplasty
of ICP management
37. Complications after head injury:
35. Continuous Neuromonitoring: a. Carotid-cavernous fistula
a. Brain tissue oxygen tension (PbO2) b. Chronic subdural hematoma
b. Cerebral perfusion pressure (CPP) c. Cranial nerve palsy
c. External ventricular drain ICP d. CSF leak due to skull base fracture
d. Glucose e. Hydrocephalus
e. Glutamate f. Internal carotid artery dissection
f. Glycerol g. Mucocele
g. Invasive microtransducer ICP h. Ossicular disruption
h. Jugular bulb venous oximetry (Svj O2) i. Pituitary dysfunction
i. Lactate/pyruvate ratio j. Pneumocephalus
j. Near infra-red spectroscopy k. Seizures
k. Non-invasive ICP l. Subdural hygroma
l. Pressure reactivity index (PRx) m. Traumatic aneurysm
n. Vasospasm

Neurosurgery Books Full


www.ketabpezeshki.com 66485438-66485457
274 PART III CRANIAL NEUROSURGERY

For each of the following descriptions, select the but did not lose consciousness and not
most appropriate answers from the list above. attend the emergency department
Each answer may be used once, more than once, 2. A 31-year-old presents 2 weeks after sus-
or not at all. taining facial trauma after a motorcycle
1. A 79-year-old presents with a 2-week his- incident which was managed without sur-
tory of difficulty walking and headache. gery. Over the last 5 days, she has noticed
His wife reports that he seems more con- visual disturbance including double vision,
fused over the last 2 days. He has a past redness of the right sclerae. On examina-
medical history of hypertension, and takes tion there is right eye conjunctival injection
aspirin and amlodipine. He sustained a fall with corkscrew scleral vessels and proptosis
in the garden 8 weeks ago and hit his head,

SBA ANSWERS
1. a—22 mild TBI: 1.5 moderate TBI: 1 sample. Additionally, in those that sustained a
severe TBI second head injury the risk of sustaining a third
head injury was 7.8-9.3 times that of an initial
A World Health Organization (WHO) system- head injury in the population. Alcohol is involved
atic review of the mTBI literature found that in one third to two thirds of cases, and 20% of
70-90% of TBI was mild in nature and that those with TBI following motor vehicle
hospital-treated mild TBI was approximately collisions.
100-300 per 100,000 in the studies it reviewed.
However, given the undertreatment and report- FURTHER READING
ing of mild TBI, the WHO estimated that the Leo P, McCrea M. Epidemiology. In: Laskowitz D, Grant G,
true yearly incidence was likely 600 per editors. Translational Research in Traumatic Brain Injury.
100,000. Average estimated incidence of TBI in Boca Raton (FL): CRC Press/Taylor and Francis
the United States 577 per 100,000, of which Group, 2016.
465 per 100,000 were treated in the emergency Hutchinson PJ, Kirkpatrick PJ. Acute head injury for the neu-
department and released, 94 per 100,000 were rologist. J Neurol Neurosurg Psychiatry 2002;73(Suppl 1):i3-i7.
hospitalized and discharged alive, and 18 per
100,000 died. In the UK, every year 1500 per 2. b—Diffuse axonal injury is a type of second-
100,000 of the population attend emergency ary brain injury
departments with a head injury, 225-300 per
100,000 are admitted to hospital, 10-15 per TBI is commonly subdivided into primary and sec-
100,000 are admitted to neurosurgical units and ondary injury. Primary injury results from the
6-10 per 100,000 die from TBI. The aforemen- mechanical forces on the brain from a combination
tioned studies constitute some of the best epide- of direct impact, penetrating injuries and shock
miological data on incidence of TBI, but the data waves, and acceleration/deceleration phenomena.
likely grossly underestimates the incidence of Examples of primary injury are diffuse axonal
mTBI. European epidemiological studies that injury, cerebral contusions/hemorrhage, and
calculated a TBI severity ratio of 22 mild TBI: extra-axial hemorrhage (EDH, SDH, SAH).
1.5 moderate TBI: 1 severe TBI (i.e. 90% of Secondary brain injury is non-mechanical in
TBI is mild). In general, TBI is much more fre- nature and due to local (e.g. cerebral edema,
quent in males than females (1.4:1), highest regional loss of cerebral autoregulation, anaerobic
among young children aged 0-4 (1337 per glycolysis/lactic acidosis, glutamate excitotoxicity)
100,000) and older adolescence aged 15-19 (896 and systemic (e.g. hypoxia, hypotension, hypogly-
per 100,000). Older adults aged 75 and above also cemia) consequences of the primary injury which
have a high rate of TBI (932 per 100,000) and aggravate it further. An intermediate set of pro-
they account for the highest rate of TBI- cesses, such as apoptosis and axonal retraction,
associated hospitalizations (339 per 100,000) occur in a delayed fashion (overlapping with time
and death (57 per 100,000). This pattern of high course of secondary injury) but arise from the pri-
rates of TBI in early childhood, late adolescence, mary injury. In general, primary brain injury is not
and in the elderly has been shown in many treatable (except for hematoma evacuation) and
population-based studies. The relative risk of a its magnitude is a limiting factor distinguishing
second TBI among those with an earlier TBI survivable from non-survivable brain injury. The
was 2.8-3 times greater than the non-injured major focus of TBI management is thus on

Neurosurgery Books Full


www.ketabpezeshki.com 66485438-66485457
21 CRANIAL TRAUMA 275

limiting/preventing secondary brain injury as following injury. MRI of the brain may assist with
much as possible to reduce unfavorable outcomes; prognostication of outcome. However, early
this is achieved by ensuring adequate oxygenation/ prognostication and utility in deciding aggres-
gas transfer, organ perfusion and control of ICP. siveness of treatment is plagued by several factors:
More recently, tertiary injuries (iatrogenic) have Inaccurate GCS recording, alcohol/drug intoxi-
been recognized to contribute to outcome just as cation, sedative drugs, intubation (no verbal
strongly as primary or secondary injury. Examples GCS score), associated trauma preventing assess-
of tertiary injury include complications from ment of GCS (e.g. maxillo-facial injury), and
a prolonged intensive care unit (ICU) stay uncorrected systemic insults (hypotension, hyp-
(e.g. ventilator-associated pneumonia, line sepsis, oxia, hypothermia) which affect consciousness.
decubitus ulcers, or medication administration The IMPACT meta-analysis reviewed reversible
errors), as well as complications of brain treatment insults present on admission and their potential to
per se, such as transfusion-related acute lung influence outcome by exacerbating secondary
injury, post-operative neurosurgical infections, injury. Hypoxia, hypotension (SBP <90 mmHg)
vasopressor-related ischemia, or hyperosmolar and hypothermia (<35 °C) were strongly associ-
renal failure. ated with poor outcomes, with hypoxia and hypo-
tension having synergistic effects. Prognostic
3. b—Hypoxia and hypoglycemia are the extra- models (e.g. IMPACT or CRASH) derived from
cranial insults which most strongly affect large prospective datasets have shown that age,
prognosis after TBI severity of primary injury (measured by GCS,
pupillary reaction, and CT scan appearances such
Functional recovery from TBI is classified on the as traumatic SAH/IVH, cistern effacement, epi-
Glasgow Outcome Scale and is commonly dural masses, midline shift) and major secondary
dichotomized into favorable outcome (5 ¼ good insults including hypotension, hypoxia and hypo-
recovery; 4 ¼ moderate disability/independent) thermia are the principal risk factors for death and
versus unfavorable outcome (3 ¼ severe disabil- long-term neurological morbidity. CT appear-
ity/dependent; 4 ¼ vegetative state; 1 ¼ death). ances alone can also be used for prognostication
The overwhelming majority of patients with mild via Marshall or Rotterdam CT criteria. For illus-
TBI (GCS 13-15) make a good recovery even trative purposes only, CRASH prognostic calcu-
though cognitive deficits and symptoms are com- lators is shown for isolated head injury (bilateral
mon in the acute stage, and overall mortality is reactive pupils, CT evidence of petechial hemor-
<1% (reflects the fact that the vast majority of rhage, SAH, cistern effacement and MLS without
mild TBIs are GCS 15; mortality rate may be hematoma) scenarios below as ranges of outcome
up to 30% in those with GCS 13 and reactive for mild (GCS 13-14), moderate (9-12) and
pupils but significant brain injury/hematoma on severe (3-8) TBI groups. The CRASH score also
CT head). Mortality rate in moderate TBI is incorporates adjustment according to high
approximately 15% overall. Patients presenting income (shown) versus low-middle income coun-
in coma with severe TBI have a 40% mortality tries, and GCS 14 or less. IMPACT only consid-
rate and a further 20% survive with major disabil- ered moderate and severe head injuries (but only
ity. Predicting outcome for an individual patient motor score is put in) and can be used with or
is, however, notoriously difficult and a clear prog- without CT (Marshall) criteria.
nosis often emerges only over the days and weeks

MRC CRASH trial prognosis for a patient with isolated head injury, bilateral reactive pupils, CT
evidence of petechial hemorrhage, SAH, cistern effacement/obliteration of third ventricle and
midlineshift (but without non-evacuated hematoma) in a high income country

GCS Age Risk of 14-day mortality Risk of unfavorable outcome at 6 months


Mild (13-14) 40 years 14.3-16.4% 38-41.9%

75-year-old 66.2-69.8% 87.7-89.4%


Moderate (9-12) 40 years 18.7-27.2% 45.9-58.1%

75-year-old 73.1-81.5% 90.8-94.2%


Severe (3-8) 40 years 30.5-49.7% 62-78.8%
75-year-old 83.8-92.1% 95-97.7%

Neurosurgery Books Full


www.ketabpezeshki.com 66485438-66485457
276 PART III CRANIAL NEUROSURGERY

FURTHER READING generally focus on the need for and timing of neu-
The MRC CRASH Trial Collaborators. Predicting outcome roimaging for which more useful rules incorporat-
after traumatic brain injury: practical prognostic models based ing these features exist (e.g. Canadian CT head
on large cohort of international patients. BMJ 2008;336:425. rules, New Orleans Criteria). In a recent systematic
Predicting outcome after traumatic brain injury: Development review of 23,079 adults presenting with minor head
and international validation of prognostic scores based trauma (GCS 13-15 who appear well on examina-
on admission characteristics. PLoS Med. 2008;5(8):e165. tion). The prevalence of severe intracranial injury
Website: www.tbi-impact.org. (subdural, epidural, ventricular or parenchymal
hematoma, subarachnoid hemorrhage, herniation,
Kolias AG, Guilfoyle MR, Helmy A. Traumatic brain injury
in adults, Pract Neurol. 2013;13(4):228-235.
or depressed skull fracture, small intracranial hem-
orrhages requiring observation in the hospital,
4. c—Clinically significant brain injury is pre- neurosurgical evaluation, or operative interven-
sent in 10-15% of cases tion) was 7.1% (95% CI, 6.8-7.4%) and the prev-
alence of injuries leading to death or requiring
Concussion (mild TBI) is a traumatically induced neurosurgical intervention was 0.9% (95% CI,
alteration in consciousness (confusion, amnesia 0.78-1.0%). In those with abnormal CT head not
with or without an associated loss of conscious- requiring surgery or those with normal CT head
ness) due to a non-penetrating injury. It usually but if GCS <15, seizures or coagulopathy inpatient
occurs immediately following the blow or within observation for 24 h is recommended due to risk of
minutes of it. The majority do not have loss of con- developing intracranial complications (e.g. cerebral
sciousness hence fail to be recognized. Levels of swelling, delayed hematoma) and need for repeat
glutamate rise after concussion and the brain CT head before discharge. In those with normal
enters a hyperglycotic and hypermetabolic state CT head and normal GCS with none or mild
which may persist for 7-10 days after injury (i.e. symptoms may be able to be observed at home
the period after which the vast majority of patients by a responsible adult aware of signs requiring
are symptom free again), and may make the brain immediate medical assessment. Further manage-
more susceptible to a second impact as altered ment on discharge relates to post-concussion
cerebral autoregulation may produce much more syndrome, risk of second impact syndrome (partic-
severe sequelae (malignant cerebral edema resis- ularly return to play guidelines in athletes and
tant to treatment and almost certainly fatal). Mild contraindications to returning to contact sport),
TBI has been classified as GCS of 13-15 (or more risk of post-traumatic epilepsy and, in those with
recently 14-15 depending on series) but the vast multiple concussions, risk of chronic traumatic
majority are initially GCS 15/15 hence other encephalopathy.
features must be used to assess risk of further dete-
rioration due to underlying significant brain injury FURTHER READING
Easter JS, et al. Will Neuroimaging Reveal a Severe Intracra-
requiring treatment or observation (i.e. a propor- nial Injury in This Adult With Minor Head Trauma?: The
tion may actually be occult moderate/potentially Rational Clinical Examination Systematic Review. JAMA.
severe TBI: “talk and die patients” with a lucid 2015;314(24):2672-2681.
interval before deterioration). Clinical grading
systems for concussion based on duration/ 5. e—Dysfunction of cerebral autoregulation
presence of confusion, post-traumatic amnesia after an initial concussion is thought to
and LOC have been used, but key concerns underlie the risk for second impact syndrome

Complications of Concussion (Mild TBI)


Complication Description

Post-concussion Post-concussion symptoms may result from brain injury or from trauma involving head
syndrome and neck structures. These include headache (commonest), dizziness (including vertigo
and nonspecific dizziness), neuropsychiatric symptoms, and cognitive impairment.
These typically develop in the first days after mild traumatic brain injury (TBI) and
generally resolve within a few weeks to a few months. Post-traumatic vertigo may be
due to direct injury to cochlear/vestibular structures, labyrinthine concussion, BPPV,
perilymphatic fistulae, vertebral artery dissection amongst others

Continued

Neurosurgery Books Full


www.ketabpezeshki.com 66485438-66485457
21 CRANIAL TRAUMA 277

Complication Description
Second impact syndrome Diffuse cerebral swelling occurs after a second concussion, while an athlete is still
symptomatic from an earlier concussion. Some have suggested a similarity with this
phenomenon and the shaken baby syndrome. The cause is hypothesized to be
disordered cerebral autoregulation causing cerebrovascular congestion and
malignant cerebral edema with increased intracranial pressure. The second impact
syndrome is a rare and somewhat controversial complication. It is unclear why this is
not a more frequently reported occurrence in boxers who seem at very high risk for
repeated concussions within a short time span

Chronic traumatic Defined as a slowly progressive disease (tauopathy) that takes years to decades
encephalopathy to develop, often providing a significant latent period between when the
neurotrauma occurs and when symptoms develop. It was first described in
Boxers, and is thought to be due to repetitive head injury. Clinical features include
behavioral disturbances such as impulsivity, depression, and lack of oversight,
followed later by cognitive impairment. Pathological criteria include
neurofibrillary tangles (NFTs) in a perivascular distribution and within superficial
cortical areas with occasional amyloid and TDP-43 protein aggregations.
neurotrauma may have many lasting deleterious consequences, including the
potential for increased risk and accelerated development of Alzheimer’s disease
and motor neuron disease
Seizures Mild TBI is associated with a twofold increase in the risk of epilepsy for the first 5 years
after injury. Seizures occurring within the first week of injury are acute symptomatic
events and are not considered epilepsy. Half of the seizures consistent with post-
traumatic epilepsy will occur in the first year; 80% will occur within 2 years.
Prophylactic treatment with anticonvulsants does not prevent post-traumatic
epilepsy and is not recommended

6. c—Return to play guidelines are based on syndrome based on current concepts regarding
risk of second impact syndrome its pathophysiology. More specific guidance is
outlined below:
AAN sports concussion guidelines 2013 advised • Players who experience symptoms suggestive
that any athlete suspected of having sustained a of concussion, such as blurry or double vision,
concussion should be immediately removed from confusion, dizziness, headache, nausea, memory
play to minimize the risk for further injury. The loss, or other cognitive or behavioral problems,
risk of further injury refers to the evidence that a must have full resolution of their symptoms
single concussion predisposes to a second one, (off medication) and approval by an LHCP to
and this risk of a second concussion is particu- return to play. Supplemental neurocognitive
larly increased in the 10 days following the first testing, including comparisons with age-
concussion. The reason for this increased risk for matched normal profiles or a patient's baseline
a second injury is unknown, but given that it profile can be used to aid decision making
mirrors the time taken for >90% of those with • An athlete cannot return to play that day if a
concussion to become symptom-free again (i.e. concussion had been diagnosed, even if symp-
7-10 days) the most likely hypothesis is that toms had cleared. This may be inadvertently
impaired cognition or physical reflexes due to circumvented if the athlete hides their symp-
the first concussion increase the player's suscep- toms and has a normal examination, or player
tibility to injury. Due to a lack of robust evidence with a witnessed head injury whose concussive
regarding their respective mechanisms concerns symptoms don’t appear until after the game
regarding the risks of repetitive head injury (sec- (who would have already been exposed to the
ond impact syndrome or chronic traumatic risk of a second impact)
encephalopathy) are not the basis for current • Concussion is a clinical diagnosis. In a propor-
recommendations, although the general prin- tion of those with concussion, a CT head
ciple of avoiding a second impact while still should be obtained based on the presence of
symptomatic from the first may be necessary risk factors (e.g. Canadian CT head rules,
but not sufficient to avoid second impact New Orleans criteria) to rule out clinically

Neurosurgery Books Full


www.ketabpezeshki.com 66485438-66485457
278 PART III CRANIAL NEUROSURGERY

important brain injury requiring admission to FURTHER READING


hospital for observation or neurosurgery Giza CC, Kutcher JS, Ashwal S. Summary of evidence-based
• Athletes with multiple concussions and contin- guideline update: evaluation and management of concussion
ued impairment, should undergo formal neu- in sports: report of the Guideline Development Subcommittee
rologic and cognitive assessment and be of the American Academy of Neurology, Neurology. 2013;80
(24):2250-2257.
counseled on the risk for developing chronic
neurobehavioral or cognitive impairment and
retirement recommended (lower threshold 7. c—Isolated post-traumatic seizure
for professional vs. amateur athletes)

UK NICE Guidelines for CT Head Scan in Patients Presenting with TBI


Risk Factors for Clinically Significant Brain Injury in Those
With Head Injury Actions Required

GCS less than 13 on initial assessment in the emergency CT head scan within 1 h of any one risk factor
department being identified
GCS less than 15 at 2 h after the injury on assessment in the
emergency department
Suspected open or depressed skull fracture
Any sign of basal skull fracture (hemotympanum, “panda” eyes,
cerebrospinal fluid leakage from the ear or nose, Battle's sign)
Post-traumatic seizure
Focal neurological deficit
More than 1 episode of vomiting

If some loss of consciousness or amnesia since the injury and CT head scan within 8 h of head injury
at least one of the following risk factors:
Age 65 years or older
Any history of bleeding or clotting disorders
Dangerous mechanism of injury (a pedestrian or cyclist
struck by a motor vehicle, an occupant ejected from a motor
vehicle or fall from a height >1 m or 5 stairs)
More than 30 min of retrograde amnesia of events immediately
before the head injury
Head injury on warfarin (even without other risk factors) CT head scan within 1 h

FURTHER READING “raccoon” eyes, CSF otorrhea/rhinorrhea, Battle's


National Institute for Health and Clinical Excellence. Head sign); (4) two or more episodes of vomiting; and
injury (2014) NICE guideline CG176. (5) patient is 65 years of age or older. CT head is also
8. c—Canadian CT head rule medium risk cri- recommended in patients in the medium-risk cate-
teria include dangerous mechanism of head gory who may have clinically important brain inju-
injury ries that may require admission: (1) greater than
30 min of retrograde amnesia or (2) injury via a
Many physicians follow a set of criteria for selecting “dangerous mechanism” (e.g. motor vehicle acci-
which patients should receive a head CT following dent versus pedestrian, ejection from motor vehicle,
mild TBI (concussion). The Canadian CT Head fall from greater than 3 feet or down five or more
Rule (CCR) was derived from a study of 3121 stairs). The New Orleans criteria for CT head in
patients presenting to 10 Canadian hospitals with mild TBI applies to emergency department patients
a GCS score of 13-15/15 after head injury (exclud- with a GCS 15/15 only with LOC and a normal neu-
ing those <16 years, bleeding disorders or warfarin, rological examination. In this situation, CT head
obvious open skull fracture). CT head is mandatory should be performed if any one of the following risk
if one or more of the following high-risk criteria for factors present: age >60 years, headache, vomiting,
neurosurgical intervention are present: (1) GCS drug/alcohol intoxication, persistent anterograde
score less than 15 at 2 h after head injury; (2) sus- amnesia, post-traumatic seizure, and evidence of
pected open or depressed skull fracture; (3) any sign trauma above the clavicles. In a recent systematic
of basal skull fracture (e.g. hemotympanum, review of 23,079 adults presenting with minor head

Neurosurgery Books Full


www.ketabpezeshki.com 66485438-66485457
21 CRANIAL TRAUMA 279

trauma (GCS 13-15 who appear well on examina- was 14 and the most frequent intracranial injuries
tion). The prevalence of severe intracranial injury were acute subdural hematoma, diffuse cerebral
(requiring prompt intervention) was 7.1% (95% edema and cerebral contusion. In about 14% of
CI, 6.8-7.4%) and the prevalence of injuries leading these patients with GCS 13 at admission, initial
to death or requiring neurosurgical intervention CT was normal but became abnormal during
was 0.9% (95% CI, 0.78-1.0%). Features most pre- hospitalization, especially because of develop-
dictive of severe intracranial injury on CT were ment of diffuse cerebral edema. Among the most
examination findings suggestive of skull fracture, important factors relating to death are: delays in
GCS score 13/15, 2 or more vomiting episodes, diagnosis of lesion through CT scan (as initially
any decline in GCS and pedestrians struck by motor appear well), delays in the transfer to a specialized
vehicles. Absence of any of the features of the Cana- center, failure to identify risk factors for deterio-
dian CT Head Rule (high and medium risk) lowered ration, inadequate prevention of secondary
the probability of severe injury to 0.31% (95% CI, injury, inappropriate correction of underlying
0-4.7%). The absence of any New Orleans Criteria coagulopathy and loss of the opportunity for
findings lowered the probability of severe intracra- definitive neurosurgical treatment.
nial injury to 0.61% (95% CI, 0.08-6.0%).
10. a—GFAP
FURTHER READING
Pardini J, Bailes JE, Maroon JC. Mild Traumatic Brain In future, early biomarkers may facilitate deci-
Injury in Adults and Concussion in Sports. In Winn HR sions to perform CT head in mild TBI (e.g.
(ed), Youmans Neurological Surgery, 6th ed., Elsevier, improving early identification of patients likely
Saunders, 2011. to “talk and die”) and later biomarkers to predict
9. c—Mortality is around 30% prolonged complications or to monitor TBI
recovery.
TBI with GCS scores of 9-12 is considered mod- Numerous candidate biomarkers have proven
erate, though some now consider those with a prognostic value with TBI outcome, such as: glial
GCS of 13 within this category too given that a fibrillary acidic protein (GFPA; glial cell injury),
third have abnormal CT head findings. They unbiquitin C-terminal hydrolase-L1 (UCH-L1;
account for 5-7% of head injury attendances in neuronal cell body injury), SBDP150/SBDP145
the emergency department (22 mild: 1.5 moder- (spectrin breakdown products; axon and presyn-
ate: 1 severe) and affects the young adult popula- aptic terminal necrosis), S100B (Calcium-
tion involved in traffic accidents, is associated Binding Protein B; elevated blood and urine
with alcohol or illicit drugs, and with extracranial levels in glial injury), and NSE (neuron-specific
injuries. Individuals with moderate TBI have enolase; neuronal injury). However, they gener-
approximately a 30% chance of having a brain ally show low specificity or sensitivity when used
lesion (intra- or extra-axial), a 30% chance that individually hence combining biomarkers into a
such injuries progress in their volume or mass screening panel may provide more information
effect (new bleeding, rebleeding, edema) and a than individual biomarkers (e.g. GFAP/UCH-
30% chance that these individuals suffer deterio- L1, NSE/S100B).
ration or worsening in their neurological status.
Mortality in moderate TBI is around 15%, FURTHER READING
>50% have cognitive sequelae and only 20% Acute biomarkers of traumatic brain injury: relationship
recover without significant disability. Most “talk between plasma levels of ubiquitin C-terminal hydrolase-L1
and die” patients (i.e. lucid interval; patients pre- and glial fibrillary acidic protein. J Neurotrauma. 2014;31
senting with a verbal GCS score 3 who were (1):19-25.
thought to have sustained a survivable head injury Brain injury biomarkers may improve the predictive power of
who later deteriorate and die to due potentially the IMPACT outcome calculator. J Neurotrauma. 2012;29
treatable head injury) should also be in the mod- (9):1770-1778.
erate TBI category. The authors concluded that
morbidity and mortality in these patients might Neuron-specific enolase and S100BB as outcome predictors in
be reduced by early diagnosis and more aggres- severe diffuse axonal injury. J Trauma Acute Care Surg.
2012;72(6):1654-1657.
sive treatment of raised ICP. More recent studies
have shown that patients who “talk and die” are
11. e—Diffuse injury type II (i.e. cisterns present
most frequently adult men and the most common
mechanisms of trauma are falls, motor vehicle with midline shift <5 mm and/or lesion deni-
sites present, no high or mixed density lesion
accidents and violence. In these studies, the aver-
>25 ml)
age GCS at admission to emergency department

Neurosurgery Books Full


www.ketabpezeshki.com 66485438-66485457
280 PART III CRANIAL NEUROSURGERY

The principle of ICP monitoring is to maintain FURTHER READING


adequate cerebral perfusion and oxygenation to Godoy DA, Rubiano A, Rabinstein AA, et al. Moderate
meet metabolic demands. Raised ICP reduces Traumatic Brain Injury: The Grey Zone of Neurotrauma,
CPP and CBF, which exacerbates secondary Neurocrit Care. 2016 Feb 29.
injury. Several studies have shown that patients
with an ICP <20 mmHg have a reduced risk of 12. e—Lesion volume 35 cm3 with atrophic
neurological deterioration, compared with higher brain without midline shift
ICPs (ICP 25 mmHg). Mortality also increases
dramatically from 17% to 47% when an average The clinical scenario is of a right frontotemporal
ICP >20 mmHg. However, Level I and level II EDH with a swirling appearance on imaging due
evidence suggesting ICP monitoring improves to leakage of serum from clot or active bleeding.
outcome in TBI is lacking. In one randomized EDH is seen in 2.7-4% of traumatic brain injury,
trial of ICP versus clinical examination plus imag- and 9% of severe TBI. Mortality approximates
ing, there was no significant improvement in sur- 10%. Peak incidence of EDH is in the second
vival between the study groups. Similar findings decade, and the mean age of patients with EDH
have been confirmed by other studies that have is between 20 and 30 years of age. EDH can result
concluded that ICP-targeted/CPP-targeted from injury to the middle meningeal artery
intensive care tends to prolong mechanical venti- (90%), the middle meningeal vein, the diploic
lation and increases therapy intensity without evi- veins, or the venous sinuses. Venous epidural less
dence of improved outcome. Nonetheless, ICP common—usually post fossa, usually pediatric,
monitoring is recommended for the management can extend across tentorium to be both supra
of severe TBI (GCS 3-8) with abnormal CT scan, and infratentorial (transverse/sigmoid sinus),
and in patients with normal CT scan but >2 of paramedian/vertex (superior sagittal), middle cra-
age >40 years, SBP <90 mmHg, decorticate pos- nial fossa floor (sphenoparietal sinus). Injury can
turing, decerebrate posturing. Indications in result in pseudoaneurysm of meningeal artery or
those with moderate TBI are less clear, but a dural AV fistula if both artery and vein lacer-
include: (a) Postoperative period after removal ated. Presentation is with coma (one third to a
of acute subdural hematoma or multiple cerebral half), lucid interval (less than a half) and the
contusion. In these cases, sudden changes in ICP remainder remain conscious. Pupillary abnor-
could signal hemorrhages due to decompression malities are present in approximately 20-40%.
or reperfusion, new extra-axial collections or The hematoma volume can be estimated quickly
worsening brain swelling. (b) GCS of 9-11 and from the head CT scan by using the formula
cerebral contusion (temporal or bifrontal) with- A  B  C/2, which approximates the volume of
out surgical intervention. In these instances, an ellipsoid. On the CT slice with the largest area
ICP monitoring can help recognize progression of hemorrhage, A is the greatest hemorrhage
of the contusions. (c) Diffuse injury type III (Mar- diameter and B is the largest diameter perpendic-
shall CT classification: cisterns compressed or ular to A. Giving each slice a value of 1 or 0.5 if
absent with MLS <5 mm, no high or mixed den- the area is >75% or 25-75% of the index slice
sity lesion >25 ml). Due to the high probability used to measure A and B, their sum is multiplied
of intracranial hypertension and poor outcome, by slice thickness in cm to give C. An epidural
ICP monitoring in these cases is indispensable. hematoma (EDH) greater than 30 cm3 should
(d) General anesthesia for emergency non-cranial be surgically evacuated regardless of the patient's
surgery (especially in the presence of conserva- Glasgow Coma Scale (GCS) score. An EDH less
tively treated intracranial lesions) due to loss than 30 cm3 and with less than a 15-mm thickness
of clinical evaluation and potential effects of and with less than a 5-mm midline shift (MLS) in
anesthetics on cerebrovascular autoregulation. patients with a GCS score greater than 8 without
(e) Concomitant severe chest trauma requiring focal deficit can be managed non-operatively with
deep sedation, high PEEP levels, recruitment serial computed tomographic (CT) scanning and
maneuvers or prone ventilation which may cause close neurological observation in a neurosurgical
hypercapnia or impair cerebral venous return, center, but enlargement occurs in 20% of cases. It
causing cerebral vasodilation and increased ICP. is strongly recommended that patients with an
(f) Concomitant intra-abdominal compartment acute EDH in coma (GCS score <9) with aniso-
syndrome (associated with intracranial hyperten- coria undergo surgical evacuation as soon as pos-
sion). (g) Prolonged traumatic shock (risk of sible (i.e. within 1 h). Bilateral EDH is
cerebral edema). comparatively uncommon entity, accounting for
approximately 2-5% of adults with extradural

Neurosurgery Books Full


www.ketabpezeshki.com 66485438-66485457
21 CRANIAL TRAUMA 281

hematoma. Kett-White and Martin classified Image from Jakherea, SG, Yadava DA, Jaina, DG, Bala-
such cases into two distinct groups: patients with subramaniamb, S; Does the Mount Fuji Sign always sig-
bilateral but separate extradural hematoma nify “tension” pneumocephalus? An exception and a
reappraisal, European Journal of Radiology Extra,
located in the convexities, resulting from dura 2011;78(1): e5-e7.
being stripped from the skull independently on
both sides, either simultaneously or sequentially; 14. c—Compression or loss of visualization of
and, more rarely, patients with bilateral extra- the temporal horns
dural hematoma straddling the midline, resulting
from injury to the sagittal sinus. Although all The clinical scenario suggests a Cushing’s reflex
identified cases with bilateral extradural hema- secondary to an expanding right sided posterior
toma resulting from sagittal sinus injury under- fossa extradural hematoma, most probably due
went urgent surgical evacuation, the operative to transverse sinus laceration, and evidence of
technique varied regarding preserve the midline 4th ventricular effacement and obstructive
skull vault and use dural tenting sutures, or—in hydrocephalus. Upward herniation of cerebellar
selected cases—expose the sagittal sinus and vermis through the tentorial incisura is also seen.
attempt primary repair. Indications for surgery include neurological
dysfunction or deterioration referable to the
Image with permission from Kirmi O, Sheerin F, lesion, or presence of mass effect: distortion, dis-
Patel N, Imaging of the meninges and the extra-
axial spaces, Semin Ultrasound CT MR. 2009;30 location, or obliteration of the fourth ventricle;
(6):565–593. compression or loss of visualization of the basal
cisterns, or the presence of obstructive hydro-
FURTHER READING cephalus. Evacuation via suboccipital craniect-
Bullock MR, Chesnut R, Ghajar J, et al. Surgical management omy should be performed as soon as possible
of acute epidural hematomas. Neurosurgery. 2006;58(3 because these patients can deteriorate rapidly,
Suppl):S7-S15; discussion Si-Sv. Review. thus, worsening their prognosis. Patients with
Bimpis A, Marcus HJ, Wilson MH. Traumatic bifrontal extra- lesions and no significant mass effect on CT scan
dural haematoma resulting from superior sagittal sinus injury: and without signs of neurological dysfunction
case report. J R Soc Open 2015;6. may be managed by close observation and serial
imaging.
13. b—Burr hole decompression
Image from Naidich T, Castillo M, Cha S. Smirniotopou-
Pneumocephalus is the presence of intracranial los J, Imaging of the Brain, Elsevier, Saunders, 2013.
air, which invariably resolves spontaneously or
with conservative treatment. However, clinical FURTHER READING
deterioration can occur in tension pneumocepha- Bullock MR, Chesnut R, Ghajar, J et al. Surgical management
lus where a progressive accumulation of intracra- of posterior fossa mass lesions. Neurosurgery. 2006;58(3
Suppl):S47-S55; discussion Si-Siv. Review.
nial air which cannot escape (ball valve
mechanism) exerts mass effect on the brain and
can lead to coma, herniation and death. Tension 15. b—Diffuse axonal injury
PC is commonly caused by intra or extra-cranial
Diffuse axonal injury is the most common cause
surgeries like drainage of chronic subdural
of coma in the head-injured patient without an
hematoma, craniofacial surgery, otorhinolaryn-
intracranial mass lesion. It is characterized path-
gological procedures, shunt operations/CSF
ologically by diffusely spread axonal swellings
drainage, trauma (fractures through skull base
affecting the white matter, corpus callosum, and
involving air sinuses), meningitis/otitis, anesthesia
upper brainstem. These foci are usually hemor-
(spinal, ventilation, nitric oxide induced), and
rhagic. The etiology is thought to be due to
tumors. Accumulation of trapped air in subdural
shearing forces on axons in certain susceptible
and interhemispheric space bilaterally, is seen as
regions of the brain, notably those that are partic-
hypodense collections causing compression and
ularly vulnerable to rotational forces, such as the
separation of the frontal lobes (Mount Fuji sign).
subcortical white matter, corpus callosum, and
Emergency management is by decompression,
upper brainstem. On MRI the spots will appear
usually by opening up previous burr hole (if
as T2 bright lesions and appear dark on GRE.
present) or making a new one. Definitive
Classically, DAI has been considered to be a
management will involve identifying the site of
major neuropathological feature of severe TBI,
air entry and intracranial or extracranial repair.
but most experts now agree that almost all TBI

Neurosurgery Books Full


www.ketabpezeshki.com 66485438-66485457
282 PART III CRANIAL NEUROSURGERY

is usually accompanied by some degree of DAI FURTHER READING


and brain volume loss. There have been no Kolias AG, Chari A, Santarius T, Hutchinson PJ. Chronic
evidence-based effective treatments specifically subdural haematoma: modern management and emerging
for DAI, hence avoidance of secondary brain therapies, Nat Rev Neurol. 2014;10(10):570-578.
injury, and medial and/or surgical management
of ICP are the mainstay. 17. b—Single burr hole with subdural drain

Image from Haaga JR, et al. (Eds). CT and MRI of the Medical management of Subacute SDH or
Whole Body, 5th ed., Elsevier, Mosby, 2009. CSDH generally involves correction of bleeding
disorders/anticoagulation and any interventions
16. e—They are more common in females than required to improve anesthetic fitness (e.g. treat
males at all ages infections, optimize gas exchange, etc.). Steroids
may be useful as adjunctive therapy preopera-
A chronic subdural hematoma (CSDH) is a col- tively or postoperatively, or as monotherapy as
lection of blood breakdown products in the sub- an alternative to surgical intervention (in small,
dural space. Estimates of the incidence of CSDH mildly symptomatic CSDH), but their role
range from 8.2 to 14.0 per 100,000 person-years. remains ill-defined in the absence of level I evi-
Presentation is predominantly in the seventh dence. Preoperative antiepileptic prophylaxis
decade onwards and is more frequent in males has been shown to affect postoperative seizure
(approximately 3:1 male to female ratio across rate but not discharge outcome, hence is not par-
all age groups). CSDH arises at the dural border ticularly advocated. In patients with mild or no
cell layer (between the dura mater and arachnoid symptoms and relatively small collections, or
mater) which is prone to separation creating a moribund patients with poor baseline function
“subdural space,” particularly in those with cere- may both be managed non-operatively. However,
bral atrophy (e.g. elderly, alcoholics). Minor pre- surgical treatment of symptomatic CSDH results
cipitant trauma can generate CSDH either from in rapid improvement of symptoms, and produces
the incomplete breakdown/reabsorption of a favorable outcome in over 80% of patients.
ASDH or recurrent microhemorrhage into sub- Coupled with relatively low surgical risk,
dural hygromas. ASDH may occur due to tearing surgical evacuation is currently the mainstay of
of the bridging veins that traverse the dural bor- management for symptomatic patients. Three
der cell layer or, less commonly, tearing of corti- primary surgical techniques are used: twist drill
cal arteries or veins. In contrast, a subdural craniostomy (TDC) involving small openings
hygroma (CSF collection), is caused by splitting (<10 mm) made using a twist drill, burr hole cra-
of the dural border cell layer at points of tension niostomy (BHC; most popular) involving open-
between the dura mater and arachnoid mater. ings of 10-30 mm, and craniotomy involving
Neovascularization occurs in the newly formed larger openings. The BHC technique involves
subdural space and hemorrhage from these new the drilling of burr holes over the cerebral con-
vessels leads to the formation of a CSDH. In both vexity followed by durotomy, usually under gen-
cases, the opening of the dural border cell layer eral anesthetic. Insertion of a subdural drain in
into a subdural space triggers a reparatory the anterior hole connected to a closed drainage
response which either causes hematoma/ system for approximately 48 h decreased CSDH
hygroma resolution or hematoma enlargement. recurrence after BHC from 24.3% to 9.3%
The latter is thought to be due to a localized (P ¼ 0.003) and 6-month mortality from 18.1%
inflammatory reaction, which results in hyperfi- to 9.6% (P ¼ 0.042). Twist drill craniostomy can
brinolysis of the clot and production of angio- be performed under local anesthesia either at
genic factors that promote neovascularization the bedside or in the operating theatre, making
and further bleeding from fragile capillaries. For- it an attractive option for the elderly patient with
mation of neomembranes is one of the main fea- multiple comorbidities. A recent modification to
tures of CSDH—the inner (visceral) membrane is the original technique involves the insertion of a
less vascular and usually thinner than the outer hollow screw to set up a closed drainage system.
(parietal) membrane. Risk factors for CSDH This technique, unlike the traditional TDC tech-
include advancing age, a history of falls, minor nique, does not require the blind insertion of a
head injury, use of anticoagulants or antiplatelet catheter in the subdural space, thereby minimiz-
drugs, bleeding diatheses, alcohol (contributing ing the risks of brain laceration and bleeding from
to globalized brain atrophy, increased risk of falls, cortical vessels. Craniotomy involves general
and hepatogenic coagulopathy), epilepsy, intra- anesthesia and is commonly reserved for recur-
cranial hypotension, and hemodialysis. rent CSDH with extensive organization and

Neurosurgery Books Full


www.ketabpezeshki.com 66485438-66485457
21 CRANIAL TRAUMA 283

membrane formation (preventing brain re- diffuse brain swelling (e.g. hypoxic brain injury)
expansion) or primary evacuation of a CSDH that is likely to cause a more symmetrical uncal herni-
has a substantial acute component. Systematic ation without this phenomenon. With midbrain
reviews comparing the three techniques have caudal compression, there is progressive narrow-
generally found either no difference between ing of the normal angle ( 90°) between the ven-
TDC and BHC, or lower recurrence rates with tral midbrain and pons with subsequent
BHC and craniotomy, higher morbidity with cra- compression of basilar artery perforators and
niotomy (and in one review BHC), and a higher potentiation for secondary midbrain hemor-
mortality with craniotomy. rhagic infarct—Duret hemorrhage. Ascending
transtentorial cerebellar herniation through the
Image with permission from Takemotoa, Y, Hashigu- tentorial notch due to a mass lesion in the poste-
chib, A, Morokib, K et al. Chronic subdural hematoma rior fossa causes rapid loss of consciousness, non-
with persistent hiccups: A case report, Interdisciplinary
Neurosurgery, 2016;3:1-2. communicating hydrocephalus and PCA/SCA
territory infarction. Descending transalar hernia-
tion occurs as a result of frontal lobe mass effect
FURTHER READING with posterior and inferior displacement of the
Kolias AG, Chari A, Santarius T, Hutchinson PJ. Chronic
subdural haematoma: modern management and emerging posterior aspect of the frontal lobe orbital surface
therapies, Nat Rev Neurol. 2014;10(10):570-578. over the sphenoid wing and MCA compression.
Ascending transalar herniation is produced by
Santarius T et al. Use of drain versus no drains after burr hole middle cranial fossa or temporal lobemass effect
evacuation of chronic subdural haematoma: a randomised resulting in displacement of the temporal lobe
controlled trial. Lancet. 2009;364:1067-1073. superiorly and anteriorly across the sphenoid
ridge causing compression of the ICA. Tonsillar
18. b—Unilateral uncal herniation cerebellar herniation though the foramen mag-
num may be chronic/congenital (e.g. Chiari I
Subfalcine herniation is the commonest type of malformation), due to primary posterior fossa
herniation patter as it is measured by midline shift mass lesions or secondary to descending transten-
of the septum pellucidum, and reflects cingulate torial herniation from a supratentorial mass
gyrus, anterior cerebral artery, internal cerebral lesion; compression of the brainstem against the
vein and lateral ventricle compression. Complica- clivus results in compression of respiratory and
tions of subfalcine herniation are contralateral cardiac centers.
lateral ventricle hydrocephalus due to obstruction
of the foramen of Monro, and ipsilateral ACA 19. c—GCS score less than 9, with or without
compression against the falx resulting in infarc- ICP >20 mmHg
tion and contralateral leg weakness. Following
subfalcine herniation, descending transtentorial Acute SDH complicates 11% of all TBI, and 10-
herniation is the second most common type of 30% of severe TBI. Most SDH are caused by
herniation syndrome and may occur unilaterally motor vehicle-related accidents (MVA), falls,
or bilaterally. With increasing supratentorial and assaults. The elderly population is particu-
mass effect, the uncus of the anteromedial tempo- larly susceptible due to increased fragility of ves-
ral lobe is driven medially to efface the suprasellar sel walls, falls and greater use of antithrombotic
cistern (uncal herniation). Medial shift of the hip- and anticoagulants agents. Between 37% and
pocampus and parahippocampal gyrus then fol- 80% of patients with acute SDH present with ini-
lows with subsequent effacement of the tial GCS scores of 8 or less. A lucid interval has
mesencephalic and quadrigeminal cisterns and been described in 12-38% of patients before
compression of the midbrain. Kernohan’s notch admission but there is no conclusive evidence that
phenomenon is an uncommon but important sce- this correlates with outcome. The definition of
nario described by this case where a unilateral lucid interval is vague. Authors interpret the lucid
hemispheric expanding mass lesion causing ipsi- interval differently and analysis of its frequency
lateral uncal herniation (descending transtentor- requires documentation during the prehospital
ial herniation; associated with 3rd nerve palsy) phase. Pupillary abnormalities are observed in
results in compression of the contralateral cere- 30-50% of patients on admission or before sur-
bral peduncle against the tentorium cerebelli cre- gery. Compared with EDH, the degree of under-
ating a “notch” in it. This leads to the confusing lying brain damage associated with ASDH is
picture of hemiparesis on the same side as the more severe, and mortality rates are greater
mass lesion (false localizing sign). By contrast, especially in older patients with poor initial

Neurosurgery Books Full


www.ketabpezeshki.com 66485438-66485457
284 PART III CRANIAL NEUROSURGERY

GCS, and other associated brain or systemic inju- contusion, depressed skull fracture) as it has been
ries. Studies looking at patients from all age groups shown to reduce the incidence of early seizures;
with GCS scores between 3 and 15 with SDH however, there is no evidence supporting the
requiring surgery quote mortality rates between use of long-term prophylaxis to prevent post-
40 and 60%. Mortality among patients presenting traumatic epilepsy.
to the hospital in coma with subsequent surgical
evacuation is between 57% and 68%. Only 30- 21. c—Associated with ascending transalar
40% of SDH requiring surgery are isolated herniation
lesions. In the majority of cases, the SDH is asso-
ciated with other intracranial and extracranial inju- Bifrontal contusions are present in about 20% of
ries. An acute subdural hematoma (SDH) with a cases of moderate TBI with cerebral contusion
thickness greater than 10 mm or a midline shift and are due to acceleration/deceleration forces
greater than 5 mm on computed tomographic pushing the inferior frontal lobes and temporal
(CT) scan should be surgically evacuated, regard- pole against the irregular skull base. Hemor-
less of the patient's Glasgow Coma Scale (GCS) rhagic swelling of these contusions may disrupt
score. All patients with acute SDH in coma the median forebrain bundle, gyrus rectus, and
(GCS score less than 9) should undergo intracra- anterior hypothalamic nuclei. These structures
nial pressure (ICP) monitoring. A comatose are involved in behavior control and are associ-
patient (GCS score less than 9) with an SDH less ated with changes in personality, volition, motiva-
than 10 mm thick and a midline shift less than tion, judgment, and social interactions. When
5 mm should undergo surgical evacuation of the these contusions swell, the brain is displaced pos-
lesion if the GCS score decreased between the teriorly (descending transalar herniation) and
time of injury and hospital admission by 2 or more abrupt deterioration because of descent of the
points on the GCS and/or the patient presents brain stem into the posterior fossa with stretching
with asymmetric or fixed and dilated pupils and/ and deformity of the small perforating blood ves-
or the ICP exceeds 20 mmHg. In patients with sels of the basilar artery with risk of death due to
acute SDH and indications for surgery, surgical respiratory arrest, sudden coma, and autonomic
evacuation should be performed as soon as possi- changes. These lesions are also very often associ-
ble, with or without bone flap removal and dura- ated with disturbances of sodium and water
plasty. Patients with less severe ASDH can be metabolism (e.g. diabetes insipidus or SIADH).
monitored clinically; after 7-10 days an ASDH Aggressive surgical resection of these contusions
may liquefy, to become drainable with burr holes, may worsen the late neurological deficit and neu-
thus avoiding the major morbidity of craniotomy. ropsychological consequences. Surgical decom-
pression requires bifrontal decompressive
Image from Fujimoto K, Otsuka T, Yoshizato K, Kuratsu craniotomy and cutting of the falx and sagittal
J. Predictors of rapid spontaneous resolution of acute sinus on the frontal cranial fossa. This is a major
subdural hematoma, Clin Neurol Neurosurg. procedure, caries hemorrhage risk, and requires
2014;118:94-97.
delayed cranioplasty 2-3 months later. The risk
FURTHER READING of this surgical procedure must, therefore, be bal-
Bullock MR, et al. Surgical management of acute subdural anced against risk of death due to herniation. The
hematomas. Neurosurgery. 2006;58(3 Suppl):S16-S24; timing of deterioration is variable, but brain
discussion Si-Siv. Review. edema will usually peak around the 5th to the
10th day, with resolution of the swelling after this
20. b—5% time hence observation for up to 2 weeks with
serial CT scanning every 2-3 days is necessary
Post-traumatic seizures are classified into early to exclude progression of the lesions. When the
(<1 week post-injury) and late (>1 week post- patients are unable to obey commands, are very
injury). Generally children are more likely to restless, or deteriorate, ICP monitoring with
develop early seizures, whereas adults are more titrated osmotherapy may be an option with
likely to develop post-traumatic epilepsy. Overall, decompression the next stage. Particular prob-
the rate of post-traumatic epilepsy after TBI is lems are posed by the patient with progressive
approximately 2% over the subsequent 10 years. swelling, especially posterior shift of the third
However, frequency increases with severity of ventricle, but preserved capacity to obey com-
TBI: the risk is only marginally higher than the mands. For these patients, prophylactic de-
general population in mild TBI, less than 5% in compressive bifrontal craniectomy is usually
moderate TBI and affects 10-15% of patients preferable to the risk of sudden death or perma-
after severe TBI. Short-term anti-epileptic pro- nent disability that can result from rapid
phylaxis for high-risk cases (e.g. cortical herniation.

Neurosurgery Books Full


www.ketabpezeshki.com 66485438-66485457
21 CRANIAL TRAUMA 285

Image from Vincent JL, et al. (Eds.), Textbook of Critical shorter operating time, a lower incidence of struc-
Care, 6th ed., Elsevier, Saunders, 2011. tural laceration or transfusion requirements.
Equally, an additional benefit for an exposure that
FURTHER READING exceeds 15 or 18 cm could not be found. A small
Kolias AG, Guilfoyle MR, Helmy A. Traumatic brain injury in
adults, Pract Neurol. 2013;13(4):228-235.
proportion of craniectomies extended over the
sagittal midline and did not result in a statistically
Godoy DA, Rubiano A, Rabinstein AA, et al. Moderate Trau- significant increase in structural laceration or
matic Brain Injury: The Grey Zone of Neurotrauma, Neuro- development of hygroma, but were associated
crit Care. 2016 Feb 29. with significantly more contusions/hemorrhages,
higher rate of meningitis and shunt-dependency,
22. b—12 cm but also with better GOS outcome.
Unilateral DC (also termed hemicraniectomy) is FURTHER READING
usually performed in cases with predominantly Wagner S, Schnippering H, Aschoff A, et al. Suboptimum
unilateral hemispheric edema—a feature that is hemicraniectomy as a cause of additional cerebral lesions in
evident on brain imaging as a midline shift to patients with malignant infarction of the middle cerebral
the contralateral side. Bifrontal DC, which artery, J Neurosurg. 2001;94(5):693-696.
extends from the floor of the anterior cranial fossa Tanrikulu L, Oez-Tanrikulu A2, Weiss C, et al. The bigger,
anteriorly to the coronal suture posteriorly and to the better? About the size of decompressive hemicraniec-
the pterion laterally, is usually performed in tomies, Clin Neurol Neurosurg. 2015;135:15-21.
patients with diffuse brain edema. Removal of
the inferior part of the temporal bone to the floor 23. a—DECRA trial has shown the utility of early
of the middle cranial fossa is an important maneu- decompressive craniectomy in neuroprotection
ver for both types of DC, especially in the pres-
ence of temporal pole lesions or edema causing In the modern era of TBI management, DC can
brainstem compression. It is now well recognized be grouped into two major categories: primary or
that, during DC, the dura mater has to be widely secondary. Primary decompressive craniectomy
opened as bony decompression alone cannot is usually performed during evacuation of an
sufficiently accommodate severe brain swelling. acute subdural hematoma (ASDH), either
Leaving the dura open while covering the brain because the brain is swollen beyond the confines
with a sheet of hemostatic material (such as of the skull or because the patient is thought to be
Surgicel®, Ethicon Inc., Somerville, NJ) is our pre- at high risk of worsening of brain swelling within
ferred option as it allows for faster closure with a the ensuing few days. Secondary decompressive
low chance of complications. If a duraplasty is per- craniectomy is usually undertaken as a last-tier
formed, it should be wide enough to accommodate (life-saving) therapy when a patient has intracra-
further brain expansion. A decompressive hemicra- nial hypertension that is sustained at 20-
niectomy diameter of 12 cm has been postulated to 35 mmHg and refractory to medical manage-
represent the minimum size for effective decom- ment. However, secondary DC can also be
pression, as the incidence of hemicraniectomy- undertaken earlier (that is, before the stages of
associated lesions increases sharply with defects last-tier therapy) and in individuals with less-
of smaller size. Cerebral herniation, shear stress sustained periods of intracranial hypertension.
along a bony ridge due to steep pressure gradients In such cases, secondary DC can be regarded as
as well as compression of cortical veins and aggra- a neuroprotective measure. This was examined
vated swelling are examples of the mechanisms in the DECRA study where 155 adults aged
thought to contribute to new postoperative contu- <60 with diffuse TBI, <72 h post injury and
sions, hemorrhage and ischemia. As a consequence, moderate intracranial hypertension (ICP
a minimum diameter exceeding 12 cm is com- exceeded 20 mmHg for more than 15 min within
monly pursued, with adequate decompression to a 1-h period, and if they did not respond to opti-
the floor of the middle cranial fossa being crucial mized first-tier interventions) were randomly
to prohibit uncal herniation. In a recent study assigned to receive either standard medical man-
where all patients had ICP control and all decom- agement alone or medical management plus
pressions were >12 cm in AP diameter, no signif- bifrontal DC. At 6-month follow-up, the investi-
icant difference in outcome or complication rate gators observed a higher rate of unfavorable out-
was seen in those <18 cm versus those >18 cm comes in the DC group than in the control group
in AP diameter, craniectomies of 12-15 or 18 cm, (70% vs. 51%). However, 27% of patients in the
though commonly perceived to be somewhat faster surgical arm had bilaterally unreactive pupils
and less invasive and traumatic, were not associated compared with only 12% in the control arm. As
with a more favorable procedural risk profile, i.e. pupil reactivity is known to be a major prognostic

Neurosurgery Books Full


www.ketabpezeshki.com 66485438-66485457
286 PART III CRANIAL NEUROSURGERY

indicator of outcome following TBI, the investi- Raised ICP after head injury increases in cerebral
gators performed a post-hoc adjustment for pupil blood volume due to reduced venous outflow
reactivity at baseline, which revealed that the (mechanical obstruction of intracranial or extracra-
between-group difference in terms of unfavorable nial venous structures, head-down position,
outcome was not significant. In contrast to the obstructed ventilation, high positive end-expiratory
DECRA study of early DC for intracranial hyper- pressure [PEEP], tight neck collar, etc.) or increased
tension, the RESCUEicp trial is examining the CBF (loss of vascular autoregulation at low or high
effectiveness of DC as a last-tier therapy for CPP, increase in PaCO2, hypoxia, acidosis, cou-
patients with refractory intracranial hyperten- pling to increased metabolism). Many current clin-
sion. The RESCUEicp study differs from ical strategies aimed at reducing ICP are based on
DECRA in a number of features: sample size the described pathophysiologic mechanisms, for
(400 patients in RESCUEicp vs. 155 patients in example, correction of hypoxia and hypotension
DECRA); surgical technique (bifrontal DC or (prevention of cellular brain edema), hyperosmolar
hemicraniectomy versus bifrontal DC alone); agents (reflex vasoconstriction to reduce CBF), head
threshold for ICP (25 mmHg vs. 20 mmHg); dura- elevation (increase in venous outflow), low-end nor-
tion of refractory intracranial hypertension (at least mocapnia (reduction in CBF and therefore CBV via
1 h vs. 15 min); timing of randomization (any time decreased PaCO2), sedation and muscle paralysis,
when inclusion criteria are met versus within 72 h normothermia (reduction in metabolism with a
post-injury); and follow-up period (2 years vs. coupled reduction in CBF/CBV), optimization of
6 months). CPP (improvement in O2 delivery without increas-
ing vasogenic edema), ventriculostomy (CSF diver-
FURTHER READING sion) and/or removal of mass lesions, and
Kolias AG, Guilfoyle MR, Helmy A. Traumatic brain injury in decompressive craniectomy (mechanical increase
adults, Pract Neurol. 2013;13(4):228-235. in intracranial volume). Regarding the role of hypo-
thermia, evidence to date suggests that early institu-
tion (i.e. at tier 2 in patients with TBI in last 10 days
recently admitted to intensive care setting in Euro-
ANSWERS 24–33
therm trial to treat ICP, or within 2.5 h of injury in
Additional answers 24–33 available on NABIS:HII as a neuroprotectant) does not improve
ExpertConsult.com outcome.

FURTHER READING
Gupta K, Gelb AW (Eds.), Essentials of Neuroanesthesia and
Neurointensive Care, Elsevier, Saunders, 2008.
EMI ANSWERS
35. 1—j, Near-infrared spectroscopy, 2—f,
34. 1—k, Thiopentone, 2—c, External ventricu- Glycerol, 3—i, Lactate/pyruvate ratio, 4—c,
lar drain and/or hematoma evacuation, 3—b, External ventricular drain ICP, 5—l, Pressure
Decompressive craniectomy reactivity index

Multi-Modality Monitoring in Traumatic Brain Injury


Modality Description

ICP monitor Single measurements not clinically as relevant as its long-term average, variability in
time, presence of waveforms, and correlation with other variables in brain monitoring.
ICP also provides information regarding autoregulation of cerebral blood flow, pressure
reactivity, and compliance of the cerebrospinal system. Intraparenchymal systems
may be inserted through an airtight support bolt (e.g. Codman or Camino systems) or
tunneled subcutaneously from a bur hole either at the bedside or after craniotomy
(Codman system). With the most common intraparenchymal arrangement, the
measured pressure may be local and not necessarily representative of CSF pressure,
and cannot generally be recalibrated after insertion, and zero drift may occur with
long-term monitoring

Continued

Neurosurgery Books Full


www.ketabpezeshki.com 66485438-66485457
21 CRANIAL TRAUMA 287

Modality Description
Cerebral microdialysis

Glycerol Release from phospholipids cell membrane degradation after cell injury. Systemic lipolysis
or the administration of glycerol-containing drugs may affect its cerebral levels

Glucose Reduced CBF (ischemia) or increased consumption of glucose may lead to a decrease in its
extracellular concentration. Low extracellular glucose levels are associated with poor
outcome after traumatic brain injury (TBI)

Lactate/glucose Marker of ischemia or increased glycolysis


ratio

Lactate/pyruvate Under normal conditions glucose is metabolized to pyruvate and lactate, and the former
ratio is used as substrate for the citric acid cycle to generate ATP. There is normally a relative
balance between glucose, lactate, and pyruvate concentrations in ECF. Ischemia and
increased anaerobic respiration increases the L/P ratio and relates to poorer neurologic
outcome

Glutamate Cerebral extracellular glutamate levels increase in tissue ischemia and hypoxia

Near-infra red Differential absorption and scatter of near-infrared light allow assessment of changes in the
spectroscopy chromophores oxyhemoglobin (HbO2), deoxyhemoglobin (Hb), and cytochrome oxidase.
Provides regional cerebral hemoglobin oxygen saturation, cerebral blood volume (CBV),
and cerebrovascular responses to therapeutic interventions

Jugular bulb A method of estimating global cerebral oxygenation and metabolism. Although Sjv O2
venous oximetry does not give quantitative information about either cerebral blood flow (CBF) or the
(Sjv O2) cerebral metabolic rate of oxygen (CMRO2), it does reflect any mismatch between them.
Low Sjv O2 values indicate either low oxygen delivery to the brain (low CBF or arterial
O2 content) or high CMRO2 and increased oxygen extraction. High Sjv O2 values reflect
high oxygen delivery (hyperemia, arteriovenous mixing) or low CMRO2. The two most
common causes of jugular bulb desaturation (Sjv O2 <55%) are decreased CPP (raised
ICP or reduced MAP) or hypocapnia. In head-injured patients, Sjv O2 values less than 50%
have been shown to increase mortality. samples may be contaminated by
extracranial venous blood

PbO2 (LICOX) Brain tissue oxygen tension (PbO2) is the partial pressure of oxygen in the extracellular fluid
of the brain and reflects the availability of oxygen for oxidative energy metabolism. It
represents the balance between oxygen delivery and consumption. Exact localization of the
sensor tips on CT after insertion is essential for accurate interpretation and use. To exclude
insertion-related microhemorrhage or sensor damage, FiO2 can be increased transiently to
confirm appropriate corresponding increases in PbO2. An equilibration time of about half an
hour after insertion is required before readings are stable

EVD ICP Still considered the “gold standard” for measurement of ICP due to ability to recalibrate and
drain CSF, but may be impossible in patients with advanced brain swelling

CPP Calculated by subtracting mean ICP from mean arterial blood pressure (CPP ¼ MAP  ICP)

Pressure The correlation between spontaneous waves in ABP and ICP is dependent on the ability
reactivity index of cerebral vessels to autoregulate. With intact autoregulation, a rise in ABP produces
(PRx) vasoconstriction, a decrease in cerebral blood volume, and a fall in ICP. With disturbed
autoregulation, changes in ABP are transmitted to the intracranial compartment and
result in a passive pressure effect. The correlation coefficient between slow changes in
mean ABP and ICP is negative when cerebral vessels are pressure reactive (i.e. intact
autoregulation). A positive correlation coefficient indicates disturbed cerebrovascular
pressure reactivity. This index may fluctuate with time as ICP and CPP vary, but on average it
expresses most of the phenomena related to cerebral hemodynamics and volume
expansion processes
Non-invasive ICP Options under investigation include transcranial Doppler examination,
tympanic membrane displacement, and ultrasound “time-of-flight” techniques
have been suggested

Neurosurgery Books Full


www.ketabpezeshki.com 66485438-66485457
288 PART III CRANIAL NEUROSURGERY

36. 1—h, Hemorrhagic progression of contu- autologous cranial bone grafts can be harvested
sion, 2—n, Subdural hygroma, 3—a, Aseptic with ease and have an enhanced survival time rel-
bone flap resorption ative to other types of bone. When the cranial
bone grafts are split, reconstruction of the donor
General frequency of complications related to site is greatly simplified, which reduces donor site
decompressive craniectomy (TBI specific where morbidity. Autologous split-thickness bone grafts
available): have become the graft of choice in craniofacial
New ipsilateral hematoma 12.9%, New contra- reconstructions in children. Autologous bone
lateral/remote hematoma 8.6%, Hemorrhagic can be preserved either by cryopreservation or
progression of contusion 12.6%, Meningitis/ by placement in a subcutaneous abdominal
ventriculitis 6.1%, deep complications in total pocket. Both of these methods may be equally
(meningitis, ventriculitis, cerebral abscess, extra- efficacious for storage in a non-traumatic brain
dural/subdural empyema) 5.1%, Hydrocephalus injury setting. However, in a traumatic brain
14.8%, CSF leak 6.7%, Syndrome of the trephined injury setting, the subcutaneous pocket may be
10%, superficial complications 8.1% (wound necro- the preferred method of storage because cryo-
sis/poor healing, wound infection, subgaleal infec- preservation may have a higher surgical site infec-
tion), Subdural hygroma 27.4%. Specific tion rate. Many studies have validated the
frequencies not available for paradoxical herniation efficacy, low infection rate, and low cost of storing
or falls onto unprotected cranium. a cranioplasty flap in the subcutaneous pouch of
General frequency of complications related to the abdominal wall. Furthermore, in a battlefield
cranioplasty following decompressive craniect- setting where injured soldiers are often trans-
omy (TBI specific where available): ported off the battlefield, storage of cranioplasty
New ipsilateral hematoma 5.4%, meningitis/ flaps in the subcutaneous abdominal wall ensures
ventriculitis 4.5%, deep complications in total (men- that the flap will not be lost in transport.
ingitis, ventriculitis, cerebral abscess, extradural/ Although preferred, autologous bone transplants
subdural empyema) 4.8%, Hydrocephalus 6.2%, are not without risks. A common complication in
CSF leak 6.8%, superficial complications 5.4% pediatric patients is bone flap resorption, which
(wound necrosis/poor healing, wound infection, results in structural breakdown. This necessitates
subgaleal infection), Subdural hygroma 6.5%, bone reoperation and replacement with plastic, metal,
flap/prosthesis infection 5.4%, aseptic bone flap or other materials. Cranioplasty depends on
reabsorption 13.5% adults and 39.2% children, osteoconduction, whereby the bone graft pro-
bone flap depression/cosmetic defect 3.1%. vides the structure to allow osteoprogenitor cells
Cranioplasty is mainly performed following to enter and take root. This requires a matrix,
craniectomy for traumatic injuries. For all age which could potentially be destroyed when the
groups, tumor removal or decompressive craniec- flap is frozen or autoclaved. This explains the
tomies are the main reasons for cranioplasty. higher resorption rate of autologous bone grafts.
Contraindications for cranioplasty include infec- autologous bone grafts had the highest rates of
tion, hydrocephalus, and brain swelling. Delaying infection than polymethylmethacrylate (PMMA),
cranioplasty could prevent devitalized autograft alumina ceramics, and titanium mesh. Synthetic
or allograft infections. The ideal material used materials are largely being considered as an alter-
for cranioplasty would be radiolucent, resistant native to prevent the complications of bone
to infections, not conductive of heat or cold, resorption, infection, donor site morbidity, and
resistant to biomechanical processes, malleable reduced strength and malleability for
to fit defects with complete closure, inexpensive, esthetic contour.
and ready to use. Replacement of the original
bone removed during craniectomy is optimal as
no other graft or foreign materials are intro- FURTHER READING
duced. In pediatric patients, this is preferable as Kurland DB, et al. Complications Associated with Decom-
the child’s original skull material will become pressive Craniectomy: A Systematic Review. Neurocrit Care.
reintegrated as he or she matures. Moreover, 2015;23(2):292-304.

Neurosurgery Books Full


www.ketabpezeshki.com 66485438-66485457
21 CRANIAL TRAUMA 289

37. 1—b, Chronic subdural hematoma, 2—a, Carotid-cavernous fistula (high flow)
Complications after TBI may include:

Complications of Traumatic Brain Injury


Bony/Dural/CSF Vascular Parenchymal
Air sinus injury Traumatic SAH Contusion
Mucocele EDH Hypopituitarism
Ossicular disruption ADH GH deficiency
CSF fistula Dissection Hypogonadotrophic hypogonadism
Hydrocephalus Carotid-cavernous fistula Hypothyroidism
Meningitis Traumatic aneurysm Secondary adrenal insufficiency
Pneumocephalus Diabetes insipidus
Subdural hygroma Seizures
Cranial nerve injury

Neurosurgery Books Full


www.ketabpezeshki.com 66485438-66485457
CHAPTER 22A

CRANIAL VASCULAR NEUROSURGERY I:


ANEURYSMS AND AVMs
SINGLE BEST ANSWER (SBA) QUESTIONS
1. Which one of the following pathologies is a. Anterior communicating artery aneurysm
most likely to give the appearances shown? b. Basilar tip aneurysm
c. Posterior communicating artery aneurysm
d. Posterior inferior cerebellar artery aneurysm
e. Middle cerebral artery aneurysm

3. A 33-year-old man presents with spontaneous


tinnitus and nausea. Which one of the follow-
ing is most likely based on the imaging shown?

a. Atrial myxoma
b. Ehlers-Danlos syndrome
c. Fibromuscular dysplasia
d. Marfan syndrome
e. Polycystic kidney disease
2. Which one of the following pathologies is a. Arteriovenous malformation
most likely to give the appearances shown? b. Cavernous angioma
c. Hemangioblastoma
d. Intracerebral hemorrhage
e. Medulloblastoma

4. A 49-year-old man attends the Emergency


department complaining of headache and
vomiting for the last 2 days and now he has
clumsiness of his left hand. There is no history
of trauma. His GCS is 15/15. CT head shows
there is a right sided acute subdural hematoma
with midline shift of 5 mm. Which one of the
following would you perform next?
a. CT intracranial angiogram
b. CT head with contrast
c. CT perfusion scan
d. MRI head with diffusion weighted sequences
e. Transcranial Doppler
290
Neurosurgery Books Full
www.ketabpezeshki.com 66485438-66485457
22A CRANIAL VASCULAR NEUROSURGERY I: ANEURYSMS AND AVMs 291

5. Which one of the following is most likely 7. A 37-year-old man presents with seizures.
given the image below? Which one of the following is NOT thought
to increase risk of hemorrhage in this type of
lesion?

a. Arachnoid cyst
b. Cavernous sinus meningioma
c. Craniopharyngioma a. Deep venous drainage
d. Giant MCA aneurysm b. Intranidal aneurysm
e. Pituitary macroadenoma c. Prior hemorrhage
d. Single draining vein
6. A 46-year-old female presents with sudden e. Smoking
onset right facial numbness, hearing loss, and
diplopia. CT head was unremarkable. Which 8. A 54-year-old woman presents with sudden
one of the following therapies may be most onset severe headache. Her GCS is 15/15 with
appropriate based on the subsequent imaging no neurological deficit on examination. Rup-
shown? ture of a vertebrobasilar aneurysm is thought
to be responsible for the imaging appearances
shown in what proportion of cases?

a. Anticoagulant therapy
b. Balloon Angioplasty
a. 2%
c. Intra-arterial nimodipine
b. 4%
d. Surgical clipping
c. 6%
e. Thrombolytic therapy
d. 8%
e. 10%

Neurosurgery Books Full


www.ketabpezeshki.com 66485438-66485457
292 PART III CRANIAL NEUROSURGERY

9. Which one of the following supraclinoid 12. Which one of the following clinical findings
internal carotid artery aneurysm locations is would you look for in this patient?
most frequent?
a. Anterior choroidal artery aneurysm
b. Carotid bifurcation aneurysm
c. Hypophyseal artery aneurysm
d. Posterior communicating artery aneurysm
e. Supraopthalmic aneurysm

10. Which one of the following pathologies is


most likely demonstrated by the angiogram?

A B

a. Abducens palsy
b. Absent corneal reflex
c. Bitemporal hemianopia
A B
d. Oculomotor palsy
e. Pituitary dysfunction
a. Anterior choroidal artery aneurysm 13. Which one of the following pathologies is
b. Basilar tip aneurysm most likely demonstrated by the angiogram?
c. MCA bifurcation
d. PCA aneurysm
e. Supraopthalmic aneurysm

11. Which one of the following is most likely


based on this AP view of a right ICA
injection?

A B

a. Anterior choroidal artery aneurysm


b. Anterior communicating artery aneurysm
c. Pericallosal aneurysm
d. Supraopthalmic aneurysm
e. Trigeminal artery aneurysm

a. A1 branch of ACA
b. Acomm artery
c. MCA bifurcation
d. M3 branch of MCA
e. Superior hypophysial artery

Neurosurgery Books Full


www.ketabpezeshki.com 66485438-66485457
22A CRANIAL VASCULAR NEUROSURGERY I: ANEURYSMS AND AVMs 293

14. The aneurysm type shown below constitutes 16. The following appearances are seen during
which one of the following proportions of all endovascular treatment of an anterior com-
intracranial aneurysms? municating artery aneurysm. What is the
next appropriate management step?

A B

a. 1%
b. 5%
c. 15%
d. 25%
e. 35%
a. Ask the anesthetist to reduce systolic
15. Which one of the following pathologies is blood pressure to 100 mmHg
most likely demonstrated by the angiogram? b. Check pupillary reflexes and perform
CT head
c. Continue with endovascular treatment
d. ICP monitoring
e. Insertion of external ventricular drain

17. Which one of the following pathologies is


most likely demonstrated by the angiogram?

a. Anterior communicating artery aneurysm


b. Basilar artery aneurysm
c. Basilar invagination
d. Left PCA artery aneurysm A B
e. Superior hypophyseal artery aneurysm
a. A1 branch of ACA aneurysm
b. Corpocallosal AVM
c. MCA bifurcation aneurysm
d. Pericallosal aneurysm
e. Posterior communicating artery aneurysm

Neurosurgery Books Full


www.ketabpezeshki.com 66485438-66485457
294 PART III CRANIAL NEUROSURGERY

18. Which one of the following is most likely 20. Which one of the following pathologies is
demonstrated by the angiogram? most likely demonstrated by the angiogram?

A B

a. Anterior communicating artery


b. MCA bifurcation aneurysm
c. Pericallosal aneurysm
d. Posterior communicating artery
e. Terminal ICA aneurysm
a. Asymmetric fusion of caudal divisions of
19. Which one of the following pathologies is fetal ICA
most likely demonstrated by the angiogram? b. Basilar fenestration
c. Basilar invagination
d. Fetal origin of PCA
e. Hypoplastic posterior communicating
artery

21. Which one of the following clinical findings


would you look for in this patient?

a. Anterior inferior cerebellar artery aneurysm


b. Basilar tip aneurysm
c. Posterior cerebral artery aneurysm
d. Posterior inferior cerebellar artery aneurysm
e. Superior cerebellar artery aneurysm

a. Abducens palsy
b. Internuclear opthalmoplegia
c. Occulomotor palsy
d. Parinaud’s syndrome
e. Trochlear palsy

Neurosurgery Books Full


www.ketabpezeshki.com 66485438-66485457
22A CRANIAL VASCULAR NEUROSURGERY I: ANEURYSMS AND AVMs 295

22. Which one of the following pathologies is EXTENDED MATCHING ITEM (EMI)
most likely demonstrated by the angiogram?
QUESTIONS
33. Posterior circulation stroke syndromes:
a. Anton syndrome
b. Babinski-Nageotte syndrome
c. Balint syndrome
d. Benedikt syndrome
e. Claude syndrome
f. Dejerine syndrome
g. Dejerine-Roussy syndrome
h. Foville syndrome
i. Locked-in syndrome
j. Marie-Foix syndrome
k. Millard-Gubler syndrome
l. Raymond syndrome
m. Top of basilar syndrome
n. Wallenberg syndrome
o. Weber syndrome

For each of the following descriptions, select the


most appropriate answers from the list above.
Each answer may be used once, more than once
or not at all:
1. Contralateral hemisensory loss and contra-
a. Basilar fenestration
lateral hemibody pain
b. Basilar tip aneurysm
2. Visual deficit without recognition of
c. Persistent trigeminal artery
blindness
d. Persistent otic artery
3. Ipsilateral loss of facial sensation/pain, ipsi-
e. Vertebral artery occlusion
lateral Horner’s syndrome, ipsilateral
ataxia, loss of gag reflex, and contralateral
23. Which one of the following mechanisms is
hemibody loss of pain and temperature
most likely responsible for the finding in
4. Ipsilateral oculomotor palsy and contralat-
the angiogram?
eral temor and ataxia

34. Arterial supply:


a. Anterior choroidal artery
b. Anterior inferior cerebellar artery
c. Anterior temporal artery
d. Anterior thalamoperforating arteries
e. Calcarine artery
f. Inferolateral trunk of ICA (C4)
g. Labyrinthine artery
h. Lateral posterior choroidal artery
A
i. Medial and lateral lenticulostriate
B
arteries
j. Medial posterior choroidal artery
a. Connective tissue disorder k. Ophthalmic artery
b. Infection l. Posterior inferior cerebellar artery
c. Traumatic dissection m. Recurrent artery of Heubner
d. Neoplasia n. Thalamogeniculate arteries
e. Toxin
For each of the following descriptions, select the
most appropriate answers from the list above.
QUESTIONS 24–32 Each answer may be used once, more than once
or not at all:
Additional questions 24–32 available on 1. Supplies visual cortex
ExpertConsult.com 2. Arises from P2 segment to supply pulvinar
and superior colliculus

Neurosurgery Books Full


www.ketabpezeshki.com 66485438-66485457
296 PART III CRANIAL NEUROSURGERY

35. Venous drainage: For each of the following descriptions, select the
a. Thalamostriate veins most appropriate answers from the list above.
b. Superior anastomotic vein of Trolard Each answer may be used once, more than once
c. Inferior anastamostic vein of Labbe or not at all:
d. Internal cerebral vein 1. Prospective study of natural history of
e. Great cerebral vein of Galen unruptured intracranial aneurysms
f. Superficial middle cerebral (Sylvian) vein 2. Randomized trial showing superiority of
g. Basal vein of Rosenthal mechanical thrombectomy over thrombol-
h. Anterior cerebral vein ysis in acute ischemic stroke
i. Deep middle cerebral vein 3. Randomized trial showing reduction in
j. Straight sinus cerebral ischemic events in subarachnoid
k. Transverse sinus hemorrhage in patients taking oral
l. Occipital sinus nimodipine
m. Inferior petrosal sinus 4. Randomized trial showing no difference in
n. Superior petrosal sinus favorable outcome when simvastatin given
o. Cavernous sinus in aneurysmal subarachnoid hemorrhage
p. Superior saggital sinus
q. Inferior saggital sinus 37. Triangle of cavernous sinus and middle
fossa:
For each of the following descriptions, select the a. Anteromedial triangle
most appropriate answers from the list above. b. Medial (Hakuba’s) triangle
Each answer may be used once, more than once c. Paramedian triangle
or not at all: d. Parkinson’s triangle
1. Drains from the Sylvian fissure to the sup- e. Anterolateral (Mullan’s) triangle
erior saggital sinus f. Lateral triangle
2. Drains from the Sylvian fissure to the g. Posterolateral (Glasscock’s) triangle
transverse sinus h. Posteromedial (Kawase’s) triangle
3. Drains into the cavernous sinus, vein of i. Inferomedial triangle
Trolard, and vein of Labbe j. Inferolateral triangle

36. Trials in Vascular Neurosurgery: For each of the following descriptions, select the
a. ARUBA most appropriate answers from the list above.
b. BRANT Each answer may be used once, more than once
c. BRAT or not at all:
d. ISAT 1. Access to entire intracavernous ICA
e. ISUIA 2. Access to petroclival area, anterolateral
f. JAM brainstem, and vertebrobasilar junction
g. NASCET 3. Access horizontal intrapetrosal ICA for
h. STASH proximal control or bypass graft
i. STICH
j. ESCAPE/EXTEND-IA/SWIFT-
PRIME

SBA ANSWERS
1. c—Fibromuscular dysplasia. FMD occurs observed, the degree of stenosis is usually
predominantly in middle-aged women and modest (less than 40%). FMD can occasion-
most often affects the cervical ICA (75%). ally be observed intracranially and is associ-
The vertebral (12%) and external carotid ated with aneurysms.
arteries may also be involved. Disease is bilat- Image with permission from Adam A, et al. (Eds.).
eral in 60% of cases. Angiographic images, Grainger & Allison's Diagnostic Radiology, 6th ed.,
almost always with non-invasive techniques, Elsevier, Churchill Livingstone, 2014.
demonstrate alternating luminal narrowing
and dilatation, the resulting appearance often 2. a—Anterior communicating artery aneu-
described as a “string of beads.” This “corru- rysm. The classical flame shaped hemorrhage
gation” typically affects the mid ICA, usually associated with acute rupture of these aneu-
2 cm distal to bulb. Uni- or multifocal tubu- rysms is depicted.
lar stenoses are less common, and where

Neurosurgery Books Full


www.ketabpezeshki.com 66485438-66485457
22A CRANIAL VASCULAR NEUROSURGERY I: ANEURYSMS AND AVMs 297

Image with permission from Adam A, et al. (Eds.). thrombolytics have only been used in selected
Grainger & Allison's Diagnostic Radiology, 6th ed., cases.
Elsevier, Churchill Livingstone, 2014.
Image with permission from Loevner L. Brain Imaging:
Case Review Series, 2nd ed., Elsevier, Mosby, 2009.
3. b—Cavernous angiomas are mulberry-like
lesions consisting of vascular spaces with lit- 7. e—Smoking. Brain arteriovenous malforma-
tle intervening tissue and hemorrhage of dif- tions (AVMs) are abnormal vascular anoma-
ferent ages. The incidence of clinically lies within the brain, presumably congenital
symptomatic hemorrhage remains uncertain, in nature, but tend to present later in life
but is less frequent than with cerebral AVMs (20-40 years). There are several subgroups,
or dural fistulae. A previous bleed and infra- including the glomerular (most common)
tentorial location are the main prognostic and fistulous (less common) types of AVMs.
factors for recurrent hemorrhage. Lesions AVMs, often pial-based, are defined by
in or close to the cerebral cortex may cause presence of arteriovenous shunting through
epilepsy. They are occasionally intraventric- a nidus of coiled and tortuous vascular con-
ular or arise on a cranial nerve. They appear nections that connect feeding arteries to
as relatively well-defined, dense, or calcified draining veins, without a capillary bed. Most
lesions on CT, which may show patchy con- (approximately 60-70%) of AVMs are
trast enhancement. On MRI they appear located in the cerebral hemispheres, 11-18%
multilobular with mixed but predominantly within the cerebellum, and 13-16% in the
elevated T2 signal intensity centrally sur- brainstem; 8-9% are deep-seated. Factors
rounded by a dark hemosiderin rim. Not sur- that increase risk of hemorrhage from an
prisingly, susceptibility-based sequences are AVM include history of hypertension or pre-
the most sensitive. They may be multiple, vious hemorrhage, flow-related aneurysm,
particularly in familial cases. In many clinical intranidal aneurysm, deep venous drainage,
situations the discovery of a cavernoma rep- deep (periventricular) location, small nidus
resents an incidental finding. size (<3 cm), high feeding artery pressure,
Image with permission from Adam A, et al. (Eds.). slow arterial filling, and venous stenosis.
Grainger & Allison's Diagnostic Radiology, 6th ed., Else- Presence of intracranial hemorrhage indi-
vier, Churchill Livingstone, 2014.
cates a poorer prognosis and is associated
with an increasing morbidity and mortality.
4. a—CT intracranial angiogram
Presence of AVMs can lead to arterial steal
Occasionally, rupture of a cerebral aneurysm may phenomenon, venous congestion, gliosis, or
cause an acute subdural hematoma, most frequently hydrocephalus.
a posterior communicating artery aneurysm lying Image with permission from Fatterpekar GM, Naidich
next to the free edge of the tentorium cerebelli. TP, Som PM. The Teaching Files: Brain and Spine, Else-
vier, Saunders, 2012.
A dural arteriovenous fistula may also bleed into
the subdural space. Angiography is therefore
indicated following a spontaneous acute subdural 8. b—4%—Nontraumatic, nonaneurysmal
hematoma, particularly in a young patient prior perimesencephalic hemorrhage (PMH), pre-
to craniotomy and evacuation of the clot. sumably of venous origin, accounts for 96%
of all perimesencephalic hemorrhage. The
5. d—Giant MCA aneurysm remaining 4% have been reported to result
from rupture of vertebrobasilar aneurysm.
On MRI imaging, giant aneurysms have a charac- Nontraumatic, nonaneurysmal perimesence-
teristic appearance, as in this case. Findings phalic hemorrhage is characterized by rela-
include signal void consistent with flow in the pat- tively mild symptoms at onset, confinement
ent lumen; phase artifact related to flow, as is seen of the extravasated blood in the perimesence-
in this case; and heterogeneous signal intensity phalic cisterns, and absence of aneurysm. It
representing thrombi of varying ages. has a benign clinical course and excellent
prognosis.
Image with permission from Loevner L. Brain Imaging:
Image with permission from Loevner L. Brain Imaging:
Case Review Series, 2nd ed., Elsevier, Mosby, 2009.
Case Review Series, 2nd ed., Elsevier, Mosby, 2009.

6. a—Anticoagulation. The MRI shows a spon- 9. d—Posterior communicating artery aneurysm


taneous right vertebral artery dissection—the
main treatment for which is anticoagulation Thirty-five percent of all intracranial aneurysms
or antiplatelet therapy once subarachnoid arise at one of the following five sites along the
hemorrhage has been excluded. Intra-arterial supraclinoid ICA

Neurosurgery Books Full


www.ketabpezeshki.com 66485438-66485457
298 PART III CRANIAL NEUROSURGERY

the ICA is convex medially. In this location


Frequency
of all IC they lie lateral to the pituitary stalk and point
aneurysms medially under the optic chiasm. Medial
Type Site (%) expansion of the aneurysm may compromise
supraopthalmic Upper surface of 5 the perforating arteries to the floor of the
aneurysm ICA at the origin third ventricle, the optic nerves, the chiasm,
of the ophthalmic the pituitary stalk, and the hypophysial vascu-
artery lar supply.
Hypophyseal Medial wall of ICA at 1 Image with permission from Naidich T, Castillo M, Cha
artery the origin of the S, Smirniotopoulos J. Imaging of the Brain, Elsevier,
aneurysm superior Saunders, 2013.
hypophyseal artery

Posterior Posterior wall of 25


12. d—Oculomotor palsy
communicating ICA immediately
artery superolateral to the The posterior communicating artery arises from
aneurysm origin of the the posterior wall of the ICA where it forms a
PcommA posteriorly convex curve as it ascends to its termi-
Carotid Apex of the 5 nal bifurcation under the anterior perforated sub-
bifurcation terminal ICA stance. These aneurysms arise near the apex of
aneurysm bifurcation into the posteriorly convex turn, immediately superior
ACA and MCA to the distal edge of the origin of the posterior
Anterior Posterior wall of 5 communicating artery. They point downward
choroidal the ICA immediately and posteriorly toward the oculomotor nerve,
artery superior to the so the posterior communicating artery is usually
aneurysm origin for the found inferomedial to the neck of the aneurysm
anterior choroidal
artery
(the anterior choroidal artery is found superior
or superolateral to the neck of the aneurysm).
The oculomotor nerve enters the dural roof of
the cavernous sinus lateral to the posterior clinoid
10. e—Supraopthalmic aneurysm. These typi- process and medial to a dural band that runs
cally arise from the superior wall of the between the tentorium cerebelli and the anterior
carotid artery at the distal edge of the origin clinoid process. Posterior communicating artery
of the ophthalmic artery close to the roof of aneurysms larger than 4-5 mm may compress
the cavernous sinus. At this point, the ICA the oculomotor nerve at its entrance into the
changes direction from superior toward pos- dural roof, causing opthalmoplegia.
terior, so the maximal hemodynamic force is Images with permission from Naidich T, Castillo M,
directed toward the superior wall of the Cha S, Smirniotopoulos J. Imaging of the Brain,
carotid artery just distal to the ophthalmic Elsevier, Saunders, 2013.
artery. Therefore, these aneurysms project
13. a—Anterior choroidal artery aneurysm
upward toward the optic nerve and are often
large with complex, multi-lobulated shape. Should the posteriorly convex curve of the supra-
Surgical exposure may be difficult as the oph- clinoid ICA form its apex at the level of the ante-
thalmic artery has a variable origin and rior choroidal artery the hemodynamic force is
course and because multiple folds of the dura shifted distally from the origin of the posterior
enclose the region of the optic foramen and communicating artery to the origin of the ante-
clinoid process. Many are wide-necked aneu- rior choroidal artery. The anterior choroidal
rysms that may require remodeling tech- aneurysms form just distal, superior, or superolat-
niques. Unruptured aneurysms may become eral to the origin of the anterior choroidal artery.
symptomatic due to headaches or compres- They also point posterior or posterolaterally but
sion of cranial nerves. are usually well above the oculomotor nerve.
Images with permission from Naidich T, Castillo M, Aneurysms arising from the choroidal segment
Cha S, Smirniotopoulos J. Imaging of the Brain, commonly have more perforating branches stret-
Elsevier, Saunders, 2013.
ched around their neck than those arising from
the communicating or ophthalmic segment, beca-
11. e—Superior hypophysial artery aneurysms use the choroidal segment has a greater number
arise just distal to the origin of the superior of perforating branches arising from it and the
hypophysial artery from the medial or poste- majority arise from the posterior wall, where the
rior wall of the ICA where the curvature of neck of the aneurysm is situated.

Neurosurgery Books Full


www.ketabpezeshki.com 66485438-66485457
22A CRANIAL VASCULAR NEUROSURGERY I: ANEURYSMS AND AVMs 299

Images with permission from Naidich T, Castillo M, Occlusion of the recurrent artery of Heubner
Cha S, Smirniotopoulos J. Imaging of the Brain, may cause hemiparesis or aphasia.
Elsevier, Saunders, 2013.
Image with permission from Naidich T, Castillo M,
14. b—5% Cha S, Smirniotopoulos J. Imaging of the Brain,
Elsevier, Saunders, 2013.
Aneurysms arise at the apex of the T-shaped
16. b—Check pupillary reflexes and perform CT
carotid bifurcation and point superiorly in the
head. This angiogram shows active extravasa-
direction of the long axis of the pre-bifurcation
tion of contrast material into the subarach-
segment of the artery. As they grow, they lie lateral
noid spaces, suggesting acute rupture of
to the optic chiasm and may indent the undersur-
this aneurysm necessitating clinical reassess-
face of the anterior perforated substance. The
ment and surgical intervention if appropriate.
perforating branches arising from the choroidal
segment of the internal carotid and the proximal Image with permission from Naidich T, Castillo M,
segments of the anterior and middle cerebral Cha S, Smirniotopoulos J. Imaging of the Brain,
Elsevier, Saunders, 2013.
arteries are stretched around the posterior aspect
of the neck and wall of the aneurysm.
17. d—Pericallosal aneurysm
Images with permission from Naidich T, Castillo M,
Cha S, Smirniotopoulos J. Imaging of the Brain, The second most common aneurysm of the ACA
Elsevier, Saunders, 2013. is the so-called pericallosal aneurysm, which
arises at the origin of the callosomarginal artery
15. a—Anterior communicating artery aneurysm
from the pericallosal artery, usually in close prox-
Aneurysms of the ACA typically form close to the imity to the anterior portion of the corpus callo-
anterior communicating artery complex. They sum, near the point where the genu of the ACA
constitute about 30% of all intracranial aneu- has its greatest angulation. Pericallosal aneurysms
rysms and are considered one of the most com- account for approximately 3% of all intracranial
mon types of aneurysm. They are frequently aneurysms. They point distally into the window
associated with anatomical variants. Aneurysms between the junction of the pericallosal and callo-
often occur when one A1 segment is hypoplastic somarginal arteries.
and the dominant A1 gives rise to both A2s. In Images with permission from Naidich T, Castillo M, Cha
such case, the aneurysm arises at the level of S, Smirniotopoulos J. Imaging of the Brain, Elsevier,
the anterior communicating artery at the point Saunders, 2013.
where the dominant A1 segment bifurcates to
18. b—MCA bifurcation aneurysm
give rise to both the left and right A2 segments.
The direction in which the dome of the aneurysm Approximately 15% of all saccular aneurysms
points is determined by the course of the domi- arise from the MCA. Typically they originate at
nant A1 segment proximal to its junction with the level of the first major bifurcation or trifur-
the anterior communicating artery. Thus, these cation of the artery and point laterally in the
aneurysms usually point away from the dominant direction of the long axis of the pre-bifurcation
segment toward the opposite side. Approaches to segment of the MCA. The more proximal the
anterior communicating artery aneurysms must bifurcation, the greater the number of lenticu-
ensure that the anterior communicating artery lostriate branches arising distal to the bifurcation
and the adjacent recurrent artery of Heubner that may be stretched around the neck of the
remain patent. The AcomA gives rise to small MCA aneurysm. When unruptured, these aneu-
perforating branches for the dorsal surface of rysms are typically clinically silent. Proximal
the optic chiasm and suprachiasmatic area that M1 segment aneurysms at origins of lenticulostri-
perfuse the fornix, corpus callosum, and septal ate arteries are exceedingly rare but when present
region. Occlusion of the anterior communicating tend to point upward toward the anterior perfo-
artery may lead to personality disorders, even if rated substance. MCA aneurysms may also arise
both A2 segments are perfused from their respec- from the temporopolar branch of the M1 seg-
tive A1 segments. The recurrent artery of ment. When present, these tend to point inferi-
Heubner arises, variably, from the distal A1, the orly. Aneurysms distal to the MCA bifurcation
proximal A2, or the frontopolar branch of are rare and are typically encountered in the set-
the ACA before looping forward on the gyrus ting of infectious diseases.
rectus or the posterior part of the orbital surface
of the frontal lobe and then passing back over Images with permission from Naidich T, Castillo M,
the carotid bifurcation to accompany the MCA Cha S, Smirniotopoulos J. Imaging of the Brain,
and enter the anterior perforating substance. Elsevier, Saunders, 2013.

Neurosurgery Books Full


www.ketabpezeshki.com 66485438-66485457
300 PART III CRANIAL NEUROSURGERY

19. d—Posterior inferior cerebellar artery to the cranium, lower cranial nerves, and the
aneurysm complex surgical approaches to this region.
Endovascular embolization of aneurysms at an
Most aneurysms of the vertebral artery take origin unfused basilar artery is an alternative to surgery.
at the posterior inferior cerebellar artery (PICA), However, it must be recognized that both limbs
especially when the origin of the PICA falls at the of the unfused basilar artery have to be preserved,
apex of a superiorly directed curve of the vertebral that the neck of such aneurysms is often broad,
artery. These aneurysms almost invariably point and that the aneurysm may regrow due to the
upward and usually communicate widely with unfavorable hemodynamics at the site of an
the PICA. The size of the territory supplied by unfused segment.
the PICA varies widely, and will influence the best
approach to aneurysm therapy. Common ana- Image with permission from Naidich T, Castillo M,
tomic variants associated with the vertebral artery Cha S, Smirniotopoulos J. Imaging of the Brain,
Elsevier, Saunders, 2013.
include unilateral agenesis/hypoplasia, double
(duplicated, fenestrated) origin, and extracranial 21. c—Oculomotor palsy
or epidural origin. There are close reciprocal
inverse relationships among the sizes of the hemi- Basilar artery aneurysms at the level of the SCA
spheric territories supplied by the PICA, AICA, often arise where the upper basilar artery curves
and SCA. Any one may annex (part of) the terri- and tilts, so the hemodynamic thrust created by
tory of the adjacent vessel, commonly leading to flow along the basilar artery impacts just above
variations such as the AICA-PICA trunk. PICA the origin of the SCA rather than at the basilar
supply to both cerebellar hemispheres is very apex. SCA aneurysms often have a broad connec-
uncommon but does occur and must be consid- tion with the SCA, a rather large neck, and a
ered prior to endovascular procedures. neck-to-dome ratio that makes endovascular
therapy demanding. Endovascular therapy must
Image with permission from Naidich T, Castillo M, attempt to preserve this artery, because this is
Cha S, Smirniotopoulos J. Imaging of the Brain,
Elsevier, Saunders, 2013. the major vessel to supply the deep nuclei of
the cerebellum. Large SCA aneurysms may cause
20. b—Basilar fenestration oculomotor nerve palsies by direct impression on
the oculomotor nerve as it courses through the
The incidence of basilar artery aneurysms interpeduncular cistern just cranial to the SCA.
increases when the basilar system shows anoma-
lous or variant architecture, including basilar Image with permission from Naidich T, Castillo M,
nonfusion (fenestration), asymmetric or caudal Cha S, Smirniotopoulos J. Imaging of the Brain,
Elsevier, Saunders, 2013.
fusion of the caudal divisions of the fetal ICA,
hypoplastic communicating artery, or fetal (per- 22. b—Basilar tip aneurysm
sistent carotid) origin of the posterior cerebral
artery. Proximal non-dissecting basilar artery About 15% of saccular aneurysms occur in the
aneurysms are rare and typically arise in patients vertebrobasilar system and of these 60% arise at
with failure to form a single basilar artery during the basilar bifurcation where the posterior cere-
embryologic development. The single basilar bral arteries branch off from the tip of the basilar
artery normally develops by union of paired lon- artery. At the aneurysm site the blood flow
gitudinal neural arteries that fuse together by changes from vertical to nearly horizontal, so
about the fifth fetal week (when the embryo is these aneurysms project upward in the direction
9 mm long). Each of the longitudinal neural of the long axis of the basilar artery. The largest
arteries gives rise to the perforating arteries for and most important perforators to arise from the
its own side of the brain stem. Failed fusion of basilar tip are the posterior thalamoperforate
the neural arteries is often associated with aneu- arteries (retromammillary arteries). These origi-
rysms at the proximal portion of the nonfused nate from the basilar tip and P1, enter the brain
artery. The lateral walls of the unfused arteries through the posterior perforated substance in
have normal intrinsic architecture. At the base the interpeduncular fossa medial to the cerebral
of the medial wall, however, the media is absent, peduncles, and ascend through the midbrain to
the elastic is discontinuous, and the subendothe- the thalamus. The risks from occlusion of these
lium is thinned. These segments are more likely vital perforating vessels include visual loss, paral-
to develop arterial aneurysms when subject to ysis, sensory disturbances, weakness, memory def-
secondary “offensive” triggers such as hemody- icits, autonomic and endocrine imbalance,
namic stress. The surgical treatment of these abnormal movements, diplopia, and depression of
aneurysms is difficult due to their relationship consciousness. Endovascular approaches have been

Neurosurgery Books Full


www.ketabpezeshki.com 66485438-66485457
22A CRANIAL VASCULAR NEUROSURGERY I: ANEURYSMS AND AVMs 301

widely adopted to treat basilar apex aneurysms, intracranial dissection is an uncommon disease but
because the surgical approach is associated with a has been increasingly recognized as a cause for
higher morbidity. This is especially true for the SAH with an unfavorable prognosis and a high
more posterior basilar tip aneurysms, because rebleeding rate. One percent to 10% of all intracra-
greater numbers of vital thalamoperforators are nial nontraumatic SAH is thought to result from
affected as the aneurysm enlarges and projects ruptured intracranial dissections. This rate may rise
more deeply into the interpeduncular fossa. to 5-20% in young patients. The choice of treatment
and its timing continue to be controversial. Acutely
Image with permission from Naidich T, Castillo M, Cha ruptured dissections are unstable. Up to 70% of
S, Smirniotopoulos J. Imaging of the Brain, Elsevier,
Saunders, 2013.
cases rebleed, often soon after the initial hemor-
rhage, with a mortality rate from rebleeding as
23. c—Microtrauma high as 50%. After SAH, 70% of rebleeding occurs
within the first 24 h, with 80% occurring within the
Intradural traumatic aneurysms most commonly first week. The rebleeding rate decreases consider-
involve the internal carotid and vertebral arteries ably beyond the first week after initial hemorrhage,
at their transdural portions. Traumatic aneu- and only 10% of rebleeding occurs more than
rysms may result from penetrating injuries such 1 month after the initial hemorrhage. The dissection
as a stabbing accident, a high-velocity gunshot may lead to an extensive mural hematoma that may
wound, or iatrogenic trauma (e.g., third ventricu- compress perforating arteries close to the site of
lostomy). Similarly, traumatic arterial aneurysms dissection. Treatment should be targeted at exclud-
have been described as involving the ACA along ing the damaged vessel wall segment from the
the falx and the tentorium, either following major circulation, either endovascularly or via surgical
head injuries or as part of the shaken baby syn- approaches.
drome. Distal posterior cerebral artery aneurysm
are most likely dissecting in nature. They typi- Image with permission from Naidich T, Castillo M,
Cha S, Smirniotopoulos J. Imaging of the Brain,
cally appear at the junction between the P2 and Elsevier, Saunders, 2013.
P3 segments, where the PCA crosses the tentor-
ium resulting in microtrauma. Angiographic cri-
teria for spontaneous dissections are the stagnation ANSWERS 24–32
of the contrast medium in an aneurysmal pouch,
the presence of stenotic segments proximal and/or Additional answers 24–32 available on
distal to the ectasia, and a fusiform appearance ExpertConsult.com
of the aneurysm. Spontaneous hemorrhagic

EMI ANSWERS
33. 1—g, Dejerine-Roussy syndrome; 2—a, Anton syndrome; 3—n, Wallenberg’s syndrome; 4—d,
Benedikt’s syndrome

Name Features Localization Supply


Claude syndrome Ipsilateral CNIII palsy Red nucleus, III nucleus, P1 PCA
contralateral ataxia and tremor superior cerebral
Contralateral hemiparesis peduncle
Contralateral hemiplegia of Corticospinal tract
lower facial muscles/tongue/ Corticobulbar tract
shoulder

Weber syndrome Ipsilateral CNIII palsy Medial midbrain/cerebral P1 PCA


Contralateral hemiplegia peduncle

Benedikt’s syndrome Ipsilateral CNIII palsy III nucleus P1 PCA


Contralateral ataxia and tremor Red nucleus

Dejerine-Roussy Contralateral hemisensory loss Thalamus P1 PCA


syndrome (thalamic pain Contralateral hemibody pain
syndrome)

Continued on following page

Neurosurgery Books Full


www.ketabpezeshki.com 66485438-66485457
302 PART III CRANIAL NEUROSURGERY

Name Features Localization Supply


STN Contralateral hemiballismus STN P1 PCA

Anton syndrome (cortical Visual agnosia (deficit without Bilateral occipital lobe Bilateral P2
blindness) recognition of blindness) PCA/top of basilar

Balint syndrome Bilateral loss of voluntary but Bilateral parieto-occipital Bilateral P2 PCA
not reflex eye movements lobe
Bilateral optic ataxia
Asimultagnosia

Top of Basilar syndrome Opthalmoplegia Bilateral rostal midbrain Basilar


Behavioral abnormalities and posterior thalamus
Somnolence/hallucinations
Usually no motor deficit

Wallenberg’s syndrome Ipsilateral loss facial pain and Trigeminal nucleus PICA or VA
(lateral medullary) sensation Cerebellar peduncle
Ipsilateral Horner’s syndrome Nucleus ambiguus
Ipsilateral ataxia Vestibular nucleus
Dysarthria/dysphagia/loss of Solitary nucleus
gag reflex Spinothalamic tract
Vertigo/nystagmus Cuneate/gracile nuclei
Loss of taste
Contralateral body pain and
temperature loss
Ipsilateral numbness

Dejerine syndrome Contralateral hemiparesis VA/BA/ASA


(medial medullary) Contralateral hemisensory loss
Ipsilateral CNXII palsy

Babinski-Nageotte Combination of Wallenberg’s Hemimedullary infarct VA (proximal to


syndrome and Dejerine syndromes PICA and spinal
artery)

Marie-Foix syndrome Ipsilateral ataxia Lateral inferior pons AICA


Contralateral hemiparesis
Contralateral hemisensory loss

Locked-in syndrome Quadriplegia Bilateral ventral pons Basilar perforators


Bilateral VI/VII weakness
Aphonia (lower CN)
Intact upgaze and blinking

Foville syndrome Ipsilateral facial palsy Inferior medial pons Basilar perforators
Ipsilateral lateral gaze palsy PPRF/CN VI
Contralateral hemiparesis

Millard-Gubler syndrome Ipsilateral abducens palsy Ventral pons Basilar perforators


Ipsilateral facial palsy
Contralateral hemiparesis

Raymond syndrome Ipsilateral abducens palsy CNVI nucleus Basilar perforators


Contralateral hemiparesis Corticospinal tracts

34. 1—e, Calcarine artery; 2—n, Thalamogeniculate arteries

35. 1—b, Superior anastomotic vein of Trolard; 2—c, Inferior anastamostic vein of Labbe; 3—f,
Superficial middle cerebral (Sylvian) vein

Neurosurgery Books Full


www.ketabpezeshki.com 66485438-66485457
22A CRANIAL VASCULAR NEUROSURGERY I: ANEURYSMS AND AVMs 303

36. 1—e, ISUIA; 2—j, ESCAPE/EXTEND-IA/SWIFT-PRIME; 3—b, BRANT; 4—h, STASH

STASH Simvastatin in aneurysmal In the STASH trial, within 96 h of ictus patients were
subarachnoid hemorrhage randomized in a 1:1 fashion to a 21 day course of either
40 mg simvastatin or placebo. The analysis of mRS at
6 months follow-up (primary outcome) showed no
difference between favorable outcome. Fewer patients
receiving simvastatin required extended hypervolemic
therapy (21% vs. 29%, respectively; p ¼ 0.009), but there was
no significant difference in the rates of DIND or mortality
between the two groups

STICH Early surgery versus initial Patients with spontaneous supratentorial intracerebral
conservative treatment in patients hemorrhage in neurosurgical units show no overall benefit
with spontaneous supratentorial from early surgery (<24 h after randomization) when
intracerebral hematomas in the compared with initial conservative treatment (with delayed
International Surgical Trial in surgery if judged necessary)
Intracerebral Hemorrhage (STICH)

STICH II Early surgery versus initial In conscious patients with spontaneous superficial (<1 cm
conservative treatment in patients from cortical surface; volume 10-100 ml) intracerebral
with spontaneous supratentorial hemorrhage without intraventricular hemorrhage early
lobar intracerebral hematomas surgery (<12 h after randomization) does not increase the
(STICH II) rate of death or disability at 6 months compared to initial
conservative management (with delayed surgery if judged
necessary). Patients in the STICH II trial with a poor
prognosis did better with early surgery, whereas those with
a good prognosis did not

ISAT International Subarachnoid In contrast to the statistically significant difference in death


Aneurysm Trial or dependence [modified Rankin Scale (mRS) 3-6] at 1 year
after ruptured aneurysm coiling as compared to clipping
(23.5% vs. 30.9%, respectively; p ¼ 0.0001), at 5-year follow-
up no statistically significant difference in independence
was seen. However, the risk of death remained significantly
lower in the endovascular cohort at 5-year follow-up
[relative risk (RR) 0.77; 95% CI 0.61-0.98]. At 10-year follow-
up, rates of independence still did not differ between the
two groups, however, the probability of being alive and
independent was more likely in the endovascular cohort
(OR 1.34; 95% CI 1.07-1.67). The risk of rebleeding was
higher in the endovascular group than the clipping group
(1.56 bleeds per 1000 patient years vs. 0.49 bleeds per 1000
patient years, respectively)

ISUIA International Study of Unruptured Risk of rupture 0.05% per year if <10 mm and no previous
(1998, Intracranial Aneurysms SAH from different aneurysm
2003) Risk of rupture 0.5% per year if <10 mm but previous SAH
from different aneurysm
Risk of rupture 1% per year if >10 mm (with or without
previous SAH)
Ninety percent of unruptured aneurysms in anterior circulation
Criticism: Selection bias, retrospective, 7.5 years follow up and
Pcomm aneurysms included in posterior circulation

ARUBA A Randomized trial of Unruptured Comparing no treatment and any treatment modality
Brain AVMs (embolization, radiosurgery, microsurgery, alone or in
combination) in the management of unruptured brain AVMs.
The primary end point was death or symptomatic stroke. After
a mean follow-up of 33.3 months, the primary end point was
observed in 10.1% of the patients receiving no treatment and in
30.7% of patients assigned to treatment (RR 0.33; 95% CI 0.18-
0.61). Main criticism is expected benefit from radiosurgery
would not be seen within such short follow-up times (used in

Continued on following page

Neurosurgery Books Full


www.ketabpezeshki.com 66485438-66485457
304 PART III CRANIAL NEUROSURGERY

alone or in combination in more than half of patients assigned


to treatment in this study). Higher rate of hemorrhage could
not be attributed to radiosurgery or surgical treatment and
therefore likely underscores a prohibitively high rate of
overzealous and/or partial embolization employed in the
study. This highly unusual practice has poor external validity
to high volume centers that treat AVMs. Although 68% of
patients in the treatment arm were designated Spetzler-Martin
Grade I or II (93% Grades I-III), only five patients (4.3%) were
treated with microsurgery alone

BRAT Barrow Ruptured Aneurysm Trial At 1 year intention to treat analysis a poor outcome (mRS
score >2) was observed in 33.7% of the patients assigned to
aneurysm clipping and in 23.2% of the patients assigned to
coil embolization (OR 1.68, 95% CI 1.08-2.61; p ¼ 0.02)
At 3 years analysis was based on received treatment (rather
than ITT) which showed the risk of a poor outcome in
patients assigned to clipping compared with those assigned
to coiling (35.8% vs. 30%) was no longer significant (OR
1.30, 95% CI 0.83-2.04, p ¼ 0.25). In addition, the degree of
aneurysm obliteration (p ¼ 0.0001), rate of aneurysm
recurrence (p ¼ 0.01), and rate of retreatment (p ¼ 0.01) were
significantly better in the group treated with clipping
compared with the group treated with coiling
Criticisms: 38% crossed over to clipping, no intention to
treat analysis in 3-year data

BRANT British Aneurysm Nimodipine Trial At 3 months after subarachnoid hemorrhage in patients
given nimodipine the incidence of cerebral infarction was
22% (61/278) compared with 33% (92/276) in those given
placebo, a significant reduction of 34% (95% confidence
interval 13-50%). Poor outcomes (death, vegetative state,
severe disability) were also significantly reduced by 40%
(95% confidence interval 20-55%) with nimodipine [20% (55/
278) in patients given nimodipine vs. 33% (91/278) in those
given placebo]

JAM Japanese Adult Moyamoya trial Compared medical management and EC-IC bypass for
prevention of rebleeding in Japanese adults with
hemorrhagic Moyamoya disease over a 5-year period.
Forty-two were randomized to surgery and 38 to
conservative management. The primary outcome was
defined as adverse events (stroke, transient ischemic attack,
or recurrent hemorrhage) and it occurred in 14.3% of
patients in the surgical group and 34.2% in the conservative
care group (p ¼ 0.048). The secondary outcome, rebleeding,
was observed in 11.9% of patients in the surgical group and
31.6% in the conservative care group (p ¼ 0.042)

ESCAPE/ Endovascular treatment for Small Core All three of these trials aiming to validate mechanical
EXTEND- and Anterior circulation Proximal thrombectomy for acute ischemic stroke were prematurely
IA/SWIFT- occlusion with Emphasis on halted due to positive interim analyses compared to
PRIME minimizing CT to recanalization times intravenous thrombolysis
Extending the time for Thrombolysis in
Emergency Neurological Deficits with
Intra-Arterial therapy
Solitaire™ With the Intention For
Thrombectomy as PRIMary treatment
for acute ischemic stroke

Neurosurgery Books Full


www.ketabpezeshki.com 66485438-66485457
22A CRANIAL VASCULAR NEUROSURGERY I: ANEURYSMS AND AVMs 305

37. 1—d, Parkinson’s triangle; 2—h, Kawase’s triangle; 3—g, Glasscock’s triangle

Triangle Borders Access to


Anteromedial Medial IIIrd, lateral optic nerve, tentorial edge Clinoid ICA
triangle between IIIrd and optic nerve entry to optic
canal

Medial Connect points: Lateral margin supraclinoid Horizontal portion of cavernous ICA
(Hakuba’s) ICA, dural entrance of IIIrd, anterolateral
triangle posterior clinoid process

Paramedian Lateral edge IIIrd, medial IVth, tentorial edge Medial loop of intracavernous ICA and
triangle between entry points of III and IV meningohypophyseal trunk

Parkinson’s Lateral aspect of IV, medial aspect of V1, dural Entire intracavernous ICA (lateral to
triangle edge between entry points of IV and V1 proximal ring)
Entire intracavernous course VI (Dorello’s
canal to SOF)

Anterolateral Between V1, V2 and bone between foramen Superior orbital vein, CN VI, carotid-
(Mullan’s) rotundum (V2 exit) and SOF cavernous fistulae
triangle

Lateral triangle Between V2, V3 and bone between foramen Sphenoidal emissary vein (between
rotundum (V2 exit) and foramen ovale (V3 cavernous sinus and pterygoid venous
exit) plexus), masses extending laterally in the
cavernous sinus

Posterolateral Greater superficial petrosal nerve, posterior Identifies the triangular area of bone that
(Glasscock’s) aspect of V3, and line between foramen must be removed to access horizontal
triangle spinosum and arcuate eminence of petrous intrapetrosal ICA for proximal control or
bone bypass graft

Posteromedial Quadrangular space. Posterior border of V3, Removal of bone here gives access to
(Kawase’s) arcuate eminence, greater superficial petrosal petroclival area, anterolateral brainstem,
triangle nerve, petrous ridge/superficial petrosal sinus and vertebrobasilar junction. Corridor
between V and VII/VIII

Inferomedial Posterior clinoid process, VI at Dorello’s canal, Dural opening to Dorello’s canal (Gruber’s
triangle IV at edge of tentorium, petrous apex ligament forms lateral wall)

Inferolateral Lateral to inferomedial triangle. VI at Dorello’s Dural opening into Meckel’s cave
triangle canal, IV at edge of tentorium, petrosal vein at
superior petrosal sinus, petrous apex at base

Neurosurgery Books Full


www.ketabpezeshki.com 66485438-66485457
CHAPTER 22B

CRANIAL VASCULAR
NEUROSURGERY II: CEREBRAL
REVASCULARIZATION AND STROKE
SINGLE BEST ANSWER (SBA) QUESTIONS
1. A 75-year-old man with a history of recent hypertension or malignancy. Non-contrast
memory impairment is admitted with CT scan and GRE MRI are shown. Which
headache, confusion, and a left hom- oneofthefollowingisthemostlikelycauseofthis
onymous hemianopsia. There is no history of patient’s symptoms and signs?

a. Multi-infarct dementia d. Undiagnosed hypertension


b. Mycotic aneurysm e. Gliomatosis cerebri
c. Amyloid angiopathy

306
Neurosurgery Books Full
www.ketabpezeshki.com 66485438-66485457
22B CRANIAL VASCULAR NEUROSURGERY II: CEREBRAL REVASCULARIZATION AND STROKE 307

2. A 71-year-old presents with sudden headache a. Based on a prospective study of 275


and confusion. CT is shown. Which one of patients with aneurysmal subarachnoid
the following is most likely cause? hemorrhage treated in a single center
b. Assesses risk of vasospasm on initial diag-
nostic cerebral angiography
c. Intended to guide timing of aneurysm
clipping based on grades of surgical risk
d. Meningeal reaction alone does not
increase surgical hazard
e. Suggests that in the absence of ICH, grade
III patients should be operated on early

5. Which one of the following statements


regarding the Fisher and Modified Fisher
scales is LEAST accurate?
a. Fisher Scale (1980) was proposed to predict
cerebral vasospasm after aneurysmal SAH
and retrospectively validated in 47 patients
b. Fisher scale (1980) utilized blood clot
thickness measurements still calculable
from modern CT scans
c. Fisher scale (1980) grade 4 SAH is charac-
terized by localised clot and/or vertical
layer within the subarachnoid space
>1 mm thick
d. After adjusting for early angiographic vaso-
spasm, history of hypertension, neurologi-
cal grade, and elevated admission mean
arterial pressure, the Fisher scale (1980)
a. Vasculitis remains a significant predictor of vasospasm
b. Mycotic aneurysm e. Odds ratio of symptomatic vasospasm in
c. Amyloid angiopathy Modified Fisher grade 4 SAH is two-fold
d. Undiagnosed hypertension higher than those with Grade 0-1 SAH
e. Gliomatosis cerebri
6. Which one of the following statements
3. Which one of the following statements regarding moyamoya disease is LEAST
regarding the World Federation of Neuro- accurate?
logical Surgeons (WFNS) subarachnoid a. Incidence is higher in Japan compared to
grading scale is most accurate? Western countries
a. It was derived from statistical analysis of a b. More prevalent in females
large cohort of consecutive SAH patients c. Adults usually present with progressive
from a single center cerebral ischemia
b. It was derived from statistical analysis of a d. Progression of disease is more commonly
large cohort of consecutive SAH patients seen in children
from multiple centers e. Ischemic symptoms can be triggered by
c. It was created based on expert opinion crying in children with moyamoya disease
using the results of the International
Cooperative Aneurysm Study 7. Which one of the following statements
d. It is inferior to the Hunt & Hess grading regarding moyamoya in pregnancy/lifestyle
scale in terms of predicting outcome at is LEAST accurate?
3 months a. Contraception increases risk of bypass
e. It is superior to the modified Fisher score graft thrombosis
in predicting risk of vasospasm b. Blood donation may increase the risk of
TIA/stroke in moyamoya patients
4. Which one of the following statements c. Cesarean section is advisable in those with
regarding the Hunt & Hess subarachnoid Suzuki stage 3 moyamoya
hemorrhage grading scale is most accurate? d. Aspirin should be continued throughout
pregnancy

Neurosurgery Books Full


www.ketabpezeshki.com 66485438-66485457
308 PART III CRANIAL NEUROSURGERY

e. Good cerebral circulation on SPECT or 11. A 73-year-old patient has sudden onset left
absence of frequent symptoms due to facial droop left hemiparesis 3/5 and slurred
moyamoya disease within 1 year before speech, left facial droop developing 90 min
pregnancy may reduce complications ago. CT head scan does not show any large
associated with vaginal delivery infarct of hemorrhage. ASPECT score is
10. CT angiography is performed and shown
8. Which one of the following statements below. CT perfusion shows elevated mean
regarding diagnosis of moyamoya disease is transit time, reduced cerebral blood flow,
most accurate? and preserved cerebral blood volume in the
a. Unilateral moyamoya disease can be diag- right MCA territory. There is no past medi-
nosed with MRI and MRA cal history and he has not had any recent
b. Suzuki stage 1 shows first evidence of surgery. BP is 179/95, HR 102. Which one
developing moyamoya vessels at the base of the following evidence-based strategies is
of the brain appropriate?
c. Diagnosis does not requires exclusion of
causes of secondary “moyamoya syndrome”
d. Bilateral stenosis at the terminal portion of
the ICA and abnormal vascular networks in
the vicinity of the stenotic lesion in the
arterial phase on angiography and no sec-
ondary cause for this appearance would
be sufficient to diagnose moyamoya disease
e. Suzuki stage 6 is characterized solely by the
complete absence of moyamoya vessels

9. Which one of the following is the most


appropriate indications of cerebral revascu-
larization surgery in moyamoya disease?
a. Suzuki stage 3 moyamoya disease on cere-
bral angiography
b. Intracranial hemorrhage
c. Recurrent ischemic episodes in a child
triggered by crying
d. Asymptomatic moyamoya
e. Planned pregnancy

10. Which one of the following statements


regarding complications of cerebral bypass
surgery for moyamoya disease is LEAST
accurate?
a. Cerebral hyperperfusion syndrome gen-
erally occurs 2-6 days after STA-MCA
bypass a. Intravenous thrombolysis, if unsuccess-
b. Cerebral hyperperfusion can result in ful in 30 min proceed to mechanical
hemorrhagic conversion of moyamoya thrombectomy
disease b. Mechanical thrombectomy without
c. Watershed shift phenomenon is com- thrombolysis
moner in adults than children c. Intravenous thrombolysis followed by
d. Watershed shift phenomenon describes heparin infusion
retrograde blood supply from STA- d. Aspirin 300 mg
MCA bypass may interfere with the anter- e. Warfarinization
ograde blood flow from the
proximal MCA 12. Which one of the following statements
e. Mechanical compression by swollen regarding stroke imaging is LEAST
temporal muscle flap can result in cerebral accurate?
edema

Neurosurgery Books Full


www.ketabpezeshki.com 66485438-66485457
22B CRANIAL VASCULAR NEUROSURGERY II: CEREBRAL REVASCULARIZATION AND STROKE 309

a. ASPECT score of >7 is associated with c. SPACE


unfavorable outcome with thrombolysis d. ICSS
b. Acute infarction is visible earlier on diffu- e. CREST
sion weighted imaging
c. MRI gradient echo (GRE) is useful for 16. Which of the following trials demon-
demonstrating microbleeds strated 2-year stroke reduction benefit for
d. CT/MR angiography can assess tissue symptomatic intracranial atherosclerotic dis-
perfusion ease patients with baseline hemodynamic
e. Prenumbra shows a normal or elevated insufficiency as defined by decreased cere-
regional cerebral blood volume, whereas bral blood flow and cerebrovascular reactiv-
this is reduced in infarcted tissue. ity on acetazolamide challenge as measured
on SPECT who underwent STA-MCA
13. Which one of the following statements bypass?
regarding randomized clinical trials compar- a. EC-IC Bypass Study (1985)
ing carotid endarterectomy to best medical b. Japanese EC-IC Bypass Trial (JET)
therapy is LEAST accurate? c. Carotid Occulsion Surgery Study
a. NASCET and ECST showed that the d. SAMMPRIS
degree of benefit individual symptomatic e. St. Louis Carotid Occlusion Study
patients gained from carotid endarterec-
tomy was directly proportional to the risk 17. In the modern era, which one of the follow-
they faced without surgery ing is the commonest indication for cerebral
b. Symptomatic male patients and those bypass surgery?
>70 years old benefit from CEA more a. Moyamoya disease
c. Risk of stroke recurrence at 30 days is 3% b. Complex intracranial aneurysm
after first ever stroke c. Skull base tumor causing carotid
d. Patients with carotid “near-occlusion” compression
benefitted most from surgery d. Traumatic ICA dissection
e. Carotid endarterectomy is 70-99% e. Extracranial carotid occlusion
carotid stenosis
18. Which one of the following statements
14. A 59-year-old man with a background of dia- regarding cerebral bypass grafts is LEAST
betes mellitus, hyperlipidemia, and hyperten- accurate?
sion presented with sudden left hemiparesis, a. STA graft patency is >95% at 2 years
hemisensory loss, and dysarthria due to right b. STA is a medium-high flow graft (initially
MCA infarct. He was treated with systemic 40-80 ml/min)
thrombolysis and made some recovery. A c. Saphenous vein graft patency is 82% at
carotid duplex ultrasound and CT angiogram 5 years
of the carotid arteries confirmed a severe (70- d. Initial flow through saphenous vein graft
99%) stenosis in the right internal carotid is 70-140 ml/min
artery (ICA) secondary to a 23-mm-long ath- e. High risk of vasospasm with radial artery
erosclerotic plaque extending from the interposition graft
carotid bifurcation to the level of C3 verte-
bral body with evidence of intraluminal 19. Which one of the following statements
thrombus. The left ICA showed moderate regarding management of complex intracra-
(50-69%) stenosis. Which one of the follow- nial aneurysms not amenable to clip recon-
ing is most appropriate? struction or coiling is LEAST accurate?
a. Carotid endarterectomy a. Flow-diverting stents can be used to
b. Carotid stent reconstruct the parent artery and are
c. Carotid angioplasty deployed across the neck of the aneurys-
d. Best medical therapy mal segment
e. EC-IC bypass b. Flow-diverting stents become endothelia-
lized over time
15. Which one of the following randomized clin- c. Intracranial-intracranial bypass may be
ical trials comparing carotid endarterectomy required distal to complex aneurysms
and carotid artery stenting showed non- d. A radial interposition graft may be used in
inferiority of carotid artery stenting to endar- short segment IC-IC bypass
terectomy in high-risk patients? e. Flow diverting stents are usually used in
a. SAPPHIRE conjunction with detachable coils for wide
b. EVA-3S necked aneurysms

Neurosurgery Books Full


www.ketabpezeshki.com 66485438-66485457
310 PART III CRANIAL NEUROSURGERY

SBA ANSWER
1. c—Amyloid angiopathy without later craniotomy for patients who have
deterioration. STICH II trial focused on those
Cerebral amyloid angiopathy (CAA) as a cause of with spontaneous, superficial lobar ICH (<1 cm
ICH has been implicated in as many as 15% from cortical surface; volume 10-100 ml) without
patients older than 60 years of age and almost intraventricular hemorrhage in conscious patients
20% of patients 70 years of age and older. Depo- and found that early surgery (<12 h after random-
sition of β-amyloid protein in the vessel walls of ization) does not increase the rate of death or dis-
small and medium sized arteries within the aging ability at 6 months compared to initial conservative
brain predisposes to both ICH and dementia. management (with delayed surgery if judged nec-
The locations of the bleeds are lobar or essary). Patients in the STICH II trial with a poor
cortical-subcortical as opposed to the basal gang- prognosis (GCS 9-12) did better with early
lia location in hypertensive IPH. Most com- surgery, whereas those with a good prognosis
monly, bleeds are seen in the frontal lobe, did not (as the ability to observe and operate on
followed by the parietal, occipital, and temporal only those who deteriorate is more beneficial over-
lobes. Hemorrhage into the deep gray matter all for this group).
or cerebellum is uncommon and there may be
evidence of prior macrohemorrhages or microhe- Image with permission from Mirvis SE, et al., editors.
Problem Solving in Emergency Radiology, Saunders,
morrhages. Patients with CAA are at substantial
Elsevier, 2015.
increased risk for recurrent hemorrhage,
estimated at approximately 10% annually. On FURTHER READING
CT it is common to see multiple microhemor- Mendelow AD, Gregson BA, Rowan EN, Murray GD,
rhages and hematomas of varying ages. Magnetic Gholkar A, Mitchell PM. STICH II Investigators. Early sur-
resonance imaging including GRE and/or gery versus initial conservative treatment in patients with
susceptibility-weighted imaging (SWI) is recom- spontaneous supratentorial lobar intracerebral haematomas
mended as a further step in evaluation of patients (STICH II): a randomised trial. Lancet 2013 Aug 3;382
suspected of CAA. MRI including MRA/MRV is (9890):397-408. doi: 10.1016/S0140-6736(13)60986-1. Epub
2013 May 29. Erratum in: Lancet. 2013 Aug 3;382
reasonably sensitive at identifying secondary (9890):396. PubMed PMID: 23726393; PubMed Central
causes of hemorrhage. A catheter angiogram PMCID: PMC3906609.
may be considered if clinical suspicion is high
or noninvasive studies are suggestive of an under- Mendelow AD, Gregson BA, Fernandes HM, Murray GD,
lying vascular cause. On GRE sequences micro- Teasdale GM, Hope DT, Karimi A, Shaw MD, Barer DH.
bleeds are round, punctate, hypointense foci less STICH investigators. Early surgery versus initial conservative
than 5-10 mm in size in brain parenchyma seen treatment in patients with spontaneous supratentorial intra-
cerebral haematomas in the International Surgical Trial in
in 80% of patients with primary ICH (hyperten- Intracerebral Haemorrhage (STICH): a randomised trial.
sion and amyloid angiopathy), 25% of patients Lancet 2005 Jan 29-Feb 4;365(9457):387-97. PubMed
with ischemic stroke, and 8% of elderly people. PMID: 15680453.
They correspond to hemosiderin-laden macro-
phages lying adjacent to the vessels and indicate 2. d—Undiagnosed hypertension
prior extravasation of blood. Microbleeds have
been suggested to be predictors of bleeding-prone Hypertensive ICH accounts for over 50% of
angiopathy. Some studies have shown that patients cases. Hemorrhage occurs most commonly in
with microbleeds may be at increased risk for ICH the basal ganglia /thalamus (80%), pons (5-10%)
after anticoagulation or thrombolytic treatment; and the cerebellar hemispheres (5-10%).
however, this is controversial and not confirmed Over 90% of the patients are older than 45 years
in all studies. The STICH trial randomized of age. The bleeding results from the rupture of
patients with spontaneous supratentorial ICH small penetrating arteries. In 1868 Charcot and
(<72 h; unlikely due to vascular malformation/ Bouchard described the cause of the bleeding as
aneurysm/tumor), hematoma >2 cm, and GCS 5 rupture of Charcot-Bouchard microaneurysms
or more where there was clinical equipoise about in the walls of small penetrating arterioles (micro-
hematoma evacuation to either early surgery or aneurysms). Intraparenchymal hemorrhage may
initial conservative management with possibility result from a large and heterogeneous group of
of ICH evacuation if deemed appropriate by the causes, including primary causes such as hyperten-
treating physicians. They found that except for sion and amyloid angiopathy or secondary causes
possibly those with superficial ICHs, craniotomy such as AVM, intracranial aneurysms, cavernous
at 1 day or longer after onset is not better than angiomas, dural venous sinus thrombosis,
initial conservative medical treatment with or

Neurosurgery Books Full


www.ketabpezeshki.com 66485438-66485457
22B CRANIAL VASCULAR NEUROSURGERY II: CEREBRAL REVASCULARIZATION AND STROKE 311

intracranial neoplasms, coagulopathy, vasculitis, disability [GOS 4-5] and an unfavorable outcome
drug use, and hemorrhagic ischemic stroke. was severe disability, a vegetative state, or death
Underlying vascular abnormalities must always [GOS 1-3]) admission WFNS was shown to be
be considered and excluded if suspected due to predictive of outcome (p < 0.0001).
the high risk for recurrent hemorrhage and the
availability of treatment options. Clinical symp-
toms suggesting a secondary cause include pro- Unfavorable
drome of headache or neurologic deficits before Outcome (GOS1-3)
the onset of the accident or other clinical findings WFNS Description at 3 months
that suggest an underlying disease. Imaging find- I GCS 15 without 13%
ings suggestive of secondary causes include the motor deficit
presence of SAH and ICH at the same time, II GCS 13-14 without 24%
unusual shape of the hematoma, increased edema motor deficit
compared to the size of the hematoma, and visual-
III GCS 13-14 with 48%
ization of a masslike lesion or abnormal vessels. motor deficit
Hematoma expansion occurs in approximately
one third of acute primary IPH cases and is IV GCS 7-12 with or 55%
without motor
associated with high mortality, disability, deficit
and functional deterioration. The CT shown
demonstrates basal ganglia hemorrhage with V GCS 3-6 with or 66%
without motor
intraventricular extension and surgical manage- deficit
ment will be directed towards developing hydro-
cephalus via external ventricular drainage.
Image with permission from Hernalsteen D, Dignac A, FURTHER READING
Oppenheim C, et al. Hyperacute intraventricular he- Report of World Federation of Neurological Surgeons
morrhage: detection and characterization, a compar- Committee on a Universal Subarachnoid Hemorrhage
ison between 5 MRI sequences, J Neuroradiol 2007 Grading Scale. J Neurosurg 1988;68:985.
Mar;34(1):42-8.
Rosen DS, Macdonald RL. Grading of subarachnoid hemor-
3. c—It was created based on expert opinion rhage: modification of the World Federation of Neurosurgical
using the results of the International Cooper- Societies scale on the basis of data for a large series of patients.
ative Aneurysm Study Neurosurgery 2004;54:566-75.

In 1988, an expert opinion committee proposed Rosen DS, Macdonald RL. Subarachnoid hemorrhage
grading scales: a systematic review. Neurocrit Care 2005;2
the WFNS Scale based on the committee mem-
(2):110-18.
bers’ opinions that a SAH scale should (a) include
five grades, (b) be based on the GCS, and (c) 4. c—Intended to guide timing of aneurysm clip-
acknowledge the presence of a focal neurological ping based on grades of surgical risk
deficit. They considered data from the Interna-
tional Cooperative Aneurysm Study that assessed The Hunt and Hess scale (1968) aimed to retro-
the prognostic importance of headache, stiff spectively create an index of surgical risk and to
neck, and major focal neurological deficits. aid neurosurgeons in deciding on the appropriate
The analysis showed that Hunt and Hess grades time after SAH at which the neurosurgeon should
1 and 2 were prognostically the same because, operate. Surgical risk was felt to be best estimated
as long as consciousness was normal, headache by the intensity of meningeal inflammatory reac-
and/or stiff neck had no significant effect on out- tion, the severity of neurological deficit/level of
come. Secondly, the most important predictor of arousal (indicating arterial spasm, ischemia, and
mortality and disability was level of conscious- brain edema and thus greater vulnerability to
ness, and lastly the most important predictor of manipulation), and the presence of associated
disability (but not mortality) was hemiparesis disease. Their practice at that time was to take
and/or aphasia. The WFNS Scale compresses grade I and II to surgery as soon as a diagnosis
the GCS into five grades, with the addition of a could be made (ideally <24 h admission), while
fourth axis (focal neurological deficit) to differen- graded III-V treated conservatively until they
tiate grades 2 and 3. In a series of approximately improved to Grade I or II (except in the case of
3500 patients with SAH who were graded pro- multiple rebleeds or life-threatening ICH). After
spectively and assessed for outcome on the retrospective review of 275 cases, they concluded
GOS 3 months after aneurysmal clipping (favor- that aneurysm clipping can be accomplished with
able outcome was good recovery or moderate an extremely low mortality rate in the absence of

Neurosurgery Books Full


www.ketabpezeshki.com 66485438-66485457
312 PART III CRANIAL NEUROSURGERY

severe meningeal reaction, neurological deficit, FURTHER READING


or serious associated disease (preop Grade I Hunt W, Hess R. Surgical risk as related to time of interven-
1.4% versus Grade II 22% versus Grade III-IV tion in the repair of intracranial aneurysms. J Neurosurg
approx. 40%) and that meningeal reaction alone 1968;28:14.
(Grade II) increases surgical hazard. Thus they Rosen DS, Macdonald RL. Subarachnoid hemorrhage grad-
suggested prompt surgical intervention is impor- ing scales: a systematic review. Neurocrit Care 2005;2
tant for patients admitted in good condition, (2):110-18.
while for the more seriously ill conservative ther-
apy should be utilized until their condition 5. e—Odds ratio of symptomatic vasospasm for
improves. In 1974, Hunt and Kosnik proposed Modified Fisher grade 4 SAH is two-fold
a modification of their SAH scale by adding a zero higher than those with Grade 0-1 SAH
grade for unruptured aneurysms and 1a grade for
a fixed neurological deficit in the absence of other The Fisher Scale (1980) was proposed to predict
signs of SAH. Although the Hunt and Hess scale cerebral vasospasm after aneurysmal SAH and
is easy to administer, the classifications are arbi- prospectively validated in 47 patients: slight to
trary, some of the terms are vague (e.g. drowsy, severe vasospasm was seen in 4/11 (36%) grade
stupor, and deep coma) and some patients may pre- 1, 3/7 (43%) grade 2, 24/24 (100%) of grade
sent with initial features that defy placement within 3, and 2/3 (67%) of grade 4 patients. Limitations
a single grade. In one study which compared Hunt of this original scale include: (i) poor resolution
and Hess Scale with GCS, and WFNS Scale in a compared to current CT scans, (ii) blood thick-
series of 185 patients with aneurysmal SAH showed ness measurements used were actual measure-
that it had the strongest predictive power for GOS ments on printed CT scan images and had no
at 6 months, though half of poor-grade patients relationship to the real clot thickness, (iii) No
achieved good recoveries suggesting that current SAH and SAH <1 mm in true thickness (Grades
admission grading scales are not accurate enough 1 and 2) are both uncommon, and (iv) it does not
to be the sole basis for treatment decisions. They account for patients with thick SAH with ICH/
also found that scores on the day of operation were IVH or those with ICH/IVH alone. More recent
of more prognostic value than values observed evidence that the Fisher scale may not correlate
immediately after hospitalization. Furthermore with risk of vasospasm resulted in the Fisher
other studies have struggled to find outcome scale being compared to the Modified Fisher
differences between the individual grades, but did scale (2006) in 1355 patients with SAH (in
when lower grades were merged suggesting the placebo arm of RCT for tirilazad), of whom
possibility of an oversplitting error weakening the 33% developed vasospasm. Early angiographic
prognostic power of the scale. vasospasm, history of hypertension, neurological
grade, and elevated admission mean arterial
pressure were identified as risk factors for
symptomatic vasospasm. After adjusting for
Hunt and Hess SAH Scale (1968) these variables, the modified Fisher scale
Description
remained a significant predictor of symptomatic
NOTE: Grade must be increased by one vasospasm (adjusted OR 1.28, p ¼ 0.01) while the
level in the presence of serious systemic original Fisher scale was not (adjusted OR 1.1,
disease (hypertension, diabetes, severe p ¼ 0.488).
arteriosclerosis, COPD) or severe
Grade vasospasm on angiography.
I Asymptomatic or minimal headache and
slight nuchal rigidity
II Moderate to severe headache, nuchal
rigidity, cranial nerve palsy (but no other
neurological deficit)
III Drowsy, confusion, or mild focal deficit
IV Stupor, moderate-to-severe hemiparesis,
possibly early decerebrate rigidity, and
vegetative disturbances
V Deep coma, decerebrate rigidity,
moribund appearance

Neurosurgery Books Full


www.ketabpezeshki.com 66485438-66485457
22B CRANIAL VASCULAR NEUROSURGERY II: CEREBRAL REVASCULARIZATION AND STROKE 313

Risk of Vasospasm Based on Fisher and Modified Fisher Scales


Vasospasm Modified Fisher Vasospasm
Grade Fisher Scale OR Scale OR
0 No SAH or IVH -
1 No subarachnoid hemorrhage - Focal or diffuse -
thin SAH, no IVH
2 Diffuse or vertical layer of subarachnoid blood 1.3 Focal or diffuse 1.6
<1 mm thick (Diffuse thin SAH) thin SAH, with
IVH
3 Localized clot and/or vertical layer within the 2.2 Focal or diffuse 1.6
subarachnoid space >1 mm thick (focal or diffuse thick SAH, no
thick SAH) IVH
4 Diffuse thin SAH or no SAH, with ICH or IVH 1.7 Focal or diffuse 2.2
thick SAH, with
IVH

FURTHER READING population. It is more prevalent among women.


Fisher CM, Kistler JP, Davis JM. Relation of cerebral Genome-wide association study identified the
vasospasm to subarachnoid hemorrhage visualized by comput- RNF213 gene in the 17q25 region as a suscepti-
erized tomographic scanning. Neurosurgery 1980;6:1-9. bility gene for moyamoya disease among East
Frontera JA, Claasen J, Schmidt JM, Wartenberg KE, Temes Asians. Secondary causes of “moyamoya syn-
R, Connolly ES, Loch Macdonald R, Mayer SA. Prediction of drome” include infection, autoimmunity, other
symptomatic vasospasm after subarachnoid haemorrhage: the inflammatory conditions, and cranial irradiation.
modified Fisher scale. Neurosurgery 2006;58(7):21-27. Presentation may be: ischemic 63.4%, hemor-
rhagic 21.6%, epileptic 7.6%, and “other”
Rosen DS, Macdonald RL. Subarachnoid hemorrhage grad-
7.5%. The ischemic type of MMD predominates
ing scales: a systematic review. Neurocrit Care 2005;2
(2):110-18.
in childhood, making up 69% of cases in patients
under 10 years old and Ischemic symptoms are
6. c—Adults usually present with progressive often instigated by hyperventilation (e.g. crying).
cerebral ischemia The symptoms may present repetitively and can
result in motor aphasia, cortical blindness, mental
Moyamoya disease is characterized by bilateral retardation, and low IQ over the long term. The
stenosis or occlusion of the terminal portion of hemorrhagic type of MMD occurs in 66% of
the ICAs and/or the proximal portions of the adult cases exhibit hemorrhages with a higher
ACAs and MCAs. Moyamoya disease is also char- occurrence in females. Progression of occlusion
acterized by irregular perforating vascular net- is more common in children than adults. Due
works, called moyamoya vessels, near the to a poor response to medical therapy, direct
occluded or stenotic regions corresponding to and indirect cerebral bypass techniques have been
the lenticulostriate and thalamoperforate arteries. devised with the goals of promoting neoangio-
The associated tuft of collateral vessels that forms genesis, inducing collateral vessel formation,
at the base of the skull gives the angiographic and restoring perfusion to oxygen-deprived areas
appearance of a hazy “puff of smoke,” or “moya- of the brain. The direct techniques can immedi-
moya” in Japanese. Incidence rate in Japan is ately augment the blood supply as well as pro-
0.5-1 per 100,000 people, with a prevalence of mote neoangiogenesis. surgical intervention
10.5 patients per 100,000. Improved diagnostic improves the outcomes of patients with symp-
measures and prognosis for these patients may tomatic MMD. Direct revascularization is the
have contributed to the increase in the incidence treatment of choice and may lead to immediate
and prevalence of the disease. MMD cases in the improvement of symptoms, but is technically
US show a lack of bimodal age of onset, preva- challenging and is associated with risks such as
lence of the ischemic type at all ages, more hemorrhage and cerebral hyperperfusion syn-
benign symptoms at presentation, and better re- drome. The pediatric population is typically
sponse to surgical treatment. The incidence of treated with indirect revascularization because
MMD in California was only 0.087 per 100,000 (i) the likelihood of angiogenesis is higher in
from 1987 to 1998, even with a higher Asian children than in adults and (ii) direct bypass

Neurosurgery Books Full


www.ketabpezeshki.com 66485438-66485457
314 PART III CRANIAL NEUROSURGERY

is technically challenging and more prone to Clinical diagnosis of moyamoya disease is based
thrombosis in children. Disadvantages of indirect on (A) cerebral angiography (gold standard)
revascularization relate to longer time for collat- and/or (B) MRI/MRA, and (C) exclusion of sec-
eral formation and angiogenesis and that it may ondary moyamoya syndrome (arteriosclerosis,
preclude the option of subsequent direct bypass autoimmune, meningitis, brain tumor, Down's
in symptomatic children if the STA is used or syndrome, NF-1, head trauma, cranial irradia-
compromised. tion, sickle cell disease, and others). Historically,
only bilateral cases could be diagnosed defini-
FURTHER READING tively while unilateral cases with appropriate
Burke GM, et al. Moyamoya disease: a summary. Neurosurg criteria could be termed probable moyamoya dis-
Focus 2009;26(4):E11. ease. However, the most recent Japanese
Baaj AA, et al. Surgical management of Moyamoya disease: a diagnostic criteria state that definitive diagnosis
review. Neurosurg Focus 2009;26(4):E7. of moyamoya disease requires catheter angiogra-
phy in unilateral cases while bilateral cases could
7. c—Cesarean section is advisable in those with be promptly diagnosed by either catheter angiog-
Suzuki stage 3 moyamoya raphy or magnetic resonance (MR) imaging/
angiography. Cerebral angiography should show
General lifestyle advice for pre- or post-bypass (i) stenosis or occlusion at the terminal portion of
moyamoya patients includes: (i) avoid oral contra- the ICA and/or at the proximal portion of the
ceptives or hormone replacement therapy due to ACAs and/or the MCAs and (ii) abnormal vascu-
the risk of cerebral thrombosis (especially lar networks in the vicinity of the occlusive or ste-
through bypass graft), (ii) lifelong aspirin, (iii) notic lesions in the arterial phase. Cerebral
ensure headgear/helmets do not constrict blood angiography is not mandatory when MR imaging
supply to the bypass, (iv) avoid donating blood and MR angiography clearly demonstrate: (i)
due to risk of TIA/stroke from loss of intravascu- bilateral stenosis or occlusion at the terminal por-
lar volume, (v) normal pregnancy and vaginal tion of the ICA and at the proximal portion of the
delivery is possible under specialist joint care. ACAs and MCAs on MR angiography, and (ii)
Cerebral infarction and intracranial hemorrhage bilateral abnormal vascular network in the basal
are the major concerns in pregnancies with moya- ganglia on MR angiography (>2 apparent flow
moya disease because these conditions greatly voids are observed in 1 side of the basal ganglia
influence the prognoses of the mother and new- on MR imaging). Cerebral angiography serves
born infant. Intrapartum, cerebral blood flow for diagnosis, surgical planning, and monitoring
decreases due to hyperventilation and increases progression via Suzuki staging. The utility of
due to elevation of blood pressure caused by pain Suzuki staging may be mostly in children (as
and uterine contractions. These increases and many adults remain within the same stage), and
decreases of cerebral blood flow cause cerebral even then most cases belong to stages 3-5, and
ischemia and hemorrhage. However, vaginal stages are not strongly related to clinical
delivery is possible if cerebral blood flow can be symptoms.
controlled, and this may be achieved by control-
ling blood flow to the brain with epidural
anesthesia. When vaginal delivery is selected, Suzuki Angiographic Staging
there is evidence to suggest that good cerebral of Moyamoya Disease
circulation on SPECT or absence of frequent
symptoms due to moyamoya disease within 1 year Suzuki
before pregnancy is important for avoiding Stage Angiographic Findings
complications. 1 Stenosis of suprasellar ICA, usually
bilateral
FURTHER READING 2 Development of moyamoya vessels at
Stanford Medicine (Neurosurgery): http://med.stanford.edu/ base of brain
neurosurgery/moyamoya/faq.html
3 Increasing ICA stenosis and
prominence of moyamoya vessels
Tanaka H, Katsuragi S, Tanaka K, et al. Vaginal delivery in (most cases diagnosed at this stage)
pregnancy with Moyamoya disease: experience at a single
institute. J Obstet Gynaecol Res 2015;41:517-22. 4 Entire circle of Willis and PCAs
occluded, extracranial collaterals start
8. d—Bilateral stenosis at the terminal portion of to appear, moyamoya vessels begin to
the ICA and abnormal vascular networks in diminish
the vicinity of the stenotic lesion in the arterial 5 Further progression of Stage 4
phase on angiography and no secondary cause 6 Complete absence of moyamoya
for this appearance would be sufficient to diag- vessels and major cerebral arteries
nose moyamoya disease
Neurosurgery Books Full
www.ketabpezeshki.com 66485438-66485457
22B CRANIAL VASCULAR NEUROSURGERY II: CEREBRAL REVASCULARIZATION AND STROKE 315

FURTHER READING 10. d—Watershed shift phenomenon is more


Suzuki J, Takaku A. Cerebrovascular “moyamoya” disease. common in adults than children
Disease showing abnormal net-like vessels in base of brain.
Arch Neurol 1969;20:288-99. Surgical complications of moyamoya disease
Research Committee on the Pathology and Treatment of
include both neurological and non-neurological
Spontaneous Occlusion of the Circle of Willis; complications, and neurological complications
Health Labour. include perioperative cerebral infarction and
cerebral hyperperfusion syndrome. Mechanisms
Sciences Research Grant for Research on Measures for Intrac- for ischemia include “watershed shift phenome-
table Diseases (2012) Guidelines for diagnosis and treatment non” where retrograde blood supply from STA-
of moyamoya disease (spontaneous occlusion of the circle of
MCA bypass may interfere with the anterograde
willis). Neurol Med Chir (Tokyo) 2012;52:245-66.
blood flow from the proximal MCA, and thus
9. c—Recurrent ischemic episodes in a child result in the temporary decrease in CBF at the
triggered by crying cortex supplied by the adjacent branch of
MCA—particularly in pediatric moyamoya dis-
Direct revascularization surgery such as ease. Secondly, thrombo-embolic complications
STA-MCA anastomosis, as well as indirect revas- related to the anastomosed site and the mechan-
cularization, is established as an effective proce- ical compression by swollen temporal muscle flap
dure for the moyamoya disease patients with could also cause cerebral ischemia in the acute
ischemic symptoms. However, the recently pub- stage. STA-MCA bypass may temporarily lead
lished results of the Japanese Adult Moyamoya to heterogeneous hemodynamic condition even
trial have provided us with level I evidence for within the hemisphere operated on. Cerebral
the potential benefit of direct cerebral revascular- hyperperfusion syndrome is one of the most seri-
ization for preventing recurrent bleeds in adults ous complications and may occur in nearly 40%
with hemorrhagic moyamoya disease. In sum- of adult patients with moyamoya disease 2-6 days
mary, 80 adult patients with a recent history after STA-MCA bypass. Rapid focal increase in
(<12 months) of cerebral hemorrhage were CBF (hyperemia) at the site of the anastomosis
randomly assigned to either direct extracranial- could result in vasogenic edema and/or hemor-
to-intracranial (EC-IC) bypass or medical rhagic conversion in moyamoya disease. Focal
management and followed for 5 years. Significant cerebral hyperperfusion can cause temporary
morbidity was seen in 34% of the patients man- focal neurological deficit such as aphasia, hemi-
aged conservatively compared with 14.3% of the paresis, and dysarthria in a blood pressure depen-
patients in the surgical group. Similarly, patients dent manner. In general, good perioperative
in the nonsurgical arm were approximately 3 times hydration, hemoglobin control, and routine use
more likely to experience a recurrent bleed than of anti-platelet agent are essential to avoid ische-
patients who underwent surgery (31.6% vs. mic complications.
11.9%) suggesting a preventive effect of direct
bypass against rebleeding. JAM trial strongly FURTHER READING
encourages direct revascularization surgery for Fujimura M, et al. Current status of revascularization surgery
reducing the risk for rebleeding in adult moya- for moyamoya disease: special consideration for its “internal
moya disease patients presenting with intracranial carotid-external carotid (IC-EC) conversion” as the physio-
logical reorganization system. Tohoku J Exp Med
hemorrhage, although the statistical significance
2015;236:45-53.
was marginal. Finally, revascularization surgery
for asymptomatic moyamoya disease patients is 11. a—Intravenous thrombolysis, if unsuccessful
not recommended due to the uncertainty of the in 30 minutes proceed immediately to
natural history of this patient population. mechanical thrombectomy

FURTHER READING In 1995 with the validation of intravenous recom-


Howard BM, Barrow DL. Cerebral revascularization: which binant tissue plasminogen activator (IV rtPA) in
patients should be bypassed and which patients should be studies demonstrating improved clinical out-
passed by? World Neurosurg 2015 Mar;83(3):288-90. comes in patients treated within 3 h of stroke
Fujimura M, et al. Current status of revascularization surgery ictus dramatically improved stroke therapy.
for moyamoya disease: special consideration for its “internal Despite the clinical benefits of IV rtPA, disap-
carotid-external carotid (IC-EC) conversion” as the physio- pointments remained concerning modest recana-
logical reorganization system. Tohoku J Exp Med lization rates, ranging between 4.4% for distal
2015;236:45-53. internal carotid artery occlusion, 4% for basilar

Neurosurgery Books Full


www.ketabpezeshki.com 66485438-66485457
316 PART III CRANIAL NEUROSURGERY

artery occlusions and 30% for middle cerebral gradient echo can demonstrate hemorrhage
artery (MCA) M1 and M2 segment occlusions. (and microhemorrhage), while DWI can demon-
Initial trials of endovascular therapy versus IV strate acute infarction very early on (e.g. 10 min
rtPA alone (e.g. MERCI, IMS III, SYNTHESIS in animal models). CT and MR perfusion imag-
Expansion, MR RESCUE trials) failed to defini- ing can be used to assess the proportion of sal-
tively demonstrate superiority of mechanical vageable (prenumbra) and non-salvageable
embolectomy—possibly due to use of first gener- (infarcted) tissue before deciding on endovascular
ation stent retrievers with poor recanalization therapy utilizing parameters shown below (TTP,
rates, and limited availability of advanced imaging MTT, CBF, CBV).
to confirm vessel occlusion and identify prenum-
bral pattern/infarct core. Recently, however, 5
rTTP MTT rCBF rCBV
published randomized controlled studies
(ESCAPE, MR CLEAN, SWIFT Prime, Tissue Increased Increased Reduced Normal
at risk or
EXTEND IA, REVASCAT) using new genera- (prenumbra) elevated
tion devices with recanalization rates 58-88%
Infarct/dead Significantly Increased Reduced Reduced
and advanced CT/MR imaging for patient selec- increased
tion (assessing collateral circulation, mismatch
ratio and ischemic core volume) have provided TTP, time to peak; MMT, mean transit time, CBF, cerebral
overwhelming evidence in support of IV rtPA blood flow, CBV, cerebral blood volume (area under
plus mechanical thrombectomy for acute ische- the curve).
mic stroke in patients with proximal large artery
(ICA/MCA) compared to IV rtPA alone. In the 13. d—Patients with carotid “near-occlusion”
studies, the odds of a favorable outcome (mRS benefitted most from surgery
2 or less at 90 days) in the endovascular group
were at least twice that in controls, without any The estimated 30-day risk of stroke recurrence
difference in 30-day mortality or symptomatic after first stroke is 3% at 30 days and 26% at
ICH between groups. Furthermore, the benefit 5 years. The NASCET investigators stratified
was maintained in old age (>80) and those with patients into groups with “low moderate”
severe stroke (based on NIHSS). Further studies (<50%), “high moderate” (50-69%) and
underway to assess its role in those with wake up “severe” (70-99%) carotid stenosis. For the
strokes and those outside treatment timeframe as severe stenosis, the risk of any major stroke or
stratified by advanced imaging. death was 32.3% in the medical group and
Image with permission from Quiñones-Hinojosa A.
15.8% in the surgical group at two years and sta-
Schmidek and Sweet's Operative Neurosurgical Tech- tistically significant. Furthermore, the degree of
niques, 6th ed. Saunders, Elsevier, 2012. benefit individual symptomatic patients gained
from carotid endarterectomy was directly pro-
FURTHER READING portional to the risk they faced without surgery:
Palaniswami and Yan: Mechanical Thrombectomy Is Now the patients with 50-69% stenosis had attenuated
Gold Standard for Acute Ischemic Stroke: Implications for
benefit, therefore would be expected to face a
Routine Clinical Practice. Intervent Neurol 2015;4:18-29.
lower risk without surgery. Patients with
12. d—CT/MR angiography can assess tissue <50% stenosis did not achieve a significant
perfusion reduction in the risk of ipsilateral stroke. In
ECST, the method of measurement of carotid
Advance imaging in stroke has and increasing role stenosis differed, but when the trial results were
in patient selection, and may be crucial in future reanalyzed using the NASCET method, similar
to discriminate where endovascular therapy may benefit for CEA was demonstrated. The five-
be of benefit in those outside of established treat- year risk reduction of “stroke or surgical death”
ment windows or with wakeup strokes. Alberta in ECST patients with 70-99% stenosis random-
Stroke Program Early Computed Tomography ized to CEA rather than medical treatment was
Score (ASPECTS) is a 10 point score used in 21.2%. In patients with 50-69% stenosis, the risk
MCA infarcts whereby 1 point is deducted for reduction was 5.7%. As expected, patients with a
every vascular region involved. An ASPECT lesser degree of stenosis did not benefit from sur-
score of 7 or below is associated with a worse gery. However, an additional important finding
functional outcome at 3 months, higher risk of is that patients with “near-occlusion”—those
symptomatic hemorrhage and unfavorable out- with evidence of collapse of the distal vessel indi-
come with thrombolysis. CT and MR angiogra- cating poor run-off flow in the carotid—did not
phy can demonstrate occlusion and length of benefit significantly from surgery. Indeed, many
clot and recanalization post-thrombolysis. MR patients with near-occlusion will progress to

Neurosurgery Books Full


www.ketabpezeshki.com 66485438-66485457
22B CRANIAL VASCULAR NEUROSURGERY II: CEREBRAL REVASCULARIZATION AND STROKE 317

complete occlusion of the artery, which pre- North American Symptomatic Carotid Endarterectomy Trial
cludes intervention. The risk/benefit ratio most Collaborators. Beneficial effect of carotid endarterectomy in
favors surgery over medical treatment in men symptomatic patients with high-grade carotid stenosis. N
Engl J Med 1991;325:445-453.
and in the elderly. In addition, the overall benefit
of surgery is diminished as the time between Barnett HJ, Taylor DW, Eliasziw M, Fox AJ, Ferguson GG,
symptoms and surgery increases, strongly argu- Haynes RB, Rankin RN, Clagett GP, Hachinski VC, Sackett
ing for intervention in the stable patient within DL, Thorpe KE, Meldrum HE, Spence JD. Benefit of carotid
two weeks of last symptoms. The rate of ipsilat- endarterectomy in patients with symptomatic moderate or
eral stroke in patients with asymptomatic carotid severe stenosis. North American Symptomatic Carotid
Endarterectomy Trial Collaborators. N Engl J Med
stenosis is much lower: possibly <0.5% per year
1998;339:1415-1425.
in patients with 50% stenosis treated with best
medical therapy. Two large randomized trials Randomized trial of endarterectomy for recently symptom-
provide much of the data available to address atic carotid stenosis: final results of the MRC European
the question of whether prophylactic CEA in Carotid Surgery Trial (ECST). Lancet 1998;351:1379-
the asymptomatic patient prevents stroke. The 1387.
Asymptomatic Carotid Atherosclerosis Study Endarterectomy for asymptomatic patients with high-grade
(ACAS) randomized 1662 patients with an stenosis. Executive Committee for the Asymptomatic Carotid
asymptomatic carotid stenosis of 60% or greater Atherosclerosis Study. JAMA 1995;273:1421-1428.
(measured using NASCET criteria) as detected Halliday A, Mansfield A, Marro J, Peto C, Peto R, Potter J,
on cerebral angiography or computerized Thomas D. MRC Asymptomatic Carotid Surgery Trial
tomography angiogram (CTA) either to daily (ACST) Collaborative Group. Prevention of disabling and
aspirin with risk factor management (BMT) or fatal strokes by successful carotid endarterectomy in patients
to CEA plus BMT. CEA reduced the rate of rate without recent neurological symptoms: randomised con-
of ipsilateral stroke or any perioperative stroke or trolled trial. Lancet 2004;363:1491-1502.
death from 11.0% to 5.1%. The Asymptomatic
Carotid Surgery Trial (ACST) randomized 14. a—Carotid endarterectomy
3,120 patients with 60-99% carotid stenosis on
Best medical therapy (BMT) alone may be pre-
Doppler ultrasound to either “immediate” end-
ferred for asymptomatic carotid occlusion if (i)
arterectomy (with half of patients being operated
patient life expectancy is less than the time to
on within one month after randomization) or
achieve stroke reduction benefit from revasculari-
deferral of CEA until a clinician considered there
zation (i.e. 2-3 years for CEA, and 5 years for
to be a clear indication for surgery. When
CAS), or (ii) if established periprocedure risk of
perioperative adverse events were combined
death/stroke is small enough (<3%) and/or high
with subsequent strokes over a five-year period,
future stroke risk factors (e.g. plaque ulceration,
CEA reduced the rate of events from 11.8%
contralateral ICA occlusion, male, intraluminal
to 6.4% (p < 0.0001) and subsequently main-
thrombus, young age) to ensure benefit despite
tained at 10 year follow up in those operated
small absolute risk reduction seen in studies (6%
<75. The results of these two trials, when com-
at 5 years in ACAS). In contrast, symptomatic
bined in meta-analysis with the asymptomatic
patients benefit almost immediately from CEA
patients of the Veterans Affairs Cooperative
due to their higher short-term stroke risk. How-
Studies, appear to support the practice of
ever, even symptomatic patients with dense hemi-
endarterectomy for asymptomatic carotid stenosis
spheric neurological deficits, significant dementia,
to reduce the risk of ipsilateral stroke over three
or severely limited functional status attributable
years but this risk reduction was marginal (6%
to poor cardiac, renal, or pulmonary reserve are
reduction over 10 years) compared to that in
unlikely to benefit from any form of carotid revas-
symptomatic patients, and that to be worthwhile
cularization. Factors outlining the decision
the surgical risk (MI, stroke, death) must be low
between CEA and CAS in appropriate individuals
and patient must also be prepared to incur an early
with symptomatic carotid stenosis are shown
risk of perioperative stroke to reduce the risk of
below, although patient preference and operator
a stroke that might not happen for many years.
experience/center specific outcomes are just as
In subgroup analyses, surgical intervention
important.
appeared to benefit men more than women and
younger patients more than older.

FURTHER READING
Doig D, Brown M. Carotid stenting versus endarterectomy.
Annu Rev Med 2012;63:259-76.

Neurosurgery Books Full


www.ketabpezeshki.com 66485438-66485457
318 PART III CRANIAL NEUROSURGERY

Patient Selection in Carotid Endarterectomy Versus Stenting


Factors Carotid Endarterectomy (CEA) plus BMT Carotid Artery Stent (CAS) plus BMT
Age Safer than CAS in patients 70 years Significantly increased risk of
of age. periprocedural stroke or death in CAS-
treated patients 70 years of age.
Life expectancy to At least 3 years At least 5 years
see stroke reduction
relative to BMT alone
Stroke risk factors Ipsilateral neurology in last 6 months Not suitable for lesions at high risk
present High grade (70-99%) stenosis of periprocedural embolization (e.g.
High risk plaque (ulcer, hemorrhage etc.) intraluminal thrombus)
Contralateral carotid occlusion increases
risk of CEA (inadequate collateral
circulation)
Neck anatomy Challenging if hostile neck anatomy Preferred for patients with hostile neck
Unclear if preferable to CAS for recurrent (previous neck surgery/radiotherapy,
carotid artery stenosis contralateral laryngeal nerve palsy,
tracheostomy)
Carotid/aortic arch Better outcomes than CAS if: ICA-CCA Preferred if hostile carotid: lesion distal to
anatomy angulation >60 deg., ICA lesion >10- C2 vertebral body or proximal to clavicle
15 mm long, ostial involvement of lesion, Increased risk of periprocedural stroke if
excessive calcification aortic arch challenging configuration,
tortuous, calcified.
Stroke risk reduction 16% absolute risk reduction at 2 years Conflicting evidence as to whether non-
and outcome compared to BMT alone in symptomatic inferior to CEA. Worse outcomes in
patients in NASCET (overall 10% risk symptomatic patient and >70. Non-
reduction once 6% perioperative risk inferior to CEA in high risk patients
considered)
Complications Greater risk of MI and cranial nerve injury Higher stroke risk and death
compared to CAS Lower risk of MI
Lower risk of periprocedure stroke risk
than CAS

15. a—SAPPHIRE meant that in symptomatic patients CAS is a


viable option in high risk patients, those with
Several large trials have shown the superiority of stenosis distal to C2 vertebral body (difficult to
carotid endarterectomy to best medical manage- treat with CEA), or if being performed by an
ment in patients with significant carotid stenosis experienced operator with established outcomes
(>70-99%) in terms of reducing risk of recurrent equivalent to CEA. Due to fewer trials demon-
stroke. Many head to head trials of carotid strating non-inferiority of CAS for asymptomatic
end-arterectomy (e.g. EVA-3S, SPACE, ICSS) patients, it should at present only be conside-
have failed to prove non-inferiority of CAS red if being performed by an experienced
compared to CEA in standard risk patients. operator with established outcomes equivalent
The CREST trial found CAS to be non-inferior to CEA.
to CEA in symptomatic or asymptomatic stan-
dard risk patients, with no difference in rates of FURTHER READING
death, stroke or MI at 4 years (but did find a sta- Hussain MA, et al. Carotid artery revascularization: what’s the
best strategy? Circulation 2015;131:2226-31.
tistically higher 30-day risk of stroke for CAS and
MI for CEA). The Stenting and Angioplasty with 16. b—Japanese EC-IC Bypass Trial (JET)
Protection in Patients at High Risk for Endarter-
ectomy (SAPPHIRE) trial, showed non- Symptomatic ICAD portends a high rate of
inferiority of CAS in high-risk patients at 3 years recurrent, disabling ischemic strokes. In fact, sev-
(e.g. clinically significant cardiac disease [conges- eral large clinical trials have documented recur-
tive heart failure, abnormal stress test, or need for rence rates of 14% -19% over 2 years, with the
open-heart surgery], contralateral carotid disease, majority of events occurring in the 1st year. In
severe pulmonary disease). These results, have the Warfarin-Aspirin Symptomatic Intracranial

Neurosurgery Books Full


www.ketabpezeshki.com 66485438-66485457
22B CRANIAL VASCULAR NEUROSURGERY II: CEREBRAL REVASCULARIZATION AND STROKE 319

Disease (WASID) study –a retrospective, multi- occlusion. Based on this finding, the Carotic
center trial that compared the efficacy of warfarin Occlusion Surgery Study (COSS; 2011) was a
with aspirin for the prevention of major vascular prospective, randomized, open-label, blinded-
events –73% of patients with recurrent strokes adjudication trial that randomly assigned 195
had ischemic lesions in the territory of the symp- patients with i) angiographically demonstrated
tomatic artery. One recent clinical trial –Stenting complete occlusion of the ICA causing either
versus Aggressive Medical Management for TIA/ischemic stroke within 120 days and ii)
Preventing Recurrent stroke in Intracranial Ste- hemodynamic cerebral ischemia indicated by an
nosis (SAMMPRIS) –also demonstrated that in increased OEF on PET imaging to either STA-
patients with symptomatic ICAD, best medical MCA bypass or medical management. The
therapy was superior to angioplasty and stenting. STA-MCA arterial bypass patency rate was
451 patients with symptomatic major intracranial 98% at the 30-day postoperative visit and 96%
artery stenosis of 70% -99% were randomized to at the last follow-up examination. The STA-
stenting with the Wingspan system or “aggressive MCA arterial bypass markedly improved,
medical therapy” alone. Both groups received although it did not normalize, the level of elevated
aspirin, clopidogrel, and management of cardio- OEF in the symptomatic cerebral hemisphere.
vascular risk factors including blood pressure However, STA-MCA bypass failed to afford
and cholesterol. The trial included 94 patients patients in the surgical arm any benefit over
with intracranial carotid stenosis. The overall patients in the medical group, primarily because
30-day rate of stroke or death for all trial patients of better than expected stroke reduction in the
was 14.7% in the stenting group and 5.8% in the non-operative arm. COSS trial design has been
medical management group, a significant differ- criticised for counting severe, disabling stroke as
ence that prompted the trial's data and safety an endpoint and not giving equal weight to the
monitoring board to recommend termination of effects of living with chronic hypoperfusion or
enrollment. In the medical management group, continuous TIA events. These trials also do not
12.2% of patients reached the primary endpoint address `hot patients' such as those with postural
(stroke or death within 30 days after enrollment hypoperfusion, crescendo TIAs and recurrent
or after a revascularization procedure for the stroke. In contrast, the Japanese EC/IC Bypass
qualifying lesion, or ischemic stroke in the terri- Trial (JET) demonstrated 2-year stroke reduction
tory of the qualifying artery after day 31) at one benefit for patients with baseline hemodynamic
year, a lower figure than that reported in the med- insufficiency as defined by decreased cerebral
ical management arms of the trials of CEA for blood flow and cerebrovascular reactivity on acet-
carotid stenosis. Although these cohorts might azolamide challenge as measured on SPECT who
not be directly comparable, this lower rate of underwent STA-MCA bypass, suggesting that
stroke may again reflect advances in medical ther- better patient selection using SPECT and PET
apy in recent decades. The EC-IC Bypass Study may be required to determine those likely to ben-
(1985) was the first prospective, multicenter efit. Patients with known carotid occlusive disease
international study comprising 1377 patients with were screened for hemodynamic insufficiency
symptomatic intracranial ICA stenosis who were by measuring cerebral blood flow and cerebrovas-
randomly assigned to best medical care and EC- cular reactivity on SPECT as well as OEF on
IC bypass (superficial temporal or occipital artery positron emission tomography. The study pro-
to MCA). Despite a bypass patency rate of 96%, spectively enrolled 49 patients, and STA-MCA
30-day surgical mortality and major stroke mor- statistically reduced strokes compared with medi-
bidity rates were 0.6% and 2.5%, respectively. cal therapy (0.7% vs. 6.5%) for patients with
The study was highly criticized because the reduced cerebral blood flow and cerebrovascular
authors were unable to identify a subgroup of reactivity and increased OEF.
patients for whom EC/IC bypass may yield ben-
efit. However, in light of the high bypass patency FURTHER READING
rate, hope remained that if an appropriate cohort Powers WJ, Clarke WR, Grubb RL Jr, Videen TO, Adams
could be identified, patients with symptomatic HP Jr, Derdeyn CP; COSS Investigators. Extracranial-
cerebral ischemia could still benefit from surgical intracranial bypass surgery for stroke prevention in
revascularization. The St. Louis Carotid Occlu- hemodynamic cerebral ischemia: the Carotid Occlusion
Surgery Study randomized trial. JAMA. 2011 Nov 9;306
sion Study, which showed that patients with cere-
(18):1983-92.
bral hemodynamic insufficiency demonstrated by
increased oxygen extraction fraction (OEF) on Bauer AM, Bain MD, Rasmussen PA. Chronic Cerebral Ische-
PET were at the greatest risk of stroke after med- mia: Where "Evidence-Based Medicine" Fails Patients.
ical management for atherosclerotic carotid World Neurosurg. 2015 Sep;84(3):714-8.

Neurosurgery Books Full


www.ketabpezeshki.com 66485438-66485457
320 PART III CRANIAL NEUROSURGERY

Howard BM, Barrow DL. Cerebral revascularization: which


Type Comments Demand Initial Flow Patency
patients should be bypassed and which patients should be
passed by? World Neurosurg. 2015 Mar;83(3):288-90. doi: Superficial Pedicle vessel Low flow 15-25 ml/ >95%
temporal min (may (e.g. EC-
10.1016/j.wneu.2014.12.045. Epub 2015 Jan 14. PubMed artery increase IC bypass
PMID: 25596433. with time) trial,
COSS)
Radial Artery Same size as Moderate- 40-80 ml/ >90% at
17. a—Moyamoya disease interposition M2 branch. High flow min 5 years
graft Risk of
vasospasm.
Yasargil first described intracranial arterial Saphenous Greater High flow 70-140 ml/ 82%
vein graft length but min patency
bypass, and indications for surgery quickly has valves at 5 years
expanded to include treatment of complex aneu-
rysms, moyamoya disease, extensive skull base
neoplasms, and occlusive vasculopathy. How-
ever, the rates of bypass are falling for all of these FURTHER READING
Regli L, Piepgras DG, Hansen KK. Late patency of long
indications due to advances in (medical, endo-
saphenous vein bypass grafts to the anterior and posterior
vascular and radiotherapy) and well as better evi- cerebral circulation. J Neurosurg 1995;83:806-11.
dence showing absence of long term benefit
(except in highly selected patients). Innovations Houkin K, Kamiyama H, Kuroda S, et al. Long-term patency
and increased experience in endovascular tech- of radial artery graft bypass for reconstruction of the internal
niques (e.g. flow-diverting stents) have limited carotid artery. J Neurosurg 1999;90:786-90.
the number of complex aneurysms that require
open surgical trapping and bypass. Similarly, 19. e—Flow diverting stents are usually used in
skull base tumors, previously treated by resec- conjunction with detachable coils for wide
tion and bypass, are often managed with less necked aneurysms
invasive surgery combined with newer, more
effective chemotherapy and radiosurgery regi- Complex aneurysms (e.g. dolichoectactic and
mens. Microsurgical bypass in most centers thrombosed aneurysms) not amenable to coil-
following publication of the EC/IC Bypass ing, direct clipping or clip reconstruction in
Study and the Carotid Occlusion Surgery Study the modern era can be managed with cerebral
(COSS). Although extracranial/intracranial bypass, or increasingly with flow diverting
(EC/IC) bypass remains a viable treatment stents like the Pipeline Embolization Device
for moyamoya angiopathy, particularly in (ev3, Irvine, California, USA) for large, giant,
adults, patients are increasingly being treated wide-necked, failed treatment and fusiform
with various indirect revascularization proce- aneurysms by reconstructing the parent artery.
dures, such as encephaloduroarteriosynangiosis During a flow-diversion/Pipeline™ procedure,
(EDAS). a microcatheter is navigated past the aneurysm
(without entering it) then, the flow-diverting
FURTHER READING stent is deployed across the neck of the aneu-
Howard BM, Barrow DL. Cerebral revascularisation: which rysm in the parent blood vessel, immediately
patients should be bypassed and which patients should be reducing blood flow to the aneurysm and induc-
passed by? World Neurosurg 2015;83(3):288-90. ing thrombosis within it. The device becomes
endothelialized, forming a permanent biological
18. b—STA is a medium-high flow graft seal across the diseased (aneurysmal) segment of
(initially 40-80 ml/min) the parent artery. The recently published
results of the International Retrospective Study
Intravascular thrombosis is more likely when of the Pipeline Embolization Device demon-
grasping the intima, if there is vessel stenosis at strated an overall combined neurologic morbid-
the anastomotic line (ideally fish mouthing ity and mortality rate of 8.4%. The combined
required) or if the vessel is overly stretched. Main- morbidity and mortality rate in patients with
taining patency is favored by fish mouthing ICA aneurysms 10 mm was 9.5% and in
the anastomosis in end to side grafts, using inter- patients with posterior circulation aneurysms
rupted sutures to allow maximum expansion, and was 16.4%. Since the introduction of the EC/
other factors such as flow demand through the IC bypass, several intracranial/intracranial
bypass and length of the donor graft used in the (IC-IC) bypass methods, which represent an
bypass. Properties of common grafts are shown elegant solution to otherwise untreatable aneu-
above. rysms, have been described. The in situ bypass

Neurosurgery Books Full


www.ketabpezeshki.com 66485438-66485457
22B CRANIAL VASCULAR NEUROSURGERY II: CEREBRAL REVASCULARIZATION AND STROKE 321

requires 2 parallel arteries (e.g. ACA, PCA, originates from the aneurysm itself is an advan-
SCA, distal PICA, MCA branches) which are tage of bypass when endovascular therapy
anastomosed side-to-side distal to the aneu- would risk occluding the perforating branch
rysm. Alternatively, reanastomosis of the parent or clip reconstruction would risk incomplete
artery or branches there of proximal and distal neck occlusion and aneurysm recurrence.
to the aneurysm after complete resection of the
diseased portion of the vessel is an attractive FURTHER READING
technique in places where the parent artery Kallmes DF, et al. International retrospective study of the
has sufficient slack. Short segment intracranial/ pipeline embolization device: a multicenter aneurysm treat-
intracranial bypass with a radial artery interpo- ment study. AJNR Am J Neuroradiol 2015 Jan;36(1):108-15
sition graft is often a good alternative to EC/IC (Erratum in: AJNR Am J Neuroradiol 2015 May;36(5)).
bypass, due to graft artery diameter being Howard BM, Barrow DL. Cerebral revascularization: which
similar to intracranial vessels. Finally, the ability patients should be bypassed and which patients should be
to reimplant an important arterial branch that passed by? World Neurosurg 2015 Mar;83(3):288-90.

Neurosurgery Books Full


www.ketabpezeshki.com 66485438-66485457
CHAPTER 23

CRANIAL ONCOLOGY
SINGLE BEST ANSWER (SBA) QUESTIONS
1. In the UK, which one of the following state- c. Colorectal cancer has a higher propensity
ments regarding driving restrictions due to for brain metastases than breast cancer
neurological disorders is LEAST accurate? d. Melanoma is the third most commonly
a. Driving can be reconsidered 6 months diagnosed type of brain metastases
after craniotomy for a benign meningi- e. Prostate cancer is the most frequent can-
oma if there is no seizure history cer of males but has a low propensity to
b. Driving can be considered after 12 months metastasize to the brain
for most craniotomies
c. Driving can be considered whenever there is 4. A 67-year-old patient presents with left
no residual impairment likely to affect driv- hemisensory change. Postcontrast MRI is
ing after trans-sphenoidal pituitary surgery shown below, and diffusion weighted imag-
d. Driving can be considered after 6 months ing shows the lesion to be dark on DWI
for after craniotomy for a benign brain- and bright on ADC map. Which one of the
stem tumor if asymptomatic following options is most appropriate next?
e. Driving can be considered 3 years after cra-
niotomy for high-grade glioma if safe to do
so and no evidence of tumor progression

2. Which one of the following lists of primary


brain tumors is in order of frequency (highest
to lowest)?
a. Glioblastoma multiforme, meningioma,
nerve sheath tumors, diffuse astrocytoma,
pituitary tumors
b. Meningioma, glioblastoma multiforme,
diffuse astrocytoma, pituitary tumors,
nerve sheath tumors
c. Meningioma, glioblastoma multiforme,
pituitary tumors, nerve sheath tumors,
diffuse astrocytoma
d. Meningioma, pituitary tumors, glioblas-
toma multiforme, nerve sheath tumors,
diffuse astrocytoma
e. Pituitary tumors, meningioma, glioblas-
toma multiforme, nerve sheath tumors,
diffuse astrocytoma
a. Urgent image-guided drainage of lesion
3. Which one of the following statements b. CT of chest, abdomen and pelvis with
regarding brain metastases in adults is contrast
LEAST accurate? c. Imaging surveillance
a. Brain metastases are over twice as com- d. Intravenous antibiotics
mon in small cell lung cancer than non- e. Lumbar puncture
small cell lung cancer
b. Distribution of brain metastases in the
CNS is proportional to amount of arterial
blood supplied

322
Neurosurgery Books Full
www.ketabpezeshki.com 66485438-66485457
23 CRANIAL ONCOLOGY 323

5. Which one of the following indications for c. SRS could be considered in multiple brain
stereotactic biopsy of a brain lesion is metastases (4-10) when the primary
LEAST appropriate? tumor is known to be radiotherapy
a. Deep seated lesions resistant
b. Infiltrative lesion d. Surgical resection should be considered in
c. Lesions in eloquent cortex the setting of a dominant hemisphere
d. Lesions not curable by surgical excision metastasis in a critical location
(e.g. brainstem tumors) e. SRS could be considered in oligometas-
e. Suspected frontal renal cell carcinoma tases if they are greater than 4 cm in
brain metastasis diameter

6. Which one of the following statements 9. A 55-year-old right handed male presents
regarding biopsy of brainstem lesions is with headache and cognitive slowing. There
LEAST accurate? is no significant past medical history. MRI is
a. Contralateral extraventricular transfron- shown. Which one of the following manage-
tal approach is suited to more lateral pon- ment strategies is most appropriate?
tine lesions
b. Ipsilateral transfrontal approach may have a
higher risk of intraventricular hemorrhage
c. Is more commonly used in adults com-
pared to children
d. Occipital transtentorial approach is rou-
tinely used
e. Suboccipital, transcerebellar approach is
associated with greater postoperative pain

7. Which one of the following statements


regarding average prognosis of patients pre-
senting with Karnofsky of score less than 70
is most accurate?
a. A Karnofsky performance score less than
70 is associated with a median survival of
2 months
b. A Karnofsky performance score less than
70 is associated with a median survival of
4 months
c. A Karnofsky performance score less than
70 is associated with a median survival of
6 months
a. Surveillance imaging
d. A Karnofsky performance score less than
b. Awake craniotomy with goal of maximal
70 is associated with a median survival of
safe resection
8 months
c. Cerebral angiogram
e. A Karnofsky performance score less than
d. Gross total resection under general
70 is associated with a median survival of
anesthetic
12 months
e. Stereotactic biopsy for molecular
classification
8. Which one of the following statements
regarding management of brain metastases
is LEAST accurate?
a. Chemotherapy/biologics should be con-
sidered alone when asymptomatic brain
metastasis is found on screening before
planned systemic therapy
b. Whole brain radiotherapy should be con-
sidered in the setting of multiple brain
metastasis (4-10) especially if primary
tumor is known to be radiotherapy
sensitive

Neurosurgery Books Full


www.ketabpezeshki.com 66485438-66485457
324 PART III CRANIAL NEUROSURGERY

10. A 44-year-old patient with a known history 13. Two-year survival in glioblastoma multi-
of relapsing remitting multiple sclerosis forme patients receiving post-surgery temo-
presents with worsening memory. MRI is zolomide and radiotherapy verus in the
shown below. MRI spectroscopy shows randomized controlled trial by Stupp and
reduced NAA and myoinositol, increased colleagues (2005) is which one of the
choline and lipid, lactate peaks. Perfusion following?
weighted MR shows markedly elevated a. 16.5%
cerebral blood flow in the rim of the b. 26.5%
necrotic mass. Which one of the following c. 36.5%
best explains his new deterioration? d. 46.5%
e. 56.5%

14. Which one of the following statements


regarding radiological phenomena follow-
ing modern treatment of high-grade glio-
mas is most accurate?
a. Tumor recurrence has a lower FDG
PET uptake than radiation necrosis
b. Radiation necrosis typically occurs
2-3 months after radiotherapy
c. Pseudoprogression is associated with
anti-VEGF pharmacotherapy
d. Pseudoresponse typically occurs 6-12
months after temozolomide chemora-
diotherapy
e. Recurrent tumors usually show a lower
ADC than radiation necrosis on diffu-
sion weighted MRI

15. Which one of the following statements


regarding advanced imaging in gliomas is
a. Tumefactive multiple sclerosis LEAST accurate?
b. Glioblastoma a. Anaplastic astrocytoma
c. Lymphoma b. Oligodendrogliomas commonly show
d. Oligodendroglioma calcification (central, peripheral or
e. Choroid plexus carcinoma ribbon like)
c. FDG-PET imaging of WHO grade III
11. Which one of the following factors is most gliomas shows an uptake greater than
important in improving length of survival in white matter but lower than gray matter,
gliomas? whereas grade II gliomas have an uptake
a. 1p19q codeletion similar to white matter
b. ATRX mutation d. MR perfusion imaging shows elevated
d. EGFR mutation regional cerebral blood flow in grade
e. IDH1/2 mutations III oligodendrogliomas compared to
c. TERT mutation grade II oligodendrogliomas
e. Glioblastomas show reduced NAA and
12. Median survival advantage in glioblastoma myoinositol peaks and increased choline,
multiforme patients undergoing 5-ALA lipid and lactate peaks on MR spec-
fluorescence assisted tumor resection versus troscopy
conventional surgery in the randomized
controlled trial by Stummer and colleagues 16. Which one of the following PET tracers is
(2006) was which one of the following? LEAST appropriate for detection of de
a. No advantage novo low-grade glioma?
b. 1 month advantage a. 18F-FET
c. 3 month advantage b. 18F-FDG
d. 5 month advantage c. 18F-DOPA
e. 7 month advantage d. 11C-MET
e. 18F-DOPA

Neurosurgery Books Full


www.ketabpezeshki.com 66485438-66485457
23 CRANIAL ONCOLOGY 325

17. A 34-year-old male presents with seizures. a. Functional mapping is a perquisite for
He has no significant past medical history. resection
MRI is shown (FLAIR) and T1 postcontrast b. MRS findings may include increased
imaging does not show any enhancement. 2-hydroxyglutarate
Which one of the following management c. Prognosis is related to extent of resection
strategies is most appropriate? d. The majority of patients with dominant
hemisphere lesions of this type present
with seizures
e. Tumor margins are usually seen best on
T1 + gad MRI sequences

19. Which one of the following statements


regarding surgical management of low grade
glioma (LGG) is LEAST accurate?
a. Gross total resection involves taking the
tumor until its border as visualized on
T2/FLAIR MRI sequences
b. Craniotomy under GA with functional
mapping is the standard of care for low-
grade gliomas in eloquent cortex
c. Biopsy of low-grade gliomas are prone to
histological undergrading as they may
miss anaplastic foci
d. Extent of resection correlated with
survival
e. PET imaging can facilitate biopsy targets
a. Imaging surveillance until starts to show in low-grade glioma
focal enhancement
b. Gamma knife surgery 20. Which one of the following statements
c. Methotrexate chemotherapy regarding brain mapping is LEAST
d. Dexamethasone accurate?
e. Maximal safe resection a. Connectome refers to the organization of
the CNS into parallel networks, which are
18. A 27-year-old presents with a generalized dynamic, interactive and able to compen-
tonic-clonic seizure. On examination there is sate for each other
no residual neurological deficit or speech dis- b. Hodotopy suggests individual functions of
turbance. Some spots of calcification are seen the brain are supported by extensive circuits
on CT therefore MRI is performed. Which comprising both the cortical nodes and
one of the following statements regarding this connections between them created by asso-
type of tumor is LEAST accurate? ciating bundles of white matter
c. Cortical map reorganization for a given
function can occur as long as the subcor-
tical white matter tracts subserving it are
preserved
d. Brodmann areas represent a connectionist
model of brain function
e. Neuroplasticity is seen in slow growing
low-grade gliomas more than in acute
insults such as stroke

21. Which one of the following chemotherapy


options is most likely to be utilized in the
context of anaplastic oligodendroglioma?
a. Anti-VEGF
b. Cyclophosphamide
c. Etoposide
d. PCV
e. Temozolomide

Neurosurgery Books Full


www.ketabpezeshki.com 66485438-66485457
326 PART III CRANIAL NEUROSURGERY

22. Which one of the following statements d. Mutant IDH is the molecular basis of the
regarding O(6)-Methylguanine-DNA Methyl CpG island methylator phenotype
Transferase (MGMT) promoter methylation (CIMP) in gliomas, leading to global dys-
status is LEAST accurate? regulation of gene expression
a. MGMT is a DNA repair enzyme that e. IDH1/2 mutations co-segregate with 1p/
reverts the naturally occurring mutagenic 19q codeletion in oligodendrogliomas
O6-methylguanine back to guanine, pre- f. IDH1/2 mutation is associated with an
venting errors during DNA replication improved prognosis in grade II
b. Functional MGMT increases the effec- astrocytomas
tiveness of cancer chemotherapy
c. Hypermethylation of the MGMT pro-
QUESTIONS 25–40
moter results in less DNA repair activity
of MGMT Additional questions 25–40 available on
d. Temozolomide chemotherapy is more ExpertConsult.com
effective in glioblastoma multiforme cells
with a hypermethylated MGMT promoter
e. MGMT promoter-unmethylated tumors
have no survival benefit from temozolo-
mide chemotherapy EXTENDED MATCHING ITEM (EMI)
QUESTIONS
23. Which one of the following statements
regarding telomerase reverse transcriptase 41. Supratentorial lesions:
(TERT) promotor mutations in gliomas is a. Anaplastic oligodendroglioma
LEAST accurate? b. Central neurocytoma
a. Overexpression of TERT in cells results c. Diffuse astrocytoma
in an increase in the number of times a cell d. DNET
can successfully divide e. Germinoma
b. TERT normally prevents telomerase f. Glioblastoma multiforme
repair ensuring that cells become replica- g. Metastatic melanoma
tively senescent h. Non-germinomatous germ cell tumor
c. TERT promoter mutations were also i. Oligodendroglioma
strongly associated with 1p/19q codele- j. Pineoblastoma
tion and co-occur with IDH1/2 in k. Primary CNS lymphoma
oligodendrogliomas l. Subependymal giant cell astrocytoma
d. TERT and IDH1/2 mutations are largely
mutually exclusive in GBM and For each of the following descriptions, select the
astrocytomas most appropriate answers from the list above.
e. Gliomas with TERT promoter mutations Each answer may be used once, more than once
but no IDH1/2 mutation have been or not at all.
shown to have poor overall survival 1. A 30-year-old man with AIDS develops
headaches and left hemiparesis and is
24. Which one of the following statements found to have a right frontal white matter
regarding isocitrate dehydrogenase 1 or 2 homogeneously enhancing lesion. The
(IDH1/2) gene mutations in gliomas is lesion shows increased uptake on
LEAST accurate? Thallium SPECT
a. The most common mutation ( 90%) in 2. A 13-year-old presents with diabetes
glial brain tumors causes a substitution insipidus
of the amino acid Arginine to Histidine 3. A 17-year-old presents with Parinaud syn-
at codon 132 of the IDH1 gene drome. Blood tests show a markedly raised
b. IDH1 and 2 are homologous enzymes HCG, but normal AFP
decarboxylate isocitrate to α-ketoglutarate
(αKG)
c. IDH1/2 mutations result in accumulation 42. Posterior fossa lesions:
of 2-hydroxyglutarate to high levels in a. Brainstem glioma
glioma tissues possibly promoting onco- b. Choroid plexus papilloma
genic transformation through epigenetic c. Dysplastic cerebellar gangliocytoma
mechanisms d. Ependymoma

Neurosurgery Books Full


www.ketabpezeshki.com 66485438-66485457
23 CRANIAL ONCOLOGY 327

e. Epidermoid d. MGMT promoter methylation


f. Hemangioblastoma e. 1p19q co-deletion
g. Medulloblastoma f. EGFRvIII
h. Meningioma g. Histone H3
i. Pilocytic astrocytoma h. ATRX
j. Vestibular schwannoma i. BRAF
j. 10q loss
For each of the following descriptions, select the
most appropriate answers from the list above. For each of the following descriptions, select the
Each answer may be used once, more than once most appropriate answers from the list above.
or not at all. Each answer may be used once, more than once
1. Commonest posterior fossa primary brain or not at all.
tumor in adults 1. A mutation which gives patients with IDH
2. Commonest posterior fossa primary brain wild type, ATRX mutant (negative) astro-
tumor in children cytoma a similar prognosis to glioblastoma
3. Associated with mutation in PTEN tumor multiforme
suppressor gene 2. Highly prevalent in all gliomas except
primary glioblastoma multiforme
3. Improves prognosis in oligodendroglio-
43. Molecular classification of brain tumors: mas, irrespective of IDH1/2 status
a. TERT 4. Target for glioblastoma vaccine trials
b. TP53 5. Predictive of benefit from temozolomide
c. IDH1/2 mutation in primary GBM

SBA ANSWERS
1. e—Driving can be considered 3 years after • Pituitary tumor surgery: driving can resume
craniotomy for high-grade glioma if safe to when safe after trans-sphenoidal surgery but
do so and no evidence of tumor progression if a craniotomy is required 6 months off
driving
The guidelines below relate to car/motorcycle • Chronic neurological disorders (e.g. multi-
use (not heavy goods vehicles) and will vary based ple sclerosis, motor neuron disease, Alzhei-
on individual risk assessment: mer’s) DVLA should be informed, complete
application for driving license holders state
• First seizure: 6 months off driving if the of health
license holder has undergone assessment by • Syncope: simple faint: no restriction, single
an appropriate specialist and no relevant episode, explained and treated: 4 weeks off,
abnormality has been identified on investiga- single episode, unexplained: 6 months off,
tion, for example, EEG and brain scan where two or more episodes: 12 months off.
indicated. For patients with established epi- • Stereotactic radiosurgery: Do not drive for
lepsy they must be fit free for 12 months 1 month after treatment
before being able to drive • Benign brainstem/posterior fossa tumor:
• Stroke or TIA: 1 month off driving, multi- can return to driving as soon as recovered
ple TIAs over a short period of time: from surgery but let DVLA know (you do
3 months off driving not need to tell DVLA about acoustic
• Craniotomy for low-grade tumor: 1 year off neuromas unless you have dizziness).
driving (if the tumor is a benign meningi-
oma and there is no seizure history, license 2. d—Meningioma, pituitary tumors, glioblas-
can be reconsidered 6 months after surgery toma multiforme, nerve sheath tumors, diffuse
if remains seizure free) astrocytoma
• Craniotomy for high-grade tumor: 2 years
off driving, and no evidence of tumor pro- The commonest intracranial tumors are brain
gression before metastases (just over 50%). Incidence of primary

Neurosurgery Books Full


www.ketabpezeshki.com 66485438-66485457
328 PART III CRANIAL NEUROSURGERY

brain tumors is approximately 20-30 per 100,000 The majority of brain metastases diagnosed origi-
in adults and 5 per 100,000 children. Approxi- nate from lung, breast, melanoma, renal and colo-
mately one third of primary brain tumors in adults rectal primary tumors—reflecting how common
are malignant whereas they account for two thirds those primary cancers are, but not necessarily their
in childhood. Frequency of WHO subgroups and respective propensity for metastasizing to the
specific tumors is given: brain. Propensity for spread to brain parenchyma
is high in melanoma, small cell lung cancer, cho-
riocarcinoma, and other germ cell tumors; inter-
Brain Tumour Frequency mediate in breast cancer, non-small cell lung
cancer (adenocarcinoma > squamous cell), and
Commonest Primary renal cell carcinoma; low in prostate, colorectal,
Brain Tumors by Commonest Primary Brain ovarian carcinoma, thyroid cancer and sarcomas.
WHO Group Tumors by Subtype
Metastases spread via the circulation and seed at
Meninges (37.6%) Meningioma (36%) the gray-white matter junction, and particularly
Neuroepithelial Pituitary tumors (15.5%) watershed areas (most obviously PCA vs. MCA
tissue (29.9%) Glioblastoma multiforme border) in a distribution proportional to amount
Sellar region (16.3%) (15.1%)
Cranial and spinal Nerve sheath tumors (8.1%)
of arterial blood supplied: 80% occur in cerebral
nerves (8.1%) Diffuse astrocytoma (2.3%) hemispheres, 15% in posterior fossa and 5% in
Unclassified (5.6%) Primary CNS lymphoma (2%) the brainstem. The frequency of metastases found
Lymphoma and Ependymal (1.9%) at autopsy is much higher than that detected
hematopoietic Anaplastic astrocytoma during the illness.
(2.1%) (1.7%)
Germ cell tumors Pilocytic astrocytoma (1.4%)
and cysts (0.4%) Neuronal and neuronal- FURTHER READING
mixed (1.2%) Schouten LJ, Rutten J, Huveneers HA, Twijnstra A. Incidence
Oligodendroglioma (1.1%) of brain metastases in a cohort of patients with carcinoma of
Oligoastrocytoma (0.9%) the breast, colon, kidney, and lung and melanoma. Cancer.
Craniopharyngioma (0.8%) 2002;94(10):269.
Anaplastic
oligodendroglioma (0.5%) Barnholtz-Sloan JS, et al. Incidence proportions of brain
metastases in patients diagnosed (1973-2001) in the Metropol-
itan Detroit Cancer Surveillance System. J Clin Oncol.
2004;22(14):2865.
Ostrom QT, Gittleman H, Fulop J, et al. CBTRUS Statistical
Report: Primary Brain and Central Nervous System Tumors Cancer Research UK, http://www.cancerresearchuk.org/
Diagnosed in the United States in 2008-2012. Neuro Oncol health-professional/cancer-statistics/incidence/common-
2015;17 Suppl 4:iv1. cancers-compared, Accessed February 2016.

3. c—Colorectal cancer has a higher propensity Brain Metastases. In: Oncology of CNS tumours. Springer
for brain metastases than breast cancer Verlag, Berlin 2010 pp.345-346.

Frequency of Metastatic Cancer and Relationship to Occurence of Brain Metastases


Cancer in Cancer in Cancer in Propensity for Radiological Contribution to Total
Order of Order of Order of Brain Metastases at 5 Years Brain Metastases
Frequency Frequency Frequency Follow Up (Descending Diagnosed (in
(Overall) (Males) (Females) Frequency) Descending Frequency)

(1) Breast (1) Prostate (1) Breast (1) Lung (16-20%; SCLC 29% (1) Lung
(2) Prostate (26%) (31%) vs. NSCLC 12%) (2) Breast
(3) Lung (2) Lung (2) Lung (2) Melanoma (7%) (3) Melanoma
(4) Colorectal (14%) (12%) (3) Renal cell carcinoma (4) Colon
(5) Uterus (3) Colorectal (3) Colorectal (7-10%) (5) Prostate
(6) Malignant (13%) (11%) (4) Breast (5%) (6) Liver/pancreas
melanoma (4) Bladder (4) Uterus (5) Colorectal (1%)
(4%) (5%)
(5) Kidney (5) Malignant
(4%) melanoma
(6) NHL (4%) (4%)
(7) Malignant (6) Ovarian
melanoma (4%)
(4%)

Neurosurgery Books Full


www.ketabpezeshki.com 66485438-66485457
23 CRANIAL ONCOLOGY 329

4. b—CT of chest, abdomen and pelvis with much earlier diagnosis of brain tumors. Although
contrast some tumors have a characteristic appearance on
imaging, no imaging modality is yet able to pro-
MRI shows a peripherally enhancing, centrally vide sufficient diagnostic information to direct
necrotic lesion in the right thalamus, with DWI subsequent aggressive therapy. The goal of
pattern consistent with a relatively unrestricted biopsy is to provide a representative sample for
diffusion in the center of the mass hence this is pathologic diagnosis to guide subsequent treat-
most likely a metastasis (given previous history ment, which can include cytoreductive surgery,
of breast cancer) or a primary tumor. As such, ini- radiotherapy, or chemotherapy. The main stimu-
tial management in a patient should consist of a lus for the adoption of stereotactic biopsy over an
search for the primary tumor based on a full clin- open operative procedure is to achieve a higher
ical examination and staging CT of the body, fol- rate of diagnostic accuracy while minimizing
lowed by discussion in the primary tumor site potential adverse effects. Diagnostic accuracy is
multidisciplinary meeting to decide on options important for dictating appropriate adjuvant
for tissue diagnosis and further management, as therapy. Particular characteristics of the tumor
well as the neuro-oncology MDT. The primary that favor the use of stereotactic biopsy over open
neoplasms that most commonly metastasize to biopsy include (1) lesions not requiring emergent
the brain are carcinoma of the lung, breast, malig- surgery or that are not curable by surgical exci-
nant melanoma, renal cell carcinoma, and GI can- sion, such as metastases or malignant intrinsic
cers (e.g. colorectal). Generally, metastases brain tumors; (2) deep-seated lesions or those
appear as multiple rounded lesions with a ten- occupying space in eloquent cortex or deep nuclei
dency to seed peripherally in the cerebral sub- (i.e. basal ganglia, thalamus), where open resec-
stance, at the gray/white matter junction. They tion would lead to unacceptable morbidity/mor-
can, however, occur anywhere in the cerebrum, tality; and (3) infiltrative lesions (i.e. gliomatosis
brainstem or cerebellum, and can also spread to cerebri) that do not have a clear brain-tumor mar-
the meninges. Metastases are characterized by gin and are unlikely to be excised completely
edema in the surrounding white matter which is without significant loss of normal brain paren-
often disproportionate to the size of the tumor chyma. Moreover, if the lesion's appearance on
itself. On T2 images, the neoplastic nodule may imaging or the course of the disease suggests an
blend with the surrounding edema, giving a picture alternative cause such as an infectious or demye-
of widespread vasogenic edema and obscuring the linating process rather than a neoplastic one, ste-
diagnosis. Most metastases enhance strongly with reotactic biopsy is a more appropriate first step
IV contrast medium, either uniformly, or ring-like than a large open procedure. Relative contraindi-
if the metastasis has outgrown its blood supply. cations include vascular tumors (e.g. metastatic
Most metastases from lung and breast are similar renal cell carcinoma, choriocarcinoma, or meta-
in density to normal brain parenchyma on CT, static melanoma) where diagnosis and biopsy
but some types are spontaneously dense, particu- the primary neoplasm instead is generally recom-
larly deposits from malignant melanoma. Hemor- mended, close proximity to a major blood vessel/
rhage occurs in about 10% of metastases, resulting sylvian fissure/cavernous sinus/brain-pial border
in high signal on T1 images and high or low signal all increase the risk of hemorrhage.
on T2 images. Similar signal characteristics can
also occur in non-hemorrhagic metastases from 6. d—Occipital transtentorial approach is
melanoma, due to the paramagnetic properties of routinely used
melanin. Small metastases and those that are not
made conspicuous by surrounding edema are often Lesions within the brainstem have long been
only detected on contrast-enhanced studies. considered challenging to diagnose and treat.
Increasing the contrast dose or relaxivity of gado- Although brainstem tumors represent only about
linium compounds can improve the sensitivity for 2% of all intracranial tumors in adults as compared
detection of metastases on MRI. with about 10-15% in the pediatric population,
radiologic diagnosis of brainstem lesions in adults
Image with permission from Kang TW, Kim ST, Byun is inaccurate 10-20% of the time whereas in chil-
HS, et al. Morphological and functional MRI, MRS, per- dren the majority are gliomas (diagnosable on
fusion and diffusion changes after radiosurgery of brain
metastasis, Eur J Radiol. 2009;72(3):370-380. MRI). In general, because most adult brainstem
tumors are not amenable to surgical excision, ste-
5. e—Suspected frontal renal cell carcinoma reotactic biopsy is important for obtaining a path-
brain metastasis ologic diagnosis enabling replacement of empirical
treatment modalities with more specific therapies,
The significant development of intracranial as well as determination of a more accurate prog-
imaging over the past few decades has allowed nosis. Equally, given the great diversity of

Neurosurgery Books Full


www.ketabpezeshki.com 66485438-66485457
330 PART III CRANIAL NEUROSURGERY

Approaches for Brainstem Biopsy


Brainstem Biopsy Approach Comments
Occipital, transtentorial Uncommon as crosses the pia superior and inferior to the tentorium and places
vital vasculature and cranial nerves at risk

Suboccipital, transcerebellar For lesions of the lower midbrain, pons, and rostral medulla projects through the
ipsilateral middle cerebellar peduncle, pain associated with muscle dissection
before making the twist drill hole in the skull. A more decisive contraindication to
this approach is the risk for posterior fossa subdural hematomas caused by blind
puncture of the cerebellar cortex

Ipsilateral transfrontal Midbrain, upper pons, and medulla. For lower pons or medulla biopsies, an
ipsilateral transfrontal approach necessitates a trajectory traversing the lateral
ventricle before entering the anterior thalamus, cerebral peduncle, and then the
brainstem. Lateral midbrain, a trajectory lateral to the ventricle suffices.
approaching the pons and medulla with this method often requires entering the
frontal horn of the lateral ventricle, which violates two ependymal surfaces and
places the patient at risk for intraventricular hemorrhage. Second,
ventriculostomy may lead to intraoperative loss of cerebrospinal fluid, which
may contribute to postoperative headache and shifting of intracranial contents,
thereby possibly altering the location of the target. Third, this route is limited to
midline regions of the pons and midbrain by the tentorial incisura

Contralateral Lesions below lower pons, and lateral pontine lesions. An intraparenchymal
extraventricular transfrontal trajectory is projected to avoid the lateral ventricle, the tentorium, major vessels,
and the cerebral aqueduct

brainstem lesions, patients most likely to benefit Oncology Group (RTOG) brain metastases trials
from stereotactic biopsy are those who are given and resulted in three groups being identified:
a better prognosis based on biopsy results or Class 1 (20%)—Patients who had a Karnofsky
who are spared a course of debilitating therapy. performance score 70 or higher, were less
Such patients include those with radiation necro- than 65 years of age, and had a controlled
sis, chemotherapy-sensitive metastasis, a lym- primary tumor without extracranial metas-
phoma, or an abscess rather than a malignant tases had a favorable prognosis (median sur-
glioma. In terms of complications, 6.6% have tran- vival was 6-7 months).
sient or mild symptoms and 1.8% have permanent Class 2 (65%)—Patients with a Karnofsky
deficits. performance score 70 or higher, but with
other unfavorable characteristics (e.g. uncon-
trolled primary tumor, other extracranial
7. a—A Karnofsky performance score less than metastases, age >65 years) had an intermedi-
70 is associated with a median survival of ate prognosis (median survival 4 months).
2 months From a further management point of view,
they are treated as either RPA Class 1 or
The median survival of patients who receive sup- RPA Class 3, depending largely upon the
portive care with brain metastases and are treated likelihood of controlling systemic disease.
only with corticosteroids is approximately 1- Class 3 (15%)—Patients with a Karnofsky
2 months. The use of whole brain radiation therapy performance score less than 70 have a poor
in large series increased the average survival to 3- prognosis (median survival of 2 months).
6 months, and larger gains were seen in carefully
selected subsets. The key parameters that deter- FURTHER READING
mine survival after the diagnosis of brain metastases Uptodate. Overview of the clinical manifestations, diagnosis,
are performance status, the extent of extracranial and management of patients with brain metastases. Topic
disease, and age, as well as the primary diagnosis. 5217 Version 16.0.
Recursive partitioning analysis (RPA) system was
8. e—SRS could be considered in oligometas-
based upon an analysis of prognostic factors in
1200 patients from three Radiation Therapy tases if they are greater than 4 cm in diameter

Neurosurgery Books Full


www.ketabpezeshki.com 66485438-66485457
23 CRANIAL ONCOLOGY 331

Brain Metastases: Indications for Different Treatment Modalities

Consider chemotherapy/biologics in brain metastasis:


• From highly chemotherapy-sensitive primary tumor
• Asymptomatic/found on screening MRI with planned systemic therapy
• Primary tumor with identified molecular alteration amenable to targeted therapy
• Other options exhausted and there is a reasonable drug available

Consider WBRT in brain metastasis when:


• CNS and systemic progression of disease, with few systemic treatment options and poor performance status
• Multiple (4-10) brain metastasis especially if primary tumor known to be radiotherapy sensitive (NB current data
support SRS use in up to 3 metastasis but there is a growing trend to use it in up to 10).
• Large (>4 cm) brain metastasis not amenable to SRS
• Postsurgical resection of a dominant hemisphere brain metastasis with multiple (4-10) remaining BMs
• Salvage therapy for recurrent BM after SRS or WBRT failure

Consider SRS when:


• Oligometastases (1-3) or multiple (4-10) metastases especially if primary tumor is known to be radiotherapy
resistant
• Postsurgical resection of a single BM, especially if 3 cm or bigger and in the posterior fossa
• Local relapse after surgical resection of a single brain metastasis
• Salvage therapy for recurrent oligometastases (1-3) after WBRT

Consider surgical resection when:


• Uncertain diagnosis of CNS lesion(s)
• 1-2 BMs, especially when associated with extensive cerebral edema
• Dominant BM in a critical location

No treatment is reasonable when:


• Systemic progression of disease with few treatment options and poor performance status

Table with permission from Lin X, DeAngelis LM. Treatment of Brain Metastases. J Clin Oncol. 2015;33(30):3475-3484.

hyperintensity is to a lesser degree than areas of


FURTHER READING central necrosis and surrounding edema, which
Lin X, DeAngelis LM. Treatment of Brain Metastases. J Clin are similar to CSF. The solid portion of the glio-
Oncol. 2015;33(30):3475-3484. blastomas may show complete or partial or
enhancement with contrast. The standard treat-
9. b—Awake craniotomy with goal of maximal
ment for glioblastoma (GBM) consists of surgery
safe resection
(with a variable extent of resection depending on
Glioblastoma (WHO grade IV) is the commonest tumor location and the patient's clinical status),
primary intracranial neoplasm in adults (fourth followed by a combination of radiotherapy and
commonest intracranial tumor after metastases, chemotherapy with temozolomide.
meningioma and pituitary tumors). About 90% Image with permission from Zikou AK, Alexiou GA,
of glioblastomas arise de novo (primary glioblas- Kosta P, et al. Diffusion tensor and dynamic susceptibil-
toma) and 10% are from malignant transforma- ity contrast MRI in glioblastoma, Clin Neurol Neuro-
tion of lower-grade astrocytomas (secondary surg. 2012;114(6):607-612.
glioblastoma). The two groups have different
genetic characteristics: primary glioblastomas, 10. b—Glioblastoma
which occurs in a slightly older age group, show
EGFR overexpression and secondary glioblasto- Tumefactive multiple sclerosis, high-grade gli-
mas show IDH mutations like the lower-grade oma (GBM), PCNSL and occasionally an abscess
gliomas from which they arise. Methylation of can appear similar on imaging. Tumefactive MS
the DNA repair gene MGMT is associated with refers to patients with known MS developing
a better response to temozolomide and better large tumefactive demyelinating plaques (as
prognosis in glioblastomas. The MRI appear- opposed to patients presenting with tumefactive
ances of glioblastomas are heterogeneous, show- demyelinating lesions who rarely go on to
ing a mixture of solid tumor portions, central develop MS).
necrosis and surrounding edema. The solid por-
Image with permission from Loevner, L, Brain Imaging:
tion is usually T1 hypointense, but T2/FLAIR Case Review Series, 2nd ed., 2009, Elsevier, Mosby.

Neurosurgery Books Full


www.ketabpezeshki.com 66485438-66485457
332 PART III CRANIAL NEUROSURGERY

Advanced MRI for Heterogenously Enhancing Lesions (Non-infective)


MR
MRI MR Spectroscopy Perfusion

Tumefactive 50% show enhancement, usually an open ring Elevation glutamate/ No elevation
demyelination with incomplete portion facing grey matter glutamine peaks in rCBV
Mildly increased diffusion (unlike abscess) Red NAA
Inc cho, lipid, lactate

High-grade Peripheral, heterogeneous enhancement with Reduced NAA Marked


glioma nodules and necrosis Reduce Myoinositol elevation
Can be ring enhancing Inc lipids, choline and rCBV
Solid parts have diffusion similar to normal white lactate
matter, but appears markedly elevated relative to
facilitated diffusion in edema/necrotic/cyst

Primary CNS Homogeneous enhancement common Large choline peak Modest


Lymphoma Ring enhancing in HIV/immunocompromise Reversed Cho/Cr ratio elevation
Restricted diffusion (lower ADC than metastasis Markedly reduced rCBV
or HGG) NAA
Lactate peak possible

11. b—ATRX mutation low glucose metabolism (FDG uptake) and low
rCBV, both of which tend to be high in tumor
FURTHER READING recurrence. On DCE perfusion imaging, recur-
Almeida JP, Chaichana KL, Rincon-Torroella J, Quinones- rent tumors show a much higher maximum slope
Hinojosa A. The value of extent of resection of glioblastomas: of enhancement than radiation necrosis. ADC
clinical evidence and current approach. Curr Neurol Neurosci measurements of the enhancing components in
Rep. 2015;15(2):517.
recurrent tumor are significantly lower than in
12. d—5 month advantage (16.8 months vs. radiation necrosis, mirroring the higher cellular
11.8 months) density in recurrent neoplasms. The assessment
of tumor response and progression in GBM had
FURTHER READING traditionally been based on measurements of
Stummer W, Pichlmeier U, Meinel T, Wiestler OD, Zanella enhancing tumor portions known as Macdonald
F, Reulen HJ; ALA-Glioma Study Group. Fluorescence- criteria. With the advent of combined chemora-
guided surgery with 5-aminolevulinic acid for resection of diation as standard therapy and antiangiogenic
malignant glioma: a randomised controlled multicentre phase drugs as second-line treatment, new phenomena
III trial. Lancet Oncol. 2006;7(5):392-401. PubMed such a pseudoprogression and pseudoresponse
PMID:16648043.
have to be taken into account and have made an
13. b—26.5% assessment solely based on assessment of enhanc-
ing tumor portion unreliable. Pseudoprogression
FURTHER READING (therapy induced necrosis) is due to an inflamma-
Hegi ME, et al. MGMT gene silencing and benefit from tory reaction, which results in a temporary
temozolomide in glioblastoma. N Engl J Med. 2005;352 increase of contrast enhancement and edema in
(10):997-1003. 20% of patient, usually within 12 weeks of
Stupp R, et al. Radiotherapy plus concomitant and adjuvant
temozolomide chemoradiotherapy, and subsides
temozolomide for glioblastoma. N Engl J Med. 2005;352 subsequently without additional treatment. Pseu-
(10):987-996. doprogression is more frequently observed in
patients with methylation of the DNA repair gene
14. e—Recurrent tumors usually show a lower MGMT, and is associated with a better prognosis
ADC than radiation necrosis on diffusion (longer overall survival). Advanced MR imaging
weighted MRI such as DSC and DCE perfusion imaging shows
promise in differentiating these two conditions
Radiation necrosis is a late complication of radio- from true tumor progression. Pseudoresponse is
therapy or gamma knife surgery, and can present characterized by a decrease of enhancement and
as an enhancing mass lesion 6-12 months after edema following the administration of antiangio-
radiotherapy, difficult to distinguish from recur- genic drugs without improved survival. In pseu-
rent tumor on conventional imaging. FDG- doresponse the tumor progresses by infiltrative
PET, PWI and DWI may help to distinguish patterns without neoangiogenesis, resulting in
between radiation necrosis and tumor recurrence. an increase of non-enhancing T2/FLAIR hyper-
In radiation necrosis the enhancing lesion has a intense tumor portions

Neurosurgery Books Full


www.ketabpezeshki.com 66485438-66485457
23 CRANIAL ONCOLOGY 333

FURTHER READING management of LGG is often more conservative,


Hygino da Cruz Jr LC, et al. Pseudoprogression and pseudor- even with an initial period of close observation,
esponse: imaging challenges in the assessment of posttreat- with serial imaging being considered in some
ment glioma. AJNR 2011;32:197801985. cases. Common molecular and genetic features
that are considered in the overall management
15. d—MR perfusion imaging shows elevated approach for gliomas include 1p/19q deletion sta-
regional cerebral blood flow in grade III oli- tus and isocitrate dehydrogenase (IDH) mutation
godendrogliomas compared to grade II status. With advances in MRI and positron emis-
oligodendrogliomas sion tomography (PET) imaging, there have been
developments to better characterize tumors non-
With recent advances in our understanding of invasively with respect to grade, known molecu-
prognostic and predictive factors of gliomas and lar, and genetic factors such as 1p/19q deletion
within current paradigms of care, glioma grade status and additional physiological features
and molecular genetic features frequently guide including tumor vascularity and metabolism. In
our management approach. In general, high- the future, the use of multiparametric/multimod-
grade gliomas are treated aggressively with up- ality imaging more routinely may make preoper-
front surgical resection followed by radiotherapy ative distinction between grade II and grade III
with or without chemotherapy. In contrast, the gliomas more sensitive.

Differential Appearances of Glioma Subtypes on Multi-modal Imaging


Glioma CT MRI MRS/Perfusion PET

Diffuse (infiltrative) Hyperdense T1 iso-hypointense, No restricted FDG uptake similar


astrocytoma (WHO No enhancement T2/FLAIR diffusion (unlike to while matter
grade II) (possibly wispy in hyperintense white infarct) Most
gemistocytic) matter lesion Elevated choline, low hypermetabolic
May be cystic expanding cortex NAA, elevated Ch/Cr area on FDG,
No enhancement ratio, myoinositol and 18F-Chlorine and
ml/Cr ratio 11C-Chlorine PET
Increased 2- useful for biopsy
hydroxyglutarate in
LGG if mutant IDH1/2

Anaplastic Heterogeneous low Heterogeneous Reduce NAA, FDG uptake greater


astrocytoma (WHO density signal intensities— reduced creatinine than white matter
grade III) Intense and predominantly T1 Increasing choline,
heterogeneous isointense, T2 lipids and lactate
enhancement hyperintense, ring- Increased rCBV
like enhancement,
mass effect

Oligodendroglioma Hypodense Hypointense T1 Increased regional FDG uptake similar


(WHO grade II) Calcification (central, Hyperintense T2/ TBV (?only if 1p19q to normal white
peripheral or ribbon FLAIR loss) matter
like) Minimal/no edema Elevated rCBV 11C-Methionine
No enhancement in None or dot-like lacy compared to studies can be used
50%, rest variable enhancement anaplastic to differentiate ODs
Cysts uncommon oligodendroglioma from anaplastic
(due to chicken wire oligodendrogliomas
capillaries)
Increased 2-
hydroxyglutarate in
LGG if mutant IDH1/2

Anaplastic Similar to grade II Similar to grade II More diffusion FDG uptake similar
oligodendroglioma oligodendroglioma oligodendroglioma restriction (low ADC) to normal grey
(WHO grade III) than grade II oligo matter
Lower rCBV 11C-Methionine
compared to grade II studies can be used
oligodendroglioma to differentiate ODs
from anaplastic
oligodendrogliomas

Continued on following page

Neurosurgery Books Full


www.ketabpezeshki.com 66485438-66485457
334 PART III CRANIAL NEUROSURGERY

Differential Appearances of Glioma Subtypes on Multi-modal Imaging (Continued)


Glioma CT MRI MRS/Perfusion PET
Primary or secondary Iso to hyperdense Within white matter, GRE/SWI may show FDG uptake similar
glioblastoma (cellularity)  T1 hypo-isointense blood products inside or higher than
multiforme (WHO hypodense (necrotic) with heterogeneous necrotic pockets. normal grey matter
grade IV) center. Marked center (bleed, Heterogeneous DWI
edema and mass necrosis). T2/FLAIR pattern—restricted in
effect. Intense, hyperintense, solid component,
irregular, edema, flow voids. facilitated in
heterogeneous Enhancement edematous and
enhancement of present but cystic. Low
margins heterogeneous, ring- ADC ¼ GBM, high
like but nodules. ADC ¼ low grade.
Elevated rCBV. NAA
and myoinositol
decreased, increased
choline/lipid/lactate

FURTHER READING can be an alternative imaging modality to rule


Chung C, Metser U, Ménard C. Advances in Magnetic Res- out recurrence even when 18F-FDG PET-CT
onance Imaging and Positron Emission Tomography Imaging is negative. In general, practical and experimen-
for Grading and Molecular Characterization of Glioma. tal roles of PET imaging in glioma management
Semin Radiat Oncol. 2015 Jul;25(3):164-171. doi: 10.1016/j. include: (1) Grading tumors and estimating
semradonc.2015.02.002. Epub 2015. Review. PubMed
prognosis; (2) Localizing the optimum biopsy
PMID:26050586.
site, (3) Defining target volumes for radiother-
Law M, Yang S, Wang H, et al. Glioma grading: sensitivity, apy (RT), (4) Assessing response to therapy,
specificity, and predictive values of perfusion MR imaging and and (5) Detecting tumor recurrence and distin-
proton MR spectroscopic imaging compared with conventional guishing it from radionecrosis.
MR imaging. AJNR Am J Neuroradiol. 24(10):1989-1998.
FURTHER READING
16. b—18F-FDG Chung C, Metser U, Ménard C. Advances in Magnetic
Resonance Imaging and Positron Emission Tomography
F-FDG PET was the first tracer used for assess- Imaging for Grading and Molecular Characterization of
ment of brain tumors; however, it has a low Glioma. Semin Radiat Oncol. 2015;25(3):164-171. doi:
tumor-to-background ratio in brain, limiting its 10.1016/j.semradonc.2015.02.002. Epub 2015. Review.
utility. 18F-FDG uptake correlates with tumor PubMed PMID:26050586.
grade, with high-grade gliomas (grades III and 17. e—Maximal safe resection
IV) showing higher uptake than low-grade glio-
mas. Therefore, in spite of its limitations, Diffused astrocytomas are WHO grade II (low-
18
F-FDG PET-CT is used for imaging of grade) gliomas. Infiltrating low-grade tumors
high-grade glioma. Amino acid PET radio- which occur typically in the cerebral hemispheres
tracers including 18F-FDOPA display superior of young adults, involving cortex and white mat-
contrast to 18F-FDG because of low uptake of ter with less well-defined borders than pilocytic
amino acids in normal brain tissue. They have astrocytomas. They frequently show IDH1 and
particularly special value in the detection of IDH2 mutations, which have a favorable impact
low-grade gliomas. However, 18F-FDOPA on overall survival. WHO grade II astrocytomas
tumor uptake cannot provide reasonable predic- appear iso- or hypodense on CT and show areas
tions about tumor grade and proliferation in of calcification in up to 20%. MRI is better in
recurrent tumors that have undergone treat- defining the extent of the low-grade gliomas.
ments. Also, their difficult synthesis or need They are hyperintense on T2 images and FLAIR
for an on-site cyclotron limits their widespread images and hypo/isointense on T1 images and
use. The present case shows the utility of 18F- show no contrast enhancement as opposed to
FDOPA PET-CT in detection of a recurrent pilocytic (WHO grade I) and anaplastic (WHO
high-grade AA that was missed by 18F-FDG grade III) astrocytomas. All diffuse astrocytomas
PET-CT. It highlights that 18F-FDG PET- (as well as other low-grade gliomas such as grade
CT can be falsely negative, even in high-grade II oligodendrogliomas and oligoastrocytoma
recurrent gliomas and, therefore, in cases with have) will inevitably transform into a higher
strong clinical suspicion 18F-FDOPA PET-CT

Neurosurgery Books Full


www.ketabpezeshki.com 66485438-66485457
23 CRANIAL ONCOLOGY 335

high-grade glioma usually within 3-10 years. As areas of cortex and corresponding white matter,
such, from a surgical management perspective their slow growth and tumor-induced plasticity
they are considered collectively. Current stan- means that seizures rather than functional deficits
dard of care for low-grade gliomas is maximal safe are by far the most common presenting features.
anatomical resection based on tumor borders as As with astrocytomas, standard of care is maximal
seen on FLAIR MRI, with or without early radio- safe resection to the FLAIR border, often requir-
therapy to the tumor bed (especially if residual) or ing awake intraoperative functional mapping for
late radiotherapy at the time of tumor progres- eloquent area tumors due to distortion of cortical
sion on imaging. More recently, some have functional maps by the tumor.
argued that since tumor cells are likely to have
spread significantly further along white matter Image with permission from Kaye, AH, Brain Tumors an
Encyclopedic Approach, 3rd ed., 2012, Elsevier.
than visualized on FLAIR MRI the limit of tumor
resection should instead be based on functional
limits identified by continuous intraoperative 19. b—Craniotomy under GA with functional
functional mapping/testing during surgery. Adju- mapping is the standard of care for low-grade
vant treatment with stereotactic radiosurgery is gliomas in eloquent cortex
under investigation.
The primary goal in the initial treatment of LGG
Image with permission from Senft C, Franz K, Ulrich CT, is maximum safe resection in the majority of
et al. Low field intraoperative MRI-guided surgery of gli- patients. The goal of achieving more extensive
omas: a single center experience. Clin Neurol Neuro- resection (over biopsy alone) is often favored
surg. 2010;112(3):237-243. because, in retrospective analyses, it is associated
with prolonged survival, greater seizure control
18. e—Tumor margins are usually seen best on and reduced risk of transformation to a higher
T1 + gad MRI sequences grade. However, surgical treatment of suspected
LGGs poses a special challenge for the neurosur-
Oligodendrogliomas account for 10-15% of all geon for the following reasons: (1) Gross total
gliomas and occur predominantly in adults. They resection is difficult as their diffusely infiltrative
are diffusely infiltrating neoplasms, which are growth pattern means that intraoperative identi-
found almost exclusively in the cerebral hemi- fication of the exact tumor border in an LGG is
spheres, most commonly in the frontal or frequently not possible with certainty, hence
temporo-frontal region, and typically involving image guidance based on tumor limits on FLAIR
subcortical white matter and cortex. The WHO MRI is key. (2) Histopathological undergrading is
classification distinguishes between WHO grade common with biopsy as LGG generally exhibits
II (well-differentiated low-grade) and WHO focal areas of malignant transformation (anaplas-
grade III (anaplastic high-grade) oligodendro- tic foci) therefore the surgical goal is to perform
gliomas. The former are slowly growing tumors precise tissue sampling from a potential anaplastic
with rounded homogeneous nuclei; the latter focus to avoid this and reduce subsequent treat-
have increased tumor cell density, mitotic activ- ment failure. (3) “Eloquent” tumor localization
ity, microvascular proliferation and necrosis. and infiltrative growth pattern means that awake
Both low- and high-grade oligodendral tumors surgery and/or functional mapping are generally
express proangiogenic mitogens and may contain employed to avoid new postoperative neurologi-
regions of increased vascular density with finely cal deficits. (4) precise localization of the tumor
branching capillaries that have a “chicken wire” and its relation to cortical surface anatomy and
appearance. This contributes to their appearance vasculature in the preoperative planning phase
on contrast-enhanced MRI and MR perfusion. as well as during the operative approach is
Up to 90% of oligodendrogliomas contain visible required to avoid morbidity (e.g. insular gliomas).
calcification on CT, which can be central, periph-
eral or ribbon-like. On MRI, intratumoral calci- 20. d—Brodmann areas represent a connection-
fication appears typically T2 hypo- and T1 ist model of brain function
hyperintense and causes marked signal loss on
T2* or SWI images. Contrast enhancement is The locationist view that each region of the
variable and often heterogeneous. Unlike in brain corresponds to a given function (e.g.
astrocytomas, contrast enhancement is not a reli- Brodmann areas) producing a topographical
able indicator of tumor grade in oligodendroglio- map has been used for over a century, and
mas: it occurs in about 20% of WHO grade II implies that brain injury in an “eloquent” area
tumors and in over 70% of WHO grade III oligo- will result in a massive and permanent neurolog-
dendrogliomas. Low-grade oligodendrogliomas ical deficit. However, it is unable to explain the
may also have an elevated rCBV on PWI. Despite many observations of recovery after brain dam-
commonly being located in functional “eloquent” age, even in eloquent regions as well as fMRI

Neurosurgery Books Full


www.ketabpezeshki.com 66485438-66485457
336 PART III CRANIAL NEUROSURGERY

and intraoperative mapping evidence of pre- (within a few weeks of surgery) or delayed radio-
and postoperative cortical topographic map therapy (at time of clinical or imaging progres-
reorganization in diffuse low-grade glioma sion). Controversy exists on its optimal timing,
patients with “eloquent” lesions. More recently, particularly due to the long term neurocognitive
the idea of a brain connectome where the CNS side effects in LGG patients who are mostly
is organized into parallel networks, which are young adults. Early adverse effects of radiation
dynamic, interactive and able to compensate include headache, dizziness, ear inflammation,
for each other (at least to a certain extent) has nausea, vomiting, seizure, altered level of con-
gained favor. This is underpinned by the hodo- sciousness, alopecia, dermatitis, urinary inconti-
topic principle where functions of the brain are nence, and personality change. Late clinical
supported by extensive circuits comprising both consequences of brain irradiation include leu-
the cortical nodes (topos) and connections koencephalopathy, neurocognitive decline,
between them created by associating bundles reduced quality of life, and tissue necrosis that
of white matter (hodos). Neurological function may mimic tumor progression. The toxic effects
arises from the synchronization between differ- of radiotherapy must be carefully weighed against
ent nodes, working in phase during a given task, the benefits for tumor control, including an
and the same node may take part in several func- improvement of seizures. SRS can produce
tions depending on the other cortical areas with long-term control with an acceptable toxicity
which it is temporarily connected at any one profile, and is generally reserved for inoperable
time. Functional maps may be reorganized tumors in close proximity to critical structures.
within remote networks over time, making neu- Similarly, chemotherapy has potential either as
roplasticity mechanisms possible. Modest redis- a concurrent treatment or substitute for radio-
tribution of neurosynaptic networks occurs in therapy and can also improve seizure control.
acute injuries (e.g. stroke) explaining the limited Studies have focused primarily on a three-drug reg-
recovery, whereas massive redistribution of imen of procarbazine, lomustine, and vincristine
function can occur in chronic slowly progressive (PCV) or single agent temozolomide (temozolo-
injuries (e.g. DLGG) so that an “eloquent” loca- mide response may be predicted by IDH1/2 muta-
tion generally does not result in functional def- tion). Ongoing randomized controlled trials are
icit. Cortical map reorganization mechanisms evaluating whether temozolomide can substitute
involve recruitment of areas around the lesion for radiotherapy, or whether concurrent temozolo-
and/or within the hemisphere remote to the gli- mide and radiotherapy is superior to radiotherapy
oma and/or contralateral to the lesion. However, alone for postoperative tumor control.
the plasticity index is high for cortical reorganiza-
tion but very limited for subcortical connectivity 22. b—Functional MGMT increases the effec-
hence it is crucial to preserve functionally impor- tiveness of cancer chemotherapy
tant subcortical white matter (determined by
intraoperative white matter functional mapping), The O(6)-Methylguanine-DNA Methyl Trans-
allowing post injury/operative cortical map reorga- ferase (MGMT) is a DNA repair enzyme that
nization and avoidance of permanent neurological reverts the naturally occurring mutagenic O6-
deficit. In this way, supratotal resection of diffuse methylguanine back to guanine. This prevents
low-grade gliomas (i.e. to the functional limit errors during DNA replication. In the context of
defined by intraoperative white matter mapping, chemotherapy with alkylating agents (e.g. temozo-
rather than to the tumor limit defined by FLAIR lomide) it removes a cytotoxic lesion, thus coun-
MRI) has been supported by some. While this teracting the chemotherapeutic effects of the
approach may tackle microscopic tumor spread, drug. Aberrant, cancer related methylation of
all cases must be done awake with functional the MGMT promoter region leads to its silencing,
mapping, requires a large craniotomy, and is seen a reduction of the MGMT enzyme expression and
as too aggressive by many at present. subsequently to less repair activity of DNA dam-
age, including that induced by temozolomide
FURTHER READING (TMZ). MGMT promoter methylation in GBM
Duffau H. Nat Rev Neurol 2015;11:255-265. is a prognostic and predictive biomarker indicating
response to chemoradiation. It has no diagnostic
21. d—PCV value. This was demonstrated in the EORTC
NCIC registration trial for TMZ in newly diag-
LGG management involves surgery, radiother- nosed GBM where patients with MGMT pro-
apy, chemotherapy, or a combination of these moter methylated tumors derived most benefit
modalities. Surgery is first-line therapy whose when treated with TMZ. Patients with tumors
chief role is to provide tissue to confirm the diag- with methylated MGMT promoter had a survival
nosis. The next most common step in manage- benefit when treated with TMZ and radiotherapy,
ment is radiotherapy: either early radiotherapy compared with those who received radiotherapy
Neurosurgery Books Full
www.ketabpezeshki.com 66485438-66485457
23 CRANIAL ONCOLOGY 337

only, whereas absence of MGMT promoter meth- in GBM and astrocytomas but co-occur in most
ylation resulted in a smaller and statistically insig- oligodendrogliomas. In a study of 400 gliomas
nificant difference in survival between the patients with TERT promoter mutations alone
treatment groups. Further studies showed that (i.e. no IDH mutation) had the poorest overall sur-
patients with MGMT promoter-unmethylated vival (median 11.3 months), patients with tumors
tumors had no survival benefit from chemother- without TERT or IDH1/2 mutations had a
apy, regardless of whether given at diagnosis slightly better survival (median 16.6 months),
together with RT or as a salvage treatment. Pro- whereas patients with IDH-only mutant GBM
spective randomized trials (NOA-08), the Nordic had the best survival (median 42.3 months).
trial and RTOG 0525 concluded that MGMT Although an earlier study with 358 patients found
promoter methylation is a useful predictive bio- no significant difference in overall survival
marker to stratify elderly (>70 years of age; the between TERT mutant and TERT wild-type
Stupp protocol was studied in GBM patients (IDH wt) GBM, the role of TERT mutations
under 70) GBM patients for RT versus alkylating may in the future provide a tool to identify non-
agent chemotherapy and should be tested for. IDH1/2 mutant GBMs and suggests that com-
bined IDH1/2 and TERT promoter genotyping
FURTHER READING will be useful for patient management.
Brandner S, et al. Invited review: diagnostic, prognostic and
predictive relevance of molecular markers in gliomas. Neuro- FURTHER READING
pathology and Applied Neurobiology 2015;41:694-720. Brandner S, et al. Invited review: diagnostic, prognostic and
predictive relevance of molecular markers in gliomas. Neuro-
pathology and applied neurobiology 2015;41:694-720.
23. b—TERT normally prevents telomerase
repair ensuring that cells become replica- 24. f—IDH1/2 mutation is associated with an
tively senescent improved prognosis in grade II astrocytomas
Telomere (repetitive nucleotide sequences at the Pathogenic mutations in the isocitrate dehydroge-
ends of chromosomes) length shortens with each nase genes 1 (IDH1; chromosome 2q) or 2 (IDH2;
cell division, normally leading to replicative senes- chromosome 15q) were discovered in next-
cence and thus a limit to the number of times a cell generation sequencing studies of 22 GBM, includ-
can divide. Cancer cells bypass this limit in various ing in secondary GBM. Likely to be a tumor initi-
ways, including an increased ability to maintain ating or driver mutation in astrocytomas and
telomere length. The enzyme telomerase reverse oligodendrogliomas, even in the presence of a pre-
transcriptase (TERT) plays a critical role in existing mutation of the tumor suppressor p53
extending the telomeres in normal cells and muta- (TP53) gene. The most common mutation
tions in the TERT promoter, resulting in overex- ( 90%) in glial brain tumors causes a substitution
pression of TERT is a feature of most human of the amino acid Arginine to Histidine at codon
cancers including gliomas. It has been suggested 132 of the IDH1 gene (IDH1 R132H); alterna-
that tumors derived from cell populations with tively, a mutually exclusive mutation in codon 172
low self-renewal capacity generally rely on alter- of the mitochondrial IDH2 gene can occur. These
ations that restore/gain telomerase activity, while homologous enzymes decarboxylate isocitrate to α-
epigenetic mechanisms maintain/prevent loss of ketoglutarate (αKG) and this “neomorphic” muta-
telomerase activity in tumor types derived from tion renders the IDH enzyme to reduce αKG into
self-renewing stem cells. In contrast, ATRX or 2-hydroxyglutarate (2-HG) in an NADPH depen-
DAXX mutations have been shown to underlie a dent manner. Accumulation of 2-HG to high levels
telomere maintenance mechanism not involving in glioma tissues may cause epigenetic alterations in
telomerase (“alternative lengthening of telo- both, DNA and histones, altering gene expression
meres;” ALT). TERT is essential in maintaining and promoting oncogenic transformation. Reorga-
telomere length and its activity is pathologically nization of the methylome due to mutant IDH is
increased in a number of human cancers, includ- the molecular basis of the CpG island methylator
ing GBM. Analysis of TERT promoter mutations phenotype (CIMP) in gliomas, leading to global
in 1515 CNS tumors showed 327 mutations, pre- dysregulation of gene expression.
dominantly in adult patients, with a strong associ-
ation with older age. Mutations were seen in IDH1 mutations occur early, with a high fre-
gliosarcomas (81%), oligodendrogliomas (78%), quency, in WHO grade II and III astrocytic and
oligoastrocytomas (58%) and primary GBMs oligodendroglial tumors and in secondary GBMs,
(54%). TERT promoter mutations were also which develop from astrocytomas. IDH mutations
strongly associated with 1p/19q codeletion and in gliomas are early events in their pathogenesis,
inversely associated with loss of ATRX expression and are associated with several clinically relevant
and IDH1/IDH2 mutations. In general, TERT parameters including patient age, histopatho-
and IDH mutations are largely mutually exclusive logical diagnosis, combined 1p/19q deletion
Neurosurgery Books Full
www.ketabpezeshki.com 66485438-66485457
338 PART III CRANIAL NEUROSURGERY

(co-segregate in oligodendrogliomas), TP53


mutation, MGMT promoter hypermethylation ANSWERS 25–40
and patient survival. Diagnostically it serves 2 roles Additional answers 25–40 available on
when used with ATRX and 1p19q status: ExpertConsult.com
(1) differentiate grade II/III oligodendrogliomas
from other lesions with potentially similar histo-
logical appearance or “oligodendroglial-like
differentiation” (e.g. primary GBM, clear cell
ependymomas, neurocytomas or pilocytic astrocy-
tomas), and (2) high-grade astrocytomas or oligo- EMI ANSWERS
dendrogliomas from “primary GBMs with
oligodendroglial differentiation.” The association 41. 1—k, Primary CNS lymphoma; 2—e,
between IDH1/2 mutation and a favorable prog- Germinoma; 3—h, Non-germinomatous
nosis is better established in high-grade gliomas. germ cell tumor (most likely choriocarci-
The majority of studies reporting mutant IDH1 noma given markedly raised HCG)
as a favorable factor in WHO grade II tumors
often include oligodendroglial tumors (which will 42. 1—f, Hemangioblastoma; 2—i, Pilocytic
also have 1p19q loss hence better chemotherapy astrocytoma; 3—c, Dysplastic cerebellar
response) and studies comprising low-grade astro- gangliocytoma (Lhermitte-Duclos disease)
cytomas only showed no prognostic value of
mutant IDH1/2. The NOA4 trial identified that 43. 1—f, Histone H3; 2—g, IDH1/2 mutation;
MGMT promoter methylation is prognostic for 3—a, 1p19q co-deletion; 4—e, EGFRvIII;
patients with IDH1/2-mutant WHO grade III 5—h, MGMT methylation
gliomas, but in patients with IDH-wild-type BRAF gene mutations are diagnostically useful in
tumors it was predictive for benefit from alkylating differentiating low-grade glial and glioneuronal
chemotherapy. (pilocytic astrocytoma, PXA, ganglioglioma,
FURTHER READING SEGA) from diffuse astrocytomas, but not prog-
Brandner S, et al. Invited review: diagnostic, prognostic and nostically relevant. The table below reflects com-
predictive relevance of molecular markers in gliomas. Neuro- monest patterns of molecular status according to
pathology and applied neurobiology 2015;41:694-720. histological grades.

Molecular Classification of Gliomas


Other
IDH1/2 MGMT Promoter Molecular
WHO Grade 1p19q Status Status ATRX Status Methylation Markers
Astrocytoma Preserved Mutant IDH in ARTX loss in IDH
(grade II) 19q loss only majority mutants
exceptionally

Oligodendroglioma Codeleted in vast Mutant IDH in ARTX loss in TERT promoter


(grade II) majority majority 25%, methylation
but rare if 1p19q (co-occurs with
loss IDH
mutation unlike in
GBM)

Anaplastic Preserved Mutant IDH in ATRX loss in Associated with H3 Histone


astrocytoma 19q loss only majority, IDH1/2 mutants ¼ better prognosis in mutation predicts
(Grade III) in minority. confers better favorable IDH mutants worse prognosis
prognosis prognosis Predictive for (similar
than benefit from to GBM)
wild type temozolomide in
IDH wild type

Anaplastic Codeleted in vast Mutant IDH in ARTX loss in Associated with TERT promoter
oligodendroglioma majority, better majority, 7%, but rare if better prognosis in methylation
(Grade III) prognosis and 1p loss confers better 1p19q loss IDH mutants
confers response prognosis Predictive for
to TMZ/PCV than benefit from
wild type IDH temozolomide in
IDH wild type

Continued

Neurosurgery Books Full


www.ketabpezeshki.com 66485438-66485457
23 CRANIAL ONCOLOGY 339

Other
IDH1/2 MGMT Promoter Molecular
WHO Grade 1p19q Status Status ATRX Status Methylation Markers
Primary GBM Preserved Wild type IDH ARTX loss rare in Predictive for 7p gain; 10q loss;
19q loss only in all adult primary GBM benefit from TERT
in a small minority primary GBM ATRX loss seen temozolomide promoter
If IDH mutant in IDH mutant methylation;
present likely secondary GBM EGFR
represents and pediatric amplification;
secondary GBM Histone H3
GBM mutation in
younger patients

FURTHER READING Louis DN, Perry A, Reifenberger G, von Deimling A, et al.


Brandner S, et al. Invited review: diagnostic, prognostic and The 2016 World Health Organization Classification of
predictive relevance of molecular markers in gliomas. Neuro- Tumors of the Central Nervous System: a summary. Acta
pathology and Applied Neurobiology 2015;41:694-720. Neuropathol. 2016 Jun; 131(6):803-20.

Neurosurgery Books Full


www.ketabpezeshki.com 66485438-66485457
CHAPTER 24

SKULL BASE AND


PITUITARY SURGERY
SINGLE BEST ANSWER (SBA) QUESTIONS
PITUITARY
1. A 45-year-old man presents with sudden
onsent headache, nausea, vomiting and visual
disturbance. On examination he is apyrexial,
hypotensive, and tachycardic but does not
have neck stiffness or photophobia. Visual
fields appear to be constricted on confronta-
tion. CT head is abnormal therefore MRI is
done. Which one of the following is appro-
priate acute management?

a. Pituitary profile
b. Formal visual field assessment
c. Image guided biopsy
d. Lumbar puncture
e. ICP monitoring

3. A 57-year-old undergoes elective transsphe-


noidal surgery for a non-functioning macro-
adenoma compressing the optic chiasm.
Recovery was uneventful and routine bloods
on postoperative day 1 were normal, and
endocrine profile showed FT4 12 pmol/L
(11.5-22), TSH 0.9 mU/L (0.35-5.5) and
9 am cortisol 154 nmol/L. Which one of
a. Dexamethasone the following statements regarding further
b. Formal visual field tests management is LEAST accurate?
c. Pituitary profile and start empirical intra- a. If thyroid function tests are normal on day
venous hydrocortisone 3 or 4 a further assessment should take
d. Lumbar puncture place at 4-8 weeks
e. Transsphenoidal surgery b. Evening steroid dose should not be omit-
ted if checking 9 am cortisol
2. A 64-year-old male presented with severe c. If day 2 postoperative 9 am cortisol
headache and visual disturbance. Plain CT >550 nmol/L does not require steroid
imaging revealed a sellar mass but no other replacement
acute abnormality. Contrast imaging was per- d. If day 2 postoperative 9 am cortisol 400-
formed (below). Which one of the following 550 nmol/L requires hydrocortisone only
is the most important next management step? during severe illness

340
Neurosurgery Books Full
www.ketabpezeshki.com 66485438-66485457
24 SKULL BASE AND PITUITARY SURGERY 341

e. If day 2 postoperative 9 am cortisol a. Serum and CSF HCG and AFP


<400 nmol/L does requires regular oral b. Ultrasound pelvis/ovaries
hydrocortisone c. Insulin tolerance test
d. Short synacthen test
4. A 65-year-old man undergoes transsphenoi- e. Visual field tests
dal surgery for a non-functioning macroade-
noma. Postoperatively on day 1 the nurses 6. A 33-year-old female who presents with a
notice he is passing large volumes of dilute several week history of headache, extreme
urine via his catheter. Which one of the fatigue and malaise. She gave birth to her first
following criteria is LEAST suggestive of child 2 months ago via vaginal delivery.
diabetes insipidus? Examination was unremarkable. Routine
a. Urine specific gravity <1.003 or osmolal- bloods were normal, and endocrine profile
ity <200 mOsmol/kg showed: Prolactin 801 mU/L (100-550),
b. Urine output >250 ml/h for 2 consecutive FT4 10 pmol/L (11.5-22), TSH 0.1 mU/L
hours (0.35-5.5). Short synacthen test: 0 h cortisol
c. Serum sodium normal or raised 55 nmol/L, 30 min cortisol 155 nmol/L.
d. Primary adrenal insufficiency MRI is shown. Which one of the following
e. Inability to concentrate urine to >300 is most likely?
mOsmol/L in the presence of clinical
dehydration

5. A 17-year-old female presents with an


8 month history of secondary amenorrhea.
More recently she has noticed that she is more
thirsty and is passing large volumes of urine.
Her examination is otherwise unremarkable.
Routine blood tests are normal and endocrine
profile shows: FSH 5.5 U/L (follicular 0.5-5,
mid-cycle 8-33, luteal 2-8), LH 2.8 U/L (fol-
licular 3-12, mid-cycle 20-80, luteal 3-16),
estradiol 32 pmol/L (follicular 17-260, luteal
180-1100), prolactin 990 mU/L (60-620),
9 am cortisol 400 nmol/L, fasting blood glu-
cose 5.5 mmol/L, serum calcium 2.35 mmol/
L (2.2-2.6). Water deprivation test: serum
osmolality 300 mOsmol/kg, urine 200 mOs- a. Lymphocytic hypophysitis
mol/kg at 6 h, post-DDAVP urine osmolality b. Sheehan’s syndrome
800 mOsmol/kg. MRI head is shown. Which c. Non-functioning adenoma
one of the following would you perform next? d. Langerhans cell histiocytosis
e. Sarcoidosis

7. A 48-year-old male presents with low mood,


malaise and reduced exercise tolerance. He
has a past history of IHD and a non-functioning
pituitary adenoma resected 5 years ago result-
ing in partial anterior hypopituitarism. He is
normally on thyroxine and hydrocortisone
replacement therapy. On examination his
BMI is 39 kg/m2 and shows central adiposity.
His endocrine tests show 9 am cortisol
410 nmol/L, IGF-1 8 nmol/L (16-118), FT4
16 pmol/L (11.5-22) and TSH 0.03 mU/L

Neurosurgery Books Full


www.ketabpezeshki.com 66485438-66485457
342 PART III CRANIAL NEUROSURGERY

(0.35-5.5). Which one of the following is most


appropriate to confirm the diagnosis?
a. Growth hormone levels
b. GHRH-arginine stimulation test
c. IGF-binding protein 3 measurement
d. Insulin tolerance test
e. Water deprivation test

8. A 56-year-old presents with a 2-month his-


tory of headache and visual disturbance.
Confrontation testing revealed a bitemporal
hemianopia. Routine bloods were normal
and endocrine profile is: FT4 pmol/L 8.5
(11.5-22), TSH 0.5 mU/L (0.35-5.5), FSH
1.0 U/L (1.4-18.1), LH 2.5 U/L (3-8), pro-
lactin 900 mU/L (45-375), testosterone
3.5 nmol/L (8.4-28.7), 9 am cortisol
a. Stereotactic radiosurgery
405 nmol/L. MRI is shown. Which one of
b. Transsphenoidal debulking followed by
the following is most likely?
stereotactic radiosurgery
c. Cabergoline
d. Somatostatin
e. Petrous sinus sampling

10. Which one of the following statements


regarding diagnosis and management of pro-
lactinomas is LEAST accurate?
a. Prolactinomas are managed medically
with dopamine receptor agonists
b. Hook effect describes spuriously high
prolactin levels due to macroprolactin
complexes
c. Compression of the pituitary stalk can
cause a mildly raised prolactin
d. Prolactin levels in a prolactinoma are usu-
ally >2000 mU/L
e. Hypothyroidism may result in hyperpro-
lactinemia
a. Non-functioning adenoma
b. Thyrotropinoma 11. Which one of the following statements
c. GH-secreting pituitary adenoma regarding management of incidentally found
d. Prolactinoma pituitary adenomas (except prolactinomas) is
e. Cushing’s disease most accurate?
a. Anterior pituitary hormone profile should
9. A 50-year-old male presents with a 12 month only be performed if the patient is
history of reduced libido and lethargy. On symptomatic
examination there were mild left sided 3rd b. MRI surveillance for lesions <10 mm and
and 6th nerve palsies. Routine bloods were >10 mm is the same
normal and endocrine profile showed: FT4 c. Indications for surgery include secreting
8 pmol/L (11.5-22), TSH 0.4 mU/L (0.35- tumours
5.5), FSH 2.2 U/L (1.4-18.1), LH 3.5 U/L d. Macroincidentalomas should be reimaged
(3-8), testosterone 6.8 nmol/L (8.4-28.7), at 12 months
IGF-1 35 nmol/L (16-118), prolactin e. Formal visual field testing should be per-
880 mU/L (45-375). MRI is shown. Which formed as a baseline
one of the following is most appropriate next?

Neurosurgery Books Full


www.ketabpezeshki.com 66485438-66485457
24 SKULL BASE AND PITUITARY SURGERY 343

12. Which one of the following statements 16. A 43-year-old female presents with coarse
regarding medical management of secreting facial appearance, macroglossia and large
pituitary adenomas is LEAST accurate? hands and feet. She has also been experienc-
a. Octreotide used in patients with ing worse glycemic control on a background
acromegaly of type II diabetes mellitus. Routine bloods
b. Cabergoline is a dopamine receptor are normal and endocrine profile shows:
antagonist IGF-1 102 nmol/L (16-118), prolactin
c. Bromocriptine is used in the treatment of 610 mU/L (45-375), FT4 12.5 pmol/L
prolactinoma (11.5-22), and TSH 1.5 mU/L (0.35-4.5).
d. Pegvisomant is a GH receptor blocker Which one of the following would you per-
and useful if somatostatin fails in form to confirm your biochemical diagnosis?
acromegaly a. Oral glucose tolerance test
e. Mitotane used for Cushing’s disease b. Insulin tolerance test
c. Short synacthen test
13. A 36-year-old patient with Cushing’s syn- d. Dexamethasone suppression test
drome but normal ACTH levels is referred. e. Domperidone test
There is no visual compromise. Pituitary
MRI shows a 3 mm hypodense area in the lat- 17. Endoscopic view from within sphenoid sinus.
eral aspect of the pituitary gland. Which one Which one of the following labels appropri-
of the following is the next appropriate ately identifies the opticocarotid recess?
management?
a. Laparoscopic adrenalectomy
b. Transsphenoidal surgery
c. Inferior petrosal sinus sampling
d. High-dose dexamethasone test D A
e. Start octreotide
B
E C
14. Which one of the following statements G
regarding Cushing’s disease is most accurate?
a. It is often due to ACTH-secreting pitui-
tary adenoma
b. Primary management is surgical resection F
of the tumor
c. High-dose dexamethasone suppression
test is able to lateralize the side of
ACTH-secreting microadenoma within
the pituitary gland
d. It may be caused by ectopic ACTH pro-
ducing tumors 18. A patient presents 7 days after transsphenoi-
e. Can cause amenorrhea in females and dal resection of pituitary adenoma with
infertility in males headache, neck stiffness, fever and clear
fluid discharge from his nose. Which one
15. Which one of the following would be most of the following is the most appropriate
appropriate treatment following failure of management?
transsphenoidal surgery to treat Cushing’s a. Endoscopic repair
disease? b. External ventricular drain
a. Surveillance imaging c. Bed rest and lumbar puncture
b. Repeat transsphenoidal surgery d. Lumbar drain
c. Cabergoline e. Empirical antibiotics and CT head
d. Octreotide
e. Bilateral adrenalectomy
QUESTIONS 19–37
Additional questions 19–37 available on
ExpertConsult.com

Neurosurgery Books Full


www.ketabpezeshki.com 66485438-66485457
344 PART III CRANIAL NEUROSURGERY

EXTENDED MATCHING ITEM (EMI) e. Hyperprolactinemia


f. Hyperthyroidism
QUESTIONS g. Hypothyroidism
h. Nelson’s syndrome
38. Sellar and parasellar lesions: i. Panhypopituitarism
a. ACTH-secreting pituitary adenoma j.Pituitary apoplexy
b. Arachnoid cyst k. Primary adrenal insufficiency (Addison’s
c. Craniopharyngioma disease)
d. Germinoma l. Secondary adrenal insufficiency
e. GH-secreting pituitary adenoma m. Syndrome of inappropriate ADH
f. Gonadotropin-secreting adenoma secretion
g. Hypothalamic hamartoma
h. Langerhans cell histiocytosis For each of the following descriptions, select the
i. Lymphocytic hypophysitis most appropriate answers from the list above.
j. Non-functioning pituitary adenoma Each answer may be used once, more than once
k. Optic pathway glioma or not at all.
l. Prolactin-secreting pituitary adenoma 1. A 37-year-old male presents with abdomi-
m. Thyrotropinoma nal pain, fatigue, hypotension and skin
hyperpigmentation. Neither short or long
For each of the following descriptions, select the synacthen tests show cortisol response.
most appropriate answers from the list above.
2. A 57-year-old male undergoes transsphe-
Each answer may be used once, more than once
noidal surgery for non-functioning ade-
or not at all.
noma. On day 2 postoperatively his serum
1. A 24-year-old female presents with a 3 year
sodium rises to 149 mmol/L and urine out-
history of menstrual irregularities. Prior to
put is 300 ml/h for greater than 2 h despite
this she had regular menstrual cycles.
previously normal fluid balance. Serum
Examination was unremarkable. Routine
osmolality is 295 mOsmol/kg and urine
bloods were normal, and endocrine profile
osmolality is 105 mOsmol/kg.
showed: FSH 45 U/L (follicular 0.5-5, mid-
cycle 8-33, luteal 2-8), LH 2.5 U/L (follic- 3. A 44-year-old on pituitary hormone
ular 3-12, mid-cycle 20-80, luteal 3-15), replacement with bitemporal hemianopia
estradiol 1332 pmol/L (follicular 17-260, and right ocular paresis. She was diagnosed
luteal 180-1100), prolactin 604 mU/L with a brain tumor but had an operation on
(45-375). Ultrasound of pelvis shows bilat- her abdomen to stop it causing symptoms.
eral enlarged and cystic ovaries. MRI shows
40. Skull base foramina:
a pituitary tumor without optic chiasm
compression.
2. A 49-year-old female presents with a
6 month history of palpitations, weight loss,
increased sweating and shortness of breath. A I
Her examination was unremarkable except J
for sinus tachycardia. Routine bloods were B K
normal and TFTs showed: FT4 27 pmol/L C L
(9-19), FT3 pmol/L (8 (2.6-5.7), TSH 7 D M
(0.35-5.5). Further tests show alpha- E
subunit to TSH ratio >1. F N
3. A 9-year-old girl presents with bitemporal G
O
hemianopia and cafe-au-lait spots. Autoan- H
tibody screen in normal. MRI shows a lesion
involving the chiasm and hypothalamus.

39. Pituitary dysfunction:


a. Acromegaly
b. Conn’s syndrome
c. Cushing’s disease
For each of the following descriptions, select the
d. Diabetes insipidus
most appropriate answers from the image above.

Neurosurgery Books Full


www.ketabpezeshki.com 66485438-66485457
24 SKULL BASE AND PITUITARY SURGERY 345

Each answer may be used once, more than once i. Transcondylar and far lateral approach
or not at all. j. Translabyrinthine
1. Contains maxillary nerve (CNV2) k. Transotic
2. Contains greater petrosal nerve (VII) which l. Transpetrous
joins with deep petrosal nerve to form For each of the following descriptions, select the
vidian nerve (of the pterygoid canal). most appropriate answers from the list above.
3. Contains facial nerve (VII), vestibuloco- Each answer may be used once, more than once
chlear (VIII) or not at all.
1. Presigmoid approach that uses a mastoidec-
tomy and skeletonization of the sigmoid
41. Skull base approaches:
sinus
a. Anterior clinoidectomy
b. Anterior petrosectomy 2. Includes drilling the posterior and superior
c. Combined transpetrosal approach external auditory canal and sacrificing mid-
d. Frontotemporal orbitozygomatic approach dle ear structures and inner ear (cochlea),
e. Middle cranial fossa approach and rerouting the facial nerve to provide
f. Retrolabyrinthine access to the anterior cerebellopontine
g. Retrosigmoid angle, petrous apex, and ventral brainstem.
h. Transcochlear

SBA ANSWERS
1. c—Pituitary profile and start empirical intra- the majority even before the apoplectic episode.
venous hydrocortisone Clinically, the most crucial deficit is that of adre-
nocorticotroph hormone (ACTH) in up to 70%
Pituitary apoplexy is characterized by sudden onset of the patients. Thyrotrophin and gonadotrophin
headache, nausea, vomiting, with or without acute deficiencies are observed in 50% and 75% of the
visual disturbance and cranial nerve palsy (2nd, patients, respectively. Hyponatremia has been
3rd, 4th, 6th) due to pituitary hemorrhage and/ reported in up to 40% of the patients because of
or infarction. Precipitating factors in those with either the syndrome of inappropriate antidiuretic
or without an underlying pituitary tumor include hormone secretion or hypocortisolism. Patients
hypertension, major surgery, dynamic testing of with pituitary apoplexy who have low serum pro-
pituitary function, anticoagulation/coagulopathy, lactin (PRL) levels at presentation have the highest
estrogen therapy, dopamine receptor agonist initi- intrasellar pressure and are the least likely to
ation/withdrawal, radiotherapy, pregnancy and recover from hypopituitarism after decompressive
head trauma. Assessment should focus on history surgery. Formal visual fields assessment, using
and examination findings consistent with pre- Humphrey visual field analyzer or Goldmann
existing pituitary dysfunction. Anterior pituitary perimeter must be undertaken when the patient
function tests (FT4, TSH, IGF-1, random corti- is clinically stable, preferably within 24 h of the
sol, prolactin, growth hormone, FSH/LH, and suspected diagnosis. MRI of the pituitary is
testosterone in men or estradiol in women) should required to confirm the diagnosis. Hemorrhage
be checked urgently but those with hemodynamic appears hyperintense on non-enhanced T1W
instability (i.e. Addisonian crisis) should be started images and, in the acute stage, hyperdense on
on empirical hydrocortisone. Indications for CT and may rarely contain a fluid level (as in this
empirical steroid therapy in patients with pituitary case) with evidence of optic chiasm being stretched
apoplexy are hemodynamic instability, altered across the top of the mass. Patients with pituitary
consciousness level, reduced visual acuity and apoplexy who are without any neuro-ophthalmic
severe visual field defects. Patients who do not ful- signs or mild and stable signs can be considered
fill the criteria for urgent empirical steroid therapy for conservative management with careful moni-
should be considered for treatment with steroids, if toring. In patients with reduced visual acuity or
their 0900 serum cortisol is less than 550 nmol/L; defective visual fields, formal assessment of visual
The majority of the patients (nearly 80%) will have fields and acuity should be performed every day
deficiency of one or more anterior pituitary hor- until a clear trend of improvement is observed.
mones at presentation. As most of the patients have Indications for urgent neurosurgical decompres-
underlying macroadenomas, partial hypopituita- sion include deteriorating level of consciousness,
rism would be expected to have been present in severely reduced visual acuity, severe/persistent/

Neurosurgery Books Full


www.ketabpezeshki.com 66485438-66485457
346 PART III CRANIAL NEUROSURGERY

worsening visual field defect. Ocular paresis and consider changing over to maintenance
because of involvement of III, IV or VI cranial dosage when stable. These patients will
nerves in cavernous sinus in the absence of visual need further assessment at 4-8 weeks with
field defects or reduced visual acuity is not in itself anterior hormone profile and short
an indication for immediate surgery. Resolution synacthen test to determine whether they
will typically occur within days or weeks with will need long-term steroids;
conservative management. Surgery should be • FT4 and TSH should be assessed on day 3
performed preferably within the first 7 days of onset or day 4 and thyroid hormone replacement
of symptoms by an experienced pituitary surgeon. should be considered if deficient.
• If FT4 and TSH normal further assessment
Image with permission from Adam A, et al. (Eds.),
Grainger & Allison's Diagnostic Radiology, 6th ed. Else- should take place at 4-8 weeks.
vier, Churchill Livingstone, 2014.
FURTHER READING
FURTHER READING Rajasekaran S, Vanderpump M, Baldeweg S, Drake W, Reddy
Rajasekaran S, Vanderpump M, Baldeweg S, Drake W, Reddy N, Lanyon M, Markey A, Plant G, Powell M, Sinha S, Wass J.
N, Lanyon M, Markey A, Plant G, Powell M, Sinha S, Wass J. UK guidelines for the management of pituitary apoplexy. Clin
UK guidelines for the management of pituitary apoplexy. Clin Endocrinol (Oxf) 2011;74(1):9-20.
Endocrinol (Oxf) 2011;74(1):9-20.
4. d—Primary adrenal insufficiency
2. d—Lumbar puncture
Diabetes insipidus cannot occur in primary adre-
Imaging above is a CT angiogram demonstrating a nal insufficiency (hypocortisolism/hypoaldoster-
giant ICA aneurysm expanding the sella. The most onism) because mineralocorticoid activity is
important next step is to clarify the likelihood that required for kidneys to produce free water.
it has ruptured, and if so the patient will need to be
started on subarachnoid hemorrhage treatment 5. a—Serum and CSF HCG and AFP
and the aneurysm secured. Options include lumbar
puncture (though many would avoid this given the Water deprivation test involves serial 2 hourly mea-
risk of aneurysm rupture) or assessing hemosiderin surement of serum and urine osmolality (and body
on MRI. Others may argue that given the history weight) over a water deprivation period of 6-8 h—if
and the size of the aneurysm, treatment should serum osmolality >290 and urine <300 by the end it
be performed expediently as in aneurysmal sub- is suggestive of DI and DDAVP is given. If kidneys
arachnoid hemorrhage patients. are functioning normally, DDAVP should cause a
rise in urine osmolality to >750 suggesting a cranial
Image with permission from Adam A, et al. (Eds.), (failure of posterior pituitary vasopressin secretion,
Grainger & Allison's Diagnostic Radiology, 6th ed. as in this case) cause for the DI. This case describes
Elsevier, Churchill Livingstone, 2014.
an adolescent presenting with a hypopituitarism
including cranial diabetes insipidus, with an homo-
3. b—Evening steroid dose should not be omit- geneously enhancing suprasellar mass and smaller
ted if checking 9 am cortisol enhancing pineal cyst. The main concern is of a ger-
minoma (synchronous suprasellar and pineal) given
If preoperative steroid reserve adequate or her age and imaging, rather than lymphocytic hypo-
unknown: physitis which presents later although the two may
• Check 9 am serum cortisol on day 2 and on be difficult to distinguish. MRI of the whole spine
day 3 after surgery, in patients with no with contrast should also be performed given these
evidence of cortisol deficiency before oper- findings. In a small proportion of germinomas,
ation. If already on hydrocortisone replace- there may be non-germinomatous elements secret-
ment, omit the evening dose for the ing BHCG or AFP hence serum and CSF should be
previous day before checking. checked urgently. If these are both negative biopsy
In patients without Cushing’s disease (9 am corti- should be performed to confirm a diagnosis of ger-
sol: >550 nmol/L no requirement for hydrocor- minoma and start radiotherapy with or without
tisone, 400-550 nmol/L hydrocortisone only chemotherapy.
during severe illness or stress, <400 nmol/L start
regular oral hydrocortisone). Image with permission from Jensen AW, Laack NN,
Buckner JC. Long-term follow-up of dose-adapted and
If preoperative steroid reserve deficient: reduced-field radiotherapy with or without chemother-
• In patients with proven cortisol deficiency apy for central nervous system germinoma. Int J Radiat
before surgery, continue hydrocortisone Oncol Biol Phys 2010;77(5):1449-56.

Neurosurgery Books Full


www.ketabpezeshki.com 66485438-66485457
24 SKULL BASE AND PITUITARY SURGERY 347

Water Deprivation Test Interpretation


Post-Dehydration Post-Dehydration Post-DDAVP Urine
Serum Osmolality Urine Osmolality Osmolality Diagnosis

280-90 >750 Do not give DDAVP Normal

>290-300< <300 <300 Nephrogenic DI

>290-300< <300 >750 Cranial DI

>290-300< 300-750 > 750 Partial cranial DI

<290 300-750 300-750 Partial nephrogenic DI, chronic


psychogenic polydipsia
<290 300-750 >750 Psychogenic polydipsia

6. a—Lymphocytic hypophysitis 1 level may be present in 60-70% of patients with


GH deficiency, and may suggest the need for
The clinical picture is that of anterior pituitary dynamic tests of GH secretion. Equally, a low
pathology with early involvement/impairment of IGF-1 in the presence of 3 or more other anterior
corticotrophs (short synacthen test suggesting pituitary hormone deficiencies in an otherwise
adrenal insufficiency; normally increase should be healthy individual is a strong predictor of GH defi-
>200 nmol/L and 30 min value >600 nmol/L) ciency and treatment should be considered with-
and thyrotrophs (low TSH). This is in contrast to out dynamic testing. Contraindications to GH
non-functioning pituitary adenomas where there therapy is active malignancy (other than pituitary).
is usually early involvement of gonadotrophs. If dynamic testing is required, the gold standard is
Lymphocytic hypophysitis is most commonly insulin tolerance test (ITT) where insulin admin-
seen during pregnancy or postpartum period and istration should provoke a rise in GH (peak level
represents an inflammatory/autoimmune process <10 mU/L or <3 μg/L is suggestive of severe
affecting the pituitary gland and the pituitary stalk GH deficiency). However, ITT is contraindicated
(infundibulum). Presentation is with pituitary in those with IHD and epilepsy due to the risks
failure such as lethargy, amenorrhea, diabetes associated with hypoglycemia and may give a false
insipidus and visual disturbance (if there is optic result in obese patients. In this situation, other
chiasm compression). MRI may show a homoge- dynamic tests include GNRH-arginine stimula-
neously enhancing sellar/infundibular mass and tion and glucagon stimulation test which have a
T1WI may even show loss of posterior pituitary higher sensitivity than IGFBP-3 levels.
bright spot (normally present due to vasopressin
storage) in cranial diabetes insipidus. Where there 8. a—Non-functioning adenoma
is a high clinical suspicion (e.g. postpartum, other
autoimmune conditions, ipilimumab use, absence Pituitary adenomas are the most common neo-
of serum markers for sarcoidosis) treatment with plasms in the sellar region and comprise
steroids and supportive therapy can be initiated. 10-15% of all primary brain tumors. They are
However, if the picture is uncertain or no response classified as microadenomas (<10 mm) and
to steroids biopsy should be performed as it is the macroadenomas (>10 mm). Furthermore, they
only way to make the diagnosis. may be non-functional pituitary adenomas
(clinically not hormonally active) or functional
Image with permission from Hess CP, Dillon WP. Imag- (clinically showing signs of hormone excess).
ing the pituitary and parasellar region. Neurosurg Clin N Non-functional adenomas account for 15-30%
Am 2012;23(4):529-42.
of pituitary adenomas and show gonadotroph
(FSH, LH) deficiency in 80%, followed by
7. b—GHRH-arginine stimulation test somatotrophy (GH secretion) deficiency, and
later thyrotrophs (TSH) and corticotrophs
The clinical features of growth hormone defi- (ACTH) deficiency in 20-50%. Gonadotrophs
ciency are nonspecific and include lethargy, low are the commonest primary cell type involved
mood, poor quality of life, loss of muscle mass in NFPA. As such, patients tend to present with
and central adiposity. GH is normally secreted hypogonadotrophic hypogonadism early, and
in a pulsatile fashion (5 per 24 h) hence random features of GH deficiency, secondary hypothy-
measurement of levels is not helpful. A low IGF- roidism and secondary adrenal insufficiency later

Neurosurgery Books Full


www.ketabpezeshki.com 66485438-66485457
348 PART III CRANIAL NEUROSURGERY

on. Hypogonadotrophic hypogonadism is charac- >2000 mU/L (note that hypothyroidism may
terized by Large NFSs compress the pituitary also cause hyperprolactinemia). Macroprolactin
stalk, causing impairment of dopamine transport with its longer half-life and biological inert
to the lactotrophs and a slight rise in prolactin nature needs to be measured, and may cause a
secretions above normal (but prolactinoma is spuriously high prolactin level. Equally, depend-
usually associated with levels >200 mU/L). ing on the assay used, if prolactin levels are truly
Macroadenomas of either type (functional or very high they may saturate the assay giving a
non-functional) may present with mass effect on near-normal result (Hook effect). Estrogen con-
adjacent structures (e.g. chiasmal compression) or taining oral contraceptives can stimulate lacto-
pituitary apoplexy. MRI with and without contrast trophs and cause hyperprolactinemia. The
is the imaging modality of choice. Management of primary treatment of prolactin-secreting micro-
nonfunctioning pituitary adenoma is with elective adenomas is medical and the role of imaging in
transsphenoidal surgery if there is evidence of optic cases of hyperprolactinemia is to assess size and
chiasm compression (as in this case with suprasellar any chiasmal compression. Bromocriptine and
extension)—otherwise endocrine, ophthalmologic cabergoline are both safe options for females
and imaging surveillance is appropriate. in the reproductive age group presenting with
a microprolactinoma if they are planning a preg-
Image with permission from Roser F, Honegger J, nancy in the near future, and have not been
Schuhmann MU, Tatagiba MS. Meningiomas, nerve
sheath tumors, and pituitary tumors: diagnosis and associated with fetal malformation. The therapy
treatment. Hematol Oncol Clin North Am 2012;26 can be stopped once pregnancy is confirmed.
(4):855-79. Dopamine agonists reduce prolactin levels and
induce ovulation, hence females started on them
9. b—Transsphenoidal debulking followed by are possibly at higher risk of pregnancy and
stereotactic radiosurgery should be given advice regarding contraception
at initiation if they do not wish to become preg-
There is extensive cavernous sinus involvement nant. Management of prolactinomas during
hence transsphenoidal surgery alone is unlikely to pregnancy is challenging as prolactin measure-
be sufficient (but will decompress/separate tumour ments are unreliable. There is <5% chance of
mass from optic nerve), and given the features of a microprolactinoma regrowth and 15-40%
non-functional pituitary adenoma stereotactic chance of macroprolactinoma regrowth during
radiosurgery is the appropriate next action as med- pregnancy. As such, patients need to be kept
ical therapy is unlikely to be of benefit. under close surveillance with periodic formal
visual field assessments. Imaging in other secret-
Image with permission from Loevner L. Brain Imaging: ing pituitary adenomas (TSH, GH, ACTH) is
Case Review Series, 2nd ed. Elsevier, Mosby, 2009. also important to localize the exact position of
the tumor for surgical treatment. In particular,
10. b—Hook effect describes spuriously high given the high morbidity of Cushing’s disease
prolactin levels due to macroprolactin if no lesion is found on MRI then inferior petro-
complexes sal and/or cavernous sinus venous sampling may
be necessary to grossly lateralize an adenoma for
Prolactinomas are the most common secreting surgery.
pituitary adenomas and tend to arise laterally
within the anterior lobe of the pituitary gland. 11. c—Indications for surgery include a
They may depress the floor of the sella turcica secreting tumor
or expand one side of the gland, causing a subtle
upwardly convex bulge and contralateral Due to improvements in imaging there has been a
displacement of the infundibulum. Hyperprolac- rise in the number of incidental pituitary adeno-
tinemia in men interferes with sperm production mas discovered on cranial imaging with rates of 5-
(infertility) and testosterone production (leth- 30% for those <10 mm (microincidentaloma)
argy, reduced libido, reduced muscle mass), and 0.1-0.25% for those >10 mm in size (macro-
galactorrhea, loss of pubic/axillary hair, and incidentaloma). They should be investigated for
erectile dysfunction (and small gonads in pre- subclinical disease even if asymptomatic includ-
pubertal boys). In females, hypeprolactinemia ing endocrinological history and examination,
reduces estradiol production and this causes anterior pituitary profile, MRI pituitary (if lesion
irregular menstrual cycles, amenorrhea, galac- found on CT) and, if clinical visual field defect or
torrhea and premature menopausal symptoms evidence of chiasmal compression on imaging,
(does not cause similar symptoms in postmeno- formal visual field testing. If all results are normal,
pausal women). Blood tests show prolactin levels microincidentalomas should be rescanned at

Neurosurgery Books Full


www.ketabpezeshki.com 66485438-66485457
24 SKULL BASE AND PITUITARY SURGERY 349

1 year then every 1-2 years for the first 3 years comfortable examining the pituitary for small ade-
then less frequently after that. Macroincidentalo- nomas. ACTH levels from the right and left petro-
mas should be rescanned at 6 months then at least sal sinuses obtained during the sampling study may
yearly for the first 3 years. Repeat anterior pitui- tell the neurosurgeon in which side of the pituitary
tary hormone profile should only be performed if gland the tumor is likely to be found, but this
symptomatic or MRI changes are seen. Indica- information is not 100% accurate. If IPSS shows
tions for surgery include visual field deficit, optic no gradient in ACTH levels, start the search for
nerve/chiasm compression, ophthalmoplegia, a carcinoid tumor (e.g. lung, adrenal, GI) and
pituitary apoplexy with severely reduced visual treatment as appropriate.
acuity or field defect, and evidence of secreting
tumor (e.g. Acromegaly, Cushing’s disease). FURTHER READING
Samuels MH. Cushing syndrome. In: McDermott MT (Ed.),
FURTHER READING Endocrine secrets. Saunders, Elsevier, 2013.
Freda PU, Beckers AM, Katznelson L. Pituitary incidenta-
loma: an endocrine society clinical practice guideline. J Clin 14. e—Can cause amenorrhea in females and
Endocrinol Metab 2011;96(4):894-904. infertility in males

12. b—Cabergoline is a dopamine receptor Cushing’s syndrome (hypercortisolism) can pre-


antagonist sent in children and young adults with an
increase in body weight, central obesity and
Dopamine receptor agonists (e.g. bromocriptine, growth retardation. In adults, it is associated
cabergoline) augment physiological inhibition of with mood facies, proximal myopathy, bruising,
prolactin secretion. Octreotide is a somatostain abdominal striae, oligomenorrhea/amenorrhea
analog used in the treatment of acromegaly is in females, impotence in males, infertility, hir-
associated with the development of gallstones sutism, hypertension and diabetes. Adrenal
and reduced gall bladder contractility, but causes tumors are commoner in children under 10
>2% tumor reduction in 75% of patients treated. (neuroblastoma) causing ACTH-independent
GI side effects are common, and glycemic Cushing’s syndrome with signs of virilization.
control also. Cushing’s disease due to ACTH-secreting pitu-
itary tumor is commoner in older children and
13. c—Inferior petrosal sinus sampling adults. In adults, ectopic ACTH production
due to lung cancer or carcinoid/neuroendocrine
This finding is nonspecific and occurs in up to tumor can also occur. Initial screening tests such
10% of healthy people. It may or may not be as 24 h urinary cortisol collection, 9 am and mid-
related to Cushing syndrome. The odds are good night cortisol (loss of diurnal variation; remains
that the patient has a pituitary tumor, but the MRI high), and overnight (low-dose) dexamethasone
findings do not prove this. The MRI is diagnostic suppression test help confirm the diagnosis of
only if it shows a large tumor. One option is to pro- hypercortisolism. Second line localization tests
ceed directly to pituitary surgery because a patient aim to differentiate hypercortisolism based on
with abnormal MRI findings has a 90% chance of whether it is secondary to increased ACTH
having an ACTH-secreting pituitary tumor. To secretion (e.g. ACTH-secreting pituitary tumor
achieve more diagnostic certainty, one has to per- or ectopic/carcinoid tumor) or not (e.g. adrenal
form bilateral simultaneous inferior petrosal sinus tumor). Third line tests aim to differentiate loca-
sampling (IPSS) for ACTH levels. Catheters are tion of ACTH-dependent hypercortisolism as
advanced through the femoral veins into the infe- pituitary (reduces ACTH/cortisol production
rior petrosal sinuses, which drain the pituitary in response to high-dose dexamethasone; gradi-
gland, and blood samples are obtained for ACTH ent on petrosal sinus sampling) or ectopic tumor
levels. If ACTH levels in the petrosal sinuses are (no reduction in ACTH/cortisol secretion in
significantly higher than those in peripheral sam- response to high-dose dexamethasone; cortisol
ples, the pituitary gland is the source of excessive rise if CRH stimulation test; no cortisol rise on
ACTH. If there is no gradient between petrosal CRH stimulation; no gradient on petrosal sinus
sinus and peripheral levels of ACTH, the patient sampling).
probably has a carcinoid tumor somewhere. The
accuracy of the test is further increased if ACTH 15. b—Repeat transsphenoidal surgery
responses to injection of exogenous CRH are mea-
sured. If sinus sampling is positive for a gradient Patients with inadequately treated Cushing's syn-
transsphenoidal surgery (TSS) should be sched- drome have a markedly increased mortality rate
uled with an experienced neurosurgeon who is (four- to fivefold above the normal rate), usually

Neurosurgery Books Full


www.ketabpezeshki.com 66485438-66485457
350 PART III CRANIAL NEUROSURGERY

from cardiovascular disease or infections. Hyper- of “fetal face,” where the forehead corresponds to
tension, impaired glucose tolerance, dyslipide- the planum sphenoidale, the eyes to the two opti-
mia, and visceral obesity all contribute to the cocarotid recesses, the eyebrows to the two bony
excess risk for cardiovascular mortality. This protuberances covering the optic nerves, the nose
excess mortality normalizes with adequate ther- to the sellar floor, and the mouth to the clival
apy. If TSS does not cure a patient with Cushing’s indentation, laterally limited by the two paraclival
disease, alternative therapies must be tried carotid arteries, representing the cheeks.
because patients with inadequately treated hyper-
cortisolism have increased morbidity and mortal- Image with permission from Laws ER, Lanzino G (Eds.),
Transsphenoidal Surgery, Saunders, Elsevier, 2010.
ity rates. Of the various options after failed
surgery, none is ideal. Patients may require repeat
pituitary surgery, radiation therapy, medical ther- 18. e—Empirical antibiotics and CT head
apy to block adrenal cortisol secretion (e.g. keto-
conazole, metyrapone, mitotane, or etomidate), CT head should be done initially to exclude
centrally acting agents that suppress ACTH development of postoperative hydrocephalus
secretion, and glucocorticoid receptor blocker and exclude subdural hematoma formation from
(mifepristone). Bilateral adrenalectomy can be intracranial hypotension due to CSF leak. In
safely performed via a laparoscopic approach, patients who have a traumatic leak and normal
with low morbidity in experienced hands. How- CSF pressure, conservative treatment consists
ever, this procedure leads to lifelong adrenal of bed rest with head of bed elevation and lumbar
insufficiency and dependence on exogenous glu- drainage of CSF for 5-10 days. With conservative
cocorticoids and mineralocorticoids. The other management, there is a reported risk ranging
main drawback is the development of Nelson syn- from 7% to 30% of ascending meningitis.
drome in up to 30% of patients after adrenalec- The incidence of spontaneous resolution with
tomy. Nelson syndrome is the appearance, conservative management is reported to be
sometimes years after adrenalectomy, of an 70%. The general consensus among practicing
aggressive corticotroph pituitary tumor. otolaryngologist is that antibiotics should not
be used for conservative management unless
16. a—Oral glucose tolerance test there is a very large defect with comminuted bone
of the skull base as a simple CSF leak carries a 7%
Acromegaly is due to excess secretion of GH/ infection rate (meningitis, intracranial abscess,
IGF-1 and results in coarsening of facial features, cellulitis/abscess, and osteomyelitis) and prophy-
increased ring/shoe size, prognathism, macro- lactic antibiotics have not been shown to decrease
glossia, widely spaced teeth, enlargement of the the risk of infection. After endoscopic repair,
extremities and increased sweating. Due to pulsa- antibiotics are generally recommended for 24-
tile release of GH, serum GH levels are rarely 48 h. This is done to cover possible contamina-
helpful—the main test should be random IGF-1 tion at the time of surgery in a non-sterile field
serum level which is useful in diagnosis and mon- with concomitant sealing of the sterile to non-
itoring of subsequent treatment. However, a nor- sterile flushing of an active leak. A reconstructive
mal IGF-1 level does not exclude acromegaly and ladder should be used to help determine the type
an oral glucose tolerance test (OGTT) with mea- of repair performed. For simple, small (less than
surements of GH remains the gold standard. Fail- 1 cm) defects, a fat plug harvested from the ear-
ure of oral glucose administration to suppress GH lobe or abdomen can be used to plug the defect.
level below 1 ng/mL (or higher sensitivity if The next option includes a simple overlay graft
<0.4 ng/mL cutoff) is supportive of a diagnosis harvested from the nasal floor mucosa, turbinate
of acromegaly. In those suspected to have acro- mucosa, or nasal septum. If a more complex,
megaly, MRI pituitary should be performed. larger reconstruction is in order, a composite
(underlay and overlay) graft can be used consist-
17. E—Opticocarotid recess. ing of an intracranial underlay of bone or carti-
lage from nasal septum, auricular cartilage or
A—planum sphenoidale, B—chiasmatic groove turbinate bone, and an overlay graft of mucosa
(CG), C—tuberculum sellae, D—optic nerve (free or pedicled) as above. Local pedicled flaps
prominence, E—opticocarotid recess, F—clival should include the nasoseptal flap, which is sup-
recess, G—prominence of the internal carotid plied by the posterior nasal septal artery, a termi-
artery nal branch of the sphenopalatine artery.
The panoramic view provided by the endo- Additional grafts that can be useful in larger
scope of the bony prominences and depressions defects include temporal fascia or tensor fascia
inside the sphenoid sinus allows one to see a sort lata grafts. These grafts are often bolstered in

Neurosurgery Books Full


www.ketabpezeshki.com 66485438-66485457
24 SKULL BASE AND PITUITARY SURGERY 351

the sinonasal cavity with abdominal fat, a naso- of pituitary adenoma on MRI, lack of TSH
septal flap or both. In complex situations of exten- response to TRH stimulation testing and lack
sive defects or poor local tissue, such as in of a family history of thyroid problems. Sur-
chemoradiated patients, a craniotomy with peri- gery is the treatment of choice, but octreotide
cranial flap or free flap reconstruction of the skull may be used in non-surgical candidates or sur-
base may be necessary. A multitude of studies gical failure. 3—k, Optic pathway glioma.
over the past 20 years have shown high success Increased association with NF-1.
rates of primary repair around 90%, and second-
ary repair around 97%. These success rates com- 39. 1—k, Primary adrenal insufficiency (Addi-
pare favorably to traditional craniotomy son’s disease). Short synacthen tests can dem-
approaches with reported success rates between onstrate adrenal insufficiency (lack of cortisol
70% and 80% that carry a higher morbidity production in response to synthetic ACTH).
profile. Long synacthen tests differentiate between
primary causes (e.g. adrenal infarction) which
FURTHER READING fail to respond even after several doses, com-
Scholes MA. ENT Secrets, 4th ed. Elsevier, 2016. pared to secondary causes (e.g. non-function-
ing adenoma causing reduced ACTH
release) which start to produce cortisol after
several doses. 2—d, Diabetes insipidus.
ANSWERS 19–37
3—h, Nelson-Salassi syndrome. When bilat-
Additional answers 19–37 available on eral adrenalectomy is performed for ACTH-
ExpertConsult.com secreting pituitary adenoma (Cushing’s dis-
ease) to treat the associated hypercortisolism,
loss of negative cortisol feedback to hypo-
thalamus causes increased secretion of corti-
cotropin releasing hormone and growth of
EMI ANSWERS pituitary tumors. This was seen before the
advent of transsphenoidal surgery and radio-
38. 1—f, Gonadotropin-secreting adenoma. This therapy, when adrenalectomy was more com-
a rare tumor presenting with ovarian cysts and monly performed.
menstrual abnormalities, high or normal
estradiol and usually suppressed LH due to 40. 1—k, Foramen rotundum; 2—d, Foramen
secretion of FSH by the pituitary adenoma. lacerum, 2—h, Internal auditory meatus
Treatment is surgery. 2—m, TSH-secreting
tumor (thyrotropinoma, TSHoma). Inappro- a—Lesser, greater wings of sphenoid bone,
priately high or normal TSH in the presence b—Carotid groove, c—Pituitary fossa,
of high free T3 and T4 levels is suggestive of d—Foramen lacerum, e—Condylar fossa (man-
either TSH-secreting tumor or thyroid hor- dibular), f—Facial hiatus, g—Temporal bone,
mone resistance syndromes. TSH-secreting h—Internal auditory meatus, i—Optic canal,
tumors represent 0.5-1% of pituitary adeno- j—Superior orbital Fissure, k—Foramen rotun-
mas and present with features of thyrotoxico- dum, l—Foramen ovale, m—Foramen spinosum,
sis and goiter, and is distinguished from n—Foramen jugulare, o—Hypoglossal Canal.
hormone resistance by its alpha-subunit to
Image with permission from Flint PW (Eds.), Cummings
TSH ratio >1 (if >5.7 is diagnostic), presence Otolaryngology, 6th ed. Saunders, Elsevier, 2015.

Contents of Skull Base Foramina


Skull Base
Foramina Contents

Cribiform plate CN I, anterior ethmoidal artery (branch of ophthalmic artery)

Opitc canal CN II, ophthalmic artery (branch of ICA), meninges, optic nerve sheath

Superior orbital Middle part: nasociliary nerve (CN V1 branch), oculomotor nerve (III), abducens nerve (CN VI)
fissure Lateral part: Trochlear nerve (CN IV), frontal nerve (V1), lacrimal nerve (V1), orbital branch of
medial meningeal artery, superior ophthalmic vein

Continued on following page

Neurosurgery Books Full


www.ketabpezeshki.com 66485438-66485457
352 PART III CRANIAL NEUROSURGERY

Contents of Skull Base Foramina (Continued)


Skull Base
Foramina Contents
Foramen Maxillary nerve (CNV2)
rotundum

Foramen ovale Mandibular nerve (CNV3), emissary vein (cavernous sinus to pterygoid plexus)

Foramen lacerum Connective tissue, meningeal branches of ascending pharyngeal artery, and emissary vein
(cavernous sinus to pterygoid plexus). Greater petrosal nerve (VII) joins with deep petrosal
nerve to form vidian nerve (of the pterygoid canal). Lacerum segment of ICA courses above
as it exits the carotid canal.

Foramen Meningeal branch of CN V3, medial meningeal artery (branch of maxillary artery)
spinosum

Carotid canal Petrous ICA, internal carotid venous and sympathetic plexus

Internal acoustic Facial nerve (VII), vestibulocochlear (VIII): superior vestibular, inferior vestibular, cochlear
meatus nerve, labyrinthine artery (branch of basilar artery), labyrinthine veins

Jugular foramen Rostromedial: Inferior petrosal sinus, meningeal branch of ascending pharyngeal artery
Middle part: Cranial nerves IX, X, XI
Caudolateral: Internal jugular vein, meningeal branch of occipital artery, meningeal branch
of X

Hypoglossal canal Hypoglossal nerve (XII) and venous plexus

Foramen magnum Meninges, marginal sinus, vertebral arteries, anterior spinal artery, medulla oblongata/
spinal cord, spinal accessory nerve (XI)

41. 1—f, Retrolabyrinthine; 2—h, Transcochlear depending on tumor size and location (e.g. middle
fossa, retrosigmoid, retrolabyrinthine). Use of other
Serviceable hearing includes a pure tone average approaches sacrifices hearing, hence other criteria
threshold better than 50 dB and/or speech discrim- such as surgical exposure gained and risk of facial
ination greater than 50% (50/50 rule), and may nerve injury become prime considerations.
favor the use of hearing-sparing approaches

Surgical Approaches to the Cerebellopontine Angle (CPA)


Approach Rationale Indications Contraindications

Retrosigmoid Easy and rapid access to the Extra-axial lesions in the Patients must have patent
craniotomy CPA. CPA and intra-axial lesions contralateral transverse
Extended retrosigmoid arising along the petrosal and sigmoid sinuses
approach permits access surface of the cerebellum, before manipulation of the
ventral to the brainstem and cerebellar peduncles, or sinuses ipsilateral to the
near the tentorium (safe brainstem. approach.
alternative to more radical
cranial base approaches).

Translabyrinthine Sacrificing the labyrinth (i.e. Indications include removal Lesions extending
hearing) to give direct of CPA lesions with anteriorly to prepontine
access to the internal preoperative unserviceable cistern
auditory canal (IAC) and hearing, regardless of lesion Ipsilateral chronic otitis
cerebellopontine angle size. media (relative)
without cerebellar retraction Only hearing ear
Exposure of the posterior
fossa and 320-degree
exposure of the IAC
circumference.

Continued

Neurosurgery Books Full


www.ketabpezeshki.com 66485438-66485457
24 SKULL BASE AND PITUITARY SURGERY 353

Surgical Approaches to the Cerebellopontine Angle (CPA) (Continued)


Approach Rationale Indications Contraindications
Transcochlear Extension of the CPA tumors with anterior Ipsilateral chronic otitis
translabyrinthine approach extension and unserviceable media (relative)
anteriorly, which includes hearing. Only hearing ear (relative)
drilling the posterior and Extensive petrous apex
superior external auditory lesions with inner ear
canal and sacrificing middle compromise (e.g. petrous
ear structures and inner ear apex cholesteatomas).
(cochlea), and rerouting the Petroclival lesion with
facial nerve to provide extension ventral to the
access to the anterior brainstem.
cerebellopontine angle, Temporal bone and clival
petrous apex, and ventral lesions with extension to the
brainstem. posterior fossa (e.g.
chordomas)

Retrolabyrinthine The retrolabyrinthine Expose widely the posterior This approach is unable to
approach approach is a hearing- petrous face and cisternal access the internal
preserving presigmoid portions of cranial nerves VII auditory canal or petrous
approach that uses a and VIII with a minimal apex directly because of
mastoidectomy and degree of cerebellar the interposition of the
skeletonization of the retraction identify and labyrinthine and cochlear
sigmoid sinus to expose the expose the superior structures between the
presigmoid dura between petrosal sinus, as a first step surgeon and these regions.
the labyrinth (semicircular for division of the tentorium
canals) and sigmoid sinus.

Transotic The objective of this CPA or the temporal bone


approach approach is to obtain a direct lesions with invasion of the
lateral exposure and the IAC or the otic capsule in
widest possible access to the patients with no serviceable
CPA through the medial wall hearing. In this clinical
of the temporal bone, from setting, the transotic
the superior petrosal sinus approach offers the best
to the jugular bulb, and from possible exposure for tumor
the internal carotid artery to extirpation and preservation
the sigmoid sinus. The of facial nerve with minimal
tympanic and mastoid morbidity.
portions of the fallopian
canal are left in situ. This
transtemporal access is
achieved at the expense of
bony exenteration, rather
than cerebellar retraction.

Middle cranial a largely extradural Convex floor of the middle Tumors with significant
fossa approach approach to the bony fossa is the most posterior fossa extension
structures that straightforward region to Tumors caudal to the IAC
make up the floor of the access with this approach,
middle fossa. this approach is commonly
the starting point for anterior
transpetrosal approaches to
the internal auditory canal
(IAC) or petroclival junction.

Neurosurgery Books Full


www.ketabpezeshki.com 66485438-66485457
CHAPTER 25

CRANIAL INFECTION
SINGLE BEST ANSWER (SBA) QUESTIONS
1. A 35-year-old transplant patient develops 4. A 27-year-old man presents to his primary care
headache, neck stiffness, photophobia and doctor with a low-grade fever, headache, and
fever. Cerebrospinal fluid (CSF) testing with neck stiffness, which have become more both-
India ink stain reveals a fungal infection. ersome over the past 1-2 weeks. Serum is pos-
Which one of the following is the cause of itive for Borrelia burgdorferi IgM. CSF
this patient’s fungal meningitis? polymerase chain reaction (PCR) is also posi-
a. Aspergillus tive for this organism. The cranial nerve most
b. Blastomyces commonly affected in this disease is most likely?
c. Candida a. Abducens nerve
d. Cryptococcus b. Facial nerve
e. Mucor c. Glossopharyngeal nerve
d. Oculomotor nerve
2. An 82-year-old previously healthy female e. Trigeminal nerve
with a recent history of upper respiratory tract
infection presents with generalized weakness, 5. An 11-year-old girl is bitten on her upper
headache, and blurry vision. Other symptoms arm by a stray dog while on holiday in South
include fever, vomiting and eye pain on move- America. Two weeks later she develops
ment with mild photosensitivity. She has no throat spasms and confusion. Which one of
drug allergies. Examination findings include the following is most accurate regarding the
temperature of 102.5 °F (39.1 °C), nuchal causative virus?
rigidity, and drowsiness GCS is E3V4M5. a. Cause of death is usually dehydration
Her blood pressure is 82/56 and she is tachy- b. Commonly causes progression to quadri-
cardic, with only a transient response to fluid plegia in 80%
challenge. Which one of the following is the c. Inducing an artificial coma obviates the
next most appropriate action in this case? need for post-exposure prophylaxis
a. Cranial imaging then perform a lumbar d. Post-exposure prophylaxis consists of a
puncture vaccine
b. Give the patient a prescription for oral co- e. Spreads retrogradely in peripheral nerves
amoxiclav (Augmentin)
c. Immediately start intravenous antibiotics 6. A 64-year-old female presents with progres-
d. Immediately start oral dexamethasone sive cognitive impairment, tremors, gait
e. Obtain CSF and blood cultures and ataxia, and myoclonic jerks over the course
observe the patient in high dependency of 6 months. There is no relevant family his-
unit the results come back tory. MRI of the head reveals a subtle
increase in T2 signal in the basal ganglia
3. A 9-year-old is brought into the emergency bilaterally. EEG reveals disorganized back-
room lethargic with a stiff neck and fever. ground activity with periodic sharp-wave dis-
Despite aggressive therapy in the ITU the charges that occur repetitively at 1-s intervals
child dies. Postmortem evaluation reveals that and extend over both sides of the head. There
the child had primary amoebic meningoen- was also evidence of diffuse slowing and tri-
cephalitis. This condition is usually acquired phasic waves. The clinical picture is most
through which one of the following means? consistent with which one of the following?
a. Animal bites a. Alzheimer's dementia
b. Drinking contaminated water b. Friedreich's ataxia
c. Eating contaminated meat c. Multi-infarct dementia
d. Freshwater swimming d. Parkinson's disease
e. IV drug abuse e. Spongiform encephalopathy

354
Neurosurgery Books Full
www.ketabpezeshki.com 66485438-66485457
25 CRANIAL INFECTION 355

7. An 85-year-old woman has 3 days of gradu-


ally worsening fever and headache. She then
develops blurry vision and a stiff neck. MRI
with contrast has an enhancement pattern
suggesting rhombencephalitis. CSF shows a
mild pleocytosis with no organisms. All blood
and CSF cultures are negative. Which one of
the following is the most likely organism
responsible for the patient's condition?
a. Borrelia burgdorferi
b. E. coli
c. HTLV-1
d. Listeria monocytogenes
e. MRSA

8. A 75-year-old left-handed woman presented


to the emergency room with what at first
was thought to be a stroke. There is no clear
history of recent infection. MRI head was
performed and T1 + gadolinium, DWI and C
ADC map are shown. Which one of the fol-
lowing is the most appropriate next step?

a. Cerebral angiogram
b. Image guided attempted gross total
resection
c. Lumbar puncture and oligoclonal bands
d. Stereotactic biopsy
e. Stereotactic needle aspiration

9. During formation of an abscess capsule,


when does necrosis begin?
a. Days 1-3
b. Days 4-9
c. Days 10-13
d. Days 14-20
e. Day 21 onwards

10. A 40-year-old ex-IV drug abuser presents to


the emergency room with a seizure. CT head
with contrast shows 3 cm diameter ring
enhancing lesion periventricular location.
A MRI is performed and the lesion is bright
on DWI and dark on ADC map. Which
one of the following would be the appropri-
ate next step in management?
a. Blood cultures and external ventricular
drain then start intravenous antibiotics
b. Craniotomy and excision of abscess then
start intravenous antibiotics
c. Image-guided aspiration of abscess then
start intravenous antibiotics
d. Endoscopic aspiration and irrigation
e. Real-time ultrasound-guided excision of
abscess and start intravenous antibiotics
B

Neurosurgery Books Full


www.ketabpezeshki.com 66485438-66485457
356 PART III CRANIAL NEUROSURGERY

11. A 37-year-old HIV positive male presents


with headache, confusion, new right-sided
weakness progressing over the previous
week. He has been on highly active retroviral
therapy for the last 10 years and not had any
problems. On examination, his responses are
slow and he has some difficulty sustaining
attention. He has a right hemiparesis with
increased reflexes on the right. FBC, U + E
and CRP are normal. T1 contrast MRI is
shown. Which one of the following organ-
isms is the most likely cause?

a. Basal meningitis
b. Cerebral abscess
c. Epidural abscess
d. Resolving hematoma
e. Subdural abscess

14. A 29-year-old ex-IVDU who previously


underwent aspiration of a brain abscess
and prolonged inpatient antibiotic course
re-presents after several generalized tonic-
clonic seizures progressing into status epilep-
ticus. Contrast CT head is shown. What is
the next appropriate action?

a. Cryptococcus neoformans
b. Herpes zoster
c. Pneumocystic jerovecii
d. Toxoplasma gondii
e. Tuberculosis

12. A 45-year-old renal transplant patient pre-


sents with a 3-week history of worsening right
headache, ear pain, and pyrexia. His medica-
tions include tacrolimus and cyclosporin. He
is diagnosed with malignant fungal otitis
externa and secondary osteomyelitis of the
skull base. Which one of the following organ-
isms is most likely?
a. Actinomyces
b. Aspergillus
c. Candida
d. Cryptococcus neoformans
e. Naegleria fowleri a. Cerebral angiogram
b. Decompressive craniectomy
13. A 67-year-old with chronic sinusitis presents c. External ventricular drain
with severe frontal headache and change in d. Lumbar drain
smell. T1-gadolinium enhanced MRI head is e. Palliative care
shown. Which one of the following is most
likely?

Neurosurgery Books Full


www.ketabpezeshki.com 66485438-66485457
25 CRANIAL INFECTION 357

15. A 27-year-old immigrant from South a. Amphotericin B


America presents with a generalized seizure. b. Fluconazole
After awakening, he relates that he has had c. Highly active antiretroviral therapy
two or three episodes of unexplained loss (HAART)
of consciousness in the past 2 years. d. Intravenous acyclovir
He has otherwise been healthy and worked e. Intravenous rifampicin
as a farmer. His examination is normal.
MRI head was performed and T2 and 17. A 52-year-old woman with acquired immune
T1 +GAD images are shown. What is the deficiency syndrome (AIDS) presents to the
most likely organism? emergency room with mild left hemiparesis
and altered mental status. A CT scan reveals
several ring-enhancing lesions with minimal
mass effect. Which one of the following is
the best next step in management?
a. Get a cerebral angiogram
b. Order a ventricular CSF aspiration
c. Perform a lumbar puncture and include
CSF for Epstein-Barr virus (EBV) PCR
in tests ordered
d. Stop all antiretroviral therapy
A B e. Treat with intravenous acyclovir

18. A 12-year-old boy has left body weakness. A


brain magnetic resonance imaging (MRI)
a. Histoplasma scan reveals a polycystic lesion. Which one
b. Mucor of the following is most likely?
c. Shistosoma mansonii
d. Taenia solium
e. Toxoplasma gondii

16. A 35-year-old woman has progressive


numbness of the right arm and difficulty
seeing objects in the right visual field. She
is known to be HIV positive, but has not
consistently taken medications in the past.
On examination, she appears healthy, but
has a right homonymous hemianopsia and
decreased sensory perception in her right
upper extremity and face. Her CD4 count
is 75 cells per μL, and her MRI is consistent
with a demyelinating lesion of the left
parieto-occipital area. CSF PCR for JC virus
is positive. Which one of the following is the
most appropriate treatment in this case?
a. Diphyllobothrium latum
b. Echinococcus
c. Schistosoma hematobium
d. Schistosoma japonicum
e. Taenia solium

QUESTIONS 19–22
Additional questions 19–22 available on
ExpertConsult.com
A B

Neurosurgery Books Full


www.ketabpezeshki.com 66485438-66485457
358 PART III CRANIAL NEUROSURGERY

EXTENDED MATCHING ITEM (EMI) On examination, he has mild left hemiparesis.


MRI shows abnormal signal, with enhance-
QUESTIONS ment, in the right anterior temporal lobe.
3. An 8-year-old girl shows rapidly deteriorating
23. Intracerebral abscess: school performance and personality change
a. Actinomyces over several weeks, then experiences a general-
b. Aspergillus ized tonic-clonic seizure. A neurologist
c. Bacillus examining the child discovers chorioretinitis,
d. Candida ataxia, and bilateral upper motor neuron signs.
e. E. coli Her EEG exhibits periodic bursts of high-
f. Klebsiella voltage slow waves, followed by recurrent
g. Listeria monocytogenes periods of burst suppression. CSF shows an
h. Nocardia increased γ-globulin fraction.
i. Pseudomonas
j. Propionibacterium 25. Infections affecting the CNS:
k. Staphylococcus aureus a. Amoebiasis
l. Toxoplasma gondii b. Aspegillosus
m. Viridans streptococci c. Bartonella hensae
d. Candidiasis
For each of the following descriptions, select the e. Cryptococcosis
most appropriate answers from the list above. f. Echinococcosis
Each answer may be used once, more than once g. Leptospirosis
or not at all. h. Neurocystercercosis
1. Commonest cause of brain abscess post- i. Tabes dorsalis
neurosurgery j. Toxoplasma
2. Brain abscess secondary to otitis externa k. Whipple disease
3. Commonest cause of pyogenic brain abscesses
For each of the following descriptions, select the
24. Viral encephalitis:
most appropriate answers from the list above.
a. Cytomegalovirus
Each answer may be used once, more than once
b. Enterovirus
or not at all.
c. Epstein Barr Virus 1. A 50-year-old presents with a several-month
d. Herpes simplex virus history of difficulty with bladder control,
e. HIV an unsteady gait, and pain in his legs. The
f. Human herpes virus 6 pain in his legs is sharp, stabbing, and parox-
g. JC virus ysmal. He has no history or tests suggestive
h. Measles of diabetes. He has absent deep tendon
i. Mumps reflexes in his legs, markedly impaired
j. Rabies virus vibration sense in his feet, and a positive
k. Varicella zoster virus Romberg sign. He has no cerebellar signs
or weakness.
For each of the following descriptions, select the 2. A 29-year-old HIV positive female pet store
most appropriate answers from the list above. worker presents with 2 days of confusion
Each answer may be used once, more than once and seizures, which progressed to status epi-
or not at all. lepticus. General examination discloses epi-
1. A 35-year-old intravenous drug abuser pre-
trochlear lymphadenopathy. CSF analysis is
sents with inability to control his left hand.
normal. MRI brain is unremarkable.
He reports that at times he will button his shirt
3. A 54-year-old woman presents with
with his right hand, only to find that his left
6 months of progressive memory loss. She
hand is unbuttoning the shirt against his con-
has limited vertical eye movements, and on
trol. He has a history of thrush. He is alert and
examination, she has rhythmic, synchro-
oriented. MRI shows an increased T2 signal
nous grimacing and eye closure movements.
affecting the subcortical white matter of the
Jejunal biopsy reveals periodic acid-Schiff
right parietal lobe without enhancement.
(PAS)-positive cells.
2. A 21-year-old male presents with 3 days of
headaches and fever, followed by hallucina-
tions, speech disturbance, and lethargy.

Neurosurgery Books Full


www.ketabpezeshki.com 66485438-66485457
25 CRANIAL INFECTION 359

SBA ANSWERS
1. d—Cryptococcus and raccoons). After introduction of the virus, the
incubation period until fulminant infection
Cryptococcosis is usually acquired through the appears extends from a few days to over 1 year,
lungs and spreads to the CNS through the blood- but usually ranges from 1 to 2 months. Bites of
stream. In the CNS, it may produce either a men- the head and face carry the greatest risk of causing
ingitis or a meningoencephalitis. Other examples fatal disease. Early after exposure, the patient will
include histoplasma, and candida (very rare and often complain of pain or paresthesias at the site
usually in premature babies). Fungal CNS of the animal bite. Animals transmitting the virus
infections occur in immunodeficiency states include. Furious form (80%) is commonly associ-
(e.g. AIDS, or immunosuppressive drugs, lym- ated with hydrophobia, where viral multiplication
phoproliferative disorders). Aspergillus, Candida, in the salivary glands results in painful spasms of
Mucor, and Rhizopus can also cause CNS fungal throat/larynx, especially when saliva production
infections, but rarely meningitis. Aspergillus tends is increased associated with drinking or the
to cause abscesses in immunocompromised indi- thought of it. Dehydration is no longer likely
viduals, and Mucor affects mostly diabetics. because intravenous fluids can be given to
completely replace what the hydrophobic patient
2. c—Immediately start intravenous antibiotics cannot consume by mouth. Other complications
of rabies include a paralytic form of the disease
This patient has presented with probable menin- that progresses to quadriplegia (dumb rabies) in
gitis, but with evidence of septic shock hence anti- 20% of patients. With the classic form of the dis-
biotics should be administered at the resuscitation ease, the patient will also exhibit intermittent
stage after taking blood cultures. Cranial imaging hyperactivity. Post-exposure prophylaxis consists
should be performed before a lumbar puncture of a single dose of rabies immunoglobulin and 3-4
(at least when no previous imaging is available), doses of rabies vaccine. Patients have also been
but delaying antibiotics until this has all been treated by inducing a coma (Milwalkee protocol)
done could be catastrophic. with some success even without post-exposure
prophylaxis.
3. d—Freshwater swimming
6. e—Spongiform encephalopathy
Primary amoebic meningoencephalitis (PAM) is
caused by Naegleria fowleri found in warm bodies The clinical, EEG and MRI findings are typical
of freshwater, and although rare is nearly always of a spongiform encephalopathy most probably
fatal. The parasites enter the nervous system due to Creutzfeldt-Jakob disease. This prion
through the cribriform plate at the perforations disease can be transmitted via infected nervous
for the olfactory nerves. system tissue, including dura mater grafts, and
occasionally via growth hormone preparations
4. b—Facial nerve acquired from cadaver pituitary glands. CSF
is usually normal, but may show slightly ele-
Facial weakness may be the only neurological sign
vated protein, increased IgG, oligoclonal bands,
of Lyme disease, and may be bilateral. The neu-
and may contain a specific protein (14-3-3 pro-
rological deficits usually appear weeks after the
teinase inhibitor). Vascular causes are unlikely
initial rash. The facial palsy or optic neuritis that given the gradual deterioration and imaging
develops with CNS disease is characteristically
findings. Friedreich disease may produce some
associated with meningitis. B. burgdorferi is a spi-
dementia, but it is not a prominent part of the
rochete usually transmitted to humans through
clinical deterioration and usually affects youn-
tick bites. Another feature is erythema chronicum
ger patients.
migrans, an expanding reddish discoloration of
the skin that spreads away from the site of the bite 7. d—Listeria monocytogenes
as a ring of erythema; evolving over 3-4 weeks
then spontaneously clearing. If there is meningeal The presentation is highly suggestive of Listeria
involvement, high-dose intravenous antibiotics is monocytogenes meningitis. This infection com-
given for 10-14 days. monly develops in renal transplant recipients,
patients with chronic renal disease, immunosup-
5. e—Spreads retrogradely in peripheral nerves pressed persons, and occasionally in otherwise
unimpaired persons. It may also affect neonates.
Rabies virus is usually spread through the saliva of
This type of meningitis is not usually seen in
an infected animal (e.g. dogs, bats, skunks, foxes,

Neurosurgery Books Full


www.ketabpezeshki.com 66485438-66485457
360 PART III CRANIAL NEUROSURGERY

older children. It may on occasion lead to intrace- invasion. Later (days 4-9), the zone of cerebritis
rebral abscess formation. Third-generation ceph- expands, and necrosis develops, with pus forming
alosporins are inactive against Listeria, and at the center of the lesion. CT scanning reveals
ampicillin and gentamicin are recommended some ring enhancement with diffusion of contrast
therapy. Neither ampicillin nor penicillin alone material into the necrotic center. The early cap-
is bactericidal. sule stage (days 10-13) demonstrates the establish-
ment and maturation of a well-formed
8. e—Stereotactic needle aspiration collagenous capsule associated with a reduction
in the degree of cerebritis and some regression
Brain abscess are commonly due to hematological in the local edema. At the late capsule stage (day
(e.g. pneumonia, endocarditis, dental work) or 14 and beyond), there is continued maturation
direct local spread of infection (e.g. mastoiditis, of a thick capsule with extracapsular gliosis and
chronic otitis). They usually start from a micro- dense ring enhancement with little contrast diffu-
scopic focus of infection at the gray-white matter sion on CT scan. Capsule formation and ring
junction and takes the following course: early cer- enhancement on imaging studies are generally
ebritis 1-3 days, late cerebritis 4-9 days, early cap- thinner and less complete on the ventricular side
sule 10-13 days, late capsule >14 days. As the of the abscess. This situation is probably related
infection develops, a cerebritis appears, and sub- to the relatively poor vascularity of the deep white
sequently this focus of infection becomes necrotic matter and reduced migration of fibroblasts into
and liquefies. Around the enlarging abscess, there the area. This thinner area of capsule predisposes
is usually a disproportionately large area of to ventricular rupture of the abscess.
edema. Mature abscess collagen capsule thinner
on ventricular side, presence of dimple or small 10. c—Image-guided aspiration of abscess then
evagination for ring enhancing lesion should start intravenous antibiotics
suspect abscess but this is not always distinguish-
able. Patient commonly present with headache, Image-guided craniotomy for excision of brain
seizures, and focal neurological deficit. Strepto- abscess with its capsule has a lower recurrence
coccal bacteria occur in more than half of all brain rate compared to aspiration methods. Excision
abscesses; Staphylococcus aureus most often occurs of brain abscesses is useful in: large (more than
in patients who have had penetrating head 2.5 cm) superficial abscesses, multi-loculated
wounds or have undergone neurosurgical proce- abscesses, failure of resolution after several aspi-
dures. Enteric bacteria (eg, Escherichia coli, Pro- rations; some posterior fossa lesions; some fungal
teus, and Pseudomonas) account for twice as abscesses; post-traumatic abscesses with retained
many abscesses as S. aureus. The important differ- bone fragments or foreign bodies; and gas-
entials of a ring enhancing brain lesion include containing abscesses, usually signifying the pres-
primary tumor, metastasis, and abscess (although ence of an associated CSF fistula. For surgical
other causes are demyelination, maturing hema- excision of a brain abscess that has failed to
toma, and radiation necrosis). Due to the high respond to aspiration and antimicrobial therapy
mortality associated with intraventricular rupture or is in a particularly dangerous location, such
of a brain abscess, emergency MRI should be per- as a posterior fossa abscess associated with edema,
formed to determine if the lesion is diffusion mass effect, and impending or actual obstructive
restricting (bright on DWI, dark on ADC) and hydrocephalus, image-guided craniotomy is
thus likely to require emergency neurosurgical favored to excise the lesion, relieve the mass effect
drainage. Although uncommon, diffusion restric- on the brain stem, and reduce the chances of
tion has been reported in metastases and glioblas- recurrence. Excision is not the procedure of
tomas and other modalities such as dynamic choice in the cerebritis stage or in deep-seated
contrast-enhanced perfusion MRI may help dis- brain abscesses, especially in eloquent areas. Dur-
tinguish between brain abscess and tumor; ing the stage of cerebritis, antimicrobials are used
abscesses have a lower relative cerebral blood vol- with serial neurologic examinations and imaging
ume in their enhancing rim than gliomas. studies to guide therapy. In most other settings,
however, surgical intervention is undertaken. In
Image with permission from Adam A, et al. (Eds.), the obtunded patient with a severe neurologic
Grainger & Allison's Diagnostic Radiology, 6th ed.,
Elsevier, Churchill Livingstone, 2014. deficit and an encapsulated lesion, surgery for
diagnosis and decompression is carried out emer-
9. b—Days 4-9 gently. If multiple lesions are discovered, those
greater than 2.5 cm in diameter should be aspi-
Early cerebritis (days 1-3) is a poorly circum- rated. If all lesions are less than 2.5 cm and do
scribed lesion characterized by acute inflamma- not exert mass effect, the largest or most accessi-
tion and cerebral edema associated with bacterial ble one should be aspirated for culture.

Neurosurgery Books Full


www.ketabpezeshki.com 66485438-66485457
25 CRANIAL INFECTION 361

11. d—Toxoplasma gondii sinuses, resulting in septic thrombophlebitis.


The treatment consists of antibiotic therapy, usu-
The timing of this presentation suggests that he ally in combination with surgical drainage to pre-
has now developed AIDS. While fungal abscesses vent progression to subdural empyema. Drainage
develop with unusual frequency in patients with of the epidural abscess can be done either by a
AIDS, T. gondii is considerably more common minimally invasive approach or by a craniotomy
than fungi as the cause of abscess formation. Cere- with removal of infected bone. A higher rate of
bral toxoplasmosis usually presents as multiple recurrence was reported after burr hole drainage.
ring-enhancing lesions in the basal ganglia, thala- In preceding sinusitis, a combination of neurosur-
mus, or corticomedullary junction. Note the gical and ENT approaches are needed.
“eccentric target sign,” which is shown best in
the right parahippocampal lesion. Although this Image with permission from Adam A, et al. (Eds.),
sign is not sensitive, it is fairly specific for toxoplas- Grainger & Allison's Diagnostic Radiology, 6th ed., Else-
vier, Churchill Livingstone, 2014.
mosis. The combination of sulfadiazine and pyri-
methamine is proper treatment for T. gondii 14. c—External ventricular drain
infection. Neurosurgical drainage of the lesions
is usually not indicated. The fungi that do produce Axial CT shows dilated lateral ventricles that con-
abscesses in persons with AIDS are most often tain intermediate attenuation debris suggestive of
Cryptococcus, Candida, Mucor, and Aspergillus. pyogenic ventriculitis, with a rim of low-
Tuberculosis meningitis and abscesses are also com- attenuation interstitial edema surrounding the
mon in immunocompromised individuals. ventricles. The intraventricular rupture of a brain
abscess occurs with progressive growth of the
Image with permission from Aiken AH. Central nervous
system infection. Neuroimaging Clin N Am 2010;20
lesion. As the pus increases, the abscess expands
(4):557-80. toward the ventricle and may rupture, resulting
in the sudden, catastrophic deterioration of the
12. b—Aspergillus patient. The diagnosis is confirmed by the pres-
ence of hydrocephalus and enhancement of the
Fungal otitis exerna is usually caused by Aspergil- ventricular walls. Immediate ventricular drainage,
lus (black exudate), followed by Candida (cheesy intraventricular instillation of antibiotics, evacua-
white exudate) and Actinomyces. Although Asper- tion of the remaining abscess, and systemic anti-
gillus is the most common cause of fungal biotic therapy are still associated with a
abscesses, it is a relatively uncommon cause of management mortality rate of greater than 80%
fungal meningitis or meningoencephalitis.
Image with permission from Adam A, et al. (Eds.),
13. c—Epidural abscess Grainger & Allison's Diagnostic Radiology, 6th ed., Else-
vier, Churchill Livingstone, 2014.
There is left subfrontal epidural fluid collection 15. d—Taenia solium
surrounded by an enhancing rim of thickened
dura, which differentiates cranial epidural abscess MRI shows extensive parenchymal and subarach-
from a sterile collection. They arise between the noid cysticercosis. Most of the lesions are in the
skull and the dura, usually in the frontal region, as vesicular stage showing thin-walled cysts with lit-
a result of contiguous spread of infections from tle or no enhancement and a scolex (worm head)
adjacent structures, such as the paranasal sinuses in the center of the cyst. Cysticercosis is produced
or the mastoid cells. It is associated with paranasal by the larval form (cysticercus) of the pork tape-
sinusitis, cranial osteomyelitis, or head trauma or worm, Taenia solium. This is the most common
occurs post-surgery. Risk factors for intracranial neurological infection throughout the world,
epidural abscess include prior craniotomy, head occurring most commonly in South America,
injury, sinusitis, otitis media, and mastoiditis. Southeast Asia, and Africa. It is transmitted by
The bacteriology of these lesions is analogous fecal-oral contact; tapeworm eggs hatch in the
to that of brain abscess. Generally, the epidural human GI tract, invade the bowel mucosa, and
abscess is an indolent lesion when compared to migrate throughout the body, particularly into
subdural empyema. Neurologic symptoms and CNS, muscle, eye, and subcutaneous tissues.
complications are rare, because the dura mater Cysticercal infection of muscles produces a non-
protects the brain parenchyma, and the tight specific myositis. Brain involvement may lead to
adherence of the dura to the skull limits the seizures. The lesions in the brain may calcify
spread. The most common presenting symptoms and often appear as multiple small cysts spread
are fever, headache, and neurologic signs. throughout the cerebrum. Treatment of neuro-
Untreated, this parameningeal focus can extend cysticercosis depends on whether the cyst is dead
intracranially and involve the dural venous (antiseizure medication) or viable. Viable cysts in

Neurosurgery Books Full


www.ketabpezeshki.com 66485438-66485457
362 PART III CRANIAL NEUROSURGERY

the presence of vasculitis/arachnoiditis/encepha- Image with permission from Adam A, et al. (Eds.),
litis, immunosuppressant therapy is usually given Grainger & Allison's Diagnostic Radiology, 6th ed.,
Elsevier, Churchill Livingstone, 2014.
before anticystercal drugs (albendazole and prazi-
quantel). Neurosurgical management may be
indicated when intraventricular or racemose cysts
cause hydrocephalus. ANSWERS 19–22
Image with permission from Adam A, et al. (Eds.), Additional answers 19–22 available on
Grainger & Allison's Diagnostic Radiology, 6th ed., Else- ExpertConsult.com
vier, Churchill Livingstone, 2014.

16. c—Highly active antiretroviral therapy


(HAART)

Progressive multifocal leukoencephalopathy is EMI ANSWERS


caused by the JC virus, which is a double-stranded
DNA virus transmitted via respiratory droplets. 23. 1—k, Staphylococcus aureus; 2—i, Pseudomonas;
MRI demonstrates widespread hyperintense sig- 3—m, Viridans streptococci
nal change in the left corona radiate with typical
involvement of the subcortical U fibers, and DWI 24. 1—g, JC Virus. Progressive multifocal leu-
shows a hyperintense edge of the lesion medially koencephalopathy is a progressive leukoen-
reflecting zones of active demyelination. The cephalopathy seen in immunocompromised
prognosis is poor, but HAART has been known patients, most notably those with AIDS.
to be effective in improving survival. Lesions in the occipital or parietal regions
can result in visual complaints or “alien hand
Image with permission from Adam A, et al. (Eds.), syndrome.” 2—d, Herpes simplex virus. Her-
Grainger & Allison's Diagnostic Radiology, 6th ed., Else- pes simplex virus (HSV) encephalitis is the
vier, Churchill Livingstone, 2014.
most common form of sporadic encephalitis
17. c—Perform a lumbar puncture and include in the United States. Patients may present
CSF for Epstein-Barr virus (EBV) PCR in with acute onset of seizures or with a sub-
tests ordered. acute course characterized by deficits refer-
able to temporal lobe structures, such as
The most common etiologies of rim-enhancing amnesia, aphasia, or psychosis. Motor deficits
brain lesions in AIDS patients are primary CNS also often occur. 3—h, Measles virus. Sub-
lymphoma and Toxoplasma gondii infection. Other acute sclerosing panencephalitis (SSPE) usu-
etiologies such as bacterial or fungal abscess are ally develops in children and is rarely seen
also possible. CSF EBV PCR test is highly sensi- after the age of 18. Most affected children
tive and specific for primary CNS lymphoma. have had a bout of measles (rubeola) that
Because there is no mass effect, it is safe to do a occurred before they were 2 years old. SSPE
lumbar puncture, so a ventricular CSF aspiration may not appear for as long as 6-8 years after
is not necessary. A cerebral angiogram should be the episode of measles. Death usually occurs
done if you suspect an aneurysm or vascular mal- within 1-3 years after the onset of symptoms.
formation. These are unlikely in this case. There SSPE produces a CSF pattern similar to that
is no reason to stop all antiretroviral therapy. seen with multiple sclerosis, whose features
Intravenous acyclovir is used to treat herpes include an increase in the γ-globulin fraction
encephalitis, which is unlikely in this case. and the presence of oligoclonal bands. The
measles virus appears to be directly responsi-
18. b—Echinococcus ble for this demyelinating disease, and the
oligoclonal bands that appear in the CSF
Echinococcosis is usually acquired by ingesting include a substantial proportion of measles-
material contaminated with fecal matter from specific antibody.
sheep or dogs. Children are more likely to
develop cerebral lesions than adults, but people 25. 1—i, Tabes dorsalis. Treponema pallidum
at any age may develop this encephalic hydatido- causes syphilis, and neurosyphilis may take
sis, which entails the development of a major cyst the form of general paresis (dementia,
with multiple compartments in which smaller personality change, myoclonus, rhombence-
cysts are evident. This hydatid cyst of the brain phalitis) or tabes dorsalis (leptomeningitis).
behaves like a tumor and may become massive The posterior columns of the spinal cord
enough to cause focal deficits.

Neurosurgery Books Full


www.ketabpezeshki.com 66485438-66485457
25 CRANIAL INFECTION 363

and the dorsal root ganglia are hit especially meningitis. In HIV-infected individuals, it
hard by degenerative changes associated with may produce a more virulent encephalitis
this form of neurosyphilis. The bladder is associated with status epilepticus. 3—k,
usually hypotonic (flaccid), and megacolon Whipple disease. Treponema whippeli can
may develop. Patients with tabes dorsalis cause a rare multisystem disorder with symp-
routinely exhibit abnormal (Argyll Robert- toms of bowel and CNS infection. Central
son) pupils and optic atrophy. 2—c, Barto- nervous system (CNS) infection may produce
nella hensae. Cat-scratch disease produces a seizures, myoclonus, ataxia, supra-nuclear
regional adenitis, frequently involving epi- gaze disturbances, hypothalamic dysfunction,
trochlear nodes caused by scratches on the and dementia. Oculomasticatory myorhyth-
patient's arm from an animal infected with mia (pendular convergence movements of
Bartonella hensae. CNS involvement occurs the eyes in association with contractions of
in only 5% of cases. In immunocompetent the masticatory muscles) may occur and is
hosts, it may produce a self-limited aseptic considered pathognomonic.

Neurosurgery Books Full


www.ketabpezeshki.com 66485438-66485457
PART IV
SPINAL NEUROSURGERY
CHAPTER 26

SPINE: GENERAL PRINCIPLES


SINGLE BEST ANSWER (SBA) QUESTIONS
1. The attachments of the ligamentum flavum d. Vertebral artery passes through C1 fora-
are best described as: men transversarium
a. The body of the axis to the sacrum bind- e. Possesses concave superior facets and
ing the anterior aspect of the vertebral convex inferior facets
bodies and intervertebral discs together
b. Extending from the axis to the sacrum 4. Which one of the following statements is most
c. The ventral aspect of the superior lamina accurate regarding the C2 vertebra (axis)?
and the dorsal aspect of the inferior lamina a. Atlantodens interval is normally <11 mm
d. The spinous processes of adjacent vertebrae in children
e. Dorsal to the spinous process and in b. Superior articular facet is located postero-
continuity with the ligamentum nuchae laterally to the spinal canal
c. Inferior articular facet is located antero-
2. The transverse sinus is located in close laterally to the spinal canal
proximity to which one of the landmarks d. Atlantodens interval is normally >3 mm
below? in adults
e. Pars interarticularis may fracture in
hyperflexion or hyperextension

1
5. Which one of the following best describes
the uncovertebral (Lushka's) joints in subax-
2 ial cervical spine?
a. Betweensuperiorandinferiorarticularfacets
3
b. Between vertebra below and inferolateral
4
uncinate processes of the vertebra above
c. Between vertebra above and the supero-
5 lateral uncinate processes of the vertebral
6
body below
d. Between transverse processes of adjacent
vertebrae
e. Between spinous processes of adjacent
vertebrae

3. Which one of the following statements is 6. Which one of the following is NOT a unique
most accurate regarding the C1 vertebra? anatomic feature of the C7 vertebra?
a. Possesses a groove for the vertebral artery a. Inferior articular process of C7 is oriented
on its anterior-superior aspect in a relatively perpendicular direction
b. It is weakest where the anterior and poste- b. It has the thinnest lateral mass in the
rior arches connect to the lateral masses cervical spine
c. Load bearing occurs through the anterior c. Its transverse process only possesses a pos-
tubercle terior tubercle
364
Neurosurgery Books Full
www.ketabpezeshki.com 66485438-66485457
26 SPINE: GENERAL PRINCIPLES 365

d. Its foramen transversarium is the entry 11. Which one of the following best describes
point for the vertebral artery Batson's plexus?
e. It has a long, non-bifid spinous process a. System of valveless veins located within
the vertebral canal and body
7. Orientation of facet joint planes in the subaxial b. Sympathetic plexus located along the
cervical spine are which one of the following? anterior prevertebral tissues in the region
a. 75° in the sagittal plane and 30° in the of L5 vertebral body and L5/S1 disc
coronal plane c. Paired segmental arteries from L1 to L4
b. 60° in the sagittal plane and 15° in the vertebrae arising from the aorta
coronal plane d. Parasympathetic plexus located along
c. 45° in the sagittal plane and 0° in the anterior prevertebral tissues in the
the coronal plane region of L4 vertebral body and L3/L4
d. 30° in the sagittal plane and 15° in the disc
coronal plane e. Nervous supply to the bladder
e. 15° in the sagittal plane and 30° in the
coronal plane 12. Vertebral artery V2 segment is best described
as which one of the following?
8. Which one of the following best describes a. Runs from transverse foramen of C6 to
the constituents of the intervertebral disc? transverse foramen of C2
a. Annulus fibrosus type I collagen, nucleus b. Curves around the C1 lateral mass
pulposus type IV collagen c. Runs from transverse foramen of C2 to
b. Annulus fibrosus type I collagen, nucleus the dura
pulposus type II collagen d. Ascends anterior to the roots of the
c. Annulus fibrosus type II collagen, nucleus hypoglossal nerve
pulposus type II collagen e. Runs from the posterior atlanto-occipital
d. Annulus fibrosus type IV collagen, membrane to the origin of PICA
nucleus pulposus type II collagen
e. Annulus fibrosus type IV collagen, 13. Panjabi & White's definition of spinal stabil-
nucleus pulposus type III collagen ity is most accurately described by which one
of the following?
9. Site of entry for thoracic pedicle screw is best a. The loss of the ability of the spine under
described as which one of the following? physiological loads to maintain relation-
a. Where the facet joint and transverse ships between vertebrae to prevent pain
process intersect or deformity
b. Where the pars interarticularis and lam- b. The loss of the ability of the spine
ina intersect under physiological loads to maintain
c. Where the superior facet and lamina relationships between vertebrae to
intersect prevent pain
d. Where the inferior facet and lamina c. The loss of the ability of the spine under
intersect physiological loads to maintain relation-
e. Where the transverse process and lamina ships between vertebrae to prevent pain,
intersect deformity or neurological injury
d. The loss of the ability of the spine under
10. Which one of the following labels denotes physiological loads to maintain relation-
the pedicle in this lumbar vertebra? ships between vertebrae to prevent neuro-
logical injury
b a
e. The loss of the ability of the spine under
f physiological loads to maintain relation-
ships between vertebrae to prevent pain
or neurological injury
e
14. Load-sharing concept of spinal biomechanics
holds that in the normal lumbar spine:
c
a. Approximately 80% of axial load is carried
by the anterior spinal column and the
d
remaining 20% is transmitted through
the posterior spinal column

Neurosurgery Books Full


www.ketabpezeshki.com 66485438-66485457
366 PART IV SPINAL NEUROSURGERY

b. Approximately 70% of axial load is carried d. Increase the rate of spinal arthrodesis
by the anterior spinal column and the e. Avoid the need for pedicle screw placement
remaining 30% is transmitted through
the posterior spinal column 18. The spinal construct shown below would
c. Approximately 60% of axial load is carried most likely have been used for which one
by the anterior spinal column and the of the following
remaining 40% is transmitted through
the posterior spinal column
d. Approximately 50% of axial load is carried
by the anterior spinal column and the
remaining 50% is transmitted through
the posterior spinal column
e. Approximately 40% of axial load is carried
by the anterior spinal column and the
remaining 60% is transmitted through
the posterior spinal column

15. Placement of lumbar pedicle screws at L3


and L4 level should use which one of the fol-
lowing entry point?
a. The site where the transverse process
a. Basilar invagination in rheumatoid arthritis
joins the superior articular process just lat-
b. C3 vertebral body metastasis
eral to the pars interarticularis
c. Hangman fracture
b. The site where the lamina joins the supe-
d. Jefferson fracture
rior articular process just lateral to the
e. Type II odontoid peg fracture
pars interarticularis
c. The site where the transverse process
19. The normal anatomical variant ponticulus
joins the inferior articular process just lat-
posticus may increase the risk of vertebral
eral to the pars interarticularis
artery injury during which one of the
d. The site where the lamina joins the infe-
following?
rior articular process just lateral to the
a. C1 lateral mass screw placement
pars interarticularis
b. C2 pars screw placement
e. The pars interarticularis
c. C2 pedicle screw placement
d. C2 translaminar screw placement
16. Which one of the following is most accurate
e. C1-C2 transarticular screws
regarding placement of thoracic pedicle screws?
a. Varies widely depending on level
b. Straight-head screw trajectory is parallel QUESTIONS 20–28
to the facet joint
c. Anatomic screw trajectory is along the Additional questions 20–28 available on
long axis of the pedicle ExpertConsult.com
d. Entry point is where the transverse pro-
cess joins the superior articular process
just medial to the pars interarticularis
e. The pars interarticularis is the LEAST
useful landmark EXTENDED MATCHING ITEM (EMI)
QUESTIONS
17. The primary goal of dynamic stabilization
techniques in the spine is best described as 29. For each of the following descriptions, select
which one of the following? the most appropriate answers from the image
a. Produce less stress on adjacent vertebral below. Each answer may be used once, more
segments than once or not at all:
b. Reduce implant failure
c. Reduce motion between segments com-
pared to currently available constructs

Neurosurgery Books Full


www.ketabpezeshki.com 66485438-66485457
26 SPINE: GENERAL PRINCIPLES 367

5 6 78
1. Dentate ligament
2. Dorsal root ganglion
3. Ramus communicans
4. Dorsal ramus
5. Ventral root of spinal nerve
4
9
3
2 10

1 11
12

13

18 17 16 15 14

30. For each of the following descriptions, select the most appropriate answers from the image below.
Each answer may be used once, more than once or not at all:

1 8
2 19
3 9
10
4
11
20
5 12

6 13

14

15

16

17
18

1. Apical ligament e. Lateral reticulospinal


2. Tectorial membrane f. Vestibulospinal tract
3. Transverse ligament g. Lateral corticospinal
4. Anterior atlanto-occipital membrane h. Septomarginal fasciculus
5. Alar ligament i. Tract of Lissauer
For each of the following descriptions, select the
most appropriate answers from the list above.
31. Descending tracts: Each answer may be used once, more than once
a. Anterior Corticospinal or not at all:
b. Rubrospinal 1. Fine skilled movements in hand, foot, and
c. Tectospinal lower limb.
d. Medial Reticulospinal

Neurosurgery Books Full


www.ketabpezeshki.com 66485438-66485457
368 PART IV SPINAL NEUROSURGERY

2. Controlling movements of the hands and ipsilateral anterior lobe of the cerebellum.
digits by facilitating flexor muscles and Relay touch, pressure and proprioception
inhibiting extensors in the upper limb. from ipsilateral upper trunk and upper limb.
3. Head and neck movements while maintain- 3. Reflex movement of eyes, head, and upper
ing gaze fixation on an object. body towards painful stimulus.
4. Autonomic and reflex responses (cardiac,
32. Ascending tracts: endocrine) to pain.
a. Cuneocerebellar tract
b. Dorsal spinocerebellar tract 33. Spinal laminae and nuclei:
c. Fasciculus cuneatus a. Lamina I
d. Fasciculus gracilis b. Lamina II/III
e. Rostral spinocerebellar tract c. Lamina III/IV/V
f. Spinohypothalamic tract d. Lamina VI
g. Spinomesencephalic tract e. Lamina VII
h. Spinoreticular tract f. Lamina VIIII
i. Spinotectal tract g. Lamina IX
j. Spinothalamic tract h. Lamina X
k. Ventral spinocerebellar tract For each of the following descriptions, select the
For each of the following descriptions, select the most appropriate answers from the list above.
most appropriate answers from the list above. Each answer may be used once, more than once
Each answer may be used once, more than once or not at all:
or not at all: 1. Contains Onuf's nucleus in sacral region
1. First-order neurons synapse in nucleus dor- 2. Contains sympathetic outflow and Clarke's
salis (Clarke's column) and second-order column (nucleus dorsalis)
neurons ascend ipsilaterally to the inferior 3. Substantia gelatinosa
cerebellar peduncle (restiform body), where
third-order mossy fibers project to cerebel-
lar vermis. Relays touch, pressure and pro-
prioception from ipsilateral lower trunk
and lower limb. QUESTIONS 34–35
2. First-order neurons ascend synapse in the
Additional questions 34–35 available on
accessory cuneate nucleus and second-order ExpertConsult.com
neurons ascend ipsilaterally to the inferior
cerebellar peduncle (restiform body), where
third-order mossy fibers project to

SBA ANSWERS
1. c—The ventral aspect of the superior lamina and the dorsal aspect of the inferior lamina

Ligament Comments
Anterior longitudinal Runs from body of the axis (C2) to the sacrum binding the anterior aspect of the
ligament (ALL) vertebral bodies and intervertebral discs together
Posterior longitudinal Extends from the axis to the sacrum binding the posterior aspect of the vertebral
ligament (PLL) bodies and intervertebral discs together
Ligamentum flavum Attaches to the ventral aspect of the superior lamina and the dorsal aspect of the
inferior lamina—segments run from C2 to first segment of the sacrum. Laterally,
the ligamentum flavum is in continuity with the facet capsules
Interspinous ligament Lies between spinous processes
Supraspinous ligament Lies dorsal to spinous process and is in continuity with the ligamentum nuchae

Neurosurgery Books Full


www.ketabpezeshki.com 66485438-66485457
26 SPINE: GENERAL PRINCIPLES 369

2. a—1: inion (external occipital protuberance)

The transverse sinus is located in close proximity to the inion, and below the superficial nuchal lines
extending laterally. The occipital area in the midline below the inion is the ideal location for screw inser-
tion for occipitocervical fixation as it is the thickest portion of the occiput and below the transverse sinus.

1 Inion (external occipital protuberance) with superior nuchal lines extending laterally
2 C1 transverse process
3 C2 Odontoid peg
4 Foramen magnum: posterior border (opisthion), anterior border (basion/clivus; not shown)
5 Inferior nuchal line
6 C3 lateral mass

Image adapted with permission from Devlin VJ. Spine Secrets Plus, 2nd ed., Elsevier, Mosby, 2012.

3. b—Weakest where the lateral mass connects 4. e—Pars interarticularis may fracture in
to anterior and posterior arches hyperflexion or hyperextension

The ring-like atlas (C1) is unique because during The axis (C2) receives its name from its odontoid
development its body fuses with the axis (C2) to process (dens), which forms the axis of rotation
form the odontoid process. Thus, it is composed in the atlantoaxial joint. The dens has an anterior
of two thick, load-bearing lateral masses, with hyaline articular surface for the anterior arch of
concave superior and inferior articular facets. C1, and a posterior articular surface articulates
There is a short anterior arch with a tubercle for the transverse ligament. Relative to the spinal
and articular facet on its posterior aspect for artic- canal, the superior articular processes are located
ulation with the dens (odontoid process). anterolaterally while the inferior articular pro-
The posterior arch is longer and curved, and cesses are located posterolaterally—they are con-
has a grove on its posterior-superior surface for nected by the pars interarticularis. Hyperflexion
the vertebral artery. The transverse process or hyperextension injuries may subject C2 to
of the atlas has a single tubercle, which protrudes shear stresses, resulting in a fracture through
laterally and can be palpated in the space between the pars region (hangman's fracture). The C2
the tip of the mastoid process and the ramus pedicle is a narrow area between the vertebral
of the mandible. It is weakest at the junction of body and the pars. The atlantodens interval
the anterior and posterior arches with the lateral (ADI) is the space between the hyaline cartilage
masses explaining the nature of Jefferson (burst) surfaces of the anterior tubercle of the C1 and
fracture of C1. the anterior dens; it is <3 mm adults and
5 mm children.
Anterior tubercle Odontoid process

Odontoid Anterior arch


process of C2

Transverse Lamina
process
C2 facet of
C1-2 joint

Body
Bifid spinous of axis
Foramen process
transversarium
C2 facet Foramen
of C2-3 joint transversarium
Posterior Transverse
Image with permission from Devlin VJ. Spine Secrets
arch ligament
Plus, 2nd ed., Elsevier, Mosby, 2012.
Image with permission from Devlin VJ. Spine Secrets
Plus, 2nd ed., Elsevier, Mosby, 2012.

Neurosurgery Books Full


www.ketabpezeshki.com 66485438-66485457
370 PART IV SPINAL NEUROSURGERY

5. c—Between vertebra above and the supero- 7. c—45° in the sagittal plane and 0° in the
lateral uncinate processes of the vertebral coronal plane
body below
The orientation of the facet joints is a major factor
Typical cervical vertebra (C3-C6) have an anterior in the range of motion of the cervical spine.
body and a posterior arch formed by lamina and Approximately 50% of cervical flexion-extension
pedicles. The lamina blends into the lateral masses, occurs at the occiput-C1 level. Approximately
which comprises the bony region between the 50% of cervical rotation occurs at the C1-C2 level.
superior and inferior articular facets/processes. Lesser amounts of flexion-extension, rotation, and
The uncovertebral (Lushka's) joints are formed lateral bending occur segmentally between C2 and
by uncinate processes that extend upward from C7 where facet joints are oriented 45° in the sag-
the lateral margin of the superior surface of the ver- ittal plane and 0° in the coronal plane. These are
tebral body and limit lateral flexion/guide flexion- the most horizontally oriented regional facet
extension. Spinal nerves exit via the intervertebral joints in the spinal column. The orientation of
foramina formed between adjacent pedicles, facet these facets allows flexion-extension (greatest at
joint and posterior aspect of the vertebral body. the C5-C6 and C6-C7), lateral flexion, and rota-
The transverse processes of the lower cervical spine tion of the lower cervical spine. Laxity of the joint
are directed anterolaterally and composed of an capsule permits sliding motion to occur and
anterior costal element and a posterior transverse explains why unilateral or bilateral dislocation
element. The transverse foramen, located at the without fracture may occur.
base of the transverse process, permits passage of
the vertebral artery. The spinous process originates 8. b—Annulus fibrosus type I collagen, nucleus
in the midsagittal plane at the junction of the lamina pulposus type II collagen
and is bifid between C2 and C6.
Each intervertebral disc is composed of a central
Neurocentral lips
gel-like nucleus pulposus surrounded by a
Foramen Anterior
tubercle
peripheral fibrocartilaginous annulus fibrosus.
transversarium
The annulus fibrosus (type I collagen) attaches
Posterior to the cartilaginous endplates via collagen fibers,
tubercle of
transverse which run obliquely at a 30° angle to the surface
process of the vertebral body and in a direction opposite
Lateral mass
to the annular fibers of the adjacent layer. The
nucleus pulposus (glycosaminoglycans and type
Intervetebral II collagen) can bind large amounts of water. In
foramen
a normal healthy disc, loads acting on the disc
are transferred to the annulus by intradiscal pres-
Bifid spinous
sure generated by the nucleus. With aging, then
process nucleus binds less water and becomes dehydrated
Image with permission from Devlin VJ. Spine Secrets resulting in increased loading of the annulus.
Plus, 2nd ed., Elsevier, Mosby, 2012. Fissuring and disruption of the annulus predis-
poses to herniation of nuclear material through it.

6. d—Its foramen transversarium is the entry 9. a—Where the facet joint and transverse pro-
point for the vertebral artery cess intersect

The unique anatomic features of the C7 vertebra The paired pedicles arise from the posterior-
reflect its location as the transitional vertebra at superior aspect of the vertebral bodies. The
the cervicothoracic junction: superior-inferior pedicle diameter is consistently
• Long non-bifid spinous process larger than the medial-lateral pedicle diameter.
• Its foramen transversarium usually contains Pedicle widths are narrowest at the T4-T6 levels,
vertebral veins only (vertebral artery enters with medial-lateral pedicle diameter increasing
at the C6 level) both above (T1-T3) and below this region. The
• The C7 transverse process is large in size medial pedicle wall is two to three times thicker
and possesses only a posterior tubercle than the lateral pedicle wall across all levels of
the thoracic spine. The medial angulation of the
• The C7 lateral mass is the thinnest lateral
pedicle axis decreases from T1 to T12. The site
mass in the cervical spine
for entry into the thoracic pedicle from a posterior
• The inferior articular process of C7 is ori- spinal approach is in the region where the facet
ented in a relatively perpendicular direction joint and transverse process intersect and varies
(like a thoracic facet joint) slightly, depending on the specific thoracic level.
Neurosurgery Books Full
www.ketabpezeshki.com 66485438-66485457
26 SPINE: GENERAL PRINCIPLES 371

10. c—C
Segment Course
Lumbar vertebral bodies are kidney-shaped with V1 Passes from the subclavian artery
the transverse diameter exceeding the antero- anterior to C7 to enter the C6 transverse
foramen
posterior diameter. An imaginary line passing
beneath the pedicles divides it into upper and V2 Enters the transverse foramina at C6 and
lower halves, with three posterior elements continues to the level of the atlas; lies
lateral to the vertebral body and in front
aligned above (superior facet, transverse process, of the lateral mass; gradually shifts to an
pedicle) and three below (lamina, inferior facet, anterior and medial position, thereby
spinous process). The pars interarticularis (con- placing the artery at greater risk of injury
cave lateral part of the lamina that connects the during anterior decompressive
superior and inferior articular facets) is located procedures at the upper cervical levels
along this imaginary dividing line (fracture here V3 Exits C2 transverse foramen laterally,
is termed spondylolysis). travels vertically to enter C1 transverse
1. Vertebral bodies increase in size from L1 foramen then exits horizontally to curve
to L5 around the C1 lateral mass, running
2. Pedicle width 6 mm at L1 and 18 mm at L5, medially along the cranial surface of the
posterior arch of C1 in its sulcus, before
and become more medially inclinated (12° passing through the atlanto-occipital
at L1 and 30° at L5). membrane and entering the foramen
3. Lateral border of pars aligns with medial magnum. The artery stays at least
border of pedicle at L1-L4, but middle of 12 mm lateral from midline of C1,
the pedicle at L5. making this a safe zone for dissection
4. The inferior articular facet of the vertebra V4 Intradural segment ascending anterior to
above is located posteromedially to the roots of XII, joining contralateral V4 to
superior articular facet of the vertebrae form basilar artery at lower boder of
pons. Gives off PICA and spinal arteries
below; facets are oriented in sagittal plane
(allowing flexion-extension only) except at
L5/S1 facet oriented coronal plane (allow-
ing rotation only).
5. The transverse processes are long and thin
Basilar artery
except at L5, where they are thick and A B Anterior aspect
broad and possess ligamentous attachments V4 V4
to the pelvis.
C1
Image adapted with permission from Devlin VJ. Spine V3
C2 V3
Secrets Plus, 2nd ed., Elsevier, Mosby, 2012.
Articular surface
of lateral mass Vertebral artery
11. a—Batson's plexus is a system of valveless C3 penetrating
atlanto-occipital
veins located within the spinal canal and V2 membrane
C4
around the vertebral body. It is an alternate and dura mater
route for venous drainage to the inferior vena C5
cava system. Because it is a valveless system,
any increase in abdominal pressure (e.g. prone C6 V1
positioning) can cause blood to flow preferen-
tially toward the spinal canal and surrounding C7
bony structures. Batson's plexus also serves as
a preferential pathway for metastatic tumor Subclavian artery
and infection spread to the lumbar spine.

12. a—Runs from transverse foramen of C6 to Image adapted with permission from Haines DE. Funda-
mental Neuroscience for Basic and Clinical Applica-
transverse foramen of C2 tions, 4th ed., Saunders, Elsevier, 2013.

The vertebral artery is the first branch off


the subclavian artery and provides the major 13. c—The loss of the ability of the spine under
blood supply to the cervical spinal cord, nerve physiological loads to maintain relationships
roots, and vertebrae. It can be divided into four between vertebrae to prevent pain, deformity
segments. or neurological injury.

Neurosurgery Books Full


www.ketabpezeshki.com 66485438-66485457
372 PART IV SPINAL NEUROSURGERY

Stabilization of the spine can be provided by spi- to achieve fixation when lamina are deficient. The
nal implants in the short-term, but long-term sta- disadvantages of pedicle screws include technical
bilization occurs only if vertebral bony fusion is challenges related to screw placement and the
successful. Non-fusion will ultimately result in potential for neurologic, vascular, and visceral
spinal implant failure. The surgical factors injury due to misplaced screws. Pedicle screws
include fusion technique, appropriate location may be: fixed head (monoaxial), mobile head (poly-
(anterior, posterior, or combined anterior and axial), or bolts (require a separate connector for
posterior column fusion), and use of appropriate attachment to the longitudinal member).
spinal implants to adequately support the spine
during bony fusion. Spinal implants: 16. c—Anatomic screw trajectory is along the
1. Immobilize spinal segments during the long axis of the pedicle
fusion process to increase the rate of suc-
cessful arthrodesis In the thoracic region, screw placement is initi-
2. Restore spinal stability lost due to pathologic ated at the lateral aspect of the pedicle. The ped-
processes (e.g. tumor, infection, fracture) icle entry site is determined by referencing the
3. Correct spinal deformities (e.g. scoliosis, transverse process, the superior articular process,
kyphosis, spondylolisthesis) and the pars interarticularis. Exact position of the
4. Maintain stability/prevent post-surgical entry site is adjusted depending on the specific
spinal deformity when extensive decom- level of the thoracic spine and whether the screw
pression of the neural elements is required trajectory is straight-ahead (perpendicular to ver-
(e.g. spinal stenosis). tebral body) or anatomic (along true axis of ped-
icle; angulated relative to vertebral body).
FURTHER READING
White AA, Panjabi MM. Clinical biomechanics of spine. Abnor- 17. a—Produce less stress on adjacent vertebral
mal flexion-extension mobility—Paradoxical motion. Kinemat- segments
ics of Spine Chap. 2; 89., Philadelphia, JB Lippincott, 1990.
Dynamic stabilization is a concept of placing
14. a—Approximately 80% of axial load is car- anchors (generally pedicle screws) into the spine
ried by the anterior spinal column and the and connecting these anchors with a flexible lon-
remaining 20% is transmitted through the gitudinal member (e.g. rod, cable, spring). The
posterior spinal column goal of this type of implant is to constrain but
not eliminate motion. Proponents of this concept
This 80/20 relationship between anterior and pos- believe this type of implant will produce less stress
terior columns is termed the load-sharing concept, on the adjacent spinal segments and may prevent
and it becomes clear that any anterior column some of the complications observed following
incompetence would require the entire axial load spinal fusion (e.g. adjacent-level degenerative
to pass through the posterior column (exceeding changes). Opponents worry that without concur-
the strength of any posterior spinal implant). In this rent spinal arthrodesis, these implants may loosen
situation, posterior spinal implants will fail by or fail prematurely and require revision surgery.
fatigue, permanent deformation, or implant migra- Currently, there are limited data to prove or dis-
tion through bone if used alone—hence it is critical prove the scientific utility of this concept.
to assess the need to reconstruct an incompetent
anterior spinal column. 18. a—Basilar invagination in rheumatoid
arthritis
15. a—The site where the transverse process
joins the superior articular process just lateral Image with permission from Devlin VJ. Spine Secrets
to the pars interarticularis Plus, 2nd ed., Elsevier, Mosby, 2012.

In the lumbar region, the entry site for screw place-


ment is located at the upslope where the transverse 19. a—C1 lateral mass screw placement
process joins the superior articular process just lat-
eral to the pars interarticularis. This site can be A bony bridge, the arcuate foramen (ponticulus
approximated by making a line along the midpoint posticus) may overlie the vertebral artery due to cal-
of the transverse process and a second line along the cification of the oblique atlanto-occipital ligaments
lateral border of the superior articular process. The and may be mistaken for the C1 lateral mass. The
crossing point of these two lines defines the entry entry point for C1 lateral mass screws is at the
site to the pedicle. Advantages of pedicle screws junction of the C1 lateral mass with the undersurface
include secure fixation, the ability to apply forces of the C1 posterior arch. The extensive venous
to both the anterior and posterior columns of the plexus in this region makes dissection challenging
spine from a posterior approach, and the capability and the C2 nerve root is in close proximity to the

Neurosurgery Books Full


www.ketabpezeshki.com 66485438-66485457
26 SPINE: GENERAL PRINCIPLES 373

screw entry point and must be retracted distally.


Screws are directed with 5-10° of convergence and ANSWERS 20–28
parallel to the C1 arch. Alternatively, C1 pedicle Additional answers 20–28 available on
screw placement has an entry point on the dorsal ExpertConsult.com
aspect of the posterior arch into the lateral mass.
With either technique, excessive superior C1 screw
angulation will violate the occiput-C1 joint. An ex-
cessively long C1 screw may potentially compro-
mise the internal carotid artery or hypoglossal nerve.

EMI ANSWERS
29. 1—2, 2—1, 3—14, 4—11, 5—18

Subdural space Periosteum

Dura mater Epidural space

Dorsal root of spinal nerve


Arachnoid membrane
Pia mater
Dentate ligament Subarachnoid space

Dorsal root ganglion Dorsal ramus


Spinal nerve

Ventral ramus of spinal nerve

Ramus communicans
Periosteum
Ventral root of spinal nerve Medulla spinalis

Dura mater

Image adapted with permission from Devlin VJ. Spine Secrets Plus, 2nd ed., Elsevier, Mosby, 2012.

30. 1—3, 2—9, 3—12, 4—1, 5—19

Attachment of tectorial
membrane
Anterior edge of
Anterior atlas-occipital membrane foramen magnum
Vertical cruciform ligament Alar ligament
Tectorial membrane
Apical ligament
Vertebral
artery
Anterior arch of atlas
Atlas
Deep tectorial membrane
Anterior atlas-axis membrane Transverse ligament

Anterior longitudinal ligament Origin of tectorial membrane

Posterior longitudinal ligament

Spinous process (axis)

Atlas

Transverse ligament
Dens (odontoid process)

Image adapted with permission from Devlin VJ. Spine Secrets Plus, 2nd ed., Elsevier, Mosby, 2012.
Neurosurgery Books Full
www.ketabpezeshki.com 66485438-66485457
374 PART IV SPINAL NEUROSURGERY

31. 1—g, Lateral corticospinal tract, 2—b, Rubrospinal tract, 3—f, Medial vestibulospinal tract

Anatomy Function
Direct (pyramidal) tracts
Fibers from primary motor cortex, secondary motor cortex (PMA, SMA), and somatosensory cortex
Cortex > Corona radiate > Genu/posterior limb of internal capsule > Midbrain crus (basis pedunculi) >
Pons > Medullary pyramids and decussation
Lateral Formed by decussating fibers (85%) Fine skilled movements hand, foot and
corticospinal lower limb
Sensory input modulation
Ventral Formed by non-decussating (15%) fibers Control of axial and proximal upper limb
corticospinal muscles
Corticonuclear Diverges from corticospinal tract at multiple Cranial nerve function
levels in brainstem to bilaterally innervate
cranial nerve somatic motor or
branchiomotor nuclei (but not directly to III,
IV, VI)
Indirect (extrapyramidal) tracts
Rubrospinal Leave red nucleus, decussate in anterior Controlling movements of the hands and
midbrain tegmentum, descends to digits by facilitating flexor muscles and
terminate in anterior horn motor neurons inhibiting extensors in the upper limb
Tectospinal Superior colliculus, decussate at level of red Important in reflexive movements of the
nucleus, descends through medulla in eyes, head, neck and upper trunk in
medial longitudinal fasciculus and response to visual, auditory and vestibular
continues in spinal cord to interneurons in stimuli
cervical/upper thoracic segments
Medial (pontine) From pontine reticular nuclei descending Excitatory to trunk/proximal limb extensor
Reticulospinal ipsilaterally to all levels of cord synapsing muscles and inhibit flexors. Acts in concert
with interneurons/motor neurons with the vestibulospinal tracts
Lateral (medullary) From medullary reticular nucleus Excitatory to trunk/proximal limb flexors
reticulospinal descending bilaterally to intermediate zone and inhibit extensors
grey matter Autonomic system output
Medial From medial vestibular nucleus, descending Head and neck movements while
Vestibulospinal ipsilaterally in MLF then continues in SC to maintaining gaze fixation on an object
tract cervical and upper thoracic interneurons or
alpha motor neurons
Lateral From lateral vestibular nucleus, descend Maintenance of posture and balance by
vestibulospinal ipsilaterally to all cord levels terminating on facilitating limb extensor (antigravity)
tract excitatory interneurons muscles and inhibiting limb flexors

32. 1—Posterior (dorsal) spinocerebellar tract, 2—Cuneocerebellar tract, 3—Spinotectal tract,


4—Spinohypothalamic tract

Ascending tract Anatomy Function


Dorsal column-Medial lemniscal
Fasciculus cuneatus* First-order neuron with cell body in DRG Fine touch, pressure, proprioception
(only present above supplying spinal cord level T6 and higher from upper trunk, upper limbs and neck
T6) ascends ipsilaterally to synapse with
second-order neurons in nucleus cuneatus
(medulla) which decussate (internal
arcuate fibers) then ascend as medial
lemniscus. They synapse with third-order
neurons in VPL thalamus which ascend in
posterior limb of internal capsule to
terminate in somatosensory cortex

Continued

Neurosurgery Books Full


www.ketabpezeshki.com 66485438-66485457
26 SPINE: GENERAL PRINCIPLES 375

Ascending tract Anatomy Function


Fasciculus gracilis* First-order neuron with cell body in DRG Fine touch, pressure, proprioception
supplying spinal cord levels below T6 from lower limbs and inferior trunk
ascends ipsilaterally to synapse with
second-order neurons in nucleus gracilis
(medulla) which decussate (internal
arcuate fibers) then ascend as medial
lemniscus. They synapse with third-order
neurons in VPL thalamus which ascend in
posterior limb of internal capsule to
terminate in somatosensory cortex
Somatosensory to cerebellum (subconscious)
Posterior (dorsal) First-order neurons synapse in nucleus Relay touch, pressure and proprioception
spinocerebellar tract dorsalis (Clarke's column) and second- from ipsilateral lower trunk and lower
order neurons ascend ipsilaterally to the limb. Posture and coordination of lower
inferior cerebellar peduncle (restiform limb movement
body), where third-order mossy fibers
project to cerebellar vermis
Cuneocerebellar tract First-order neurons ascend in fasciculus Relay touch, pressure and proprioception
cuneatus to synapse in the accessory from ipsilateral upper trunk and upper
cuneate nucleus and second-order limb. Coordinated movement of head
neurons ascend ipsilaterally to the inferior and upper limb
cerebellar peduncle (restiform body),
where third-order mossy fibers project to
ipsilateral anterior lobe of the cerebellum
Anterior (ventral) First-order neurons synapse in laminae Relays proprioception. Posture and
spinocerebellar tract V-VII and second-order neurons coordination of lower limb movement
decussated and ascend to the superior
cerebellar peduncle where third-order
mossy fibers project to cerebellar vermis
and decussate again to side of origin
Rostral First-order neurons synapse in laminae VII Movement of head and upper limb
spinocerebellar tract and second-order neurons ascend
ipsilaterally to inferior cerebellar peduncle
where third-order mossy fibers project to
cerebellum
Anterolateral
Anterior Second-order neuron from dorsal horn Light touch
spinothalamic tract* decussates to synapse with third-order
neuron in thalamus which terminates in
somatosensory cortex
Lateral spinothalamic Second-order neuron from dorsal horn Pain and temperature
tract* decussates to synapse with third-order
neuron in thalamus which terminates in
somatosensory cortex
Spinoreticular tract Second-order neurons remain ipsilateral Increased arousal following nociceptive,
(majority) or decussate (minority) and thermal, non-discriminatory (crude)
ascend bilaterally to reticular formation touch
where they synapse on reticulothalamic
fibers
Spinomesencephalic Terminate in periaqueductal grey and Descending pain inhibiting system
tract midbrain raphe nuclei Emotional component of pain
Terminate in parabranchial nucleus (then
amygdala)
Spinotectal tract Terminate in superior colliculus Reflex movement of eyes, head, and
upper body towards painful stimulus
Spinohypothalamic Terminate in hypothalamus where Autonomic and reflex responses (e.g.
synapse with neurons that give rise to cardiac, endocrine) to pain
hypothalamospinal tract

*Pathways transmitting consciously perceived stimuli.

Neurosurgery Books Full


www.ketabpezeshki.com 66485438-66485457
376 PART IV SPINAL NEUROSURGERY

33. 1—g, Lamina IX, 2—e, Lamina VII, 3—b, Lamina II/III

Rexed Lamina Description


Lamina I Substantia marginalis (marginal nucleus; receives/relays pain and temperature from
Lissauer's tract)
Lamina II/III Substantia gelatinosa (first-order neurons of spinothalamic tract synapse)
Lamina III/IV/V Nucleus proprius (first-order neurons of spinothalamic tract synapse)
Lamina VI Base of the dorsal horn
Lamina VII Intermediolateral nucleus (T1-L2; sympathetic output) and nucleus dorsalis (Clarke's
column; proprioception via spinocerebellar tract)
Lamina VIII Motor interneurons
Lamina IX Lateral and medial motor neurons; cervical phrenic and spinal accessory nuclei; sacral
Onuf's nucleus (S2 level; pudendal nerve origin; micturition, defecation)
Lamina X Substantia gelatinosa centralis

ANSWERS 34–35

Additional answers 34–35 available on


ExpertConsult.com

Neurosurgery Books Full


www.ketabpezeshki.com 66485438-66485457
CHAPTER 27

SCOLIOSIS AND SPINAL DEFORMITY


SINGLE BEST ANSWER (SBA) QUESTIONS
1. Which one of the following statements about 3. Which one of the following statements
angle X is LEAST accurate? regarding management of neuromuscular
scoliosis is LEAST accurate?
a. Neuromuscular curves do not respond
well to orthotic treatment, and spinal sur-
gery is frequently required
b. Observation is reasonable for patients with
small curves (30°)
c. Surgery is not recommended before the
age of 5
X
d. Earlier surgical treatment is advised for
patients with Duchenne’s muscular
dystrophy (when curves reach 20°)
e. Curves up to 90° are most commonly
treated with posterior spinal instrumenta-
tion and fusion

4. Which one of the following statements about


infantile idiopathic scoliosis is LEAST
accurate?
a. Idiopathic adolescent curves are com-
monly right thoracic, whereas infantile
scoliosis curves are left sided
a. Only applies to coronal plane deformity b. Genetic testing for scoliosis curve
b. Requires identification of proximal and progression is currently not available
distal end vertebrae c. Infantile scoliosis commonly affects girls,
c. The largest angle defines the major (struc- whereas adolescent idiopathic scoliosis
tural) scoliosis curve predominantly affects boys
d. Can be reduced in a non-structural curve d. Infantile scoliosis is associated with
by side bending plagiocephaly
e. Is a reflection of the sagittal balance e. A rib vertebral angle (RVA) of >20°
is associated with curve progression
2. Which one of the following statements
regarding development of scoliosis in patients 5. Which one of the following statements
with neuromuscular disorders is most accurate? regarding juvenile idiopathic scoliosis is
a. Neuromuscular curves tend to be longer LEAST accurate?
and involve more vertebrae than a. It shows increasing female predominance
idiopathic scoliosis curves with older ages
b. Neuromuscular curves form due to the b. Refers to scoliosis presenting at age
Heuter-Volkmann principle 4-10 years
c. Neuromuscular scoliosis tends to develop c. The majority of curves require treatment
later than most cases of idiopathic scoliosis d. Surgical treatment is usually considered
d. Neuromuscular spinal deformities are when curves reach 50-60°
less likely to progress in severity e. Single-staged posterior fusion procedures
e. Neuromuscular curves are rarely associ- are the preferred surgical treatment
ated with pelvic obliquity

377
Neurosurgery Books Full
www.ketabpezeshki.com 66485438-66485457
378 PART IV SPINAL NEUROSURGERY

6. Which one of the following statements b. A left-sided thoracic curve is more likely
regarding adolescent idiopathic scoliosis is in this patient
most accurate? c. Scoliosis is reported in 25-85% of
a. Lumbar and thoracolumbar curves are syringomyelia cases.
more likely to progress than thoracic d. She may have abnormal abdominal reflexes
curves because they lack the inherent sta- e. Decompression of the syrinx causes
bility provided by the rib cage improvement of the scoliosis in most cases
b. It is commoner in females and tends to
10. Which one of the following statements
have a right thoracic curve
regarding adult scoliosis is most accurate?
c. Female sex is not a risk factor for curve
a. Type 1 adult scoliosis is associated with
progression
degenerative disease of the spine
d. Genetic risk stratification is not an option
b. Curves 30-50° at skeletal maturity pro-
e. Curves measuring 50-75° at maturity
gress on average 3° per year
progress steadily at a rate of approxi-
c. Curves between 50° and 75° progress at
mately 5° per year
an average of 5° per year.
d. Leg length discrepancy is a cause of Type
7. Which one of the following statements
2 adult scoliosis
regarding the management of adolescent idi-
e. Adult idiopathic scoliosis generally does
opathic scoliosis is LEAST accurate?
not progress if curves are less than 50°
a. Untreated adult patients with a history of
adolescent idiopathic scoliosis do not have 11. Which one of the following statements
increased mortality regarding congenital scoliosis is LEAST
b. In the absence of hypokyphosis, cardiore- accurate?
spiratory complications occur with curves a. The defect arises between weeks 4 and 6
of 90° or more of embryogenesis.
c. Bracing is contraindicated in the presence b. Fully segmented hemivertebrae results in
of skeletal maturity rate of progression of 5° per year
d. Observation is appropriate for curves less c. Block vertebrae are defects of segmentation
than 20° d. Wedge vertebrae are defects of formation
e. Skeletally mature adolescents with curves e. Incarcerated hemivertebrae produce little
of >30° should undergo posterior instru- or no spinal deformity
mented fusion
12. Which one of the following is NOT included
in the VACTERLS association?
8. Which one of the following statements
a. Eye abnormalities
regarding the Lenke classification of scoliosis
b. Tracheo-esophageal fistula
curves is LEAST accurate?
c. Cardiac abnormalities
a. There are six types based on the number
d. Single umbilical artery
of structural and non-structural curves
e. Anorectal abnormalities, eye abnor-
present
malities
b. It does not take into account thoracic
kyphosis 13. Which one of the following statements
c. Curve types are subclassified by the rela- regarding sagittal plane deformities is
tionship of the center sacral vertical line LEAST accurate?
to the lumbar spine a. Type I congenital kyphosis defects lead to
d. Curve classification takes the form of a sharp angular kyphosis that may cause
curve type, lumbar modifier and thoracic paraplegia
sagittal modifier b. Thoracolumbar kyphosis in achondropla-
e. Cervical curves are not included in the sic dwarfs resolves in the majority of cases
classification by 12-18 months of age
c. Type II congenital kyphosis is due to a
9. A 12-year-old girl with scoliosis was found to defect of vertebral body segmentation
have a fluid-filled cavity within the spinal d. Posterior in situ fusion should be consid-
cord on a routine preoperative MRI scan. ered for a young child with a kyphosis
Which one of the following statements is measuring less than 50°
LEAST accurate? e. Bracing prevents deformity progression
a. There is an increased risk of neurological and may provide long-term correction
deficit with spinal distraction and instru- of a congenital kyphotic deformity in skel-
mentation in this patient etally immature patients

Neurosurgery Books Full


www.ketabpezeshki.com 66485438-66485457
27 SCOLIOSIS AND SPINAL DEFORMITY 379

14. A 15-year-old presents with back pain and d. Major curve


kyphotic deformity in the thoracic spine with e. Minor curve
a Cobb angle of 60° between T5 and T12. He f. Non-structural curve
is unable to correct the deformity by active g. Rib-vertebral angle difference
extension. Sagittal CT spine is shown. Which h. Risser sign
one of the following statements regarding i. Sagittal balance
this condition is LEAST accurate? j. Stable vertebra
k. Structural curve

For each of the following descriptions, select


the most appropriate answer from the list shown
above. Each answer may be used once, more than
once or not at all.
1. Scoliosis curves that do not correct
completely on bending
2. A useful guide to assessing skeletal matu-
rity based on ossification of the iliac
apophysis.
3. A vertical line extending cephalad from the
center of the sacrum and through the S1
spinous process
4. The vertebra bisected by the center
sacral line
5. Horizontal distance between a perpendicu-
lar line dropped from the center of the body
of C7 to the floor and the posterior superior
corner of the S1 segment

16. Scoliosis management:


a. Anterior or posterior fusion of the
lumbar curve
b. Anterior release followed by posterior
instrumentation of the thoracic curve
c. Boston brace (TLSO)
d. Fusion of the whole spine including pelvis
e. Growth rod application
f. Kyphectomy
a. The condition is often accompanied by a g. Milwaukee brace (CTLSO)
lumbar hyperlordosis h. Posterior instrumentation of curve
b. This is the most common cause of i. Posterior instrumented fusion of thoracic
thoracic back pain in adolescents and lumbar curve
c. MRI spine should be performed
preoperatively For each of the following descriptions, select the
d. Extension bracing is not appropriate even most appropriate answer from the list shown
in skeletally immature patients above. Each answer may be used once, more than
e. Ligamentum flavum excision should be once or not at all.
performed at the apex of the curve during 1. A 17-year-old girl presents with a right
surgery thoracic curve of 55° and a lumbar curve
of 40°. On the bending views, the
thoracic curve corrects to 33° and the lum-
bar to 18° with no significant apical verte-
EXTENDED MATCHING ITEM (EMI) bral rotation.
QUESTIONS 2. An 11-year-old premenarchal girl presents
with a right low thoracic curve (apex at
15. Scoliosis terms: T10) of 38°. MRI spine and craniocervical
a. Center sacral vertebral line junction are normal.
b. Cobb angle 3. A 15-year-old boy wheelchair user with
c. Coronal balance cerebral palsy presents with a 70° lumbar
curve with associated pelvic obliquity.

Neurosurgery Books Full


www.ketabpezeshki.com 66485438-66485457
380 PART IV SPINAL NEUROSURGERY

SBA ANSWERS
1. a—Only applies to coronal plane deformity often before age 10 and are more likely to progress
in severity due to the early age of onset of neuromus-
The Cobb angle (X) is obtained on the frontal PA cular disease. Neuromuscular causes of scoliosis may
radiograph and is calculated by finding the verte- be myopathic or neuropathic. Myopathic causes
brae that are maximally tilted at the cranial and include muscular dystrophies (Duchenne, limb-
caudal portions of the curve being evaluated (prox- girdle, and fascioscapulohumeral), myotonic dystro-
imal and distal end vertebrae). Lines are drawn phy and congenital hypotonia. Neuropathic causes
parallel to the superior endplate of the proximal may be upper motor neuron (cerebral palsy, Frider-
end vertebrae and parallel to the inferior endplate ich's ataxia, syringomyelia, quadriplegia), lower
of the distal end vertebrae; 90° perpendicular lines motor neuron (spinal muscular atrophy, poliomyeli-
are then drawn for each of these lines so that they tis, dysautonomia) or mixed. The prevalence of spi-
intersect. The angle created by these two intersect- nal deformities in different neuromuscular
ing lines is the Cobb angle. The apex of a curve is diseases is variable: cerebral palsy (25%),
the most lateral vertebral body on frontal radio- myelodysplasia (60%), spinal muscular atrophy
graphs and is considered cervical if its apex is (67%), Friedreich's ataxia (80%), Duchenne's mus-
between C2 and C6, cervicothoracic if between cular dystrophy (90%), and spinal cord injury before
C7 and T1, thoracic if between T2 and T11, thor- 10 years of age (100%).
acolumbar if between T12 and L1, lumbar if
between L2 and L4, or lumbosacral if at L5 or FURTHER READING
below. While there should not be any curve in Mardjetko S, Devlin VJ. Neuromuscular spinal deformities.
the coronal place normally, the sagittal Cobb angle In: Devlin VJ (Ed.), Spine Secrets Plus, 2nd ed., Elsevier,
can be used to measure normal cervical lordosis, Mosby, 2012 [chapter 41].
thoracic kyphosis and lumbar lordosis (40-60°)
3. c—Surgery is not recommended before the
using a lateral radiograph.
age of 5
Image with permission from Czervionke LF, Fenton DS.
Imaging Painful Spine Disorders, Copyright © 2011, Evaluation of neuromuscular scoliosis requires
Mayo Foundation for Medical Education and Research. assessment of the spinal deformity as well as mul-
Published by Saunders, Elsevier Inc. tidisciplinary evaluation of underlying neuro-
muscular disease (e.g. developmental, seizures,
FURTHER READING
musculoskeletal, infections). Observation is
Czervionke LF, Fenton DS. Imaging Painful Spine Disorders.
Copyright © 2011 by Mayo Foundation for Medical Educa- reasonable for curves <30°, large curves without
tion and Research Published by Saunders, Elsevier Inc. functional loss in severe developmental disabil-
ity, and those not fit for major spinal reconstruc-
2. a—Neuromuscular curves tend to be longer tive surgery. In most cases of neuromuscular
and involve more vertebrae than idiopathic scoliosis, a spinal orthosis will not prevent curve
scoliosis curves progression but serves to (i) help nonambulatory
patients to sit and (ii) slows progression of spinal
The Heuter-Volkmann principle states that deformities until the onset of puberty (permits
decreased loading across an epiphyseal growth growth of the spine prior to definitive treatment
plate inhibits growth and increased pressure tends with spinal instrumentation and fusion).
to accelerate growth, hence imbalance of forces Orthotic management is challenging in neuro-
across vertebral end plates due to neuromuscular muscular disorders because of poor muscle con-
disease results in scoliosis. A wide spectrum of spi- trol, impaired sensation, pulmonary
nal deformities may develop including scoliosis compromise, impaired gastrointestinal function,
(most common), hyperkyphosis, hyperlordosis, obesity, and difficulty with cooperating with
and complex multiplanar deformities. Neuromus- brace wear. In general, operative treatment is
cular curves are typically long, sweeping C-shaped considered when progressive curves exceed 40°
curves that extend to the pelvic region. The curve or when patients develop trunk decompensation,
apex is usually in the thoracolumbar or lumbar and there is no absolute minimum age to con-
region. When secondary curves develop, they are sider surgery. Earlier surgical treatment is
usually unable to restore coronal balance. Significant advised for patients with Duchenne's muscular
sagittal plane deformity often accompanies coronal dystrophy (when curves reach 20°) due to pre-
plane deformity. Pelvic obliquity is common and dictable pulmonary deterioration associated with
poses a major problem because it creates an uneven further curve progression. It is not necessary to
sitting base. Neuromuscular scoliosis develops at delay surgery until skeletal maturity. Curves up
an earlier age than most cases of idiopathic scoliosis, to 90° are most commonly treated with posterior

Neurosurgery Books Full


www.ketabpezeshki.com 66485438-66485457
27 SCOLIOSIS AND SPINAL DEFORMITY 381

spinal instrumentation and fusion. Curves fusion is avoided due to: (1) restriction of thoracic
exceeding 90° or curves with severe stiffness cage and lung development, and (2) the risk of
are considered for more complex procedures. crankshaft phenomenon. In extreme cases, a com-
Combined anterior (i.e. ATDF - anterior tho- bined anterior and posterior fusion procedure is an
racic discectomy and fusion) and posterior option but will consequently limit development of
approaches may help deformity correction, the thorax, lungs, and normal trunk height.
enhance fusion and avoid the crankshaft phe-
nomenon (by destroying anterior growth plates FURTHER READING
in skeletally immature patients). Devlin VJ. Idiopathic scoliosis. In: Devlin VJ (Ed.), Spine
Secrets Plus, 2nd ed., Elsevier, Mosby, 2012 [chapter 39].

FURTHER READING 5. e—Single-staged posterior fusion proce-


Mardjetko S, Devlin VJ. Neuromuscular spinal deformities. dures are the preferred surgical treatment
In: Devlin VJ (Ed.), Spine Secrets Plus, 2nd ed., Elsevier,
Mosby, 2012 [chapter 41]. Juvenile idiopathic scoliosis represents a gradual
transition from the characteristics of infantile idi-
4. c—Infantile scoliosis commonly affects girls, opathic scoliosis to those of adolescent idiopathic
whereas adolescent idiopathic scoliosis pre- scoliosis. It is less common than adolescent idio-
dominantly affects boys pathic scoliosis, increasing female predominance
is noted with increasing age from 4- to 10-year-
The criterion for diagnosis of scoliosis is a coronal olds, and right thoracic and double major curve
plane spinal curvature of 10° or more as measured
types are commonest. Approximately 70% of
by the Cobb method. Curves less than 10° are
curves progress and require some forms of treat-
referred to as spinal asymmetry. MRI spine includ- ment (bracing or surgery). Orthotic treatment is
ing craniocervical junction is required to exclude
initiated for curves in the 25-50° range. Surgical
CNS causes (e.g. syrinx, Chiari malformation,
treatment is considered when curve magnitude
tethered cord). Idiopathic scoliosis is defined as a
exceeds 50-60°. Surgical decision making is com-
spinal deformity characterized by lateral bending
plex in view of the wide age range of patients pre-
and fixed rotation of the spine in the absence of
senting in this group. Major concerns include the
any known cause. Idiopathic scoliosis is classified
effect of treatment on remaining growth and
according to age at onset into infantile (birth-3
potential for development of crankshaft phenom-
years), juvenile (3-10 years), and adolescent (after
enon if a single-stage posterior fusion procedure
10 years) subtypes. An alternative classification
is performed. Dual growing rod instrumentation
distinguishes early-onset scoliosis (0-5 years) from is considered for early juvenile scoliosis patients.
late-onset scoliosis (after 5 years) due to increased
Combined anterior and posterior fusion with
cardiopulmonary risk associated with early-onset
posterior instrumentation is an option for older
scoliosis due to rapid curve progression. In gen- patients.
eral, the younger the age at diagnosis, the more
likely the deformity will progress and require FURTHER READING
treatment. Infantile idiopathic scoliosis is com- Devlin VJ. Idiopathic scoliosis. In: Devlin VJ (Ed.), Spine
mon in Europe but rare in the USA, and is charac- Secrets Plus, 2nd ed., Elsevier, Mosby, 2012 [chapter 39].
terized by male predominance, commonly a left
thoracic curve, and is associated with plagioce- 6. b—It is commoner in females and tends to
phaly, developmental delay, congenital heart dis- have a right thoracic curve
ease, and developmental hip dysplasia; it is
divided into resolving (85%) and progressive Adolescent idiopathic scoliosis is the most common
(15%) types, with progression likely in those with type of scoliosis in children (prevalence is 3% in the
a rib-vertebral angle difference >20° and increas- general population), but few adolescent patients
ing rib phase (overlap of rib head and apical verte- (0.3%) develop curves requiring treatment. Com-
bral body). Resolving curves are observed with monly patients are female (especially larger curves),
advice to sleep in the prone position. Progressive have a right thoracic curve and do not have severe
curves are treated with serial derotational (plaster) pain. Risk factors for curve progression in skeletally
casting followed by orthotic treatment with a Mil- immature patients are future growth potential (e.g.
waukee brace. Curves that continue to progress age at onset, Risser stage, Tanner stage, menarche,
despite orthotic treatment require surgery. peak height velocity, triradiate physeal closure, skel-
Growth preserving options (permit delay of defin- etal age as determined by hand radiographs), curve
itive fusion until the child has achieved additional magnitude, curve pattern, female sex and genetic
growth) include posterior spinal instrumentation risk score. Curves measuring less than 30° at matu-
without fusion or the vertically expandable pros- rity are least likely to progress. Curves measuring
thetic titanium rib. Posterior spinal instrumented 30-50° are likely to progress an average of 10-15°

Neurosurgery Books Full


www.ketabpezeshki.com 66485438-66485457
382 PART IV SPINAL NEUROSURGERY

over the course of a normal lifetime. Curves mea- FURTHER READING


suring 50-75° at maturity progress steadily at a rate Devlin VJ. Idiopathic scoliosis, chapter 39. In: Devlin VJ (ed),
of approximately 1° per year. Lumbar and thoraco- Spine Secrets Plus, 2nd ed, 2012, Elsevier, Mosby.
lumbar curves are more likely to progress than tho-
racic curves because they lack the inherent stability 8. b—It does not take into account thoracic
provided by the rib cage. kyphosis

FURTHER READING The Lenke classification is based on assessment


Devlin VJ. Idiopathic scoliosis. In: Devlin VJ (Ed.), Spine of PA, lateral, and side-bending radiographs
Secrets Plus, 2nd ed., Elsevier, Mosby, 2012 [chapter 39]. (the latter aim to distinguish structural from
non-structural curves) and measuring the Cobb
7. e—Skeletally mature adolescents with curves angle of all curves present; the curve with the
of >30° should undergo posterior instru- largest Cobb angle is the major curve, which is
mented fusion always structural, while other minor curves can
be classified as structural or non-structural on
The mortality rate of untreated adult patients the basis of Cobb angle >25° on side bending
with adolescent idiopathic scoliosis is comparable films and degree of kyphosis. Various combina-
with that of the general population, unlike those tions at proximal thoracic, main thoracic and
with early-onset scoliosis (before age 5) who thoracolumbar/lumbar levels result in 6 curve
develop severe curves (90°) with cor pulmonale types: (1) main thoracic, (2) double thoracic,
and right ventricular failure, resulting in prema- (3) double major, (4) triple major, (5) thoracolum-
ture death. Common reasons for presentation bar/lumbar, and (6) thoracolumbar/lumbar-main
are back pain and cosmesis. The treatment thoracic. Next, a lumbar spine modifier is applied
options for adolescent idiopathic scoliosis include depending on the relationship of the center
observation, orthoses, and operation. The sacral vertebral line to the lumbar spine: (A)
purpose of observation for adolescent idiopathic between pedicles, (B) touches apical body, and
scoliosis is to identify and document curve pro- (C) completely medial. Finally, a thoracic sagittal
gression and thereby facilitate timely interven- modifier describes degree of T5-T12 kyphosis:—
tion. Bracing aims to prevent progression, and (hypo) if <10°, N (normal) if 10-40°, and +
the general types of orthoses used for adolescent (hyper) if >40°. The classification takes the form
idiopathic scoliosis are CTLSO (Milwaukee of (1-6) + (A, B or C) + ( , N, +), e.g. 1BN.
brace, most efficacious for curve apex above
T8), TLSO (e.g. Boston brace; curves with an
FURTHER READING
apex at T8 or below; better tolerated), bending
Devlin VJ. Idiopathic scoliosis. In: Devlin VJ (Ed.), Spine
brace and flexible brace. Contraindications to Secrets Plus, 2nd ed., Elsevier, Mosby, 2012 [chapter 39].
brace treatment include skeletal maturity, curves
>40°, thoracic lordosis (worsens cardiopulmo- 9. e—Decompression of the syrinx resolves
nary restriction), not tolerating. Surgical decision scoliosis in most cases
making is based on the coronal Cobb angle, sag-
ittal plane alignment, rotational deformity, the Syringomyelia, a fluid-filled cavity within the spi-
natural history of the patient's curve, and the nal cord, may lead to scoliosis that can be mistak-
patient’s skeletal maturity. enly attributed to idiopathic scoliosis. Associated
spinal curvature has been reported in 25-85%.
Syrinx related curves are rapidly progressive,
atypical and usually left sided, and associated with
Major abnormal abdominal reflexes. Other musculo-
Curve Management skeletal features include pes cavus, wasting of
<20° Observation intrinsic hand muscles and Charcot joints. A
symptomatic syrinx requires surgical treatment,
20-29° If Risser 0-1, premenarchal: Immediate which may improve neurologic deficits and pre-
bracing
If Risser 2: brace if progresses by 5° vent curve progression. Surgical treatment (spinal
during observation distraction and instrumentation) of scoliosis with-
out recognition of syringomyelia can result in
30-40° Brace if skeletally immature
increased neurological complications.
>40° If skeletally immature and failed
bracing, offer surgery
If skeletally mature, wait till >50° before FURTHER READING
surgery Kontio K, et al. Management of scoliosis and syringomyelia in
Children. J Pediatr Orthopaed 2002;22(6):771-9.

Neurosurgery Books Full


www.ketabpezeshki.com 66485438-66485457
27 SCOLIOSIS AND SPINAL DEFORMITY 383

10. a—Type 1 adult scoliosis is associated with 13. e—Bracing prevents deformity progression
degenerative disease of the spine and may provide long-term correction of a
congenital kyphotic deformity in skeletally
Adult scoliosis can be divided into type 1 adult immature patients.
scoliosis, which is degenerative; type 2 idiopathic
adolescent scoliosis, which progresses into adult- Type I is a defect of vertebral body formation
hood; and type 3 secondary adult scoliosis, which (hemivertebra), type II is a defect of vertebral
may be due to a leg length discrepancy, hip body segmentation (block vertebra or bar), and
pathology, or may be secondary to a metabolic type III is a mixed or combined lesion. Type I
bone disease such as osteoporosis combined with defects are more common and more serious
asymmetric arthritic disease. Adult idiopathic because they lead to a sharp angular kyphosis that
scoliosis: curves <30° generally do not progress, may cause paraplegia. Bracing does not prevent
curves 30-50° progress 10-15° during life, and deformity progression or provide long-term cor-
curves 50-75° progress at 1° per year. Degenera- rection of a congenital kyphotic deformity. Non-
tive scoliosis usually develops after the age of 50 surgical management does not play a role in the
and is typically associated with disk degeneration, treatment of congenital kyphosis. Congenital
facet arthritis, thickening/hypertrophy of the kyphosis does not respond to non-operative treat-
ligamentum flava, loss of lumbar lordosis, and lat- ment. Posterior in situ fusion should be consid-
eral listhesis. Degenerative scoliosis can lead to ered for a young child (1-5 years old) with a
neurogenic claudication, radicular pain, and kyphosis measuring less than 50°. Kyphosis
back pain. greater than 50° and older children require an
anterior and posterior fusion. Symptomatic neu-
FURTHER READING ral compression at the apex of the kyphosis
Czervionke LF, Fenton DS. Imaging Painful Spine Disorders, requires decompression. In select deformities,
Copyright © 2011 by Mayo Foundation for Medical Educa- circumferential decompression and fusion may
tion and Research Published by Saunders, Elsevier Inc. be achieved through a single-stage posterior sur-
gical approach. Extensive preoperative evaluation
11. b—Fully segmented hemivertebrae results in
is required, including cardiopulmonary assess-
rate of progression of 5° per year
ment, evaluation of the genitourinary system,
Congenital scoliosis is due to vertebral anomalies detailed neurologic examination, MRI of the neu-
that produce a frontal plane growth asymmetry. ral axis, and a computed tomography scan to
The anomalies are present at birth, but the curva- define osseous abnormalities. Thoracolumbar
ture may take years to become clinically evident. kyphosis is the most common sagittal plane defor-
During weeks 4-6 of the embryonic period. mity among achondroplasic dwarfs. The kyphosis
Defects of segmentation include block vertebra is generally evident at birth, progresses as the
(bilateral failure of segmentation), unilateral bar child begins to sit, and resolves in approximately
alone, or unilateral bar with contralateral hemi- 70% of cases with ambulation at 12-18 months.
vertebra. Defects of formation include hemiver- Radiographs show anterior wedging at the apex
tebra (unilateral complete failure of formation) of the deformity. Progression can lead to a focal
or wedge vertebra (unilateral partial failure of for- kyphosis and possible neural compression, which
mation). Most rapidly progressive deformities are may be masked by the lumbar stenosis associated
unilateral unsegmented bar  hemivertebra at with achondroplasia. Anterior and posterior
5-6° per year. Fully segmented hemivertebra fusion is reserved for children with progressive
progresses at 1-2° per year. Semi-segmented, deformity, thoracolumbar kyphosis greater than
incarcerated, and non-segmented hemivertebrae 50° at age older than 5 years, or neural compro-
produce little or no deformity. mise attributed to compression in the kyphotic
region.
FURTHER READING
Karlin LI. Congenital spinal deformities. In: Devlin VJ (Ed.), FURTHER READING
Spine Secrets Plus, Elsevier, Mosby, 2012 [chapter 42]. Gupta MC, Devlin VJ. Sagittal plane deformities in paediatric
patients. In: Devlin VJ (Ed.), Spine Secrets Plus, Elsevier,
12. a—VACTERL association is usually Mosby, 2012 [chapter 40].
defined by the presence of 3 of: vertebral,
anorectal, cardiac, tracheo-esophageal fis- 14. d—Extension bracing is not appropriate even
tula, renal abnormalities and limb dysplasia. in skeletally immature patients
Extensions of the association include lung
abnormalities and single umbilical artery, Scheuermann's kyphosis is a kyphotic deformity
amongst others. of >45° in the thoracic spine with >5° of anterior

Neurosurgery Books Full


www.ketabpezeshki.com 66485438-66485457
384 PART IV SPINAL NEUROSURGERY

wedging across three consecutive vertebrae, and surgery include (1) skeletally immature adoles-
is the commonest cause of thoracic back pain in cent patients with painful kyphosis >75° with
children and adolescents. The exact cause is local wedging >10° not responsive to 6 months
unknown but may involve avascular necrosis of of bracing, (2) skeletally mature patients with
the vertebral body ring apophysis. Type I painful deformity resistant to bracing, (3) curves
Scheuermann's kyphosis is a rigid, angular >80° in skeletally mature patients. Surgery
thoracic kyphosis and has a hereditary compo- entails a posterior spinal fusion with dual-rod
nent while type II is thoracolumbar, more pain- instrumentation/anterior release and interbody
ful, and affects predominantly athletes and fusion. The fusion level should stop distally at
laborers. A male or female approaching the the vertebra which is parallel to the floor (usually
end of skeletal growth presents with back defor- the L3 level). A ligamentum flavum excision
mity and/or pain. The increased thoracic kypho- should be performed at the apex to prevent
sis is accentuated with forward-bending, but not buckling of the ligament and therefore decrease
corrected by active extension (unlike in postural the risk of neurological deficit.
kyphosis). The condition is often associated with
a lumbar hyperlordosis. Other radiological Image with permission from Waldman MD, Campbell
features include vertebral endplate irregularities, RS. Imaging of Pain, Elsevier, Saunders, 2011.
Schmorl's nodes, and decreased disk space
height are additional radiographic findings that
FURTHER READING
may be present. An MRI scan is indicated to look Gupta MC, Devlin VJ. Sagittal plane deformities in paediatric
for disk herniation, cord abnormalities and spi- patients. In: Devlin VJ (Ed.), Spine Secrets Plus, Elsevier,
nal stenosis. Extension bracing is appropriate Mosby, 2012 [chapter 40].
for curves between 45° and 74° with 2 years of
growth remaining and greater than 5° wedging.
An apex at T9 or above is traditionally treated EMI ANSWERS
with a Milwaukee type brace. A thoracolumbar
orthosis (TLSO) is considered if the apex is 15. 1—k, Structural curve; 2—h, Risser sign;
below T9. Braces should be updated every 4-6 3—a, Center sacral vertebral line; 4—j,
months to maximize deformity correction and Stable vertebra; 5—i, Sagittal balance
weaned with skeletal maturity. Indications for

Common Terms in Scoliosis Surgery


Major curve The curve with the largest Cobb measurement and is always a structural curve
Minor curve Other curves associated with the major curve, which can be either structural or
nonstructural
Non-structural curve Curves that correct completely when the patient bends toward the convexity of
the curve. Nonstructural curves permit the shoulders and pelvis to remain level to
the ground and permit the head to remain centered in the midline above the pelvis.
For this reason, nonstructural curves are also referred to as compensatory curves
Structural curve Curves that do not correct completely on bending (e.g. A 60° curve corrects to 45°
on bending)
Sagittal balance Sagittal balance is the horizontal distance between a perpendicular line dropped
(lateral radiograph) from the center of the body of C7 to the floor and the posterior superior corner of
the S1 segment (normally within 2 cm). Relative to the posterior superior corner
of S1, the C7 plumb line may fall posteriorly (negative sagittal balance), directly
through it (neutral balance) or anteriorly (positive sagittal balance). Positive sagittal
balance greater than 2 cm is more significantly associated with pain and disability
than curve magnitude, curve location, or coronal imbalance.
Coronal balance The horizontal distance between a vertical plumb line drawn through the C7
(frontal radiograph) vertebral body and the center sacral vertebral line. The C7 plumb line should
normally pass within 2 cm of the Centerl Sacral Vertical Line, either to the right
(positive coronal balance), directly through it (neutral), or to the left (negative
coronal balance)
Center sacral A vertical line extending cephalad from the center of the sacrum and through
vertebral line the S1 spinous process

Continued

Neurosurgery Books Full


www.ketabpezeshki.com 66485438-66485457
27 SCOLIOSIS AND SPINAL DEFORMITY 385

Risser sign Ossification of the iliac apophysis used as a guide to assessing skeletal maturity.
The iliac crest is divided into quarters, and the stage of ossification is used as a
guideline to assess skeletal maturity: grade 0 (absent), grade 1 (0-25%), grade 2
(26-50%), grade 3 (51-75%), grade 4 (76-100%), and grade 5 (fusion of apophysis
to the ilium). Risser stage 4 correlates with the end of spinal growth in females,
and Risser stage 5 correlates with the end of spinal growth in males. For the
same curve angle, the risk of progression is lower with greater skeletal maturity
(i.e. higher Risser stage)
Rib-vertebral angle A line perpendicular to the endplate of the apical vertebra is bisected by a line
difference drawn along the central axis of its rib, producing a smaller angle inferiorly and
a lager angle superiorly (both adding to 180°). The difference between these two
RVAs is calculated
Stable vertebra The vertebra bisected by the center sacral line
Neutral vertebra The first nonrotated vertebra at the caudal and cranial end of a curve. Rotation
is assessed based on the radiographic appearance of the vertebral pedicle shadow

16. 1—i, Posterior instrumentation of thoracic patient, bracing should be trialed given a
curve. In this case of a skeletally mature curve of 30-40°. Skeletal maturity (Risser 5)
patient with a non-structural minor curve is achieved at 16 years in females and 18 years
(i.e. corrects to <25°) bracing is contraindi- in males, with Risser 1 stage appearing pre-
cated and posterior instrumentation of the puberty/early puberty. 3—d, Spinal fusion
major curve is the optimal choice. 2—c, Bos- down to pelvis. Due to pelvic obliquity any
ton brace. In this case of adolescent idio- surgical treatment should include instrumen-
pathic scoliosis in a skeletally immature tation of the pelvis.

Neurosurgery Books Full


www.ketabpezeshki.com 66485438-66485457
CHAPTER 28

SPINAL TRAUMA AND


ACUTE PATHOLOGY
SINGLE BEST ANSWER (SBA) QUESTIONS
1. Which one of the following statements 3. Which one of the following statements
regarding pre-hospital spinal immobilization regarding clearing the cervical spine in sus-
is LEAST accurate? pected spinal injury is most accurate?
a. Pre-hospital spine immobilization should a. NEXUS low-risk criteria includes absence
be routinely used in the setting of pene- of a flexion-distraction mechanism of injury
trating trauma b. In awake, symptomatic patients three-
b. Time spent on a spinal board is associated view radiographs are the initial imaging
with pressure ulcer development in the study for cervical spine injury
next 8 days c. In obtunded or comatose patients a high-
c. The probability of a noncontiguous spinal quality CT scan of the entire spinal axis is
injury in the setting of a known injury is recommended
approximately 20% d. Canadian C-spine rules require assess-
d. Firm application of a cervical collar may ment of the range of movement of the
be associated with an ICP rise of cervical spine to justify not imaging the
2-5 mmHg cervical spine
e. Pre-hospital selective spine immobiliza- e. STIR MRI is used to confirm spinal
tion protocols can be up to 99% sensitive instability
in identifying trauma patients with cervi-
cal injuries requiring immobilization 4. An 18-year-old male is brought into the
emergency room after diving into a shallow
2. Which one of the following statements pool. He is awake and alert, has intact cranial
regarding clinical assessment of SCI using nerves (CNs), and is able to move his shoul-
the American Spinal Injury Association Scale ders, but has a flaccid quadriplegia and a sen-
is most accurate? sory level at C5. Which one of the following
a. ASIA E is where no sensory or motor statements regarding medical management
function is preserved below the level of of SCI is most accurate?
injury or in the sacral segments S4-S5 a. Methylprednisolone should be adminis-
b. ASIA D is where sensory but not motor tered if the injury was within 3 h of arrival
function is preserved below the neurolog- to the emergency department (30 mg/kg
ical level and includes the sacral segments bolus followed by a 5.4-mg/kg/h infusion
S4-S5 for 24 h)
c. ASIA C describes preserved motor func- b. Class II studies have shown therapeutic effi-
tion below the neurological level, and cacy of methylprednisolone in improving
more than half of key muscles below motor function in acute spinal cord injury
the neurological level have a muscle c. Class I studies have shown significant
grade <3 harmful side effects associated with meth-
d. ASIA B describes preserved motor func- ylprednisolone in spinal cord injury
tion below the neurological level, and d. Maintenance of mean arterial pressure
at least half of key muscles below the from 85 to 90 mmHg in the first 7 days
neurological level have a muscle grade 3 may improve spinal cord perfusion and
e. ASIA A describes normal sensory and outcome after SCI
motor function e. Cardiovascular instability is unlikely to
develop if the patient is initially stable
on admission

386
Neurosurgery Books Full
www.ketabpezeshki.com 66485438-66485457
28 SPINAL TRAUMA AND ACUTE PATHOLOGY 387

5. A 21-year-old male is involved in an RTA.


Examination shows that he is able to move
over half of muscles but not against gravity
(ASIA C) and CT trauma protocol shows a
C4 burst fracture, 25% loss of vertebral body
height, 70% encroachment of the spinal canal
due to retropulsion. MRI shows evidence of
injury to the posterior ligamentous complex
and cervical cord compression. Which one
of the following statements regarding the
STASCIS trial is most accurate?
a. The results are not applicable to traumatic
cervical spinal cord injury
b. Those with ASIA A injuries at presenta-
tion were excluded
c. Early decompression (<24 h) was associ-
ated with significantly more patients with
a 2 grade improvement in ASIA impair- a. Complete spinal cord injury
ment scale at 6-month follow-up b. Central cord syndrome
d. Early decompression (<24 h) was associ- c. Anterior cord syndrome
ated with significantly more patients with d. Posterior cord syndrome
a only 1 grade improvement in ASIA e. Brown-Sequard syndrome
impairment scale at 6-month follow-up
e. Odds of a 2 grade improvement in ASIA 8. A patient develops Wallenberg syndrome
impairment scale at 6-month follow-up 4 days after C5-C6 fracture dislocation:
were fourfold higher in the early decom- Which one of the following statements
pression (<24 h) versus late decompres- regarding further management is most
sion (>24 h) group accurate?
a. CT head, CT angiogram and start
6. Which one of the following statements anticoagulation
regarding spinal cord perfusion pressure b. Diffusion weighted MRI
(SCPP) and intradural spinal pressure (ISP) c. Clinical observation
in spinal cord injury patients is most accurate? d. CT head and anticoagulation
a. Inotropes cause an increase in ISP e. CT head and commence antiplatelet
and MAP but with a net increase in SCPP
b. Mannitol administration causes a reduc- 9. Which one of the following statements
tion in ISP and increases MAP regarding VTE prophylaxis in SCI is most
c. Reduction in PaCO2 reduces ISP accurate?
d. Sevoflurane increases ISP a. Prophylactic IVC filter insertion is appro-
e. Laminectomy plus expansion duraplasty priate in those with complete SCI
provided a higher ISP and SCPP b. Low molecular weight heparin should
be discontinued after 3 months post-
7. A 75-year-old man falls sustaining a hyperex- injury in those without other risk factors
tension injury to his neck. On examination, for VTE
he has 3/5 strength in his deltoids, elbow c. Oral anticoagulation is the new standard
and wrist flexors and extensors bilaterally in of care for VTE prophylaxis in SCI in
the upper limbs. In the lower limbs, he has the first 28 days post-injury
4/5 strength in his hip flexors, knee flexors, d. Patients tolerating mechanical DVT pro-
extensors, ankle dorsiflexors and plantarflex- phylaxis do not require anticoagulation
ors bilaterally. Sensation is intact throughout e. SCI patients with retained lower limb
the limbs and saddle area. T2 weighted axial function have similar DVT rate to those
MRI is shown. Which one of the following is with complete injury
most likely?

Neurosurgery Books Full


www.ketabpezeshki.com 66485438-66485457
388 PART IV SPINAL NEUROSURGERY

10. Regarding Punjabi and white stability cri- 14. Which one of the following statements
teria, which one of the following weightings regarding management of occipital condyle
is inaccurate? fractures is LEAST accurate?
a. Anterior elements destroyed or unable to a. Occipito-cervical fusion is generally
function (2 points) recommended in the context of bilateral
b. Posterior elements destroyed or unable fractures with overt instability
to function (2 points) b. Halo immobilization is not commonly
c. Relative sagittal plane translation used in the management of unilateral
>3.5 mm (or >20% AP vertebral width) OC fractures
on X-ray (2 points) c. Cervical collars are the mainstay of
d. Relative sagittal plane rotation >11° on treatment
X-ray (2 points) d. Cervical collars are contraindicated in the
e. Cord or root damage (2 points) presence of cranial nerve palsy after uni-
lateral fracture
11. An 18-year-old male was admitted following e. MRI is recommended to assess the integ-
a fall from a height of 20 m. His American rity of the craniocervical ligaments
Spinal Injury Association (ASIA) motor
score was 43/100 and ASIA Impairment Scale 15. A 16-year-old is brought into the emergency
(AIS) A. Cervical spine CT showed a C7/T1 department intubated and ventilated after a
flexion-distraction injury. MRI indicated high speed MVA. Which one of the follow-
complete disruption of discoligamentous ing statements regarding the injury shown
complex with persistent spinal cord compres- is LEAST accurate?
sion. Which one of the following is the most
likely SLIC score?
a. 7
b. 8
c. 9
d. 10
e. 11

12. Which one of the following statements


regarding closed reduction of cervical
fracture-dislocations is LEAST accurate?
a. MRI is essential before attempted closed
reduction of cervical fracture-dislocations
b. MRI results should be available before
open reduction of fractures as a significant
disc herniation may favor an anterior cer-
vical approach
c. Closed reduction of cervical fracture-
dislocations is safe in awake patients
d. Risk of transient injury with closed reduc- a. Associated with a Power’s ratio <0.8
tion is 2-4% b. Associated with BAI or BDI >12 mm
e. Risk of permanent neurological injury c. Associated with increased condyle-C1
from closed reduction is 1% interval in children
d. Associated with a high mortality
13. Which one of the following is a reasonable e. Associated with high speed motor vehicle
guide for weight increments used in cervical accidents
traction with Gardener Wells tongs?
a. 1 lb per level
b. 5 lb per level
c. 10 lb per level
d. 15 lb per level
e. 20 lb per level

Neurosurgery Books Full


www.ketabpezeshki.com 66485438-66485457
28 SPINAL TRAUMA AND ACUTE PATHOLOGY 389

16. A 21-year-old male complains of 1 year dura-


tion of neck pain. He denies any recent
trauma. He has noticed intermittent episodes
of gait imbalance and difficulty with button- a b
ing his shirt over the past 3 months. Physical
exam shows normal strength in all four
extremities and hyper-reflexic patellar ten-
dons. Neutral and flexion radiographs are
awaited. Which one of the following is the
most appropriate treatment?
a. Conservative management
b. Cervical collar
c. Halo immobilization for 12 weeks
d. C1-C2 instrumented fusion
e. Minerva vest

QUESTIONS 18–37

Additional questions 18–37 available on


ExpertConsult.com

EXTENDED MATCHING ITEM (EMI)


QUESTIONS
a. Physiotherapy
b. Conservative management with avoid- 38. Spinal Fractures:
ance of contact sport a. Atlanto-occipital dissociation
c. Soft collar during sports b. Avulsion fracture
d. Posterior C1-C2 fusion c. Burst fracture
e. Anterior C1-C2 fusion d. Chance fracture
e. Clay-shoveler’s fracture
17. A 76-year-old falls but does not lose con- f. Compression fracture
sciousness, but does complain of neck pain. g. Extension teardrop fracture
He is neurologically intact. CTs are shown h. Flexion teardrop fracture
and sum of displacement of C1 lateral masses i. Hangman’s fracture
on C2 is 9 mm. j. Jefferson’s fracture
Which one of the following would be k. Occipital condyle fracture
appropriate management in this case? l. Unilateral facet dislocation

For each of the following descriptions, select the


most appropriate answers from the list above.
Each answer may be used once, more than once,
or not at all.
1. A 32-year-old male is transferred after
accidentally diving into a shallow pool.
He was found to be quadriplegic with loss
of pain and temperature sensation. CT
shows sustaining a highly unstable fracture
at C4
2. An 11-year-old child is involved in a road
accident and is found to be quadriplegic

Neurosurgery Books Full


www.ketabpezeshki.com 66485438-66485457
390 PART IV SPINAL NEUROSURGERY

at scene. CT of the cervical spine showed an h. Posterior C2-C3 transarticular screws


increased Condyle-C1 interval and cranio- i. Posterior decompression without pedicle
cervical subarachnoid hemorrhage screw fixation
3. A 67-year-old female has significant neck j. Posterior wiring
pain after falling and striking her head. X-
rays of the cervical spine demonstrated For each of the following descriptions, select the
the bow tie sign at C5. Axial CT show an most appropriate answers from the list above.
uncovered left C5 facet Each answer may be used once, more than once,
or not at all.
39. Mechanism of Injury: 1. Jefferson’s fracture in 57-year-old
a. Axial compression 2. Bilateral facet dislocation in patient with
b. Extension neurological compromise
c. Extension and axial compression
d. Flexion 41. Craniocervical measurements:
e. Flexion and axial compression a. Atlantodens interval (ADI)
f. Flexion-distraction b. Basion axial interval (BAI)
g. Rotational flexion-dislocation c. Basion dens interval (BDI)
h. Rotational flexion-distraction d. C1-Condyle interval (CCI)
i. Shear and rotation e. Chamberlain
j. Tension f. McGregor
g. McRae
For each of the following descriptions, select the h. Pavlov (torg) ratio
most appropriate answers from the list above. i. Power’s ratio
Each answer may be used once, more than once, j. Ranawat
or not at all. k. Space available for the cord (SAC)
l. Wackenheim’s line
1. Bilateral facet dislocation
2. Flexion teardrop For each of the following descriptions, select the
3. Jefferson’s fracture most appropriate answers from the list above.
4. Hangman fracture Each answer may be used once, more than once,
or not at all.
40. Surgical approaches for spine trauma: 1. A value less than 0.80 is considered a risk
a. Anterior corpectomy and combined factor for neurological injury after minor
anterior-posterior instrumented fusion trauma.
b. Anterior corpectomy, cage and plate 2. A value less than 14 mm is considered a risk
c. Anterior discectomy and fusion factor for neurological injury in patients
d. Anterior odontoid screw with rheumatoid arthritis.
e. C1-C2 posterior instrumented fusion 3. A line drawn from the center of the C2 ped-
f. Closed reduction with cervical traction icle to the C1 arch.
g. Halo immobilization

SBA ANSWERS
1. e—Pre-hospital spine immobilization should to die as those who are not (due to proper appli-
be routinely used in the setting of penetrating cation of spinal immobilization delaying patient
trauma resuscitation) hence routine use is not recom-
mended. Pre-hospital spinal immobilization is
Penetrating trauma (stab and gunshot) rarely advised when there is: (i) spinal pain or tender-
causes spinal instability even when specifically ness, including any neck pain with a history of
injuring the spine and those who are placed in trauma, (ii) significant multiple system trauma,
spinal immobilization at scene are twice as likely (iii) severe head or facial trauma, (iv) numbness

Neurosurgery Books Full


www.ketabpezeshki.com 66485438-66485457
28 SPINAL TRAUMA AND ACUTE PATHOLOGY 391

or weakness in any extremity after trauma, (v)


loss of consciousness caused by trauma, (vi) American Spinal Injury Association
mental status is altered (including drugs, alcohol, Impairment Scale
trauma) and no history is available, or the
ASIA A No sensory or motor function is
patient is found in a setting of possible trauma preserved below the level of injury or in
(e.g. lying at the bottom of stairs or in the the sacral segments S4-S5
street); or the patient experienced near drown-
ing with a history or probability of diving, (vii) ASIA B Sensory but not motor function is
preserved below the neurological level
any significant injury distracting the patient and includes the sacral segments S4-S5
from reporting spinal pain/symptoms. These
criteria are 99% sensitive in identifying trauma ASIA C Preserved motor function below the
patients with cervical injuries requiring immobi- neurological level, and more than half
lization, but extra vigilance is required in old of key muscles below the neurological
level have a muscle grade <3
(>67 years) and very young patients. As such,
immobilization of trauma patients who are ASIA D Preserved motor function below the
awake, alert, and are not intoxicated, who are neurological level, and at least half of
without neck pain or tenderness, who do not key muscles below the neurological
level have a muscle grade 3
have an abnormal motor or sensory examination
and who do not have any significant associated ASIA E Normal sensory and motor function
injury that might detract from their general eval-
uation is not recommended. The probability of a
noncontiguous spinal injury in the setting of a
known injury is approximately 20%, necessitat-
ing the need for complete spinal immobilization.
FURTHER READING
Limiting untoward spinal motion during trans- Hadley MN, Walters BC, Aarabi B, Dhall SS, Gelb DE,
portation of patients with cervical spine injuries Hurlbert RJ, Rozzelle CJ, Ryken TC, Theodore N. Clinical
is considered essential to preserve neurological assessment following acute cervical spinal cord injury. Neuro-
function and to limit further injury from spinal surgery. 2013;72 Suppl 2:40-53.
instability. Cervical SCIs have a high incidence
of airway compromise and pulmonary dysfunc-
3. c—In obtunded or comatose patients a high-
tion; therefore, respiratory support measures
quality CT scan of the entire spinal axis is
should be available during transport. Immobili-
recommended
zation is associated with modest morbidity such
as rises in ICP due to cervical collar placement, In awake trauma patients in the emergency
risk of pressure sores in the next 8 days (propor- department, cervical spine imaging is recom-
tional to length of time on rigid spinal mended unless they meet all of the NEXUS
board; also if not turned in the first 2 h), risk low-risk criteria: absent posterior midline cervical
of aspiration and impaired respiratory function, tenderness, no evidence of intoxication, a normal
and must take into account pre-existing spinal level of alertness and consciousness, absence of
deformity (e.g. ankylosing spondylitis, occipital any focal neurological deficit, absence of any dis-
recess in children)—favoring removal as soon tracting injuries (painful enough to distract the
as safe to do so. patient from another, particularly cervical, injury
e.g. long-bone fracture; a visceral injury; a signif-
FURTHER READING
Theodore N, et al. Prehospital cervical spinal immobilization
icant laceration, degloving or crush, large burns).
after trauma. Neurosurgery. 2013;72 Suppl 3:22-34. Alternatively, the Canadian C-spine rules can be
applied to stable trauma patients with a GCS
Theodore N, et al. Transportation of patients with acute trau- 15/15 to determine the need for imaging based
matic cervical spine injuries. Neurosurgery. 2013;72 Suppl
on the presence/absence of high risk factors
2:35-39.
necessitating radiography (age >65, significant
mechanism, paresthesias in extremities), presence
2. c—ASIA C describes preserved motor func- or absence of low-risk factors allowing safe assess-
tion below the neurological level, and more ment of cervical ROM (simple rear-end collision,
than half of key muscles below the neurolog- sitting up in ED, delayed onset of neck pain,
ical level have a muscle grade <3 absence of midline cervical tenderness) and

Neurosurgery Books Full


www.ketabpezeshki.com 66485438-66485457
392 PART IV SPINAL NEUROSURGERY

(if safe to do so) the ability to rotate neck actively FURTHER READING
45° to right and left. Imaging would need to be Ryken TC, Hadley MN, Walters BC, et al. Radiographic
performed if (i) high risk factors are present, assessment. Neurosurgery. 2013;72 Suppl 2:54-72.
(ii) low-risk factors are absent, or (iii) if low-risk Doi:10.1227/NEU.0b013e318276edee. Review. PubMed
factors present but the physician is unable to PMID: 23417179.
complete the range of motion assessment. Stiell IG, Wells GA, Vandeheem K, et al. The Canadian C-
Although not 100% sensitive, clinicians can easily Spine rule study for alert and stable trauma patients. JAMA.
apply the NEXUS criteria or Canadian C-spine 2001;286:1841-1848.
rules to deciding whether to request further cer- Stiell IG, Clement CM, McKnight RD, et al. The Canadian
vical spine imaging for an awake and asymptom- C-Spine rule versus the NEXUS low risk criteria in patients
atic patient. In awake, symptomatic patients (neck with trauma. New England Journal of Medicine.
pain and/or neurology) CT of the cervical spine 2003;349:2510-2518.
should be the initial imaging study. Traditional
three-view radiographs (anteroposterior, lateral 4. d—Maintenance of mean arterial pressure
and open-mouth odontoid view) should be from 85 to 90 mmHg in the first 7 days
obtained only if it is not possible to obtain a may improve spinal cord perfusion and out-
high-quality CT scan, but should be supplemen- come after SCI
ted with CT as soon as it becomes available if
there is high suspicion of injury or poor visualiza- Administration of methylprednisolone for the
tion on plain X-ray. If the CT scan is normal and treatment of acute spinal cord injury is no longer
the patient continues to have neck pain then sev- recommended based on evidence, although vari-
eral options exist: (i) continue cervical immobili- ation in practice persists due to medicolegal and
zation until asymptomatic, (ii) discontinue other contentions. There is no Class I or Class
cervical immobilization after normal MRI II medical evidence supporting clinical benefit
(<48 h post-injury) and/or adequate dynamic in the treatment of acute SCI (e.g. NASCIS I
flexion/extension radiographs, or (iii) discontinue and III), but Class I, II, and III evidence does exist
immobilization at the discretion of the treating suggesting that high-dose steroids are associated
physician. MRI including short T1 inversion with harmful side effects including death, GI
recovery (STIR) fat suppressed sequences to hemorrhage, pneumonia, steroid-induced myop-
identify damaged ligaments that indirectly sug- athy and wound infection. A variety of Class III
gests potential laxity in the joints and vertebrae medical evidence (e.g. NASCIS II) has been pub-
(i.e. potential instability), which could cause a lished supporting the neuroprotective effect of
subluxation and narrow the spinal canal. True methylprednisolone in SCI but generally, these
cervical spinal instability can only be directly con- studies suffer from 1 of 2 significant limitations:
firmed with cervical flexion-extension lateral limited sample size derived retrospectively from
radiographs. These films must be performed much larger study populations and/or incomplete
under controlled conditions to ensure that the data reporting in which omitted data are likely to
patient does not move his/her neck past the point have negated the proposed beneficial effect.
of worsening pain or symptoms, and the lateral Additionally, the claimed beneficial effects have
views must include the C7-T1 disc space to been inconsistent (e.g. sensory only, motor only,
ensure the entire cervical spine can be imaged. or other type of neurological recovery) and not
If CT, MRI and dynamic flexion/extension views necessarily clinically/functionally meaningful.
are normal in a symptomatic patient (i.e. most For example, although NASCIS III was a rando-
likely muscle spasm or soft tissue trauma) one mised double-blind trial (without a placebo arm)
can either remove the collar or continue immobi- assessing effect of starting steroids within 8 hours
lization until the patient is reviewed in a few of SCI the only positive results came from an
weeks, at which point the collar can be removed arbitrary post-hoc analysis (i.e. decision to split
without further imaging if the patient has a stable into <3 h and 3-8 h groups, which showed a
and normal neurological examination or repeat 5-point motor improvement at 1 year in the latter
dynamic X-rays if still symptomatic. In obtunded group with a p¼0.053) which cannot be classed as
or comatose patients a high-quality CT scan of level I evidence. In light of both significant meth-
the entire spinal axis is recommended initially (as odological errors and inconsistent neurological
there is a risk of noncontiguous injury that would outcomes, the beneficial effects of MP can as eas-
otherwise remain occult). If the CT scan is normal, ily be ascribed to random chance as to any true
MR imaging within 48 h may identify subtle signs therapeutic effect. In head injured patients, the
of cervical spine injury. If the MR scan is normal or CRASH trial showed the administration of ste-
performed after 48 h, the clinician must determine roids led to a worse outcome and they should
whether to continue cervical collar immobilization not be used in this context either. Where methyl-
on an individual patient basis. prednisolone is still given the accepted dose is a

Neurosurgery Books Full


www.ketabpezeshki.com 66485438-66485457
28 SPINAL TRAUMA AND ACUTE PATHOLOGY 393

30-mg/kg bolus followed by a 5.4-mg/kg/h infu- More contemporary studies suggest that medical
sion for either 24 h (if started <3 h post-injury) or complication rates are actually lower in patients
48 h (3-8 h post-injury). In general, ICU/HDU who undergo early surgery, which allows for ear-
management of patients with an acute cervical lier mobilization and reduced length of intensive
spinal cord injury should include cardiac, hemo- care unit and hospital stay. The nonrandomized,
dynamic, and respiratory monitoring to detect Surgical Timing in Acute Spinal Cord Injury
cardiovascular dysfunction and respiratory insuf- Study (STASCIS), compared 6-month outcomes
ficiency. Hypotension, hypoxemia, pulmonary in those with acute cervical SCI (ASIA A-D) who
dysfunction, and cardiovascular instability, are received surgery within 24 h after injury (n ¼ 131)
frequent despite initial stable cardiac and pulmo- to those whose surgery was performed later
nary function. Life-threatening cardiovascular (n ¼ 91). It found that 19.8% of patients undergo-
instability and respiratory insufficiency may be ing early surgery showed a 2 grade improve-
transient and episodic and may be recurrent ment in ASIA impairment scale compared to
in the first 7-10 days after injury. Prompt treat- 8.8% in the late decompression group. After
ment of these events in patients with acute SCI adjusting for glucocorticoid treatment and injury
reduces cardiac- and respiratory-related morbid- severity, there was a 2.8-fold higher odds of 2
ity and mortality. Hypotension may be due to grade improvement in ASIA impairment scale
hypovolemia, direct severe spinal cord trauma with early surgery (but no difference between
itself, or a combination of the two and contributes groups of patients with only 1 AIS grade improve-
to secondary injury after acute SCI by further ment). Mortality and complications were similar
reducing spinal cord blood flow and perfusion. in both patient groups. The role of early surgery
Correction of hypotension in spinal cord injury with a complete TSCI (ASIA grade A) is debat-
(systolic blood pressure >90 mmHg) and volume able and although surgery to stabilize the spine
expansion have improved ASIA scores in patients is performed it is not immediate. Most clinicians
with acute SCI compared with historical controls. consider deteriorating neurologic function after
Maintenance of mean arterial blood pressure incomplete TSCI to be an indication to perform
between 85 and 90 mmHg for the first 7 days is surgery as early as possible if there are no contra-
safe and may improve spinal cord perfusion and indications (e.g. hemorrhagic shock, blood dys-
ultimately neurological outcome. crasias) hence shorter time intervals (within 6-
12 h) are preferred. Criticisms of the study
FURTHER READING include problems related to baseline differences
Hurlbert RJ, Hadley MN, Walters BC, et al. The acute car- in epidemiological characteristics of the groups
diopulmonary management of patients with cervical spinal under study (patient age, injury morphological
cord injuries. Neurosurgery. 2013;72 Suppl 2:84-92. subtype, steroid administration, and neurological
status at admission), biases related to treatment
5. c—Early decompression (<24 h) was associ- peculiarities (patient allocation by different phy-
ated with significantly more patients with a sicians’ discretion and variations in surgical tech-
2 grade improvement in ASIA impairment nique), and further concerns regarding data
scale at 6-month follow-up analysis (loss of follow-up, specific criteria for
outcomes evaluation—such as AIS >2, and
There are currently no standards regarding the unclearly reported findings).
role, timing, and method of vertebral decompres-
sion in acute spinal cord injury. Options include FURTHER READING
closed reduction using traction and open surgical Fehlings MG, Vaccaro A, Wilson JR, et al. Early versus
procedures. Goals for surgical intervention in delayed decompression for traumatic cervical spinal cord
TSCI include stabilization of the spine (preceded injury: results of the Surgical Timing in Acute Spinal Cord
by closed or open reduction of dislocations if Injury Study (STASCIS). PLoS One. 2012;7(2):e32037.
required) and decompression of neural elements.
Neurologically intact patients are treated nono- 6. a—Inotropes cause an increase in ISP and
peratively unless there is instability of the verte- MAP but with a net increase in SCPP
bral column. Indications for cervical spine
surgery include significant cord compression with Intraspinal pressure (ISP) and spinal cord perfu-
neurologic deficits, especially those that are pro- sion pressure (SCPP) at the site of injury in severe
gressive, that are not amenable or do not respond TSCI (ASIA grade A-C) has been monitored via
to closed reduction, or an unstable vertebral frac- laminectomy and insertion of a pressure trans-
ture or dislocation. Most penetrating injuries ducer between the swollen spinal cord and the
require surgical exploration to ensure that there dura for up to 1 week. After severe TSCI, ISP is
are no foreign bodies imbedded in the tissue, high (typically 20-40 mmHg) and SCPP low (typ-
and also to clean the wound to prevent infection. ically 40-60 mmHg). Interestingly, although

Neurosurgery Books Full


www.ketabpezeshki.com 66485438-66485457
394 PART IV SPINAL NEUROSURGERY

mannitol administration, reduction in PaCO2, variable involvement of the sensory system and
and increase in sevoflurane dose are known to a variable effect on bladder function. Regardless
have a major effect on intracranial pressure, they of the mechanism, nearly 70% of patients suffer-
had little effect on ISP after TSCI. Increasing ing from incomplete spinal cord injuries will have
the dose of inotropes caused an increase in ISP central cord syndrome. Despite this, it is the
and MAP but with a net increase in SCPP. By mechanism and associated degree of instability,
intervening to increase SCPP, we could improve biomechanical failure, urgency of spinal cord
outcome in some patients as assessed using motor decompression, and the need for internal fixation
evoked potentials and a limb motor score. In addi- of a potentially unstable cervical spine which will
tion to bone, dura is a major cause of spinal cord influence management hence early CT and MRI
compression after TSCI and may explain why are recommended. Approximately 10% of
studies of bony decompression without dural patients with ATCCS have MRI evidence of sig-
opening have not convincingly shown a beneficial nal change within the spinal cord with no other
effect on outcome. Spinal decompression com- radiographic abnormality. It is recommended
bined with dural decompression (expansion duro- that these patients be managed medically cardiac,
plasty) safely and effectively improves ISP, SCPP hemodynamic, and respiratory monitoring, and
and spinal cord pressure reactivity after TSCI. maintenance of mean arterial blood pressure at
Compared with the laminectomy group, the lami- 85-90 mmHg for the first week after injury to
nectomy plus duroplasty group had greater improve spinal cord perfusion. Roughly 20% of
increase in intradural space at the injury site and patients present with an acute disc herniation as
more effective decompression of the injured cord. the cause of ATCCS. Surgical intervention is
In the laminectomy+duroplasty group, ISP was recommended for this group. Nearly 30% of
lower, SCPP higher, and sPRx lower, (i.e. patients with ATCCS have cervical spine skeletal
improved vascular pressure reactivity), compared injuries in the form of fracture subluxation inju-
with the laminectomy group. Laminectomy + dur- ries. In this group of patients, early re-alignment
oplasty caused cerebrospinal fluid leak that settled of the spinal column (closed or open) with spinal
with lumbar drain in one patient and pseudome- cord decompression is recommended. The last
ningocele that resolved completely in five group of patients (approximately 40%) have spi-
patients. Change in ASIA grade (ASIA grade at nal stenosis without evidence of bony or ligamen-
follow-up minus ASIA grade at presentation), tous injury and management remains
walking ability, bladder function, and bowel func- controversial due to the variable degree of spon-
tion were better in the laminectomy+duroplasty taneous recovery of neurological function.
versus the laminectomy group, though not signif- Patients with central cord syndrome usually
icant (p < 0.05) and assessed at significantly differ- regain bowel and bladder function and their abil-
ent time points post-injury (10 months vs. ity to ambulate. Return of upper extremity func-
25 months post-injury, respectively). tion is less reliable, and patients are often left with
deficits in their upper extremity, worse distally,
FURTHER READING characterized by "clumsy" hands.
Phang I, Werndle MC, Saadoun S, et al, Expansion duroplasty
improves intraspinal pressure, spinal cord perfusion pressure, Image with permission from Fatterpekar GM, Naidich
and vascular pressure reactivity index in patients with trau- TP, Som PM. The Teaching Files: Brain and Spine,
matic spinal cord injury: injured spinal cord pressure evalua- Elsevier, Saunders, 2012.
tion study. J Neurotrauma. 2015;32(12):865-874.
Werndle MC, Saadoun S, Phang I, et al, Monitoring of spinal FURTHER READING
cord perfusion pressure in acute spinal cord injury: initial find- Aarabi B, Hadley MN, Dhall SS, Gelb DE, Hurlbert RJ, Roz-
ings of the injured spinal cord pressure evaluation study*. Crit zelle CJ, Ryken TC, Theodore N, Walters BC. Management
Care Med. 2014;42(3):646-655. of acute traumatic central cord syndrome (ATCCS). Neuro-
surgery. 2013;72 Suppl 2:195-204.
Phang I, Papadopoulos MC. Intraspinal Pressure Monitoring
in a Patient with Spinal Cord Injury Reveals Different
Intradural Compartments: Injured Spinal Cord Pressure 8. a—CT head, CT angiogram and start
Evaluation (ISCoPE) Study. Neurocrit Care. 2015;23 anticoagulation
(3):414-418.
The incidence of vertebral artery injury may be as
7. b—Central cord syndrome high as 11% after nonpenetrating cervical spinal
trauma in patients meeting specific clinical and
Acute traumatic central cord syndrome is an physical exam criteria. The modified Denver
incomplete spinal cord injury in which the upper Screening Criteria for BCVI are the most com-
extremities are weaker, (at least 10 points in ASIA monly used: lateralizing neurologic deficit (not
Motor Score) than the lower extremities with explained by CT head), infarct on CT head scan,

Neurosurgery Books Full


www.ketabpezeshki.com 66485438-66485457
28 SPINAL TRAUMA AND ACUTE PATHOLOGY 395

cervical hematoma (nonexpanding), massive epi- thromboembolism in several Class III studies,
staxis, anisocoria/Homer’s syndrome, GCS other Class I, Class II, and Class III medical evi-
<8 without significant CT findings, cervical spine dence indicates that better alternatives than low-
fracture, basal skull fracture, severe facial fracture dose heparin therapy exist. These alternatives
(LeForte II or III only), seatbelt sign above clavi- include the use of low molecular weight heparin,
cle, and presence of cervical bruit or thrill. Gold adjusted dose heparin, or anticoagulation in con-
standard of imaging is catheter angiography but junction with rotating beds, pneumatic compres-
there is Class I evidence supporting CTA as a sion devices or electrical stimulation. Oral
highly accurate alternative to catheter angiogra- anticoagulation alone does not appear to be as
phy for screening for VAI in blunt injury trauma effective as these other measures used for prophy-
patients, with a very high negative predictive laxis. There appears to be a DVT prophylaxis ben-
value. It appears that the majority of patients efit to early anticoagulation in acute spinal cord
with VAI are asymptomatic, and those who with injury patients. Class II medical evidence supports
symptomatic VAI have neurological deficit attrib- beginning mechanical and chemical prophylaxis
utable to the initial blunt traumatic injury; no upon admission after SCI and holding chemical
definitive longitudinal study has defined the prophylaxis 1 day prior to and 1 day following sur-
future stroke risk of either of these groups, with gical intervention. The incidence of thromboem-
or without anticoagulation/antiplatelets. While bolic events appears to decrease over time and the
no conclusive medical evidence supports treat- prolonged use of anticoagulant therapy is associ-
ment for VAI, most clinicians support treatment ated with a definite incidence of bleeding compli-
for patients with symptomatic VAI with either cations. There are multiple reports of the
anticoagulation or antiplatelet therapy individual- beneficial effects of the prophylaxis therapy for
ized based on the patient’s vertebral artery inju- 6-12 weeks following spinal cord injury. Class II
ries, associated traumatic injuries, and the medical evidence indicates that the majority of
relative risk of bleeding associated with that form thromboembolic events occur in the first
of therapy. Because of an increased relative risk of 3 months following acute SCI and very few
hemorrhagic complications from anticoagulation occur thereafter. For these reasons, it is recom-
therapy for VAI, without clear superior efficacy, mended that prophylactic therapy be discontin-
anticoagulation therapy is not considered ideal ued after 3 months unless the patient is at high
treatment in multiple trauma patients with either risk for a future VTE event (previous thrombo-
symptomatic or asymptomatic VAI. Antiplatelet embolic events, obesity, advanced age). It is
therapy (aspirin the most studied) appears to be reasonable to discontinue therapy earlier in
a safe and comparable option for symptomatic patients with retained lower extremity motor
patients with VAI after blunt trauma. For asymp- function after spinal cord injury, as the incidence
tomatic patients with documented VAI, no treat- of thromboembolic events in these patients is sub-
ment is comparable to antiplatelet therapy but stantially lower than among those patients with
the potential to reduce future stroke risk favors motor complete injuries. Although the guidelines
the use of aspirin if there are no contraindications. author group concluded that caval filters appeared
to be efficacious for the prevention of PE in SCI
FURTHER READING patients in the 2002 guideline on this topic, more
Rozzelle CJ, Aarabi B, Dhall SS, Gelb DE, Hurlbert RJ, recent medical evidence suggests that prophylac-
Ryken TC, Theodore N, Walters BC, Hadley MN. Spinal tic filters may be more morbid than initially
cord injury without radiographic abnormality (SCIWORA). believed. Caval filters still have a role for SCI
Neurosurgery. 2013;72 Suppl 2:227-233. patients who have suffered thromboembolic
events despite anticoagulation, and for SCI
9. b—Low molecular weight heparin should be patients with contraindications to anticoagulation
discontinued after 3 months post-injury in and/or the use of pneumatic compression devices.
those without other risk factors for VTE
FURTHER READING
Thromboembolic disease is a common occur- Dhall SS, et al. Deep venous thrombosis and thromboembo-
rence in patients who have sustained a cervical spi- lism in patients with cervical spinal cord injuries. Neurosur-
nal cord injury and is associated with significant gery 72:244-54, 2013.
morbidity. Class I medical evidence exists demon-
strating the efficacy of several means of prophy- 10. e—Cord or root damage (2 points)
laxis for the prevention of thromboembolic
events. Therefore, patients with SCI should be In 1990, White and Punjabi described a formula for
treated with a regimen aimed at VTE prophylaxis. evaluating fracture stability in the subaxial cervical
Although low-dose heparin therapy has been spine based on cadaveric studies utilizing radio-
reported to be effective as prophylaxis for graphs. Under normal physiological conditions,

Neurosurgery Books Full


www.ketabpezeshki.com 66485438-66485457
396 PART IV SPINAL NEUROSURGERY

cervical spine movements are smooth, effortless,


SLIC Scale Points
pain-free, and do not produce neurological symp-
toms. One should consider the fact that White Burst +1 (¼2)
and Panjabi’s stability checklist was based on radio-
Distraction (facet perch, hyperextension) 3
graphs (not CT/MRI) and that some suggested
maneuvers, such as stretch testing or dynamic stud- Rotation/translation (facet dislocation, 4
ies, may not be compatible with the present stan- unstable teardrop, advance stage flexion
dards of cervical spine clearance in patients with compression injury)
traumatic brain or cervical spine injuries. The Discoligamentous complex
checklist has never been validated nor its reliability
measured but it remains in use. A total of 5 points or Intact 0
more suggests spinal instability: anterior elements
Indeterminate (isolated interspinous 1
destroyed or unable to function [2 points], posterior widening, MRI signal change only)
elements destroyed or unable to function [2 points],
relative sagittal plane translation >3.5 mm (or Disrupted (widening of disc space, facet 2
>20% AP vertebral width) on X-ray [2 points], rel- perch or dislocation)
ativesagittalplanerotation >11° on X-ray[2points], Neurological status
positive stretch test [2 points], cord damage
[2 points], root damage [1 point], developmentally Intact 0
narrow spinal canal (sagittal <13 mm or Pavlov’s
Root injury 1
ratio <0.8) [1 point], abnormal disc narrowing
[1 point], dangerous loading anticipated [1 point]. Complete Cord Injury 2

Incomplete cord injury 3


FURTHER READING
Aarabi B, et al. Subaxial cervical spine injury classification sys- Continuous cord compression in setting +1 (¼1)
of neurological deficit (neuro modifier)
tems. Neurosurgery. 2013;72:170-186.

11. c—9

The Subaxial cervical spine Injury Classification


(SLIC) and severity scale (0-10) is recommended
FURTHER READING
as a classification system for spinal cord injury. Vaccaro AR, Hurlbert RJ, Patel AA, et al.; Spine Trauma
This system includes morphology of the ana- Study Group. The subaxial cervical spine injury classification
tomical injury, including the discoligamentous system: a novel approach to recognize the importance of mor-
complex and neurological condition of the phology, neurology, and integrity of the disco-ligamentous
patient. The total score in this case was 9. The complex. Spine (Phila Pa 1976). 2007;32(21):2365-2374.
patient was treated with circumferential (anterior Arabi B, et al. Subaxial cervical spine injury classification
cervical discectomy and fusion and posterior systems. Neurosurgery. 2013;72:170-186.
spinal fusion) fusion of the cervical spine. Mor-
phology in this case is eligible for a score of 12. a—MRI is essential before attempted closed
4 (translation/rotation), DLC a score of 2 (com- reduction of cervical fracture-dislocations
plete disruption), and neurology a score of 2 for
complete spinal cord injury (+1 for persistent). Closed reduction of cervical fracture-dislocations
The Cervical Spine Injury Severity Score may obviate surgery and promote neurologic
(CSISS) is limited to clinical trials rather than improvement in some cases. Early reports raised
daily practice. a concern that closed reduction in the setting of
associated disc disruption and/or herniation has
the potential to exacerbate neurologic injury but
Subaxial Cervical Spine Injury studies have shown this not to be true in practice.
Classification and Severity Scale (SLIC) In the clinical scenario of traumatic cervical spine
fractures and cervical facet dislocation injuries,
SLIC Scale Points narrowing of the spinal canal caused by displace-
Morphology ment of fracture fragments or subluxation of one
vertebra over another frequently produces spinal
No abnormality 0 cord compression and injury, necessitating urgent
Compression 1 reduction of the dislocation which may improve
neurologic outcome. Closed reduction of frac-
Continued ture/dislocation injuries of the cervical spine by

Neurosurgery Books Full


www.ketabpezeshki.com 66485438-66485457
28 SPINAL TRAUMA AND ACUTE PATHOLOGY 397

traction-reduction appears to be safe and effective Position, correct bed type, angle of traction,
in awake patients. Approximately 80% of patients X-ray check 15 min post adding weight. Repeat
will have their cervical fracture dislocation injuries CT cervical spine 6-8 weeks.
reduced with this technique. The overall perma-
nent neurological complication rate of closed 14. d—Cervical collars are contraindicated in the
reduction is approximately 1%. The associated risk presence of cranial nerve palsy after unilat-
of a transient injury with closed reduction appears eral fracture
to be 2-4%. Closed traction-reduction appears to
be safer than MUA. Pre-reduction MRI has not OCF is an uncommon injury (1-3% frequency of
been shown to improve the safety or efficacy of OCF in patients sustaining blunt craniocervical
closed traction-reduction of patients with acute trauma) and requires CT imaging to establish the
cervical fracture dislocation injuries, hence may diagnosis. Patients sustaining high-energy blunt
unnecessarily delay spinal column realignment craniocervical trauma, particularly in the setting
for decompression of the spinal cord. The ideal of loss of consciousness, impaired consciousness,
timing of closed reduction of cervical spinal frac- occipito-cervical pain or motion impairment, and
ture dislocation injuries is unknown, but many lower cranial nerve deficits, should undergo CT
investigators favor reduction as rapidly as possible imaging of the craniocervical junction. Magnetic
after injury to maximize the potential for neurolog- resonance imaging (MRI) is recommended to
ical recovery. Patients who fail attempted closed assess the integrity of the craniocervical ligaments.
reduction of cervical fracture injuries have a higher OCFs have been classified by Anderson and Mon-
incidence of anatomic obstacles to reduction, tesano into three types: Type I (comminuted),
including facet fractures and disc herniations. Type II (extension of a linear basilar skull fracture),
Patients who fail closed reduction should undergo and Type III (avulsion of a fragment). Untreated
more detailed radiographic study/MRI before patients with OCF can develop lower cranial nerve
attempts at open reduction. The presence of a sig- deficits that usually recover or improve with non-
nificant disc herniation in this setting is a relative rigid external immobilization (cervical collar).
indication for an anterior decompression proce- Nonsurgical treatment with external cervical
dure, either in lieu of or preceding a posterior pro- immobilization is sufficient to promote bony
cedure. Patients with cervical fracture dislocation union/healing and recovery or cranial nerve deficit
injuries who cannot be examined because of head improvement in nearly all types of OCF. Bilateral
injury or intoxication cannot be assessed for neuro- OCF injuries should prompt consideration for
logical deterioration during attempted closed more rigid external immobilization in a halo vest
reduction. For this reason, an MRI before device. Surgical treatment (occipito-cervical
attempted reduction (open or closed) is recom- instrumented fusion) may be indicated in patients
mended as a treatment option on the basis of Class with OCF who have overt instability, neural com-
III medical evidence. pression from displaced fracture fragments, or who
have associated occipito-atlantal or atlanto-axial
FURTHER READING injuries (e.g. Atlanto-occipital dissociation).
Gelb DE et al. Initial closed reduction of cervical spinal
fracture-dislocation injuries. Neurosurgery 72:73-83, 2013. FURTHER READING
Theodore N, et al. Occipital condyle fractures. Neurosurgery.
13. b—5 lb per level 2013;72:106-113.

This technique involves use of longitudinal 15. a—Associated with a Power’s ratio <0.8
traction using skull tongs or a halo headpiece.
An initial weight of 5-15 pounds is applied; Atlanto-occipital dislocation (dissociation)
this is increased in 5 lb increments, taking lateral accounts for <1% of all acute cervical spine inju-
X-rays after each increment is applied. The more ries. It is usually seen in high-speed motor vehicle
rostral the dislocation, the less weight is used, usu- accidents and results from hyperextension and
ally about three to five pounds per vertebral level. distraction of the cervical spine. Atlanto-occipital
While weights up to 70 pounds are sometimes dislocation is more commonly seen in children
used, we suggest that after 35 pounds is applied, because the pediatric occipital condyles are small,
patients be observed for at least an hour with are almost horizontal, and lack inherent stability.
repeat cervical spine X-rays before the weight is Atlanto-occipital dislocation is often immediately
cautiously increased further. Administration of a fatal because of associated injury to the brainstem
muscle relaxant or analgesic, such as diazepam or and there is a high incidence of neurologic deficits
meperidine, may help facilitate reduction. in survivors. Patients who survive AOD injuries

Neurosurgery Books Full


www.ketabpezeshki.com 66485438-66485457
398 PART IV SPINAL NEUROSURGERY

often have neurological impairment including extension views (plain film and dynamic CT). It
lower cranial nerve deficits, unilateral or bilateral may be orthotopic (moves with the anterior arch
weakness, or quadriplegia. Nearly 20% of patients of C1 on flexion/extension) or dystopic (function-
with acute traumatic AOD will have a normal neu- ally fused to the basion, potentially subluxing
rological examination on presentation. The lack anterior to the arch of C1). Plain dynamic radio-
of localizing findings and/or global neurological graphs in flexion and extension have been used to
deficits from severe brain injury may impede/hin- depict the degree of abnormal motion between
der the diagnosis of AOD in patients with normal- C1 and C2 and narrowest canal diameter. Most
appearing initial cervical radiographs. A high index often, there is anterior instability, with the os
of suspicion must be maintained in order to odontoideum translating forward in relation to
diagnose AOD. Prevertebral soft tissue swelling the body of C2. However, at times, one will see
on a lateral cervical radiograph should prompt either no discernible instability or “posterior
CT imaging to rule out AOD. Commonly used instability” with the os odontoideum moving pos-
radiological parameters suggesting AOD include: teriorly into the spinal canal during neck exten-
craniocervical subarachnoid hemorrhage, Powers sion. The degree of C1-C2 instability identified
ratio (basion-posterior atlas arch distance divided on cervical X-rays does not correlate with the
by the opisthion-anterior atlas arch distance) >1, presence of myelopathy. A sagittal diameter of
basion axial interval or basion dental interval the spinal canal at the C1-C2 level of 13 mm does
>12 mm (Harris rule of 12), Condyle-C1 interval correlate with myelopathy detected on clinical
(highest diagnostic sensitivity in pediatric AOD). examination. MRI can depict spinal cord com-
AOD is classified into Type I (anterior), Type II pression and signal changes within the cord that
(longitudinal), and Type III (posterior) disloca- correlate with the presence of myelopathy. Man-
tions. All patients with AOD should be treated agement is surveillance or surgery based on
with craniocervical fixation and fusion. Without degree of instability, neurological deficits or risk
treatment, nearly all patients developed neurolog- of future spinal cord injury. Patients who have no
ical worsening, many of whom never fully recover. neurological deficit and no instability at C1-C2
Treatment of AOD with traction is associated on flexion and extension studies can be managed
with 10% risk of neurological deterioration and without operative intervention. However, many
external immobilization has a high failure rate. favor operative stabilization and fusion of C1-
C2 instability associated with os odontoideum
Image with permission from Saraf-Lavi E. Spine Imag- because of the increased likelihood of future spi-
ing: Case Review Series, 3rd ed., Elsevier, Saunders,
2014.
nal cord injury following minor trauma (if not
already present). Posterior C1-C2 internal fixa-
FURTHER READING tion with arthrodesis in the treatment of os odon-
Theodore N, et al. The diagnosis and management of trau- toideum provides effective stabilization of the
matic altanto-occipital dislocation injuries. Neurosurgery. atlantoaxial joint in the majority of patients. Neu-
2013;72:114-126. ral compression in association with os odontoi-
deum has been treated with a reduction of
16. d—Posterior C1-C2 fusion deformity, dorsal decompression of irreducible
deformity, and ventral decompression of irreduc-
Os odontoideum is an ossicle with smooth cir- ible deformity, each in conjunction with C1-C2
cumferential cortical margins representing the or occipito-cervical fusion with internal fixation.
odontoid process that has no osseous continuity Each of these combined approaches has provided
with the body of C2. The origin of os odontoi- satisfactory results. Odontoid screw fixation has
deum remains debated in the literature with no role in the treatment of this disorder.
evidence for both acquired and congenital causes.
Image with permission from Brecknell JE, Malham GM.
There are 3 groups of patients with os odontoi- Os odontoideum: report of three cases, J Clin Neurosci.
deum: those with occipito-cervical pain alone, 2008;15(3):295-301.
those with myelopathy, and those with intracra-
nial symptoms or signs from vertebrobasilar FURTHER READING
ischemia. Patients with os odontoideum and mye- Rozzelle CJ, et al. Os odontoideum. Neurosurgery.
lopathy have been subcategorized further into 2013;72:159-169. Doi:10.1227/NEU.0b013e318276ee69.
those with transient myelopathy (commonly after
trauma), those with static myelopathy, and those 17. d—C1-C2 instrumented fusion
with progressive myelopathy. It is usually found
on imaging for other causes, and plain cervical Fractures of C1 are usually classified by the
spine radiographs are sufficient to obtain a diag- Landell’s and Von Petegham classification.
nosis but will necessitate dynamic flexion/ A central issue in the management of atlas

Neurosurgery Books Full


www.ketabpezeshki.com 66485438-66485457
28 SPINAL TRAUMA AND ACUTE PATHOLOGY 399

fractures has been the importance placed on the Consideration of the potential complications of
integrity of the transverse atlantal ligament. Cri- halo immobilization, particularly in the elderly,
teria proposed to determine transverse atlantal lig- is suggested and must be balanced against the
ament injury with associated C1-C2 instability potential morbidity/mortality associated with sur-
include the sum of the displacement of the lateral gical treatment for these fracture injuries.
masses of C1 on C2 of 6-9 mm on a plain open-
mouth X-ray (or 8.1 mm, the rule of Spence cor- Image A with permission from Naidich TP. Imaging of
rected for magnification), a predental space of the Spine, Elsevier, Saunders, 2011. Image B with per-
>5 mm in adults, and evidence of transverse atlan- mission from Dane B, Bernstein MP. Imaging of Spine
Trauma, Seminars in Roentgenology, Elsevier, 2016,
tal ligament disruption or avulsion on MRI. in press.

FURTHER READING
Ryken T, et al. Management of isolated fractures of the atlas in
Management of Atlas (C1) Fractures adults. Neurosurgery. 2013;72:127-131.

Atlas (C1)
Fracture Treatment
Type Description Options
ANSWERS 18–37
Type I Anterior or Collar 8-12
posterior arch weeks Additional answers 18–37 available on
fractures ExpertConsult.com

Type II Burst fracture of If stable: Collar


anterior and or Halo (10-12
posterior arch at weeks)
three or more If unstable: Halo
points (12 weeks) or EMI ANSWERS
(Jefferson’s C1-C2
fracture is 4 point stabilization
break). and fusion
38. 1—h, Flexion “teardrop” fracture, 2—a,
Transverse Atlanto-occipital dissociation, 3—l, Unilat-
atlantal ligament eral facet dislocation.
which is intact
(stable) or 39. 1—f, Flexion-distraction, 2—e, Flexion
disrupted
(unstable) and axial compression, 3—a, Axial com-
pression, 4—c, Extension and axial
Type III Unilateral C1 Comminuted: compression
lateral mass Collar or Halo
fracture 8-12 weeks Although there may be discrepancies in terminol-
Transverse
process: Collar ogy depending on classification system, main
causes are described below.

Initial Management of Spinal Fractures

Upper cervical spine

Distraction Atlanto-occipital Unstable Occipito-cervical fusion

Rotation Atlanto-axial subluxation Potentially Closed reduction and Halo immobilization


(rotatory fixation) Unstable for 8-10 weeks

Axial compression Jefferson’s fracture Unstable Halo immobilization

Hyperextension and Hangman’s fracture Unstable Halo immobilization


axial compression

Flexion (or less Odontoid peg fracture Type II and Halo immobilization (surgery in Type II if
commonly extension) III unstable >50-years old)

Continued on following page

Neurosurgery Books Full


www.ketabpezeshki.com 66485438-66485457
400 PART IV SPINAL NEUROSURGERY

Initial Management of Spinal Fractures (Continued)


Subaxial cervical spine

Axial compression Burst fracture Unstable Anterior and posterior instrumented fusion

Hyperflexion Clay shoveler’s (spinous Stable Cervical collar


process avulsion)

Flexion and axial Compression (wedge) Stable Cervical collar


compression

Flexion and axial Flexion teardrop Unstable Open reduction and instrumented fusion
compression (severe)

Flexion-distraction Facet dislocation; Unstable Open reduction and instrumented fusion


posterior ligamentous
injury

Extension Extension teardrop Stable Conservative or orthosis


(avulsion)

Spinous process/lamina Stable Cervical collar


fracture

Extension and axial Lateral mass/facet Stable or Stable—cervical collar 6 weeks


loading fracture unstable Unstable—closed or open reduction
and anterior/posterior/combined
stabilization

Thoracolumbar spine

Axial compression Burst Unstable TLSO or Surgical stabilization

Flexion and axial Compression (wedge) Stable Conservative or TLSO


compression

Flexion and axial Flexion teardrop Unstable Open reduction and surgical stabilization
compression (severe)

Extension Extension teardrop Stable Conservative or TLSO


(avulsion)

Flexion Chance fracture Unstable Surgical stabilization

Flexion-distraction Facet dislocation; PLC Unstable Open reduction and surgical stabilization
injury

Shear/rotation Translocation (fracture Unstable Open reduction and surgical stabilization


dislocation)

40. 1—j, Halo immobilization, 2—a, Anterior A number of radiological lines and spaces are used
corpectomy and combined anterior- when assessing a rheumatoid cervical spine and
posterior instrumented fusion relate to atlantoaxial subluxation and cranial set-
tling (or basilar invagination). They include:
41. 1—h, The Pavlov (or Torg) ratio, 2—k,
Space available for the cord (SAC), 3—j,
Ranawat’s line

Neurosurgery Books Full


www.ketabpezeshki.com 66485438-66485457
28 SPINAL TRAUMA AND ACUTE PATHOLOGY 401

Radiological Lines Used to Assess Instability at the Craniocervical Junction


Line Description Utility

Power’s ratio Ratio of BC line (basion to anterior lamina Ratio 1 is normal


of C1) to AO line (Posterior C1 anterior arch If >1.0 concern for anterior dislocation. If
to posterior lip of foramen magnum) ratio <1.0 raises concern for posterior
atlanto-occipital dislocation, odontoid
fractures, ring of atlas fractures

Atlantodens interval Gap between anterior C1 arch and C2 Instability is present when there is a
(ADI) dens. Assessed on both flexion and 3.5-mm difference on each view; a 7-mm
extension views difference suggests disruption of the alar
ligaments; a difference of more than 9-
mm is associated with an increase in
neurological injury and indication for
surgery

Wackenheim’s (clivus) A line extended inferiorly from the slope of Normally should pass through C2 dens
line the clivus or be tangential to it. Abnormal in AOD or
basilar invagination.

Space available for the Sagittal diameter of spinal canal. May Less than 14 mm is considered a risk
cord (SAC) change with flexion/extension if instability factor for neurological injury and
present. indication for surgery

Ranawat’s line Center of the C2 pedicle to the C1 posterior Normally 17 mm; a distance less than
arch 13 mm would suggest cranial settling

McRae’s line Foramen magnum line drawn from the The tip of the dens should not cross this
anterior margin of the foramen to the line. If it does it suggests basilar
posterior margin invagination

McGregor’s line Hard palate to the posterior occipit curve If tip of dens >4.5 mm above line
suggests basilar invagination

Chamberlain’s line Posterior end of the hard palate to the Normally the tip of the dens is no more
posterior lip of the foramen magnum than 3 mm above this line. >3 mm
suggest basilar invagination

Pavlov (torg) ratio Ratio of sagittal canal diameter to total Normally equals 1. A value less than 0.80
vertebral body width is considered a risk factor for
neurological injury after minor trauma
such as hyperextension.

Basion dens interval Gap between basion and tip of dens Harris rule of 12: If BDI or BAI >12 mm
(BDI) or suggest atlanto-occipital dissociation
or Gap between basion and posterior C2
Basion axial interval (axial) line
(BAI)

C1-Condyle interval Distance between upper border of C1 and Increased in atlanto-occipital


(CCI) occipital condyle dissociation

Sum of lateral Sum of overhang of C1 lateral masses on If >8.1 mm in adults then a transverse
mass displacement C2 ligament rupture is assured or C1
fracture and the injury pattern is
considered unstable

Neurosurgery Books Full


www.ketabpezeshki.com 66485438-66485457
402 PART IV SPINAL NEUROSURGERY

Anterior rim of
foramen magnum (clivus)
Posterior rim of
foramen magnum
Wackenheim

Chamberlain
McRae

McGregor
Hard palate
Ranawat

SAC

ADI

BAI
B O

A C
C1 C1
C1C2

PAL C2C3

Image A with permission from Miller MD, Thompson SR. Miller's Review of Orthopaedics, 7th ed., Elsevier, 2016;
Images B and C with permission from Winn, HR, Youman’s Neurological Surgery, 4-Volume Set, 6th ed., 2011,
Elsevier, Saunders.

Neurosurgery Books Full


www.ketabpezeshki.com 66485438-66485457
CHAPTER 29

DEGENERATIVE SPINE
SINGLE BEST ANSWER (SBA) QUESTIONS
CERVICAL c. Cervical traction systems are generally
recommended
1. A 56-year-old female presents with neck pain d. Use of soft cervical collars should not
worsened by activity over the last 6 months. exceed 3 months
On examination, she has full power bilater- e. Cervical epidural injections are more
ally in the upper and lower extremities. She effective in those with neck pain and
has a normal gait and no difficulties with radiculopathy
manual dexterity. Which one of the follow-
ing is the most appropriate next step in 3. A 41-year-old male presents with left arm
management? pain of 4 weeks duration. On examination
there is weakness of triceps and wrist flexion.
Axial MRI is shown. Which level is the
pathology shown likely to be at?

a. C4/5
b. C5/6
a. Posterior laminectomy C5-C7 c. C6/7
b. Physiotherapy d. C7/T1
c. Cervical epidural injection e. T1/T2
d. C5/C6 ACDF
e. C5/C6 foraminotomy 4. Which one of the following statements
regarding radiculopathy is most accurate?
2. Which one of the following statements a. Hoffman’s sign is suggestive of C5
regarding nonoperative management of cer- radiculopathy
vical disc and degenerative disorders is most b. Spurling’s test involves rotation, exten-
accurate? sion and axial compression applied to
a. Nonsteroidal anti-inflammatories should the cervical spine
be avoided
b. Neck pain related to cervical spondylosis
improves in only one third of patients

403
Neurosurgery Books Full
www.ketabpezeshki.com 66485438-66485457
404 PART IV SPINAL NEUROSURGERY

c. Cervical foraminotomy is indicated in


radiculopathy secondary to posterolateral
disc protrusion
d. ACDF is not appropriate for patients pre-
senting with radiculopathy and neck
pain alone
e. Shoulder abduction test is positive if radi-
culopathic pain worsens

5. A 65-year-old male presents to your office


with difficulty ambulating and buttoning
his shirt. It started 2 years ago but has wors-
ened significantly over the last year. On
examination he is unable to perform a tan- a. C3-C6 laminectomy with lateral mass
dem gait and is Hoffman's sign positive bilat- screw fixation
erally, but has flexor plantar reflexes b. C3-C6 ACDF without instrumentation
bilaterally. He has 4/5 power in his hands, c. Cervical epidural steroid injection
but 5/5 power in all other muscle groups. d. C3-C6 laminectomy
MRI sequences are shown. Which one of e. C3-C6 bilateral foraminotomies
the following is the most appropriate next
step in management? 7. A 68-year-old female presents with progres-
sive loss of ability to ambulate and dexterity
problems with her hands. Six months ago
she was able to walk with a cane, but now
has difficulty with ambulating with a walker.
She also reports difficulty with her hands and
needs assistance with eating. Physical exam
shows limited neck extension. Flexion exten-
sion X-rays do not show any dynamic insta-
bility. MRI is shown. Which one of the
following is the most appropriate next step
in management?
a. C5/6 ACDF
b. C5/6 and C6/7 ACDF
c. C6 corpectomy
d. C5-C6 laminectomy
e. C5-C6 laminectomy and instrumented
fusion

6. A 55-year-old man presents to your office


with difficulty ambulating and buttoning
his shirt. It started 2 years ago but has wors-
ened significantly over the last year. On phys-
ical exam he is unable to perform a tandem
gait and has a positive Hoffman's sign bilater-
ally, however he has no ankle clonus and
flexor plantars bilaterally. He has 4/5
strength in his hands, but 5/5 strength in
all other muscle groups. Sagittal MRI is
shown. Which one of the following is the
most appropriate next step in management?

a. C5-C6 posterior decompression and


instrumented fusion
b. Physiotherapy and NSAIDs

Neurosurgery Books Full


www.ketabpezeshki.com 66485438-66485457
29 DEGENERATIVE SPINE 405

c. C5-C6 laminoplasty
d. C5-C6 decompressive laminectomy
e. C5/6 and C6/7 anterior cervical decom-
pression and fusion with anterior plate
fixation

8. A 44-year-old presents with worsening gait


and loss of fine motor control in his hands.
Examination reveals normal cranial nerve
examination, negative jaw jerk, hyperreflexia
in all four limbs, positive bilateral Hoffman’s
signs and Babinski positive. There is no evi-
dence of dynamic instability on flexion/
extension radiographs. MRI shown below. a. Posterior foraminotomy
Which one of the following is the most b. Anterior cervical decompression
appropriate next step in management? c. C6 laminectomy
d. C6 arthroplasty
e. C6 laminectomy and C5-C7 instrumen-
ted fusion

10. A 55-year-old presents with 4 months of gait


impairment and sustains a fall. On examina-
tion, there is spastic tetraparesis 4-/5 globally
without evidence of any cerebellar signs.
MRI cervical spine is shown. Which one of
the following is the most appropriate next
step in management?

a. C3-C6 laminectomy
b. C3-C6 unilateral foraminotomy
c. C3-C6 bilateral foraminotomy
d. C3-C6 anterior cervical decompression
e. C3-C6 microdiscectomy
a. C3-C7 laminectomy
9. A 42-year-old with a 3-month history of b. C3-C7 laminoplasty
shooting pains down his left arm which is c. C3/4, C4/5.C5/6, and C6/7 ACDF
still severe 7/10 and not improved with d. C3-C5 corpectomy, srut graft, plates
physiotherapy or epidural steroid injection. and screws
On examination, Spurling’s sign is positive e. C3-C6 corpectomy, srut graft, plates
(left side), left wrist extensor weakness and screws with resection of posterior
4+/5, decreased sensation to pinprick in left longitudinal ligament
C6 distribution. No hyperreflexia or
Babinski sign. Cervical MRI shown below.
Flexion/extension views do not show any
evidence of instability. Which one of the
following is the most appropriate next step
in management?

Neurosurgery Books Full


www.ketabpezeshki.com 66485438-66485457
406 PART IV SPINAL NEUROSURGERY

11. Postoperatively following C3/4 ACDF a SpO2 is 79% and falling. Which one of the
patient notices decreased sweating on one following is the most appropriate next step
side of her face and slight eyelid droop on in management?
the same side. Which one of the following a. Stat neck X-ray to look for cage migration
is most likely? b. Urgent neck CT
a. Retraction injury to vagus nerve in carotid c. Immediately reopen neck incision at the
sheath bedside
b. Intraoperative stroke due to disruption of d. Intramuscular adrenaline
carotid plaque e. CT pulmonary angiogram
c. Injury to sympathetic chain along longus
colli muscle 15. A 52-year-old female underwent a C5/6
d. Traction injury to cutaneous cervical sen- ACDF for cervical radiculopathy through a
sory nerves giving reduced sensation of left-sided approach 2 years ago. She has
sweating had an altered voice since this operation.
e. Thermal injury to branches of facial nerve Recently, the patient has developed myelo-
within the parotid gland pathic symptoms including gait instability
and dexterity problems with her hands. Flex-
12. Which one of the following have been asso- ion extension X-rays do not reveal any
ciated with reduced risk of recurrent laryn- dynamic instability. MRI is shown. Laryn-
geal nerve palsy? goscopy demonstrates abnormal function of
a. Left sided neck dissection the vocal cords on the left hand side. Which
b. Deflating ETT pressure after retractor one of the following is the most appropriate
insertion next step in management?
c. Same side approach for revision surgery
d. Sharp dissection during exposure
e. Anterior cervical plate fixation

13. A 54-year-old female undergoes an C6/7


ACDF via right sided approach and does
not have any new postoperative deficits.
Three days later, she develops burning pain
to her left shoulder region followed by loss
of shoulder abduction and weakness of elbow
flexion. Post-operative MRI of the cervical
spine was unremarkable. Which one of the
following is most likely?
a. C3/4 and C4/5 ACDF via right sided
a. Cage migration
anterior approach
b. Epidural hematoma
b. C3/4 and C4/5 ACDF via left sided ante-
c. Traction injury to brachial plexus from
rior approach
taping of shoulders
c. C3-C7 laminectomy with instrumented
d. Parsonage-Turner syndrome
fusion
e. C7 palsy
d. C3-C7 laminoforaminotomy
e. C3-C5 laminoplasty
14. Six hours after a C5-C6 ACDF, the nurse
calls you that the patient is having difficulty
swallowing and breathing. By the time you
arrive to the bedside, the patient is in distress,
SOB, tachycardic, tachypneic, despite being
on a nonrebreathe oxygen facemask. The
hemovac drain reservoir showed minimal
bloody drainage and the tubing appeared to
be clotted and his neck is extremely tense
and swollen. She appears cyanosed and

Neurosurgery Books Full


www.ketabpezeshki.com 66485438-66485457
29 DEGENERATIVE SPINE 407

16. A 73-year-old female presents with neck pain a. Transoral odontoid resection may be
and clumsiness in her hands. Past medical required in reducible migration
history includes rheumatoid arthritis. On b. Cervicomedullary angle < 135° suggests
exam she has 4 +/5 power in the lower limbs, impeding neurological compromise
hyperreflexia and extensor plantars. Flexing c. Occipitocervical (O-C2) fusion is usually
her neck produces an electric shock-like sen- appropriate if deformity reduction if
sation down her spine. Which one of the fol- possible
lowing is the most appropriate next step in d. Ranawat C1-C2 index is the most repro-
management? ducible radiological sign
e. The tip of the dens is >4.5 mm above
McGregor’s line

18. A 29-year-old male presents with numbness


and tingling in his lower extremities and gait
instability for 2 weeks. Physical exam shows
3+ brisk patellar reflexes. MRI is shown.
Which one of the following is the LEAST
appropriate next step in management?

a. Rigid collar for 6-12 weeks


b. Halo immobilization for 12 weeks
c. Transoral odontoid resection
d. Anterior odontoid screw fixation
e. Posterior C1-C2 fusion

17. Which one of the following statements


regarding the condition shown is LEAST
accurate?

a. Laminectomy and discectomy


b. Anterior thoracic discectomy
c. Transpedicular approach and discectomy
d. Transcostovertebral approach and
discectomy
e. Costotransversectomy and discectomy

19. A 35-year-old female presents for evaluation


of new onset lumbar spine pain. Which one
of the following physical exam findings is
indicative of an organic cause of low back
pain symptoms?
a. Superficial and nonanatomic tenderness
b. Pain with axial compression of the
lumbar spine
c. Negative straight-leg raise with patient
distraction
d. Regional disturbances which do not fol-
low a logical dermatomal pattern
e. Nocturnal pain in the thoracic spine

Neurosurgery Books Full


www.ketabpezeshki.com 66485438-66485457
408 PART IV SPINAL NEUROSURGERY

20. A 35-year-old female presents for evaluation 36. Neurological signs and tests:
of new onset lumbar spine pain. Which one a. Abdominal reflex
of the following is a yellow flag for back pain? b. Adam’s test
a. Recent history of violent trauma c. Babinski reflex
b. Constant, progressive, nonmechanical pain d. Bowstring test
c. Reduced activity levels due to avoidance e. Bulbocavernosus reflex
d. Focal kyphosis f. Femoral stretch test
e. Unexplained weight loss g. Finger escape sign
h. Hoffman’s test
i. Inverted radial reflex
QUESTIONS 21–34 j. Lasegue’s test
k. Lhermitte’s sign
Additional questions 21–34 available on
ExpertConsult.com
l. Oppenheim test
m. Schober’s test
n. Spurling’s test

For each of the following descriptions, select the


EXTENDED MATCHING ITEM (EMI) most appropriate answers from the list above.
Each answer may be used once, more than once
QUESTIONS or not at all.
1. Snapping or flicking the middle fingernail
35. Back pain: results in flexion of thumb.
a. Ankylosing spondylitis 2. Supinator reflex elicits finger flexion only
b. Cauda equina syndrome 3. Scratch along the crest of the patient's tibia
c. Degenerative lumbar disc disease in a downward motion produces extensor
d. Destructive spondyloarthropathy plantar response
e. Diffuse idiopathic skeletal hyperostosis 4. It is one of the first reflexes to return after
(DISH) spinal shock
f. Mechanical back pain
g. Ossification of the posterior longitudinal 37. Intervertebral disc disease:
ligament (OPLL) a. Central disc
h. Osteoporosis b. Disc bulge
i. Rheumatoid arthritis c. Disc extrusion
j. Spinal stenosis d. Disc protrusion
k. Spondylolisthesis e. Far lateral disc
l. Vascular claudication f. Herniated disc
g. Migrated disc
For each of the following descriptions, select the h. Neural foraminal disc
most appropriate answers from the list above. i. Paracentral disc
Each answer may be used once, more than once j. Sequestered disc
or not at all.
1. A 68-year-old male presents with buttock For each of the following descriptions, select the
and bilateral leg pain made worse by walk- most appropriate answers from the list above.
ing and relieved on bending forward. Each answer may be used once, more than once
2. A 70-year-old diabetic male presents with or not at all.
back pain and stiffness. Plain radiographs 1. A herniated disc where the base of the her-
of the thoracolumbar spine showed the niation is smaller than the anterior-
presence of nonmarginal osteophytes at posterior dimension
three successive levels. 2. A herniated disc where the base of the her-
3. A 48-year-old hemodialysis patient pre- niation is wider than the anterior-posterior
sents with back pain. Radiographs of the dimension
thoracolumbar spine showed a destruction 3. Extruded disc material that has broken
of three adjacent vertebrae and two away from the site of extrusion
intervening discs. 4. Extruded disc material extending in the cra-
4. A 38-year-old male presents with an insid- niocaudal plane but still in continuity
ious onset of back pain. Spine radiographs with disc
revealed the presence of vertical osteo-
phytes giving a “bamboo” appearance.

Neurosurgery Books Full


www.ketabpezeshki.com 66485438-66485457
29 DEGENERATIVE SPINE 409

SBA ANSWER
1. b—Physiotherapy 2. e—Cervical epidural injections are more
effective in those with neck pain and
Cervical spondylosis is characterized by degenera- radiculopathy
tion of the disc, both facet joints and both uncover-
tebral joints of the cervical motion segment most Many cases of acute neck pain may arise from soft
commonly at C5/6 and C6/7 levels where the tissue sprains and muscle strains, but ongoing
majority of flexion/extension occurs. Risk factors neck pain is more suggestive of a spondylotic
include age (5th decade onwards), males, excessive source. The natural histories of most nonmyelo-
driving, smoking, lifting, and professional athletes pathic spondylotic cervical disorders are statisti-
(e.g. jockeys, rugby, gymnastics). It involves disc cally favorable, with 40-50% becoming pain
degeneration (loss of height, bulging, and hernia- free/no recurrence and 20-30% getting worse/
tion), joint degeneration (uncinate spurring, facet persisting (i.e. 70-80% improve to varying
hypertrophy), ligamentous changes (thickening, degree). Modifiable factors have been identified,
infolding, bowstringing), and kyphosis. Presenta- including smoking, obesity, occupational hazards,
tion is usually with discogenic neck pain, radiculo- and psychological factors. Initial treatment of
pathy or myelopathy. Nerve root compression acute pain can include a brief trial of rest and
may be due to foraminal stenosis due to spondylo- immobilization with a soft cervical collar. Medi-
tic changes (e.g. chondrosseous spurs of facet cations including narcotics, NSAIDs, oral ste-
and uncovertebral joints), posterolateral disc her- roids, and antidepressants can be beneficial.
niation or disc-osteophyte complex in the lateral Although short-term (<2 weeks) use of cervical
recess, and foraminal soft disc herniation. Cervical collars may be beneficial, prolonged immobiliza-
cord compression due to central canal stenosis tion should be avoided to prevent atrophy of the
leads to a clinical picture of myelopathy and occurs cervical musculature. Traction (at home) should
with a canal diameter <13 mm (normal is 17 mm), be avoided in myelopathic patients to prevent
worse during neck extension when the central cord stretching of a compromised spinal cord. Partic-
becomes pinched between degenerative disc (ante- ipation in an active rehabilitation protocol seems
riorly) hypertrophic facets and infolded ligamen- much more likely to be successful than use of pas-
tum (posteriorly). X-rays may show degenerative sive modalities. Cervical manipulation should not
changes of uncovertebral and facet joints, osteo- be undertaken without an adequate radiographic
phyte formation, disc space narrowing, vertebral examination to screen for potential instability,
endplate sclerosis, sagittal canal diameter given complications that include radiculopathy,
<13 mm and spondylolisthesis but often do not myelopathy, spinal cord injury, and vertebrobasi-
correlate with symptoms (70% of patients by lar artery injury. Cervical epidural injections, or
70 years of age will have degenerative changes). selective root blocks, help most in those with neck
Flexion and extension views should be assessed pain and radiculopathy but it is unclear whether
for instability and compensatory subluxation above they alter the natural history of radiculopathy
or below the spondylotic segment, and oblique or surgical management. Complications of cervi-
views for foraminal stenosis. MRI can assess status cal steroid injections are rare but devastating
of soft tissues and identifies neural compression when they occur (dural puncture, meningitis, epi-
(CT myelography in patients that cannot have dural abscess, intraocular hemorrhage, adreno-
an MRI). Nerve conduction studies may help in cortical suppression, and epidural hematoma).
cases where clinical and imaging findings are Patients with myelopathy, severe or progressive
unable to distinguish between central versus neurologic symptoms, or failure to improve with
peripheral causes. Management in initially nono- time are good candidates for surgery whereas
perative (physiotherapy, NSAIDs, and a cervical those with axial neck pain alone from disc degen-
collar) but can be escalated if this fails or the eration are not.
patient develops radiculopathy/myelopathy (e.g.
foraminotomy, laminectomy, anterior cervical 3. c—C6/7 (i.e. C7 radiculopathy)
discectomy).
Pedicle/nerve root mismatch between cervical
Image with permission from Miller, M, Hart, JA, and thoracolumbar spine is that in the cervical
MacKnight JM. Essential Orthopaedics, Saunders, Else- spine a given nerve root exits above the pedicle
vier, 2010.
belonging to its named vertebra (e.g. C6 nerve
root exits at the C5/6 intervertebral foramen).
However, since C8 nerve root exits at C7/T1 level

Neurosurgery Books Full


www.ketabpezeshki.com 66485438-66485457
410 PART IV SPINAL NEUROSURGERY

below this all names nerve roots exit below the Posterior cervical foraminotomy is highly effec-
pedicle of its named vertebrae (e.g. L5 nerve root tive in treating patients with cervical radiculopa-
exits at L5/S1 intervertebral foramen). As such, thy. The approach is effective in decompressing
nerve roots in the cervical spine have a horizontal lateral spinal roots that are compromised by soft
path whereas those in the lumbar spine are more disc herniations or osteophytic spurs. It also
vertically orientated. This is important, because of reduces the risk of iatrogenic injury with anterior
the resultant differential effect of posterolateral approaches. Long-term radiographic follow-up
and foraminal disc herniations in the lumbar spine shows no significant trend toward kyphosis and
but not the cervical spine. For example, in the cer- improved long-term pain scores compared to
vical spine at C5/6 either a posterolateral disc her- nonoperative treatment. Advantages of ACDF
niation or a foraminal disc herniation will cause a include increased fusion rates (with graft inser-
C6 radiculopathy. In contrast, in the lumbar spine tion in the disc space) and decompression of the
at L4/5 a posterolateral disc herniation will affect neural foramina by increasing its cephalocaudal
the traversing nerve root (i.e. L5 nerve root) while dimension. On the other hand, the posterior
foraminal disc will affect the exiting L4 nerve root. approach maintains spinal alignment and does
Central disc prolapses in the cervical or thoracic not require fusion, but increases risk of neck pain
spine will result in myelopathy if significant, (from posterior muscle dissection).
whereas in the thoracolumbar (T12-L2) and
lower lumbar spine they may cause conus medul- 5. a—C5/6 ACDF
laris syndrome (mixture of UMN/LMN signs and
bladder involvement) or cauda equina syndrome The incidence of cervical myelopathy is difficult
respectively. Common patterns of radiculopathy to ascertain due to subtly of early findings and
in the cervical spine include: overlap with features of “old age.” The natural
C5 radiculopathy leads to deltoid and biceps history of spondylotic cervical myelopathy is
weakness, characterized by slow progression in a pattern
C6 radiculopathy leads to brachioradialis and of stepwise deterioration following periods of sta-
wrist extension weakness, ble symptoms. Patients often complain of balance
C7 radiculopathy leads to triceps and wrist issues, numbness and weakness in their hands,
flexion weakness, and and difficulty with fine motor tasks. Examination
C8 radiculopathy leads to finger flexion may reveal Hoffman’s sign and finger escape sign
weakness. in a myelopathic hand, and long tract signs in the
legs (e.g. clonus, extensor plantar response), and
Image with permission from Saraf-Lavi E. Spine Imag- difficulty with tandem gait. Factors that are asso-
ing: Case Review Series, 3rd ed., Elsevier, Saunders,
2014. ciated with worse outcomes with nonoperative
treatment include segmental kyphosis and cir-
4. b—Spurling’s test involves rotation, exten- cumferential spinal cord compression. Clinical
sion and axial compression applied to the classification systems for cervical myelopathy
cervical spine include Nurick, Ranawat, and the Japanese
Orthopaedic Association. Imaging shows loss of
Incidence of cervical radiculopathy was found to the CSF signal around the cord, and intramedul-
be 83 per 100,000 population, with a peak inci- lary hyperintensity on T2 weighted imaging.
dence in the 6th decade of life. While central disc Sometimes myelopathy may be due to dynamic
prolapses can cause myelopathy in cervical/tho- cord compression, still producing high T2 signal
racic spine, or conus medullaris/cauda equina in the cord, without cord compression in the neu-
syndromes in the lumbar spine. While in the lum- tral position hence flexion extension X-rays or
bar spine posterolateral disc prolapse causes a dynamic imaging may be required. In this case,
radiculopathy affecting the traversing nerve root there is extrusion of the C5/6 disc with migration
and far lateral/foraminal disc prolapses affect the caudally and ACDF at this level is indicated.
exiting nerve root, both types of disc herniation
affect the same nerve root in the cervical spine Image with permission from Townsend CM, Beacu-
champ D, Evers BM, Mattox KL (Eds.), Sabiston Text-
(as travel horizontally). The two major provoca- book of Surgery, 19th ed., Saunders, Elsevier, 2012.
tive tests for cervical radiculopathy include the
Spurling test and the shoulder abduction test. 6. a—C3-C6 laminectomy with lateral mass
They state that acute cervical radiculopathy has screw fixation
75% rate of spontaneous improvement with non-
surgical treatment. If surgery is necessary, either This patient has cervical myelopathy with multi-
anterior cervical discectomy and fusion (ACDF) level anterior and posterior compression. Presence
or posterior laminoforaminotomy is warranted. of a rigid kyphotic deformity favors use of an

Neurosurgery Books Full


www.ketabpezeshki.com 66485438-66485457
29 DEGENERATIVE SPINE 411

anterior approach (can correct kyphotic deformity with laminectomy. Pre-operative flexion/extension
as well as decompress cord; posterior surgery is films should be performed as if any evidence of
unable to correct deformity and may make it worse instability this may favor instrumented fixation with
if noninstrumented). Posterior laminectomy and lateral mass screws. Intraoperative concerns include
instrumented fusion is preferable for patients with maintaining MAP >70 mmHg in myelopathic
a lordotic cervical spine and either three or more patients, avoiding dural tears, ensuring sufficient
levels of compression, primarily posterior com- width of the laminectomy (approx. 15 mm), avoid-
pression or diffuse congenital stenosis. ing violation of the facet joints (<50%) and mini-
mizing risk factors for C5 palsy.
Images with permission from (a) Quiñones-Hinojosa A.
Schmidek and Sweet's Operative Neurosurgical Tech- Image with permission from Saraf-Lavi E. Spine Imag-
niques, 6th ed., Saunders, Elsevier, 2012. (b) Du W, ing: Case Review Series, 3rd ed., Elsevier, Saunders,
Zhang P, Shen Y, et al. Enlarged laminectomy and lat- 2014.
eral mass screw fixation for multilevel cervical degener-
ative myelopathy associated with kyphosis, Spine J 14
(1);2014:57-64. 9. a—Posterior foraminotomy

7. e—C5/6 and C6/7 anterior cervical decom- The case describes a left C6 radiculopathy which
pression and anterior plate fixation has failed conservative management, and MRI
shows foraminal stenosis. As such posterior
In patients with cervical myelopathy due to canal foraminotomy is the most appropriate first line
stenosis, surgical approach will be influenced by operative treatment. The goal is to open the
the sagittal alignment of the cervical spine. Lat- intervertebral foramen to decompress exiting
eral cervical radiographs in neutral position and nerve root, which can be done unilaterally or
flexion and extension can identify kyphotic defor- bilaterally at one or more levels for patients with
mity and determine whether it is rigid or not. The radiculopathy. Care must be taken to avoid
C2-C7 angle is determined by intersecting lines resecting over 50% of the facet otherwise this
extended from the posterior borders of the C2 could lead to instability. The other choices are
and C7 vertebral bodies respectively, whereas indicated when myelopathy is present due to
the local kyphotic angle is based on the posterior canal stenosis/disc herniation.
borders of the vertebral bodies that immediately
Image with permission from Czervionke LF, Fenton DS.
flank the kyphotic segment. If patients have sig- Imaging Painful Spine Disorders, 2011 by Mayo Foun-
nificant kyphosis, the spinal cord is draped over dation for Medical Education and Research, Published
the anterior compressive elements, and a poste- by Saunders, Elsevier.
rior approach alone is not recommended. The
mainstay of treatment in most patients with 10. b—C3-C7 laminoplasty
multi-level disease would be laminectomy with
posterior fusion or laminoplasty (if kyphosis is OPLL is a common cause of cervical myelopathy
<10-13°) or a combined anterior and posterior in the Asian population, men >women, C4-C6
approach (if kyphosis is >10-13°). Possible treat- levels Risk factors diabetes obesity high salt-low
ment options in this case could be (1) C5/6 and meat diet poor calcium absorption mechanical
C6/7 anterior cervical decompression and fusion stress on posterior longitudinal ligament. Presen-
with anterior plate fixation, (2) C5 corpectomy tation may be with myelopathy, neck pain or
with ACDF at C5/6 and anterior plate fixation, asymptomatic. Lateral radiographs often shows
or (3) a C5 and C6 corpectomy, anterior plate fix- ossification of PLL important to evaluate sagittal
ation, followed by posterior decompression and alignment of cervical spine. MRI is study of choice
instrumented fusion (any two level corpectomy to evaluate spinal cord compression, whereas CT
needs to be stabilized posterior due to the high will delineate bony anatomy of ossified posterior
rate of graft migration). longitudinal ligament. Given the propensity for
progression, nonoperative management may only
Image with permission from Yeh KT, Lee RP, Chen IH. be indicated in those with mild symptoms and/or
Laminoplasty with adjunct anterior short segment who are not candidates for surgery. Most symp-
fusion for multilevel cervical myelopathy associated tomatic patients will undergo surgery, with ante-
with local kyphosis. J Chin Med Assoc 2015;78(6):364-9.
rior or posterior approaches. Anterior approaches
8. a—C3-C6 laminectomy may involves interbody fusion (limiting changes
in sagittal canal diameter with flexion/extension
This case shows cervica myelopathy due to canal hence best for dynamic myelopathy) or corpect-
stenosis at multiple levels without any kyphotic omy with/without resection of the OPLL (can just
deformity. In this situation, the goal is decompres- be left to float in corpectomy site, and reduces risk
sion of the spinal cord which will only be adequate of dural tear when trying to dissect PLL off thecal

Neurosurgery Books Full


www.ketabpezeshki.com 66485438-66485457
412 PART IV SPINAL NEUROSURGERY

sac) and must be used in those with existing 13. d—Parsonage-Turner syndrome
kyphotic deformity. Posterior laminoplasty or
laminectomy with fusion is only appropriate in The most common nerve injury with anterior and
lordotic cervical spine and is safer and preferable posterior cervical spine surgery is C5 nerve palsy,
approach due to the difficulty of resecting the with an incidence of about 5%; decompressive
OPLL off the dura from an anterior approach. procedures for myelopathy have the highest rate
Where laminectomy (rather than laminoplasty) of this complication. Several theories potentially
is performed instrumented fusion to avoid post- explain nerve root palsies, including direct trauma
operative kyphosis is recommended. or a traction phenomenon from displacement of
the spinal cord after decompression, or segmental
Image with permission from Jandial R, Garfin SR. Best cord gray matter dysfunction. Patients with C5
Evidence for Spine Surgery: 20 Cardinal Cases, Saun-
ders, Elsevier, 2012.
palsy generally present in a delayed fashion
(within 1 week) postoperatively, sometimes even
11. c—Injury to sympathetic chain along longus as long as 1 month postoperatively. The most
colli muscle (post-op Horner’s syndrome) common presentation is with deltoid and biceps
weakness.
Generally, if a patient awakens from surgery
Advantages of Anterior Disadvantages of
with upper extremity weakness, the differential
Approach Anterior Approach should include shoulder traction (diffuse distribu-
tion) caused by positioning or intraoperative
Direct decompression Requires fusion nerve trauma (root and side specific). When pre-
of anterior pathology Increased surgical time,
Muscle sparing and morbidity with
senting in a delayed fashion, the main differentials
approach multiple levels include compressive causes (cage/bone graft
Reduced blood loss, Risk of recurrent migration) which can be excluded on MRI, C5
infection, LOS laryngeal nerve injury nerve palsy and Parsonage-Turner syndrome
High fusion rate Risk of dysphagia (PTS). PTS is typically characterised by severe
Access to multiple Risk of injury to
levels via small incision vascular/visceral pain followed shortly by the onset of weakness,
Good for correction of structures and the weakness, sensory deficit, and pain usu-
kyphotic deformity Access limited to C2-T1 ally do not all correspond to the same nerve root
Transverse incision Visible scar or peripheral nerve distribution. By contrast, the
allows access up to Limited access after
three levels previous surgery/
C5 palsy is predominantly motor disturbance.
radiotherapy EMG is useful at 1-4 weeks after onset to clarify
distribution. If no compression is noted, patients
are treated symptomatically with physical therapy
12. a—Left-sided neck dissection and pain control. Given the lack of deltoid func-
tion and the possibility of a traction phenomenon,
Right recurrent laryngeal nerve ascends in the patients with C5 palsy are given a sling for
neck after passing around the subclavian vessels, comfort (steroids are not routinely given). Most
courses medially and cranially at the C6-C7 level, patients recover within 6 months.
often along with the inferior thyroid artery. In
contrast, the left recurrent laryngeal nerve curves
around the aortic arch and then ascends along the
tracheoesophageal groove in a more midline and
protected position. This has led some to suggest a
left-sided approach is safer especially when lower
cervical segments are approached, but studies Common Complications of Cervical
favoring either a right- or a left-sided approach Spine Surgery
are found in the literature. Arguments supporting Anterior Approach Posterior Approach
a left-sided approach have been based on anatom-
ical factors described and the possible occurrence Hypoglossal nerve injury C2 nerve root injury (C1
of a nonrecurrent inferior laryngeal nerve on the Superior laryngeal nerve lateral mass screws)
injury Third occipital nerve
right, while proponents of a right-sided approach Recurrent laryngeal injury (occipital
have argued that it is more comfortable for right- nerve injury neuralgia)
handed surgeons, that a left-sided approach puts Sympathetic chain injury C5 nerve root palsy
the thoracic duct at risk (at C7-T1 level), and that (Horner’s syndrome 5%
1%) C8 nerve root palsy
the esophagus lies anatomically slightly to the left, C5 palsy 5% (C7/T1 osteotomy)
which renders a right-sided approach safer.

Neurosurgery Books Full


www.ketabpezeshki.com 66485438-66485457
29 DEGENERATIVE SPINE 413

FURTHER READING FURTHER READING


Brown JM, Yee A, Ivens RA, Dribben W, Mackinnon SE. Palumbo MA, et al. Airway compromise due to wound hema-
Post-cervical decompression parsonage-turner syndrome toma following anterior cervical spine surgery. Open Orthop J
represents a subset of C5 palsy: six cases and a review of 6;2012:108-13.
the literature: case report. Neurosurgery. 2010 Dec;67(6):
E1831-43; discussion E1843-4. doi: http://dx.doi.org/10.1227/ 15. c—C3-C7 laminectomy with instrumented
NEU.0b013e3181f8254b. Review. PubMed PMID: 21107152. fusion
14. c—immediately reopen neck incision at Adjacent level disease is a relatively frequent clin-
bedside ical finding after cervical spine surgery (approx.
3%). Whenever possible, nonoperative treat-
One of the most serious adverse events associated ment should be attempted, but it may be less suc-
with anterior cervical spine surgery is postopera- cessful than in de novo cervical spondylotic
tive airway obstruction due to wound hematoma. syndromes. If nonoperative treatment fails, radi-
The reported incidence of this complication has culopathy or myelopathy caused by adjacent level
varied from 0.2% to 1.9%. Hematoma following disease can be treated operatively much in the
anterior cervical spine surgery may be the result of same manner as de novo disease. Relevant con-
inadequate control of arterial or venous bleeding siderations in the operative treatment of adjacent
during the operation, and has been reported due segment disease include anterior versus posterior
to superior thyroid artery dissection. In approaches, fusion versus motion-preserving
other instances, a hematoma can form after sur- procedures, and single-level versus multilevel
gery irrespective of adequate intraoperative surgical procedures. Revision ACDF has good
hemostasis. Postoperative hemorrhage may occur outcomes, but suspected pseudoarthrosis at the
secondary to coagulopathy, increased blood pres- index level and recurrent laryngeal nerve status
sure during emergence from anesthesia, or ele- should be considered. The right and left recur-
vated venous pressure due to the Valsalva effect rent laryngeal nerves innervate the posterior cri-
of coughing at the time of extubation. There are coarytenoid muscles which open the vocal cords.
two potential pathophysiologic mechanisms by Injury to the ipsilateral recurrent laryngeal nerve
which hematoma can produce airway compro- can occur in 1.5-6% of patients after ACDF, with
mise. The first is direct mechanical compression resultant unilateral paralysis of the posterior cri-
leading to reduction in the cross-sectional area coarytenoid muscle. Although paralysis of this
of the airway lumen. The second mechanism muscle unilaterally is usually benign, bilateral
involves the development of intrinsic airway paralysis can lead to severe airway difficulties
edema in response to the mass effect of collected and the need for tracheostomy. If revision sur-
blood within the surgical wound impaired venous gery is planned from the opposite side, the vocal
drainage (more likely as requires lower pressure to cords need to be evaluated with laryngoscopy
obstruct). Although delayed hematoma is possible preoperatively. If there was asymptomatic/occult
beyond the first 12 h, alternative causes of airway (left) RLN injury from the initial surgery, then
obstruction (e.g. pharyngeal/prevertebral edema, the opposite side approach is inadvisable for fear
spinal construct failure, cerebrospinal fluid collec- of developing bilateral vocal cord paralysis and its
tion or retropharyngeal abscess) become more catastrophic complications. Posterior decom-
probable. The primary treatment objective is to pression and fusion can provide a high fusion rate
establish and maintain patency of the airway by and avoid revision ACDF at the index level.
placement of an endotracheal tube—in a noncri- Patients with multilevel spondylotic compression
tical airway this is attempted in the operating may also benefit from a posterior approach to
suite, whereas in the critical airway compromise address multiple levels of disease. Posteriorly
(e.g. air hunger, excessive salivation, a rocking based nonfusion options include laminoforami-
motion of the head and chest with the respiratory notomy for single-level radiculopathic symptoms
cycle, use of accessory muscles of respiration, and laminoplasty for multilevel disease. Contra-
inspiratory stridor and, eventually, central cyano- indications to laminoplasty include cervical
sis) this is done at the bedside. Once definitive air- kyphosis, which does not allow the spinal cord
way control is achieved, the patient can undergo to drift posteriorly and be indirectly decom-
wound exploration in theater. However, if the ini- pressed, and significant preoperative neck pain.
tial intubation attempt fails (or airway manage- The rate of postoperative neck pain after lamino-
ment equipment is not immediately available), plasty is high, and patients should be counseled
the surgical wound should be opened and blood accordingly. Given the relatively recent advent
clot removed at the bedside followed by reassess- of cervical disc arthroplasty, clinical data are cur-
ment of airway and need for further intubation rently not sufficient to recommend for or against
attempts/cricothyroidotomy. arthroplasty at a level adjacent to a fusion. This

Neurosurgery Books Full


www.ketabpezeshki.com 66485438-66485457
414 PART IV SPINAL NEUROSURGERY

case describes myelopathy with evidence of mul- disruption of the transverse ligament following
tilevel disease, both above and below the fused joint inflammation around it. The subluxation
segment, without any kyphotic deformity, but can be anterior, posterior, lateral, or rotatory.
in the presence of an occult left recurrent laryn- An atlantoaxial subluxation occurs in 60-80%
geal nerve palsy. These features favor a posterior of cases of rheumatoid arthritis (RA) as the result
approach from C3 to C7, which could be either of pannus formation at the synovial joints
laminoplasty or laminectomy with instrumented between the dens and the ring of C1. Patients
fusion. with symptomatic instability or no symptoms
but ADI > 10 mm and SAC < 14 mm are gener-
Image with permission from Shen FH, et al. (Eds.), Text- ally managed with operative stabilization. If the
book of the Cervical Spine, Saunders, Elsevier, 2015.
subluxation is reducible, a posterior approach
FURTHER READING and C1-C2 instrumented fixation are used. Occi-
Regan C, Lim MR. Adjacent segment disease. In: Shen FH, pitocervical fusion (O-C2) may be considered in
et al. (Eds.), Textbook of the Cervical Spine, Saunders, this patient population if co-existent basilar
Elsevier, 2015. invagination is present or likely in future, or if
anterior compression from pannus requires C1
16. e—Posterior C1-C2 fusion posterior arch resection. When the subluxation
is not reducible or when it is associated with
Although the occurrence of radiographic evi- anterior pannus compressing the upper cervical
dence of disease as atlantoaxial subluxation in spine, anterior release of odontoid is generally
asymptomatic patients is common, the most fre- required before posterior fusion.
quent presenting symptom is pain. It is usually a
combination of occipital and neck pain that Image with permission from Ogihara N, Takahashi J,
Hirabayashi H, et al. Long-term results of computer-
either is caused by mechanical instability or is assisted posterior occipitocervical reconstruction.
radicular, as a result of compression of C1 and World Neurosurg 2010;73(6):722-8.
C2 nerves. A positive Sharp-Purser test is a click-
ing sensation in extension that results with spon- 17. a—Transoral odontoid resection may be
taneous reduction of atlantoaxial subluxation. required in reducible migration
Neurologic manifestations are less common
and are caused by mechanical neurovascular With RA progression, the atlanto-occipital and
compression on the cervical spine and cervico- atlantoaxial joints and lateral masses are
medullary junction. Patients may present with destroyed, resulting in cranial migration of the
cervical myelopathy manifesting as gait dystaxia, odontoid process and hence “settling” and rheu-
hand clumsiness, and difficulty with dexterity. matoid basilar invagination in 40%. This condi-
Objective findings of myelopathy include weak- tion leads to variable degrees of neurovascular
ness, hyperreflexia, and positive Hoffmann, cervicomedullary compression. Multiplanar CT
Babinski, and Lhermitte signs. Cruciate paralysis and MRI studies that delineate the bony and
and even sudden death from respiratory arrest the neurovascular anatomy, respectively, should
have also been reported. The deep tendon reflex always be used during the workup because they
may not be elicited in RA because of appendicu- also facilitate the diagnosis. Surgery is indicated
lar joint destruction. Anterior subluxation of the when patients are symptomatic, when radio-
atlas on the axis results from weakening and graphic evidence of instability is present, or when

Radiological Features of Atlantoaxial Instability Versus Basilar Invagination


Atlantoaxial Instability/Subluxation Radiological Features Basilar Invagination Radiological Features

ADI difference of >3.5 mm between flexion and extension Ranawat C1-C2 index <13 mm (most reproducible;
suggests instability (not an indication for surgery) normally 17 mm in men, 15 mm in women)
ADI >5 mm is diagnostic of subluxation Tip of dens >4.5 mm above McGregor’s line
ADI difference of >7 mm of motion may indicate disruption Dens is above McRae’s line
of alar ligament
ADI difference of >10 mm motion of associated with Dens >3 mm above Chamberlain’s line
increased risk of neurologic injury and an indication for Cervicomedullary angle <135°
surgery
PADI (SAC) <14 mm associated with increased risk of
neurologic injury and is an indication for surgery
PADI (SAC) >13 mm may predict complete neural recovery
after decompressive surgery

Neurosurgery Books Full


www.ketabpezeshki.com 66485438-66485457
29 DEGENERATIVE SPINE 415

the degree of compression of cervicomedullary Image with permission from Quiñones-Hinojosa A.


junction is severe (cervicomedullary angle Schmidek and Sweet's Operative Neurosurgical Tech-
<135°). The surgical approach depends on the niques, 6th ed., Saunders, Elsevier, 2012.
ability to achieve reduction preoperatively. Flex-
ion and extension imaging is helpful in determin- FURTHER READING
Simmons NE. Surgical techniques in the management of tho-
ing the extent or absence of reduction. Often, racic disc herniations. In: Quiñones-Hinojosa A (Ed.), Schmi-
preoperative traction can also be used in achiev- dek and Sweet's Operative Neurosurgical Techniques, 6th ed.,
ing reduction and is successful in 75% or 80% Saunders, Elsevier, 2012.
of the cases. When reduction occurs, dorsal occi-
pitocervical fusion (O-C2), with or without sub- 19. e—Nocturnal pain in the thoracic spine
occipital decompression, is sufficient. When the
invagination is not reducible, transoral resection Nonorganic signs of low back pain (i.e. Waddell
of the odontoid/pannus should precede dorsal Signs) include superficial and nonanatomic ten-
occipitocervical fusion. derness, pain with axial compression or simulated
rotation of the lumbar spine, negative straight-leg
Image with permission from Saraf-Lavi E. Spine Imag-
ing: Case Review Series, 3rd ed., Elsevier, Saunders,
raise with patient distraction, regional distur-
2014. bances which do not follow a logical dermatomal
pattern, and overreaction to physical examina-
18. a—Laminectomy and discectomy tion. Nocturnal pain is a red flag symptom,
especially considering the majority of spinal
The clinical presentation and imaging studies are metastases are thoracic.
consistent with a thoracic disc herniation with
spinal cord compression causing symptoms of
thoracic myelopathy. This is an indication for sur- FURTHER READING
gery. Thoracic level disc herniations are treated Waddell G, McCulloch JA, Kummel E, Venner RM. Nonor-
with anterior discectomy with or without fusion. ganic physical signs in low-back pain. Spine (Phila Pa 1976) 5
(2);1980:117-25.
• Thoracic disc herniations, although uncom-
mon, are encountered by spine surgeons. 20. c—Reduced activity levels due to avoidance
Relative immobility of the thoracic spine
as compared with the cervical and lumbar Red flags are indicators of serious spinal pathol-
regions and thus the low incidence of degen- ogy: age of onset less than 20 years or more than
erative changes. The majority are T8-T11. 55 years, recent history of violent trauma; con-
• Laminectomy is an unacceptable treatment stant progressive, nonmechanical pain (no relief
for thoracic disc herniations due to high with bed rest); thoracic pain; past medical his-
rates of paralysis and death in the earliest tory of malignant tumor; prolonged use of cor-
published studies (1950-1960s). Since the ticosteroids; drug abuse, immunosuppression,
abandonment of laminectomy for thoracic HIV; systemically unwell; unexplained weight
disc herniations, morbidity and mortality loss; widespread neurological symptoms (includ-
rates have dropped significantly. ing cauda equine syndrome); spinal deformity;
• Posterior techniques include the transpedi- fever. Yellow flags are psychosocial factors indic-
cular, Stillerman’s transfacet pedicle sparing, ative of long-term chronicity and disability: a
transcostovertebral, costotransversectomy, negative attitude that back pain is harmful or
and lateral extracavitary. potentially severely disabling; fear avoidance
• Posterior approaches are generally favored behavior and reduced activity levels; an expecta-
in cases of more lateral, noncalcified, extra- tion that passive, rather than active, treatment
dural disc herniations and in those not fit will be beneficial; a tendency to depression,
enough for an anterior approach (e.g. pul- low morale, and social withdrawal; social or
monary disease. financial problems.
• Anterior approaches include the transtho-
racic (below T4), retropleural, and trans-
sternal (above T4).
ANSWERS 21–34
• Anterior techniques offer better ventral
exposure for discs that are centrally located, Additional answers 21–34 available on
calcified, and/or intradural. ExpertConsult.com
• Complication rates for the common poste-
rior and anterior procedures are similar.

Neurosurgery Books Full


www.ketabpezeshki.com 66485438-66485457
416 PART IV SPINAL NEUROSURGERY

EMI ANSWER skeletal hyperostosis (DISH). Defined by


the presence of nonmarginal osteophytes
35. 1—j, Spinal stenosis. Patients with neuro- at three or more successive levels. 3—d,
genic claudication improve with bending Destructive spondyloarthropathy. Seen in
forward, and have pain radiating from prox- hemodialysis patients with chronic renal
imally to distally. In contrast, claudication failure, it typically involves three adjacent
due to peripheral vascular disease usually vertebrae and their intervening disc spaces.
comes on after walking a fixed distance, 4—a, Ankylosing spondylitis. Vertical or
starts as a cramp or tightness in the calf marginal osteophytes produce a bamboo
and relieved by rest. 2—e, Diffuse idiopathic spine appearance.

36. 1—h, Hoffman’s test; 2—i, Inverted radial reflex; 3—l, Oppenheim test; 4—e, Bulbocavernosus
reflex

Neurological Examination Findings


Test or Sign Technique and Interpretation

Babinski reflex Extensor plantar response on stroking sole of foot suggesting UMN lesion

Spurling’s test Examiner turns the patient's head to the affected side while extending and applying
downward pressure to the top of the patient's head (foraminal compression). A positive
test (Spurling’s sign) is when the pain arising in the neck radiates in the direction of the
corresponding painful dermatome ipsilaterally

Hoffman’s test Quickly snapping or flicking the middle fingernail; if positive, the tip of the index finger, ring
finger, and/or thumb suddenly flex in response, and this indicates cervical myelopathy

Inverted radial Said to be present when the supinator reflex associated with the brachioradialis muscle
reflex elicits finger flexion (abnormal) rather than elbow flexion (normal). This is due pathology
causing to an absent biceps jerk (C5-C6) and an exaggerated triceps jerk (C7). It occurs
because a lower motor neuron lesion of C5 root is combined with an upper motor neuron
lesion affecting reflexes below C5

Finger escape sign Ask the patient to hold fingers extended and adducted. Little finger spontaneously abducts
and flexes due to weak intrinsic muscles indicating cervical myelopathy

Lhermitte’s sign Electric shock-like sensation shooting down the spine when flexing the cervical spine

Oppenheim test Scratch along the crest of the patient's tibia in a downward motion. A normal (negative)
response is no reaction. An abnormal (positive) response is an extensor plantar response
suggesting upper motor neuron lesion

Adam’s test Not reliable in the presence of lower limb length discrepancy. The patient bends forward at
the waist until the back comes in the horizontal plane, with feet together, arms hanging and
knees extended. The palms are held together. The examiner looks from behind, along the
horizontal plane of the column vertebrae. The examiner looks for indicators of scoliosis,
such as spinal asymmetry, nonlevel shoulders, scapula asymmetry, nonlevel hips, the
head that does not line up with the pelvis or a rib hump. An increased or decreased lordosis/
kyphosis can also be a sign for scoliosis. The rotation deformity or rib hump can be
measured with a scoliometer

Schober’s test A mark is made at the level of the posterior iliac spine on the vertebral column, i.e.
approximately at the level of L5. The examiner then places one finger 5 cm below this mark
and another finger at about 10 cm above this mark. The patient is then instructed to touch
his toes. If the increase in distance between the two fingers on the patients spine is less than
5 cm then this is indicative of a limitation of lumbar flexion

Femoral stretch Knee is passively flexed and the hip is passively extended with the patient in the prone
test position. Test is positive if results in anterior thigh pain, most often in L2-L3 and L3-L4 disc
herniation (less so or negative in L4/5 and L5/S1 herniation)

Lasegue’s test Straight leg raise. Positive for herniated disc if the patient experiences sciatica when the
straight leg is at an angle of 30-70°

Continued

Neurosurgery Books Full


www.ketabpezeshki.com 66485438-66485457
29 DEGENERATIVE SPINE 417

Test or Sign Technique and Interpretation


Bowstring test Performed after a positive straight leg raise is elicited; the angle of hip elevation is
decreased to decrease the radicular pain and then pressure is applied to the popliteal fossa,
over the nerve, to reproduce symptoms

Abdominal reflex Stroke on the abdominal skin from lateral to the medial aspect in all four quadrants A
normal positive response usually involves a contraction of the abdominal muscles, and the
umbilicus moving towards the source of the stimulation. Polysynaptic T7-T12 reflex.
Absence can be pathological or physiological (e.g. obesity, multiparity, tolerance, children)

Bulbocavernosus Polysynaptic S2-S4 reflex. Monitoring internal/external anal sphincter contraction in


reflex response to squeezing the glans penis or clitoris, or tugging on an indwelling Foley
catheter. In the context of acute spinal cord injury, absence suggests spinal shock, whereas
presence suggests a severed cord. It is one of the first reflexes to return after spinal shock at
48 h. Absence after lumbar trauma may be due to conus medullaris or cauda equina injury

Shoulder Patients with radiculopathy have improvement of their symptoms with elevation of the arm
abduction test above the head. This is an important test to distinguish cervical pathology from other
sources of shoulder/arm pain

37. 1—d, Disc protrusion; 2—c, Disc extrusion; 3—j, Sequestered disc; 4—g, Migrated disc

Terminology Relating to Intervertebral Disc Pathology

Herniated disc A localized displacement of nucleus, cartilage, fragmented apophyseal vertebral bone, and/
or annular tissue beyond the intervertebral disc space

Disc bulge A disc which extends beyond the edges of the disc space, over greater than 50% (180°) of the
circumference of the disc and usually less than 3 mm beyond the edges of the vertebral body
apophyses

Disc extrusion A herniated disc where the base of the herniation (transverse dimension) is smaller than the
AP dimension

Disc protrusion A herniated disc where the base of the herniation (transverse dimension) is wider than the AP
dimension; >25% but <50% (180°) of disc herniation

Central disc A disc that extrudes or protrudes in the midline

Paracentral disc A disc that extrudes or protrudes just off the midline but still in the spinal canal (i.e. right
central or left central)

Neural foraminal A disc that extrudes or protrudes into the neural foramen
disc

Far lateral disc A disc that extrudes or protrudes anterolateral to the neural foramen outside of the spinal
canal

Sequestered disc A subtype of extruded disc that has broken away from the from the site of extrusion (i.e. free
disc material in canal)

Migrated disc A subtype of extruded disc that extends in the craniocaudal plane but still maintains
continuity with the disc

Neurosurgery Books Full


www.ketabpezeshki.com 66485438-66485457
CHAPTER 30

SPINAL INFECTION
SINGLE BEST ANSWER (SBA) QUESTIONS
1. A 39-year-old presents with 4 days of neck 2. A 54-year-old renal transplant patient presents
pain and unsteady gait. He had recently com- with back pain, fever, and a new thoracic
pleted a course of antibiotics for cellulitis. On kyphotic deformity. She is otherwise neurologi-
examination he was myelopathic, with sensory cally intact on examination. T2W MRI is shown.
impairment below C4 level. Reflexes were Which one of the following is most likely?
brisk globally with upgoing plantars. He is
pyrexial with a temperature of 38.2 °C. Blood
cultures grow Staph aureus. T1 contrast MRI
is shown. Which one of the following is most
likely?

a. Arachnoiditis a. Epidural abscess


b. Epidural abscess b. Intramedullary abscess
c. Epidural lipomatosis c. Pyogenic vertebral osteomyelitis
d. Epidural metastasis d. Spinal subdural empyema
e. Subdural empyema e. Tuberculosis spondylitis

418
Neurosurgery Books Full
www.ketabpezeshki.com 66485438-66485457
30 SPINAL INFECTION 419

3. Which one of the following factors is LEAST c. Multiple myeloma


likely to predict good candidates for nonsurgi- d. Pyogenic vertebral osteomyelitis
cal management of vertebral osteomyelitis? e. Spinal tuberculosis
a. Over the age of 60
b. Immunocompetent 6. Which one of the following statements regard-
c. No neurological deficit ing spinal subdural empyema is most accurate?
d. No kyphotic deformity a. Two thirds are associated with discitis/
e. Cultures positive for Staphylococcus aureus osteomyelitis
f. Primarily discitis (minimal involvement of b. Are commoner than cranial subdural
adjacent vertebrae) empyemas
c. Lumbar puncture should be attempted
4. Which one of the following statements about before proceeding to open surgery
surgical management of vertebral osteomyeli- d. MRI is definitive in distinguishing subdural
tis is LEAST accurate? empyema from epidural abscesses
a. Simple laminectomy is the primary pro- e. Commonly occur in adolescents
cedure for emergency management of
neural element compression in vertebral 7. A 19-year-old male presents with neck pain,
osteomyelitis fever, and numbness in his legs and trunk up
b. Anterior or combined anterior-posterior to his arm pits. His neurological examination
approaches are required in the majority is consistent with an upper motor neuron
of cases lesion, and you note a small midline sinus on
c. Posterior approach alone can be considered the back of his neck. Blood cultures grow
if there is disc space infection below the Streptococcus epidermis, T1 MRI with contrast
conus without anterior column instability is shown. Which one of the following is the
d. Refractory back pain after a period of non- next appropriate step?
operative management is an indication for
surgery
e. Correction of progressive spinal deformity
is an indication for surgery

5. A 32-year-old recent immigrant from Southeast


Asia presents with back pain and a new kyphotic
deformity after tripping up in his flat. T1 and T2
W MRI are shown. Which one of the following
is most likely diagnosis?

a. MRI head with contrast


b. CSF analysis
c. Cardiac echo
A B d. Dental examination
e. Start IV antibiotics immediately
a. Ankylosing spondylitis
b. Discitis

Neurosurgery Books Full


www.ketabpezeshki.com 66485438-66485457
420 PART IV SPINAL NEUROSURGERY

SBA ANSWERS
1. b—Epidural abscess Image with permission from Herkowitz et al. Rothman-
Simeone The Spine, 6th ed., Elsevier, Saunders, 2011.
Epidural abscess can result from hematogenous
FURTHER READING
spread, local extension, or direct inoculation. Devlin VJ, Steinmann JC. Spinal infections. In: Devlin VJ
This condition is usually found in adults; risk fac- (Ed.), Spine Secrets Plus, 2nd ed., Elsevier, Mosby, 2012
tors include intravenous drug abuse, diabetes (Chapter 67).
mellitus, prior spine trauma, renal failure, and
pregnancy. The majority of cases are located in 2. c—Pyogenic vertebral osteomyelitis
the thoracic spine. Causative organisms are S.
aureus (70%), other staphylococcal species, aero- Pyogenic vertebral osteomyelitis accounts for
bic streptococci, Enterobacteriaceae (mainly E. coli) 2-7% of all osteomyelitis, and at-risk groups
Pseudomonas species mixed bacterial infections include the elderly, diabetics, AIDS, IV drug
and fungi. The initial presentation includes local- abusers, and the immunosuppressed. The most
ized pain and fever with elevation of the ESR, common method for bacteria to spread to the
CRP, and leukocyte count. Blood cultures are spine is by the hematogenous route. Common
positive in 60% of patients. MRI is able to visual- sources of infection include infected catheters,
ize the degree of cord compression and extent of urinary tract infection, dental caries, IV drug
abscess in all directions, and discitis/vertebral use, and skin infections. The next most common
osteomyelitis which commonly accompanies it. route is local extension from an adjacent soft tis-
Areas of infection have characteristically high sig- sue infection or paravertebral abscess, followed
nal intensity on T2-weighted image. Without by direct inoculation via trauma, puncture, or fol-
treatment, significant neurologic deficits occur lowing spine surgery. The nucleus pulposus is rel-
and eventually paralysis may develop. Significant atively avascular, providing little or no immune
neurologic recovery is observed in patients with response, and thus is rapidly destroyed by bacte-
mild neurologic deficits or paralysis of less than rial enzymes. The disc is nearly always involved in
36 h duration who undergo surgical intervention. pyogenic vertebral infections, unlike in tubercu-
Complete paralysis of greater than 36-48 h dura- lous spondylitis (granulomatous). The most con-
tion has not shown recovery. The death rate asso- sistent symptom is back or neck pain, fever,
ciated with epidural abscess has been reported as neurologic deficits, radicular pain, weight loss,
12%. The surgical approach is determined by the and kyphosis. The spinal areas affected in des-
location of the epidural abscess. An abscess cending order are lumbar, thoracic, and cervical.
located posteriorly and extending over multiple Staphylococcus aureus is the most common organ-
levels is best treated by multiple-level lamino- ism and has been identified in over 50% of cases.
tomies or laminectomy, taking care to preserve However, gram-negative organisms (Escherichia
the facet joints. Alternatively, debridement of coli, Pseudomonas spp., Proteus spp.) are associated
the spinal canal through fenestrations removing with spinal infections following genitourinary
the ligamentum flavum and portions of adjacent infections or procedures. Intravenous drug
lamina, and use of catheters can be considered. abusers have a high incidence of Pseudomonas
An abscess located anteriorly and associated with infections. Anaerobic infections are common in
vertebral osteomyelitis is most directly treated diabetics and following penetrating trauma.
with an anterior surgical approach. If an abscess Investigations should include FBC, CRP, ESR,
involves both the anterior and posterior epidural and blood cultures. Vertebral body and adjacent
space, an anterior and posterior approach com- discs appear hypointense on T1-weighted and
bined with spinal stabilization using posterior hyperintense on T2 weighted MRI, and both
instrumentation is considered. A symptomatic enhance on T1 + contrast imaging. Positive radio-
epidural abscess is considered a medical and sur- graphic findings are not evident for at least
gical emergency. The combination of surgical 4 weeks after the onset of symptoms: the earliest
and antibiotic treatment is required for a symp- detectable finding is disc space narrowing, fol-
tomatic epidural abscess. Nonoperative manage- lowed by localized osteopenia and finally destruc-
ment is considered in patients who are extremely tion of the vertebral endplates. Technetium-99m
high-risk surgical candidates and in patients with bone scanning is valuable in the early diagnosis of
an established complete neurologic deficit for pyogenic vertebral osteomyelitis because it dem-
greater than 72 h. In addition, neurologically onstrates positive findings before X-ray changes.
intact patients without sepsis can be considered CT is best at defining extent of bony destruction
for a trial of culture-specific antibiotic therapy and localization of lesions for biopsies. In the
under close clinical supervision. absence of positive blood cultures, biopsy of the

Neurosurgery Books Full


www.ketabpezeshki.com 66485438-66485457
30 SPINAL INFECTION 421

site of presumed vertebral osteomyelitis or discitis involvement, minimal involvement of adjacent


is essential to provide a definitive diagnosis, iden- vertebrae, no kyphotic deformity, and who is
tify the causative organism, and guide treatment. not debilitated by systemic disease or immune
The biopsy ideally should be performed before suppression. The most consistent predictors of
initiation of antibiotics. If antibiotics have success for nonoperative treatment include:
been given, they should be discontinued for 3 days • patients younger than 60 years
before the biopsy. Computed tomography (CT)- • patients who are immunocompetent
guided, closed Craig needle biopsy is safe and • infections with Staphylococcus aureus
effective and yields the etiologic organism in • decreasing ESR and CRP with treatment.
70% of cases. If a closed biopsy is negative after
two attempts, an open biopsy can be considered. FURTHER READING
Tissue samples should be sent for Gram stain, Devlin VJ, Steinmann JC. Spinal infections. In: Devlin VJ
acid-fast stain, aerobic and anaerobic cultures, (Ed.), Spine Secrets Plus, 2nd ed., Elsevier, Mosby, 2012
and fungal and tuberculosis (TB) cultures. Bacte- (Chapter 67).
rial cultures should be observed for at least 10 days
to detect low-virulence organisms. TB cultures 4. a—Simple laminectomy is the primary pro-
may take weeks to grow. Histology studies should cedure for emergency management of neural
also be performed to detect neoplastic processes element compression in vertebral osteomyelitis
and to differentiate acute versus chronic infec-
tion. The goals in treating vertebral osteomyelitis Indications for operative treatment of pyogenic
include early definitive diagnosis, eradication of vertebral osteomyelitis include: (1) need for open
infection, relief of axial pain, prevention or rever- biopsy; (2) failure of nonsurgical management
sal of neurologic deficits, preservation of spinal (high ESR/CRP, refractory back pain); (3) need
stability, and correction of spinal deformity. for open drainage of abscess; (4) neural decom-
pression; (5) spinal stabilization; and (6) correction
Image with permission from Saraf-Lavi E. Spine Imag- of progressive spinal deformity. As such, the
ing: Case Review Series, 3rd ed., Elsevier, Saunders, location of the infection, presence/absence of
2014. abscess, extent of bone destruction, and need for
stabilization are key. Spinal discitis/osteomyelitis
FURTHER READING primarily affects the anterior spinal column, hence
Devlin VJ, Steinmann JC. Spinal infections. In: Devlin VJ
(Ed.), Spine Secrets Plus, 2nd ed., Elsevier, Mosby, 2012 anterior only or combined anterior and posterior
(Chapter 67). approaches are indicated in the majority of spinal
infections. Posterior approaches may be consid-
3. a—Over the age of 60 ered in special circumstances such as posterior
epidural abscesses, disc space infections below
Nonoperative treatment includes antibiotic the conus with satisfactory anterior column sup-
administration, treatment of underlying disease port, and in the absence of significant paraverteb-
processes, nutritional support, and spinal immo- ral abscess. Laminectomy alone is associated with
bilization with an orthosis. Antibiotic selection deformity progression, instability, and neurologic
is based on identification and sensitivity testing. deterioration, hence it is not recommended.
Consultation with an infectious disease specialist Surgery should achieve complete debridement
is recommended. Intravenous antibiotics gener- of nonviable and infected tissue, decompression
ally should be continued for 6 weeks, provided of neural elements, and long-term stability
that satisfactory clinical results and reduction in through fusion (use of autogenous graft material
ESR and CRP occur. In the setting of a broadly is gold standard). The surgical approach generally
sensitive organism and rapid clinical resolution, should include anterior debridement and grafting
intravenous antibiotics may be replaced with oral followed by a staged or simultaneous posterior
antibiotics at 4 weeks. Relapse of infection has spinal stabilization procedure. While placing im-
been reported in up to 25% of patients who plants in infected environments would normally
receive intravenous antibiotic treatment for less be avoided, bone infections are better controlled
than 4 weeks. Contemporary mortality rates with antibiotics and bone stabilization than with
resulting from pyogenic spinal infections range antibiotics alone in an unstable bony environment.
from 2% to 17%. Nonoperative treatment is Use of titanium alloys is preferable to stainless
reported as successful in up to 75% of appropri- steel due to increased bacterial adherence to stain-
ately treated patients when criteria for success less steel implants. In this setting, advantages of
focus on infection cure, infection recurrence, posterior spinal instrumentation include:
and neurologic status following treatment. The 1. preservation of spinal alignment and resto-
ideal patient for nonoperative treatment is a neu- ration of spinal stability following radical
rologically intact patient with primarily disc space debridement

Neurosurgery Books Full


www.ketabpezeshki.com 66485438-66485457
422 PART IV SPINAL NEUROSURGERY

2. increased fusion rates multinucleated giant-cell reaction surrounding a


3. ability to correct kyphotic deformities central region of caseating necrosis. Molecular
4. avoidance of graft collapse or dislodgement detection of mycobacterium DNA or RNA is use-
5. rapid patient mobilization and early reha- ful for rapid diagnosis and for determining drug
bilitation without the need for an external resistance. Radiographs: A clue to diagnosis is
orthosis. the presence of extensive vertebral destruction
out of proportion to the amount of pain. Typically,
FURTHER READING the intervertebral discs are preserved in the early
Devlin VJ, Steinmann JC. Spinal infections. In: Devlin VJ stages of this disease. Chest radiographs can be
(Ed.), Spine Secrets Plus, 2nd ed., Elsevier, Mosby, 2012 useful in demonstrating pulmonary MRI: The
(Chapter 67). imaging modality of choice for diagnosis of spinal
TBCT: Plays a role in defining the extent of bony
5. e—Spinal tuberculosis destruction and localization for biopsies. Chemo-
therapy (four-drug regimen, for a minimum of
Tuberculosis is the most common granulomatous 6-month duration, includes isoniazid, rifampin,
infection of the spine. The three patterns of spinal pyrazinamide, and ethambutol) and brace immo-
involvement are peridiscal (commonest), central, bilization are the initial treatment except in
and anterior. Peridiscal occurs adjacent to the ver- patients presenting with neurologic deficit or pro-
tebral endplate and spreads around a single inter- gressive deformity. The indications for surgery
vertebral disc as the abscess material tracks and the principles of surgical reconstruction are
beneath the anterior longitudinal ligament (the similar to those advised for pyogenic spinal
disc is usually spared unlike in pyogenic infec- infections.
tions). Central involvement occurs in the middle
of the vertebral body and eventually leads to ver- Image with permission from Saraf-Lavi E. Spine Imaging:
Case Review Series, 3rd ed., Elsevier, Saunders, 2014.
tebral collapse and kyphotic deformity. This pat-
tern of involvement can be mistaken for a 6. e—Commonly occur in adolescents
tumor. Anterior infections begin beneath the ante-
rior longitudinal ligament, causing scalloping of Spinal subdural empyemas are uncommon and
the anterior vertebral bodies, and extend over mul- tend to occur in thoracolumbar spine, in those
tiple levels. The presentation is highly variable. in 5th-7th decades. Suggestions for why they
Mild back pain is the most common symptom. are less common than cranial subdural empyema
Patients with tuberculous infections may present and spinal epidural abscesses include the lack of
with malaise, fevers, night sweats, and weight loss. air sinuses in the spine, the fact that the epidural
In addition, chronic infections may result in cuta- space in the spine is an actual rather than poten-
neous sinuses, neurologic deficits (in up to 40% of tial space, and blood is directed centripetally
patients), and kyphotic deformities. Certain factors in the spine, whereas it is directed centrifugally
define the high-risk population and should raise in the brain. The pathogenesis of these infec-
suspicion. Patients from countries with a high inci- tions can be categorized into one of four major
dence of tuberculosis, such as Southeast Asia, categories: hematogenous spread of primary
South America, and Russia are considered high infection, iatrogenic (lumbar puncture, spinal
risk. Patients who live in confinement with others, injections), direct extension into the subdural
such as homeless centers and prisons, are also at space (dysraphism, penetrating trauma, spinal
risk. Elderly adults, chronic alcoholics, patients infection), and cryptogenic. The most common
with AIDS, and patients with a family member organism is Staphylococcus aureus, other Staphylo-
or a household contact with tuberculosis are addi- coccus species, Streptococcus, Escherichia coli,
tional high-risk groups. The leukocyte count may Pseudomonas aeruginosa, Streptococcus pneumoniae,
be normal or mildly elevated. The ESR is mildly and Peptococcus magnus. The classical presenta-
elevated (typically <50), but may be normal in tion of an SSE includes fever, neck/back pain,
up to 25% of cases. Although the purified protein followed by symptoms of spinal cord/cauda
derivative (PPD) skin test may detect active infec- equina compression. Presence of spinal tender-
tion or past exposure, this test is unreliable because ness may favor epidural abscess rather than sub-
false-negative results may occur in malnourished dural empyema. Routine tests include FBC,
and immunocompromised patients. Anergy panel ESR, CRP; lumbar puncture is not performed
testing should be included for this reason. Urine due to risks of contaminating deeper meningeal
cultures, sputum specimens, and gastric washings layers. Contrast MRI is imaging of choice as
may be helpful for diagnosis if the primary source allows better visualization of the spinal cord, ver-
is unknown. The most reliable test for diagnosis is tebrae, disc spaces, extent of lesion, and extent of
CT-guided biopsy. The characteristic finding on compression. The major limitation of MRI,
histology is a granuloma, which is described as a however, remains its inability to distinguish

Neurosurgery Books Full


www.ketabpezeshki.com 66485438-66485457
30 SPINAL INFECTION 423

whether the lesion is intradural or extradural; the postoperative, after penetrating trauma. Most
presence of discitis/osteomyelitis accompanies commonly no cause is found (cryptogenic).
two thirds of SEAs. The mainstay of treatment Causative organisms are Staphylococcus and Strep-
for these infections is surgical decompression tococci, with other significant organisms Actino-
(laminectomy) with irrigation and drainage of myces, Proteus mirabilis, Pneumococcus, Listeria
the subdural space followed by appropriate anti- monocytogenes, Hemophilus, and Escherichia coli.
biotic therapy. The exposure should encompass Cases of contiguous spread via a dermal sinus
the extent of the abscess. After copious irriga- tract are most commonly due to Staphylococcus
tion, most authors advocate the primary closure epidermidis, S. aureus, Enterobacteriaceae, anaer-
of the dura. The arachnoid should be preserved obes, and Proteus mirabilis. Postsurgical (contig-
if possible. A significant indication for surgery is uous) cases are most often due to S. epidermidis,
obtaining a definitive organism to treat; there- S. aureus, Enterobacteriaceae, and Pseudomonas
fore cultures should be obtained before using aeruginosa. The cases that arise from hematoge-
antibiotic irrigation. The use of postoperative nous spread reflect the site of primary infection.
antibiotics should be based on the given organ- The presenting signs and symptoms in patients
ism found during surgery. Empiric antibiotic with ISCAs almost always involve motor deficits,
coverage for these infections must cover gram- sensory impairment, loss of sphincteric control,
positive cocci. Some advocate the additional pain, and fever. Acute infections (<2 weeks) par-
use of corticosteroids (dexamethasone) during tial transverse myelitis commonly associated
the perioperative period as a prophylaxis against with fever and leucocytosis, while subacute
the development of thrombophlebitis. Among (2-6 weeks) and chronic (>6 weeks) present like
the surgically treated group, 82.1% made a com- intramedullary tumors. Investigations include
plete recovery or improved, whereas 17.9% WCC, CRP, ESR, CSF analysis (usually
died. In the conservatively treated group, 80% negative unless meningitis). Imaging of choice
died (4 of 5 patients) and only 20% (1 of 5 is contrast MRI which may differentiate between
patients) improved. On the basis of these num- early and late myelitis. CT myelography may
bers, the current recommendations are for show widening of the cord at a focal segment
aggressive surgical treatment followed by antibi- with obstruction to CSF flow. X-rays are usually
otic therapy. normal at presentation. Other features are oste-
omyelitis, spinal deformity, spinal stenosis, and
FURTHER READING spinal dysraphism, current recommendations
Javahery RJ, Levi AD. Spinal intradural infection. In:
are for immediate surgical treatment. The sur-
Herkowitz et al. (Eds.), Rothman-Simeone the Spine, 6th ed., gery should include laminectomies at the
Elsevier, Saunders, 2011 (Chapter 90). involved levels, intradural exploration, midline
myelotomy, and irrigation and drainage of the
7. a—MRI head with contrast abscess cavity. Aggressive treatment with anti-
biotics requires empirical therapy until an
Intramedullary spinal cord abscesses (ISCAs) are organism has been isolated. The choice of anti-
rare, and patients are young. ISCAs occur biotics for empirical therapy should be based on
throughout the spine, but they are most fre- the suspected source of infection and then
quently found in the thoracic region. The path- adjusted on the basis of the operative culture
ogenesis of ISCA can be divided into two broad results. The current recommendation is a mini-
categories: hematogenous spread and direct mum of 4-6 weeks of parenteral therapy.
implantation. The more complex of the two is Patients presenting with acute symptoms have
hematogenous spread via arterial supply (septic a worse prognosis in terms of neurologic recov-
emboli), venous drainage (increased intratho- ery. Overall, the death of a patient diagnosed
racic/abdominal pressure causing backflow in with an ISCA is most frequently due to the pres-
low pressure spinal venous system), or lym- ence of multiple CNS abscesses and, specifically,
phatics (draining mediastinum, abdomen con- to brain or brainstem abscesses.
nect with Virchow Robins paces in spinal cord
via channels in spinal nerves). The most com- Image with permission from Saraf-Lavi E. Spine Imag-
mon primary infection is pulmonary, endocardi- ing: Case Review Series, 3rd ed., Elsevier, Saunders,
tis, urinary tract infections, peritonitis, and 2014.
peripheral skin infections. The other major
FURTHER READING
route for the pathogenesis of ISCA is direct Javahery RJ, Levi AD. Spinal intradural infection. In:
implantation via a congenital midline neuro- Herkowitz et al. (Eds.), Rothman-Simeone the Spine, 6th
ectodermal defect (e.g. dermal sinus tract), ed., Elsevier, Saunders, 2011 (Chapter 90).

Neurosurgery Books Full


www.ketabpezeshki.com 66485438-66485457
CHAPTER 31

SPINAL ONCOLOGY
SINGLE BEST ANSWER (SBA) QUESTIONS
1. Which one of the following statements presents with mechanical back pain, stab-
regarding spinal tumors is most accurate? bing in nature, which she is more aware of
a. The commonest extradural tumor is an at night. On examination there is non-
osteoid osteoma specific tenderness in the mid-thoracic
b. Intradural tumors in adults are predomi- spine, but no focal neurological signs.
nantly intramedullary rather than d. A 79-year-old with a past medical history of
extramedullary benign prostatic hyperplasia presents with
c. The commonest intramedullary tumor in mechanical back pain after a fall. On exam-
adults is ependymoma ination there is tenderness in the paraspinal
d. Intramedullary metastases are more com- muscles, but no focal neurological signs.
mon than extradural metastases e. A 57-year-old with a past medical history
e. The commonest intramedullary tumor in of small cell lung cancer presents with
children is ganglioglioma mechanical back pain. On examination
he appears kyphotic there is tenderness
2. Which one of the following groups of primary in the mid-thoracic spine. Neurological
malignancies represents the most frequent examination shows brisk reflexes and
causes of spinal column metastases? extensor plantars in both lower limbs.
a. Breast, lung, kidney, melanoma, and
colorectal 4. Which one of the following statements
b. Breast, lung, thyroid, melanoma, and regarding the imaging of spinal metastatic
colorectal disease is LEAST accurate?
c. Breast, lung, kidney, prostate, and thyroid a. Plain radiographs may only show changes
d. Breast, lung, prostate, melanoma, and in approximately half of affected vertebrae
colorectal b. Nuclear scintigraphy detects areas of
e. Breast, prostate, lung, kidney, thyroid increased metabolic activity in bone but
this is not specific to metastatic lesions.
3. Which one of the following clinical presenta- c. SPECT can be used to differentiate
tions is most concerning for spinal metastatic between malignant and benign lesions
disease? d. PET scans can be used to identify the
a. A 47-year-old with breast cancer treated biopsy site
with mastectomy and lymph node dissec- e. Angiography is not likely to be of benefit
tion, on tamoxifen, presents with moderate prior to resection of vertebral metastases
lower back pain worse at the end of the from renal cell carcinoma
day, relieved by rest, not exacerbated by
movement. 5. Which one of the following statements
b. A 54-year-old with previous Dukes B regarding prognostic scoring for patients
colorectal cancer who underwent bowel presenting with metastatic disease of the
resection and end ileostomy, followed by spine is LEAST accurate?
chemotherapy presents with gait distur- a. Tokuhashi score considers Karnofsky
bance. On examination he has a stocking performance status, number of extra-
distribution of sensory loss in the lower spinal metastases and neurological status
limbs only. whereas the Tomita score does not
c. A 35-year-old with a past medical history b. A lower Tomita score favors more aggres-
of wide local excision of melanoma sive surgical treatment

424
Neurosurgery Books Full
www.ketabpezeshki.com 66485438-66485457
31 SPINAL ONCOLOGY 425

c. A higher Tokuhashi score predicts a b. Preoperative radiotherapy should not be


poorer prognosis carried out on patients with MSCC if sur-
d. Both Tokuhashi and Tomita scores con- gery is planned
sider the treatability of visceral metastases c. Radiotherapy can be given as soon as
e. Tokuhashi score assesses neurological sta- there is suspected MSCC on imaging
tus using the Frankel grading system d. Spinal instability is a relative contraindi-
cation to radiotherapy
6. Which one of the following statements e. Teratoma is one of the rare causes of spi-
regarding management of pain related to spi- nal cord compression where chemother-
nal metastasis is most accurate? apy is more effective than radiotherapy
a. Bisphosphonates have shown utility in man-
agement of pain resistant to conventional 9. Which one of the following statements
analgesia and/but not in reducing the risk regarding surgery in patients with metastatic
of malignant spinal cord compression disease of the spine is LEAST accurate?
b. Intrathecal morphine pump insertion is a. Indicated in those with metastatic spinal
inappropriate in patients with intractable cord compression who are ambulant and
pain from spinal metastases without significant neurological deficit if
c. NSAIDs are inappropriate for manage- they are expected to survive greater than
ment of pain related to spinal metastases 3 months
due to their platelet inhibiting effect b. Indicated in those with metastatic spinal
d. Single fraction palliative radiotherapy for cord compression with less than 24 h of
spinal metastases causing non-mechanical complete paralysis who otherwise have a
pain is not appropriate in those with com- good prognosis
plete paralysis c. Indicated in those with spinal instability
e. Vertebroplasty has been shown to be effec- and evidence of structural spine failure
tive in the setting of pain from metastatic to prevent malignant spinal cord com-
spinal cord compression in non-surgical pression, even if their pain is controlled
candidates d. Indicated in those with spinal instability
related mechanical back pain resistant to
7. A 57-year-old with who recently underwent analgesia
right upper lobectomy for small cell lung can- e. Indicated only in those patients expected
cer presents due to worsening back pain over to survive at least 12 months
the last 3 days. It initially started at night over
the last 2 weeks or so, but is now exacerbated by 10. Which one of the following statements
physical movement. He denies any trauma. On regarding the 2005 RCT (Patchell et al.,
examination he is tender to palpation in the Lancet 366:643-648) comparing decompres-
mid thoracic spine. Neurological examination sive resection plus adjuvant radiotherapy ver-
reveals mild weakness of both lower limbs sus radiotherapy alone for metastatic spinal
bilaterally, brisk knee and ankle reflexes, and cord compression is LEAST accurate?
extensor plantars. Sensory examination reveals a. The surgical arm and radiotherapy only
a sensory level at the umbilicus. Which one of arm both received 30 Gy of external-
the following would you do next? beam radiation delivered in 10 fractions
a. Standing spine X-ray b. The surgical arm did not have increased sur-
b. MRI whole spine MRI and CT whole vival time compared to radiotherapy alone
spine c. The surgical arm had greater return of
c. ensure they are Nurse flat with neutral ambulation after treatment compared to
spine alignment radiotherapy alone
d. CT lumbar spine d. The surgical arm remained ambulatory for
e. Dexamethasone longer compared to radiotherapy alone
e. The surgical plus radiotherapy, and radio-
8. Which one of the following statements therapy alone groups both excluded those
regarding definitive oncological manage- with radio-sensitive tumors
ment of patients presenting with MSCC is
LEAST accurate? 11. Which one of the following statements
a. In patients who present with MSCC with- regarding surgical management of spinal
out a known diagnosis of malignancy metastatic disease is LEAST accurate?
radiotherapy can usually start sooner than a. The goal of surgery is preservation of
chemotherapy neurological function, pain relief, and
mechanical stabilization

Neurosurgery Books Full


www.ketabpezeshki.com 66485438-66485457
426 PART IV SPINAL NEUROSURGERY

b. Expected patient survival should exceed a. Gross total resection is the next
12 months before surgical treatment of appropriate step
spinal metastases is considered b. Craniospinal axis MRI should be
c. Percutaneous biopsy (or excisional biopsy performed
during surgery) often required for tissue c. External beam radiotherapy is critical
diagnosis as 10-20% of spine metastases after gross total resection
have no known source d. Etoposide chemotherapy is critical after
d. Seeding and recurrence along the biopsy subtotal resection
needle track can occur with some primary e. Conservative management with surveil-
tumors, such as chordomas lance imaging is recommended
e. Posterior and posterolateral approaches
are preferred to deal with vertebral body 14. Which one of the following is most likely
tumor in the setting of spinal metastases based on the T2-Weighted MRI shown
where possible below?

12. Which one of the following statements


regarding radiation myelopathy is LEAST
accurate?
a. A history of radiation therapy in doses suf-
ficient to result in injury must be present
b. The region of the irradiated cord must lie
slightly below the dermatome level of
expression of the radiation myelitis
c. Local tumor progression must be ruled
out before a diagnosis can be made
d. A latent period from the completion of
treatment to the onset of injury is usually
within 20-30 months
e. The probability of dying from radiation
myelopathy is approximately 70% with
cervical lesions

13. A 31-year-old man presents with back pain


and erectile dysfunction. MRI is performed.
Which one of the following statements
regarding further management is key?

a. Ependymoma
b. Lipoma
c. Neurofibroma
d. Primary CNS lymphoma
e. Schwannoma

Neurosurgery Books Full


www.ketabpezeshki.com 66485438-66485457
31 SPINAL ONCOLOGY 427

15. A 34-year-old male presents with a several 17. A 39-year-old presents with a 6-month history
month history of neck pain, with intermittent of tingling in her fingers and in the last few
episodes of arm and leg numbness. MRI is months, her legs. There is no bowel or bladder
shown. Which one of the following is most disturbance. On examination, there is cape-like
likely? pattern of pain and temperature loss in her
upper limbs. Upper limbs are weak distally
4/5, and reflexes are globally brisk with extensor
plantar responses bilaterally. MRI is shown.
Which one of the following is most likely?

a. Ependymoma
b. Ganglioglioma
c. Meningioma
d. Neurofibroma
e. Schwannoma

16. A 21-year-old with asymmetrical, bilateral sen-


sorineural hearing loss presents and progres-
sive gait disturbance. T1-weighted MRI with a. Astrocytoma
gadolinium is shown. Which one of the fol- b. Ganglioglioma
lowing chromosomes is likely to be mutated? c. Hemangioblastoma
d. Meningioma
e. Metastasis

QUESTIONS 18–24

Additional questions 18–24 available on


ExpertConsult.com

EXTENDED MATCHING ITEM (EMI)


QUESTIONS
25. Intramedullary tumors:
a. Astrocytoma
b. Ependymoma
c. Ganglioglioma
a. Chromosome 7 d. Hemangioblastoma
b. Chromosome 9 e. Hemangioma
c. Chromosome 11 f. Medulloblastoma
d. Chromosome 17 g. Oligodendroglioma
e. Chromosome 22 h. Teratoma
i. PCNSL
j. Metastasis

Neurosurgery Books Full


www.ketabpezeshki.com 66485438-66485457
428 PART IV SPINAL NEUROSURGERY

For each of the following descriptions, select the c. Eosinophilic granuloma


most appropriate answers from the list above. d. Hemangioma
Each answer may be used once, more than once e. Multiple myeloma
or not at all. f. Metastasis
1. Commonest primary intramedullary tumor g. Neuroblastoma
in adults h. Neurofibroma
2. Second commonest intramedullary tumor i. Osteoblastoma
in children j. Osteochondroma
k. Osteoid osteoma
26. Intradural extramedullary lesions: l. Osteosarcoma
a. Myxopapillary ependymoma
b. Epidermoid For each of the following descriptions, select the
c. Lipoma most appropriate answers from the list above.
d. Meningioma Each answer may be used once, more than once
e. Neurofibroma or not at all.
f. Paraganglioma 1. A 37-year-old male sustained a wedge frac-
g. Schwannoma ture of L1 following a fall down a flight of
h. Leptomeningeal drop metastasis stairs. Jailhouse striations (honeycomb pat-
i. Teratoma tern) of the vertebra were also seen with
j. Arachnoid cyst a high signal on STIR MRI
2. A 7-year-old boy presents with mid-
For each of the following descriptions, select the thoracic back pain. Spinal X-rays show a
most appropriate answers from the list above. vertebra plana at T10
Each answer may be used once, more than once 3. An 11-year-old boy has had back pain for
or not at all. the last 6 months that has failed to improve
1. Important in risk stratification of patients with observation and simple analgesia. X-rays
presenting with PNETs showed a reactive bone around a 1 cm radio-
2. The commonest intradural, extramedullary lucent nidus in the lamina of L1
lesion in China and Japan 4. A 49-year-old male presents with lower
3. The commonest intradural, extramedullary abdominal pain. X-rays and CT pelvis
lesion in Western countries showed a lytic lesion of the anterior sacrum
and histology after wide-margin surgical
27. Extradural tumors: excision reveals cells with a foamy physali-
a. Chordoma ferous appearance
b. Chondrosarcoma

SBA ANSWERS
1. c—The commonest intramedullary tumor in intramedullary (in children it is approximately
adults is ependymoma equal)—with the incidence of intradural spinal
Extradural tumors in adults: 90% metastatic cord tumors 3-10 per 100,000 per year. Intradural
tumors. The highest proportion of spinal meta- extramedullary lesions are most commonly tho-
static deposits come from breast cancer (16.5%), racic, and most commonly meningiomas and
lung cancer (15%), prostate cancer (10%), and nerve sheath tumors in equal proportions (spinal
renal cell carcinoma (7%) and 10-20% will schwannomas are much more common than
have no known primary. Therefore primary meningiomas in China and Japan). Intramedul-
extradural tumors only make up <10% of all spi- lary “lesions”: ependymoma 41%, astrocytoma
nal column tumors, and may be benign or malig- (WHO I-II) 15%, astrocytoma (WHO III-IV)
nant. The commonest benign lesions are osteoid 5%, ganglioglioma 3.2%, lipoma 2.8%, subepen-
osteomas, osteoblastomas, osteochondromas, dymoma 0.9%, metastasis 0.6%.
aneurysmal bone cysts and hemangiomas (and
Langerhans cell histiocytosis in children). The FURTHER READING
commonest malignant lesions are osteosarcoma, Petteys RJ et al. Epidural tumours and metastases. In: J-C
chondrosarcoma, malignant fibrous histiocy- Tonn et al. Oncology of CNS tumours. Springer-Verlag,
toma, giant cell tumor, plasmacytoma (solitary Berlin, 2010, pp. 719-737.
myeloma), Ewing’s sarcoma and chordoma. In Goldbrunner R. Intradural extramedullary tumours. In: J-C
adults, approximately two thirds of intradural Tonn et al. Oncology of CNS tumours. Springer-Verlag,
lesions are extramedullary and one third Berlin, 2010, pp. 709-718.

Neurosurgery Books Full


www.ketabpezeshki.com 66485438-66485457
31 SPINAL ONCOLOGY 429

Westphal M. Intramedullary tumours. In: J-C Tonn et al. mid-thoracic spine. Neurological examination
Oncology of CNS tumours. Springer-Verlag, Berlin, 2010, shows brisk reflexes and extensor plantars in
pp. 689-708. both lower limbs
2. e—Breast, prostate, lung, kidney, thyroid Symptoms associated with spinal metastatic dis-
ease may be systemic (weight loss, anorexia, organ
The commonest cancers which metastasize to dysfunction) or local (pain and/or neurology).
bone are breast, lung, prostate, kidney and thy- The most common symptom is pain, which
roid (in contrast to brain metastasis where the may predate neurological symptoms by weeks
commonest primary tumors are breast, lung, kid- or months, and may even be the presenting symp-
ney, melanoma, and colorectal). Bony metastasis tom of an undiagnosed cancer. Three classically
most commonly occur in the spine. At autopsy defined types of pain often affect patients with
30-90% of patients who die of cancer are found symptomatic spinal metastases, including local
to have spinal metastases. Symptomatic second- (periosteal stretching and inflammation due to
ary metastases are estimated to occur in approxi- tumor and often nocturnal, usually deep ache),
mately 10% of all cancer patients. Up to 50% of mechanical (impending or established spinal
spinal metastases require some form of treatment, instability due to deformity/collapse of verte-
and 5-10% require surgical management. The brae), and radicular pain (nerve root compression
highest incidence of spinal metastases is found directly from tumor or narrowing of interverteb-
in individuals 40-65 years of age, corresponding ral foramen due to vertebral collapse). Neurolog-
to the period of highest cancer risk. Males are ical symptoms may be due to radiculopathy or
slightly more prone to the development of spinal metastatic spinal cord compression (MSCC).
metastases, probably reflecting the slightly higher MSCC can present with motor, sensory or auto-
prevalence of lung cancer in men, and of prostate nomic disturbance (bladder, bowel or sexual dys-
cancer over breast cancer. Metastatic spread is via function or rarely even spinal shock) due to direct
hematogenous seeding, direct extension or inva- compression of neural structures by tumor, or to a
sion, and by seeding in the CSF. Hematogenous pathological fracture that leads to retropulsion of
spread of tumors usually results in multicentric bone fragments into the spinal canal or neural
disease of the spine, which also occurs with foramina. The patient’s neurological function
CSF seeding (e.g. after surgical manipulation of when a diagnosis of spinal cord compression is
cerebral or cerebellar metastatic or primary made usually correlates well with their prognosis.
lesions). Spinal tumors may be extradural, This observation underscores the importance of
intradural-extramedullary, and intramedullary; diagnosis before motor or autonomic deficits
the overwhelming majority of all spinal metasta- occur. Most patients will have pain before these
ses are found in the extradural compartment; that deficits appear. However, because reports of back
is, the bony spine and associated tissues. Metasta- pain are very common in the general population,
ses to the extradural compartment are found most with a lifetime prevalence of up to 84% in some
commonly in the VB, with or without extension studies, a delay in diagnosis occurs in many cases
into the posterior elements, followed by the of vertebral metastasis in which the initial com-
paravertebral regions and the epidural space, plaint is one of new-onset back or neck pain.
respectively. Intradural extramedullary and intra-
medullary metastases are very rare, and are often FURTHER READING
due to CSF seeding. The thoracic spine is by far Sciubba DM, Petteys RJ, Dekutoski MB, Fisher CG, Fehlings
the most frequent site for metastasis (70%), fol- MG, Ondra SL, Rhines LD, Gokaslan ZL. Diagnosis and
lowed by the lumbar spine (20%), cervical spine, management of metastatic spine disease. A review. J Neuro-
and sacrum, respectively. surg Spine. 2010;13(1):94-108.

FURTHER READING 4. e—Angiography is not likely to be of benefit


Sciubba DM, Petteys RJ, Dekutoski MB, Fisher CG, Fehlings prior to resection of vertebral metastases from
MG, Ondra SL, Rhines LD, Gokaslan ZL. Diagnosis and renal cell carcinoma
management of metastatic spine disease. A review. J Neuro-
surg Spine. 2010;13(1):94-108. doi: 10.3171/2010.3.
Plain radiographs are a useful screening test to
SPINE09202. Review. PubMed PMID:20594024.
identify lytic or sclerotic lesions, pathological frac-
tures, spinal deformities, and large masses. Breast
3. e—A 57-year-old with a past medical history
or prostate cancers may produce sclerotic or blas-
of small cell lung cancer presents with
tic lesions, but most spinal metastases are lytic, and
mechanical back pain. On examination he
plain radiographs may not reveal changes until up
appears kyphotic there is tenderness in the
to half of the VB is affected. Due to this relative

Neurosurgery Books Full


www.ketabpezeshki.com 66485438-66485457
430 PART IV SPINAL NEUROSURGERY

insensitivity, diagnosis is often obtained with Tokuhashi score for metastatic spine tumor prog-
other imaging techniques. Nuclear scintigraphy nosis (irrespective of treatment modality).
(bone scan) is a sensitive method for identifying
areas of increased metabolic activity throughout
the skeletal system. They are not specific for met- Modified Tokuhashi Score Criteria Points
astatic lesions, because this activity may be related General condition (Karnofsky)
to inflammation or infection. A more advanced
form of nuclear bone scan, SPECT, provides 3D Poor (10-40%) 0
imaging of suspected vertebral metastases can be
Moderate (50-70%) 1
used to differentiate between metastatic and
benign lesions. Positron emission tomography Good (80-100%) 2
with FDG is also commonly used for whole-body
surveillance in the detection of metastatic disease Number of extra-spinal bone
metastases foci
and cancer staging PET scans can also be used to
identify cystic or necrotic areas of tumor, informa- 3 0
tion that may increase the diagnostic yield of
biopsy sampling and assist planning of surgical 1-2 1
intervention. However, the resolution of PET is 2 points: 1 2
limited, and correlation with CT or MR imaging
is required. Additionally, PET scanning is time- Number of metastases in the vertebral
consuming and expensive. CT scanners provide body
highly detailed imaging of the osseous anatomy 3 0
of the spine and the degree of tumor involvement,
can aid surgical planning, and may be combined 1-2 1
with CT angiography (vascular supply and tumor 1 2
drainage) and myelography. MRI is most sensitive
at detecting lesions within the vertebral column, Metastases to the major internal organs
The MR images also elucidate the bone- to soft-
Unremovable 0
tissue interface, providing accurate anatomical
detail of tumor invasion or related compression Removable 1
of osseous, neural, and paraspinal structures. Dif-
fusion weighted studies, although not routinely No metastases 2
used, may help distinguish benign and pathologi- Primary site of the cancer
cal compression fractures. Digital subtraction
angiography is useful for lesions that originate Lung, osteosarcoma, stomach, bladder, 0
from primary tumors with abundant vascularity esophagus, pancreas
(i.e. renal cell, thyroid, angiosarcoma, leiomyosar- Liver, gallbladder, unidentified 1
coma, hepatocellular, and neuroendocrine
tumors), knowledge of the vascular supply of Others 2
metastases may prove invaluable if surgery is con-
Kidney, uterus 3
sidered. Angiography may also permit preopera-
tive embolization of metastases, which can be an Rectum 4
effective alternative treatment for patients who
are not candidates for surgical treatment. Emboli- Thyroid, breast, prostate, carcinoid tumor 5
zation reduces intraoperative blood loss and facil- Palsy
itates complete resection of the lesion. In addition
to limiting intraoperative hemorrhage, reducing Complete (Frankel A, B) 0
the vascularity of metastases can also potentially
Incomplete (Frankel C, D) 1
shorten operating times and prevent the develop-
ment of postoperative hematomas, which can None (Frankel E) 2
cause wound breakdown and neurological decline. Total score 0-8: 85% live <6 months thus conservative
treatment or palliative surgery appropriate. Total score 9-11:
FURTHER READING 73% live >6 months (and 30% >1 year) thus palliative
Sciubba DM, Petteys RJ, Dekutoski MB, Fisher CG, Fehlings surgery or (exceptionally) excisional surgery an option. Total
MG, Ondra SL, Rhines LD, Gokaslan ZL. Diagnosis and score 12-15: 95% live >1 year thus excisional surgery an
management of metastatic spine disease. A review. J Neuro- option.
surg Spine. 2010;13(1):94-108. With permission from Tokuhashi Y, Matsuzaki H, Oda H,
et al., A revised scoring system for preoperative evaluation
of metastatic spine tumor prognosis, Spine (Phila Pa 1976).
5. c—A higher Tokuhashi score predicts a 2005;30(19):2186-2191.
poorer prognosis
Neurosurgery Books Full
www.ketabpezeshki.com 66485438-66485457
31 SPINAL ONCOLOGY 431

compression. Offer patients with spinal metasta-


Tomita Score Criteria Points
ses causing non-mechanical spinal pain 8 Gy sin-
Primary tumor gle fraction palliative radiotherapy even if they
are completely paralysed, but not with the inten-
Slow growth (e.g. breast, prostate, 1
thyroid)
tion of preventing MSCC. In the absence of
Moderate growth (e.g. kidney, uterus) 2 MSCC or spinal instability, consider vertebro-
plasty or kyphoplasty for patients who have verte-
Rapid growth (e.g. lung, liver, stomach, 4 bral metastases causing mechanical pain resistant
colon, primary unknown) to conventional analgesia, or vertebral body
Visceral metastases collapse.

No visceral metastases 0 FURTHER READING


Metastatic Spinal Cord Compression: Diagnosis and Manage-
Treatable 2 ment of Patients at Risk of or with Metastatic Spinal Cord
Compression. NICE Clinical Guidelines, No. 75. National
Untreatable 4 Collaborating Centre for Cancer (UK). Cardiff (UK):
Bone metastases (including spine) National Collaborating Centre for Cancer (UK); 2008 Nov.

Solitary/isolated 1 7. c—Ensure he is nursed flat with neutral spine


alignment
Multiple 2
Acute management should include spinal precau-
tions, steroids, and usually MR imaging. Patients
with severe mechanical pain suggestive of spinal
Treatment goal & surgical strategy based on instability, or any neurological symptoms or signs
total score: suggestive of MSCC, should be nursed flat with
2-3 points: long-term local control (mean sur- neutral spine alignment (including “log rolling”
vival 50 months) ! wide or marginal excision or turning beds, with use of a slipper pan for toilet)
4-5 points: middle-term local control (mean until bony and neurological stability are ensured
survival 23.5 months) ! marginal or intrale- and cautious remobilization may begin. For
sional excision patients with MSCC, once any spinal shock has
6-7 points: short-term palliation (mean sur- settled and neurology is stable, carry out close
vival 15 months) ! palliative surgery monitoring and interval assessment during grad-
8-10 points: terminal care (mean survival ual sitting from supine to 60° over a period of
6 months) ! supportive care, no surgery 3-4 h. When patients with MSCC begin gradual
sitting, if their blood pressure remains stable and
FURTHER READING no significant increase in pain or neurological
Tomita K, Kawahara N, Kobayashi T et al. Surgical strategy for symptoms occurs, continue to unsupported sitting,
spinal metastases, Spine (Phila Pa 1976). 2001;26(3):298-306. transfers and mobilization as symptoms allow. If
a significant increase in pain or neurological symp-
6. a—Bisphosphonates have shown utility in toms occurs when patients with MSCC begin grad-
management of pain resistant to conventional ual sitting and mobilization, return them to a
analgesia and but not in reducing the risk of position where these changes reverse and reassess
malignant spinal cord compression the stability of their spine. After a full discussion
of the risks, patients who are not suitable for defin-
Offer conventional analgesia (including NSAIDs, itive treatment should be helped to position them-
non-opiate and opiate medication) as required to selves and mobilize as symptoms permit with the
patients with painful spinal metastases in escalating aid of orthoses and/or specialist seating to stabilize
doses as described by the WHO three-step pain the spine, if appropriate. Unless contraindicated
relief ladder. Consider referral for specialist pain (including a significant suspicion of lymphoma)
care including invasive procedures (such as epidural offer all patients with MSCC a loading dose of at
or intrathecal analgesia) and neurosurgical inter- least 16 mg of dexamethasone as soon as possible
ventions for patients with intractable pain from spi- after assessment, continue dexamethasone 16 mg
nal metastases (e.g. intrathecal morphine pump). daily in patients awaiting surgery or radiotherapy
Bisphosphonates should only be used (if con- for MSCC. After surgery or the start of radiother-
ventional analgesia fails) for pain relief in cases apy the dose should be reduced gradually over
of vertebral metastases from breast, myeloma 5-7 days and stopped. If neurological function
or prostate cancer only, and should not be deteriorates at any time the dose should be
used as prophylaxis for malignant spinal cord increased temporarily. In those not proceeding to

Neurosurgery Books Full


www.ketabpezeshki.com 66485438-66485457
432 PART IV SPINAL NEUROSURGERY

surgery or radiotherapy reduce gradually and stop MSCC if surgery is planned, but postoperative
dexamethasone as tolerated. fractionated radiotherapy should be offered rou-
tinely to all patients with a satisfactory surgical
FURTHER READING outcome once the wound has healed. In those
Metastatic Spinal Cord Compression: Diagnosis and Manage- with MSCC who are not suitable for surgery,
ment of Patients at Risk of or with Metastatic Spinal Cord urgent radiotherapy should be offered (<24 h)
Compression. NICE Clinical Guidelines, No. 75. National
unless they have had complete tetraplegia or
Collaborating Centre for Cancer (UK). Cardiff (UK):
National Collaborating Centre for Cancer (UK); 2008 Nov.
paraplegia for more than 24 h and their pain
is already well controlled; or their overall progno-
8. c—Radiotherapy can be given as soon as there sis is judged to be too poor. Chemotherapy is gen-
is suspected MSCC on imaging erally not indicated as the immediate treatment
for malignant spinal cord compression and its
Start definitive treatment, if appropriate, before main role is following the initial treatment with
any further neurological deterioration and decompressive spinal surgery, or sometimes fol-
ideally within 24 h of the confirmed diagnosis lowing local radiotherapy. Patients who present
of MSCC. In deciding on definitive treatment, with malignant spinal cord compression, without
establishing primary histology and staging (sites a previous known malignancy, generally require
and extent of visceral and bony metastases) are a tissue diagnosis, and in most cases immediate
key. Other important factors are the preferences surgery to decompress the spinal cord before
of patients, neurological deficit, functional the diagnosis is made, so that a biopsy would
status, general health and fitness, previous treat- be obtained as part of the procedure. Rarely,
ments, magnitude of surgery, likelihood of radiological appearances may strongly suggest
complications, fitness for general anesthesia and lymphoma, and needle biopsy, rather than
overall prognosis. In particular, early decisions immediate surgery is occasionally warranted, in
should be made about aggressiveness of MSCC which case immediate radiotherapy, rather than
treatment in those with (i) a poor performance chemotherapy is given, as a provisional diagnosis
status and widespread metastatic disease or (ii) can be obtained in an emergency within 24 h, and
completely paraplegic or tetraplegic for more the correct chemotherapy usually requires a more
than 24 h, and (iii) too frail or unfit for specialist detailed pathological diagnosis, which takes lon-
treatment. Major surgical treatments should only ger. Most chemo-sensitive tumors are also
be considered in those patients expected to sur- radio-sensitive, and it is often preferable to give
vive 3 months or more, and use of the revised local radiotherapy in such cases, to deal with
Tokuhashi scoring system and American Society the anatomical cause of the cord compression
of Anaesthetists (ASA) grading will help define its without having to consider the fitness of the
type and extent. Management options include patient for what may be life-threatening treat-
mobilizing with bracing, palliation, radiotherapy ment with chemotherapy. Teratoma, yolk sac
(most common), chemotherapy (e.g. localized tumor, choriocarcinoma or a malignant molar
non-Hodgkin’s lymphoma and germ cell tumors) pregnancy, are the (rare) causes of spinal cord
and surgery. Consider patients with MSCC who compression where chemotherapy is more effec-
have severe mechanical pain and/or imaging evi- tive than radiotherapy, and should be the treat-
dence of spinal instability, but who are unsuitable ment of choice following initial tissue diagnosis.
for surgery, for external spinal support (for exam-
ple, a halo vest or cervico-thoraco-lumbar ortho- FURTHER READING
sis). In those with non-mechanical pain related to Metastatic Spinal Cord Compression: Diagnosis and Manage-
extradural spinal metastases only (i.e. without ment of Patients at Risk of or with Metastatic Spinal Cord
Compression. NICE Clinical Guidelines, No. 75. National
MSCC or spinal instability) offer fractionated
Collaborating Centre for Cancer (UK). Cardiff (UK):
radiotherapy as the definitive treatment. In those National Collaborating Centre for Cancer (UK); 2008 Nov.
with MSCC confirmed on imaging there must be
a cancer diagnosis established before radiother- 9. e—Indicated in those patients expected to sur-
apy can start. Relative contraindications to radio- vive at least 12 months
therapy include no histological diagnosis of
cancer, radio-resistant tumor if surgery is an To be considered for surgery the patient must be
option (renal cell carcinoma, sarcoma, melanoma surgically fit, no pre-existing neurology, ambu-
etc.), vertebral displacement/spinal instability, lant/weak/<24 h paralysis, single area of cord
poor general condition (irreversible) due to co- compression (this can include several contiguous
morbidities, and previous radiotherapy (to cord spinal or vertebral segments), expected to survive
tolerance) to same spinal site. Preoperative radio- 6 months or at least >3 months. Indications for
therapy should not be carried out on patients with

Neurosurgery Books Full


www.ketabpezeshki.com 66485438-66485457
31 SPINAL ONCOLOGY 433

surgery in the context of spinal metastastic disease 10. b—The surgical arm did not have increased
will generally occur in the following scenarios: (i) survival time compared to radiotherapy alone
to stabilize the spine and prevent MSCC in those
with imaging evidence of structural spinal failure In 2005, Patchell et al. reported the results of the
with spinal instability; (ii) to stabilize the spine in first prospective randomized controlled trial of
those with mechanical pain resistant to conven- direct decompressive resection plus adjuvant
tional analgesia, irrespective of neurological sta- radiotherapy versus radiotherapy alone for meta-
tus, or (iii) to decompress the cord (usually with static spinal cord compression. Their study
spinal stabilization if vertebral involvement) in showed surgery plus radiotherapy to be far supe-
those with MSCC who are can walk, have rior to radiotherapy alone, and the trial was
<24 h complete paralysis, or have little (but stopped after 50% recruitment. Both groups of
some) neurological function with very good prog- patients received 30 Gy of external-beam radia-
nosis giving them a chance of functional recovery. tion delivered in 10 fractions, and the surgical
If surgery is appropriate in patients with MSCC group underwent operations intended to decom-
attempt to achieve both spinal cord decompres- press the spinal cord, resect tumor bulk, and sta-
sion and durable spinal column stability before bilize the spine. This approach was associated
they lose the ability to walk. If there is the slight- with statistically superior post-treatment ambula-
est doubt as to the underlying pathology, partic- tory rate (84% vs. 57%, p ¼ 0.001), duration of
ularly where there is a solitary bony lesion, ambulation (median 122 days vs. 13 days,
further investigations including percutaneous p ¼ 0.003), maintenance of ambulation after treat-
bone biopsy should be carried out before defini- ment (94% vs. 74%, p ¼ 0.024), return of ambula-
tive surgery. In those with a good prognosis but tion after treatment (62% vs. 19%, p ¼ 0.012),
only residual distal sensory or motor function functional ability (Frankel scores), muscle
should still be offered surgery in an attempt to strength (American Spinal Injury Association
recover useful function, regardless of their ability scores), continence, and survival time than those
to walk. Patients with MSCC who have been treated with radiotherapy alone. The median sur-
completely paraplegic or tetraplegic for more vival time in the surgery plus radiotherapy group
than 24 h should only be offered surgery if spinal was 126 days, versus 100 days in the radiotherapy
stabilization is required for pain relief. Posterior alone group (p ¼ 0.033). However, those with
decompression alone should not be performed highly radio-sensitive tumors (e.g. lymphoma,
in patients with MSCC except in the rare circum- myeloma, and small cell lung carcinoma) were
stances of isolated epidural tumor or neural arch excluded from both groups hence it should be
metastases without bony instability. If spinal seen as proving the superiority of this approach
metastases involve the vertebral body or threaten for MSCC due to radio-resistant tumors. In
spinal stability, posterior decompression should patients with radio-sensitive primary tumors,
always be accompanied by internal fixation with radiotherapy alone is still indicated for MSCC
or without bone grafting. Consider vertebral presenting without spinal instability, rapidly pro-
body reinforcement with cement for patients with gressive neurological decline without significant
MSCC and vertebral body involvement who are bone intrusion of the spinal canal, or with
suitable for instrumented decompression but expected survival time <3 months. Surgical
are expected to survive for less than 1 year. Con- decompression and stabilization is indicated in
sider vertebral body reconstruction with anterior patients with spinal instability, bony cord com-
bone graft for patients with MSCC and vertebral pression, rapid decline due to non-bony cord
body involvement who are suitable for instru- compression, recurrent tumor despite radiother-
mented decompression, are expected to survive apy, MSCC caused by radio-resistant tumors, and
for 1 year or longer and who are fit to undergo cases in which tissue diagnosis is necessary. Total
a more prolonged procedure. En bloc excisional en bloc resection and spondylectomy may be
surgery with the objective of curing the cancer indicated with curative resection possible for
should not be attempted, except in very rare cir- patients with solitary metastases of relatively
cumstances (e.g. confirmed solitary renal or thy- indolent course, such as renal cell carcinoma
roid metastasis following complete staging). without systemic metastases.

FURTHER READING
Metastatic Spinal Cord Compression: Diagnosis and Manage-
FURTHER READING
Patchell RA, Tibbs PA, Regine WF, et al. Direct decompres-
ment of Patients at Risk of or with Metastatic Spinal Cord
sive surgical resection in the treatment of spinal cord compres-
Compression. NICE Clinical Guidelines, No. 75. National
sion caused by metastatic cancer. A randomised trial. Lancet
Collaborating Centre for Cancer (UK). Cardiff (UK):
2005;366:643-648.
National Collaborating Centre for Cancer (UK); 2008 Nov.

Neurosurgery Books Full


www.ketabpezeshki.com 66485438-66485457
434 PART IV SPINAL NEUROSURGERY

11. b—Expected patient survival should exceed portion of the spine in metastatic disease, and
12 months before surgical treatment of spinal therefore, anterior approaches provide the great-
metastases is considered est ability to resect the lesion and decompress the
spinal canal. However, these approaches are asso-
Curative treatment is often not possible; there- ciated with increased surgery-related morbidity
fore, therapeutic objectives are focused on preser- and mortality, especially since the thoracic spine
vation of neurological function, pain relief, and is the commonest site. Therefore, a transpedicular
mechanical stabilization. Surgical intervention posterior or posterolateral approach is frequently
can successfully achieve these goals, but patient used for T1-T4, Three-column decompression
variables (such as age, tumor burden, life expec- and stabilization can be achieved with this
tancy, and functional status) overwhelmingly approach, especially with circumferential involve-
influence the choice of therapy as much as stability ment and/or multiple levels. A right-sided thora-
and neurology. Developments in surgical tech- cotomy, which minimizes risk to the great
nique and anterior and posterior stabilization of vessels and aortic arch, permits access to the
the spine that allow improved decompression mid-thoracic spine (T5-12). If, however, the
and tumor resection with acceptable morbidity. majority of tumor bulk is on the left, a left-sided
Long-term disease-free survival is possible in thoracotomy is indicated. Decompression of the
some cases, specifically in solitary renal cell carci- thoracolumbar junction (T11-L1) may necessitate
noma metastases. Additionally, most clinicians a combined thoracotomy and retroperitoneal
would agree that the expected patient survival approach. The lumbar spine (L2-5) and sacrum
should exceed 3 months before surgical treatment may be approached via anterior approaches, but
of spinal metastases is considered. The principles posterior excision and stabilization is usually ade-
used to develop these scoring systems were quate in metastatic disease. Vertebral body resec-
designed to assist surgeons in selecting patients tion requires subsequent reconstruction, often
who may benefit from surgical intervention and with titanium distractible or static mesh cages or
to determine the extent of surgical invasiveness with PMMA and anterolateral plating. Posterior
that is appropriate. Practically speaking, the calcu- stabilization with pedicle screw instrumentation
lated scores from the Tomita and Tokuhashi sys- is indicated for resections at high-stress areas, such
tems are not binding in the choice of treatment, as the cervicothoracic and thoracolumbar junc-
especially with the recent development of other tion, and for patients with two or more adjacent
treatment modalities like SRS. Moreover, once vertebrectomies, kyphosis, or circumferential
patients have been deemed appropriate candidates involvement.
for surgical intervention, the determination of
operative approach and stabilization requires a FURTHER READING
comprehensive understanding of the anatomy Sciubba DM, Petteys RJ, Dekutoski MB, Fisher CG, Fehlings
and histopathological features of the metastatic MG, Ondra SL, Rhines LD, Gokaslan ZL. Diagnosis and
tumor and its surrounding structures, as well as management of metastatic spine disease. A review. J Neuro-
the biomechanics of the spine and changes surg Spine. 2010;13(1):94-108.
induced by vertebral metastases. Advances in
imaging technology have improved the detection 12. b—The region of the irradiated cord must lie
of cancerous lesions, but tissue from spinal masses slightly below the dermatome level of expres-
is often still required for definitive diagnosis as sion of the radiation myelitis
10-20% of spine metastases have no known
source. If surgery and excisional biopsy are not Radiation myelopathy may present as a transient
immediately indicated, percutaneous biopsy may early-delayed or late-delayed reaction. Transient
be required, because most treatment decisions will (acute) radiation myelopathy is clinically mani-
be dictated by the tumor histological findings. fested by Lhermitte’s sign developing 3-4 months
When a primary tumor is considered a possibility, after treatment and spontaneously resolves over
the surgeon should be consulted in planning the the following 3-6 months without therapy. It is
biopsy procedure, because seeding and recurrence attributed to transient demyelination caused by
along the biopsy needle track can occur with some radiation-induced inhibition of myelin producing
primary tumors, such as chordomas. The surgical oligodendroglial cells in the irradiated spinal cord
approach to resection or decompression in spinal segment. Irreversible radiation myelopathy usu-
metastases is in large part determined by the spinal ally is not seen earlier than 6-12 months after com-
segment involved, the location of the tumor within pletion of treatment. Typically, half of the patients
the vertebra, the tumor’s histological features, and who develop radiation-induced myelopathy in the
the type of spine reconstruction necessary. The cervical or thoracic cord region will do so within
vertebral body is the most commonly affected 20 months of treatment and 75% of cases will

Neurosurgery Books Full


www.ketabpezeshki.com 66485438-66485457
31 SPINAL ONCOLOGY 435

occur within 30 months. The signs and symptoms region (filum terminale), are well-differentiated,
are typically progressive over several months, but and are often encapsulated but can seed the
acute onset of plegia over several hours or a few CSF, typically with “drop metastases” at the thecal
days is possible. It is thought to be multifactorial sac. Myxopapillary ependymomas often progress
in origin, involving demyelination and white mat- slowly and cause milder-than-expected neuro-
ter necrosis ultimately resulting from oligoden- logic deficits for their size. These tumors repre-
droglial cell depletion and microvascular injury. sent a special variant of ependymoma found
Radiation myelopathy is a diagnosis of exclusion almost exclusively in the region of the filum termi-
with the following characteristics: (1) a history of nale, although occasionally they have been found
radiation therapy in doses sufficient to result in higher in the spinal cord or, rarely, in the brain.
injury must be present; (2) the region of the irradi- They may occur at any age, but most arise in the
ated cord must lie slightly above the dermatome fourth decade. Myxopapillary ependymomas
level of expression of the lesion; (3) the latent characteristically form a sausage-shaped mass in
period from the completion of treatment to the the lumbosacral region, displacing spinal nerve
onset of injury must be consistent with that roots of the cauda equina. Their biologic behavior
observed in radiation myelopathy; and (4) local is usually benign, but because of their location they
tumor progression must be ruled out. There are are often associated with significant compression-
no pathognomonic laboratory tests or imaging induced paralysis. Treatment consists of local
studies that conclusively diagnose radiation mye- excision, which must often be only partial because
lopathy. MRI findings include swelling of the spi- of the tumor’s location; approximately 20% recur
nal cord with hyperintensity on the T2-weighted even after complete initial resection. Metastases
images with or without areas of contrast enhance- infiltrating the CSF and extradural space may
ment. There is no known consistently effective occur, but transformation to anaplastic variants
treatment for radiation myelitis. The probability is extremely rare. Myxopapillary subtypes appear
of dying from radiation myelopathy is approxi- to be associated with a favorable prognosis, poten-
mately 70% with cervical lesions and 30% with tially because of ease of resection because of their
thoracic spinal cord injury. Radiation side effects anatomic location. Patients who are able to
in children include growth abnormalities such as achieve GTR have improved outcomes and the
decreased vertebral height, kyphosis, and scoliosis. upfront addition of radiation therapy is of ques-
Secondary malignant disease, including bone or tionable benefit. However, one study suggests that
soft-tissue sarcomas and glioblastoma, has been pediatric patients with this tumor had higher
reported after irradiation of spinal cord tumors. recurrence rates, even in the setting of GTR,
and appeared to benefit from postoperative irradi-
13. b—Craniospinal axis MRI should be ation. A retrospective review from the Rare Can-
performed cer Network suggests that higher postoperative
radiation dose (>50.4 Gy) for the myxopapillary
Myxopapillary ependymomas are most commonly subtype may be associated with improved PFS.
benign and localize most often to the filum termi-
nale and conus medullaris. They differ from other Image with permission from Yachnis AT, Rivera-
ependymomas morphologically and biologically Zengotita ML, Neuropathology, High-Yield Pathology
Series, Saunders, Elsevier, 2014.
and often resemble chordomas or chondrosarco-
mas; immunohistochemical analysis is frequently
required for differentiation. Myxopapillary epen- 14. a—Ependymoma
dymomas manifest in younger individuals, in com-
However, studies have shown that ependymomas
parison with cellular ependymomas, and are also
more common in male patients. They display large have a predilection for the caudal spinal cord,
with 50% of ependymomas arising in the lumbo-
variations in size and are associated with scalloping
sacral cord or filum terminale and the remaining
of the vertebral body and enlargement of the neu-
50% occurring nonpreferentially along the cervi-
ral foramina. On T1-weighted imaging, myxopa-
cal or thoracic spinal cord. On imaging, anaplastic
pillary ependymomas are most often isointense or
ependymomas may be distinguished by their
hypointense; however, in some instances, they
larger size, numerous cysts, and heterogeneous
have displayed hyperintensity on T1-weighted
postcontrast enhancement. Anaplastic ependy-
imaging because of hemorrhage or their mucin
momas are uncommon, comprising only 5% of
content. On T2-weighted imaging, these tumors
all ependymomas, but they are characterized by
are most often hyperintense. Polar cysts are also
common findings in myxopapillary ependymo- anaplastic features (i.e. vascular proliferation,
mitotic figures, cellular pleomorphism, and
mas. Myxopapillary ependymomas are low-grade
necrosis) on histologic analysis. Patients
tumors that typically occur in the lumbosacral

Neurosurgery Books Full


www.ketabpezeshki.com 66485438-66485457
436 PART IV SPINAL NEUROSURGERY

experience higher rates of tumor recurrence and dissemination. In most but not all series, the out-
decreased rates of survival. Classic radiographic come for STR followed by EBRT appears to be
features of spinal cord ependymomas include dis- similar to that of complete resection. In patients
tinct tumor-spinal cord border, an associated syr- with tumors at high risk of seeding, when pretreat-
inx, cysts within or adjacent to the mass, and ment CSF cytologic studies reveal malignant cells,
hemosiderin deposits or “caps” near the poles or if the spinal MRI scan shows evidence of lepto-
of the tumor on T1 and T2. The treatment of meningeal disease, the craniospinal axis should be
choice is gross total surgical resection. treated. There is no strong body of evidence
thus far demonstrating that the addition of
Image with permission from Herkowitz et al., Rothman- chemotherapy to EBRT improves the outcome,
Simeone The Spine, 6th ed., Elsevier, Saunders, 2011.
but it is used in pediatric patients with anaplastic
ependymoma or ependymoblastoma are routinely
15. a—Ependymoma given chemotherapy.
Ependymomas arise from ependymal cells and Image with permission from Yachnis AT, Rivera-
typically occur in the central canal of the spinal Zengotita ML, Neuropathology, High-Yield Pathology
cord, the filum terminale, and the white matter Series, Saunders, Elsevier, 2014.
adjacent to a ventricular surface. They are the com-
monest intramedullary spinal cord tumor in adults 16. e—Chromosome 22
and commoner in males than females (the com-
monest intramedullary tumors in children are MISME syndrome (multiple inherited schwan-
astrocytoma, ganglioglioma then ependymoma). nomas, meningiomas, and ependymomas) is seen
The mean age at presentation is 30-40 years with in NF-2, which has a locus on chromosome 22.
long duration of symptoms (e.g. 2-4 years). Two NF-1 gene locus is on chromosome 17, tuberous
thirds occur in the lumbosacral region (40% of sclerosis on chromosomes 9 and 16, neuroblas-
these arise from the filum terminale (myxopapil- toma on chromosome 11.
lary ependymoma). Because of the propensity of
Image with permission from Holland K, Kaye AH, Spinal
these tumors for seeding the craniospinal axis, tumors in neurofibromatosis-2: management consider-
CSF evaluation and MRI of the whole craniosp- ations—a review, J Clin Neurosci. 2009;16(2):169-177.
inal axis is strongly recommended. The three
main subsets of ependymomas are cellular (this 17. a—Astrocytoma
case), myxopapillary, and anaplastic. Cellular
ependymomas are most often located in the cervi- Astrocytomas are the second most common intra-
cal spine. On T1-weighted MRI, they are isoin- medullary spinal cord tumors in adults (30%),
tense to hypointense, whereas on T2-weighted compared to children in which they are the com-
MRI, they are hyperintense and there may be a monest. Almost 60% of these tumors occur in the
syrinx in 50% of cases. Factors prognostic for a cervical and cervicothoracic region, and 20%
favorable outcome include patient age younger have an associated syrinx. Back pain and motor
than 40 years; tumors with a lumbosacral location, deficits are the most common presenting symp-
myxopapillary histologic findings, or a grade of tom in astrocytomas. The most significant prog-
WHO grade I; tumors amenable to GTR or nostic factors in patients with primary spinal cord
STR; and good preoperative function of the astrocytoma are tumor histology, tumor grade,
patient. Whether volume of residual disease age, and performance status. Because of the rare
correlates with a worse outcome after EBRT is nature of this disease, almost all data are based
controversial. Most intradural extramedullary on retrospective reviews fraught with selection
ependymomas are myxopapillary and are often bias. Therefore, neither the extent of resection
amenable to complete surgical resection if they nor treatment with adjuvant irradiation appears
are not multifocal. The goal of surgery is GTR. to be prognostic, although this is controversial.
Every attempt should be made to remove tumors The classic MRI appearance of intramedullary
as a whole as opposed to piecemeal removal, astrocytoma is cord enlargement with a central
because of the risk of seeding, including upward lesion with poorly defined margins, cysts, peritu-
seeding to the cranial nerves. Typically, complete moral edema and patchy enhancement (no
resection is achievable in 80-100% of modern enhancement in 30%). It is typically isointense
series, with 10-year survival for all spinal cord to hypointense on T1-weighted images and
ependymomas is 70-100%. Postoperative EBRT hyperintense on T2-weighted images. The treat-
appears to improve local control in patients with ment of choice for intramedullary astrocytomas is
STR ependymomas and also for patients with complete excision of the tumor, when it can be
high-grade lesions and those with neuraxis safely accomplished without neurologic

Neurosurgery Books Full


www.ketabpezeshki.com 66485438-66485457
31 SPINAL ONCOLOGY 437

compromise. Otherwise, an incomplete excision et al. for intracranial glioblastoma, temozolomide


is typically performed for grade I lesions, and has emerged as a treatment strategy in high-grade
biopsy alone is the surgical strategy for the intramedullary tumors.
non-exophytic component of an infiltrative gli-
oma. GTR is typically extremely difficult to Image with permission from Adam A et al. (Eds),
Grainger & Allison's Diagnostic Radiology, 6th ed.,
achieve because of the infiltrative nature of all 2014, Elsevier, Churchill Livingstone.
but the pilocytic lesions, with most authors
reporting a 0-50% likelihood of GTR for spinal
cord astrocytoma. In patients with favorable
prognostic factors (low-grade histologic findings, ANSWERS 18–24
good performance status, and young age), obser-
vation with serial imaging studies, reserving irra- Additional answers 18–24 available on
ExpertConsult.com
diation for local recurrence, is an appropriate
management option, particularly for young chil-
dren. Radiation should be considered for high-
grade tumors, inoperable tumors, tumors remain-
ing after surgery, and recurring tumors. In the
remainder of patients, adjuvant irradiation is EMI ANSWERS
usually recommended because progression of
tumor in the spinal cord may lead to significant 25. 1—b, Ependymoma; 2—c, Ganglioglioma
neurologic impairment. The overall outcomes
are similar for patients with low-grade gliomas 26. 1—h, Leptomeningeal drop metastasis; 2—g,
of the spinal cord treated either by GTR or Schwannoma; 3—d, Meningioma
STR or biopsy followed by external beam irradi-
ation (EBRT), with most series reporting OS at 27. 1—c, Hemangioma; 2—c, Eosinophilic
5 years of 55-100%. With high-grade tumors in granuloma; 3 ¼ k, Osteoid osteoma; 4—a,
adults and children, the median survival time is Chordoma
quite poor (4-10 months) despite surgery and
EBRT. Extrapolating from the results of Stupp

Neurosurgery Books Full


www.ketabpezeshki.com 66485438-66485457
438
Summary of Epidemiology, Histology, and Imaging Findings of Primary Vertebral Tumors
Associated

PART IV
Sex Syndromes or Histological
Tumor Behavior Age Range Predilection Diseases Location Findings CT Findings MRI Findings

Aneurysmal Benign 1st-3rd Female Posterior “Blood-filled “Soap bubble”, Variable on T1


bone cyst decades (slight) elements sponges” of lytic and and T2;

SPINAL NEUROSURGERY
cavernous cysts multilobulated gadolinium
with walls of enhances
woven bone

Chondroma/ Benign 2nd-3rd Male (2:1) Olier Cervical Hypocellular, Calcified, lytic T1 low to
enchondroma decades syndrome, thoracic chondrocytes intermediate
Maffucci lumbar within lacunae signal; T2
syndrome hyperintense

Eosinophillic Benign 2nd decade Male (2-5:1) Langerhans cell Thoracic “Tennis racquet”- Lytic destruction T1 isointense; T2
granuloma histiocytosis lumbar; shaped Birbeck of vertebral hyperintense
vertebral granules in bodies may cause with soft tissue
bodies and cytoplasm, stains collapse (vertebra swelling
anterior for CD1a plana)
elements

Fibrous Benign 1st-2nd Equal McCune- No preference Sheets of broken Lytic and T1 and T2 with
dysplasia decades Allbright or retracted expansive with variable intensity
syndrome fibroblasts with “ground glass”
woven bone; and sclerotic
foamy margins
macrophages,
cystic changes,
hemorrhage

Hemangioma Benign 4th-6th Female No preference; Multiple small “Honeycomb” T1 isointense; T2


decades (slight) vertebral vessels with pattern on plain hyperintense
bodies sclerotic radiograph; polka gadolinium
trabeculated bone dots of bone on CT enhances

Osteoid osteoma Benign 1st-2nd Male (2-3:1) Lumbar; “Cloudlike;” well- Low-attenuation T1 nidus with
decades posterior circumscribed nidus with intermediate
elements hypodense tumor sclerotic boarder signal intensity;
and surrounding T2 hypointense;
normal trabecula gadolinium nidal
(osteoblastomas enhancement
similar appearing
but >2 cm)

Neurosurgery Books Full


www.ketabpezeshki.com 66485438-66485457
Chordoma Locally 4th-6th Male (2:1) Sacrum, “Physalipharous” Expansive, lytic, T1 isointense; T2
aggressive decades cervical bubble-containing sclerotic hyperintense
(malignant) cytoplasm and gadolinium,
large, round cells; variable
Brachyury,
epithelial
membrane
antigen, 5-100
staining

Giant cell tumor Locally 2nd-4th Female Sacrum Multinucleated Lytic, cystic T1 isointense; T2
aggressive decades (slight) giant cells in hyperintense
(malignant) spindle cell stroma with areas of
hemorrhage;
gadolinium
enhances

Chondrosarcoma Malignant 4th decade Male (2:1) Thoracic Large or Expansive and Markedly T2
binucleated lytic; scalloping of hyperintense;
pleomorphic cells cortex with gadolinium
with small nucleoli cortical expansion “rings and arcs”
pattern

Ewing sarcoma Malignant 1st-3rd Male Sacral Small round cells Lytic T1 isointense; T2
decades (slight) with uniform hyperintense to
nuclei; CD99 is hyperintense
sensitive nut not gadolinium
specific enhances

Multiple Malignant 5th decade Male (2:1) Thoracic Sheets of plasma Lytic, osteoporotic T1 isointense; T2
myeloma/ cells, cytologic hyperintense
solitary atypia gadolinium
plasmacytoma enhances

31
Osteosarcoma Malignant 3rd-6th Male Sacrum Spindle cells with Lytic and T1 hypointense;

SPINAL ONCOLOGY
decades (slight) nuclear destructive with T2 hyperintense
pleomorphism; matrix
osteoid or bone mineralization
production with
the tumor

With permission from Ropper AE, Cahill KS, Hanna JW, et al., Primary vertebral tumors: a review of epidemiologic, histological, and imaging findings, Part I: benign tumors, Neurosurgery.
2011;69(6):1171-1180.

439
Neurosurgery Books Full
www.ketabpezeshki.com 66485438-66485457
CHAPTER 32

SPINAL VASCULAR NEUROSURGERY


SINGLE BEST ANSWER (SBA) QUESTIONS
1. The artery of Adamkiewicz is best described as a. Aneurysm of artery of Adamkiewicz
which one of the following? b. Cavernoma
a. Centripetal artery c. Hemangioblastoma
b. Radiculomedullary artery d. Type I Dural AVF
c. Radiculopial artery e. Type II intramedullary glomus AVM
d. Segmental artery
e. Sulcocommissural artery (centrifugal) 5. A 15-year-old boy presents with neck pain and
bilateral upper limb weakness. MRI is shown
2. Which one of the following best describes below. Which one of the following is most likely?
Foix-Alajouanine syndrome?
a. Involves descending sensory loss
b. Involves hemorrhage from multiple spinal
AVMs
c. Involves normal cerebrospinal fluid protein
level
d. Involves progressive paraparesis due to
congestive myelopathy
e. Involves spinal venous thrombosis on path-
ological examination

3. In the literature, Type II spinal Arteriovenous


malformations (AVMs) generally refer to
which one of the following?
a. Conus medullaris AVM
a. Astrocytoma
b. Dural arteriovenous fistula (AVF)
b. Ependymoma
c. Intradural perimedullary AVF
c. Transverse myelitis
d. Intramedullary glomus AVM
d. Type I Dural (dorsal intradural) AVF
e. Juvenile or mixed AVM
e. Type III juvenile (extradural-intradural)
AVM
4. A 55-year-old male presents with progressive
gait disturbance and impotence. Coronal
6. A 35-year-old with Osler-Weber-Rendu syn-
Magnetic resonance angiogram and angio-
drome presents with gait disturbance. T2W
grams are shown in the lumbar spine. There
MRI is shown below. Which one of the fol-
is no evidence of hemorrhage. VHL screen
lowing is most likely?
is negative. Which one of the following diag-
noses is most likely?

A B

440
Neurosurgery Books Full
www.ketabpezeshki.com 66485438-66485457
32 SPINAL VASCULAR NEUROSURGERY 441

a. Hemangioblastoma temperature sensation on the contralateral


b. Neurenteric cyst side. Which one of the following would you
c. Spinal artery aneurysm consider the appropriate next management
d. Type III juvenile (extradural-intradural) step?
AVF
e. Type IV-C perimedullary (ventral intra-
dural) AVF

7. Which one of the following is most likely from


the images below?

a. CT myelogram
b. Epidural hematoma
c. Prothrombin complex concentrate
d. Spinal angiogram
A B C
e. Spinal decompression
f. Spinal traction
a. Cavernous angioma
b. Hemangioblastoma
c. Hemangioma QUESTIONS 10–11
d. Type I dural (dorsal intradural) AVF
Additional questions 10–11 available on
e. Type II intramedullary (Glomus) AVM ExpertConsult.com

8. A 33-year-old presents with sudden onset neck


pain. MRI is shown below. Which one of the 12. A 54-year-old with a Type II glomus AVM
following is most likely? who has been under surveillance presents
with worsening myelopathy such that they
are now unable to work. Repeat MRI shows
increased T2 signal in the cord at the L1
level. Which one of the following would be
appropriate next?
a. Embolization followed by surgical
decompression at level of AVM only
b. Embolization followed by surgical exci-
sion with interruption of arterial side of
AVM first
c. Embolization followed by surgical exci-
sion with interruption of venous side of
AVM first
e. Endovascular obliteration of primary
a. Cavernous angioma arterial feeders alone
b. Multiple sclerosis f. Surveillance
c. Syringomyelia
d. Type I dural AVF
e. Type II intramedullary glomus AVM

9. A 54-year-old male presents with sudden onset


stabbing neck pain two days ago, and is now
has a left hemiparesis with impaired pain and

Neurosurgery Books Full


www.ketabpezeshki.com 66485438-66485457
442 PART IV SPINAL NEUROSURGERY

EXTENDED MATCHING ITEM (EMI) 14. Venous drainage of spine:


QUESTIONS E

13. Arterial supply to spine:


G
N

M
F
L
J
J K A
E I
D F
F
C G
D
H
I
A

H
C
B

A For each of the following descriptions, select the


most appropriate answer from the image above.
Each answer may be used once, more than once
or not at all.
For each of the following descriptions, select the 1. Anterior median vein
most appropriate answer from the image above. 2. Coronal vein
Each answer may be used once, more than once 3. Dorsal sulcal vein
or not at all. 4. Intramedullary anastamosis
1. Anterior spinal artery 5. Radiculomedullary vein
2. Paravertebral longitudinal anastamosis
3. Radiculomedullary artery 15. Vascular disorders of the spine:
4. Radiculopial artery a. Cavernous angioma
5. Segmental artery b. Conus medullaris AVM
c. Dural AVF
d. Extradural AVF
e. Extradural-intradural (juvenile) AVM
f. Hemangioblastoma
g. Intradural dorsal (dural) AVF
h. Intradural ventral (perimedullary) AVF

Neurosurgery Books Full


www.ketabpezeshki.com 66485438-66485457
32 SPINAL VASCULAR NEUROSURGERY 443

i. Intramedullary glomus AVM Each answer may be used once, more than once
j. Paraspinal AVM or not at all.
k. Spinal artery aneurysm 1. Associated with hereditary hemorrhagic
telangiectasia
For each of the following descriptions, select the 2. Associated with Cobb syndrome
most appropriate answer from the list above.

SBA ANSWERS
1. b—Radiculomedullary artery the ASA is found angiographically at the
lower thoracic and lumbar levels.
The simplified algorithm for the vascular supply
at each segmental level is major arterial trunk 2. d—Involves progressive paraparesis due to
(vertebral artery, aorta) ! spinal/segmental congestive myelopathy
artery ! radicular branches which give off branches
to the paraspinous musculature, vertebral body, The exact etiology of the process which resulted
and dura before forming: in the progressive and fatal neurological deterio-
1. Radiculopial arteries supply the nerve roots by ration the two patients described by Foix and
means of smaller branches and then run ven- Alajouanine in 1926 is still a cause for debate, lead-
tral to either the dorsal or ventral nerve root ing to confusion about it's actual definition. Both
to supply the pial network. Although they patients presented with a progressive lower limb
anastomose with pial branches of the ASA, weakness, incontinence and sensory level; patho-
they do not supply the ASA directly. There logical examination in both cases revealed myelitis
are more dorsal than ventral radiculopial and spinal cord necrosis, with extensive hypertro-
arteries, and the dorsal radiculopial arteries phy of intradural vessels (veins on cord surface,
are the dominant supply to the PSA. Their extramedullary veins, intramedullary veins and
number varies from three to four in the cer- to a lesser extent some arteries); histological anal-
vical region, from six to nine in the thoracic ysis showed vessels endo-meso-vasculitis with
region, and from zero to three in the lumbo- necrotizing tendencies with widely patent vessel
sacral region. lumens. Despite the authors specifically excluding
2. Radiculomedullary arteries are the dominant thrombosis on pathological examination, the
supply to the ASA. After giving off their Foix-Alajouanine syndrome was initially associ-
radicular branches to the nerve roots, they ated with spinal artery thrombosis leading to mye-
run along the ventral surface of the nerve lopathy. However, some patients diagnosed with
root, occasionally give off pial collateral Foix-Alajouanine syndrome improved, making
branches, and continue to the ASA. On aver- thrombosis of the spinal artery an unlikely cause.
age, there are two to four radiculomedullary Afterward, in 1989 Criscuolo et al. also pointed
arteries in the cervical region, two to three in against its association with thrombosis and
the thoracic region, and zero to four in the explained that the symptoms of this syndrome
lumbosacral region. The largest radiculome- could be explained by congestive myelopathy,
dullary artery of the thoracolumbar segment which is a reversible process. As such, nowadays
is also known as the artery of Adamkiewicz. In it may also refer to spinal AVM (usually intradural
75% of patients, the AKA arises between T9 AVF) patients with clinically subacute to chronic
and T12, more commonly on the left. When progressive neurological symptoms due to con-
its origin is above T8 or below L2, another gestive myelopathy without hemorrhage.
major contributor to the ASA can be found
either cranially or caudally. In 30-50% of FURTHER READING
cases, it also contributes significantly to the Criscuolo GR, Oldfield EH, Doppman JL. Reversible acute
PSA. Generally, a pair of arteries arises in and subacute myelopathy in patients with dural arteriovenous
the cervical region from the intradural seg- fistulas. Foix-Alajouanine syndrome reconsidered. J Neuro-
ment of each vertebral artery that fuse to surg 1989;70(3):354-9.
one “Y”-shaped ASA running in the subpial
space in the ventral sulcus of the spinal cord Ferrell AS, Tubbs RS, Acakpo-Satchivi L, Deveikis JP,
(dorsal to the anterior spinal vein) to the ter- Harrigan MR. Legacy and current understanding of the
often-misunderstood Foix-Alajouanine syndrome. Historical
minal film. The typical hairpin anastomosis vignette. J Neurosurg 2009;111(5):902-6.
between the radiculomedullary arteries and

Neurosurgery Books Full


www.ketabpezeshki.com 66485438-66485457
444 PART IV SPINAL NEUROSURGERY

3. d—Intramedullary glomus AVM FURTHER READING


Spetzler RF, Detwiler PW, Riina HA, Porter RW. Modified
Classification of spinal AVMs is another area of classification of spinal cord vascular lesions. J Neurosurg
controversy. Two salient classifications for neu- 2002;96:145-56.
rosurgeons are shown below:
4. d—Type 1 Dural AVF
FURTHER READING
Bao YH, Ling F. Classification and therapeutic modalities of Dural arteriovenous fistulas comprise 80-85% of
spinal vascular malformations in 80 patients. Neurosurgery spinal AVMs. These lesions show a male predom-
1997;40(1):75-81. inance (80-90%) and generally present in late
adulthood, ages 40-60. Presentation is generally
Spinal Cord Vascular Lesions (Spetzler with radiculomyelopathy, followed by slow but
et al. 2002) progressive neurological deterioration. Site of
pathology in these lesions is within the dural root
Modified Classification Previous Name sleeve, where a direct arteriovenous fistula
AV fistulas
develops, generally with a single dural artery
feeder vessel. Venous drainage of the AV fistula
Extradural AVF Epidural AVF is by a high pressure, low-flow arterialized vein
Dorsal intradural AVF Dural AVF (IA) intradurally. Chronic venous hypertension yields
chronic spinal cord ischemia, cell loss, and cord
Ventral intradural AVF Perimedullary AVF atrophy. Impaired autoregulation yields direct
Arteriovenous malformation transmission of changes in systemic arterial
pressure to the spinal cord without the normal
Extradural-intradural AVM Juvenile AVM
dampening effect of the venous plexus. Goal of
Intramedullary AVM Glomus AVM treatment is isolation and obliteration of the fistula
and draining veins, which normalizes venous
Conus medullaris AVM -
pressure and corrects venous hypertension.
Neoplastic vascular lesion
Image with permission from Krings T, Lasjaunias PL,
Hemangioblastoma - Hans FJ, et al. Imaging in spinal vascular disease. Neu-
Cavernous malformation - roimaging Clin N Am 2007;17(1):57-72.

Spinal cord aneurysm -

Classification of Spinal Vascular Malformations (Bao & Ling 1997)


Type Name Description Age

I Dural AVF AVFs between a radicular or radiculomedullary artery and a medullary vein, with 40-60
the fistulous connection located in the dural root sleeve. These malformations are
likely often acquired and arise predominantly at thoracic and thoracolumbar
levels, almost universally dorsal. Most authors have classified this lesion as type A
if fed by a single arterial feeder and type B if fed by two or more feeders

II Intramedullary Well-defined, completely or partially intramedullary lesions consisting of a distinct 20-30


glomus AVM conglomeration of dysmorphic arteries and veins in direct communication without
an intervening capillary bed. Type II AVMs of the cervical spinal cord frequently
have multiple feeding vessels of anterior spinal, posterior spinal, and
radiculomedullary arteries, whereas Type II AVMs of the thoracic spinal cord or
conus are often supplied via a single enlarged branch of the anterior spinal artery.
Latent anastomotic channels invariably exist, however, which may emerge after
proximal ligation or endovascular occlusion of the primary feeding vessel

III Juvenile or Rare. Consist of diffuse arteriovenous shunts with variable degrees of involvement 10-20
mixed AVM of the spinal cord, vertebral, and paraspinal tissues. Other metameric anomalies of
associated organs and the skin are commonly associated with these lesions

IV Intradural Most occur in the thoracolumbar region as a fistula between the anterior spinal artery 20-40
perimedullary (ASA) and vein (ASV) on the ventral spinal cord surface
AVF Type IV-A malformation is a small, simple direct fistula with a single ASA feeder
Type IV-B lesions are medium-sized fistulas and have additional, smaller feeding
vessels arising from either the ASA or posterior spinal artery (PSA)
Type IV-C fistulas are giant sized and demonstrate several enlarged ASA and PSA
feeding branches with dilated venous outflow

Neurosurgery Books Full


www.ketabpezeshki.com 66485438-66485457
32 SPINAL VASCULAR NEUROSURGERY 445

5. e—Type III juvenile (extradural- technically difficult and may jeopardize


intradural) AVM the spinal cord. Treatment is hence
through combination of endovascular abla-
Juvenile spinal AVMs are extremely rare lesions. tion, followed by surgical excision of
These lesions are again true AVMs, with an intra- retained elements.
medullary nidus which may occupy the entire spinal
canal at the involved level. Cord tissue is present Image with permission from Krings T, Lasjaunias PL,
within the AVM interspaces. Extramedullary and Hans FJ, et al. Imaging in spinal vascular disease. Neu-
roimaging Clin N Am 2007;17(1):57-72.
even extraspinal extension of the lesion is possible.
Juvenile AVMs are large and complex lesions, with 7. e—Type II intramedullary (glomus) AVM
multiple arterial feeding vessels often arising from
different cord levels. Hemodynamically, this lesion Type II intramedullary (glomus) AVMs are charac-
manifests both high flow and high pressure, often terized by a compact intramedullary nidus, with
yielding an auscultatable spinal bruit over involved feeding vessels arising from the anterior or posterior
levels. They occur most commonly in adolescents spinal arteries, or both, and drainage into an arteria-
and young adults. Presentation and treatment are lized coronal venous plexus. Unlike spinal AV fistu-
similar to Type II AVMs; however, prognosis for las they are high pressure, low-flow lesions (rapid
these lesions, considering their size and vascular filling on angiogram and early venous drainage).
complexity, is understandably very poor. MR imag- Affect men and women equally, and mean age of
ing typically demonstrates prominent flow voids presentation in 20s. The clinical course of these
suggestive of underlying vascular malformation. lesions is marked by progressive and fluctuating
Edema or gliosis can also be seen on MR imaging. myelopathy, often overlaid by periods of acute neu-
rologic deterioration secondary to hemorrhage
Image with permission from Fatterpekar GM, Naidich TP,
Som PM (Eds.), Teaching Files: Brain and Spine, Elsevier, within the AVM. Sudden apoplectic presentation,
Saunders, 2012. often with profound neurologic impairment and
possible transverse myelopathy is common. SAH
6. e—Type IV-C perimedullary AVF often occurs in these lesions, occurring in 50% of
cases. True intramedullary AVMs occur throughout
Perimedullary (ventral intradural) AVFs have a fistu- the cord hence presentation with upper extremity
lous connection which is intradural but symptoms is possible. Imaging demonstrates intra-
extramedullary, with feeding vessel(s) from the ante- medullary AVM at C3-C4 levels with associated
rior spinal artery. Venous drainage is via an enlarged hematoma and flow voids from arterialized coronal
coronal venous plexus. These lesions may present in venous plexus of the spinal cord, while angiogram
young adults, but presentation in the third to sixth demonstrates feeders arising from the left vertebral
decade is more likely. SAH is possible with subse- artery. Treatment involves initial embolization of
quent acute neurologic deterioration, but a gradual feeding vessels using particulate matter. Immediate
progressive neurologic deterioration is common. clinical improvement is often noted after emboliza-
Three subcategories of intradural spinal AV fistulas tion, through reduction in arterial steal and
have been recognized, with different treatment improved cord perfusion; however, recanalization
options appropriate for each. may occur over time, with continued risk of hemor-
• Type IVa has a single feeding vessel, often rhage. Hence, surgical resection of residual nidus
the artery of Adamkiewicz, with low flow after embolization is generally reasonable.
through the arteriovenous shunt and mod-
erate venous enlargement. Endovascular Image with permission from Kim DH, et al. (Eds.), Sur-
techniques are difficult with these lesions gical Anatomy and Techniques to the Spine, 2nd ed.,
Elsevier, Saunders, 2013.
due to the small size of feeding vessels
hence surgical excision is often mandated. 8. a—Cavernous angioma
• Type IVb AV fistulas are medium-sized, often
with multiple feeding vessels, and more Central nervous system (CNS) cavernomas are
marked venous enlargement. Embolization in rare vascular malformations consisting of closely
these lesions is easier, due to the increased size packed large sinusoid-like vascular channels with
of feeding vessels. In cases of incomplete shunt little or no intervening nervous tissue. They can
obliteration with an endovascular approach, occur anywhere in the CNS but favor the cerebral
direct surgical excision may be necessary. hemisphere. Spinal cord cavernomas are uncom-
• Type IVc are giant, multipediculated, high- mon, accounting for 3-5% of all cavernous mal-
flow fistulas with large, tortuous draining formations. Most commonly seen in the thoracic
veins. Spinal ischemia may develop in these region (50%), they are uncommonly identified
lesions secondary to vascular steal. Due to in the conus (10%); the cervical cord accounts
the size of these lesions, surgery is for 40% of such lesions. Sudden onset paraplegia

Neurosurgery Books Full


www.ketabpezeshki.com 66485438-66485457
446 PART IV SPINAL NEUROSURGERY

in a young adult. MR is the imaging modality of collection with variable cord compression. MR will
choice. The typical imaging features are of a demonstrate a lentiform collection in the epidural
well-defined lesion causing focal expansion of compartment, the signal intensity of which will vary
the cord with mixed signal intensity on T1WI with the age of the hemorrhage (Fat-suppressed
and T2WI. These lesions are typically surrounded T1WI can be used to distinguish blood from fat).
by a complete hypointense rim owing to hemo- Early surgical decompression is standard, and if
siderin deposition. Typically, no or only sparse the exact location of the hematoma cannot be
edema is associated with this lesion. Enhancement detected and confirmed by MRI, the dura should
following contrast administration is variable. be opened to exclude a subdural hematoma. Con-
servative treatment has also been documented, usu-
Image with permission from Fatterpekar GM, Naidich TP, ally when neurological deficits improved in the
Som PM (Eds.), Teaching Files: Brain and Spine, Elsevier,
Saunders, 2012.
early phase or with the coexistence of coagulopathy
and multilevel acute epidural hematomas.
9. e—Spinal decompression
FURTHER READING
Epidural hematoma can occur secondary to trauma, Kreppel D, Antoniadis G, Seeling W. Spinal hematoma: a
iatrogenic procedures, or vascular malformation or literature survey with metaanalysis of 613 patients. Neurosurg
Rev 2003;26(1):1-49.
sometimes can be seen as a spontaneous occur-
rence, such as in patients with coagulopathy. Spinal Image with permission from Fatterpekar GM, Naidich TP,
extradural haematoma (EDH) is seen to occur Som PM (Eds.), Teaching Files: Brain and Spine, Elsevier,
most commonly in the thoracic and lumbar region Saunders, 2012.
in adults, whereas in children, it is seen to occur in
the cervicothoracic region. The dorsal aspect of the
canal is the most common site for an epidural hema-
toma. Most spinal EDHs have a venous source of ANSWERS 10–11
hemorrhage. Spinal hemorrhage (epidural, sub- Additional answers 10–11 available on
dural or subarachnoid) presents with intense, stab- ExpertConsult.com
bing pain at the location of the hemorrhage (“coup
de poignard of Michon”) that may be followed in
some cases by a pain-free interval of minutes to
days, after which there is progressive paralysis 12. b—Embolization followed by surgical exci-
below the affected spinal level. CT scan will dem- sion with interruption of arterial side of
onstrate lentiform or biconvex hyperdense AVM first

Summary of Management of Spinal Cord Vascular Lesions


Type Management Notes

I (Dural AVF) Surgical interruption of the intradural draining vein


More complex/recurrent fistula excision definitively prevents re-establishment of retrograde
intradural venous drainage through collateral longitudinal extradural venous channels at
adjacent radicular levels. Several millimeters of the feeding radicular artery and intradural
draining vein may be cauterized, divided, and contiguously excised along with a small
window of dura on the root sleeve. Fistula obliteration rate is significantly lower with
embolization, and progressive myelopathy is a risk due to delay in definitive treatment

II (Glomus AVM) Classic AVM—require surgical excision with or without preoperative embolization.
Interruption of the venous side of an AVM first can lead to hazardous elevations in pressure in
the remaining venous drainage system, producing either excessive bleeding around the
AVM or rupture of associated venous aneurysms

III (juvenile AVM) Most difficult to treat—no well-defined margin for resection
Partial treatment through embolization, surgical decompression and limited arterial clip
ligation may produce some clinical benefit but it is unlikely to last and is not without
significant risk

IV (perimedullary Surgical ligation is definitive for small (Type IV-A) shunts


AVF) For more complex, higher-flow lesions (Types IV-B and C), endovascular obliteration of the
shunt may be the preferred primary treatment or at least a preoperative adjunct

Cavernoma For pial-based lesions, a circumscribing pial incision allows detachment and delivery of the
cavernoma from the superficial substance of the spinal cord. Deeper intramedullary lesions
are exposed by a midline myelotomy. Although unencapsulated, these malformations are
generally well circumscribed and present a clear dissection plane

Neurosurgery Books Full


www.ketabpezeshki.com 66485438-66485457
32 SPINAL VASCULAR NEUROSURGERY 447

Successful treatment of spinal vascular malforma- 14. 1 ¼ D, 2 ¼ C or J, 3 ¼ G, 4 ¼ F, 5 ¼ H


tions requires the total obliteration or excision
of the abnormal shunt. Procedures that only
Posterior median
partially reduce the shunt or address only proxi- vein
mal feeders may provide temporary benefit but Dorsal sulcal
all too often lead to delayed recurrences and vein
intervening neurological decline. Management
options are outlined below, but surgery and/or
endovascular approaches will be dictated by clin-
ical, MRI and spinal angiography findings. Radial/coronal
veins Dorsal nerve
Intraoperative angiography (or other vessel imag- root
ing) may be used to confirm shunt elimination, Intramedullary
and electrophysiological monitoring may be use- anastomosis
Anterior median
ful in either setting. Whether endovascular vein
Radiculomedullary
vein
techniques occupy a primary or adjunctive role Ventral longitudinal
depends not only on the type of vascular lesions vein Dorsal nerve
root
but also on institutional experience.
Radial/coronal
veins Ventral nerve root

EMI ANSWERS
Image adapted with permission from Mathis JM, Shai-
bani A, Wakhloo AK. Spine anatomy. In: Mathis JM
13. 1 ¼ L, 2 ¼ E, 3 ¼ F, 4 ¼ M, 5 ¼ A (Ed.), Image-Guided Spine Interventions. New York,
Springer-Verlag, 2004.

Posterior
spinal arteries
15. 1 ¼ h, Perimedullary AVF, 2 ¼ d, Extradural-
intradural (juvenile) AVF
Radiculopial
artery

Radiculomedullary
Osler-Weber-Rendu syndrome (Hereditary
artery
Anterior spinal
Hemorrhagic Telangiectasia) is an autosomal
Radicular branches artery dominant syndrome and consists of two geno-
to the ventral nerve root
Radicular branches
types (Types I and II). Type I is associated with
to the dorsal nerve root mucocutaneous telangiectasia, pulmonary AVF,
Paravertebral longitudinal Dural sheath
anastomosis and arteriovenous shunts of the central nervous
Radiculomedullary
Dorsospinal trunk
artery system. The associated spinal arteriovenous
Intercostal artery
or muscular branch
Dorsal somatic shunts are most often seen in the pediatric pop-
branch
ulation and are always pial AVF (subtype C, ven-
tral intradural AVF, or Type IV). The
endothelial cells in this syndrome lack the mol-
ecule endoglin and form abnormal vessels, espe-
cially after injury.
Spinal nerve
Cobb syndrome is a synonym for the complete
manifestation of the juvenile/metameric type of
spinal vascular malformation where skin, muscle,
Somatic branches
bone, dura, and spinal cord are involved. Klippel-
for vertebral body Trenaunay and Parkes-Weber syndromes consist
supply
of vascular malformations involving primarily
Segmental artery the lower limbs, with the following dominant
features: cutaneous capillary malformation,
Image adapted with permission from Mathis JM,
Shaibani A, Wakhloo AK. Spine anatomy. In Mathis varicose veins, and limb hypertrophy. Klippel-
JM (Ed.), Image-Guided Spine Interventions. New York, Trenaunay syndrome is composed mainly of
Springer-Verlag, 2004. venous anomalies; Parkes-Weber syndrome has
more arteriovenous shunts.

Neurosurgery Books Full


www.ketabpezeshki.com 66485438-66485457
PART V
FUNCTIONAL
NEUROSURGERY
CHAPTER 33

PAIN SURGERY
SINGLE BEST ANSWER (SBA) QUESTIONS
1. A 55-year-old male with known ischemic heart 3. Which one of the following is not a potential
disease develops a crushing chest pain which site for ulnar nerve entrapment?
radiates into his neck and left arm. The phe- a. Arcade of Struthers
nomenon of referred pain is best explained b. Heads of the flexor carpi ulnaris
by which one of the following mechanisms? c. Intermuscular septum
a. Pain transmission along a given afferent d. Osbourne's fascia
nerve is transferred to another afferent e. Sublimis arch
pathway due to ephaptic transmission
b. Lateral inhibition of secondary afferent 4. A 43-year-old woman is referred with a 2-
fibers by a single primary afferent year history of sharp, shooting pains radiat-
c. Convergence of primary afferent fibers ing into the right side of her jaw. Each attack
from a specific part of the body onto lasts for a few seconds but they seem to merge
second-order neurons that normally together so it can seem like several minutes of
receive primary afferents from a different pain. After this she may be pain free for sev-
body part eral hours. It is often triggered by eating/
d. Disruption of dorsal root ganglia drinking, brushing her teeth, touching her
e. The blockade of substance P and gluta- face, and talking. She denies any tearing,
mate in the dorsal horn eye/eyelid changes or nasal congestion/rhi-
norrhea. There is no other past medical or
2. The descending pathway for central control family history. Neurological examination is
of nociception is best described as involving normal and MRI head was unremarkable.
which one of the following structures? Which one of the following is most appropri-
a. Anterior cingulate cortex fibers that syn- ate first line therapy?
apse directly on dorsal horn neurons a. Baclofen
b. Periaqueductal gray fibers that synapse b. Carbamazepine
nucleus raphe magnus neurons, which in c. Gabapentin
turn synapse on dorsal horn neurons d. Lamotrigine
c. Thalamic neurons that synapse upon red e. Pimozide
nucleus neurons, which in turn synapse
upon dorsal horn cells 5. A 22-year-old female presents to her GP with
d. Hypothalamic fibers that synapse upon a 3-month history of sharp, stabbing jaw pain
neurons of the nucleus solitarius that then and blurred vision on the right side. She is
synapse upon neurons of the dorsal horn about to sit her examinations and is finding
e. Thalamic fibers that synapse upon inhib- it difficult to study because of her symptoms.
itory interneurons in the dorsal horn Which one of the following would you wish
to exclude as a priority?

448
Neurosurgery Books Full
www.ketabpezeshki.com 66485438-66485457
33 PAIN SURGERY 449

a. Tolosa-Hunt syndrome c. Development of an catheter tip granuloma


b. Multiple sclerosis is particularly associated with morphine
c. Intracranial tumor d. Intrathecal baclofen has a potent analgesic
d. Classical trigeminal neuralgia effect
e. Psychogenic pain syndrome e. Indications for intrathecal drug delivery
do not include patient preference.
6. Which one of the following statements
regarding surgical treatments for trigeminal 10. Which one of the following statements
neuralgia is LEAST accurate? regarding motor cortex stimulation for pain
a. Microvascular decompression is a favored is LEAST accurate?
option for trigeminal neuralgia in MS a. Electrodes may be placed in the subdural or
b. Radiofrequency thermocoagulation of extradural space
gasserian ganglion is most likely to cause b. Subdural placement of electrodes may be
anesthesia dolorosa required to achieve lower limb pain coverage
c. Percutaneous destructive techniques c. Risk of seizures is highest intraoperatively
directed at the gasserian ganglion may and during programming
cause a trigemino-cardiac reflex d. Is appropriate for deafferentation pain
d. Balloon compression is usually performed syndromes
under general anesthetic e. Is not indicated in patients with multiple
e. Stereotactic radiosurgery of the trigemi- sclerosis related chronic pain syndromes
nal root only produces pain relief in a due to high seizure risk
delayed fashion

7. Gate control theory of pain is best described QUESTIONS 11–15


by which one of the following statements?
Additional questions 11–15 available on
a. The thalamus gates pain perception in the ExpertConsult.com
cortex
b. The substantia gelatinosa gates nocicep-
tive signals before they reach the thalamus
c. The dorsal root ganglion gates nocicep- EXTENDED MATCHING ITEM (EMI)
tive signals before they reach the substan-
tia gelatinosa QUESTIONS
d. The nociceptive nerve endings gate pain
signals by altering the ratio of C and 16. Sensory disturbances:
A-delta fibers activated a. Allodynia
e. The dorsal columns gate nociceptive b. Analgesia
signals by ephaptic transmission c. Anesthesia dolorosa
d. Causalgia
8. Which one of the following is the commonest e. Dysesthesia
vessel thought to cause compression of the tri- f. Hyperesthesia
geminal root in cases of trigeminal neuralgia? g. Hyperalgesia
a. Anterior inferior cerebellar artery h. Hyperpathia
b. Basilar artery i. Hypoesthesia
c. Dandy's vein j. Hypoalgesia
d. Posterior inferior cerebellar artery k. Neuropathic pain
e. Superior cerebellar artery l. Nociceptive pain
f. Vertebral artery m. Paresthesiae

9. Which one of the following statements For each of the following descriptions, select the
regarding intrathecal drug delivery systems most appropriate answers from the list above.
is LEAST accurate? Each answer may be used once, more than once
a. The intrathecal route results in higher or not at all.
subarachnoid drug concentrations, lower 1. An unpleasant abnormal sensation, whether
absolute drug doses, and avoidance of side spontaneous or evoked.
effects associated with systemic dosing 2. A painful syndrome characterized by an
b. Ziconotide is exclusively intrathecal analge- abnormally painful reaction to a stimulus,
sic which blocks a N-type calcium channel especially a repetitive stimulus, as well as
an increased threshold.

Neurosurgery Books Full


www.ketabpezeshki.com 66485438-66485457
450 PART V FUNCTIONAL NEUROSURGERY

3. A syndrome of sustained burning pain, allo- h. Post-spinal cord injury


dynia, and hyperpathia after a traumatic nerve i. Post-thoracotomy pain syndrome
lesion, often combined with vasomotor and j. Post-traumatic pain
sudomotor dysfunction and later trophic
changes. For each of the following descriptions, select the
4. Pain in an area or region which is most appropriate answers from the list above.
anesthetic. Each answer may be used once, more than once
5. Pain due to a stimulus that does not nor- or not at all.
mally provoke pain. 1. Persistent or recurring low back pain, with
or without sciatica, after one or more spine
17. Craniofacial pain syndromes: surgeries.
a. Anesthesia dolorosa 2. Pain after acute rash has healed accompa-
b. Cluster headache nied by pain, allodynia, paresthesia, or dys-
c. Geniculate neuralgia esthesia. The pain usually affects a single
d. Glossopharyngeal neuralgia dermatome.
e. Migraine 3. Present with severe pain that is disproportion-
f. Occipital neuralgia ate to the inciting event, most commonly
g. Persistent idiopathic facial pain affecting the hand or foot but that can spread
h. Temporomandibular joint disorder to other body regions. The affected body parts
i. Tension-type headache may display sensory disturbances, tempera-
j. Trigeminal neuralgia ture changes, abnormal patterns of sweating,
edema, reduced joint range of motion, move-
For each of the following descriptions, select the ment abnormalities such as weakness, tremor,
most appropriate answers from the list above. or dystonia, trophic changes such as skin
Each answer may be used once, more than once atrophy or altered hair and nail growth, and
or not at all. localized osteoporotic changes.
1. A 58-year-old presents with paroxysmal
attacks of stabbing pain in the throat which 19. Surgical procedures for pain:
occasionally shoots to her right ear, usually a. Anterolateral cordotomy
lasting less than 1 min. She experiences some b. Cingulotomy
relief with cocainization of the right tonsil. c. Deep brain stimulation
2. Brief paroxysms of pain felt deep in the d. DREZ lesions
auditory canal. A trigger area is present in e. Intrathecal morphine
the posterior wall of the auditory canal. f. Midline myelotomy
May benefit from transection of nervus g. Mesencephalic tractotomy
intermedius. h. Motor cortex stimulator
3. Pain attacks of severe or very severe unilat- i. Spinal cord stimulation
eral orbital, supraorbital, and/or temporal j. Sympathectomy
pain lasting 15-180 min if untreated. May k. Thalamotomy
be accompanied by ipsilateral conjunctival
injection and/or lacrimation, nasal conges- For each of the following descriptions, select the
tion, and/or rhinorrhea, eyelid edema, fore- most appropriate answers from the list above.
head and facial sweating, miosis and/or Each answer may be used once, more than once
ptosis and a sense of restlessness or agita- or not at all.
tion. Attacks have a frequency from 1 every 1. Involves disconnection of the anterior
other day to 8 per day. commissure
2. Involves lesioning Lissauer's tract
18. Spine and peripheral pain syndromes: 3. Involves a cord lesion just anterior to the
a. Brachial plexus avulsion dentate ligament
b. Chronic low back pain 4. Involves a lesion of extralemniscal pathways
c. CRPS lateral to the spinothalamic tract
d. Diabetic peripheral neuropathic pain 5. Aims to treat the motivational-affective
e. Failed back surgery syndrome component of pain that contributes to fear,
f. Phantom limb pain suffering, and anxiety.
g. Post-herpetic neuralgia

Neurosurgery Books Full


www.ketabpezeshki.com 66485438-66485457
33 PAIN SURGERY 451

SBA ANSWER
1. c—A convergence of primary afferent fibers proximally by cutting intermuscular septum and
from a given region onto second-order arcade of Struthers if present; assess position in full
neurons that also normally receive primary range of motion to ensure no subluxation. Recur-
afferents from a different body part rence of symptoms after decompression alone may
suggest kinking/tethering at the intermuscular
Referred pain is a phenomenon in which pain septum (or arcade of Struthers if not divided ini-
impulses, usually arising from primary visceral tially) requiring release.
afferent fibers from one part of the body, terminate
on dorsal horn projection neurons that normally 4. b—Carbamazepine
receive cutaneous afferents from a different part
of the body (such as the arm). It is the convergence The history is suggestive of trigeminal neuralgia
of these distinctly different inputs onto the same (tic douloureux). First line treatment is carbamaz-
projection neurons that provides the basis for this epine. Common side effects of CBZ: dizziness,
phenomenon. However, other theories including vertigo, ataxia, diplopia, blood disorders, drowsi-
central sensitization, thalamic convergence, and ness, skin reactions. If CBZ not tolerated due to
hyperexcitability have also been proposed. Pain side effects could try oxcarbazepine (higher risk
signaling in the spinal cord dorsal horn is via glu- of hyponatremia), or if allergic start gabapentin.
tamate (C fibers and A-delta fibers) and substance Other useful drugs include baclofen, phenytoin,
P (C fibers). The opioid peptides (enkephalin and and lamotrigine.
dynorphin) are released by inhibitory interneurons
in the dorsal horns, medulla, and PAG and inhibit 5. b—Multiple sclerosis
transmission of pain impulses.
While the majority of cases of trigeminal neural-
2. b—Periaqueductal gray that synapse on neu- gia are due to vascular compression or idiopathic
rons of the nucleus raphe magnus that then (Classical), in a small proportion of cases it may be
synapse on dorsal horn cells the presenting feature of significant underlying
condition such as multiple sclerosis (brainstem
Periaqueductal gray stimulation activates plaque causing ephaptic transmission), basilar
enkephalin-releasing neurons that project to the artery aneurysms, acoustic schwannomas, and
nucleus raphe magnus, 5-HT release activates pro- posterior fossa meningiomas, all of which may
jections to inhibitory interneurons in Laminae II cause injury to the fifth cranial nerve by compres-
(substantia gelatinosa). This results in release of sion. Red flag symptoms which should be
either enkephalin or dynorphin (endogenous opi- excluded include: sensory changes, deafness, dif-
oid neurotransmitters), which bind to mu opioid ficulty achieving pain control, poor response to
receptors on the axons of incoming nociceptive carbamazepine, history of skin or oral malignancy
C and A-delta fibers, inhibiting the release of sub- that could lead to perineural spread, symptoms in
stance P/glutamate from then and activation of ophthalmic distribution (more likely herpes zos-
ascending pain pathways. ter), under age of 40, symptoms suggestive of
optic neuritis, family history of MS. The
3. e—Sublimis arch (fibrous arch between Tolosa-Hunt syndrome is a presumably inflam-
flexor digitorum superficialis) matory disorder that produces ophthalmoplegia
associated with headache and loss of sensation
Ulnar nerve at the elbow lies in the postcondylar over the forehead. Pupillary function is usually
groove, then under the aponeurosis (Osbourne's spared, and the site of pathology is believed to
fascia) then under the FCU itself. Initial benefit be in the superior orbital fissure or the cavernous
following ulnar nerve transposition at the cubital sinus. It is usually not associated with trigeminal
tunnel may be complicated by kinking at the neuralgia.
arcade of Struthers more proximally, which may
necessitate release at this level. Technique for 6. a—Microvascular decompression is a favored
ulnar nerve decompression  transposition is: option for trigeminal neuralgia in MS
incision, identify, and protect/mobilize medial
antebrachial cutaneous nerve, identify ulnar nerve Microvascular decompression is the gold standard
proximal to Osbourne's fascia, decompress distally in good surgical candidates (i.e. classical trigemi-
through leading edge of FCU (simple decompres- nal neuralgia, evidence of vascular compression
sion); for transposition, extend decompression on MRI, short duration of disease, no previous

Neurosurgery Books Full


www.ketabpezeshki.com 66485438-66485457
452 PART V FUNCTIONAL NEUROSURGERY

surgery); pain relief is usually immediate and 60- various midbrain regions are thought to contrib-
70% remaining pain free at 10-20 years. Indica- ute to the motivational-affective component of
tions include patients whose pain is no longer pain. Primary somatosensory cortex, thalamus,
controlled by drugs and whose quality of life has spinothalamic tract, and local nerve endings are
markedly deteriorated, young patients, side involved in the sensory component of pain.
effects from antiepileptic drugs. The procedure
requires general anesthesia, and its adverse effects
FURTHER READING
include aseptic meningitis, ipsilateral hearing loss Bourne S, et al. Basic anatomy and physiology of pain path-
(in less than 5%), CSF leak, low risk of sensory ways. Neurosurg Clin N Am 2014;25:629-638.
loss, and death (0.4%). Stereotactic radiosurgery
targeting the trigeminal root does not require 8. e—Superior cerebellar artery
general anesthesia, but its pain-relieving effects
are not immediate. Adverse effects include facial Neurovascular compression is found in 88% of
numbness, paresthesias, and sensory complica- cases on imaging, but an average of 7% of explo-
tions. Percutaneous destructive neurosurgical rations reveal no pathology intraoperatively. The
techniques (radiofrequency thermocoagulation, superior cerebellar artery is the most common
glycerol rhizolysis, or balloon compression) can (70-80%), and is usually compressing the rostral
achieve immediate pain relief and can therefore and anterior portion of the nerve in patients with
be considered for emergency management, but V2 or V3 symptoms. The anterior inferior cere-
50% of patients have recurrence at 5 years. Seda- bellar artery is the compressive vessel in 10% of
tion or even GA is required, and it carries a risk of cases and occurs in the caudal and posterior por-
trigeminal-vagal reflex effects on the heart during tion of the nerve closest to nerve VI, while veins
lesioning and a very small risk of carotid injury or impact the nerve in 5-13% of cases. Microvascu-
intracranial infection. Balloon compression lar decompression involves separating offending
carries a risk of temporary trigeminal motor vessels from the nerve and inserting a synthetic
dysfunction. Adverse effects include a facial sponge or Teflon felt between them to maintain
numbness, corneal numbness (risk of keratitis), the separation. The trigeminal nerve must be
dysesthesias, and anesthesia dolorosa. carefully and circumferentially inspected along
its entire intracranial course from the root entry
FURTHER READING zone to its entrance laterally into Meckel's cave.
Zakrzewska Joanna M, Linskey Mark E. Trigeminal neural- This procedure ordinarily provides pain relief
gia. BMJ 2015;350:h1238. without any facial sensory loss and has a greater
potential for producing long-lasting pain relief.
7. b—The substantia gelatinosa gates nocicep- Interestingly, tic pain does not always stop imme-
tive signals before they reach the thalamus
diately following a microvascular decompression,
and nerve manipulation by itself (e.g. where no
The 1965 gate control theory of pain by Melzack
offending vessel found, or without moving the
and Wall proposed that there were three spinal
artery) transiently stops the tic pain although it
cord systems involved in pain transmission: the
will soon recur thereafter.
substantia gelatinosa, dorsal column fibers, and
central transmission cells in the DH. The sub-
FURTHER READING
stantia gelatinosa functions as a gate that modu- Grant GA, Loeser JD. Trigeminal neuralgia. In: Ellenbogen
lates signals before they reach the brain. Large RG et al., editors. Principles of Neurological Surgery. 3rd
diameter fibers have inhibitory effects to “shut ed. p. 729-736 (Chapter 48).
the gate” whereas small diameter fibers carrying
noxious stimuli open the gate to pain transmis- 9. a—Intrathecal baclofen has a potent analge-
sion. In a simplistic view of this model, rubbing sic effect
of the injured area promotes proprioceptive (i.e.
large diameter) fiber input and reduces pain The continuous administration of analgesics via
perception. In the late 1990s, Melzack proposed the intrathecal route results in higher subarach-
the neuromatrix theory, adding higher cortical noid drug concentrations, lower absolute drug
functions as key elements of pain transmission doses, and avoidance of side effects associated
and interpretation. Today, the experience of pain with systemic dosing (especially at high doses).
is described as distributed in three dimensions: Additionally, concerns about opiate diversion
cognitive, affective, and sensory. Frontal and and analgesic compliance are reduced. Indica-
limbic areas are believed to subserve the tions for intrathecal drug delivery for pain
cognitive-evaluative component of pain. Limbic control:
cortex, cingulum, hypothalamus, thalamus, and

Neurosurgery Books Full


www.ketabpezeshki.com 66485438-66485457
33 PAIN SURGERY 453

1. An established pain diagnosis has been 10. e—Is not indicated in patients with multiple
made classifying the symptoms as neuro- sclerosis related chronic pain syndromes due
pathic, nociceptive, or mixed. to high seizure risk
2. Pain is chronic or both chronic and pro-
gressive in nature owing to either a malig- Chronic stimulation of the precentral gyrus
nant or nonmalignant cause. below the threshold to produce a motor response
3. Pain should be present throughout nearly is able to alleviate certain types of deafferentation
the entire day. pain and MCS has shown efficacy for a number of
4. Patients have failed to achieve analgesia deafferentation pain syndromes (e.g. trigeminal,
with conservative nonpharmacologic central post-stroke pain, anesthesia dolorosa,
modalities. post-herpetic neuralgia, multiple sclerosis, phan-
5. Patient is refractory or intolerant to orally tom limb pain, and spinal cord injury). The mech-
administered analgesics. anism of action has been attributed to modulation
6. Corrective treatment addressing the pain of deafferentation-induced pathologic hyperactiv-
generator is not warranted. ity in thalamic relay nuclei and/or increased sensi-
7. Surgical contraindications to implanting tivity of higher order pain pathway neurons.
prosthetic hardware and accessing the Intraoperatively, the central sulcus is localized using
intrathecal space are absent (e.g. bacter- SSEPs and a contact paddle electrode is placed in
emia, anticoagulation). the epidural space overlying the facial or upper
Although several agents are commonly used for extremity region of the motor cortex. The elec-
chronic intrathecal delivery, only three medica- trode is then used for motor evoked potentials with
tions currently have FDA approval for long-term electromyography to confirm motor activity. Iced
intrathecal use: baclofen, morphine, and zicono- saline is prepared for irrigation if a seizure is
tide. Ziconotide is exclusively intrathecal form induced. The minimum thresholds for motor activ-
of ω-conotoxin MVIIa that blocks a N-type ity and any seizure activity are noted. After confir-
calcium channel on small myelinated and unmy- mation, a paddle electrode is sutured to the dura
elinated nociceptive afferents that are primarily over the precentral gyrus over the motor area that
localized in the superficial Rexed laminae (I and corresponds to the patient's pain distribution. Sub-
II). Intrathecal baclofen, a GABA-B agonist, dural placement is associated with greater energy
is primarily used to treat spasticity but may help efficiency, but also an increased rate of complica-
with pain associated with spasticity and dystonias. tions, including subdural hematomas and a higher
Intrathecal delivery of medications can result reported rate of seizures. However, opening of
in several potential adverse events including the dura may be necessary anyway for coverage
sedation, cognitive impairment, nausea, vomit- of lower extremity pain, which requires placement
ing, pruritus, urinary retention, constipation, of an electrode along the medial part of the hemi-
hormonal dysfunction, and edema. The develop- sphere. MCS has an overall complication rate of
ment of an inflammatory mass (granuloma) is about 5%: wound breakdown or infection (5.1%),
particularly associated with morphine and hardware breakage from trauma, and seizures
increasingly hydromorphone, with high drug (12%). Stimulation of the motor cortex is known
concentrations combined with low flow rate to be associated with the potential to induce sei-
increase the risk of granuloma development. Pre- zures, and most seizures observed during MCS
sentation is with loss on pain control or new- occur during programming sessions.
onset pain complaints or progressive myelopathy.
CT myelography or MRI with gadolinium con- FURTHER READING
trast of the catheter tip region is necessary to con- Ostergard T, et al. Motor cortex stimulation for chronic pain.
firm the diagnosis. In asymptomatic and Neurosurg Clin N Am 2014;25:693-698.
nonprogressive patients, weaning of intrathecal
medications and initiation of saline infusion can
produce spontaneous disintegration of the mass. ANSWERS 11–15
In patients with progressive or severe neurologic
compromise, urgent surgical decompression and Additional answers 11–15 available on
excision are recommended. ExpertConsult.com

FURTHER READING
Bolash R, et al. Intrathecal pain pumps: indications, patient
selection, techniques, and outcomes. Neurosurg Clin N Am
2014;25:735-742.

Neurosurgery Books Full


www.ketabpezeshki.com 66485438-66485457
454 PART V FUNCTIONAL NEUROSURGERY

EMI ANSWER International Association for the Study of Pain


Taxonomy:
16. 1—e, Dysesthesia; 2—h, Hyperpathia; 3—d, Pain is an unpleasant sensory and emotional
Causalgia; 4—c, Anesthesia dolorosa; 5—a, experience associated with actual or potential tis-
Allodynia sue damage, or described in terms of such damage.

Pain: Definitions
Term Definition

Hypoesthesia Decreased sensitivity to stimulation, excluding the special senses

Paresthesiae An abnormal sensation, whether spontaneous or evoked (not unpleasant)

Dysesthesia An unpleasant abnormal sensation, whether spontaneous or evoked

Hyperesthesia Increased sensitivity to stimulation, excluding the special senses, includes both allodynia and
hyperalgesia, but the more specific terms should be used wherever they are applicable

Allodynia Pain due to a stimulus that does not normally provoke pain

Hyperalgesia Increased pain from a stimulus that normally provokes pain

Hypoalgesia Diminished pain in response to a normally painful stimulus

Hyperpathia A painful syndrome characterized by an abnormally painful reaction to a stimulus, especially


a repetitive stimulus, as well as an increased threshold

Neuropathic Pain caused by a lesion or disease of the somatosensory nervous system. May be central or
pain peripheral

Nociceptive Pain that arises from actual or threatened damage to non-neural tissue and is due to the
pain activation of nociceptors. Describes pain occurring with a normally functioning
somatosensory nervous system to contrast with the abnormal function seen in neuropathic
pain

Analgesia Absence of pain in response to stimulation which would normally be painful

Anesthesia Pain in an area or region which is anesthetic


dolorosa

Causalgia A syndrome of sustained burning pain, allodynia, and hyperpathia after a traumatic nerve
lesion, often combined with vasomotor and sudomotor dysfunction and later trophic changes

FURTHER READING
http://www.iasp-pain.org/Education/

17. 1—d, Glossopharyngeal neuralgia; 2—c, Geniculate neuralgia; 3—b, Cluster headache

Craniofacial Pain Syndromes: Clinical Features and Management


Syndrome Comments Surgical Options

Cluster headache Pain attacks of severe or very severe unilateral Occipital nerve stimulation
orbital, supraorbital, and/or temporal pain Sphenopalatine ganglion stimulation
lasting 15-180 min if untreated. May be Radiofrequency ablation of
accompanied by ipsilateral conjunctival sphenopalatine ganglion
injection and/or lacrimation, nasal congestion, Gamma knife SRS to trigeminal root
and/or rhinorrhea, eyelid edema, forehead Hypothalamic DBS
and facial sweating, miosis and/or ptosis and a Transection of nervus intermedius
sense of restlessness or agitation. Attacks
have a frequency from 1 every other day to
8 per day. Episodic form is six times more
common than the chronic form (attacks
occurring for >1 year without remission or
with remissions lasting <1 month)

Continued
Neurosurgery Books Full
www.ketabpezeshki.com 66485438-66485457
33 PAIN SURGERY 455

Syndrome Comments Surgical Options


Migraine Usually a throbbing headache with or without Botulinum toxin A prophylactic
sensory aura injections
Trigger site deactivation surgery
Occipital nerve stimulation

Occipital neuralgia Paroxysmal stabbing pain, with or without Peripheral neurectomy


persistent aching between paroxysms, in the C2 dorsal root ganglionectomy
distribution of the greater, lesser, and/or third Microvascular decompression (C2 root)
occipital nerves Percutaneous neurolysis C2 root
Occipital nerve stimulation
RF lesioning of greater/lesser occipital
nerves
Pulsed RF treatment C2 dorsal root
ganglion

Trigeminal neuralgia Brief strong, sharp, unilateral shooting pain in Microvascular decompression
one or more branches of trigeminal nerve. Gamma knife surgery to trigeminal
Combined V2 and V3 symptoms commonest, DREZ
V1 symptoms alone rarest presentation Peripheral ablation of gasserian
ganglion

Glossopharyngeal Paroxysmal attacks of facial pain lasting from MVD


neuralgia a fraction of a second to 2 min. Characterized Rhizotomy of IX ( upper rootlets of X)
by unilateral, sharp, stabbing, and severe pain Gamma knife surgery
in the distribution within the posterior part of Motor cortex stimulation
the tongue, tonsillar fossa, pharynx, or Trigeminal tractotomy  nucleotomy
beneath the angle of the lower jaw and/or in
the ear. Triggers include swallowing,
chewing, talking, coughing, and yawning

Geniculate neuralgia Brief paroxysms of pain felt deep in the Transection of nervus intermedius
auditory canal. A trigger area is present in the (with or without geniculate ganglion
posterior wall of the auditory canal removal)
Microvascular decompression

Persistent idiopathic Persistent facial pain that does not have the CT-guided percutaneous trigeminal
facial pain characteristics of cranial neuralgias or cannot tractotomy-nucleotomy
be attributed to another disorder. Usually a Nucleus caudalis DREZ lesioning
throbbing pain situated deep in the eye and pulsed radiofrequency to the
malar region, often radiating to the ear, neck, sphenopalatine ganglion
and shoulders

Anesthesia dolorosa Uncommon complication of surgical Motor cortex stimulation


treatments for neuralgias: excruciating pain Deep brain stimulation
perceived in an insensate region of the face Nucleus caudalis DREZ lesioning

Temporomandibular Recurrent pain in one or more regions of the Botulinum toxin


joint disorder head and/or face precipitated by jaw Arthrocentesis/arthroscopy
movements and/or chewing of hard or tough TMJ disc surgery or joint replacement
food. Other findings are a reduced range of or Denervation of TMJ
irregular jaw opening, noise from one or both
TMJs during jaw movements, tenderness of
the joint capsule

FURTHER READING
Gutierrez, et al. Introduction to neuropathic pain syndromes. Neurosurg Clin N Am 2014;2:639-662.

Neurosurgery Books Full


www.ketabpezeshki.com 66485438-66485457
456 PART V FUNCTIONAL NEUROSURGERY

18. 1—e, Failed back surgery syndrome; 2—g, Post-herpetic neuralgia; 3—c, Complex regional pain
syndrome

Neuropathic Pain Syndromes: Features and Surgical Management


Syndrome Comments Surgical Options

Failed back Persistent or recurring low back pain, with or Lumbar epidural steroid injection
surgery without sciatica, after one or more spine Percutaneous epidural adhesiolysis
syndrome surgeries. Incidence of FBSS has increased with Percutaneous ozone injection (intradiscal/
increasing rates of spine surgery: 10-40% patients paravertebral/intraforaminal)
may develop FBSS after lumbar spinal surgery. Facet joint rhizotomy
Success rate of lumbar spinal surgery falls with Intrathecal drug delivery
each successive surgery on the same patient Peripheral nerve field stimulation
Spinal cord stimulation
Deep brain stimulation
Revision surgery

Chronic low Chronic, recurrent, or long-lasting pain localized Lumbar fusion


back pain below the costal margin and above the inferior Total disc replacement
gluteal folds lasting for at least 6 months. Spinal endoscopic adhesiolysis
Prevalence: 8.1-10.2% of the US population Caudal epidural steroid injection
Spinal cord stimulation

Post-spinal Neuropathic pain and dysesthesia in areas with DREZ lesion


cord injury sensory deficit and can be spontaneous or Spinal cord stimulation
stimulus evoked. Quality can be burning, Cordotomy
smarting, shooting, aching, pricking, and tingling Intrathecal drug delivery
 paresthesia. Approximately 65-85% of people Motor cortex stimulation
have pain (nociceptive and neuropathic) after
spinal cord injury and it is severe in one third

Post-herpetic Pain after acute rash has healed accompanied by Botulinum toxin A
neuralgia pain, allodynia, paresthesia, or dysesthesia. The Sympathetic nerve block
pain usually affects a single dermatome Spinal cord stimulation
Gamma knife SRS (trigeminal/thalamic)
Peripheral nerve stimulation
DREZ lesioning

Post- Pain that recurs or persists along a thoracotomy Peripheral nerve field stimulation
thoracotomy incision at least 2 months after the surgical Peripheral nerve stimulation
pain procedure. The gentlest of stimulation can trigger Intercostal nerve block
syndrome intense burning and stabbing pain, with Intercostal nerve cryoablation
dysesthesia DREZ lesion
5% of patients experience severe and disabling Spinal cord stimulation
pain Pulsed RF intercostal nerve or DRG

Brachial The typical root avulsion pain is a constant dull, DREZ lesion
plexus crushing, or burning pain with superimposed
avulsion lightening jolts of severe sharp pain shooting
down the arm. Most occur as a consequence of
motorcycle accidents. Prevalence of neuropathic
pain ranges from 34% to 95% of cases, with a
quarter of patients experience intractable long-
term pain of the upper limb

Phantom limb The pain presents as short-lasting and rarely Neurostimulation (e.g. dorsal root
pain occurring painful shocks or as constant, ganglion)
excruciatingly painful experience in the missing Sympathectomy
body part. Prevalence of phantom pain among DREZ lesion
amputated patients: 50-90% Cordotomy
Rhizotomy

Complex CRPS type I (Reflex Sympathetic Dystrophy) has Epidural clonidine


regional pain no definable nerve lesion, while a peripheral Regional anesthetic block
syndrome nerve lesion can be demonstrated in CRPS type II Local anesthetic block  Botox
(causalgia). Present with severe pain that is Intrathecal baclofen
disproportionate to the inciting event, most Sympathectomy
commonly affecting the hand or foot but that can Repair of injured nerve (CRPS II)

Continued

Neurosurgery Books Full


www.ketabpezeshki.com 66485438-66485457
33 PAIN SURGERY 457

Syndrome Comments Surgical Options


spread to other body regions. The affected body Amputation
parts may display sensory disturbances, Spinal cord stimulation
temperature changes, abnormal patterns of TENS
sweating, edema, reduced joint range of motion, Repetitive TMS
movement abnormalities such as weakness, PNS
tremor, or dystonia, trophic changes such as skin Dorsal root ganglion stimulation
atrophy or altered hair and nail growth, and DBS (for pain and dystonia)
localized osteoporotic changes. Alterations in Motor cortex stimulation
body perception or schema may be present

Post-traumatic Symptoms such as hypesthesia, paresthesia, Brachial plexus grafting


pain allodynia, hyperpathia, and hyperalgesia may be Epidural steroid injection
seen within days of nerve damage or months Nerve resection/end neuroma relocation/
later neurolysis
Post-traumatic neuropathic pain (from accidental Spinal cord stimulation
or surgical injury) is one of the most common Motor cortex stimulation
causes of chronic pain Deep brain stimulation
Peripheral nerve stimulation
DREZ lesion  selective rhizotomy

Diabetic Presents with burning-type pain, paresthesia, and Surgical decompression


peripheral numbness of mild to moderate severity. These Spinal cord stimulation
neuropathic symptoms may be accompanied by loss of
pain proprioception, temperature sensitivity, and
eventually pain sensation. Prevalence among
adults with diabetes in the US population:
27-50%. Of these, 11% have neuropathic pain

FURTHER READING
Gutierrez, et al. Introduction to neuropathic pain syndromes. Neurosurg Clin N Am 2014;25:639-662.

19. 1—f, Midline myelotomy; 2—d, DREZ lesion; 3—a, Anterolateral cordotomy; 4—g, Mesence-
phalic tractotomy; 5—b, Cingulotomy

Surgical Approaches for the Management of Neuropathic Pain

DREZ lesions DREZ lesioning can be thought of as a treatment for pain that is believed to be confined to a
unilateral limb. The Lissauer tract is a key pathway that conducts nociceptive information at
least two segments above and below the DREZ (hence pain could be arising up to two
segments above or below an involved dermatomal segment). If dorsal root fibers are avulsed,
as commonly seen in pain associated with brachial or lumbar-sacral plexus trauma anatomy
is more difficult and complications more likely. Lesions that inadvertently are placed too far
laterally and injure the corticospinal tract, resulting in permanent ipsilateral weakness below
the lesion. Alternatively, if lesions deviate too medially from the DREZ, ipsilateral loss of
proprioception and light touch may occur due to dorsal column injury

Anterolateral The spinothalamic tract lies just anterior to the dentate ligament and near the anterolateral
cordotomy surface of the cord, while the corticospinal tract is just posterior. The anterior spinal artery
is a significant vascular structure whose midline position must be appreciated and avoided
during open transection of the spinothalamic tract. Lesioning the spinothalamic tract
removes pain below the level of the lesion, but levels of adequate pain control are several
levels below the lesion. Good candidates are patients who experience severe pain that
originates from cancer involving the pelvis, leg, hip, and lower trunk. Those with
nonmalignant pain syndromes are not ideal due to recurrence rate of pain within a few
years or the emergence of new central neuropathic pain (burning dysesthesias below the
level of the lesion). Complications are urinary retention or incontinence, permanent
dysesthesias, transient hemiparesis, and respiratory complications (cervical cordotomies).
Mirror pain is a unique complication of open thoracic cordotomies in patients with cancer
in which a similar pain develops contralaterally within weeks to months after the
cordotomy. Other complications that are shared with open dural procedures and
laminectomies include possible mechanical spinal instability, CSF leak, and meningitis

Continued on following page

Neurosurgery Books Full


www.ketabpezeshki.com 66485438-66485457
458 PART V FUNCTIONAL NEUROSURGERY

Surgical Approaches for the Management of Neuropathic Pain (Continued)


Midline In patients with bilateral lower extremity pain and, in particular, with involvement of the
myelotomy pelvis and lower abdominal organs, a single lesion disconnecting the anterior commissure
(commissurotomy) through a lower thoracic approach has been quite effective in relieving severe refractory
pain. The typical patient is one with pelvic cancer or sarcoma that invades bilateral
structures in the pelvis and lower extremities. Loss of bowel and bladder function, and
proprioception with relative preservation of motor function (leg weakness in one third)
may be acceptable in this group who may otherwise be bedridden with severe pain. The
risks of respiratory and sympathetic damage (fibers located near the central gray matter) in
creating a mid to upper cervical midline myelotomy are likely the reasons why this
technique has not been used much for upper trunk and arm pain

Mesencephalic Mesencephalic tractotomy has been successfully used for the treatment of denervation
tractotomy pain, such as central dysesthesia, in the upper extremity, head, or neck. Potential
candidates are those who fail medical management, neuromodulation, intrathecal
infusions, and thoracic or cervical cordotomy, or those patients whose pain is from
structures more superior than what cervical cordotomy can treat. In particular, neuropathic
pain from head or neck malignancy could be a potential indication. Patients with chronic
nonmalignant pain do not respond well to this technique. It involves a lesion of
extralemniscal pathways lateral to the spinothalamic tract and medial lemniscus can result
in relief of intractable pain without loss of sensation or dysesthesia

Cingulotomy Because the motivational-affective component of pain contributes to the fear, suffering,
and anxiety of pain, cingulectomy and cingulotomy were proposed to treat this component
of chronic pain. The anterior midcingulate cortex is implicated as an area of overlap
between negative affect, pain, and cognitive control based on functional MR imaging and
DTI studies. The first open resection of 4 cm of the anterior cingulate gyrus for intractable
pain, called cingulectomy. Unilateral and bilateral cingulotomies that affect a large volume
of the cingulate fasciculus were subsequently developed, and more recently stereotactic
cingulotomy. Its main success has been in malignant pain of the head and neck with
associated sensations of respiratory distress, and for the discomfort of chronic dyspnea in
a patient with malignant mesothelioma. Patients with significant preexisting brain disease,
sociopathic personalities, or advanced age are generally not thought good candidates for
cingulotomy

FURTHER READING
Konrad P. Dorsal root entry zone lesion, midline myelotomy and anterolateral cordotomy. Neurosurg Clin N Am
2014;25:699-722.

Neurosurgery Books Full


www.ketabpezeshki.com 66485438-66485457
CHAPTER 34

ADULT AND PEDIATRIC


EPILEPSY SURGERY
SINGLE BEST ANSWER (SBA) QUESTIONS
1. Which one of the following is the most epi- a. 10-20%
leptogenic primary brain tumor? b. 20-30%
a. DNET/ganglioglioma c. 30-40%
b. Glioblastoma d. 40-50%
c. Low grade glioma e. 50-60%
d. Meningioma
e. Metastasis 6. Which one of the following is most appropri-
ate for predicting postoperative language and
2. Which one of the following types of epilepsy memory impairment in epilepsy surgery
is LEAST likely to benefit from epilepsy sur- candidates?
gery referral? a. BOLD functional MRI
a. Hemimegalencephaly b. EEG
b. Rasmussen's syndrome c. Hippocampal depth electrodes
c. Rolandic epilepsy d. Ictal SPECT
d. Sturge-Weber syndrome e. Video telemetry
e. West syndrome with focal malformation f. Wada test
of cortical development
7. Neuropsychological testing preoperatively is
3. A 37-year-old male with medically refractory
unable to:
epilepsy undergoes workup for surgery. MRI
a. Aid lateralization of the epileptogenic
shows frontal cortical dysgenesis, and a concor-
zone
dant EEG shows that the seizure focus involves
b. Predict postoperative deficits
the posteriorly located motor cortex. Which
c. Assess patients mental reserve capacity
one of the following operative approaches
d. Define the epileptogenic zone preopera-
could you consider when seizure activity
tively
extends beyond the area of resection and into
e. Assess for depression and anxiety
eloquent cortex?
a. Corpus callosotomy
8. Prolonged delay between ictal behavior onset
b. Deep brain stimulation
and first appearance of ictal EEG discharge
c. Hemispherectomy
d. Multiple subpial transection during video telemetry (continuous video-
EEG recording) is most likely due to:
e. Vagal nerve stimulation
a. Malfunction of scalp electrodes
4. Which one of the following is the best pre- b. Drowsy patient
dictor for seizure-free outcome after epilepsy c. Remote/distant site of seizure onset
surgery (assuming total lesionectomy)? d. Hyperventilation
a. Preoperative EEG and MRI concordance e. Withdrawal of antiepileptic medication
b. Extratemporal seizure focus
c. History of febrile seizures 9. A 28-year-old male presents with intractable
d. Mesial temporal sclerosis epilepsy. Sleep EEG is unable to localize the
e. Low grade temporal glioma focus. Coregistered MRI and PET studies
cannot identify an epileptic focus. Subtracted
5. The frequency of abnormal interictal EEG ictal and interictal SPECT was performed
findings in the investigation of seizures is which showed a hypermetabolic focus in
which one of the following? the left posterolateral temporal lobe. Which

459
Neurosurgery Books Full
www.ketabpezeshki.com 66485438-66485457
460 PART V FUNCTIONAL NEUROSURGERY

one of the following statements are most


appropriate in this patient?
a. The next appropriate step would be
to place subdural grids/strips
b. The next appropriate step would be to per-
form a left insular corticectomy
c. The next appropriate step would be to
perform multiple subpial resection A B

d. The next appropriate step would be to


place depth electrodes (stereotactic EEG)
e. The next appropriate step would be to a. Depth electrodes
perform a left temporal lobectomy b. Subdural grid
c. Subdural strip
10. Which one of the following is the common- d. AVM resection
est neurological complication after VNS? e. Temporal lobectomy
a. Arrhythmia
b. Bradycardia 14. A 49-year-old woman is referred with a 1-year
c. Dysphonia history of medically refractory epilepsy. Elec-
d. Facial numbness trographically, both interictally and ictally, this
e. Hypotension patient's seizures were consistent with left
mesial temporal onset. However, no definite
11. A 10-year-old child undergoes hemispherec- abnormalities were observed on MRI. Assum-
tomy. At 2 years post-op, he is only having ing left hemisphere dominance for language,
nocturnal seizures now. Which one of the neuropsychological data were suggestive of left
following Engel Epilepsy surgery outcome temporal lobe dysfunction. Some intact verbal
classes does he fall into? memory scores raised a question that the left
a. I mesial temporal lobe structures were intact.
b. II Wada test: Language lateralized to the left
c. III hemisphere. She underwent invasive electro-
d. IV corticographic (ECoG) monitoring with a left
e. V mesial temporal (LMT) strip, basal temporal
strips (anterior, LAT and posterior, LPT),
12. Which one of the following is an unlikely pri- and an 8  8 electrode grid with the upper five
mary indication for using invasive EEG mon- rows located frontoparietally (contacts 1-40)
itoring as part of evaluation for epilepsy and the lower three rows over the lateral tem-
surgery? poral neocortex (contacts 41-64). All seizures
a. Seizure onsets are lateralized but not were electrographically stereotyped. Electro-
localized graphic onset preceded clinical onset and con-
b. Seizure onsets are localized but not clearly sisted of fast activity of 80-100 Hz isolated to
lateralized (i.e. bilateral) the contact at LG41. Six to nine seconds later
c. Seizure onset near eloquent cortex how- this abruptly transitioned to a 2.5-Hz spike and
ever unknown focus wave pattern involving the mesial and basal
d. Dual pathology in opposite hemispheres temporal contacts. Which one of the following
e. Multiple cortical lesions (e.g. tuberous statements is most accurate?
sclerosis)
f. Predicting postoperative memory deficit

13. A 29-year-old man with a history of 1


9
febrile seizures as a child has developed 17
25
medication-refractory complex partial 33
41
49 LAT LPT
seizures within the past 2 years. EEG sug- 57
gests a left temporal focus, with evidence of LMT

concordant PET imaging. An MRI as shown


below, reveals the abnormality. Which one of
the following is the next appropriate step in
management?

Neurosurgery Books Full


www.ketabpezeshki.com 66485438-66485457
34 ADULT AND PEDIATRIC EPILEPSY SURGERY 461

a. Seizure onset is from left mesial temporal d. Fimbria


structures e. Subiculum
b. Seizures onset is from the left orbitofron-
tal region 20. Imaging in epilepsy:
c. Seizure onset is from the left anterior a. Arterial spin-labeling MRI
temporal pole b. Diffusion-tensor MRI
d. Seizure onset is from the left anterosuper- c. Diffusion-weighted MRI
ior temporal gyrus d. FLAIR MRI
e. Hippocampal depth electrodes should e. Functional MRI
have been placed f. Gradient echo MRI
g. Ictal and interictal subtraction SPECT
(SISCOM)
QUESTIONS 15–18 h. MR spectroscopy
i. PET imaging
Additional questions 15–18 available on
ExpertConsult.com
j. Susceptibility-weighted imaging MRI
k. Volumetric T1 MRI

For each of the following descriptions, select the


most appropriate answers from the list above.
EXTENDED MATCHING ITEM (EMI) Each answer may be used once, more than once
or not at all.
QUESTIONS 1. Increasingly being used to screen language
lateralization
19. For each of the following descriptions, select 2. Sensitive for detection of hemosiderin
the most appropriate answers from the image deposition (e.g. cavernoma)
below. Each answer may be used once, more 3. Best for identifying mesial temporal sclerosis
than once or not at all.
A 21. Surgical options in epilepsy:
B a. Anatomical hemispherectomy
b. Corpus callosotomy
C c. Deep brain stimulation
D d. Functional hemispherectomy
e. Multiple subpial transection
E
f. Repetitive cortical stimulation
g. Selective amygdalohippocampectomy
F h. Tailored temporal lobectomy
i. Topectomy
j. Vegal nerve stimulation
k. Vertical parasagittal hemispherotomy

For each of the following descriptions, select the


G most appropriate answers from the list above.
1. Considered to be an established palliative
surgical option for generalized seizures.
2. Intraoperative option when part of the
H epileptogenic zone overlaps an area of
eloquent cortex
3. Classically associated with late postoperative
I complication of superficial cerebral siderosis
4. Temporal epilepsy with a lesion in lateral
J
temporal lobe in the dominant hemisphere
5. Resection of cortex sparing underlying white
K
matter often used in extratemporal lesions
L

a. Choroid plexus
b. Dentate gyrus
c. Entorhinal cortex

Neurosurgery Books Full


www.ketabpezeshki.com 66485438-66485457
462 PART V FUNCTIONAL NEUROSURGERY

22. Epilepsy surgery workup: 2. A 17-year-old boy reports involuntary jerk-


a. Sleep-deprived EEG ing movements in his arms when he
b. Video EEG awakens. This has occurred during the
c. MRI day after a nap as well as in the morning
d. PET after a full night's sleep.
e. Neuropsychological testing 3. A 21-year-old man with septicemia induced
f. Psychiatric evaluation DIC develops continuous rhythmic jerking
g. Wada study of the right thumb and wrist lasting several
h. Functional MRI hours. CT head shows a small hemorrhage
i. Ictal and interictal subtraction SPECT in the left posterior frontal region. He is oth-
(SISCOM) erwise conscious and obeying commands.
j. Magnetoencephalography 4. A 21-year-old presents with unpleasant
k. Visual field testing olfactory hallucinations.

For each of the following descriptions, select the 24. Medical treatment of epilepsy:
most appropriate answers from the list above. a. ACTH
Each answer may be used once, more than once b. Clonazepam
or not at all. c. Ethosuximide
1. Aids in localizing language and motor func- d. Felbamate
tion but not sufficient for predicting post- e. Levetiracetam
operative verbal memory impairment. f. Lorazepam
2. Important before resuming driving if sei- g. Magnesium sulfate
zure free after temporal lobectomy. h. Phenobarbital
3. Suspicion of nonepileptic attack disorder. i. Primidone
4. Can help assess risk of postoperative amne- j. Phenytoin
sia and likelihood of seizure-free outcome k. Sodium valproate
5. Epileptic focus often appears hypometa-
bolic For each of the following descriptions, select the
most appropriate answers from the list above.
23. Seizure semiology: Each answer may be used once, more than once
a. Complex partial or not at all.
b. Epilepsia partialis continua 1. A 7-month-old boy develops generalized
c. Generalized tonic-clonic limb extension and neck flexion spasms
d. Generalized absence that occur more than 20 times daily and
e. Jacksonian March are associated with altered consciousness.
f. Myoclonic EEG reveals diffuse, high-voltage,
g. Nonconvulsive status epilepticus polyspike-and-slow-wave discharges bet-
h. Pseudoseizures ween spasms and suppression of these bursts
i. Rolandic epilepsy during the spasms. A sibling died with a
j. Simple partial sensory brainstem glioma, and the father has several
k. Status epilepticus large areas of hypopigmented skin in the
l. Uncinate seizures shape of ash leaves. The infant had obvious
psychomotor retardation even before the
For each of the following descriptions, select the appearance of the spasms.
most appropriate answers from the list above. 2. A 5-year-old girl has frequent staring spells
Each answer may be used once, more than once and does not respond when her mother
or not at all. calls her name during these episodes. She
1. A 37-year-old man develops involuntary never falls down or bites her tongue, but
twitching movements in his left thumb, she does have occasional lip smacking dur-
which spread to his entire left hand and ing episodes. EEG reveals a 3 Hz spike-
forearm and the left side of his face then and-wave pattern that occurs for less than
blacked out. Witnesses report that he fell 10 s at a time but several times an hour.
down and the entire left side of his body The child has normal motor and cognitive
appeared to be twitching, followed by unre- development.
sponsive episode for a few minutes and con- 3. A 35-year-old pregnant woman at term
fusion for 15 min. During the episode, he is admitted to the hospital for delivery.
bit the side of his tongue and was inconti- She has headaches and visual blurring. Her
nent of urine. blood pressure is 180/100. On examination,

Neurosurgery Books Full


www.ketabpezeshki.com 66485438-66485457
34 ADULT AND PEDIATRIC EPILEPSY SURGERY 463

she is edematous. Reflexes are increased. Pro- ing home. An ambulatory EEG demon-
tein is found in the urine. She then develops a strates evolving spike activity originating
generalized tonic-clonic convulsion. in the left temporal lobe during one of
4. A 19-year-old woman describes recurrent the episodes. The EEG pattern does not
memory problems. Her fiancé reports that generalize. CT and MRI scanning of the
she seems to be inattentive for minutes at a brain reveal no structural abnormalities.
time several times a week. She never Conversations with the woman's parents
injures herself during these episodes, but reveal that she had febrile seizures when
she cannot recall what happened, and, on she was 3 years old, which abated with
one occasion, she became lost while walk- antipyretic treatment alone.

SBA ANSWERS
1. a—DNET/ganglioglioma are characterized by multiple daily seizures,
intractability to standard AEDs, developmental
The most epileptogenic tumors are DNET/ arrest or decline, and presumed/known epilepto-
gangliogliomas, but these are rare compared to genic pathology. Examples include Sturge-
meningiomas which are thus the commonest pri- Weber syndrome, seizures due to malformations
mary intracranial tumor (34%). However, brain of cortical development, infantile spasms (West
metastases account for 50% of all intracranial syndrome) due to malformations of cortical
mass lesions—with lung and breast metastasis development and Rasmussen's syndrome. Rolan-
making up the majority due to their prevalence, dic epilepsy is benign and spontaneously remits in
but melanoma having a higher propensity to adolescence hence surgery is not indicated.
metastasize to the brain (but is less common).
3. d—Multiple subpial transection

The technique of multiple subpial transections


of cortex aimed to address the management of
% of Primary
Brain Tumors Approximate that portion of the epileptogenic zone that was
(Adults and Seizure electrocorticographically demonstrated to be
Tumor Children) Frequency functionally eloquent cortex. Instead of topect-
omy or cortical resection, the cortex is instead
DNET/ 1% 80-100%
ganglioglioma disrupted by parallel subpial cuts transecting
the gray matter every 3-5 mm. The intention
Low grade 9% 60-85% is to preserve centripetal axonal outflow while
glioma
isolating silos of epileptogenic neurons by
Brain N/A 24% (melanoma disrupting lateral dendritic communication
metastasis 67%)
across gyri, thereby preventing Jacksonian pat-
Glioblastoma 17% 25-40% terns of spreading cortical propagation of sei-
Meningioma 34% 25-40% zures. The benefit and indications of this
technique remain controversial. A newer
approach for treatment of focal epilepsy arising
from eloquent cortex is chronic subthreshold
FURTHER READING subdural cortical stimulation.
Uptodate. Meningioma: clinical presentation and diagnosis.
Topic 5220 Version 10.0.
FURTHER READING
Uptodate. Seizures in patients with primary and metastatic Maciunas RJ. Surgical treatment of medically intractable epi-
brain tumours. Topic 5181 Version 25.0. lepsy. In: Werz MA, editors. Epilepsy Syndromes. Elsevier,
Saunders; 2010.
American Brain Tumour Association. Brain tumour statistics.

2. c—Rolandic epilepsy 4. e—Low grade temporal glioma

The catastrophic epilepsies of infancy and child- In patients with intractable epilepsy it is impor-
hood (named due to the deleterious effect on tant to determine if the seizures arise from the
development) are recognizable early and referral temporal lobes because temporal lobectomy is
for surgical workup should not be deferred. They known to achieve seizure freedom in the majority

Neurosurgery Books Full


www.ketabpezeshki.com 66485438-66485457
464 PART V FUNCTIONAL NEUROSURGERY

of this group. A recent randomized trial by Wiebe 6. f—Wada test


and associates showed the superiority of temporal
lobectomy to ongoing medication therapy, with Mesial temporal structures, especially the hippo-
about 60% becoming completely seizure free campal formation, are critical for the formation of
compared with 8% in those continuing on med- new memories. Both sides are involved in memory
ical management with antiepileptic drugs. In this consolidation, although the left may be more signif-
study, 70% of both groups had mesial temporal icant in verbal memories and the right in visuospatial
lobe epilepsy, compared to 10-15% who had a (nonverbal) memories. Thus, removal of the mesial
different temporal lesion (e.g. low grade glioma, temporal structures in a temporal lobectomy may
cortical dysplasia, vascular malformation), and lead to memory decline. Many patients with tempo-
10-15% who had a normal MRI. The only death ral lobe epilepsy have bilateral disease, hence a uni-
in the study occurred in the medically managed lateral temporal lobectomy in a patient with poor
group and was secondary to a seizure. However, function on the contralateral side could have cata-
in studies looking at surgery for low grade tempo- strophic results, such as an amnestic syndrome.
ral lobe tumours specifically, gross total tumour The main means of assessing language dominance
resection results in seizure freedom in closer to in epilepsy patients includes clinical presentation,
80% of patients. The most common complication neuropsychological assessment, functional MRI
of temporal lobectomy is a visual field defect and Wada (sodium barbital infusion) test. While
caused by interruption of fibers from the optic functional MRI and/or magnetoencephalography
tracts passing over the temporal horn of the lat- may aid in language lateralization, Wada is per-
eral ventricles. Superior quadrantanopia is more formed not only to demonstrate language domi-
common than hemianopsia. Some deficits may nance but also to assess the potential for verbal
improve if the injury does not completely damage memory reduction postoperatively by evaluating
the nerves. Language deficits, particularly dysno- the behavioral function of each brain hemisphere
mia, occur less frequently. Hemiparesis is uncom- independent of the contralateral hemisphere. Injec-
mon (<2%), because the surgery is performed at a tion ofsodiumamytalintoa carotidartery transiently
distance from the motor fibers of the corticosp- shuts down the hemisphere supplied, so that the
inal tract. Other neurological problems that can memory and verbal function of the contralateral
occur include diplopia caused by extraocular hemisphere can be assessed; usually the side of resec-
nerve deficits and facial paresis. tion is injected first to determine the functioning of
the nonresection side. Patients at high risk of postop-
erative memory impairment are those without MRI
FURTHER READING
Wiebe S, Blume WT, Girvin JP, Eliasziw M. A randomized, lesion, average-low average memory on preopera-
controlled trial of surgery for temporal-lobe epilepsy. N Engl tive neuropsychology, good memory when side con-
J Med 2001;345:311-318. tralateral to planned resection is injected (i.e. good
memory on side of resection) and poor memory
Englot DJ, Han SJ, Berger MS, Barbaro NM, Chang EF. Extent when side of planned resection is injected (i.e. poor
of surgical resection predicts seizure freedom in low-grade tem-
memory on side to be preserved postoperatively).
poral lobe brain tumors. Neurosurgery. 2012 Apr;70(4):921-8;
discussion 928. doi: 10.1227/NEU.0b013e31823c3a30. Review.
Equally, lower risk candidates are those with a unilat-
PubMed PMID: 21997540. eral mesial temporal sclerosis (MTS), poor material-
specific memory on neuropsychological testing (i.e.
5. e—50-60%, with further increase in yield by poor verbal memory if left MTS, or visual memory if
repeated or prolonged recordings that sam- right MTS), good memory on injection of side to be
ple drowsiness and sleep. Routine EEGs resected and poor memory on injection of hemi-
aim to answer the following questions: sphere to be preserved.
1. Are there any interictal epileptiform dis-
charges (IEDs; sharp waves, spikes, FURTHER READING
spike-and-wave complexes)? Shoenberg M. Intracarotid sodium amytal procedure (the
Wada test). In: Werz MA, editors. Epilepsy Syndromes.
2. Are the IEDs diagnostic of an idiopathic
Elsevier, Saunders; 2010.
generalized syndrome (i.e. not appropri-
ate for surgery)?
3. Are the IEDs confined to one hemisphere 7. d—Define the epileptogenic zone preopera-
or bilateral? tively
4. If unilateral, are IEDs confined to one
The neuropsychological evaluation can (1) assist
area/lobe or are they multifocal?
in lateralizing and localizing brain dysfunction

Neurosurgery Books Full


www.ketabpezeshki.com 66485438-66485457
34 ADULT AND PEDIATRIC EPILEPSY SURGERY 465

presumably related to the area of seizure origin, lobe. In general, the role of interictal PET in
(2) predict cognitive risks from surgical treat- extratemporal epilepsy is to guide the surgeon
ment, (3) study the cognitive and behavioral in where to place electrodes for invasive EEG
effects of focal and generalized epilepsies over monitoring (electrocorticography). SPECT stud-
time, and (4) assess for cognitive and mood effects ies in epilepsy are for the evaluation of regional
of antiepileptic medications. Mood can strongly cerebral blood flow, which is decreased interic-
impact on patient performance. Therefore, tally and increased ictally (up to threefold) in
measures of mood and affect (e.g. anxiety and the epileptic focus. The sensitivity of 99mTc-
depression) are frequently administered as a mea- HMPAO SPECT in detecting abnormalities
sure of test validity/task engagement. Thus, the with interictal SPECT is about 70%, while ictal
neuropsychological evaluation is to estimate the or early postictal imaging localizes epileptic foci
cognitive capabilities/reserve of the patient; to in temporal lobe epilepsy between 80% and
discern what cognitive risks may occur with the 90%. As with PET, extratemporal epileptic foci
proposed surgical treatment for a patient with are not as easily localized and the response rate
refractory epilepsy that combines results from to surgery of extratemporal foci seen on SPECT
neuropsychological evaluation, intracarotid is not as good as with temporal lobe foci. Addi-
sodium amobarbital procedure (Wada test), and tionally, SPECT is also technically difficult due
structural neuroimaging. The epileptogenic zone to tracer half-life and need to inject the tracer
is the zone whose resection or disruption is both within the first seconds of seizure onset to
necessary and sufficient to eliminate seizures and increase the likelihood of identifying just the
hence is not determinable preoperatively. area of seizure origin rather than the area of
seizure propagation. PET has increased spatial
FURTHER READING resolution compared with SPECT and has a
Schoenberg M. Neuropsychology in epilepsy. In: Werz MA, higher positive predictive value compared with
editors. Epilepsy Syndromes. Elsevier, Saunders; 2010. SPECT in the interictal state. But interictal
PET combined with ictal and interictal SPECT
8. c—Remote/distant site of seizure onset can result in a better identification of an epileptic
focus than either alone. Coregistration of sub-
The time taken for discharge to reach scalp surface tracted ictal and interictal SPECT data with
electrode can vary depending on its distance from MRI (SISCOM) will add to this armory
the site of ictal onset. Video EEG is done in con-
junction with sleep deprivation and reduction/ FURTHER READING
cessation of AEDs to maximize the chance of Faulhaber P. Nuclear imaging and epilepsy. In: Werz MA,
recording a seizure during the observation period. editors. Epilepsy Syndromes. Elsevier, Saunders; 2010.
The goals of video EEG are to:
1. Further characterize the interictal discharges 10. c—Dysphonia
and correlate ictal EEG with behavior
2. Detect, characterize and quantify the The vagal nerve stimulator (VNS) may provide
patients habitual seizures—are they having relief to patients who are not candidates for intra-
more than one type? cranial surgery, but at a cost of limited efficacy for
3. Lateralize and localize seizure onset to allow complete seizure control and MRI incompatibil-
comparison with neuroimaging findings ity. VNS should not be viewed as a low-morbidity
alternative to intracranial surgery, since it typi-
9. a—The next appropriate step would be to cally only offers a 50% reduction in seizure fre-
place subdural grid/strips quency in approximately two thirds of children
and makes minimal difference in seizure control
An epileptic focus appears as an area of hypome- in about one fourth of patients (though a subset
tabolism on an interictal FDG-PET scan and sur- may experience a marked reduction in seizure fre-
gical outcomes are better in patients with quency). The electrode is attached by three small
concordant PET and MRI foci. PET may be of flexible coils to the vagus nerve in the carotid
more importance in extratemporal lobe epilepsy, sheath in the groove between the carotid artery
where the proportion of patients with normal and jugular vein, and connects to a generator
anatomic imaging is higher than in temporal lobe placed in the anterior chest wall. The signal
epilepsy. However, interictal PET for extratem- travels up the vagus nerve to the brain stem and
poral epilepsy is less sensitive than in the temporal spreads across the diencephalon and cerebral

Neurosurgery Books Full


www.ketabpezeshki.com 66485438-66485457
466 PART V FUNCTIONAL NEUROSURGERY

cortex. The VNS output is gradually ramped up


Engel
at approximately 2-week intervals over a period Class Description Comments
of several months. Some children experience a
mild cough or hoarse voice during the first day I Free of Completely seizure free;
after the VNS is ramped up, but they quickly disabling nondisabling, simple
seizures partial seizures only;
adapt to this. Typically, a few months are some disabling seizures,
required before efficacy of the VNS for a partic- but free of disabling
ular patient will become evident. It is difficult to seizures for at least
predict preoperatively who will obtain better sei- 2 years; generalized
convulsion with
zure control, and while seizure frequency may not antiepileptic drug
decrease overall it may become short enough to withdrawal only
avoid emergency measures, or reduce the impact
II Rare disabling Initially free of disabling
at school. The VNS generator typically has seizures seizures, but rare seizures
enough energy to last 3-5 years, depending on now; rare disabling
the settings and the amount of magnet usage. seizures since surgery;
The VNS is not compatible with current model more than rare disabling
seizures, but rare seizures
MRIs with risk of thermal injury to the vagus for at least 2 years;
nerve. Thus, all patients should ideally have a rel- nocturnal seizures only
atively recent MRI before the VNS is implanted.
III Worthwhile Worthwhile seizure
The VNS is relatively contraindicated in patients improvement reduction; prolonged
with tumors that will require serial imaging, such seizure-free intervals
as children with tuberous sclerosis. The overall amounting to more than
morbidity of the surgical procedure is relatively half the follow-up period,
low, with the most common risks being bleeding, but not less than 2 years
infection, and nerve injury. A fracture in the lead IV No No significant seizure
worthwhile reduction; no appreciable
is suggested when a previously effective VNS improvement change; seizures worse
loses efficacy, adequate generator energy is pre-
sent, and the DC current is high when the
VNS is interrogated. The lead fracture is occa-
12. f—Predicting postoperative language/
sionally apparent on a plain radiograph but can
memory deficit
be difficult to appreciate preoperatively. Removal
of the lead and its replacement should only be Scalp electrodes give the best overview of interic-
considered for those children who have demon- tal epileptiform activity because they sample
strated a significant improvement in their quality extensive areas of the cranium, but they are lim-
of life, because the risks of bleeding and nerve ited by their low sensitivity due to intervening
injury are much higher than with the initial sur- high-resistance tissue and poor ability to sample
gery. When a child undergoes other surgical pro- activity from deep structures. Invasive recordings
cedures with anesthesia, it is generally resolve this limitation (improving both sensitivity
recommended that VNS output current be set and spatial localization) but have the risk of
to zero with the programmer. In emergencies sampling error as a result of the limited spatial
the magnet can be taped over the generator. Fam- distribution selected for monitoring. Stereotactic
ilies need to be counseled to advise their other implantation of depth electrode arrays to define
health care providers of the need to reprogram the epileptogenic zone provides unparalleled
the VNS so arrangements can be made. sampling from deep cortical anatomy not directly
accessible by other means, allowing for the defin-
FURTHER READING
Robinson S. Pediatric epilepsy surgery. In: Werz MA, editors.
itive lateralization and localization of mesial tem-
Epilepsy Syndromes. Elsevier, Saunders; 2010. poral lobe, insular region, mesial frontoparietal
and pericingulate, orbitofrontal, and submerged
11. b—II perisulcal cortical onset epilepsy. Bilateral tempo-
ral and frontal implantation of subdural electrode
The Engel Epilepsy surgery outcome scale is out- strips through enlarged bur holes, though less
lined below, however it is worth noting that precise, are associated with a lower risk of intra-
the ILAE have introduced a newer postsurgical cerebral hemorrhage. If localization (rather than
scale which may be more sensitive and reduce lateralization) is the priority, unilateral craniot-
ambiguity. omy for placement of subdural electrode grids

Neurosurgery Books Full


www.ketabpezeshki.com 66485438-66485457
34 ADULT AND PEDIATRIC EPILEPSY SURGERY 467

allows one to achieve refined definition of the epi- hemianopsia due to disruption of Meyer's loop of
leptogenic zone through localization of the irrita- visual fibers in the periventricular white matter sur-
tive zone and the ictal onset zone, and also to rounding the temporal horn of the lateral ventricle.
carry out extraoperative cortical mapping for pre- More extensive resection of mesial temporal
operative delineation of functionally eloquent structures, including the amygdala and hippocam-
cortex that must be spared at resection. However, pus has improved seizure control but injury to the
this comes with the risks of craniotomy, mass anterior choroidal and posterior cerebral artery
effect from the grids and a higher infection risk, branches risks contralateral hemiparesis. Equally,
but the information gained can guide the choice some have advocated sparing of the lateral temporal
between anterior temporal lobectomy and amyg- neocortex in cases of epilepsy due to mesial temporal
dalohippocampectomy and inform the tailoring sclerosis (trans-sylvian amygdalohippocampect-
of dominant temporal lobectomies to spare lateral omy). Gamma Knife Radiosurgery targeting of
cortical regions exhibiting speech arrest with mesial temporal lobe structures is available in some
stimulation (reducing postoperative language centers.
deficits while maximizing the extent of lateral
temporal resection). Craniotomy and grid Image with permission from Loevner L. Brain Imaging:
Case Review Series, 2nd ed. Elsevier, Mosby; 2009.
placement can be combined with subdural strip
electrodes (passed around the temporal pole,
underneath the temporal lobe, and under the 14. d—Seizure onset is from the left anterosu-
orbitofrontal cortex) or frameless image-guided perior temporal gyrus
implantation of depth electrodes targeting the
The epileptogenic zone is the zone whose resec-
amygdala and hippocampus. Craniotomy for
tion or disruption is both necessary and sufficient
placement of subdural electrode grids and strips
to eliminate seizures hence is only determinable
is frequently employed to guide tailored extra-
postoperatively once seizure freedom has been
temporal cortical resections.
gained. As such, epilepsy surgery targets the ictal
13. e—Temporal lobectomy onset zone and areas involved in early seizure
organization, which generally tend to coincide
Invasive monitoring is unnecessary when there or intersect with the epileptogenic zone. The ictal
is concordance between interictal and ictal onset zone is defined as the area where the ictal
video-EEG scalp recordings localizing to the discharge is first detected, regardless of its mor-
nondominant temporal lobe, with ipsilateral phology, before the clinical manifestations of
mesial temporal sclerosis on MRI and contralat- the seizure. Identifying that area (using ECoG
eral language and memory dominance on Wada and/or nuclear imaging), a major prerequisite
testing (with congruent neuropsychological for successful resective epilepsy surgery, requires
testing), ipsilateral PET hypometabolism of the familiarity with electrographic ictal patterns.
temporal lobe, and perhaps magnetoencephalo- Early ictal patterns seen on the ECoG include
graphic data. In such cases where a safe resection rhythmic sinusoidal waves, irregular spike dis-
is possible, proceeding directly to surgery is charge, spike and wave activity, low-voltage fast
usually indicated. As many as 50-90% of such activity, and high-frequency oscillations. An
patients will be rendered free of seizures postop- appropriate broad definition of an electrocortico-
eratively. Anterior Medial Temporal Lobectomy graphic ictal discharge is any electrodecremental
represents the gold standard of surgical manage- or rhythmic pattern that represents a consider-
ment of temporal lobe epilepsy. More lateral tem- able deviation from the baseline, whether or not
poral resection is carried out with the intention of it contains apiculate waveforms. In general, it is
maximizing disruption of the circuit of the epilep- believed that ictal onsets consisting of fast fre-
togenic zone while sparing the superior temporal quency activity indicate the proximity of the
gyrus (and limiting middle temporal gyrus resec- recording electrodes to the ictal onset zone
tion more on the dominant side) to minimizing whereas slower ictal onsets tend to represent
interference with language function in the dom- propagated activity. In this case, seizure onset
inant hemisphere. Alternatively, awake intrao- was from the anterosuperior temporal gyrus
perative cortical stimulation producing speech (LG41) with spread to the hippocampus (mesial
arrest to map critical language regions can be used and basal contacts). There are several caveats to
as a guide to tailor the extent of resection of the this: the epileptogenic zone may be more exten-
dominant temporal lobe. Resections that extend sive than the ictal onset zone hence resection
more posterior may be associated with an may not eliminate seizures (or adjacent areas
increased incidence of visual field deficits, ranging become capable of initiating seizures), the major-
from contralateral superior quadrantanopia to ity of epilepsy surgery patients only remain

Neurosurgery Books Full


www.ketabpezeshki.com 66485438-66485457
468 PART V FUNCTIONAL NEUROSURGERY

seizure free on antiepileptic drugs, the epilepto- 20. 1—e, Functional MRI; 2—j, Susceptibility
genic zone and the ictal onset zone may be sepa- weighted MRI; 3—d, FLAIR MRI
rate, e.g. the epileptogenic zone in a “clinically
silent” area and the seizure becomes clinically 21. 1—b, Corpus callosotomy; 2—e, Multiple
manifest only after it propagates to the temporal subpial transection; 3—a, Anatomical hemi-
lobe (ECoG will localize ictal onset to temporal spherectomy; 4—h, Tailored temporal lobe
lobe but resection will not eliminate the seizure resections are directed by the results of the
generator). initial stage of depth or subdural grid elec-
trode invasive monitoring. Because the cases
Image with permission from Werz MA. Epilepsy Syn- typically involve discordant noninvasive find-
dromes. Elsevier, Saunders; 2010.
ings, they are more likely to demonstrate
complex pathologic findings, including
purely lateral temporal lesions such as corti-
ANSWERS 15–18 cal dysplasia, tumors, vascular malforma-
tions, “dual pathology” of mesial temporal
Additional answers 15–18 available on sclerosis plus another lesion, or the absence
ExpertConsult.com of any definitive pathologic etiologic diagno-
sis. 5—i, Topectomy. Extratemporal resec-
tions are tailored by the findings of invasive
monitoring to include the epileptogenic
lesion and the epileptogenic zone while spar-
EMI ANSWER ing functionally eloquent cortical regions.
Topectomy, or the resection of cortex spar-
19. a—D, b—F, c—I, d—E, e—H ing underlying white matter, is employed.
Resection of epileptogenic cortex while
a—Lateral ventricle, b—Alveus, c—hippocam- incompletely removing the epileptogenic
pus, d—choroid plexus, e—fimbria, f—dentate lesion is associated with a lower incidence
gyrus, g—performant path, h—subiculum, of seizure control. On the other hand, resec-
i—entorhinal cortex, j—collateral sulcus, tion of the lesion alone, whether tumor or
k—alvear path, l—two-way connections with vascular malformation, is associated with a
sensory association areas lower incidence of seizure control than resec-
tion of the lesion plus surrounding epilepto-
Image with permission from Mtui E, Gruener
G, Dockery P. Fitzgerald's Clinical Neuroanatomy genic zone. This surrounding epileptogenic
and Neuroscience, 7th ed. Elsevier; 2016. zone may at times prove to contain cortical
dysplasia on pathologic evaluation.

22. 1—h, functional MRI; 2—k, Visual field test; 3—f, Psychiatric evaluation; 4—g, Wada test; 5—d,
PET imaging

Epilepsy Work-up: Rationale for Investigations


Investigation Comments

Sleep-deprived EEG Many routine EEGs are normal, especially if the patient has been taking
antiepileptic medication before the study. A prolonged (1 h) sleep-deprived EEG
may show epileptiform discharges during nonrapid-eye-movement sleep.
Video EEG Admission for monitoring while medication is tapered with the aim of recording a
minimum number of seizures. May also require sleep deprivation,
hyperventilation, photic stimulation, caffeine, vigorous exercise. Sphenoidal
electrodes should be placed when mesial temporal lobe origin is suspected. Can be
complicated by concomitant nonepileptic seizures.
MRI Standard sequences include T1, T2, and FLAIR with coronal sequences through
mesial temporal lobe. Contrast is given if inflammatory, infectious or neoplastic
etiology suspected.
Continued

Neurosurgery Books Full


www.ketabpezeshki.com 66485438-66485457
34 ADULT AND PEDIATRIC EPILEPSY SURGERY 469

Investigation Comments
PET Seizure focus is often hypometabolic on FDG-PET. In patients with frequent
seizures, if a seizure happens immediately before or during the scan the
epileptogenic focus may appear hypermetabolic and be missed—hence concurrent
EEG should be used in these patients.
Neuropsychological Identifies epileptogenic region (and associated dysfunctioning areas),
testing lateralization/dominance of language and memory, prediction of postoperative
deficits, and preoperative psychological issues needing treatment (depression,
anxiety).
Psychiatric evaluation Aim to identify and treat psychological problems which require surgery to be
delayed/withheld or arise postoperatively, e.g. depression/anxiety, nonepileptic
seizures, adjustment to seizure freedom.
Wada study Indicated preoperatively in patients with TLE or involvement of areas critical for
language. It aids localizing language and memory, assess risk of postoperative
amnesia, risk of memory decline and likelihood of seizure-free outcome.
Angiography must exclude cerebral vessel cross filling before injection of sodium
amobarbital otherwise results will be affected. Ipsilateral carotid injection allows
testing of contralateral hemisphere memory and language for 15 min.
fMRI Aids in localizing language and motor function, and interictal spikes. Not sufficient
for predicting postoperative verbal memory impairment.
Ictal SPECT with SISCOM Technically difficult hence used when preceding workup has not identified an
epileptogenic focus—usually in the case of extratemporal nonlesional epilepsy.
Interictal SPECT is performed and then the patient undergoes video EEG. As soon
as seizure onset is detected radioactive tracer injected which concentrates at the
focus (delays in tracer injection result in poorer distinction between onset zone and
propagation zone). Subtraction Ictal SPECT Coregistered to MRI (SISCOM) allows
better localization of the seizure focus and operative targeting.
Magnetoencephalography Records interictal magnetic brain waves and maps them in 3D onto MRI images of
the patient's brain. Aids localization of seizure focus in extratemporal nonlesional
epilepsy and preoperative functional mapping.
Visual field testing Testing should be done where seizure foci are in temporal, parietal or occipital
lobes; preoperative testing may reveal deficits unnoticed by patients, and new
postoperative deficits.

23. 1—e, Jacksonian March. With a Jacksonian muscles are most frequently affected. 4—l,
March, or sequential seizure, the patient Uncinate fits are characterized by seizure
develops focal seizure activity that is primar- activity involving portions of the anterior
ily motor and spreads. This type of seizure aspect of the temporal lobe resulting in
often secondarily generalizes, at which point unpleasant olfactory hallucinations. The
the patient loses consciousness and may have structures most often implicated include the
a generalized tonic-clonic seizure. The hand uncus, parahippocampal gyrus, the region
is a common site for the start of a Jacksonian of the amygdala and adjoining tissue, and
March. The face may be involved early the piriform cortex.
because the thumb and the mouth are situ-
ated near each other on the motor strip of 24. 1—a, ACTH. This child has West syndrome,
the cerebral cortex. 2—f, Myoclonic seizures a generalized seizure disorder of infants char-
may be generalized or partial benign juvenile acterized by recurrent spasms, the EEG pat-
myoclonic epilepsy (BJME). The episodes tern of hypsarrhythmia, and retardation.
occur when the affected person wakes up Several different diseases cause West syn-
rather than when he or she is falling asleep. drome. The family history in this case sug-
Myoclonic jerks may be triggered by light gests tuberous sclerosis as the underlying
flashes or loud sounds, and patients may also problem. ACTH is the best of the given
have generalized tonic-clonic seizures. 3—b, choices. 2—c, Ethosuximide. This girl has
Epilepsia partialis continua is persistent focal generalized absence attacks. This may be a
motor seizure activity (i.e. a focal motor sta- manifestation of a more complex epilepsy
tus epilepticus). The distal hand and foot syndrome or may occur as an isolated

Neurosurgery Books Full


www.ketabpezeshki.com 66485438-66485457
470 PART V FUNCTIONAL NEUROSURGERY

finding. Generalized absence attacks have no lished eclampsia. The dose is 4-5 g intrave-
aura and no postictal period. The affected nously, followed by a 1 g/h intravenous
child has no warning that an attack is about infusion. Magnesium sulfate was shown to
to occur and is usually unaware that one result in a reduction in recurrent seizures
has occurred unless it is more than a few sec- and in maternal morbidity and mortality
onds long. In fact, generalized absence sei- compared with both diazepam and phenyt-
zures are most often only a few seconds oin. In addition, the fetus should be deliv-
long. Ethosuximide is the drug of choice, ered as quickly as possible, using cesarean
but it may cause gastrointestinal distress. section if necessary. Eclampsia is an example
Divalproex sodium is effective in many of of the posterior leukoencephalopathy syn-
the children who cannot tolerate ethosuxi- drome. 4—e, Levetiracetam. This young
mide or who are not well controlled on that woman is having complex partial seizures
antiepileptic. If the absence seizures are asso- without secondary generalization. She has
ciated with generalized tonic-clonic seizures, episodic altered consciousness associated
divalproex sodium is a better choice. Some with a temporal lobe seizure focus and ante-
antiepileptic drugs, such as the sodium chan- dated by febrile seizures. Levetiracetam is
nel blockers phenytoin and carbamazepine, the best choice because of its relatively good
can actually worsen generalized from onset efficacy and adverse effect profile. Felba-
seizures. 3—g, Magnesium sulfate. Recent mate, phenobarbital, primidone, and dival-
studies have established that magnesium sul- proex sodium may also be effective at
fate (MgSO4) is the optimal treatment both controlling the seizures, but all have side
to prevent seizures in women with hyperten- effect profiles making them poor first
sion at the time of admission for delivery choices in this case.
(preeclampsia) and to treat seizures in estab-

Neurosurgery Books Full


www.ketabpezeshki.com 66485438-66485457
CHAPTER 35

ADULT MOVEMENT DISORDERS


SINGLE BEST ANSWER (SBA) QUESTIONS
1. A 40-year-old man has a family history of a. Serotonin (5-HT)
hereditary neurodegenerative disease and b. Acetylcholine
early death. His father died of a rapidly pro- c. Enkephalin
gressive dementia at the age of 50 and he d. Dopamine
has previously been told that he has the e. GABA
defective gene. He presents with personality
change and non-purposeful, slow, rhythmic 5. Which one of the following statements
movements of his hands and face. A magnetic regarding dystonia is most accurate?
resonance imaging (MRI) indicates atrophy a. Isolated foot dystonia is very rare and may
in the head of the caudate nucleus. Which suggest an underlying Parkinsonian disor-
one of the following neurotransmitter defi- der or brain structural abnormality
ciencies is most likely to be responsible? b. Torticollis is the least common dystonia
a. Dopamine in the substantia nigra c. Segawa's disease is a primary dystonia
b. Substance P in the subthalamic nucleus responsive to antidopaminergic drugs
c. Acetylcholine and gamma-aminobutyric d. Trihexyphenidyl is the primary treatment
acid in the striatum for drug-induced dystonia
d. Serotonin in the globus pallidus e. Blepharospasm is involuntary opening of
e. Glutamate in the cortex eyelids resulting in paradoxical blinking

2. A 43-year-old man has a father who died 6. A 65-year-old man presents with difficulty
from Huntington's disease. The son was getting out of chairs and problems walking
tested and found to have the gene for Hun- over the last year with multiple falls. On
tington's disease. Which one of the following examination, his face appeared mask-like
is true regarding the offspring of those with with reduced blinking and slight drooling,
Huntington's disease? and he had a pill-rolling tremor at rest
a. All children are at risk only if the affected bilaterally, and on passive movement demon-
parent is male strated cogwheel rigidity. His handwriting
b. 1 in 10 are at risk only if the affected became smaller towards the end of a
parent is female sentence. There was no weakness, sensory
c. 1 in 8 are at risk for the disease problems, or abnormalities in his reflexes.
d. 1 in 4 are at risk for the disease There are no autonomic symptoms. He had
e. 1 in 2 are at risk for the disease a stooped posture and walked with a slow
shuffling gait, with a noticeably reduced
3. Damage to the subthalamic nucleus is associ- arm swing. Which one of the following
ated with which one of the following statements regarding this condition is most
a. Myoclonus accurate?
b. Dystonic tremor a. It may present with a 3 Hz rest pill
c. Hemiballism rolling tremor
d. Levodopa-induced dyskinesia b. The major neuronal loss is in the substan-
e. Tardive dyskinesia tia nigra pars reticularis
c. It is associated with a fluent aphasia
4. Tardive dyskinesia is most likely the result d. Loss of smell is a non-motor symptom
of receptor changes causing hypersensitivity of this disease
to which one of the following neurotransmit- e. Dyskinesias are common in non-
ters? medicated patients

471
Neurosurgery Books Full
www.ketabpezeshki.com 66485438-66485457
472 PART V FUNCTIONAL NEUROSURGERY

7. Which one of the followings statements c. STN stimulation


regarding Parkinson's plus syndromes is d. Red nucleus stimulation
LEAST accurate? e. Ventral striatal/ventral internal capsule
a. MSA-P (Striatonigral degeneration) is stimulation
characterized by Parkinsonism with auto-
nomic failure 11. Which one of the following targets is most
b. Dementia with Lewy bodies is often asso- commonly used for the treatment of dystonia?
ciated with detailed visual hallucinations a. GPi
c. Tremor is rare in progressive supranu- b. Vim thalamus
clear palsy c. Vo thalamus
d. Corticobasal degeneration may include d. STN
cortical sensory loss and apraxia e. Nucleus accumbens
e. MSA-C (olivopontocerebellar atrophy) is
characterized by Parkinsonism with pro- 12. Which one of the following statements
gressive ataxia regarding thalamotomy is LEAST accurate?
a. It is most appropriate for those with
8. A 67-year-old man had idiopathic tremulous predominantly unilateral symptoms
Parkinson's disease diagnosed 15 years ago, b. Lesions placed to laterally may result in
and has had a 10-year history of progressive contralateral weakness
worsening in wearing-off phenomenon. c. Bilateral thalamotomy is the treatment
Despite increasing the dose and frequency of choice in bilateral tremor dominant
of levodopa this had still deteriorated, and Parkinson's disease when deep brain stim-
higher doses induced peak-dose dyskinesias. ulation is not possible
High-frequency stimulation of which one d. Speech disturbance is a common problem
of the following brain structures is most likely e. Lesions placed too posteriorly may
to improve his symptoms? cause numbness and paresthesias of the
a. Unilateral STN mouth
b. Bilateral STN
c. Unilateral Vim
d. Bilateral Vim QUESTIONS 13–18
e. Bilateral GPi
Additional questions 13–18 available on
ExpertConsult.com
9. A 25-year-old man has had motor tics since
age 13. They seem to be getting worse, and
now he also has involuntary obscene vocali-
zations. Which one of the following stimula- EXTENDED MATCHING ITEM (EMI)
tion targets would you use in this patient?
a. Centromedian nucleus-parafascicular com- QUESTIONS
plex of thalamus
b. Vim thalamus 19. Movement disorder signs:
c. Posterior hypothalamus a. Akithisia
d. Subthalamic nucleus b. Asterixis
e. Hippocampus c. Ataxia
d. Athetosis
10. A 22-year-old male is involved in a road e. Ballism
traffic collision and sustains a significant f. Bradykinesia
traumatic brain injury. CT head shows trau- g. Chorea
matic subarachnoid hemorrhage and hemor- h. Clonus
rhagic contusions extending into the basal i. Dyskinesia
ganglia. After a long period of rehabilitation j. Dystonia
he is left with a residual left arm tremor. k. Myoclonus
Which one of the following could be consid- l. Rigidity
ered to manage his tremor? m. Stereotypy
a. Motor cortex stimulation n. Tic
b. VIM thalamic stimulation o. Tremor

Neurosurgery Books Full


www.ketabpezeshki.com 66485438-66485457
35 ADULT MOVEMENT DISORDERS 473

For each of the following descriptions, select the right hand. On examination, he has a
most appropriate answers from the list above. tremor of the right hand, which disappears
Each answer may be used once, more than once when he reaches to grab a pen. Movements
or not at all. are slower on the right than the left. He has
1. A 5-year-old child with Rett syndrome cogwheel rigidity of the right arm
exhibits hand wringing 3. A 56-year-old presents 4 months after a
2. A 45-year-old patient with hepatic enceph- thalamic stroke with a left arm 3 Hz
alopathy exhibits sudden and involuntary action tremor with a “wing beating”
relaxation of a dorsiflexed hand appearance
3. A 35-year-old with personality change and 4. A 47-year-old patient presents with a jerky,
jerky limb movements, which he often low-frequency 2 Hz high-amplitude action
attempts to mask by incorporating them tremor. He also had impairment in finger-
into seemingly purposeful actions nose and heel shin testing
4. A 67-year-old presents with a reduction in
movement velocity 21. Chorea, athetosis, and ballism:
5. A 70-year-old presents with involuntary, a. Ataxia telangiectasia
high-amplitude arm flinging episodes b. Benign hereditary chorea
3 months after an ischemic stroke c. Cerebrovascular disease
6. A 34-year-old schizophrenic started on an d. Chorea gravidarum
antipsychotic presents 3 months later with e. Huntington's disease
involuntary, non-suppressible lip smack- f. Mitochondrial disease (Leigh's disease)
ing, pouting and tongue protrusion g. Neuroacanthocytosis
h. Spinocerebellar ataxia
20. Tremor: i. Sydenham's chorea
a. Cerebellar tremor j. Wilson's disease
b. Drug-induced tremor
c. Dystonic tremor For each of the following descriptions, select the
d. Essential tremor most appropriate answers from the list above.
e. Holme's tremor Each answer may be used once, more than once
f. Neuropathic tremor or not at all.
g. Palatal tremor 1. A 12-year-old girl presents with acute rheu-
h. Parkinsonian tremor matic fever develops rhythmic, writhing
i. Physiological tremor movements in all four limbs
j. Primary orthostatic tremor 2. A 35-year-old patient with Kaiser-Fleischer
k. Psychogenic tremor rings and a low serum ceruloplasmin
3. A 48-year-old patient with new onset atrial
For each of the following descriptions, select the fibrillation who presents with hemichorea
most appropriate answers from the list above. 4. A 19-year-old, left-handed woman has had
Each answer may be used once, more than once several weeks of nausea, vomiting, and
or not at all. weight gain. She has been taking cyclizine
1. A 40-year-old literary agent has had wors- with some reduction in vomiting. She has
ening tremor of the hands. This has been also noticed the recent onset of an involun-
present for 2 years, but has increasingly tary relatively rapid and fluid, but not
impaired her work ability because she is rhythmic, limb and trunk movements.
frequently required to take her clients to Neurological examination is otherwise
lunch, and she is embarrassed by her unremarkable. Which one of the following
inability to eat and drink normally. A glass is the most likely diagnosis?
of wine with the meal typically helps 5. A 26-year-old presents with orofacial dys-
somewhat. On examination, there is a tonia and psychiatric disturbance. Periph-
mild head tremor, but no rest tremor of eral blood smear shows acanthocytes
the hands. When she holds a pen by the
tip at arm's length, however, a coarse 22. Dystonia:
tremor is readily apparent. Examination a. DYT1 dystonia (Oppenheim's dystonia)
is otherwise normal b. DYT5 dystonia (Segawa's disease)
2. A 64-year-old man has noticed dragging of c. DYT11 Myoclonic-dystonia
the right leg and tremor and stiffness of the d. Huntington's disease

Neurosurgery Books Full


www.ketabpezeshki.com 66485438-66485457
474 PART V FUNCTIONAL NEUROSURGERY

e. Lesch-Nyhan syndrome i. Rotigotine


f. Meige syndrome j. Selegiline
g. Multiple system atrophy
h. Neuroacanthocytosis For each of the following descriptions, select the
i. Spinocerebellar ataxia most appropriate answers from the list above.
j. Wilson's disease Each answer may be used once, more than once
or not at all.
For each of the following descriptions, select the 1. First line therapy for elderly patients pre-
most appropriate answers from the list above. senting with Parkinson's disease
Each answer may be used once, more than once 2. First line transdermal therapy for early
or not at all. onset Parkinson's disease
1. A 53-year-old woman is unable to stop 3. May have neuroprotective effect if given
blinking forcefully and has frequent grima- early in Parkinson's disease
cing movements of the face. At times, she 4. Extending the duration of action of
protrudes her tongue against her will. She levodopa-carbidopa
has never taken any medications
2. A 14-year-old boy from an Ashkenazi 25. Motor complications in Parkinson's
Jewish family presents with dystonia affect- disease:
ing his right leg which is not responsive to a. Acute akinesia
levodopa b. Akathisia
3. A 9-year-old boy presents with left foot c. Camptocormia
dystonia which shows a diurnal fluctuation d. Diphasic dyskinesia
and improves with levodopa treatment e. Dystonia
f. Freezing of gait
23. Adverse Effects of DBS: g. No-on phenomenon
a. Ataxia h. Peak-dose dyskinesia
b. Blepharospasm i. Unpredictable off periods
c. Diplopia j. Wearing off phenomenon
d. Dysarthria
e. Infection For each of the following descriptions, select the
f. Intracranial hemorrhage most appropriate answers from the list above.
g. Microthalamotomy effect Each answer may be used once, more than once
h. Paresthesia or not at all.
i. Photopsias 1. A 72-year-old man presented with idio-
j. Tonic contraction pathic Parkinson's disease has been on
levodopa treatment for 5 years. His dose
For each of the following descriptions, select the requirements have been steadily increasing,
most appropriate answers from the list above. and after the most recent dose increase he
Each answer may be used once, more than once complains of odd movements in his limbs
or not at all. about an hour after taking his tablets.
1. Overstimulation of the subthalamic 2. An 82-year-old with Parkinson's disease is
nucleus admitted to hospital as her symptoms are
2. Stimulation posterior to subthalamic so severe she is frozen and cannot take
nucleus her levodopa dose. Her temperature is
3. Stimulation posteromedial to globus palli- 38.6 °C (101.8 °F), RR 30, O2 Sat 91%
dus interna (air) and CXR demonstrates right lower
4. Stimulation posterior to Vim thalamus lobe consolidation
5. Stimulation anterior to Vim thalamus 3. A 73-year-old patient diagnosed 4 years
ago with Parkinson's disease and is cur-
24. Medical therapy for Parkinson's disease: rently taking levodopa-carbidopa every
a. Amantidine 4 h. He presents with recurrence of brady-
b. Apomorphine kinesia and tremor 3 h after each dose of
c. Benztropine levodopa
d. Entacapone 4. A 67-year-old patient on levodopa-carbidopa
e. Levodopa-carbidopa for Parkinson's disease diagnosed 7 years
f. Nortriptyline ago presents with early morning painful
g. Rivastigmine (or donepezil) flexion and inversion postures of the feet
h. Ropinerole (or pramipexole) and toes

Neurosurgery Books Full


www.ketabpezeshki.com 66485438-66485457
35 ADULT MOVEMENT DISORDERS 475

SBA ANSWER
1. c—Acetylcholine and gamma-aminobutyric It is believed to occur as a result of an imbalance
acid in the striatum in the output signals of the basal ganglia, with
overactivity of the direct pathway relative to the
Huntington's disease is characterized by loss of indirect pathway. This is in contrast to Parkin-
striatal neurons resulting in reduced levels of cho- son's disease, where overactivity of the indirect
line acetyltransferase, glutamic acid decarboxyl- pathway relative to the direct pathway results in
ase, and GABA in the striatum resulting in a bradykinesia.
relative excess of dopamine causing a hyperki-
netic movement disorder with writhing and jerk- 4. d—Dopamine
ing movements of the limbs (chorea). Dopamine
antagonists, such as haloperidol, may be used to Tardive dyskinesia results from treatment with
suppress chorea, but also carry the risk of provok- dopamine receptor blocking agents. Tardive syn-
ing tardive dyskinesia. Dopaminergic drugs, such dromes are less frequently caused by atypical than
as L-dopa, bromocriptine, and lisuride, may by typical neuroleptics. The most common pat-
unmask chorea but should not be used diagnosti- tern of tardive dyskinesia is stereotyped and
cally as it may not abate. repetitive movement of the face (e.g., tongue-
thrusting and involuntary chewing movements
2. e—1 in 2 are at risk for the disease is often accompanied by a feeling of restlessness).
This akathisia may be localized and reported
Huntington's disease is transmitted in an autoso- as a “burning” sensation, often of the genitals
mal dominant fashion. The age at which the or mouth.
patient becomes symptomatic is variable and has
no effect on the probability of transmitting the 5. a—Isolated foot dystonia is very rare and may
disease. The defect underlying this degenerative suggest an underlying Parkinsonian disorder
disease is an abnormal expansion of a triplicate or brain structural abnormality
repeat (CAG) sequence in the HTT gene on
chromosome 4 (normally coding for the protein Focal dystonias produce abnormal sustained mus-
huntingtin). Normal individuals have between 6 cle contractions in a single region of the body:
and 34 copies of this CAG section; patients with • Neck (torticollis): most commonly affected
Huntington's disease may have from 36 to more site with a tendency for the head to turn to
than 100 repeats. People with the adult-onset one side.
form of Huntington's disease typically have more • Eyelids (blepharospasm): involuntary clo-
than 36 CAG repeats in the HTT gene (although sure of the eyelids that leads to excessive
those with 36-39 still may not develop signs/ eye blinking, sometimes with persistent
symptoms) and those with the juvenile form of eye closure and functional blindness.
the disorder tend to have more than 60 CAG • Mouth (oromandibular dystonia): involun-
repeats. Individuals who have 27-35 CAG repeats tary contraction of muscles of the mouth,
in the HTT gene do not develop Huntington's tongue, or face.
disease, but they are at risk of having children • Hand (writer's cramp).
who will develop the disorder. As the gene is
passed from parent to child, the size of the Isolated foot dystonia is very rare and may suggest
CAG trinucleotide repeat may lengthen into the an underlying Parkinsonian disorder or brain
range associated with Huntington's disease (36 structural abnormality. Generalized dystonia
repeats or more). Once expanded beyond 40 cop- affects multiple areas of the body and can lead
ies, the repeats are unstable and may further to marked joint deformities. Primary dystonia
increase as they are passed on from one generation can be sporadic or heritable (e.g., DYT1 dystonia,
to the next. An increased number of repeats in suc- Segawa's disease). Secondary dystonia results
cessive generations can lead to earlier disease from basal ganglia insults (stroke, demyelination,
onset, a phenomenon termed anticipation. hypoxia, trauma), Huntington's disease, Wilson's
disease, Parkinson's syndromes, and lysosomal
3. c—Hemiballism storage diseases. Acute or chronic (tardive) dyski-
nesias can occur with dopaminergic antagonists
A lesion of the subthalamic nucleus results in (e.g., antipsychotics, antiemetics). In addition to
hemiballism, a form of dyskinesia in which the removing any offending drug, trihexyphenidyl
patient displays severe involuntary movements. is an anticholinergic drug which is used to

Neurosurgery Books Full


www.ketabpezeshki.com 66485438-66485457
476 PART V FUNCTIONAL NEUROSURGERY

manage chorea, dystonia and dyskinesias (by cor- 7. e—MSA-C (olivopontocerebellar atrophy) is
recting the imbalance between dopamine and characterized by Parkinsonism with progres-
acetylcholine in the basal ganglia). sive ataxia

6. d—Loss of smell is a non-motor symptom of Many other disorders present with Parkinsonian
this disease features early in their course, and the more char-
acteristic features of some of these “Parkinson's
Idiopathic Parkinson's disease is a neurodegenera- plus” syndromes (e.g., gaze paralysis in progres-
tive condition caused by progressive loss of dopa- sive supranuclear palsy or autonomic dysfunction
minergic cells in the substantia nigra pars in multiple system atrophy) may not become
compacta projecting to the striatum. This pro- apparent until several years after symptom onset.
duces a hypokinetic movement disorder character- An incorrect diagnosis of early Parkinson's dis-
ized by bradykinesia, rigidity and resting tremor. ease is probably made between 10% and 20%
More specific symptoms include: a slow shuffling of the time, even among Parkinson's disease spe-
gait with a tendency to move progressively faster cialists. An accurate early diagnosis of Parkinson's
(festinating gait); micrographia; mask-like facial disease is becoming increasingly important as the
expression with reduced eye blinking; and diffi- long-term effects of early treatment on natural
culty getting out of a chair; quiet monotonous history are better understood. Secondary
voice (hypophonia); muscle rigidity (lead-pipe Parkinsonism may be idiopathic, drug-induced,
rigidity); pill-rolling rest tremor 4-8 Hz, which toxin-induced, due to cerebrovascular disease or
combines with rigidity to produce “cogwheeling” structural lesions, or post-traumatic. Clinically
on passive flexion by the examiner. Non-motor similar to Parkinson's disease, symptomatic
symptoms may also have a major impact on quality Parkinsonism may be identified by history
of life: drooling (reduced swallowing), dementia, (e.g., medication history or exposure to toxins)
REM sleep disorders, loss of smell, constipation, or by symptoms and signs consistent with an
mood disorder (especially depression), orthostatic underlying disorder. Clues suggesting secondary
hypotension, bladder and erectile dysfunction. Parkinsonism include acute/subacute onset, sym-
Dopamine cannot cross the blood-brain barrier, metric symptoms, rapid progression or static
hence medical therapy must increase striatal dopa- course, poor response to dopaminergic drugs,
mine by other means (e.g., prevent catecholamine history of exposure to causative drugs/toxins/
breakdown [MAOI-B], provide the immediate CNS infection/cerebrovascular disease, and signs
precursor to dopamine [levodopa], or other dopa- of underlying metabolic or structural brain
minergic receptor agonists). disease.

Parkinson's Plus Syndrome Description

Multiple system atrophy (Shy- Classified into MSA-P (striatonigral degeneration) and MSA-C
Drager syndrome) (olivopontocerebellar atrophy) subtypes. Predominance of rapidly progressive
Parkinsonian features (rigidity, postural instability, tremor and gait freezing) in
MSA-P and cerebellar features (gait, limb, and speech ataxia) in MSA-C. Both are
accompanied by autonomic failure (urinary and bowel dysfunction, impaired
potency and libido, decreased sweating and orthostatic hypotension)

Progressive supranuclear Progressive supranuclear palsy, or Steele-Richardson-Olszewski syndrome, is


palsy a rare, progressive brain disorder that causes serious and permanent problems
with control of gait and balance. In particular, ocular motor control problems
(blurred vision and difficulties looking up or down/saccades) and problems with
speech and swallowing are commonly seen in progressive supranuclear palsy.
Tremor is rare in these patients. Often there is retrocollis (hence tend to fall
backwards) and a characteristic wide-eyed/astonished facial expression

Corticobasal degeneration Corticobasal degeneration is a progressive neurological disorder with


symptoms similar to those of Parkinson's disease, such as poor coordination,
akinesia, rigidity, impaired balance, and limb dystonia. Both cortex and basal
ganglia affected—cortical sensory loss and apraxia are important clinical signs
Dementia with Lewy bodies Neurodegenerative dementia associated with Parkinsonism. Well-formed and
detailed visual hallucinations; may be accompanied by delusions

Neurosurgery Books Full


www.ketabpezeshki.com 66485438-66485457
35 ADULT MOVEMENT DISORDERS 477

8. b—Bilateral STN limbic (anteromedial) or motor (posteroventral)


regions of GPi. A double-blind, randomized cross-
A number of prospective and randomized trials over trial on GPi DBS has reported a significant
comparing STN and GPi DBS have contributed improvement in tic severity, with an overall
to a better definition of the differences between acceptable safety profile.
these two targets. STN stimulation is superior
for rigidity, bradykinesia, cost (medication reduc- FURTHER READING
tion and less frequent battery change) and is more Fasano A, Lozano AM. Deep brain stimulation for movement
popular for tremor due to proximity to zona disorders: 2015 and beyond. Curr Opin Neurol 2015;28:423-36.
incerta. GPi stimulation is superior for dyskine-
sia, dystonia (including levodopa-unresponsive; 10. b—VIM thalamic stimulation
STN only works for levodopa-responsive), cog-
nition, mood and apathy, axial motor symptoms, DBS of the ventro-intermedius nucleus (Vim) of
does not adversely affect speech/swallowing, can the thalamus is an effective treatment for essential
be implanted unilaterally, and requires less fre- tremor, tremor dominant PD and other types of
quent programming (initially). The motor bene- tremor. In essential tremor, a loss of benefit is
fits can be similar with each target, but STN sometimes observed over time. In both short-
offers greater benefit for off-symptoms, facilitates and long-term studies, dysarthria and disequilib-
medication dose lowering and is most cost effi- rium are the most frequent reported adverse
cient, whereas dyskinesia suppression and long- effects, especially with bilateral stimulation. The
term effects on postural stability and cognitive very mild cerebellar ataxia displayed by ET
function favor GPi. STN stimulation may risk patients may also be improved by the stimulation
behavior and impulse control disorders, but med- of the posterior subthalamic area (STA), beneath
ication reduction reduces these. the inferior border of the Vim, where the
dentato-thalamic tract runs—conversely strong
FURTHER READING stimulation worsens cerebellar side effects.
Fasano A, Lozano AM. Deep brain stimulation for movement
disorders: 2015 and beyond. Curr Opin Neurol 2015;28:423-36. FURTHER READING
Fasano A, Lozano AM. Deep brain stimulation for movement
9. a—Centromedian-parafascicular nucleus of disorders: 2015 and beyond. Curr Opin Neurol 2015;28:423-36.
thalamus
11. a—GPi
Tourette's syndrome is defined by the onset of
motor and vocal tics before 18 years of age that GPi DBS in isolated dystonias (either generalized
cannot be ascribed to another medical condition. or segmental) is supported by strong evidence of
Tics must occur multiple times over at least 1 year success, with improvement. Improvement is sus-
and must evolve over time. The first tics are usu- tained up to 10 years after surgery, although some
ally observed around the age of 5 or 6, and tic patients need the implantation of additional GPi
severity peaks 4-5 years later, and is lowest in electrodes. In addition, there are encouraging
patients' early 20s, coincident with frontal lobe results in cervical dystonia, myoclonus-dystonia
maturation. Tics are worsened by heightened and tardive dystonia, in which the outcome of
emotional states, stress, and fatigue. The tics of DBS is usually excellent, rapid and sustained.
Tourette's syndrome are commonly accompa- Case reports have also shown that cranial/cervical
nied by attention-deficit/hyperactivity disorder dystonias may not recur when DBS has stopped
(ADHD) and obsessive-compulsive disorder working (for a variety of reasons) suggesting
(OCD). In Tourette's syndrome, obsessions chronic changes to circuits.
center on concerns with symmetry, fear of
violent thoughts, and a need to perform activities FURTHER READING
in a particular manner (rather than fears of con- Fasano A, Lozano AM. Deep brain stimulation for movement
tamination and checking seen in primary OCD). disorders: 2015 and beyond. Curr Opin Neurol 2015;28:423-36.
These obsessions may lead to self-injurious behav-
ior. There is evidence that it shows a sex-linked 12. c—Bilateral thalamotomy is the treatment of
autosomal dominant mode of inheritance, and is choice in bilateral tremor dominant Parkin-
due to dysfunction in the corticostriatal- son's disease when deep brain stimulation is
thalamocortical loop. The most commonly used not possible
targets for DBS have been the thalamic nuclei
(centromedian nucleus-parafascicular complex The best candidates for thalamotomy are patients
and ventral oral nuclei) followed by either the with tremor-predominant PD or those with

Neurosurgery Books Full


www.ketabpezeshki.com 66485438-66485457
478 PART V FUNCTIONAL NEUROSURGERY

incapacitating benign essential tremor. Less pre- while right thalamic lesions are associated with
dictable outcomes are seen with tremor and impaired visuospatial memory and nonverbal per-
hemiballismus/chorea due to damage of the cer- formance abilities. Bilateral thalamotomies are
ebellar tracts from cerebrovascular accidents, associated with deficits in memory/cognition
trauma or multiple sclerosis, and primary and sec- and speech problems (e.g., hypophonia, dysar-
ondary dystonias. It is important to confirm the thria, dysphasia, and abulia) in up to 60%, hence
clinical diagnosis of idiopathic PD or benign should not be undertaken routinely—where they
essential tremor since Parkinson's plus syn- must be done it should be staged and slight vari-
dromes have a much poorer prognosis after tha- ation in the target coordinates between sides may
lamotomy. Evidence of cognitive decline, reduce major side effects.
speech disorders, serious systemic disease, and
advanced age are also considered contraindica-
tions to surgery. Specific complications of thala-
motomy are due to inaccurate lesion placement ANSWERS 13–18
or overly large lesions. Lesions placed too later-
ally may result in contralateral weakness due to Additional answers 13–18 available on
ExpertConsult.com
injury of the posterior limb of the internal capsule
(face and arm). Lesions placed too posterior may
cause contralateral hemisensory deficits due to
injury of the VC nucleus (e.g., numbness or
paresthesias of the mouth or fingers). A signifi-
cant proportion have transient dysarthria or
EMI ANSWER
dysphasia, and transient confusion and may
persist permanently in some. Left thalamic 19. 1—m, Stereotypy; 2—b, Asterixis; 3—g,
lesions are associated with an increased risk for Chorea; 4—f, Bradykinesia; 5—e, Ballism;
deficits in learning, verbal memory and dysarthria 6—i, Dyskinesia

Movement Disorder Symptoms

Bradykinesia Bradykinesia refers to a decrease in movement velocity (a reduction in amplitude is termed


hypokinesia). The term akinesia, when properly used, refers to a complete lack of movement or
an inability to initiate movement

Rigidity A function of enhanced static or postural reflexes and either a “lead pipe” or “cogwheel” quality
(tremor superimposed on rigidity) and is typically asymmetric

Chorea Chorea refers to an involuntary, continual, irregular hyperkinetic disorder in which movements or
movement fragments with variable rate and direction occur unpredictably and randomly. All
body parts may be involved, and usually worsen during attempted voluntary action. Individuals
may generate semivolitional movements that attempt to mask the involuntary choreic
movements or incorporate them into seemingly purposeful movements, such as touching the
face (parakinesias)

Ballism An involuntary, high-amplitude, flinging movements typically generated proximally. May be


brief or continual and may occur with chorea. Where there is hemiballism (one side of the body
affected, usually due to a STN lesion), it may become milder and evolve into chorea over time

Athetosis Slow, writhing, continuous, involuntary movements (not sustained postures like dystonia). It
often accompanies basal ganglia disorders producing chorea or dystonia

Akithisia Akathisia refers either to an uncomfortable sensation of inner restlessness or to the voluntary
activity performed to relieve that restlessness. It is often manifested by an inability to remain
seated, crossing and uncrossing the legs, or pacing. Causes include neuroleptic medication, but
may be difficult to distinguish from tics and restless legs syndrome (RLS)

Dystonia Dystonia refers to a group of disorders characterized by involuntary muscle contractions


(sustained or spasmodic) that lead to abnormal body movements or postures

Continued

Neurosurgery Books Full


www.ketabpezeshki.com 66485438-66485457
35 ADULT MOVEMENT DISORDERS 479

Dyskinesia Dyskinesia refers to any disordered (non-rhythmic) and involuntary, non-suppressible


movement including chorea/athetosis/ballism (and to a lesser degree dystonia). The limbs, neck,
and face are the most frequently affected, but axial symptoms may also occur. When dyskinesia
occurs in the face, the features may appear wry or overanimated. Head bobbing, blinking, lip
smacking, and tongue protrusion are common. When dyskinesia occurs in the limbs, the
movements may be proximal or distal and of either high or low amplitude. The limbs may tap,
whirl, or writhe. Dyskinesia syndromes include abdominal (belly dancer's) dyskinesia, levodopa-
induced dyskinesia, tardive dyskinesia, and the paroxysmal dyskinesias

Tremor Involuntary rhythmic oscillation of a body part about a set point. The tremor may be regular or
irregular, unilateral or bilateral, symmetrical or asymmetric, and present in one or several body
regions. The frequency and amplitude of a tremor depend heavily on its underlying cause

Myoclonus A sudden, arrhythmic, involuntary movement that is “shock-like” in its rapidity. When multiple,
these movements do not flow into one another, which distinguishes them from chorea. True
myoclonus is due to brief synchronous firing of agonist and antagonist muscles. Positive
myoclonus occurs with active muscle contraction (e.g., hypnic jerks, a sudden body-wide
contraction that occurs as a person drifts between sleep and wakefulness). Myoclonus is most
often encountered as one of a collection of symptoms rather than as a pathology's primary
manifestation. Symptomatic myoclonus may be a feature of any process involving cortical, basal
ganglionic, or cerebellar degeneration, such as Creutzfeldt-Jakob disease or PD; Hepatic, renal,
endocrine, and other metabolic derangements; myoclonic epilepsies, periodic leg movements of
sleep and others

Ataxia Clumsy or poorly organized movements due to deficits in the cerebellar, vestibular, or
proprioceptive pathways. It may affect speech, manual dexterity, or gait and patients often
complain of feeling as though they are drunk. Pure ataxia is not associated with deficits in
strength or motor planning. Movements are poorly aimed or timed; patients have difficulty
properly estimating the distance required to reach a target or terminating an action at the proper
moment

Clonus Rhythmic movement from hyperactive stretch reflex

Asterixis An example of negative myoclonus and consists of sudden and involuntary relaxation of a
dorsiflexed hand or other body part. The EMG pattern of negative myoclonus is distinctive, with
aperiodic electrophysiologic silences in the antagonist muscle groups

Tic Brief movements (motor tic) that are commonly preceded by a feeling of discomfort that builds
until the tic appears, followed by a temporary feeling of relief. These preceding “premonitory
urges” may consist of a feeling of itching or tension in the affected body part (sensory tic). One of
the hallmarks of tics is that they are temporarily suppressible, although they typically rebound
with increased frequency and severity after conscious suppression. They are both purposefully
executed but performed out of a feeling of need (“semivoluntary” or “involuntary”). Tics can be
clonic (i.e., brief), dystonic (i.e., sustained), or phonic (vocal). Simple tics consist of isolated
actions, such as throat clearing or winking. Complex tics consist of speech or coordinated
actions. They sometimes include obscene gestures (copropraxia) or vocalizations (coprolalia)

Stereotypies Stereotypies are repetitive movements or vocalizations that mimic a purposeful action, are
performed outside that action's normal context, and are involuntary or semivoluntary (e.g., hand
wringing of Rett's syndrome). Stereotypies should be differentiated from automatisms (epilepsy)
and perseverative/repetitive behavior (e.g., ADHD, OCD)

FURTHER READING 20. 1—d, Essential tremor; 2—h, Parkinsonian


Clinical overview of movement disorders, In: Winn HR (Ed.), tremor; 3—e, Holmes’ tremor; 4—a,
Youman's Neurological Surgery, vol. 4, 6th ed., Elsevier, Cerebellar tremor
Saunders, 2011 [Chapter 75].

Neurosurgery Books Full


www.ketabpezeshki.com 66485438-66485457
480 PART V FUNCTIONAL NEUROSURGERY

Types of Tremor

Essential tremor The most common tremor disorder and is a low-amplitude, bilateral action and
postural tremor with a frequency of 6-8 Hz. The tremor usually has its onset in
adulthood and worsens over time, but it may begin in childhood and can coexist with
other movement disorders. ET involves the upper limbs in more than 90% of patients. It
less commonly involves the head, legs, or voice. Patients commonly first complain of
difficulty with tasks requiring fine coordination, such as threading a needle, tying knots,
or writing. Alcohol may temporarily alleviate symptoms, and family history is often
positive. May be due to nonstructural cerebellar dysfunction with ET patients
commonly have an intention tremor and difficulty with tandem gait

Dystonic tremor Dystonic tremor is a jerky postural and task/action 3-8 Hz tremor that is abolished by
complete rest and occurs in a body part affected by dystonia (e.g., limbs, trunk, head,
vocal cords, or face). This contrasts with essential tremor in which the kinetic component
is more or less constant throughout all postures. Dystonic tremor may occur with other
involuntary movements including blepharospasm, torticollis, or spasmodic dysphonia.
Isolated head tremor can be particularly challenging to distinguish between essential
tremor and dystonic tremor (the latter of which can be exquisitely responsive to
pharmacological treatment or botulinum toxin injections). The tremor caused by
cervical dystonia is usually present as “no-no” kind of head shaking

Physiological tremor Non-pathologic postural tremor, which typically has a frequency of 8-12 Hz. Both
ET and physiologic tremor can be elicited by posture, both are fairly symmetrical,
and both occur predominantly in the arms. Observing the progression of a tremor over
time will eventually reveal whether a given patient has ET or physiologic tremor

Holme's tremor Although predominantly an action tremor, Holmes tremor frequently has a significant
resting component. The amplitude of movement tends to be large and it can
sometimes adopt a “wing-beating” appearance. It is also among the slowest tremors,
with frequencies often less than 4 Hz. It can occur with lesions affecting not only the
red nucleus and rubral spinal tract in the brainstem but also the cerebellum and
thalamus. The tremor may appear weeks to months after a known lesion (e.g., stroke),
and some patients may have associated dystonia

Neuropathic tremor Tremor may accompany diseases of the anterior horn cell (e.g., amyotropic lateral
sclerosis) and peripheral neuropathies. It is unclear whether tremor associated with
peripheral neuropathy is due to enhanced physiologic tremor secondary to
weakness, to an abnormality in the central nervous system, or both

Drug-induced tremor The onset of tremor should be temporally related to drug ingestion and may be due to
an enhancement of physiologic tremor (e.g., amiodarone, antidepressants,
antiepileptic medications, beta-agonist bronchodilators, caffeine, lithium,
neuroleptics, nicotine, steroids, and sympathomimetics) or production of cerebellar
tremor (immunosuppressive agents and acute/chronic alcohol)

Psychogenic tremor Clinical features suggesting psychogenic tremor include sudden onset with severe
presentations, inconsistent combinations of resting and postural or kinetic tremor,
entrainment (change in frequency of tremor to that of a task performed in another
body part, e.g., a patient with left hand tremor who taps at various frequencies with
the right hand will have a left hand that acquires those frequencies), and tremor that
diminishes with distraction

Parkinsonian tremor A 4-9 Hz low-amplitude rest tremor, often with a “pill-rolling” quality. A typical pattern
of spread is for the dominant hand to be affected first, followed by the dominant foot
and then the non-dominant hand. Re-emergent tremor occurs while sustaining a
prolonged position and most likely represents a rest tremor that has been reset by the
relative stasis of a persistent position. Postural tremor that begins immediately on
adopting a position is seen in as many as 93% of patients with PD and correlates with
the degree of functional disability. Treatment with levodopa improves bradykinesia
and rigidity more reliably than it does tremor

Cerebellar tremor Cerebellar tremor is characterized as a jerky, low-frequency (2-5 Hz), high-amplitude
action tremor. This tremor may be accompanied by other cerebellar signs such as
ataxia, dysdiadochokinesia, dysarthria, dysmetria, and telegraphic speech

Post-traumatic tremor The character of the tremor depends on the region of the brain that is damaged.
Damage to the brainstem may produce rest tremor if it affects the substantia nigra
and related pathways. Damage to the cerebellum may result in a low-frequency

Continued

Neurosurgery Books Full


www.ketabpezeshki.com 66485438-66485457
35 ADULT MOVEMENT DISORDERS 481

action tremor. Because multiple regions are usually damaged, post-traumatic


tremors are generally mixed in character. Post-traumatic tremor is often
accompanied by myoclonus

Primary orthostatic Uncommon condition that starts in late adulthood with feeling of tremulousness in
tremor the legs when standing (but not sitting or lying). The diagnosis is helped by specific
surface EMG showing a 14-18-Hz oscillating tremor in the musculature of the legs
when standing, disappearing with rest or movement

Palatal tremor (Palatal A rare disorder presenting as unilateral or bilateral rhythmic involuntary
myoclonus/nystagmus) movements of the soft palate. The movement consists of repetitive rather than
oscillatory movements of agonist muscles only, thus having some similarity with
myoclonus. The tremors in essential palatal tremor produce audible click due to
the contraction of the tensor valipalatini muscle which disappear during sleep,
whereas in symptomatic palatal tremor there is no audible click but it continues
during sleep

21. 1—i, Sydenham's chorea. psychiatric symptoms, and seizures. Choreoa-


canthocytosis is an autosomal recessive disease with
A delayed complication of infection with group A an age at onset of 20-40 years and orofacial dystonia
β-hemolytic streptococci that usually develops 4- (e.g., lip and tongue biting, involuntarily push food
8 weeks after the infection, and the most common out of their mouths with their tongue when eating),
cause of acute childhood chorea in the world. The generalized chorea, dystonia, and tics can also
typical age at onset of Sydenham's chorea is 8-9 occur. Abnormalities of saccadic eye movement
years; it is rarely seen in children younger than may develop, similar to those in HD. Symptoms
5 years. The chorea usually generalizes but some of McLeod's syndrome include limb weakness, cho-
patients remain hemichoreic. Associated symp- rea, cognitive decline, paranoia, and schizophrenia.
toms include tics, OCD, and ADHD. The disease
is self-limited and spontaneously remits after 8-9 • Genetic causes include Huntington's dis-
months in a large percentage of patients, but up to ease, neuroacanthocytosis, and Wilson's
50% may still have chorea 2 years after infection. disease.
Huntington's disease typically presents with neuro- • Infectious causes include rheumatic fever
psychiatric symptoms that predate choreiform causing Sydenham's chorea, chorea gravi-
movements. 2—j, Wilson's disease. An autosomal darum during pregnancy, and spongiform
recessive disorder of copper metabolism that causes encephalopathy (e.g., prion disease).
both liver and basal ganglia damage (hepatolenticular • Drug-induced causes include oral contra-
degeneration). usually appear between the ages of 11 ceptives, tricyclic antidepressants, cimeti-
and 25. Patients typically have a Parkinsonian dine, digoxin, verapamil, baclofen,
akinetic-rigid syndrome, generalized dystonia, or steroids, and antiepileptics.
a proximal postural/action tremor with ataxia and • Strokes in the thalamic area of the brain
dysarthria. Although pure chorea may be seen in can also cause choreiform movements.
Wilson's disease, it is an unusual manifestation of
the syndrome. Psychiatric findings include pseudo- 22. 1—f, Meige syndrome.
bulbar affect, impulsivity, and depression. Cerulo-
plasmin testing may be normal hence elevated 24- Meige syndrome is a form of focal dystonia char-
h urinary copper levels is diagnostic. The classic acterized by blepharospasm, forceful jaw open-
Kaiser-Fleischer rings—flecks of copper visible in ing, lip retraction, neck contractions, and
the cornea under slit-lamp examination—are tongue thrusting. Sometimes these features are
almost universally present in patients with neuro- produced by phenothiazine or butyrophenone
logical symptoms. 3—c, Cerebrovascular disease. use, but they may also occur idiopathically, more
4—d, Chorea gravidarum. Chorea in pregnancy often in women than men, with onset in the sixth
usually presents in the first or second trimester, decade. Botulinum toxin injection has been more
and may be autoimmune in nature. The severity effective in treatment than any oral medication.
of the chorea tends to decrease as the pregnancy 2—a, DYT1 dystonia (Oppenheim's dystonia).
progresses. Approximately a third of patients see Primary dystonias are either sporadic mutations
their symptoms resolve after delivery. 5—g, Neu- or heritable. The most common cause of early-
roacanthosis. Includes choreoacanthocytosis and onset generalized dystonia is DYT1 dystonia
McLeod's syndrome, which both show acanthocytes (Oppenheim's dystonia). It occurs relatively fre-
on peripheral smear, peripheral neuropathy, quently in the Ashkenazi Jewish population, with

Neurosurgery Books Full


www.ketabpezeshki.com 66485438-66485457
482 PART V FUNCTIONAL NEUROSURGERY

a prevalence of 1 in 2000. DYT1 is inherited in an responsive proximal myoclonic jerks. The dystonia
autosomal dominant fashion with a penetrance of is usually mild and present in 50% of patients. Cer-
30-40%. The onset of symptoms is usually in late vical dystonia and writer's cramp are the most com-
childhood/early adolescence, and they generally mon. Many adults with myoclonus-dystonia
begin in one leg and later generalize. Dystonia syndrome report dramatic improvement in their
plus syndromes are conditions in which Parkinson- symptoms with alcohol ingestion.
ism, tremor, or myoclonus develop in addition
to dystonia. Dystonia plus can be divided Secondary dystonia is caused by basal ganglia
into dystonia-Parkinsonism (DYT3 and DYT12), insults (stroke, demyelination, hypoxia, trauma),
dopa-responsive dystonia (DYT5), paroxysmal Huntington's disease, Wilson's disease, Parkin-
dystonia (DYT8, DYT9, and DYT10), son's syndromes, drugs and lysosomal storage
and myoclonus-dystonia (DYT11). 3—b, DYT5 diseases.
dystonia (Segawa's disease). DYT5, or Segawa's
disease, is a childhood-onset levodopa-responsive 23. 1—b, Blepharospasm; 2—h, Paresthesia;
dystonia. The initial symptom is generally foot dys- 3—j, Tonic contraction; 4—h, Paresthesias;
tonia, with a marked diurnal fluctuation that atten- 5—a, Ataxia
uates with age. A postural tremor typically develops
in adulthood, followed later by bradykinesia. Rest General risks of DBS surgery include infection,
tremor is absent. Response to levodopa is marked intracerebral hemorrhage, electrode malposition,
and sustained. Both autosomal dominant and and lead-related complications (fractures/infec-
autosomal recessive subtypes have been identified. tion). Stimulation related side effects related to
DYT11 has its onset in childhood or adolescence direction of current spread and adjacent struc-
and is manifested as dystonia with alcohol- tures and are outlined below:

Target Current Spread Site Affected Side Effect

STN STN STN Blepharospasm

Anterior Corticospinal tract Tonic contractions


Hypothalamus Sweating/flushing

Posterior Medial lemniscus Paresthesias

Medial Red nucleus Ataxia

Inferomedial CNIII/Supranuclear oculomotor center Diplopia

Lateral Corticobulbar tract Dysarthria

GPi Anterior Optic tract Photopsias

Posterior Medial lemniscus Paresthesias

Posteromedial Internal capsule Tonic contraction

VIM thalamus Target VIM thalamus Microthalamotomy effect

Anterior Cerebellothalamic tract Ataxia

Posterior VPL thalamus Paresthesia

Lateral Internal capsule Tonic contraction

24. 1—e, Levodopa-carbidopa; 2—i, Rotigotine; and patient lifestyle. The activities of daily living
3— j, Selegiline; 4—d, Entacapone subscale of the UPDRS is useful to assess disabil-
ity. Current opinion is divided about when to
It is generally accepted that symptomatic therapy start therapy with levodopa-carbidopa. Delay
should start when a patient becomes functionally may minimize the risks of motor complications
impaired, although functional impairment is and theoretical progression of disease by oxidant
highly individualized depending on symptoms radical formation from the metabolism of

Neurosurgery Books Full


www.ketabpezeshki.com 66485438-66485457
35 ADULT MOVEMENT DISORDERS 483

Medical Management of Parkinson's Disease


Drug Class Example Role

MAOI-B Selegiline May have neuroprotective effect hence given early in disease. Selegiline
Rasagiline blocks free radical formation during dopamine oxidation

Levodopa Levodopa- Usually first line in elderly patients; can lead to motor complications with
carbidopa prolonged treatment. Conversion of L-dopa to dopamine occurs outside
the CNS in a wide variety of tissues, but once converted to dopamine in the
periphery, the drug becomes inaccessible to the brain. Peripheral
conversion of L-dopa to dopamine is routinely inhibited by adding a
dopa decarboxylase inhibitor (carbidopa). Because it is cannot cross the
blood-brain barrier, carbidopa cannot inhibit the conversion of L-dopa to
dopamine in the brain

Dopamine Pramipexole Usually first line in young patients (early onset PD). For elderly patients,
agonists Ropinerole dopamine agonists are often a second-line option. The effects of these
Rotigotine agents are independent of degenerating dopaminergic neurons.
(transdermal) Therefore, the use of dopamine agonists may avoid problems associated
with levodopa-carbidopa. The dopamine agonists do not, however,
improve all types of symptoms and have specific dopaminergic adverse
effects. Rotigotine is a dopamine agonist that is administered
transdermally and offers continuous dopaminergic stimulation. It is
approved by the FDA for early-stage idiopathic Parkinson's disease

Apomorphine For the treatment of acute, intermittent hypomobility and “off” episodes

NMDA Amantidine Potentiates dopaminergic response with a mild anticholinergic effect


antagonist and can be used for early Parkinsonism, as well as for reduction of
levodopa-induced dyskinesias associated with later-stage Parkinson's
disease

Anticholinergic Benztropine Treatment of isolated tremor

COMT inhibitor Entacapone Reduce motor fluctuation by extending the duration of action of levodo
pa-carbidopa, thus having a dose-sparing effect and reducing “off-time”

Antidepressant SSRI and TCA TCAs are contraindicated in patients taking MAOI

Cholinesterase Rivastigmine Treatment of mild to moderate dementia includes improve cognition


inhibitor and activities of daily living in about 15% of patients

levodopa. Most specialists, in fact, delay its intro- Motor fluctuations are alterations between
duction, giving less potent medications a trial periods of being “on,” during which the patient
first, especially in younger patients. Levodopa- experiences a positive response to medication,
carbidopa is generally the first-choice medica- and being “off,” during which the patient experi-
tion for most elderly patients and is almost uni- ences a re-emergence of the Parkinson's symp-
versally effective. Combination therapy begins toms. Levodopa-induced dyskinesia encompass
with the addition of a COMT inhibitor to a variety of involuntary movements, including
levodopa-carbidopa. Dopamine agonists and chorea, dystonia, ballism, and myoclonus. Dyski-
levodopa-carbidopa can be tried next. In some nesia tend to appear when the patient is “on” and
patients, it may be necessary to use all three are usually choreiform. Dyskinesia in the “off ”
agents. Parkinson's related depression is the state is more commonly dystonic but can be any
most common nonmotor symptom in Parkin- of those listed above. Early morning dystonic
son's disease, affecting 40% of patients. inversion of a foot (usually on the side of greater
Parkinsonian involvement) occurs as a with-
drawal reaction because of the long interval with-
25. 1—h, Peak dose dyskinesia; 2—a, Acute aki- out medication overnight. Dyskinesia are
nesia; 3—j, Wearing-off phenomenon; 4—e, sometimes mistaken for manifestations of pro-
Dystonia gressive PD or confused with tremor by patients
and their families, rather than recognized as

Neurosurgery Books Full


www.ketabpezeshki.com 66485438-66485457
484 PART V FUNCTIONAL NEUROSURGERY

Motor Complications of Parkinson's Disease


Motor Phenomena
in PD Description

Wearing off Re-emergence of Parkinsonian motor problems as the effect of levodopa diminishes near
phenomenon the end of the dose interval (i.e., usually before 4 h)

Unpredictable off Transitions from being “on” to being “off” bearing no obvious relationship with the
periods time of levodopa administration. May be due to erratic absorption of levodopa from the
gut and pharmacodynamic changes in the brain

Freezing of gait Occurs as a transient “off” phenomenon or randomly at variable frequency in patients
with advanced PD. Patients suddenly become immobilized for seconds to minutes at
a time, usually when initiating walking, in a confined space such as a doorway or a closet,
or when getting on or off an elevator and may cause falls. Random freezing is poorly
responsive to anti-Parkinson's medications and DBS

No-on Lack of an “on” response following a dose of levodopa. May be due to delayed gastric
phenomenon motility, especially when no-on preceded by an excessively prolonged or severe “off”
period

Acute akinesia Sudden severe exacerbation of PD including an akinetic state that lasts for several days
and does not respond to treatment with anti-Parkinson's medication. Commonly due to
systemic infection or other intercurrent medical problem

Peak-dose Peak dose or “on” dyskinesia is usually choreiform in type: appearance of restlessness
dyskinesia and continuous jerky, involuntary movements of the extremities, head, face, trunk, or
respiratory muscles, typically starting 30-90 min after a dose of levodopa. Remarkably
well tolerated by most patients as prefer being “on” with dyskinesia to being “off,” but
severe dyskinesia may take the form of large amplitude, ballistic movements that interfere
with function and become very disturbing to patients and their families

Diphasic dyskinesia Uncommon form where dyskinesia peaks twice after each dose—first when patients turn
“on” and again when they begin to turn “off.” In the second phase, dyskinesia (often
involving the legs) in one body part may coexist with the emergence elsewhere in the
body of Parkinsonian signs such as tremor and bradykinesia

Dystonia Dystonia can be a manifestation of early untreated PD or may appear as a complication of


levodopa treatment. Dystonia in young-onset PD most commonly involves the foot,
typically taking onset as exercise-induced cramp-like discomfort, first noticed in the toes
and later evolving into inversion of the affected foot, sometimes bringing the person to a
halt (kinesigenic foot dystonia). Dystonia is also common in non-medicated patients
with Parkinson's plus syndromes (e.g., retrocollis, blepharospasm). Dystonia due to
levodopa can occur either as a peak levodopa effect or during “off” periods due to
levodopa withdrawal. Withdrawal dystonia most commonly occurs in the early morning
when it produces painful flexion and inversion postures of the feet and toes

Akathisia Another form of levodopa withdrawal is akathisia or motor restlessness, which may
resemble restless legs syndrome, and usually occurs at night, several hours after the
last dose of levodopa. This is managed by providing slow release levodopa or a dopamine
agonist before retiring

Camptocormia Camptocormia is an apparently fixed trunkal flexion deformity that disappears in


recumbent position

reversible consequences of levodopa treatment. anti-Parkinsonian medication or manipulating


Dyskinesia occurs in 30-40% of patients treated dietary protein intake.
with levodopa during the first 5 years of use
and nearly 60% or more by 10 years. Manage- FURTHER READING
ment usually involves adjusting the levodopa Motor fluctuations and dyskinesia in Parkinson disease.
doses and dosing schedule, adding an additional Uptodate. Topic 4893 Version 20.0.

Neurosurgery Books Full


www.ketabpezeshki.com 66485438-66485457
CHAPTER 36

SURGERY FOR PSYCHIATRIC


DISORDERS
SINGLE BEST ANSWER (SBA) QUESTIONS
1. In anorexia nervosa, which one of the follow- 6. Anterior cingulotomy involves which one of
ing surgical targets has been most used? the following?
a. Anterior nucleus of thalamus a. Bilateral burr holes and thermocoagulation
b. Hypothalamus b. Parasagittal approach to cingulate gyrus
c. Inferior thalamic peduncle c. Division of fibers connecting the orbital
d. Subgenual anterior cingulate cortex to subcortical and limbic areas
e. Subthalamic nucleus d. Exclusion of obsessive-compulsive symp-
toms preoperatively
2. In Alzheimer's disease, which one of the fol- e. The target site for the lesion is 20-25 mm
lowing targets for deep brain stimulation has posterior to the anterior horn of the lateral
shown promise? ventricles, 7 mm from the midline and
a. Caudate 20 mm above corpus callosum.
b. Fornix
c. Globus pallidus externa 7. Which one of the following components is
d. Hippocampus LEAST important in the mechanism of vagal
e. Nucleus accumbens nerve stimulation for treatment-refractory
depression is most accurate?
3. Circuit pathophysiology in OCD has been a. Amygdala
localized to which one of the following? b. Hippocampus
a. Corticoreticular circuit c. Locus coereleus
b. Corticostriatal-thalamocortical circuit d. Prefrontal cortex
c. Nigro-striatal circuit e. Red nucleus
d. Nigro-pallidal circuit
e. Thalamocortical circuit 8. Which one of the following statements
regarding indications for lesional neurosurgi-
4. Targets for deep brain stimulation for cal procedures for mental disorder is most
obsessive-compulsive disorder do not include accurate?
which one of the following? a. Cerebrovascular disease and pre-existing
a. Anterior limb of the internal capsule epilepsy are absolute contraindications
b. Centromedian-parafascicular nucleus of b. Patients can be treated in their best inter-
thalamus ests with neurosurgery for mental
c. Inferior thalamic peduncle disorder
d. Subthalamic nucleus c. Used occasionally to control affective or
e. Ventral capsule/ventral striatum obsessional symptoms due to active organic
or degenerative brain disease
5. Which one of the following targets is not cur- d. Can be considered in personality disorder
rently used in deep brain stimulation for and schizophrenia if the aim of the sur-
treatment-resistant depression? gery is restricted to chronic intractable
a. Anterior limb of the internal capsule affective or obsessional comorbid
b. Nucleus accumbens symptoms
c. Subgenual anterior cingulate e. Not currently performed in a multidisci-
d. Subthalamic nucleus plinary team setting
e. Ventral capsule/ventral striatum

485
Neurosurgery Books Full
www.ketabpezeshki.com 66485438-66485457
486 PART V FUNCTIONAL NEUROSURGERY

EXTENDED MATCHING ITEM For each of the following descriptions, select the
most appropriate answers from the list above.
(EMI) QUESTIONS Each answer may be used once, more than once
or not at all.
9. Surgery for psychiatric disorders:
a. Anterior capsulotomy 1. Generates a magnetic field that traverses
b. Anterior cingulotomy the cranium and induces an electrical field
c. Electroconvulsive therapy in the cortex. Used for measuring and mod-
d. Limbic leucotomy ulating cortical plasticity approved for use
e. Nucleus accumbens DBS in medication-refractory depression.
f. Subcaudate tractotomy 2. Division of fibers connecting the orbital
g. Subthalamic nucleus DBS cortex to subcortical and limbic areas (e.g.
h. Transcranial magnetic stimulation thalamus, basal ganglia, amygdala). Lesions
i. Vagal nerve stimulation are placed in the white matter of the sub-
j. Ventral capsule/Ventral striatum DBS stantia innominata, below the head of the
caudate nucleus.

SBA ANSWERS
1. d—Subgenual anterior cingulate OCD has a prevalence of 1-3% of the population
and is characterized by recurrent, intrusive anx-
A number of small recent studies have described ious thoughts (obsessions) accompanied by
the possible use of DBS in the treatment of repetitive stereotyped behaviors or mental rou-
anorexia nervosa and associated mood include sub- tines (compulsions) that are frequently per-
genual anterior cingulate and nucleus accumbens. formed in an effort to reduce distress caused
by obsessions. These can significantly hinder
FURTHER READING interpersonal relationships, social and occupa-
Fitzgerald PB, Segrave RA. Deep brain stimulation in mental tional functioning, and the ability to carry out
health: review of evidence for clinical efficacy. Aust N Z J Psy- basic activities of daily living; it is associated with
chiatry 2015. pii: 0004867415598011. [Epub ahead of print]
a higher lifetime risk of suicide (up to 27% of
Review. PubMed PMID: 26246408.
patients), as well as major depression. Treatment
2. b—Fornix for OCD typically involves pharmacotherapy
(e.g. selective serotonin reuptake inhibitors)
The two targets being investigated clinically that which is often combined with psychotherapy
hold promise for dementias are the fornix for Alz- (e.g. cognitive behavioral therapy/exposure and
heimer's disease, and the nucleus basalis of Mey- response prevention). Up to half of OCD
nert for dementia with Lewy bodies. patients do not obtain adequate benefit with
standard treatment approaches, and approxi-
FURTHER READING mately 10% experience severe treatment-
Zhang Q, Kim YC, Narayanan NS. Disease-modifying ther- refractory symptoms. Identification of DBS tar-
apeutic directions for Lewy-Body dementias. Front Neurosci gets for OCD has been based on a combination
2015;9:293. of experience from lesional psychosurgery pro-
3. b—Corticostriatal-thalamocortical circuit cedures, following observations of response to
surgery for other conditions (e.g. STN DBS
Functional neuroimaging studies have implicated for PD) as well as gradual target refinement fol-
corticostriatal-thalamocortical circuitry in the lowing ongoing evaluation of clinical outcomes
pathophysiology of OCD. More specifically, in relation to lead location. The neuroanatomi-
in patients with OCD, there is abnormal (predom- cal targets used for DBS in the treatment of
inately increased) metabolic activity in the orbito- OCD have included the anterior limb of the
frontal cortex (OFC), the anterior cingulate cortex internal capsule, nucleus accumbens, ventral
(ACC), medial prefrontal cortex and the caudate capsule/ventral striatum, STN, and the inferior
nucleus, particularly its ventral division. OFC thalamic peduncle.
and caudate hyperactivity are directly correlated
with symptom severity, and these changes partially FURTHER READING
normalize with successful treatment. Fitzgerald PB, Segrave RA. Deep brain stimulation in mental
health: review of evidence for clinical efficacy. Aust N Z J
4. b—Centromedian-parafascicular nucleus of Psychiatry 2015. pii: 0004867415598011 [Epub ahead of
thalamus print] Review. PubMed PMID: 26246408.

Neurosurgery Books Full


www.ketabpezeshki.com 66485438-66485457
36 SURGERY FOR PSYCHIATRIC DISORDERS 487

5. d—Subthalamic nucleus on stimulation of the anterior limb of the internal


capsule and the associated ventral capsule/ventral
Depression is extremely common with a lifetime striatum (including the nucleus accumbens). Less
prevalence of 15-20%, with 30% of patients not commonly described targets include superolateral
responding to standard medications or psychother- branch of the median forebrain bundle, inferior
apy. The management of treatment-resistant thalamic peduncle, and lateral habenula.
depression includes repeated trials of medication,
psychotherapy, and forms of brain stimulation
(transcranial magnetic stimulation and electrocon- FURTHER READING
Fitzgerald PB, Segrave RA. Deep brain stimulation in mental
vulsive therapy). However, there is a significant
health: review of evidence for clinical efficacy. Aust N Z J
subgroup of patients (10-20%) who remain chron- Psychiatry 2015. pii: 0004867415598011. [Epub ahead of
ically treatment refractory, and relapse and devel- print] Review. PubMed PMID: 26246408.
opment of resistance to ECT also poses
problems. Targets for DBS applications in depres-
sion have been proposed based on (a) extrapolation 6. a—Bilateral burr holes and thermocoagulation.
from sites targeted in lesional psychosurgical
procedures and (b) from the results of neuroimag- FURTHER READING
ing experiments. The majority of research to date Christmas D, Morrison C, Eljamel MS, Matthews K.
has focused on DBS implantation in the white mat- Neurosurgery for mental disorder. Adv Psychiatr Treat
ter adjacent to the subgenual anterior cingulate and 2004;10:189-199

Lesional Surgery for Psychiatric Disorders


Procedure Description Indications

Anterior Thermal damage or gamma knife surgery to target Indications for anterior capsulotomy
capsulotomy fibers connecting the ventromedial cortex, vary across Europe. In Sweden, it is
orbitofrontal cortex, and anterior cingulate gyrus used for generalized anxiety disorder,
with the thalamus, amygdala, and hippocampus. agoraphobia with panic disorder, and
These fibers pass through the anterior one-third obsessive-compulsive disorder,
of the anterior limb of the internal capsule whereas in the UK it is mainly used for
depression and obsessive-compulsive
disorder
Anterior Targets are the supracallosal fibers of the cingulum Initially developed for the treatment
cingulotomy bundle (part of the Papez circuit) as it travels of intractable pain, but its other
through the anterior cingulate gyrus. The lesion indications include anxiety disorders,
procedure also results in damage to a localized depressive disorders, and obsessive-
area of anterior cingulate cortex. The target site for compulsive disorder
the lesion is 20-25 mm posterior to the anterior
horn of the lateral ventricles, 7 mm from the
midline and 2-3 mm above corpus callosum
Limbic The fibers targeted are those of the anterior Its main indications are for depression,
leucotomy cingulate cortex, the cingulum bundle, and obsessive-compulsive disorder, and
frontostriatolimbic circuits. It is a combination of anxiety disorder. Used as a treatment
anterior cingulotomy and stereotactic subcaudate alternative where anterior cingulotomy
tractotomy, although the frontal lesions are slightly has resulted in non-sustained or
smaller than those conventionally created using partial benefit
stereotactic subcaudate tractotomy
Stereotactic Division of fibers connecting the orbital cortex to This procedure has been used to treat
subcaudate subcortical and limbic areas (e.g. thalamus, basal depression, obsessive-compulsive
tractotomy ganglia, amygdala). Lesions are placed in the white disorder, anxiety disorder, and chronic
matter of the substantia innominata, below the pain, although it is no longer offered
head of the caudate nucleus. Typically, the lesions within the UK
would be created using radioactive yttrium-90
rods inserted using stereotactic guidance

Neurosurgery Books Full


www.ketabpezeshki.com 66485438-66485457
488 PART V FUNCTIONAL NEUROSURGERY

7. e—Red nucleus restricted to chronic intractable affective or


obsessional comorbid symptoms
Vagus nerve stimulation has become established
for treatment-resistant, partial-onset seizure dis- Neurosurgery for mental disorder has suffered
order. The vagus nerve is not only a parasympa- historically from relatively “crude” lesional pro-
thetic efferent nerve—around 80% of its fibers cedures (e.g. anterior cingulotomy, limbic leucot-
are afferent sensory fibers transmitting informa- omy, subcaudate tractotomy), and a lack of
tion to the brain. There are sensory afferent con- rigorous investigation regarding both effective-
nections of the vagus nerve in the nucleus tractus ness and adverse effects on personality and cogni-
solitarius that, in turn, send ascending projec- tion. Inclusion criteria are a secure diagnosis and
tions to the forebrain, mainly through the para- the ability to provide informed consent. In all
brachial nucleus and locus ceruleus. Further cases, such surgery can only be offered following
connections offer potential routes of communi- careful and detailed consideration of the potential
cation with the amygdala, hippocampus, hypo- costs and benefits to the individual on a case-by-
thalamus, insular cortex, dorsal thalamus, case basis by the multidisciplinary team. Obvious
orbitofrontal cortex, and other important limbic contraindications include patients with affective
regions linked to mood regulation. The initial or obsessional symptoms due to active organic
rationale for using VNS for the treatment of or degenerative brain disease, or where pervasive
refractory depression resulted from mood developmental disorder is likely (although cere-
improvements in epilepsy patients treated with brovascular disease or pre-existing epilepsy is
VNS, irrespective of the presence or absence not an absolute contraindication). There is no
of beneficial effects on seizure frequency. VNS evidence that personality disorders, anorexia ner-
demonstrated steadily increasing improvement vosa, or schizophrenia respond to lesional neuro-
of depressive symptoms with full benefit after surgery and these patients should not be
6-12 months, sustained for up to 2 years. These considered unless the aim of the surgery is
studies reported response rates of 30-40% and restricted to chronic intractable affective or
remission rates of 15-17% after 3-24 months obsessional comorbid symptoms. Difficulties
of treatment. Preclinical animal studies may sug- can arise in determining the suitability of patients
gest that VNS exerts its antidepressant effects where illness onset was at a sufficiently early age
through a rapid increase in the concentration to have had an adverse impact on personality
of the monoamines, which then enhance neuro- development. Neurosurgery for mental disorder
nal plasticity/neurogenesis in the hippocampus. is contraindicated if the patient is not fit for sur-
Newborn cells could then functionally integrate gery because of a tendency to bleed, local infec-
and restore the disturbed corticolimbic networks tion, or a high anesthetic risk.
in depressed patients, and may explain the ther-
apeutic lag of VNS in the treatment of
FURTHER READING
depression. Christmas D, Morrison C, Eljamel MS, Matthews K. Neuro-
surgery for mental disorder. Adv Psychiatr Treat 2004;10:
FURTHER READING 189-199
Grimonprez A, Raedt R, Baeken C, Boon P, Vonck K. The
antidepressant mechanism of action of vagus nerve stimula-
tion: evidence from preclinical studies. Neurosci Biobehav
Rev 2015;56:26-34. doi:10.1016/j.neubiorev.2015.06.019
[Epub 2015 Jun 25]. Review. PubMed PMID: 26116875. EMI ANSWERS
8. d—Can be considered in personality disorder 9. 1—h, Transcranial magnetic stimulation;
and schizophrenia if the aim of the surgery is 2—f, Subcaudate tractotomy

Neurosurgery Books Full


www.ketabpezeshki.com 66485438-66485457
PART VI
PERIPHERAL NERVE
SURGERY
CHAPTER 37

PERIPHERAL NERVE
SINGLE BEST ANSWER (SBA) QUESTIONS
1. Which one of the following type of nerve fiber a. Fibrillation potentials
carries fast pain and temperature sensation? b. Increased number of motor unit
a. Type A alpha potentials
b. Type A beta c. Positive sharp spikes
c. Type A delta d. Reduced CMAP
d. Type A gamma e. Reduced interference pattern
e. Type B
f. Type C 6. Which one of the following best describes a
motor unit potential?
2. Which one of the following tests is best for a. Sum of conduction velocities of motor
confirming an isolated cervical radiculopathy? neurons
a. Compound muscle action potential (CMAP) b. Seen during muscle necrosis
b. F-wave c. Reinnervation potential
c. H-reflex d. Sensory axon loss
d. Insertional activity on EMG e. Sum of electrical activity from muscle
e. Sensory Nerve Action Potential (SNAP) fibers supplied by the same motor neuron

3. EMG abnormalities in which one of the fol- 7. The F-wave is useful when trying to confirm
lowing muscles is LEAST specific for C5 which one of the following:
radiculopathy? a. Bilateral carpal tunnel syndrome
a. Infraspinatus b. Isolated C7 nerve root lesion
b. Levator scapulae c. Isolated S1 nerve root lesion
c. Pronator teres d. Multiple proximal motor root
d. Rhomboids compromise
e. Supraspinatus e. Multiple sensory root compromise

4. Fibrillation potentials are an indicator of: 8. The H-reflex is used in which one of the fol-
a. Dorsal root ganglion injury lowing scenarios:
b. Motor axon loss a. Suspected common peroneal nerve palsy
c. Muscle necrosis b. Suspected L4 radiculopathy
d. Reinnervation c. Suspected L5 radiculopathy
e. Sensory axon loss d. Suspected lumbar plexopathy
e. Suspected S1 radiculopathy
5. Which one of the following is the earliest
electrophysiological change seen in 9. Which one of the following are features of a
radiculopathy? primarily demyelinating neuropathy?

489
Neurosurgery Books Full
www.ketabpezeshki.com 66485438-66485457
490 PART VI PERIPHERAL NERVE SURGERY

a. Fibrillation potentials d. Positive sharp spikes


b. Increased insertional activity e. Reduced conduction velocity
c. Polyphasic waves

10. A 76-year-old woman presented with numbness and tingling of the hands that had persisted for the
previous 5 weeks. Examination revealed mild limitation of neck movements. Strength and reflexes were
normal. She had mild atrophy of the right thenar muscles. Pain sensation was decreased in the radial 3½
digits; Phalen’s test was positive bilaterally. Neurologic examination was otherwise negative and the
EMG was done.
Motor nerve studies
Nerve and Site Latency (ms) Amplitude (mV) Conduction Velocity (m/s)
Median nerve R. Normal  4.2 Normal  6 Normal  50
Wrist 7.8 1.7 -
Elbow 12.2 1.7 55
Ulnar nerve R. Normal  3.6 Normal  8 Normal  50
Wrist 3.0 11 -
Elbow 6.4 10 56
Median nerve L. Normal  4.2 Normal  6 Normal  50
Wrist 4.3 6 -
Elbow 8.4 6 55

F-wave studies
Nerve Latency (ms) Normal Latency  (ms)
Median nerve R. 39.4 30
Ulnar nerve R. 27.2 30
Median nerve L. 28.1 30

Sensory nerve studies

Normal Normal
Onset Onset Peak Peak Normal Conduction Normal
Latency Latency Latency Latency Amp Amp ≥ Velocity Conduction
Nerve (ms)  (ms) (ms)  (ms) (μV) (μV) (m/s) Velocity ≥ (m/s)
Median 6.5 2.6 7.0 3.1 5 20 20 50
nerve R.
Ulnar 2.4 2.6 2.9 3.1 25 13 50 50
nerve R.
Median 3.4 2.6 3.9 3.1 26 20 38 50
nerve L.

EMG data
Insert Pos
Muscle Activity Fibs Waves Fasc Amp Dur Poly Pattern
Brachioradialis R. Norm None None None Norm Norm None Full
Flexor carpi Norm None None None Norm Norm None Full
ulnaris R.
Extensor digitorum Norm None None None Norm Norm None Full
communis R.
Abductor pollicis Norm None None None Lg Inc None Red
brevis R.
First dorsal Norm None None None Norm Norm None Full
interosseous R.

Results from Bertorini TE. Neuromuscular Case Studies, 1st ed., Butterworth-Heinemann, Elsevier, Case 1, 2008.
Neurosurgery Books Full
www.ketabpezeshki.com 66485438-66485457
37 PERIPHERAL NERVE 491

Which one of the following explains her symptoms?


a. C6 and C7 radiculopathy
b. C6 radiculopathy
c. Carpal tunnel syndrome
d. Musculocutaneous nerve entrapment
e. Ulnar nerve entrapment

11. A 57-year-old man presented with an 8 week history of progressive weakness and wasting in the
left hand and mild numbness in the little finger. His past medical history was positive for polio-
myelitis as a child, which left residual weakness in both legs. He had an almost complete recovery
except for a mild deformity of the left foot. He did have occasional fasciculations in the leg. On
examination, the left hand interossei muscles were wasted but without fasciculations. He had a
positive Froment's sign and a Tinel's sign below the left elbow. When extending his fingers,
the last two digits had a tendency to remain flexed. He could not “cup” the left hand. Adson's
maneuver was negative. There was decreased pain sensation in the left little finger and the ulnar
half of the ring finger with equivocal decreased pain sensation in the ulnar aspect of the dorsum of
the hand. All reflexes were mildly depressed.
Motor nerve studies

Nerve and Site Latency (ms) Amplitude (mV) Conduction Velocity (m/s)
Median nerve L. Normal  4.2 Normal  6 Normal  50
Wrist 4.7 11 -
Elbow 9.8 10 53
Ulnar nerve L. Normal  3.6 Normal  8 Normal  50
Wrist 3.5 3 -
Below elbow 8.3 3 57
Above elbow 11.4 1 39
Axilla 13.2 1 67
Peroneal nerve L. Normal  5.7 Normal  3 Normal  40
Ankle 5.2 4 -
Fibular head 12.3 4 44
Knee 14.3 4 50
Median nerve R. Normal  4.2 Normal  6 Normal  50
Wrist 4.3 15 -
Elbow 9.3 14 54
Ulnar nerve R. Normal  3.6 Normal  8 Normal  50
Wrist 2.7 10 -
Below elbow 7.2 10 58
Above elbow 9.6 10 50

F-wave studies

Nerve Latency (ms) Normal Latency  (ms)


Median nerve L. 30.7 30
Ulnar nerve L. 33.0 30
Peroneal nerve L. 54.9 54
Median nerve R. 30.8 30
Ulnar nerve R. 32.1 30

Neurosurgery Books Full


www.ketabpezeshki.com 66485438-66485457
492 PART VI PERIPHERAL NERVE SURGERY

Inching technique: ulnar nerve L.

Stimulation Site Latency (ms) Amplitude (mV)


4 cm below elbow 8.1 3.3
3 cm below elbow 8.3 3.1
2 cm below elbow 9.0* 1.4
1 cm below elbow 9.7* 1.3
Ulnar groove 9.8 1.3
1 cm above elbow 9.9 1.3
2 cm above elbow 10.0 1.1
3 cm above elbow 10.1 1.1

*0.2 ms difference allowed between stimulation sites.

Sensory nerve studies

Normal Normal
Onset Onset Peak Peak Normal Conduction Normal
Latency Latency Latency Latency Amp Amp ≥ Velocity Conduction
Nerve (ms) (ms) (ms) (ms) (μV) (μV) (m/s) Velocity ≥ (m/s)
Median 3.2 2.6 3.7 3.1 11 20 41 50
nerve L.
Ulnar nerve L. NR 2.6 NR 3.1 NR 13 NR 50
Dorsal ulnar NR * NR 2.3 NR 12 NR *
cut. L.
Sural nerve L. 2.9 3.5 3.4 4.0 14 11 48 40
Median 2.6 2.6 3.1 3.1 9 20 50 50
nerve R.
Ulnar nerve R. 2.1 2.6 2.6 3.1 5 13 57 50
Dorsal ulnar 1.5 * 2.0 2.3 12 12 53 *
cut. R.

*Normal data not available.

EMG data

Insert Pos
Muscle Activity Fibs Waves Fasc Amp Dur Poly Pattern
Biceps brachii L. Norm None None None Norm Norm Few Full
Triceps L. Norm None None None Lg Inc None Full
Flexor carpi radialis L. Norm None None None Norm Norm None Full
Flexor carpi ulnaris L. Norm None None None Norm Norm None Full
1st dorsal interosseous L. Inc 2+ 2+ None Lg Inc None Red
Abductor pollicis brevis L. Norm None None None Norm Norm None Full
Flexor digitorum profundus L. Inc 2+ 2+ None Lg Inc None Red
Vastus lateralis L. Inc None 2+ None Lg Inc None Red
Tibialis anterior L. Norm None None None Lg Inc None Red
Gastrocnemius L. Inc None 1+ None Lg Inc None Full

Results from Bertorini TE. Neuromuscular Case Studies, 1st ed., Butterworth-Heinemann, Elsevier, Case 7, 2008.

Neurosurgery Books Full


www.ketabpezeshki.com 66485438-66485457
37 PERIPHERAL NERVE 493

Which one of the following explains his acute symptoms?


a. Bilateral carpal tunnel syndrome
b. Diabetic neuropathy
c. Lower trunk brachial plexopathy
d. Poliomyelitis
e. Ulnar nerve entrapment below the elbow

12. A 46-year-old woman had a 4-month history of left shoulder and arm pain, and a 3-week history of
numbness in the dorsum of the hand with some arm weakness and now wrist drop. She had severe
weakness of the brachioradialis muscle, wrist, and finger extensors on the left. Strength of the
triceps muscle was difficult to evaluate because of severe pain. The left brachioradialis reflex was
absent, and ankle reflexes were diminished bilaterally; other reflexes were normal. There was
decreased sensation in the left hand first web space. The remainder of her neurologic examination
was normal.
Motor nerve studies

Nerve and Site Latency (ms) Amplitude (mV) Conduction Velocity (m/s)
Median Nerve L. Normal  4.2 Normal  6 Normal  50
Wrist 3.5 17 -
Elbow 6.8 16 61
Ulnar nerve L. Normal  3.6 Normal  8 Normal  50
Wrist 3.0 14 -
Below elbow 6.4 14 57

F-wave studies

Nerve Latency (ms) Normal Latency  (ms)


Median nerve L. 26.8 30
Ulnar nerve L. 25.6 30

Sensory nerve studies

Normal Normal
Onset Onset Peak Peak Normal Conduction Normal
Latency Latency Latency Latency Amp Amp ≥ Velocity Conduction
Nerve (ms) (ms) (ms) (ms) (μV) (μV) (m/s) Velocity ≥ (m/s)
Median 2.1 2.6 2.6 3.1 40 20 62 50
nerve L.
Ulnar 2.1 2.6 2.6 3.1 26 13 57 50
nerve L.
Radial NR 2.6 NR 3.1 NR 30 NR 50
nerve L.
Radial 2.3 2.6 2.8 3.1 36 30 60 50
nerve R.

EMG data

Insert Pos
Muscle Activity Fibs Waves Fasc Amp Dur Poly Pattern
Cervical paraspinals L. Norm None None None Norm Norm None Full
Deltoid L. Norm None None None Norm Norm None Full
Biceps brachii L. Norm None None None Norm Norm None Full

Continued on following page


Neurosurgery Books Full
www.ketabpezeshki.com 66485438-66485457
494 PART VI PERIPHERAL NERVE SURGERY

Insert Pos
Muscle Activity Fibs Waves Fasc Amp Dur Poly Pattern
Flexor carpi radialis L. Norm None None None Norm Norm None Full
Brachioradialis L. Inc 1+ 1+ None Norm Norm Few Red
Extensor digitorum com. L. Inc 3+ 3+ None * * * *
Extensor carpi radialis L. Inc 3+ 3+ None * * * *
1st dorsal interosseous L. Norm None None None Norm Norm None Full

*No motor units recruited.


Results from Bertorini TE. Neuromuscular Case Studies, 1st ed., Butterworth-Heinemann, Elsevier, Case 9, 2008.

Abnormalities can be localized to which one of the following?


a. C7 nerve root
b. C8 nerve root
c. Median nerve
d. Radial nerve
e. Ulnar nerve

13. A 65-year-old woman developed pain in the right arm and shoulder with some neck discomfort.
A few days later, she presented with a rash on the arm and could no longer raise the arm. Examination
revealed an erythematous vesicular rash in the outer aspect of the right arm, and she had weakness of
the right deltoid of 2/5. Supraspinatus were 5 /5 and brachioradialis was 3/5. Biceps strength was
3/5; triceps, forearm, and hand muscles were normal. The left arm and leg strength were normal.
The right biceps and brachioradialis reflexes were absent; the triceps was normal. Reflexes in the legs
and left arm were normal. Sensory examination showed diffuse hypoalgesia in the right lower arm;
the affected dermatome was hard to localize. The rest of the neurologic examination was normal.
Motor nerve studies

Nerve and Site Latency (ms) Amplitude (mV) Conduction Velocity (m/s)
Median nerve R. Normal  4.2 Normal  6 Normal  50
Wrist 4.1 10 -
Elbow 7.0 10 62
Ulnar nerve R. Normal  3.7 Normal  8 Normal  50
Wrist 3.7 9 -
Below elbow 6.9 9 53
Above elbow 9.2 9 52

F-wave studies

Nerve Latency (ms) Normal Latency  (ms)


Median nerve R. 26.4 30
Ulnar nerve R. 27.6 30

Sensory nerve studies

Normal Normal
Onset Onset Peak Peak Normal Conduction Normal
Latency Latency Latency Latency Amp Amp ≥ Velocity Conduction
Nerve (ms)  (ms) (ms)  (ms) (μV) (μV) (m/s) Velocity ≥ (m/s)
Median 2.4 2.6 2.9 3.1 25 20 54 50
nerve R.
Ulnar 2.3 2.6 2.8 3.1 18 13 52 50
nerve R.

Neurosurgery Books Full


www.ketabpezeshki.com 66485438-66485457
37 PERIPHERAL NERVE 495

EMG data

Insert Pos
Muscle Activity Fibs Waves Fasc Amp Dur Poly Pattern
Cervical paraspinals R. Inc 2+ 2+ None Norm Norm Few Full
Infraspinatus R. Norm None None None Norm Norm None Full
Rhomboids R. Norm None None None Norm Norm None Full
Deltoid R. Inc 2+ 2+ None Lg Inc Few Red
Biceps brachii R. Inc 2+ 2+ None Lg Inc Few Red
Brachioradialis R. Inc 2+ 2+ None Lg Inc Few Red
Extensor digitorum Norm None None None Norm Norm None Full
comm. R.
Flexor carpi radialis R. Norm None None None Norm Norm None Full
Flexor carpi ulnaris R. Norm None None None Norm Norm None Full
First dorsal Norm None None None Norm Norm None Full
interosseous R.

Results from Bertorini TE. Neuromuscular Case Studies, 1st ed., Butterworth-Heinemann, Elsevier, Case 15, 2008.

Which one of the following is most likely?


a. C5 radiculopathy
b. C6 radiculopathy
c. Musculocutaneous mononeuritis
d. Suprascapular neuropathy
e. Upper trunk brachial plexopathy

14. A 44-year-old woman came with nearly a year long history of neck pain radiating to the right arm.
Neurologic examination showed normal mentation and cranial nerves. There was no weakness, but
there was diminished brachioradialis reflex on the right compared with the left. Sensory examina-
tion was normal. The rest of the examination was also normal. EMG findings are shown.

Motor nerve studies

Nerve and Site Latency (ms) Amplitude (mV) Conduction Velocity (m/s)
Median nerve R. Normal  4.2 Normal  6 Normal  50
Wrist 3.6 13 -
Elbow 7.3 12 59
Ulnar nerve R. Normal  3.6 Normal  8 Normal  50
Wrist 2.8 18 -
Below elbow 6.0 16 61

F-wave studies

Nerve Latency (ms) Normal Latency  (ms)


Median nerve R. 23.4 30
Ulnar nerve R. 25.2 30

Neurosurgery Books Full


www.ketabpezeshki.com 66485438-66485457
496 PART VI PERIPHERAL NERVE SURGERY

Sensory nerve studies

Normal Normal
Onset Onset Peak Peak Normal Conduction Normal
Latency Latency Latency Latency Amp Amp ≥ Velocity Conduction
Nerve (ms)  (ms) (ms)  (ms) (μV) (μV) (m/s) Velocity ≥ (m/s)
Median 2.5 2.6 3.0 3.1 34 20 52 50
nerve R.
Ulnar 2.1 2.6 2.6 3.1 33 13 57 50
nerve R.

EMG data

Insert Pos
Muscle Activity Fibs Waves Fasc Amp Dur Poly Pattern
Cervical paraspinals R. Norm None None None Norm Norm None Full
Rhomboids R. Inc None 2+ None Norm Norm None Full
Supraspinatus R. CRDs None None None Norm Norm None Full
Infraspinatus R. Inc None None None Norm Norm None Full
Deltoid R. Inc None 1+ None Norm Norm None Full
Biceps brachii R. Inc None 2+ None Norm Norm None Red
Flexor carpi radialis R. Norm None None None Norm Norm None Full
Flexor carpi ulnaris R. Norm None None None Norm Norm None Full
Extensor dig. communis R. Norm None None None Norm Norm None Full
1st dorsal interosseous R. Norm None None None Norm Norm None Full

Results from Bertorini TE. Neuromuscular Case Studies, 1st ed., Butterworth-Heinemann, Elsevier, Case 16, 2008.

Which one of the following is most likely? 21. Which one of the labels in the image below
a. Axillary nerve pathology corresponds to the innervation from the lat-
b. C5 radiculopathy eral plantar nerve?
c. C6 radiculopathy
d. Dorsal scapular nerve pathology Deep peroneal nerve
e. Musculocutaneous nerve pathology
Calcaneal branch (tibial nerve)

QUESTIONS 15–20 Lateral plantar nerve

Medial plantar nerve


Additional questions 15–20 available on
ExpertConsult.com Saphenous nerve

Superficial peroneal nerve

Sural nerve

Dorsal surface Plantar surface

Neurosurgery Books Full


www.ketabpezeshki.com 66485438-66485457
37 PERIPHERAL NERVE 497

22. Which one of the following segments is c. C6,C7


responsible for finger extension? d. C7,C8
a. C4,C5 e. C8,T1
b. C5,C6

EXTENDED MATCHING ITEM (EMI) QUESTIONS


23. Brachial plexus:

15

1 16

2
3
17
4

5
18 19
6
20
7 21

8 22
9
10 23

25 24
11
26
12

27
13 28

14 29

For each of the following descriptions, select the most appropriate answers from the list above.
Each answer may be used once, more than once or not at all.
a. Long thoracic nerve d. Suprascapular nerve
b. Musculocutaneous nerve e. Ulnar nerve
c. Radial nerve

Neurosurgery Books Full


www.ketabpezeshki.com 66485438-66485457
498 PART VI PERIPHERAL NERVE SURGERY

24. Lumbar plexus: For each of the following descriptions, select the
most appropriate answers from the list above.
A
T12 Each answer may be used once, more than once
or not at all.
L1 a. Common peroneal part of sciatic nerve
B b. Nerve to piriformis
c. Inferior gluteal nerve
C L2
d. Pudendal nerve
D Ventral e. Tibial part of sciatic nerve
rami of
E L3 spinal nn.
26. Upper limb nerve injuries:
a. Anterior interosseous nerve syndrome
L4
b. Carpal tunnel syndrome
c. Cheiralgia paresthetica
d. Cubital tunnel syndrome
F L5
e. Erb-Duchenne palsy
f. Guyon's canal syndrome
G g. Klumpke's palsy
h. Pronator syndrome
H i. Radial tunnel syndrome
j. Saturday night palsy
k. Supinator syndrome
For each of the following descriptions, select the l. Thoracic outlet syndrome
most appropriate answers from the list above.
Each answer may be used once, more than once For each of the following descriptions, select the
or not at all. most appropriate answers from the list above.
1. Femoral nerve Each answer may be used once, more than once
2. Genitofemoral nerve or not at all.
3. Lateral cutaneous nerve of the thigh 1. Weakness of elbow extension, wrist extension
(wrist drop), finger extension and sensory loss
25. Sacral plexus: in 1st web space.
2. Weakness in FPL, pronator quadratus and
Anterior rami FDP (Digits 2 + 3); Abnormal pinch sign; no
L4 sensory loss.
Dorsal divisions
Ventral divisions 3. Sensory loss in palmar ulnar 1½ fingers and
hand (dorsal sensory branch arises before
1 L5 wrist); weakness and wasting of hypothenar,
all interossei, lumbricals 3 + 4, deep head of
S1
FPB, adductor pollicis. Froment's sign due
2 Sacral plexus to weak thumb adduction (flexes IPJ instead).
Clawing of 4 + 5th digits when attempting to
3
S2 extend fingers.
11
S3
27. Lower limb nerve entrapment:
4
a. Anterior tarsal tunnel syndrome
S4
b. Deep peroneal nerve entrapment
5 c. Distal tarsal tunnel syndrome
S5 Coccygeal plexus d. Exertional compartment syndrome
6
Co e. Femoral nerve compression syndrome
7
f. Fibular tunnel syndrome
8 12
g. Meralgia paresthetica
9 13 h. Obturator syndrome
10 14 i. Piriformis syndrome
16
15 j. Proximal tarsal tunnel syndrome

Neurosurgery Books Full


www.ketabpezeshki.com 66485438-66485457
37 PERIPHERAL NERVE 499

For each of the following descriptions, select the 29. Innervation of the upper limb:
most appropriate answers from the list above.
Anterior Posterior
Each answer may be used once, more than once
or not at all.
1. Entrapment of lateral cutaneous nerve of thigh
(femoral n) under inguinal ligament. Sensory
loss, burning dysesthesias in anterolateral
thigh; no motor weakness.
2. Sensory loss anteromedial thigh; weakness and
wasting of quadriceps femoris; point tender- A
ness in groin; impaired knee jerk. Pain and B
numbness in saphenous nerve distribution C
(anterior knee, medial leg). D
3. Pain and parasthesias in toes and sole of E
foot (heel spared as sensory branches arise F
proximal to tunnel), clawing of toes due to G
H
weakness of intrinsic foot muscles, typically
I
worse at night. Tinel's test positive. Ankle J
eversion+ dorsiflexion combined with toe K
dorsiflexion can reproduce pain (dorsiflexion- L
eversion test).

28. Dermatomes:
a. C4
b. C5
c. C6
d. C7
e. C8 For each of the following descriptions, select the
f. T1 most appropriate answers from the list above.
g. L2 Each answer may be used once, more than once
h. L3 or not at all.
i. L4 1. Ulnar nerve
j. L5 2. Median nerve
k. S1 3. Lateral cutaneous nerve of forearm
l. S2 4. Superficial branch of radial
m. S3
30. Myotomes:
For each of the following descriptions, select the a. C3, C4
most appropriate answers from the list above. b. C5, C6
Each answer may be used once, more than once c. C6, C7
or not at all. d. C7, C8
1. Obliquely from lateral thigh, lateral aspect of e. C8, T1
knee/calf, anterior shin and dorsum of foot f. L1, L2
including medial four toes and sole of foot g. L2, L3
including heel. h. L3, L4
2. Medial arm, forearm and dorsal and palmar i. L4, L5
aspects of hand and ring + little fingers. j. L5, S1
3. Lateral shoulder, arm, forearm, anatomical k. S1, S2
snuff box, thenar eminence and whole thumb.

Neurosurgery Books Full


www.ketabpezeshki.com 66485438-66485457
500 PART VI PERIPHERAL NERVE SURGERY

For each of the following descriptions, select the 32. Innervation of the lower limb:
most appropriate answers from the list above.
Each answer may be used once, more than once
or not at all. 1
13
12

1. Wrist extension 2 14
2. Hip flexion 3 15
3. Ankle dorsiflexion
4
31. Clinical signs: 5
16
a. Ape hand 6 17
b. Claw hand
18
c. Froment's sign
19
d. Hand of benediction
e. Hoffman's sign
f. Hoover test 20
7
g. Inverted radial reflex 21
h. Lasegue's sign 8

i. Lhermitte's sign
j. Phalen's test 9
k. Spurling's sign 10 22
l. Tinel's test 11
23
m. Volkman's contracture

For each of the following descriptions, select the


most appropriate answers from the list above. For each of the following descriptions, select the
Each answer may be used once, more than once most appropriate answers from the list above.
or not at all. Each answer may be used once, more than once
1. Diminished brachioradialis reflex with reflex or not at all.
contraction of finger flexors. a. Saphenous nerve
2. On attempting to make a fist, only 4th and 5th b. Sural nerve
digits flex at IPJs. c. Obturator
3. Loss of thumb opposition and abduction.

SBA ANSWERS
1. c—Type A delta

Diameter Conduction
(μm) Myelination Velocity (m/s) Modality
Type A alpha 13-20 Myelinated 70-120 Motor
Type A beta 6-12 Myelinated 30-70 Touch and pressure
Type A 3-6 Myelinated 15-30 Proprioception
gamma
Type A delta 1-6 Myelinated 12-30 Fast pain and cold temperature
Type B Myelinated 3-15 Autonomic, preganglionic sympathetic
Type C 0.5-1.5 Unmyelinated 0.5-2 Slow pain and warm temperature, autonomic,
postganglionic sympathetic, polymodal
receptors

Neurosurgery Books Full


www.ketabpezeshki.com 66485438-66485457
37 PERIPHERAL NERVE 501

2. d—Insertional activity on EMG (in a pattern obscure the baseline (termed a full interference
excluding other root lesions) pattern). Incomplete interference pattern is con-
sidered to be a reflection of loss of motor units in
Over 1-2 weeks denervated muscle fibers a muscle, though it can also be seen with dimin-
becomes progressively more mechanically irrita- ished effort. The table below shows the sequence
ble such that electrical discharges provoked by of EMG changes in radiculopathy:
movement of the needle can be prolonged
(increased insertional activity). Muscle fibers also
become chemically sensitive to their microenvi- Days After Onset Electrophysiological
ronment and their membranes can also become of Radiculopathy Abnormalities
unstable enough to produce spontaneously activ- >0 Reduced number of MUPs
ity (fibrillation potentials and positive sharp (reduced interference pattern)
waves; disappear with complete degeneration of Fasciculations
the denervated muscle fiber). The finding of H-reflex prolonged latency
Reduced F waves
fibrillations and positive sharp waves is the most
reliable and objective test that there is for damage >1 week Fibrillation potentials and
to motor axons to the muscle after 1 week at least positive sharp spikes in
paraspinal muscles
up to 12 months after the damage. If there is
ongoing damage such as in amyotrophic lateral >2 weeks Fibrillation potentials and
positive sharp spikes in
sclerosis one can see ongoing denervation. Rein- proximal limb muscles
nervation of muscle is an ongoing process, occur-
ring whenever a muscle is partially denervated. >3 weeks Fibrillation potentials and
positive sharp spikes in distal
This process typically involves the development limb muscles
of sprouts from adjacent, unaffected motor nerve
fibers that ultimately contact at least some of the
denervated muscle fibers. These reinnervated
muscle fibers cluster right in the area of other,
normally innervated muscle fibers. This process 6. e—MUP is the sum of the electrical signals
results in the development of clumps of reinner- arising from the discharge of the several mus-
vated muscle fibers attached to individual motor cle fibers within recording distance of the tip
neurons, producing larger motor units more of the needle that innervated by the same
irregular potentials (polyphasic).This process motor neuron. The amplitude of the MUP
takes months to develop and indicates the pres- is dependent on the density of the muscle
ence of chronic denervation. fibers attached to that one motor neuron
(also to the proximity of the MUP). As the
3. c—Pronator teres (C6, C7) degree of contraction is slowly increased,
more motor units are recruited.
The typical needle EMG examination requires
sampling several muscles. Its ability to localize a 7. d—Multiple motor root compromise
lesion depends on sampling muscles innervated
by the same nerve but different nerve roots, mus- The F-wave (originally recorded in the foot,
cles innervated by the same nerve root but differ- hence the name) is a late response that occurs
ent nerves and muscles innervated at different in muscles during a motor nerve conduction
locations along the course of the nerves. Para- study long after the initial contraction of the mus-
spinal muscles can be very useful in this regard cle (CMAP). CMAP usually appears within sev-
because nerve root damage will tend to produce eral milliseconds but another response can be
abnormalities in these muscles as well as within normally recorded in the muscle slightly later
the muscles of the limbs (helping to distinguish (25-55 ms). The electrical impulse is transmitted
a radiculopathy from a plexopathy or peripheral proximally along the motor axon from the site of
neuropathy, for example). Sometimes precise initiation of the action potential. When this anti-
localization can be difficult due to the overlap dromic depolarization reaches the motor neurons
in innervation of the various nerve root levels. in the spinal cord, a percentage of these motor
neurons are activated a second time. This results
4. b—Motor axon loss in an electrical signal being conducted in the nor-
mal (orthodromic) direction from the spinal cord
5. e—Reduced interference pattern to the muscles innervated by the nerve. This sec-
ond, later activation produces a small muscle con-
During EMG assessment of a muscle during con- traction that is termed the F-wave. Because the
traction, the electrical activity should fully number of motor neurons that are re-activated

Neurosurgery Books Full


www.ketabpezeshki.com 66485438-66485457
502 PART VI PERIPHERAL NERVE SURGERY

is somewhat unpredictable, the amplitude of this axon all the way back to the spinal cord before
signal is variable and, therefore, amplitude mea- synapsing on the motor neuron, and since the
surements are usually not used. However, delay motor response must then traverse the length
in the F-wave indicates some slowing of conduc- of the motor axon to reach the triceps surae
tion of the motor axon. Since the F-response muscle, this reflex takes a long time (i.e., late).
traverses more proximal portions of the motor Theoretically, this reflex can be elicited from vir-
axons (twice) it may be useful in the investigation tually any muscle but only the triceps surae mus-
of proximal nerve pathology. Since the anti- cle produces H-reflexes that are reliable enough
dromic impulse in motor axons in a single periph- to be clinically useful. Therefore H-reflex evalu-
eral nerve will test the multiple nerve roots ates the integrity of the reflex arc from the tibial
forming it, F-wave is not useful for isolated nerve, sciatic nerve, S1 sensory root, spinal cord,
radiculopathy but is valuable where multiple S1 motor root and back to the triceps surae.
roots may be involved (e.g., Guillain-Barre syn- Damage to any portion of the reflex arc can
drome, or chronic inflammatory demyelinating result in loss or slowing of the reflex response.
polyradiculopathy). Since the H-reflex is mediated primarily over
the S1 nerve root (just like the ankle jerk reflex),
8. e—Suspected S1 radiculopathy it is a sensitive test for S1 radiculopathy. How-
ever, once the reflex arc has been damaged, it
H-reflex is named in honor of Hoffmann, who often does not return to normal (making the test
first described this response in 1918. The less useful in investigating the question of recur-
H-reflex is most commonly tested by electrical rent radiculopathy).
stimulation of the tibial nerve, with recordings
from the gastrocnemius/soleus muscle complex 9. e—Reduced conduction velocity (or
(triceps surae). Therefore, this response utilizes conduction block)
the same neural pathway as the ankle jerk reflex.
Electrical stimulation will depolarize the largest, The table below shows general patterns of
most heavily myelinated nerve fibers at a lower peripheral neuropathies due to demyelination
stimulus intensity than is required to activate versus those due to axonal degeneration, though
other smaller nerve fibers. Since the largest in reality each can cause secondary damage to
nerve fibers in a peripheral nerve are those aris- the other and electrophysiology may be mixed.
ing from muscle stretch receptors, there should Pathology which may affect both myelin and
be a stimulus intensity that activates muscle axons equally include diabetes, uremia and para-
stretch afferent nerve fibers without directly acti- proteinemia. Radiculopathies (root lesions) and
vating many motor nerve axons eliciting a mono- neuropathies (e.g., MND, herpes zoster) are
synaptic reflex contraction in the muscle. not included in this table, although they may
Because this response must traverse the sensory mimic peripheral neuropathy.

Focal Multifocal
EMG/NCS (Mononeuropathy) (Mononeuritis Multiplex) Generalized (Polyneuropathy)
Demyelination Nerve entrapment Paraproteinemia Guillain-Barre syndrome
Diphtheria CIPD
Leprosy Lymphoma
Multiple myeloma
Amiodarone
Hereditary
Axonal Severe nerve Diabetes Diabetes
degeneration entrapment Vasculitis Alcohol
Neoplastic Drugs/toxin
HIV Critical illness
Sarcoidosis Multiple myeloma
Amyloidosis Hereditary
Lyme disease

Neurosurgery Books Full


www.ketabpezeshki.com 66485438-66485457
37 PERIPHERAL NERVE 503

10. c—Carpal tunnel syndrome 12. d—Radial nerve

It showed prolonged median nerve distal motor The left median and ulnar nerve SNAPs conduc-
latency on the right with low-amplitude com- tion velocities and amplitudes were normal,
pound muscle action potential (CMAP). The whereas the superficial radial SNAP was absent.
elbow to wrist conduction velocity was normal. Radial nerve motor conduction could not be stud-
The right ulnar conduction velocity, CMAP ied because of discomfort. The needle test
amplitude, and distal latency were normal. The showed denervation potentials in radial-
right median sensory nerve action potential innervated muscles, including wrist extensor
(SNAP) had a prolonged latency and slow conduc- and the brachioradialis, indicating that the lesion
tion velocity. The left median CMAP distal occurred above the elbow. The normal median
latency was mildly prolonged and of normal and ulnar-innervated muscles negate a C7 radicu-
amplitude; the conduction velocity was normal. lopathy or a lesion of the middle trunk of the bra-
The left median SNAP was prolonged and had chial plexus. The normal EMG of her deltoid is
slow conduction velocity. The ulnar SNAP was evidence against a posterior cord lesion. Denerva-
normal. The F-response on the right median tion in the brachioradialis and extensor carpi
nerve was prolonged, likely secondary to the pro- radialis and the absent radial SNAP indicate that
longed distal motor latency. The needle test the main trunk of the radial nerve was affected
showed large motor units potentials only in the with axonal degeneration.
abductor pollicis brevis muscle. There was thus
distal median nerve demyelination and chronic 13. b—C6 radiculopathy
axonal degeneration in the right, causing a low-
amplitude CMAP and large motor unit potentials The sensory and motor nerve conduction studies
with reduced recruitment. It was concluded that were normal. The needle test showed denerva-
this patient had carpal tunnel syndrome from tion potentials with reduced recruitment of
RA and likely also from a compression from the motor units in the deltoid, biceps, brachioradialis,
use of a walker. and cervical paraspinal muscles, indicating axonal
degeneration of the C6 roots sparing the infraspi-
11. e—Ulnar nerve entrapment below the elbow natus and rhomboids, as well as forearm and
intrinsic hand muscles.
Absent were the left digital ulnar and dorsal cuta-
neous SNAPs. The motor ulnar nerve conduction 14. a—C5 radiculopathy
velocity from elbow to wrist was normal but was
slow at 39 m/s across the elbow. Conduction The median and ulnar motor conduction veloci-
velocity of less than 50 m/s, or 10 m/s slower than ties, distal latencies, compound muscle action
the elbow-wrist segment, or 10 m/s slower than potential amplitudes, F-responses, and sensory
the velocity across the elbow in the opposite side, nerve action potentials were normal. Needle
are considered abnormal. The “inching,” or short EMG showed denervation potentials in the right
(1 cm) segment increment study, revealed a signif- biceps, deltoid, and rhomboids, and complex
icant prolongation of 0.7 ms (normal, 0.5 ms) at repetitive discharges in the supraspinatus muscle.
2 cm distal to the medial epicondyle where there The motor units appeared normal in these mus-
was also a drop in amplitude over 50%. The needle cles. Paraspinal muscles were normal. This EMG
EMG showed denervation potentials in the left was suggestive of a C5 radiculopathy (despite no
ulnar-innervated intrinsic hand muscles, but not paraspinal denervation). This was concluded
in the median-innervated muscles. There were because there was involvement of muscles inner-
also denervation potentials in the flexor digitorum vated only by C5; in particular, the rhomboid that
profundus although not in the flexor carpi ulnaris. is innervated by the dorsal scapular nerve that
The test also showed electrophysiologic evidence originates directly from the C5 root and not the
of bilateral median neuropathy at the wrist. In brachial plexus.
addition, there were electrophysiologic findings
consistent with the previous history of polio, char-
acterized by large motor unit potentials in the
lower extremities with mild active denervation in
ANSWERS 15–20
the gastrocnemius muscle.
Additional answers 15–20 available on
ExpertConsult.com

Neurosurgery Books Full


www.ketabpezeshki.com 66485438-66485457
504 PART VI PERIPHERAL NERVE SURGERY

21. c—appears to be superficial peroneal, query E?


Image with permission from Sen CK, Roy S. Wound healing. In: Neligan P (ed.), Plastic Surgery, 6 volumes, 3rd ed.,
Elsevier, 2013.

22. d—C7/C8 roots

Joint Action Segments Major Muscle Nerve Reflex


Scapula Elevation C3,4 Upper trapezius Spinal acc. (XI)
Levator scapulae Dorsal scapular
Shoulder Flexion C5,6 Deltoid Axillary
Pectoral Pectoral
Coracobrachialis Musculocut.
Extension C5,6 Deltoid Axillary
Teres minor Suprascapular
Infraspinatus Lower subscap.
Teres major Thoracodorsal
Lat. dorsi
Adduction C5,6 Pectoral
Lat dorsi
Coracobrachialis
Abduction C5,6 Deltoid
Supraspinatus
Internal rotation C5,6 Subscapularis
Teres major
Lat dorsi
Anterior deltoid
External C5,6 Infraspinatus Suprascapular
rotation Posterior deltoid Axillary
Teres minor
Elbow Extension C6,7 Triceps Radial Triceps
Flexion C5,6 Biceps brachii Musculocutaneous Biceps
Brachialis Brachioradialis
Brachioradialis
Wrist Flexion C6,7 FCR, PL Median
FCU Ulnar
Extension C6,7 ECRL Radial
ECRB, ECU PIN
Finger Flexion C8,T1 FDS,FDP,FPL Ulnar/median
Extension C7,8 ED, EI, EPL Posterior interosseous
Abduction C8,T1 APB, dorsal interossei Median/ulnar
Adduction C8,T1 Palmar interossei Ulnar

Neurosurgery Books Full


www.ketabpezeshki.com 66485438-66485457
37 PERIPHERAL NERVE 505

EMI ANSWERS
23. a—20, b—8, c—10, d—3, e—12

From C4

Dorsal scapular nerve C5

To phrenic nerve
Suprascapular nerve
C6
Nerve to subclavius

Lateral pectoral nerve


C7 To scaleni
Lateral cord
Long thoracic nerve
Posterior cord C8

Musculocutaneous nerve T1
Axillary nerve
Radial nerve From T2

First intercostal nerve


Medial pectoral nerve
Median nerve
Medial cord
Ulnar nerve

Upper subscapular nerve


Medial cutaneousnerve Thoracodorsal nerve
of forearm
Medial cutaneous nerve Lower subscapular nerve
of arm
Image with permission from Mancall, Elliott L. Gray's Clinical Neuroanatomy: The Anatomic Basis for Clinical Neu-
roscience, Elsevier, Saunders, 2011.

Important Motor
Major Nerves Roots Cord Supply Motor Test Sensory Supply
Axillary C5-6 Posterior Deltoid Shoulder abduction Superior lateral
Teres minor arm
Radial C5-T1 Posterior Triceps brachii Elbow extension Dorsal hand and
Brachioradialis Brachioradialis jerk radial 3½ digits
ECRL and ECRB Wrist extension
Posterior interosseous Finger extension
nerve
Forearm extensors:
supinator, ECU, ED,
EDM, APL, EPL, EPB,
EI.
Musculocutaneous C5-7 Lateral Coracobrachialis, Elbow flexion Lateral forearm
biceps, brachioradialis

Continued on following page

Neurosurgery Books Full


www.ketabpezeshki.com 66485438-66485457
506 PART VI PERIPHERAL NERVE SURGERY

Important Motor
Major Nerves Roots Cord Supply Motor Test Sensory Supply
Median C6-T1 Lateral Forearm flexors (PT, Forearm pronation Palmar aspect of
+Medial FDS, PL, FCR) Wrist flexion radial 3½ digits
Thenar (Lumbricals 1 PIPJ flexion and nailbeds
+ 2, opponens pollicis, PIPJ extension
abductor pollicis, FPB) (index, middle)
Anterior interosseous DIPJ flexion (index,
nerve middle)
Forearm flexors: Thumb abduction,
median FDP, FPL, opposition; Pinch
pronator quadratus sign;
Ulnar C7-T1 Medial FCU Wrist flexion and Palmar and
FDP (ulnar: ring and adduction, little dorsal aspects of
little) finger abduction, ulnar 1½ fingers
Adductor pollicis MCPJ flexion (little
Hypothenar: ADM, finger), DIPJ flexion
ODM, FDM (little, ring fingers)
Dorsal and palmar finger adduction,
interossei, lumbricals Froment's sign
3+4

24. 1—f, 2—d, 3—e

T12
Subcostal nerve (T12)

L1
Iliohypogastric nerve

Ilioinguinal nerve L2

Genitofemoral nerve Ventral


Lateral femoral cutaneous rami of
nerve L3 spinal nn.

L4

Femoral nerve L5

Obturator nerve

Lumbosacral trunk
Image with permission from Waldman SD. Pain Review, Elsevier, Saunders, 2009.

Major Nerves Important Motor Supply Sensory


Iliohypogastric (L1) Posterolateral buttock
Ilioinguinal (L1) Inguinal and suprapubic
Genitofemoral (L1-2) Genital branch to Femoral triangle
cremaster muscle
Lateral cutaneous nerve of - Anterolateral thigh
thigh (L2-3)
Femoral nerve (L2-4) Iliacus and quadriceps Anterior thigh; saphenous nerve branch supplies
medial leg/foot
Obturator nerve (L2-4) Motor to thigh adductors Medial thigh

Neurosurgery Books Full


www.ketabpezeshki.com 66485438-66485457
37 PERIPHERAL NERVE 507

25. a—5, b—4, c—3, d—14, e—7


Anterior rami
L4
Dorsal divisions
Ventral divisions

Lumbosacral trunk L5

S1
Superior gluteal Sacral plexus

S2
Inferior gluteal
Pelvic splanchnic nerve
To piriformis muscle S3

S4
Sciatic nerve
S5 Coccygeal plexus
Common fibular part
Co
Tibial part
To quadratus femoris and inferior Anococcygeal nerves
gemelius muscles
To obturator internus muscles To levator ani, coccygues, and external anal sphincter muscles
Posterior femoral cutaneous nerve Pudendal nerve
Pelvic splanchnic nerves
Perforating cutaneous nerve
Image with permission from Drake RL, Vogl AW, Mitchell A, Tibbitts R, Richardson P (Eds.), Gray's Atlas of Anatomy,
2nd ed., Elsevier, Churchill Livingstone. Copyright 2015.

Major Nerves Important Motor Supply Motor Test Sensory Supply


Superior gluteal Gluteus medius Hip abduction;
Gluteus minimus Trendelenburg
Tensor fascia lata sign
Inferior gluteal Gluteus maximus Hip extension
Sciatic Nerve Hamstrings Knee flexion
(L4-S3) Adductor magnus (medial)
Tibial nerve Gastrocneumius/soleus Ankle Medial sural cutaneous
(L4-S3) Tibialis posterior plantarflexion Lower posterolateral calf
Flexor digitorum longus Ankle inversion Lateral aspect of foot
Flexor halluces longus Medial and lateral calcaneal
Medial and lateral plantar nerves Heel of foot
Intrinsic foot muscles Medial plantar
Digits 1-3 and medial sole of foot
Lateral plantar
4th + 5th digits and lateral sole
of foot

Continued on following page

Neurosurgery Books Full


www.ketabpezeshki.com 66485438-66485457
508 PART VI PERIPHERAL NERVE SURGERY

Major Nerves Important Motor Supply Motor Test Sensory Supply


Common Superficial peroneal Lateral sural cutaneous
peroneal nerve Peroneus longus Upper posterolateral calf
(L4-S2) Peroneus brevis Superficial peroneal
Deep peroneal Dorsum of foot and medial 3 toes
Tibialis anterior Deep peroneal
EDL/EDB Space between 1st and 2nd toes
EHL/EDB
Peroneus tertius
Pudendal nerve Inferior rectal nerve Inferior rectal nerve
(S2-4) External anal sphincter Below pectinate line
Perineal nerve Perineal nerve
External urethral sphincter, Posterior scrotum
bulbospongiosus, Dorsal nerve of penis/clitoris
ischiocavernosus Skin of penis/clitoris

26. 1—j, Saturday night palsy, 2—a, Anterior interosseous nerve syndrome, 3—f, Guyon's canal
syndrome

Entrapment Syndrome Key Features


Klumpke's palsy Lower brachial plexus injury—weakness and sensory loss in C8 and T1
distribution. Associated with Claw hand and Horner's syndrome
Erb-Duchenne palsy Upper brachial plexus injury—C5 and C6 weakness and sensory loss
predominantly affecting axillary nerve, musculocutaneous nerve and
suprascapular nerve territories. At rest the arm hangs by the side, medially
rotated, forearm extended and pronated resulting in “waiter's tip” posture
Radial nerve
Saturday night/crutch palsy Radial nerve compression against proximal humerus. Weakness of elbow
extension, wrist extension (wrist drop), finger extension and sensory loss in first
web space
Mid-arm Radial nerve compression in spiral groove of humerus (after branch to triceps).
Wrist drop, weakness finger extension, sensory loss first web space
Posterior interosseous nerve Compression of posterior interosseous nerve (branch of radial nerve) at the
syndrome (radial tunnel lateral intermuscular septum of the arm. Pain in dorsal aspect of upper forearm
syndrome) and when extending middle finger against resistance
Supinator syndrome Compression of posterior interosseous nerve (branch of radial nerve) at the
arcade of Frohse. Sensory loss to area of superficial radial nerve supply with
wrist drop
Cheiralgia paresthetica Compression of superficial branch of radial nerve causing pain, paresthesias,
(Handcuff neuropathy) numbness in first web space. No motor weakness
Median nerve
Pronator syndrome Median nerve entrapment in the proximal forearm. Proximal forearm pain and
tenderness; FPL and abductor pollicis brevis weakness; paresthesias in radial 3½
digits; positive Tinel's sign
Anterior interosseous nerve Entrapment of anterior interosseous nerve (branch of median nerve) causing
syndrome weakness in FPL, pronator quadratus and FDP (Digits 2 + 3); Abnormal pinch
sign; no sensory loss

Continued

Neurosurgery Books Full


www.ketabpezeshki.com 66485438-66485457
37 PERIPHERAL NERVE 509

Entrapment Syndrome Key Features


Carpal tunnel syndrome Median nerve entrapment at the wrist. Pain, paresthesias, numbness and
reduced two-point discrimination in thumb and radial 2½ fingers; thenar
weakness and wasting; positive Tinel's sign and Phalen's sign
Ulnar nerve
Cubital tunnel syndrome Ulnar nerve entrapment at the elbow. Sensory loss in ulnar 1½ fingers and ulnar
aspect of hand; weakness and wasting of ulnar intrinsic hand muscles and FDP;
positive Tinel's sign; ache in medial elbow and forearm
Guyon's canal syndrome Ulnar nerve entrapment at the wrist. Sensory loss in palmar ulnar 1½ fingers and
hand (dorsal sensory branch arises before wrist); weakness and wasting of
hypothenar, all interossei, lumbricals 3 + 4, deep head of FPB, adductor pollicis.
Froment's sign due to weak thumb adduction (flexes IPJ instead). Clawing of 4
+ 5th digits when attempting to extend fingers—MCPJ hyperextension due to
intact long extensors, loss of interossei and lumbricals 3 + 4 (normally act to flex
MCPJ, extend IPJs) means long flexors unopposed particularly in digits 4 + 5.
Clawing less pronounced with higher ulnar nerve injuries due to weakness of
ulnar half of FDP

27. 1—Meralgia paresthetica, 2—Femoral nerve entrapment, 3—Proximal tarsal tunnel syndrome

Meralgia paresthetica Entrapment of lateral cutaneous nerve of thigh (femoral n) under inguinal
ligament. Sensory loss, burning dysesthesias in anterolateral thigh; no motor
weakness
Piriformis syndrome Compression of sciatic nerve by piriformis muscle. Pain, tingling and sensory loss in the
buttocks and sciatic nerve distribution
Obturator syndrome Sensory loss superomedial thigh; weakness of thigh adduction
Femoral nerve Sensory loss anteromedial thigh; weakness and wasting of quadriceps femoris; point
entrapment tenderness in groin; impaired knee jerk. Pain and numbness in saphenous nerve
distribution (anterior knee, medial leg)
Fibular tunnel Common peroneal nerve entrapment/trauma, commonly at fibular head. Sensory loss
syndrome of dorsal foot and lateral leg; foot drop; Tinel's sign over fibular head; anterior and
lateral compartment atrophy
Exertional Exercise induced pain which is relieved by rest, and may be associated with weakness
compartment and paresthesia
syndrome
Distal tibial tarsal Tarsal tunnel syndrome due to entrapment of distal branches (medial and lateral
tunnel syndrome plantar) of tibial nerve. Distal entrapment of lateral plantar nerve present with chronic
heel pain (difficult to distinguish from plantar fasciitis)
Deep peroneal nerve Can occur anywhere along its course, but compression at the ankle beneath
entrapment the extensor retinaculum is termed anterior tarsal tunnel syndrome. Pain
over dorsum of foot (possibly first web space); if proximal lesion foot drop or EHL
weakness
Proximal tarsal tunnel Entrapment of the tibial nerve anywhere along its course, but often seen in the tarsal
syndrome tunnel (posterior and inferior to medial malleolus). Pain and paresthesias in toes and
sole of foot (heel spared as sensory branches arise proximal to tunnel), clawing of toes
due to weakness of intrinsic foot muscles, typically worse at night. Tinel's test positive.
Ankle eversion + dorsiflexion combined with toe dorsiflexion can reproduce pain
(dorsiflexion-eversion test)

Neurosurgery Books Full


www.ketabpezeshki.com 66485438-66485457
510 PART VI PERIPHERAL NERVE SURGERY

28. 1—j, L5, 2—e, C8, 3—c, C6 Spinal


Nerve Dermatome
C2
C2
T12 Suprapubic level
C3
C4 L1 Inguinal and pubic levels
C5
C4 C6
C5
T1
C7
T1 C8
L2 Obliquely from upper lateral thigh to
T2
T2
T3
T3 upper medial thigh (borders S2
T4
T4
T5 T6
T5 dermatome posterior thigh)
T6 T7
T8
T7
T8 T10
T9
C8
L3 Obliquely from upper lateral thigh to lower
T11 C7
T9
T10
T12
L1 medial thigh and medial aspect of knee
T11 L2
L3
T12 L4
L5
L4 Obliquely from lateral thigh, anterior
L1
C6
C8 S2 S2
C6 aspect of knee, medial aspect of calf and
C7
L2 C8 C7
medial side of foot
L3
S1
L5 Obliquely from lateral thigh, lateral aspect
of knee/calf, anterior shin and dorsum of
foot including medial 4 toes and sole of
L4
foot including heel
L5 S2
S1 S1 Posterolateral aspect of buttock, thigh,
popliteal fossa, calf and lateral aspect of
ankle (lateral malleolus) and foot
S1
S1 L5
including little toe
L5 S1 L4
S2 Posteromedial aspect of buttock, thigh,
popliteal fossa, calf and heel; scrotum
Image with permission from Standring S. Gray's Anat-
S3 Perineum; glans penis/clitoris
omy, 41st ed., Elsevier, 2015.
S4/5 Perianal (S5 is within 1cm of
mucocutaneous junction of anal canal)

Spinal
Nerve Dermatome 29. 1—l, 2—k, 3—h, 4—j
C2 Occipital
C3 Upper neck
Anterior Posterior
C4 Supraclavicular and acromioclavicular
joint
C5 Clavicles, anterolateral aspect of arm and
forearm until wrist.
C6 Lateral shoulder, arm, forearm,
anatomical snuff box, thenar eminence
and whole thumb Supraclavicular, C3,C4
Upper lateral cutaneous of arm, C5, C6
C7 Posterior arm, forearm, dorsum of hand,
Posterior cutaneous of arm, C5, C6, C7, C8
dorsal and palmar aspects of index and Intercostobrachial, T2
middle finger Medial cutaneous of arm, C8, T1
C8 Medial arm, forearm and dorsal and Posterior cutaneous of forearm, C5, C6, C7, C8
Medial cutaneous of forearm, C8, T1
palmar aspects of hand and ring + little
Lateral cutaneous of forearm, C5, C6
fingers
Lower lateral cutaneous of forearm, C5, C6
T1 Anteromedial arm and forearm until wrist Superficial branch of radial, C7, C8
Median, C6,C7,C8
T2 Chest and axilla Ulnar, C8, T1

T4 Level of nipples/fourth intercostal space


T6 Level of the xiphoid process
T8 Level half way between xiphoid process
and umbilicus Image with permission from Mancall, Elliott L. Gray's
T10 Level of umbilicus Clinical Neuroanatomy: The Anatomic Basis for Clinical
Neuroscience, Elsevier, Saunders, 2011.
Continued

Neurosurgery Books Full


www.ketabpezeshki.com 66485438-66485457
37 PERIPHERAL NERVE 511

30. 1—c, 2—f, 3—i


31. 1—g, Inverted radial reflex, 2—d, Hand of benediction, 3—a, Ape hand

Finding Key Features


Ape hand Loss of thumb opposition and abduction due to median nerve damage
Hand of On attempting to make a fist, only 4th and 5th digits flex at IPJs. Loss of flexion of digits
benediction 2 + 3 at MCPJ and IPJs due to median nerve palsy affecting lumbricals 1 + 2, median
part of FDP
Volkman's Permanent flexion contracture of the hand at the wrist resulting in clawing of hand and
contracture fingers. Usually due to ischemia of long forearm flexors which then become fibrotic and
short
Claw hand On attempting to extend fingers, digits 4 + 5 remain in clawed position (MCPJ extension,
flexed IPJs). Affects 4th and 5th digits with low ulnar nerve lesions (below mid-forearm), but
complete claw hand can be seen if both low ulnar and median nerve injury occur together.
Ulnar nerve lesions above mid-forearm do not produce clawing as there is also weakness of
ulnar half of FDP
Hoover test Aimed to distinguish organic from non-organic leg weakness using principle of
synergistic contraction. Perform by holding the heel of the normal leg while asking
patient to straight leg raise the weak leg against resistance. Normally examiner will feel
heel push down as they try to raise the weak leg—absence suggests lack of effort to
either leg
Tinel's test Percussion over an (irritated) nerve elicits a sensation of pins and needles in the distribution
of the nerve
Froment's sign While grasping a piece of paper between thumb and index finger (palm flat) as it is pulled
away, weakness of adductor pollicis (ulnar nerve) will result in compensatory flexion of
thumb PIPJ to try and hold on to it
Hoffman's reflex Tapping the nail or flicking the terminal phalanx of the middle/ring finger results in flexion of
the terminal phalanx of the thumb. Suggests cervical cord pathology
Spurling's sign Hyperextension of head and rotation towards symptomatic extremity (+/ pressing down
on patients head) reproduces radicular symptoms due to narrowing of intervertebral
foramina
Inverted radial Diminished brachioradialis reflex with reflex contraction of finger flexors. Suggests C5
reflex pathology
Phalen's test Holding wrist in complete forced flexion for up to 1 min, aiming to draw lumbricals into the
carpal tunnel and compress the median nerve to reproduce symptoms. Reverse Phalen's
test involves forced extension (prayer position)
Lasegue's sign Straight leg raising test is considered positive if pain or paresthesia occur in a radicular
distribution at less than 60° of elevation. Lowering the leg and dorsiflexing the ankle will
exacerbate symptoms. Allowing the foot to rest on the table by flexing the knee will reduce
pain (bowstring sign). Most specific for L5 or S1 root compression
Fajersztajn sign Crossed straight leg raising test is usually positive with a large central disc protrusion.
Raising the unaffected leg with patient supine produces radicular pain in the affected
extremity
Femoral stretch Useful for distinguishing between sciatica involving L2 and L3 nerves and those involving L4
test to S1. With the patient lying prone, the knee is passively flexed and the hip passively
extended to elicit thigh pain
Pinch sign Patient attempts to forcefully pinch the tips of index finger and thumb together to make an
“OK” sign but AIN palsy causing weakness of FDP to digits 2 + 3 and FPL results in extension
of terminal phalanges. As a result finger pulp rather than tips touch
Wartenberg's Abducted little finger at rest due to weakness of 3rd palmar interosseous muscle in ulnar
sign nerve palsy

Neurosurgery Books Full


www.ketabpezeshki.com 66485438-66485457
512 PART VI PERIPHERAL NERVE SURGERY

32. a—8 & 21, b—10 & 22, c—5 & 17

Subcostal, T12 Lliohypogastric L1


Subcostal, T12
Femoral branch of genitofemoral L1, L2 Dorsal rami, L1, L2, L3
Llioinguinal, L1 Dorsal rami, S1, S2, S3

Lateral cutaneous of thigh, L2, L3


Obturator, L2, L3, L4
Lateral cutaneous of thigh L2, L3,
Medial and intermediate cutaneous of thigh, L2, L3 Obturator, L2, L3, L4

Medial cutaneous of thigh, L2, L3


Posterior cutaneous of thigh, S1, S2, S3

Lateral cutaneous of calf, L4, L5, S1


Lateral cutaneous of calf, L5, S1, S2
Saphenous L3, L4
Saphenous, L3, L4

Superficial peroneal L4, L4, S1

Sural, S1, S2 Sural, L5, S1, S2


Deep peroneal
Medial calcaneal branches of tibial, S1, S2

Image with permission from Mancall, Elliott L. Gray's Clinical Neuroanatomy: The Anatomic Basis for Clinical Neu-
roscience, Elsevier, Saunders, 2011.

Neurosurgery Books Full


www.ketabpezeshki.com 66485438-66485457
PART VII
PEDIATRIC NEUROSURGERY
CHAPTER 38

PEDIATRIC NEUROSURGERY:
GENERAL PRINCIPLES AND
NORMAL DEVELOPMENT
SINGLE BEST ANSWER (SBA) QUESTIONS
1. Development of which one of the following d. Microcephaly is due to postnatal cause
domains has the best predictive value of e. Isolated microcephaly
future intelligence?
a. Gross motor 4. A 1-year-old infant has a head circumference in
b. Adaptive the 98th centile for age, he is at 75th centile for
c. Visuomotor weight and height. At birth, his head
d. Language circumference was just above the 95th centile.
e. Social His father and mother have large heads. The
child has achieved appropriate developmental
2. Which one of the following combinations of milestones for age, and normal examination.
birthweight, height, and head circumference Which one of the following is most accurate?
most likely represent normality? a. He needs cranial ultrasound to rule out
a. Weight 3500 g, height 50 cm, head hydrocephalus
circumference 35 cm b. He is likely to have subtle cerebral anomalies
b. Weight 3000 g, height 40 cm, head c. He is at risk for learning disabilities later
circumference 30 cm in life
c. Weight 2500 g, height 50 cm, head d. He probably has familial macrocephaly
circumference 40 cm and needs no further workup
d. Weight 2000 g, height 50 cm, head e. He is at risk for craniosynostosis
circumference 25 cm
e. Weight 1500 g, height 30 cm, head 5. You are involved in a trauma call for a 2-year-
circumference 20 cm old child fallen from a climbing frame and
sustained a head injury. Hypotensive, and
3. A neonate born at term has a weight of 3400 g, you are asked to prescribe a fluid bolus for
a length of 50 cm and a head circumference of him (he weighs 30 kg). What volume of fluid
30 cm. There are no abnormal neurological would you prescribe as per ATLS protocol?
findings or syndromic features. Which one a. 600 ml
of the following statements regarding the b. 150 ml
child's apparent microcephaly is true? c. 300 ml
a. He has a secondary microcephaly d. 450 ml
b. Microcephaly is due to the fact they are e. 900 ml
small for gestational age
c. Cranial ultrasound should be performed 6. You are asked to prescribe maintenance
as first choice for microcephaly fluids for an infant weighing 10 kg. Which

513
Neurosurgery Books Full
www.ketabpezeshki.com 66485438-66485457
514 PART VII PEDIATRIC NEUROSURGERY

one of the following volumes of maintenance a. A single examination and confirmatory


fluid should they receive over 24 h? ancillary test are sufficient to make a
a. 1000 ml diagnosis of brain death in most children
b. 100 ml b. The diagnosis of brain death cannot be
c. 500 ml made in preterm infants less than 37 weeks
d. 240 ml of gestational age
e. 750 ml c. MRI head is a commonly used
ancillary test
7. What is the normal heart rate and respiratory d. Apnea testing must have been performed
rate for a child under 1 year? for a diagnosis of brain death
a. HR < 100 and RR < 40 e. An observation period of 12 h is recom-
b. HR < 120 and RR < 30 mended between clinical examinations
c. HR < 160 and RR < 60 determining brain death
d. HR < 140 and RR < 50
e. HR < 150 and RR < 30
EXTENDED MATCHING ITEM (EMI)
8. Which one of the following presentations QUESTIONS
most accurately suggests moderate blood vol-
ume loss (30-45%) in pediatric patients? 11. Primitive reflexes:
a. A 6-year-old child with increased HR, a. Asymmetric tonic neck
weak thready peripheral pulses, normal b. Babinski
SBP, normal pulse pressure, confused, c. Crossed extensor
mottled skin, and capillary refill 3 s. Urine d. Galant
output is 0.5 ml/kg/h. e. Heel
b. An 8-year-old child with markedly f. Moro
increased HR, weak thready central g. Palmar grip
pulses, low normal SBP, normal pulse h. Plantar grip
pressure, confused, prolonged CRT, and i. Rossolimo
urine output 0.2 ml/kg/h. j. Suprapubic extensor
c. A 5-year-old child with bradycardia, weak
central pulses, hypotension, widened For each of the following descriptions, select the
pulse, unresponsive to painful stimuli, most appropriate answers from the list above.
cold, and anuric. Each answer may be used once, more than once
d. An 8-month-old child with increased HR, or not at all.
weak thready peripheral pulses, normal 1. Scratching the skin of the infant's back
SBP, normal pulse pressure, irritable, from the shoulder downward, 2-3 cm lat-
mottled skin, and capillary refill 3 s. Urine eral to the spinous processes results in
output is 2 ml/kg/h. incurvation of the trunk, with the concavity
e. An 8-year-old child with markedly on the stimulated side.
increased HR, weak thready central 2. Passive total flexion of one lower extremity
pulses, low normal SBP, narrowed pulse results in extension of the other lower limb,
pressure, dulled response to pain, mark- with adduction and internal rotation into
edly prolonged CRT, and urine output talipes equinus.
0.2 ml/kg/h. 3. Sudden head extension produced by a light
drop of the head results in abduction
9. Which one of the following is NOT a feature followed by adduction and flexion of upper
of Down's syndrome? extremities.
a. Atrial septal defect
b. Occipital (and nasal) flattening 12. Developmental milestones:
c. Brushfield spots a. 1 month
d. Epicanthic folds b. 2 months
e. Simian crease c. 4 months
f. Duodenal atresia d. 6 months
g. Clinodactyly e. 9 months
h. Butterfly erythema f. 12 months
g. 15 months
10. Which one of the following statements about h. 18 months
brain death in children is most accurate? i. 2 years

Neurosurgery Books Full


www.ketabpezeshki.com 66485438-66485457
38 PEDIATRIC NEUROSURGERY: GENERAL PRINCIPLES AND NORMAL DEVELOPMENT 515

j. 3 years 3. The child's parents report that she can sit


k. 4 years but may need support, rolls in both direc-
l. 5 years tions, reaches with one hand, transfers
m. 6 years object between hands, babbles and seems
to recognize an object or person as
For each of the following descriptions, select the unfamiliar.
most appropriate answers from the list above. 4. The child's parents report that she raises
Each answer may be used once, more than once her head slightly in the prone position, fol-
or not at all. lows with her eyes to the midline only, her
1. The child's parents report that she can ride hands are tightly fisted, startles to sound
a bicycle, write her own name, identifies and fixes on people's faces.
written letters and numbers, knows right 5. The child's parents report that she sits
from left and knows all the color names. without support, crawls, pulls to stand, uses
2. The child's parents report that she can run, a pincer grasp, finger feeds, imitates speech
walks upstairs with her hand held, stoops sounds (nonspecific mama, dada), under-
and recovers, builds a three block tower, stands the meaning of “no,” plays gesture
can use a spoon and scribbles spontane- games (pat-a-cake) and understands her
ously. Additionally, she uses up to 25 words, own name. They have also noticed that
points to body parts when asked, uses words she now knows hidden objects still remain
to communicate wants and needs, and plays there and is anxious around people she
near other children. doesn't know.

SBA ANSWERS
1. d—Language circumference standard for children in all coun-
tries or ethnic groups. The standard growth
2. a—Weight 3500 g, height 50 cm, head curves are not appropriate for monitoring the
circumference 35 cm head size of children with certain medical condi-
tions associated with macrocephaly (e.g. achon-
For the average term newborn: droplasia, neurofibromatosis) and other curves
• Weight is 3500 g (low birthweight is are available for these specific conditions.
<2500 g, very low birthweight <1500 g
and extremely low <1000 g). Neonates FURTHER READING
with birthweight below the 10th percentile Boom JA. Normal growth patterns in infants and prepubertal
are termed small for gestational age, and children. UpToDate Topic 2845 Version 15.0.
may be genetic or due to intrauterine
growth restriction. 3. e—Isolated microcephaly
• Height is 50 cm
• Occipitofrontal head circumference (OFC) Microcephaly is generally defined as an occipito-
is 32.5-38 cm (mean 35 cm). frontal circumference more than 2 standard devi-
ations (SD) below the mean for a given age, sex,
Head circumference increases approximately and gestation (i.e. <3rd percentile); using this
• By 2 cm in first month definition approximately 2% of the general popu-
• By 6 cm in the first 4 months lation would be considered microcephalic even
• Approximately 1 cm/month during the first though many of these individuals are simply at
year of life the low end of the population distribution.
• Brain weight doubles by 4-6 months of age Because head growth is driven by brain growth,
and triples by 1 year of age microcephaly usually implies microencephaly
• The majority of head growth is complete by (small brain size) except in cases of generalized cra-
4 years of age niosynostosis in which skull growth is restricted,
but microencephaly may be present in children
The measuring tape should encircle the head and with normal OFC. In general, microencephaly
include an area 1-2 cm above the glabella anteri- can result either from abnormal brain develop-
orly and the most prominent portion of the occi- ment or insult to a previous normal brain. Multiple
put posteriorly. Measurement of OFC in the classifications: primary microcephaly is present at
newborn may be unreliable until the 3rd or 4th birth, while secondary develops postnatally;
day of life since it may be affected by caput others include genetic vs. environments, isolated
succedaneum, cephalohematoma, or molding. vs. syndromic, symmetric vs. asymmetric. The
It may be inappropriate to use a single head causes of microcephaly are summarized below:
Neurosurgery Books Full
www.ketabpezeshki.com 66485438-66485457
516 PART VII PEDIATRIC NEUROSURGERY

timing of these changes in relation to closure


Microcephaly: Causes of the fontanelles and sutures. Changes in
the volume of any component before the clo-
Genetic— Present at birth and
isolated uncomplicated by other
sure of the fontanelles and sutures may alter
anomalies; genetic failure to the OFC. Evaluation for macrocephaly should
produce enough neurons, be initiated when a single OFC measurement
relatively normal brain anatomy, is abnormal, when serial measurements reveal
no neurological signs but some progressive enlargement (i.e. crossing of one or
degree of learning difficulty
more major percentile lines [e.g. 10th, 25th,
Genetic— Down syndrome, Trisomy 18, 50th, 75th, 90th] between health supervision
syndromic Trisomy 13, Fetal alcohol
syndrome, and many others
visits), or when there is an increase in OFC of
>2 cm/month (for infants aged 0-6 months).
Genetic or Neural Tube Defect, Macrocephaly is defined as an OFC greater
other—CNS holoprosencephaly,
lissencephaly, schizencephaly,
than two standard deviations (SD) above the
polymicrogyria, pachygyria, mean for a given age, sex, and gestation
hydrancephaly, fetal brain (i.e. 97th percentile). The most common type
disruption sequence of anatomic megalencephaly is benign familial
Metabolic Aminoaciduria (e.g. megalencephaly where children are born with
phenylketonuria), storage large heads and normal body size, and during
disorders, and others infancy OFC increases to greater than the
Environmental Antenatal, perinatal, postnatal 90th percentile, typically 2-4 cm above, but par-
infection (e.g. TORCH), in utero allel to, the 98th percentile; head growth veloc-
drug/toxin exposure, hypoxic- ity slows to a normal rate by approximately
ischemic insult, IVH/stroke,
malnutrition
6 months of age. In children with a normal
neurologic examination, normal development,
no clinical features suggestive of a specific
FURTHER READING syndrome, and no family history of abnormal
Boom JA. Microcephaly in infants and children: etiology and neurologic or developmental problems, familial
evaluation. UpToDate. megalencephaly can be confirmed by measuring
the patient's parents' head circumferences and
4. d—He probably has familial macrocephaly by using Weaver curves. If the child's OFC falls
and needs no further workup within the normal ranges as estimated using the
Weaver curves, radiologic evaluation is not
Head growth is affected by growth and alter- necessary. Other causes of macrocephaly are
ations in the contents of the cranium and the summarized below:

Macrocephaly: Causes

Increased brain Anatomic (increased cells): Metabolic (deposition of substances in


(megalencephaly) • Familial megalencephaly brain tissue):
(commonest) • Leukodystrophies
• Neurophakomatoses • Lysosomal storage disorders
• Autistic spectrum disorders
• Achondroplasia
• Cerebral gigantism (Sotos
syndrome)
• Fragile X syndrome
• Cowden syndrome
• Gorlin syndrome
Increased CSF Hydrocephalus
Benign enlargement of subarachnoid space
Increased blood Hemorrhage (extradural, subdural, parenchymal, intraventricular).
Arteriovenous malformation
Increased bone Thalassaemia (marrow expansion)
Primary bone disorders
Intracranial mass Cyst, tumor, abscess

Neurosurgery Books Full


www.ketabpezeshki.com 66485438-66485457
38 PEDIATRIC NEUROSURGERY: GENERAL PRINCIPLES AND NORMAL DEVELOPMENT 517

FURTHER READING
Moderate Severe
Boom JA. Macrocephaly in infants and children: etiology and
Mild Blood Blood Loss Blood Loss
evaluation. UpToDate. System Loss (<30%) (30-45%) (>45%)
5. a—600 ml thready HR bradycardia
peripheral Weak Weak/absent
Current ATLS protocol for fluid boluses in pulses, thready central
pediatric trauma is 20 ml/kg (0.9% saline or ringer's normal SBP central pulses,
(80-90 + 2  pulses hypotension
lactate). If hypotensive after two boluses of fluid, age in years), Low normal (<70+ 2
packed red cells should be administered (10 ml/kg). normal pulse SBP (70-80 age),
pressure + 2  age), widened
6. a—1000 ml narrowed pulse
pulse pressure or
pressure undetectable
In general, maintenance fluid requirement for diastolic
children (excluding losses from drains, etc.) can pressure
be calculated as:
CNS Anxious, Lethargic, Comatose
• 100 ml/kg/24 h for the first 10 kg of weight irritable, dulles
• 50 ml/kg/24 h for the next 10 kg of weight confused response to
• 20 ml/kg/24 h for each kg over 20 kg pain
For example, in 24 h maintenance fluid require- Skin Cool, Cyanotic, Pale and cold
ment for a child weighing 25 kg would be: mottled, markedly
(10 kg  100 ml/kg) + (10 kg  50 ml/kg) prolonged prolonged
+ (5 kg  20 ml/kg) ¼ 1000 ml + 500 ml CRT CRT
+100 ml¼ 1600 ml (i.e. run at 1600/24 ¼ 66.7 ml/h) Urine Low to very Minimal Anuric
output low
7. c—HR < 160 and RR < 60
FURTHER READING
ATLS Student Course Manual: Advanced Trauma Life Sup-
Normal Values in Children
port. J Am Coll Surg 2012; ISBN-10: 1880696029, ISBN-13:
Weight Heart Urinary 978-1880696026.
Age Range Rate BP RR Output
9. h—Butterfly erythema
Infant (0-12 0-10 <160 >60 <60 2.0 ml/kg/h
months) Down's syndrome affects 1 in 1000 births and is
Toddler (1-2 10-14 <150 >70 <40 1.5 ml/kg/h due to trisomy 21. Manifestations are various, sys-
years) tems are summarized (non-exhaustive) below:
Preschool 14-18 <140 >75 <35 1.0 ml/kg/h
(3-5 years)
Down Syndrome: Clinical Features
School age 18-36 <120 >80 <30 1.0 ml/kg/h
(6-12 years) CNS Mental retardation, early dementia,
Adolescent 36-70 <100 >90 <30 0.5 ml/kg/h hearing impairment
(>13 years) Cardiac Endocardial cushion defect: ASD, VSD,
PDA, tetralogy of Fallot
FURTHER READING Facial Round face, occipital/nasal flattening,
ATLS Student Course Manual: Advanced Trauma Life Sup- Brushfield spots (speckled iris), open
port. J Am Coll Surg 2012; ISBN-10: 1880696029, ISBN-13: mouth and protruding tongue, upslanting
978-1880696026. palpebral fissure, epicanthal folds
Hands and Single transverse palmar (Simian)
8. b—An 8-year-old child with markedly feet crease, short fingers, curved little finger
increased HR, weak thready central pulses, (clinodactyly), sandal gap between big
low normal SBP, narrowed pulse pressure, toe and adjacent toe
dulled response to pain, markedly prolonged Spine/MSK Hypotonia, atlanto-axial instability
CRT, and urine output 0.2 ml/kg/h. GI Duodenal atresia/stenosis, annular
pancreas, omphalocele,
Hemorrhagic Shock in Children Hirschsprung's disease, imperforate
anus, tracheoesophageal fistula
Moderate Severe
Mild Blood Blood Loss Blood Loss Immune Impaired cellular immunity (more
System Loss (<30%) (30-45%) (>45%) infections), autoimmune disorders,
increased childhood risk of AML-M7
CVS Increased HR Markedly Tachycardia and ALL
Weak, increased followed by Endocrine Hypo/hyperthyroidism, type 1 diabetes
Continued mellitus, infertility in males
Neurosurgery Books Full
www.ketabpezeshki.com 66485438-66485457
518 PART VII PEDIATRIC NEUROSURGERY

10. b—The diagnosis of brain death cannot be FURTHER READING


made in preterm infants less than 37 weeks Spinello IM. Brain death determination. J Intens Care Med
of gestational age 2015;30(6):326-337;
Academy of Medical Royal Colleges. A code of practice for the
The most common causes of brain death in chil- diagnosis and confirmation of death; 2008;
dren are trauma, anoxic encephalopathy, infec-
tions, and cerebral neoplasms. In the UK, in Nakagawa TA, Ashwal S, Mathur M, Mysore M. Guidelines
children >2 months the criteria used to establish for the determination of brain death in infants and children:
an update of the 1987 task force recommendations. Pediatrics
death should be the same as those in adults; it is
2011;128(3):e720-e740.
also appreciated that between 37 weeks of gesta-
tion and 2 months of age, it is rarely possible
confidently to diagnose death as a result of ces-
sation of brainstem reflexes, and below 37 weeks EMI ANSWERS
of gestation the criteria to establish this cannot
be applied. Although the definition of brain 11. 1—d, Galant; 2—c, Crossed extensor;
death and the declaration process in children is 3—f, Moro
very similar to adult patients, there are several
specific recommendations made by the Ameri- Primitive reflexes are brainstem-mediated, com-
can Academy of Pediatrics in 2011 (below). plex, automatic movement patterns that com-
These guidelines are based in large part on con- mence as early as the 25th week of gestation,
sensus opinion as evidence is limited and com- are fully present at birth in term infants, and dis-
mittees in several countries decided to declare appear with central nervous system maturation
brain death only in children >2 months of age, (when voluntary motor activity and thus cortical
while others requiring serial examinations in inhibition emerges). Infants with cerebral palsy
younger infants. have been known to demonstrate persistence

American Academy of Pediatrics (2011) Guidlines for Determination of Brain Death


in Children

Waiting period before initial brain 24 h following cardiopulmonary resuscitation or severe acute brain injury
death examination is suggested if there are concerns about the neurologic examination or if
dictated by clinical judgment
Clinical examination Required
Core body temperature >35 °C (95 °F)
Number of examinations Two exams, irrespective of ancillary study results (if ancillary testing is
being done in lieu of initial examination elements that cannot be safely
performed, the components of the second examination that can be done
must be completed)
Number of examiners Two (different attending physicians must perform the first and second
exam)
Observation interval between Age dependent
neurologic examinations Term newborn (37 weeks gestation) to 30 days of age: 24 h
31 days to 18 years: 12 h
Reduction of observation period Permitted for both age groups if EEG or CBF consistent with brain death
between exams
Apnea testing Two apnea tests required unless clinically contraindicated
Final PCO2 threshold for apnea 60 mmHg and 20 mmHg above the baseline PaCO2
testing
Ancillary study recommended Not required except in cases where the clinical examination and apnea
test cannot be completed

• Term newborn (37 weeks gestation) to 30 days of age: EEG or CBF are
less sensitive in this age group. CBF may be preferred.
• >30 days to 18 years: EEG and CBF have equal sensitivity
Time of death Time of the second examination and apnea test (or completion of ancillary
study and the components of the second examination that can be safely
completed)

Neurosurgery Books Full


www.ketabpezeshki.com 66485438-66485457
38 PEDIATRIC NEUROSURGERY: GENERAL PRINCIPLES AND NORMAL DEVELOPMENT 519

of primitive reflexes or a delay in their disappear- infants, varying from flexor to extensor accord-
ance. Persistence of obligatory primitive reflexes ing to the intensity of the stimulus used but it
beyond 12 months of age is an indicator of a poor is generally accepted and that extensor plantar
prognosis regarding ambulation. The major response matures to flexor by the end of the first
primitive motor reflexes or patterns that have year in most normal infants. The Babinski sign
been described include Moro, palmar and plan- refers to the extensor toe response observed in
tar grasp, rooting, sucking, placing, Moro, corticospinal tract pathology but there is ongo-
Galant (or truncal incurvation), asymmetric ing debate as to whether a true Babinski sign
tonic neck reflex, crossed extensor, tonic laby- (dorsiflexion of the great toe and fanning of
rinthine reflex, and others. Special emphasis the remaining toes) is present as a primitive
should be placed on the plantar response: differ- reflex in infants where it is part of flexion with-
ent types of responses have been elicited in drawal of the leg.

Primitive Reflexes
Age at
Reflex Position Method Response Disappearance

Palmar grip Supine Placing the index finger in the Flexion of fingers, fist-making 6 months
palm of the infant
Plantar grip Supine Pressing a thumb against the Flexion of toes 15 months
sole just behind the toes
Galant Prone Scratching the skin of the Incurvation of the trunk, with 4 months
infant's back from the shoulder the concavity on the
downward, 2-3 cm lateral to the stimulated side
spinous processes
Asymmetric Supine Rotation of the infant's head to Extension of the extremities 3 months
tonic neck one side for 15 s on the chin side and flexion of
those on the occipital side
Suprapubic Supine Pressing the skin over the pubic Reflex extension of both lower 4 weeks
extensor bone with the fingers extremities, with adduction
and internal rotation into
talipes equinus
Crossed Supine Passive total flexion of one Extension of the other lower 6 weeks
extensor lower extremity limb, with adduction and
internal rotation into talipes
equinus
Rossolimo Supine Light tapping of toes 2-4 at their Tonic flexion of the toes at the 4 weeks
plantar surfaces first metacarpophalangeal
joint
Heel Supine Tapping on the heel with a Rapid reflex extension of the 3 weeks
hammer, with the infant's hip lower extremity in question
and knee joints flexed and the
ankle joint in neutral position
Moro Supine Sudden head extension Abduction followed by 6 months
produced by a light drop of the adduction and flexion of upper
head extremities
Babinski Supine Striking along the lateral aspect Combined extensor response: Presence
of the sole, extending from the simultaneous dorsiflexion of always
heel to the head of the fifth the great toe and fanning of abnormal
metatarsal the remaining toes

FURTHER READING
Zafeiriou DI. Primitive reflexes and postural reactions in the neu- 12. 1—m, 6 years; 2—h, 18 months; 3—d,
rodevelopmental examination. Pediatr Neurol 2004;31(1):1-8. 6 months; 4—a, 1 month; 5—e, 9 months

Neurosurgery Books Full


www.ketabpezeshki.com 66485438-66485457
520 PART VII PEDIATRIC NEUROSURGERY

Normal Developmental Milestones in Children


Age Gross Motor Fine (Visual) Motor Language Social/Adaptive

1 month Raises head slightly in Follows with eyes Alerts/startles to sound Fixes on faces
prone position to midline only
Hands tightly fisted
2 months Raises chest and head Regards object and Coos and vocalizes Social smile,
off bed in prone position follows 180 arc; reciprocally recognizes parent
briefly holds rattle
4 months Lifts onto extended Reaches for object Orients to voice, Initiates social
elbows in prone with both hands laughs, and squeals interaction
position; steady head together, bats at
control with no head lag, objects, grabs and
rolls over front to back retains objects
6 months Sits but may need Reaches with one Babbles Recognizes object or
support, rolls in both hand, transfers person as unfamiliar
directions object between
hands
9 months Sits without support Uses pincer grasp, Imitates speech Plays gesture games
Crawls finger feeds sounds (nonspecific (pat-a-cake),
Pulls to stand mama, dada), understands own
understands meaning name, object
of “no” permanence,
stranger anxiety
12 months Cruises furniture, stands Can voluntarily Discriminative use of Imitates, comes
alone, takes a few release items, mama, dada; one to when called,
independent steps four other words, cooperates with
follows command with dressing
gesture
15 months Walks well Builds two block Four to six more words; Begins to use cup;
independently tower; throws ball uses jargon; responds indicates wants or
underhand to one-step verbal needs
command
18 months Runs; walks upstairs Builds three block Uses up to 25 words; Uses words to
with hand held; stoops tower; uses spoon; points to body parts communicate wants
and recovers spontaneous when asked; uses and needs; plays
scribbling words to communicate near but not with
wants and needs other children
2 years Walks unassisted up and Builds 4-6 brick Uses 50+ words, two- Removes simple
down stairs; kicks ball; tower, uses fork and three-word clothing; parallel
throws ball overhand; and spoon, copies a phrases; uses I and me; play
jumps with two feet off straight line 50% of speech
floor intelligible to stranger
3 years Pedals tricycle; broad Copies a circle Uses 5-8 word Knows age and
jumps sentences; 75% of gender, engages in
speech intelligible to group play, shares
stranger
4 years Balances on one foot Copies a cross; Tells a story; 100% of Dresses self, puts on
catches ball speech intelligible to shows, washes and
stranger dries hands,
imaginative play
5 years Skips with alternating Draws person with Asks what words mean Names four colors;
feet six body parts plays cooperative
games; understands
rules and abides by
them
6 years Rides a bicycle Writes name Identifies written Knows right from
letters and numbers left; knows all color
names

FURTHER READING
Fine KS. Paediatric board recertification review. Lippincott, Williams and Wilkins; 2008. p. 2.

Neurosurgery Books Full


www.ketabpezeshki.com 66485438-66485457
CHAPTER 39

CRANIOSYNOSTOSIS
SINGLE BEST ANSWER (SBA) QUESTIONS
1. Features demonstrated in the picture below 3. Which one of the following most likely to
are most likely the result of: cause the appearances shown below?

A B

a. Bicoronal synostosis
b. Crouzon's syndrome
c. Metopic synostosis
d. Pierre-Robin sequence
e. Sagittal synostosis
a. Anterior plagiocephaly 4. Given the clinical features demonstrated in
b. Deformational posterior plagiocephaly this 6-month-old child which one of the fol-
c. Kleeblattschadel deformity lowing is most likely:
d. Oxycephaly
e. Turricephaly
2. What is the most likely craniosynostosis
depicted in this image?

a. Right anterior plagiocephaly due to left


coronal synostosis
b. Left anterior plagiocephaly due to left
coronal synostosis
a. Bilateral Coronal synostosis c. Right anterior plagiocephaly due to right
b. Cloverleaf (Kleeblattschadel) coronal synostosis
c. Lambdoid synostosis d. Left anterior plagiocephaly due to right
d. Metopic synostosis coronal synostosis
e. Sagittal synostosis

521
Neurosurgery Books Full
www.ketabpezeshki.com 66485438-66485457
522 PART VII PEDIATRIC NEUROSURGERY

e. Left anterior plagiocephaly due to bilat- 8. Virchow's law states


eral coronal synostosis a. Skull growth is arrested in the direction
f. Right anterior plagiocephaly due to bilat- perpendicular to the fused suture and
eral coronal synostosis reduced at the sites of unaffected sutures
b. Skull growth is arrested in the direction
5. Features demonstrated in the bird's eye dia- parallel to the fused suture and expanded
gram below are most consistent with: at the sites of unaffected sutures
c. Skull growth is arrested in the direction
perpendicular to the fused suture and
expanded at the sites of affected sutures
d. Skull growth is arrested in the direction
parallel to the fused suture and expanded
at the sites of affected sutures
e. Skull growth is arrested in the direction
perpendicular to the fused suture and
expanded at the sites of unaffected sutures

9. Features of raised intracranial pressure on


plain radiograph show:
a. Bilateral Harlequin sign
b. Copper beating
c. Frontal bossing
d. Temporoparietal bossing
e. Open fontanelle

10. Fronto-orbital advancement surgery is most


a. Right posterior plagiocephaly due to
likely considered in which one of the follow-
lambdoid synostosis
ing scenarios
b. Left posterior plagiocephaly due to lamb-
a. Apert's syndrome
doid synostosis
b. Fibrous dysplasia
c. Right posterior plagiocephaly due to
c. Paget's disease
bilateral lambdoid synostosis
d. Sagittal synostosis
d. Rightposteriordeformationalplagiocephaly
e. Left posterior deformational plagiocephaly
11. Which one of the following most likely to
f. Left posterior plagiocephaly due to bilat-
cause the appearances shown below?
eral lambdoid synostosis

6. Craniosynostosis occurs in approximately


how many live births?
a. 1/1000 to 1/1500
b. 1/1500 to 1/2000
c. 1/2000 to 1/2500
d. 1/2500 to 1/3000
e. 1/3000 to 1/3500

7. Frequency of different types of single suture


synostoses is approximately:
a. Lambdoid 18%, Metopic 25%, Sagittal
60%, Unicoronal 3%
b. Lambdoid 18%, Metopic 60%, Sagittal a. Lambdoid synostosis
25%, Unicoronal 3% b. Metopic synostosis
c. Lambdoid 25%, Metopic 18%, Sagittal c. Positional plagiocephaly
3%, Unicoronal 60% d. Sagittal synostosis
d. Lambdoid 3%, Metopic 25%, Sagittal e. Unicoronal suture synostosis
60%, Unicoronal 18%
e. Lambdoid 60%, Metopic 18%, Sagittal
3%, Unicoronal 25%

Neurosurgery Books Full


www.ketabpezeshki.com 66485438-66485457
39 CRANIOSYNOSTOSIS 523

12. Which one of the following most likely to a. Cerebral abscess


cause the appearances shown below? b. Fibrous dysplasia
c. Multiple myeloma
d. Neurofibromatosis type 1
e. Raised intracranial pressure

15. Which one of the following most likely to


cause the appearances shown below?

A B

a. Aqueduct stenosis
b. Fibrous dysplasia
c. Hyperparathyroidism
d. Langerhan's histiocytosis
e. Multiple myeloma
a. Arachnoid cyst
13. Which one of the following most likely to
b. Epidermoid
cause the appearances shown below?
c. Fibrous dysplasia
d. Leukemia
e. Raised intracranial pressure

16. Which one of the following most likely to


cause the appearances shown below?

a. Hyperparathyroidism
b. Leukemia
c. Neurofibromatosis (NF)
d. Pfeiffer's syndrome
e. Saethre-Chotzen

14. Which one of the following most likely to


cause the appearances shown below?

a. Dermoid
b. Epidermoid
c. Raised intracranial pressure
d. Saethre-Chotzen syndrome
e. Tuberculosis

Neurosurgery Books Full


www.ketabpezeshki.com 66485438-66485457
524 PART VII PEDIATRIC NEUROSURGERY

EXTENDED MATCHING ITEM (EMI) 18. Causes of craniosynostosis:


a. Blood dyscrasias (e.g. thalassemia, sickle
QUESTIONS cell anemia, polycythemia vera)
b. Fibroblast growth factor receptor mutations
17. Craniofacial terminology: c. Holoprosencephaly
a. Acrocephaly d. Hyperthyroidism
b. Anterior plagiocephaly e. MSX-2 transcription factor
c. Brachycephaly f. Mucopolysaccharidoses (e.g. Hurler syn-
d. Cloverleaf skull drome, Morquio syndrome)
e. Dolichocephaly g. Overshunted hydrocephalus
f. Harlequin sign h. Rickets disease
g. Oxycephaly i. Teratogens (Valproic acid, Retinoic acid)
h. Posterior plagiocephaly j. Transforming growth factor-beta receptors
i. Scaphocephaly k. TWIST transcription factor
j. Trigonocephaly
k. Turricephaly For each of the following descriptions, select the
most appropriate answers from the list above.
For each of the following descriptions, select the Each answer may be used once, more than once
most appropriate answers from the list above. or not at all.
Each answer may be used once, more than once 1. Implicated in Crouzon, Apert, Pfeiffer and
or not at all. Jackson-Weiss syndromes
1. Cephalic index <74 2. Implicated in intrauterine head constraint-
2. Cephalic index >83 related craniosynostosis—d
3. Implicated in Saethre-Chotzen syndrome—c

SBA ANSWERS
1. c—The picture demonstrates a newborn
infant with cloverleaf skull deformity-frontal
towering, bitemporal expansion, bilateral
supraorbital recession (with proptosis),
hypertelorism, and midface hypoplasia. One
should also examine for broad great toe/
thumb (Pfeiffer syndrome) and syndactyly
(Apert's syndrome). It is caused by premature
closure of sagittal, coronal, and lambdoid
sutures and can occur in any severe
craniosynostosis.
Image with permission from Ellenbogen RG, Abdulrauf
SI, Sekhar LN. Principles of Neurological Surgery, 3rd
ed., Elsevier, Saunders, 2012.

2. d—Metopic synostosis. Features demon-


strated are trigonocephaly, significant ridg-
ing over the metopic suture, supraorbital Image with permission from Ellenbogen RG, Abdulrauf
recession and hypotelorism (see CT below). SI, Sekhar LN. Principles of Neurological Surgery, 3rd
Due to reduced anterior cranial volume ed., Elsevier, Saunders, 2012.
resulting in back pressure into the posterior
fossa, there is an association with Type 1
Chiari malformations (30%). It is also associ- 3. e—Sagittal synostosis
ated with frontal dysmorphology-corpus cal- Image with permission from Ellenbogen RG, Abdulrauf
losum dysgenesis, holoproscencephaly. SI, Sekhar LN. Principles of Neurological Surgery, 3rd
ed., Elsevier, Saunders, 2012.

Neurosurgery Books Full


www.ketabpezeshki.com 66485438-66485457
39 CRANIOSYNOSTOSIS 525

4. b—Left anterior plagiocephaly due to left cor- 11. e—Unicoronal suture synostosis. The skull
onal synostosis. Features seen are significant radiograph exhibits the classic ‘harlequin’
left supraorbital retrusion, left forehead flat- sign.
tening, and compensatory right frontal bos-
sing. Other features to look for in unicoronal Image with permission from Coley BD. Caffey’s Pediat-
ric Diagnostic Imaging, 12th ed., Elsevier, Saunders,
synostosis are ipsilateral perisutural ridging, 2013.
ipsilateral nasal root displacement, anterior
displacement of ipsilateral ear, contralateral 12. a—Aqueduct stenosis. The appearances
chin deviation, and the pathognomonic feature above are termed “copper beaten skull” asso-
of ipsilateral orbital elevation (harlequin sign). ciated with raised intracranial pressure in
Strabismus is common (50-60%) due to children. Common causes are craniosynosto-
mechanical effect on superior oblique, and sis, obstructive hydrocephalus, intracranial
anterior plagiocephaly is commoner on the masses and hypophosphatasia.
right side (3:2).
Image with permission from Coley BD. Caffey's Pediat-
Image with permission from Ellenbogen RG, Abdulrauf ric Diagnostic Imaging, 12th ed., Elsevier, Saunders,
SI, Sekhar LN. Principles of Neurological Surgery, 3rd 2013.
ed., Elsevier, Saunders, 2012.
13. d—Pfeiffer's syndrome (Cloverleaf skull).
5. d—Right posterior deformational plagioce-
phaly. Posterior deformational plagiocephaly Image with permission from Coley BD. Caffey's Pediatric
is characterized by a parallelogram shaped Diagnostic Imaging, 12th ed., Elsevier, Saunders, 2013.
head, anterior displacement of ipsilateral
ear, and ipsilateral frontal bossing. In con- 14. d—Angiomas and neurofibromas of the scalp
trast, unilateral lambdoid synostosis is may affect the underlying skull and cause
marked by a trapezoid shaped skull, posterior deformities, bony defects, and regional hyper-
displacement of ipsilateral ear and contralat- ostoses. Plain film findings of NF include lytic
eral occipital bossing. defect in the lambdoid suture, absence of the
orbital roof and floor, elevated lesser sphenoid
Image with permission from Ellenbogen RG, Abdulrauf wing, enlarged middle cranial fossa, enlarged
SI, Sekhar LN. Principles of Neurological Surgery, 3rd cranial nerve foramina, unilateral orbital
ed., Elsevier, Saunders, 2012. enlargement, and J-shaped sella turcica.
6. c—1/2000 to 1/2500 Image with permission from Coley BD. Caffey's Pediatric
Diagnostic Imaging, 12th ed., Elsevier, Saunders, 2013.
7. d—Lambdoid 3%, Metopic 25%, Sagittal
60%, Unicoronal 18%. Sagittal (3.5-7:1) and 15. b—Epidermoid
metopic (75%) synostosis is commoner in boys,
while unicoronal is commoner in girls (3:2). A small oval defect in the parietal bone with a
True lambdoid synostosis is rare, and must be sharply defined sclerotic border. Epidermoids
distinguished from posterior deformational are ectodermal rests or inclusions that may be
plagiocephaly (positional molding) where there located in the scalp, in the diploic spaces, or
is occipital flattening without suture fusion, between the internal surface of the inner table
possibly due to supine sleeping position and the dura. Epidermoids are usually benign
instituted to reduce sudden infant death and grow slowly. If they protrude into the cranial
syndrome. cavity, they may be the source of cerebral symp-
toms. When epidermoids grow within the bone
8. e—Skull growth is arrested in the direction or impinge on it, they produce local destruction
perpendicular to the fused suture and of bone that appears radiographically as a sharply
expanded at the sites of unaffected sutures, demarcated lucency surrounded by a smooth scle-
leading to characteristic calvarial deformations rotic margin, which sometimes may be scalloped.
The margin is due to flaring of the edge of the
9. b—Copper beating. The thinned out skull is bone into a marginal ridge. Most cases are found
usually an indicator of chronic hydrocephalus. in children younger than 3 years. The lesions usu-
ally disappear within a few years of discovery.
10. a—Apert's syndrome
Image with permission from Coley BD. Caffey's Pediat-
ric Diagnostic Imaging, 12th ed., Elsevier, Saunders,
2013.

Neurosurgery Books Full


www.ketabpezeshki.com 66485438-66485457
526 PART VII PEDIATRIC NEUROSURGERY

16. e—Multiple destructive tuberculosis foci in EMI ANSWERS


the calvaria of a 3-year-old girl. Tuberculosis
of the calvaria usually manifests as a painless 17. 1—e, Dolichocephaly; 2—c, Brachycephaly
subgaleal scalp swelling with a discharging
sinus. The lesions are usually either small, Cephalic index (CI) ¼ biparietal diameter (BPD)/
circumscribed, punched-out lytic areas or occipitofrontal diameter (OFD)  100. Normal
spreading, circumscribed sclerotic areas; or range 74-83, and it can be altered by breech pre-
a combination of the two. The differential sentation, ruptured membranes, twin pregnancy
diagnosis of multiple lytic lesions in the when measured intrauterine.
skull—TB, multiple myeloma, Langerhans
cell histiocytosis. 18. 1—b, Fibroblast growth factor receptor muta-
tions; 2—j, Transforming growth factor-beta
Image with permission from Coley BD. Caffey's Pediat-
ric Diagnostic Imaging, 12th ed., Elsevier, Saunders,
receptors; 3—k, TWIST transcription factor.
2013.

Neurosurgery Books Full


www.ketabpezeshki.com 66485438-66485457
CHAPTER 40

CONGENITAL CRANIAL AND


SPINAL DISORDERS
SINGLE BEST ANSWER (SBA) QUESTIONS
1. Which one of the following is most likely
based on the radiograph shown below?

a. Achondroplasia
b. Klippel-Feil
c. Osteogenesis imperfecta
d. Posterior spina bifida 3. Which one of the following is most likely
e. Segmental spinal dysgenesis based on the MRI shown?

2. A young child with an FGFR-3 mutation is


shown in the picture below. Which one of
the following disorders is he NOT at
increased risk of?
a. Cervicomedullary compression
b. Hydrocephalus
c. Posterior circulation aneurysms
d. Spinal canal stenosis A B
e. Thoracolumbar kyphosis

a. Basilar invagination
b. Corpus callosum agenesis
c. Hydrocephalus
d. Lissencephaly
e. Type II Chiari malformation

527
Neurosurgery Books Full
www.ketabpezeshki.com 66485438-66485457
528 PART VII PEDIATRIC NEUROSURGERY

4. MRI shows the appearances below, and the 6. Which one of the following is most likely
lesion is bright on T2WI and dark on fat sup- based on the MRI shown?
pression sequences and does not show any
contrast enhancement. Which one of the fol-
lowing is most likely?

a. Blake's pouch cyst


b. Chiari III malformation
c. Dandy-Walker Malformation
a. Colloid cyst d. Joubert's syndrome
b. Craniopharyngioma e. Mega cysterna magna
c. Intracranial lipoma
d. Intraventricular hemorrhage 7. Which one of the following is most likely
e. Pericallosal aneurysm based on the MRI shown?

5. Which one of the following is likely based on


the MRI shown?

a. Arachnoid cyst
b. Dandy-Walker variant
c. Epidermoid cyst
d. Medulloblastoma
e. Type II Chiari malformation

a. Closed-lip schizencephaly
b. Cobblestone cortex
c. Septo-optic dysplasia
d. Subcortical heterotopia
e. Tuberous sclerosis

Neurosurgery Books Full


www.ketabpezeshki.com 66485438-66485457
40 CONGENITAL CRANIAL AND SPINAL DISORDERS 529

8. A 31-year-old undergoes MRI for headache. 10. Which one of the following is most likely
On axial views the 4th ventricle is enlarged based on the imaging shown?
and on certain sequences choroid plexus
can be seen under and posterior to the vermis
entering the superior aspect of the lesion.
There is no restricted diffusion. Which one
of the following is most likely?

A B

a. Band heterotopia
b. Marginal glioneuronal heterotopia
c. Subcortical heterotopia
d. Subependymal heterotopia
e. Type II lissencephaly

11. A 45-year-old woman with intractable struc-


tural epilepsy who had experienced partial
a. Arachnoid cyst complex and secondarily generalized seizures
b. Blake's pouch cyst from the age of 8 years. Coronal FLAIR MRI
c. Cystic metastasis is shown. Which one of the following is most
d. Epidermoid likely?
e. Type II Chiari malformation

9. A child undergoes MRI scan as part of epi-


lepsy evaluation after two seizures unrelated
to febrile episodes. There are no symptoms
of hydrocephalus. MRI appearances are
shown, and there is no diffusion restriction.
Which one of the following is most likely?

a. Abscess
b. Focal cortical dysplasia
c. Lissencephaly
d. Mesial temporal sclerosis
e. Tuberous sclerosis
a. Arachnoid cyst
b. Dandy-Walker malformation
c. Epidermoid cyst
d. Mega cisterna magna
e. Type III Chiari malformation

Neurosurgery Books Full


www.ketabpezeshki.com 66485438-66485457
530 PART VII PEDIATRIC NEUROSURGERY

12. Which one of the following is most likely 14. Which one of the following is most likely
based on the imaging shown below? based on the imaging shown below?

A B
A B

a. Cobblestone cortex a. Lissencephaly with band heterotopia


b. Focal cortical dysplasia b. Porencephalic cyst
c. Hemimegalencephaly c. Pachygyria
d. Lissencephaly with band heterotopia
d. Schizencephaly
e. Periventricular nodular heterotopia e. Semilobar holoprosencephaly
13. Which one of the following is most likely 15. Which one of the following is most likely
based on the imaging shown? based on the imaging shown below?

a. Cobblestone cortex
a. Agyria
b. Focal cortical dysplasia
b. Lissencephaly with band heterotopia
c. Lissencephaly with band heterotopia
c. Pachygyria
d. Periventricular nodular heterotopia
d. Polymicrogyria
e. Polymicrogyria
e. Walker-Warburg syndrome

Neurosurgery Books Full


www.ketabpezeshki.com 66485438-66485457
40 CONGENITAL CRANIAL AND SPINAL DISORDERS 531

16. Which one of the following is most likely


based on the imaging shown?

A B

A B

a. Dermal sinus tract


b. Dorsal enteric fistula
a. Alobar holoprosencephaly c. Lipomyelomeningocele
b. Anencephaly d. Myelocele
c. Dandy-Walker malformation e. Type II Chiari malformation
d. Porencephalic cyst
e. Schizencephaly 19. The MRI shown below demonstrates
features of which one of the following
17. Which one of the following is most likely conditions?
based on the imaging shown below?

a. Atretic cephalocele
b. Encephalocele (meningoencephalocele)
c. Gliocele
d. Meningocele
e. Meningoencephalocystocele

18. A newborn infant has an apneic event with


cyanosis and he was brought to the hospital.
His evaluation was remarkable for develop- a. Chiari I malformation
mental delay, bilateral lower-extremity b. Chiari II malformation
frog-leg flaccid paralysis, with a large open c. Chiari III malformation
defect in his lower back. At birth, the defect d. Chiari IV malformation
was covered by a transparent membrane, e. Basilar invagination
which subsequently ruptured and drained
clear fluid for several weeks. Based on the
imaging below, what is the cause of the
infant's apneic events?

Neurosurgery Books Full


www.ketabpezeshki.com 66485438-66485457
532 PART VII PEDIATRIC NEUROSURGERY

20. You plan to perform a foramen magnum 22. Which one of the following is likely to detect
decompression on the patient shown below. tethering of the spinal cord earliest?
Which one of the following investigations a. Spinal CT
may be helpful? b. Spinal MRI
c. Spinal ultrasound
d. Spinal XR
e. Urodynamic testing

23. Which one of the following patients with teth-


ered cord syndrome (TCS) is LEAST likely to
benefit from detethering surgery?
a. A 1-day-old boy with bilateral 3/5 leg
weakness and a midline open lumbar
lesion leaking clear fluid.
b. A 6-week-old boy with static bilateral
3/5 leg weakness. Urodynamic testing
shows evidence of neurogenic bladder
which is being managed with intermittent
catheterization.
c. A 2-year-old boy with a tuft of hair on his
lumbar spine. He does not have any gross
neurology in the lower limbs but MRI
shows the conus at L2/3. Urodynamic
testing shows evidence of neurogenic
bladder disturbance.
d. A 9-year-old boy with progressive scolio-
sis. MRI shows a tight filum terminale.
e. A 12-year-old boy with back pain. MRI
a. CSF flow study shows a tight filum terminale. Urody-
b. CT cisternography namic testing is equivocal.
c. CT cervical angiogram
d. Flexion/extension X-rays 24. Which one of the following is most likely
e. Nerve conduction studies based on the findings in the image below?
21. Which one of the following is most likely
based on the clinical image shown below?

a. Lipomyelomeningocele
b. Meningocele
c. Myelocele
d. Myelocystocele
e. Myelomeningocele
a. Achondroplasia
b. Closed spinal dysraphism
c. Klippel-Feil syndrome QUESTIONS 25–33
d. Sturge-Weber syndrome
Additional questions 25–33 available on
e. Wyburn-Mason syndrome ExpertConsult.com

Neurosurgery Books Full


www.ketabpezeshki.com 66485438-66485457
40 CONGENITAL CRANIAL AND SPINAL DISORDERS 533

EXTENDED MATCHING ITEM (EMI) 3. A subcutaneous lipoma tethers the spinal


cord (lipoma-placode interface) inside the
QUESTIONS spinal canal via defect in vertebral elements.
4. Small ependymal lined cavity in the conus
34. Spinal dysraphism: medullaris which may undergo cystic
a. Caudal agenesis dilatation.
b. Distematomyelia 5. Cysts usually in intradural extramedullary
c. Dermal sinus plane of endodermal origin, usually lined
d. Dorsal enteric fistula with GI or respiratory epithelium.
e. Filar lipoma
f. Intradural lipo 35. Congenital cranial abnormalities:
g. Hemimyelocele a. Adrenoleukodystrophy
h. Hemimyelomeningocele b. Arachnoid cyst
i. Lipomyelocele c. Band heterotopia
j. Lipomyelomeningocele d. Blake's pouch cyst
k. Meningocele e. Chiari II malformation
l. Myelocele f. Dandy-Walker malformation
m. Myelocystocele g. Focal cortical dysplasia
n. Myelomeningocele h. Holoprosencephaly
o. Neurenteric cyst i. Mega cisterna magna
p. Persistent terminal ventricle j. Porencephalic cyst
q. Segmental dysgenesis k. Schizencephaly
r. Tight filum terminale l. Subependymal heterotopia
For each of the following descriptions, select the For each of the following descriptions, select the
most appropriate answer from the list above. most appropriate answer from the list above.
Each answer may be used once, more than once Each answer may be used once, more than once
or not at all. or not at all.
1. Epithelium-lined fistula which extends 1. An intraparenchymal vascular territory
from skin to meninges within the spinal CSF cyst lined by white matter, communi-
canal, possibly opening into the subarach- cating with the ventricles and/or subarach-
noid space or connecting to filum termi- noid space.
nale, lipoma and may also be associated 2. A full-thickness cerebral cleft extending
with a spinal dermoid. from the pial surface of the cortex to the
2. Myelomeningocele affecting one of two ependymal lining of the lateral ventricle,
hemicords in split cord malformation. and is almost always lined by abnormal
gray matter.

SBA ANSWERS
1. b—Klippel-Feil syndrome I—fusion of many cervical and upper thoracic ver-
tebrae, Type II—fusion of 2-3 vertebrae with asso-
KFS is a congenital anomaly that is caused by the ciated hemivertebrae/occipito-atlantal fusion/
failure of the spine to segment properly during other abnormality, Type III—cervical fusion with
embryonic development, resulting in fusion of lower thoracic/lumbar vertebral fusion. Fetal cer-
two or more cervical vertebrae. Incidence is 1 in vical vertebrae should be evaluated with ultra-
40,000 with a slight female predominance (3:2). sound (US) for cervical fusions, blocking, and
Clinically patients may have a shortened neck, a hemivertebrae. MRI can also be useful to deter-
low posterior neckline, and limited neck mobility mine whether the cervical malformation is causing
(less than half possess all three). Associated find- compression of the brain, brainstem, or spinal
ings that may provide diagnostic clues include cord. Treatment varies depending on the severity
other skeletal abnormalities, orofacial anomalies, of the fusions, adjacent segment degenerative
and visceral defects. Congenital fusions can occur disease, degree of instability present and the
at any level of the cervical spine, although 75% underlying diagnosis. Isolated KFS is generally
occur in the region of the first three cervical verte- well tolerated. Initial treatment strategies include
brae. The most prevalent fusion is between C2 modification of activities, bracing, and traction,
and C3. KFS is classified into three types: Type all of which may delay surgery and prevent

Neurosurgery Books Full


www.ketabpezeshki.com 66485438-66485457
534 PART VII PEDIATRIC NEUROSURGERY

neurologic compromise. Indications for surgical sinus preoperatively. The use of polysom-
stabilization are symptomatic instability or neuro- nography to assess patients for the presence of
logic compromise. central and/or obstructive sleep apnea has been
reported to identify central/mixed apnea in up
Image with permission from Dornbos D 3rd, Ikeda DS, to 60% of unselected children with achondropla-
Slivka A, Powers C. Vertebral artery dissection after
neck extension in an adult patient with Klippel-Feil syn-
sia. Indications for surgery include myelopathy
drome. J Clin Neurosci 2014;21(4):685-8. with upper motor neuron signs such as clonus
and hyperreflexia, and/or central apnea as docu-
2. c—Posterior circulation aneurysms mented on polysomnography, or the presence
of a syrinx, with evidence of a narrow foramen
Achondroplasia is the most common form of magnum and/or T2 signal change in the spinal
human short-limbed dwarfism and is one of a cord on MR imaging.
spectrum of diseases caused by mutations in the
FGFR3 gene. Occurs in 1 in 10,000-30,000 live Image with permission from Moore KL. Developing
births. The disease is autosomal dominant, but Human, 10th ed. Elsevier, 2016.
80% of patients have new mutations. Classic fea-
FURTHER READING
tures include a long, narrow trunk with short King JA, Vachhrajani S, Drake JM, Rutka JT. Neurosurgical
limbs, macrocephaly with frontal bossing/promi- implications of achondroplasia. J Neurosurg Pediatr 2009;4
nence and facial hypoplasia. Other features (4):297-306.
include hypotonia in infancy (motor delay),
hyperextensible joints, short and broad trident 3. b—Corpus callosum agenesis
hands, and thoracolumbar kyphosis. Abnormal
compression at a number of levels along the neur- Complete corpus callosum agenesis is the most
axis may result in hydrocephalus, cervicomedul- common type of commissural agenesis, and may
lary compression, spinal canal stenosis (both be associated with absence of the hippocampal
cervical and lumbar), syringomyelia, and spinal commissure; in partial agenesis, the genu and
instability. Diminished growth of the skull base anterior body are formed, whereas portions that
in achondroplasia results in cranial foraminal ste- develop later like the posterior body, isthmus,
nosis and intracranial venous hypertension result- and splenium are absent. Mid-sagittal MR images
ing in impaired CSF absorption, macrocephaly; are diagnostic, and also show an everted cingulate
additionally, obstructive hydrocephalus may gyrus and longitudinal Probst bundles containing
result from cervicomedullary compression. Mon- non-crossing callosal axons, lateral ventricles are
itoring of head growth should be performed at shifted laterally and closed medially by rolled up
regular intervals and compared with control white matter lamina (which should be forming
charts for children with achondroplasia to avoid the leaf of the septum pellucidum). The inner
unnecessary CSF shunting. If raised ICP is sus- walls of the lateral ventricles are concave medially
pected, US can show ventricular size, but MRI as a result of encroachment of the Probst bundles
will detect transependymal spread of CSF, assess on the ventricular lumen. In addition, the roof of
craniocervical junction and venous stenosis. the third ventricle bulges upward. The frontal horns
Despite mild-moderate ventriculomegaly, the of the lateral ventricles might be underdeveloped,
majority of patients with macrocephaly stabilize whereas the dilated temporal horns invaginate into
spontaneously, and thus insertion of a VP shunt the core of the parahippocampal gyri because of
should be avoided if possible. Stress on the cra- decreased white matter. The lateral ventricles run
niocervical junction by a large head with weak parallel to each other, with marked dilation of the
cervical musculature can produce cervicomedul- trigone and occipital horns (colpocephaly). Devel-
lary compression. Patients may present with neck opment of the cerebrum and cerebellar hemi-
pain, apneic episodes, bulbar dysfunction, bladder spheres occurs at the same time, hence associated
dysfunction, paresis, hyperreflexia, and hyperto- Chiari malformation, Dandy-Walker malforma-
nia with clonus. Indeed, “normal” reflexes may tion, neuronal migration anomalies, and midline
reflect spasticity in normally hypotonic children facial anomalies (facial cleft, encephalocele). Peri-
with achondroplasia. Acute deterioration may ventricular or subcortical heterotopia may also
occur after minor trauma and there is an be seen.
increased incidence of sudden death at <4 years Image with permission from Loevner L. Brain Imaging:
of age. Investigations include MR imaging and Case Review Series, 2nd ed. Elsevier, Mosby, 2009.
formal polysomnography. MRI may show tight
foramen magnum, flexion/extension sequences 4. c—Intracranial lipoma
may demonstrate transient cervicomedullary
compression or CSF flow obstruction, and Intracranial lipomas are thought to be maldifferen-
MRV may show a persistent occipital venous tiations of the meninx primitive (undifferentiated)

Neurosurgery Books Full


www.ketabpezeshki.com 66485438-66485457
40 CONGENITAL CRANIAL AND SPINAL DISORDERS 535

mesenchyme that surrounds the developing agenesis lead to a box-like shape of the frontal
brain. Evolution of the inner meninx primitiva horns in coronal planes, low-lying fornix (due to
leads to formation of the subarachnoid spaces, absent septum), pituitary hypoplasia/empty
the pre-pontomedullary cistern is the first to sella/ectopic posterior pituitary gland, hypo-
develop, followed by cisterns around the brain- thalamic hypoplasia, optic nerve and/or optic
stem and cerebral hemispheres, the quadrigem- chiasm hypoplasia (visualization difficult as mild
inal plate, and finally the suprasellar system. form—ophthalmological findings more reliable).
Meninx primitiva surrounding the dorsum of In addition, the remainder of the brain paren-
the lamina terminalis is the last to become chyma might have a variety of other congenital
evolved. The most common locations of an anomalies like malformations of cortical deve-
intracranial lipoma are in the deep interhemi- lopment (schizencephaly and gray matter
spheric fissure (40-50%), quadrigeminal plate heterotopia), olfactory hypoplasia (arrhinence-
cistern (30%), suprasellar/interpedicular cistern phaly), hypoplasia of white matter, and/or
(10-20%), cerebellopontine angle cistern (10%), ventriculomegaly.
and sylvian fissures (5%). Because by embryolo-
gic definition lipomas occupy the subarachnoid Image with permission from Adam A, et al. (Eds.).
Grainger & Allison's Diagnostic Radiology, 6th ed.
space, blood vessels and cranial nerves course Elsevier, Churchill Livingstone, 2014.
through them. Most intracranial lipomas are
asymptomatic, so they are diagnosed inciden- 6. c—Dandy-Walker malformation
tally. On CT scan, a lipoma is a well-defined,
fat density mass within a cistern. MR appear- The classic triad of the full-blown Dandy-Walker
ances show T1 hyperintensity, T2 hyperinten- malformation is complete or partial agenesis of
sity, fat suppression and no enhancement. the vermis, cystic dilation of the 4th ventricle,
Chemical shift artifact seen around the hyperin- and an enlarged posterior fossa with upward dis-
tensity confirms the fatty origin of the mass as placement of the transverse sinuses, tentorium,
opposed to hemorrhage. Interhemispheric lipo- and torcula. Hydrocephalus seen in 80% of cases
mas are invariably associated with hypogenesis but is not a part of the essential criteria. The pres-
of the corpus callosum. A pericallosal lipoma ence of vermian agenesis and cystic dilatation
might also show multiple signal voids because of the fourth ventricle without an enlarged poste-
of a combination of traversing vessels and calci- rior fossa is termed Dandy-Walker variant, and
fication. Small lipomas might not demonstrate remains in the Dandy-Walker spectrum of
chemical shift artifact. In such cases, fat satura- rhombencephalon roof development disorders
tion can be very helpful in differentiating this (though hydrocephalus is less common). Failure
lesion from other T1 bright lesions. of incorporation of the anterior membranous area
(AMA) into the choroid plexus leads to its persis-
Image with permission from Perkin GD, et al. Atlas tence between the caudal edge of the developing
of Clinical Neurology, 3rd ed. Elsevier, Saunders, 2011.
vermis and the cranial edge of developing choroid
5. c—Septo-optic dysplasia plexus. CSF pulsations cause the AMA to balloon
out into a cyst that displaces the hypoplastic ver-
Coronal T2-weighted MRI shows an absent sep- mis superiorly so that it appears to be rotated in a
tum pellucidum, right optic nerve hypoplasia, counterclockwise fashion. The posterior mem-
and “point-down” appearance of the frontal horns branous area can persist unopened or become
of the lateral ventricles. The main features of patent, accounting for the reportedly variable
septo-optic-pituitary dysplasia include hypoplasia patency of the foramen of Magendie and associa-
or absence of the septum pellucidum, optic nerve tion of hydrocephalus. Global enlargement of the
hypoplasia, and hypothalamic-pituitary dysplasia PF may result from arrested development of the
or dysfunction. SOPD is sporadic in most of the tentorium, straight sinus and torcula, with failure
cases, with no etiologic factor identified although of migration of the straight sinus from the vertex
seen with maternal diabetes and intrauterine to the lambda, possibly because of the abnormal
CMV infection. Clinical presentation is mainly distention of the 4th ventricle. Imaging in the
related to seizures (50%), pituitary dysfunction Dandy-Walker spectrum aims to distinguish a
or visual symptoms such as nystagmus or dilated 4th ventricle from extra ventricular cysts
decreased visual acuity. Ophthalmoscopic exami- and assess hydrocephalus (the foramen of Magen-
nation shows optic nerve hypoplasia, a pale optic die is usually not patent, whereas the foramina of
nerve head, and isolated tortuosity of the retinal Luschka generally are).
vein. Fifty percent of patients might present Image with permission from Loevner L. Brain Imaging:
with seizures. MRI may show corpus callosum Case Review Series, 2nd ed. Elsevier, Mosby, 2009.

Neurosurgery Books Full


www.ketabpezeshki.com 66485438-66485457
536 PART VII PEDIATRIC NEUROSURGERY

7. a—Arachnoid cyst looks virtually identical to an arachnoid cyst pos-


terior and inferior to the vermis. However, the
An arachnoid cyst usually is evident as a CSF isoin- choroid plexus in a Blake pouch cyst at times
tense, non-enhancing, space-occupying cyst with may be identified as being displaced into the cyst
walls that normally are too thin to visualize. It results along its superior wall (under and posterior to the
from splitting of arachnoid membrane with inner vermis), though this can be mimicked by a prom-
and outer leaflet surrounding a cyst cavity which inent inferior vermian vein. No MRI sequence
can undergo progressive expansion due to a ball- allows clear differentiation of a mega cisterna
valve mechanism, fluid secretion by the cyst wall, magna from a Blake pouch cyst, but generally it
or a small osmotic gradient. Arachnoid cysts are appears as a CSF collection with normal non-
filled with CSF and do not communicate with the rotated cerebellar vermis, enlarged 4th ventricle
surrounding subarachnoid space and ventricular (though may be normal at times) and brainstem
system; they usually are not associated with brain compression, without other brain abnormalities.
maldevelopment and may be present anywhere in
the posterior fossa. When the arachnoid cyst is pre- Image with permission from Small JE, Schaefer PW
sent inferior to the vermis, it may compress the infe- (Eds.). Neuroradiology: Key Differential Diagnoses
and Clinical Questions, Elsevier, Saunders, 2013.
riorvermis,butidentification ofwhitemattertoeach
vermian lobule suggests compression rather than 9. d—Mega cisterna magna
vermian hypoplasia seen in Dandy-Walker spec-
trum. Confusion may arise when they are large The cisterna magna is a normal subarachnoid cis-
and located posterior and inferior to the cerebellum. tern located below the cerebellum and behind the
Imaging features suggestive of arachnoid cyst are medulla. Embryologically, it originates from the
similar intensity to CSF, mass effect, large cyst with permeabilization of the Blake's pouch, which allows
obstructive hydrocephalus and scalloping of the CSF flow out of the 4th ventricle via the foramen of
occipital bone. The only definitive diagnosis of Magendie. As such, it is in communication superi-
arachnoid cyst can be made with CT cisternogra- orly with the 4th ventricle and inferiorly with the
phy: arachnoid cysts should not demonstrate peri-medullary subarachnoid spaces. The normal
enhancement (or only after a delay), while cysts that size limit is debated, but a mega cisterna magna
fill with contrast immediately are regarded as diver- refers to a cystic posterior fossa malformation that
ticula of the subarachnoid space. Mega cisterna is characterized by large cisterna magna, an intact
magna usually does not demonstrate mass effect vermis, and absence of hydrocephalus. It is most
on the cerebellum and vermis even if large, and likely the result of cerebellar volume loss and asso-
hydrocephalus is generally absent. Arachnoid cysts ciated with cerebellar insults such as infection,
are not associated with an enlarged 4th ventricle, infarction, and inflammation, as well as chromo-
whereas a persistent Blake's pouch is. It is essential somal abnormalities. The appearance is similar to
to differentiate arachnoid cyst from epidermoid cyst a persistent Blake's pouch except for the consistent
which can be easily done with the use of FLAIR absence of hydrocephalus. The cerebellum and
images and DWI (epidermoid cysts are diffusion brainstem are typically normal. The vermis is
restricting). Most arachnoid cysts are an incidental intact, and there is usually no distortion of the cer-
finding and patients are usually asymptomatic, but ebellum (unlike arachnoid cysts) though occipital
symptoms due to mass effect or hydrocephalus can scalloping may be seen when very large. Patients
be managed with fenestration or cystoperitoneal with mega cisterna magna do not usually have
shunt. any neurological signs of involvement of PF. By
itself mega cisterna magna is asymptomatic, and
Image with permission from Small JE, Schaefer PW is usually discovered incidentally. The incidence
(Eds.). Neuroradiology: Key Differential Diagnoses
and Clinical Questions, Elsevier, Saunders, 2013. is high and represents approximately 50% of all cys-
tic PF malformations. The CSF in the enlarged cis-
8. b—Blake's pouch cyst terna magna freely communicates with the
surrounding CSF spaces and does not obstruct
Blake's pouch is an embryologic midline evagina- CSF circulation and hence hydrocephalus is absent.
tion of the embryonic fourth ventricular roof If there is presence of hydrocephalus mega cisterna
lined by ependyma, glia, and choroid plexus. In magna is not the correct diagnosis and it should
normal early fetal development, this evagination then steer towards the diagnosis of a persistent
along the inferior surface of the vermis ruptures Blake's pouch. There is no role for shunt surgery
and forms the foramen of Magendie that opens even if the cisterna magna is extremely large.
into the subarachnoid space. If the evagination
fails to rupture, the diverticulum continues to Image with permission from Small JE, Schaefer PW
(Eds.). Neuroradiology: Key Differential Diagnoses
expand and eventually forms an uncommon mid- and Clinical Questions, Elsevier, Saunders, 2013.
line posterior fossa cyst (Blake pouch cyst) which

Neurosurgery Books Full


www.ketabpezeshki.com 66485438-66485457
40 CONGENITAL CRANIAL AND SPINAL DISORDERS 537

10. d—Subependymal heterotopia stimulation during awake craniotomy: case presenta-


tion and systematic review of the literature. World Neu-
Heterotopias are focal collections of morpholog- rosurg. 2015;84(2):470-4.
ically normal neurons in abnormal locations sec-
ondary to interrupted neuronal migration from 12. d—Lissencephaly with band heterotopia
the ependyma of lateral ventricles to cortex.
As progenitor cells begin to proliferate, they nor-
The most common clinical presentation is seizure
mally migrate outward from the germinal matrix
disorder in the second or third decade. They are
to form the cortex. Lissencephaly is a severe form
classified by location:
1. Subependymal heterotopia (commonest): of abnormal neuronal migration characterized by
unilateral or bilateral periventricular nodular an absence of gyri with a thickened cortex (agyria)
heterotopia, the latter is X-linked dominant or the presence of few broad fat gyri with a
disorder only seen in females (lethal in males). thickened cortex (pachygyria), both leading to a rel-
2. Subcortical heterotopia: no longer includes atively smooth featureless brain. Classical lissen-
band heterotopia, which is included with cephaly presents with diffuse hypotonia, early
lissencephaly due to common genetic developmental delay, spastic quadriplegia, opistho-
background. tonus, and severe mental retardation. Development
3. Marginal glioneuronal heterotopia— of medically refractory epilepsy at a very early age
overmigration of neurons and glial cells with increasingly complex seizure patterns is very
into the leptomeninges but microscopic common. There are two main types:
1. Type 1 “classical” lissencephaly (lissence-
and not visible on imaging.
phaly with band heterotopia): only four cor-
Imaging shows focal ovoid lesions which match
tical layers.
the gray matter in signal intensity on all the
2. Type 2 lissencephaly (cobblestone cortex):
sequences, lack edema and do not enhance with
no cortical layers.
contrast. Differential diagnosis is subependymal
Two separate mutations, a hemizygous mutation
hamartomas of tuberous sclerosis (irregular, iso-
in the DCX/XLIS gene on chromosome
to hypointense to white matter, may enhance,
Xq22.3q23 and a heterozygous (dominantly
associated tubers) and subependymal metastases.
inherited) mutation in the LIS1 gene on chromo-
Image with permission from Loevner L. Brain Imaging: some 17p13.3 can lead to classical lissencephaly.
Case Review Series, 2nd ed. Elsevier, Mosby, 2009. The DCX mutation also results in subcortical
band heterotopia consists of a discrete extensive
11. b—Focal cortical dysplasia plate or band of gray matter situated between
the cortex and the lateral ventricle. On imaging,
FCD is the most commonly found malformation classical lissencephaly shows a smooth brain with
of cortical development in the surgical series and vertical orientation of the sylvian fissures giving
accounts for approximately 20-50% of all cases the cerebrum a “figure 8” appearance on axial
that have undergone epilepsy surgery. Attempts images, while band heterotopia shows a very
at classifying FCD have been incomplete and characteristic layered pattern from cortex to mid-
unsatisfactory because of the absence of a scienti- line: normal thickness cortex with shallow sulci, a
fically proven etiology. The latest classification variable thickness white matter band, an inter-
system uses a multimodality approach which posing gray matter band (the heterotopic band),
includes a combination of histopathological exam- and a deeper white matter layer extending to
ination, imaging, and genetic findings. The most the ventricular margin or midline.
common clinical presentation is partial seizures
with an otherwise normal neurological examina- Image with permission from Loevner L. Brain Imaging:
tion. The classical imaging features of FCD seen Case Review Series, 2nd ed. Elsevier, Mosby, 2009.
on T2 or FLAIR images is focal cortical thinning
with hyperintensity and volume loss of the under- 13. a—Cobblestone cortex
lying white matter. High resolution thin section
In cobblestone cortex (Type 2 lissencephaly) the
images may show focal blurring of the gray-white
brain surface is irregular because of the presence
matter junction. Rarely, FCD may be associated
of heterotopic tissue which results from over-
with neuroglial tumors, such as dysembryoplastic
migration of glioneural elements. In general,
neuroepithelial tumor and ganglioglioma and may
this malformation shows a cobblestone cortex,
also be seen in association with mesial temporal
dilated ventricles, abnormal white matter, a small
sclerosis.
brainstem, a hypoplastic vermis, and cerebellar
Image with permission from Nowacki A, Seidel K, polymicrogyria. It is associated with eye malfor-
Schucht P, et al. Induction of fear by intraoperative mations as well as congenital muscular dystrophy,

Neurosurgery Books Full


www.ketabpezeshki.com 66485438-66485457
538 PART VII PEDIATRIC NEUROSURGERY

e.g., Walker-Warburg syndrome, Muscle- 15. d—Polymicrogyria


eye-brain disease, Fukuyama congenital muscular
dystrophy. Polymicrogyria (multiple malformed convulsions)
is caused by disturbances in the late stages of neu-
Image with permission from Coley BD. Caffey's Pediat- ronal migration or in the early stages of cortical
ric Diagnostic Imaging, 12th ed. Elsevier, Saunders,
organization (typically between 17 and 25-26 weeks
2013.
of gestation). These disturbances result in the
14. d—Schizencephaly abnormal development of the deep layer of cere-
bral cortex which manifests as multiple small gyri
Schizencephaly (clefted brain) is a brain malfor- separated by small sulci generating an irregular
mation characterized by a full-thickness cerebral bumpy cortical surface. Causes include intra-
cleft. This cleft extends from the pial surface of uterine ischemia, intrauterine infection (CMV or
the cortex to the ependymal lining of the lateral toxoplasmosis), metabolic disorders (e.g., peroxi-
ventricle, and is almost always lined by abnormal somal storage disorders, pyruvate dehydrogenase
gray matter (polymicrogyria). Schizencephaly deficiency), or genetic syndromes (e.g., Aicardi
may result from disruption of any of the three syndrome, DiGeorge syndrome, and Warburg
phases of cortical development (proliferation, Micro syndrome). The most common sites for
migration, and organization), but polymicrogyric PMG are the sylvian cortex (80%) and frontal lobes
cortex surrounding the cleft suggests that it is a dis- (70%) followed by the parietal, temporal and occip-
order of cortical organization, probably secondary ital lobes. Involvement may be either bilateral
to hypoperfusion or ischemic cortical injury. As (60%) or unilateral (40%), focal or diffuse, symmet-
such, it may represent a spectrum where mild dam- ric or asymmetric. MRI shows a bumpy irregular
age results in polymicrogyria while severe damage appearance of the outer surface of the cortex
may involve the deep radial-glial fibers and result because of multiple small gyri, diffuse cortical
in schizencephaly. It may be unilateral (60%) or thickening, and an irregular corrugated appearance
bilateral, and either: of the inner cortical surface (gray-white matter
1. Closed-lip schizencephaly (left hemisphere junction). Most common presentation is seizures
in MRI shown), the margins of the cerebro- (80%), but diffuse polymicrogyria can present with
spinal fluid (CSF)-filled, gray matter lined microcephaly, hypotonia, and infantile seizures
cleft are closely opposed to one another with marked developmental delay (also possibly
along the entire length of the cleft. A small contralateral hemiplegia).
dimple is often seen in the ventricular wall
where the cleft enters. Image with permission from Kanekar S, Gent M. Malfor-
mations of cortical development. Semin Ultrasound CT
2. Open lip schizencephaly (right hemisphere MR 2011;32(3):211-27.
in MRI shown) the cleft margins are widely
separated, lined with polymicrogyria, 16. a—Alobar holoprosencephaly
absent septum pellucidum, dilated ventri-
cles, scalloping and thinning of the inner Holoprosencephaly (HPE) is the most common
vault of the calvarium (direct transmission developmental defect of the forebrain, with a live
of CSF pulsations), and the contralateral birth prevalence of approximately 1 in 10,000.
cerebral cortex may be dysplastic. On The formation of two hemispheres from a single
T2WI, a large vessel may be seen traversing telencephalic vesicle begins around the 37th
the CSF cleft. embryonic day. This division takes place as a
Unilateral closed-lip schizencephaly commonly result of induction by bone morphogenetic pro-
presents with epilepsy, while open lip schizence- tein from the midline roof plate. The etiology
phaly generally presents with microcephaly, of HPE is heterogeneous and might be due to
contralateral hemiparesis, and mental retarda- either genetic causes (Sonic hedgehog pathway
tion. Bilateral schizencephaly manifests as severe most common) or environmental factors, includ-
mental retardation with early onset epilepsy. ing maternal diabetes and exposure to teratogens
Blindness because of optic nerve hypoplasia such as alcohol. SHH is a protein that encodes a
may be seen in 30% of cases. Porencephalic cysts morphogen which mediates notochordal-ventral
are intraparenchymal and lined by white matter. neural tube and development of craniofacial
structures (facial deformities are also seen in asso-
Image with permission from Kanekar S, Gent M. Malfor- ciation with the HPE spectrum). In HPE, the
mations of cortical development. Semin Ultrasound CT
MR 2011;32(3):211-27. forebrain fails to divide into two separate hemi-
spheres; rather, it develops into a single, unpaired

Neurosurgery Books Full


www.ketabpezeshki.com 66485438-66485457
40 CONGENITAL CRANIAL AND SPINAL DISORDERS 539

forebrain called the holoprosencephalon. The crescentic folds and is inserted inferior to the
degree of failure of hemisphere cleavage is classi- petrous ridge, leading to a narrow, funnel-shaped
fied as alobar, semilobar, and lobar HPE. Alobar lower posterior fossa. The falx is usually thin,
HPE is the most common and most severe form, hypoplastic, and may either attach to the superior
resulting in either stillbirth or a very short life- margin of the defect or herniate into the encepha-
span. In alobar HPE, the holosphere remains locele. Because of traction, the cerebral paren-
undivided as a single flattened mass of brain sur- chyma is pulled posteriorly, and nonherniated
rounding a midline holoventricle that is large and brain may assume abnormal positions in the skull.
shaped like an inverted “U” or crescent. It is usu- The anterior commissure, septum pellucidum,
ally associated with severe facial deformities like and fornices are absent in 80% of cases. Hydro-
premaxillary agenesis, cleft lip/palate, ocular cephalus may affect the entire ventricular system
hypotelorism, ethmocephaly (proboscis between or it may be limited to the extracranial portion
the eyes), and in severe cases, cyclopia. On imag- of the ventricles. Other associated anomalies
ing, the holosphere is noted to be displaced in the like cerebellar cortical dysplasia, heterotopias,
most cephalad part of the intracranial cavity. Chiari or Dandy-Walker malformation, and par-
There is complete absence of the interhemi- tial/complete absence of corpus callosum may be
spheric fissure, falx cerebri, and corpus callosum. seen. By definition, Type III Chiari malformation
The gyri recti are also absent. This is associated includes an occipital or cervicooccipital ence-
with aplasia of the olfactory bulbs and optic phalocele with herniation of the medulla, 4th vent-
nerves. The basal ganglia and thalami are fused ricle and cerebellum, and sometimes the occipital
and located in the floor of the holoventricle. lobes (rare).
The sylvian fissures and third ventricle are not
present. A dorsal cyst is frequently seen commu- Image with permission from Coley BD. Caffey's Pediatric
Diagnostic Imaging, 12th ed. Elsevier, Saunders, 2013.
nicating with the monoventricle.
18. e—Type II Chiari malformation
Image with permission from Winn HR. Youman's Neuro-
logical Surgery, vol. 4, 6th ed. Elsevier, Saunders, 2011.
MRI depicting cerebellar vermian displacement,
17. b—Encephalocele downward displacement of cerebellar tonsils and
brainstem, cervical hydrosyrinx, ventriculomegaly,
Cephaloceles are complex neural axis malforma- and pachygyria. (Right) T2-weighted MRI of the
tions which manifest as herniation of the menin- lumbosacral spine depicting hydrosyrinx, tethered
ges and often cerebral tissue through a defect in cord, and unrepaired lumbosacral myelomeningo-
the calvarium. Incidence is 1 in 10,000—1 in cele. In a Chiari II malformation, there is displace-
1000 depending on series. They are usually mid- ment of the cerebellum, part of the brainstem, and
line, but vary in location with occipital (75%) site the fourth ventricle into the cervical canal (below
commonest in Europe and North America. They the basion-opisthion line). A lumbar myelomenin-
are thought to be attributable to nonseparation of gocele is seen in almost all the cases. Several prom-
neural and surface ectoderm leading to defective inent theories exist regarding etiology:
formation of the occipital bone. They can be clas- 1. Traction theory: primary defect is tethering
sified by contents: of the spinal cord which leads to abnormal
1. Meningocele: contains CSF and lined by traction and pulling of the posterior fossa
meninges. contents into the cervical canal.
2. Gliocele: contains CSF and lined by glial 2. Crowding theory: primary defect is in the
tissue. mesodermal development involving the
3. Encephalocele (meningoencephalocele): cranial base rather than neuroectodermal
contains CSF and brain. tissue, resulting in a smaller posterior fossa,
4. Meningoencephalocystocele: contains underdevelopment of the occipital bone,
CSF, brain and ventricles. and basal chondrocranium, which is unable
5. Atretic cephalocele: small nodule of fibrous to accommodate rapidly developing neural
fatty tissue. tissue which herniates through the foramen
Occipital cephaloceles are often large but usually magnum.
covered with normal skin and hair, with herniation 3. Unified Theory/Hydrodynamic Oligo-CSF
of the infra and/or supratentorial structures states that neurulation is the primary defect.
through a narrow pedicle. Herniated brain tissue There is lack of expression of the specific
may be normal, dysplastic, or may show new/old surface molecules required for neural tube
ischemic or hemorrhagic changes because of closure, leading to incomplete occlusion of
strangulation of the blood vessels at the neck of the neural tube and leakage of CSF through
the sac. The tentorium is frequently reduced into the neural tube. Subsequent hypotension

Neurosurgery Books Full


www.ketabpezeshki.com 66485438-66485457
540 PART VII PEDIATRIC NEUROSURGERY

develops within the ventricular system. This 19. a—Chiari I malformation


disruption in the CSF dynamics has an effect
all along the neural tube (from rostral to the
caudal). At the level of lateral ventricle, the
Chiari Malformation: Terminology
germinal matrix is disrupted, leading to mal-
formation of the cortical development. At Accepted terms
the level of the third ventricle, there is
enlargement of massa intermedia because Chiari I Downward herniation of cerebellar
tonsils through the foramen magnum
of extended contact between the thalami.
In the posterior fossa, the rhombencephalic Chiari II Herniation of medulla, 4th ventricle and
vesicle fails to expand, therefore stopping cerebellar vermis through the foramen
magnum associated with
the induction of the perineural mesenchyma myelomeningocele
of the posterior fossa. All of this leads to a
Chiari III Herniation of medulla, 4th ventricle and
smaller posterior fossa, which is unable to cerebellum into an occipitocervical
accommodate the developing rhomben- encephalocele
cephalon. The result is displacement of cer-
Uncommon terms
ebellum and brainstem into the cervical
canal. Chiari IV Primary cerebellar aplasia/hypoplasia
4. Primary defect in the genetic programming without hindbrain herniation
of hindbrain segment and growth associ- Chiari V Herniation of occipital lobe through
ated structures of chondrocranium. foramen magnum (absent cerebellum)

Image with permission from Kaye E. Apnea in a refugee Chiari 0 Symptoms without 5 mm cerebellar
child. Pediatr Neurol 2014;50(1):119-20. tonsillar descent
Chiari 1.5 Downward herniation of cerebellar
tonsils, medulla and 4th ventricle
through the foramen magnum
Chiari II Malformation: Imaging
Location Main Imaging Findings Chiari I malformation is defined as inferior
Posterior 1. Small posterior fossa with low- displacement of the cerebellar tonsils below the
fossa lying tentorium and torcula basion-opisthion line. Tonsillar herniation can
2. Herniation of the cerebellar tonsils be due to multiple causes:
and vermis through widened 1. Intracranial pressure: both intracranial
foramen
3. Brainstem appears pulled down hypertension and intracranial hypotension.
with elongated 4th ventricle 2. Congenital or acquired osseous anomaly or
4. Tectal beaking: inferior colliculus pathology of the posterior fossa and cranio-
elongated posteriorly causing vertebral junction caused by softening of the
angulation and stenosis of aqueduct
resulting in hydrocephalus
skull base, leading to a decrease in the size
5. Cervicomedullary kinking (as of the posterior fossa (e.g., osteogenesis
dentate ligament stops cord imperfecta, Paget's disease, platybasia, or bas-
descending further) ilar invagination).
6. Scalloping of petrous 3. Congenital causes: mesodermal abnormali-
temporal bone
ties leading to a short clivus, reduced height
Supratentorial 1. Obstructive hydrocephalus of the supraocciput, and increased slope of
2. Corpus callosal agenesis and tentorium severely reduce posterior fossa
absent septum pellucidum
3. Stenogyria/polymicrogyria volume with subsequent overcrowding of
4. Fenestration of falx cerebri with the contents and inferior displacement of
interdigitation of gyri the tonsils and/or vermis.
Spinal 1. Myelomeningocele Stridor and hindbrain dysfunction are the most
2. Syringohydromyelia common clinical presentations during the first
3. Klippel-Feil syndrome, scoliosis 3 months in a child with a type I Chiari malforma-
tion. The stridor usually disappears by 3 months of
age. Another common clinical presentation in
children is a headache, which may be generalized,
or can be localized to occipital region. In children,
this headache becomes exacerbated by physical
exercise, straining, or coughing. Other symptoms
related to cerebellar or brainstem dysfunction,

Neurosurgery Books Full


www.ketabpezeshki.com 66485438-66485457
40 CONGENITAL CRANIAL AND SPINAL DISORDERS 541

such as cranial nerve palsy or otoneurologic distur- (occult) spinal dysraphism. Other midline lesions
bances, such as tinnitus, vertigo, and dizziness, to look for include a. The clinical presentation
dysmetria (tremors and down-beating nystag- varies to some degree by age. Younger children
mus). Disorders of motor, sensation, and reflexes tend to present with cutaneous markers that lead
are seen when there is an associated syrinx in the to an evaluation for CSD, but on formal testing,
cervical/thoracic cord. MRI is done to assess most have mild signs of lower motor neuron dys-
whether clinically significant hindbrain hernia- function and abnormalities on urodynamic test-
tion is present and likely cause; normal tonsillar ing. Older children and adolescents tend to
descent below the basion-opisthion line is up to present with either cutaneous stigmata or with
6 mm in 5- to 15-year-olds and up to 5 mm in any- progressive neurologic deficits. Some affected
one over 15 years. Secondary features include a individuals remain asymptomatic into adulthood,
pointed appearance of the cerebellar tonsils, com- at which time they may develop back pain with or
pression of cerebellar cistern (demonstrated by without radiculopathy and perineal dysesthesias.
effacement of vallecula and cisterna magna), retro- Features associated with closed spinal dysraphism
flexion of the odontoid process, compression of are summarized below:
the fourth ventricle, and syringohydromyelia.
Image with permission from Coley BD. Caffey's Pediatric
Diagnostic Imaging, 12th ed. Elsevier, Saunders, 2013. Closed Spinal Dysraphism: Features

20. a—CSF flow study Cutaneous Lipoma (most common sign of


spinal dysraphism), faun's tail (V
shaped patch of long silky hair),
Syringohydromyelia may be seen in the cervical dermal sinuses (hair at ostium),
cord in 30-50% of Chiari I malformations. In true tails, telangiectasias, capillary
healthy patients, an increase in the cerebral blood malformations, aplasia cutis
volume during the cardiac systole causes displace- congenita, naevi, pigmentation
ment of the corresponding amount of CSF from abnormalities and other
subcutaneous masses
the basal cistern into the cervical subarachnoid
space. In diastole with cerebral venous outflow, Neurological Autonomic dyfunction, sphincteric
there is a caudo-cranial diastolic CSF wave. In dysfunction, sensori-motor
deficits in legs, meningitis
hindbrain herniation, syrinx formation may occur (ruptured dermal sinus/cyst); if
due to partial obstruction of the CSF flow chan- symptoms progressive rather than
nel around the craniocervical junction and Ven- static more likely to be due to TCS
turi effect with enhanced intramedullary pulse or compression by mass lesion
pressure causing extracellular fluid accumulation Urological Neurogenic bladder dysfunction
in the distended cord. The syrinx may form any- (TCS) or urogenital
where along the cord, hence whole spine MRI malformations. Diagnosis of
bladder dysfunction is often
is advised in patients with tonsillar herniation not recognized in pre-toilet trained
symptoms, such as sensori-motor weakness of children
the extremities, and/or loss of bladder/bowel con-
Orthopedic Scoliosis, kyphosis, lordosis, leg
trol. MRI CSF flow studies can also be performed length discrepancy, and foot
to study the CSF motion in and around the fora- deformities
men magnum to predict likely benefits of decom- Gastrointestinal Imperforate anus
pressive surgery. In addition, before surgical
intervention, a low tentorium should be evaluated
with MR venography to document a low-lying If present, clinical evaluation for TCS should
torcula and transverse sinus to avoid damage dur- be performed as far as possible given the age of
ing decompressive surgery. the patient. If there is a high clinical suspicion
(or age >3-5 months or bulky lesion precluding
Image with permission from Grainger RG, et al. (Eds.).
Grainger and Allison's Diagnostic Radiology: A Textbook USS) spinal MRI should be performed and
of Medical Imaging, 4th ed. Harcourt, London, 2001. depending on significance of findings, referral
to neurosurgeon or neurologist made. Spinal
21. b—Closed spinal dysraphism (spina bifida US can be performed in those under 3-5 months
occulta) and low clinical suspicion of TCS, but inappro-
priate visualization will necessitate MRI.
This infant displays segmental infantile hemangi-
omas, a dimple, a pseudotail, and a deviated glu- Image with permission from Bolognia J, et al. Dermatol-
teal cleft suggesting the presence of a closed ogy Essentials, Elsevier, 2014.

Neurosurgery Books Full


www.ketabpezeshki.com 66485438-66485457
542 PART VII PEDIATRIC NEUROSURGERY

22. e—Urodynamic testing and plain films/CT for evaluation of bony abnor-
malities. MRI may show a low-lying conus medul-
TCS is stretch-induced dysfunction of the caudal laris but can also be normal. Urodynamic testing
spinal cord and conus, caused by attachment of can detect preclinical urologic dysfunction in chil-
the filum terminale to inelastic structures caudally. dren with CSD. Urodynamic testing is often used
TCS may occur independently (primary) or sec- for preoperative evaluation of children who might
ondary to spinal dysraphism (open or closed), spi- benefit from neurosurgery for tethered cord
nal cord trauma or other pathology. The filum release.
terminale is normally viscoelastic in nature, and
serves to dampen movements of the spine during 23. b—A 6-week-old boy with static bilateral 3/5
flexion and extension, without applying undue leg weakness. Urodynamic testing shows evi-
traction to the moving spinal cord. In TCS, the dence of neurogenic bladder which is being
spinal cord is attached to abnormally inelastic managed with intermittent catheterization.
structures caudally, such as a fibrous or fat-
infiltrated filum, tumor, meningoceles or myelo- Although no clear consensus exists, the main indi-
meningoceles, scars, or septa (as seen in SSCM). cation for neurosurgery is new onset or progres-
This causes the caudal portion of the spinal cord sion of neurologic symptoms related to the CSD
to stretch between the point of tethering and the or TCS. Early neurosurgical intervention also is
dentate ligaments that fix the cord proximally. warranted for severe neonatal symptoms such as
Progressive dysfunction occurs because of bowel obstruction. Additional indications for
repeated extension or flexion of the spine and/or neurosurgical intervention include cases where
differential growth of the vertebral column as the spinal cord is internally exposed to decrease
compared to the spinal cord. The clinical presen- the risk of infection and meningitis, spinal insta-
tation of TCS is broad and varies with age at pre- bility or for pain relief. In contrast, severely dis-
sentation as well as features associated with the abled patients with static deficits related to
underlying cause (e.g., cutaneous lesions of closed CSD are unlikely to benefit from surgery. More
spinal dysraphism). Features commonly described controversial indications for surgical intervention
as a direct cause of cord tethering are: include radiographic demonstration of a tethered
• Neurological—back pain, leg weakness, cord in asymptomatic patients, or abnormal
progressive gait disturbance, calf muscle findings on urodynamic studies in a patient
atrophy, absent deep tendon reflexes, der- with CSD. The rationale for surgery in such
matomal sensory loss. The TCS causes spi- cases is that even infants and children who are
nal dysfunction caudal to the T12/L1spinal asymptomatic or mildly symptomatic may go on
level, and does not explain upper motor to develop progressive and irreversible neuro-
neuron signs logic deficits. Conservative management with
• Urological—neurogenic bladder dysfunction watchful monitoring is also an acceptable
• Orthopedic—progressive scoliosis and foot approach in patients who are asymptomatic or
deformities mildly symptomatic, given the highly variable
In the classic progression of symptoms with TCS, natural history of CSD. In CSD cases associated
children begin to stumble after they have learned with cord tethering, surgery involves removal of
to walk normally. Then they start dribbling urine any anatomic structure that is acting to tether
after having achieved successful toilet training. the spinal cord, and may include transection of
Later, they develop musculoskeletal signs and the filum, resection of transitional lipoma, lysis
symptoms; common findings include foot drop, of adhesions, and excision of dermal sinus tracts.
painless sores, and scoliosis. Older children will In addition, some data suggest that fashioning a
often complain of back pain exacerbated by exer- large intradural compartment, with duraplasty if
cise, while younger children tend to have increased needed, is associated with a reduced risk of devel-
irritability and refuse to perform certain activities oping arachnoid adhesions and cord Asymptom-
and movements, though without a frank complaint atic patients with CSD who do not have surgery
of pain. Back pain, leg pain, and scoliosis are the still require close monitoring to watch for the
primary symptoms of TCS in adults, and these onset of neurologic, genitourinary or gastrointes-
may be difficult to distinguish from other more tinal symptoms, especially with respect to incon-
common causes of chronic back pain. The earliest tinence or constipation. Patients who have
sign of motor dysfunction in the older child and surgery for CSD should remain under close mon-
adult with TCS is usually weakness of ankle dorsi- itoring because of the risk of future worsening,
flexion. Sensory symptoms usually are patchy and which can occur with spinal cord retethering or
vague, especially when related to TCS. Imaging progression of a preexisting syrinx. The earliest
modality of choice is MRI, with US playing a role indication of retethering is usually urologic symp-
prior to ossification of posterior elements (<4 m) toms. In addition, non-neurological symptoms

Neurosurgery Books Full


www.ketabpezeshki.com 66485438-66485457
40 CONGENITAL CRANIAL AND SPINAL DISORDERS 543

may continue to progress postoperatively, as can region and a Chiari II malformation is invitable
be seen with preexisting scoliosis and pain. Uro- due to CSF leak. Surgical repair and closure of
dynamics are generally considered to be a good the defect is required as soon as possible. While
monitoring tool for both nonoperative patients imaging is not necessarily required before
and postoperative patients, and particularly for closure, MR imaging should be performed to
early detection of cord retethering. assess for associated pathology at other levels
(e.g., split cord malformation, lipoma, dermoid/
FURTHER READING epidermoid, Chiari II malformation) and is cru-
Khoury C. Closed spinal dysraphism: pathogenesis and types. cial in patients presenting with progressive neu-
Uptodate, 2015. rological deficits postoperatively (e.g., exclude
cord ischemia, arachnoid cyst, scar tethering).
24. e—Myelomeningocele
Image with permission from Carlson BM. Human
Open spinal dysraphism (OSD) is a clinical diag- Embryology and Developmental Biology, 5th ed.
nosis and a neurosurgical emergency. Myelome- Elsevier, Saunders, 2014.
ningocele accounts for 99% of OSD, myelocele is
rare, and hemimyelocele and hemimyelomenin-
gocele extremely rare. In all cases there is defec-
ANSWERS 25–33
tive closure of the primary neural tube (primary
neurulation) resulting in the neural placode being Additional answers 25–33 available on
exposed through a midline skin defect on the ExpertConsult.com
back. In myelomeningocele, neural placode pro-
trudes above skin surface, whereas in myelocele,
the placode is flush with skin surface. The abnor-
mality is most commonly found at lumbosacral

EMI ANSWERS
34. 1—c, Dermal sinus; 2—h, Hemimyelomeningocele; 3—i, Lipomyelocele; 4—p, Persistent terminal
ventricle; 5—o, Neurenteric cyst

Spinal Dysraphism: Classification


Type Description Stage Affected

Open spinal dysraphism


Myelomeningocele Abnormal spinal cord exposed in the midline via defect Primary neurulation
in the dura, posterior vertebral elements, facia and skin.
Defect produces Chiari II malformation in all cases and
hydrocephalus. Neurosurgical emergency due to CSF
leak and hydrocephalus
Myelocele As myelomeningocele, but subarachnoid space ventral
to placode is not expanded resulting in a lesion flush
with skin
Hemimyelocele Myelocele affecting one of two hemicords in split cord Gastrulation and primary
malformation neurulation

Hemimyelomeningocele Myelomeningocele affecting one of two hemicords in


split cord malformation
Closed spinal dysraphism with subcutaneous mass
Lipomyelomeningocele A subcutaneous lipoma tethers the spinal cord (lipoma- Primary neurulation
placode interface) outside the spinal canal via defect in
vertebral elements
Lipomyelocele A subcutaneous lipoma tethers the spinal cord (lipoma-
placode interface) inside the spinal canal via defect in
vertebral elements
Continued on following page

Neurosurgery Books Full


www.ketabpezeshki.com 66485438-66485457
544 PART VII PEDIATRIC NEUROSURGERY

Spinal Dysraphism: Classification (Continued)


Type Description Stage Affected
Meningocele Posterior—Herniation of CSF-filled meningeal sac Unknown
through posterior bony defect (may contain nerve roots
but not spinal cord). Dura may have defect but
overlying skin is intact. Anterior meningoceles are part
of the caudal regression syndrome
Terminal Expansion of terminal central canal (syringocele) Secondary neurulation
myelocystocele surrounded by a meningocele with no communication and retrogressive
between two components. Syringocele is caudal to the differentiation
meningocele and herniates through a wide spina bifida
to cause a intergluteal cystic swelling

Closed spinal dysraphism without subcutaneous mass


Simple dysraphic state
Posterior spina bifida Simple defect of fusion of posterior neural arch of a
vertebra, usually at L5 or S1
Intradural lipoma Lipomas originate from early disjunction between Primary neurulation
neuroectoderm and ectoderm—the surrounding
mesenchyme creeps between and adheres to primitive
ependyma which induces it to transform into fat
Filum terminale lipoma Fibrolipomatous thickening of the filum terminale Secondary neurulation
and retrogressive
differentiation
Tight filum terminale Short, hypertrophic filum terminale producing
tethering and impaired ascent of conus
Persistent terminal Small ependymal lined cavity in the conus medullaris
ventricle which may undergo cystic dilatation
Complex dysraphic state
Split cord malformation Type 1—bony midline septum dividing cord into two Gastrulation (notochordal
separate cords in own dural sleeves integration)
Type 2—single thecal sac containing split cords with an
intervening fibrous band
Neurenteric cyst Cysts usually in intradural extramedullary plane of
endodermal origin, usually lined with GI or respiratory
epithelium
Dorsal enteric fistula Most severe. Cleft connecting the bowel with dorsal
skin surface through the prevertebral soft tissues, split
vertebral bodies and spinal cord, neural arch,
subcutaneous tissue
Dermal sinus Epithelium-lined fistula which extends from skin to
meninges within the spinal canal, possibly opening into
the subarachnoid space or connecting to filum
terminale, lipoma and may also be associated with a
spinal dermoid
Caudal regression Agenesis of spinal column, imperforate anus, genital Gastrulation (notochordal
syndrome anomalies, bilateral renal dysplasia, pulmonary formation)
hypoplasia and lower limb abnormalities. Syndromic
associations include OEIS, VACTERL and Currarino triad
Segmental dysgenesis Segmental agenesis of thoracolumbar spine: vertebral,
segmental spinal cord hypoplasia/aplasia with caudal
bulkiness, congenital paraparesis, lower limb
deformities

FURTHER READING 35. 1—j, Porencephalic cyst


Tortori-Donati P, Rossi A, Cama A. Spinal dysraphism: a
review of neuroradiological features with embryological cor- Porencephalic cysts are due to an encephaloclastic
relations and proposal for a new classification. Neuroradiol- insult (e.g., intrauterine infections and ischemia),
ogy. 2000;42(7):471-91. lined by white matter, and communicate with
Neurosurgery Books Full
www.ketabpezeshki.com 66485438-66485457
40 CONGENITAL CRANIAL AND SPINAL DISORDERS 545

the ventricles and/or the subarachnoid space. (this depends on the age at which the insult
They commonly become symptomatic in the 1st occurred). Importantly the cyst is not lined by
year of life with evidence of spasticity, seizures, gray matter, helpful in distinguishing them
and developmental delay. Imaging Well-defined from arachnoid cysts and schizencephaly. Typi-
CSF cyst (T1 hypointense, T2 hyperintense, cally the cyst seen to communicate with the ventri-
FLAIR dark, no restricted diffusion, no enhance- cles and/or the subarachnoid space and there is no
ment) and often corresponds to a vascular terri- mass effect. Management is supportive.
tory. The cyst is lined by white matter, which
may or may not demonstrate evidence of gliosis 2—k, Schizencephaly

Neurosurgery Books Full


www.ketabpezeshki.com 66485438-66485457
CHAPTER 41

PEDIATRIC NEUROSURGERY:
GENERAL AND HYDROCEPHALUS
SINGLE BEST ANSWER (SBA) QUESTIONS
1. Which one of the following statements 5. A 9-year-old child with VP shunt comes with
LEAST accurately describes CSF production? headache and lethargy. CT head, blood,
a. Neonates produce CSF at a rate of 25 ml/day unchanged and shunt series are normal. Risk
b. Infants have a total CSF volume of 25 ml of CSF infection secondary to percutaneous
c. Adults produce CSF at a rate of approxi- shunt tap is which one of the following?
mately 0.5 ml/min a. 0.0002%
d. Ventricles contain 25 ml of CSF b. 0.002%
e. Raised intracranial pressure does not affect c. 0.02%
formation of CSF at the choroid plexus d. 0.2%
e. 2%
2. The estimated frequency of hydrocephalus in
6. Which one of the following statements
children is which one of the following?
a. 1 in 100 regarding the management of pediatric
hydrocephalus is LEAST accurate?
b. 1 in 250
a. Preoperative antibiotics reduces the risk
c. 1 in 500
of subsequent shunt infection in patients
d. 1 in 1500
with hydrocephalus
e. 1 in 5000
b. There is insufficient evidence to recom-
mend the routine use of endoscopic guid-
3. An infant has a head CT performed because of ance in ventricular shunt placement
a large head and failure to thrive. The diagnosis c. There is insufficient evidence to recom-
of hydrocephalus is made. Congenital hydro- mend occipital over frontal point of entry
cephalus is most commonly caused by which for ventricular catheters
one of the following maternal infections? d. There is no clear advantage for one shunt
a. Toxoplasmosis valve type over another
b. Rubella e. Antibiotic-impregnated shunt tubing may
c. Influenza be associated with a lower risk of shunt
d. HIV infection compared to conventional
e. Group B Streptococci silver-impregnated hardware

4. An 18-month-old child presents with poor 7. Which one of the following statements regard-
feeding, and motor delay. Papilledema is pre- ing the choice of endoscopic third ventriculost-
sent on fundoscopy. CT head shows a het- omy or ventriculoperitoneal shunt CSF
erogeneously enhancing midline posterior diversion in children is LEAST accurate?
fossa mass suspicious of medulloblastoma a. ETV is the standard of care for post-
causing severe obstructive hydrocephalus. hemorrhagic hydrocephalus in infants
This child’s risk of developing chronic (<24 months old)
hydrocephalus at 6 months postoperatively b. VP shunt is usually most appropriate for
is which one of the following? communicating hydrocephalus
a. 7% c. ETV should be considered first for hydro-
b. 19% cephalus due to congenital aqueduct stenosis
c. 42% d. Late ETV failure (2 years) is less common
d. 56% than late VP shunt failure
e. 80% e. Majority of ETV failure occurs in the first
f. 97% 3 months
546
Neurosurgery Books Full
www.ketabpezeshki.com 66485438-66485457
41 PEDIATRIC NEUROSURGERY: GENERAL AND HYDROCEPHALUS 547

8. An 8-month-old previously healthy child 10. Which one of the following statements
presents with macrocephaly and delayed regarding the management of post-
milestones. MRI is shown below. Which hemorrhagic hydrocephalus LEAST correct:
one of the following is his most likely ETV a. Ventriculosubgaleal shunts increase the
success score? need for daily CSF aspiration compared
with ventricular access devices
b. The use of prophylactic serial lumbar
puncture is not recommended as it does
not reduce the need for shunt placement
or avoid the progression of hydrocephalus
in premature infants compared to
observation alone
c. Intraventricular thrombolytic agents are
not recommended as methods to reduce
the need for shunt placement in prema-
ture infants with PHH
d. Acetazolamide and furosemide are not
recommended as methods to reduce the
need for shunt placement in premature
infants with PHH
e. There is insufficient evidence to recom-
mend a specific infant weight or CSF
parameter to direct the timing of shunt
placement in premature infants with PHH
a. 40 f. There is insufficient evidence to recom-
b. 50 mend the use of endoscopic third ventri-
c. 60 culostomy (ETV) in premature infants
d. 70 with PHH
e. 80
11. Which one of the following statements
9. A preterm neonate is born at 28 weeks gesta- regarding the treatment of CSF shunt infec-
tion with a birth weight of 1000 g. Cranial US tion is most accurate?
performed in the first 24 h of birth due to a a. Evidence recommends shunt externaliza-
bulging fontanelle and episodes of apnoea tion over complete shunt removal as the
revealed Papile grade III germinal matrix preferred surgical strategy in manage-
hemorrhage with hydrocephalus. Which ment of CSF shunt infection
one of the following statements is most b. Evidence recommends the combination
accurate? of intrathecal and systemic antibiotics
a. The majority of intraventricular hemor- for patients with CSF shunt infection
rhage in low birthweight preterm infants when the infected shunt hardware cannot
is Grade III and IV be fully removed
b. The proportion of infants with post- c. Evidence recommends the combination
hemorrhagic hydrocephalus who require of intrathecal and systemic antibiotics
permanent shunt placement is lower in for patients with CSF shunt infection
preterm than term births when caused by gram-negative organisms
c. Sunset phenomenon consists of impaired d. Evidence recommends supplementation
downgaze of antibiotic treatment with partial (exter-
d. Term infants usually present with sponta- nalization) or with complete shunt hard-
neous apnea or bradycardia in the first ware removal
24 h after IVH e. Evidence recommends the combination
e. The rate of intraventricular hemorrhage of intrathecal and systemic antibiotics
in both term and preterm babies is 30%, for patients with CSF shunt infection
but the mean Papile grade is higher in when the shunt must be removed and
preterms immediately replaced

Neurosurgery Books Full


www.ketabpezeshki.com 66485438-66485457
548 PART VII PEDIATRIC NEUROSURGERY

12. A 4-month-old child with a history of Escher- 14. A 16-year-old child presents with cough
ichia coli neonatal meningitis returns with related headache and sensory changes in
poor feeding and a bulging fontanelle. CT her hands. Neurological examination is oth-
head is done which shows hydrocephalus erwise normal and there is no papilledema.
and MRI is done for surgical planning. MRI head and spine show a Chiari I malfor-
Which one of the following is likely to be mation with 8 mm tonsillar decent and a
the surgical goal in this patient? small cervical syrinx at C3//4. Which one
of the following statements regarding surgi-
cal management is most accurate?
a. Durotomy and tonsillectomy would be
mandatory in this case
b. C1-C3 laminectomy should also be per-
formed in this case to treat the syrinx
adequately
c. Due to minimal tonsillar descent, primary
treatment should be directed towards the
syrinx (e.g. syringostomy) if her symp-
toms progress
d. Lumbar puncture should be performed
to exclude neurological cause of her
symptoms
e. Intraoperature ultrasound is less accurate
at predicting those who do not need dur-
otomy in the presence of tonsillar descent
below C1 lamina

15. An 18-month old is treated for meningitis.


After initially improvement, becomes less
responsive on day 10 and imaging is per-
formed. Which one of the following is
a. Complex VP shunt with at least two prox- LEAST appropriate?
imal catheters
b. Single VP shunt following endoscopic
fenestration of loculations
c. ETV and endoscopic fenestration of
loculations
d. Endoscopic aqueductoplasty
e. Ventriculocisternal shunt

13. Which one of the following statements


regarding Lundberg waves seen during ICP
monitoring is most accurate?
a. They represent aortic valve closure seen
in normal patients
b. They are dicrotic representing arterial
pulsation seen in normal patients
c. B wave is mean ICP with pressure
20-50 mmHg lasting 30 s to a few minutes
during sleep
d. A waves represent the cyclic variation in
systolic pressure due to oscillations in
autoregulation
e. C waves suggest ICP exceeding limits of
cerebral compliance

Neurosurgery Books Full


www.ketabpezeshki.com 66485438-66485457
41 PEDIATRIC NEUROSURGERY: GENERAL AND HYDROCEPHALUS 549

a. Subduroperitoneal shunt i. Proximal catheter obstruction


b. Percutaneous needle drainage j. Shunt site infection
c. Transventricular tap k. Silicone allergy
d. External subdural drain
e. Burr hole drainage For each of the following descriptions, select the
most appropriate answers from the list above.
Each answer may be used once, more than once
QUESTIONS 16–25 or not at all.
Additional questions 16–25 available on 1. A 7-year-old boy with a VP shunt in situ
ExpertConsult.com
presents to the emergency department
after being found by his parents uncon-
scious in his room after 2 days of a headache
and vomiting. His pupils are 5 mm and
sluggish bilaterally, and he is hypertensive
EXTENDED MATCHING ITEM (EMI) with bradycardia of 38 bpm. Shunt reser-
QUESTIONS voir tap is unable to aspirate CSF.
2. An 8-month-old infant presents 1 month
26. Congenital and Genetic hydrocephalus: after VP shunt for post-hemorrhagic
a. Arachnoid cyst hydrocephalus with swelling over abdomi-
b. Aqueduct stenosis nal wound, without erythema or heat. XR
c. Chiari malformation shunt series shows that the shunt is in con-
d. Dandy-Walker tinuity and a bird’s nest arrangement of the
e. Foramen of Monro atresia peritoneal catheter.
f. Joubert syndrome 3. A 9-year-old boy presents with a fixed pres-
g. Neural tube defect sure VP shunt inserted for symptomatic
h. Posterior fossa defect aqueduct stenosis which was inserted sev-
i. VACTERL-H eral months ago. He has been experiencing
j. Walker-Warburg syndrome intermittent headaches on standing for the
k. X-linked hydrocephalus last few weeks, and feeling nauseous. Now
the headaches have become permanent
For each of the following descriptions, select the even when lying flat and he is finding it dif-
most appropriate answers from the list above. ficult to keep food down. Shunt reservoir is
Each answer may be used once, more than once filling adequately.
or not at all.
1. An infant presents with the most common 28. Neonatal meningitis:
heritable non-syndromic from of hydro- a. Citrobacter koseri
cephalus, usually with mutation in LCAM1 b. Coagulase negative staphylococcus
gene. c. Escherichia coli
2. An infant presents with hypotonia, sleep d. Hemophilus influenza
apnea, developmental delay. MRI head e. Listeria monocytogenes
demonstrates midline cerebellar vermis f. Neisseria meningitidis
hypoplasia with deepened interpeduncular g. Proprionobacter
fossa (Molar tooth sign) and thick elongated h. Staphylococcus epidermis
superior cerebellar peduncles. i. Streptococcus pneumonia
j. Tuberculosis
27. Shunt complications:
a. Abdominal pseudocyst For each of the following descriptions, select the
b. Acute overdrainage most appropriate answers from the list above.
c. Chronic overdrainage Each answer may be used once, more than once
d. CSF ascites or not at all.
e. CSF infection 1. Highest risk of associated brain abscess
f. Disconnection 2. Multiple tiny parenchymal nodules and
g. Erosion into abdominal viscus basal meningeal enhancement
h. Migration of distal catheter

Neurosurgery Books Full


www.ketabpezeshki.com 66485438-66485457
550 PART VII PEDIATRIC NEUROSURGERY

SBA ANSWERS
1. b—Infants have a total CSF volume of 25 ml aqueduct stenosis are also in this group, with Toxo-
plasma being the most frequent intrauterine infec-
Infants have a total CSF volume of 50 ml (150 ml in tion associated with it, and viral causes including
adults), and in both infants and adults there is 25 ml mumps and CMV.
of CSF in the ventricles. Neonates produce CSF at a
rate of 25 ml/day, which increases to two thirds of 4. e—80%
the adult capacity as an infant. Adults produce
CSF at a rate of 0.35 ml/min ¼ 20 ml/h (approxi- Propensity of midline, and mostly posterior fossa,
mately 500 ml/day). tumors in children causes a high incidence of
hydrocephalus. Preoperative shunting is no longer
FURTHER READING routine, and most surgeons opt to remove the
Hdeib A, Cohen, AR. Hydrocephalus in Children and Adults. tumor and monitor for the development of hydro-
In: Ellenbogen RG, editor. Principles of Neurological Sur- cephalus. External ventricular drain insertion at
gery, 3rd ed. Saunders, Elsevier; 2012. the time of tumor removal is common for tumors
2. c—1 in 500 within the fourth ventricle but may be avoided in
cerebellar hemispheric tumors. While preopera-
It is estimated that hydrocephalus may occur with tive ETV can reduce hydrocephalus and does
the frequency of 1 in every 500 children. Causes not burden the child with hardware, some of
include genetic (e.g. X-linked aqueduct stenosis), these third ventriculostomies may be unnecessary
other congenital causes (e.g. myelomeningocele because a proportion of children will not develop
and Chiari malformation) and more common progressive hydrocephalus after tumor removal.
acquired causes such as intraventricular The validated Canadian preoperative prediction
hemorrhage, trauma, tumors, and infection. rule for hydrocephalus in children with posterior
fossa neoplasms predicts the risk of developing
FURTHER READING postoperative hydrocephalus based on age, papille-
Flannery AM, et al. Pediatric hydrocephalus: systematic litera- dema, severity of hydrocephalus, metastatic dis-
ture review and evidence-based guidelines. Part 1: Introduction ease, and estimated preoperative tumor type.
and methodology. J Neurosurg Pediatr 2014;14 Suppl. 1:3-7. Evaluating these factors allows a more informed
3. a—Toxoplasmosis discussion with patients and families and possibly
the selective use of endoscopic third ventriculost-
Congenitally acquired TORCHES infections omy before tumor surgery. Following surgery for
(Toxoplasma, Other, Rubella, CMV, HErpes, tumors in the lateral ventricle that are associated
Syphilis) are known to cause intrauterine growth with hydrocephalus, the surgical tract may lead
restriction, microcephaly, intracranial calcifications, to postoperative decompression of the hydroceph-
conjunctivitis, hearing loss, rash, hepatosplenome- alus into the subdural space. When this collection
galy and thrombocytopenia. Post-infectiouscauses persists as a subdural hygroma, it may require
of intrauterine and neonatal hydrocephalus due to treatment with a subdural shunt.

FURTHER READING
Kestle JD. Hydrocephalus in children: approach to the
Canadian Preoperative Prediction Rule for
Hydrocephalus in Children with Posterior patient. In: Winn RH, editor. Youmans Neurological Surgery,
Fossa Neoplasms 6th ed. Saunders, Elsevier; 2011.

Risk of Riva-Cambrin J, Lamberti-Pasculli M, Armstrong D, et al.


Hydrocephalus at 6 The validation of a preoperative prediction score for chronic
Predictor Score Months by Score hydrocephalus in pediatric patients with posterior fossa
Age <2 3 0 points ¼ 7%, tumours. J Neurosurg 2005;102:A798.
Papilledema 1 1 ¼ 11%, 2 ¼ 19%,
Moderate to severe 2 3 ¼ 29%, 4 ¼ 42%,
5. d—0.2%
hydrocephalus 5 ¼ 56%, 6 ¼ 69%,
present 7 ¼ 80%, 8 ¼ 88%, In a recent study, of 542 shunt taps performed in
Cerebral metastasis 3 9 ¼ 93%, 10 ¼ 96% 266 children using a standard protocol (by neuro-
Likely Diagnosis of 1 surgical department personnel only), there were
medulloblastoma,
ependymoma, or
14 infected shunts but only one child (with red-
dorsally exophytic ness over shunt track) whose first CSF tap was
brainstem glioma negative but then returned 11 days later with
fever and irritability and whose repeat CSF tap

Neurosurgery Books Full


www.ketabpezeshki.com 66485438-66485457
41 PEDIATRIC NEUROSURGERY: GENERAL AND HYDROCEPHALUS 551

grew Staphylococcus aureus. They state: “Assuming 7. a—ETV is the standard of care for post-
that this patient's shunt infection was secondary hemorrhagic hydrocephalus in infants
to the shunt tap, the infection rate would be 1 (<24 months old)
(0.18%) in 542. If one removes the 14 infected
shunts, because theoretically a shunt tap leading The optimum treatment for hydrocephalus is con-
to another infection might be masked by the anti- troversial. Aside from obstructive hydrocephalus
biotics used to treat the first diagnosed infection, in children older than 2 years and adults, in whom
the rate would be 1 (0.19%) in 528. Because there ETV is often used, VPS placement remains the
were often multiple taps in the same patient, the standard of care. But the indications for perform-
incorporation of a time separation between the ing ETV have recently broadened to communi-
first and second tap can be used to ensure that cating types of hydrocephalus and the success of
the second tap was indeed negative. If one ETV in young infants for all causes of hydroceph-
assumes a reasonable time interval to be 3 months, alus has been increased by the addition of choroid
that would eliminate 162 taps, changing the inci- plexus cauterization. No completed randomized
dence to 1 (0.27%) in 366.” trials have compared endoscopic and shunt treat-
ment for pediatric hydrocephalus, and the Inter-
FURTHER READING national Infant Hydrocephalus Study is ongoing
Spiegelman L, Asija R, Da Silva SL, Krieger MD, McComb (direct comparisons of VPS versus ETV for
JG. What is the risk of infecting a cerebrospinal fluid- infants [24 months of age] with aqueduct steno-
diverting shunt with percutaneous tapping? J Neurosurg sis). Another randomized prospective trial is cur-
Pediatr 2014;14(4):336-9. http://dx.doi.org/10.3171/2014.7. rently underway at CURE Children’s Hospital
PEDS13612. Epub 2014 Aug 8. PubMed PMID: 25105511.
of Uganda to compare ETV plus CPC versus
VPS alone in infants younger than 6 months of
6. e—Antibiotic-impregnated shunt tubing may
age with post-infectious hydrocephalus. The best
be associated with a lower risk of shunt infec-
criteria to determine optimum hydrocephalus
tion compared to conventional silver-
treatment are not known, since while acute symp-
impregnated hardware
toms may be alleviated ventriculomegaly may not
FURTHER READING completely resolve and continue to cause subtle
Baird LC, Mazzola CA, Auguste KI, Klimo P Jr, Flannery AM; white matter injury or impair cognitive outcome,
Pediatric Hydrocephalus Systematic Review and Evidence- hence brain volume may correlate better with cog-
Based Guidelines Task Force. Pediatric hydrocephalus: sys- nitive outcome better than CSF volume.
tematic literature review and evidence-based guidelines. Part
5: Effect of valve type on cerebrospinal fluid shunt efficacy. FURTHER READING
J Neurosurg Pediatr 2014;14 Suppl. 1:35-43. http://dx.doi. Kahle KT, et al. Hydrocephalus in children. Lancet 2015,
org/10.3171/2014.7.PEDS14325. Review. PubMed PMID: http://dx.doi.org/10.1016/S0140-6736(15)60694-8.
25988781.
Limbrick DD Jr, Baird LC, Klimo P Jr, Riva-Cambrin J, Flan-
Kemp J, et al. Pediatric hydrocephalus: systematic literature nery AM. Pediatric Hydrocephalus Systematic Review and
review and evidence-based guidelines. Part 9: Effect of ven- Evidence-Based Guidelines Task Force. Pediatric hydroceph-
tricular catheter entry point and position. J Neurosurg Pediatr alus: systematic literature review and evidence-based guide-
2014;14 Suppl. 1:72-6. PubMed PMID: 25988785. lines. Part 4: Cerebrospinal fluid shunt or endoscopic third
ventriculostomy for the treatment of hydrocephalus in
Flannery AM, et al. Pediatric hydrocephalus: systematic litera- children. J Neurosurg Pediatr 2014;14 Suppl. 1:30-4. http://
ture review and evidence-based guidelines. Part 3: Endoscopic dx.doi.org/10.3171/2014.7.PEDS14324. Review. PubMed
computer-assisted electromagnetic navigation and ultrasonog- PMID: 25988780.
raphy as technical adjuvants for shunt placement. J Neurosurg
Pediatr 2014;14 Suppl. 1:24-9. PubMed PMID: 25988779. 8. d—70
Klimo P Jr, et al. Pediatric hydrocephalus: systematic literature
review and evidence-based guidelines. Part 6: Preoperative Most ETV failures occur within the first 6 months
antibiotics for shunt surgery in children with hydrocephalus: of surgery. ETV success score is a simple means
a systematic review and meta-analysis. J Neurosurg Pediatr to predict 6-month success rate of ETV, with
2014;14 Suppl. 1:44-52. PubMed PMID: 25988782. scores ranging from 0 (meaning virtually no
chance of ETV success) to 90 (meaning a roughly
Klimo P Jr, et al. Pediatric hydrocephalus: systematic litera-
90% chance of ETV success). ETVSS is calcu-
ture review and evidence-based guidelines. Part 7:
Antibiotic-impregnated shunt systems versus conventional
lated as Age Score + Etiology Score + Previous
shunts in children: a systematic review and meta-analysis. J Shunt Score from the table above, and has
Neurosurg Pediatr 2014;14 Suppl. 1:53-9. Review. PubMed demonstrated internal and external validity.
PMID: 25988783.

Neurosurgery Books Full


www.ketabpezeshki.com 66485438-66485457
552 PART VII PEDIATRIC NEUROSURGERY

term infants have no or transient ventricular dila-


ETV Success Score tion in the period immediately after IVH but
Previous
significant proportion may eventually require a
Score Age Etiology Shunt shunt, usually during the first year of life.
0 <1 month Post-infectious Yes FURTHER READING
10 1 month to – No Mazzola CA, et al. Pediatric hydrocephalus: systematic litera-
<6 months ture review and evidence-based guidelines. Part 2: Manage-
ment of posthemorrhagic hydrocephalus in premature
20 – Myelomeningocele, –
infants. J Neurosurg Pediatr 2014;14 Suppl. 1:8-23.
IVH, non-tectal tumor
30 6 months Aqueduct stenosis, – Hu YC, et al. Infantile posthemorrhagic hydrocephalus. In:
to 1 year tectal tumor, other Winn RH, editor. Youmans Neurological Surgery, 6th ed.
Saunders, Elsevier; 2011.
40 1 year to – –
<10 years
10. a—VSG shunts increase the need for daily
50 >10 years – –
CSF aspiration compared with VADs

Preterm infants with higher grade (II/IV) IVH


Image with permission from Kahle KT, Kulkarni AV, are also usually the smallest and have the most
Limbrick DD Jr, Warf BC. Hydrocephalus in children, comorbidities; they are poor surgical candidates
Lancet 2015. pii:S0140-6736(15)60694-8. due to poor nutrition, immature immune system,
and other comorbidities (e.g. anemia of prematu-
FURTHER READING
rity treated with erythropoietin, red blood cell,
Kulkarni AV, Riva-Cambrin J, Browd SR. Use of the ETV Suc-
cess Score to explain the variation in reported endoscopic third and platelet transfusion). Interventions for symp-
ventriculostomy success rates among published case series of tomatic hydrocephalus are offered in a stepwise
childhood hydrocephalus. J Neurosurg Pediatr 2011;7(2):143-6. progression to identify patients in whom perma-
nent CSF diversion is not required. This process
9. b—The proportion of infants with post- allows surgery to be avoided in all but the few who
hemorrhagic hydrocephalus who require per- definitely need it. Lumbar punctures are useful
manent shunt placement is lower in preterm for drawing off CSF as an immediate treatment
than term births for elevated intracranial pressure or CSF sam-
pling in infants with communicating PHH, but
The incidence of IVH increases inversely with routine use of LP (to reduce CSF blood load)
decreasing birthweight or EGA. In extremely does not eliminate the need for a VP shunt hence
low birthweight preterm infants who survived should not be performed. Diuretic therapy,
and could be followed up, 33% had a history of including acetazolamide and furosemide, is not
intraventricular hemorrhage (IVH) of which effective in this population and may increase
40% was Grade III or IV but eventually only 3% the risk of nephrocalcinosis and other complica-
required VP shunt placement for post- tions, and intraventricular streptokinase is also
hemorrhagic hydrocephalus (PHH). In term not effective. Temporary surgical approaches
babies, 15% had peri/intraventricular hemorrhage (e.g. ventricular access device, ventriculosubga-
and nearly all have grade I/II IVH. In preterm leal shunt) should be used in those that do not
babies, most IVH occurs within the first 72 h of show stabilization of the head circumference on
life and is diagnosed by bedside cranial US due a reasonable growth curve and stabilization/
to deterioration over several days. Infants with reduction of ventriculomegaly on cranial US. In
IVH should be observed closely with daily mea- these cases, temporary shunts are used to treat
surement of the occipitofrontal circumference symptomatic hydrocephalus and allow the infant
(OFC): increase in growth rate from 0.5 to 1 cm/ to grow and recover from other complications of
day for 2-3 consecutive days often suggests symp- prematurity before undergoing definitive surgical
tomatic hydrocephalus. Other features include therapy. Ventricular taps are reserved for life-
bulging fontanelle and splayed sutures, episodes threatening emergencies because they are associ-
of spontaneous apnea or bradycardia, refractory ated with a markedly increased risk of infection
seizures, lethargy, and impaired upward gaze and loculated hydrocephalus in childhood. EVDs
(“sunset” phenomenon). Term infants with IVH have a much higher risk of complications in pre-
typically present with lethargy or seizures, but a term infants than in older neurosurgical patients.
subset of infants presents with distress at birth, Definitive surgery (permanent ventriculoperito-
and the remainder often present within the first neal shunts) are required in only a small minority
week. In contrast to ELBW preterm infants, many of preterm infants who suffer IVH. These

Neurosurgery Books Full


www.ketabpezeshki.com 66485438-66485457
41 PEDIATRIC NEUROSURGERY: GENERAL AND HYDROCEPHALUS 553

children are prone to cerebral palsy, epilepsy, FURTHER READING


cognitive delay, and behavioral abnormalities Tamber MS, et al. Pediatric hydrocephalus: systematic litera-
regardless of whether the CSF diversion is effec- ture review and evidence-based guidelines. Part 8: Manage-
tive. Permanent shunt insertion at an older age in ment of cerebrospinal fluid shunt infection. J Neurosurg
the neonatal period also likely decreases the need Pediatr 2014;14 Suppl. 1:60-71.
for shunt revisions throughout childhood.
12. b—Single VP shunt following endoscopic
FURTHER READING fenestration of loculations
Hu YC, et al. Infantile posthemorrhagic hydrocephalus, In:
Winn RH, editor. Youmans Neurological Surgery, 6th ed. Given the high risk of failure of an ETV due to
Saunders, Elsevier; 2011. age and post-infectious etiology (ETVSS ¼ 10),
a VP shunt is more acceptable in this case but
Mazzola CA, et al. Pediatric hydrocephalus: systematic litera- endoscopic fenestration could reduce the number
ture review and evidence-based guidelines. Part 2: Manage- of proximal catheters required for adequate
ment of posthemorrhagic hydrocephalus in premature
drainage of CSF to one (as well as reducing the
infants. J Neurosurg Pediatr 2014;14 Suppl. 1:8-23.
odds of shunt failure as there are less catheters/
connection points). Patients with both multilocu-
11. d—Supplementation of antibiotic treatment
lated and uniloculated hydrocephalus, isolated
with partial (externalization) or with com-
lateral and fourth ventricles, arachnoid cysts,
plete shunt hardware removal is the standard
and slit ventricle syndrome have not always
of care responded to simple shunting systems. The ven-
tricular system may become trabeculated and
Current management of CSF shunt infection is
encysted following bacterial meningitis or germi-
dictated not by evidence, but rather by physician
nal matrix hemorrhage. Hydrocephalus arising
preference and other possibly relevant patient-
from intraventricular septations is known as com-
level factors (for example, patient surgical risk,
plex or loculated. The lateral ventricle may
ventricle size, and complexity of the shunt sys-
become trapped due to obstruction of the Monro
tem). It is not surprising that there is significant
variation in CSF shunt infection treatment pro- foramina by noncolloid neuroepithelial cysts
(ependymal, choroid plexus, or arachnoid),
tocols between centers. An infected ventricular
termed unilocular hydrocephalus, and in other
shunt, as an infected foreign body, is difficult if
not impossible to sterilize using antibiotics cases there may be multiple encysted compart-
ments (multilocular). Traditional treatment is
alone. We therefore accept not only that shunt
by placement of multiple shunts or multiperfo-
removal (and eventual replacement once CSF
rated catheters which were multiple shunt revi-
sterility is achieved) requires multiple surgeries,
sions and high morbidity and mortality rates.
but also the risk of introducing secondary infec-
Multiple shunts have increased the risks of infec-
tion during a variable period of external drain-
tion and mechanical obstruction, and their
age. Variations in whether the infected shunt
removal has been problematic, with its associated
was externalized or completely removed, and
risk of intraventricular hemorrhage. Transcallo-
whether supplemental intrathecal antibiotics
sal fenestration via craniotomy may reduce shunt
were administered contribute to significant
between-study heterogeneity. Shunt infection revision rates or achieve shunt independence in
both multiloculated hydrocephalus and unilocu-
should be ideally managed with antibiotics, com-
lated hydrocephalus, but craniotomy itself carries
plete shunt removal, and placement of a temporary
external ventricular drain, followed by reimplanta- concomitant risks. The specific risks associated
with the transcallosal approach include venous
tion after CSF sterilization (48 h after last negative
infarction from sacrificed bridging veins, damage
CSF). Although intrathecal administration of anti-
to the pericallosal arteries, disconnection syn-
biotics appears to make theoretical sense because
dromes after splitting the corpus callosum, and
of enhanced CSF antibiotic concentrations, its
damage to the fornices and subcortical nuclei.
practical application is controversial, owing in
Authors of many reports stress the operative sim-
large part to the potential adverse effects of intra-
plicity of the stereotactic procedure in unilocu-
thecal therapy, including neurotoxicity. The indi-
lated hydrocephalus; however, it was associated
cations for intrathecal therapy are not well
with a high recurrence rate (up to 80%) because
established and presently range from use in any
shunt infection, use in only those infections in the cyst wall could not be widely fenestrated,
making it unsuitable in cases of multiloculated
which the CSF cannot be sterilized by systemic
hydrocephalus. In the present day where endo-
antibiotics alone (for example, persistent positive
scopic fenestration (of loculations, cyst walls, sep-
cultures), or use in those ventricular shunt infec-
tum pellucidum, etc.) is available, the aim of
tions caused by specific organisms (for example,
surgery is to control hydrocephalus, simplify
gram-negative infections).

Neurosurgery Books Full


www.ketabpezeshki.com 66485438-66485457
554 PART VII PEDIATRIC NEUROSURGERY

complex shunts, reduce the shunt revision rate, Each individual ICP waveform has three peaks:
avoid implanting a shunt if possible, and decrease percussion wave (P1) representing arterial pulsa-
operative morbidity. Fenestration of the septum tion, tidal wave (P2) representing intracranial
pellucidum is indicated when there is an obstruc- compliance and dicrotic wave (P3) representing
tion of one foramen of Monro causing the ipsilat- aortic valve closure. Lundberg waves describe
eral ventricle to dilate from trapped CSF. patterns of mean ICP in patients.
Membranous/parenchymal obstructions of the
foramen of Monro causing unilateral hydroceph- 14. e—Intraoperature ultrasound is less accurate
alus have been reported and endoscopic at predicting those who do not need durot-
foraminoplasty ¼/- stent can be considered. omy in the presence of tonsillar descent
below C1 lamina
Image with permission from Winn RH, editor. Youmans
Neurological Surgery, 6th ed. Saunders, Elsevier; 2011. Chiari malformation Type I (CM-I) is a craniocer-
vical junction disorder that is associated with
FURTHER READING deformity and elongation of the cerebellar tonsils
Teo C, et al. Endoscopic management of complex hydroceph-
and is specifically characterized by tonsils’ descent
alus. World Neurosurg 2013;79 (2S):S21.e1-7.
of more than 5 mm below the foramen magnum
El-Ghandour NMF. Endoscopic cyst fenestration in the treat- into the spinal canal—a change in the flow of
ment of multiloculated hydrocephalus in children. J Neuro- CSF at the level of the foramen magnum is fre-
surg Paediatr 2008;1(3):217-22. quently associated with development of syringo-
myelia. Patients with Chiari I malformation
13. c—B wave is mean ICP with pressure commonly undergo foramen magnum/suboccipi-
20-50 mmHg lasting 30 s to a few minutes tal decompression (FMD) in order to restore free
during sleep flow of CSF across the craniocervical junction,
treating symptoms either related to raised ICP,
brainstem compression or syringomyelia. There
is no consensus on surgical indication, but a lower
Lundberg ICP Waves threshold may be expected in symptomatic hind-
brain herniation with syringomyelia compared
Lundberg
Type Criteria Pathophysiology to asymptomatic hindbrain hernia with syrinx or
symptomatic hindbrain herniation without syrinx.
A wave Mean wave ICP Low CPP results in Variation in FMD technique is also possible at
>50 mmHg lasting vasodilatation
5-20 min before (raised CBV and multiple stages: extent of bony decompression,
returning to ICP) and ischemia indication for opening the dura and arachnoid,
elevated baseline (Cushing response need for tonsils coagulation and dural closure. A
to restore CPP). recent review of all published studies of surgical
Suggests ICP
exceeding limits of
treatment of Chiari I showed that there was slight
cerebral variation in pediatric (97% FMD, 81% dural
compliance, and opening, 47% arachnoid opening/dissection and
ongoing ischemia. 21% tonsillar resection) versus adult (100%
B wave Mean wave ICP Seen in sleep; FMD, 97% dural opening, 70% arachnoid open-
20-50 mmHg respiratory ing/dissection and 16% tonsillar resection) prac-
lasting 30 s to changes and tice. In series reporting on syrinx association, the
3 min variations in CBF. incidence was 69% in adult series, 40% in pediat-
Suggests
qualitative rise in ric series, and 78% in mixed series. The traditional
ICP and that A operation for CM-1 is FMD with bony decom-
waves may form. pression and dural opening with or without dura-
C wave Mean wave ICP ICP transmission of plasty. Some have contended that not all patients
<20 mmHg cyclic Traube- with CM-1 need durotomy, with bony decom-
occurring every Hering-Meyer pression alone having been demonstrated as ade-
10 s variation in SBP quate/efficacious in a proportion of patients with
due to oscillations
in baroreceptor
CM-1, possibly as the dura of children may still
and have some elasticity and expand following bony
chemoreceptor decompression, leading to better CSF dynamics
control. at the craniocervical junction. The main advantage
Sometimes seen in of not needing to open the dura is that complica-
normal ICP
waveform tions such as pseudomeningocoele, CSF leak,
meningitis and hydrocephalus are reduced.

Neurosurgery Books Full


www.ketabpezeshki.com 66485438-66485457
41 PEDIATRIC NEUROSURGERY: GENERAL AND HYDROCEPHALUS 555

Preoperative and intraoperative prediction of


those who could be adequately treated without ANSWERS 16–25
durotomy/duraplasty (or conversely those who Additional answers 16–25 available on
definitely should have it) would ensure the efficacy ExpertConsult.com
of the surgical intervention while minimizing the
rate of complications and need for reoperation.
Adequacy of cerebellar tonsillar or CSF pulsation
at the craniocervical junction as judged on intrao-
perative ultrasound may be of value in this respect, EMI ANSWERS
although may be less reliable in predicting patients
who will do well with bony decompression alone 26. 1—k, X-linked hydrocephalus; 2—f, Joubert
when there is moderate (below C1 lamina)/severe syndrome
(below C2 lamina) tonsillar descent.
Infants may have hydrocephalus at birth, caused
FURTHER READING by a congenital malformation that prevents ade-
Arnautovic A, Splavski B, Boop FA, Arnautovic KI. Pediatric quate circulation of cerebrospinal fluid. Acquired
and adult Chiari malformation Type I surgical series 1965-
hydrocephalus may arise after infection, intracra-
2013: a review of demographics, operative treatment, and out-
comes. J Neurosurg Pediatr 2015;15(2):161-77. http://dx.doi.
nial hemorrhage, or structural or mass lesions.
org/10.3171/2014.10.PEDS14295. Epub 2014 Dec 5. Most patients with non-syndromic congenital
PubMed PMID: 25479580. hydrocephalus have aqueduct stenosis. Of these,
X-linked hydrocephalus is the most common her-
McGirt MJ, Attenello FJ, Datoo G, Gathinji M, Atiba A, itable form, accounting for about 10% of cases in
Weingart JD, Carson B, Jallo GI. Intraoperative ultrasonog- boys. Mutations in L1CAM, encoding the L1 cell
raphy as a guide to patient selection for duraplasty after sub-
adhesion molecule, are the most common cause.
occipital decompression in children with Chiari malformation
Type I. J Neurosurg Pediatr 2008;2(1):52-7.
Primary ciliopathies such as Joubert’s syndrome
and Meckel-Gruber syndrome are associated
15. c—Transventricular tap with congenital hydrocephalus in human beings.

Subdural effusion occurs in 40-60% of infants 27. 1—i, Proximal catheter obstruction. Chil-
and young children with proven meningitis. dren with treated hydrocephalus face many
Small collections generally subside with observa- potential long-term complications, often
tion alone and surgical drainage is only required relating to treatment. Shunt failure, usually
when if symptomatic or causing mass effect. from mechanical obstruction, needing some
Options include serial percutaneous needle form of intervention occurs in 40% of chil-
drainage, burr hole drainage, external subdural dren within the first 2 years after original
drain and subduroperitoneal shunt (SDP). Exter- placement with continued risk of failure
nal drains were placed whenever the fluid was thereafter. Failure is diagnosed by imaging
purulent, when the collection was estimated to evidence of increased ventricle size compared
be too large to be cured with SDP alone, or when with baseline (although this is not always the
raised ICP recurred after SDP, and be kept in case) with symptoms of headache, vomiting,
place for 1-2 weeks until fluid becomes clearer. irritability, decreased level of consciousness,
At the end of that period, whenever the patient and, in infants, bulging fontanel and acceler-
was dependent on the external drain, a subduro- ated head growth. Evidence suggests that the
peritoneal drain was inserted. Removal of internal type of shunt valve used has no effect on fail-
subdural shunts should ideally be after a few ure incidence. Shunt obstruction is treated
months. In rare cases of obstruction of the drain- with urgent surgery, either EVD (if in extre-
age associated with thick subdural membranes mis) or shunt revision in patients with near
exerting a mass effect on the brain, craniotomy normal preoperative GCS. In situations in
with membrane resection can be performed. which symptoms are more subtle (e.g.
chronic headache or deteriorating school
Image with permission from Khanna PC, Shaw DWW. performance) intracranial pressure monitor-
Neuroimaging. In: Fuhrman BP, editor. Pediatric Critical ing can sometimes be helpful to establish if
Care, 4th ed. Mosby, Elsevier; 2011. shunt obstruction is the cause. 2—h, Migra-
tion of distal peritoneal catheter. Swelling
FURTHER READING over the abdominal wound is most likely a
Vinchon M, Joriot S, Jissendi-Tchofo P, Dhellemmes P.
Postmeningitis subdural fluid collection in infants: changing pat-
CSF seroma due to migration of the distal
tern and indications for surgery. J Neurosurg 2006;104:383-7. catheter out of the peritoneal cavity and into

www.ketabpezeshki.com 66485438-66485457
556 PART VII PEDIATRIC NEUROSURGERY

subcutaneous tissues (producing a bird’s nest revising the shunt valve to one with higher
pattern of coiled distal catheter). CSF ascites resistance, or both. Approximately 40-60%
arises due to malabsorption of CSF by peri- of children with CSF shunts develop small,
toneum, and may require needle paracentesis slit-like ventricles, which can be appreciated
to exclude infection, whereas abdominal on radiographic imaging after CSF diversion.
pseudocysts also present with abdominal pain Over time with chronic overdrainage 10% of
and distension but, by definition, form these patients may develop what is known as
around the distal catheter in the peritoneal slit ventricle syndrome—typically presenting
cavity (perhaps due to low virulence organ- with cyclical signs and symptoms consistent
isms like Proprionobacter and Corynebacterium) with increased ICP, with slit-like ventricles
and often require removal or the shunt and unchanged from prior imaging studies. Inter-
antibiotic treatment. 3—b, Acute overdrai- mittent obstruction of the proximal catheter
nage. If drainage of CSF occurs too rapidly, and loss of the ability to compensate for tran-
collapse of the ventricular system and brain sient changes in intracranial volume may
may occur along with the creation of a poten- underlie this picture.
tial space between the dura and the cortical
surface of the brain. Extra-axial CSF collec- FURTHER READING
tions may develop, but if bridging cortical Lee P, et al. Evaluation of suspected cerebrospinal fluid shunt
veins are disrupted, subdural hemorrhage complications in children. Clin Pediatr Emerg Med
can occur. These collections may respond 2008;9:76-82.
to conservative measures, but some necessi-
tate draining the extra-axial collections, 28. 1—a, Citrobacter koseri; 2—j, Tuberculosis

www.ketabpezeshki.com 66485438-66485457
CHAPTER 42

PEDIATRIC NEURO-ONCOLOGY
SINGLE BEST ANSWER (SBA) QUESTIONS
1. Which one of the following lists of primary 4. A 17-year-old boy patient presents with left
brain tumors in children is from most com- arm clumsiness. MRI is shown. Which one
mon to LEAST common (all age groups of the following is most likely?
combined)?
a. Astrocytoma, craniopharyngioma, medul-
loblastoma, ependymoma, germ cell
tumors
b. Astrocytoma, medulloblastoma, cranio-
pharyngioma, ependymoma, germ cell
tumors
c. Ependymoma, astrocytoma, medulloblas-
toma, craniopharyngioma, germ cell
tumors
d. Medulloblastoma, astrocytoma, cranio-
pharyngioma, ependymoma, germ cell
tumors
e. Medulloblastoma, craniopharyngioma,
ependymoma, astrocytoma, germ cell
tumors

2. Which one of the following is associated with


the highest incidence of brain metastases in
children?
a. Neuroblastoma
b. Sarcoma a. Hemangioblastoma
c. Wilms tumor b. Medulloblastoma
d. Leukemia c. Neuroblastoma metastasis
e. Lymphoma d. Pilocytic astrocytoma
e. Vestibular schwannoma
3. Which one of the following statements
regarding posterior fossa tumors in children
is LEAST accurate?
a. Pilocytic astrocytomas account for 35% of
posterior fossa tumors in children
b. Medulloblastoma account for 30% of pos-
terior fossa tumors in children
c. Ependymomas account for 20-30% of
posterior fossa tumors in children
d. Brainstem gliomas account for 10-20% of
posterior fossa tumors in children
e. Hemangioblastomas account for 10% of
posterior fossa tumors in children

557

www.ketabpezeshki.com 66485438-66485457
558 PART VII PEDIATRIC NEUROSURGERY

5. A 5-year-old with features of NF-1 presents


with visual disturbance. T1 postcontrast
MRI is shown. Which one of the following
statements regarding management of these
lesions is most accurate?

a. Diffuse astrocytoma
a. Surgical decompression is prophylactic to b. DNET
save bilateral vision c. GBM
b. Chemotherapy is second line manage- d. Pilocytic astrocytoma
ment in children under 9 years old e. Pleomorphic xanthoastrocytoma
c. Biopsy should only be performed if there
are features of NF-1 7. A 2-year-old presents after his mother noted
d. Somatostatin has been shown to improve an abnormal white reflection in his right eye.
outcome MRI is shown. Which one of the following
e. Cranial irradiation is contraindicated in would you look for?
this case

6. A 16-year-old male patient who previously


underwent excision of a right temporo-
occipital lesion 7 years previously, with good
control of seizures since, presents with
increased frequency of tonic-clonic seizures
and headaches. Neurological examination
was unremarkable. MRI is shown. Which
one of the following is most likely?

a. AT/RT
b. Hypothalamic hamartoma
c. Optic glioma
d. Pineoblastoma
e. Pontine glioma

www.ketabpezeshki.com 66485438-66485457
42 PEDIATRIC NEURO-ONCOLOGY 559

8. A 4-year-old presents with torticollis, facial 10. A 5-year-old child presents with gait ataxia.
asymmetry and ophthalmoparesis. Which MRI is shown. Which one of the following
one of the following is appropriate next management strategies is appropriate in this
management? case?

a. Biopsy
b. Radiotherapy
c. Chemotherapy a. Complete resection followed by
d. Midline suboccipital resection surveillance
e. Transoral resection b. Complete resection followed by focal
radiotherapy boost
9. Which one of the following conditions is c. Neoadjuvant chemotherapy followed by
most likely? resection and postoperative adjuvant
chemotherapy
d. Debulking surgery followed by cranio-
spinal irradiation
e. Stereotactic radiosurgery

a. NF-1
b. NF-2
c. Tuberous sclerosis
d. Li-Fraumeni syndrome
e. Cowden syndrome

www.ketabpezeshki.com 66485438-66485457
560 PART VII PEDIATRIC NEUROSURGERY

11. A 9-year-old girl presents with headache and a. Craniopharyngioma


vomiting. On examination there is bilateral b. Germinoma
papilledema but she is otherwise neurologi- c. Langerhans cell histiocytosis
cally stable. MRI is performed. Which one d. NGGCT
of the following is the next appropriate step? e. Pineoblastoma

13. A 12-year-old boy presents with precocious


puberty and headache. CT head demon-
strates obstructive hydrocephalus and pineal
region mass on CT. Serum HCG is
8178 IU/l, AFP is 5 ng/ml. What would be
the most appropriate next step in
management?
a. Stereotactic biopsy
b. Endoscopic biopsy
c. Endoscopic third ventriculostomy with-
out biopsy
d. Tumor debulking
e. Ventriculoperitoneal shunt

14. An 8-year-old girl presents with symptoms of


raised intracranial pressure over the last
2 weeks. T1 + GAD MRI is shown. Which
one of the following is most likely?

a. ICP monitoring
b. Lumbar puncture
c. Serum AFP and HCG
d. External ventricular drain
e. Endoscopic third ventriculostomy and
biopsy of lesion

12. A 13-year-old presents with polydipsia and


polyuria. Clinical examination is unremark-
able. Serum blood tests show a Na 149 mEq/l
and serum osmolality is 315 mOsm/kg H2O,
with urine osmolality of 115 mOsm/l. MRI is
shown. Subsequent serum AFP and BHCG
are within normal limits. Which one of the
following is most likely?

a. Arachnoid cyst
b. AT/RT
c. Choroid plexus papilloma
d. DNET
e. Medulloblastoma

www.ketabpezeshki.com 66485438-66485457
42 PEDIATRIC NEURO-ONCOLOGY 561

15. A 6-year-old child presents with clumsiness EXTENDED MATCHING ITEM (EMI)
and slurred speech. On examination and
papilledema. CT head is abnormal therefore
QUESTIONS
MRI of the craniospinal axis is performed
showing the lesion above. Spinal MRI is nor- 22. For each of the following descriptions, select
mal. Which one of the following best the most appropriate answer from the image
describes conventional treatment? below. Each answer may be used one, more
than once, or not at all.
a

c
j

d
i

e h

f g
T1 Post

1. Striae medullaris
a. Neoadjuvant chemotherapy, surgical 2. Sulcus limitans
resection, postoperative focal radiotherapy 3. Area postrema
b. Surgical resection followed by focal radio- 4. Facial colliculus
therapy plus craniospinal irradiation 5. Median sulcus
c. Surgical resection followed by focal radio-
therapy plus adjuvant chemotherapy
d. Surgical resection followed by 23. Pediatric posterior fossa tumors:
chemotherapy a. Atypical teratoid/rhabdoid tumor
e. Surgical resection followed by cranio- b. Dermoid
spinal irradiation and focal radiotherapy c. Diffuse intrinsic pontine glioma
with concurrent chemotherapy d. Ependymoma
e. Epidermoid
f. Hemangioblastoma
QUESTIONS 16–21 g. Medulloblastoma
h. Meningioma
Additional questions 16–21 available on i. Pilocytic astrocytoma
ExpertConsult.com
j. Sarcoma metastases
k. Vestibular schwannoma

For each of the following descriptions, select the


most appropriate answer from the list above.
Each answer may be used one, more than once,
or not at all.
1. A 15-year-old boy with multiple angioma-
toses of the retina and cysts of the kidney
and pancreas.

www.ketabpezeshki.com 66485438-66485457
562 PART VII PEDIATRIC NEUROSURGERY

2. An 11-year-old presents with features of For each of the following descriptions, select the
hydrocephalus. Imaging shows a cystic most appropriate answer from the list above.
mass with an enhancing mural nodule, Each answer may be used one, more than once,
which lacks flow voids. or not at all.
1. A 9-year-old developmentally delayed girl
24. Pediatric sellar and pineal lesions: has precocious puberty and poorly con-
a. Arachnoid cyst trolled seizures. Her seizures are typically
b. Craniopharyngioma preceded by episodes of uncontrollable
c. Germinoma laughter.
d. Histiocytosis X 2. An 11-year-old presents with hydrocepha-
e. Hypothalamic hamartoma lus and diabetes insipidus. MRI shows an
f. Optic pathway glioma enhancing lesion in the pineal region and
g. Pituitary adenoma another in the suprasellar region.
h. Pituitary macroadenoma 3. A 5-year-old boy with neurofibromatosis 1
i. PNET presents with visual field defect and pendu-
j. Rathke's cleft cyst lar nystagmus.

SBA ANSWER
1. b—Astrocytoma, medulloblastoma, cranio- of age). Roughly 60-70% are gliomas; specific
pharyngioma, ependymoma, germ cell frequency of tumor types is astrocytomas 30%,
tumors medulloblastoma (15-20%), craniopharyngioma
(10-15%), ependymomas (5-10%), germ cell
Overall incidence approximately 5 per 100,000 tumors (5-10%). Commonest tumors vary by
persons per year (compared to 20-30 per age (see below). Incidence in 19 years or younger
100,000 in adults). Incidence rates of tumor in the Central Brain Tumor Registry of the
types vary by age, hence although posterior United States:
fossa tumors are much more common in children
than adults, in children infratentorial tumors FURTHER READING
may only be commoner than supratentorial Uptodate. Epidemiology of central nervous system tumours in
tumors within certain age groups (e.g. 4-10 years children. Topic 6251 Version 17.0.

Epidemiology of Childhood Brain Tumours


Primary childhood CNS tumors incidence
by WHO group Primary childhood CNS tumor incidence by diagnosis

Neuroepithelial tissue 3.7 per 100,000 Pilocytic astrocytoma 0.84 per 100,000 per year
- Astrocytic 1.3 per 100,000 per year Medulloblastoma/PNET 0.65 per 100,000 per year
- Ependymoma 0.28 per 100,000 per year Pituitary and craniopharyngioma 0.73 per 100,000 per
Sellar region 0.73 per 100,000 per year year
Embryonal CNS tumors 0.65 per 100,000 per year Ependymoma 0.28 per 100,000 per year
Neuronal and mixed neuronal-glial 0.37 per 100,000 per
year
Unclassified 0.3 per 100,000 per year
Cranial and paraspinal nerves 0.27 per 100,000 per year
Meninges 0.22 per 100,000 per year
Germ cell tumors and cysts 0.21 per 100,000 per year
Lymphoma and hemopoetic 0.03 per 100,000 per year
0-4 years 5-9 year 10-14 years 15-19 years

Gliomas (excluding PA) Gliomas (excluding PA) Gliomas (excluding PA) Gliomas (excluding
Pilocytic astrocytoma Pilocytic astrocytoma Pilocytic astrocytoma PA) Pituitary tumors
Medulloblastoma Medulloblastoma Pituitary tumors Pilocytic astrocytoma
PNET Pituitary tumors Medulloblastoma Medulloblastoma
Pituitary tumors PNET PNET PNET

www.ketabpezeshki.com 66485438-66485457
42 PEDIATRIC NEURO-ONCOLOGY 563

2. b—Sarcoma due to microcystic and necrotic areas. The solid


and mural components enhance prominently.
The commonest primary neoplasms in children Enhancement of the cyst wall suggests tumor infil-
uncommonly demonstrate hematogenous seed- tration of the capsule. They maintain their WHO
ing resulting in brain metastasis—the overall rate grade I status for years; they only rarely show
approximates 4% in patient followed up malignant transformation (anaplastic pilocytic
>10 years. In a pooled analysis of multiple studies astrocytomas). A large percentage of pilocytic
in over 2000 children, cerebral hematogenous astrocytomas, particularly those arising within
metastases were reported in 4.4% of 429 patients the cerebellar hemisphere, have demonstrated
with neuroblastoma, 1.9% of 574 rhabdomyosar- alterations in the BRAF gene but this is not prog-
coma patients, 6.5% of 386 patients with osteo- nostically significant. Resection is the treatment of
sarcoma, 3.3% of 487 Ewing sarcoma patients, choice for well-circumscribed lesions and extent
3.6% of 44 melanoma patients, 13.5% of 37 of resection most strongly associated with out-
patients with germ cell tumors, and 1.3% of the come. Gross total resection leads to over 90%
78 patients with Wilms tumor. long-term survival. Cerebellar lesions are gener-
ally completely resectable hence adjuvant therapy
is not indicated, whereas. Those arising from the
Epidemiology of Childhood Cancer brainstem are often not and require adjuvant che-
motherapy, usually including carboplatin and vin-
Commonest Commonest Commonest cristine, and consideration of radiotherapy at
cancers cancers 1-10 cancers progression. Trials of BRAF and MAPK pathway
in <1 year years >10 years inhibitors, and antiangiogenic agents bevacizu-
(1) Neuroblastoma Leukemias Leukemias mab and linalidomide are ongoing. The main dif-
(2) Leukemia CNS tumors CNS tumors ferential diagnosis is hemangioblastoma, which
(3) Germ cell Neuroblastoma Lymphoma usually presents in young adults but may present
tumors Wilms tumor Bone tumors/ in children with VHL but these may be multiple,
(4) CNS tumors Germ cell Soft tissue
(5) Wilms tumor tumors/Soft sarcomas show greater vascularity/abnormal vessels associ-
tissue sarcoma Germ cell ated with enhancing portion/nodule, are usually
tumors abutting a pial surface, and have a considerably
higher rCBV than pilocytic astrocytomas on
perfusion imaging.
FURTHER READING
Curless RG, Toledano SR, Ragheb J, Cleveland WW, Image with permission from Perry A, Brat DJ. Practical
Falcone S. Hematogenous brain metastasis in children. Surgical Neuropathology: A Diagnostic Approach,
Churchill Livingstone, Elsevier, 2010.
Pediatr Neurol 2002 Mar;26(3):219-21. Review. PubMed
PMID: 11955930.
FURTHER READING
Uptodate. Epidemiology of central nervous system tumours in Aquilina K. Posterior fossa tumours in children. ACNR
children. Topic 6251 Version 17.0. 2013:13:4:24-8.

Graphic 99680 Version 1.0. 5. e—Cranial irradiation is contraindicated in


this case
3. e—Hemangioblastomas account for 10% of
posterior fossa tumors in children Optic pathway gliomas are congenital WHO grade
I astrocytoma of optic nerves or chiasm, and may
4. d—Pilocytic astrocytoma be associated with NF-1 (50-60%) or be sporadic.
Sporadic tumors typically arise within the
Pilocytic astrocytomas are the most frequent pos- chiasmatic-hypothalamic region or in other brain
terior fossa tumors in children (approximately structures adjacent to, or involving the optic tract,
35%). Peak age is 5-13 years; approximately half and do not typically involve the optic nerves. In
arise in the midline vermis and half from the cer- contrast, OPG associated with neurofibromatosis
ebellar hemispheres. They are circumscribed, dis- type 1 characteristically involve optic nerve, chiasm
crete, slow-growing lesions, often associated and optic radiation, including the geniculate
with cysts within and around the tumor. On CT ganglion. Tumors involving optic chiasm and
they are large cystic lesions with a hypodense hypothalamus are of a higher histologic grade
solid component which enhances avidly on (grade 2) than tumors confined to optic nerve and
contrast administration. On T1WI, the solid com- may cause considerable morbidity and even death.
ponent tends to be iso- to hypointense and on Usually discovered during first decade due to visual
T2WI it is hyperintense, with heterogeneity is impairment, endocrine deficit, focal neurological

www.ketabpezeshki.com 66485438-66485457
564 PART VII PEDIATRIC NEUROSURGERY

deficit, hypothalamic behavioral disturbance appears to be a less frequent finding. On CT,


(appetite, rage, obesity), stationary or progressive PXAs are usually seen as well-defined, partially
visual loss in one eye or both. Examination may cystic masses of various sizes, which show
show iris hamartomas (Lisch nodules) and/or eye- enhancement after contrast administration. The
lid plexiform neurofibromas, visual acuity and/or solid component of the neoplasm tends to be het-
visual field loss, afferent pupil defect, pendular erogeneous and isodense to gray matter, and cal-
(monocular or seesaw) nystagmus, optic disc cification may be present. MRI reveals that most
abnormality, proptosis if large intraorbital compo- of these tumors are predominately superficial,
nent. MRI shows intrinsic mass of optic nerves, cortical, partially cystic masses which are hypo-
optic chiasm, optic tracts, or hypothalamus and to isointense to gray matter on unenhanced
may also show hamartomas elsewhere that are typ- T1-weighted images while mildly hyperintense
ical of neurofibromatosis type 1. By definition, due on T2-weighted images. Gadolinium-enhanced
to their location, surgery will inevitably damage the T1-weighted images typically show marked
optic nerve or pathway (plus risk vascular supply to enhancement of the solid component. The most
hypothalamus and pituitary) and biopsy cannot consistent findings are peripheral location of
usually be justified if imaging is diagnostic (it is the tumor, leptomeningeal contact of the tumor
done only if imaging is non-diagnostic/atypical), and marked enhancement of the solid tumor
and where useful vision is maintained ipsilateral component. The radiologic differential diagnosis
to tumor debulking or resective surgery seems of PXAs includes mainly other astrocytomas,
unnecessary. However, when there is unilateral ganglioglioma, meningioma, meningiosarcoma
failing or useless vision debulking, resective surgery and fibrous xanthoma. The treatment of choice
may be appropriate to protect vision on the good for pleomorphic xanthoastrocytomas is complete
side, potentially controlling hydrocephalus and surgical excision which is favored by the superfi-
delaying radiotherapy in what is often a young cial location, relative circumscription, and cyst/
patient group. Surgical treatment may also be mural nodule architecture of the tumor.
offered for optic nerve gliomas causing disfiguring
proptosis in a blind eye (excision of optic nerve Image with permission from Douis H, Andronikou, S,
von Bezing, H. Pleomorphic xanthoastrocytoma: case
only). Chemotherapy is used for patients under series with radiologic-pathological correlation and
age 9 with glioma involving the chiasm and/or review of the literature, Eur J Radiol Extra 2008;68(1):
hypothalamus and who have severe or worsening pp.5-8.
visual or hypothalamic dysfunction or signs of
tumor growth (to delay radiotherapy and neuro- 7. d—Pineoblastoma.
cognitive side effects, or in NF-1 for additional risk
of secondary moyamoya disease and intracranial The T2-weighted MRI shows left sided retino-
tumors). Cranial radiation is contraindicated in blastoma. Retinoblastomas are bilateral in 30-
NF-1 but may have a vision sparing effect in spo- 40% of cases, and may be associated with pineo-
radic tumors in older children and adults. Visual blastoma (“trilateral retinoblastoma” as the pineal
function often stable in untreated patients, but gland has been historically labeled as the “inner/
visual decline may result from tumor progression. third eye”) in 15% of cases.
Image with permission from Trobe JD, Rapid Diagnosis in Image with permission from LaPlante JK, Pierson NS,
Ophthalmology: Neuro-Ophthalmology, Elsevier, 2008. Hedlund GL. Common pediatric head and neck congen-
ital/developmental anomalies, Radiol Clin North Am
6. e—Pleomorphic xanthoastrocytoma 2015 Jan;53(1):181-96.

Pleomorphic xanthoastrocytoma is a very rare 8. b—Radiotherapy (diffuse intrinsic pontine


brain tumor accounting for only 1% of all brain glioma)
neoplasms. It is however important to be familiar
with the radiological findings of this tumor Brainstem gliomas account for 10-20% of all
because it has a characteristic imaging appearance CNS tumors in children. They are broadly
and is highly amenable to surgical resection with a classified into diffuse or focal. Diffuse intrinsic
good prognosis. PXAs predominantly occur in pontine gliomas are high-grade fibrillary astrocy-
children and young adults and there is no gender tomas with median overall and progression-free
predilection. PXA predominately affects the cere- survival of up to eleven and nine months respec-
bral hemispheres, showing mainly involvement of tively. They present with a short history, often
the temporal and parietal lobes. The most com- characterized by cranial nerve palsies and ataxia.
mon clinical presentation of this tumor is sei- They are diagnosed radiologically and when typ-
zures, whilst increased intracranial pressure ical, do not require biopsy. They are hyperintense

www.ketabpezeshki.com 66485438-66485457
42 PEDIATRIC NEURO-ONCOLOGY 565

on T2- and hypointense on T1-weighted images, mid-20s, even in the absence of SEGA symptoms.
with ill-defined boundaries and diffuse enlarge- Options include surgery or pharmacotherapy. Sur-
ment of the brain stem. They generally do not gical tumor resection has traditionally been the
enhance with contrast. Surgical resection has no standard therapy for SEGA and earlier interven-
role in these tumors. Radiation is indicated to tion has been proposed to avoid sequelae of hydro-
the tumor with a margin to a dose of approxi- cephalus. Indications for surgery include:
mately 54 Gy. Symptoms generally resolve with asymptomatic SEGAs with documented tumor
treatment. Unfortunately, duration of symptom growth or enlargement of the ventricles is
relief is usually short lived, on the order of observed; symptomatic SEGA (e.g. behavioral
6 months. This disease is uniformly fatal, and changes, worsening of seizures, symptoms of
the median survival for these children is 1 year. raised ICP). Prognosis is good, particularly in cases
Despite several clinical trials over the last fifteen where surgery is performed early for small lesions;
years, based on various chemotherapeutic agents in most cases resection is curative if tumor removal
and radiotherapy delivery techniques, there has is complete. The goal of pharmacotherapy with
been no improvement in clinical outcome. Focal mTOR pathway inhibitor (everolimus) is to shrink
brainstem tumors are well-circumscribed masses or stabilize the tumor in adults and children 3 years
that may be intrinsic, exophytic or cervicomedul- or older with SEGA associated with TSC who
lary, solid or cystic, under 2 cm in diameter, and require therapeutic intervention but are not ame-
are commonly low grade astrocytomas. In one nable to surgery (e.g. contraindications exist for
retrospective study of focal brainstem gliomas, anesthesia or surgery in general, total resection
following 52 children over a mean of ten years, unlikely to be possible, or in rare cases of bilateral
the survival rate was 98% at five years and 90% fornix lesions with high risk of morbidity). Treat-
at ten years; 36. 5% underwent gross or near total ment reduces tumor size, and the effect is sustained
resection. If there is clinical or MRI progression, in EXIST-1 trial. The role of stereotactic radio-
options include resection (for accessible tumors surgery has not yet been fully elucidated.
where the patient and family understand the high
risk of new neurological deficit) or stereotactic Image with permission from Perry A, Brat DJ. Practical
Surgical Neuropathology: A Diagnostic Approach,
biopsy followed by radiation. Churchill Livingstone, Elsevier, 2010.
Image with permission from Kaye AH. Brain Tumors an
Encyclopedic Approach, 3rd ed. Elsevier, 2012. FURTHER READING
Józwiak, Sergiusz; Nabbout, Rima; Curatolo, Paolo. Manage-
ment of subependymal giant cell astrocytoma (SEGA) associ-
FURTHER READING
Aquilina K. Posterior fossa tumours in children. ACNR ated with tuberous sclerosis complex (TSC): clinical
2013;13:4:24-28. recommendations. Eur J Paediatr Neurol 2013;17(4):348-52.

Klimo P Jr et al. Management and outcome of focal low-grade 10. b—Complete resection followed by focal
brainstem tumors in pediatric patients: the St. Jude experience. radiotherapy boost
J Neurosurg Pediatr 2013.
Ependymoma is the third most common pediatric
9. c—Tuberous sclerosis brain tumor; over 50% of cases arise in children
under 5 years of age. Chromosomal abnormalities
Tuberous sclerosis complex (TSC) is a genetic, include 22q; chromosome 1q gain has been found
multi-organ condition characterized by the devel- in up to 22% of childhood ependymomas, and is
opment of benign tumors in the brain, heart, kid- associated with posterior fossa location, anaplastic
neys, liver, and lungs. Subependymal giant cell features and a poor prognosis. Infratentorial
astrocytomas (SEGAs), develop in 10-15% of indi- ependymomas in children are classified as
viduals with TSC. SEGAs can be unilateral or WHO grade 2 or 3 (grade 1 being reserved only
bilateral, developing from those benign subepen- for subependymoma and myxopapillary ependy-
dymal nodules (hamartomas) located near the moma). Infratentorial ependymomas arise from
foramen of Monro. SEGA development is a grad- the floor or roof of the fourth ventricle and grow
ual process that generally occurs within the first 2 into the ventricular lumen. They have a propen-
decades of life. These are slow-growing, glial neu- sity to extend through the foramen of Luschka
ronal tumors, usually developing over 1-3 years. into the cerebellopontine cistern and around
SEGAs are usually asymptomatic until they block the brainstem, as well as down through the fora-
circulation of cerebrospinal fluid (CSF), leading to men magnum. They are well-delineated, soft,
increased intracranial pressure and hydrocephalus. heterogeneous tumors, often with cystic, necrotic
Typically, serial neuroimaging is performed every and hemorrhagic elements. On CT, ependymo-
1-3 years in pediatric TSC patients and until the mas are iso- or hyperdense lesions, up to 50%

www.ketabpezeshki.com 66485438-66485457
566 PART VII PEDIATRIC NEUROSURGERY

have punctate calcification, and they enhance het- by surgery and radiotherapy), or another pineal
erogeneously on contrast administration. On region tumor (e.g. pineoblastoma requiring
MRI, they are iso- to hypointense on T1- surgery followed by radiotherapy). The tumor
weighted sequences and hypointense on T2 but markers alpha-fetoprotein (AFP), human chori-
calcification, cysts, areas of necrosis and hemor- onic gonadotropin (HCG), placental alkaline
rhage cause heterogeneity within the tumor mass phosphatase, and lactic dehydrogenase isoen-
on enhanced and non-enhanced sequences. Lep- zymes are useful in the diagnosis and treatment
tomeningeal dissemination at presentation is less monitoring of germ cell tumors. Elevations in
common than in medulloblastoma; full spinal levels of AFP alone in cerebrospinal fluid (CSF)
MRI at diagnosis is imperative as part of the stag- and serum are found in pure endodermal sinus
ing process. The extent of surgical resection is a tumor, and may or may not be seen in mature ter-
major determinant of outcome. In historical atomas. Elevated levels of both HCG and AFP
series, five-year overall survival for ependymoma are found in embryonal carcinoma, and high
has ranged from 50 to 64%. Despite several levels of HCG alone are found in choriocarci-
multi-institutional studies, mostly including noma. The serum and CSF levels in cells of
platinum-based agents, no single chemothera- AFP may be 10-100 times baseline. Serum
peutic regimen has demonstrated significant HCG levels may be 100 times baseline, and there
survival benefit for ependymoma. The role of may be a CSF/serum gradient, especially when
chemotherapy alongside postoperative radiother- lumbar CSF is assayed. Normal levels are gener-
apy remains unclear. In a recent single-institution ally thought to be less than 1.5 IU/l or 1 IU/l for
study, conformal radiotherapy, administered β-hCG, however, and less than 1.5 ng/ml for
immediately after surgery, led to better overall AFP. Modest elevations of HCG may be found
survival rates, up to 85% at five years, compared in germinoma (CSF more likely), usually in the
to earlier studies with up to 73% at five years. presence of elevated placental alkaline phospha-
This may be partly attributable to the high rate tase and/or lactic dehydrogenase isoenzymes,
of gross total resection (82%) and use of radio- whereas a serum or CSF HCG >50 IU/l and/
therapy for the first time in children under or an AFP >25 ng/ml in the presence of a midline
three years. CNS tumor is supportive of a diagnosis of
NGGCT. Germinomas may secrete low levels
Image with permission from Yachnis AT, Rivera- of B-HCG if it contains some syncytiotropho-
Zengotita ML. Neuropathology, High-Yield Pathology
Series, Saunders, Elsevier, 2014. blasts (unlike the very high levels secreted by cho-
riocarcinomas >2000 IU/l) but pure germinomas
FURTHER READING do not, although they may secrete placental alka-
Aquilina K. Posterior fossa tumours in children. ACNR line phosphatase. In this situation, biopsy of the
2013;13(4):24-28. tumor is required to distinguish between
germinoma/non-secreting germ cell tumor and
Merchant TE, Li C, Xiong X, Kun LE, Boop FA, Sanford RA.
Conformal radiotherapy after surgery for pediatric ependy-
a pineal parenchymal tumor. This is ideally done
moma: a prospective study. Lancet Oncol 2009;10:258-66. at the same time as endoscopic third ventriculost-
omy. Additionally, endodermal sinus tumors
secrete alpha fetoprotein, and embryonal carci-
11. c—Serum AFP and HCG nomas secrete a mixture of beta human chorionic
gonadotropin and alpha fetoprotein. Pineal ger-
Pineal region masses have a wide differential minomas most often occur in male patients,
diagnosis, and considering she is neurologically pineal parenchymal neoplasms occur with equal
stable, the priority is to narrow the differential frequency in male and female patients. Both
diagnosis ideally before any surgical intervention tumors can arise at any age, but pineoblastomas
is required. In general germ cell tumors account peak during the first decade of life, whereas pine-
for 60%, pineal parenchymal tumors for 30%, ocytomas peak during the second and third
supporting cells/meninges for 10%, and cysts, decades. On CT imaging, pineal parenchymal
vascular lesions and metastases. Approximately tumors classically demonstrate a rim of
60-70% of germ cell tumors are highly radiosen- “exploded” calcification that can be helpful in dis-
sitive germinomas, while the rest are classed tinguishing them from germ cell tumors which
as non-germinomatous germ cell tumors have a central calcified pineal gland. Teratomas
(NGGCT: teratoma, choriocarcinoma, embryo- possess unique imaging characteristics because
nal carcinoma, yolk sac tumors). The main ques- of fat and calcium, and choriocarcinomas may
tion is whether the lesion is likely to be a hemorrhage; these characteristics help identify
germinoma (requiring non-surgical treatment these entities. The remainder of the NGGCTs
with craniospinal irradiation), a NGGCT have no unique characteristics that can be used
(requiring neoadjuvant chemotherapy, followed to confidently distinguish them.

www.ketabpezeshki.com 66485438-66485457
42 PEDIATRIC NEURO-ONCOLOGY 567

Image with permission from Quiñones-Hinojosa A. 14. c—Choroid plexus papilloma.


Schmidek and Sweet's Operative Neurosurgical Tech-
niques, 6th ed. Saunders, Elsevier, 2012. Choroid plexus papillomas and carcinomas repre-
sent 0.4-0.6% of all intracranial tumors. They are
12. b—Germinoma more common in the pediatric population and
account for approximately 3% of childhood brain
This child is presenting with features of diabetes tumors. More commonly, the lesions are seen in
insipidus, with MRI evidence of enhancing lesions the lateral and third ventricle, but 30% are seen in
in the hypothalamus-sellar area and the pineal the fourth ventricle, and less than 5% occur in the
region consistent with bifocal (synchronous) ger- cerebellopontine angle. The lesions usually pre-
minoma. Typical neuroimaging findings of CDI sent with obstruction of the CSF pathways as well
include absence of the posterior pituitary hyperin- as overproduction of CSF with resultant hydro-
tensity and normal or thickened infundibulum but cephalus. These lobulated intraventricular masses
they are nonspecific as they may progress to may have internal small flow voids and possibly
develop germinoma, Langerhans cell histiocyto- calcifications. Choroid plexus carcinomas are
sis, or lymphocytic hypophysitis. Less common more heterogeneous in appearance than are cho-
causes include lymphoma and granulomatous dis- roid plexus papillomas and usually extend beyond
eases (such as tuberculosis and sarcoidosis). Germ the margins of the ventricle. CT shows an iso- to
cell tumors are the most common neoplasm aris- hyperdense mass with punctate calcification and
ing from the pineal region, accounting for roughly homogeneous enhancement. On MRI the papil-
two thirds of pineal region neoplasms. They are lomas appear as lobulated, intraventricular masses
much more common in people of Asian descent of heterogeneous, predominantly intermediate
within the second and third decades of life, with signal intensity on both T1 and T2 images, with
men affected 10 times more frequently than intense contrast enhancement. Choroid plexus
women. The majority arise in the pineal region, carcinomas are rare, highly malignant tumors that
and the rest are suprasellar or in the basal ganglia. invade the adjacent brain parenchyma to a greater
On imaging, they classically engulf calcification, degree than papillomas. On MR spectroscopy,
enhance avidly, and demonstrate intrinsic hyper- choroid plexus papillomas tend to have an ele-
density on CT and isointensity to gray matter on vated myoinositol peak, whereas choroid plexus
most MRI sequences. Because they are histologi- carcinomas are more often associated with an ele-
cally unencapsulated, they sometimes invade the vated choline peak.
adjacent thalamus or tectum and disseminate via
the CSF. For this reason, imaging of the entire Image with permission from Koob M, Girard N. Cerebral
neural axis for detection of metastases is vital dur- tumors: specific features in children, Diagn Interv Imag-
ing the initial workup. Pure germinomas are very ing 2014 Oct;95(10):965-83.
radiosensitive, and patients typically have an excel-
lent prognosis. 15. e—Surgical resection followed by cranio-
spinal irradiation and focal radiotherapy with
Image with permission from Perry A, Brat DJ. Practical concurrent chemotherapy
Surgical Neuropathology: A Diagnostic Approach,
Churchill Livingstone, Elsevier, 2010. Medulloblastoma is a primitive neuroectoder-
mal tumor (PNET) arising from aberrant prolif-
13. c—Endoscopic third ventriculostomy with- eration of granule neuron precursor cells that go
out biopsy on to constitute the external granular layer of the
cerebellum. It represents 30% of posterior fossa
The history is suggestive of a non-germinomatous tumors and is the commonest malignant brain
germ cell tumor, given the precocious puberty and tumor in children with an incidence of 6.5 per
raised serum HCG (e.g. choriocarcinoma). As 1,000,000 children per year. It is a WHO grade
such, tumor biopsy is not necessary in the initial 4 tumor and has a propensity to leptomeningeal
management of this patient (but may be required dissemination. Associations include familial can-
later if there is no response to therapy). Instead, cer syndromes such as Gorlin syndrome, Li-
initially endoscopic third ventriculostomy can be Fraumeni syndrome, Turcot syndrome, Gard-
performed and CSF samples taken for cytology ner syndrome, and Cowden syndrome. It is a
and tumor markers to further stratify treatment. midline lesion in 75%—a cerebellar location is
Beyond this, NGGCT treatment requires neoad- associated with older age and desmoplastic his-
juvant chemotherapy followed by resective sur- tology. The mass is hypointense on T1- and
gery and postoperative radiation to the tumor T2-weighted images; it enhances heteroge-
bed with or without craniospinal irradiation. neously on gadolinium administration, may have

www.ketabpezeshki.com 66485438-66485457
568 PART VII PEDIATRIC NEUROSURGERY

cysts, and is diffusion restricting on DWI (due to FURTHER READING


hypercellularity). Leptomeningeal disease is Aquilina K. Posterior fossa tumours in children. ACNR
identified as enhancing nodules on the surface 2013;13(4):24-8.
of the brain and spinal cord (“sugar coating”). Gandola L, et al. Hyperfractionated accelerated radiotherapy
Histologically, medulloblastoma is composed in the Milan strategy for metastatic medulloblastoma. J Clin
of small blue round cells with a high nuclear to Oncol 2009;27:566-71.
cytoplasmic ratio in four distinct pathological
subgroups: classical (65-80%), desmoplastic/ Gajjar A, et al. Risk-adapted craniospinal radiotherapy fol-
nodular (15-25%), medulloblastoma with exten- lowed by high-dose chemotherapy and stem-cell rescue in
children with newly diagnosed medulloblastoma (St Jude
sive nodularity (15-25%), and an anaplastic/ Medulloblastoma-96): long-term results from a prospective,
large cell variant (4-5%). The desmoplastic var- multicentre trial. Lancet Oncol 2006;7:813-20.
iant is commoner in older children and is associ-
ated with a better prognosis. The large cell and
anaplastic variants are associated with a poor ANSWERS 16–21
prognosis. Molecular classification is based on
Wnt and SHH signaling abnormalities, which Additional answers 16–21 available on
confer a very good and intermediate prognosis ExpertConsult.com
respectively. Treatment is maximal safe resec-
tion (including via transvermian, transcortical,
or telovelar-cerebellar) followed by further adju-
vant treatment depending on whether they are
classified/staged as standard or high-risk cases EMI ANSWER
based on preoperative craniospinal axis MRI
(macroscopic metastases) and lumbar CSF cytol- 22. a—11, b—2, c—6, d—3, e—12
ogy two weeks postoperatively to avoid false posi-
tives early after resection (microscopic metastases). 1—Rostral apex, 2—Sulcus limitans, 3—Facial
High-risk patients include all children <3 years, colliculus, 4—Lateral aperture, 4—Vestibular
those with positive CSF, macrometastases on area, 5—Area postrema, 6—Caudal apex (obex),
MRI, and >1.5 cm of residual tumor visible on 7—Vagal trigone, 8—Hypoglossal trigone,
postoperative MRI (done within 24-72 h). 9—Acoustic tubercle, 10—Striae medullaris,
Children older than three with anaplastic histology 11—Median sulcus
or c-myc amplification are also considered Image modified from Cohen AR. Surgical Disorders of
high risk. Children >3 years old classified as stan- the Fourth Ventricle, Cambridge, MA: Blackwell
dard risk undergo craniospinal irradiation Science; 1996
(23.4 Gy), commenced within 40 days of surgery
with a posterior fossa boost to a total dose of 23. 1—f, Hemangioblastoma, 2—i, Pilocytic
54-55.8 Gy and concurrent chemotherapy once astrocytoma
weekly (“PACKER” regime, consisting of cis-
platin, vincristine, CCNU) which yields a 5-year 24. 1—e, Hypothalamic hamartoma, 2—c, Ger-
event-free survival of up to 80%. In children with minoma (bifocal), 3—f, Optic pathway
high-risk disease, 5-year progression-free survival glioma
has historically been approximately 40% but in
recent studies using a higher dose to the craniosp- Grossly, they can be classified as follows:
inal axis, hyperfractionated radiotherapy with 1. Sellar tumors: Pituitary adenoma (0.5-
higher posterior fossa boosts, and myoablative 2.5%), arachnoid cyst;
myeloablative courses of chemotherapy followed 2. Sellar-suprasellar tumors and cysts: Cra-
by peripheral blood stem cell rescue, 5-year pro- niopharyngiomas (6-9%), Rathke's cleft
gression-free survival of up to 73% has been seen. cyst, arachnoid cysts;
Furthermore, as the neurocognitive sequelae of 3. Suprasellar tumors: Chiasmatic/hypotha-
radiotherapy are more severe in young children lamic glioma (4-8%), arachnoid cyst;
repeated cycles of chemotherapy have been used 4. Lesions of the stalk: Germ cell tumors
after surgery in those <3 years in an attempt to (1-2%), histiocytosis X;
prevent progression until they become eligible 5. Miscellaneous lesions: Hypothalamic
for radiotherapy. hamartoma, metastatic lesions, PNET.
Image with permission from Kumar Selvarajan S, Hsu L.
Intraventricular posterior fossa yumors. In: Small JE,
Schaefer PW, editors, Neuroradiology: Key Differential
Diagnoses and Clinical Questions, Saunders,
Elsevier, 2013.

www.ketabpezeshki.com 66485438-66485457
CHAPTER 43

PEDIATRIC HEAD AND


SPINAL TRAUMA
SINGLE BEST ANSWER (SBA) QUESTIONS
1. You see a 9-month-old girl in the emergency GCS14/15, altered mental status or palpa-
department after she slipped out of her ble skull fracture is approximately 4%
father’s arms, falling 4 ft and hit her head c. In a child under 2 years, likelihood of clin-
on a hard floor. There is a tense bruise over ically important traumatic brain injury
the occiput. She is moving all four limbs without a scalp hematoma, LOC >5 s,
spontaneously but intermittently, inconsis- altered behavior, or severe mechanism
tently inconsolable, moaning, and opens eyes of injury is <0.02%
to voice. Which one of the following best d. Severe mechanism of injury include falls
describes her Glasgow Coma Scale (GCS)? of more than 2 ft if <2 years old or 6 ft
a. 9 if >2 years old
b. 10 e. In a child over 2 years, CT head or obser-
c. 11 vation are appropriate if the child has
d. 12 severe headache
e. 13
4. Which one of the following risk factors in the
2. A 12-year-old child sustains a head injury and UK NICE head injury guidelines for chil-
is found to have evidence of venous sinus dren is not sufficient alone to justify CT head
thrombosis on cranial imaging. Prior to dis- scan within 1 h when identified?
charge home, he is started on aspirin 25 mg a. Suspicion of non-accidental injury
od by the neurologist and the parents are b. Post-traumatic seizure but no history of
advised that he is at increased risk of Reye epilepsy
syndrome if he develops a febrile illness or c. GCS less than 14/15, or for children
viral infection. Which one of the following under 1 year GCS less than 15/15 on ini-
best describes Reye syndrome? tial assessment
a. Vomiting, encephalopathy, and hepatic d. At 2 h after the injury, GCS less than 15
dysfunction e. Witnessed loss of consciousness lasting
b. Rash, encephalopathy, and renal dysfun- more than 5 min
ction
c. Encephalopathy, renal dysfunction, and 5. In a ventilated infant following closed head
hepatic dysfunction injury which one of the following ICP mon-
d. Wheeze, encephalopathy, and hepatic itoring values is considered the upper limit of
dysfunction normal?
e. Vasculitis, encephalopathy, and renal a. 5 mmHg
dysfunction b. 7.5 mmHg
c. 10 mmHg
3. Which one of the following statements d. 12.5 mmHg
regarding the US PECARN pediatric head e. 15 mmHg
trauma algorithm is LEAST accurate?
a. It aims to calculate the likelihood of clin- 6. An 11-month-old girl was admitted to the
ically important traumatic brain injury hospital because of blunt head trauma.
needing computed tomography (CT) Her initial neurological examination was
imaging based on the age group completely normal except that a depressed
b. The likelihood of clinically important fracture was palpated in her right parietal
traumatic brain injury in children with region. CT of the head is shown.

569

www.ketabpezeshki.com 66485438-66485457
570 PART VII PEDIATRIC NEUROSURGERY

8. Brain trauma foundation guidelines for med-


ical management of TBI in children, which
one of the following statements is incorrect:
a. Rewarming after hypothermia should not
occur faster than 1 °C/h
b. A minimum CPP of 40 mmHg may be
considered
c. 3% hypertonic saline should be consid-
ered for the treatment of intracranial
hypertension
d. Continuous infusion of propofol for
either sedation or the management of
refractory intracranial hypertension
should be avoided
e. The use of corticosteroids is not recom-
mended to improve outcome or
reduce ICP

9. Regarding Brain Trauma Foundation guide-


lines for surgical management of TBI in chil-
dren which one of the following is LEAST
a. Comminuted fracture accurate?
b. Growing skull fracture a. ICP should be treated when exceeding
c. Linear skull fracture 15 mmHg in children
d. Ping-pong fracture b. Simultaneous EVD and lumbar drainage
e. Positional plagiocephaly of CSF can be used in refractory intracra-
nial hypertension
7. An 8-month-old female child who previously c. Decompressive craniectomy should only
fell from a changing table. She presented on be considered after the onset of late signs
referral from her pediatrician for evaluation of neurologic deterioration
of a left frontoparietal mass. Which one of d. Decompressive craniectomy improves
the following management strategies is most long-term neurological outcome com-
appropriate? pared to aggressive medical management
of raised ICP
e. ICP monitor placement is recommended
for early detection of expanding intracere-
bral hematomas in coagulopathic children

a. Cranioplasty
b. Conservative management
c. Dural repair
d. Duraplasty and autologous cranioplasty
e. Head bandage

www.ketabpezeshki.com 66485438-66485457
43 PEDIATRIC HEAD AND SPINAL TRAUMA 571

10. A 4-month-old girl presenting with apnea c. The lifetime additional risk of cancer due
and loss of consciousness. CT head scan is to CT scans in children is 1 extra case for
shown. Which one of the following is most 10,000 children scanned
likely? d. The baseline incidence of any form of
cancer in a child (before the age of 14) is
1 in 1000
e. The baseline incidence of any form of can-
cer is 1 in 20 in men before the age of 50.

13. Which one of the following factors does not


predispose children to cervical cord injuries
above C4 level?
a. Large head-to-body ratio
b. Horizontal, shallow facet joints
c. Ligamentous laxity
d. Increased spinal column elasticity
e. Age over 9 years

14. Which one of the following statements


regarding history of spine trauma is LEAST
accurate?
a. In young children aged 0-9 years, the pre-
dominant cause of injury is falls and
automobile-versus-pedestrian accidents
(>75%)
b. In children aged 10-14 years, motor vehi-
cle accidents (40%) are the major cause
of lumbar fractures, and falls and
a. Fall from 3 ft height automobile-versus-pedestrian accidents
b. Moya disease are less prevalent
c. Traumatic vertebral artery dissection c. In children 15-17 years of age, motor
d. Shaken baby syndrome vehicle and motorcycle accidents become
e. Benign enlargement of the subdural space the leading cause of spine injuries
(>70%), and there is also an increase in
11. Which one of the following is not a sensitive sports-related spine trauma
finding of non-accidental injury in children? d. SCI should be suspected if the child
a. A fractured femur in a pre-mobile child. reports transient neurological symptoms
b. Bruising away from bony prominences at the time of injury, even if they are
c. Torn labial frenulum in a mobile child now resolved
d. Genital and perineal burns e. Children with Down's syndrome are at
e. Multiple fractures and/or fractures of higher risk of cervical injuries as it is asso-
different ages ciated with congenital fusion of cervical
vertebrae
12. A 12-year-old child is brought to the emer-
gency department following an occipital head 15. A 9-year-old girl is involved in a MVA and
injury without loss of consciousness, but experiences a hyperflexion injury to the neck.
complains of significant headache and has She develops numbness and tingling in her
vomited twice. The parents are concerned arms and legs which progressed to an incom-
that he requires a scan. Which one of the plete quadraparesis. GCS is 15/15. CT cervical
following statements regarding the risk of spine does not show any fracture. MRI is
cancer from CT scans in general is most shown below, and STIR sequences do not
accurate? show any ligamentous disruption. Which one
a. For a cumulative dose of between 50 and of the following statements is most accurate?
60 mGy to the head there is a fivefold
increase in the risk of brain tumors
b. For a cumulative dose of between 50 and
60 mGy dose to bone marrow there is a
fivefold increase in the risk of leukemia

www.ketabpezeshki.com 66485438-66485457
572 PART VII PEDIATRIC NEUROSURGERY

b. In children older than 9 years, open


mouth X-ray views are not recommended
c. Children over 3 years of age should not
have cervical spine imaging if they are
alert, have no neurological deficit, no
midline cervical tenderness, no painful
distracting injury, no unexplained hypo-
tension, and are not intoxicated.
d. In a child under 3 years of age with a GCS
of 14/15 cervical spine imaging is manda-
tory after trauma
e. If gross ligamentous instability is sus-
pected on static radiographs MRI should
be performed urgently

18. A 10-year-old child presents with an abnor-


mal head posture as shown below after a
recent respiratory tract infection. Which
one of the following is most likely?

a. She should be managed in halo


immobilization
b. Flexion-extensions should be done at the
earliest opportunity
c. Somatosensory evokes potentials are
likely to be of value in localizing the injury
d. Spinal angiography should be performed
to exclude dissection
e. In the presence of normal dynamic cervi-
cal X-rays, cervicothoracic bracing for
12 weeks is appropriate initially

16. Which one of the following is not a normal


variant seen in the pediatric cervical spine?
a. Pseudosubluxation of C2 on C3
b. Localized kyphosis in mid-cervical area
c. Overriding C1 over tip of odontoid peg in
extension a. Atlanto-occipital dislocation
d. Persistence of basilar odontoid b. DYT-1 dystonia
synchondrosis c. Atlanto-axial rotatory subluxation (or
e. Anterior wedging of vertebral bodies fixation)
d. Odontoid epiphysiolysis
17. Which one of the following statements e. Subaxial cervical subluxation
regarding imaging of the cervical spine after
trauma in children is most accurate?
a. Overriding of the anterior atlas in relation QUESTIONS 19–27
to the odontoid on extension is a normal
finding on cervical radiographs in young Additional questions 19–27 available on
ExpertConsult.com
children

www.ketabpezeshki.com 66485438-66485457
43 PEDIATRIC HEAD AND SPINAL TRAUMA 573

EXTENDED MATCHING ITEM (EMI) 5. A 2-year-old child is rushed into hospital in


circulatory shock has multiple nonblanch-
QUESTIONS ing purple lesions of various sizes scattered
about on the trunk and extremities; pete-
28. Pediatric injuries: chiae are noted, and oozing from the vene-
a. Cephalhematoma puncture site has been observed.
b. Copper deficiency
c. Disseminated intravascular coagulation 29. Pediatric spinal injury:
d. Fibrous dysplasia a. Atlanto-occipital dislocation
e. Idiopathic thrombocytopenic (dissociation)
purpura (ITP) b. Atlanto-axial instability
f. Mongolian blue spot c. Atlas (C1) Fracture
g. Non-accidental injury d. C2-C3 subluxation and dislocation
h. Osteogenesis imperfecta e. Hangman fracture
i. Osteomalacia f. Odontoid epiphysiolysis
j. Preterm birth and osteopenia g. Pseudosubluxation
k. Rickets h. Thoracolumbar burst fracture
i. Thoracolumbar compression fracture
For each of the following descriptions, select the j. Thoracolumbar flexion-distraction
most appropriate answers from the list above.
Each answer may be used once, more than once For each of the following descriptions, select the
or not at all. most appropriate answers from the list above.
1. A 1-week-old infant of African origin pre- Each answer may be used once, more than once
sents to you for the first time with a large, or not at all.
fairly well-defined, purple lesion over the 1. A 4-year-old falls from a climbing frame
buttocks bilaterally. The lesion is not palpa- and complains of persistent neck pain. Lat-
ble, and it is not warm or tender. This oth- eral c-spine films show anterior displace-
erwise well-appearing infant is growing and ment of C2 on C3 of up to 4 mm or 40%
developing normally and appears normal displacement.
upon physical examination. 2. A 10-year-old who underwent tonsillec-
2. A 6-month-old infant is brought in 3 days tomy 3 days ago presents to the emergency
after a fall due to reduced conscious level. department with his head fixed in a “cock-
On examination there is a bulging anterior robin” position.
fontanelle, and retinal hemorrhages. CT 3. A 6-year-old is involved in a high-speed
head demonstrates subdural hematoma RTA and has an incomplete quadraparesis
adjacent to the falx and over the tentorium. on examination. CT cervical spine demon-
3. A 1-day-old healthy infant with a superficial strates a basion dens interval (BDI) is
swelling over the right parietotemporal 14 mm, Power’s ratio is 1.2 and C1-condyle
region that does not cross the suture lines interval of 9 mm.
and without bruising.
4. Two weeks after a viral syndrome, a 2-year-
old child develops bruising and generalized
petechiae, more prominent over the legs.
No hepatosplenomegaly or lymph node
enlargement is noted. The examination is
otherwise unremarkable. Laboratory test-
ing shows the patient to have a normal
hemoglobin, hematocrit, and white blood
count and differential. The platelet count
is 15,000/μL.

www.ketabpezeshki.com 66485438-66485457
574 PART VII PEDIATRIC NEUROSURGERY

SBA ANSWERS
1. d—12 (E3V3M6)

Pediatric GCS score applies to children under the age of 5 years

Eyes Opening Verbal Response Best Motor Response

4—Spontaneous 5—Smiles, oriented to sounds, follows objects 6—Spontaneous and purposeful


3—To voice 4—Cries (but consolable), inappropriate interactions 5—Withdraws from touch
2—To pain 3—Inconsistently inconsolable, moaning 4—Withdraws from pain
1—No response 2—Inconsolable (persistently), agitated 3—Flexion to pain
1—No response 2—Extension to pain
1—No response

2. a—Vomiting, encephalopathy, and hepatic be used to calculate likelihood of clinically impor-


dysfunction tant TBI (ciTBI) and need for CT imaging. CT is
recommended in all children with GCS 14 or
Reye syndrome occurs most frequently after a altered mental status (agitation, somnolence,
viral illness and is characterized by the onset of repetitive questioning, slow response to verbal
severe vomiting followed by the development communication) or palpable skull fracture—4.3-
of encephalopathy and hepatic dysfunction. 4.4% risk of ciTBI. If these are not present,
The onset of these symptoms typically occurs criteria for scanning are based on age of patient.
within several days after the onset of the viral ill- In those under 2 years old, inpatient observation
ness and commonly during a period when the or CT head should be considered if there is
child seems to be recovering from this illness. occipital/parietal/temporal scalp hematoma,
In association with severe—often projectile— LOC for >5 sec, not acting normally according
vomiting, which occurs for a transient period, to parent, or a severe mechanism of injury (e.g.
are progressive encephalopathic changes that motor vehicle crash with patient ejection, death
may follow stages from delirium through confu- of another passenger, or rollover; pedestrian/
sion, agitation, and lethargy to coma if cyclist without helmet struck by motor vehicle;
untreated. It is associated with the ingestion of fall of more than 3 ft if <2 years or 5 ft if
aspirin during the antecedent viral illness, hence >2 years old; head struck by a high impact
aspirin is no longer recommended or used for object). In those over 2 years old, inpatient obser-
the treatment of febrile illnesses in children. vation or CT head should be considered if there is
Alternative antipyretics, including nonsteroidal history of LOC, or vomiting, or a severe mecha-
anti-inflammatory drugs and acetaminophen nism of injury, or severe headache. In the absence
(Tylenol), have replaced aspirin as the primary of any of these age-group specific criteria, the risk
therapy for such illnesses. Children with some of clinically important TBI is <0.02% (under 2)
disorders, including juvenile rheumatoid arthri- and <0.05% (over 2) and CT head is not recom-
tis and Kawasaki disease, continue to be given mended. Decisions regarding whether to observe
aspirin to treat these disorders (and in this case or scan a patient should be based on clinical expe-
CVST). Efforts to reduce the risk for develop- rience, multiple findings rather than isolated,
ment of Reye syndrome in these children have worsening condition, age <3 months and paren-
included influenza vaccination annually and vac- tal preference.
cination against chickenpox. Careful monitoring
of these children also is necessary to ensure early FURTHER READING
recognition and treatment of Reye syndrome Kuppermann N, Holmes JF, Dayan PS, et al. Pediatric Emer-
should it occur. gency Care Applied Research Network (PECARN). Identifi-
cation of children at very low risk of clinically-important brain
3. d—Severe mechanism of injury include falls injuries after head trauma: a prospective cohort study. Lancet
of more than 2 ft if <2 years old or 6 ft if 2009;374(9696):1160-70.
>2 years old.

For children presenting after head injury with


GCS 14 or 15 the PECARN algorithm may

www.ketabpezeshki.com 66485438-66485457
43 PEDIATRIC HEAD AND SPINAL TRAUMA 575

4. e—Witnessed loss of consciousness lasting In the UK, NICE guidelines (CG176) cover cri-
more than 5 min teria for CT scanning in children with head
injuries

Risk Factors Actions Required

Suspicion of non-accidental injury CT head scan within 1 h if any one risk factor present
Post-traumatic seizure but no history of epilepsy
On initial assessment, GCS less than 14, or for children
under 1 year GCS (pediatric) less than 15.
GCS less than 15 at 2 h after the injury.
Suspected open or depressed skull fracture or tense
fontanelle.
Any sign of basal skull fracture (hemotympanum,
“panda” eyes, cerebrospinal fluid (CSF) leakage from
the ear or nose, Battle's sign).
Focal neurological deficit.
For children under 1 year, presence of bruise, swelling
or laceration of more than 5 cm on the head

Loss of consciousness lasting more than 5 min CT head scan within 1 h if more than one risk factor
(witnessed). present
Abnormal drowsiness. Observe for 4 h if only one risk factor present, but
Three or more discrete episodes of vomiting. CT head within 1 h if develops GCS <15, further
Dangerous mechanism of injury (high-speed road vomiting or further episode of abnormal drowsiness
traffic accident either as pedestrian, cyclist or vehicle
occupant, fall from a height of greater than 3 m, high-
speed injury from a projectile or other object).
Amnesia (antegrade or retrograde) lasting more than
5 min.
Head injury (without other risk factors) on warfarin CT head scan within 1 h

5. a—5 mmHg younger than 1 year forms an inward buckling


of the bones forming a “cup shape,” termed a
The upper limit of normal when measuring ICP in “ping-pong fracture.” In the neonate, ping-pong
a child (aged 4-16 years) using lumbar puncture ball or pond fractures occur with indentation of
is generally considered to be 18 cm CSF the bone surface without disruption of the conti-
(12.9 mmHg). However, this applies to children nuity of the bone (similar to green stick fractures
who are comfortable in the lateral decubitus posi- of the long bones). Typically the outer table is
tion with the knees flexed. A crying, distressed fractured around the periphery, while the inner
child will cause the ICP to rise and will be difficult table fractures at the center. In newborns, the
to interpret. In an infant undergoing ICP mea- main cause of the depressed fractures is birth
surement in the horizontal position while venti- trauma, which includes various perinatal factors
lated following acute head injury, an upper limit such as sacral promontory, uterine fibroids, exos-
of 5 mmHg is considered normal. An older child tosis of the lumbar vertebra, symphysis pubis, and
with a closed skull with the same circumstances ischial spine. However, in the postnatal period,
and in the same position will have ICP values sim- the main cause is head trauma. For children with-
ilar to an adult with an upper limit of normal of out evidence of neurological or radiographic
10 mmHg. intracranial lesions, there is no differences
between conservative and surgical management
FURTHER READING strategy in terms of future neurological sequels
Wiegand C, Richards P. Measurement of intracranial pressure or seizures. The natural history of these
in children: a critical review of current methods. Dev Med depressed skull fractures is variable, with some
Child Neurol 2007;49(12):935-41. Review. PubMed PMID: elevating spontaneously over time and others
18039242.
remaining depressed. Several nonsurgical eleva-
6. d—Ping-pong fracture tion techniques that use the fact that the bone
is in partial continuity have been demonstrated;
Depressed skull fractures occur in 7-10% of the these techniques include elevation using digital
children admitted to hospital with a head injury. pressure, and vacuum devices such as a breast
Depressed skull fractures that occur in children pump and a vacuum extractor which carry the

www.ketabpezeshki.com 66485438-66485457
576 PART VII PEDIATRIC NEUROSURGERY

disadvantages of patient discomfort, inability to may also contribute to enlargement of the skull
obtain complete correction of the depression, defect, such that cerebral herniation through it
and creation of local cephalhematoma with the causes a new neurological deficit. Presentation
procedure. It has been demonstrated that the dee- is with scalp swelling at the site of the fracture,
per the depressed bone (>1 cm), the higher the skull defects, persistent or progressive neurolog-
risk of dural laceration and cortical laceration in ical deficits, and seizures. Ideally, young children
adults and older children, but less clear in neonate with a linear skull fracture managed conserva-
and infant populations. Surgical treatment is tively should be followed up (e.g. at 1 year) to
required in cases where the fragments are exclude the development of a growing skull frac-
depressed to the depth of at least one thickness ture. Once the diagnosis of growing skull fracture
of the skull, and in those with intracranial hema- is made management is surgical resection of the
toma, dural laceration/CSF leak, cosmetically leptomeningeal cyst and degenerated brain tissue,
deforming defects, gross wound contamination, water-tight repair of the dural defect (either pri-
and established wound infection. mary or with duraplasty) and closure of the bony
skull defect. The dural defect may be larger than
Image with permission from Law M, Som P, Naidich T. the bone defect, such that sinus bleeding/injury at
Problem Solving in Neuroradiology, Saunders, Else-
vier; 2011. one or both ends of the dural tear. In late stages of
growing skull fractures, the size of the bone defect
FURTHER READING increases and the deformity of the bone near the
Zalatimao O, et al. Treatment of depressed skull fractures in fracture is usually severe. In younger children
neonates using percutaneous microscrew elevation. J Neuro- (especially infants) it is technically difficult to split
surg Pediatrics 2012;9(6):676-9. the skull bone to make enough materials for clo-
sure of the defect. Issues around cranioplasty
7. d—Duraplasty and autologous cranioplasty which are controversial: age of the patient, tim-
ing, cranioplasty type, and resorption of the
A rare complication after linear skull fracture in autologous bone. If GSF is diagnosed in the early
young children (usually younger than 2 or 3 years) stages, especially the prephase of GSF, these
is a “growing” skull defect at the fracture site. In problems can be easily resolved.
these cases, the dura is torn under a linear skull
fracture and a pouch of arachnoid passing Image with permission from Carter R, Anslow P. Imag-
through the defect and expands, acting as a ing of the calvarium. Semin Ultrasound CT MR 2009;30
(6):465-91.
one-way valve that traps CSF and causes progres-
sive pressure erosion of the fractured edges to
enlarge the fracture. Brain growth which pro- FURTHER READING
duces pulsating, spreading tensile pressure forces Liu X. Growing skull fracture stages ad treatment strategy.
on the edges of an unrepaired dural laceration J Neurosurg Pediatrics 2012;9:670-5.

8. a—Rewarming after hypothermia should not occur faster than 1 °C/h

Therapy Guidance

CPP A minimum CPP of 40 mmHg may be considered in children with TBI


A CPP threshold of 40-50 mmHg may be considered; there may be age-specific thresholds
with infants at the lower end and adolescents at the upper end of this range

Brain If brain oxygen monitoring is used, maintenance of oxygen tension 10 mmHg may be
oxygenation considered

Hyperosmolar 3% hypertonic saline (0.1-1 ml/kg of body weight per hour) should be considered for the
therapy treatment of intracranial hypertension

Hyperventilation Avoidance of prophylactic severe hyperventilation to a PaCO2 < 30 mmHg may be


considered in the initial 48 h after injury
If hyperventilation is used in the management of refractory intracranial hypertension,
advanced neuromonitoring for evaluation of cerebral ischemia may be considered
Temperature Moderate hypothermia (32-33 °C) beginning early after severe TBI for only 24-h duration
control should be avoided
Moderate hypothermia (32-33 °C) beginning within 8 h after severe TBI for up to 48-h
duration should be considered to reduce intracranial hypertension
If hypothermia is induced for any reason, rewarming at a rate of >0.5 °C/h should be avoided

Continued

www.ketabpezeshki.com 66485438-66485457
43 PEDIATRIC HEAD AND SPINAL TRAUMA 577

Therapy Guidance
Barbiturates High-dose barbiturate therapy may be considered in hemodynamically stable patients with
refractory intracranial hypertension despite maximal medical and surgical management
When high-dose barbiturate therapy is used to treat refractory intracranial hypertension,
continuous arterial blood pressure monitoring and cardiovascular support to maintain
adequate cerebral perfusion pressure are required

Corticosteroids The use of corticosteroids is not recommended to improve outcome or reduce ICP for
children with severe TBI

Anesthetic drugs Etomidate may be considered to control severe intracranial hypertension; however, the risks
resulting from adrenal suppression must be considered
Thiopental may be considered to control intracranial hypertension
As stated by the FDA, a continuous infusion of propofol for either sedation or the
management of refractory intracranial hypertension in infants and children with severe TBI
is not recommended

Antiseizure drugs Prophylactic use of antiseizure therapy is not recommended for children with severe TBI for
preventing late post-traumatic seizures
Prophylactic antiseizure therapy may be considered as a treatment option to prevent early
post-traumatic seizures in young pediatric patients and infants at high risk of seizures after
head injury
Nutrition Evidence does not support the use of immune-modulating diet to improve outcome

Adapted from Kochanek PM, Carney N, Adelson PD, et al. Guidelines for the acute medical management of severe traumatic
brain injury in infants, children, and adolescents—second edition. Pediatr Crit Care Med. 2012;13(Suppl. 1):S1-82.

9. c—Decompressive craniectomy should only ideal timing and method of craniectomy, as well
be considered after the onset of late signs as its impact on long-term outcome.
of neurologic deterioration.
FURTHER READING
Consider ICP monitoring in infants and children Kochanek PM, Carney N, Adelson PD, et al. Guidelines for
with severe TBI and treating when ICP exceeds the acute medical management of severe traumatic brain
20-25 mmHg. The presence of coagulopathy injury in infants, children, and adolescents—second edition.
Pediatr Crit Care Med 2012;13(Suppl. 1):S1-82.
would contraindicate the placement of an ICP
monitor, in which situation the Cushing reflex 10. d—Shaken baby syndrome
and autonomic dysfunction might be the only
indicators of increased ICP. CSF drainage Abusive head trauma (shaken baby syndrome)
through an external ventricular drain may be remains the most common cause of death in chil-
considered in the management of increased dren who are victims of non-accidental injury
ICP, with optional addition of a lumbar drain (NAI), and this usually occurs during the first
in refractory intracranial hypertension despite a year of life. The diagnosis is often missed since
functioning external ventricular drain, open basal no history of head trauma is provided, and the
cisterns, and no evidence of a mass effect. CSF signs and symptoms the child displays may be
drainage via EVD resulted in ICP control in non-specific, such as vomiting, poor feeding,
87% of pediatric patients. These three studies irritability or lethargy. Primary injuries (conse-
also confirmed that refractory raised ICP is asso- quence of the initial trauma or impact of
ciated with poor outcome, with 100% mortality force) include epidural hemorrhages, subdural
in all patients with refractory intracranial hyper- hemorrhages, subarachnoid hemorrhages, skull
tension after CSF drainage. Decompressive fractures, intraventricular hemorrhages, cortical
craniectomy with duraplasty may be considered contusions, diffuse axonal injury (DAI) and intra-
for patients who are showing early signs of parenchymal hematomas. Epidural hemorrhages
neurologic deterioration or herniation or are in children require a direct impact of forces and
developing intracranial hypertension refractory are generally venous bleeds which result from
to medical management during the early stages tears in the dural sinus or diploic veins. Subdural
of their treatment. Multiple small case series hemorrhages do not require direct impact
also show that craniectomy is an effective rescue and may result from inertial shearing or rota-
intervention in patients with sustained ICP tional forces but most commonly result from
greater than 20 mmHg, clearly demonstrating abrupt deceleration. Subdural hemorrhages
craniectomy has a role in ICH management. Cer- occur in a space created by the traumatic separa-
tain questions are still unanswered, including the tion of the arachnoid from the dura mater and

www.ketabpezeshki.com 66485438-66485457
578 PART VII PEDIATRIC NEUROSURGERY

are caused by bleeding from bridging veins. Sub- regression, iridiodialysis, retinal dialysis or
dural hematoma suggestive of abuse may be detachment, intraocular hemorrhage, optic atro-
acute, subacute or chronic, frequently bilateral, phy or papilledema. Multiple mechanisms of
closely related to the falx, layering over the retinal hemorrhage have been postulated,
tentorium and accompanied by hemispheric including direct tracking of blood from intra-
hypodensity (HH; hypodense edematous cortex cranial hemorrhage, hemorrhage secondary to
and underlying white matter in multiple cere- raised intracranial pressure or retinoschisis.
brovascular territories) due to secondary insults
(e.g. cardiac arrest, hypoxia and hypotension). Image with permission from Adam D, editor. Grainger &
Allison's Diagnostic Radiology, 6th ed. Churchill Living-
DAI results from sudden acceleration and stone, Elsevier; 2014.
deceleration forces which may be combined with
rotational forces, which disrupts fiber tracts. FURTHER READING
Infants are more susceptible to DAI due to their Paul AR, Adamo MA (n.d.). Non-accidental trauma in pedi-
large head-to-body ratio, weak neck musculature atric patients: a review of epidemiology, pathophysiology,
and thinner skull. DAI typically affects subcorti- diagnosis and treatment. Transl Pediatr. Retrieved from
cal white matter, the corpus callosum, the brain- http://www.thetp.org/article/view/4150/5031.
stem and internal capsule. Intraparenchymal
hematomas can result from shearing-straining 11. c—Torn labial frenulum in a mobile child
injuries due to rupture of small intraparenchymal
blood vessels and typically occurs in the fronto- Abuse can be classified broadly as physical, emo-
temporal white matter. The differential diag- tional, sexual and neglect, with overlaps occurring
nosis of a child with intracranial hemorrhage in most cases. It has been estimated that 10-15%
includes accidental trauma or NAT, birth of childhood injuries resulting in emergency
trauma, coagulopathy, congenital vascular mal- department visits are caused by abuse. Fractures
formations, spontaneous SDH (related to benign are the second most common presentation after
enlargement of the subdural space), and meta- soft-tissue injury and bruising. In general, any
bolic deficiencies such as glutaric aciduria type delay in presentation, any injury that does not
I. Associated ocular findings which increase the fit with the explanation offered or the develop-
likelihood of NAI (in the absence of verifiable mental stage of the child, or a changing or
history) include retinal hemorrhage, periorbital conflicting account should raise suspicion.
hematoma, eyelid laceration, subconjunctival/ Important differential diagnosis of NAI includes
intraocular hemorrhage, subluxed or dislocated accidental injury, osteogenesis imperfecta, ITP,
lens, cataracts, glaucoma, anterior chamber angle Mongolian blue spot, and scalded skin syndrome.

Non-accidental Injury: Clinical Features


Extra-CNS Signs
of Physical Abuse Situations Highly Suspicious of NAI

Fractures • children under 18 months with a fracture


• where the fracture is inconsistent with the developmental stage of the child
• multiple fractures and/or fractures of different ages
• rib fractures in children with normal bones and no history of major trauma
• a fractured femur in a pre-mobile child.

Bruises Bruises cannot be aged accurately. Bruising that suggests the possibility of physical
abuse includes:
• bruising in children who are not independently mobile
• bruising in babies
• bruising away from bony prominences
• bruising to the face, back, abdomen, arms, buttocks, ears, and hands
• multiple bruises in clusters
• multiple bruises of uniform shape
• bruising that carries an imprint (of an implement or cord).

Oral injuries and bites There is not enough evidence in the literature to support the view that a pre-mobile
child with a torn labial frenum in isolation is diagnostic of child abuse. A full
assessment should be performed.
• 1% of emergency department attendances are due to bites.
• Human bites can be distinguished from animal bites.

Continued

www.ketabpezeshki.com 66485438-66485457
43 PEDIATRIC HEAD AND SPINAL TRAUMA 579

Extra-CNS Signs
of Physical Abuse Situations Highly Suspicious of NAI
Burns A high percentage of childhood burns are due to abuse (2-35% overall; up to 45% for
genital and perineal burns). The two kinds of burns most often seen in abused
children are scald burns (from contact with hot liquids) and thermal burns (contact
with hot objects). In accidental burns, the head, neck, anterior trunk, and arms are
the most often affected. In cases of abuse, hands, legs, feet, and buttocks were more
likely to be involved. The anterior aspect of the hand was more likely to be involved
in accidental burns and the dorsum of the hand was more likely involved in abuse
cases. Suspicious burns include patterned contact burns in clear shape of hot object
(fork, clothing iron, curling iron, cigarette lighter) and classic forced immersion burn
patterns with sharp stocking-and-glove demarcation and sparing of flexed protected
areas. In addition, splash/spill burn patterns in children which is not consistent with
the history of the child’s developmental level should be considered suspicious.
Cigarette burns should always raise concern for abuse, as should any evidence of
delay in seeking medical treatment

FURTHER READING Cancer Research UK, http://www.cancerresearchuk.org/health-


Roderick C, Davies E, Rabb L, Bowley DM. What does the professional/cancer-risk-statistics; 2016 [accessed 01.02.16].
surgeon need to know about safeguarding children? How to
Pearce MS, Salotti JA, Little MP, et al. Radiation exposure
spot signs of abuse. RCS Bull 2015;349-532. http://dx.doi.
from CT scans in childhood and subsequent risk of leukemia
org/10.1308/rcsbull.2015.345.
and brain tumors: a retrospective cohort study. Lancet
12. c—The lifetime additional risk of cancer due 2012;380(9840):499-505.
to CT scans in children is 1 extra case for Mathews JD, Forsythe AV, Brady Z, et al. Cancer risk in
10,000 children scanned. 680,000 people exposed to computed tomography scans in
childhood or adolescence: data linkage study of 11 million
Children are considerably more sensitive to radia- Australians. Br Med J 2013;346:f2360.
tion than adults, have a longer life expectancy than
adults (a larger window of opportunity for expres- 13. e—Age over 9 years
sing radiation damage), and may receive a higher
radiation dose than necessary if CT settings are The pediatric cervical spine does not become
not adjusted for their smaller body size. For a cumu- adult like until about the age of 8 years. These fac-
lative dose of between 50 and 60 mGy to the head tors increase the risk of injury/instability to the
(i.e. 2-3 CT head scans), the investigators reported levels of C1-C3 include large head-to-body ratio,
a threefold increase in the risk of brain tumors; the ligamentous laxity, relative paraspinal muscle
same dose to bone marrow (i.e. 5-10 CT head scans) weakness, horizontal/shallow facets. Cervical
resulted in a threefold increase in the risk of leuke- spine injuries occur mainly in the upper cervical
mia. However, it is important to stress that the abso- spine above C4 in patients 8 years of age or youn-
lute additional cancer risks associated with CT ger which most often involve the occiput, C1, and
scans (1 additional case for every 2000 people C2 complex and thus carries increased risk
scanned) are small compared to the baseline cancer of fatality. Patients older than 8 years of age
risk (i.e. 1 in 35 (men) or 1 in 20 (female) risk of typically sustain more injuries below C4 and carry
cancer before the age of 50, or 1 in 5 lifetime risk). a much lower fatality rate. Because of these
In children, the lifetime extra risk of cancer from biomechanical differences, younger children
a single CT scan was small—about one case of (0-8 years) tend to have fewer fractures and
cancer for every 10,000 scans performed on chil- greater incidence of SCIWORA (spinal cord
dren (baseline 1 in 500 risk of developing some injury without radiographic abnormality).
form of cancer before the age of 14).
14. e—Children with Down's syndrome are at
FURTHER READING higher risk of cervical injuries as it is associ-
http://www.cancer.gov/about-cancer/causes-prevention/risk/ ated with congenital fusion of cervical
radiation/pediatric-ct-scans. vertebrae.
Berrington de Gonzále A, Mahesh M, Kim KP, Bhargavan M, Spine fractures are usually the result of high-
Lewis R, Mettler F, Land C. Projected cancer risks from com-
speed and impact injuries such as a motor vehicle
puted tomographic scans performed in the United States in
2007. Arch Intern Med 2009;169:2071-7.
accident or a fall from great height. Spine

www.ketabpezeshki.com 66485438-66485457
580 PART VII PEDIATRIC NEUROSURGERY

fractures in children represent 1-3% of all pediat- transient neurological signs or symptoms refer-
ric fractures. The incidence of pediatric spine able to the spinal cord after a traumatic event
injuries peaks in 2 age groups; children but normal neurological examination can develop
<5 years old and children >10 years old. There delayed onset SCIWORA, hence there is debate
is a seasonal peak of pediatric spinal injuries from as to whether they should just be managed as such
June to September, during summer break. There initially despite the absence of objective signs. By
is another seasonal peak in the 2 weeks surround- definition, normal acute flexion/extension X-rays
ing the Christmas holiday. The mechanism of are required for a diagnosis of SCIWORA. If
injury in the pediatric population varies with paraspinous muscle spasm, pain, or uncoopera-
age. In young children aged 0-9 years, the pre- tion prevents dynamic studies, they recom-
dominant causes of injury are falls and mended external immobilization until the child
automobile-versus-pedestrian accidents can cooperatively flex and extend the spine for
(>75%). In children aged 10-14 years, motor dynamic X-ray assessment. Although concern
vehicle accidents (40%) are the major cause of exists for the late development of pathological
lumbar fractures, and falls and automobile-ver- intersegmental motion in children with SCI-
sus-pedestrian accidents are less prevalent. In WORA following normal flexion and extension
children 15-17 years of age, motor vehicle and studies, there has been no documentation of such
motorcycle accidents become the leading cause instability ever developing. MRI is recommended
of spine injuries (>70%), and there is also an in children with potential SCIWORA as it may
increase in sports-related spine trauma. A spinal include identifying signal change or intramedul-
cord injury should be suspected if the child has lary injury (prognostic value), excluding compres-
a history of numbness, tingling, or brief paralysis sive lesions of the cord/roots needing surgery,
even if it has recovered subsequently. Some chil- exclude spinal ligamentous disruption that might
dren are predisposed to cervical injuries more warrant surgical intervention (in situations where
than others includes children with Down's Syn- dynamic flexion/extension radiographs cannot be
drome (atlanto-axial instability), Klippel-Feil done or would be superfluous preoperatively),
syndrome (congenital fusion of cervical spine), guiding treatment regarding length of external
previous cervical spine surgery, and other syn- immobilization (e.g. evidence of residual ligamen-
dromes affecting the cervical spine. Hyperflexion tous injury), and/or determining when to allow
injuries are most common and are associated with patients to return to full activity.
wedge fractures of the anterior cervical bodies Pang has also recommended somatosensory
with disruption of posterior aspects. The classic evoked potential (SSEP) screening of children
triad of symptoms of cervical spine injury is local- with presumed SCIWORA to detect subtle pos-
ized neck tenderness, muscle spasm and terior column dysfunction when clinical findings
decreased range of motion. are inconclusive, evaluating head-injured, coma-
tose, or pharmacologically paralyzed children,
FURTHER READING distinguishing between intracranial, spinal, or
Rozzelle CJ, et al. Management of pediatric cervical spine and peripheral nerve injuries, and/or providing a
spinal cord injuries. Neurosurgery 2013;72:205-26. baseline for comparison with subsequent evalu-
ations. Neither spinal angiography nor myelo-
15. e—In the presence of normal dynamic cervi- graphy is recommended in the evaluation of
cal X-rays, cervicothoracic bracing for patients with SCIWORA. Because subluxation
12 weeks is appropriate initially and/or malalignment/ligamentous instability
are, by definition, absent in SCIWORA, the
SCIWORA is a widely recognized form of spinal mainstay of treatment has been immobilization
cord injury, occurring almost exclusively in chil- and avoidance of activity that may either lead
dren (due to relative elasticity of the spinal col- to exacerbation of the present (ligamentous
umn relative to the spinal cord), and is and spina cord) strain/injury or increase the
characterized by objective signs of myelopathy potential for recurrent injury. Medical manage-
as a result of trauma in the absence of any radio- ment issues such as blood pressure support and
graphically evident fracture, dislocation, or liga- pharmacological therapy apply. Treatment con-
mentous instability (on static or dynamic X-rays sisting of cervicothoracic bracing for patients
or CT). Children presenting with a history of with cervical-level SCIWORA for 12 weeks

www.ketabpezeshki.com 66485438-66485457
43 PEDIATRIC HEAD AND SPINAL TRAUMA 581

and avoidance of activities that encourage flex- the odontoid may angulate posteriorly in 4% of
ion and extension of the neck for an additional children; (6) C2-C3 pseudosubluxation (can be
12 weeks (i.e. 6 months total) has not been asso- assessed with Swischuk’s line); (7) the ossification
ciated with recurrent injury. Patients with normal center of the anterior arch of C1 may be absent
MRI and SSEP findings following transient def- in the first year of life; (8) the atlanto-dens interval
icits or “symptoms only” may be managed with a may be as wide as 4.5 mm and still be normal; (9) the
cervical collar for 1-2 weeks. Despite this, it is width of the prevertebral soft tissues varies widely,
unclear what role immobilization plays given that especially with crying, and may be mistaken for
dynamic radiographs have confirmed the absence swelling; and (10) horizontal facets in young chil-
of instability required for diagnosing SCI- dren can be mistaken for a fracture.
WORA, and furthermore, that follow-up with
dynamic radiographs in these children has not 17. c—Children over 3 years of age should not
shown development of delayed pathological have cervical spine imaging if they are alert,
intersegmental motion. However, if (normal) have no neurological deficit, no midline cer-
physiological motion of the spinal column can vical tenderness, no painful distracting injury,
potentiate spinal cord injury in these patients no unexplained hypotension, and are not
when there is no malalignment, subluxation, or intoxicated.
lesion causing cord compression, then immobili-
zation may be warranted. Anteroposterior (AP) and lateral cervical spine
radiography (plus open mouth views if >9 years
Image with permission from Klimo Jr P, Ware ML, old) or high-resolution CT is recommended to
Gupta N, Brockmeyer D. Cervical spine trauma in the assess the cervical spine in children. High-
pediatric patient. Neurosurg Clin N Am 2007;18
(4):599-620.
resolution CT scan with attention to the suspected
level of neurological injury, radiographic abnor-
FURTHER READING mality or area not adequately visualized on plain
Rozzelle CJ, et al. Spinal cord injury without radiographic films. Flexion and extension cervical radiographs
abnormality (SCIWORA). Neurosurgery 2013;72:227-33. or fluoroscopy are recommended to exclude gross
ligamentous instability if suspected following static
16. b—Localized kyphosis in mid-cervical area radiographs/CT. MRI is recommended to exclude
spinal cord or nerve root compression, evaluate lig-
Ten unique features of the pediatric cervical spine amentous integrity, or provide information regard-
that can cause confusion during the trauma evalua- ing neurological prognosis. Common normal
tion: (1) the apical ossification center can be mis- findings on cervical spine radiographs obtained
taken for a fracture; (2) the synchondrosis at the on young children which may be mistaken for acute
base of the odontoid can be mistaken for a fracture; traumatic injuries are pseudosubluxation of C2 on
(3) vertebral bodies appear rounded-off or wedged, C3, overriding of the anterior atlas in relation to
simulating a wedge compression fracture; (4) sec- the odontoid on extension, exaggerated atlanto-
ondary centers of ossification at the tips of the spi- dens intervals, and the radiolucent synchondrosis
nous processes can be mistaken for a fracture; (5) between the odontoid and C2 body.

Cervical Spine Imaging in Children: Indications

Do not image cervical spine under 3 years of age who have Do not image cervical spine over 3 years of age
experienced trauma and who: who have experienced trauma and who:

• have a GCS greater than 13, • are alert,


• have no neurological deficit, • have no neurological deficit,
• have no midline cervical tenderness, • have no midline cervical tenderness,
• have no painful distracting injury, • have no painful distracting injury,
• are not intoxicated, • do not have unexplained hypotension,
• do not have unexplained hypotension (i.e. due to SCI), • and are not intoxicated.
• and do not have motor vehicle collision (MVC), a fall from
a height >10 ft, or non-accidental trauma (NAT) as a
known or suspected mechanism of injury.

www.ketabpezeshki.com 66485438-66485457
582 PART VII PEDIATRIC NEUROSURGERY

FURTHER READING Type IV is a rare and usually fatal injury that


Rozzelle CJ, et al. Management of pediatric cervical spine and involves rotatory fixation with a posterior shift.
spinal cord injuries. Neurosurgery 2013;72:205-26. If the diagnosis of AARF is suspected after clinical
examination and plain radiographic study, a
18. c—Atlanto-axial rotatory subluxation (or
dynamic CT study should be obtained (three-
fixation)
position CT with C1-C2 motion analysis). The
Fixed rotatory subluxation of the atlanto-axial longer AARF is present before attempted treat-
complex (AARF) is not unique to children but ment, the less likely reduction can be accom-
is more common during childhood. AARF may plished or maintained. Acute AARF (<4 weeks
present following minor trauma (30%), in associ- since onset) that has not reduced spontaneously
ation with an upper respiratory infection, or with- should undergo attempted reduction with manip-
out an identifiable inciting event. It is associated ulation or halter traction. Chronic AARF (4 weeks
with Down’s syndrome, Morquio syndrome, duration or more since onset) should undergo
spondyloepiphyseal dysplasia, Larsen’s syn- attempted reduction with halter or skull tong/
drome, achondroplasia and Grisel’s syndrome halo traction. Reductions achieved with manipu-
(post-inflammatory). The head is rotated to one lation or halter traction should be immobilized
side with the head tilted to the other side (due with a cervicothoracic (Minerva) brace, while
to SCM spasm) causing the so-called “cock- those requiring tong/halo traction should be
robin” appearance mimicking torticollis, unable immobilized in a halo. Length of immobilization
to turn his/her head past the midline, attempts should be proportional to the length of time that
to move the neck are often painful and usually the subluxation was present before treatment.
there is no neurological deficit. Plain cervical Surgical arthrodesis can be considered for those
spine radiographs may reveal the lateral mass of with irreducible subluxations, recurrent subluxa-
C1 rotated anterior to the odontoid on a lateral tions, or subluxations present for 3 months’
view. Fielding and Hawkins type I rotatory sub- duration.
luxation is characterized by rotatory fixation Green’s skeletal trauma in children. Fielding
without anterior shift of the atlas (transverse lig- and Hawkins classification of atlanto-axial
ament intact), type II consists of rotatory sublux- rotatory displacement.
ation with an anterior shift (ADI) of greater than Image with permission from Klimo Jr P, Ware ML,
3 mm but less than 5 mm due to transverse liga- Gupta N, Brockmeyer D. Cervical spine trauma in the
ment compromise and may also be associated pediatric patient. Neurosurg Clin N Am 2007;18
with spinal instability. Type III involves rotatory (4):599-620.
subluxation with an ADI of greater than 5 mm.

A B C D
A, Type I, no anterior displacement, and the odontoid acts as pivot. B, Type II, anterior displacement of 3-5 mm and
one lateral articular process acting as the pivot. C, Type III, anterior displacement of more than 5 mm. D, Type IV,
posterior displacement. (With permission from Dormans JP. Evaluation of children with suspected cervical spine
injury. Instr Course Lect 51:407, 2002.)

FURTHER READING ANSWERS 19–27


Rozzelle CJ, et al. Management of pediatric cervical spine and
spinal cord injuries. Neurosurgery 2013;72:205-26. Additional answers 19–27 available on
ExpertConsult.com

www.ketabpezeshki.com 66485438-66485457
43 PEDIATRIC HEAD AND SPINAL TRAUMA 583

EMI ANSWERS coagulopathy and hemorrhage. Clotting


studies show prolonged PT and PTT,
28. 1—f, Mongolian blue spots. These are decreased fibrinogen concentration, and an
bluish-gray lesions located over the buttocks, increase in fibrin degradation products.
lower back, and occasionally, the extensor Blood film will show fragmented cells and
surfaces of the extremities. They are com- few platelets. Microvascular thrombosis
mon in infants of African, Asian, and Latin (fibrin clot formation) can lead to tissue
American origin. They tend to disappear by ischemia and necrosis, further capillary dam-
1-2 years of age, although those on the age, release of thromboplastic substances,
extremities may not fully resolve. 2—g, and worsen DIC. Simultaneous activation
Non-accidental injury. This case is suspi- of the fibrinolytic system produces increased
cious due to the delayed presentation, retinal amounts of fibrin degradation products,
hemorrhage and distribution of the subdural which inhibit thrombin activity. Treatment
hemorrhage on CT scan. 3—a, Cephalhema- of the precipitating cause is critical.
tomas. These are subperiosteal hematomas
which thus do not cross the suture line, do 29. 1—g, Pseudosubluxation. Normal anterior
not discolor the scalp, and swelling usually translation that can occur between C2 and
progresses over the first few hours of life. C3 and less frequently between C3 and C4
Most cephalhematomas resolve within the in patients younger than 8 years. May be
first few weeks or months of life without seen in 40% of children at C2-C3 level and
residual findings. By contrast, caput succeda- in 14% of children at the C3-C4 level occurs
neum is soft-tissue swelling of the scalp because of increased ligamentous laxity,
involving the presenting delivery portion of more horizontal nature of facet joint (30°
the head. This lesion is sometimes ecchy- versus 60-70° in adult); in children, fulcrum
motic and can extend across the suture lines. of motion that is relatively greatest at
The edema resolves in the first few days of C2-C3 level (compared w/C5-C6 in the
life. 4—e, Idiopathic thrombocytopenic pur- adult). The posterior cervical line (spinolami-
pura (ITP). This is the most common form of nar line of Swischuk) is used to distinguish
thrombocytopenic purpura, usually triggered pathologic displacement from normal ante-
by a viral infection causing immune- rior displacement. A line is constructed con-
mediated sequestration and destruction of necting the anterior aspect of the spinous
platelets in the spleen. Treatment for processes of C1-C3. If the anterior aspect
ITP consists of observation and/or gamma of the spinous process of C2 is more than
globulin and steroids. Splenectomy is 1.5 mm from this line, an injury should be
reserved for the most severe and chronic suspected. 2—b, Atlanto-axial instability.
forms. 5—c, Disseminated intravascular The history is that of atlanto-axial rotatory
coagulation. The disorder, which can be trig- subluxation, which is termed Grisel’s syn-
gered by a variety of disorders (e.g. endotoxin drome if secondary to inflammatory condi-
shock, trauma, malignancy) results in micro- tions in the neck. 3—a, Atlanto-occipital
vascular thrombosis causing a consumptive dislocation (dissociation).

www.ketabpezeshki.com 66485438-66485457
CHAPTER 44

PEDIATRIC VASCULAR NEUROSURGERY


SINGLE BEST ANSWER (SBA) QUESTIONS
1. Which one of the following pathologies is d. Tentorial AVM
most likely to produce the ultrasound appear- e. Vein of galen malformation
ance seen below?
3. This 14-year-old presented with intraven-
tricular hemorrhage. Which one of the fol-
lowing management options is likely to be
discussed based on this right ICA angiogram?

a. Band heterotopia
b. Germinal matrix hemorrhage
c. Hydrocephalus
d. Schizencephaly
e. Subependymal nodules

2. Which one of the following is most likely in a


child with the below MRI appearance pre-
senting with increasing head circumference
and shortness of breath?
a. Coil embolization
b. Extracranial-Intracranial Bypass
c. Intravenous antibiotics
d. Onyx embolization
e. Stereotactic radiosurgery

4. Choroidal vein of Galen malformations pre-


sent most commonly as:
a. Congestive heart failure
b. Delayed developmental milestones
c. Focal neurological deficit
d. Increasing head circumference
e. Subarachnoid hemorrhage

a. Cavernous malformation
b. Developmental venous anomaly
c. Dural AV fistula

584

www.ketabpezeshki.com 66485438-66485457
44 PEDIATRIC VASCULAR NEUROSURGERY 585

5. Which treatment would be preferred in a a. Enlarging aneurysm


vein of Galen malformation patient with a b. Hydrocephalus
extremely high flow shunt and in which per- c. Maturation of hemorrhagic contusion
forating arteries are of small caliber? d. Meningitis
a. Surgical excision e. Vasospasm
b. Transfemoral transarterial embolization
c. Transfemoral transvenous embolization 10. A 22 month old male child with a history of iron
d. Transtorcula deficiency anemia developed progressive leth-
e. VP shunting argy and vomiting over several days, culminat-
ing in flaccid quadriparesis. Brain MRI and
6. Wyburn-Mason syndrome is most accurately MRV confirmed cerebral venous sinus throm-
described by which one of the following bosis with subcortical white matter infarction.
statements? Lumbar puncture revealed elevated opening
a. Facial angioma and ipsilateral pressure, normal cerebrospinal fluid cell count,
parietooccipital AVM protein and glucose. Laboratory studies
b. Multiple visceral, muscosal, and showed Hb 7.5 mg/dl with iron deficiency,
cerebral AVM and a positive lupus anticoagulant. The most
c. Optic tract glioma and midbrain AVM appropriate management strategy is:
d. Retinal angiomatosis, facial hemangioma, a. IV rehydration
and midbrain AVM b. IV rehydration and systemic heparin
e. Retinal angioma and posterior fossa c. IV rehydration and thrombolysis
hemangioblastoma d. IV rehydration and warfarinization
e. IV rehydration and venous stenting
7. Which one of the following statements about 11. Which one of the following extracranial to
pediatric intracranial aneurysms is NOT intracranial bypass grafts is LEAST likely
likely? to be possible in children?
a. Infectious aneurysms are more common a. Encephaloduroarteriosynangiosis
in children than adults b. Encephalomyoarteriosynangiosis
b. Internal carotid bifurcation aneurysms are c. Encephalomyosynangiosis
the commonest type d. Superficial temporal artery to MCA
c. Posterior circulation aneurysms are e. Transposed temporalis muscle forms an
more common that anterior circulation anastomosis
aneurysms
d. There is an association with coarctation of 12. Small, deep-seated pediatric arteriovenous
the aorta malformations may be most appropriately
e. They occur more commonly in males than treated with which one of the following?
females a. Endovascular coil embolization
b. Fractionated stereotactic radiotherapy
8. The approach LEAST likely to be utilized c. Proton beam therapy
during surgery for a dorsolateral brainstem d. Stereotactic radiosurgery
cavernous malformation is: e. Surgical excision
a. Anterior petrosectomy
b. Retrosigmoid
c. Suboccipital EXTENDED MATCHING ITEM (EMI)
d. Supracerebellar infratentorial QUESTIONS
e. Transcallosal
13. Pediatric Vascular Pathology:
9. A 9-year-old boy sustains a left sided penetrat- a. Autosomal Dominant Polycystic Kidney
ing brain injury secondary to air rifle accident. Disease
Initial CT head shows traumatic SAH and b. Coarctation of the aorta
hemorrhagic contusion in the left temporal c. Ehlers-Danlos syndrome
lobe. GCS recovers to 15 and he is discharged d. Fibromuscular dysplasia
after 7 days. He is readmitted 5 days later e. Klippel-Trenaunay syndrome
with intermittent dysphasia and right facial f. Marfan's syndrome
weakness. Which one of the following surgi- g. Osler-Weber-Rendu syndrome
cally treatable conditions are you concerned h. Parkes-Weber syndrome
about? i. Pseudoxanthoma Elasticum

www.ketabpezeshki.com 66485438-66485457
586 PART VII PEDIATRIC NEUROSURGERY

j. Sturge-Weber syndrome 2. Characterized by arachnodactyly, lens dis-


k. Tuberous Sclerosis location, spontaneous pneumothorax,
l. Wyburn-Mason syndrome mitral valve prolapse and associated with
intracranial aneurysms.
For each of the following descriptions, select the 3. Characterized by facial angioma and ipsi-
most appropriate answers from the list above. lateral parietooccipital AVM.
Each answer may be used once, more than once
or not at all.
1. Characterized by spontaneous recurrent
epistaxis, telangiectasias and AVM affecting
lung, liver, brain, and spine.

SBA ANSWERS
1. b—Grade IV germinal matrix hemorrhage 3. b—Extracranial-Intracranial Bypass

Papile's classification: Adult moyamoya patients often present with hem-


Grade I—hemorrhage is confined to the germinal orrhage, leading to rapid diagnosis. In contrast,
matrix children usually present with TIAs or strokes.
Grade II—intraventricular hemorrhage without Other symptoms include seizures, headache, chor-
ventricular dilatation eoatheform movements, ICH/ICH. Risk factors
Grade III—intraventricular hemorrhage with include Asian origin, cranial radiotherapy, NF-1,
ventricular dilatation Down's syndrome among others. Moyamoya
Grade IV—intraventricular rupture and hemor- syndrome should be considered and diagnostic
rhage into the surrounding white matter evaluation begun in any child who presents with
symptoms of cerebral ischemia, especially if the
Image with permission from Coley BD. Caffey's Pediat- symptoms are precipitated by physical exertion,
ric Diagnostic Imaging, 12th ed. Elsevier, Saunders,
hyperventilation, or crying. Hemorrhage from
2013.
moyamoya vessels can be readily diagnosed on
FURTHER READING head CT, with the most common sites of hemor-
Papile LA, Burstein J, Burstein R, Koffler H. Incidence and rhage being the basal ganglia, ventricular system,
evolution of subependymal and intraventricular hemorrhage: medial temporal lobes, and thalamus. Most
a study of infants with birth weights less than 1,500 gm. J suggestive of moyamoya on MRI is the finding
Pediatr 1978 Apr;92(4):529-34. of diminished flow voids in the internal carotid
and middle and anterior cerebral arteries coupled
2. e—Vein of Galen Malformation with prominent collateral flow voids in the
basal ganglia and thalamus. The diagnosis of
Persistence of the single median prosencephalic moyamoya is said to require bilateral symmetrical
vein of Markowski into fetal life results in a dilated stenosis or occlusion of the terminal portion
venous sac in the midline, the Vein of Galen malfor- of the intracranial carotid arteries as well as
mation. Normal deep veins are absent and venous the presence of dilated collateral vessels that
drainage is routed through the median prosence- develop at the base of the brain producing the
phalic vein (or primitive internal cerebral vein). This classic “puff of smoke” appearance on angiogra-
temporary vein drains the choroid plexus from phy. External carotid imaging is essential to
both sides almost between 5th and 10th week of identify pre-existing collateral vessels, so that
fetal life. With simultaneous development of basal surgery, if performed, will not disrupt them.
ganglia, thalamus, and cerebral vascularization, Aneurysms or arterio venous malformations
paired internal cerebral veins appear to progressi- (AVMs), known to be associated with some cases
vely annex the venous drainage of the midline struc- of moyamoya, can also be best detected by con-
tures. The falcine sinus becomes the normal outlet ventional angiography.
for the persistent median prosencephalic vein, Image with permission from Loevner L. Brain imaging:
with the straight sinus hypoplastic or stenosed. case review series, 2nd ed. Elsevier, Mosby, 2009.
Such persistence of fetal pattern maintains the fetal
hemodynamics and as a result some dural sinuses 4. a—Congestive heart failure
may fail to develop altogether.
In the Lasjaunicas classification, VGMs are
Image with permission from Loevner L. Brain imaging: divided into choroidal or mural types. In choroidal
case review series, 2nd ed. Elsevier, Mosby, 2009.

www.ketabpezeshki.com 66485438-66485457
44 PEDIATRIC VASCULAR NEUROSURGERY 587

type VGMs, the arteriovenous shunt is subarach- the shunt completely. In the majority of infants
noid located in the tela choroidea along the cis- and children, it becomes necessary to stage the
terna velum interpositum with arterial feeders embolization ranging from a few weeks to a few
(choroidal arteries, thalamo-perforator arteries, months based on the angioarchitecture and clinical
and pericallosal artery) entering the anterior status. The follow-up endovascular approach is
segment of the median prosencephalic vein. Mul- based on the residual shunt and the architecture of
tiple high-flow fistulas (large volume of shunt) the malformation. Occlusive venopathy is a well-
results in cardiac failure in neonates. The aggres- known delayed event causing progressive neurolog-
sive volume overload results in cerebral ischemia ical deterioration. The acquired venopathy may be
and encephalomalacia. fatal. It is postulated that too long intervals between
In mural type VGMs, a single arteriovenous the embolization procedures would result in high
fistula is situated in the inferolateral wall of the venous pressures in the dural sinuses and cortical
median prosencephalic vein fed by collicular or veins. The high flow venopathy is transmitted to
posterior choroidal arteries (i.e. fewer fistulas, the medullary veins and cortical veins which would
low-flow). They present later in infancy or early result in progressive parenchymal calcifications and
childhood with failure to thrive and macrocephaly refractory seizures.
due to hydrocephalus. Hydrocephalus develops
mainly due to impaired absorption because of 6. d—Retinal angiomatosis, facial hemangi-
absence or stenosis of dural sinuses. oma, and midbrain AVM

7. c—Posterior circulation aneurysms are more


FURTHER READING common that anterior circulation aneurysms.
Rao VR, Mathuriya SN. Pediatric aneurysms and vein of
Galen malformations. J Pediatr Neurosci 2011 Oct; 6(Suppl.
1):S109-17.
8. e—Transcallosal

5. c—Transfemoral transvenous embolization Common approaches to the dorsolateral


brainstem:
Surgical treatment and natural history are associated
with high morbidity and mortality. Adjuvant ven- Lesion Approach
tricular shunting is not necessary unless definitive location
endovascular treatment is unavailable immediately. Dorsal Supracerebellar infratentorial
Emergency embolization is indicated in infants with midbrain
refractory cardiac failure, acute or symptomatic Lateral Subtemporal; orbitozygomatic
hydrocephalus, rapid neurological deterioration or midbrain
when parenchymal calcifications appear on follow- Floor of 4th Suboccipital
up scanning of brain. Neonates or infants having ventricle
encephalomalacia or severe brain damage or severe
Lateral pons Anterior/posterior petrosectomy
parenchymal loss do not need any aggressive treat- and retrosigmoid
ment because of poor clinical outcome in spite of
successful closure of the shunt by embolization. Dorsolateral Suboccipital and far lateral
medulla
Transarterial embolization is employed when the
diagnostic angiogram demonstrates accessible feed-
ing arterial branches from the choroidal and perfo- 9. a—Enlarging aneurysm
rator arteries having sufficient caliber to permit
navigation of the microcatheters. Transvenous tech- Traumatic aneurysms account for 14-39% of all
nique is particularly employed when the perforating pediatric aneurysms. Children with neurological
arteries are not of sufficient caliber to permit nego- deterioration after head injury should be suspected
tiation of the microcatheters. If the shunt is very to have these lesions and investigated accordingly.
large with extremely high flow the venous approach These aneurysms can result from direct trauma
is preferred to avoid migration of the embolic mate- (gunshot wounds, stab wounds, or surgical proce-
rial when delivered by the transarterial route. Occa- dures). Indirect causes are trauma by falcine edge,
sionally it is necessary to use a combination of both sphenoid ridge, and sharp edge of fractured bone.
techniques. In high flow choroidal malformations The patient can present with devastating hemor-
with no accessible femoral approach, occipital bone rhage in 50%. It could be SAH, subdural hematoma
over the torcular is penetrated with a large bore nee- (SDH), ICH or extradural hematoma (EDH).
dle for catheterization of the varix in neonates. How- Aneurysm can bleed 5 h to 10 years after trauma
ever it is necessary to make a burr-hole in older with a mean of 3 weeks. The patient can present
children. The goal of treatment is to reduce the vol- with irritability/unconsciousness or focal signs
ume load initially and attempt to finally obliterate due to enlargement of aneurysm. Infraclinoid

www.ketabpezeshki.com 66485438-66485457
588 PART VII PEDIATRIC NEUROSURGERY

aneurysm can present with diabetes insipidus, cra- further ischemic injury by increasing collateral
nial nerve deficits, unilateral blindness, recurrent blood flow to hypoperfused areas of cortex, using
massive epistaxis, and features of carotid-cavernous the external carotid circulation as a donor supply.
fistula. Rupture carries mortality of 32-50%. Direct anastomosis procedures, most commonly
superficial temporal artery (STA) to middle cere-
10. b—IV rehydration and systemic heparin bral artery (MCA) bypasses, may achieve instant
therapy
improvement in focal cerebral perfusion, but
these procedures are often technically difficult
Symptoms of cerebral venous sinus thrombosis are
to perform because small pediatric patients often
diverse and nonspecific, including most commonly
the triad of headache, vomiting, and depressed do not have a large enough donor scalp artery or
recipient middle cerebral artery to allow for a
mental status. Seizures are common, especially in
anastomosis large enough to supply a significant
neonates. Clinical signs include reduced GCS,
papilledema, cranial nerve abnormalities (6th nerve amount of additional collateral blood supply. A
variety of indirect anastomotic procedures have
palsy), hemiparesis, quadriparesis, ataxia, and
been described: encephaloduroarteriosynangiosis
hyper-reflexia. In neonatal CSVT, the most com-
(EDAS) whereby the STA is dissected free over a
mon signs and symptoms are lethargy, vomiting, a
course of several inches and then sutured to the
full fontanelle, and seizures. In newborns, acute sys-
cut edges of the opened dura; encephalomyo-
temic illness with dehydration and infection are the
synangiosis (EMS) in which the temporalis mus-
most common predisposing conditions. Among
cle is dissected and placed onto the surface of
previously healthy children, acute head and neck
the brain to encourage collateral vessel deve-
infections, dehydration, and iron deficiency anemia
lopment; and the combination of both, enceph-
are common. Chronic diseases associated with
childhood CSVT include inflammatory bowel dis- alomyoarteriosynangiosis (EMAS). Moyamoya
patients are at particular risk of ischemic events
ease, cancer, autoimmune disorders, and chronic
in the perioperative period. Crying and hyper-
liver or renal disease. Specific prothrombotic
ventilation, common occurrences in children at
abnormalities that have been explored in pediatric
times during hospitalization, can lower PaCO2
CSVT include factor V Leiden and prothrombin
and induce ischemia secondary to cerebral vaso-
gene mutations, protein C, S and antithrombin defi-
constriction. Any techniques to reduce pain—
ciencies, antiphospholipid antibodies, lipoprotein
including the use of perioperative sedation, pain-
(a), methyl tetrahydrofolate reductase mutations,
less wound dressing techniques, and absorbable
homocysteine, fibrinogen, plasminogen, factor
wound suture closures—helped to reduce the
VIII, and heparin cofactor II. While CT imaging
is fast and convenient, this modality may miss the incidence of strokes, TIAs, and length of stay in
a recent study. A further perioperative consider-
diagnosis of CSVT in up to 40% of cases, and is best
ation is the use of monitoring, such as intraopera-
identified using MRI with MR venogram (MRV).
Ultrasound is convenient in infants with an open tive EEG or near-infrared spectroscopy, used to
identify and ameliorate ischemic events detected
fontanel but is less sensitive than either CT or
while the patient is under general anesthesia.
MRI. Primary treatment for CVST is anticoagula-
tion with heparin. Additional therapies for CSVT
include thrombolysis or surgical thrombectomy, FURTHER READING
but are associated with intracranial hemorrhage Smith ER, Scott RM. Surgical management of moyamoya
hence usually used when there is a failure to respond syndrome. Skull Base. 2005 Feb;15(1):15-26.
to anticoagulation.
12. d—Stereotactic radiosurgery
FURTHER READING
Witmer C, Ichord R. Crossing the blood-brain barrier: clini-
cal interactions between neurologists and hematologists in
pediatrics—advances in childhood arterial ischemic stroke EMI ANSWERS
and cerebral venous thrombosis. Curr Opin Pediatr 2010
Feb;22(1):20-7. 13. 1—g, Osler-Weber-Rendu syndrome
(HHT), 2—f, Marfan's syndrome, 3—j,
11. d—Superficial temporal artery to MCA Sturge-Weber syndrome
In moyamoya disease, surgery is generally recom-
mended for the treatment of patients with recur- FURTHER READING
rent or progressive cerebral ischemic events and Xu HW, Yu SQ, Mei CL, Li MH. Screening for intracranial
associated reduced cerebral perfusion reserve. aneurysm in 355 patients with autosomal-dominant polycystic
kidney disease. Stroke 2011 Jan; 42(1):204-6.
Many different operative techniques have been
described, all with the main goal of preventing

www.ketabpezeshki.com 66485438-66485457
CHAPTER 45

PEDIATRIC MOVEMENT
DISORDERS AND SPASTICITY
SINGLE BEST ANSWER (SBA) QUESTIONS
1. Spasticity is best described as: 4. Which one of the following is LEAST likely to
a. Velocity-dependent resistance to passive be a therapeutic end goal for intrathecal
muscle stretch usually due to upper motor baclofen?
neuron lesion a. Absence of limb spasticity
b. Velocity-dependent resistance to active b. Facilitating care
muscle stretch usually due to upper motor c. Increasing range of motion
neuron lesion d. Reducing painful muscle spasms
c. Velocity-dependent resistance to passive e. Slowing the development of muscle
muscle stretch usually due to lower motor contractures
neuron lesion
d. Force-dependent resistance to passive 5. Intrathecal baclofen treatment in children
muscle stretch usually due to upper motor with cerebral palsy is most commonly associ-
neuron lesion ated with?
e. Force-dependent resistance to active a. Increased generalized itching
muscle stretch usually due to lower motor b. Increased headache
neuron lesion c. Increased progression of scoliosis
f. Force-dependent resistance to passive d. Increased seizure frequency
muscle stretch usually due to lower motor e. Reduced physostigmine requirements
neuron lesion 6. A 27-year-old patient with spastic diplegia
presents 2 years after baclofen pump inser-
2. The most common cause of spasticity in chil- tion with increasing baclofen requirements.
dren is likely to be: Until 6 months ago he had been stable for
a. Cerebral palsy 1 year with a requirement of 300 μg/day.
b. Multiple sclerosis On the X-ray image of his baclofen pump
c. Stroke below, which port would allow you to deter-
d. Traumatic spinal injury mine catheter patency?
e. Wilson's disease

3. You see a 4-year-old child in clinic with a


right spastic hemiparesis from a previously
ruptured left basal ganglia cavernoma. She
is currently taking oral baclofen and gaba-
pentin. On examination, the right upper
limb has increased tone throughout the
range of movement but is easy to move,
while the right leg has a considerable
increased in muscle tone with passive move-
ment being difficult. What are the Modified
Ashworth scale grades for the right arm
and leg?
a. Right arm ¼ 1, Right leg ¼ 2
b. Right arm ¼ 1 +, Right leg ¼ 2
c. Right arm ¼ 1 +, Right leg ¼ 3
d. Right arm ¼ 2, Right leg ¼ 3
e. Right arm ¼ 3, Right leg ¼ 4
589

www.ketabpezeshki.com 66485438-66485457
590 PART VII PEDIATRIC NEUROSURGERY

a. Blue arrow d. Pallidotomy


b. Purple arrow e. Selective dorsal rhizotomy
c. Red circle
d. Yellow arrow 11. A child with cerebral palsy and a long-term
e. Access port is usually in-line with catheter intrathecal infusion pump presents with
in lumbar region fever, seizures, and rebound spasticity. This
is most likely to be:
7. The fluoroscopic image below most likely
a. Acute dystonic reaction
shows:
b. Baclofen withdrawal
c. Malignant hyperthermia
d. Neuroleptic malignant syndrome
e. Status dystonicus

EXTENDED MATCHING ITEM (EMI)


QUESTIONS
12. Drug treatment of spasticity:
a. Carbidopa-levodopa
b. Cyproheptadine
c. Dantrolene
d. Diazepam
e. Gabapentin
f. Intrathecal baclofen
g. Oral baclofen
h. Physostigmine
a. Catheter leak i. Pregabalin
b. Curling of catheter in subcutaneous tissues j. Tetrabenazine
c. Disconnection at pump catheter k. Tizanidine
d. Extradural position of catheter l. Trihexyphenidyl
e. Kinking of catheter
For each of the following descriptions, select the
8. Selective dorsal rhizotomy for spastic cere- most appropriate answers from the list above.
bral palsy works by: Each answer may be used once, more than once
a. Electrical stimulation of dorsal root entry or not at all.
zone 1. Half-life is 4-5 h, therefore 24 h is required
b. Interruption of a subset of alpha-motor to achieve steady-state concentration after a
neurons change in dosage
c. Interruption of gamma-motor neurons 2. Reduces intracellular calcium by binding
d. Interruption of the spinal reflex arc ryanodine receptor in skeletal muscle
e. Restoration of GABAergic inhibition of 3. Alpha2 adrenergic antagonist
alpha-motor neurons
13. Dystonia:
9. Which one of the following treatments for a. Dopa-responsive dystonia (Segawa's
pediatric dystonia works via vesicular mono- disease)
amine transporter inhibition? b. Dystonic cerebral palsy
a. Baclofen c. DYT-1 dystonia
b. Carbamazepine d. Gangliosidoses
c. Carbidopa-levodopa e. Glutaric aciduria
d. Clonazepam f. Leigh's disease
e. Tetrabenazine g. Mitochondrial disorders
f. Trihexiphenidyl h. Pantothenate kinase-associated
neurodegeneration
10. The surgical treatment of choice for primary i. Post-head injury dystonia
dystonia is likely to be: j. Post-stroke dystonia
a. Bilateral GPi DBS k. Rett's syndrome
b. Epidural motor cortex stimulation l. Wilson's disease
c. Intrathecal baclofen pump

www.ketabpezeshki.com 66485438-66485457
45 PEDIATRIC MOVEMENT DISORDERS AND SPASTICITY 591

For each of the following descriptions, select the e. Globus pallidus externus
most appropriate answers from the list above. f. Nucleus accumbens
Each answer may be used once, more than once g. Pedunculopontine nucleus
or not at all. h. Posterior hypothalamic
1. Dystonia worse in late afternoon, parkin- i. Subgenual cortex
sonism, and spastic gait but no history con- j. Subthalamic nucleus
sistent with cerebral palsy, low biopterin k. Ventral PL thalamus
and homovanillic acid levels
2. Childhood onset dystonia associated with For each of the following descriptions, select the
TorsinA gene mutation most appropriate answers from the list above.
Each answer may be used once, more than once
14. Neuromodulation: or not at all.
a. Anterior limb of internal capsule 1. Primary generalized dystonia
b. Anterior thalamic nucleus 2. Epilepsy
c. Cingulate cortex
d. Globus pallidus internus

SBA ANSWERS
1. a—Velocity-dependent resistance to passive
Ashworth Modified Ashworth
muscle stretch usually due to upper motor
neuron lesion. Symptoms include muscle 0 ¼ Normal muscle tone 0 ¼ Normal muscle tone
tightness, cramping/pain, and fatigue. It is 1 ¼ Slight increase/catch 1 ¼ Slight increase/catch
due to a loss of descending GABA inhibition when limb moved and release/minimal
of muscle groups resulting in hypertonia (co- passively resistance at end of ROM
activation of agonist and antagonist muscles 2 ¼ More marked 1+ ¼ Slight increase/
during volitional movement). increase but limb easily catch and minimal
flexed resistance <50% of ROM
2. a—Cerebral palsy. Occurs in 1.5-3 per 1000 3 ¼ Considerable 2 ¼ More marked
and accounts for 75% of spasticity in chil- increase in muscle tone increase through most of
dren. This is defined as a range of non- ROM but affected parts
progressive syndromes of posture and motor easily moved
impairment due to an insult to the developing 4 ¼ Limb in rigid flexion 3 ¼ Considerable
nervous system. It is often accompanied by or extension increase, passive
disturbances in sensation (visual and tactile), movement difficult
cognition/behavior, communication, and 4 ¼ Affected parts in rigid
flexion or extension
epilepsy (30%). CP is classified into spastic,
dyskinetic, or mixed. Spastic CP is the most
common and divided into spastic quadraple-
gia, diplegia, hemiplegia, or monoplegia.
Severity of spasticity is graded according to
Ashworth or Modified Ashworth scales. Dys- 4. a—Absence of limb spasticity. This is
kinetic cerebral palsy can be divided into dys- because spasticity can help people with stand-
tonic and choreoathetoid forms. ing, walking or transferring and complete
removal of spasticity can result in loss of
3. d—Right arm ¼ 2, Right leg ¼ 3. Generally function in some patients and thus intrathecal
patients with an Ashworth score of 3 are baclofen pump insertion is inappropriate.
candidates for intrathecal baclofen, although
those with less severe spasticity may also 5. e—Headache. CSF leaks occur in 5-15% of
benefit depending on the clinical context. children with cerebral palsy, compared to
Additionally, during ITB test dose (25- 3% in adults. This is thought to be due to
50 μg in children) for spasticity an improve- smaller size, thinner tissues, malnutrition of
ment of one point or greater is considered chronically disabled children and occult
positive. hydrocephalus in cerebral palsy.

www.ketabpezeshki.com 66485438-66485457
592 PART VII PEDIATRIC NEUROSURGERY

6. a—Blue arrow. The general parts in the most


commonly used pump (Medtronic Syn-
chromed) consist of pump roller (red ring),
pump reservoir port for filling (yellow arrow),
catheter access port (blue arrow) and pump-
catheter connector (purple arrow). In general,
catheter malfunction can be assessed with
plain radiographs (disconnection, kinking,
migration) and either fluoroscopic or CT
imaging after injection of contrast material A B
via the catheter access port (catheter leak or
extrathecal catheter-see image below).
Image with permission from Miracle AC, Fox MA,
Ayyangar RN, et al. Imaging evaluation of intrathecal
baclofen pump-catheter systems, AJNR Am J Neurora-
diol Aug;32(7):1158-64, 2011.

C D

E F

Image with permission from Winn HR. Youman's Neu-


rological Surgery, 4-Volume Set, 6th ed., Elsevier,
Image with permission from Miracle AC, Fox MA, Saunders, 2011.
Ayyangar RN, et al. Imaging evaluation of intrathecal
baclofen pump-catheter systems, AJNR Am J Neurora-
diol Aug;32(7):1158-64, 2011.
9. e—Tetrabenazine
7. a—Catheter leak
10. a—Bilateral GPi DBS
Image with permission from Winn HR. Youman's
Neurological Surgery, 4-Volume Set, 6th ed., Elsevier, 11. b—Baclofen withdrawal
Saunders, 2011.

8. d—Interruption of the spinal reflex arc. Defi- EMI ANSWERS


cient descending inhibition of spinal reflexes
leads to hyperreflexia, hypertonia hence 12. 1—f, Intrathecal baclofen, 2—c, Dantrolene,
selective interruption of afferent fibers from 3—k, Tizanidine.
spastic muscles will stop this. Briefly, in
SDR (see diagram below) the conus is 13. 1—a, Dopa-responsive dystonia, 2—c, DYT-
exposed and L2-S2 dorsal roots separated 1 dystonia.
from motor roots using a silastic sheet, Each
dorsal root is divided into 3-5 fascicles using a 14. 1—d, Globus pallidus internus, 2—b, Ante-
Scheer needle, and each of these is examined rior thalamic nucleus.
with EMG. Only rootlets which display a sig-
nificant EMG response after stimulation are
sectioned, while others are spared. L1 rhizot-
omy is required to reduce spasticity in hip
flexors.

www.ketabpezeshki.com 66485438-66485457
CHAPTER 46

NEUROSURGERY AND PREGNANCY


SINGLE BEST ANSWER (SBA) QUESTIONS
1. Which one of the following best describes a. Most AVMs rupture during the first and
the risk of subarachnoid hemorrhage during second trimesters
pregnancy? b. Most AVMs rupture during the puer-
a. Lowest during the first trimester perium
b. Highest during the puerperium c. Most AVMs rupture during the second
c. Highest during the second trimester trimester
d. Lowest during the third trimester d. Most AVMs rupture during the second
e. Highest during labor and third trimesters
f. Not increased during pregnancy, labor, or e. Most AVMs rupture during labor
puerperium f. Same rate of AVM rupture throughout
pregnancy and puerperium
2. A 28-year-old left-handed female who is
14 weeks pregnant presented to the emer- 5. Back pain in pregnancy is most likely due to the
gency department with worsening headache, action which one of the following hormones?
vomiting and drowsiness. CT head scan a. FSH
showed a large right frontal tumor with sig- b. Oestrodiol
nificant edema. After 48 h of dexamethasone c. Oxytocin
her headache has improved but she still has a d. Progesterone
mild left-sided arm weakness. Which one of e. Relaxin
the following would be most appropriate?
a. Permit gestational advancement to second 6. A 36-year-old female who is 20 weeks preg-
trimester nant presents with sciatica, saddle anesthesia
b. Craniotomy and tumor resection followed and urinary incontinence. MRI of the lumbo-
by chemoradiotherapy sacral spine reveals a significant L5/S1 disc
c. Cesarean section followed by neuro- prolapse with compression of cauda equina
surgery nerve roots. What would be your preferred
d. Iatrogenic termination followed by neuro- option?
surgery a. Anterior discectomy in supine position
e. Radiotherapy alone until fetal maturity with 30° lateral decubitus tilt
established b. Discectomy in lateral decubitus position
c. Discectomy with patient prone on four-
3. At what gestational age is it generally poster frame
accepted that a ruptured cerebral aneurysm d. Iatrogenic termination and proceed to
should be managed by cesarean section under discectomy in prone position
general anesthesia followed by immediate e. Laminectomy only in lateral decubitus
aneurysm exclusion? position
a. 26 weeks
b. 28 weeks 7. The additional risk of childhood cancer
c. 30 weeks above the natural risk (1 in 500) per CT
d. 32 weeks head performed on a pregnant female is
e. 34 weeks approximately:
a. Less than 1 in 1,000,000
4. According to available evidence, which one of b. 1 in 1,000,000 to 1 in 100,000
the following statements is most correct c. 1 in 100,000 to 1 in 10,000
regarding the timing of cerebral arteriovenous d. 1 in 10,000 to 1 in 1000
malformation rupture during pregnancy? e. 1 in 1000 to 1 in 200

593

www.ketabpezeshki.com 66485438-66485457
594 PART VII PEDIATRIC NEUROSURGERY

8. A 31-year-old female who is 34 weeks preg- EXTENDED MATCHING ITEM (EMI)


nant is involved in a high-speed RTA. She
is immobilized by paramedics at the scene
QUESTIONS
and transferred to your emergency depart-
ment. She is GCS 15/15 but becomes hypo- 12. Shunt malfunction in pregnancy:
tensive and tachycardic. What action would a. Cesarean section under epidural anesthesia
you take immediately? b. Cesarean section under GA
a. Place in left lateral position c. CT or MRI during pregnancy
b. Manually push the uterus to the left d. Induced hypocarbia
c. Start a vasopressor e. Magnesium sulfate
d. Intermittent fetal monitoring f. Preconception CT or MRI
e. Continuous cardiotocography g. Prophylactic antibiotics
f. Fetal blood sampling h. Revision of ventriculoperitoneal shunt
i. Shunt tap for pressure and CSF MCS
9. Which one of the following is a major poten- j. Vaginal delivery with assisted second
tial risk to the fetus exposed to MRI stage
sequences employing time-varying gradient k. Ventriculoatrial shunt or third
electromagnetic fields? ventriculostomy
a. Developmental delay
b. Acoustic noise damage For each of the following descriptions, select the
c. Magnetophosphenes most appropriate answers from the list above.
d. Peripheral nerve and muscle stimulation Each answer may be used once, more than once
e. Implant or not at all:
1. A 32-year-old with VP shunt placement
10. A 31-year-old female who is 28 weeks preg- 8 years ago for hydrocephalus following
nant presents to your emergency department foramen magnum decompression is consid-
with a self-terminating tonic-clonic seizure ering pregnancy.
and currently has postictal confusion. She is 2. A 36-year-old with VP shunt is undergoing
under close monitoring for her hypertension a planned vaginal delivery with assisted
and proteinuria. Which one of the following second stage.
would you administer first? 3. A 23-year-old with VP shunt develops
a. Lorazepam headache, nausea, vomiting and a single
b. Diazepam generalized tonic-clonic seizure which
c. Valproate self-terminates after 1 min. She does not
d. Magnesium have a pre-existing diagnosis of epilepsy
e. Phenytoin and is not on antiepileptics. Blood pressure
f. Levetiracetam is 140/90 and urine dip shows 2 + leuko-
cytes, negative nitrites, no ketones and no
11. Women with aneurysmal subarachnoid hem- protein.
orrhage 10 weeks pregnant and oculomotor
13. Neurological disease in pregnancy:
palsy. She agrees to DSA and coil embolization
a. Cerebral venous sinus thrombosis
and the aneurysm is secured. Postoperatively
b. Choriocarcinoma
she asks about the possibility of terminating
c. Chorea gravidarum
her fetus due to the probable harmful effects
d. Idiopathic intracranial hypertension
of the radiation exposure required in her treat-
e. Lymphocytic hypophysitis
ment. Which one of the following actions
f. Lymphoma
would you take in the first instance?
g. Pituitary macroadenoma
a. Organize a medical termination while she
h. Pre-eclampsia/Eclampsia
is still an inpatient
i. Reversible posterior leukoencephalopa-
b. Explain that the likely dose was much
thy syndrome
lower than the generally accepted thresh-
j. Sheehan's syndrome
old for causing fetal harm
c. Recommend amniocentesis
For each of the following descriptions, select the
d. Advise her to discuss the risks with a
most appropriate answers from the list above.
medical physicist
e. Arrange an obstetric review

www.ketabpezeshki.com 66485438-66485457
46 NEUROSURGERY AND PREGNANCY 595

Each answer may be used once, more than once polyuria. Her past medical history includes
or not at all: with systemic lupus erythematosus. Initial
1. A 32-year-old female with a previous history investigations reveal sodium 155 and a diag-
of molar pregnancy 6 months ago presents nosis of diabetes is made after further
with a generalized tonic-clonic seizure. workup. MRI shows a homogeneously
CT head shows a right frontal hemorrhagic enhancing sellar mass with thickening of
space occupying lesion and CT chest shows the pituitary stalk producing a “pear-
multiple pulmonary lesions. shaped” appearance.
2. A 27-year-old female who is 3 months post-
partum presents with headache, fatigue and

SBA ANSWERS
1. f—Not increased during pregnancy, labor, or and age-appropriate neurocognitive (IQ, atten-
puerperium. tion, behavior, memory) development. Estima-
tions of the absorbed fetal dose were between
FURTHER READING 0.01 and 0.1 Gy (10-100 mGy) for patients who
Algra AM, et al. Female risk factors for subarachnoid haemor- received whole brain RT by a 3D conformal tech-
rhage: a systematic review. Neurology 2012;79(12):1230-6. nique and many of the toxic effects will only be
induced above the deterministic threshold of
2. b—Craniotomy and tumor resection 0.1 Gy. Most studies reporting on the administra-
tion of radiotherapy to brain tumors showed that
Treatment should adhere to the treatment options the fetal exposure never exceeded this threshold
as in nonpregnant women. The optimal time to dose. These radiotherapy schedules are therefore
perform the procedure during pregnancy is still considered safe. Still, proper shielding should
a matter of debate. It is recommended to delay sur- always be used to further reduce the fetal dose
gery if possible until after the first trimester to and it is recommended to discuss treatment with
reduce the miscarriage risk—surgery during the a radiation physicist and to use a phantom to esti-
second and third trimesters surgery is considered mate the fetal dose as accurate as possible in order
safe. Delay can cause progressive neurologic dete- to counsel parents on the potential risks of
rioration and increasing risk of urgent interven- radiation-induced toxicity.
tion (resection and cesarean section). Due to the
significant complications of prematurity (e.g. FURTHER READING
respiratory diseases, bradycardia, necrotizing Tewari KS, Cappuccini F, Asrat T, et al. Obstetric emergen-
enterocolitis, intraventricular hemorrhage, hypo- cies precipitated by malignant brain tumors. Am J Obstet
glycemia and feeding problems, sepsis and Gynecol 2000;182:1215-21; Verheecke M, Halaska MJ, Lok
seizures) iatrogenic preterm birth should be CA, et al. Primary brain tumours, meningiomas and brain
avoided whenever possible by postponing or con- metastases in pregnancy: report on 27 cases and review of lit-
tinuing treatment until a term delivery can be erature. Eur J Cancer 2014;50(8):1462-71; Amant F, Van Cal-
steren K, Halaska MJ, et al. Long-term cognitive and cardiac
achieved. The decision of performing an elective
outcomes after prenatal exposure to chemotherapy in children
cesarean section preterm is often based upon the aged 18 months or older: An observational study. Lancet
risk of increased intracranial pressure associated Oncol 2012;13(3):256-64.
with bearing-down efforts during the second stage
of labor. Nonetheless, if patients are clinically sta- 3. e—34 weeks
ble and carefully discussed, and the individual risk
of rapid tumor growth has been evaluated, gesta- For ruptured cerebral aneurysms in pregnant
tional advancement until fetal maturity should women generally the aneurysm should be treated
be considered, as well as the attempt to have a vag- first and the pregnancy allowed to continue to
inal delivery. Balance between waiting for fetal term, except in cases of rupture during labor
maturation and risk of intrauterine death (second- when delivery should be completed prior to aneu-
ary to maternal death) remains difficult in patients rysm treatment. For gestational ages less than
with highly malignant tumors. 26 weeks, proceed as best for the mother and if
A recent study summarized long-term data of aneurysm treatment is successful vaginal delivery
children after antenatal exposure to chemother- should be attempted. For gestational ages beyond
apy (and/or radiotherapy) found a cardiac out- 34 weeks, cesarean section under general anesthe-
come equal to the general population, and no sia, followed immediately by aneurysm exclusion,
adverse effects of treatment on the general health is advised. Between 26 and 34 weeks, aneurysm

www.ketabpezeshki.com 66485438-66485457
596 PART VII PEDIATRIC NEUROSURGERY

exclusion should proceed and, if the fetus is stable, 6. c—Discectomy with patient prone on four-
pregnancy allowed to continue to term. Deciding poster frame
whether to undertake endovascular coiling or sur-
gical clipping is difficult. In view of the progres- Pregnant women who have progressive neuro-
sive hormonal and hemodynamic changes in logical deficit at 34-36 weeks' gestation or later
pregnancy, ISAT data may not be applicable; should undergo induction of delivery or cesarean
additionally complications such as coil prolapse section before, or at the same time as, they un-
require antiplatelet agents that need to be con- dergo spinal surgery; pre-partum surgical treat-
sidered in unexpected labor or emergency cesar- ment should be considered in patients who
ean section soon after coiling. Such issues develop progressive neurological deficits before
certainly require detailed discussion between 34 weeks. The decision regarding timing of spinal
the neurosurgeon, neuroanesthetist, obstetrician surgery should be made in close consultation with
and patient. the obstetrician, as uncertain dating of gestational
age could greatly affect the infant's outcome. In
FURTHER READING addition, inducing labor before the neurological
Ng J, Kitchen N. Neurosurgery and pregnancy. JNNP injury is treated could cause increased neurolog-
2008;79:745-52. ical injury in the patient because of the rise in epi-
dural venous pressure that occurs during labor. In
4. a—Most occur during the second and third cases of true cauda equina syndrome or severe
trimesters motor weakness occurring at later gestational
ages (34 weeks), cesarean delivery should be
It is accepted that the overall rate of hemorrhage strongly considered over induction of labor to
from cerebral AVMs is not increased during avoid more severe neurological deficits after
pregnancy compared to nonpregnant periods of delivery. Brookfield et al. used the prone position
life. However, in pregnant patients with intracra- in pregnant patients with lumbar disc herniation
nial AVMs it is important to know that most rup- after 20- and 32-weeks' gestation by use of a four-
tures occur in the second and third trimester, and poster laminectomy frame to provide pressure
not during the first trimester, labor or puerpe- relief over the abdomen; it is unnecessary to use
rium. The definitive management of AVMs in the technically difficult lateral decubitus position.
pregnancy thus follows standard neurosurgical Diagnostic imaging in women of child-bearing
guidelines. In general, those with fully treated age is limited to MRI scanning as the initial,
AVMs before 35 weeks gestation unassisted vag- and when possible the only, confirmatory and
inal delivery should be possible. In those with surgical planning diagnostic procedure. Intrao-
unruptured intracranial AVM, the risk of hemor- perative fluoroscopy is unlikely to deliver terato-
rhage during vaginal delivery is recognized to be genic fetal radiation doses but if any question
low with the use of epidural analgesia and an about termination of pregnancy arises input from
assisted second stage. In contrast, elective cesar- both an obstetrician and a medical physicist who
ean section has been advocated for women with can accurately calculate the exact dose of radia-
an untreated or partially treated AVM, especially tion to which the fetus was exposed. In the preg-
if it has bled during pregnancy. nant patient, only the surgical procedure that is
necessary to alleviate neurological deficit should
FURTHER READING be performed.
Ng J, Kitchen N. Neurosurgery and pregnancy. JNNP
2008;79:745-52; Liu XJ, Wang S, Zhao YL, et al. Risk of cere- FURTHER READING
bral arteriovenous malformation rupture during pregnancy
Brookfield KF, Brown MD. How should pregnant women
and puerperium. Neurology 2014;82(20):1798-803. with spinal disease be managed? Nat Clin Pract Neurol
2008 Dec;4(12):652-3.
5. e—Relaxin
7. a—Less than 1 in 1,000,000
This is a hormone released during pregnancy to
cause ligamentous laxity in preparation for partu- Fetal radiation doses of less than 50 mGy are
rition. Women with severe pelvic girdle pain in not associated with increased fetal anomalies or
pregnancy have significantly higher serum levels fetal loss throughout pregnancy; fortunately,
of relaxin than those who are pain free. radiation doses of all diagnostic imaging examina-
tions using ionizing radiation routinely used in a
FURTHER READING trauma evaluation should be well below this
MacLennan AL, Nicholson R, Green RC, et al. Serum relaxin
threshold (by comparison fetal dose from natural
and pelvic pain of pregnancy. Lancet 1986;2:243-5.

www.ketabpezeshki.com 66485438-66485457
46 NEUROSURGERY AND PREGNANCY 597

background radiation during pregnancy is FURTHER READING


0.5-1.0 mGy): Health Protection Agency. RCE-9: protection of pregnant
patients during diagnostic medical exposures to ionizing radi-
ation: advice from the Health Protection Agency, The Royal
College of Radiologists and the College of Radiographers.
Typical March 2009; Raptis CA, Mellnick VM, Raptis DA, et al. Imag-
Fetal Additional Risk of ing of trauma in the pregnant patient. Radiographics 2014;34
Maternal Dose Childhood Cancer (3):748-63.
Examination (mGy) per Examination
XR Skull, CT head 0.001- <1 in 1,000,000 8. a—Left lateral position
0.01
CT Pulmonary 0.01-0.1 1 in 1,000,000 to 1 Trauma, which affects 5-7% of all pregnancies, is
angiogram in 100,000 the leading cause of nonobstetric maternal mortal-
Cerebral ity. Fetal loss rates approach 1-5% in minor injuries
angiography and 40-50% in life-threatening trauma, but as
XR Abdomen, XR 0.1-1.0 1 in 100,000 to 1 in minor trauma is much more common it is the major
Hip, XR pelvis, CT 10,000 cause of fetal loss. Stabilization of the mother
chest involves resuscitation used with any trauma patient,
XR lumbar spine, 1.0-10 1 in 10,000 to 1 in bearing in mind that if she is more than 20 weeks
CT lumbar spine, 1000 pregnant, she should be placed in the 30° left lateral
CT abdomen decubitus position to prevent systemic hypotension
CT pelvis, CT 10-50 1 in 1000 to 1 in 200 caused by compression of the inferior vena cava by
pelvis-abdomen the gravid uterus. For imaging studies that require
the patient to lie flat for an extended time, use of the
30% left lateral decubitus position during imaging
should be strongly considered. In addition, blood
Body CT examinations of pregnant trauma products should be administered to maintain a
patients should be performed with intravenous hematocrit level higher than 30% for optimal fetal
iodinated contrast as it improves detection of both oxygenation. After the patient has been stabilized,
maternal and fetal injuries by providing vascular ultrasound should be performed to determine the
contrast in organs and opacification of vascular gestational age of the fetus and whether a fetal heart
structures, including the placenta. The use of iodin- rate is present. For a fetus older than 24-26 weeks of
ated contrast material to obtain one diagnostic CT gestational age, continuous external fetal monitor-
study is preferable to obtaining a nonenhanced CT ing (cardiotocography; CTG) should be used as it
study that may be nondiagnostic and necessitate would be viable outside the uterus and should be
repeat imaging. In a seriously injured pregnant delivered if there is evidence of fetal distress.
patient, multiple or repeat imaging examinations
could result in a fetal radiation dose that exceeds FURTHER READING
50 mGy. In these situations, it is important to Raptis CA, Mellnick VM, Raptis DA, Kitchin D, et al. Imaging
recognize the risks of ionizing radiation to the fetus, of trauma in the pregnant patient. Radiographics 2014;34
which depend on the stage of the pregnancy: (3):748-63.

9. b—Acoustic noise damage


Fetal
Radiation
Gestation Dose (mGy) Risks MRI Field Type Risks
<2 weeks 50-100 Failure of blastocyst Static field Vertigo, nausea,
implantation (if magnetophosphenes, metallic
survives, no other taste, projectiles, implant
deleterious effect malfunction and movement,
expected) monitoring device malfunction
and movement
2-20 weeks 50-150 Teratogenesis
Radiofrequency Heating effect (specific energy
Any time 50 Carcinogenesis: pulse absorption rate, SAR), Induced
doubles risk of fatal current burns
childhood cancer (from
1 in 500 to 1 in 250) Time-varying Acoustic noise damage,
Increases overall gradient EMF peripheral nerve stimulation,
lifetime risk of cancer muscle stimulation (arrhythmia
by 2% in extreme cases)

www.ketabpezeshki.com 66485438-66485457
598 PART VII PEDIATRIC NEUROSURGERY

To minimize these potential risks, it is recom- Aneurysms are diagnosed by digital subtraction
mended that MR imaging of pregnant patients is angiography (DSA) or, increasingly, by CT angi-
performed at field strengths of 1.5 T or less. In ography. Furthermore, coil embolization
addition, MR imaging protocols for pregnant requires prolonged use of DSA. Concerns exist
patients should be tailored to include the mini- regarding fetal radiation exposure. A phantom
mum number of sequences required to answer study has demonstrated that the effective radia-
the particular clinical question. Gadolinium is tion dose to the fetus during DSA for coil embo-
considered a pregnancy category C drug by the lization is so small that it confers no additional
FDA, which means that animal studies have risk to the fetus. If there is still concern then a
shown adverse effects but adequate data are not medical physicist should be consulted.
available in humans, and the potential benefits
may warrant its use in pregnant women if it is FURTHER READING
considered critical for evaluation. Typically, the Marshman LA, Rai MS, Aspoas AR. Comment to “Endovascu-
use of gadolinium-based contrast material is not lar treatment of ruptured intracranial aneurysms during
necessary in pregnant trauma patients because pregnancy: report of three cases”. Arch Gynecol Obstet
2005;272:93.
essential clinical information can be obtained
with nonenhanced MR imaging. Gadolinium-
based contrast material can be used for imaging
pregnant trauma patients in rare circumstances EMI ANSWERS
when it is believed to be absolutely necessary
for diagnosis. 12. 1—f, Preconception CT or MRI, 2—g,
Prophylactic antibiotics, 3—i, Shunt tap for
FURTHER READING pressure and CSF MCS
Raptis CA, Mellnick VM, Raptis DA, et al. Imaging of trauma
in the pregnant patient. Radiographics 2014;34(3):748-63. Pregnancy is associated with a higher incidence of
shunt complications, and more women with
10. d—Magnesium shunts are surviving into child-bearing age. Other
causes of raised intracranial pressure (e.g. CVST)
Empirical evidence supports the effectiveness of should also be excluded in the workup. General
magnesium sulfate in preventing and treating management is outlined below.
eclamptic seizures. Therapeutic levels of magne- Preconception management
sium can be obtained by administering a 6-g 1. Baseline CT/MRI should be done in those
intramuscular loading dose followed by 2 g/h considering pregnancy with a shunt in situ.
intravenous infusion, or alternatively with a
2. Review potentially teratogenic medications
2- to 4-g intravenous bolus followed by a (e.g. anticonvulsants).
1 g/min infusion, or a combination of both.
3. If shunt was inserted for a neural tube
The goal serum concentration is considered to
defect, there is a 2-3% chance that the baby
be 4-8 mg/dL (2.0-3.5 mol/L). Magnesium is
will also have a neural tube defect. There-
excreted in the urine; thus, impaired renal func-
fore, relevant patients require genetic
tion may affect serum levels. Magnesium therapy
has a narrow therapeutic index and symptoms of counseling and judicious measures taken
to limit the risk factors for neural tube
toxicity include loss of deep tendon reflexes at
defects (e.g. folate supplementation)
blood levels of 8-12 mg/dL, respiratory depres-
Management during pregnancy
sion at concentrations of >14 mg/dL, muscular
paralysis and respiratory arrest at levels >15- 1. Raised ICP can mimic pre-eclampsia/
17 mg/dL. Cardiac arrest can occur above eclampsia and a low index of suspicion is
30 mg/dL. Recommended treatment for toxicity required.
includes calcium gluconate. 2. If increasing intracranial pressure is sus-
pected, a CT or MRI brain should be per-
FURTHER READING formed and compared with the baseline. If
The Eclampsia Trial Collaborative Group. Which anticonvul- there is no change from preoperative imag-
sant for women with eclampsia? Evidence from the Collabora- ing, the shunt should be tapped, the ICP
tive Eclampsia Trial. Lancet 1995;345(8963):1455-63. measured and cerebrospinal fluid samples
taken for culture.
11. b—Explain that the likely dose was much 3. If intracranial pressure is normal, and cul-
lower than the generally accepted threshold tures are negative, physiological changes
for causing fetal harm. may be responsible. Treatment is bed rest

www.ketabpezeshki.com 66485438-66485457
46 NEUROSURGERY AND PREGNANCY 599

and the shunt may be pumped to aid cere- FURTHER READING


brospinal fluid flow. Ng J, Kitchen N. Neurosurgery and pregnancy. JNNP
4. If there is an increase in ventricle size or if 2008;79:745-52; Wisoff JH, Kratzert KJ, Handwerker SM,
intracranial pressure is raised on shunt tap, et al. Pregnancy in patient with cerebrospinal fluid shunts:
report of a series and review of the literature. Neurosurgery
shunt revision is required. In the first and 1991;29:827-31.
second trimesters, this may be performed
as in the nonpregnant. In the third trimes-
13. 1—b, Choriocarcinoma, 2—e, Lymphocytic
ter, a ventriculoatrial shunt or third ventri-
hypophysitis
culostomy may be considered as an
alternative, thereby avoiding the risks of
Choriocarcinoma is a rare tumor of trophoblastic
uterine trauma or induction of labor.
origin; 90% have lung metastasis at presentation
Intrapartum management and cerebral metastases are a common mani-
1. Prophylactic antibiotics are recommended festation. Approximately 15% of tumors follow
during labor and delivery. Antibiotics normal pregnancies, but most are discovered
should cover coliforms and be in accordance months after pregnancies characterized by sponta-
with local guidelines. Colonization with neous abortion or by vaginal bleeding, premature
group B streptococcus has been associated labor, and an enlarged uterus due to a molar preg-
with postpartum shunt infection following nancy. Women with cerebral metastases present
cesarean section and extended antibiotic with seizures, hemorrhage, infarction, or gradu-
regimens should be considered in such ally progressive deficits. The tumor may invade
cases. the sacral plexus, cauda equina, or spinal canal.
2. If the patient has no symptoms of raised A ratio of serum: CSF hCG of >1:60 suggests
intracranial pressure, vaginal delivery is safe the presence of choriocarcinoma brain metastasis.
and the preferred option, as there is a lower A pituitary mass presenting in late pregnancy
risk of adhesions, which may subsequently or up to 1 year postpartum may be lymphocytic
result in shunt malfunction and shunt infec- hypophysitis and is also associated with other
tion. A shortened second stage is suggested, autoimmune disease. Presentation is with head-
as increases in cerebrospinal fluid pressure ache, panhypopituitarism (as inflammation dam-
at this time is greater than during any other ages anterior pituitary, posterior pituitary and
Valsalva maneuver and may lead to func- pituitary stalk equally unlike in adenomas) and
tional shunt obstruction. pressure effects on the chiasm/cavernous sinus.
3. If a patient becomes symptomatic during MRI shows a homogeneously enlarged pituitary
labor, cesarean section under general anes- gland and stalk, often with a pear-shaped appear-
thesia is indicated: epidural anesthesia is ance. First-line treatment is with steroids and
contraindicated with elevated intracranial correction of endocrine abnormalities.
pressure.

www.ketabpezeshki.com 66485438-66485457
INDEX

Note: Page numbers followed by f indicate figures, and t indicate tables.

A Anesthesia, neuroprotection during, 186, 192


Abducens nerve, 3, 3f, 14 Anesthesiologist, in anterior cervical spine surgery, 406, 413
Abetalipoproteinemia, 137, 153 Anesthetic agents, in neonatal intensive-care unit (NICU),
Acetazolamide, 258, 258f, 264 188, 198
Acetylcholine, Huntington’s disease and, 471, 475 Anesthetic drugs, 123, 126
Acid-base balance, disturbance in, 188, 199 Aneurysm, proportion of, 293, 293f, 299
Acoustic neuroma Aneurysmal subarachnoid hemorrhage
resection of, in deaf patient, 177, 181 cognitive outcome in, 236, 239
stereotactic radiosurgery for, 227, 232 fetal radiation exposure and, 594, 598
Acoustic noise damage, 594, 597, 597t return to work in, 237, 239
Action potential Angiomatous meningioma, 95, 95f, 113
generation of, 35, 43, 43–44f Ankle dorsiflexion, myotomes of, 500, 511
phenomena relevant to, 35, 44 Anterior atlanto-occipital membrane, 367, 367f, 373, 373f
Acute diffuse hypoxia/anoxia, 68, 68f, 80 Anterior choroidal artery aneurysm, 292, 292f, 298
Acute disseminating encephalomyelitis (ADEM), 131, Anterior commissure
131f, 146 in development of brain, 23, 30
Acute intermittent porphyria, 136, 152 fenestration at, 259, 259f, 266
Acute metabolic acidosis, 188, 199 Anterior communicating artery aneurysm, 290, 290f, 293,
Acute overdrainage, 549, 555 293f, 296, 299
Acute refractory bradycardia, with metabolic acidosis, treatment of, 293, 293f, 299
121, 124 Anterior inferior cerebellar artery (AICA), 3–5, 3–4f, 14–15
Acute respiratory alkalosis, 188, 199 Anterior ischemic optic neuropathy, 164, 172
Acute respiratory distress syndrome (ARDS), 187, 187f, 197 Anterior plate fixation, for cervical myelopathy, 404, 404f, 411
AD. See Alzheimer’s disease (AD) Anterior thalamic nucleus, for neuromodulation, 591–592
ADEM. See Acute disseminating encephalomyelitis (ADEM) Anti-acetylcholine receptor antibodies, in myasthenia gravis,
Adies tonic pupil, right, 164, 164f, 172 133, 133f, 148
Adolescent idiopathic scoliosis, 378, 381 Anti-aquaporin 4 antibody
management of, 378, 382 disorders in, 139, 156
Adolescents, spinal subdural empyemas in, 419, 422 for neuromyelitis
Adult and pediatric epilepsy surgery, 459–470 optica, 130, 145
Adult scoliosis, type 1, 378, 383 Antibodies, disorders in, 139, 156
Affect and personality, in neuropsychological tests, 237, 240 Anticoagulants, 122, 126
Age, in spontaneous epilepsy remission, 203, 208 Anticoagulation, 291, 291f, 297
AICA. See Anterior inferior cerebellar artery (AICA) Anticonvulsants, 122, 125
Alar ligament, 367, 367f, 373, 373f Anti-GQ1b antibody
Alcohol-related neurological disorder, 135, 150 disorders in, 139, 156
Alobar holoprosencephaly, 531, 531f, 538 Anti-muscle specific kinase antibody, disorders in, 139, 156
Alpha, in EEG, 208, 213 Anti-voltage gated calcium channel antibody, 134, 149
Alpha-fetoprotein (AFP), in bilateral papilledema, 560, disorders in, 139, 156
560f, 566 Anti-voltage gated potassium channel antibody, disorders in,
Alport syndrome, 179, 182 139, 156
ALS. See Amyotrophic lateral sclerosis (ALS) Antoni A schwannoma, 98f, 115
Alzheimer’s disease (AD), 77, 77f, 84, 127, 127f, 141 Antoni B schwannoma, 98, 98f, 116
Amenorrhea, and Cushing’s syndrome, 343, 349 Ape hand, 500, 511, 511t
American Society of Anesthesiologists (ASA) grade, Apert’s syndrome, 522, 525
186, 194 Apical ligament, 367, 367f, 373, 373f
American Spinal Injury Association Scale, clinical assessment Apparent diffusion coefficient maps, in true diffusion
of spinal cord injury using, 386, 391 restriction, 216, 222
Amiodarone, 189, 200 Aqueduct stenosis, 523, 523f, 525
Amoebic meningoencephalitis, 76, 76f, 84 Arachnoid cysts, 528, 528f, 536
Amyotrophic lateral sclerosis (ALS), 77, 77f, 84, 129, 143 Arachnoid granulations, 66, 66f, 78
Anaplastic astrocytoma, 87, 87f, 102 Arcuate fasciculus, 6, 218, 218f, 220, 223–224
Anaplastic ependymoma, 91, 91f, 108 ARDS. See Acute respiratory distress syndrome (ARDS)
Anaplastic meningioma, 96, 96f, 114 Area postrema, 34, 40, 561, 561f, 568
Anaplastic oligodendroglioma, 88, 88f, 105 Argyll Robertson pupils, 165, 165f, 172
chemotherapy for, 325, 336 Arrhythmia, 189, 200

600

www.ketabpezeshki.com 66485438-66485457
INDEX 601

Arsenic, 135, 150 Brachial plexus, 497, 497f, 505, 505f


Arterial supply, 295, 302 Brain death, in children, 514, 518, 518t
to spine, 442, 442f, 447, 447f Brain herniation, complications relating to, 272, 285
Arteriovenous malformations (AVMs), 227–228, 232, 234 Brain mapping, and Brodmann areas, 325, 335
rupture, during pregnancy, 593, 596 Brain metastases
Ascending tracts, 368 colorectal cancer and, 322, 328, 328t
Ash-leaf spot, 56, 56f, 62 CT scan for, 322, 322f, 329
Aspergillosis, 75, 75–76f, 84 Karnofsky performance score for, 323, 330
Aspergillus, causing fungal otitis externa, 356, 361 stereotactic radiosurgery for, 323, 330
Astrocytoma, 71, 71f, 82, 427, 427f, 436 stereotactic radiotherapy for, 226, 231
Asymmetric tonic neck reflex, 519t Brain stem auditory evoked response (BAER), 177, 180f
Ataxia telangiectasia, 137, 153 Brain stem compression, 66, 66f, 78
Athetosis, 473, 481 Brain Trauma Foundation guidelines, 570, 576, 576–577t
Atlanto-axial instability, 573, 583 Brain tumors
Atlanto-axial rotatory subluxation, 572, 572f, 582 in children, 557, 562, 562t
Atlanto-occipital dislocation (dissociation), 388, 388f, 397, molecular classification of, 327, 338, 338–339t
573, 583 primary, frequency of, 322, 327, 328t
Atropine, 189, 200 prognosis, 55, 60
Atypical meningiomas, 96, 96f, 114 Brainstem
Atypical teratoid/rhabdoid tumor, 72, 72f, 82, 93, 93f, 111 lesions, in biopsy of, 323, 329, 330t
Audiogram symbols, 179, 184 Breast, spinal column metastases in, 424, 429
Auditory brainstem implant, in cochlear nucleus, 177, 181 Broca’s aphasia, 141, 160
Auditory brainstem response, 178, 182 Burr hole decompression, 270, 270f, 281
Autologous cranioplasty, for linear skull fracture, 570,
570f, 576
Autonomic failure, 140, 158–159 C
Autonomic nervous system, 5, 16 C1 lateral mass screw placement, vertebral artery injury risk
via ventral roots and white rami, 1, 10 during, 366, 372
Autoregulatory vasoconstriction, 121, 123 C1 (ring-like atlas) vertebra, 364, 369, 369f
AVMs. See Arteriovenous malformations (AVMs) C1-C2 instrumented fusion, for C1 fractures, 389,
Axon hillock, 35, 44 389f, 398
Axonal degeneration, 502t C2 vertebra (axis), 364, 369, 369f
Axonal transport, 34, 41 C3-C6 laminectomy
for cervical myelopathy, 405, 405f, 411
with lateral mass screw fixation, for cervical myelopathy,
B 404, 404f, 410
Babinski reflex, 519t C3-C7 laminectomy, with instrumented fusion, for adjacent
Back pain, 408, 416 level disease, 406, 406f, 413
yellow flag for, 408, 415 C3-C7 laminoplasty, for ossification of the posterior
Baclofen pump, catheter patency of, 589, 589–590f, 592, 592f longitudinal ligament, 405, 405f, 411
Baclofen withdrawal, 590, 592 C4 (cavernous), internal carotid artery segment, 2, 2–3f, 13
BAER. See Brain stem auditory evoked response (BAER) C5 palsy, 406, 412
Ballism, 473, 481 C5 radiculopathy, 495–496t, 503
Bartonella hensae, affecting CNS, 358, 362 C5/6 ACDF, for cervical myelopathy, 404, 404f, 410
Basal ganglia, 4, 4f, 15 C5/6 and C6/7 anterior cervical decompression, for cervical
Basal turn of cochlea, 3, 3f, 14 myelopathy, 404, 404f, 411
Basal vein of Rosenthal, 5f C6 radiculopathy, 494, 494–495t, 503
Basilar fenestration, 294, 294f, 300 C6/7, pedicle/nerve root mismatch in, 403, 403f, 409
Basilar invagination, in rheumatoid arthritis, 366, 366f, 372 C7 (communicating), internal carotid artery segment, 13
Basilar tip aneurysm, 295, 295f, 300 C7 vertebra, unique anatomic feature of, 364, 370
Batson’s plexus, 365, 371 C7/C8 roots, for finger extension, 497, 504, 504t
Benign paroxysmal positional vertigo (BPPV), 179, 182 Cabergoline, 343, 349
Benign Rolandic epilepsy, 207, 207f, 212 CADASIL. See Cerebral autosomal dominant arteriopathy
Beta, in EEG, 208, 213 with subcortical infarcts and leukoencephalopathy
Bilateral burr holes, in anterior cingulotomy, 485, 487 (CADASIL)
Bilateral GPi DBS, 590, 592 Caloric testing, in right ear, 178, 181
Bilateral internuclear ophthalmoplegia (INO), 168, 168f, 174 Canadian CT head rule, and mild head injury, 269, 278
Bilateral subthalamic nucleus (STN), high-frequency Canavan disease, 138, 155
stimulation of, 472, 477 Cancer, risk of
Blake’s pouch cyst, 529, 529f, 536 in CT scans, 571, 579
Bleeding, 122, 126 pregnancy and, 593, 596, 597t
time, in coagulation assays, 188, 199 Capsulotomy, anterior, 487t
Bleeding diatheses, 188, 198 Carbamazepine, for trigeminal neuralgia, 448, 451
Blood loss management techniques, intraoperative, 187, 194 Cardiac arrhythmia, 140, 158–159
Blood volume Cardiac monitoring, 121, 124
loss, pediatric, 514, 517, 517t Cardiovascular risk factors, treating, 128, 143
for neonate, 187, 197 Carotid endarterectomy, 309, 317, 318t
B-mode ultrasound, 220, 224 “near-occlusion” and, 309, 316
BOLD functional MRI, 220, 224 Carpal tunnel syndrome, 490, 490t, 502
BPPV. See Benign paroxysmal positional vertigo (BPPV) Catheter leak, 590, 590f, 592

www.ketabpezeshki.com 66485438-66485457
602 INDEX

Caudal pons, 9, 9f, 20 CIDP. See Chronic inflammatory demyelinating


Caudal regression syndrome, 543–544t polyneuropathy (CIDP)
Cavernous angiomas, 290, 290f, 297, 441, 441f, 445 Cingulotomy, anterior, 487t
Cavernous sinus Cingulum bundle, 218, 218f, 223
location in, in developing edema, 227, 232 Circulatory shock, 187, 197
triangle of, 296, 305, 305t Cisterna magna, 7, 7f, 19
Cavernous sinus lesion, 169, 174 Citrobacter koseri, in neonatal meningitis, 549, 556
CBF. See Cerebral blood flow (CBF) CJD. See Creutzfeldt-Jakob disease (CJD)
Center late anaphase, in mitosis, 87, 87f Claw hand, 511t
Central cord syndrome, 387, 387f, 394 Clear cell ependymomas, 91, 91f, 108
Central nervous system Clear cell meningioma, 96, 96f, 114
disorders of, 24, 33 Clinical signs, 500, 511, 511t
embryological stages of, 22, 25 Closed spinal dysraphism, 532, 532f, 541, 541t
formation, 24, 31 Clotting abnormalities, reversal of, 123
infections affecting, 358, 362 Cloverleaf skull. See Pfeiffer’s syndrome
Central nervous system (CNS) cells, 38, 53 Cluster headache, 140, 159, 228, 234
Central neurocytoma, 94, 94f, 112 Coagulation assays, 188, 199
Centromedian-parafascicular nucleus of thalamus, 485–486 Cobblestone cortex, 530, 530f, 537
in Tourette’s syndrome, 472, 477 Cochlea, 228, 234
Cephalhematomas, 573, 583 Cochlear anatomy, 179, 179f, 183, 183–184f
Cephalic index (CI), 524, 526 Cochlear nerve, 3, 3f, 14
Cerebellar cognitive affective syndrome, 236, 238 Cochlear nucleus, auditory brainstem implant in, 177, 181
Cerebellar cortex, 39, 39f, 53 Cognitive decline, following cranial irradiation, 236, 239
Cerebellar development, 23, 28 Colloid cyst
Cerebellum, 39, 39f, 53 of third ventricle, 74, 74f, 83
Cerebral abscess, 216, 216f, 222 Compensated respiratory acidosis, 188, 199
Cerebral amyloid angiopathy, 306, 306f, 310 Complex partial seizures, 207, 213
Cerebral autoregulation, 185, 190, 191f Concussion
Cerebral blood flow (CBF), 185, 189–190, 191f return to play and, 268, 277
Cerebral bypass surgery second impact syndrome and, 268, 276
Moyamoya as indication for, 309, 320 Conduction velocity, demyelinating neuropathy and, 489,
STA grafts for, 309, 320, 320t 502, 502t
Cerebral cortex formation, 23, 29 Cones, of resting membrane potential, 36, 48
Cerebral cryptococcosis, 75, 75f, 84 Congenital Horner’s syndrome, 164, 164f, 172
Cerebral ischemia, 56, 61 Congenital scoliosis, 378, 383
Cerebral palsy, spasticity and, 589, 591 Congestive heart failure, vein of Galen malformations and,
Cerebral protection, during cerebrovascular surgery, 186, 193 584, 586
Cerebral veins, 5, 16 Continuous monitoring, 273, 286, 286–287t
Cerebral venous sinus thrombosis, management of, 585, 588 Contrast nephropathy, N-acetylcysteine for, 216, 221
Cerebrovascular surgery, cerebral protection during, 186, 193 Contrecoup contusions, 70, 70f, 81
Cervical epidural injections, for cervical disc and degenerative Copper beating, 522, 525
disorders, 403, 409 Cord or root damage, 388, 395
Cervical spine, in suspecting spinal injury, 386, 391 Corona radiata, 217, 217f, 222
Cervical spine trauma Coronal plane deformity, 377, 377f, 380
imaging of, 572, 581, 581t Corpus callosum, 217, 217f, 222
Cervical traction, with Gardener Wells tongs, 388, 397 Corpus callosum agenesis, 527, 527f, 534
Chemotherapy, 122, 126 Corticospinal tract, 217, 217f, 222
Chiari I malformation, 531, 531f, 540, 540t Cortico-striatal-thalamo-cortical circuit, in OCD, 485–486
management of, 548, 554 Cranial and spinal disorders, congenital, pediatric, 527–545
Chiari II malformation, 67, 67f, 79 Cranial infection, 354–363
Chiari malformation, 132, 147 Cranial irradiation, contraindication of, 558, 563
Chiasm Cranial nerve nuclei, 7–8, 19
fenestration at, 259, 259f, 266 Cranial nerves, 5, 16
Chiasmatic cistern, 7, 7f, 19 Cranial oncology, 322–339
Childhood absence epilepsy, 203, 209 Cranial trauma, 268–289
Chloride, 35, 42 Craniocervical measurements, 390, 400, 400–402f
Choline (Cho), 221, 224–225 Craniofacial pain syndromes, 450, 454
Cholinergic neurotransmission, 36, 45 Craniofacial surgery, 521–526
Chordoid meningioma, 96, 96f, 114 Craniopharyngioma
Chorea, 473, 481 adamantinomatous type, 101, 101f, 119
Choriocarcinoma, 594, 599 radiation treatment for, 227, 233
hemorrhagic tumors and, 101, 101f, 118 Craniospinal axis MRI, of myxopapillary ependymomas, 426,
Choroid plexus, 461, 461f, 468 426f, 435
Choroid plexus carcinoma, 92, 92f, 109 Craniosynostosis
Choroid plexus cells, CSF by, 34, 40 causes of, 524, 526
Choroid plexus papilloma, 92, 92f, 109, 560, 560f, 567 live births in, 522, 525
Chromosome 22, mutation of, 427, 427f, 436 Craniotomy
Chronic inflammatory demyelinating polyneuropathy during pregnancy, 593, 595
(CIDP), 136, 152 Creatine (Cr), 221, 224–225
CI. See Cephalic index (CI) Creutzfeldt-Jakob disease (CJD), 128, 128f, 140, 142, 158

www.ketabpezeshki.com 66485438-66485457
INDEX 603

Crossed extensor reflex, 519t Duchenne muscular dystrophy (DMD), 134, 134–135f, 149
Cryptococcus, causing fungal meningitis, 354, 359 Dural arteriovenous fistulas, 440, 440f, 444
CSF disorders, and general neurosurgery, 257–267 Duraplasty, for linear skull fracture, 570, 570f, 576
CSF EBV PCR test, for AIDS, 357, 362 Dynamic stabilization, primary goal of, 366, 372
CSF flow study, for foramen magnum decompression, 532, Dysembryoplastic neuroepithelial tumor (DNET), 94,
532f, 541 94f, 112
CSF pseudocyst, 259, 267 ganglioglioma and, 459, 463
CSF shunt infection, treatment of, 547, 553 Dysphonia, as complication after vagal nerve stimulator,
CSF volume, of infants, 546, 550 460, 465
CT angiogram, of Wallenberg syndrome, 387, 394 Dystonia, 471, 473, 475, 481
CT intracranial angiogram, for acute subdural hematoma, DYT-1 dystonia, 590, 592
290, 297
Cytomegalovirus ventriculitis, 76, 76f, 84
E
Early subacute subdural hematoma, characteristics of,
D 216, 221
Dandy-Walker malformation, 67, 67f, 79, 528, 528f, 535 Ecchordosis physaliphora, 74, 74f, 83
Dantrolene, 186, 194 Echinococcus, 357, 357f, 362
for spasticity, 590, 592 EC-IC Bypass Trial, 309, 318
Decompression, of syrinx, 378, 382 EEG. See Electroencephalogram (EEG)
Decompressive craniectomy, for traumatic brain injury, Electrical synapses, 35, 44
570, 577 Electrochemical equilibrium, 42
Decompressive craniotomies, 272, 285 Electrodeposition, of EEG, 208, 214, 214f
complications of, 273, 288 Electroencephalogram (EEG)
Deep brain stimulation (DBS), adverse effects of, 474, 482 electrode position of, 208, 214, 214f
Deep peroneal nerve entrapment, 509t terminology in, 208, 213
Degenerative spine, 403–417 Electrolyte disturbance, 188, 200
cervical, 403–408 Embryology, 24, 32
Delirium tremens, 135, 150 terms, 23, 30
Delta, in EEG, 208, 213 Embryonal carcinoma, 100, 100f, 118
Dementia, 139, 158 Encephalocele, 531, 531f, 539
Demyelination, 132, 132f, 147, 502t Endoscopic third ventriculostomy, 546, 551
Dentate gyrus, 461, 461f, 468 without biopsy, 560, 567
Dentate ligament, 366–367f, 367, 373, 373f Engel Epilepsy surgery, 460, 466
Dermal sinus, 543–544t Enlarging aneurysm, 585, 587
Dermatomes, 499, 510, 510f, 510t Enterovirus, 55, 60
Dermatomyositis, 133, 148 Entorhinal cortex, 461, 461f, 468
Descending tracts, 367, 374 Enzymatic conversion pathways, 36
Detethering surgery, for TCS, 532, 542 Ependymal rosettes, 90, 90f, 107
Developmental milestones, 514, 519, 520t Ependymoma, 424, 426–428, 426–427f, 435
Developmental venous anomaly, 69, 69f, 81 Ependymoma tumor, 89, 89f, 106
Dexamethasone, mechanism of, 121, 123 Epidermoid, 523, 523f, 525
Diffusion restriction, MR sequences in, 216, 222 Epidural abscess, 356, 356f, 361, 418, 418f, 420
Diffusion weighted imaging, 221, 225 Epidural hematoma, 441, 441f, 446
in true diffusion restriction, 216, 222 Epilepsy
Dilute Russell’s viper venom time, in coagulation assays, benign Rolandic, 207, 207f, 212
188, 199 childhood absence, 203, 209
Discectomy imaging in, 461, 468
with patient prone on four-poster frame, 593, 596 juvenile
for thoracic disc herniation, 407, 407f, 415 absence, 203, 209
DISH. See Diffuse idiopathic skeletal hyperostosis (DISH) myoclonic, 203, 209
Disseminated intravascular coagulation, 573, 583 medical treatment of, 462, 469
in bleeding diatheses, 188, 198 mimics of, 208, 215
Distal peritoneal catheter, migration of, 549, 555 with refractory seizures, 203, 208
Distal tibial tarsal tunnel syndrome, 509t sudden unexpected death in, 203, 208
DMD. See Duchenne muscular dystrophy (DMD) surgery workup of, 462, 468
DNET. See Dysembryoplastic neuroepithelial tumor surgical options in, 461, 468
(DNET) Ergotism, 135, 150
Dopamine, causing tardive dyskinesia, 471, 475 Evening steroid dose, for transsphenoidal surgery, 340, 346
Dopaminergic neurotransmission, 36, 46 Executive function, in neuropsychological tests, 237, 240
Dopa-responsive dystonia, 590, 592 Exertional compartment syndrome, 509t
Dorsal enteric fistula, 543–544t External ventricular drain, of brain abscess, 356, 356f, 361
Dorsal ramus, 366–367f, 367, 373, 373f Extracranial-intracranial bypass, 584, 584f, 586
Dorsal root entry zone, of trigeminal nerve in the pons, Extradural tumors, 428, 437
227, 231
Dorsal root ganglion, 366–367f, 367, 373, 373f
Down syndrome, cervical injuries in, 571, 579 F
Down’s syndrome, butterfly erythema and, 514, 517, 517t Facet joints, orientation of, 365, 370
Driving, in UK, restrictions due to neurological disorders in, Facial artery, as 4th branch of external carotid artery of neck,
322, 327 1, 10

www.ketabpezeshki.com 66485438-66485457
604 INDEX

Facial colliculus, 561, 561f, 568 Glioblastoma multiforme (GBM), 87, 87f, 103, 103f
Facial nerve, 4, 4f, 15 median survival advantage in, 324, 332
internal auditory canal and, 2, 2f, 13 two-year survival in, 324, 332
large cerebellopontine angle tumor in, 177, 181 Glioma
in Lyme disease, 354, 359 high-grade, treatment, of and radiation necrosis, 324, 332
Factor Xa activity, 121, 123 improving length of survival in, 324, 332
Fajersztajn sign, 511t low-grade
FDG-PET CT, 220, 224 craniotomy of, 325, 335
Femoral nerve entrapment, 509t de novo, 18F-FDG for, 324, 334
Femoral stretch test, 511t MR perfusion imaging and, 324, 333, 333–334t
Fibromuscular dysplasia, 290, 290f, 296 Gliomatosis cerebri, 71, 71f, 82, 88, 88f, 105
Fibrous (fibroblastic) meningioma, 95, 95f, 113 Globus pallidus internus (GPi)
Fibular tunnel syndrome, 509t for neuromodulation, 591–592
FIESTA, 220, 224 for treatment of dystonia, 472, 477
Fimbria, 461, 461f, 468 “Glomeruloid tuft”, 87, 87f, 103, 103–104f
Flat bed rest, for spontaneous intracranial hypotension, Glutamatergic neurotransmission, 36, 45
130, 130f, 145 Gorlin syndrome, 58, 58f, 62
Flow-diverting stents, for intracranial aneurysms, 309, Greater superficial petrosal nerve, 4, 4f, 15
320–321
Fluid bolus, ATLS protocol for, 513, 517
Fluid requirement, for children, 513, 517 H
Focal cortical dysplasia, 529, 529f, 537 HAART. See Highly active antiretroviral therapy (HAART)
Foix-Alajouanine syndrome, 440, 443 Hair cells, gentamicin in, 177, 181
Fornix, 217, 217f, 222 Hand of benediction, 500, 511, 511t
in Alzheimer’s disease, 485–486 HCG. See Human chorionic gonadotropin (HCG)
fenestration at, 259, 259f, 266 Head and spinal injury, pediatric, 569–583
Fourth ventricular floor, 8, 8f, 19 Headache, 140–141, 159
Fourth ventricular subependymoma, 72, 72f, 82 Hearing loss, 178, 182
4th ventricle, entrapped, and foramen magnum Heart rate, in children, 514, 517, 517t
decompression, 258, 264 Heel, in primitive reflexes, 519t
Fragile X syndrome, 136, 151 Hemangioblastoma, 73, 73f, 83, 101, 101f, 118, 557, 561,
Freshwater swimming, primary amoebic meningoencephalitis 563, 568
in, 354, 359 Hemangiopericytomas, 94, 94f, 112
Friedreich’s ataxia, 78, 78f, 85, 137, 153 Hemiballism, damage to subthalamic nucleus and, 471, 475
Froment’s sign, 511t Hemimyelocele, 543–544t
Frontal lobe executive function, in shunt insertion, Hemimyelomeningocele, 543–544t
236, 238 Hemophilia, in bleeding diatheses, 188, 198
Frontal pole, seizures in, 206, 206f, 212 Hereditary ataxia, 137, 153
Fronto-parietal operculum, seizures in, 206, 211 Hereditary sensory and motor neuropathy (HSMN), 136, 152
Hering-Breuer inflation and deflation, 1, 11
Herpes simplex virus (HSV), viral encephalitis and, 358, 362
G HHT. See Osler-Weber-Rendu syndrome (HHT)
GABA-B receptor, 35, 42 High frequency hearing loss, 177, 181
GABAergic neurotransmission, 36, 47 Highly active antiretroviral therapy (HAART), for progressive
GABA(A) receptor activation, 121, 123 multifocal leukoencephalopathy, 357, 357f, 362
Gait ataxia, management of, 559, 559f, 565 Hip flexion, myotomes of, 500, 511
Galant reflex, 519t Histology, neuropathology III, 86–120
Gamma Knife surgery, 226, 228–229, 234 HIV patients, lesions in, 129, 129f, 144
Gamma motor neurons, 37, 51, 51f Hoffman’s reflex, 511t
Gamma-aminobutyric acid, Huntington’s disease and, Homer Wright rosette, tumor cells and, 90, 90f, 108
471, 475 Hoover test, 511t
Ganglion cell tumors, 93, 93f, 112 Horner’s syndrome, 162, 162f, 169, 171, 174, 174f
Ganglion cells, 36, 47 congenital, 164, 164f, 172
Gastroesophageal reflux, 208, 215 urinary catecholamines in, 164, 164f, 172
Gastrulation, 22, 26 House-Brackmann grade IV facial palsy, 177, 181
Gate control theory of pain, 449, 452 HSMN. See Hereditary sensory and motor neuropathy
Gaucher disease, 139, 155–156 (HSMN)
GBM. See Glioblastoma multiforme (GBM) HSV. See Herpes simplex virus (HSV)
Gelastic seizures, 204, 204f, 209 Human chorionic gonadotropin (HCG), in bilateral
Gemistocytic astrocytoma, 86, 86f, 102 papilledema, 560, 560f, 566
Genetic mutations, 56, 56f, 62 Hunt & Hess subarachnoid hemorrhage grading scale, 307,
Germinal matrix hemorrhage, 68, 68f, 81, 584, 584f, 586 311–312, 312t
Germinoma, 100, 100f, 117, 560, 560f, 562, 567–568 Hunter syndrome, 139, 156–157
Gestational age, in ruptured cerebral aneurysms, 593, 595 Hunter’s angle, description of, 219, 219f, 224
Glasgow Coma Scale (GCS), pediatric, 569, 574, 574t Huntington’s disease, 77, 77f, 85, 140, 158
Glatiramer acetate, for multiple sclerosis, 131, 132f, 146 offspring and, 471, 475
Glia, cytopathology of, 59, 65 Hurler syndrome, 139, 156–157
Glioblastoma Hydrocephalus
awake craniotomy for, 323, 323f, 331, 331t adjustment of shunt in, 258, 263
remitting multiple sclerosis and, 324, 324f, 331, 332t excess CSF production and, 257, 261, 262t

www.ketabpezeshki.com 66485438-66485457
INDEX 605

Hydrocephalus (Continued) Intracranial compliance, 185, 189, 190f


frequency of, 546, 550 Intracranial hypertension, in intracranial pressure pulse
management of, 546, 551 waveform, 185, 190, 190f
normal pressure Intracranial lipoma, 528, 528f, 534
ICP monitoring and, 259, 265, 265t Intracranial pressure (ICP), 185, 191f
management of, 258, 263 management of, 273, 286
risk of, 546, 550 pulse waveform of, 190, 190f
Hydrocephalus neurosurgery, pediatric, 546–556 Intractable seizures, sudden unexpected death in epilepsy,
Hydrostatic, in malignant hypertension, 56, 61 203, 208
Hypercalcemia, 137, 152 Intradural extramedullary lesions, 428, 437
Hyperflexion injury, cervicothoracic bracing and, 571, Intradural lipoma, 543–544t
571–572f, 580 Intramedullary glomus AVM, 440, 444
Hyperkalemia, 189, 200 Intramedullary tumors, 427, 437
Hyperkalemia periodic paralysis, 134, 149 Intrathecal baclofen
Hypertension, undiagnosed, 307, 307f, 310–311 for cerebral palsy, 589, 591
Hypocalcemia, 189, 200 for spasticity, 590, 592
Hypokalemia, 189, 200 Intrathecal drug delivery systems, 449, 452
Hypothalamic hamartoma, 562, 568 Intravenous antibiotics
Hypothalamic-pituitary axis, 39, 54 for brain abscess, 355, 360
Hypothermia, in traumatic brain injury, 186, 193 for patient with meningitis, 354, 359
Hypothyroidism, 140, 158 Invasive EEG monitoring, primary indication for,
Hypoxia-ischemia, in neonatal seizures, 206, 211 460, 466
Inverted radial reflex, 500, 511, 511t
Iron overload, in massive blood transfusion, 186, 194
I Ischemic optic neuropathy, anterior, 164, 172
ICP. See Intracranial pressure (ICP) Isocitrate dehydrogenase 1 or 2 (IDH1/2) gene mutations,
ICP monitoring, for closed head injury, 569, 575 in glioma, 326, 337
Ictal EEG, abnormalities in, 204, 204f, 209 ITP. See Idiopathic thrombocytopenic purpura (ITP)
Idiopathic intracranial hypertension (IIH), 141, 159, 161,
161f, 170
Idiopathic thrombocytopenic purpura (ITP), 573, 583 J
IFOF. See Inferior fronto orbital fasciculus (IFOF) JC virus, viral encephalitis and, 358, 362
IIH. See Idiopathic intracranial hypertension (IIH) Jervell Lange-Nielsen syndrome, 179, 182
Image guided-aspiration, of brain abscess, 355, 360 Joubert syndrome, 549, 555
Imaging modalities, 220, 224 Juvenile absence epilepsy, 203, 209
Inclusion body myositis, 133, 148 Juvenile myoclonic epilepsy, 203, 209
Indium-111 Diethylenepentaacetic acid study, 220, 224
Infantile idiopathic scoliosis, 377, 381
Infarction K
combinations of findings in, 216, 222 Karnofsky performance scores (KPS), 236, 238
in regional cerebral blood flow, 186, 192 Kearns-Sayre syndrome, 139, 157, 169, 174
Inferior anastomotic vein of Labb, 5f Kernicterus, 70, 70f, 81
Inferior colliculus Ketamine, 188, 198
in brainstem auditory evoked response, 177, 180, 180f Ki-67, as marker for tumor proliferation, 55, 60
Inferior fronto orbital fasciculus (IFOF), 219, 219f, 223 Kidney, spinal column metastases in, 424, 429
Inferior fronto-occipital fasciculus, 220, 224 Kleeblattschadel deformity, 521, 521f, 524
Inferior longitudinal fasciculus, 219–220, 219f, 223–224 Klippel-Feil syndrome, 527, 527f, 533
Inferior petrosal sinus sampling, for Cushing’s syndrome, KPS. See Karnofsky performance scores (KPS)
343, 349
Infertility, male, and Cushing’s syndrome, 343, 349
Infundibulum, fenestration at, 259, 259f, 266 L
Inion (external occipital protuberance), transverse sinus L4/5 interspace, in iliac crests, 2, 12
location in, 364, 364f, 369 LACS. See Lacunar syndrome (LACS)
Inner hair cells, 179, 183–184f Lactate, 221, 224–225
Inotropes, in spinal cord perfusion pressure and intradural Lambert-Eaton myasthenic syndrome (LEMS), 137, 152
spinal pressure, 387, 393 Laminectomy
Insertional activity, on EMG, 489, 501 for thoracic disc herniation, 407, 407f, 415
Intensity-modulated radiotherapy, 228, 234 Landau-Kleffner syndrome, 205, 210
Interference pattern, reduced, in radiculopathy, 489, Language
501, 501t disorders of, 141, 160
Interictal EEG, abnormal, frequency of, 459, 464 intelligence and, 513, 515
Intermammillary space, fenestration at, 259, 259f, 266 Large cell/anaplastic medulloblastoma, 93, 93f, 111
Internal vesicle sphincter, inhibition of, nervi erigentes Large cerebellopontine angle tumor, in facial nerve,
and, 2, 13 177, 181
Interpeduncular cistern, 7, 7f, 19 Lasegue’s sign, 511t
Intervertebral disc, constituents of, 365, 370 Lateral plantar nerve, 496, 496f, 504
Intervertebral disc disease, 408, 417 Lead, 135, 150
Intracellular calcium ion concentration, 34, 41 Leber’s hereditary optic neuropathy, 139, 157
Intracerebral abscess, 358, 362 Left abducens nerve palsy, 163, 163f, 171
Intracerebral hemorrhage, and enoxaparin, 257, 260, 260t Left acoustic neuroma, 177, 177f, 181

www.ketabpezeshki.com 66485438-66485457
606 INDEX

Left anterior plagiocephaly, due to coronal synostosis, 521, Medulloblastoma, 92, 92f, 110
521f, 525 cerebellar mutism and, 236, 238
Left fourth nerve palsy, 166, 166f, 173 conventional treatment for, 561, 561f, 567
Left internuclear ophthalmoplegia, 161, 161f, 171 Mega cisterna magna, 529, 529f, 536
Left lateral decubitus position, high-speed RTA and, Melanoma, 74, 74f, 83
594, 597 MELAS. See Mitochondrial encephalomyopathy, lactic
Left one-and-a-half syndrome, 168, 168f, 174 acidosis, and stroke-like episodes (MELAS)
Left sided neck dissection, reducing risk of recurrent Membrane potentials, 35, 42
laryngeal nerve palsy, 406, 412 Meniere’s disease, 176, 176f, 179–180, 182
Legius syndrome, 57, 57f, 62 Meningioma, 72, 72f, 82
LEMS. See Lambert-Eaton myasthenic syndrome (LEMS) intraventricular, 72, 72f, 83
Lennox-Gastaut syndrome, 205, 210 tumors and, 113
Leptomeningeal angiomatosis, 58, 58f, 63 Meningocele, 543–544t
Leukodystrophies, 138, 155 Meningothelial meningioma, 94, 94f, 113
Lewy body dementia, 128, 142 Mental disorders, surgery for, 485–488
Lhermitte-Duclos disease. See Dysplastic cerebellar Meralgia paresthetica, 509t
gangliocytoma (Lhermitte-Duclos disease) Mercury, 135, 150
Ligamentum flavum, 364, 368 Mesial temporal sclerosis, as predictor for seizure-free
Limb spasticity, absence of, intrathecal baclofen and, outcome after epilepsy surgery, 459, 463
589, 591 Metachromatic leukodystrophy, 138, 155
Limbic leukotomy, 487t Methotrexate, for HIV, 129, 144
Lipomyelocele, 543–544t O(6)-Methylguanine-DNA Methyl Transferase (MGMT)
Lipomyelomeningocele, 543–544t promoter methylation, 326, 336
Lissencephaly, with band heterotopia, 530, 530f, 537 Methylprednisolone, IV, for multiple sclerosis, 131,
Listeria monocytogenes, causing meningitis, 355, 359 131f, 146
Lobar holoprosencephaly, 67, 67f, 78 Metopic synostosis, 521, 521f, 524, 524f
Localized kyphosis, in mid-cervical area, 572, 581 Microcephaly, isolated, 513, 515, 516t
Loss of smell, in idiopathic Parkinson’s disease, 471, 476 Microglia, 34, 40, 53
Low molecular weight heparin, monitoring of, 121, 123 Microtrauma, 295, 295f, 301
Lower limb Microvascular decompression, for trigeminal neuralgia,
innervation of, 500, 500f, 512, 512f 449, 451
nerve entrapment in, 498, 509, 509t Microvascular proliferation (glioblastoma multiforme), 87,
Low-grade glioma, radiotherapy for, 227, 231 87f, 103, 103–104f
Lumbar pedicle screws, placement of, 366, 372 Mid pons, 9, 9f, 20
Lumbar plexus, 498, 498f, 506, 506f, 506t Midazolam, 188, 198
Lumbar puncture, 340, 340f, 346 Middle cerebral artery aneurysm, 294, 294f, 299
for AIDS, 357, 362 giant, 291, 291f, 297
Lumbar puncture tap test, 257, 257–258f, 262 Middle fossa, triangle of, 296, 305, 305t
Lundberg waves, ICP in, 548, 554, 554t Migraine with aura, 140, 159
Lung, spinal column metastases in, 424, 429 Miller Fisher syndrome, 169, 174
Lymphocytic hypophysitis, 341, 341f, 347, 594, 599 Mitochondrial encephalomyopathy, lactic acidosis, and
stroke-like episodes (MELAS), 139, 157
Mitosis, 87, 87f, 104
M Mixing (50:50) test, in coagulation assays, 188, 199
Macrocephaly, familial, 513, 516, 516t MMN. See Multifocal motor neuropathy (MMN)
Magnesium, for tonic-clonic seizure, 594, 598 Modified Ashworth scale grades, for spasticity, 589, 591, 591t
Magnetic resonance imaging (MRI) Modified Fisher scales, 307, 312, 313t
before closed reduction of cervical fracture-dislocations, Modified Rankin and Glasgow Outcome Scale scores,
388, 396 236–237
head with contrast, of intramedullary spinal cord abscesses, Modiolus, 178, 178f, 181
419, 419f, 423 MOM. See Management of Myelomeningocele (MOM)
sequences, 220, 224 Mongolian blue spots, 573, 583
Malignant infarction, 69, 69f, 81 Mononeuritis multiplex, 137, 152
Malignant peripheral nerve sheath tumors, 98, 98f, 116 Moro reflex, 519t
Malignant spinal cord compression (MSCC), for acute Morquio syndrome, 139, 156–157
management of, 425, 431 Motor axon loss, 489, 501
Mammillary bodies, fenestration at, 259, 259f, 266 Motor cortex stimulation, for pain, 449, 453
Manganese, 135, 150 Motor processing, in neuropsychological tests, 237, 240
Marcus-Gunn phenomenon, 133, 147 Motor unit potential, 489, 501
Marcus-Gunn pupil, 169, 174 Movement disorders, 471–484
Marfan’s syndrome, 585, 588 signs of, 472, 478
Maxillary division of CN V (V2), 3, 3f, 15 Moyamoya
Maximal safe resection, for diffuse astrocytomas, 325, bilateral stenosis and, 308, 314, 314t
325f, 334 caesarean section and, 307, 314
Measles virus, viral encephalitis and, 358, 362 cerebral revascularization for, 308, 315
Mechanical ventilation, 187, 195 progressive cerebral ischemia and, 307, 313
Meckel’s cave, 2, 2f, 12, 12f watershed shift phenomenon and, 308, 315
Median sulcus, 561, 561f, 568 MR spectroscopy, 220, 224
Medulla, 9, 9f MRI. See Magnetic resonance imaging (MRI)
vagal nuclei and, 9, 20 MSCC. See Malignant spinal cord compression (MSCC)

www.ketabpezeshki.com 66485438-66485457
INDEX 607

Mucopolysaccharidoses, 139, 157 Nonfunctioning adenoma, 342, 342f, 347


Multifocal motor neuropathy (MMN) Normal development, pediatric general principles and,
with conduction block, 136, 152 513–520
Multiloculated hydrocephalus, shunt for, 548, 548f, 553 NS tumors, molecular assays of, 59, 64
Multiple metastasis, 74, 74f, 83 Nuclear/hyperchromasia, 56, 61
Multiple motor root compromise, F-wave for, 489, 501 Nuclei, 368
Multiple sclerosis Nutritional deficiency, 137, 154–155
trigeminal neuralgia and, 448, 451
Multiple subpial transection, for seizure activity, 459, 463
Mutism, cerebellar cognitive affective syndrome and, O
236, 238 Obsessive compulsive disorder, 228, 234
Myasthenia gravis, 162, 162f, 171 Obturator syndrome, 509t
Mycobacterium tuberculosis, 75, 75f, 84 Occipital condyle fractures, management of, 388, 397
Mycotic aneurysm, 69, 69f, 81 Occipital cortical dysgenesis, 207, 212
Myelocele, 543–544t Ocular myasthenia gravis, 162, 162f, 171
Myelomeningocele, 532, 532f, 543, 543–544t Oculomotor nerve, 3, 3f, 14
Myositis, inclusion body, 133, 148 III, 3, 3f, 15
Myxopapillary ependymoma, 91, 91f, 109 Oculomotor palsy, 292, 292f, 294, 294f, 298, 300
Olfaction, 37, 49
Oligodendroglioma, 88, 88f, 104
N anaplastic, 105
N-acetylcysteine, for contrast nephropathy, 216, 221 tumor margins in, 325, 325f, 335
NAI. See Non-accidental injury (NAI) Operative approaches, to CSF disorder, 259, 266, 266–267t
Necrosis, beginning of, 355, 360 Ophthalmoplegia, 168, 174
Neonatal seizures, hypoxia-ischemia in, 206, 211 Optic chiasm, 3, 3f, 14
Nerve palsy nerves and, 228, 234
left fourth, 166, 166f, 173 Optic neuritis, 162, 171
left oculomotor, 167, 167f, 173 Optic pathway gliomas, 558f, 562, 568
Neural tube, dorsoventral patterning of, 23, 28 Optic vesicle, 22, 27
Neurally mediated (vasovagal) syncope, 140, 158–159, 208, Opticocarotid recess, 343, 343f, 350
215 Oral glucose tolerance test, for acromegaly diagnosis,
Neurenteric cyst, 543–544t 343, 350
Neuroblastoma, 169, 174 Orbital apex syndrome, 165, 165f, 173, 173t
Neurocysticercosis, 76, 76f, 84 Organ of Corti, 179, 179f, 183f
Neurofibromas, 97, 97f, 115 Osler-Weber-Rendu syndrome (HHT), 585, 588
Neurofibromatosis, 523, 523f, 525 Osseous spiral lamina, 179, 183–184f
Neurofibromatosis type 1, 55, 55f, 57–58, 57–58f, 60, 62 Ossicular dislocation, 178, 182
Neurofibromatosis type 2, 58, 58f, 62 Otoacoustic emission testing, 178, 182
Neurointensive care, 185–200 Outer hair cells, 179, 183–184f
Neurological rehabilitation, 236–240 Oval window, 179f, 183f
Neurology Oxygen, inhaling, in cluster headache, 130, 145
signs and tests of, 408, 416 Oxygen-dissociation curve, 187, 195, 195f
stroke and, 127–160
Neuromuscular scoliosis, management of, 377, 380
Neuromyelitis optica (NMO), 130, 130–131f P
Neurons Pacemaker, 189, 200
cytopathology of, 59, 65 Pachygyria, 67, 67f, 79–80
passive membrane properties of, 35, 42 Pain, surgical procedures for, 450, 457
Neuro-otology, 176–184 Pain surgery, 448–458
Neuropathology I, basics, 55–65 Palmar grip reflex, 519t
Neuropathology II, 66–85 Palmar keratosis. see Cowden syndrome
Neuropathy, 136, 152 Pancoast tumor, 169, 174
Neurophysiology, 34–54 Papile grade III germinal matrix hemorrhage, with
Neuropsychological tests, 237, 240, 459, 464 hydrocephalus, 547, 552
Neuropsychology, 236–240 Papillary ependymoma, 90, 90f, 108
Neuroradiology, 216–225 Papillary meningioma, 97, 97f, 115
Neurosyphilis, 140, 158 Paraneoplastic cerebellar degeneration, 137, 152
Neurotoxins, 38, 52 Paraneoplastic disorders, 137, 152
Neurotransmission Paraproteinemic neuropathy, 137, 152
at chemical synapses, 36, 44 Parietal seizures, 207, 212
at neuromuscular junction, 37, 50 Parinaud syndrome, 163, 163f, 169, 172, 174
Neurotransmitters, 38, 52 Parkinson-plus syndromes, 472, 476
Neurulation, 24, 32 Parkinson’s disease
secondary versus retrogressive, 22, 26 medical therapy for, 474, 482
Newborn, average term, measurements of, 513, 515 motor complications in, 474, 483
NF-1 associated optic nerve glioma, 71, 71f, 82 Partially compensated respiratory acidosis, 188, 199
NMO. See Neuromyelitis optica (NMO) PCNSL. See Primary CNS lymphoma (PCNSL)
Nocturnal pain, in thoracic spine, 407, 415 Pediatric movement disorders, 589–592
Noise-induced hearing loss, 178, 182 Pediatric neuro-oncology, 557–568
Non-accidental injury (NAI), 573, 578–579t, 583 Pedicle, in lumbar vertebra, 365, 365f, 371

www.ketabpezeshki.com 66485438-66485457
608 INDEX

Pelizaeus-Merzbacher disease, 138, 155 Postmitotic young neurons, 23, 29


Periaqueductal gray, 448, 451 Postop Horner’s syndrome, 406, 412
Pericallosal aneurysm, 293, 293f, 299 Post-traumatic amnesia, in traumatic brain injury, 237, 239
Perilymph fistula, 178, 182 Post-traumatic epilepsy, risk of development of, 272, 284
Periodic paralysis, hyperkalemia, 134, 149 Potassium, 35, 42
Perioperative care, 185–200 Prader-Willi syndrome, 136, 151
Peripheral nerve, 40, 40f, 54, 489–512 Prednisolone, for temporal arteritis, 134, 149
injury, 37, 50 Pregnancy, neurosurgery and, 593–599
rabies virus spreading retrogradely in, 354, 359 Pre-hospital spinal immobilization, 386, 390
Peripheral neuropathy, 169, 175 Primary adrenal insufficiency, and diabetes insipidus,
Periungual fibroma, 57, 57f, 62 341, 346
Periventricular leukomalacia, 68, 68f, 81 Primary CNS lymphoma (PCNSL), 73, 73f, 83, 99,
Periventricular nodular heterotopias, 68, 68f, 80 99f, 116
Persistent terminal ventricle, 543–544t Primary neurulation, 22, 26
Persistent trigeminal artery, 1, 11, 11f Primitive reflexes, 514, 518, 519t
Personality disorder, lesional neurosurgical procedures for, Procyclidine, 121, 124
485, 488 Progressive multifocal leukoencephalopathy (PML), 76, 76f,
Pfeiffer’s syndrome, 523, 523f, 525 84, 129, 129–130f, 144
PFO. See Patent foramen ovale (PFO) Projection/association tracts, 4, 6, 15–16, 18
Phalen’s test, 511t Prolactinoma
Pharmacology, 121–126 Hook effect and, 342, 348
Pharyngeal arch, derivatives, 24, 32 Pronator teres, 489, 501
Phase-contrast, 220, 224 Propofol, 188, 198
Physiotherapy, for cervical spondylosis, 403, 403f, 409 Prostate, spinal column metastases in, 424, 429
Pick disease, 77, 77f, 84, 127, 127–128f, 142 Prothrombin complex concentrate, 121, 123
Pilocytic astrocytoma, 557, 557f, 561, 563, 568 Proton beam therapy, 226, 230, 230f
of tumors, 88, 88f, 105 Proximal catheter obstruction, 549, 555
Pinch sign, 511t Proximal tarsal tunnel syndrome, 509t
Pineal body, 22, 27 Psammomatous meningiomas, 95, 95f, 113
Pineal parenchymal tumor of intermediate differentiation, 99, Pseudomonas, intracerebral abscess and, 358, 362
99f, 117 Pseudosubluxation, 573, 583
Pineoblastoma, 99, 99f, 117, 558, 558f, 564 Pseudotumor cerebri, and optic nerve sheath fenestration,
Pineocytoma, 99, 99f, 117 258, 264
Ping-pong fracture, 569, 569–570f, 575 PTV. See Planning target volume (PTV)
Piriformis syndrome, 509t Pupillary abnormalities, 169, 174
Pituitary adenomas, 73, 73f, 83, 101, 101f, 119 Pure-tone audiometry, 178, 182
stereotactic radiosurgery for, 227, 233 Purulent meningitis, 75, 75f, 83
surgery for, secreting tumor as indicator for, 342, 348 Pyogenic vertebral osteomyelitis, 418, 418f, 420
transsphenoidal resection of, CT head after, 343, 350 Pyridoxine deficiency, 138, 154–155
Pituitary dysfunction, 344, 351
Pituitary profile, 340, 340f, 345
Planning target volume (PTV), 226, 229 R
Plantar grip reflex, 519t Radial nerve, abnormalities of, 493, 493t, 503
Play audiometry, 178, 182 Radiation myelopathy, 426, 434
Pleomorphic xanthoastrocytoma, 89, 89f, 105, 558, 558f, 564 Radiculomedullary artery, 440, 443
PML. See Progressive multifocal leukoencephalopathy (PML) Radiobiology, five Rs of, 230
PNET. See Primitive neuroectodermal tumor (PNET) Radiosurgery, 226–235
Polymicrogyria, 530, 530f, 538 intracranial targets for, radiobiological response of,
Pontine paramedian reticular formation, 161, 170, 170f 226, 229
Porencephalic cysts, 82, 533, 544 stereotactic, 226, 228
Porencephaly, 70, 70f for acoustic neuroma, 227, 232
Posterior C1-C2 fusion indications in, 228, 234
for atlantoaxial subluxation, 406, 407f, 414 for pituitary adenomas, 227, 233
for os odontoideum, 389, 389f, 398 for trigeminal neuralgia, 227, 231
Posterior circulation aneurysms, 527, 527f, 534 tissue tolerance to, 228, 234
pediatric, 585, 587 Radiotherapy
Posterior circulation stroke syndromes, 295, 301, 301–302t for diffuse intrinsic pontine glioma, 559, 559f, 564
Posterior communicating artery aneurysm, 292, 292f, 297, for low-grade glioma, 227, 231
298t for malignant spinal cord compression, 425, 432
Posterior deformational plagiocephaly, right, 522, 522f, 525 stereotactic, 226–235
Posterior foraminotomy, for left C6 radiculopathy, 405, for brain metastasis, 226, 231
405f, 411 Ramus communicans, 366–367f, 367, 373, 373f
Posterior fossa lesions, 326, 338 Rankin and Glasgow Outcome Scale scores, modified,
Posterior inferior cerebellar artery aneurysm, 294, 236–237
294f, 300 Rathke’s cleft cyst, 74, 74f, 83
Posterior pituitary, of brain, 86, 86f, 102 Red nucleus, 1, 11
Posterior reversible encephalopathy syndrome, 132, for vagus nerve stimulation, 485, 488
132–133f, 147 Reduction, in fractionation, 226, 230
Posterior semicircular canal, 4, 4f, 15 Referred pain, phenomenon of, 448, 451
Postherpetic neuralgia, 140, 159 Reflex anoxic seizure, 208, 215

www.ketabpezeshki.com 66485438-66485457
INDEX 609

Regional cerebral blood flow, 185, 189 SFOF. See Superior fronto-orbital fasciculus (SFOF)
infarction in, 186, 192 Shaken baby syndrome, 571, 571f, 577
Relaxin, for back pain, 593, 596 Shunt allergy, 259, 267
Repeat transsphenoidal surgery, for Cushing’s syndrome, Shunt malfunction, in pregnancy, 594, 598
343, 349 Shunt tap, CSF infection in, 546, 550
Respiratory rate, 187, 196 Single burr hole, with subdural drain, 271, 271f, 282
in children, 514, 517, 517t Single-staged posterior fusion procedures, for juvenile
Resting membrane potential, 34, 41 idiopathic scoliosis, 377, 381
Retinal telangiectasias, 133, 148 Skull base
Retinoblastoma, 90, 90f, 107 approaches for, 345, 352
Rett syndrome, 136, 151 foramina of, 344, 344f, 351
Reversible encephalopathy syndrome, posterior, 132, SLF. See Superior longitudinal fasciculus (SLF)
132–133f, 147 SLIC. See Subaxial cervical spine Injury Classification (SLIC)
Reye syndrome, 569, 574 Smoking, and hemorrhage, 291, 291f, 297
Rhabdoid meningioma, 97, 97f, 114 Solitary nucleus, 2, 13
Rhombencephalon, 23, 27 Spasticity, 589, 591
Right Adies tonic pupil, 164, 164f, 172 Spectroscopy, 55, 60
Right PCA infarct, 69, 69f, 81 Sphingolipidosis, 138, 155–156
Rods, of resting membrane potential, 36, 48 Spina bifida, posterior, 543–544t
Rolandic epilepsy, epilepsy surgery referral for, 459, 463 Spina bifida occulta. See Closed spinal dysraphism
Rosenthal fibers (RFs), 86, 86f, 102 Spinal biomechanics, load-sharing concept of, 365, 372
Rossolimo reflex, 519t Spinal cord, development of, 23, 30
Rostral medulla, 9, 9f, 20 Spinal cord injury (SCI)
Rostral pons, 10, 10f, 21 medical management of, 386, 392
rehabilitation in, goals of, 237, 240
Spinal deformity, 377–385
S Spinal dysraphism, 533, 543, 543–544t
S1 radiculopathy Spinal fractures, 389, 399
suspected, H-reflex for, 489, 502 Spinal infection, 418–423
Sacral plexus, 498, 498f, 507, 507f Spinal laminae, 368
Sagittal plane deformities, 378, 383 Spinal metastatic disease
Sagittal synostosis, 521, 521f, 524 clinical presentations of, 424, 429
SAPPHIRE, 309, 318 imaging of, 424, 429
Sarcoidosis, 136, 152 management of pain related to, 425, 431
Sarcoma, brain metastases, 557, 563, 563t prognostic scoring for, 424, 430
Scala media, 179, 179f, 183f surgery for, 425, 432
Scala vestibuli, 179, 179f, 183f surgical management of, 425, 434
Scheuermann’s kyphosis, 379, 379f, 383 Spinal oncology, 424–439
Schizencephaly, 70, 70f, 82, 530, 530f, 533, 538, 544 Spinal reflex arc, interruption of, dorsal rhizotomy and, 590,
Schizophrenia, lesional neurosurgical procedures for, 485, 488 592, 592f
Schwannomas, 73, 73f, 83, 98, 115 Spinal stability, Punjabi &White’s definition of, 365, 371
SCI. See Spinal cord injury (SCI) Spinal trauma
Scoliosis, 377–385 acute pathology and, 386–402
curves, Lenke classification of, 378, 382 mechanism of, 390, 399
development of, 377, 380 predispose children to, 571, 579
management of, 379, 385 surgical approaches for, 390, 400
terms of, 379, 384 Spinal tuberculosis, 419, 419f, 422
SCS. See Spinal cord stimulation (SCS) Spinal vascular malformations, successful treatment of, 441, 446
Secondary neoplasm, risk of, after stereotactic radiosurgery, Spinal vascular neurosurgery, 440–447
227, 233 Spine, general principles of, 364–376
Secondary ossification centers, of C2 vertebra, 2, 12 Spine and peripheral pain syndromes, 450, 456
Secretory meningioma, 95, 95f, 113 Spinocerebellar ataxia, 137, 153
Segmental dysgenesis, 543–544t Splanchnic nerves, 2, 13
Seizure onset, 460, 460f, 467 Split cord malformation, 543–544t
remote/distant site of, 459, 465 Spongiform encephalopathy, 354, 359
Seizure semiology, 462, 469 Spontaneous epilepsy remission, predictor of, 203, 208
Seizures, 203–215 Spontaneous intracranial hypotension, 141, 159
complex partial, 207, 213 SPORT. See Spinal patient outcomes research trial (SPORT)
gelastic, 204, 204f, 209 Spurling’s sign, 511t
intractable, 203, 208 Spurling’s test, radiculopathy and, 403, 410
isolated post-traumatic, 269, 278, 278t SRS. See Stereotactic radiosurgery (SRS)
neonatal, 206, 211 Staphylococcus aureus, intracerebral abscess and, 358, 362
parietal, 207, 212 Steele-Richardson-Olszewski syndrome, 169, 174
Sellar and parasellar lesions, 344, 351 Stereotactic biopsy, for brain lesion, 323, 329
Sensorineural deafness, 57, 57f, 62 Stereotactic needle aspiration, for brain abscess, 355,
Sensory decussation, medulla at, 8, 8f, 20 355f, 360
Sensory disturbances, 449, 454 Stereotactic radiosurgery (SRS), 226, 228, 342, 342f, 348
Sensory receptors, 38, 52 for acoustic neuroma, 227, 232
Septo-optic dysplasia, 528, 528f, 535 for arteriovenous malformations, 585, 588
Serum CSF and HCG and AFP, 341, 341f, 346 indications in, 228, 234

www.ketabpezeshki.com 66485438-66485457
610 INDEX

Stereotactic radiosurgery (SRS) (Continued) Task oriented therapy, 237, 239


for pituitary adenomas, 227, 233 Taste sensation, 37, 50
for trigeminal neuralgia, 227, 231 Tay-Sachs disease, 138, 155–156
Stereotactic radiotherapy, 226–235 Tectorial membrane, 179, 179f, 183f, 367, 367f, 373, 373f
for brain metastasis, 226, 231 Telomerase reverse transcriptase (TERT) promoter
Striae medullaris, 561, 561f, 568 mutations, in gliomas, 326, 337
Stroke Temporal arteritis, 134, 149
CT/MR angiography for, 308, 316, 316t Temporal horns, compression or loss of visualization of, 270,
intravenous thrombolysis, 308, 308f, 315 270f, 281
neurology and, 127–160 Temporal lobectomy, for febrile seizures, 460, 460f, 467
Sturge-Weber syndrome, 585, 588 Teratoma, germ cell element in cartilage, 100, 100f, 118
Subarachnoid hemorrhage, during pregnancy, 593, 595 Terminal myelocystocele, 543–544t
Subaxial cervical spine Injury Classification (SLIC), 388, 396 Tetrabenazine, 590, 592
Subcaudate tractotomy, 486, 488 Thalamotomy, 472, 477
stereotactic, 487t Thalamus, 6, 6f, 17
Subclavian artery, left, 1, 10, 10f Thermocoagulation, in anterior cingulotomy, 485, 487
Subcommissural organ, of intact blood-brain barrier, 34, 40 Thiamine deficiency, 138, 154–155
Subdural grid/strips, for epileptic focus, 459, 465 Third nerve palsy, 169, 175
Subdural hematomas Thoracic pedicle screw
chronic, and females, 271, 282 placement of, 366, 372
indications for surgery in, 272, 272f, 283 site of entry for, 365, 370
Subependymal giant cell astrocytoma, 89, 89f, 106 3-Hz spike, in ictal EEG, 204, 204f, 209
Subependymal heterotopia, 529, 529f, 537 Thrombin clotting time, in coagulation assays, 188, 199
Subependymoma, as benign glial neoplasm, 92, 92f, 109 Thrombocytopenia, in bleeding diatheses, 188, 198
Subfalcine herniation, 66, 66f, 78 Thromboembolism, and intracranial bleeding, 257, 260, 261t
Subgenual anterior cingulate, for anorexia nervosa, 485–486 Thunderclap headache, 141, 159
Subiculum, 461, 461f, 468 Thyroid, spinal column metastases in, 424, 429
Sublimis arch, for ulnar nerve entrapment, 448, 451 Tight filum terminale, 543–544t
Subthalamic nucleus, 1, 11, 485, 487 Tinel’s test, 511t
Success score, in endoscopic third ventriculostomy, 547, 547f, Tissue tolerance, to radiosurgery, 228, 234
551, 552t Tizanidine, for spasticity, 590
Sudden unexpected death in epilepsy (SUDEP), in intractable TLICS. See Thoracolumbar injury classification system
seizures, 203, 208 (TLICS)
SUDEP. See Sudden unexpected death in epilepsy (SUDEP) Torn labial frenulum, in mobile child, 571, 578
Sulci and gyri, 7–8, 7f, 19 Toxoplasma gondii, AIDS and, 356, 356f, 361
Sulcus limitans, 561, 561f, 568 Toxoplasmosis, hydrocephalus and, 546, 550, 550t
Sumatriptan, for migraine, 130, 145 Tractography, 220, 224
Superficial middle cerebral vein of Silvius, 5f Transalar herniation, ascending, 272, 272f, 284
Superficial temporal artery, bypass grafts and, in children, Transcallosal, for dorsolateral brainstem cavernous
585, 588 malformations, 585, 587
Superior anastomotic vein of Trolard, 5f Transcortical sensory aphasia, 141, 160
Superior cerebellar artery Transcranial magnetic stimulation, 486, 488
trigeminal neuralgia and, 449, 452 Transfemoral transvenous embolization, 585, 587
Superior fronto-occipital fasciculus, 220, 224 Transient global amnesia, 139, 158
Superior fronto-orbital fasciculus (SFOF), 218, 218f, 223 Transmembrane electrical and ionic gradients, in brain,
Superior hypophyseal artery aneurysm, 292, 292f, 298 186, 191
Superior longitudinal fasciculus (SLF), 218, 218f, 220, Transoral odontoid resection, for RA progression, 407,
223–224 407f, 414
Superior ophthalmic vein, 1, 11 Transventricular tap, meningitis and, 548, 548–549f, 555
Superior temporal gyrus (Heschl’s gyrus), 36, 47 Transverse ligament, 367, 367f, 373, 373f
Superior vestibular nerve, 4, 4f, 15 Traumatic brain injury (TBI)
vestibular schwannoma in, 176, 180 complications after, 273, 289, 289t
Supraophthalmic aneurysm, 292, 292f, 298 diffuse axonal injury and, 268, 271, 271f, 274, 281
Suprapubic extensor reflex, 519t GFAP as biomarker for, 269, 279
Supratentorial lesions, 326, 338 hypothermia in, 186, 193
Supratentorial primitive neuroepithelial tumor (PNET), 93, hypoxia/hypoglycemia and, 268, 275, 275t
93f, 111 ICP placement in, and diffuse injury type II, 269, 279
Surgical arm, for metastatic spinal cord compression, 425, 433 indication for surgery in, 269, 269f, 280
Surgical Timing in Acute Spinal Cord Injury Study mild, 268, 276, 276–277t
(STASCIS) trial, 387, 393 moderate, 269, 279
Survival motor neuron gene testing, 129, 143 ratios for, 268, 274
Susceptibility weighted imaging, 220, 224 Tremor, 473, 479
Suture synostoses, frequency of, 522, 525 Trochlear nerve, in annulus of Zinn, 2, 12, 12f
Syncope, 140, 158 Tuber cinereum, fenestration at, 259, 259f, 266
Tuberculosis, 523, 523f, 526
in neonatal meningitis, 549, 556
T Tuberous sclerosis, 56, 56f, 62, 71, 71f, 82, 559, 559f, 565
Tabes dorsalis, affecting CNS, 358, 362 Tuberous sclerosis complex, 89, 89f, 106
Taenia solium, causing cysticercosis, 357, 357f, 361 Tumor markers, 59, 65
Tanycytic ependymomas, 91, 91f, 108 Tumor resection, during pregnancy, 593, 595

www.ketabpezeshki.com 66485438-66485457
INDEX 611

Tumor syndrome, 59, 64 Ventro-intermedius nucleus (Vim) thalamic stimulation, for


Type A delta, of nerve fiber, 489, 500, 500t tremor, 472, 477
Type II Chiari malformation, 531, 531f, 539, 540t Verocay bodies, 98, 98f, 116
Type II intramedullary (glomus) AVM, 441, 441f, 445 Vertebral artery V2 segment, 365, 371, 371f
Type III juvenile (extradural intradural) AVM, 440, Vertebral osteomyelitis
440f, 445 good candidates for nonsurgical management of, 419, 421
Type IV-C perimedullary (ventral intradural) AVF, 440, surgical management of, 419, 421
440f, 445 Vertebrobasilar aneurysm, proportion of rupture of, 291,
291f, 297
Vestibular schwannoma, 179, 182
U in superior vestibular nerve, 176, 180
UK NICE head injury guidelines, for children, 569, Vestibular-ocular reflex, description of, 176, 180
574, 575t Vestibulocochlear nerve, 4, 4f, 15
Ulnar nerve entrapment, below the elbow, 491, 491–492t, 503 Viral encephalitis, 358, 362
Uncal herniation, 169, 175 Virchow’s law, 522, 525
Uncinate fasciculus, 219, 219f, 224 Viridans streptococci, intracerebral abscess and, 358, 362
Uncovertebral (Luschka’s) joints, in subaxial cervical spine, Visual processing, events during, 37, 48
364, 370, 370f Vitamin B12 deficiency, 138, 154–155
Unicoronal suture synostosis, 522, 522f, 525 Vitamin E deficiency, 138, 154–155
Unilateral uncal herniation, 271, 283 Volkman’s contracture, 511t
Upper limb VTE. See Venous thromboembolism (VTE)
innervation of, 499, 499f, 510, 510f
nerve entrapment in, 498, 508
Urinary catecholamines, in Horner’s syndrome, 164,
164f, 172
W
Waardenburg syndrome, 179, 182
Urodynamic testing, 532, 542
US PECARN Pediatric head trauma algorithm, 569, 574 Wada test, for postoperative language and memory
impairment, 459, 464
Warfarin, clotting factor affected by, 121, 123
V Wartenberg’s sign, 511t
VACTERL association, 378, 383 Wave discharge, in ictal EEG, 204, 204f, 209
VAE. See Venous air embolism (VAE) WBRT. See Whole brain radiotherapy (WBRT)
Vagal maneuver plus adenosine, 189, 200 Wernicke encephalopathy, 135, 150
Vagal nuclei, 9, 20 Wernicke-Korsakoff syndrome, 135, 150
Vancomycin, 121, 124 Wernicke’s aphasia, 141, 160
Vascular dementia, 78, 78f, 85 West syndrome, 205, 205f, 209
Vascular disorders, of spine, 442, 447 Whipple disease, affecting CNS, 358, 362
Vascular neurology, pediatric, 584–588 Whole brain radiotherapy (WBRT), 228, 234
Vascular neurosurgery, trials in, 296, 303, 303–304t Williams syndrome, 136, 151
Vascular territories, 5, 16 World Federation of Neurological Surgeons (WFNS)
Vasogenic edema, 66, 66f, 78 subarachnoid grading scale, 307, 311, 311t
Vein of Galen, 5f Wright, James Homer, 108
Vein of Galen malformation, 584, 584f, 586 Wrist extension, myotomes of, 500, 511
Velocardiofacial syndrome, 136, 151 Wyburn-Mason syndrome, 585, 587
Venous drainage, 296, 302
of spine, 442, 442f, 447, 447f
Venous thromboembolism (VTE) X
prophylaxis, in SCI, 387, 395 X-linked adrenoleukodystrophy, 138, 155
Ventral induction, of brain, 22, 27 X-linked hydrocephalus, 549, 555
Ventral root, of spinal nerve, 366–367f, 367, 373, 373f
Ventriculitis, 75, 75f, 83
Ventriculosubgaleal shunts, posthemorrhagic hydrocephalus Y
and, 547, 552 Yolk sac tumor, 100, 100f, 118

www.ketabpezeshki.com 66485438-66485457
This page intentionally left blank

www.ketabpezeshki.com 66485438-66485457

You might also like